Vous êtes sur la page 1sur 486

exercise judicial restraint where the issue before it has already been mooted by

G.R. No. 191988. August 31, 2010.* subsequent events. More importantly, the constitutional requirement of the
ATTY. EVILLO C. PORMENTO, petitioner, vs. JOSEPH “ERAP” EJERCITO existence of a “case” or an “actual controversy” for the proper exercise of the power
ESTRADA and COMMISSION ON ELECTIONS, respondents. of judicial review constrains us to refuse the allure of making a grand
Courts; Judicial Review; Moot and Academic Issues; While the novelty and pronouncement that, in the end, will amount to nothing but a non-binding opinion.
complexity of the constitutional issue involved in this case present a temptation that The petition asks whether private respondent Joseph Ejercito Estrada is
magistrates, lawyers, legal scholars and law students alike would find hard to covered by the ban on the President from “any reelection.” Private respondent was
resist, prudence dictates that this Court exercise judicial restraint where the issue elected President of the Republic of the Philippines in the general elections held
before it has already been mooted by subsequent events.—The novelty and on May 11, 1998. He sought the presidency again in the general elections held on
complexity of the constitutional issue involved in this case present a temptation May 10, 2010. Petitioner Atty. Evillo C. Pormento opposed private respondent’s
that magistrates, lawyers, legal scholars and law students alike would find hard candidacy and filed a petition for disqualification. However, his petition was
to resist. However, prudence dictates that this Court exercise judicial restraint denied by the Second Division of public respondent Commission on Elections
where the issue before it has already been mooted by subsequent events. More (COMELEC).1 His motion for reconsideration was subsequently denied by the
importantly, the constitutional requirement of the existence of a “case” or an COMELEC en banc.2
“actual controversy” for the proper exercise of the power of judicial review Petitioner filed the instant petition for certiorari3 on May 7, 2010. However,
constrains us to refuse the allure of making a grand pronouncement that, in the under the Rules of Court, the filing of such petition would not stay the execution
end, will amount to nothing but a non-binding opinion. of the judgment, final order or resolution of the COMELEC that is sought to be
Same; Same; Same; Words and Phrases; When a case is moot, it becomes non- reviewed.4 Besides, petitioner did not even pray for the issuance of a temporary
justiciable; An action is considered “moot” when it no longer presents a justiciable restraining order or writ of preliminary injunction. Hence, private respondent was
controversy because the issues involved have become academic or dead or when the able to participate as a candidate for the position of President in the May 10,
matter in dispute has already been resolved and hence, one is not entitled to judicial
intervention unless the issue is likely to be raised again between the parties.—As a _______________
rule, this Court may only adjudicate actual, ongoing controversies. The Court is
not empowered to decide moot questions or abstract propositions, or to declare 1 Resolution dated January 10, 2010 penned by Commissioner Nicodemo T.
principles or rules of law which cannot affect the result as to the thing in issue in Ferrer and concurred in by Commissioners Lucenito N. Tagle and Elias R.
the case before it. In other words, when a case is moot, it becomes non-justiciable. Yusoph. Rollo, pp. 21-46.
An action is considered “moot” when it no longer presents a justiciable controversy 2 Resolution dated May 4, 2010 penned by Commissioner Armando C. Velasco
because the issues involved have become academic or dead or when the matter in and concurred in by Chairperson Jose A.R. Melo and Commissioners Rene V.
dispute has already been resolved and hence, one is not entitled to judicial Sarmiento, Nicodemo T. Ferrer, Lucenito N. Tagle, Elias R. Yusoph and Gregorio
intervention unless the issue is likely to be raised again between the parties. There Y. Larrazabal. Id., pp. 47-51.
is nothing for the court to resolve as the determination thereof has been overtaken 3 Under Rule 65 in relation to Rule 64 of the Rules of Court.
by subsequent events. 4 See Section 8, Rule 64 of the Rules of Court.
SPECIAL CIVIL ACTION in the Supreme Court. Certiorari. 533
The facts are stated in the resolution of the Court.
VOL. 629, AUGUST 31, 2010 533
George Erwin M. Garcia, Joan M. Padilla and Julie Ann V. Chang for private
respondent Joseph “Erap” Ejercito Estrada. Pormento vs. Estrada
2010 elections where he garnered the second highest number of votes.5
RESOLUTION Private respondent was not elected President the second time he ran. Since the
issue on the proper interpretation of the phrase “any reelection” will be premised
CORONA, C.J.: on a person’s second (whether immediate or not) election as President, there is no
What is the proper interpretation of the following provision of Section 4, Article case or controversy to be resolved in this case. No live conflict of legal rights
VII of the Constitution: “[t]he President shall not be eligible for any reelection?” exists.6 There is in this case no definite, concrete, real or substantial controversy
The novelty and complexity of the constitutional issue involved in this case that touches on the legal relations of parties having adverse legal interests. 7 No
present a temptation that magistrates, lawyers, legal scholars and law students specific relief may conclusively be decreed upon by this Court in this case that will
alike would find hard to resist. However, prudence dictates that this Court benefit any of the parties herein.8 As such, one of the essential requisites for the
532 exercise of the power of judicial review, the existence of an actual case or
controversy, is sorely lacking in this case.
532 SUPREME COURT REPORTS ANNOTATED
As a rule, this Court may only adjudicate actual, ongoing controversies. 9 The
Pormento vs. Estrada Court is not empowered to decide moot questions or abstract propositions, or to
declare principles or rules of law which cannot affect the result as to the thing in poration, 494 U.S. 472 [1990]). The second of these requirements is absent in this
issue in the case before it.10 In other words, when a case is moot, it becomes non- case. It is highly speculative and hypothetical that petitioner would be subjected
justiciable.11 to the same action again. It is highly doubtful if he can demonstrate a substantial
likelihood that he will “suffer a harm” alleged in his petition. (See Honig v.
_______________ Doe, supra.)

5 Benigno Simeon C. Aquino III garnered the highest number of votes and was 12 Santiago v. Court of Appeals, G.R. No. 121908, 26 January 1998, 285 SCRA
therefore proclaimed as President. 16.
6 See discussion on the concept of “case” or “controversy” in Cruz, Isagani,
Philippine Political Law, 2002 Edition, p. 259.
7 Id.
8 Id.
9 Honig v. Doe, 484 U.S. 305 (1988).
10 Id.
11 While there are exceptions to this rule, none of the exceptions applies in this
case. What may most probably come to mind is the “capable of repetition yet requirements, subject to the observance of fundamental and essential
evading review” exception. However, the said exception applies only where the requirements of due process in justiciable cases presented before them. (Samalio
vs. Court of Appeals, 454 SCRA 462 [2005])
following two circumstances concur: (1) the challenged action is in its duration too
short to be fully litigated prior to its cessation or expiration and (2) there is a
reasonable expectation that the same complaining party would be subjected to the ——o0o——
same action again (Lewis v. Continental Bank Cor-
534 G.R. No. 191618. November 23, 2010.*
534 SUPREME COURT REPORTS ANNOTATED ATTY. ROMULO B. MACALINTAL, petitioner, vs. PRESIDENTIAL
ELECTORAL TRIBUNAL, respondent.
Pormento vs. Estrada
Remedial Law; Constitutional Law; Locus Standi; Parties; On more than one
An action is considered “moot” when it no longer presents a justiciable occasion we have characterized a proper party as one who has sustained or is in
controversy because the issues involved have become academic or dead or when immediate danger of sustaining an injury as a result of the act complained of.—On
the matter in dispute has already been resolved and hence, one is not entitled to more than one occasion we have characterized a proper party as one who has
judicial intervention unless the issue is likely to be raised again between the sustained or is in immediate danger of sustaining an injury as a result of the act
parties. There is nothing for the court to resolve as the determination thereof has complained of. The dust has long settled on the test laid down in Baker v. Carr:
been overtaken by subsequent events.12 “whether the party has alleged such a personal stake in the outcome of the
Assuming an actual case or controversy existed prior to the proclamation of a controversy as to assure that concrete adverseness which sharpens the
President who has been duly elected in the May 10, 2010 elections, the same is no presentation of issues upon which the court so largely depends for illumination of
longer true today. Following the results of that election, private respondent was difficult questions.” Until and unless such actual or threatened injury is
not elected President for the second time. Thus, any discussion of his “reelection” established, the complainant is not clothed with legal personality to raise the
will simply be hypothetical and speculative. It will serve no useful or practical constitutional question.
purpose. Same; Same; Same; Same; Estoppel; His failure to raise a seasonable
Accordingly, the petition is denied due course and is hereby DISMISSED. constitutional challenge at that time, coupled with his unconditional acceptance of
SO ORDERED. the Tribunal’s authority over the case he was defending, translates to the clear
Carpio, Carpio-Morales, Velasco, Jr., Nachura, Leonardo-De Castro, absence of an indispensable requisite for the proper invocation of this Court’s power
Bersamin, Del Castillo, Abad, Villarama, Jr., Perez, Mendoza and Sereno, JJ., of judicial review.—Although there are recognized exceptions to this requisite, we
concur. find none in this instance. Petitioner is unmistakably estopped from assailing the
Brion, J., On Leave. jurisdiction of the PET before which tribunal he had ubiquitously appeared and
Peralta, J., On Official Leave. had acknowledged its jurisdiction in
Petition dismissed.
_______________
_______________
* EN BANC.
784
784 SUPREME COURT REPORTS ANNOTATED Same; Same; Ut Magis Valeat Quam Pereat.—Last, ut magis valeat quam
pereat—the Constitution is to be interpreted as a whole. We intoned thus in the
Macalintal vs. Presidential Electoral Tribunal landmark case of Civil Liberties Union v. Executive Secretary,194 SCRA 317
2004. His failure to raise a seasonable constitutional challenge at that time, (1991): It is a well-established rule in constitutional construction that no one
coupled with his unconditional acceptance of the Tribunal’s authority over the case provision of the Constitution is to be separated from all the others, to be considered
he was defending, translates to the clear absence of an indispensable requisite for alone, but that all the provisions bearing upon a particular subject are to be
the proper invocation of this Court’s power of judicial review. Even on this score brought into view and to be so interpreted as to effectuate the great purposes of
alone, the petition ought to be dismissed outright. the instrument. Sections bearing on a particular subject should be considered and
Same; Statutory Construction; Presidential Electoral Tribunal; The Supreme interpreted together as to effectuate the whole purpose of the Constitution and one
Court has original jurisdiction to decide presidential and vice-presidential election section is not to be allowed to defeat another, if by any reasonable construction,
protests while concurrently acting as an independent Electoral Tribunal.— the two can be made to stand together. In other words, the court must harmonize
Petitioner, a prominent election lawyer who has filed several cases before this them, if practicable, and must lean in favor of a construction which will render
Court involving constitutional and election law issues, including, among others, every word operative, rather than one which may make the words idle and
the constitutionality of certain provisions of Republic Act (R.A.) No. 9189 (The nugatory.
Overseas Absentee Voting Act of 2003), cannot claim ignorance of: (1) the Same; Presidential Electoral Tribunal (PET); Powers of the Presidential
invocation of our jurisdiction under Section 4, Article VII of the Constitution; and Electoral Tribunal is plenary; Unmistakable from the foregoing is that the exercise
(2) the unanimous holding thereon. Unquestionably, the overarching of our power to judge presidential and vice-presidential election contests, as well as
framework affirmed in Tecson v. Commission on Elections, 424 SCRA 277 (2004), the rule-making power adjunct thereto, is plenary; it is not as restrictive as
is that the Supreme Court has original jurisdiction to decide presidential and vice- petitioner would interpret it.—Unmistakable from the foregoing is that the exercise
presidential election protests while concurrently acting as an independent of our power to judge presidential and vice-presidenti al election contests, as well
Electoral Tribunal. as the rule-making power adjunct thereto, is plenary; it is not as restrictive as
Same; Words and Phrases; Verba Legis; Verba legis dictates that wherever petitioner would interpret it. In fact, former Chief Justice Hilario G. Davide, Jr.,
possible, the words used in the Constitution must be given their ordinary meaning who proposed the insertion of
except where technical terms are employed, in which case the significance thus 786
attached to them prevails.—Verba legis dictates that wherever possible, the words
786 SUPREME COURT REPORTS ANNOTATED
used in the Constitution must be given their ordinary meaning except where
technical terms are employed, in which case the significance thus attached to them Macalintal vs. Presidential Electoral Tribunal
prevails. This Court, speaking through former Chief Justice Enrique Fernando, the phrase, intended the Supreme Court to exercise exclusive authority to
in J.M. Tuason & Co., Inc. v. Land Tenure Administration, 31 SCRA 413 (1970), promulgate its rules of procedure for that purpose. To this, Justice Regalado
instructs: As the Constitution is not primarily a lawyer’s document, it being forthwith assented and then emphasized that the sole power ought to be without
essential for the rule of law to obtain that it should ever be present in the people’s intervention by the legislative department. Evidently, even the legislature cannot
consciousness, its language as much as possible should be understood in the sense limit the judicial power to resolve presidential and vice-presidential election
they have in common use. What it says according to the text of the provision to be contests and our rule-making power connected thereto.
construed compels acceptance and negates the power of the courts to alter it, based Constitutional Law; Courts; Supreme Court; The conferment of additional
on the postulate that the framers and the people mean what they say. Thus these jurisdiction to the Supreme Court, with the duty characterized as an “awesome”
are cases where the need for construction is reduced to a minimum.785 task, includes the means necessary to carry it into effect under the doctrine of
VOL. 635, NOVEMBER 23, 2010 785 necessary implication.—The conferment of additional jurisdiction to the Supreme
Court, with the duty characterized as an “awesome” task, includes the means
Macalintal vs. Presidential Electoral Tribunal necessary to carry it into effect under the doctrine of necessary implication. We
Same; Statutory Construction; Ratio Legis Et Anima; A doubtful provision cannot overemphasize that the abstraction of the PET from the explicit grant of
must be examined in light of the history of the times, and the condition and power to the Supreme Court, given our abundant experience, is not unwarranted.
circumstances surrounding the framing of the Constitution.—Where there is A plain reading of Article VII, Section 4, paragraph 7, readily reveals a grant of
ambiguity or doubt, the words of the Constitution should be interpreted in authority to the Supreme Court sitting en banc. In the same vein, although the
accordance with the intent of its framers or ratio legis et anima. A doubtful method by which the Supreme Court exercises this authority is not specified in the
provision must be examined in light of the history of the times, and the condition provision, the grant of power does not contain any limitation on the Supreme
and circumstances surrounding the framing of the Constitution. In following this Court’s exercise thereof. The Supreme Court’s method of deciding presidential and
guideline, courts should bear in mind the object sought to be accomplished in vice-presidential election contests, through the PET, is actually a derivative of the
adopting a doubtful constitutional provision, and the evils sought to be prevented exercise of the prerogative conferred by the aforequoted constitutional provision.
or remedied. Consequently, the intent of the framers and the people ratifying the Thus, the subsequent directive in the provision for the Supreme Court to
constitution, and not the panderings of self-indulgent men, should be given effect. “promulgate its rules for the purpose.”
Senate Electoral Tribunal; House of Representatives Electoral Tribunal; The PET. Thus, a microscopic view, like the petitioner’s, should not constrict an
different electoral tribunals, with the Supreme Court functioning as the absolute and constitutional grant of judicial power.
Presidential Electoral Tribunal (PET), are constitutional bodies.—Particularly Same; Pleadings and Practice; Baseless Petitions.—One final note. Although
cogent are the discussions of the Constitutional Commission on the parallel this Court has no control over contrary people and naysayers, we reiterate a word
provisions of the SET and the HRET. The discussions point to the inevitable of caution against the filing of baseless petitions which only clog the Court’s docket.
conclusion that the different electoral tribunals, with the Supreme Court The petition in the instant case belongs to that classification.
functioning as the PET, are constitutional bodies, independent of the three PETITION to question the constitution of the Presidential Electoral Tribunal.
departments of government—Executive, Legislative, and Judiciary—but not The facts are stated in the opinion of the Court.
separate therefrom.787 NACHURA, J.:
VOL. 635, NOVEMBER 23, 2010 787 Confronting us is an undesignated petition1 filed by Atty. Romulo B.
Macalintal (Atty. Macalintal), that questions the constitution of the Presidential
Macalintal vs. Presidential Electoral Tribunal Electoral Tribunal (PET) as an illegal and unauthorized progeny of Section
Same; Same; The Presidential Electoral Tribunal is not a separate and 4,2 Article VII of the Constitution:
distinct entity from the Supreme Court, albeit it has functions peculiar only to the “The Supreme Court, sitting en banc, shall be the sole judge of all contests
Tribunal.—By the same token, the PET is not a separate and distinct entity from relating to the election, returns, and qualifications of the President or Vice-
the Supreme Court, albeit it has functions peculiar only to the Tribunal. It is President, and may promulgate its rules for the purpose.”
obvious that the PET was constituted in implementation of Section 4, Article VII While petitioner concedes that the Supreme Court is “authorized to promulgate
of the Constitution, and it faithfully complies—not unlawfully defies—the its rules for the purpose,” he chafes at the creation of a purportedly “separate
constitutional directive. The adoption of a separate seal, as well as the change in tribunal” complemented by a budget allocation, a seal, a set of personnel and
the nomenclature of the Chief Justice and the Associate Justices into Chairman confidential employees, to effect the constitutional mandate. Petitioner’s averment
and Members of the Tribunal, respectively, was designed simply to highlight the is supposedly supported by the provisions of the 2005 Rules of the Presidential
singularity and exclusivity of the Tribunal’s functions as a special electoral court. Electoral Tribunal (2005 PET Rules),3 specifically:
Same; Same; Electoral Contests.—The set up embodied in the Constitution
and statutes characterizes the resolution of electoral contests as _______________
essentially an exercise of judicial power.
Same; Same; The power wielded by Presidential Electoral Tribunal (PET) is 1 Rollo, pp. 3-9.
a derivative of the plenary judicial power allocated to courts of law, expressly 2 Paragraph 7.
provided in the Constitution.—With the explicit provision, the present 3 On May 4, 2010, the 2010 Rules of the Presidential Electoral Tribunal (2010
Constitution has allocated to the Supreme Court, in conjunction with latter’s PET Rules) took effect.
exercise of judicial power inherent in all courts, the task of deciding presidential
and vice-presidential election contests, with full authority in the exercise thereof.
789
The power wielded by PET is a derivative of the plenary judicial power allocated
to courts of law, expressly provided in the Constitution. On the whole, the VOL. 635, NOVEMBER 23, 2010 789
Constitution draws a thin, but, nevertheless, distinct line between the PET and Macalintal vs. Presidential Electoral Tribunal
the Supreme Court. (1) Rule 3 which provides for membership of the PET wherein the Chief
Same; Same; The Presidential Electoral Tribunal, as intended by the framers Justice and the Associate Justices are designated as “Chairman and Members,”
of the Constitution, is to be an institution independent, but not separate, from the respectively;
judicial department, i.e., the Supreme Court.—We have previously declared that (2) Rule 8(e) which authorizes the Chairman of the PET to appoint employees
the PET is not simply an agency to which Members of the Court were designated. and confidential employees of every member thereof;
Once again, the PET, as intended by the framers of the Constitution, is to be an (3) Rule 9 which provides for a separate “Administrative Staff of the
institution independent, but not separate, from the judicial department, i.e., the Tribunal” with the appointment of a Clerk and a Deputy Clerk of the Tribunal
Supreme Court. McCulloch v. State of Maryland proclaimed that “[a] power who, at the discretion of the PET, may designate the Clerk of Court (en banc) as
without the means to use it, is a nullity.” The vehicle for the exercise of this power, the Clerk of the Tribunal; and
as intended by the Constitution and specifically mentioned by the Constitutional (4) Rule 11 which provides for a “seal” separate and distinct from the
Commissioners during the discussions on the grant of power to this Court, is the Supreme Court seal.
788 Grudgingly, petitioner throws us a bone by acknowledging that the invoked
788 SUPREME COURT REPORTS ANNOTATED constitutional provision does allow the “appointment of additional personnel.”
Further, petitioner highlights our decision in Buac v. COMELEC4which
Macalintal vs. Presidential Electoral Tribunal
peripherally declared that “contests involving the President and the Vice-
President fall within the exclusive original jurisdiction of the PET, x x x in the Macalintal vs. Presidential Electoral Tribunal
exercise of quasi-judicial power.” On this point, petitioner reiterates that the We winnow the meanderings of petitioner into the singular issue of whether
constitution of the PET, with the designation of the Members of the Court as the constitution of the PET, composed of the Members of this Court, is
Chairman and Members thereof, contravenes Section 12, Article VIII of the unconstitutional, and violates Section 4, Article VII and Section 12, Article VIII of
Constitution, which prohibits the designation of Members of the Supreme Court the Constitution.
and of other courts established by law to any agency performing quasi-judicial or But first, we dispose of the procedural issue of whether petitioner has standing
administrative functions. to file the present petition.
The Office of the Solicitor General (OSG), as directed in our Resolution dated The issue of locus standi is derived from the following requisites of a judicial
April 6, 2010, filed a Comment5 thereon. At the outset, the OSG points out that inquiry:
the petition filed by Atty. Macalintal is unspecified and without statutory basis; 1. There must be an actual case or controversy;
“the 2. The question of constitutionality must be raised by the proper party;
3. The constitutional question must be raised at the earliest possible
_______________ opportunity; and
4. The decision of the constitutional question must be necessary to the
4 465 Phil. 800, 810; 421 SCRA 92, 103 (2004). determination of the case itself.8
5 Rollo, pp. 12-38. On more than one occasion we have characterized a proper party as one who
790 has sustained or is in immediate danger of sustaining an injury as a result of the
790 SUPREME COURT REPORTS ANNOTATED act complained of.9 The dust has long settled on the test laid down in Baker v.
Carr:10 “whether the party has alleged such a personal stake in the outcome of the
Macalintal vs. Presidential Electoral Tribunal
controversy as to assure that concrete adverseness which sharpens the
liberal approach in its preparation x x x is a violation of the well known rules of presentation of issues upon which the court so largely depends for illumination of
practice and pleading in this jurisdiction.” difficult questions.”11 Until and unless such actual or threatened in-
In all, the OSG crystallizes the following issues for resolution of the Court:
I
_______________
WHETHER x x x PETITIONER HAS LOCUS STANDI TO FILE THE INSTANT
PETITION.
II 8 Cruz, Philippine Political Law, 1998 ed., p. 257.
WHETHER x x x THE CREATION OF THE PRESIDENTIAL ELECTORAL 9 Province of North Cotabato v. Government of the Republic of the Philippines
TRIBUNAL IS UNCONSTITUTIONAL FOR BEING A VIOLATION OF Peace Panel on Ancestral Domain, G.R. Nos. 183591, 183752, 183893, 183951, and
PARAGRAPH 7, SECTION 4 OF ARTICLE VII OF THE 1987 CONSTITUTION. 183962, October 14, 2008, 568 SCRA 402, 456.
III 10 369 U.S. 186 (1962).
WHETHER x x x THE DESIGNATION OF MEMBERS OF THE SUPREME 11 Province of Batangas v. Hon. Romulo, 473 Phil. 806; 429 SCRA 736 (2004).
COURT AS MEMBERS OF THE PRESIDENTIAL ELECTORAL TRIBUNAL IS 792
UNCONSTITUTIONAL FOR BEING A VIOLATION OF SECTION 12, ARTICLE 792 SUPREME COURT REPORTS ANNOTATED
VIII OF THE 1987 CONSTITUTION.6 Macalintal vs. Presidential Electoral Tribunal
In his Reply,7 petitioner maintains that: jury is established, the complainant is not clothed with legal personality to raise
1. He has legal standing to file the petition given his averment of the constitutional question.
transcendental importance of the issues raised therein; Our pronouncements in David v. Macapagal-Arroyo12 illuminate:
2. The creation of the PET, a separate tribunal from the Supreme Court, “The difficulty of determining locus standi arises in public suits. Here, the
violates Section 4, Article VII of the Constitution; and plaintiff who asserts a “public right” in assailing an allegedly illegal official action,
3. The PET, being a separate tribunal, exercises quasi-judicial functions does so as a representative of the general public. He may be a person who is
contrary to Section 12, Article VIII of the Constitution. affected no differently from any other person. He could be suing as a “stranger,” or
in the category of a “citizen,” or “taxpayer.” In either case, he has to adequately
_______________ show that he is entitled to seek judicial protection. In other words, he has to make
out a sufficient interest in the vindication of the public order and the securing of
6 Id., at pp. 15-16. relief as a” citizen” or “taxpayer.”
7 Id., at pp. 42-58. xxxx
791 However, to prevent just about any person from seeking judicial interference
VOL. 635, NOVEMBER 23, 2010 791 in any official policy or act with which he disagreed with, and thus hinders the
activities of governmental agencies engaged in public service, the United States Contrary to the well-settled actual and direct injury test, petitioner has simply
Supreme Court laid down the more stringent “direct injury” test in Ex Parte alleged a generalized interest in the outcome of this case, and succeeds only in
Levitt, later reaffirmed in Tileston v. Ullman. The same Court ruled that for a muddling the issues. Paragraph 2 of the petition reads:
private individual to invoke the judicial power to determine the validity of an “2. x x x Since the creation and continued operation of the PET involves the use
executive or legislative action, he must show that he has sustained a direct of public funds and the issue raised herein is of
injury as a result of that action, and it is not sufficient that he has a 794
general interest common to all members of the public. 794 SUPREME COURT REPORTS ANNOTATED
This Court adopted the “direct injury” test in our jurisdiction. In People v.
Vera, it held that the person who impugns the validity of a statute must have “a Macalintal vs. Presidential Electoral Tribunal
personal and substantial interest in the case such that he has sustained, transcendental importance, it is petitioner’s humble submission that, as a citizen,
or will sustain direct injury as a result.” The Vera doctrine was upheld in a a taxpayer and a member of the BAR, he has the legal standing to file this petition.”
litany of cases, such as, Custodio v. President of the Senate, Manila Race Horse But even if his submission is valid, petitioner’s standing is still imperiled by
Trainers’ Association v. De la Fuente, Pascual v. Secretary of Public the white elephant in the petition, i.e., his appearance as counsel for former
Works and Anti-Chinese League of the Philippines v. Felix. President Gloria Macapagal-Arroyo (Macapagal-Arroyo) in the election protest
filed by 2004 presidential candidate Fernando Poe, Jr. before the Presidential
_______________ Electoral Tribunal,13 because judicial inquiry, as mentioned above, requires that
the constitutional question be raised at the earliest possible opportunity. 14 Such
12 G.R. Nos. 171396, 171409, 171485, 171483, 171400, 171489, and 171424, appearance as counsel before the Tribunal, to our mind, would have been the first
May 3, 2006, 489 SCRA 160, 216-221. (Citations omitted.) opportunity to challenge the constitutionality of the Tribunal’s constitution.
Although there are recognized exceptions to this requisite, we find none in this
793
instance. Petitioner is unmistakably estopped from assailing the jurisdiction of the
VOL. 635, NOVEMBER 23, 2010 793 PET before which tribunal he had ubiquitously appeared and had acknowledged
Macalintal vs. Presidential Electoral Tribunal its jurisdiction in 2004. His failure to raise a seasonable constitutional challenge
However, being a mere procedural technicality, the requirement of locus at that time, coupled with his unconditional acceptance of the Tribunal’s authority
standimay be waived by the Court in the exercise of its discretion. This was done over the case he was defending, translates to the clear absence of an indispensable
in the 1949 Emergency Powers Cases, Araneta v. Dinglasan, where the requisite for the proper invocation of this Court’s power of judicial review. Even on
“transcendental importance” of the cases prompted the Court to act liberally. this score alone, the petition ought to be dismissed outright.
Such liberality was neither a rarity nor accidental. In Aquino v. Comelec, this Prior to petitioner’s appearance as counsel for then protestee Macapagal-
Court resolved to pass upon the issues raised due to the “far-reaching Arroyo, we had occasion to affirm the grant of original jurisdiction to this Court as
implications” of the petition notwithstanding its categorical statement that a Presidential Electoral Tribunal in the auspicious case of Tecson v. Commission
petitioner therein had no personality to file the suit. Indeed, there is a chain of on Elections.15 Thus—
cases where this liberal policy has been observed, allowing ordinary citizens,
members of Congress, and civic organizations to prosecute actions involving the _______________
constitutionality or validity of laws, regulations and rulings.
xxxx 13 Poe v. Macapagal-Arroyo, P.E.T. Case No. 002, March 29, 2005, 454 SCRA
By way of summary, the following rules may be culled from the cases decided 142.
by this Court. Taxpayers, voters, concerned citizens, and legislators may be 14 Cruz, Philippine Political Law, 1998 ed., p. 263.
accorded standing to sue, provided that the following requirements are met: 15 G.R. Nos. 161434, 161634, and 161824, March 3, 2004, 424 SCRA 277, 324-
(1) cases involve constitutional issues; 325. (Emphasis supplied.)
(2) for taxpayers, there must be a claim of illegal disbursement of 795
public funds or that the tax measure is unconstitutional; VOL. 635, NOVEMBER 23, 2010 795
(3) for voters, there must be a showing of obvious interest in the
validity of the election law in question; Macalintal vs. Presidential Electoral Tribunal
(4) for concerned citizens, there must be a showing that the issues “Petitioners Tecson, et al., in G.R. No. 161434, and Velez, in G.R. No. 161634,
raised are of transcendental importance which must be settled invoke the provisions of Article VII, Section 4, paragraph 7, of the 1987
early; and Constitution in assailing the jurisdiction of the COMELEC when it took cognizance
(5) for legislators, there must be a claim that the official action of SPA No. 04-003 and in urging the Supreme Court to instead take on the petitions
complained of infringes upon their prerogatives as legislators.” they directly instituted before it. The Constitutional provision cited reads:
“The Supreme Court, sitting en banc, shall be the sole judge of all President, election protest or quo warranto may be filed after the proclamation of
contests relating to the election, returns, and qualifications of the President the winner.”17
or Vice-President, and may promulgate its rules for the purpose.” Petitioner, a prominent election lawyer who has filed several cases before this
The provision is an innovation of the 1987 Constitution. The omission in the Court involving constitutional and election law issues, including, among others,
1935 and the 1973 Constitution to designate any tribunal to be the sole judge of the constitutionality of certain provisions of Republic Act (R.A.) No. 9189 (The
presidential and vice-presidential contests, has constrained this Court to declare, Overseas Absentee Voting Act of 2003),18 cannot claim igno-
in Lopez vs. Roxas, as “not (being) justiciable” controversies or disputes involving
contests on the elections, returns and qualifications of the President or Vice- _______________
President. The constitutional lapse prompted Congress, on 21 June 1957, to enact
Republic Act No. 1793, “An Act Constituting an Independent Presidential Electoral 16 Id., at p. 363.
Tribunal to Try, Hear and Decide Protests Contesting the Election of the President- 17 Id., at pp. 431-432.
Elect and the Vice-President-Elect of the Philippines and Providing for the Manner 18 Atty. Macalintal v. Commission on Elections, 453 Phil. 586; 405 SCRA 614
of Hearing the Same.” Republic Act 1793 designated the Chief Justice and the (2003).
Associate Justices of the Supreme Court to be the members of the 797
tribunal. Although the subsequent adoption of the parliamentary form of
VOL. 635, NOVEMBER 23, 2010 797
government under the 1973 Constitution might have implicitly affected
Republic Act No. 1793, the statutory set-up, nonetheless, would now be Macalintal vs. Presidential Electoral Tribunal
deemed revived under the present Section 4, paragraph 7, of the 1987 rance of: (1) the invocation of our jurisdiction under Section 4, Article VII of the
Constitution.” Constitution; and (2) the unanimous holding thereon. Unquestionably,
Former Chief Justice Reynato S. Puno, in his separate opinion, was even more the overarching framework affirmed in Tecson v. Commission on Elections19 is
categorical: that the Supreme Court has original jurisdiction to decide presidential and vice-
“The Court is unanimous on the issue of jurisdiction. It has no jurisdiction on presidential election protests while concurrently acting as an independent
the Tecson and Valdez petitions. Petitioners cannot invoke Article VII, Section 4, Electoral Tribunal.
par. 7 of the Constitution which provides: Despite the foregoing, petitioner is adamant on his contention that the
“The Supreme Court, sitting en banc shall be the sole judge of all provision, as worded, does not authorize the constitution of the PET. And although
contests relating to the election, returns and quali- he concedes that the Supreme Court may promulgate its rules for this purpose,
796 petitioner is insistent that the constitution of the PET is unconstitutional.
796 SUPREME COURT REPORTS ANNOTATED However, petitioner avers that it allows the Court to appoint additional personnel
for the purpose, notwithstanding the silence of the constitutional provision.
Macalintal vs. Presidential Electoral Tribunal Petitioner’s pastiche arguments are all hurled at the Court, hopeful that at
fications of the President or Vice President and may promulgate its rules least one might possibly stick. But these arguments fail to elucidate on the scope
for the purpose.” of the rules the Supreme Court is allowed to promulgate. Apparently, petitioner’s
The word “contest” in the provision means that the jurisdiction of this Court can concept of this adjunct of judicial power is very restrictive. Fortunately, thanks in
only be invoked after the election and proclamation of a President or Vice no part to petitioner’s opinion, we are guided by well-settled principles of
President. There can be no “contest” before a winner is proclaimed.”16 constitutional construction.
Similarly, in her separate opinion, Justice Alicia Austria-Martinez declared: Verba legis dictates that wherever possible, the words used in the Constitution
“G.R. Nos. 161434 and 161634 invoke the Court’s exclusive jurisdiction under must be given their ordinary meaning except where technical terms are employed,
the last paragraph of Section 4, Article VII of the 1987 Constitution. I agree with in which case the significance thus attached to them prevails. This Court, speaking
the majority opinion that these petitions should be dismissed outright for through former Chief Justice Enrique Fernando, in J.M. Tuason & Co., Inc. v.
prematurity. The Court has no jurisdiction at this point of time to entertain said Land Tenure Administration20 instructs:
petitions. “As the Constitution is not primarily a lawyer’s document, it being essential for
The Supreme Court, as a Presidential Electoral Tribunal (PET), the Senate the rule of law to obtain that it should ever be present
Electoral Tribunal (SET) and House of Representatives Electoral Tribunal (HRET)
are electoral tribunals, each specifically and exclusively clothed with jurisdiction _______________
by the Constitution to act respectively as “sole judge of all contests relating to the
election, returns, and qualifications” of the President and Vice-President,
19 Supra at note 15.
Senators, and Representatives. In a litany of cases, this Court has long recognized
20 No. L-21064, February 18, 1970, 31 SCRA 413, 423.
that these electoral tribunals exercise jurisdiction over election contests only after
798
a candidate has already been proclaimed winner in an election. Rules 14 and 15 of
the Rules of the Presidential Electoral Tribunal provide that, for President or Vice- 798 SUPREME COURT REPORTS ANNOTATED
Macalintal vs. Presidential Electoral Tribunal On its face, the contentious constitutional provision does not specify the
in the people’s consciousness, its language as much as possible should be establishment of the PET. But neither does it preclude, much less prohibit,
understood in the sense they have in common use. What it says according to the otherwise. It entertains divergent interpretations which, though unacceptable to
text of the provision to be construed compels acceptance and negates the power of petitioner, do not include his restrictive view—one which really does not offer a
the courts to alter it, based on the postulate that the framers and the people mean solution.
what they say. Thus these are cases where the need for construction is reduced to Section 4, Article VII of the Constitution, the provision under scrutiny, should
a minimum.” be read with other related provisions of the Constitution such as the parallel
However, where there is ambiguity or doubt, the words of the Constitution provisions on the Electoral Tribunals of the Senate and the House of
should be interpreted in accordance with the intent of its framers or ratio legis et Representatives.
anima. A doubtful provision must be examined in light of the history of the times, Before we resort to the records of the Constitutional Commission, we discuss
and the condition and circumstances surrounding the framing of the the framework of judicial power mapped out in the Constitution. Contrary to
Constitution.21 In following this guideline, courts should bear in mind the object petitioner’s assertion, the Supreme Court’s constitutional mandate to act as sole
sought to be accomplished in adopting a doubtful constitutional provision, and the judge of election contests involving our country’s highest public officials, and its
evils sought to be prevented or remedied.22Consequently, the intent of the framers rule-making authority in connection therewith, is not restricted; it includes all
and the people ratifying the constitution, and not the panderings of self-indulgent necessary powers implicit in the exercise thereof.
men, should be given effect.
Last, ut magis valeat quam pereat—the Constitution is to be interpreted as a _______________
whole. We intoned thus in the landmark case of Civil Liberties Union v. Executive
Secretary:23 24 82 Phil. 771, 775 (1949).
“It is a well-established rule in constitutional construction that no one 800
provision of the Constitution is to be separated from all the others, to be considered 800 SUPREME COURT REPORTS ANNOTATED
alone, but that all the provisions bearing upon a particular subject are to be
Macalintal vs. Presidential Electoral Tribunal
brought into view and to be so interpreted as to effectuate the great purposes of
the instrument. Sections bearing on a particular subject should be considered and We recall the unprecedented and trailblazing case of Marcos v. Manglapus:25
interpreted together as to effectuate the whole purpose of the Constitution and one “The 1987 Constitution has fully restored the separation of powers of the three
section is not to be allowed to defeat another, if by any reasonable construction, great branches of government. To recall the words of Justice Laurel in Angara v.
the two can be made to stand together. Electoral Commission, “the Constitution has blocked but with deft strokes and in
bold lines, allotment of power to the executive, the legislative and the judicial
departments of the government.” Thus, the 1987 Constitution explicitly provides
_______________
that “[t]he legislative power shall be vested in the Congress of the Philippines”
[Art. VI, Sec. 1], “[t]he executive power shall be vested in the President of the
21 McCulloch v. State of Maryland, 17 U.S. 316 (Wheat.), 1819. Philippines” [Art. VII, Sec. 1], and “[t]he judicial power shall be vested in one
22 In the Philippine context, see Civil Liberties Union v. Executive Secretary, Supreme Court and in such lower courts as may be established by law” [Art. VIII,
G.R. Nos. 83896 and 83815, February 22, 1991, 194 SCRA 317. Sec. 1]. These provisions not only establish a separation of powers by actual
23 Id., at 330-331. division but also confer plenary legislative, executive and judicial powers subject
799 only to limitations provided in the Constitution. For as the Supreme Court
VOL. 635, NOVEMBER 23, 2010 799 in Ocampo v. Cabangis pointed out “a grant of the legislative power
Macalintal vs. Presidential Electoral Tribunal means a grant of all legislative power; and a grant of the judicial power
In other words, the court must harmonize them, if practicable, and must lean means a grant of all the judicial power which may be exercised under the
in favor of a construction which will render every word operative, rather than one government.”
which may make the words idle and nugatory.” The Court could not have been more explicit then on the plenary grant and
We had earlier expounded on this rule of construction in Chiongbian v. De exercise of judicial power. Plainly, the abstraction of the Supreme Court acting as
Leon, et al.,24 to wit: a Presidential Electoral Tribunal from the unequivocal grant of jurisdiction in the
“[T]he members of the Constitutional Convention could not have dedicated a last paragraph of Section 4, Article VII of the Constitution is sound and tenable.
provision of our Constitution merely for the benefit of one person without The mirabile dictu of the grant of jurisdiction to this Court, albeit found in the
considering that it could also affect others. When they adopted subsection 2, they Article on the executive branch of government, and the constitution of the PET, is
permitted, if not willed, that said provision should function to the full extent of its evident in the discussions of the Constitutional Commission. On the exercise of
substance and its terms, not by itself alone, but in conjunction with all other this Court’s judicial power as sole judge of presidential and vice-presidential
provisions of that great document.” election contests, and to promulgate its rules
_______________ Would the Commissioner not consider that violative of the doctrine of
separation of powers?
25 G.R. No. 88211, September 15, 1989, 177 SCRA 668, 688-689. (Emphasis MR. CONCEPCION. I think Commissioner Bernas explained that this
supplied, citations omitted.) is a contest between two parties. This is a judicial power.
801 MR. SUAREZ. We know, but practically the Committee is giving to the
judiciary the right to declare who will be the President of our country, which to me
VOL. 635, NOVEMBER 23, 2010 801
is a political action.
Macalintal vs. Presidential Electoral Tribunal MR. CONCEPCION. There are legal rights which are enforceable
for this purpose, we find the proceedings in the Constitutional Commission under the law, and these are essentially justiciable questions.
most instructive: MR. SUAREZ. If the election contest proved to be long, burdensome
MR. DAVIDE. On line 25, after the words “Vice-President,” I propose to add and tedious, practically all the time of the Supreme Court sitting en
AND MAY PROMULGATE ITS RULES FOR THE PURPOSE. This refers to the banc would be occupied with it considering that they will be going over
Supreme Court sitting en banc. This is also to confer on the Supreme Court millions and millions of ballots or election returns, Madam President.28
exclusive authority to enact the necessary rules while acting as sole judge Echoing the same sentiment and affirming the grant of judicial power to the
of all contests relating to the election, returns and qualifications of the Supreme Court, Justice Florenz D. Regalado29 and Fr. Joaquin Bernas30 both
President or Vice-President. opined:
MR. REGALADO. My personal position is that the rule-making power MR. VILLACORTA. Thank you very much, Madam President.
of the Supreme Court with respect to its internal procedure is already I am not sure whether Commissioner Suarez has expressed his point. On page
implicit under the Article on the Judiciary; considering, however, that 2, the fourth paragraph of Section 4 provides:
according to the Commissioner, the purpose of this is to indicate the sole The Supreme Court, sitting en banc, shall be the sole judge of all contests
power of the Supreme Court without intervention by the legislature in relating to the election, returns and qualifications of the President or Vice-
the promulgation of its rules on this particular point, I think I will President.
personally recommend its acceptance to the Committee.26
xxxx _______________
MR. NOLLEDO. x x x.
With respect to Sections 10 and 11 on page 8, I understand that the Committee 28 Id., at pp. 420-421. (Emphasis supplied.)
has also created an Electoral Tribunal in the Senate and a Commission on 29 Supreme Court.
Appointments which may cover membership from both Houses. But my question 30 A Roman Catholic Priest of the Jesuit Order.
is: It seems to me that the committee report does not indicate which body should 803
promulgate the rules that shall govern the Electoral Tribunal and the Commission
on Appointments. Who shall then promulgate the rules of these bodies? VOL. 635, NOVEMBER 23, 2010 803
MR. DAVIDE. The Electoral Tribunal itself will establish and Macalintal vs. Presidential Electoral Tribunal
promulgate its rules because it is a body distinct and independent May I seek clarification as to whether or not the matter of determining
already from the House, and so with the Commission on Appointments the outcome of the contests relating to the election returns and
also. It will have the authority to promulgate its own rules.27 qualifications of the President or Vice-President is purely a political
matter and, therefore, should not be left entirely to the judiciary. Will the
_______________ above-quoted provision not impinge on the doctrine of separation of
powers between the executive and the judicial departments of the
26 Records of the Constitutional Commission, Vol. 2, p. 433. (Emphasis government?
supplied.) MR. REGALADO. No, I really do not feel that would be a problem. This
27 Id., at pp. 87-88. (Emphasis supplied.) is a new provision incidentally. It was not in the 1935 Constitution nor in
802 the 1973 Constitution.
802 SUPREME COURT REPORTS ANNOTATED MR. VILLACORTA. That is right.
MR. REGALADO. We feel that it will not be an intrusion into the
Macalintal vs. Presidential Electoral Tribunal separation of powers guaranteed to the judiciary because this is strictly
On another point of discussion relative to the grant of judicial power, but an adversarial and judicial proceeding.
equally cogent, we listen to former Chief Justice Roberto Concepcion: MR. VILLACORTA. May I know the rationale of the Committee because this
MR. SUAREZ. Thank you. supersedes Republic Act 7950 which provides for the Presidential Electoral
Tribunal?
FR. BERNAS. Precisely, this is necessary. Election contests are, by 32 See Defensor-Santiago v. Ramos, P.E.T. Case No. 001, February 13, 1996,
their nature, judicial. Therefore, they are cognizable only by courts. If, 253 SCRA 559; Tecson v. Commission on Elections, supra at note 15.
for instance, we did not have a constitutional provision on an electoral 805
tribunal for the Senate or an electoral tribunal for the House, normally, VOL. 635, NOVEMBER 23, 2010 805
as composed, that cannot be given jurisdiction over contests.
So, the background of this is really the case of Roxas v. Lopez. The Gentleman Macalintal vs. Presidential Electoral Tribunal
will remember that in that election, Lopez was declared winner. He filed a protest elections—from the canvassing of election returns, to the proclamation of the
before the Supreme Court because there was a republic act which created the president-elect and the vice-president elect, and even the determination, by
Supreme Court as the Presidential Electoral Tribunal. The question in this case ordinary legislation, of whether such proclamations may be contested. Unless the
was whether new powers could be given the Supreme Court by law. In effect, the legislature enacted a law creating an institution that would hear election contests
conflict was actually whether there was an attempt to create two Supreme Courts in the Presidential and Vice-Presidential race, a defeated candidate had no legal
and the answer of the Supreme Court was: “No, this did not involve the creation of right to demand a recount of the votes cast for the office involved or to challenge
two Supreme Courts, but precisely we are giving new jurisdiction to the Supreme the ineligibility of the proclaimed candidate. Effectively, presidential and vice-
Court, as it is allowed by the Constitution. Congress may allocate various presidential contests were non-justiciable in the then prevailing milieu.
jurisdictions.”804 The omission in the 1935 Constitution was intentional. It was mainly
influenced by the absence of a similar provision in its pattern, the Federal
804 SUPREME COURT REPORTS ANNOTATED
Constitution of the United States. Rather, the creation of such tribunal was left to
Macalintal vs. Presidential Electoral Tribunal the determination of the National Assembly. The journal of the 1935
Before the passage of that republic act, in case there was any contest between Constitutional Convention is crystal clear on this point:
two presidential candidates or two vice-presidential candidates, no one had Delegate Saguin. – For an information. It seems that this Constitution does
jurisdiction over it. So, it became necessary to create a Presidential not contain any provision with respect to the entity or body which will look into
Electoral Tribunal. What we have done is to constitutionalize what was the protests for the positions of the President and Vice-President.
statutory but it is not an infringement on the separation of powers President Recto. – Neither does the American constitution contain a provision
because the power being given to the Supreme Court is a judicial power.31 over the subject.
Unmistakable from the foregoing is that the exercise of our power to judge Delegate Saguin. – But then, who will decide these protests?
presidential and vice-presidential election contests, as well as the rule-making President Recto. – I suppose that the National Assembly will decide on that.33
power adjunct thereto, is plenary; it is not as restrictive as petitioner would To fill the void in the 1935 Constitution, the National Assembly enacted R.A.
interpret it. In fact, former Chief Justice Hilario G. Davide, Jr., who proposed the No. 1793, establishing an independent PET to try, hear, and decide protests
insertion of the phrase, intended the Supreme Court to exercise exclusive contesting the election of President and Vice-President. The Chief Justice and the
authority to promulgate its rules of procedure for that purpose. To this, Justice Associate Justices of the Supreme Court were tasked to sit as its Chairman and
Regalado forthwith assented and then emphasized that the sole power ought to be Members, respectively. Its composition was extended to retired Supreme Court
without intervention by the legislative department. Evidently, even the legislature Justices and incumbent
cannot limit the judicial power to resolve presidential and vice-presidential
election contests and our rule-making power connected thereto. _______________
To foreclose all arguments of petitioner, we reiterate that the establishment of
the PET simply constitutionalized what was statutory before the 1987 33 Constitutional Convention Record, Vol. X, pp. 471-472.
Constitution. The experiential context of the PET in our country cannot be 806
denied.32
806 SUPREME COURT REPORTS ANNOTATED
Consequently, we find it imperative to trace the historical antecedents of the
PET. Macalintal vs. Presidential Electoral Tribunal
Article VII, Section 4, paragraph 7 of the 1987 Constitution is an innovation. Court of Appeals Justices who may be appointed as substitutes for ill, absent, or
The precursors of the present Constitution did not contain similar provisions and temporarily incapacitated regular members.
instead vested upon the legislature all phases of presidential and vice-presidential The eleven-member tribunal was empowered to promulgate rules for the
conduct of its proceedings. It was mandated to sit en banc in deciding presidential
_______________ and vice-presidential contests and authorized to exercise powers similar to those
conferred upon courts of justice, including the issuance of subpoena, taking of
31 Records of the Constitutional Commission, Vol. 2, pp. 407-408. (Emphasis depositions, arrest of witnesses to compel their appearance, production of
supplied.) documents and other evidence, and the power to punish contemptuous acts and
bearings. The tribunal was assigned a Clerk, subordinate officers, and employees
necessary for the efficient performance of its functions.
R.A. No. 1793 was implicitly repealed and superseded by the 1973 Constitution 34 Records of the Constitutional Commission, Vol. 2, p. 408.
which replaced the bicameral legislature under the 1935 Constitution with the 808
unicameral body of a parliamentary government. 808 SUPREME COURT REPORTS ANNOTATED
With the 1973 Constitution, a PET was rendered irrelevant, considering that
the President was not directly chosen by the people but elected from among the Macalintal vs. Presidential Electoral Tribunal
members of the National Assembly, while the position of Vice-President was Clearly, petitioner’s bete noire of the PET and the exercise of its power are
constitutionally non-existent. unwarranted. His arguments that: (1) the Chief Justice and Associate Justices are
In 1981, several modifications were introduced to the parliamentary system. referred to as “Chairman” and “Members,” respectively; (2) the PET uses a
Executive power was restored to the President who was elected directly by the different seal; (3) the Chairman is authorized to appoint personnel; and (4)
people. An Executive Committee was formed to assist the President in the additional compensation is allocated to the “Members,” in order to bolster his claim
performance of his functions and duties. Eventually, the Executive Committee was of infirmity in the establishment of the PET, are too superficial to merit further
abolished and the Office of Vice-President was installed anew. attention by the Court.
These changes prompted the National Assembly to revive the PET by enacting, Be that as it may, we hasten to clarify the structure of the PET as a legitimate
on December 3, 1985, Batas Pambansa Bilang (B.P. Blg.) 884, entitled “An Act progeny of Section 4, Article VII of the Constitution, composed of members of the
Constituting an Independent Presidential Electoral Tribunal to Try, Hear and Supreme Court, sitting en banc. The following exchange in the 1986 Constitutional
Decide Election Contests in the Office of the President and Commission should provide enlightenment:
807 MR. SUAREZ. Thank you. Let me proceed to line 23, page 2, wherein it is
provided, and I quote:
VOL. 635, NOVEMBER 23, 2010 807
The Supreme Court, sitting en banc[,] shall be the sole judge of all contests
Macalintal vs. Presidential Electoral Tribunal relating to the election, returns and qualifications of the President or Vice-
Vice-President of the Philippines, Appropriating Funds Therefor and For Other President.
Purposes.” This tribunal was composed of nine members, three of whom were the Are we not giving enormous work to the Supreme Court especially
Chief Justice of the Supreme Court and two Associate Justices designated by him, when it is directed to sit en banc as the sole judge of all presidential and
while the six were divided equally between representatives of the majority and vice-presidential election contests?
minority parties in the Batasang Pambansa. MR. SUMULONG. That question will be referred to Commissioner
Aside from the license to wield powers akin to those of a court of justice, the Concepcion.
PET was permitted to recommend the prosecution of persons, whether public MR. CONCEPCION. This function was discharged by the Supreme
officers or private individuals, who in its opinion had participated in any Court twice and the Supreme Court was able to dispose of each case in a
irregularity connected with the canvassing and/or accomplishing of election period of one year as provided by law. Of course, that was probably
returns. during the late 1960s and early 1970s. I do not know how the present
The independence of the tribunal was highlighted by a provision allocating a Supreme Court would react to such circumstances, but there is also the
specific budget from the national treasury or Special Activities Fund for its question of who else would hear the election protests.
operational expenses. It was empowered to appoint its own clerk in accordance MR. SUAREZ. We are asking this question because between lines 23 to 25,
with its rules. However, the subordinate officers were strictly employees of the there are no rules provided for the hearings and there is not time limit or duration
judiciary or other officers of the government who were merely designated to the for the election contest to be decided by the Supreme Court. Also, we will have to
tribunal. consider the historical background that when R.A. 1793, which organized the
After the historic People Power Revolution that ended the martial law era and Presidential
installed Corazon Aquino as President, civil liberties were restored and a new 809
constitution was formed. VOL. 635, NOVEMBER 23, 2010 809
With R.A. No. 1793 as framework, the 1986 Constitutional Commission
transformed the then statutory PET into a constitutional institution, albeit Macalintal vs. Presidential Electoral Tribunal
without its traditional nomenclature: Electoral Tribunal, was promulgated on June 21, 1957, at least three famous
“FR. BERNAS.x x x. election contests were presented and two of them ended up in withdrawal by the
x x x. So it became necessary to create a Presidential Electoral Tribunal. What protestants out of sheer frustration because of the delay in the resolution of the
we have done is to constitutionalize what was statutory but it is not an cases. I am referring to the electoral protest that was lodged by former President
infringement on the separation of powers because the power being given to the Carlos P. Garcia against our “kabalen” former President Diosdado Macapagal in
Supreme Court is a judicial power.”34 1961 and the vice-presidential election contest filed by the late Senator Gerardo
Roxas against Vice-President Fernando Lopez in 1965.
_______________ MR. CONCEPCION. I cannot answer for what the protestants had in mind.
But when that protest of Senator Roxas was withdrawn, the results were already
available. Senator Roxas did not want to have a decision adverse to him. The votes Obvious from the foregoing is the intent to bestow independence to the
were being counted already, and he did not get what he expected so rather than Supreme Court as the PET, to undertake the Herculean task of deciding election
have a decision adverse to his protest, he withdrew the case. protests involving presidential and vice-presidential candidates in accordance with
xxxx
MR. SUAREZ. I see. So the Commission would not have any objection _______________
to vesting in the Supreme Court this matter of resolving presidential and
vice-presidential contests? 35 Id., at pp. 420-421. (Emphasis supplied.)
MR. CONCEPCION. Personally, I would not have any objection. 811
MR. SUAREZ. Thank you.
VOL. 635, NOVEMBER 23, 2010 811
Would the Commissioner not consider that violative of the doctrine of
separation of powers? Macalintal vs. Presidential Electoral Tribunal
MR. CONCEPCION. I think Commissioner Bernas explained that this is a the process outlined by former Chief Justice Roberto Concepcion. It was made in
contest between two parties. This is a judicial power. response to the concern aired by delegate Jose E. Suarez that the additional duty
MR. SUAREZ. We know, but practically the Committee is giving to the may prove too burdensome for the Supreme Court. This explicit grant of
judiciary the right to declare who will be the President of our country, which to me independence and of the plenary powers needed to discharge this burden justifies
is a political action. the budget allocation of the PET.
MR. CONCEPCION. There are legal rights which are enforceable under the The conferment of additional jurisdiction to the Supreme Court, with the duty
law, and these are essentially justiciable questions. characterized as an “awesome” task, includes the means necessary to carry it into
MR. SUAREZ. If the election contest proved to be long, burdensome effect under the doctrine of necessary implication.36 We cannot overemphasize that
and tedious, practically all the time of the Supreme Court sitting en the abstraction of the PET from the explicit grant of power to the Supreme Court,
banc would be occupied with it considering that they will be going over given our abundant experience, is not unwarranted.
millions and millions of ballots or election returns, Madam President.810 A plain reading of Article VII, Section 4, paragraph 7, readily reveals a grant
810 SUPREME COURT REPORTS ANNOTATED of authority to the Supreme Court sitting en banc. In the same vein, although the
method by which the Supreme Court exercises this authority is not specified in the
Macalintal vs. Presidential Electoral Tribunal provision, the grant of power does not contain any limitation on the Supreme
MR. CONCEPCION. The time consumed or to be consumed in this contest Court’s exercise thereof. The Supreme Court’s method of deciding presidential and
for President is dependent upon they key number of teams of revisors. I have no vice-presidential election contests, through the PET, is actually a derivative of the
experience insofar as contests in other offices are concerned. exercise of the prerogative conferred by the aforequoted constitutional provision.
MR. SUAREZ. Although there is a requirement here that the Supreme Court Thus, the subsequent directive in the provision for the Supreme Court to
is mandated to sit en banc? “promulgate its rules for the purpose.”
MR. CONCEPCION. Yes. The conferment of full authority to the Supreme Court, as a PET, is equivalent
MR. SUAREZ. I see. to the full authority conferred upon the electoral tribunals of the Senate and the
MR. CONCEPCION. The steps involved in this contest are: First, the House of Representatives, i.e., the Senate Electoral Tribunal (SET) and the House
ballot boxes are opened before teams of three, generally, a representative of Representatives Electoral Tribunal (HRET),37 which we have affirmed on
each of the court, of the protestant and of the “protestee.” It is all a numerous occasions.38
questions of how many teams are organized. Of course, that can be
expensive, but it would be expensive whatever court one would choose. _______________
There were times that the Supreme Court, with sometimes 50 teams at
the same time working, would classify the objections, the kind of
36 McCulloch v. State of Maryland, supra note 21.
problems, and the court would only go over the objected votes on which
37 CONSTITUTION, Art. VI, Sec. 17.
the parties could not agree. So it is not as awesome as it would appear
38 Sen. Defensor-Santiago v. Sen. Guingona, Jr., 359 Phil. 276, 294; 298 SCRA
insofar as the Court is concerned. What is awesome is the cost of the
756 (1998), citing Lazatin v. House Electoral Tribu-
revision of the ballots because each party would have to appoint one
812
representative for every team, and that may take quite a big amount.
MR. SUAREZ. If we draw from the Commissioner’s experience which he is 812 SUPREME COURT REPORTS ANNOTATED
sharing with us, what would be the reasonable period for the election contest to be Macalintal vs. Presidential Electoral Tribunal
decided? Particularly cogent are the discussions of the Constitutional Commission on
MR. CONCEPCION. Insofar as the Supreme Court is concerned, the the parallel provisions of the SET and the HRET. The discussions point to the
Supreme Court always manages to dispose of the case in one year. inevitable conclusion that the different electoral tribunals, with the Supreme
MR. SUAREZ. In one year. Thank you for the clarification.35
Court functioning as the PET, are constitutional bodies, independent of the three said legislation has conferred upon such Court an additional original jurisdiction
departments of government—Executive, Legislative, and Judiciary—but not of an exclusive character.
separate therefrom. Republic Act No. 1793 has not created a new or separate court. It has merely
MR. MAAMBONG. x x x. conferred upon the Supreme Court the functions of a Presidential Electoral
My questions will be very basic so we can go as fast as we can. In the case of Tribunal. The result of the enactment may be likened to the fact that courts of first
the electoral tribunal, either of the House or of the Senate, is it correct to say that instance perform the functions of such ordinary courts of first instance, those of
these tribunals are constitutional creations? I will distinguish these with the case court of land registration, those of probate courts, and those of courts of juvenile
of the Tanodbayan and the Sandiganbayan which are created by mandate of the and domestic relations. It is, also, comparable to the situation obtaining when the
Constitution but they are not constitutional creations. Is that a good distinction? municipal court of a provincial capital exercises its author-
xxxx
MR. MAAMBONG. Could we, therefore, say that either the Senate Electoral _______________
Tribunal or the House Electoral Tribunal is a constitutional body?
MR. AZCUNA. It is, Madam President. 39 Records of the Constitutional Commission, Vol. 2, pp. 111-112. (Emphasis
MR. MAAMBONG. If it is a constitutional body, is it then subject to supplied.)
constitutional restrictions? 40 Supreme Court.
MR. AZCUNA. It would be subject to constitutional restrictions intended for 41 Court of Appeals.
that body. 42 No. L-25716, July 28, 1966, 17 SCRA 756, 762-765. (Emphasis supplied.)
MR. MAAMBONG. I see. But I want to find out if the ruling in the case
of Vera v. Avelino, 77 Phil. 192, will still be applicable to the present bodies we are 814
creating since it ruled that the electoral tribunals are not separate departments of
814 SUPREME COURT REPORTS ANNOTATED
the government. Would that ruling still be valid?
MR. AZCUNA. Yes, they are not separate departments because the Macalintal vs. Presidential Electoral Tribunal
separate departments are the legislative, the ity, pursuant to law, over a limited number of cases which were previously within
the exclusive jurisdiction of courts of first instance.
_______________ In all of these instances, the court (court of first instance or municipal
court) is only one, although the functions may be distinct and,
nal, 250 Phil. 390; 168 SCRA 391 (1988); Robles v. House of Representatives even, separate. Thus the powers of a court of first instance, in the exercise of its
Electoral Tribunal, G.R. No. 86647, February 5, 1990, 181 SCRA 780. jurisdiction over ordinary civil cases, are broader than, as well as distinct and
separate from, those of the same court acting as a court of land registration or
813 a probate court, or as a court of juvenile and domestic relations. So too, the
authority of the municipal court of a provincial capital, when acting as such
VOL. 635, NOVEMBER 23, 2010 813 municipal court, is, territorially more limited than that of the same court when
Macalintal vs. Presidential Electoral Tribunal hearing the aforementioned cases which are primary within the jurisdiction of
executive and the judiciary; but they are constitutional bodies. 39 courts of first instance. In other words, there is only one court, although it may
The view taken by Justices Adolfo S. Azcuna40 and Regalado E. Maambong41 is perform the functions pertaining to several types of courts, each having some
schooled by our holding in Lopez v. Roxas, et al.:42 characteristics different from those of the others.
Section 1 of Republic Act No. 1793, which provides that: Indeed, the Supreme Court, the Court of Appeals and courts of first instance,
“There shall be an independent Presidential Electoral Tribunal x x x are vested with original jurisdiction, as well as with appellate jurisdiction, in
which shall be the sole judge of all contests relating to the election, returns, consequence of which they are both trial courts and, appellate courts, without
and qualifications of the president-elect and the vice-president-elect of the detracting from the fact that there is only oneSupreme Court, one Court of
Philippines.” Appeals, and one court of first instance, clothed with authority to discharge said
has the effect of giving said defeated candidate the legal right to contest judicially dual functions. A court of first instance, when performing the functions of a probate
the election of the President-elect of Vice-President-elect and to demand a recount court or a court of land registration, or a court of juvenile and domestic relations,
of the votes case for the office involved in the litigation, as well as to secure a although with powers less broad than those of a court of first instance, hearing
judgment declaring that he is the one elected president or vice-president, as the ordinary actions, is not inferior to the latter, for one cannot be inferior to itself. So
case may be, and that, as such, he is entitled to assume the duties attached to said too, the Presidential Electoral Tribunal is not inferior to the Supreme Court, since
office. And by providing, further, that the Presidential Electoral Tribunal “shall be it is the same Court although the functions peculiar to said Tribunal
composed of the Chief Justice and the other ten Members of the Supreme Court,” are more limited in scope than those of the Supreme Court in the exercise of its
ordinary functions. Hence, the enactment of Republic Act No. 1793, does not entail
an assumption by Congress of the power of appointment vested by the Constitution The set up embodied in the Constitution and statutes characterizes the
in the President. It merely connotes the imposition of additional duties upon the resolution of electoral contests as essentially an exercise of judicial
Members of the Supreme Court.” power.
By the same token, the PET is not a separate and distinct entity from the At the barangay and municipal levels, original and exclusive jurisdiction over
Supreme Court, albeit it has functions peculiar only to the Tribunal. It is obvious election contests is vested in the municipal or metropolitan trial courts and the
that the PET was constituted in implementation of Section 4, Article VII of the regional trial courts, respectively.
815 At the higher levels—city, provincial, and regional, as well as congressional
VOL. 635, NOVEMBER 23, 2010 815 and senatorial—exclusive and original jurisdiction is lodged in the COMELEC and
in the House of Representatives and Senate Electoral Tribunals, which are not,
Macalintal vs. Presidential Electoral Tribunal strictly and literally speaking, courts of law. Although not courts of law, they
Constitution, and it faithfully complies—not unlawfully defies—the constitutional are, nonetheless, empowered to resolve election contests which involve, in essence,
directive. The adoption of a separate seal, as well as the change in the an exercise of judicial power, because of the explicit constitutional empowerment
nomenclature of the Chief Justice and the Associate Justices into Chairman and found in Section 2(2), Article IX-C (for the COMELEC) and Section 17, Article VI
Members of the Tribunal, respectively, was designed simply to highlight the (for the Senate and House Electoral Tribunals) of the Constitution. Besides, when
singularity and exclusivity of the Tribunal’s functions as a special electoral court. the COMELEC, the HRET, and the SET decide election contests, their decisions
As regards petitioner’s claim that the PET exercises quasi-judicial functions in are still subject to judicial review—via a petition for certiorari filed by the proper
contravention of Section 12, Article VIII of the Constitution, we point out that the party—if there is a showing that the decision was rendered with grave abuse of
issue in Buac v. COMELEC43involved the characterization of the enforcement and discretion tantamount to lack or excess of jurisdiction.46
administration of a law relative to the conduct of a plebiscite which falls under the
jurisdiction of the Commission on Elections. However, petitioner latches on to the _______________
enumeration in Buac which declared, in an obiter, that “contests involving the
President and the Vice-President fall within the exclusive original jurisdiction of 45 CONSTITUTION, Art. VIII, Sec. 1, second paragraph.
the PET, also in the exercise of quasi-judicial power.” 46 See Robles v. House of Representatives Electoral Tribunal, supra note
The issue raised by petitioner is more imagined than real. Section 12, Article 38; Lazatin v. House Electoral Tribunal, supra note 38.
VIII of the Constitution reads:
817
“SEC. 12. The Members of the Supreme Court and of other courts
established by law shall not be designated to any agency performing quasi-judicial VOL. 635, NOVEMBER 23, 2010 817
or administrative functions.” Macalintal vs. Presidential Electoral Tribunal
The traditional grant of judicial power is found in Section 1, Article VIII of the It is also beyond cavil that when the Supreme Court, as PET, resolves a
Constitution which provides that the power “shall be vested in one Supreme Court presidential or vice-presidential election contest, it performs what is essentially a
and in such lower courts as may be established by law.” Consistent with our judicial power. In the landmark case of Angara v. Electoral Commission,47 Justice
presidential system of government, the function of “dealing with the settlement of Jose P. Laurel enucleated that “it would be inconceivable if the Constitution had
disputes, controversies or conflicts involving rights, duties or prerogatives that are not provided for a mechanism by which to direct the course of government along
legally demandable and enforceable”44 is apportioned to courts of jus- constitutional channels.” In fact, Angara pointed out that “[t]he Constitution is a
definition of the powers of government.” And yet, at that time, the 1935
_______________ Constitution did not contain the expanded definition of judicial power found in
Article VIII, Section 1, paragraph 2 of the present Constitution.
43 Supra note 4. With the explicit provision, the present Constitution has allocated to the
44 Javellana v. Executive Secretary, et al., 151-A Phil. 36, 131; 50 SCRA 30 Supreme Court, in conjunction with latter’s exercise of judicial power inherent in
(1973). all courts,48 the task of deciding presidential and vice-presidential election
816 contests, with full authority in the exercise thereof. The power wielded by PET is
816 SUPREME COURT REPORTS ANNOTATED a derivative of the plenary judicial power allocated to courts of law, expressly
provided in the Constitution. On the whole, the Constitution draws a thin, but,
Macalintal vs. Presidential Electoral Tribunal nevertheless, distinct line between the PET and the Supreme Court.
tice. With the advent of the 1987 Constitution, judicial power was expanded to If the logic of petitioner is to be followed, all Members of the Court, sitting in
include “the duty of the courts of justice to settle actual controversies involving the Senate and House Electoral Tribunals would violate the constitutional
rights which are legally demandable and enforceable, and to determine whether proscription found in Section 12, Article VIII. Surely, the petitioner will be among
or not there has been a grave abuse of discretion amounting to lack or excess of the first to acknowledge that this is not so. The Constitution which, in Section 17,
jurisdiction on the part of any branch or instrumentality of the Government.”45 The Article VI, explicitly provides that three Supreme Court Justices shall sit in the
power was expanded, but it remained absolute.
Senate and House Electoral Tribunals, respectively, effectively exempts the contests relating to his election, returns, and qualifications ends, and the HRET’s
Justices-Members thereof from the prohibition in Section 12, Article VIII. In the own jurisdiction begins. (Aggabao vs. Commission on Elections, 449 SCRA 400
same vein, it is the Constitution itself, in [2005])
Protestant’s widow is not a real party in interest to this election protest. (Poe
_______________ vs. Macapagal-Arroyo, 454 SCRA 142 [2005])
Under Section 20 of R.A. No. 7166, the board of canvassers is mandated to
47 63 Phil. 139 (1936). grant an objecting party 24 hours from the time of the presentation of the oral
48 See Ynot v. Intermediate Appellate Court, G.R. No. L-74457, March 20, objection to submit its opposition. (Espidol vs. Commission on Elections, 472 SCRA
1987, 148 SCRA 659, 665; Tañada and Macapagal v. Cuenco, et al., 103 Phil. 1051 380 [2005])
(1957); Alejandrino v. Quezon, 46 Phil. 83 (1924). ——o0o——
818
818 SUPREME COURT REPORTS ANNOTATED
Macalintal vs. Presidential Electoral Tribunal
Section 4, Article VII, which exempts the Members of the Court, constituting the
PET, from the same prohibition.
We have previously declared that the PET is not simply an agency to which
Members of the Court were designated. Once again, the PET, as intended by the G.R. No. 191618. June 7, 2011.*
framers of the Constitution, is to be an institution independent, but not separate, ATTY. ROMULO B. MACALINTAL, petitioner, vs.PRESIDENTIAL
from the judicial department, i.e., the Supreme Court. McCulloch v. State of ELECTORAL TRIBUNAL, respondent.
Maryland49 proclaimed that “[a] power without the means to use it, is a nullity.” Constitutional Law; Election Law; Presidential Electoral Tribunal (PET);
The vehicle for the exercise of this power, as intended by the Constitution and Presidential Electoral Tribunal (PET) is authorized by the last paragraph of
specifically mentioned by the Constitutional Commissioners during the Section 4, Article VII of the Constitution and as supported by the discussions of the
discussions on the grant of power to this Court, is the PET. Thus, a microscopic Members of the Constitutional Commission; The Constitution cannot contain the
view, like the petitioner’s, should not constrict an absolute and constitutional grant specific wording required by petitioner in order for him to accept the
of judicial power. constitutionality of the Presidential Electoral Tribunal (PET).—We reiterate that
One final note. Although this Court has no control over contrary people and the PET is authorized by the last paragraph of Section 4, Article VII of the
naysayers, we reiterate a word of caution against the filing of baseless petitions Constitution and as supported by the discussions of the Members of the
which only clog the Court’s docket. The petition in the instant case belongs to that Constitutional Commission, which drafted the present Constitution. The explicit
classification. reference by the framers of our Constitution to constitutionalizing what was
WHEREFORE, the petition is DISMISSED. Costs against petitioner. merely statutory before is not diluted by the absence of a phrase, line or word,
SO ORDERED. mandating the Supreme Court to create a Presidential Electoral Tribunal. Suffice
Corona (C.J.), Carpio, Carpio-Morales, Velasco, Jr., it to state that the Constitution, verbose as it already is, cannot contain the specific
Leonardo-De Castro, Brion, Peralta, Bersamin, Abad, wording required by petitioner in order for him to accept the constitutionality of
Villarama, Jr., Perez, Mendoza and Sereno, JJ., concur. the PET.
Del Castillo, J., On Official Leave. Same; Same; Same; Words and Phrases; Doctrine of Necessary Implication;
Petition dismissed. Under the doctrine of necessary implication, the additional jurisdiction bestowed
Notes.—Once a winning candidate has been proclaimed, taken his oath, and by the last paragraph of Section 4, Article VII of the Constitution to decide
assumed office as a Member of the House of Representatives, COMELEC’s presidential and vice-presidential elections contests includes the means necessary
jurisdiction over election to carry it into ef-

_______________ _______________

49 Supra note 21. * EN BANC.


819 240
VOL. 635, NOVEMBER 23, 2010 819 240 SUPREME COURT REPORTS ANNOTATED

Macalintal vs. Presidential Electoral Tribunal Macalintal vs. Presidential Electoral Tribunal
fect.—Judicial power granted to the Supreme Court by the same Constitution
is plenary. And under the doctrine of necessary implication, the additional
jurisdiction bestowed by the last paragraph of Section 4, Article VII of the We cannot agree with his insistence that the creation of the PET is
Constitution to decide presidential and vice-presidential elections contests unconstitutional. We reiterate that the abstraction of the Supreme Court acting as
includes the means necessary to carry it into effect. a Presidential Electoral Tribu-
MOTION FOR RECONSIDERATION of a decision of the Supreme Court.
The facts are stated in the resolution of the Court. _______________
RESOLUTION
NACHURA, J.: 2 Entitled “Biraogo v. Philippine Truth Commission” and “Lagman v.
Before us is a Motion for Reconsideration filed by petitioner Atty. Romulo B. Executive Secretary,” docketed as G.R. Nos. 192935 and 193036, respectively, and
Macalintal of our Decision1 in G.R. No. 191618 dated November 23, 2010, promulgated on December 7, 2010, 637 SCRA 78.
dismissing his petition and declaring the establishment of respondent Presidential 3 G.R. No. 192935, December 7, 2010, 637 SCRA 78.
Electoral Tribunal (PET) as constitutional. 242
Petitioner reiterates his arguments on the alleged unconstitutional creation of
242 SUPREME COURT REPORTS ANNOTATED
the PET:
1. He has standing to file the petition as a taxpayer and a concerned citizen. Macalintal vs. Presidential Electoral Tribunal
2. He is not estopped from assailing the constitution of the PET simply by nal from the unequivocal grant of jurisdiction in the last paragraph of Section 4,
virtue of his appearance as counsel of former president Gloria Macapagal-Arroyo Article VII of the Constitution is sound and tenable. The provision reads:
before respondent tribunal. “Sec. 4. x x x.
3. Section 4, Article VII of the Constitution does not provide for the creation The Supreme Court, sitting en banc, shall be the sole judge of all contests
of the PET. relating to the election, returns, and qualifications of the President or Vice-
4. The PET violates Section 12, Article VIII of the Constitution. President, and may promulgate its rules for the purpose.”
We mapped out the discussions of the Constitutional Commission on the
_______________ foregoing provision and concluded therefrom that:
“The mirabile dictu of the grant of jurisdiction to this Court, albeit found in the
1 Rollo, pp. 71-102. Article on the executive branch of government, and the constitution of the PET, is
241 evident in the discussions of the Constitutional Commission. On the exercise of
this Court’s judicial power as sole judge of presidential and vice-presidential
VOL. 651, JUNE 7, 2011 241
election contests, and to promulgate its rules for this purpose, we find the
Macalintal vs. Presidential Electoral Tribunal proceedings in the Constitutional Commission most instructive:
To bolster his arguments that the PET is an illegal and unauthorized progeny MR. DAVIDE. On line 25, after the words “Vice-President,” I propose to add
of Section 4, Article VII of the Constitution, petitioner invokes our ruling on the AND MAY PROMULGATE ITS RULES FOR THE PURPOSE. This refers
constitutionality of the Philippine Truth Commission (PTC).2 Petitioner cites the to the Supreme Court sitting en banc. This is also to confer on the
concurring opinion of Justice Teresita J. Leonardo-de Castro that the PTC is a Supreme Court exclusive authority to enact the necessary rules
public office which cannot be created by the President, the power to do so being while acting as sole judge of all contests relating to the election,
lodged exclusively with Congress. Thus, petitioner submits that if the President, returns and qualifications of the President or Vice-President.
as head of the Executive Department, cannot create the PTC, the Supreme Court, MR. REGALADO. My personal position is that the rule-making
likewise, cannot create the PET in the absence of an act of legislature. power of the Supreme Court with respect to its internal procedure
On the other hand, in its Comment to the Motion for Reconsideration, the is already implicit under the Article on the Judiciary; considering,
Office of the Solicitor General maintains that: however, that according to the Commissioner, the purpose of this
1. Petitioner is without standing to file the petition. is to indicate the sole power of the Supreme Court without
2. Petitioner is estopped from assailing the jurisdiction of the PET. intervention by the legislature in the promulgation of its rules on
3. The constitution of the PET is “on firm footing on the basis of the grant of this particular point, I think I will personally recommend its
authority to the [Supreme] Court to be the sole judge of all election contests for the acceptance to the Committee.243
President or Vice-President under paragraph 7, Section 4, Article VII of the 1987 VOL. 651, JUNE 7, 2011 243
Constitution.”
Except for the invocation of our decision in Louis “Barok” C. Biraogo v. The Macalintal vs. Presidential Electoral Tribunal
Philippine Truth Commission of 2010,3 petitioner does not allege new arguments xxxx
to warrant reconsideration of our Decision. MR. NOLLEDO x x x.
With respect to Sections 10 and 11 on page 8, I understand that the
Committee has also created an Electoral Tribunal in the Senate and a
Commission on Appointments which may cover membership from both
Houses. But my question is: It seems to me that the committee report does MR. REGALADO. We feel that it will not be an intrusion into the
not indicate which body should promulgate the rules that shall govern the separation of powers guaranteed to the judiciary because this is
Electoral Tribunal and the Commission on Appointments. Who shall then strictly an adversarial and judicial proceeding.
promulgate the rules of these bodies? MR. VILLACORTA. May I know the rationale of the Committee because
MR. DAVIDE. The Electoral Tribunal itself will establish and this supersedes Republic Act 7950 which provides for the Presidential
promulgate its rules because it is a body distinct and independent Electoral Tribunal?
already from the House, and so with the Commission on FR. BERNAS. Precisely, this is necessary. Election contests are, by
Appointments also. It will have the authority to promulgate its own their nature, judicial. Therefore, they are cognizable only by
rules. courts. If, for instance, we did not have a constitutional provision
On another point of discussion relative to the grant of judicial power, but equally on an electoral tribunal
cogent, we listen to former Chief Justice Roberto Concepcion: 245
MR. SUAREZ. Thank you. VOL. 651, JUNE 7, 2011 245
Would the Commissioner not consider that violative of the doctrine of
separation of powers? Macalintal vs. Presidential Electoral Tribunal
MR. CONCEPCION. I think Commissioner Bernas explained that for the Senate or an electoral tribunal for the House, normally, as
this is a contest between two parties. This is a judicial power. composed, that cannot be given jurisdiction over contests.
MR. SUAREZ. We know, but practically the Committee is giving to the So, the background of this is really the case of Roxas v. Lopez. The
judiciary the right to declare who will be the President of our country, which Gentleman will remember that in that election, Lopez was declared winner.
to me is a political action. He filed a protest before the Supreme Court because there was a republic
MR. CONCEPCION. There are legal rights which are enforceable act which created the Supreme Court as the Presidential Electoral
under the law, and these are essentially justiciable questions. Tribunal. The question in this case was whether new powers could be given
MR. SUAREZ. If the election contest proved to be long, burdensome the Supreme Court by law. In effect, the conflict was actually whether there
and tedious, practically all the time of the Supreme Court was an attempt to create two Supreme Courts and the answer of the
sitting en banc would be occupied with it considering that they Supreme Court was: “No, this did not involve the creation of two Supreme
will be going over millions and Courts, but precisely we are giving new jurisdiction to the Supreme Court,
244 as it is allowed by the Constitution. Congress may allocate various
jurisdictions.”
244 SUPREME COURT REPORTS ANNOTATED
Before the passage of that republic act, in case there was any contest
Macalintal vs. Presidential Electoral Tribunal between two presidential candidates or two vice-presidential candidates,
millions of ballots or election returns, Madam President. no one had jurisdiction over it. So, it became necessary to create a
Echoing the same sentiment and affirming the grant of judicial power to the Presidential Electoral Tribunal. What we have done is to
Supreme Court, Justice Florenz D. Regalado and Fr. Joaquin Bernas both opined: constitutionalize what was statutory but it is not an infringement
MR. VILLACORTA. Thank you very much, Madam President. on the separation of powers because the power being given to the
I am not sure whether Commissioner Suarez has expressed his point. On Supreme Court is a judicial power.
page 2, the fourth paragraph of Section 4 provides: Unmistakable from the foregoing is that the exercise of our power to judge
The Supreme Court, sitting en banc, shall be the sole judge of all contests presidential and vice-presidential election contests, as well as the rule-making
relating to the election, returns and qualifications of the President or Vice- power adjunct thereto, is plenary; it is not as restrictive as petitioner would
President. interpret it. In fact, former Chief Justice Hilario G. Davide, Jr., who proposed the
May I seek clarification as to whether or not the matter of insertion of the phrase, intended the Supreme Court to exercise exclusive
determining the outcome of the contests relating to the election authority to promulgate its rules of procedure for that purpose. To this, Justice
returns and qualifications of the President or Vice-President is Regalado forthwith assented and then emphasized that the sole power ought to be
purely a political matter and, therefore, should not be left entirely without intervention by the legislative department. Evidently, even the legislature
to the judiciary. Will the above-quoted provision not impinge on cannot limit the judicial power to resolve presidential and vice-presidential
the doctrine of separation of powers between the executive and the election contests and our rule-making power connected thereto.
judicial departments of the government? To foreclose all arguments of petitioner, we reiterate that the establishment of the
MR. REGALADO. No, I really do not feel that would be a problem. PET simply constitutionalized what was statutory
This is a new provision incidentally. It was not in the 1935 246
Constitution nor in the 1973 Constitution. 246 SUPREME COURT REPORTS ANNOTATED
MR. VILLACORTA. That is right.
Macalintal vs. Presidential Electoral Tribunal
before the 1987 Constitution. The experiential context of the PET in our country probably during the late 1960s and early 1970s. I do not know how
cannot be denied.”4 the present Supreme Court would react to such circumstances, but
Stubbornly, despite the explicit reference of the Members of the Constitutional there is also the question of who else would hear the election
Commission to a Presidential Electoral Tribunal, with Fr. Joaquin Bernas protests.
categorically declaring that in crafting the last paragraph of Section 4, Article VII MR. SUAREZ. We are asking this question because between lines 23 to 25,
of the Constitution, they “constitutionalize[d] what was statutory,” petitioner there are no rules provided for the hearings and there is not time limit or
continues to insist that the last paragraph of Section 4, Article VII of the duration for the election contest to be decided by the Supreme Court. Also,
Constitution does not provide for the creation of the PET. Petitioner is adamant we will have to consider the historical background that when R.A. 1793,
that “the fact that [the provision] does not expressly prohibit [the] creation [of the which organized the Presidential Electoral Tribunal, was promulgated on
PET] is not an authority for the Supreme Court to create the same.” June 21, 1957, at least three famous election contests were presented and
Petitioner is going to town under the misplaced assumption that the text of the two of them ended up in withdrawal by the protestants out of sheer
provision itself was the only basis for this Court to sustain the PET’s frustration because of the delay in the resolution of the cases. I am referring
constitutionality. to the electoral protest that was lodged by former President Carlos P.
We reiterate that the PET is authorized by the last paragraph of Section 4, Garcia against our “kabalen” former President Diosdado Macapagal in
Article VII of the Constitution and as supported by the discussions of the Members 1961 and the vice-presidential election contest filed by the late
of the Constitutional Commission, which drafted the present Constitution. 248
The explicit reference by the framers of our Constitution to constitutionalizing 248 SUPREME COURT REPORTS ANNOTATED
what was merely statutory before is not diluted by the absence of a phrase, line or
word, mandating the Supreme Court to create a Presidential Electoral Tribunal. Macalintal vs. Presidential Electoral Tribunal
Suffice it to state that the Constitution, verbose as it already is, cannot contain Senator Gerardo Roxas against Vice-President Fernando Lopez in 1965.
the specific wording required by petitioner in order for him to accept the MR. CONCEPCION. I cannot answer for what the protestants had in mind.
constitutionality of the PET. But when that protest of Senator Roxas was withdrawn, the results were
In our Decision, we clarified the structure of the PET: already available. Senator Roxas did not want to have a decision adverse
to him. The votes were being counted already, and he did not get what he
_______________ expected so rather than have a decision adverse to his protest, he withdrew
the case.
4 Atty. Romulo B. Macalintal v. Presidential Electoral Tribunal, G.R. No. xxxx
191618, November 23, 2010, 635 SCRA 783. MR. SUAREZ. I see. So the Commission would not have any
objection to vesting in the Supreme Court this matter of resolving
247
presidential and vice-presidential contests?
VOL. 651, JUNE 7, 2011 247 MR. CONCEPCION. Personally, I would not have any objection.
Macalintal vs. Presidential Electoral Tribunal MR. SUAREZ. Thank you.
“Be that as it may, we hasten to clarify the structure of the PET as a legitimate Would the Commissioner not consider that violative of the doctrine of
progeny of Section 4, Article VII of the Constitution, composed of members of the separation of powers?
Supreme Court, sitting en banc. The following exchange in the 1986 Constitutional MR. CONCEPCION. I think Commissioner Bernas explained that this is a
Commission should provide enlightenment: contest between two parties. This is a judicial power.
MR. SUAREZ. Thank you. Let me proceed to line 23, page 2, wherein it is MR. SUAREZ. We know, but practically the Committee is giving to the
provided, and I quote: judiciary the right to declare who will be the President of our country, which
The Supreme Court, sitting en banc[,] shall be the sole judge of all contests to me is a political action.
relating to the election, returns and qualifications of the President or Vice- MR. CONCEPCION. There are legal rights which are enforceable under
President. the law, and these are essentially justiciable questions.
Are we not giving enormous work to the Supreme Court especially MR. SUAREZ. If the election contest proved to be long, burdensome
when it is directed to sit en banc as the sole judge of all presidential and tedious, practically all the time of the Supreme Court
and vice-presidential election contests? sitting en banc would be occupied with it considering that they
MR. SUMULONG. That question will be referred to Commissioner will be going over millions and millions of ballots or election
Concepcion. returns, Madam President.
MR. CONCEPCION. This function was discharged by the Supreme MR. CONCEPCION. The time consumed or to be consumed in this contest
Court twice and the Supreme Court was able to dispose of each for President is dependent upon they key number
case in a period of one year as provided by law. Of course, that was 249
VOL. 651, JUNE 7, 2011 249 independence and of the plenary powers needed to discharge this burden justifies
the budget allocation of the PET.
Macalintal vs. Presidential Electoral Tribunal The conferment of additional jurisdiction to the Supreme Court, with the duty
of teams of revisors. I have no experience insofar as contests in other offices characterized as an “awesome” task, includes the means necessary to carry it into
are concerned. effect under the doctrine of necessary implication. We cannot overemphasize that
MR. SUAREZ. Although there is a requirement here that the Supreme the abstraction of the PET from the explicit grant of power to the Supreme Court,
Court is mandated to sit en banc? given our abundant experience, is not unwarranted.
MR. CONCEPCION. Yes. A plain reading of Article VII, Section 4, paragraph 7, readily reveals a grant of
MR. SUAREZ. I see. authority to the Supreme Court sitting en banc. In the same vein, although the
MR. CONCEPCION. The steps involved in this contest are: First, the method by which the Supreme Court exercises this authority is not specified in the
ballot boxes are opened before teams of three, generally, a provision, the grant of power does not contain any limitation on the Supreme
representative each of the court, of the protestant and of the Court’s exercise thereof. The Supreme Court’s method of deciding presidential and
“protestee.” It is all a questions of how many teams are organized. vice-presidential election contests, through the PET, is actually a derivative of the
Of course, that can be expensive, but it would be expensive exercise of the prerogative conferred by the aforequoted constitutional provision.
whatever court one would choose. There were times that the Thus, the subsequent directive in the provision for the Supreme Court to
Supreme Court, with sometimes 50 teams at the same time “promulgate its rules for the purpose.”
working, would classify the objections, the kind of problems, and The conferment of full authority to the Supreme Court, as a PET, is equivalent to
the court would only go over the objected votes on which the the full authority conferred upon the electoral tribunals of the Senate and the
parties could not agree. So it is not as awesome as it would appear House of Representatives, i.e., the Senate Electoral Tribunal (SET) and the House
insofar as the Court is concerned. What is awesome is the cost of of Representatives Electoral Tribunal (HRET), which we have affirmed on
the revision of the ballots because each party would have to numerous occasions.”6
appoint one representative for every team, and that may take quite
a big amount. _______________
MR. SUAREZ. If we draw from the Commissioner’s experience which he is
sharing with us, what would be the reasonable period for the election
6 Id.
contest to be decided?
251
MR. CONCEPCION. Insofar as the Supreme Court is concerned, the
Supreme Court always manages to dispose of the case in one year. VOL. 651, JUNE 7, 2011 251
MR. SUAREZ. In one year. Thank you for the clarification.”5 Macalintal vs. Presidential Electoral Tribunal
Judicial power granted to the Supreme Court by the same Constitution is Next, petitioner still claims that the PET exercises quasi-judicial power and,
plenary. And under the doctrine of necessary implication, the additional thus, its members violate the proscription in Section 12, Article VIII of the
jurisdiction bestowed by the last paragraph of Section 4, Article VII of the Constitution, which reads:
Constitution to de- “SEC. 12. The Members of the Supreme Court and of other courts
established by law shall not be designated to any agency performing quasi-judicial
_______________ or administrative functions.”
We dispose of this argument as we have done in our Decision, viz.:
5 Id. “The traditional grant of judicial power is found in Section 1, Article VIII of the
250 Constitution which provides that the power “shall be vested in one Supreme Court
250 SUPREME COURT REPORTS ANNOTATED and in such lower courts as may be established by law.” Consistent with our
presidential system of government, the function of “dealing with the settlement of
Macalintal vs. Presidential Electoral Tribunal disputes, controversies or conflicts involving rights, duties or prerogatives that are
cide presidential and vice-presidential elections contests includes the means legally demandable and enforceable” is apportioned to courts of justice. With the
necessary to carry it into effect. Thus: advent of the 1987 Constitution, judicial power was expanded to include “the duty
“Obvious from the foregoing is the intent to bestow independence to the Supreme of the courts of justice to settle actual controversies involving rights which are
Court as the PET, to undertake the Herculean task of deciding election protests legally demandable and enforceable, and to determine whether or not there has
involving presidential and vice-presidential candidates in accordance with the been a grave abuse of discretion amounting to lack or excess of jurisdiction on the
process outlined by former Chief Justice Roberto Concepcion. It was made in part of any branch or instrumentality of the Government.” The power was
response to the concern aired by delegate Jose E. Suarez that the additional duty expanded, but it remained absolute.
may prove too burdensome for the Supreme Court. This explicit grant of
The set up embodied in the Constitution and statutes characterizes the 253
resolution of electoral contests as essentially an exercise of judicial VOL. 651, JUNE 7, 2011 253
power.
At the barangay and municipal levels, original and exclusive jurisdiction over Macalintal vs. Presidential Electoral Tribunal
election contests is vested in the municipal or metropolitan trial courts and the as intended by the framers of the Constitution, is to be an
regional trial courts, respectively. institution independent, but not separate, from the judicial department, i.e., the
At the higher levels—city, provincial, and regional, as well as congressional and Supreme Court. McCulloch v. State of Maryland proclaimed that “[a] power
senatorial—exclusive and original jurisdiction is lodged in the COMELEC and in without the means to use it, is a nullity.” The vehicle for the exercise of this power,
the House of Representatives and Senate Electoral Tribunals, which are not, as intended by the Constitution and specifically mentioned by the Constitutional
strictly and literally speaking, courts of law. Although not courts of law, they Commissioners during the discussions on the grant of power to this Court, is the
are, nonetheless, empowered to resolve election contests which involve, in essence, PET. Thus, a microscopic view, like the petitioner’s, should not constrict an
an exercise of judicial power, because of the explicit constitutional empowerment absolute and constitutional grant of judicial power.”7
found in Section 2(2), Article IX-C (for the Finally, petitioner’s application of our decision in Biraogo v. Philippine Truth
252 Commission8 to the present case is an unmitigated quantum leap.
The decision therein held that the PTC “finds justification under Section 17,
252 SUPREME COURT REPORTS ANNOTATED
Article VII of the Constitution.” A plain reading of the constitutional
Macalintal vs. Presidential Electoral Tribunal provisions, i.e., last paragraph of Section 4 and Section 17, both of Article VII on
COMELEC) and Section 17, Article VI (for the Senate and House Electoral the Executive Branch, reveals that the two are differently worded and deal with
Tribunals) of the Constitution. Besides, when the COMELEC, the HRET, and the separate powers of the Executive and the Judicial Branches of government. And
SET decide election contests, their decisions are still subject to judicial review— as previously adverted to, the basis for the constitution of the PET was, in fact,
via a petition for certiorari filed by the proper party—if there is a showing that the mentioned in the deliberations of the Members of the Constitutional Commission
decision was rendered with grave abuse of discretion tantamount to lack or excess during the drafting of the present Constitution.
of jurisdiction. WHEREFORE, the Motion for Reconsideration is DENIED. Our Decision in
It is also beyond cavil that when the Supreme Court, as PET, resolves a G.R. No. 191618 STANDS.
presidential or vice-presidential election contest, it performs what is essentially a SO ORDERED.
judicial power. In the landmark case of Angara v. Electoral Commission, Justice Corona (C.J.), Carpio, Carpio-Morales, Velasco, Jr.,
Jose P. Laurel enucleated that “it would be inconceivable if the Constitution had Leonardo-De Castro, Brion, Peralta, Bersamin, Del Castillo, Abad, Villarama, Jr.,
not provided for a mechanism by which to direct the course of government along Perez, Mendoza and Sereno, JJ., concur.
constitutional channels.” In fact, Angara pointed out that “[t]he Constitution is a Motion for Reconsideration denied, judgment stands.
definition of the powers of government.” And yet, at that time, the 1935
Constitution did not contain the expanded definition of judicial power found in _______________
Article VIII, Section 1, paragraph 2 of the present Constitution.
With the explicit provision, the present Constitution has allocated to the Supreme 7 Id.
Court, in conjunction with latter’s exercise of judicial power inherent in all courts, 8 Supra note 3.
the task of deciding presidential and vice-presidential election contests, with full 254
authority in the exercise thereof. The power wielded by PET is a derivative of the
254 SUPREME COURT REPORTS ANNOTATED
plenary judicial power allocated to courts of law, expressly provided in the
Constitution. On the whole, the Constitution draws a thin, but, nevertheless, Macalintal vs. Presidential Electoral Tribunal
distinct line between the PET and the Supreme Court. Note.—The constitutional function as well as the power and the duty to be the
If the logic of petitioner is to be followed, all Members of the Court, sitting in the sole judge of all contests relating to the election, returns and qualification of the
Senate and House Electoral Tribunals would violate the constitutional President and Vice-President is expressly vested in the PET, in Section 4, Article
proscription found in Section 12, Article VIII. Surely, the petitioner will be among VII of the Constitution. (Legarda vs. De Castro, 454 SCRA 242 [2005])
the first to acknowledge that this is not so. The Constitution which, in Section 17,
Article VI, explicitly provides that three Supreme Court Justices shall sit in the ——o0o——
Senate and House Electoral Tribunals, respectively, effectively exempts the
Justices-Members thereof from the prohibition in Section 12, Article VIII. In the © Copyright 2018 Central Book Supply, Inc. All rights reserved.
same vein, it is the Constitution itself, in Section 4, Article VII, which exempts the
Members of the Court, constituting the PET, from the same prohibition.
We have previously declared that the PET is not simply an agency to which
Members of the Court were designated. Once again, the PET,
the above-cited case addressed the issue of whether petitioner therein acquired
valid title to the disputed position and so had the right to security of tenure. It
must be stressed though that while the designation was in the nature of an acting
SO ORDERED. and temporary capacity, the words “hold the office” were employed. Such holding
Puno (C.J., Chairperson), Nachura,** Bersamin and Villarama, Jr., JJ., of office pertains to both appointment and designation because the appointee or
concur. designate performs the duties and functions of the office. The 1987 Constitution in
Petition granted, judgment reversed and set aside. prohibiting dual or multiple offices, as well as incompatible offices, refers to the
Note.—A complaint must allege the assessed value of the real property subject holding of the office, and not to the nature of the appointment or designation,
of the complaint or the interest thereon to determine which court has jurisdiction words which were not even found in Section 13, Article VII nor in Section 7,
over the action. (Quinagoran vs. Court of Appeals, 531 SCRA 104 [2007]) paragraph 2, Article IX-B. To “hold” an office means to “possess or occupy” the
——o0o—— same, or “to be in possession and administration,” which implies nothing less than
the actual discharge of the functions and duties of the office.
G.R. No. 184740. February 11, 2010.* Same; Presidency; Holding of Other Office; Evidently, from this move as well
DENNIS A. B. FUNA, petitioner, vs. EXECUTIVE SECRETARY EDUARDO R. as in the different phraseologies of the constitutional provisions in question, the
ERMITA, Office of the President, SEC. LEANDRO R. MENDOZA, in his official intent of the framers of the Constitution was to impose a stricter prohibition of the
capacity as Secretary of the Department of Transportation and Communications, President and his official family in so far as holding other offices or employment in
USEC. MARIA ELENA H. BAUTISTA, in her official capacities as Undersecretary the gov-
of the Department of Transportation and Communications and as Officer-in- 310ernment or elsewhere is concerned.—Evidently, from this move as well as
Charge of the Maritime Industry Authority (MARINA), respondents. in the different phraseologies of the constitutional provisions in question, the
Justiciable Controversy; Moot and Academic; A moot and academic case is intent of the framers of the Constitution was to impose a stricter
one that ceases to present a justiciable controversy by virtue of supervening events, prohibition on the President and his official family in so far
so that a declaration thereon would be of no practical use or value. Generally, courts as holdingother offices or employment in the government or elsewhere is
decline jurisdiction over such case or dismiss it on ground of mootness.—A moot concerned.
and academic case is one that ceases to present a justiciable controversy by virtue CARPIO-MORALES, J., Concurring Opinion:
of supervening events, so that a declaration thereon would be of no practical use Public Officers; “Designation” Defined.—Designation may be loosely defined
or value. Generally, courts decline jurisdiction over such case or dismiss it on as an appointment because it likewise involves the naming of a particular person
ground of mootness. However, as we held in to a specified public office. In fact, even without a known appointment or election,
the de facto doctrine comes into play if the duties of the office were exercised under
_______________ such circumstances of reputation or acquiescence as were calculated to induce
people, without inquiry, to submit to or invoke his action, supposing him to be the
** Additional member per Special Order No. 821. officer he assumed to be.
* EN BANC. Same; Effect of assuming a second office which is incompatible with the act.—
309Public Interest Center, Inc. v. Elma, 494 SCRA 53 (2006) supervening The other proposition—that a person who assumes a second and incompatible
events, whether intended or accidental, cannot prevent the Court from rendering office is deemed to have resigned from the first office—was applied in Public
a decision if there is a grave violation of the Constitution. Even in cases where Interest Center, Inc. v. Elma, 494 SCRA 53 (2006) where the Court, by Resolution
supervening events had made the cases moot, this Court did not hesitate to resolve of March 5, 2007, clarified that the ruling did not render both appointments void.
the legal or constitutional issues raised to formulate controlling principles to guide It held that “[f]ollowing the common-law rule on incompatibility of offices,
the bench, bar, and public. respondent Elma had, in effect, vacated his first office as PCGG Chairman when
Administrative Law; Incompatible Office; While the designation was in the he accepted the second office” as Chief Presidential Legal Counsel.
nature of an acting and temporary capacity, the words “hold the office” were Same; Rule when acceptance of a second public office does not apply.—It has
employed. Such holding of office pertains to both appointment and designation also been observed that the rule of ipso facto vacancy of a public office by
because the appointee or designate performs the duties and functions of the office. acceptance of a second public office does not apply where, under applicable
The 1987 Constitution in prohibiting dual or multiple offices, as well as constitutional or statutory provisions, the holder of a public office is rendered
incompatible offices, refers to the holding of the office, and not to the nature of the ineligible for a specified time for a second public office; under such circumstances
appointment or designation, words which were not even found in Section 13, Article it is the second office which is considered vacant rather than the first office. I,
VII nor in Section 7, paragraph 2, Article IX-B. To “hold” an office means to “possess therefore, vote to GRANT the petition and further declare that Bautista was a de
or occupy” the same, or “to be in possession and administration,” which implies facto officer during her brief stint as MARINA OIC Administrator and a de
nothing less than the actual discharge of the functions and duties of the office.— jure DOTC Undersecretary during her entire tenure as such.
Respondents’ reliance on the foregoing definitions is misplaced considering that 311
SPECIAL CIVIL ACTION in the Supreme Court. Certiorari, Prohibition and A.8 Moreover, the provisions on the DOTC in the Administrative Code of 1987,
Mandamus. specifically Sections 23 and 24, Chapter 6, Title XV, Book IV do not provide any ex
The facts are stated in the opinion of the Court. officio role for the undersecretaries in any of the department’s attached agencies.
Funa, Tantuan & Fortes for petitioner. The fact that Bautista was extended an appointment naming her as OIC of
The Solicitor General for respondents. MARINA shows that she does not occupy
VILLARAMA, JR., J.:
This is a petition for certiorari, prohibition and mandamus under Rule 65 with _______________
prayer for the issuance of a temporary restraining order and/or writ of preliminary
injunction, to declare as unconstitutional the designation of respondent 3 Id., at p. 102.
Undersecretary Maria Elena H. Bautista as Officer-in-Charge (OIC) of the 4 Id., at pp. 103-104.
Maritime Industry Authority (MARINA). 5 G.R. Nos. 83896 and 83815, February 22, 1991, 194 SCRA 317.
6 G.R. No. 138965, June 30, 2006, 494 SCRA 53.
The Antecedents 7 PROVIDING FOR THE REORGANIZATION OF MARITIME FUNCTIONS IN THE
PHILIPPINES, CREATING THE MARITIME INDUSTRY AUTHORITY, AND FOR OTHER
On October 4, 2006, President Gloria Macapagal-Arroyo appointed respondent PURPOSES, approved on June 1, 1974.
Maria Elena H. Bautista (Bautista) as Undersecretary of the Department of 8 Approved on April 13, 1987.
Transportation and Communications (DOTC), vice Agustin R. Bengzon. Bautista 313it in an ex officio capacity since an ex officio position does not require any
was designated as Undersecretary for Maritime Transport of the department “further warrant or appoint.”9
under Special Order No. 2006-171 dated October 23, 2006.1 Petitioner further contends that even if Bautista’s appointment or designation
On September 1, 2008, following the resignation of then MARINA as OIC of MARINA was intended to be merely temporary, still, such designation
Administrator Vicente T. Suazo, Jr., Bautista was designated as Officer-in-Charge must not violate a standing constitutional prohibition, citing the rationale
(OIC), Office of the Administrator, MARINA, in concurrent capacity as DOTC in Achacoso v. Macaraig.10 Section 13, Article VII of the 1987 Constitution does not
Undersecretary.2 enumerate temporariness as one (1) of the exceptions thereto. And since a
On October 21, 2008, Dennis A. B. Funa in his capacity as taxpayer, concerned temporary designation does not have a maximum duration, it can go on for months
citizen and lawyer, filed the instant petition challenging the constitutionality of or years. In effect, the temporary appointment/designation can effectively
Bautista’s ap- circumvent the prohibition. Allowing undersecretaries or assistant secretaries to
occupy other government posts would open a Pandora’s Box as to let them feast on
_______________ choice government positions. Thus, in case of vacancy where no permanent
appointment could as yet be made, the remedy would be to designate one (1) of the
1 Rollo, pp. 99 and 101. two (2) Deputy Administrators as the Acting Administrator. Such would be the
2 Id., at p. 100. logical course, the said officers being in a better position in terms of knowledge and
312pointment/designation, which is proscribed by the prohibition on the experience to run the agency in a temporary capacity. Should none of them merit
President, Vice President, the Members of the Cabinet, and their deputies and the President’s confidence, then the practical remedy would be for Undersecretary
assistants to hold any other office or employment. Bautista to first resign as Undersecretary in order to qualify her as Administrator
On January 5, 2009, during the pendency of this petition, Bautista was of MARINA. As to whether she in fact does not receive or has waived any
appointed Administrator of the MARINA vice Vicente T. Suazo, Jr.3 and she remuneration, the same does not matter because remuneration is not an element
assumed her duties and responsibilities as such on February 2, 2009. 4 in determining whether there has been a violation of Section 13, Article VII of
the 1987 Constitution.11
The Case Petitioner likewise asserts the incompatibility between the posts of DOTC
Undersecretary and MARINA Administrator. The reason is that with respect to
the affairs in the maritime industry, the recommendations of the MARINA may be
Petitioner argues that Bautista’s concurrent positions as DOTC
Undersecretary and MARINA OIC is in violation of Section 13, Article VII of
the 1987 Constitution, as interpreted and explained by this Court in Civil Liberties _______________
Union v. Executive Secretary,5 and reiterated in Public Interest Center, Inc. v.
Elma.6 He points out that while it was clarified in Civil Liberties Union that the 9 Rollo, pp. 14-27.
prohibition does not apply to those positions held in ex officio capacities, the 10 G.R. No. 93023, March 13, 1991, 195 SCRA 235.
position of MARINA Administrator is not ex officio to the post of DOTC 11 Rollo, pp. 34-37.
Undersecretary, as can be gleaned from the provisions of its charter, Presidential 314the subject of counter or opposing recommendations from the Undersecretary
Decree (P.D.) No. 474,7 as amended by Executive Order (EO) No. 125- for Maritime Transport. In this case, the DOTC Undersecretary for Maritime
Transport and the OIC of MARINA have become one (1) and the same person. under the recognized exceptions to the rule against multiple offices, i.e., without
There is no more checking and counter-checking of powers and functions, and additional compensation (she did not receive any emolument as MA-
therein lies the danger to the maritime industry. There is no longer a person above
the Administrator of MARINA who will be reviewing the acts of said agency _______________
because the person who should be overseeing MARINA, the Undersecretary for
Maritime Transport, has effectively been compromised.12 14 Id., at pp. 86-87.
Finally, petitioner contends that there is a strong possibility in this case that 15 Id., at pp. 88-89.
the challenge herein can be rendered moot through the expediency of simply 316RINA OIC) and as required by the primary functions of the office. Besides,
revoking the temporary appointment/designation. But since a similar violation can Bautista held the position for four (4) months only, as in fact when she was
be committed in the future, there exists a possibility of “evading review,” and hence appointed MARINA Administrator on February 2, 2009, she relinquished her post
supervening events should not prevent the Court from deciding cases involving as DOTC Undersecretary for Maritime Transport, in acknowledgment of the
grave violation of the 1987 Constitution, as this Court ruled in Public Interest proscription on the holding of multiple offices.16
Center. Notwithstanding its mootness therefore, should it occur, there is a As to petitioner’s argument that the DOTC Undersecretary for Maritime
compelling reason for this case to be decided: the issue raised being “capable of Transport and MARINA Administrator are incompatible offices, respondents cite
repetition, yet evading review.”13 the test laid down in People v. Green,17which held that “[T]he offices must
On the other hand, the respondents argue that the requisites of a judicial subordinate, one [over] the other, and they must, per se, have the right to interfere,
inquiry are not present in this case. In fact, there no longer exists an actual one with the other, before they are compatible at common law.” Thus, respondents
controversy that needs to be resolved in view of the appointment of respondent point out that any recommendation by the MARINA Administrator concerning
Bautista as MARINA Administrator effective February 2, 2009 and the issues of policy and administration go to the MARINA Board and not the
relinquishment of her post as DOTC Undersecretary for Maritime Transport, Undersecretary for Maritime Transport. The Undersecretary for Maritime
which rendered the present petition moot and academic. Petitioner’s prayer for a Transport is, in turn, under the direct supervision of the DOTC Secretary.
temporary restraining order or writ of preliminary injunction is likewise moot and Petitioner’s fear that there is no longer a person above the Administrator of
MARINA who will be reviewing the acts of said agency (the Undersecretary for
_______________ Maritime Transport) is, therefore, clearly unfounded.18
In his Reply, petitioner contends that respondents’ argument on the
12 Id., at pp. 38-40. incompatibility of positions was made on the mere assumption that the positions
13 Id., at pp. 40-42. of DOTC Undersecretary for Maritime Transport and the administratorship of
315academic since, with this supervening event, there is nothing left to enjoin.14 MARINA are “closely related” and is governed by Section 7, paragraph 2, Article
Respondents also raise the lack of legal standing of petitioner to bring this suit. IX-B of the 1987 Constitution rather than by Section 13, Article VII. In other
Clear from the standard set in Public Interest Center is the requirement that the words, it was a mere secondary argument. The fact remains that, incompatible or
party suing as a taxpayer must prove that he has sufficient interest in preventing not, Section 13, Article VII still does not allow the herein challenged designation. 19
illegal expenditure of public funds, and more particularly, his personal and
substantial interest in the case. Petitioner, however, has not alleged any personal _______________
or substantial interest in this case. Neither has he claimed that public funds were
actually disbursed in connection with respondent Bautista’s designation as 16 Id., at pp. 90-93.
MARINA OIC. It is to be noted that respondent Bautista did not receive any salary 17 13 Sickels 295, 58 N.Y. 295, 1874 WL 11282 (N.Y.).
while she was MARINA OIC. As to the alleged transcendental importance of an 18 Id., at pp. 93-95.
issue, this should not automatically confer legal standing on a party.15 19 Id., at pp. 127-128.
Assuming for the sake of argument that the legal question raised herein needs 317 The sole issue to be resolved is whether or not the designation of
to be resolved, respondents submit that the petition should still be dismissed for respondent Bautista as OIC of MARINA, concurrent with the position of DOTC
being unmeritorious considering that Bautista’s concurrent designation as Undersecretary for Maritime Transport to which she had been appointed, violated
MARINA OIC and DOTC Undersecretary was constitutional. There was no the constitutional proscription against dual or multiple offices for Cabinet
violation of Section 13, Article VII of the 1987 Constitution because respondent Members and their deputies and assistants.
Bautista was merely designated acting head of MARINA on September 1, 2008.
She was designated MARINA OIC, not appointed MARINA Administrator. With Our Ruling
the resignation of Vicente T. Suazo, Jr., the position of MARINA Administrator
was left vacant, and pending the appointment of permanent Administrator,
The petition is meritorious.
respondent Bautista was designated OIC in a temporary capacity for the purpose
Requisites for Judicial Review
of preventing a hiatus in the discharge of official functions. Her case thus falls
The courts’ power of judicial review, like almost all other powers conferred by 22 Kilosbayan, Incorporated v. Morato, G.R. No. 118910, July 17, 1995, 246
the Constitution, is subject to several limitations, namely: (1) there must be an SCRA 540, 562-563, citing Baker v. Carr, 369 U.S. 186, 7 L.Ed.2d 663 (1962).
actual case or controversy calling for the exercise of judicial power; (2) the person 23 G.R. No. 171396 and six (6) other cases, May 3, 2006, 489 SCRA 160, 220-
challenging the act must have “standing” to challenge; he must have a personal 221.
and substantial interest in the case, such that he has sustained or will sustain, 319
direct injury as a result of its enforcement; (3) the question of constitutionality The other objection raised by the respondent is that the resolution of this case
must be raised at the earliest possible opportunity; and (4) the issue of had been overtaken by events considering the effectivity of respondent Bautista’s
constitutionality must be the very lis mota of the case.20 Respondents assert that appointment as MARINA Administrator effective February 2, 2009 and her
the second requisite is absent in this case. relinquishment of her former position as DOTC Undersecretary for Maritime
Generally, a party will be allowed to litigate only when (1) he can show that he Transport.
has personally suffered some actual or threatened injury because of the allegedly A moot and academic case is one that ceases to present a justiciable controversy
illegal conduct of the government; (2) the injury is fairly traceable to the challenged by virtue of supervening events, so that a declaration thereon would be of no
action; and (3) the injury is likely to be redressed by a practical use or value. Generally, courts decline jurisdiction over such case or
dismiss it on ground of mootness.24 However, as we held in Public Interest Center,
_______________ Inc. v. Elma,25 supervening events, whether intended or accidental, cannot prevent
the Court from rendering a decision if there is a grave violation of the Constitution.
20 Francisco, Jr. v. Nagmamalasakit na mga Manananggol ng mga Even in cases where supervening events had made the cases moot, this Court did
Manggagawang Pilipino, Inc., G.R. Nos. 160261-160263, 160277, 160292, 160295, not hesitate to resolve the legal or constitutional issues raised to formulate
160310, 160318, 160342, 160343, 160360, 160365, 160370, 160376, 160392, controlling principles to guide the bench, bar, and public.26
160397, 160403 and 160405, November 10, 2003, 415 SCRA 44, 133 citing Angara As a rule, the writ of prohibition will not lie to enjoin acts already done.
v. Electoral Commission, 63 Phil. 139 (1936). However, as an exception to the rule on mootness, courts will decide a question
318favorable action.21 The question on standing is whether such parties have otherwise moot if it is capable of repetition yet evading review. 27 In the present
“alleged such a personal stake in the outcome of the controversy as to assure that case,
concrete adverseness which sharpens the presentation of issues upon which the
court so largely depends for illumination of difficult constitutional questions.”22 _______________
In David v. Macapagal-Arroyo,23 summarizing the rules culled from
jurisprudence, we held that taxpayers, voters, concerned citizens, and legislators 24 David v. Macapagal-Arroyo, supra at 213-214, citing Province of Batangas
may be accorded standing to sue, provided that the following requirements are v. Romulo, G.R. No. 152774, May 27, 2004, 429 SCRA 736, Banco Filipino Savings
met: and Mortgage Bank v. Tuazon, Jr., G.R. No. 132795, March 10, 2004, 425 SCRA
(1) cases involve constitutional issues; 129, Vda. de Dabao v. Court of Appeals, G.R. No. 116526, March 23, 2004, 426
(2) for taxpayers, there must be a claim of illegal disbursement of public funds SCRA 91; Paloma v. Court of Appeals, G.R. No. 145431, November 11, 2003, 415
or that the tax measure is unconstitutional; SCRA 590, Royal Cargo Corporation v. Civil Aeronautics Board, G.R. Nos. 103055-
(3) for voters, there must be a showing of obvious interest in the validity of 56, January 26, 2004, 421 SCRA 21 and Lacson v. Perez, G.R. No. 147780, May 10,
the election law in question; 2001, 357 SCRA 756.
(4) for concerned citizens, there must be a showing that the issues raised 25 G.R. No. 138965, June 30, 2006, 494 SCRA 53.
are of transcendental importance which must be settled early; and 26 Id., at p. 58, citing Province of Batangas v. Romulo, supra at 757
(5) for legislators, there must be a claim that the official action complained and Chavez v. Public Estates Authority, 433 Phil. 506, 522; 384 SCRA 152, 177
of infringes upon their prerogatives as legislators. [EMPHASIS SUPPLIED.] (2002).
Petitioner having alleged a grave violation of the constitutional prohibition 27 Pimentel, Jr. v. Ermita, G.R. No. 164978, October 13, 2005, 472 SCRA 587,
against Members of the Cabinet, their deputies and assistants holding two (2) or 593, citing Tolentino v. Commission on Elections,
more positions in government, the fact that he filed this suit as a concerned citizen 320the mootness of the petition does not bar its resolution. The question of the
sufficiently confers him with standing to sue for redress of such illegal act by public constitutionality of the President’s appointment or designation of a Department
officials. Undersecretary as officer-in-charge of an attached agency will arise in every such
appointment.28
_______________ Undersecretary Bautista’s designation
as MARINA OIC falls under the stricter prohibition
21 Tolentino v. Commission on Elections, 465 Phil. 385, 402; 420 SCRA 438, under Section 13, Article
452 (2004). VII of the 1987 Constitution.
Resolution of the present controversy hinges on the correct application of 1987 Constitution to treat the President and his official family as a class
Section 13, Article VII of the 1987 Constitution, which provides: by itself and to impose upon said class stricter prohibitions.322 Such
“Sec. 13. The President, Vice President, the Members of the Cabinet, intent of the 1986 Constitutional Commission to be stricter with the President and
and their deputies or assistants shall not, unless otherwise provided in his official family was also succinctly articulated by Commissioner Vicente Foz
this Constitution, hold any other office or employment during their after Commissioner Regalado Maambong noted during the floor deliberations and
tenure. They shall not, during said tenure, directly or indirectly practice any other debate that there was no symmetry between the Civil Service prohibitions,
profession, participate in any business, or be financially interested in any contract originally found in the General Provisions and the anticipated report on the
with, or in any franchise, or special privilege granted by the Government or any Executive Department. Commissioner Foz Commented, “We actually have to be
subdivision, agency, or instrumentality thereof, including government-owned or stricter with the President and the members of the Cabinet because they exercise
controlled corporations or their subsidiaries. They shall strictly avoid conflict of more powers and, therefore, more checks and restraints on them are called for
interest in the conduct of their office.” because there is more possibility of abuse in their case.”
On the other hand, Section 7, paragraph (2), Article IX-B reads: Thus, while all other appointive officials in the civil service are
“Sec. 7. x x x allowed to hold other office or employment in the government during
their tenure when such is allowed by law or by the primary functions of
_______________ their positions, members of the Cabinet, their deputies and assistants
may do so only when expressly authorized by the Constitution itself. In
G.R. No. 148334, January 21, 2004, 420 SCRA 438, Gil v. Benipayo, G.R. No. other words, Section 7, Article IX-B is meant to lay down the general rule
148179, June 26, 2001 (Unsigned Resolution), Chief Supt. Acop v. Secretary applicable to all elective and appointive public officials and employees,
Guingona, Jr., 433 Phil. 62; 383 SCRA 577 (2002), Viola v. Hon. Alunan III, 343 while Section 13, Article VII is meant to be the exception applicable only
Phil. 184; 277 SCRA 409 (1997) and Alunan III v. Mirasol, 342 Phil. 467; 276 SCRA to the President, the Vice President, Members of the Cabinet, their
501 (1997). deputies and assistants.
28 Id., at p. 593. xxxx
321 Unless otherwise allowed by law or the primary functions of his Since the evident purpose of the framers of the 1987 Constitution is to impose
position, no appointive official shall hold any other office or employment in the a stricter prohibition on the President, Vice President, members of the Cabinet,
Government or any subdivision, agency or instrumentality thereof, including their deputies and assistants with respect to holding multiple offices or
government-owned or controlled corporations or their subsidiaries.” employment in the government during their tenure, the exception to this
In Civil Liberties Union, a constitutional challenge was brought before this prohibition must be read with equal severity. On its face, the language of Section
Court to nullify EO No. 284 issued by then President Corazon C. Aquino on July 13, Article VII is prohibitory so that it must be understood as intended to be a
25, 1987, which included Members of the Cabinet, undersecretaries and assistant positive and unequivocal negation of the privilege of holding multiple government
secretaries in its provisions limiting to two (2) the positions that appointive offices or employment. Verily, wherever the language used in the constitution is
officials of the Executive Department may hold in government and government prohibitory, it is to be understood as intended to be a positive and unequivocal
corporations. Interpreting the above provisions in the light of the history and times negation. The phrase “unless otherwise provided in this Constitution” must be
and the conditions and circumstances under which the Constitution was framed, given a literal interpretation to refer only to those particular instances cited in the
this Court struck down as unconstitutional said executive issuance, saying that it Constitution itself, to wit: the Vice President being appointed as a member of the
actually allows them to hold multiple offices or employment in direct contravention Cabinet under Section 3, par. (2), Article VII; or acting as President in those
of the express mandate of Section 13, Article VII of the 1987 instances provided under Section 7, pars. (2) and (3), Article VII; and, the Secretary
Constitution prohibiting them from doing so, unless otherwise provided in of Justice
the 1987 Constitution itself. 323being ex-officio member of the Judicial and Bar Council by virtue of Section 8
Noting that the prohibition imposed on the President and his official family is (1), Article VIII.”29 [EMPHASIS SUPPLIED.]
all-embracing, the disqualification was held to be absolute, as the holding of “any Respondent Bautista being then the appointed Undersecretary of DOTC, she
other office” is not qualified by the phrase “in the Government” unlike in Section was thus covered by the stricter prohibition under Section 13, Article VII and
13, Article VI prohibiting Senators and Members of the House of Representatives consequently she cannot invoke the exception provided in Section 7, paragraph 2,
from holding “any other office or employment in the Government”; and when Article IX-B where holding another office is allowed by law or the primary
compared with other officials and employees such as members of the armed forces functions of the position. Neither was she designated OIC of MARINA in an ex
and civil service employees, we concluded thus: officio capacity, which is the exception recognized in Civil Liberties Union.
“These sweeping, all-embracing prohibitions imposed on the President and his The prohibition against holding dual or multiple offices or employment under
official family, which prohibitions are not similarly imposed on other public Section 13, Article VII of the 1987 Constitutionwas held inapplicable to posts
officials or employees such as the Members of Congress, members of the civil occupied by the Executive officials specified therein, without additional
service in general and members of the armed forces, are proof of the intent of the compensation in an ex officiocapacity as provided by law and as required by the
primary functions of said office. The reason is that these posts do not comprise 325
“any other office” within the contemplation of the constitutional prohibition but a. To implement, enforce and apply the policies, programs, standards,
are properly an imposition of additional duties and functions on said guidelines, procedures, decisions and rules and regulations issued,
officials.30 Apart from their bare assertion that respondent Bautista did not receive prescribed or adopted by the Board pursuant to this Decree;
any compensation when she was OIC of MARINA, respondents failed to b. To undertake researches, studies, investigations and other activities and
demonstrate clearly that her designation as such OIC was in an ex projects, on his own initiative or upon instructions of the Board, and to
officio capacity as required by the primary functions of her office as DOTC submit comprehensive reports and appropriate recommendations to the
Undersecretary for Maritime Transport. Board for its information and action;
MARINA was created by virtue of P.D. No. 474 issued by President Ferdinand c. To undertake studies to determine present and future requirements for
E. Marcos on June 1, 1974. It is mandated to undertake the following: port development including navigational aids, and improvement of
(a) Adopt and implement a practicable and coordinated Maritime Industry waterways and navigable waters in consultation with appropriate agencies;
Development Program which shall include, among others, the early d. To pursue continuing research and developmental programs on expansion
replacement of obsolescent and modernization of the merchant fleet and supporting facilities taking
into consideration the needs of the domestic trade and the need of regional
_______________ economic cooperation schemes; and
e. To manage the affairs of the Authority subject to the provisions of this
29 Civil Liberties Union v. Executive Secretary, supra at 328-329, 331. Decree and applicable laws, orders, rules and regulations of other
30 Id., at pp. 331-332. appropriate government entities.
324and uneconomic vessels; modernization and expansion of the Sec. 12. Specific Powers and Functions of the Administrator.—In addition to
Philippine merchant fleet, enhancement of domestic capability for his general powers and functions, the Administrator shall;
shipbuilding, repair and maintenance; and the development of reservoir of a. Issue Certificate of Philippine Registry for all vessels being used in
trained manpower; Philippine waters, including fishing vessels covered by Presidential Decree
(b) Provide and help provide the necessary; (i) financial assistance to the No. 43 except transient civilian vessels of foreign registry, vessels owned
industry through public and private financing institutions and and/or operated by the Armed Forces of the Philippines or by foreign
instrumentalities; (ii) technological assistance; and (iii) in general, a governments for military purposes, and bancas, sailboats and other
favorable climate for expansion of domestic and foreign investments in watercraft which are not motorized, of less than three gross tons;
shipping enterprises; and b. Provide a system of assisting various officers, professionals, technicians,
(c) Provide for the effective supervision, regulation and rationalization of the skilled workers and seamen to be gainfully employed in shipping
organizational management, ownership and operations of all water enterprises, priority being given to domestic needs;
transport utilities, and other maritime enterprises.31 c. In collaboration and coordination with the Department of Labor, to look
The management of MARINA is vested in the Maritime Administrator, who into, and promote improvements in the
shall be directly assisted by the Deputy Administrator for Planning and a Deputy 326working conditions and terms of employment of the officers and crew
Administrator for Operations, who shall be appointed by the President for a term of vessels of Philippine registry, and of such officers and crew members who
of six (6) years. The law likewise prescribes the qualifications for the office, are Philippine citizens and employed by foreign flag vessels, as well as of
including such “adequate training and experience in economics, technology, personnel of other shipping enterprises, and to assist in the settlement of
finance, law, management, public utility, or in other phases or aspects of the disputes between the shipowners and ship operators and such officers and
maritime industry,” and he or she is entitled to receive a fixed annual salary. 32 The crew members and between the owner or manager of other shipping
Administrator shall be directly responsible to the Maritime Industry Board, enterprises and their personnel;
MARINA’s governing body, and shall have powers, functions and duties as d. To require any public water transport utility or Philippine flag vessels to
provided in P.D. No. 474, which provides, under Sections 11 and 12, for his or her provide shipping services to any coastal areas in the country where such
general and specific functions, respectively, as follows: services are necessary for the development of the area, to meet emergency
“Sec. 11. General Powers and Functions of the Administrator.—Subject to sealift requirements, or when public interest so requires;
the general supervision and control of the Board, the Administrators shall have e. Investigate by itself or with the assistance of other appropriate
the following general powers, functions and duties; government agencies or officials, or experts from the private sector, any
matter within its jurisdiction, except marine casualties or accidents which
_______________ shall be undertaken by the Philippine Coast Guard;
f. Impose, fix, collect and receive in accordance with the schedules approved
by the Board, from any shipping enterprise or other persons concerned,
31 P.D. No. 474, Sec. 2.
such fees and other charges for the payment of its services;
32 Id., SECS. 8 and 9.
g. Inspect, at least annually, the facilities of port and cargo operators and of the Philippines General Manager, and the Department of Trade and Industry
recommend measures for adherence to prescribed standards of safety, Secretary.34
quality and operations; Finally, the Court similarly finds respondents’ theory that being just a
h. Approve the sale, lease or transfer of management of vessels owned by “designation,” and temporary at that, respondent Bautista was never really
Philippine Nationals to foreign owned or controlled enterprises; “appointed” as OIC Administrator of MARINA, untenable. In Binamira v.
i. Prescribe and enforce rules and regulations for the prevention of marine Garrucho, Jr.,35 we distinguished between the
pollution in bays, harbors and other navigable waters of the Philippines, in terms appointment and designation, as follows:
coordination with the government authorities concerned; “Appointment may be defined as the selection, by the authority vested with the
j. Establish and maintain, in coordination with the appropriate government power, of an individual who is to exercise the functions of a given office. When
offices and agencies, a system of regularly and promptly producing, completed, usually with its confirmation, the appointment results in security of
collating, analyzing and disseminating traffic flows, port operations, tenure for the person chosen unless he is replaceable at pleasure because of the
marine in- nature of his office. Designation, on the other hand, connotes merely the impo-
327surance services and other information on maritime matters;
k. Recommend such measures as may be necessary for the regulation of the _______________
importation into and exportation from the Philippines of vessels, their
equipment and spare parts; 33 AN ACT PROMOTING THE DEVELOPMENT OF PHILIPPINE DOMESTIC SHIPPING,
l. Implement the rules and regulations issued by the Board of Transportation; SHIPBUILDING, SHIP REPAIR AND SHIP BREAKING, ORDAINING REFORMS IN
m. Compile and codify all maritime laws, orders, rules and regulations, GOVERNMENT POLICIES TOWARDS SHIPPING IN THE PHILIPPINES, AND FOR OTHER
decisions in leasing cases of courts and the Authority’s procedures and PURPOSES, approved on May 3, 2004.
other requirements relative to shipping and other shipping enterprises, 34 Reference: 2006 MARINA Annual Report, sourced from the Internet
make them available to the public, and, whenever practicable to publish athttp://www.marina.gov.ph/services/results.aspx?k=MARINA%20annual%20rep
such materials; ort&start1=1>.
n. Delegate his powers in writing to either of the Deputy Administrators or 35 G.R. No. 92008, July 30, 1990, 188 SCRA 154.
any other ranking officials of the Authority; Provided, That he informs the 329sition by law of additional duties on an incumbent official, as where, in the
Board of such delegation promptly; and case before us, the Secretary of Tourism is designated Chairman of the Board of
o. erform such other duties as the Board may assign, and such acts as may be Directors of the Philippine Tourism Authority, or where, under the Constitution,
necessary and proper to implement this Decree. three Justices of the Supreme Court are designated by the Chief Justice to sit in
With the creation of the Ministry (now Department) of Transportation and the Electoral Tribunal of the Senate or the House of Representatives. It is said that
Communications by virtue of EO No. 546, MARINA was attached to the DOTC for appointment is essentially executive while designation is legislative in nature.
policy and program coordination on July 23, 1979. Its regulatory function was Designation may also be loosely defined as an appointment because it likewise
likewise increased with the issuance of EO No. 1011 which abolished the Board of involves the naming of a particular person to a specified public office. That is the
Transportation and transferred the quasi-judicial functions pertaining to water common understanding of the term. However, where the person is merely
transportation to MARINA. On January 30, 1987, EO No. 125 (amended by EO designated and not appointed, the implication is that he shall hold the
No. 125-A) was issued reorganizing the DOTC. The powers and functions of the office only in a temporary capacity and may be replaced at will by the appointing
department and the agencies under its umbrella were defined, further increasing authority. In this sense, the designation is considered only an acting or temporary
the responsibility of MARINA to the industry. Republic Act No. 9295, otherwise appointment, which does not confer security of tenure on the person
known as the “The Domestic Shipping Development Act of named.”36 [emphasis supplied.]
3282004,”33 further strengthened MARINA’s regulatory powers and functions in Clearly, respondents’ reliance on the foregoing definitions is misplaced
the shipping sector. considering that the above-cited case addressed the issue of whether petitioner
Given the vast responsibilities and scope of administration of the Authority, therein acquired valid title to the disputed position and so had the right to security
we are hardly persuaded by respondents’ submission that respondent Bautista’s of tenure. It must be stressed though that while the designation was in the nature
designation as OIC of MARINA was merely an imposition of additional duties of an acting and temporary capacity, the words “hold the office” were employed.
related to her primary position as DOTC Undersecretary for Maritime Transport. Such holding of office pertains to both appointment and designation because the
It appears that the DOTC Undersecretary for Maritime Transport is not even a appointee or designate performs the duties and functionsof the office. The 1987
member of the Maritime Industry Board, which includes the DOTC Secretary as Constitution in prohibiting dual or multiple offices, as well as incompatible offices,
Chairman, the MARINA Administrator as Vice-Chairman, and the following as refers to the holding of the office, and not to the nature of the appointment or
members: Executive Secretary (Office of the President), Philippine Ports Authority designation, words which were not even found in Section 13, Article VII nor in
General Manager, Department of National Defense Secretary, Development Bank Section 7, paragraph 2, Article IX-B. To “hold” an office means to “possess or
occupy” the same, or “to be in possession and administration,” 37 which implies assistant secretaries as officers-in-charge of government agencies,
nothing less than the actual discharge of the functions and duties of the office. instrumentalities, or government-owned or controlled corporations.
As to respondents’ contention that the concurrent positions of DOTC
_______________ Undersecretary for Maritime Transport and MARINA OIC Administrator are not
incompatible offices, we find no necessity for delving into this matter.
36 Id., at pp. 158-159. Incompatibility of offices is irrelevant in this case, unlike in the case of PCGG
37 BLACK’S LAW DICTIONARY, Eighth Edition, p. 749. Chairman Magdangal Elma in Public Interest Center, Inc. v. Elma.40 Therein we
330 The disqualification laid down in Section 13, Article VII is aimed at held that Section 13, Article VII is not applicable to the PCGG Chairman or to the
preventing the concentration of powers in the Executive Department officials, Chief Presidential Legal Counsel, as he is not a cabinet member, undersecretary
specifically the President, Vice President, Members of the Cabinet and their or assistant secretary.41
deputies and assistants. Civil Liberties Union traced the history of the times and WHEREFORE, the petition is GRANTED. The designation of respondent Ma.
the conditions under which the Constitution was framed, and construed the Elena H. Bautista as Officer-in-Charge, Office of the Administrator, Maritime
Constitution consistent with the object sought to be accomplished by adoption of Industry Authority, in a concurrent capacity with her position as DOTC
such provision, and the evils sought to be avoided or remedied. We recalled the Undersecretary for Maritime Transport, is hereby declared
practice, during the Marcos regime, of designating members of the Cabinet, their UNCONSTITUTIONAL
deputies and assistants as members of the governing bodies or boards of various
government agencies and instrumentalities, including government-owned or _______________
controlled corporations. This practice of holding multiple offices or positions in the
government led to abuses by unscrupulous public officials, who took advantage of 39 Id., at p. 327.
this scheme for purposes of self-enrichment. The blatant betrayal of public trust 40 Supra note 6.
evolved into one of the serious causes of discontent with the Marcos regime. It was 41 Id., at p. 62.
therefore quite inevitable and in consonance with the overwhelming sentiment of 332for being violative of Section 13, Article VII of the 1987 Constitution and
the people that the 1986 Constitutional Commission would draft into the proposed therefore, NULL and VOID.
Constitution the provisions under consideration, which were envisioned to remedy, No costs.
if not correct, the evils that flow from the holding of multiple governmental offices SO ORDERED.
and employment.38 Our declaration in that case cannot be more explicit: Puno (C.J.), Carpio, Velasco, Jr., Nachura, Leonardo-De Castro, Brion,
“But what is indeed significant is the fact that although Section 7, Article IX- Peralta, Bersamin, Del Castillo, Abad, Perez andMendoza, JJ., concur.
B already contains a blanket prohibition against the holding of multiple offices or Corona, J., No Part.
employment in the government subsuming both elective and appointive public Carpio-Morales, J., Please see concurring opinion.
officials, the Constitutional Commission should see it fit to formulate another
provision, Sec. 13, Article VII, specifically prohibiting the President, Vice CONCURRING OPINION
President, members of the Cabinet, their deputies and assistants from
CARPIO-MORALES, J.:
_______________ I concur with Justice Martin Villarama, Jr. in his ponenciadeclaring
unconstitutional the designation of respondent Maria Elena Bautista (Bautista) as
38 Civil Liberties Union v. Executive Secretary, supra at 326-327. Officer-in-Charge (OIC) of the Office of the Administrator of the Maritime Industry
331holding any other office or employment during their tenure, unless otherwise Authority (MARINA) in a concurrent capacity with her position as Undersecretary
provided in the Constitution itself. for Maritime Transport of the Department of Transportation and Communications
Evidently, from this move as well as in the different phraseologies of the (DOTC).
constitutional provisions in question, the intent of the framers of the A quick rundown of the facts shows that Bautista was appointed as DOTC
Constitution was to impose a stricter prohibition on the President and Undersecretary in October 2006 and was designated as OIC Administrator of
his official family in so far as holding other offices or employment in the MARINA on September 1, 2008. On January 5, 2009, she was appointed as
government or elsewhere is concerned.”39 [emphasis supplied.] Administrator of MARINA, the duties and responsibilities of which position she
Such laudable intent of the law will be defeated and rendered sterile if we are assumed on February 2, 2009 following her relinquishment of the position of DOTC
to adopt the semantics of respondents. It would open the veritable floodgates of Undersecretary.
circumvention of an important constitutional disqualification of officials in the Bautista thus now claims mootness of the case. A moot and academic case is
Executive Department and of limitations on the President’s power of appointment one that ceases to present a justiciable controversy by virtue of supervening events,
in the guise of temporary designations of Cabinet Members, undersecretaries and so that a declaration thereon would be of no practical use or value. Aside from the
formulation of controlling principles, the grave violation of the Constitution, and that he shall hold the office only in a temporary capacity and may be replaced at
the susceptibility of recurrence as pointed will by the appointing authority.
333out by Justice Villarama, there is the presence of practical use or value to 4 Vide Lino Luna v. Rodriguez and De los Angeles, 37 Phil. 186, 192 (1917).
impel the Court to take cognizance of this case. 5 An obiter dictum has been defined as an opinion expressed by a court upon
Its mootness notwithstanding, the present petition which involves the issue of some question of law which is not necessary to the decision of the case before it. It
holding dual positions still calls for a resolution, for there remains the practical is a remark made, or opinion expressed, by a judge, in his decision upon a cause,
use or value of identifying whether one was a de facto or de jure officer in terms of “by the way,” that is, incidentally or collaterally, and not directly upon the question
the legal signification of the public officer’s acts, remuneration and accountability. before him, or upon a point not necessarily involved in the determination of the
Bautista, during her tenure as OIC Administrator of MARINA, cannot be cause, or introduced by way of illustration, or analogy or argument. Such are not
considered as a de jure officer due to the unconstitutionality of the designation. At binding as precedent. (Delta Motors Corporation v. Court of Appeals, G.R. No.
best, she can be regarded as a de facto officer in such capacity from September 1, 121075, July 24, 1997, 276 SCRA 212, 223).
2008 until she assumed her subsequent appointment as MARINA Administrator 6 With respect to the exception enunciated in the Civil Liberties Union case
on February 2, 2009. allowing posts occupied by the Executive officials specified therein without
National Amnesty Commission v. Commission on Audit1 espouses the view additional compensation in an ex officio capacity as provided by law and
that one who was not appointed but merely designated to act as such cannot be as required by the primary functions of said officials’ office.
considered as a de facto officer. To sustain this view, however, would place in limbo 335
the legal effects of a designated officer’s acts and would negate the raison d’etre of Civil Liberties Union vis-à-vis Public Interest Center
the de factodoctrine which is basically to protect the sanctity of dealings by the With respect to the legal complexion of Bautista’s position as DOTC
public with persons whose ostensible authority emanates from the State. 2 To Undersecretary, there is a need to explore the implication of nullifying the holding
deduce that Bautista, as a designated OIC Administrator, was not a de facto officer of a second position.
would effectively categorize her as an intruder or a mere volunteer, which she was Where a person is prohibited from holding two offices at the same time, his
not because she had a color of right or authority. appointment or election to a second office may operate to vacate the first or he may
A de facto officer need not show that she was elected or “appointed in its strict be ineligible for the second.7
sense,” for a showing of a color of right to the office suffices. The proposition that a person shall be declared ineligible for the second
Designation may be loosely defined as an appointment because it likewise position was followed in Civil Liberties Union v. Executive Secretary8 where the
involves the naming of a particular person to Court ordered certain cabinet members, except those who were no longer
occupying the positions complained of, “to immediately relinquish their other
_______________ offices or employment, as herein defined, in the government, including
government-owned and controlled corporations and their subsidiaries.”9
1 G.R. No. 156982, September 8, 2004, 437 SCRA 655, 670. Under this principle, Bautista would only be directed to relinquish
2 Vide Topacio v. Ong, G.R. No. 179895, December 18, 2008, 574 SCRA 817, the post of MARINA Administrator, if still being occupied, and
830. concentrate on her functions as DOTC Undersecretary.
334a specified public office.3 In fact, even without a known appointment or The other proposition—that a person who assumes a second and incompatible
election, the de facto doctrine comes into play if the duties of the office were office is deemed to have resigned from the first office—was applied in Public
exercised under such circumstances of reputation or acquiescence as were Interest Center, Inc. v. Elma10 where the Court, by Resolution of March 5, 2007,
calculated to induce people, without inquiry, to submit to or invoke his action, clarified that the ruling did not render both appointments void. It held that
supposing him to be the officer he assumed to be.4 “[f]ollowing the common-law rule on incompatibility of offices, respondent
I submit that the pronouncement in National Amnesty Commission comes in Elma had, in effect, vacated his first
the form of an obiter dictum5 since it was not necessary to the disposition of that
case where the Court disallowed the payment of honoraria to the representatives _______________
of the ex-officiomembers of the National Amnesty Commission and ruled that the
restrictions6 covering the ex-officio members apply with equal force to their 7 HECTOR DE LEON & HECTOR DE LEON, JR., THE LAW ON PUBLIC OFFICERS AND
representatives since the representative cannot have a better right than his or her ELECTION LAW. 45 (2000).
principal. 8 G.R. No. 83896, February 22, 1991, 194 SCRA 317.
9 Id., at p. 339.
_______________ 10 G.R. No. 138965, June 30, 2006, 494 SCRA53.
336office as PCGG Chairman when he accepted the second office”11 as Chief
3 Binamira v. Garrucho, Jr., G.R. No. 92008, July 30, 1990, 188 SCRA 154, Presidential Legal Counsel.
159, where the person is merely designated and not appointed, the implication is
Under this rule, Bautista would be deemed to have vacated her first 12 SECTION 13. The President, Vice President, the Members of the
office as DOTC Undersecretary when she accepted the post of OIC Cabinet, and their deputies or assistants shall not, unless otherwise provided in
Administrator of MARINA. this Constitution, hold any other office or employment during their tenure. x x x.
The Implications of the Two Propositions 13 SECTION 7. No elective official shall be eligible for appointment or
Upon a closer examination of Public Interest Center, Inc. which espouses designation in any capacity to any public office or position during his tenure.
the ipso facto vacancy rule, there appears a vacuity in such a situation where the Unless otherwise allowed by law or by the primary functions of his position, no
Court nullifies the appointment to a second office for being unconstitutional and appointive official shall hold any other office or employment in the Government or
likewise deems the first office as having been vacated. In the end, the public officer any subdivision, agency or instrumentality thereof, including government-owned
is left without an office. or controlled corporations or their subsidiaries.
In the present case, Bautista eventually voluntarily gave up her first post 338dent, Vice President, the Members of the Cabinet, and their deputies or
when she was subsequently appointed as MARINA Administrator, after five assistants” as a class by itself, necessitating the disallowance of any implied
months of concurrently discharging the functions of an appointed DOTC vacancy in such offices.
Undersecretary and a designated MARINA Officer-in-Charge. It bears noting that The Pubic Interest Center rule of implied resignation does not apply since it
what is being nullified is her designation and not the subsequent appointment as speaks of “incompatibility of office” which is irrelevant in determining a violation
Administrator. Her current position as MARINA Administrator was conferred not of Section 13, Article VII of the Constitution.
by virtue of the assailed designation but by the subsequent appointment which It has also been observed that the rule of ipso facto vacancy of a public office
effectively stands. Thus, notwithstanding the implication of Public Interest Center, by acceptance of a second public office does not apply where, under applicable
the scenario of vacancy will not occur in this peculiar case. constitutional or statutory provisions, the holder of a public office is rendered
With respect to the proposition under Civil Liberties Union—ineligibility for ineligible for a specified time for a second public office; under such circumstances
the second position only—the only peculiarity of the present case is that the it is the second office which is considered vacant rather than the first office.14
reverse thing transpired in the meantime, with Bautista giving up the I, therefore, vote to GRANT the petition and further declare that Bautista was
Undersecretary position and accepting the subsequent regular appointment as a de facto officer during her brief stint as MARINA OIC Administrator and a de
MARINA Administrator. The supposed continued validity jure DOTC Undersecretary during her entire tenure as such.
Concluding Words
_______________ The present case, in which the constitutional question posed is no longer an
unchartered sea, should once again remind all civil servants of the rationale
11 G.R. No. 138965, March 5, 2007, 517 SCRA 336, 339. behind the general rule against the holding of multiple positions.
337of her position as DOTC Undersecretary has been rendered nugatory by her One manifest purpose of a restriction on multiple holdings is to prevent offices
voluntary relinquishment of said position. of public trust from accumulating in a single person.15 Indeed, no one can claim a
Further quandary lies in the five-month interregnum. monopoly of skills.
On the one hand, following the Public Interest Center rule that deems her first “Being head of an executive department is no mean job. It is more than a full-
office vacated upon her holding of a second position, Bautista had become a de time job, requiring full attention, specialized knowledge, skills and expertise. If
facto DOTC Undersecretary from September 1, 2008 (when she assumed the maximum benefits are to be derived from a department head’s ability and
position of MARINA OIC Administrator) until she resigned therefrom. On the expertise, he should be allowed
other hand, following the Civil Liberties Union rule that merely deems her
ineligible for the second position, Bautista remained a de jure DOTC _______________
Undersecretary during her entire tenure as such.
IN FINE, I submit that the two cases provide sound formulations for two 14 63C Am. Jur. 2d §61 p. 504, that is, not merely on the ground of the
distinct situations. The Civil Liberties Union rule applies to cases involving incompatibility of office.
dual or multiple positions under Section 13 of Article VII of the 15 Supra note 7 at 45.
Constitution12 while the Public Interest Centerrule covers those under Section 339to attend to his duties and responsibilities without the distraction of other
7 of Article IX-B of the Constitution.13 governmental offices or employment. He should be precluded from dissipating his
The Civil Liberties Union formulation rendering the public officer ineligible for efforts, attention and energy among too many positions of responsibility, which
the second position comes into play, since Bautista was a department may result in haphazardness and inefficiency. Surely the advantages to be derived
undersecretary, a position covered by the prohibition under Section 13, Article VII from this concentration of attention, knowledge and expertise, particularly at this
of the Constitution. This principle underscores the primacy of the “Presi- stage of our national and economic development, far outweigh the benefits, if any,
that may be gained from a department head spreading himself too thin and taking
_______________ in more than what he can handle.”16
The same norm holds true to that of a DOTC Undersecretary for Maritime Remedial Law; Civil Procedure; Locus Standi; To have legal standing,
Transport. Now as always, the country cannot afford to have a public official who therefore, a suitor must show that he has sustained or will sustain a “direct injury”
cannot devote full time on the crucial problems, contemporary or longstanding, not as a result of a government action, or have a “material interest” in the issue affected
to mention the perennial sea tragedies, that have beleaguered the maritime by the challenged official act.—The OSG does not dispute the justiciability and
industry, an industry that is “indubitably imbued with national interest.”17 ripeness for consideration and resolution by the Court of the matter raised by the
Petition granted, designation of Ma. Elena H. Bautista as OIC, Office of petitioner. Also, the locus standi of the petitioner as a taxpayer, a concerned
Administrator, MARINA, concurrently as DOTC Undersecretary for Marine citizen and a lawyer to bring a suit of this nature has already been settled in his
Transportation declared unconstitutional. favor in rulings by the Court on several other public law litigations he brought.
Note.—Following the common-law rule on incompatibility of offices, In Funa v. Villar, 670 SCRA 579 (2012), for one, the Court has held: To have legal
respondent Elma had, in effect, vacated his first office as Presidential Commission standing, therefore, a suitor must show that he has sustained or will sustain a
on Good Government (PCGG) Chairman when he accepted the second office as “direct injury” as a result of a government action, or have a “material interest” in
Chief Presidential Legal Counsel. (Public Interest Center, Inc. vs. Elma, 517 SCRA the issue affected by the challenged official act. However, the Court has time
336 [2007]) and again acted liberally on the locus standi requirements and has
——o0o—— accorded certain individuals, not otherwise directly injured, or with
material interest affected, by a Government act, standing to sue provided
_______________ a constitutional issue of critical significance is at stake. The rule on locus
standi is after all a mere procedural technicality in relation to which the
16 Civil Liberties Union v. Executive Secretary, supra note 8 at 339. Court, in a catena of cases involving a subject of transcendental import,
17 Trans-Asia Shipping Lines, Inc.-Unlicensed Crews Employees Union— has waived, or relaxed, thus allowing non-traditional plaintiffs, such as
Assisted Labor Unions (Tasli-Alu) v. Court of Appeals, G.R. No. 145428, July 7, concerned citizens, taxpayers, voters or legislators, to sue in the public
2004, 433 SCRA 610, 621. interest, albeit they may not have been personally injured by the
© Copyright 2018 Central Book Supply, Inc. All rights reserved. operation of a law or any other government act. In David, the Court laid
out the bare minimum norm before the so-called “non-traditional suitors”
may be extended standing to sue, thusly: 1.) For taxpayers, there must be a
claim of illegal disbursement of public funds or that the tax measure is
unconstitutional; 2.) For voters, there must be a showing of obvious interest in the
validity of the election law in question; 3.) For concerned citizens, there must be a
showing that the issues raised are of transcendental importance which must be
G.R. No. 191644. February 19, 2013.* settled early; and 4.) For legislators, there must be a claim that the official action
DENNIS A.B. FUNA, petitioner, vs. ACTING SECRETARY OF JUSTICE complained of infringes their prerogatives as legislators. This case before Us is
ALBERTO C. AGRA, IN HIS OFFICIAL CONCURRENT CAPACITIES AS of transcendental importance, since it obviously has “far-reaching
ACTING SECRETARY OF THE DEPARTMENT OF JUSTICE AND AS ACTING implications,” and there is a need to promulgate rules that will guide the
SOLICITOR GENERAL, EXECUTIVE SECRETARY LEANDRO R. MENDOZA, bench, bar, and the public in future analogous cases. We, thus, assume a
OFFICE OF THE PRESIDENT, respondents. liberal stance and allow petitioner to institute the instant petition.198
Constitutional Law; Judicial Review; Limitations to the Power of Judicial 198 SUPREME COURT REPORTS ANNOTATED
Review.—The power of judicial review is subject to limitations, to wit: (1) there
must be an actual case or controversy calling for the exercise of judicial power; (2) Funa vs. Agra
the person challenging the act must have the standing to assail the validity of the Same; Same; Moot and Academic; Words and Phrases; A moot and academic
subject act or issuance, that is, he must have a personal and substantial interest case is one that ceases to present a justiciable controversy by virtue of supervening
in the case such that he has sustained, or will sustain, direct injury as a result of events, so that a declaration thereon would be of no practical use or value.—A moot
its enforcement; (3) the question of constitutionality must be raised at the earliest and academic case is one that ceases to present a justiciable controversy by virtue
opportunity; and (4) the issue of constitutionality must be the very lis mota of the of supervening events, so that a declaration thereon would be of no practical use
case. or value. Although the controversy could have ceased due to the intervening
_______________ appointment of and assumption by Cadiz as the Solicitor General during the
* EN BANC. pendency of this suit, and such cessation of the controversy seemingly rendered
197 moot and academic the resolution of the issue of the constitutionality of the
VOL. 691, FEBRUARY 19, 2013 197 concurrent holding of the two positions by Agra, the Court should still go forward
and resolve the issue and not abstain from exercising its power of judicial review
Funa vs. Agra because this case comes under several of the well-recognized exceptions
established in jurisprudence. Verily, the Court did not desist from resolving an
issue that a supervening event meanwhile rendered moot and academic if any of Secretary, 194 SCRA 317 (1991), whereby the Court held that the phrase “the
the following recognized exceptions obtained, namely: (1) there was a grave Members of the Cabinet, and their deputies or assistants” found in Section 13,
violation of the Constitution; (2) the case involved a situation of exceptional 1987 Constitution, referred only to the heads of the various executive departments,
character and was of paramount public interest; (3) the constitutional issue raised their undersecretaries and assistant secretaries, and did not extend to other public
required the formulation of controlling principles to guide the Bench, the Bar and officials given the rank of Secretary, Undersecretary or Assistant Secretary.
the public; and (4) the case was capable of repetition, yet evading review. Hence, in Public Interest Center, Inc. v. Elma, 494 SCRA 53 (2006), the Court
Administrative Law; To Hold an Office; Words and Phrases; To hold an office opined that the prohibition under Section 13 did not cover Elma, a Presidential
means to possess or to occupy the office, or to be in possession and administration Assistant with the rank of Undersecretary.200
of the office, which implies nothing less than the actual discharge of the functions 200 SUPREME COURT REPORTS ANNOTATED
and duties of the office.—It was of no moment that Agra’s designation was in an
acting or temporary capacity. The text of Section 13, 1987 Constitution, plainly Funa vs. Agra
indicates that the intent of the Framers of the Constitution was to impose a stricter Same; Office of the Solicitor General; The powers and functions of the Office
prohibition on the President and the Members of his Cabinet in so far as holding of the Solicitor General are neither required by the primary functions nor included
other offices or employments in the Government or in government-owned or by the powers of the Department of Justice, and vice versa.—Indeed, the powers
government controlled-corporations was concerned. In this regard, to hold an office and functions of the OSG are neither required by the primary functions nor
means to possess or to occupy the office, or to be in possession and administration included by the powers of the DOJ, and vice versa. The OSG, while attached to the
of the office, which implies nothing less than the actual discharge of the functions DOJ, is not a constituent unit of the latter, as, in fact, the Administrative Code of
and duties of the office. Indeed, in the language of Section 13 itself, 1987 1987 decrees that the OSG is independent and autonomous. With the enactment
Constitution, the Constitution makes no reference to the nature of the of Republic Act No. 9417, the Solicitor General is now vested with a cabinet rank,
appointment or designation. The prohibition against dual or multiple offices being and has the same qualifications for appointment, rank, prerogatives, salaries,
held by one allowances, benefits and privileges as those of the Presiding Justice of the Court
199 of Appeals. Moreover, the magnitude of the scope of work of the Solicitor General,
if added to the equally demanding tasks of the Secretary of Justice, is obviously
VOL. 691, FEBRUARY 19, 2013 199
too much for any one official to bear. Apart from the sure peril of political pressure,
Funa vs. Agra the concurrent holding of the two positions, even if they are not entirely
official must be construed as to apply to all appointments or designations, incompatible, may affect sound government operations and the proper
whether permanent or temporary, for it is without question that the avowed performance of duties.
objective of Section 13, 1987 Constitution, is to prevent the concentration of powers Same; Same; The primary functions of the Office of the Solicitor General are
in the Executive Department officials, specifically the President, the Vice- not related or necessary to the primary functions of the Department of Justice.—
President, the Members of the Cabinet and their deputies and assistants. To Clearly, the primary functions of the Office of the Solicitor General are not related
construe differently is to “open the veritable floodgates of circumvention of an or necessary to the primary functions of the Department of Justice. Considering
important constitutional disqualification of officials in the Executive Department that the nature and duties of the two offices are such as to render it improper, from
and of limitations on the President’s power of appointment in the guise of considerations of public policy, for one person to retain both, an incompatibility
temporary designations of Cabinet Members, undersecretaries and assistant between the offices exists, further warranting the declaration of Agra’s designation
secretaries as officers-in-charge of government agencies, instrumentalities, or as the Acting Secretary of Justice, concurrently with his designation as the Acting
government-owned or controlled corporations.” Solicitor General, to be void for being in violation of the express provisions of the
Same; Multiple Offices; The only two exceptions against the holding of Constitution.
multiple offices are: (1) those provided for under the Constitution, such as Section Same; Public Officers; De Facto Officers; Words and Phrases; A de facto
3, Article VII, authorizing the Vice President to become a member of the Cabinet; officer is one who derives his appointment from one having colorable authority to
and (2) posts occupied by Executive officials specified in Section 13, Article VII appoint, if the office is an appointive office, and whose appointment is valid on its
without additional compensation in ex officio capacities as provided by law and as face. He may also be one who is in possession of an office, and is discharging its
required by the primary functions of the officials’ offices.—According to Public duties under color of authority, by which is meant authority derived from an
Interest Center, Inc. v. Elma, 494 SCRA 53 (2006), the only two exceptions against appointment, however irregular or informal, so that the incumbent is not a mere
the holding of multiple offices are: (1) those provided for under the Constitution, volunteer.—A de facto officer is one who derives his appointment
such as Section 3, Article VII, authorizing the Vice President to become a member 201
of the Cabinet; and (2) posts occupied by Executive officials specified in Section 13, VOL. 691, FEBRUARY 19, 2013 201
Article VII without additional compensation in ex officio capacities as provided by
law and as required by the primary functions of the officials’ offices. In this regard, Funa vs. Agra
the decision in Public Interest Center, Inc. v. Elma, 494 SCRA 53 (2006), adverted from one having colorable authority to appoint, if the office is an appointive
to the resolution issued on August 1, 1991 in Civil Liberties Union v. The Executive office, and whose appointment is valid on its face. He may also be one who is in
possession of an office, and is discharging its duties under color of authority, by appointments or designations, claiming it to be prohibited under Section 13,
which is meant authority derived from an appointment, however irregular or Article VII of the 1987 Constitution; that during the pendency of the suit,
informal, so that the incumbent is not a mere volunteer. Consequently, the acts of President Benigno S. Aquino III appointed Atty. Jose Anselmo I. Cadiz as the
the de facto officer are just as valid for all purposes as those of a de jure officer, in Solicitor General; and that Cadiz assumed as the Solicitor General and commenced
so far as the public or third persons who are interested therein are concerned. his duties as such on August 5, 2010.2
Same; Same; Same; The Supreme Court holds that all official actions of Agra _______________
as a de facto Acting Secretary of Justice, assuming that was his later designation, 1 Rollo, p. 13.
were presumed valid, binding and effective as if he was the officer legally appointed 2 Id., at p. 172.
and qualified for the office.—In order to be clear, therefore, the Court holds that 203
all official actions of Agra as a de factoActing Secretary of Justice, assuming that VOL. 691, FEBRUARY 19, 2013 203
was his later designation, were presumed valid, binding and effective as if he was
the officer legally appointed and qualified for the office. This clarification is Funa vs. Agra
necessary in order to protect the sanctity of the dealings by the public with persons Agra renders a different version of the antecedents. He represents that on
whose ostensible authority emanates from the State. Agra’s official actions covered January 12, 2010, he was then the Government Corporate Counsel when President
by this clarification extend to but are not limited to the promulgation of resolutions Arroyo designated him as the Acting Solicitor General in place of Solicitor General
on petitions for review filed in the Department of Justice, and the issuance of Devanadera who had been appointed as the Secretary of Justice;3 that on March
department orders, memoranda and circulars relative to the prosecution of 5, 2010, President Arroyo designated him also as the Acting Secretary of Justice
criminal cases. vice Secretary Devanadera who had meanwhile tendered her resignation in order
SPECIAL CIVIL ACTION in the Supreme Court. Certiorari and Prohibition. to run for Congress representing a district in Quezon Province in the May 2010
The facts are stated in the opinion of the Court. elections; that he then relinquished his position as the Government Corporate
Funa Balayan Fortes Galandines & Villagonzalo Law Counsel; and that pending the appointment of his successor, Agra continued to
Offices for petitioner. perform his duties as the Acting Solicitor General.4
Lally C. Ortilla for respondent Alberto C. Agra. Notwithstanding the conflict in the versions of the parties, the fact that Agra
BERSAMIN, J.: has admitted to holding the two offices concurrently in acting capacities is settled,
Section 13, Article VII of the 1987 Constitution expressly prohibits the which is sufficient for purposes of resolving the constitutional question that
President, Vice-President, the Members of the Cabinet, and their deputies or petitioner raises herein.
assistants from holding any other office or employment during their tenure unless The Case
other- In Funa v. Ermita,5 the Court resolved a petition for certiorari, prohibition and
202 mandamus brought by herein petitioner assailing the constitutionality of the
designation of then Undersecretary of the Department of Transportation and
202 SUPREME COURT REPORTS ANNOTATED
Communications (DOTC) Maria Elena H. Bautista as concurrently the Officer-in-
Funa vs. Agra Charge of the Maritime Industry Authority. The petitioner has adopted here the
wise provided in the Constitution. Complementing the prohibition is Section 7, arguments he advanced in Funa v. Ermita, and he has rested his grounds of
paragraph (2), Article IX-B of the 1987 Constitution, which bans any appointive challenge
official from holding any other office or employment in the Government or any _______________
subdivision, agency or instrumentality thereof, including government-owned or 3 Id., at p. 76.
controlled corporations or their subsidiaries, unless otherwise allowed by law or 4 Id., at p. 77.
the primary functions of his position. 5 G.R. No. 184740, February 11, 2010, 612 SCRA 308.
These prohibitions under the Constitution are at the core of this special civil 204
action for certiorari and prohibition commenced on April 7, 2010 to assail the 204 SUPREME COURT REPORTS ANNOTATED
designation of respondent Hon. Alberto C. Agra, then the Acting Secretary of
Justice, as concurrently the Acting Solicitor General. Funa vs. Agra
Antecedents mainly on the pronouncements in Civil Liberties Union v. Executive
The petitioner alleges that on March 1, 2010, President Gloria M. Macapagal- Secretary6 and Public Interest Center, Inc. v. Elma.7
Arroyo appointed Agra as the Acting Secretary of Justice following the resignation What may differentiate this challenge from those in the others is that the
of Secretary Agnes VST Devanadera in order to vie for a congressional seat in appointments being hereby challenged were in acting or temporary capacities.
Quezon Province; that on March 5, 2010, President Arroyo designated Agra as the Still, the petitioner submits that the prohibition under Section 13, Article VII of
Acting Solicitor General in a concurrent capacity;1 that on April 7, 2010, the the 1987 Constitution does not distinguish between an appointment or designation
petitioner, in his capacity as a taxpayer, a concerned citizen and a lawyer, of a Member of the Cabinet in an acting or temporary capacity, on the one hand,
commenced this suit to challenge the constitutionality of Agra’s concurrent and one in a permanent capacity, on the other hand; and that Acting Secretaries,
being nonetheless Members of the Cabinet, are not exempt from the constitutional 11 Id., at p. 87.
ban. He emphasizes that the position of the Solicitor General is not an ex 12 Id., at pp. 91, 100.
officio position in relation to the position of the Secretary of Justice, considering 13 Id., at p. 94.
that the Office of the Solicitor General (OSG) is an independent and autonomous 14 Id.
office attached to the Department of Justice (DOJ). 8 He insists that the fact that 15 Id., at p. 126.
Agra was extended an appointment as the Acting Solicitor General shows that he 16 Id., at pp. 128-129.
did not occupy that office in an ex officiocapacity because an ex officio position does 206
not require any further warrant or appointment. 206 SUPREME COURT REPORTS ANNOTATED
Respondents contend, in contrast, that Agra’s concurrent designations as the
Acting Secretary of Justice and Acting Solicitor General were only in a temporary Funa vs. Agra
capacity, the only effect of which was to confer additional duties to him. Thus, as as to the time of his designation as the Acting Solicitor General and Acting
the Acting Solicitor General and Acting Secretary of Justice, Agra was not Secretary of Justice; that Agra’s concurrent designations further violated
“holding” both offices in the strict constitutional sense. 9 They argue that an the Administrative Code of 1987 which mandates that the OSG shall be
appointment, to be covered by the constitutional prohibition, must be regular and autonomous and independent.17
permanent, instead of a mere designation. Issue
_______________ Did the designation of Agra as the Acting Secretary of Justice, concurrently
6 G.R. Nos. 83896 and 83815, February 22, 1991, 194 SCRA 317. with his position of Acting Solicitor General, violate the constitutional prohibition
7 G.R. No. 138965, June 30, 2006, 494 SCRA 53. against dual or multiple offices for the Members of the Cabinet and their deputies
8 Section 34, Chapter 12, Title III, Book 4 of the Administrative Code of 1987. and assistants?
9 Rollo, p. 83. Ruling
205 The petition is meritorious.
The designation of Agra as Acting Secretary of Justice concurrently with his
VOL. 691, FEBRUARY 19, 2013 205
position of Acting Solicitor General was unconstitutional and void for being in
Funa vs. Agra violation of the constitutional prohibition under Section 13, Article VII of the 1987
Respondents further contend that, even on the assumption that Agra’s Constitution.
concurrent designation constituted “holding of multiple offices,” his continued 1.
service as the Acting Solicitor General was akin to a hold-over; that upon Agra’s Requisites of judicial review not in issue
designation as the Acting Secretary of Justice, his term as the Acting Solicitor The power of judicial review is subject to limitations, to wit: (1) there must be
General expired in view of the constitutional prohibition against holding of an actual case or controversy calling for the exercise of judicial power; (2) the
multiple offices by the Members of the Cabinet; that under the principle of hold- person challenging the act must have the standing to assail the validity of the
over, Agra continued his service as the Acting Solicitor General “until his successor subject act or issuance, that is, he must have a personal and substantial interest
is elected and qualified”10 to “prevent a hiatus in the government pending the time in the case such that he has sustained, or will sustain, direct injury as a result of
when a successor may be chosen and inducted into office”; 11 and that during his its enforcement; (3) the question of constitutionality must be raised at the earliest
continued service as the Acting Solicitor General, he did not receive any salaries oppor-
and emoluments from the OSG after becoming the Acting Secretary of Justice on _______________
March 5, 2010.12 17 Id., at p. 137.
Respondents point out that the OSG’s independence and autonomy are defined 207
by the powers and functions conferred to that office by law, not by the person VOL. 691, FEBRUARY 19, 2013 207
appointed to head such office;13 and that although the OSG is attached to the DOJ,
the DOJ’s authority, control and supervision over the OSG are limited only to Funa vs. Agra
budgetary purposes.14 tunity; and (4) the issue of constitutionality must be the very lis motaof the case.18
In his reply, petitioner counters that there was no “prevailing special Here, the OSG does not dispute the justiciability and ripeness for consideration
circumstance” that justified the non-application to Agra of Section 13, Article VII and resolution by the Court of the matter raised by the petitioner. Also, the locus
of the 1987 Constitution;15 that the temporariness of the appointment or standi of the petitioner as a taxpayer, a concerned citizen and a lawyer to bring a
designation is not an excuse to disregard the constitutional ban against holding of suit of this nature has already been settled in his favor in rulings by the Court on
multiple offices by the Members of the Cabinet;16 that Agra’s invocation of the several other public law litigations he brought. In Funa v. Villar,19 for one, the
principle of hold-over is misplaced for being predicated upon an erroneous Court has held:
presentation of a material fact To have legal standing, therefore, a suitor must show that he has sustained or
_______________ will sustain a “direct injury” as a result of a government action, or have a “material
10 Id., at p. 86. interest” in the issue affected by the challenged official act. However, the Court
has time and again acted liberally on the locus standi requirements and 21 Supra note 4.
has accorded certain individuals, not otherwise directly injured, or with 22 Id., at p. 319.
material interest affected, by a Government act, standing to sue provided 209
a constitutional issue of critical significance is at stake. The rule on locus VOL. 691, FEBRUARY 19, 2013 209
standi is after all a mere procedural technicality in relation to which the
Court, in a catena of cases involving a subject of transcendental import, Funa vs. Agra
has waived, or relaxed, thus allowing non-traditional plaintiffs, such as from exercising its power of judicial review because this case comes under several
concerned citizens, taxpayers, voters or legislators, to sue in the public of the well-recognized exceptions established in jurisprudence. Verily, the Court
interest, albeit they may not have been personally injured by the did not desist from resolving an issue that a supervening event meanwhile
operation of a law or any other government act. In David, the Court laid rendered moot and academic if any of the following recognized exceptions obtained,
out the bare minimum norm before the so-called “non-traditional suitors” namely: (1) there was a grave violation of the Constitution; (2) the case involved a
may be extended standing to sue, thusly: situation of exceptional character and was of paramount public interest; (3) the
1.) For taxpayers, there must be a claim of illegal disbursement of constitutional issue raised required the formulation of controlling principles to
public funds or that the tax measure is unconstitutional; guide the Bench, the Bar and the public; and (4) the case was capable of repetition,
2.) For voters, there must be a showing of obvious interest in the yet evading review.23
validity of the election law in question; It is the same here. The constitutionality of the concurrent holding by Agra of
_______________ the two positions in the Cabinet, albeit in acting capacities, was an issue that
18 Lawyers Against Monopoly and Poverty (LAMP) v. The Secretary of Budget comes under all the recognized exceptions. The issue involves a probable violation
and Management, G.R. No. 164987, April 24, 2012, 670 SCRA 373, 382. of the Constitution, and relates to a situation of exceptional character and of
19 G.R. No. 192791, April 24, 2012, 670 SCRA 579. paramount public interest by reason of its transcendental importance to the
208 people. The resolution of the issue will also be of the greatest value to the Bench
and the Bar in view of the broad powers wielded through said positions. The
208 SUPREME COURT REPORTS ANNOTATED
situation further calls for the review because the situation is capable of repetition,
Funa vs. Agra yet evading review.24 In other words, many important and practical benefits are
3.) For concerned citizens, there must be a showing that the issues still to be gained were the Court to proceed to the ultimate resolution of the
raised are of transcendental importance which must be settled early; and constitutional issue posed.
4.) For legislators, there must be a claim that the official action _______________
complained of infringes their prerogatives as legislators. 23 See Funa v. Villar, supra note 18, at pp. 592-593; David v. Macapagal-
This case before Us is of transcendental importance, since it obviously Arroyo, G.R. Nos. 171396, 171409, 171485, 171483, 171400, 171489 & 171424, May
has “far-reaching implications,” and there is a need to promulgate rules 3, 2006, 489 SCRA 160, 214-215.
that will guide the bench, bar, and the public in future analogous cases. 24 Javier v. Commission on Elections, Nos. L-68379-81, September 22, 1986,
We, thus, assume a liberal stance and allow petitioner to institute the 144 SCRA 194, 198.
instant petition.20 (Bold emphasis supplied) 210
In Funa v. Ermita,21 the Court recognized the locus standi of the petitioner as 210 SUPREME COURT REPORTS ANNOTATED
a taxpayer, a concerned citizen and a lawyer because the issue raised therein
involved a subject of transcendental importance whose resolution was necessary Funa vs. Agra
to promulgate rules to guide the Bench, Bar, and the public in similar cases. 2.
But, it is next posed, did not the intervening appointment of and assumption Unconstitutionality of Agra’s concurrent designation as Acting
by Cadiz as the Solicitor General during the pendency of this suit render this suit Secretary of Justice and Acting
and the issue tendered herein moot and academic? Solicitor General
A moot and academic case is one that ceases to present a justiciable controversy At the center of the controversy is the correct application of Section 13, Article
by virtue of supervening events, so that a declaration thereon would be of no VII of the 1987 Constitution, viz.:
practical use or value.22Although the controversy could have ceased due to the Section 13. The President, Vice-President, the Members of the Cabinet, and
intervening appointment of and assumption by Cadiz as the Solicitor General their deputies or assistants shall not, unless otherwise provided in this
during the pendency of this suit, and such cessation of the controversy seemingly Constitution, hold any other office or employment during their tenure. They shall
rendered moot and academic the resolution of the issue of the constitutionality of not, during said tenure, directly or indirectly practice any other profession,
the concurrent holding of the two positions by Agra, the Court should still go participate in any business, or be financially interested in any contract with, or in
forward and resolve the issue and not abstain any franchise, or special privilege granted by the Government or any subdivision,
_______________ agency, or instrumentality thereof, including government-owned or controlled
20 Id., at pp. 594-595.
corporations or their subsidiaries. They shall strictly avoid conflict of interest in Being designated as the Acting Secretary of Justice concurrently with his
the conduct of their office. position of Acting Solicitor General, therefore, Agra was undoubtedly covered by
A relevant and complementing provision is Section 7, paragraph (2), Article Section 13, Article VII,212
IX-B of the 1987 Constitution, to wit: 212 SUPREME COURT REPORTS ANNOTATED
Section 7. x x x
Unless otherwise allowed by law or the primary functions of his position, no Funa vs. Agra
appointive official shall hold any other office or employment in the Government or supra, whose text and spirit were too clear to be differently read. Hence, Agra
any subdivision, agency or instrumentality thereof, including government-owned could not validly hold any other office or employment during his tenure as the
or controlled corporations or their subsidiaries. Acting Solicitor General, because the Constitution has not otherwise so provided. 27
The differentiation of the two constitutional provisions was well stated in Funa It was of no moment that Agra’s designation was in an acting or temporary
v. Ermita,25 a case in which the petitioner herein also assailed the designation of capacity. The text of Section 13, supra, plainly indicates that the intent of the
DOTC Undersecretary as concurrent Officer-in-Charge of the Maritime Industry Framers of the Constitution was to impose a stricter prohibition on the President
Authority, with the Court reiterating its pronouncement in Civil Liberties Union and the Members of his Cabinet in so far as holding other offices or employments
v. The Executive Secretary26 on the in- in the Government or in government-owned or government controlled-corporations
_______________ was concerned.28 In this regard, to hold an office means to possess or to occupy the
25 Supra note 4. office, or to be in possession and administration of the office, which implies nothing
26 Supra note 5, at pp. 329-331. less than the actual discharge of the functions and duties of the office.29 Indeed, in
211 the language of Section 13 itself, supra, the Constitution makes no reference to the
nature of the appointment or designation. The prohibition against dual or multiple
VOL. 691, FEBRUARY 19, 2013 211
offices being held by one official must be construed as to apply to all appointments
Funa vs. Agra or designations, whether permanent or temporary, for it is without question that
tent of the Framers behind these provisions of the Constitution, viz.: the avowed objective of Section 13, supra, is to prevent the concentration of powers
Thus, while all other appointive officials in the civil service are allowed to hold in the Executive Department officials, specifically the President, the Vice-
other office or employment in the government during their tenure when such is President, the Members of the Cabinet and their deputies and assistants. 30 To
allowed by law or by the primary functions of their positions, members of the construe differently is to “open the veritable floodgates of circumvention of an
Cabinet, their deputies and assistants may do so only when expressly authorized important constitutional disqualification of officials in the Executive Department
by the Constitution itself. In other words, Section 7, Article IX-B is meant to and of limitations on the President’s power of appointment in the guise of
lay down the general rule applicable to all elective and appointive public temporary designations of Cabinet Members, under-
officials and employees, while Section 13, Article VII is meant to be the _______________
exception applicable only to the President, the Vice-President, Members 27 E.g., the Constitution, under its Section (1), Article VIII, provides that the
of the Cabinet, their deputies and assistants. Secretary of Justice sits as an ex officio member of the Judicial and Bar Council.
xxxx 28 Civil Liberties Union v. The Executive Secretary, supra note 6, at pp. 326-
Since the evident purpose of the framers of the 1987 Constitution is to impose 327.
a stricter prohibition on the President, Vice-President, members of the Cabinet, 29 Funa v. Ermita, supra note 5, at p. 329.
their deputies and assistants with respect to holding multiple offices or 30 Id., at p. 330.
employment in the government during their tenure, the exception to this 213
prohibition must be read with equal severity. On its face, the language of Section VOL. 691, FEBRUARY 19, 2013 213
13, Article VII is prohibitory so that it must be understood as intended to be a
positive and unequivocal negation of the privilege of holding multiple government Funa vs. Agra
offices or employment. Verily, wherever the language used in the constitution is secretaries and assistant secretaries as officers-in-charge of government agencies,
prohibitory, it is to be understood as intended to be a positive and unequivocal instrumentalities, or government-owned or controlled corporations.”31
negation. The phrase “unless otherwise provided in this Constitution” According to Public Interest Center, Inc. v. Elma,32 the only two exceptions
must be given a literal interpretation to refer only to those particular against the holding of multiple offices are: (1) those provided for under the
instances cited in the Constitution itself, to wit: the Vice-President being Constitution, such as Section 3, Article VII, authorizing the Vice President to
appointed as a member of the Cabinet under Section 3, par. (2), Article VII; or become a member of the Cabinet; and (2) posts occupied by Executive officials
acting as President in those instances provided under Section 7, pars. (2) and (3), specified in Section 13, Article VII without additional compensation in ex
Article VII; and, the Secretary of Justice being ex-officio member of the officio capacities as provided by law and as required by the primary functions of
Judicial and Bar Council by virtue of Section 8 (1), Article VIII. (Bold the officials’ offices. In this regard, the decision in Public Interest Center, Inc. v.
emphasis supplied.) Elma adverted to the resolution issued on August 1, 1991 in Civil Liberties Union
v. The Executive Secretary, whereby the Court held that the phrase “the Members
of the Cabinet, and their deputies or assistants” found in Section 13, supra, 34 Public Interest Center, Inc. v. Elma, supra note 7 at p. 64, with the Court
referred only to the heads of the various executive departments, their summing up at the end with the statement: “In sum, the prohibition in Section 13,
undersecretaries and assistant secretaries, and did not extend to other public Article VII of the 1987 Constitution does not apply to respondent Elma since
officials given the rank of Secretary, Undersecretary or Assistant neither the PCGG Chairman nor the (Chief Presidential Legal Counsel) is a
Secretary.33 Hence, in Public Cabinet secretary, undersecretary, or assistant secretary. x x x.”
_______________ 215
31 Id., at p. 331. VOL. 691, FEBRUARY 19, 2013 215
32 Supra note 7.
33 The clarification was the Court’s action on the motion for clarification filed Funa vs. Agra
in Civil Liberties Union v. The Executive Secretary, and revises the main opinion the Cabinet, their deputies and assistants with respect to holding other offices or
promulgated on February 22, 1991 (194 SCRA 317) totally invalidating Executive employment in the government during their tenure. Respondents’ interpretation
Order No. 284 dated July 25, 1987 (whose questioned Section 1 states: “Even if that Section 13 of Article VII admits of the exceptions found in Section 7, par. (2)
allowed by law or by the ordinary functions of his position, a member of the of Article IX-B would obliterate the distinction so carefully set by the framers of
Cabinet, undersecretary or assistant secretary or other appointive officials of the the Constitution as to when the high-ranking officials of the Executive Branch
Executive Department may, in addition to his primary position, hold not more than from the President to Assistant Secretary, on the one hand, and the generality of
two positions in the government and government corporations and receive the civil servants from the rank immediately below Assistant Secretary downwards,
corresponding compensation therefor; Provided, that this limitation shall not on the other, may hold any other office or position in the government during their
apply to ad hoc bodies or committees, or to boards, councils or bodies of which the tenure.35
President is the Chairman.”). The clarifying dictum now considered Executive To underscore the obvious, it is not sufficient for Agra to show that his holding
Order No. 284 partly valid to the extent that it included in its coverage “other of the other office was “allowed by law or the primary functions of his position.” To
appointive officials” aside from the members of the Cabi- claim the exemption of his concurrent designations from the coverage of the
214 stricter prohibition under Section 13, supra, he needed to establish herein that his
concurrent designation was expressly allowed by the Constitution. But, alas, he
214 SUPREME COURT REPORTS ANNOTATED
did not do so.
Funa vs. Agra To be sure, Agra’s concurrent designations as Acting Secretary of Justice and
Interest Center, Inc. v. Elma, the Court opined that the prohibition under Section Acting Solicitor General did not come within the definition of an ex officio capacity.
13 did not cover Elma, a Presidential Assistant with the rank of Undersecretary. 34 Had either of his concurrent designations been in an ex officio capacity in relation
It is equally remarkable, therefore, that Agra’s designation as the Acting to the other, the Court might now be ruling in his favor.
Secretary of Justice was not in an ex officio capacity, by which he would have been The import of an ex officio capacity has been fittingly explained in Civil
validly authorized to concurrently hold the two positions due to the holding of one Liberties Union v. Executive Secretary,36 as follows:
office being the consequence of holding the other. Being included in the stricter x x x. The term ex officio means “from office; by virtue of office.” It refers to an
prohibition embodied in Section 13, supra, Agra cannot liberally apply in his favor “authority derived from official character merely, not expressly conferred upon the
the broad exceptions provided in Section 7, paragraph 2, Article IX-B of the individual character, but rather annexed to the official position.” Ex
Constitution (“Unless otherwise allowed by law or the primary functions of his officio likewise denotes an “act done in an official character, or as a consequence of
position”) to justify his designation as Acting Secretary of Justice concurrently office, and without any other appointment or authority other than that conferred
with his designation as Acting Solicitor General, or vice versa. Thus, the Court has by
said― _______________
[T]he qualifying phrase “unless otherwise provided in this Constitution” in 35 Civil Liberties Union v. The Executive Secretary, supra note 6, at pp. 329-
Section 13, Article VII cannot possibly refer to the broad exceptions provided under 330.
Section 7, Article IX-B of the 1987 Constitution. To construe said qualifying phrase 36 Id., at pp. 333-335.
as respondents would have us do, would render nugatory and meaningless the 216
manifest intent and purpose of the framers of the Constitution to impose a stricter 216 SUPREME COURT REPORTS ANNOTATED
prohibition on the President, Vice-President, Members of
_______________ Funa vs. Agra
net, their undersecretaries and assistant secretaries, with the dispositive part the office.” An ex officio member of a board is one who is a member by virtue of his
of the clarificatory resolution of August 1, 1991 stating: “WHEREFORE, subject to title to a certain office, and without further warrant or appointment. x x x.
the qualification above-stated, the petitions are GRANTED. Executive Order No. xxxx
284 is hereby declared null and void insofar as it allows a member of the Cabinet, The ex officio position being actually and in legal contemplation part of the
undersecretary or assistant secretary to hold other positions in the government principal office, it follows that the official concerned has no right to receive
and government-owned and controlled corporations.” additional compensation for his services in the said position. The reason is that
these services are already paid for and covered by the compensation attached to proceedings are to be conducted outside of the Philippines the Solicitor General
his principal office. x x x. may employ
Under the Administrative Code of 1987, the DOJ is mandated to “provide the _______________
government with a principal law agency which shall be both its legal counsel and 38 Section 3, Chapter 1, Title III, Book IV of the Administrative Code of 1987.
prosecution arm; administer the criminal justice system in accordance with the 218
accepted processes thereof consisting in the investigation of the crimes, 218 SUPREME COURT REPORTS ANNOTATED
prosecution of offenders and administration of the correctional system; implement
the laws on the admission and stay of aliens, citizenship, land titling system, and Funa vs. Agra
settlement of land problems involving small landowners and members of counsel to assist in the discharge of the aforementioned responsibilities.
indigenous cultural minorities; and provide free legal services to indigent members 3. Appear in any court in any action involving the validity of any treaty, law,
of the society.”37 The DOJ’s specific powers and functions are as follows: executive order or proclamation, rule or regulation when in his judgment his
(1) Act as principal law agency of the government and as legal counsel and intervention is necessary or when requested by the Court.
representative thereof, whenever so required; 4. Appear in all proceedings involving the acquisition or loss of Philippine
(2) Investigate the commission of crimes, prosecute offenders and administer citizenship.
the probation and correction system; 5. Represent the Government in all land registration and related
(3) Extend free legal assistance/representation to indigents and poor litigants proceedings. Institute actions for the reversion to the Government of lands of the
in criminal cases and non-commercial civil disputes; public domain and improvements thereon as well as lands held in violation of the
(4) Preserve the integrity of land titles through proper registration; Constitution.
_______________ 6. Prepare, upon request of the President or other proper officer of the
37 Sections 1 and 2, Chapter 1, Title III, Book IV of the Administrative Code of National Government, rules and guidelines for government entities governing the
1987. preparation of contracts, making investments, undertaking of transactions, and
217 drafting of forms or other writings needed for official use, with the end in view of
facilitating their enforcement and insuring that they are entered into or prepared
VOL. 691, FEBRUARY 19, 2013 217
conformably with law and for the best interests of the public.
Funa vs. Agra 7. Deputize, whenever in the opinion of the Solicitor General the public
(5) Investigate and arbitrate untitled land disputes involving small landowners interest requires, any provincial or city fiscal to assist him in the performance of
and members of indigenous cultural communities; any function or discharge of any duty incumbent upon him, within the jurisdiction
(6) Provide immigration and naturalization regulatory services and implement of the aforesaid provincial or city fiscal. When so deputized, the fiscal shall be
the laws governing citizenship and the admission and stay of aliens; under the control and supervision of the Solicitor General with regard to the
(7) Provide legal services to the national government and its functionaries, conduct of the proceedings assigned to the fiscal, and he may be required to render
including government-owned or controlled corporations and their reports or furnish information regarding the assignment.
subsidiaries; and 8. Deputize legal officers of government departments, bureaus, agencies and
(8) Perform such other functions as may be provided by law.38 offices to assist the Solicitor General and appear or represent the Government in
On the other hand, the Administrative Code of 1987 confers upon the Office of cased involving their respective offices, brought before the courts and exercise
the Solicitor General the following powers and functions, to wit: supervision and control over such legal Officers with respect to such cases.
The Office of the Solicitor General shall represent the Government of the 9. Call on any department, bureau, office, agency or instrumentality of the
Philippines, its agencies and instrumentalities and its officials and agents in any Government for such service, assistance and cooperation as may be necessary in
litigation, proceeding, investigation or matter requiring the services of lawyers. fulfilling its functions and responsibilities and for this purpose enlist the services
When authorized by the President or head of the office concerned, it shall also of any government official or employee in the pursuit of his tasks.219
represent government owned or controlled corporations. The Office of the Solicitor VOL. 691, FEBRUARY 19, 2013 219
General shall discharge duties requiring the services of lawyers. It shall have the
following specific powers and functions: Funa vs. Agra
1. Represent the Government in the Supreme Court and the Court of Appeals 10. Departments, bureaus, agencies, offices, instrumentalities and
in all criminal proceedings; represent the Government and its officers in the corporations to whom the Office of the Solicitor General renders legal services are
Supreme Court, the Court of Appeals, and all other courts or tribunals in all civil authorized to disburse funds from their sundry operating and other funds for the
actions and special proceedings in which the Government or any officer thereof in latter Office. For this purpose, the Solicitor General and his staff are specifically
his official capacity is a party. authorized to receive allowances as may be provided by the Government offices,
2. Investigate, initiate court action, or in any manner proceed against any instrumentalities and corporations concerned, in addition to their regular
person, corporation or firm for the enforcement of any contract, bond, guarantee, compensation.
mortgage, pledge or other collateral executed in favor of the Government. Where
11. Represent, upon the instructions of the President, the Republic of the It is not amiss to observe, lastly, that assuming that Agra, as the Acting
Philippines in international litigations, negotiations or conferences where the legal Solicitor General, was not covered by the stricter prohibition under Section
position of the Republic must be defended or presented. 13, supra, due to such position being merely vested with a cabinet rank under
12. Act and represent the Republic and/or the people before any court, Section 3, Republic Act No. 9417, he nonetheless remained covered by the general
tribunal, body or commission in any matter, action or proceedings which, in his prohibition under Section 7, supra. Hence, his concurrent designations were still
opinion affects the welfare of the people as the ends of justice may require; and subject to the conditions
13. Perform such other functions as may be provided by law.39 _______________
The foregoing provisions of the applicable laws show that one position was not 44 Section 3, Republic Act No. 9417.
derived from the other. Indeed, the powers and functions of the OSG are neither 45 Supra note 6, at p. 339.
required by the primary functions nor included by the powers of the DOJ, and vice 221
versa. The OSG, while attached to the DOJ,40 is not a constituent unit of the VOL. 691, FEBRUARY 19, 2013 221
latter,41as, in fact, the Administrative Code of 1987 decrees that the OSG is
independent and autonomous.42 With the enactment of Republic Act No. 9417,43 Funa vs. Agra
_______________ under the latter constitutional provision. In this regard, the Court aptly pointed
39 Section 35, Chapter 12, Title III, Book IV of the Administrative Code of out in Public Interest Center, Inc. v. Elma:46
1987. The general rule contained in Article IX-B of the 1987 Constitution permits an
40 Section 34, Chapter 12, Title III, Book IV of the Administrative Code of appointive official to hold more than one office only if “allowed by law or by the
1987. primary functions of his position.” In the case of Quimson v. Ozaeta, this Court
41 Section 4, Chapter 1, Title III, Book IV of the Administrative Code of 1987. ruled that, “[t]here is no legal objection to a government official occupying two
42 Section 34, Chapter 12, Title III, Book IV of the Administrative Code of government offices and performing the functions of both as long as there is no
1987. incompatibility.” The crucial test in determining whether incompatibility exists
43 An Act to Strengthen the Office of the Solicitor General, by Expanding and between two offices was laid out in People v. Green—whether one office is
Streamlining its Bureaucracy, Upgrading Employee Skills, and Augmenting subordinate to the other, in the sense that one office has the right to interfere with
Benefits, and Appropriating funds therefor and for Other Purposes. the other.
220 [I]ncompatibility between two offices, is an inconsistency in the
functions of the two; x x x Where one office is not subordinate to the other,
220 SUPREME COURT REPORTS ANNOTATED
nor the relations of the one to the other such as are inconsistent and
Funa vs. Agra repugnant, there is not that incompatibility from which the law declares
the Solicitor General is now vested with a cabinet rank, and has the same that the acceptance of the one is the vacation of the other. The force of the
qualifications for appointment, rank, prerogatives, salaries, allowances, benefits word, in its application to this matter is, that from the nature and relations
and privileges as those of the Presiding Justice of the Court of Appeals.44 to each other, of the two places, they ought not to be held by the same
Moreover, the magnitude of the scope of work of the Solicitor General, if added person, from the contrariety and antagonism which would result in the
to the equally demanding tasks of the Secretary of Justice, is obviously too much attempt by one person to faithfully and impartially discharge the duties of
for any one official to bear. Apart from the sure peril of political pressure, the one, toward the incumbent of the other. x x x The offices must subordinate,
concurrent holding of the two positions, even if they are not entirely incompatible, one [over] the other, and they must, per se, have the right to interfere, one
may affect sound government operations and the proper performance of duties. with the other, before they are incompatible at common law. x x x.
Heed should be paid to what the Court has pointedly observed in Civil Liberties xxxx
Union v. Executive Secretary:45 While Section 7, Article IX-B of the 1987 Constitution applies in general to all
Being head of an executive department is no mean job. It is more than a full- elective and appointive officials, Section 13, Article VII, thereof applies in
time job, requiring full attention, specialized knowledge, skills and expertise. If particular to Cabinet secretaries, undersecretaries and assistant secretaries. In
maximum benefits are to be derived from a department head’s ability and the Resolution in Civil Liberties Union v. Executive Secretary, this Court already
expertise, he should be allowed to attend to his duties and responsibilities without clarified the scope of the prohibition provided in Section 13, Article VII of the 1987
the distraction of other governmental offices or employment. He should be Constitution. Citing the case of US v. Mouat, it specifically identified the
precluded from dissipating his efforts, attention and energy among too many _______________
positions of responsibility, which may result in haphazardness and inefficiency. 46 Supra note 7.
Surely the advantages to be derived from this concentration of attention, 222
knowledge and expertise, particularly at this stage of our national and economic 222 SUPREME COURT REPORTS ANNOTATED
development, far outweigh the benefits, if any, that may be gained from a
department head spreading himself too thin and taking in more than what he can Funa vs. Agra
handle.
persons who are affected by this prohibition as secretaries, undersecretaries and Secretary of Justice. A de jure officer is one who is deemed, in all respects, legally
assistant secretaries; and categorically excluded public officers who merely have appointed and qualified and whose term of office has not expired.49
the rank of secretary, undersecretary or assistant secretary. That notwithstanding, Agra was a de facto officer during his tenure as Acting
Another point of clarification raised by the Solicitor General refers to Secretary of Justice. In Civil Liberties Union v. Executive Secretary,50 the Court
the persons affected by the constitutional prohibition. The persons cited in said:
the constitutional provision are the “Members of the Cabinet, their deputies During their tenure in the questioned positions, respondents may be
and assistants.” These terms must be given their common and general considered de facto officers and as such entitled to emoluments for actual services
acceptation as referring to the heads of the executive departments, their rendered. It has been held that “in cases where there is no de jure, officer, a de
undersecretaries and assistant secretaries. Public officials given the rank facto officer, who, in good faith has had possession of the office and has discharged
equivalent to a Secretary, Undersecretary, or Assistant Secretary are not the duties pertaining thereto, is legally entitled to the emoluments of the office,
covered by the prohibition, nor is the Solicitor General affected thereby. and may in an appropriate action recover the salary, fees and other compensations
(Italics supplied). attached to the office. This doctrine is, undoubtedly, supported on equitable
It is clear from the foregoing that the strict prohibition under Section 13, grounds since it seems unjust that the public should benefit by the services of an
Article VII of the 1987 Constitution is not applicable to the PCGG Chairman nor officer de facto and then be freed from all liability to pay any one for such services.
to the CPLC, as neither of them is a secretary, undersecretary, nor an assistant Any per diem, allowances or other emoluments received by the respondents by
secretary, even if the former may have the same rank as the latter positions. virtue of actual services rendered in the questioned positions may therefore be
It must be emphasized, however, that despite the non-applicability of Section retained by them.
13, Article VII of the 1987 Constitution to respondent Elma, he remains covered _______________
by the general prohibition under Section 7, Article IX-B and his appointments 49 Topacio v. Ong, G.R. No. 179895, December 18, 2008, 574 SCRA 817, 830.
must still comply with the standard of compatibility of officers laid down therein; 50 Supra note 6, at pp. 339-340.
failing which, his appointments are hereby pronounced in violation of the 224
Constitution.47 224 SUPREME COURT REPORTS ANNOTATED
Clearly, the primary functions of the Office of the Solicitor General are not
related or necessary to the primary functions of the Department of Justice. Funa vs. Agra
Considering that the nature and duties of the two offices are such as to render it A de facto officer is one who derives his appointment from one having colorable
improper, from considerations of public policy, for one person to retain both, 48 an authority to appoint, if the office is an appointive office, and whose appointment is
incompatibility between the offices exists, further warranting the declaration of valid on its face.51 He may also be one who is in possession of an office, and is
Agra’s designation as the Act- discharging its duties under color of authority, by which is meant authority derived
_______________ from an appointment, however irregular or informal, so that the incumbent is not
47 Id., at pp. 59-63. a mere volunteer.52 Consequently, the acts of the de facto officer are just as valid
48 Summers v. Ozaeta, 81 Phil. 754, 764 (1948); see Mechem, A Treatise on the for all purposes as those of a de jureofficer, in so far as the public or third persons
Law of Public Offices and Officers, pp. 268-269 (1890). who are interested therein are concerned.53
223 In order to be clear, therefore, the Court holds that all official actions of Agra
as a de facto Acting Secretary of Justice, assuming that was his later designation,
VOL. 691, FEBRUARY 19, 2013 223
were presumed valid, binding and effective as if he was the officer legally
Funa vs. Agra appointed and qualified for the office.54 This clarification is necessary in order to
ing Secretary of Justice, concurrently with his designation as the Acting Solicitor protect the sanctity of the dealings by the public with persons whose ostensible
General, to be void for being in violation of the express provisions of the authority emanates from the State.55 Agra’s official actions covered by this
Constitution. clarification extend to but are not limited to the promulgation of resolutions on
petitions for review filed in the Department of Justice, and the issuance of
3. department orders, memoranda and circulars relative to the prosecution of
Effect of declaration of unconstitutionality of Agra’s concurrent criminal cases.
appointment; the de facto officer doctrine WHEREFORE, the Court GRANTS the petition for certiorariand prohibition;
ANNULS AND VOIDS the designation of Hon. Alberto C. Agra as the Acting
In view of the application of the stricter prohibition under Section 13, supra, Secretary of Justice in a concurrent capacity with his position as the Acting
Agra did not validly hold the position of Acting Secretary of Justice concurrently Solicitor
with his holding of the position of Acting Solicitor General. Accordingly, he was not _______________
to be considered as a de jure officer for the entire period of his tenure as the Acting 51 Dimaandal v. Commission on Audit, G.R. No. 122197, June 26, 1998, 291
SCRA 322, 330.
52 Id.; see also The Civil Service Commission v. Joson, Jr., G.R. No. 154674, ——o0o——
May 27, 2004, 429 SCRA 773, 786-787.
53 See Mechem, supra note 47, at pp. 10 and 218; Topacio v. Ong, supra note
48, at pp. 829-830. G.R. No. 191002. March 17, 2010.*
54 Id.; Seneres v. Commission on Elections, G.R. No. 178678, April 16, 2009, ARTURO M. DE CASTRO, petitioner, vs. JUDICIAL AND BAR COUNCIL (JBC)
585 SCRA 557, 575. and PRESIDENT GLORIA MACAPAGAL-ARROYO, respondents.
55 Topacio v. Ong, supra note 48 at p. 830. G.R. No. 191032. March 17, 2010.*
225 JAIME N. SORIANO, petitioner, vs. JUDICIAL AND BAR COUNCIL (JBC),
VOL. 691, FEBRUARY 19, 2013 225 respondent.
G.R. No. 191057. March 17, 2010.*
Funa vs. Agra PHILIPPINE CONSTITUTION ASSOCIATION (PHILCONSA),
General for being unconstitutional and violative of Section 13, Article VII of the petitioner, vs. JUDICIAL AND BAR COUNCIL (JBC), respondent.
1987 Constitution; and DECLARES that Hon. Alberto C. Agra was a de A.M. No. 10-2-5-SC. March 17, 2010.*
facto officer during his tenure as Acting Secretary of Justice. IN RE APPLICABILITY OF SECTION 15, ARTICLE VII OF THE
No pronouncement on costs of suit. CONSTITUTION TO APPOINTMENTS TO THE JU-
SO ORDERED.
Sereno (C.J.), Carpio, Velasco, Jr., Leonardo-De Castro, Brion, Peralta, Del _______________
Castillo, Abad, Villarama, Jr., Perez, Mendoza, Reyes, Perlas-Bernabe and Leonen,
JJ., concur.
* EN BANC.
Petition granted, designation of Hon. Alberto C. Agra as Acting Secretary of
667
Justice in a concurrent capacity with his position as Acting Solicitor General
annulled and voided. VOL. 615, March 17, 2010 667
Notes.—While the designation was in the nature of an acting and temporary De Castro vs. Judicial and Bar Council (JBC)
capacity, the words “hold the office” were employed. Such holding of office pertains DICIARY, ESTELITO P. MENDOZA, petitioner,
to both appointment and designation because the appointee or designate performs G.R. No. 191149. March 17, 2010.*
the duties and functions of the office. The 1987 Constitution in prohibiting dual or JOHN G. PERALTA, petitioner, vs. JUDICIAL AND BAR COUNCIL (JBC),
multiple offices, as well as incompatible offices, refers to the holding of the office, respondent.
and not to the nature of the appointment or designation, words which were not PETER IRVING CORVERA;
even found in Section 13, Article VII nor in Section 7, paragraph 2, Article IX-B. CHRISTIAN ROBERT S. LIM;
To “hold” an office means to “possess or occupy” the same, or “to be in possession ALFONSO V. TAN, JR.;
and administration,” which implies nothing less than the actual discharge of the NATIONAL UNION OF PEOPLE’S LAWYERS;
functions and duties of the office. (Funa vs. Ermita, 612 SCRA 308 [2010]) MARLOU B. UBANO;
Locus standi or legal standing has been defined as a personal and substantial INTEGRATED BAR OF THE PHILIPPINES-DAVAO DEL SUR CHAPTER,
interest in a case such that the party has sustained or will sustain direct injury as represented by its Immediate Past President, ATTY. ISRAELITO P. TORREON,
a result of the governmental act that is being challenged. (Galicto vs. Aquino III, and the latter in his own personal capacity as a MEMBER of the PHILIPPINE
667 SCRA 150 [2012]) BAR;
——o0o—— MITCHELL JOHN L. BOISER;
© Copyright 2018 Central Book Supply, Inc. All rights reserved. BAGONG ALYANSANG BAYAN (BAYAN) CHAIRMAN DR. CAROLINA P.
ARAULLO; BAYAN SECRETARY GENERAL RENATO M. REYES, JR.;
CONFEDERATION FOR UNITY, RECOGNITION AND ADVANCEMENT OF
GOVERNMENT EMPLOYEES (COURAGE) CHAIRMAN FERDINAND GAITE;
KALIPUNAN NG DAMAYANG MAHIHIRAP (KADAMAY) SECRETARY
GENERAL GLORIA ARELLANO; ALYANSA NG NAGKAKAISANG KABATAAN
The existence of a presumption indicating the guilt of the accused does not in
itself destroy the constitutional presumption of innocence unless the inculpating NG SAMBAYANAN PARA SA KAUNLARAN (ANAKBAYAN) CHAIRMAN KEN
presumption, together with all the evidence, or the lack of any evidence or LEONARD RAMOS; TAYO ANG PAG-ASA CONVENOR ALVIN PETERS;
explanation, proves the accused’s guilt beyond a reasonable doubt. Until the LEAGUE OF FILIPINO STUDENTS (LFS) CHAIRMAN JAMES MARK TERRY
LACUANAN RIDON; NATIONAL UNION OF STUDENTS OF THE
accused’s guilt is shown in this manner, the presumption of innocence continues.
PHILIPPINES (NUSP) CHAIRMAN EINSTEIN RE-
(Re: Conviction of Judge Adoracion G. Angeles, RTC, Br. 121, Caloocan City in
Criminal Cases Nos. Q-97-69655 to 56 for Child Abuse, 543 SCRA 196 [2008]) 668
668 SUPREME COURT REPORTS ANNOTATED that the person complaining has been or is about to be denied some right
or privilege to which he is lawfully entitled or that he is about to be
De Castro vs. Judicial and Bar Council (JBC) subjected to some burdens or penalties by reason of the statute or act
CEDES; COLLEGE EDITORS GUILD OF THE PHILIPPINES (CEGP) complained of.
CHAIRMAN VIJAE ALQUISOLA; and STUDENT CHRISTIAN MOVEMENT OF Same; Same; Taxpayer Suits; Quite often, the petitioner in a public action
THE PHILIPPINES (SCMP) CHAIRMAN MA. CRISTINA ANGELA sues as a citizen or taxpayer to gain locus standi, which is not surprising, for even
GUEVARRA; if the issue may appear to concern only the public in general, such capacities
WALDEN F. BELLO and LORETTA ANN P. ROSALES; nonetheless equip the petitioner with adequate interest to sue.—The assertion of a
WOMEN TRIAL LAWYERS ORGANIZATION OF THE PHILIPPINES, public right as a predicate for challenging a supposedly illegal or unconstitutional
represented by YOLANDA QUISUMBING-JAVELLANA; BELLEZA ALOJADO executive or legislative action rests on the theory that the petitioner represents
DEMAISIP; TERESITA GANDIONCO-OLEDAN; MA. VERENA KASILAG- the public in general. Although such petitioner may not be as adversely affected
VILLANUEVA; MARILYN STA. ROMANA; LEONILA DE JESUS; and by the action complained against as are others, it is enough that he sufficiently
GUINEVERE DE LEON. intervenors. demonstrates in his petition that he is entitled to protection or relief from the
G.R. No. 191342. March 17, 2010.* Court in the vindication of a public right. Quite often, as here, the petitioner in a
ATTY. AMADOR Z. TOLENTINO, JR., (IBP Governor–Southern Luzon), and public action sues as a citizen or taxpayer to gain locus standi. That is not
ATTY. ROLAND B. INTING (IBP Governor–Eastern Visayas), surprising, for even if the issue may appear to concern only the public in general,
petitioners, vs. JUDICIAL AND BAR COUNCIL (JBC), respondent. such capacities nonetheless equip the petitioner with adequate interest to sue.
G.R. No. 191420. March 17, 2010.* In David v. Macapagal-Arroyo, 489 SCRA 160 (2006) the Court aptly explains why:
PHILIPPINE BAR ASSOCIATION, INC., petitioner, vs. JUDICIAL AND BAR Case law in most jurisdictions now allows both “citizen” and “taxpayer” standing
COUNCIL and HER EXCELLENCY GLORIA MACAPAGAL-ARROYO, in public actions. The distinction was first laid down in Beauchamp v.
respondents. 670
Judicial Review; Locus Standi; Requisites; Words and Phrases; In public or
670 SUPREME COURT REPORTS ANNOTATED
constitutional litigations, the Court is often burdened with the determination of the
locus standi of the petitioners due to the ever-present need to regulate the invocation De Castro vs. Judicial and Bar Council (JBC)
of the intervention of the Court to correct any official action or policy in order to Silk, where it was held that the plaintiff in a taxpayer’s suit is in a different
avoid obstructing the efficient functioning of public officials and offices involved in category from the plaintiff in a citizen’s suit. In the former, the plaintiff is
public service; Black defines locus standi as “a right of appearance in a court of affected by the expenditure of public funds, while in the latter, he is but
justice on a given question.”—Black defines locus standi as “a right of appearance the mere instrument of the public concern. As held by the New York Supreme
in a court of justice on a given question.” In public or constitutional litigations, the Court in People ex rel Case v. Collins: “In matter of mere public right,
Court is often burdened with the determination of the locus standi of the however…the people are the real parties…It is at least the right, if not the
petitioners due to the ever-present need to regulate the invocation of the duty, of every citizen to interfere and see that a public offence be properly
intervention of the Court to correct any official action or policy in pursued and punished, and that a public grievance be remedied.” With
669 respect to taxpayer’s suits, Terr v. Jordan held that “the right of a citizen and
VOL. 615, March 17, 2010 669 a taxpayer to maintain an action in courts to restrain the unlawful use of
public funds to his injury cannot be denied.”
De Castro vs. Judicial and Bar Council (JBC) Same; Same; Same; The Court retains the broad discretion to waive the
order to avoid obstructing the efficient functioning of public officials and requirement of legal standing in favor of any petitioner when the matter involved
offices involved in public service. It is required, therefore, that the petitioner must has transcendental importance, or otherwise requires a liberalization of the
have a personal stake in the outcome of the controversy, for, as indicated in Agan, requirement.—The Court rules that the petitioners have each demonstrated
Jr. v. Philippine International Air Terminals Co., Inc., 402 SCRA 612 (2003): The adequate interest in the outcome of the controversy as to vest them with the
question on legal standing is whether such parties have “alleged such a requisite locus standi. The issues before us are of transcendental importance to the
personal stake in the outcome of the controversy as to assure that people as a whole, and to the petitioners in particular. Indeed, the issues affect
concrete adverseness which sharpens the presentation of issues upon everyone (including the petitioners), regardless of one’s personal interest in life,
which the court so largely depends for illumination of difficult because they concern that great doubt about the authority of the incumbent
constitutional questions.” Accordingly, it has been held that the interest President to appoint not only the successor of the retiring incumbent Chief Justice,
of a person assailing the constitutionality of a statute must be direct and but also others who may serve in the Judiciary, which already suffers from a far
personal. He must be able to show, not only that the law or any too great number of vacancies in the ranks of trial judges throughout the country.
government act is invalid, but also that he sustained or is in imminent In any event, the Court retains the broad discretion to waive the requirement of
danger of sustaining some direct injury as a result of its enforcement, and legal standing in favor of any petitioner when the matter involved has
not merely that he suffers thereby in some indefinite way. It must appear
transcendental importance, or otherwise requires a liberalization of the Tolentino and Inting, submit that the incumbent President can appoint the
requirement. successor of Chief Justice Puno upon his retirement on May 17, 2010, on the
Same; Actual Case or Controversy; Although the position is not yet vacant, ground that the prohibition against presidential appointments under Section 15,
the fact that the Judicial and Bar Council began the process of nomination Article VII does not extend to appointments in the Judiciary. The Court agrees
pursuant to its rules and practices, although it has yet to decide whether to submit with the submission.
the list of nominees to the incumbent outgoing President or to the next President, Same; Same; Statutory Construction; Had the framers intended to extend the
makes the situation ripe for judicial determination, because the next steps are the prohibition contained in Section 15, Article VII to the appointment of Members of
public interview of the candidates, the preparation of the short list of candidates, the Supreme Court, they could have explicitly done so—they could not have ignored
and the “interview of constitutional experts, as may be the meticulous ordering of the provisions.—As can be seen, Article VII is devoted
671 to the Executive Department, and, among others, it lists the powers vested by the
VOL. 615, March 17, 2010 671 Constitution in the President. The presidential power of appointment is dealt with
in Sections 14, 15 and 16 of the Article. Article VIII is dedicated to the Judicial
De Castro vs. Judicial and Bar Council (JBC) Department and defines the duties and qualifications of Members of the Supreme
needed.”—We hold that the petitions set forth an actual case or controversy Court, among others. Section 4(1) and Section 9 of this Article are the provisions
that is ripe for judicial determination. The reality is that the JBC already specifically providing for the appointment of Supreme Court Justices. In
commenced the proceedings for the selection of the nominees to be included in a particular, Section 9 states that the appointment of Supreme Court Justices can
short list to be submitted to the President for consideration of which of them will only be made by the President upon the submission of a list of at least three
succeed Chief Justice Puno as the next Chief Justice. Although the position is not nominees by the JBC; Section 4(1) of the Article mandates the President to fill the
yet vacant, the fact that the JBC began the process of nomination pursuant to its vacancy within 90 days from the occurrence of the vacancy. Had the framers
rules and practices, although it has yet to decide whether to submit the list of intended to extend the prohibition contained in Section 15, Article VII to the
nominees to the incumbent outgoing President or to the next President, makes the appointment of Members of the Supreme Court, they could have explicitly done so.
situation ripe for judicial determination, because the next steps are the public They could not have ignored the meticulous ordering of the provisions. They would
interview of the candidates, the preparation of the short list of candidates, and the have easily and surely written the prohibition made explicit in Section 15, Article
“interview of constitutional experts, as may be needed.” A part of the question to VII as being equally applicable to the appointment of Members of the Supreme
be reviewed by the Court is whether the JBC properly initiated the process, there Court in Article VIII itself, most likely in Section 4 (1), Article VIII. That such
being an insistence from some of the oppositors-intervenors that the JBC could specification was not done only reveals that the prohibition against the President
only do so once the vacancy has occurred (that is, after May 17, 2010). Another part or Acting President making appointments within two months before the next
is, of course, whether the JBC may resume its process until the short list is presidential elections and up to the end of the President’s or Acting President’s
prepared, in view of the provision of Section 4(1), Article VIII, which unqualifiedly term does not refer to the Members of the Supreme Court.
requires the President to appoint one from the short list to fill the vacancy in the Same; Same; Same; Judgments; The reference to the records of the
Supreme Court (be it the Chief Justice or an Associate Justice) within 90 days from Constitutional Commission did not advance or support the result in In Re
the occurrence of the vacancy. Appointments Dated March 30, 1998 of Hon. Mateo A. Valenzuela and Hon. Placido
Same; Same; The Court need not await the occurrence of the vacancy in the B. Vallarta as Judges of the Regional
position of the Chief Justice in order for the principal issue to ripe for judicial 673
determination by the Court.—We need not await the occurrence of the vacancy by
VOL. 615, March 17, 2010 673
May 17, 2010 in order for the principal issue to ripe for judicial determination by
the Court. It is enough that one alleges conduct arguably affected with a De Castro vs. Judicial and Bar Council (JBC)
constitutional interest, but seemingly proscribed by the Constitution. A reasonable Trial Court of Branch 62, Bago City and of Branch 24, Cabanatuan City,
certainty of the occurrence of the perceived threat to a constitutional interest is respectively (Valenzuela), 298 SCRA 408 (1998).—The reference to the records of
sufficient to afford a basis for bringing a challenge, provided the Court has the Constitutional Commission did not advance or support the result
sufficient facts before it to enable it to intelligently adjudicate the issues. Herein, in Valenzuela. Far to the contrary, the records disclosed the express intent of the
the facts are not in doubt, for only legal issues remain. framers to enshrine in the Constitution, upon the initiative of Commissioner
Presidency; Appointments; Midnight Appointment Ban; The prohibition Eulogio Lerum, “a command [to the President] to fill up any vacancy therein within
against presidential appointments under Section 15, Article VII does not extend to 90 days from its occurrence,” which even Valenzuelaconceded. The exchanges
appointments in the Judiciary.—In the consolidated petitions, the petitioners, with during deliberations of the Constitutional Commission on October 8, 1986 further
the exception of Soriano, show that the filling of a vacancy in the Supreme Court within the 90-day period
672 was a true mandate for the President.
672 SUPREME COURT REPORTS ANNOTATED Same; Same; Same; Same; The usage in Section 4(1), Article VIII of the
Constitution of the word shall—an imperative, operating to impose a duty that may
De Castro vs. Judicial and Bar Council (JBC) be enforced—should not be disregarded; Section 4(1) imposes on the President the
imperative duty to make an appointment of a Member of the Supreme Court within to the Judiciary made after the establishment of the Judicial and Bar Council
90 days from the occurrence of the vacancy—the failure by the President to do so (JBC) would not be suffering from such defects because of the JBC’s prior processing
will be a clear disobedience to the Constitution; The 90-day limitation fixed in of candidates.—Given the background and rationale for the prohibition in Section
Section 4(1), Article VIII for the President to fill the vacancy in the Supreme Court 15, Article VII, we have no doubt that the Constitutional Commission confined the
was undoubtedly a special provision to establish a definite mandate for the prohibition to appointments made in the Executive Department. The framers did
President as the appointing power.—Moreover, the usage in Section 4(1), Article not need to extend the prohibition to appointments in the Judiciary, because their
VIII of the word shall—an imperative, operating to impose a duty that may be establishment of the JBC and their subjecting the nomination and screening of
enforced—should not be disregarded. Thereby, Sections 4(1) imposes on the candidates for judicial positions to the unhurried and deliberate prior process of
President the imperative duty to make an appointment of a Member of the the JBC ensured that there would no longer be mid-
Supreme Court within 90 days from the occurrence of the vacancy. The failure by 675
the President to do so will be a clear disobedience to the Constitution. The 90-day VOL. 615, March 17, 2010 675
limitation fixed in Section 4(1), Article VIII for the President to fill the vacancy in
the Supreme Court was undoubtedly a special provision to establish a definite De Castro vs. Judicial and Bar Council (JBC)
mandate for the President as the appointing power, and cannot be defeated by night appointments to the Judiciary. If midnight appointments in the mold
mere judicial interpretation in Valenzuela to the effect that Section 15, Article VII of Aytona were made in haste and with irregularities, or made by an outgoing
prevailed because it was “couched in stronger negative language.” Such Chief Executive in the last days of his administration out of a desire to subvert the
interpretation even turned out to be conjectural, in light of the records of the policies of the incoming President or for partisanship, the appointments to the
Constitutional Commission’s deliberations on Section 4 (1), Article VIII. Judiciary made after the establishment of the JBC would not be suffering from
Same; Same; Same; Same; Valenzuela arbitrarily ignored the express intent such defects because of the JBC’s prior processing of candidates. Indeed, it is
of the Constitutional Commission to have Section 4 (1), Article VIII stand axiomatic in statutory construction that the ascertainment of the purpose of the
independently of any other provision, least of all enactment is a step in the process of ascertaining the intent or meaning of the
674 enactment, because the reason for the enactment must necessarily shed
considerable light on “the law of the statute,” i.e., the intent; hence, the enactment
674 SUPREME COURT REPORTS ANNOTATED
should be construed with reference to its intended scope and purpose, and the court
De Castro vs. Judicial and Bar Council (JBC) should seek to carry out this purpose rather than to defeat it.
one found in Article VII—a misinterpretation like Valenzuela should not be Same; Same; Same; Same; The intervention of the Judicial and Bar Council
allowed to last after its false premises have been exposed; Valenzuela now deserves (JBC) eliminates the danger that appointments to the Judiciary can be made for
to be quickly sent to the dustbin of the unworthy and forgettable.—In this the purpose of buying votes in a coming presidential election, or of satisfying
connection, PHILCONSA’s urging of a revisit and a review of Valenzuela is timely partisan considerations.—The intervention of the JBC eliminates the danger that
and appropriate. Valenzuela arbitrarily ignored the express intent of the appointments to the Judiciary can be made for the purpose of buying votes in a
Constitutional Commission to have Section 4 (1), Article VIII stand independently coming presidential election, or of satisfying partisan considerations. The
of any other provision, least of all one found in Article VII. It further ignored that experience from the time of the establishment of the JBC shows that even
the two provisions had no irreconcilable conflict, regardless of Section 15, Article candidates for judicial positions at any level backed by people influential with the
VII being couched in the negative. As judges, we are not to unduly interpret, and President could not always be assured of being recommended for the consideration
should not accept an interpretation that defeats the intent of the framers. of the President, because they first had to undergo the vetting of the JBC and pass
Consequently, prohibiting the incumbent President from appointing a Chief muster there. Indeed, the creation of the JBC was precisely intended to de-
Justice on the premise that Section 15, Article VII extends to appointments in the politicize the Judiciary by doing away with the intervention of the Commission on
Judiciary cannot be sustained. A misinterpretation like Valenzuela should not be Appointments. This insulating process was absent from the Aytona midnight
allowed to last after its false premises have been exposed. It will not do to merely appointment.
distinguish Valenzuela from these cases, for the result to be reached herein is Same; Same; Same; The fault of Valenzuela was that it accorded no weight
entirely incompatible with what Valenzuela decreed.Consequently, and due consideration to the confirmation of Justice Regalado—Valenzuela was
Valenzuela now deserves to be quickly sent to the dustbin of the unworthy and weak, because it relied on interpretation to determine the intent of the framers
forgettable. We reverse Valenzuela. rather than on the deliberations of the Constitutional Commission.—As earlier
Same; Same; Same; Judicial and Bar Council; Given the background and stated, the non-applicability of Section 15, Article VII to appointments in the
rationale for the prohibition in Section 15, Article VII, the Court has no doubt that Judiciary was confirmed by then Senior Associate Justice Regalado to the JBC
the Constitutional Commission confined the prohibition to appointments made in itself when it met on March 9, 1998 to discuss the question raised by some sectors
the Executive Department; If midnight appointments in the mold of Aytona v. about the “constitutionality of xxx appointments” to the Court of Appeals in light
Castillo, 4 SCRA 1 (1962), were made in haste and with irregularities, or made by of the forthcoming presiden-
an outgoing Chief Executive in the last days of his administration out of a desire to 676
subvert the policies of the incoming President or for partisanship, the appointments
676 SUPREME COURT REPORTS ANNOTATED Justices of the Court as Chief Justice? The question is not squarely before us at
the moment, but it should lend itself to a deeper analysis if and when
De Castro vs. Judicial and Bar Council (JBC) circumstances permit. It should be a good issue for the proposed Constitutional
tial elections. He assured that “on the basis of the (Constitutional) Convention to consider in the light of Senate President Juan Ponce Enrile’s
Commission’s records, the election ban had no application to appointments to the statement that the President can appoint the Chief Justice from among the sitting
Court of Appeals.” This confirmation was accepted by the JBC, which then justices of the Court even without a JBC list.
submitted to the President for consideration the nominations for the eight Same; Same; Same; Supreme Court; Judiciary Act of 1948; Legal Research;
vacancies in the Court of Appeals. The fault of Valenzuela was that it accorded no The express reference in Sections 4(1) and 9 of Article VIII of the Constitution to a
weight and due consideration to the confirmation of Justice Chief Justice abhors the idea that the framers contemplated an Acting Chief Justice
Regalado. Valenzuela was weak, because it relied on interpretation to determine to head the membership of the Supreme Court—otherwise, they would have simply
the intent of the framers rather than on the deliberations of the Constitutional written so in the Constitution; The framers intended the position of Chief Justice to
Commission. Much of the unfounded doubt about the President’s power to appoint be permanent, not one to be occupied in an acting or temporary capacity; It ought
during the period of prohibition in Section 15, Article VII could have been dispelled to be remembered that Section 12 of the Judiciary Act of 1948 was enacted because
since its promulgation on November 9, 1998, had Valenzuela properly the Chief Justice appointed under the 1935 Constitution was subject to the
acknowledged and relied on the confirmation of a distinguished member of the confirmation of the Commission on Appointments, and the confirmation process
Constitutional Commission like Justice Regalado. might take longer than expected.—A review of Sections 4(1) and 9 of Article VIII
Same; Same; Same; To hold like the Court did in Valenzuela that Section 15 shows that the Supreme Court is composed of a Chief Justice and 14 Associate
extends to appointments to the Judiciary further undermines the intent of the Justices, who all shall be appointed by the President from a list of at least three
Constitution of ensuring the independence of the Judicial Department from the nominees prepared by the JBC for every vacancy, which appointments require no
Executive and Legislative Departments.—To hold like the Court did confirmation by the Commission on Appointments. With reference to the Chief
in Valenzuela that Section 15 extends to appointments to the Judiciary further Justice, he or she is appointed by the President as Chief Justice, and the
undermines the intent of the Constitution of ensuring the independence of the appointment is never in an acting capacity. The express reference to a Chief
Judicial Department from the Executive and Legislative Departments. Such a Justice abhors the idea that the framers contemplated an Acting Chief Justice to
holding will tie the Judiciary and the Supreme Court to the fortunes or misfortunes head the membership of the Supreme Court. Otherwise, they would have simply
of political leaders vying for the Presidency in a presidential election. written so in the Constitution.
Consequently, the wisdom of having the new President, instead of the current 678
incumbent President, appoint the next Chief Justice is itself suspect, and cannot
678 SUPREME COURT REPORTS ANNOTATED
ensure judicial independence, because the appointee can also become beholden to
the appointing authority. In contrast, the appointment by the incumbent President De Castro vs. Judicial and Bar Council (JBC)
does not run the same risk of compromising judicial independence, precisely Consequently, to rely on Section 12 of the Judiciary Act of 1948 in order to
because her term will end by June 30, 2010. forestall the imperative need to appoint the next Chief Justice soonest is to defy
Same; Same; Same; In an extreme case, the Court can even raise a doubt on the plain intent of the Constitution. For sure, the framers intended the position of
whether a Judicial and Bar Council (JBC) list is necessary at all for the President— Chief Justice to be permanent, not one to be occupied in an acting or temporary
any President—to appoint a Chief Justice if the appointee is to come from the ranks capacity. In relation to the scheme of things under the present Constitution,
of the sitting justices of the Supreme Court.—As a matter of fact, in an extreme case, Section 12 of the Judiciary Act of 1948 only responds to a rare situation in which
we can even raise a doubt on whether a JBC list is necessary at all for the the new Chief Justice is not yet appointed, or in which the incumbent Chief Justice
President—any President—to appoint a Chief Justice if the is unable to perform the duties and powers of the office. It ought to be remembered,
677 however, that it was enacted because the Chief Justice appointed under the 1935
VOL. 615, March 17, 2010 677 Constitution was subject to the confirmation of the Commission on Appointments,
and the confirmation process might take longer than expected.
De Castro vs. Judicial and Bar Council (JBC) Same; Same; Same; Same; The lack of any appointed occupant of the office of
appointee is to come from the ranks of the sitting justices of the Supreme Chief Justice harms the independence of the Judiciary, because the Chief Justice is
Court. Sec. 9, Article VIII says: xxx. The Members of the Supreme Court xxx shall the head of the entire Judiciary.—The appointment of the next Chief Justice by the
be appointed by the President from a list of at least three nominees prepared by incumbent President is preferable to having the Associate Justice who is first in
the Judicial and Bar Council for any vacancy. Such appointments need no precedence take over. Under the Constitution, the heads of the Legislative and
confirmation. xxx The provision clearly refers to an appointee coming into the Executive Departments are popularly elected, and whoever are elected and
Supreme Court from the outside, that is, a non-member of the Court aspiring to proclaimed at once become the leaders of their respective Departments. However,
become one. It speaks of candidates for the Supreme Court, not of those who are the lack of any appointed occupant of the office of Chief Justice harms the
already members or sitting justices of the Court, all of whom have previously been independence of the Judiciary, because the Chief Justice is the head of the entire
vetted by the JBC. Can the President, therefore, appoint any of the incumbent Judiciary. The Chief Justice performs functions absolutely significant to the life of
the nation. With the entire Supreme Court being the Presidential Electoral within the discretion of the JBC. The object of the petitions for mandamus herein
Tribunal, the Chief Justice is the Chairman of the Tribunal. There being no should only refer to the
obstacle to the appointment of the next Chief Justice, aside from its being 680
mandatory for the incumbent President to make within the 90-day period from 680 SUPREME COURT REPORTS ANNOTATED
May 17, 2010, there is no justification to insist that the successor of Chief Justice
Puno be appointed by the next President. De Castro vs. Judicial and Bar Council (JBC)
Mandamus; Requisites.—Mandamus shall issue when any tribunal, duty to submit to the President the list of nominees for every vacancy in the
corporation, board, officer or person unlawfully neglects the performance of an act Judiciary, because in order to constitute unlawful neglect of duty, there must be
that the law specifically enjoins as a duty resulting from an office, trust, or station. an unjustified delay in performing that duty. For mandamus to lie against the
It is proper when the act against which it is directed is one addressed to the JBC, therefore, there should be an unexplained delay on its part in recommending
discretion of the tribunal or officer. Mandamus is not available to direct the nominees to the Judiciary, that is, in submitting the list to the President.
exercise of a judgment or discretion in a particular way. For mandamus to Words and Phrases; “Ministerial Act” and “Discretionary Act,”
679 Distinguished.—The distinction between a ministerial act and a discretionary one
has been delineated in the following manner: The distinction between a ministerial
VOL. 615, March 17, 2010 679
and discretionary act is well delineated. A purely ministerial act or duty is one
De Castro vs. Judicial and Bar Council (JBC) which an officer or tribunal performs in a given state of facts, in a
lie, the following requisites must be complied with: (a) the plaintiff has a prescribed manner, in obedience to the mandate of a legal authority,
clear legal right to the act demanded; (b) it must be the duty of the defendant to without regard to or the exercise of his own judgment upon the propriety
perform the act, because it is mandated by law; (c) the defendant unlawfully or impropriety of the act done. If the law imposes a duty upon a public
neglects the performance of the duty enjoined by law; (d) the act to be performed officer and gives him the right to decide how or when the duty shall be
is ministerial, not discretionary; and (e) there is no appeal or any other plain, performed, such duty is discretionary and not ministerial. The duty is
speedy and adequate remedy in the ordinary course of law. ministerial only when the discharge of the same requires neither the
Same; Presidency; Appointments; Judicial and Bar Council; The Judicial exercise of official discretion or judgment.
and Bar Council (JBC) has no discretion to submit the list of nominees to fill a CARPIO-MORALES, J., Dissenting Opinion:
vacancy in the Supreme Court to the President after the vacancy occurs, because Supreme Court; Words and Phrases; Primus Inter Pares; The Latin maxim
that shortens the 90-day period allowed by the Constitution for the President to primus inter pares indicates that a person is the most senior of a group of people
make the appointment.—Section 4(1) and Section 9, Article VIII, mandate the sharing the same rank or office.—Primus Inter Pares. First among equals. The
President to fill the vacancy in the Supreme Court within 90 days from the Latin maxim indicates that a person is the most senior of a group of people sharing
occurrence of the vacancy, and within 90 days from the submission of the list, in the same rank or office. The phrase has been used to describe the status, condition
the case of the lower courts. The 90-day period is directed at the President, not at or role of the prime minister in most parliamentary nations, the high-ranking
the JBC. Thus, the JBC should start the process of selecting the candidates to fill prelate in several religious orders, and the chief justice in many supreme courts
the vacancy in the Supreme Court before the occurrence of the vacancy. Under the around the world. The inclination to focus on the inter pares without due emphasis
Constitution, it is mandatory for the JBC to submit to the President the list of on the primus/prima has spawned contemporary discourse that revives the
nominees to fill a vacancy in the Supreme Court in order to enable the President original tug-of-war between domination and parity, which impasse the conceived
to appoint one of them within the 90-day period from the occurrence of the vacancy. maxim precisely intended to resolve. In the present case, several arguments
The JBC has no discretion to submit the list to the President after the vacancy attempt to depict a mirage of doomsday scenarios arising from the impending
occurs, because that shortens the 90-day period allowed by the Constitution for the vacancy of the primus in the Court as a springboard for their plea to avert a
President to make the appointment. For the JBC to do so will be unconscionable supposed undermining of the independence of the judiciary. In reality,
on its part, considering that it will thereby effectively and illegally deprive the the essential question boils down to the limitation on the appointing
President of the ample time granted under the Constitution to reflect on the power of the President.681
qualifications of the nominees named in the list of the JBC before making the VOL. 615, March 17, 2010 681
appointment.
Same; Same; Same; Same; The duty of the Judicial and Bar Council (JBC) De Castro vs. Judicial and Bar Council (JBC)
to submit a list of nominees before the start of the President’s mandatory 90-day
period to appoint is ministerial, but its selection of the candidates whose names will Presidency; Appointments; Midnight Appointment Ban; Statutory
be in the list to be submitted to the President lies within the discretion of the JBC.— Construction; It is unfortunate that the ponencia chiefly relies on the trivialities of
The duty of the JBC to submit a list of nominees before the start of the President’s draftsmanship style in arriving at a constitutional construction.—The first
mandatory 90-day period to appoint is ministerial, but its selection of the ratiocination adverts to the “organization and arrangement of the provisions of
candidates whose names will be in the list to be submitted to the President lies the Constitution” that was, as the ponenciadeclares, purposely made by the
framers of the Constitution to “reflect their intention and manifest their vision” of
the charter’s contents. It is unfortunate that the ponencia chiefly relies on the in Article VIII, for it backlashes the question that had the framers intended to
trivialities of draftsmanship style in arriving at a constitutional construction. exclude judicial appointments in Article VIII from the prohibition in Article VII,
Same; Same; Same; Same; Judicial and Bar Council; Checks and Balances; they would have easily and surely written so in the excepting proviso in Article
That the power of judicial appointment was lodged in the President is a recognized VII. Taking into account how the framers painstakingly rummaged through
measure of limitation on the power of the judiciary, which measure, however, is various sections of the Constitution and came up with only one exception with the
counterbalanced by the election ban due to the need to insulate the judiciary from need to specify the executive department, it insults the collective intelligence and
the political climate of presidential elections—to abandon this interplay of checks diligence of the ConCom to postulate that it intended to exclude the judiciary but
and balances on the mere inference that the establishment of the JBC could de- missed out on that one. To hold that the ban on midnight appointments applies
politicize the process of judicial appointments lacks constitutional mooring.—The only to executive positions, and not to vacancies in the judiciary and independent
allocation of three Articles in the Constitution devoted to the respective dynamics constitutional bodies, is to make the prohibition practically useless. It bears noting
of the three Departments was deliberately adopted by the framers to allocate the that Section 15, Article VII of the Constitution already allows the President, by
vast powers of government among the three Departments in recognition of the way of exception, to make temporary appointments in the Executive Department
principle of separation of powers. The equation, however, does not end there. Such during the prohibited period. Under this view, there is virtually no restriction on
kind of formulation detaches itself from the concomitant system of checks and the President’s power of appointment during the prohibited period. The general
balances. Section sequencing alone of Sections 14, 15 and 16 of Article VII, as rule is clear since the prohibition applies to ALL kinds of midnight appointments.
explained in the fourth ratiocination, does not suffice to signify functional 683
structuring. That the power of judicial appointment was lodged in the President is VOL. 615, March 17, 2010 683
a recognized measure of limitation on the power of the judiciary, which measure,
however, is counterbalanced by the election ban due to the need to insulate the De Castro vs. Judicial and Bar Council (JBC)
judiciary from the political climate of presidential elections. To abandon this The Constitution made no distinction. Ubi lex non distinguit nec nos
interplay of checks and balances on the mere inference that the establishment of distinguere debemos.
the JBC could de-politicize the process of judicial appointments lacks Same; Same; Same; Same; Expressio Unius; There is no clear circumstance
constitutional mooring. that would indicate that the enumeration in the exception in Section 15, Article VII
Same; Same; Same; Same; The constitutional prohibition in Section 15 found of the Constitution was not intended to be exclusive.—The exception is likewise
its roots in the case of Aytona v. Castillo, 4 SCRA 1 (1962), where among the clear. Expressio unius et exclusio alterius. The express mention of one person,
“midnight” or “last minute” appointments voided to abort the abuse of presidential thing or consequence implies the exclusion of all others. There is no clear
prerogatives or partisan efforts to fill vacant positions were one in the Supreme circumstance that would indicate that the enumeration in the exception was not
Court and two intended to be exclusive. Moreover, the fact that Section 15 was couched in
682 negative language reinforces the exclusivity of the exception. Under the rules of
statutory construction, exceptions, as a general rule, should be strictly but
682 SUPREME COURT REPORTS ANNOTATED
reasonably construed; they extend only so far as their language fairly warrants,
De Castro vs. Judicial and Bar Council (JBC) and all doubts should be resolved in favor of the general provisions rather than the
in the Court of Appeals.—The constitutional prohibition in Section 15 exception. Where a general rule is established by statute with exceptions, the court
found its roots in the case of Aytona v. Castillo, 4 SCRA 1 (1962) where will not curtail the former nor add to the latter by implication.
among the “midnight” or “last minute” appointments voided to abort the abuse of Same; Same; Same; Same; Judgments; The ponencia should not hastily
presidential prerogatives or partisan efforts to fill vacant positions were one in the reverse, on the sole basis of Justice Regalado’s opinion, the Court’s unanimous en
Supreme Court and two in the Court of Appeals. Heeding Aytona’s admonition, the banc decision In Re Appointments of Hon. Valenzuela & Hon. Vallarta, 298 SCRA
Constitutional Commission (ConCom) saw it fit to provide for a comprehensive ban 408 (1998), penned by Chief Justice Andres Narvasa, and concurred in by, inter
on midnight appointments, finding that the establishment of the JBC is not alia, Associate Justices who later became Chief Justices—Hilario Davide, Jr.,
enough to safeguard or insulate judicial appointments from politicization. Artemio Panganiban and Reynato Puno.—In its third ratiocination,
Same; Same; Same; Same; Taking into account how the framers the ponencia faults Valenzuela for not according weight and due consideration to
painstakingly rummaged through various sections of the Constitution and came up the opinion of Justice Florenz Regalado. It accords high regard to the opinion
with only one exception with the need to specify the executive department, it insults expressed by Justice Regalado as a former ConCom Member, to the exception of
the collective intelligence and diligence of the Constitutional Commission (ConCom) the opinion of all others similarly situated. It bears noting that the Court had
to postulate that it intended to exclude the judiciary but missed out on that one; To spoken in one voice in Valenzuela. The ponencia should not hastily reverse, on the
hold that the ban on midnight appointments applies only to executive positions, and sole basis of Justice Regalado’s opinion, the Court’s unanimous en banc decision
not to vacancies in the judiciary and independent constitutional bodies, is to make penned by Chief Justice Andres Narvasa, and concurred in by, inter alia, Associate
the prohibition practically useless.—It is simplistic and unreliable for Justices who later became Chief Justices—Hilario Davide, Jr., Artemio
the ponencia to contend that had the framers intended to extend the ban in Article Panganiban and Reynato Puno. The line of reasoning is specious. If that is the case
VII to appointments in the judiciary, they would have easily and surely written so
and for accuracy’s sake, we might as well reconvene all ConCom members and put Same; Same; Judicial and Bar Council; The ponencia’s interpretation that
the matter to a vote among them.684 the Judicial and Bar Council (JBC) has until 17 May 2010, at the latest, within
684 SUPREME COURT REPORTS ANNOTATED which to submit to the President the list of nominees for the position of Chief Justice
is absurd as it takes the application and nomination stages in isolation from the
De Castro vs. Judicial and Bar Council (JBC) whole appointment process—for the ponencia, the filling of the vacancy only
involves the President, and the JBC was not considered when the period was
Same; Same; Same; Same; Casus Omissus; The proper interpretation increased from 60 days to 90 days.—The ponencia also holds that the JBC has until
therefore of a Constitution depends more on how it was understood by the people May 17, 2010, at the latest, within which to submit to the President the list of
adopting it than in the framers’ understanding thereof; The people may not be of nominees for the position of Chief Justice. It declares that the JBC should start
the same caliber as Justice Regalado, but they simply could not read into Section the process of selecting the candidates to fill the vacancy in the Supreme Court
15 something that is not there—casus omissus pro omisso habendus est.— before the occurrence of the vacancy, explaining that the 90-day period in the
Providentially, jurisprudence is replete with guiding principles to proviso, “Any vacancy shall be filled within ninety days from the occurrence
ascertain the true meaning of the Constitution when the provisions as thereof,” is addressed to the President, not to the JBC. Such interpretation is
written appear unclear and the proceedings as recorded provide little absurd as it takes the application and nomination stages in isolation from the
help: While it is permissible in this jurisdiction to consult the debates and whole appointment process. For the ponencia, the filling of the vacancy only
proceedings of the constitutional convention in order to arrive at the reason and involves the President, and the JBC was not considered when the period was
purpose of the resulting Constitution, resort thereto may be had only when other increased from 60 days to 90 days. The sense of the Concom is the exact opposite.
guides fail as said proceedings are powerless to vary the terms of the Constitution The flaw in the reasoning is made more evident when the vacancy occurs by virtue
when the meaning is clear. Debates in the constitutional convention “are of value of death of a member of the Court. In that instance, the JBC could never anticipate
as showing the views of the individual members, and as indicating the reasons for the vacancy, and could never submit a list to the President before the 90-day
their votes, but they give us no light as to the views of the large majority who did period.
not talk, much less of the mass of our fellow citizens whose votes at the polls gave Same; Same; Supreme Court; It is ironic for the ponencia to recognize that the
that instrument the force of fundamental law. We think it safer to construe the President may need as much as 90 days of reflection in appointing a member of the
constitution from what appears upon its face.” The proper interpretation therefore Court, and yet abhor the idea of an acting Chief Justice in the interregnum as
depends more on how it was understood by the people adopting it than in the provided for by law, confirmed by tradition, and settled by jurisprudence to be an
framers’ understanding thereof. (underscoring supplied) The clear import of internal matter—the express allowance of a 90-day period of vacancy rebuts any
Section 15 of Article VII is readily apparent. The people may not be of the same policy argument on the necessity to avoid a vacuum of even a single day in the
caliber as Justice Regalado, but they simply could not read into Section 15 position of an appointed Chief Justice.—It is ironic
something that is not there. Casus omissus pro omisso habendus est. 686
Same; Same; Same; Considering that Section 15 of Article VII is an express
686 SUPREME COURT REPORTS ANNOTATED
limitation on the President’s power of appointment, the running of the 90-day period
is deemed suspended during the period of the ban which takes effect only once every De Castro vs. Judicial and Bar Council (JBC)
six years.—In the present case, there can only arise a legal impossibility when for the ponencia to state on the one hand that the President would
the JBC list is submitted or the vacancy occurred during the appointments ban be deprived of ample time to reflect on the qualifications of the nominees, and to
and the 90-day period would expire before the end of the appointments ban, in show on the other hand that the President has, in recent history, filled the vacancy
which case the fresh 90-day period should start to run at noon of June 30. This was in the position of Chief Justice in one or two days. It is ironic for the ponencia to
the factual antecedent respecting the trial court judges involved in Valenzuela. recognize that the President may need as much as 90 days of reflection in
There also arises a legal impossibility when the list is submitted or the vacancy appointing a member of the Court, and yet abhor the idea of an acting Chief Justice
occurred prior to the ban and no appointment was made before the ban starts, in the interregnum as provided for by law, confirmed by tradition, and settled by
685 jurisprudence to be an internal matter. The express allowance of a 90-day period
VOL. 615, March 17, 2010 685 of vacancy rebuts any policy argument on the necessity to avoid a vacuum of even
a single day in the position of an appointed Chief Justice.
De Castro vs. Judicial and Bar Council (JBC) ame; Same; Same; As a member of the Court, I strongly take exception to the
rendering the lapse of the 90-day period within the period of the ban, in ponencia’s implication that the Court cannot function without a sitting Chief
which case the remaining period should resume to run at noon of June 30. The Justice.—As a member of the Court, I strongly take exception to
outgoing President would be released from non-fulfillment of the constitutional the ponencia’s implication that the Court cannot function without a
obligation, and the duty devolves upon the new President. Considering also that sitting Chief Justice. To begin with, judicial power is vested in one Supreme
Section 15 of Article VII is an express limitation on the President’s power of Court and not in its individual members, much less in the Chief Justice alone.
appointment, the running of the 90-day period is deemed suspended during the Notably, after Chief Justice Puno retires, the Court will have 14 members left,
period of the ban which takes effect only once every six years. which is more than sufficient to constitute a quorum. The fundamental principle
in the system of laws recognizes that there is only one Supreme Court from whose Moreover, the Court does not sit to adjudicate mere academic questions to
decisions all other courts are required to take their bearings. While most of the satisfy scholarly interest, however intellectually challenging. While
Court’s work is performed by its three divisions, the Court remains one court— Mendoza and the other petitioners espouse worthy causes, they have presented
single, unitary, complete and supreme. Flowing from this is the fact that, while before this Court issues which are still subject to unfore-
individual justices may dissent or only partially concur, when the Court states 688
what the law is, it speaks with only one voice. The Court, as a collegial body, 688 SUPREME COURT REPORTS ANNOTATED
operates on a “one member, one vote” basis, whether it sits en banc or in divisions.
The competence, probity and independence of the Court en banc, or those of the De Castro vs. Judicial and Bar Council (JBC)
Court’s Division to which the Chief Justice belongs, have never depended on seen possibilities. In other words, the issues they raised are hypothetical
whether the member voting as Chief Justice is merely an acting Chief Justice or a and unripe for judicial determination.
duly appointed one. Same; Same; Advisory Opinions; The function of the courts is to determine
NACHURA, J., Separate Opinion: controversies between litigants and not to give advisory opinions.—To repeat for
Judicial Review; The consolidated petitions should be dismissed, because they emphasis, before this Court steps in to wield its awesome power of deciding cases,
do not raise an actual case or controversy ripe for judicial determination.—After there must first be an actual controversy ripe for judicial adjudication.
careful perusal of the pleadings and painstaking study of the applicable law and Here, the allegations in all the petitions are conjectural or anticipatory. No actual
jurisprudence, I controversy between real litigants exists. These consolidated petitions, in other
687 words, are a “purely academic exercise.” Hence, any resolution that this Court
might make would constitute an attempt at abstraction that can only lead to
VOL. 615, March 17, 2010 687
barren legal dialectics and sterile conclusions unrelated to actualities. Moreover,
De Castro vs. Judicial and Bar Council (JBC) the function of the courts is to determine controversies between litigants and not
earnestly believe that the consolidated petitions should be dismissed, to give advisory opinions. Here, petitioners are asking this Court to render an
because they do not raise an actual case or controversy ripe for judicial advisory opinion on what the JBC and the President should do. To accede to it is
determination. As an essential ingredient for the exercise of the power of judicial tantamount to an incursion into the functions of the executive department. This
review, an actual case or controversy involves a conflict of legal rights, an will further inappropriately make the Court an adviser of the President.
assertion of opposite legal claims susceptible to judicial resolution. The controversy Same; Same; Same; If petitioners only want guidance from this Court, then,
must be justiciable—definite and concrete—touching on the legal relations of let it be stated that enough guidance is already provided by the Constitution, the
parties having adverse legal interests. In other words, the pleadings must show an relevant laws, and the prevailing jurisprudence on the matter—the Court must not
active antagonistic assertion of a legal right, on one hand, and a denial thereof, on be unduly burdened with petitions raising abstract, hypothetical, or contingent
the other; that is, the case must concern a real and not a merely theoretical questions.—A final note. If petitioners only want guidance from this Court, then,
question or issue. There ought to be an actual and substantial controversy let it be stated that enough guidance is already provided by the Constitution, the
admitting of specific relief through a decree conclusive in nature, as distinguished relevant laws, and the prevailing jurisprudence on the matter. The Court must
from an opinion advising what the law would be upon a hypothetical state of facts. not be unduly burdened with petitions raising abstract, hypothetical, or
The rationale for this requirement is to prevent the courts through avoidance of contingent questions. As fittingly phrased by Chief Justice Puno in Lozano—
premature adjudication from entangling themselves in abstract disagreements, Given the sparseness of our resources, the capacity of courts to render efficient
and for us to be satisfied that the case does not present a hypothetical injury or a judicial service to our people is severely limited. For courts to indiscriminately
claim contingent upon some event that has not and indeed may never transpire. open their doors to all types of suits and suitors is for them to unduly overburden
Thus, justiciability requires (1) that there be an actual controversy between or their dockets, and ultimately render themselves ineffective dispensers of justice.
among the parties to the dispute; (2) that the interests of the parties be adverse; To be sure, this is an evil that clearly confronts our judiciary today.689
(3) that the matter in controversy be capable of being adjudicated by judicial power; VOL. 615, March 17, 2010 689
and (4) that the determination of the controversy will result in practical relief to
the complainant. De Castro vs. Judicial and Bar Council (JBC)
Same; Declaratory Relief; Jurisdiction; Supreme Court; Settled is the rule
that petitions for declaratory relief are outside the jurisdiction of this Court.—As to BRION, J., Separate Opinion:
the petition filed by Estelito Mendoza, while it is captioned as an administrative Judicial Review; Locus Standi; Jurisprudence is replete with precedents on
matter, the same is in the nature of a petition for declaratory relief. the liberal appreciation of the locus standi rule on issues that are of transcendental
Mendoza pleads that this Court interpret two apparently conflicting provisions of concern to the nation, and the petitioners very well qualify under these rulings.—I
the Constitution—Article VII, Section 15 and Article VIII, Section 4(1). Petitioner completely agree with the ponencia’s ruling on the parties’ standing, their locus
Mendoza specifically prays for such a ruling “for the guidance of the [JBC],” a standi, to bring their petitions and interventions in their capacities as citizens and
relief evidently in the nature of a declaratory judgment. Settled is the rule that lawyers who stand to be affected by our ruling as lawyers or by the impact of our
petitions for declaratory relief are outside the jurisdiction of this Court. ruling on the nation and the all-important electoral exercise we shall hold in May
2010. Jurisprudence is replete with precedents on the liberal appreciation of provisions, laws and regulations affecting the means and manner of the exercise
the locus standi rule on issues that are of transcendental concern to the nation, of their powers as the Supreme Court is the final authority on the interpretation
and the petitioners very well qualify under these rulings. In this sense, locus of these instruments. A prime example of the exercise of the Court’s power of
standi is not a critical issue in the present case. In fact, the concern voiced out supervision is In Re: Appointments dated March 30, 1998 of Hon. Mateo A.
during the Court’s deliberations, is more on how participation can be limited to Valenzuela and Hon. Placido B. Vallarta as Judges of the Regional Trial Court of
those who have substantial contributions, through their submissions, to the Branch 62, Bago City, and of Branch 24, Cabanatuan City, respectively, A.M. No.
resolution of the grave issues before the Court. 98-5-01-SC, November 9, 1998 (hereinafter referred to as Valenzuela) where the
Same; Same; I disagree with the ponencia’s ruling on justiciability as I believe Court nullified the oath of office taken by Judge Valenzuela, while at the same
some of the petitions before us do not reach the required level of justiciability; others, time giving its interpretation of how the election ban against appointment operates
however, qualify so that my disagreement with the lack of justiciability of some of on the Judiciary, thereby setting the guidelines on how Section 15, Article VII is
the petitions need not hinder the Court’s consideration of the main issue at hand.— to be read and interpreted. The Valenzuela case shall be discussed more fully
While the rule on locus standi can be relaxed, the rule on the need for an actual below.691
justiciable case that is ripe for adjudication addresses a different concern and VOL. 615, March 17, 2010 691
cannot be similarly treated. I disagree with the ponencia’s ruling on
justiciability as I believe some of the petitions before us do not reach the De Castro vs. Judicial and Bar Council (JBC)
required level of justiciability; others, however, qualify as discussed
below so that my disagreement with the lack of justiciability of some of Same; Same; An administrative matter that is entered in the Court’s docket
the petitions need not hinder the Court’s consideration of the main issue is either an administrative case (A.C.) or an administrative matter (A.M.) submitted
at hand. The basic requisite before this Court can rule is the presence of an actual to the Court for its consideration and action pursuant to its power of supervision;
case calling for the exercise of judicial power. This is a requirement that the An A.C. involves disciplinary and other actions over members of the Bar, based on
Constitution itself expressly imposes; in granting the Court judicial power and in the Court’s supervision over them arising from the Supreme Court’s authority to
defining the grant, the Constitution expressly states that judicial power includes promulgate rules relating to the admission to the practice of law and to the
the duty to settle actual controversies involving rights which are legally Integrated Bar; An A.M. is a matter based on the Supreme Court’s power of
demandable and enforceable. Thus, the Court does not issue advisory opinions, nor supervision—under Section 6, Article VIII, this refers to the Court’s administrative
do we pass upon hypo- supervision over all courts and the personnel thereof, and, under Section 8, it refers
690 to its supervision over the JBC.—An administrative matter that is entered in the
Court’s docket is either an administrative case (A.C.) or an administrative matter
690 SUPREME COURT REPORTS ANNOTATED
(A.M.) submitted to the Court for its consideration and action pursuant to its power
De Castro vs. Judicial and Bar Council (JBC) of supervision. An A.C. involves disciplinary and other actions over members of the
thetical cases, feigned problems or friendly suits collusively arranged Bar, based on the Court’s supervision over them arising from the Supreme Court’s
between parties without real adverse interests. Courts cannot adjudicate mere authority to promulgate rules relating to the admission to the practice of law and
academic questions to satisfy scholarly interest, however intellectually challenging to the Integrated Bar. Closely related to A.C. cases are the Bar Matter (B.M.) cases
they may be. As a condition precedent to the exercise of judicial power, an actual particularly involving admission to the practice of law. An A.M. is a matter based
controversy between litigants must first exist. on the Supreme Court’s power of supervision: under Section 6, Article VIII, this
Supreme Court; Judicial and Bar Council; The Supreme Court’s power of refers to administrative supervision over all courts and the personnel thereof;
supervision is particularly relevant in this case since the Judicial and Bar Council under Section 8, it refers to its supervision over the JBC. In using an
(JBC) was created “under the supervision of the Supreme Court,” with the administrative matter as its medium, the Mendoza petition cites as basis the effect
“principal function of recommending appointees to the Judiciary.”—This aspect of of a complete election ban on judicial appointments (in view of the already high
the power of the Court—its power of supervision—is particularly relevant in this level of vacancies and the backlog of cases) and submits this as an administrative
case since the JBC was created “under the supervision of the Supreme Court,” with matter that the Court, in the exercise of its supervision over the Judiciary, should
the “principal function of recommending appointees to the Judiciary.” In the same act upon. At the same time, it cites the “public discourse and controversy” now
manner that the Court cannot dictate on the lower courts on how they should taking place because of the application of the election ban on the appointment of
decide cases except through the appeal and review process provided by the Rules the Chief Justice, citing in this regard the very same reasons mentioned
of Court, so also cannot the Court intervene in the JBC’s authority to discharge its in Valenzuela about the need to resolve the issue and avoid the recurrence of
principal function. In this sense, the JBC is fully independent as shown by A.M. conflict thereon between the Executive and the Judiciary on the matter; and the
No. 03-11-16-SC or Resolution Strengthening The Role and Capacity of the Judicial need to “avoid any possible polemics concerning the matter.” The petition mentions
and Bar Council and Establishing the Offices Therein. In both cases, however and as well that the Court addressed the election ban issue in Valenzuela as an A.M.
unless otherwise defined by the Court (as in A.M. No. 03-11-16-SC), the Court can case, and apparently takes the lead from this decided A.M. matter.692
supervise by ensuring the legality and correctness of these entities’ exercise of 692 SUPREME COURT REPORTS ANNOTATED
their powers as to means and manner, and interpreting for them the constitutional
De Castro vs. Judicial and Bar Council (JBC) offer more flexibility in its application as the mandate for the President is to issue
appointments within 90 days from submission of the list, without specifying when
Same; Same; Declaratory Relief; The Supreme Court’s supervision over the the submission should be made.—Section 15 on its face disallows any appointment
Judicial and Bar Council (JBC), the latter’s need for guidance, and the existence of in clear negative terms (shall not make) without specifying the appointments
an actual controversy that Soriano and Tolentino cite, save the Mendoza petition covered by the prohibition. From this literal reading springs the argument that no
from being one for declaratory relief, which petition is originally cognizable by the exception is provided (except the exception found in Section 15 itself) so that even
Regional Trial Court, not by this Court.—Given the justiciable Soriano and the Judiciary is covered by the ban on appointments. On the other hand, Section
Tolentino petitions that directly address the JBC and its activities, the impact of 4(1) is likewise very clear and categorical in its terms: any vacancy in the
the above-outlined realities on the grant of a writ of prohibition, and the Court shall be filled within 90 days from its occurrence. In the way of Section 15,
undeniable supervision that the Supreme Court exercises over the JBC as well as Section 4(1) is also clear and categorical and provides no exception; the
its role as the interpreter of the Constitution—sufficiently compelling reason appointment refers solely to the Members of the Supreme Court and does not
exists to recognize the Mendoza petition as a properly filed A.M. mention any period that would interrupt, hold or postpone the 90-day requirement.
petition that should fully be heard in these proceedings to fully ventilate the Section 9 may offer more flexibility in its application as the mandate for the
supervisory aspect of the Court’s relationship with the JBC and to reflect, President is to issue appointments within 90 days from submission of the list,
once again, how this Court views the issues first considered without specifying when the submission should be made. From their wordings,
in Valenzuela. The Court’s supervision over the JBC, the latter’s need for urgency leaps up from Section 4(1) while no such message emanates from Section
guidance, and the existence of an actual controversy that Soriano and Tolentino 9; in the latter the JBC appears free to determine when a submission is to be made,
cite, save the Mendoza petition from being one for declaratory relief, which petition obligating the President to issue appointments within 90 days from the submission
is originally cognizable by the Regional Trial Court, not by this Court. of the JBC list. From this view, the appointment period under Section 9 is one that
Same; Same; Both by law and history, the Chief Justice has always been a is flexible and can move. Thus, in terms of conflict, Sections 4(1) and Sections 15
Member of the Court—although, as a primus inter pares—appointed by the can be said to be directly in conflict with each other, while a conflict is much less
President together with every other Associate Justice. For this reason, we should evident from a comparison of Sections 9 and 15. This conclusion answers the verba
dismiss the Soriano petition for lack of merit.—The use of the generic term legis argument of the Peralta petition that when the words or terms of a statute or
“Members of the Supreme Court” under Section 9, Article VIII in delineating the provision is clear and unambiguous, then no interpretation is necessary as the
appointing authority under the 1987 Constitution, is not new. This was the term 694
used in the present line of Philippine Constitutions, from 1935 to 1987, and the 694 SUPREME COURT REPORTS ANNOTATED
inclusion of the Chief Justice with the general term “Member of the Court” has De Castro vs. Judicial and Bar Council (JBC)
never been in doubt. In fact, Section 4(1) of the present Constitution itself confirms words or terms shall be understood in their ordinary meaning. In this case,
that the Chief Justice is a Member of the Court when it provides that the the individual provisions, in themselves, are clear; the conflict surfaces when they
Court “may sit en banc or, in its discretion, in divisions of three, five, or seven operate in tandem or against one another.
Members.” The Chief Justice is a Member of the En Banc and of the First Same; Same; Same; Same; Judgments; It appears clear that In Re:
Division—in fact, he is the Chair of the En Banc and of the First Division—but Appointments dated March 30, 1998 of Hon. Mateo A. Valenzuela and Hon. Placido
even as Chair is counted in the total membership of the En Banc or the Division B. Vallarta as Judges of the Regional Trial Court of Branch 62, Bago City, and of
for all purposes, particularly of quorum. Thus, at the same time that Section 4(1) Branch 24, Cabanatuan City, Respectively, A.M. No. 98-5-01-SC, November 9, 1998
speaks of a “Supreme Court. . . composed of one Chief Justice and fourteen (Valenzuela), 298 SCRA 408 (1998), should be read and appreciated for what it is—
Associate Justices,” it likewise calls all of them Members in defining how they will a ruling made on the basis of the Court’s supervision over judicial personnel that
693 upholds the election ban as against the appointment of lower court judges appointed
VOL. 615, March 17, 2010 693 pursuant to the period provided by Section 9 of Article VIII—Valenzuela’s
De Castro vs. Judicial and Bar Council (JBC) application to the filling up of a vacancy in the Supreme Court is a mere obiter
sit in the Court. Thus, both by law and history, the Chief Justice has always dictum as the Court is largely governed by Section 4(1) with respect to the period of
been a Member of the Court—although, as a primus inter pares—appointed by the appointment.—The Valenzuela decision gives the full flavor of how the election ban
President together with every other Associate Justice. For this reason, we should issue arose because of Chief Justice Narvasa’s very candid treatment of the facts
dismiss the Soriano petition for lack of merit. and the issue. Valenzuela openly stated that at the root of the dispute was the then
Same; Same; Appointments; Judiciary; From their wordings, urgency leaps existing vacancy in the Court and the difference of opinion on the matter between
up from Section 4(1) of Article VIII of the Constitution while no such message the Executive and the Court on the application of Section 15, Article VII, in
emanates from Section 9—in the latter the Judicial and Bar Council (JBC) appears relation with Section 4(1) and 9 of Article VIII, of the Constitution. What appears
free to determine when a submission is to be made, obligating the President to issue very clear from the decision, however, is that the factual situation the Court ruled
appointments within 90 days from the submission of the JBC list; Section 9 may upon, in the exercise of its supervision of court personnel, was the appointment by
the President of two RTC judges during the period of the ban. It is clear from the
decision, too, that no immediate appointment was ever made to the Court for the 696 SUPREME COURT REPORTS ANNOTATED
replacement of retired Justice Ricardo Francisco as the JBC failed to meet on the
required nominations prior to the onset of the election ban. From this perspective, De Castro vs. Judicial and Bar Council (JBC)
it appears clear to me that Valenzuela should be read and appreciated for what it one and all, that the terms of the Valenzuela ruling, if truly applicable even
is—a ruling made on the basis of the Court’s supervision over judicial to appointments to this Court, is not written in stone and remains open for review
personnel that upholds the election ban as against the appointment of lower court by this Court.
judges appointed pursuant to the period provided by Section 9 of Article VIII. Same; Same; Same; Same; Same; Completeness has a heightened meaning
Thus, Valenzuela’s application to the filling up of a vacancy in the Supreme Court when the missing Member is the head of the Judiciary and the Court in the person
is a mere obiter dictum as the Court is largely governed by Section 4(1) with of the Chief Justice.—Valenzuela rests on the reasoning that the evil that Section
respect to the period of appointment. The Section 4(1) period, of course and as 15 seeks to remedy—vote buying, midnight appointments and partisan reasons to
already mentioned above, has an impact uniquely its own that is different from influence the results of the election—is so pervasive so that the Section 15 ban
695 should prevail over everything else. The Court, however, forgot in some statements
in this case that hand in hand with Section 15 is Section 4(1) where the framers
VOL. 615, March 17, 2010 695
also recognized, in clear and absolute terms, that a vacancy in the Court should be
De Castro vs. Judicial and Bar Council (JBC) filled up because of the importance of having a Supreme Court with its full and
that created by the period provided for the lower court under Section 9. complete membership. Completeness has a heightened meaning when the missing
Same; Same; Same; Same; Stare Decisis; The stability of judgments is indeed Member is the head of the Judiciary and the Court in the person of the Chief
a glue that the Judiciary and the litigating public cannot do without if we are to Justice.
have a working and stable justice system.—I find it interesting that Peralta largely Same; Same; Same; Same; Same; What Valenzuela failed to consider, because
justifies his position that the JBC should now be prohibited from proceeding with it was looking at the disputed provisions from the prism of two Regional Trial Court
the nomination process based on Valenzuela as the prevailing rule that should be (RTC) judges, is that the reasons for the application of Section 15, Article VII may
followed under the principle of stare decisis. Tolentino apparently misappreciates not at all exist in appointments to the Supreme Court.—What Valenzuela failed to
the reach and real holding of Valenzuela, as explained and clarified above. A ruling consider, because it was looking at the disputed provisions from the prism of two
involving the appointment of lower court judges under Section 9, Article VIII RTC judges, is that the reasons for the application of Section 15, Article VII may
cannot simply be bodily lifted and applied in toto to the appointment of Members not at all exist in appointments to the Supreme Court. In the first place, Section
of the Supreme Court under Section 4(1) of the same Article. Because of his 4(1) covers only the appointment of 15 Members, not in their totality, but singly
misappreciation, Tolentino is likewise mistaken in his appeal to the principle and individually as Members disappear from the Court and are replaced. Thus,
of stare decisis. The stability of judgments is indeed a glue that the Judiciary and the evil that the Aytona case sought to remove—mass midnight appointments—
the litigating public cannot do without if we are to have a working and stable will not be present. Secondly, partisanship is hardly a reason that would apply to
justice system. Because of this role, the principle is one that binds all courts, the Supreme Court except when the Members of the Court individually act in
including this Court, and the litigating public. The principle, however, is not open- violation of their oaths or directly transgress our graft and corruption laws. Let it
ended and contains its own self-limitations; it applies only to actions in all be remembered that the Constitution itself has entrusted to the Court the final
future similar cases and to none other. Where ample room for distinction exists, and definitive recourse in election contest involving the President, the Vice-
as in this case, then stare decisis does not apply. President and Members of Congress. Because of this reposed trust on the Supreme
Same; Same; Same; Same; Same; Another aspect of stare decisis that must be Court as a body, reasons of partisanship can hardly be a reason to systemically
appreciated is that Supreme Court rulings are not written in stone so that they will place the whole Supreme Court under a ban on appointments during the election
remain unerased and applicable for all times.—Another aspect of stare decisis that period.697
must be appreciated is that Supreme Court rulings are not written in stone so that VOL. 615, March 17, 2010 697
they will remain unerased and applicable for all times. The Supreme Court’s
review of rulings and their binding effects is a continuing one so that a ruling in De Castro vs. Judicial and Bar Council (JBC)
one era may be declared by the Court at some future time to be no longer true and Same; Same; Same; Same; Same; One voice can be a big difference if the
should thus be abandoned and changed. The best and most unforgettable example missing voice is that of the Chief Justice.—The importance of the presence of one
of this kind of change happened in the United States when the US Supreme Court Member of the Court can and should never be underestimated, particularly on
overturned the ruling in Plessy v. Fergusson that upheld the constitutionality of issues that may gravely affect the nation. Many a case has been won or lost on the
racial segregation under the “separate but equal” doctrine. After half a century, basis of one vote. On an issue of the constitutionality of a law, treaty or statute, a
the US Court completely abandoned this ruling in the now famous Brown v. Board tie vote—which is possible in a 14 member court—means that the constitutionality
of Education when it ruled that “separate but equal” doctrine is inherently is upheld. This was our lesson in Isagani Cruz v. DENR Secretary, 347 SCRA 128
unequal in the context of public education. I mention this, if only as a reminder to (2000). More than the vote, Court deliberation is the core of the decision-making
696 process and one voice less is not only a vote less but a contributed opinion, an
observation, or a cautionary word less for the Court. One voice can be a big occurrence. This presupposes that the incumbent President should have the list on
difference if the missing voice is that of the Chief Justice. or before May 17, the day the vacancy occurs, so she can comply with her duty
Same; Same; Same; Same; The reasons the former Chief Justice cited in under the Constitution to make the appointment within the 90-day period provided
Valenzuela justify the application of the Section 15, Article VII as against the rule by it. Of course, the circumstances is such that the period for appointing the Chief
on appointment of lower court judges under Section 9, Article VIII—Section 9 does Justice’s replacement will span the tenure of the incumbent President (for 44 days)
not impose a hard and fast rule on the period to be observed, apparently because and her successor (for 46 days), but it is the incumbent’s call whether to exercise
the urgency of the appointment may not be as great as in the appointment of the power or pass it on. Again, assuming as correct petitioners’ view that the ban
Members of the Supreme Court.—Where, as in Valenzuela, the Chief Justice of the on midnight appointments does not apply to the judiciary, the JBC’s suspension of
Supreme Court, no less, appeared to have given up the benefit of an immediate its selection process places it in default, given its above duty in regard to the
appointment of Members of the Supreme Court, then extremely compelling submission of its list of nominees to the President within a time constraint. Under
reasons must have driven the Court to its conclusion. I fully understood though the same assumption, moreover, the petitioner citizens and members of the bar
the former Chief Justice’s conclusion in this case when I realized that he was not would have a demandable right or interest in having the JBC proceed with its
effectively ruling on Section 4(1) of Article VIII, and was in fact ruling on a case 699
involving lower court judges. For indeed, the reasons the former Chief Justice cited VOL. 615, March 17, 2010 699
in Valenzuela justify the application of the Section 15, Article VII as against the
rule on appointment of lower court judges under Section 9, Article VIII. As I have De Castro vs. Judicial and Bar Council (JBC)
shown above, Section 9 does not impose a hard and fast rule on the period to be selection process and submit its list of nominees in time for the incumbent
observed, apparently because the urgency of the appointment may not be as great President or her successor to fill up the vacancy within the period required by the
as in the appointment of Members of the Supreme Court. The period for Constitution. Alternatively, assuming that an actual controversy has not yet
appointment can move at the discretion of the JBC, although the exercise of this developed as to warrant action on the petitions filed in this case, the Court has the
discretion also carries its own butt-in and implicit limits. authority, as an incident of its power of supervision over the JBC, to see to it that
ABAD, J., Concurring Opinion: the JBC faithfully executes its duties as the Constitution requires of it.
Presidency; Appointments; Midnight Appointment Ban; Judicial Review; Same; Same; Same; Same; Same; While the President can freely choose to
Actual Case or Controversy; Under the circumstances, appoint any person who meets the basic qualifications for a position in the Executive
698 Department, he does not have such freedom of choice when it comes to appointments
in the judiciary—in the latter case, the Constitution provides in Section 9 of Article
698 SUPREME COURT REPORTS ANNOTATED
VIII that the President can choose his appointee only from a Judicial and Bar
De Castro vs. Judicial and Bar Council (JBC) Council (JBC) short list of its nominees.—Citing “In Re: Appointments dated March
the controversy is already ripe for adjudication for, assuming that the ban on 30, 1998 of Hon. Mateo A. Valenzuela and Hon. Placido B. Vallarta as Judges of
midnight appointment does not apply to the judiciary, then the Judicial and Bar the Regional Trial Court of Branch 62, Bago City and of Branch 24, Cabanatuan
Council’s (JBC’s) suspension of its selection process would constitute a violation of City,” 298 SCRA 408 (1998), the oppositors claim that the ban on midnight
its duty under the Constitution to carry on with such process until it is able to appointments applies to the judiciary. After examining the reasons for the two
submit the desired list to the incumbent President.—After taking some steps in the apparently conflicting provisions, the Court said that the need to fill up vacancies
selection process, the JBC held the process in abeyance, unable to decide as yet in the judiciary within the period the Constitution provides must yield to the ban
when and to whom it will submit its list of nominees for the position that Chief on Presidential midnight appointments. The Court explained this
Justice Puno will vacate on May 17, 2010. Under the circumstances, the ruling: Considering the respective reasons for the time frames for filling
controversy is already ripe for adjudication for, assuming that the ban on midnight vacancies in the courts and the restriction on the President’s power of
appointment does not apply to the judiciary as the petitioners would have it, then appointment, it is this Court’s view that, as a general proposition, in case
the JBC’s suspension of its selection process would constitute a violation of its duty of conflict, the former should yield to the latter. Surely, the prevention of
under the Constitution to carry on with such process until it is able to submit the vote-buying and similar evils outweighs the need for avoiding delays in
desired list to the incumbent President. If my subdivision neighbor begins filling up of court vacancies or the disposition of some cases. Temporary
constructing a shed in his yard and tells me that he has ordered 20 pigs to raise vacancies can abide the period of the ban which, incidentally and as
there, I will not wait till the pigs arrive and defecate before I bring an action to earlier pointed out, comes to exist only once in every six years. Moreover,
abate a nuisance. those occurring in the lower courts can be filled temporarily by
Same; Same; Same; Same; Judicial and Bar Council; Assuming that an designation. But prohibited appointments are long-lasting and
actual controversy has not yet developed as to warrant action on the petitions filed permanent in their effects. They may, as earlier pointed out, in fact
in this case, the Court has the authority, as an incident of its power of supervision influence the results of elections and, for that reason, their making is
over the Judicial and Bar Council (JBC), to see to it that the JBC faithfully executes considered an election offense. But the above assumes that the outgoing
its duties as the Constitution requires of it.—As mandated by the Constitution, the incumbent President can make appointments in the judiciary during
incumbent President should be able to fill up the vacancy within 90 days of its 700
700 SUPREME COURT REPORTS ANNOTATED Saklolo A. Leaño, Rita Linda V. Jimeno and Rico A. Limpengcofor petitioner
in G.R. No. 191420.
De Castro vs. Judicial and Bar Council (JBC) Estelito P. Mendoza for petitioner in A.M. No. 10-2-5-SC.
the period of the ban “to buy votes” and commit “similar evils” like denying Amador Z. Tolentino, Jr. and Roland B. Inting for petitioner in G.R. No.
the incoming President the opportunity to consider other appointees in the light of 191342.
his new policies, a point former President Diosdado Macapagal made in Aytona v. Arturo M. De Castro for and on his own behalf in G.R. No. 191002.
Castillo, 4 SCRA (1962). The fact, however, is that while the President can freely Jaime N. Soriano for himself in G.R. No. 191032.
choose to appoint any person who meets the basic qualifications for a position in Manuel M. Lazaro, et al. for petitioner in G.R. No. 191057.
the Executive Department, he does not have such freedom of choice when it comes Benjamin P. Lozada III, et al. for movant-intervenor Atty. Marlon B. Urbano.
to appointments in the judiciary. In the latter case, the Constitution provides in Pitero M. Reig for oppositor-in-intervention Board of the Integrated Bar of the
Section 9 of Article VIII that the President can choose his appointee only from a Philippines-Pasay, Parañaque, Las Piñas and Muntinlupa Chapters.
JBC short list of its nominees. Edre U. Olalia, et al. for oppositor-intervenor.
Same; Same; Same; Same; Same; The idea that the outgoing incumbent Al A. Parreño for movant oppositors-in-intervention.
President can take advantage of her appointment of a Chief Justice to buy votes in Ibarra M. Gutierrez for oppositor-intervenors Walden F. Bello and Loreta Ann
the coming elections is utterly ridiculous—she has no control over the Judicial and P. Rosales.
Bar Council’s (JBC’s) actions.—In reality, a President’s choice of Chief Justice is 702
in fact first a choice of the JBC before it is that of the President. Easily there should
702 SUPREME COURT REPORTS ANNOTATED
at least be 20,000 lawyers who are 40 years of age and have 15 years of law practice
of some kind who could qualify for Chief Justice. Yet, the President can choose only De Castro vs. Judicial and Bar Council (JBC)
from a list of three, four, or five lawyers that the JBC draws up for him. BERSAMIN, J.:
Consequently, the idea that the outgoing incumbent President can take advantage The compulsory retirement of Chief Justice Reynato S. Puno by May 17, 2010
of her appointment of a Chief Justice to buy votes in the coming elections is utterly occurs just days after the coming presidential elections on May 10, 2010. Even
ridiculous. She has no control over the JBC’s actions. before the event actually happens, it is giving rise to many legal dilemmas. May
Same; Same; Same; Same; Same; Separation of Powers; The Supreme Court the incumbent President appoint his successor, considering that Section 15, Article
that the Chief Justice heads is not a support agency under the President; The VII (Executive Department) of the Constitution prohibits the President or Acting
proposition that a Chief Justice will always be beholden to the President who President from making appointments within two months immediately before the
appoints him is a myth.—The idea that the incoming President should have the next presidential elections and up to the end of his term, except temporary
opportunity to choose a Chief Justice who will support his policies does not also appointments to executive positions when continued vacancies therein will
make sense. The Supreme Court that the Chief Justice heads is not a support prejudice public service or endanger public safety? What is the relevance of Section
agency under the President. One of the functions of the Supreme Court is to 4 (1), Article VIII (Judicial Department) of the Constitution, which provides that
provide a Constitutional check on abuses of the Executive Department. The any vacancy in the Supreme Court shall be filled within 90 days from the
proposition that a Chief Justice will always be beholden to the President who occurrence thereof, to the matter of the appointment of his successor? May the
appoints him is a myth. Former President Estrada appointed Chief Justice Hilario Judicial and Bar Council (JBC) resume the process of screening the candidates
G. Davide, Jr. who presided over his impeachment and administered the oath to nominated or being considered to succeed Chief Justice Puno, and submit the list
the incumbent President at the heels of EDSA II while President Estrada still sat of nominees to the incumbent President even during the period of the prohibition
in Malacañang. Chief Justices Artemio V. Pangani- under Section 15, Article VII? Does mandamus lie to compel the submission of the
701 shortlist of nominees by the JBC?
VOL. 615, March 17, 2010 701 Precís of the Consolidated Cases
Petitioners Arturo M. De Castro and John G. Peralta respectively commenced
De Castro vs. Judicial and Bar Council (JBC) G.R. No. 1910021 and G.R. No. 1911492 as special civil actions
ban and Reynato S. Puno voted against positions taken by the for certiorari and mandamus, praying that the JBC be compelled to submit to the
administration of the incumbent President who appointed them both to their incumbent President the list of at least three nominees for the position of the next
position. These Chief Justices like those before them were first choices of the JBC Chief Justice.
before they were those of the Presidents concerned.
SPECIAL CIVIL ACTIONS in the Supreme Court. Certiorari, Prohibition and _______________
Mandamus; Query on the Applicability of Section 15, Article VII to
Appointments to the Judiciary; and PETITION to Enjoin the JBC from
1 Filed on February 9, 2010.
Submitting a List of Nominees for the Position of Chief Justice to the
2 Begun on February 23, 2010.
President.
703
The facts are stated in the opinion of the Court.
VOL. 615, March 17, 2010 703 during the election ban—has impelled the JBC to defer the decision to whom to
send its list of at least three nominees, whether to the incumbent President or to
De Castro vs. Judicial and Bar Council (JBC) her successor.8 He opines that the JBC is thereby arrogating unto itself “the
In G.R. No. 191032,3 Jaime N. Soriano, via his petition for prohibition, judicial function that is not conferred upon it by the Constitution,” which has
proposes to prevent the JBC from conducting its search, selection and nomination limited it to the task of recommending appointees to the Judiciary, but has not
proceedings for the position of Chief Justice. empowered it to “finally resolve constitutional questions, which is the power vested
In G.R. No. 191057, a special civil action for mandamus,4 the Philippine only in the Supreme Court under the Constitution.” As such, he contends that the
Constitution Association (PHILCONSA) wants the JBC to submit its list of JBC acted with grave abuse of discretion in deferring the submission of the list of
nominees for the position of Chief Justice to be vacated by Chief Justice Puno upon nominees to the President; and that a “final and definitive resolution of the
his retirement on May 17, 2010, because the incumbent President is not covered constitutional questions raised above would diffuse (sic) the tension in the legal
by the prohibition that applies only to appointments in the Executive Department. community that would go a long way to keep and maintain stability in the judiciary
In Administrative Matter No. 10-2-5-SC,5 petitioner Estelito M. Mendoza, a and the political system.”9
former Solicitor General, seeks a ruling from the Court for the guidance of the JBC
on whether Section 15, Article VII applies to appointments to the Judiciary. _______________
In G.R. No. 191342,6 which the Court consolidated on March 9, 2010 with the
petitions earlier filed, petitioners Amador Z. Tolentino, Jr. and Roland B. Inting, 7 A.M. No. 98-5-01-SC, November 9, 1998, 298 SCRA 408.
Integrated Bar of the Philippines (IBP) Governors for Southern Luzon and Eastern 8 Petition in G.R. No. 191002, pp. 3-4.
Visayas, respectively, want to enjoin and restrain the JBC from submitting a list 9 Id., p. 5.
of nominees for the position of Chief Justice to the President for appointment
705
during the period provided for in Section 15, Article VII.
All the petitions now before the Court pose as the principal legal question VOL. 615, March 17, 2010 705
whether the incumbent President can appoint the successor of Chief Justice Puno De Castro vs. Judicial and Bar Council (JBC)
upon his retirement. That question is undoubtedly impressed with transcendental In G.R. No. 191032, Soriano offers the view that the JBC committed a grave
importance to the Nation, because the appointment of the Chief Justice is any abuse of discretion amounting to lack or excess of its jurisdiction when it resolved
President’s most important appointment. unanimously on January 18, 2010 to open the search, nomination, and selection
process for the position of Chief Justice to succeed Chief Justice Puno, because the
_______________ appointing authority for the position of Chief Justice is the Supreme Court itself,
the President’s authority being limited to the appointment of the Members of the
3 Initiated on February 10, 2010. Supreme Court. Hence, the JBC should not intervene in the process, unless a
4 Commenced on February 11, 2010. nominee is not yet a Member of the Supreme Court.10
5 Dated February 15, 2010. For its part, PHILCONSA observes in its petition in G.R. No. 191057 that
6 Filed on March 8, 2010. “unorthodox and exceptional circumstances spawned by the discordant
704 interpretations, due perhaps to a perfunctory understanding, of Sec. 15, Art. VII
704 SUPREME COURT REPORTS ANNOTATED in relation to Secs. 4(1), 8(5) and 9, Art. VIII of the Constitution” have bred “a
frenzied inflammatory legal debate on the constitutional provisions mentioned
De Castro vs. Judicial and Bar Council (JBC) that has divided the bench and the bar and the general public as well, because of
its dimensional impact to the nation and the people,” thereby fashioning
A precedent frequently cited is In Re Appointments Dated March 30, 1998 of “transcendental questions or issues affecting the JBC’s proper exercise of its
Hon. Mateo A. Valenzuela and Hon. Placido B. Vallarta as Judges of the Regional “principal function of recommending appointees to the Judiciary” by submitting
Trial Court of Branch 62, Bago City and of Branch 24, Cabanatuan City, only to the President (not to the next President) “a list of at least three nominees
respectively (Valenzuela),7 by which the Court held that Section 15, Article VII prepared by the Judicial and Bar Council for every vacancy” from which the
prohibited the exercise by the President of the power to appoint to judicial positions members of the Supreme Court and judges of the lower courts may be
during the period therein fixed. appointed.”11 PHILCONSA further believes and submits that now is the time to
In G.R. No. 191002, De Castro submits that the conflicting opinions on the revisit and review Valenzuela, the “strange and exotic Decision of the Court en
issue expressed by legal luminaries—one side holds that the incumbent President banc.”12
is prohibited from making appointments within two months immediately before Peralta states in his petition in G.R. No. 191149 that mandamuscan compel
the coming presidential elections and until the end of her term of office as the JBC “to immediately transmit to the
President on June 30, 2010, while the other insists that the prohibition applies
only to appointments to executive positions that may influence the election and, _______________
anyway, paramount national interest justifies the appointment of a Chief Justice
10 Petition in G.R. No. 191032, pp. 4-8. It will publish the opening of the position for applications or recommendations;
11 Petition in G.R. No. 191057, pp. 1-2. deliberate on the list of candidates; publish the names of candidates; accept
12 Id., at p. 11. comments on or opposition to the applications; conduct public interviews of
706 candidates; and prepare the shortlist of candidates.
706 SUPREME COURT REPORTS ANNOTATED As to the time to submit this shortlist to the proper appointing authority, in
the light of the Constitution, existing laws and jurisprudence, the JBC welcomes
De Castro vs. Judicial and Bar Council (JBC) and will consider all views on the matter.
President, within a reasonable time, its nomination list for the position of chief 18 January 2010.
justice upon the mandatory retirement of Chief Justice Reynato S. Puno, in (sgd.)
compliance with its mandated duty under the Constitution” in the event that the MA. LUISA D. VILLARAMA
Court resolves that the President can appoint a Chief Justice even during the Clerk of Court &
election ban under Section 15, Article VII of the Constitution.13 Ex-Officio Secretary
The petitioners in G.R. No. 191342 insist that there is an actual controversy, Judicial and Bar Council
considering that the “JBC has initiated the process of receiving applications for As a result, the JBC opened the position of Chief Justice for application or
the position of Chief Justice and has in fact begun the evaluation process for the recommendation, and published for that purpose its announcement dated January
applications to the position,” and “is perilously near completing the nomination 20, 2010,16 viz.:
process and coming up with a list of nominees for submission to the President, “The Judicial and Bar Council (JBC) announces the opening for application or
entering into the period of the ban on midnight appointments on March 10, 2010,” recommendation, of the position of CHIEF JUSTICE OF THE SUPREME COURT,
which “only highlights the pressing and compelling need for a writ of prohibition which will be vacated on 17 May 2010 upon the retirement of the incumbent Chief
to enjoin such alleged ministerial function of submitting the list, especially if it will Justice, HON. REYNATO S. PUNO.
be cone within the period of the ban on midnight appointments.”14 Applications or recommendations for this position must be submitted not later
than 4 February 2010 (Thursday) to the JBC Secretariat xxx:”
Antecedents
_______________
These cases trace their genesis to the controversy that has arisen from the
forthcoming compulsory retirement of Chief Justice Puno on May 17, 2010, or 15 http://jbc.judiciary.gov.ph/announcements/JBCreCJ.pdf
seven days after the presidential election. Under Section 4(1), in relation to Section 16 http://jbc.judiciary.gov.ph/announcements/jbc_announce_2009/jan22%20%
9, Article VIII, that“vacancy shall be filled within ninety days from the occurrence 2710.pdf
thereof” from a “list of at least three nominees prepared by the Judicial and Bar 708
Council for every vacancy.”
708 SUPREME COURT REPORTS ANNOTATED
On December 22, 2009, Congressman Matias V. Defensor, an ex officio member
of the JBC, addressed a letter to the JBC, requesting that the process for De Castro vs. Judicial and Bar Council (JBC)
nominations to the office of the Chief Justice be commenced immediately. The announcement was published on January 20, 2010 in the Philippine Daily
Inquirer and The Philippine Star.17
_______________ Conformably with its existing practice, the JBC “automatically considered” for
the position of Chief Justice the five most senior of the Associate Justices of the
13 Petition in G.R. No. 191149. Court, namely: Associate Justice Antonio T. Carpio; Associate Justice Renato C.
14 Petition in G.R. No. 191342. Corona; Associate Justice Conchita Carpio Morales; Associate Justice Presbitero
707 J. Velasco, Jr.; and Associate Justice Antonio Eduardo B. Nachura. However, the
last two declined their nomination through letters dated January 18, 2010 and
VOL. 615, March 17, 2010 707
January 25, 2010, respectively.18
De Castro vs. Judicial and Bar Council (JBC) Others either applied or were nominated. Victor Fernandez, the retired Deputy
Ombudsman for Luzon, applied, but later formally withdrew his name from
In its January 18, 2010 meeting en banc, therefore, the JBC passed a consideration through his letter dated February 8, 2010. Candidates who accepted
resolution,15 which reads: their nominations without conditions were Associate Justice Renato C. Corona;
“The JBC, in its en banc meeting of January 18, 2010, unanimously agreed to Associate Justice Teresita J. Leonardo-De Castro; Associate Justice Arturo D.
start the process of filling up the position of Chief Justice to be vacated on May 17, Brion; and Associate Justice Edilberto G. Sandoval (Sandiganbayan). Candidates
2010 upon the retirement of the incumbent Chief Justice Honorable Reynato S. who accepted their nominations with conditions were Associate Justice Antonio T.
Puno. Carpio and Associate Justice Conchita Carpio Morales.19 Declining their
nominations were Atty. Henry Villarica (via telephone conversation with the 21 Id.
Executive Officer of the JBC on February 5, 2010) and Atty. Gregorio M. Batiller, 22 Id., at p. 6.
Jr. (via telephone conversation with the Executive Officer of the JBC on February 710
8, 2010).20 710 SUPREME COURT REPORTS ANNOTATED
The JBC excluded from consideration former RTC Judge Florentino Floro (for
failure to meet the standards set by the JBC rules); and Special Prosecutor Dennis De Castro vs. Judicial and Bar Council (JBC)
Villa-Ignacio of the achieved by a ruling from the Court, the integrity of the process and the credibility
of whoever is appointed to the position of Chief Justice, may irreparably be
_______________ impaired.”23
Accordingly, we reframe the issues as submitted by each petitioner in the order
17 Comment of the JBC, p. 3. of the chronological filing of their petitions.
18 Id.
19 Id., at pp. 4-5. G.R. No. 191002
20 Id., at p. 5.
709 a. Does the JBC have the power and authority to resolve the constitutional
question of whether the incumbent President can appoint a Chief Justice during
VOL. 615, March 17, 2010 709
the election ban period?
De Castro vs. Judicial and Bar Council (JBC) b. Does the incumbent President have the power and authority to appoint
Office of the Ombudsman (due to cases pending in the Office of the Ombudsman).21 during the election ban the successor of Chief Justice Puno when he vacates the
In its meeting of February 8, 2010, the JBC resolved to proceed to the next step position of Chief Justice on his retirement on May 17, 2010?
of announcing the names of the following candidates to invite the public to file G.R. No. 191032
their sworn complaint, written report, or opposition, if any, not later than a. Is the power to appoint the Chief Justice vested in the Supreme Court en
February 22, 2010, to wit: Associate Justice Carpio, Associate Justice Corona, banc?
Associate Justice Carpio Morales, Associate Justice Leonardo-De Castro, Associate G.R. No. 191057
Justice Brion, and Associate Justice Sandoval. The announcement came out in a. Is the constitutional prohibition against appointment under Section 15,
the Philippine Daily Inquirer and The Philippine Star issues of February 13, Article VII of the Constitution applicable only to positions in the Executive
2010.22 Department?
b. Assuming that the prohibition under Section 15, Article VII of the
Issues Constitution also applies to members of the Judiciary, may such
appointments be excepted because they are impressed with public interest
Although it has already begun the process for the filling of the position of Chief or are demanded by the exigencies of public service, thereby justifying these
Justice Puno in accordance with its rules, the JBC is not yet decided on when to appointments during the period of prohibition?
submit to the President its list of nominees for the position due to the controversy c. Does the JBC have the authority to decide whether or not to include and
now before us being yet unresolved. In the meanwhile, time is marching in quick submit the names of nominees who mani-
step towards May 17, 2010 when the vacancy occurs upon the retirement of Chief
Justice Puno. _______________
The actions of the JBC have sparked a vigorous debate not only among legal
luminaries, but also among non-legal quarters, and brought out highly disparate 23 Petition in A.M. No. 10-2-5-SC, pp. 5-6.
opinions on whether the incumbent President can appoint the next Chief Justice 711
or not. Petitioner Mendoza notes that in Valenzuela, which involved the VOL. 615, March 17, 2010 711
appointments of two judges of the Regional Trial Court, the Court addressed this
issue now before us as an administrative matter “to avoid any possible polemics De Castro vs. Judicial and Bar Council (JBC)
concerning the matter,” but he opines that the polemics leading fested interest to be nominated for the position of Chief Justice on the
to Valenzuela “would be miniscule [sic] compared to the “polemics” that have now understanding that his/her nomination will be submitted to the next President in
erupted in regard to the current controversy,” and that unless “put to a halt, and view of the prohibition against presidential appointments from March 11, 2010
this may only be until June 30, 2010?
A.M. No. 10-2-5-SC
_______________ a. Does Section 15, Article VII of the Constitution apply to appointments to
positions in the Judiciary under Section 9, Article VIII of the Constitution?
b.May President Gloria Macapagal-Arroyo make appointments to the _______________
Judiciary after March 10, 2010, including that for the position of Chief
Justice after Chief Justice Puno retires on May 17, 2010? 24 Comment of the JBC, p. 6.
G.R. No. 191149 25 Id., at p. 7; bold emphasis is in the original text.
a. Does the JBC have the discretion to withhold the submission of the short 26 Comment of the OSG, pp. 13-14.
list to President Gloria Macapagal-Arroyo? 713
G.R. No. 191342
VOL. 615, March 17, 2010 713
a. Does the JBC have the authority to submit the list of nominees to the
incumbent President without committing a grave violation of the De Castro vs. Judicial and Bar Council (JBC)
Constitution and jurisprudence prohibiting the incumbent President from it commenced and set in motion the process of selecting the nominees to be
making midnight appointments two months immediately preceding the submitted to the President for the position of Chief Justice to be vacated by Chief
next presidential elections until the end of her term? Justice Puno;27 (c) petitioner Soriano’s theory that it is the Supreme Court, not the
b. Is any act performed by the JBC, including the vetting of the candidates President, who has the power to appoint the Chief Justice, is incorrect, and
for the position of Chief Justice, constitutionally invalid in view of the JBC’s proceeds from his misinterpretation of the phrase “members of the Supreme Court”
illegal composition allowing each member from the Senate and the House found in Section 9, Article VIII of the Constitution as referring only to the
of Representatives to have one vote each? Associate Justices, to the exclusion of the Chief Justice; 28 (d) a writ
On February 16, 2010, the Court directed the JBC and the Office of the of mandamus can issue to compel the JBC to submit the list of nominees to the
Solicitor General (OSG) to comment on the consolidated petitions, except that filed President, considering that its duty to prepare the list of at least three nominees
in G.R. No. 191342.712 is unqualified, and the submission of the list is a ministerial act that the JBC is
712 SUPREME COURT REPORTS ANNOTATED mandated to perform under the Constitution; as such, the JBC, the nature of
whose principal function is executive, is not vested with the power to resolve who
De Castro vs. Judicial and Bar Council (JBC) has the authority to appoint the next Chief Justice and, therefore, has no discretion
to withhold the list from the President; 29 and (e) a writ of mandamus cannot issue
On February 26, 2010, the JBC submitted its comment, reporting therein that to compel the JBC to include or exclude particular candidates as nominees,
the next stage of the process for the selection of the nominees for the position of considering that there is no imperative duty on its part to include in or exclude
Chief Justice would be the public interview of the candidates and the preparation from the list particular individuals, but, on the contrary, the JBC’s determination
of the short list of candidates, “including the interview of the constitutional of who it nominates to the President is an exercise of a discretionary duty. 30
experts, as may be needed.”24 It stated:25 The OSG contends that the incumbent President may appoint the next Chief
Likewise, the JBC has yet to take a position on when to submit the Justice, because the prohibition under Section 15, Article VII of the Constitution
shortlist to the proper appointing authority, in light of Section 4 (1), does not apply to appointments in the Supreme Court. It argues that any vacancy
Article VIII of the Constitution, which provides that vacancy in the in the Supreme Court must be filled within 90 days from its occurrence, pursuant
Supreme Court shall be filled within ninety (90) days from the occurrence to Section 4(1), Article VIII of the Constitution;31 that in their deliberations on the
thereof, Section 15, Article VII of the Constitution concerning the ban on manda-
Presidential appointments “two (2) months immediately before the next
presidential elections and up to the end of his term” and Section 261 (g), _______________
Article XXII of the Omnibus Election Code of the Philippines.
12. Since the Honorable Supreme Court is the final interpreter of the 27 Id., at p. 14.
Constitution, the JBC will be guided by its decision in these consolidated 28 Id., at p. 15.
Petitions and Administrative Matter. 29 Id., at pp. 20-24.
On February 26, 2010, the OSG also submitted its comment, essentially stating 30 Id., at pp. 25-27.
that the incumbent President can appoint the successor of Chief Justice Puno upon 31 Id., at pp. 29-30.
his retirement by May 17, 2010. 714
The OSG insists that: (a) a writ of prohibition cannot issue to prevent the JBC
from performing its principal function under the Constitution to recommend 714 SUPREME COURT REPORTS ANNOTATED
appointees in the Judiciary; (b) the JBC’s function to recommend is a “continuing De Castro vs. Judicial and Bar Council (JBC)
process,” which does not begin with each vacancy or end with each nomination, tory period for the appointment of Supreme Court Justices, the framers neither
because the goal is “to submit the list of nominees to Malacañang on the very day mentioned nor referred to the ban against midnight appointments, or its effects on
the vacancy arises”;26 the JBC was thus acting within its jurisdiction when such period, or vice versa;32 that had the framers intended the prohibition to apply
to Supreme Court appointments, they could have easily expressly stated so in the
Constitution, which explains why the prohibition found in Article VII (Executive National interest compels the President to make such appointment for
Department) was not written in Article VIII (Judicial Department); and that the it is particularly during this crucial period when national leaders are
framers also incorporated in Article VIII ample restrictions or limitations on the seeking fresh mandates from the people that the Supreme Court, more
President’s power to appoint members of the Supreme Court to ensure its than at any other time, represents stability. Hence, a full court is ideal to
independence from “political vicissitudes” and its “insulation from political ensure not only due deliberation on and careful consideration of issues
pressures,”33 such as stringent qualifications for the positions, the establishment but also expeditious disposition of cases.
of the JBC, the specified period within which the President shall appoint a Indeed, such function becomes especially significant in view of the
Supreme Court Justice. fact that this is the first time that the whole country will experience
The OSG posits that although Valenzuela involved the appointment of RTC automated elections.
Judges, the situation now refers to the appointment of the next Chief Justice to 37 Id., at pp. 36-37. The OSG stresses:
which the prohibition does not apply; that, at any rate, Valenzuela even recognized The possible fallouts or serious aftermath of allowing a vacuum in the
that there might be “the imperative need for an appointment during the period of position of the Chief Justice may be greater and riskier than the
the ban,” like when the membership of the Supreme Court should be “so reduced consequences or repercussions of inaction. Needless to state, the
that it will have no quorum, or should the voting on a particular important appointment of the Chief Justice of this Honorable Court (sic) is the most
question requiring expeditious resolution be divided”; 34 and that Valenzuela also important appointment vested by the 1987 Constitution to (sic) the
recognized that the filling of vacancies in the Judiciary is undoubtedly in the public President.
interest, most especially if there is any compelling reason to justify the making of 38 Id., at p. 37.
the appointments during the period of the prohibition.35 39 Id., at p. 38.
Lastly, the OSG urges that there are now undeniably compelling reasons for 716
the incumbent President to appoint the 716 SUPREME COURT REPORTS ANNOTATED

_______________ De Castro vs. Judicial and Bar Council (JBC)


On March 9, 2010, the Court admitted the following comments/oppositions-in-
32 Id. intervention, to wit:
33 Id., at pp. 32-33. (a) The opposition-in-intervention dated February 22, 2010 of Atty. Peter
34 Id., at pp. 34-35. Irving Corvera (Corvera);40
35 Id. (b) The opposition-in-intervention dated February 22, 2010 of Atty. Christian
Robert S. Lim (Lim);
715
(c) The opposition-in-intervention dated February 23, 2010 of Atty. Alfonso V.
VOL. 615, March 17, 2010 715 Tan, Jr. (Tan);
De Castro vs. Judicial and Bar Council (JBC) (d) The comment/opposition-in-intervention dated March 1, 2010 of the
next Chief Justice, to wit: (a) a deluge of cases involving sensitive political issues National Union of People’s Lawyers (NUPL);
is “quite expected”;36 (b) the Court acts as the Presidential Electoral Tribunal (e) The opposition-in-intervention dated February 25, 2010 of Atty. Marlou B.
(PET), which, sitting en banc, is the sole judge of all contests relating to the Ubano (Ubano);
election, returns, and qualifications of the President and Vice President and, as (f) The opposition-in-intervention dated February 25, 2010 of Integrated Bar
such, has “the power to correct manifest errors on the statement of votes (SOV) of the Philippines-Davao del Sur Chapter and its Immediate Past
and certificates of canvass (COC)”;37 (c) if history has shown that during ordinary President, Atty. Israelito P. Torreon (IBP- Davao del Sur);
times the Chief Justice was appointed immediately upon the occurrence of the (g) The opposition-in-intervention dated February 26, 2010 of Atty. Mitchell
vacancy, from the time of the effectivity of the Constitution, there is now even more John L. Boiser (Boiser);
reason to appoint the next Chief Justice immediately upon the retirement of Chief (h) The consolidated comment/opposition-in-intervention dated February 26,
Justice Puno;38 and (d) should the next Chief Justice come from among the 2010 of BAYAN Chairman Dr. Carolina P. Araullo; BAYAN Secretary
incumbent Associate Justices of the Supreme Court, thereby causing a vacancy, it General Renato M. Reyes, Jr.; Confederation for Unity, Recognition and
also becomes incumbent upon the JBC to start the selection process for the filling Advancement of Government Employees (COURAGE) Chairman
up of the vacancy in accordance with the constitutional mandate.39 Ferdinand Gaite; Kalipunan ng Damayang Mahihirap (KADAMAY)
Secretary General Gloria Arellano; Alyansa ng Nagkakaisang Kabataan ng
_______________ Samayanan Para sa Kaunlaran (ANAKBAYAN) Chairman Ken Leonard
Ramos; Tayo ang Pag-asa Convenor Alvin Peters; League of Filipino
Students (LFS) Chairman James Mark Terry Lacuanan Ridon; National
36 Id., at pp. 35-36. The OSG posits:
Union of Students of the Philippines (NUSP) Chairman Einstein Recedes,
College Editors Guild of the Philippines (CEGP) Chairman Vijae Alquisola;
and Student Christian Movement of the Philippines (SCMP) Chairman Ma. that such a situation will create a crisis in the judicial system and will worsen an
Cristina Angela Guevarra (BAYAN et al.); already vulnerable political situation.
Intervenors Tan, Ubano, WTLOP, Bello et al., IBP Davao del Sur Corvera, and
_______________ Boiser regard De Castro’s argument that a permanent Chief Justice is imperative
for the stability of the judicial system and the political situation in the country
40 Filed by Atty. Pitero M. Reig. when the election-related questions reach the Court as false, because there is an
717 existing law on filling the void brought about by a vacancy in the office of Chief
Justice; that the law is Section 12 of the Judiciary Act of 1948, which has not been
VOL. 615, March 17, 2010 717
repealed by Batas Pambansa Blg. 129 or any other law; that a temporary or an
De Castro vs. Judicial and Bar Council (JBC) acting Chief Justice is not anathema to judicial independence; that the designation
of an actingChief Justice is not only provided for by law, but is also dictated by
(i)The opposition-in-intervention dated March 3, 2010 of Walden F. Bello and practical necessity; that the practice was intended to be enshrined in the 1987
Loretta Ann P. Rosales (Bello et al.); and Constitution, but the Commissioners decided not to write it in the Constitution on
(j) The consolidated comment/opposition-in-intervention dated March 4, 2010 account of the settled practice; that the practice was followed under the 1987
of the Women Trial Lawyers Organization of the Philippines (WTLOP), Constitution, when, in 1992, at the end of the term of Chief Justice Marcelo B.
represented by Atty. Yolanda Quisumbing-Javellana; Atty. Belleza Alojado Fernan, Associate Justice Andres Narvasa assumed the position as Acting Chief
Demaisip; Atty. Teresita Gandionco-Oledan; Atty. Ma. Verena Kasilag- Justice prior to his official appointment as Chief Justice; that said filling up of a
Villanueva; Atty. Marilyn Sta. Romana; Atty. Leonila de Jesus; and Atty. vacancy in the office of the Chief Justice was acknowledged and even used by
Guinevere de Leon (WTLOP). analogy in the case of the vacancy of the Chairman of the Commission on Elections,
Intervenors Tan, WTLOP, BAYAN et al., Corvera, IBP Davao del Sur, and per Brillantes v. Yorac, 192 SCRA 358; and that the history of the Supreme Court
NUPL take the position that De Castro’s petition was bereft of any basis, because has shown that this rule of succession has been repeatedly observed and has
under Section 15, Article VII, the outgoing President is constitutionally banned become a part of its tradition.719
from making any appointments from March 10, 2010 until June 30, 2010, VOL. 615, March 17, 2010 719
including the appointment of the successor of Chief Justice Puno.
Hence, mandamus does not lie to compel the JBC to submit the list of nominees to De Castro vs. Judicial and Bar Council (JBC)
the outgoing President if the constitutional prohibition is already in effect. Tan
adds that the prohibition against midnight appointments was applied by the Court Intervenors Ubano, Boiser, NUPL, Corvera, and Lim maintain that
to the appointments to the Judiciary made by then President Ramos, with the the Omnibus Election Code penalizes as an election offense the act of any
Court holding that the duty of the President to fill the vacancies within 90 days government official who appoints, promotes, or gives any increase in salary or
from occurrence of the vacancies (for the Supreme Court) or from the submission remuneration or privilege to any government official or employee during the period
of the list (for all other courts) was not an excuse to violate the constitutional of 45 days before a regular election; that the provision covers all appointing heads,
prohibition. officials, and officers of a government office, agency or instrumentality, including
Intervenors Tan, Ubano, Boiser, Corvera, NULP, BAYAN et al., and Bello et the President; that for the incumbent President to appoint the next Chief Justice
al. oppose the insistence that Valenzuela recognizes the possibility that the upon the retirement of Chief Justice Puno, or during the period of the ban under
President may appoint the next Chief Justice if exigent circumstances warrant the the Omnibus Election Code, constitutes an election offense; that even an
appointment, because that recognition is obiter dictum; and aver that the absence appointment of the next Chief Justice prior to the election ban is fundamentally
of a Chief Justice or even an Associate Justice does not cause epic damage or invalid and without effect because there can be no appointment until a vacancy
absolute disruption or paralysis in the operations of the Judiciary. They insist that occurs; and that the vacancy for the position can occur only by May 17, 2010.
even without the successor of Chief Justice Puno being appointed by the incumbent Intervenor Boiser adds that De Castro’s prayer to compel the submission of
President, the Court is allowed to nominees by the JBC to the incumbent President is off-tangent because the
718 position of Chief Justice is still not vacant; that to speak of a list, much more a
submission of such list, before a vacancy occurs is glaringly premature; that the
718 SUPREME COURT REPORTS ANNOTATED
proposed advance appointment by the incumbent President of the next Chief
De Castro vs. Judicial and Bar Council (JBC) Justice will be unconstitutional; and that no list of nominees can be submitted by
sit and adjudge en banc or in divisions of three, five or seven members at its the JBC if there is no vacancy.
discretion; that a full membership of the Court is not necessary; that petitioner De All the intervenors-oppositors submit that Section 15, Article VII makes no
Castro’s fears are unfounded and baseless, being based on a mere possibility, the distinction between the kinds of appointments made by the President; and that
occurrence of which is entirely unsure; that it is not in the national interest to have the Court, in Valenzuela, ruled that the appointments by the President of the two
a Chief Justice whose appointment is unconstitutional and, therefore, void; and judges during the prohibition period were void.
Intervenor WTLOP posits that Section 15, Article VII of the 1987 Constitution The main question presented in all the filings herein—because it involves two
does not apply only to the appointments in the Executive Department, but also to seemingly conflicting provisions of the Constitution—imperatively demands the
judicial appointments, contrary to the submission of PHILCONSA; that Section 15 attention and resolution of this Court, the only authority that can resolve the
does not distinguish; and that Valenzuela already question definitively and finally. The imperative demand rests on the ever-present
720 need, first, to safeguard the independence, reputation, and integrity of the entire
720 SUPREME COURT REPORTS ANNOTATED Judiciary, particularly this Court, an institution that has been unnecessarily
dragged into the harsh polemics brought on by the controversy; second, to settle
De Castro vs. Judicial and Bar Council (JBC) once and for all the doubt about an outgoing President’s power to appoint to the
interpreted the prohibition as applicable to judicial appointments. Judiciary within the long period starting two months before the presidential
Intervenor WTLOP further posits that petitioner Soriano’s contention that the elections until the end of the presidential term; and third, to set a definite
power to appoint the Chief Justice is vested, not in the President, but in the guideline for the JBC to follow in the discharge of its primary office of screening
Supreme Court, is utterly baseless, because the Chief Justice is also a Member of and nominating qualified persons for appointment to the Judiciary.
the Supreme Court as contemplated under Section 9, Article VIII; and that, at any Thus, we resolve.
rate, the term “members” was interpreted in Vargas v. Rillaroza (G.R. No. L-1612,
February 26, 1948) to refer to the Chief Justice and the Associate Justices of the Ruling of the Court
Supreme Court; that PHILCONSA’s prayer that the Court pass a resolution
declaring that persons who manifest their interest as nominees, but with Locus Standi of Petitioners
conditions, shall not be considered nominees by the JBC is diametrically opposed
to the arguments in the body of its petition; that such glaring inconsistency
The preliminary issue to be settled is whether or not the petitioners have locus
between the allegations in the body and the relief prayed for highlights the lack of
standi.722
merit of PHILCONSA’s petition; that the role of the JBC cannot be separated from
the constitutional prohibition on the President; and that the Court must direct the 722 SUPREME COURT REPORTS ANNOTATED
JBC to follow the rule of law, that is, to submit the list of nominees only to the next De Castro vs. Judicial and Bar Council (JBC)
duly elected President after the period of the constitutional ban against midnight
appointments has expired. Black defines locus standi as “a right of appearance in a court of justice on a
Oppositor IBP Davao del Sur opines that the JBC—because it is neither a given question.”41 In public or constitutional litigations, the Court is often
judicial nor a quasi-judicial body—has no duty under the Constitution to resolve burdened with the determination of the locus standi of the petitioners due to the
the question of whether the incumbent President can appoint a Chief Justice ever-present need to regulate the invocation of the intervention of the Court to
during the period of prohibition; that even if the JBC has already come up with a correct any official action or policy in order to avoid obstructing the efficient
short list, it still has to bow to the strict limitations under Section 15, Article VII; functioning of public officials and offices involved in public service. It is required,
that should the JBC defer submission of the list, it is not arrogating unto itself a therefore, that the petitioner must have a personal stake in the outcome of the
judicial function, but simply respecting the clear mandate of the Constitution; and controversy, for, as indicated in Agan, Jr. v. Philippine International Air
that the application of the general rule in Section 15, Article VII to the Judiciary Terminals Co., Inc.:42
does not violate the principle of separation of powers, because said provision is an “The question on legal standing is whether such parties have “alleged
exception.721 such a personal stake in the outcome of the controversy as to assure that
VOL. 615, March 17, 2010 721 concrete adverseness which sharpens the presentation of issues upon
De Castro vs. Judicial and Bar Council (JBC) which the court so largely depends for illumination of difficult
constitutional questions.”43 Accordingly, it has been held that the interest
of a person assailing the constitutionality of a statute must be direct and
Oppositors NUPL, Corvera, Lim and BAYAN et al. state that the JBC’s act of
personal. He must be able to show, not only that the law or any
nominating appointees to the Supreme Court is purely ministerial and does not
government act is invalid, but also that he sustained or is in imminent
involve the exercise of judgment; that there can be no default on the part of the
danger of sustaining some direct injury as a result of its enforcement, and
JBC in submitting the list of nominees to the President, considering that the call
not merely that he suffers thereby in some indefinite way. It must appear
for applications only begins from the occurrence of the vacancy in the Supreme
that the person complaining has been or is about to be denied some right
Court; and that the commencement of the process of screening of applicants to fill
or privilege to which he is lawfully entitled or that he is about to be
the vacancy in the office of the Chief Justice only begins from the retirement on
subjected to some burdens or penalties by reason of the statute or act
May 17, 2010, for, prior to this date, there is no definite legal basis for any party
complained of.”44
to claim that the submission or non-submission of the list of nominees to the
President by the JBC is a matter of right under law.
_______________
41 Black’s Law Dictionary, 941 (6th Ed. 1991). Commission on Elections,52 this Court decided to resolve the issues raised by the
42 G.R. No. 155001, May 5, 2003, 402 SCRA 612. petition due to their “far-reaching implications,” even if the petitioner had no
43 Citing Kilosbayan, Inc. v. Morato, G.R. No. 118910, July 17, 1995, 246 personality to file the suit. The liberal approach of Aquino v. Commission on
SCRA 540, 562-563, citing Baker v. Carr, 369 U.S. 186, 7 L. Ed. 633 (1962). Elections has been adopted in several notable cases, permitting ordinary citizens,
44 Citing Kilosbayan, Inc. v. Morato, supra; Bayan v. Zamora, G.R. No. legislators, and civic organizations to bring their suits involving the
138570, October 10, 2000; 342 SCRA 449, 478. constitutionality or validity of laws, regulations, and rulings.53
723
VOL. 615, March 17, 2010 723 _______________
De Castro vs. Judicial and Bar Council (JBC)
not file suit in their capacity as taxpayers without a showing that Balikatan 02-01
It is true that as early as in 1937, in People v. Vera,45 the Court adopted involved the exercise of Congress’ taxing or spending powers, reiterated Bagong
the direct injury test for determining whether a petitioner in a public action Alyansang Makabayan v. Zamora, declaring that cases of transcendental
had locus standi. There, the Court held that the person who would assail the importance must be settled promptly and definitely and the standing requirements
validity of a statute must have “a personal and substantial interest in the case may be relaxed); and Osmeña v. Commission on Elections, G.R. No. 100318,
such that he has sustained, or will sustain direct injury as a result.” Vera was 100308, 100417,100420, July 30, 1991, 199 SCRA 750 (in which the Court held
followed in Custodio v. President of the Senate,46 Manila Race Horse Trainers’ that where serious constitutional questions were involved, the transcendental
Association v. De la Fuente,47 Anti-Chinese League of the Philippines v. importance to the public of the cases demanded that they be settled promptly and
Felix,48 and Pascual v. Secretary of Public Works.49 definitely, brushing aside technicalities of procedure).
Yet, the Court has also held that the requirement of locus standi, being a mere
procedural technicality, can be waived by the Court in the exercise of its discretion.
52 L-No. 40004, January 31, 1975, 62 SCRA 275.
For instance, in 1949, in Araneta v. Dinglasan,50 the Court liberalized the
53 E.g., Tañada v. Tuvera, G.R. No. 63915, April 24, 1985, 136 SCRA 27 (in
approach when the cases had “transcendental importance.” Some notable
which the Court held that it is sufficient that the petitioner is a citizen interested
controversies whose petitioners did not pass the direct injury test were allowed to
in the execution of the law, because the question is one of public duty and the
be treated in the same way as in Araneta v. Dinglasan.51 In the 1975 decision
enforcement of a public right, and the people are the real party-in-
in Aquino v.
interest); Legaspi v. Civil Service Commission, G.R. No. 72119, May 29, 1987, 150
SCRA 530 (in which the Court declared that where an assertion of a public right
_______________ is involved, the requirement of personal interest is satisfied by the mere fact that
the petitioner is a citizen and is part of the general public which possesses the
45 65 Phil. 56. right); Kapatiran ng mga Naglilingkod sa Pamahalaan ng Pilipinas, Inc. v.
46 G.R. No. 117, November 7, 1945 (Unreported). Tan, L. No. 81311, June 30, 1988, 163 SCRA 371 (in which the Court disregarded
47 G.R. No. 2947, January 11, 1959 (Unreported). objections to taxpayers’ lack of personality to sue in determining the validity of the
48 77 Phil. 1012 (1947). VAT Law); Albano v. Reyes, G.R. No. 83551, July 11, 1989, 175 SCRA 264 (in which
49 110 Phil. 331 (1960). the Court pronounced that although no expenditure of public funds was involved
50 84 Phil. 368 (1949) in the questioned contract, the petitioner was nonetheless clothed with the legal
51 E.g., Chavez v. Public Estates Authority, G.R. No. 133250, July 9, 2002, 384 personality under the disclosure provision of the Constitution to question it,
SCRA 152 (in which the Court ruled that the enforcement of the constitutional considering its important role in the economic development of the country and the
right to information and the equitable diffusion of natural resources are matters 725
of transcendental importance which clothe the petitioner with locus
standi); Bagong Alyansang Makabayan v. Zamora, G.R. Nos. 138570, 138572, VOL. 615, March 17, 2010 725
138587, 138680, 138698, October 10, 2000, 342 SCRA 449 (in which the Court held De Castro vs. Judicial and Bar Council (JBC)
that “given the transcendental importance of the issues involved, the Court may However, the assertion of a public right as a predicate for challenging a
relax the standing requirements and allow the suit to prosper despite the lack of supposedly illegal or unconstitutional executive or legislative action rests on the
direct injury to the parties seeking judicial review” of the Visiting Forces theory that the petitioner represents the public in general. Although such
Agreement); Lim v. Executive Secretary, G.R. No. 151445, April 11, 2002, 380 petitioner may not be as adversely affected by the action complained against as
SCRA 739 (in which the Court, albeit conceding that the petitioners might are others, it is enough that he sufficiently demonstrates in his petition that he is
724 entitled to protection or relief from the Court in the vindication of a public right.
724 SUPREME COURT REPORTS ANNOTATED Quite often, as here, the petitioner in a public action sues as
a citizen or taxpayer to gain locus standi. That is not surprising, for even if the
De Castro vs. Judicial and Bar Council (JBC) issue may appear to concern only the public in general, such capacities nonetheless
equip the petitioner with adequate interest to sue. In David v. Macapagal- Batasang Pambansa and the Regular Batasang Pambansa, and former member of
Arroyo,54 the Court aptly explains why: the Faculty of the College of Law of the University of the Philippines.
“Case law in most jurisdictions now allows both “citizen” and “taxpayer”
standing in public actions. The distinction was first laid down in Beauchamp v. _______________
Silk,55 where it was held that the plaintiff in a taxpayer’s suit is in a different
category from the plaintiff in a citizen’s suit. In the former, the plaintiff is 57 232 NC 48, 59 SE2d 359 (1950).
affected by the expenditure of public funds, while in the latter, he is but 58 Bold emphasis is in the original text.
the mere instrument of the public concern. As held by the New York Supreme 59 Petition in G.R. No. 191032, p. 2.
Court in People ex rel Case v. Collins:56 “In matter of mere public right, 60 Petition in G.R. No. 191057, pp. 3-4; citing the cases of PHILCONSA v.
however…the people are the real parties…It is at least the right, if not the Gimenez, 15 SCRA 479 (1965); PHILCONSA v. Mathay, 18 SCRA 300
duty, of every citizen to interfere and see that a public offence be properly (1966); PHILCONSA v. Enriquez, 235 SCRA 506 (1994); and Lambino v.
pursued and pun- COMELEC, 505 SCRA 160 (2006).
727
_______________
VOL. 615, March 17, 2010 727

magnitude of the financial consideration involved, indicating that public interest De Castro vs. Judicial and Bar Council (JBC)
was definitely involved); and Association of Small Landowners in the Philippines,
Inc. v. Sec. of Agrarian Reform, G.R. No. 78742, July 14, 1989, 175 SCRA 343 (in The petitioners in G.R. No. 191342 are the Governors of the Integrated Bar of
which the Court ruled that it had the discretion to waive the requirement of locus the Philippines (IBP) for Southern Luzon and Eastern Visayas. They allege that
standi in determining the validity of the implementation of the Comprehensive they have the legal standing to enjoin the submission of the list of nominees by the
Agrarian Reform Program, although the petitioners were not, strictly speaking, JBC to the President, for “[a]n adjudication of the proper interpretation and
covered by the definition of proper party). application of the constitutional ban on midnight appointments with regard to
respondent JBC’s function in submitting the list of nominees is well within the
54 David v. Macapagal-Arroyo, G.R. No. 171396, May 3, 2006, 489 SCRA 160. concern of petitioners, who are duty bound to ensure that obedience and respect
55 275 Ky 91, 120 SW2d 765 (1938). for the Constitution is upheld, most especially by government offices, such as
56 19 Wend. 56 (1837). respondent JBC, who are specifically tasked to perform crucial functions in the
726 whole scheme of our democratic institution.” They further allege that, reposed in
them as members of the Bar, is a clear legal interest in the process of selecting the
726 SUPREME COURT REPORTS ANNOTATED members of the Supreme Court, and in the selection of the Chief Justice,
De Castro vs. Judicial and Bar Council (JBC) considering that the person appointed becomes a member of the body that has
ished, and that a public grievance be remedied.” With respect to taxpayer’s constitutional supervision and authority over them and other members of the legal
suits, Terr v. Jordan57 held that “the right of a citizen and a taxpayer to profession.61
maintain an action in courts to restrain the unlawful use of public funds The Court rules that the petitioners have each demonstrated adequate interest
to his injury cannot be denied.”58 in the outcome of the controversy as to vest them with the requisite locus
Petitioners De Castro (G.R. No. 191002), Soriano (G.R. No. 191032) and standi. The issues before us are of transcendental importance to the people as a
Peralta (G.R. No. 191149) all assert their right as citizens filing their petitions on whole, and to the petitioners in particular. Indeed, the issues affect everyone
behalf of the public who are directly affected by the issue of the appointment of the (including the petitioners), regardless of one’s personal interest in life, because
next Chief Justice. De Castro and Soriano further claim standing as taxpayers, they concern that great doubt about the authority of the incumbent President to
with Soriano averring that he is affected by the continuing proceedings in the JBC, appoint not only the successor of the retiring incumbent Chief Justice, but also
which involve “unnecessary, if not, illegal disbursement of public funds.” 59 others who may serve in the Judiciary, which already suffers from a far too great
PHILCONSA alleges itself to be a non-stock, non-profit organization existing number of vacancies in the ranks of trial judges throughout the country.
under the law for the purpose of defending, protecting, and preserving the In any event, the Court retains the broad discretion to waive the requirement
Constitution and promoting its growth and flowering. It also alleges that the Court of legal standing in favor of any petitioner when the matter involved has
has recognized its legal standing to file cases on constitutional issues in several transcendental impor-
cases.60
In A.M. No. 10-2-5-SC, Mendoza states that he is a citizen of the Philippines, _______________
a member of the Philippine Bar engaged in the active practice of law, and a former
Solicitor General, former Minister of Justice, former Member of the Interim 61 Petition in G.R. No. 191342, pp. 2-3.
728
728 SUPREME COURT REPORTS ANNOTATED Intervenor Tan raises the lack of any actual justiciable controversy that is ripe
for judicial determination, pointing out that petitioner De Castro has not even
De Castro vs. Judicial and Bar Council (JBC) shown that the JBC has already completed its selection process and is now ready
tance, or otherwise requires a liberalization of the requirement.62 to submit the list to the incumbent President; and that petitioner De Castro is
Yet, if any doubt still lingers about the locus standi of any petitioner, we dispel merely presenting a hypothetical scenario that is clearly not sufficient for the
the doubt now in order to remove any obstacle or obstruction to the resolution of Court to exercise its power of judicial review.
the essential issue squarely presented herein. We are not to shirk from discharging Intervenors Corvera and Lim separately opine that De Castro’s petition rests
our solemn duty by reason alone of an obstacle more technical than otherwise. on an overbroad and vague allegation of political tension, which is insufficient
In Agan, Jr. v. Philippine International Air Terminals Co., Inc.,63 we pointed out: basis for the Court to exercise its power of judicial review.
“Standing is a peculiar concept in constitutional law because in some cases, suits Intervenor BAYAN et al. contend that the petitioners are seeking a mere
are not brought by parties who have been personally injured by the operation of a advisory opinion on what the JBC and the President should do, and are not
law or any other government act but by concerned citizens, taxpayers or voters invoking any issues that are justiciable in nature.
who actually sue in the public interest.” But even if, strictly speaking, the Intervenors Bello et al. submit that there exist no conflict of legal rights and
petitioners “are not covered by the definition, it is still within the wide discretion no assertion of opposite legal claims in any of the petitions; that PHILCONSA does
of the Court to waive the requirement and so not allege any action taken by the JBC, but simply avers that the conditional
manifestations of two Members of the Court, accented by the divided opinions and
_______________ interpretations of legal experts, or associa-

62 See, for instance, Integrated Bar of the Philippines v. Zamora, G.R. No. _______________
141284, August 15, 2000, 338 SCRA 81 (where the petitioner questioned the
validity of the deployment and utilization of the Marines to assist the PNP in law 64 Id.
enforcement, asserting that IBP was the official organization of Filipino lawyers 730
tasked with the bounden duty to uphold the rule of law and the Constitution, but
730 SUPREME COURT REPORTS ANNOTATED
the Court held that the IBP had not shown that it was so tasked: “In this case, a
reading of the petition shows that the IBP has advanced constitutional issues De Castro vs. Judicial and Bar Council (JBC)
which deserve the attention of this Court in view of their seriousness, novelty and tions of lawyers and law students on the issues published in the daily
weight as precedents. Moreover, because peace and order are under constant newspapers are “matters of paramount and transcendental importance to the
threat and lawless violence occurs in increasing tempo, undoubtedly aggravated bench, bar and general public”; that PHILCONSA fails not only to cite any legal
by the Mindanao insurgency problem, the legal controversy raised in the petition duty or allege any failure to perform the duty, but also to indicate what specific
almost certainly will not go away. It will stare us in the face again. It, therefore, action should be done by the JBC; that Mendoza does not even attempt to portray
behooves the Court to relax the rules on standing and to resolve the issue now, the matter as a controversy or conflict of rights, but, instead, prays that the Court
rather than later”, and went on to resolve the issues because the petitioner should “rule for the guidance of” the JBC; that the fact that the Court supervises
advanced constitutional issues that deserved the attention of the Court in view of the JBC does not automatically imply that the Court can rule on the issues
their seriousness, novelty, and weight as precedents). presented in the Mendoza petition, because supervision involves oversight, which
63 Supra, note 42, p. 645. means that the subordinate officer or body must first act, and if such action is not
729 in accordance with prescribed rules, then, and only then, may the person exercising
VOL. 615, March 17, 2010 729 oversight order the action to be redone to conform to the prescribed rules; that the
Mendoza petition does not allege that the JBC has performed a specific act
De Castro vs. Judicial and Bar Council (JBC) susceptible to correction for being illegal or unconstitutional; and that the
remove the impediment to its addressing and resolving the serious constitutional Mendoza petition asks the Court to issue an advisory ruling, not to exercise its
questions raised.”64 power of supervision to correct a wrong act by the JBC, but to declare the state of
the law in the absence of an actual case or controversy.
Justiciability We hold that the petitions set forth an actual case or controversy that is ripe
for judicial determination. The reality is that the JBC already commenced the
Intervenor NUPL maintains that there is no actual case or controversy that is proceedings for the selection of the nominees to be included in a short list to be
appropriate or ripe for adjudication, considering that although the selection submitted to the President for consideration of which of them will succeed Chief
process commenced by the JBC is going on, there is yet no final list of nominees; Justice Puno as the next Chief Justice. Although the position is not yet vacant, the
hence, there is no imminent controversy as to whether such list must be submitted fact that the JBC began the process of nomination pursuant to its rules and
to the incumbent President, or reserved for submission to the incoming President. practices, although it has yet to decide whether to submit the list of nominees to
the incumbent outgoing President or to the next President, makes the situation Prohibition under Section 15, Article VII does not apply to
ripe for judicial determination, because the next steps are the public interview of appointments to fill a vacancy in the Supreme Court or to other
the candidates, the preparation of the short list of candidates, and the “interview appointments to the Judiciary
of constitutional experts, as may be needed.”731 Two constitutional provisions are seemingly in conflict.
VOL. 615, March 17, 2010 731 The first, Section 15, Article VII (Executive Department), provides:
“Section 15. Two months immediately before the next presidential elections
De Castro vs. Judicial and Bar Council (JBC) and up to the end of his term, a President or Acting President shall not make
appointments, except temporary appointments to executive positions when
A part of the question to be reviewed by the Court is whether the JBC properly continued vacancies therein will prejudice public service or endanger public
initiated the process, there being an insistence from some of the oppositors- safety.”
intervenors that the JBC could only do so once the vacancy has occurred (that is, The other, Section 4 (1), Article VIII (Judicial Department), states:
after May 17, 2010). Another part is, of course, whether the JBC may resume its “Section 4. (1). The Supreme Court shall be composed of a Chief Justice and
process until the short list is prepared, in view of the provision of Section 4(1), fourteen Associate Justices. It may sit en banc or in its discretion, in division of
Article VIII, which unqualifiedly requires the President to appoint one from the three, five, or seven Members. Any vacancy shall be filled within ninety days from
short list to fill the vacancy in the Supreme Court (be it the Chief Justice or an the occurrence thereof.”
Associate Justice) within 90 days from the occurrence of the vacancy.
The ripeness of the controversy for judicial determination may not be doubted. _______________
The challenges to the authority of the JBC to open the process of nomination and
to continue the process until the submission of the list of nominees; the insistence 65 See Buckley v. Valeo, 424 U.S. 1, 113-118 (1976); Regional Rail
of some of the petitioners to compel the JBC through mandamus to submit the Reoganization Act Cases, 419 U.S. 102, 138-148 (1974).
short list to the incumbent President; the counter-insistence of the intervenors to
733
prohibit the JBC from submitting the short list to the incumbent President on the
ground that said list should be submitted instead to the next President; the strong VOL. 615, March 17, 2010 733
position that the incumbent President is already prohibited under Section 15, De Castro vs. Judicial and Bar Council (JBC)
Article VII from making any appointments, including those to the Judiciary, In the consolidated petitions, the petitioners, with the exception of Soriano,
starting on May 10, 2010 until June 30, 2010; and the contrary position that the Tolentino and Inting, submit that the incumbent President can appoint the
incumbent President is not so prohibited are only some of the real issues for successor of Chief Justice Puno upon his retirement on May 17, 2010, on the
determination. All such issues establish the ripeness of the controversy, ground that the prohibition against presidential appointments under Section 15,
considering that for some the short list must be submitted before the vacancy Article VII does not extend to appointments in the Judiciary.
actually occurs by May 17, 2010. The outcome will not be an abstraction, or a The Court agrees with the submission.
merely hypothetical exercise. The resolution of the controversy will surely settle— First. The records of the deliberations of the Constitutional Commission reveal
with finality—the nagging questions that are preventing the JBC from moving on that the framers devoted time to meticulously drafting, styling, and arranging the
with the process that it already began, or that are reasons persuading the JBC to Constitution. Such meticulousness indicates that the organization and
desist from the rest of the process.732 arrangement of the provisions of the Constitution were not arbitrarily or
732 SUPREME COURT REPORTS ANNOTATED whimsically done by the framers, but purposely made to reflect their intention and
manifest their vision of what the Constitution should contain.
De Castro vs. Judicial and Bar Council (JBC)
The Constitution consists of 18 Articles, three of which embody the allocation
of the awesome powers of government among the three great departments, the
We need not await the occurrence of the vacancy by May 17, 2010 in order for Legislative (Article VI), the Executive (Article VII), and the Judicial Departments
the principal issue to ripe for judicial determination by the Court. It is enough that (Article VIII). The arrangement was a true recognition of the principle of
one alleges conduct arguably affected with a constitutional interest, but seemingly separation of powers that underlies the political structure, as Constitutional
proscribed by the Constitution. A reasonable certainty of the occurrence of the Commissioner Adolfo S. Azcuna (later a worthy member of the Court) explained in
perceived threat to a constitutional interest is sufficient to afford a basis for his sponsorship speech:
bringing a challenge, provided the Court has sufficient facts before it to enable it “We have in the political part of this Constitution opted for the separation of
to intelligently adjudicate the issues.65 Herein, the facts are not in doubt, for only powers in government because we believe that the only way to protect freedom and
legal issues remain. liberty is to separate and divide the awesome powers of government. Hence, we
return to the separation of powers doctrine and the legislative, executive and
Substantive Merits judicial departments.”66

I
_______________ Relevantly, Valenzuela adverted to the intent of the framers in the genesis of
Section 4 (1), Article VIII, viz.:
66 Record of Proceedings and Debates of the Constitutional Commission, Vol.
V., p. 912, October 12, 1998. _______________
734
734 SUPREME COURT REPORTS ANNOTATED power of appointment, it is this Court’s view that, as a general proposition, in
case of conflict, the former should yield to the latter. Surely, the prevention of vote-
De Castro vs. Judicial and Bar Council (JBC) buying and similar evils outweighs the need for avoiding delays in filling up of
As can be seen, Article VII is devoted to the Executive Department, and, among court vacancies or the disposition of some cases. Temporary vacancies can abide
others, it lists the powers vested by the Constitution in the President. The the period of the ban which, incidentally and as earlier pointed out, comes to exist
presidential power of appointment is dealt with in Sections 14, 15 and 16 of the only once in every six years. Moreover, those occurring in the lower courts can be
Article. filled temporarily by designation. But prohibited appointments are long-lasting
Article VIII is dedicated to the Judicial Department and defines the duties and and permanent in their effects. They may, as earlier pointed out, in fact influence
qualifications of Members of the Supreme Court, among others. Section 4(1) and the results of elections and, for that reason, their making is considered an election
Section 9 of this Article are the provisions specifically providing for the offense.
appointment of Supreme Court Justices. In particular, Section 9 states that the To the contention that may perhaps be asserted, that Sections 4 (1) and 9 of
appointment of Supreme Court Justices can only be made by the President upon Article VIII should prevail over Section 15 of Article VII, because they may be
the submission of a list of at least three nominees by the JBC; Section 4(1) of the considered later expressions of the people when they adopted the Constitution, it
Article mandates the President to fill the vacancy within90 days from the suffices to point out that the Constitution must be construed in its entirety as one,
occurrence of the vacancy. single, instrument.
Had the framers intended to extend the prohibition contained in Section 15, To be sure, instances may be conceived of the imperative need for an
Article VII to the appointment of Members of the Supreme Court, they could have appointment, during the period of the ban, not only in the executive but also in the
explicitly done so. They could not have ignored the meticulous ordering of the Supreme Court. This may be the case should the membership of the court be so
provisions. They would have easily and surely written the prohibition made reduced that it will have no quorum or should the voting on a particularly
explicit in Section 15, Article VII as being equally applicable to the appointment of important question requiring expeditious resolution be evenly divided. Such a
Members of the Supreme Court in Article VIII itself, most likely in Section 4 (1), case, however, is covered by neither Section 15 of Article VII nor Section 4 (1) and
Article VIII. That such specification was not done only reveals that the prohibition 9 of Article VIII.
against the President or Acting President making appointments within two 736
months before the next presidential elections and up to the end of the President’s
or Acting President’s term does not refer to the Members of the Supreme Court. 736 SUPREME COURT REPORTS ANNOTATED
Although Valenzuela67 came to hold that the prohibition covered even judicial De Castro vs. Judicial and Bar Council (JBC)
appointments, it cannot be disputed that
V . Intent of the Constitutional Commission
_______________ The journal of the Commission which drew up the present Constitution
discloses that the original proposal was to have an eleven-member Supreme Court.
67 Supra, note 6, p. 426-427, stating: Commissioner Eulogio Lerum wanted to increase the number of Justices to fifteen.
Considering the respective reasons for the time frames for filling vacancies in He also wished to ensure that that number would not be reduced for any
the courts and the restriction on the President’s appreciable length of time (even only temporarily), and to this end proposed that
735 any vacancy “must be filled within two months from the date that the vacancy
occurs.” His proposal to have a 15-member Court was not initially adopted.
VOL. 615, March 17, 2010 735
Persisting however in his desire to make certain that the size of the Court would
De Castro vs. Judicial and Bar Council (JBC) not be decreased for any substantial period as a result of vacancies, Lerum
proposed the insertion in the provision (anent the Court’s membership) of the same
the Valenzuela dictum did not firmly rest on the deliberations of the Constitutional mandate that “IN CASE OF ANY VACANCY, THE SAME SHALL BE FILLED
Commission. Thereby, the confirmation made to the JBC by then Senior Associate WITHIN TWO MONTHS FROM OCCURRENCE THEREOF.” He later agreed to
Justice Florenz D. Regalado of this Court, a former member of the Constitutional suggestions to make the period three, instead of two, months. As thus amended,
Commission, about the prohibition not being intended to apply to the the proposal was approved. As it turned out, however, the Commission ultimately
appointments to the Judiciary, which confirmation Valenzuela even expressly agreed on a fifteen-member Court. Thus it was that the section fixing the
mentioned, should prevail.
composition of the Supreme Court came to include a command to fill up _______________
any vacancy therein within 90 days from its occurrence.
In this connection, it may be pointed out that that instruction that any 68 Id., at pp. 422-423.
“vacancy shall be filled within ninety days” (in the last sentence of Section 4 (1) of 69 Id., at p. 423.
Article VIII) contrasts with the prohibition in Section 15, Article VII, which is 70 Record of Proceedings and Debates of the Constitutional Commission, Vol.
couched in stronger negative language—that “a President or Acting President shall V., pp. 632-633.
not make appointments…” 738
The commission later approved a proposal of Commissioner Hilario G. Davide,
738 SUPREME COURT REPORTS ANNOTATED
Jr. (now a Member of this Court) to add to what is now Section 9 of Article VIII,
the following paragraph: “WITH RESPECT TO LOWER COURTS, THE De Castro vs. Judicial and Bar Council (JBC)
PRESIDENT SHALL ISSUE THE APPOINTMENT WITHIN NINETY DAYS may be enforced71—should not be disregarded. Thereby, Sections 4(1) imposes on
FROM THE SUBMISSION OF THE LIST” (of nominees by the Judicial and Bar the President the imperative duty to make an appointment of a Member of the
Council to the President). Davide stated that his purpose was to provide a “uniform Supreme Court within 90 days from the occurrence of the vacancy. The failure by
rule” for lower courts. According to him, the 90-day period should be counted from the President to do so will be a clear disobedience to the Constitution.
submission of the list of nominees to the President in view of the possibility that The 90-day limitation fixed in Section 4(1), Article VIII for the President to fill
the President might reject the list submitted to him and the JBC thus need more the vacancy in the Supreme Court was undoubtedly a special provision to establish
time to submit a new one.737 a definite mandate for the President as the appointing power, and cannot be
VOL. 615, March 17, 2010 737 defeated by mere judicial interpretation in Valenzuela to the effect that Section 15,
Article VII prevailed because it was “couched in stronger negative language.” Such
De Castro vs. Judicial and Bar Council (JBC) interpretation even turned out to be conjectural, in light of the records of the
Constitutional Commission’s deliberations on Section 4 (1), Article VIII.
On the other hand, Section 15, Article VII—which in effect deprives the How Valenzuela justified its pronouncement and result is hardly warranted.
President of his appointing power “two months immediately before the next According to an authority on statutory construction:72
presidential elections up to the end of his term”—was approved without “xxx the court should seek to avoid any conflict in the provisions of the statute
discussion.”68 by endeavoring to harmonize and reconcile every part so that each shall be
However, the reference to the records of the Constitutional Commission did not effective. It is not easy to draft a statute, or any other writing for that matter,
advance or support the result in Valenzuela. Far to the contrary, the records which may not in some manner contain conflicting provisions. But what appears
disclosed the express intent of the framers to enshrine in the Constitution, upon to the reader to be a conflict may not have seemed so to the drafter. Undoubtedly,
the initiative of Commissioner Eulogio Lerum, “a command [to the President] to each provision was inserted for a definite reason. Often by considering the
fill up any vacancy therein within 90 days from its occurrence,” which enactment in its entirety, what appears to be on its face a conflict may be cleared
even Valenzuela conceded.69 The exchanges during deliberations of the up and the provisions reconciled.
Constitutional Commission on October 8, 1986 further show that the filling of a Consequently, that construction which will leave every word operative will be
vacancy in the Supreme Court within the 90-day period was a true mandate for favored over one which leaves some word or provision meaningless because of
the President, viz.: inconsistency. But a word should not be given effect, if to do so gives the statute a
MR. DE CASTRO. I understand that our justices now in the Supreme Court, meaning contrary to the
together with the Chief Justice, are only 11.
MR. CONCEPCION. Yes. _______________
MR. DE CASTRO. And the second sentence of this subsection reads:
“Any vacancy shall be filled within ninety days from the 71 Dizon v. Encarnacion, G.R. No. L-18615, December 24, 1963, 9 SCRA 714.
occurrence thereof.” 72 Crawford, Earl. T., The Construction of Statutes, Thomas Law Book
MR. CONCEPCION. That is right. Company, St. Louis, Missouri, 262-264 (1940).
MR. DE CASTRO. Is this now a mandate to the executive to fill the 739
vacancy?
MR. CONCEPCION. That is right. That is borne out of the fact that in VOL. 615, March 17, 2010 739
the past 30 years, seldom has the Court had a complete De Castro vs. Judicial and Bar Council (JBC)
complement.70 intent of the legislature. On the other hand, if full effect cannot be given to the
Moreover, the usage in Section 4(1), Article VIII of the word shall—an words of a statute, they must be made effective as far as possible. Nor should the
imperative, operating to impose a duty that provisions of a statute which are inconsistent be harmonized at a sacrifice of the
legislative intention. It may be that two provisions are irreconcilable; if so, the one
which expresses the intent of the law-makers should control. And the arbitrary preceding a Presidential election and are similar to those which are declared
rule has been frequently announced that where there is an irreconcilable conflict election offenses in the Omnibus Election Code, viz.:
between the different provisions of a statute, the provision last in order of position xxx
will prevail, since it is the latest expression of the legislative will. Obviously, the The second type of appointments prohibited by Section 15, Article VII consists
rule is subject to deserved criticism. It is seldom applied, and probably then only of the so-called “midnight” appointments. In Aytona v. Castillo, it was held that
where an irreconcilable conflict exists between different sections of the same act, after the proclamation of Diosdado Macapagal as duly elected President, President
and after all other means of ascertaining the meaning of the legislature have been Carlos P. Garcia, who was defeated in his bid for reelection, became no more than
exhausted. Where the conflict is between two statutes, more may be said in favor a “caretaker” administrator whose duty was to “prepare for the orderly transfer of
of the rule’s application, largely because of the principle of implied repeal.” authority to the incoming President.” Said the Court:
In this connection, PHILCONSA’s urging of a revisit and a review “The filling up of vacancies in important positions, if few, and
of Valenzuela is timely and appropriate. Valenzuelaarbitrarily ignored the express so spaced as to afford some assurance of deliberate action and
intent of the Constitutional Commission to have Section 4 (1), Article VIII careful consideration of the need for the appointment and
stand independently of any other provision, least of all one found in Article VII. It appointee’s qualifications may
further ignored that the two provisions had no irreconcilable conflict, regardless of
Section 15, Article VII being couched in the negative. As judges, we are not to _______________
unduly interpret, and should not accept an interpretation that defeats the intent
of the framers.73 mands special justification.” The special justification for the reversal
Consequently, prohibiting the incumbent President from appointing a Chief of Valenzuela lies in its intrinsic unsoundness.
Justice on the premise that Section 15, Article VII extends to appointments in the
Judiciary cannot be sustained. A misinterpretation like Valenzuela should not be 75 No. L-19313, January 19, 1962, 4 SCRA 1.
allowed to last after its false premises have been exposed.74 741
VOL. 615, March 17, 2010 741
_______________
De Castro vs. Judicial and Bar Council (JBC)
73 Garcia v. Social Security Commission Legal and Collection, G.R. No. undoubtedly be permitted. But the issuance of 350 appointments in
170735, December 17, 2007, 540 SCRA 456, 472; citing Escosura v. San Miguel one night and the planned induction of almost all of them in a few
Brewery, Inc., 4 SCRA 285, (1962). hours before the inauguration of the new President may, with some
74 According to Arizona v. Rumsey, 467 U. S. 203, 212 (1984): “Although reason, be regarded by the latter as an abuse of Presidential
adherence to precedent is not rigidly required in constitutional cases, any prerogatives, the steps taken being apparently a mere partisan
departure from the doctrine of stare decisis de- effort to fill all vacant positions irrespective of fitness and other
740 conditions, and thereby to deprive the new administration of an
740 SUPREME COURT REPORTS ANNOTATED opportunity to make the corresponding appointments.”
As indicated, the Court recognized that there may well be appointments to
De Castro vs. Judicial and Bar Council (JBC) important positions which have to be made even after the proclamation of the new
It will not do to merely distinguish Valenzuela from these cases, for the result to President. Such appointments, so long as they are “few and so spaced as to
be reached herein is entirely incompatible with afford some assurance of deliberate action and careful consideration of
what Valenzuela decreed. Consequently, Valenzuela now deserves to be quickly the need for the appointment and the appointee’s qualifications,” can be
sent to the dustbin of the unworthy and forgettable. made by the outgoing President. Accordingly, several appointments made by
We reverse Valenzuela. President Garcia, which were shown to have been well considered, were upheld.
Second. Section 15, Article VII does not apply as well to Section 15, Article VII has a broader scope than the Aytona ruling. It
all other appointments in the Judiciary. may not unreasonably be deemed to contemplate not only “midnight”
There is no question that one of the reasons underlying the adoption of Section appointments—those made obviously for partisan reasons as shown by
15 as part of Article VII was to eliminate midnight appointments from being made their number and the time of their making—but also appointments
by an outgoing Chief Executive in the mold of the appointments dealt with in the presumed made for the purpose of influencing the outcome of the
leading case of Aytona v. Castillo.75 In fact, in Valenzuela, the Court so observed, Presidential election.
stating that: On the other hand, the exception in the same Section 15 of Article VII—
“xxx it appears that Section 15, Article VII is directed against two types of allowing appointments to be made during the period of the ban therein provided—
appointments: (1) those made for buying votes and (2) those made for partisan is much narrower than that recognized in Aytona. The exception allows only the
considerations. The first refers to those appointments made within the two months making of temporary appointments to executive positions when continued
vacancies will prejudice public service or endanger public safety. Obviously, the enactment should be construed with reference to its intended scope and purpose,
article greatly restricts the appointing power of the President during the period of and the court should seek to carry out this purpose rather than to defeat it.78
the ban. Also, the intervention of the JBC eliminates the danger that appointments to
Considering the respective reasons for the time frames for filling vacancies in the Judiciary can be made for the purpose of buying votes in a coming presidential
the courts and the restriction on the President’s power of appointment, it is this election, or of satisfying partisan considerations. The experience from the time of
Court’s view that, as a general proposition, in case of conflict, the former should the establishment of the JBC shows that even candidates for judicial positions at
yield to the latter. Surely, the prevention of vote-buying and similar evils any level backed by people influential with the President could not always be
outweighs the assured of being recommended for the consideration of the President, because they
742 first had to undergo the vetting of the JBC and pass muster there. Indeed, the
742 SUPREME COURT REPORTS ANNOTATED creation of the JBC was precisely intended to de-politicize the Judiciary by doing
away with the intervention of the Commission on Appointments. This insulating
De Castro vs. Judicial and Bar Council (JBC) process was absent from the Aytona midnight appointment.
need for avoiding delays in filling up of court vacancies or the disposition of some Third. As earlier stated, the non-applicability of Section 15, Article VII to
cases. Temporary vacancies can abide the period of the ban which, incidentally and appointments in the Judiciary was confirmed by then Senior Associate Justice
as earlier pointed out, comes to exist only once in every six years. Moreover, those Regalado to the JBC itself when it met on March 9, 1998 to discuss the question
occurring in the lower courts can be filled temporarily by designation. But raised by some sectors about the “constitutionality of xxx appointments” to the
prohibited appointments are long-lasting and permanent in their effects. They Court of Appeals in light of the forthcoming presidential elections. He assured that
may, as earlier pointed out, in fact influence the results of elections and, for that “on the basis of the (Constitutional) Commission’s records, the election ban had no
reason, their making is considered an election offense.”76 application to appointments to the Court of Appeals.”79 This confirmation
Given the background and rationale for the prohibition in Section 15, Article was accepted by the JBC, which then submitted to the President for consideration
VII, we have no doubt that the Constitutional Commission confined the prohibition the nominations for the eight vacancies in the Court of Appeals.80
to appointments made in the Executive Department. The framers did not need to
extend the prohibition to appointments in the Judiciary, because their
establishment of the JBC and their subjecting the nomination and screening of _______________
candidates for judicial positions to the unhurried and deliberate prior process of
the JBC ensured that there would no longer be midnight appointments to the 78 Crawford, op. cit., supra, note 72, pp. 248-249.
Judiciary. If midnight appointments in the mold of Aytona were made in haste and 79 Supra, note 6, p. 413.
with irregularities, or made by an outgoing Chief Executive in the last days of his 80 Id.
administration out of a desire to subvert the policies of the incoming President or
744
for partisanship,77 the appointments to the Judiciary made after the establishment
of the JBC would not be suffering from such defects because of the JBC’s prior 744 SUPREME COURT REPORTS ANNOTATED
processing of candidates. Indeed, it is axiomatic in statutory construction that the De Castro vs. Judicial and Bar Council (JBC)
ascertainment of the purpose of the enactment is a step in the process of The fault of Valenzuela was that it accorded no weight and due consideration
ascertaining the intent or to the confirmation of Justice Regalado. Valenzuelawas weak, because it relied on
interpretation to determine the intent of the framers rather than on the
_______________ deliberations of the Constitutional Commission. Much of the unfounded doubt
about the President’s power to appoint during the period of prohibition in Section
76 Supra, note 6, pp. 424-426; bold underscoring supplied for emphasis. 15, Article VII could have been dispelled since its promulgation on November 9,
77 Aytona v. Castillo, supra, note 74, pp. 8-10 (N.B. - In the time material 1998, had Valenzuela properly acknowledged and relied on the confirmation of a
to Aytona, there were judges of the Court of First Instance who were appointed to distinguished member of the Constitutional Commission like Justice Regalado.
districts that had no vacancies, because the incumbents had not qualified for other Fourth. Of the 23 sections in Article VII, three (i.e., Section 14, Section15, and
districts to which they had been supposedly transferred or promoted; at any rate, Section 16) concern the appointing powers of the President.
the appointments still required confirmation by the Commission on Section 14 speaks of the power of the succeeding President to revoke
Appointments). appointments made by an Acting President,81 and evidently refers only to
743 appointments in the Executive Department. It has no application to appointments
VOL. 615, March 17, 2010 743 in the Judiciary, because temporary or acting appointments can only undermine
the independence of the Judiciary due to their being revocable at will.82 The letter
De Castro vs. Judicial and Bar Council (JBC) and spirit of the Constitution safeguard that independence. Also, there is no law
meaning of the enactment, because the reason for the enactment must necessarily in the books that authorizes the revocation of appointments in the Judiciary. Prior
shed considerable light on “the law of the statute,” i.e., the intent; hence, the
to their mandatory retirement or resignation, judges of the first and second level 746 SUPREME COURT REPORTS ANNOTATED
courts and the Justices of the third level courts may only be removed for cause, but
the Members of the Supreme Court may be removed only by impeachment. De Castro vs. Judicial and Bar Council (JBC)
Section 16 covers only the presidential appointments that require confirmation litical leaders vying for the Presidency in a presidential election. Consequently,
by the Commission on Appointments. the wisdom of having the new President, instead of the current incumbent
President, appoint the next Chief Justice is itself suspect, and cannot ensure
_______________ judicial independence, because the appointee can also become beholden to the
appointing authority. In contrast, the appointment by the incumbent President
does not run the same risk of compromising judicial independence, precisely
81 Section 14. Appointments extended by an Acting President shall remain
because her term will end by June 30, 2010.
effective, unless revoked by the elected President within ninety days from his
assumption or reassumption of office. Sixth. The argument has been raised to the effect that there will be no need
82 Cruz, I., Philippine Political Law, 253 (2002); also Rilloraza v. Vargas, 80 for the incumbent President to appoint during the prohibition period the successor
of Chief Justice Puno within the context of Section 4 (1), Article VIII, because
Phil. 297 (1948).
anyway there will still be about 45 days of the 90 days mandated in Section 4(1),
745
Article VIII remaining.
VOL. 615, March 17, 2010 745 The argument is flawed, because it is focused only on the coming vacancy
De Castro vs. Judicial and Bar Council (JBC) occurring from Chief Justice Puno’s retirement by May 17, 2010. It ignores the
Thereby, the Constitutional Commission restored the requirement of need to apply Section 4(1) to every situation of a vacancy in the Supreme Court.
confirmation by the Commission on Appointments after the requirement was The argument also rests on the fallacious assumption that there will still be
removed from the 1973 Constitution. Yet, because of Section 9 of Article VIII, the time remaining in the 90-day period under Section 4(1), Article VIII. The fallacy is
restored requirement did not include appointments to the Judiciary.83 easily demonstrable, as the OSG has shown in its comment.
Section 14, Section 15, and Section 16 are obviously of the same character, in Section 4 (3), Article VII requires the regular elections to be held on the second
that they affect the power of the President to appoint. The fact that Section 14 and Monday of May, letting the elections fall on May 8, at the earliest, or May 14, at
Section 16 refer only to appointments within the Executive Department renders the latest. If the regular presidential elections are held on May 8, the period of the
conclusive that Section 15 also applies only to the Executive Department. This prohibition is 115 days. If such elections are held on May 14, the period of the
conclusion is consistent with the rule that every part of the statute must be prohibition is 109 days. Either period of the prohibition is longer than the full
interpreted with reference to the context, i.e. that every part must be considered mandatory 90-day period to fill the vacancy in the Supreme Court. The result is
together with the other parts, and kept subservient to the general intent of the that there are at least 19 occasions (i.e., the difference between the shortest
whole enactment.84 It is absurd to assume that the framers deliberately situated possible period of the ban of 109 days and the 90-day mandatory period for
Section 15 between Section 14 and Section 16, if they intended Section 15 to appointments) in which the outgoing President would be in no position to comply
cover all kinds of presidential appointments. If that was their intention in respect with the constitutional duty to fill up a vacancy in the Supreme Court. It is
of appointments to the Judiciary, the framers, if only to be clear, would have easily 747
and surely inserted a similar prohibition in Article VIII, most likely within Section VOL. 615, March 17, 2010 747
4 (1) thereof.
De Castro vs. Judicial and Bar Council (JBC)
Fifth. To hold like the Court did in Valenzuela that Section 15 extends to
safe to assume that the framers of the Constitution could not have intended such
appointments to the Judiciary further undermines the intent of the Constitution
an absurdity. In fact, in their deliberations on the mandatory period for the
of ensuring the independence of the Judicial Department from the Executive and
appointment of Supreme Court Justices under Section 4 (1), Article VIII, the
Legislative Departments. Such a holding will tie the Judiciary and the Supreme
framers neither discussed, nor mentioned, nor referred to the ban against
Court to the fortunes or misfortunes of po-
midnight appointments under Section 15, Article VII, or its effects on the 90-day
period, or vice versa. They did not need to, because they never intended Section 15,
_______________
Article VII to apply to a vacancy in the Supreme Court, or in any of the lower
courts.
83 Record of Proceedings and Debates of the Constitutional Commission, Vol. Seventh. As a matter of fact, in an extreme case, we can even raise a doubt on
V., p. 908, which indicates that in his sponsorship speech delivered on October 12, whether a JBC list is necessary at all for the President—any President—to appoint
1986 on the floor of the Constitutional Commission, Commissioner Teofisto a Chief Justice if the appointee is to come from the ranks of the sitting justices of
Guingona explained that “[a]ppointments to the judiciary shall not be subject to the Supreme Court.
confirmation by the Commission on Appointments.” Sec. 9, Article VIII says:
84 Rodriguez, Statutory Construction, 171 (1999).
746
“xxx. The Members of the Supreme Court xxx shall be appointed by the De Castro vs. Judicial and Bar Council (JBC)
President from a list of at least three nominees prepared by the Judicial and Bar JBC for every vacancy, which appointments require no confirmation by the
Council for any vacancy. Such appointments need no confirmation. Commission on Appointments. With reference to the Chief Justice, he or she is
xxx” appointed by the President as Chief Justice, and the appointment is never in an
The provision clearly refers to an appointee coming into the Supreme Court acting capacity. The express reference to a Chief Justice abhors the idea that the
from the outside, that is, a non-member of the Court aspiring to become one. It framers contemplated an Acting Chief Justice to head the membership of the
speaks of candidates for the Supreme Court, not of those who are already members Supreme Court. Otherwise, they would have simply written so in the Constitution.
or sitting justices of the Court, all of whom have previously been vetted by the JBC. Consequently, to rely on Section 12 of the Judiciary Act of 1948 in order to forestall
Can the President, therefore, appoint any of the incumbent Justices of the the imperative need to appoint the next Chief Justice soonest is to defy the plain
Court as Chief Justice? intent of the Constitution.
The question is not squarely before us at the moment, but it should lend itself For sure, the framers intended the position of Chief Justice to be permanent,
to a deeper analysis if and when circumstances permit. It should be a good issue not one to be occupied in an acting or temporary capacity. In relation to the scheme
for the proposed Constitutional Convention to consider in the light of Senate of things under the present Constitution, Section 12 of the Judiciary Act of 1948
President Juan Ponce Enrile’s statement that the President only responds to a rare situation in which the new Chief Justice is not yet
748 appointed, or in which the incumbent Chief Justice is unable to perform the duties
748 SUPREME COURT REPORTS ANNOTATED and powers of the office. It ought to be remembered, however, that it was enacted
De Castro vs. Judicial and Bar Council (JBC) because the Chief Justice appointed under the 1935 Constitution was subject to
can appoint the Chief Justice from among the sitting justices of the Court even the confirmation of the Commission on Appointments, and the confirmation
without a JBC list. process might take longer than expected.
The appointment of the next Chief Justice by the incumbent President is
preferable to having the Associate Justice who is first in precedence take over.
II
Under the Constitution, the heads of the Legislative and Executive Departments
are popularly elected, and whoever are elected and proclaimed at once become the
The Judiciary Act of 1948 leaders of their respective Departments. However, the lack of any appointed
The posture has been taken that no urgency exists for the President to appoint occupant of the office of Chief Justice harms the independence of the Judiciary,
the successor of Chief Justice Puno, considering that the Judiciary Act of 1948 can because the Chief Justice is the head of the entire Judiciary. The Chief Justice
still address the situation of having the next President appoint the successor. performs functions absolutely significant to the life of the nation. With the entire
Section 12 of the Judiciary Act of 1948 states: Supreme Court being the Presidential Electoral Tribunal, the Chief Justice is the
“Section 12. Vacancy in Office of Chief Justice.—In case of a vacancy in the Chairman of the Tribunal. There being no obstacle to the
office of Chief Justice of the Supreme Court or of his inability to perform the duties 750
and powers of his office, they shall devolve upon the Associate Justice who is first
in precedence, until such disability is removed, or another Chief Justice is 750 SUPREME COURT REPORTS ANNOTATED
appointed and duly qualified. This provision shall apply to every Associate Justice De Castro vs. Judicial and Bar Council (JBC)
who succeeds to the office of Chief Justice.” appointment of the next Chief Justice, aside from its being mandatory for the
The provision calls for an Acting Chief Justice in the event of a vacancy in the incumbent President to make within the 90-day period from May 17, 2010, there
office of the Chief Justice, or in the event that the Chief Justice is unable to perform is no justification to insist that the successor of Chief Justice Puno be appointed
his duties and powers. In either of such circumstances, the duties and powers of by the next President.
the office of the Chief Justice shall devolve upon the Associate Justice who is first Historically, under the present Constitution, there has been no wide gap
in precedence until a new Chief Justice is appointed or until the disability is between the retirement and the resignation of an incumbent Chief Justice, on one
removed. hand, and the appointment to and assumption of office of his successor, on the
Notwithstanding that there is no pressing need to dwell on this peripheral other hand. As summarized in the comment of the OSG, the chronology of
matter after the Court has hereby resolved the question of consequence, we do not succession is as follows:
find it amiss to confront the matter now. 1. When Chief Justice Claudio Teehankee retired on April 18, 1988, Chief
We cannot agree with the posture. Justice Pedro Yap was appointed on the same day;
A review of Sections 4(1) and 9 of Article VIII shows that the Supreme Court 2. When Chief Justice Yap retired on July 1, 1988, Chief Justice Marcelo
is composed of a Chief Justice and 14 Associate Justices, who all shall be appointed Fernan was appointed on the same day;
by the President from a list of at least three nominees prepared by the 3. When Chief Justice Fernan resigned on December 7, 1991, Chief Justice
749 Andres Narvasa was appointed the following day, December 8, 1991;
VOL. 615, March 17, 2010 749
4. When Chief Justice Narvasa retired on November 29, 1998, Chief Justice 86 Section 3, Rule 65, 1997 Rules of Civil Procedure.
Hilario Davide, Jr. was sworn into office the following early morning of November 87 JG Summit Holdings, Inc. v. Court of Appeals, G.R. No. 124293, November
30, 1998; 20, 2000, 345 SCRA 143.
5. When Chief Justice Davide retired on December 19, 2005, Chief Justice 752
Artemio Panganiban was appointed the next day, December 20, 2005; and 752 SUPREME COURT REPORTS ANNOTATED
6. When Chief Justice Panganiban retired on December 6, 2006, Chief Justice
Reynato S. Puno took his oath as Chief Justice at midnight of December 6, 2006.85 De Castro vs. Judicial and Bar Council (JBC)
days from the submission of the list, in the case of the lower courts. The 90-day
III period is directed at the President, not at the JBC. Thus, the JBC should start the
process of selecting the candidates to fill the vacancy in the Supreme
Writ of mandamus does not lie against the JBC Court before the occurrence of the vacancy.
May the JBC be compelled to submit the list of nominees to the President? Under the Constitution, it is mandatory for the JBC to submit to the President
the list of nominees to fill a vacancy in the Supreme Court in order to enable the
President to appoint one of them withinthe 90-day period from the occurrence of
_______________
the vacancy. The JBC has no discretion to submit the list to the President after the
vacancy occurs, because that shortens the 90-day period allowed by the
85 Comment of the OSG, p. 37. Constitution for the President to make the appointment. For the JBC to do so will
751 be unconscionable on its part, considering that it will
VOL. 615, March 17, 2010 751 thereby effectively and illegally deprive the President of the ample time granted
De Castro vs. Judicial and Bar Council (JBC) under the Constitution to reflect on the qualifications of the nominees named in
the list of the JBC before making the appointment.
The duty of the JBC to submit a list of nominees before the start of the
Mandamus shall issue when any tribunal, corporation, board, officer or person
President’s mandatory 90-day period to appoint is ministerial, but its selection of
unlawfully neglects the performance of an act that the law specifically enjoins as
the candidates whose names will be in the list to be submitted to the President lies
a duty resulting from an office, trust, or station.86 It is proper when the act against
within the discretion of the JBC. The object of the petitions for mandamus herein
which it is directed is one addressed to the discretion of the tribunal or
should only refer to the duty to submit to the President the list of nominees for
officer. Mandamusis not available to direct the exercise of a judgment or discretion
every vacancy in the Judiciary, because in order to constitute unlawful neglect of
in a particular way.87
duty, there must be an unjustified delay in performing that
For mandamus to lie, the following requisites must be complied with: (a) the
duty.88 For mandamus to lie against the JBC, therefore, there should be an
plaintiff has a clear legal right to the act demanded; (b) it must be the duty of the
unexplained delay on its part in recommending nominees to the Judiciary, that is,
defendant to perform the act, because it is mandated by law; (c) the defendant
in submitting the list to the President.
unlawfully neglects the performance of the duty enjoined by law; (d) the act to be
The distinction between a ministerial act and a discretionary one has been
performed is ministerial, not discretionary; and (e) there is no appeal or any other
delineated in the following manner:
plain, speedy and adequate remedy in the ordinary course of law.
Section 8(5) and Section 9, Article VIII, mandate the JBC to submit a list of at
least three nominees to the President for every vacancy in the Judiciary: _______________
“Section 8. xxx
(5) The Council shall have the principal function of recommending 88 Nery v. Gamolo, A.M. No. P-01-1508, February 7, 2003, 397 SCRA 110,
appointees to the Judiciary. xxx citing Musni v. Morales, 315 SCRA 85, 86 (1999).
Section 9. The Members of the Supreme Court and judges of lower courts 753
shall be appointed by the President from a list of at least three nominees VOL. 615, March 17, 2010 753
prepared by the Judicial and Bar Council for every vacancy.Such
De Castro vs. Judicial and Bar Council (JBC)
appointments need no confirmation.
For the lower courts, the President shall issue the appointments within
ninety days from the submission of the list.” “The distinction between a ministerial and discretionary act is well delineated.
However, Section 4(1) and Section 9, Article VIII, mandate the President to fill A purely ministerial act or duty is one which an officer or tribunal
the vacancy in the Supreme Court within 90 days from the occurrence of the performs in a given state of facts, in a prescribed manner, in obedience to
vacancy, and within 90 the mandate of a legal authority, without regard to or the exercise of his
own judgment upon the propriety or impropriety of the act done. If the
law imposes a duty upon a public officer and gives him the right to decide
_______________
how or when the duty shall be performed, such duty is discretionary and
not ministerial. The duty is ministerial only when the discharge of the same (d) To continue its proceedings for the nomination of candidates to fill
requires neither the exercise of official discretion or judgment.”89 other vacancies in the Judiciary and submit to the President the short list
Accordingly, we find no sufficient grounds to grant the petitions of nominees corresponding thereto in accordance with this decision.
for mandamus and to issue a writ of mandamus against the JBC. The actions for SO ORDERED.
that purpose are premature, because it is clear that the JBC still has until May Leonardo-De Castro and Perez, JJ., concur.
17, 2010, at the latest, within which to submit the list of nominees to the President Puno (C.J.), No part. JBC or respondent.
to fill the vacancy created by the compulsory retirement of Chief Justice Puno. Carpio, J., No Part, as Senior Associate Justice, I am involved either
way.755
IV VOL. 615, March 17, 2010 755
De Castro vs. Judicial and Bar Council (JBC)
Writ of prohibition does not lie against the JBC
In light of the foregoing disquisitions, the conclusion is ineluctable that only
the President can appoint the Chief Justice. Hence, Soriano’s petition for Corona, J., No part.
prohibition in G.R. No. 191032, which proposes to prevent the JBC from Carpio-Morales, J., Please see Dissenting Opinion.
intervening in the process of nominating the successor of Chief Justice Puno, lacks Velasco, Jr., I join the separate opinion of Justice Nachura.
merit. Nachura, J., Please see Separate Opinion.
On the other hand, the petition for prohibition in G.R. No. 191342 is similarly Brion, J., See my Separate Opinion.
devoid of merit. The challenge mounted against the composition of the JBC based Peralta, J., In the result. I join the opinion of Justice Brion.
on the allegedly unconstitutional allocation of a vote each to the ex officio members Del Castillo, J., In the result. I share the view of Justice Brion.
from the Senate and the House of Representatives, thereby prejudicing the Abad, J., Please see my Concurrence.
chances of some candidates for nomi- Villarama, Jr., J., I certify that Justice Villarama voted in favor of the
Decision of Justice Bersamin—Puno, C.J.
Mendoza, J., In the result. I join Justice Brion in his Separate Opinion.
_______________
DISSENTING OPINION
CARPIO-MORALES, J.:
89 Espiridion v. Court of Appeals, G.R. No. 146933, June 8, 2006, 490 SCRA “Although the Chief Justice is primus inter pares, he cannot legally decide a
273. case on his own because of the Court’s nature as a collegial body. Neither can the
754 Chief Justice, by himself, overturn the decision of the Court, whether of a division
754 SUPREME COURT REPORTS ANNOTATED or the en banc.”
De Castro vs. Judicial and Bar Council (JBC) —Associate Justice Renato C. Corona in
Complaint of Mr. Aurelio Indencia Arrienda
nation by raising the minimum number of votes required in accordance with the
against Justice Puno, 499 Phil. 1, 14; 460
rules of the JBC, is not based on the petitioners’ actual interest, because they have
SCRA 1, 15 (2005)
not alleged in their petition that they were nominated to the JBC to fill some
Primus inter pares. First among equals. The Latin maxim indicates that a
vacancies in the Judiciary. Thus, the petitioners lack locus standi on that issue.
person is the most senior of a group of people sharing the same rank or office. The
WHEREFORE, the Court:
phrase has been used to describe the status, condition or role of the prime minister
1. Dismisses the petitions for certiorari and mandamus in G.R. No. 191002
in most parliamentary nations, the high-ranking prelate in sev-
and G.R. No. 191149, and the petition for mandamus in G.R. No. 191057 for being
premature; 756
2. Dismisses the petitions for prohibition in G.R. No. 191032 and G.R. No. 756 SUPREME COURT REPORTS ANNOTATED
191342 for lack of merit; and De Castro vs. Judicial and Bar Council (JBC)
3. Grants the petition in A.M. No. 10-2-5-SC and, accordingly, directs the eral religious orders, and the chief justice in many supreme courts around the
Judicial and Bar Council: world.1
(a) To resume its proceedings for the nomination of candidates to fill The inclination to focus on the inter pares without due emphasis on
the vacancy to be created by the compulsory retirement of Chief Justice the primus/prima2 has spawned contemporary discourse that revives the original
Reynato S. Puno by May 17, 2010; tug-of-war between domination and parity, which impasse the conceived maxim
(b) To prepare the short list of nominees for the position of Chief precisely intended to resolve.
Justice; In the present case, several arguments attempt to depict a mirage of doomsday
(c) To submit to the incumbent President the short list of nominees for scenarios arising from the impending vacancy of the primus in the Court as a
the position of Chief Justice on or before May 17, 2010; and springboard for their plea to avert a supposed undermining of the independence of
the judiciary. In reality, the essential question boils down to the limitation and works against a rigid pigeonholing of executive tasks among the members of
on the appointing power of the President. the President’s official family. Notably, the Constitution inhibited from identifying
The ponencia of Justice Bersamin holds that the incumbent President can and compartmentalizing the composition of the Cabinet. In vesting ex-
appoint the next Chief Justice upon the retirement of Chief Justice Reynato S.
Puno on May 17, 2010 since the prohibition during election period3 does not extend _______________
to appointments in the judiciary, thereby reversing In re appointments of Hon.
Valenzuela & Hon. Vallarta.4 5 G.R. No. 166052, August 29, 2007, 531 SCRA 583, where the petitioner
The ponencia additionally holds that the Judicial and Bar Council (JBC) has assailed the placing of the National Commission on Indigenous Peoples as an
until May 17, 2010, at the latest, within which to submit to the President the list attached agency of the Department of Agrarian Reform on the ground that, inter
of nominees for the position of Chief Justice. alia, policy and program coordination between allegedly conceptually different
I DISSENT. government agencies is unconstitutional.
758
_______________
758 SUPREME COURT REPORTS ANNOTATED

1 Vide http://en.wikipedia.org/wiki/Primus_inter_pares (visited: March 10, De Castro vs. Judicial and Bar Council (JBC)
2010). ecutive power in one person rather than in a plural executive, the evident intention
2 Feminine ablative of primus (first among her equals). was to invest the power holder with energy.
3 Constitution, Art. VII, Sec. 15. Two months immediately before the next AMIN takes premium on the severed treatment of these reform areas in
presidential elections and up to the end of his term, a President or Acting marked provisions of the Constitution. It is a precept, however, that
President shall not make appointments, except temporary appointments to inferences drawn from title, chapter or section headings are entitled to
executive positions when continued vacancies therein will prejudice public service very little weight. And so must reliance on sub-headings, or the lack
or endanger public safety. (emphasis, italics and underscoring supplied) thereof, to support a strained deduction be given the weight of helium.
4 358 Phil. 896; 298 SCRA 408 (1998). Secondary aids may be consulted to remove, not to create doubt. AMIN’s thesis
757 unsettles, more than settles the order of things in construing the Constitution. Its
interpretation fails to clearly establish that the so-called “ordering” or
VOL. 615, March 17, 2010 757
arrangement of provisions in the Constitution was consciously adopted
De Castro vs. Judicial and Bar Council (JBC) to imply a signification in terms of government hierarchy from where a
constitutional mandate can per se be derived or asserted. It fails to
Constitutional draftsmanship style is demonstrate that the “ordering” or layout was not simply a matter of style
the weakest aid in arriving at a con- in constitutional drafting but one of intention in government
stitutional construction structuring. With its inherent ambiguity, the proposed interpretation cannot be
The first ratiocination adverts to the “organization and arrangement of the made a basis for declaring a law or governmental act unconstitutional.” 6 (emphasis
provisions of the Constitution” that was, as the ponencia declares, purposely made and underscoring supplied)
by the framers of the Constitution to “reflect their intention and manifest their Concededly, the allocation of three Articles in the Constitution devoted to the
vision” of the charter’s contents. respective dynamics of the three Departments was deliberately adopted by the
It is unfortunate that the ponencia chiefly relies on the trivialities of framers to allocate the vast powers of government among the three Departments
draftsmanship style in arriving at a constitutional construction. The petitioner in recognition of the principle of separation of powers.
in Anak Mindanao Party-List Group v. The Executive Secretary5 raised a similar The equation, however, does not end there. Such kind of formulation detaches
argument, but the Court held: itself from the concomitant system of checks and balances. Section sequencing
“AMIN goes on to proffer the concept of “ordering the law” which, so it alleges, alone of Sections 14, 15 and 16 of Article VII, as explained in the fourth
can be said of the Constitution’s distinct treatment of these three areas, as ratiocination, does not suffice to signify functional structuring.
reflected in separate provisions in different parts of the Constitution. It argues That the power of judicial appointment was lodged in the President is a
that the Constitution did not intend an over-arching concept of agrarian reform to recognized measure of limitation on the power of the judiciary, which measure,
encompass the two other areas, and that how the law is ordered in a certain however, is counterbalanced
way should not be undermined by mere executive orders in the guise of
administrative efficiency. _______________
The Court is not persuaded.
The interplay of various areas of reform in the promotion of social justice is not 6 Id., at pp. 601-603.
something implausible or unlikely. Their interlocking nature cuts across labels 759
VOL. 615, March 17, 2010 759 President is six years and under what we had voted on, there is no
reelection for him. Yet he can continue to rule the country through
De Castro vs. Judicial and Bar Council (JBC) appointments made about the end of his term to these sensitive positions.
by the election ban due to the need to insulate the judiciary from the political FR. BERNAS: At any rate, there are other checks as far as the appointment of
climate of presidential elections. To abandon this interplay of checks and balances those officers is concerned.
on the mere inference that the establishment of the JBC could de-politicize the MR. DAVIDE: Only insofar as the Commission on Appointments is concerned
process of judicial appointments lacks constitutional mooring. for offices which would require consent, and the Judicial Bar Council
The establishment of the JBC is not insofar as the judiciary is concerned.
sufficient to curtail the evils of mid- FR. BERNAS: We leave the matter to the body for a vote.8 (capitalization and
night appointments in the judiciary emphasis supplied)
The constitutional prohibition in Section 15 found its roots in the case The clear intent of the framers is thus for the ban on midnight appointments
of Aytona v. Castillo,7 where among the “midnight” or “last minute” to apply to the judiciary. The succeeding interpellations9 suggest no departure
appointments voided to abort the abuse of presidential prerogatives or partisan from this intent.
efforts to fill vacant positions were one in the Supreme Court and two in the Court
of Appeals. _______________
Heeding Aytona’s admonition, the Constitutional Commission (ConCom) saw
it fit to provide for a comprehensive ban on midnight appointments, finding that 8 RECORD OF THE 1986 CONSTITUTIONAL COMMISSION, Vol. 2, July 31, 1986, RCC
the establishment of the JBC is not enough to safeguard or insulate judicial No. 44 (CD Format).
appointments from politicization. The ConCom deliberations reveal:
9 Id. Following were the deliberations concerning the prohibition
MR. GUINGONA: Madam President.
on nepotism,wherein the deletion of the word “judiciary” was reflected in the final
THE PRESIDENT: Commissioner Guingona is recognized.
text of Section 13, Article VII of the Constitution:
MR. GUINGONA: Would the distinguished proponent accept an amendment
761
to his amendment to limit this prohibition to members of collegiate courts?
The judges of the lower courts perhaps would not have the same category VOL. 615, March 17, 2010 761
or the same standing as the others mentioned here. De Castro vs. Judicial and Bar Council (JBC)
MR. DAVIDE: Pursuant to the post amendment, we already included here For almost half a century, the seeds of Aytona, as nurtured and broadened by
government-owned or controlled corporations or their subsidiaries which the Constitution, have grown into an estab-
are not even very sensitive positions. So with more reason that the
prohibition should apply to appointments in these bodies. _______________

_______________ MR. TINGSON: Madam President, may I just ask one


question of the proponent?
7 No. L-19313, January 19, 1962, 4 SCRA 1, 8. THE PRESIDENT: Commissioner Tingson is recognized.
760 MR. TINGSON: Even though the members of the
760 SUPREME COURT REPORTS ANNOTATED President’s family are related to him,
shall we bar the men of probity,
De Castro vs. Judicial and Bar Council (JBC)
honesty and specialized technical
THE PRESIDENT: Does the Committee accept?
knowledge from being appointed?
FR. BERNAS: What is common among these people—Ministers, Deputy
MR. DAVIDE: That is precisely the core or the meat
Ministers, heads of bureaus or offices—is that they are under the control of
and the heart of the prohibition. In
the President.
effect, it is just extending it to these
MR. GUINGONA: That is correct.
sensitive positions that I have
FR. BERNAS: Whereas, the other offices the Commissioner mentioned are mentioned.
independent offices. MR. TINGSON: But in a sense would that not be
MR. DAVIDE: The idea of the proposal is that about the end of the term of the
counterproductive?
President, he may prolong his rule indirectly by appointing people to these
MR. DAVIDE: If that is the thinking of the
sensitive positions, like the commissions, the Ombudsman, the
Commissioner, he should rather
JUDICIARY, so he could perpetuate himself in power even beyond his term
propose for the deletion of the entire
of office; therefore foreclosing the right of his successor to make sentence since that is really its effect.
appointments to these positions. We should realize that the term of the
MR. TINGSON: Will the Commissioner join me if I do? JUDGES OF THE METROPOLITAN
MR. DAVIDE:No. As a matter of fact, I am expanding TRIAL COURTS.
the prohibition. But if the MR. DAVIDE: To avoid any further complication, I
Commissioner’s position is that we would agree to delete “JUDICIARY.”
might be prohibiting these capable MR. GUINGONA: Thank you.
men who are relatives of the President, MR. DAVIDE: So, on line 5, the only amendment
then the deletion would be proper, would consist of the following: after
which I am not in favor of. the word “as,” insert MEMBERS OF
MR. TINGSON: Madam President, we have already THE CONSTITUTIONAL
limited the presidency to one term, COMMISSIONS OR THE OFFICE
predicated on the fact that he will now OF THE OMBUDSMAN.
become a statesman rather than a THE PRESIDENT: Does the Committee prefer to throw
partisan politician. Then he will be this to the body?
acting for the good of our country; that MR. REGALADO: We prefer that we submit it to the
is, we base that philosophy with that body.
predicate. So I am just wondering why VOTING
we should not utilize these men who, 763
according to Commis- VOL. 615, March 17, 2010 763
762
De Castro vs. Judicial and Bar Council (JBC)
762 SUPREME COURT REPORTS ANNOTATED
De Castro vs. Judicial and Bar Council (JBC) The second ratiocination in the ponencia could thus not remove an added
lished doctrine that has weathered legal storms like Valenzuela. constitutional safeguard by pretending to have examined and concluded that the
establishment of the JBC had eliminated all encompassing forms of political
_______________ maneuverings during elections. Otherwise, reading into the Constitution such
conclusion so crucial to the scheme of checks and balances, which is neither written
sioner Uka, happen to have nor tackled, undermines the noticeable silence or restraint exercised by the
committed a crime of being related to framers themselves from making a definitive analysis.
the President. To illustrate, the instance given in the fifth ratiocination that having the
MR. DAVIDE: Is the Commissioner proposing that new President appoint the next Chief Justice cannot ensure judicial independence
as an amendment to my amendment? because the appointee can also become beholden to the appointing authority bears
MR. TINGSON: I would like to. an inconsistent stance. It does not admit or recognize that the mechanism of
MR. DAVIDE: In the sense that the Commissioner’s removal by impeachment eliminates the evils of political indebtedness. In any
amendment is to delete the entire event, that level of reasoning overlooks the risk of compromising judicial
sentence? independence when the outgoing President faces the Court in the charges that may
MR. TINGSON: Is that the Commissioner’s thinking be subsequently filed against her/him, and when the
also?
MR. DAVIDE: No, I am entirely for the opposite. _______________
MR. TINGSON: Then, I am not insisting anymore.
MR. DAVIDE: If the Commissioner is introducing it THE PRESIDENT: Those in favor of this proposed
as an amendment, I am sorry, I have to amendment of Commissioner Davide
reject his proposal. on page 9, line 5, to include these two
THE PRESIDENT: So, let us now proceed to the offices: the constitutional
amendment of Commissioner Davide. commissions and the office of the
MR. GUINGONA: Madam President, may I just offer one Ombudsman, please raise their hand.
more amendment to the distinguished (Several Members raised their hand.)
proponent? After the word Those against the proposed amendment will please raise their hand. (Few
“JUDICIARY,” we insert: EXCEPT Members raised their hand.)
The results show 24 votes in favor and 9 against; the amendment is approved.
MR. ROMULO: Madam President, we are almost at the end of our long The proclivity to innovate legal concepts is enticing. Lest the basic rule be
journey. I ask for continued patience on the part of everyone. We are now on forgotten, it helps to once more recite that when the law is clear, it is not
Section 20. We have consolidated all the amendments for presentation by one susceptible to interpretation and must be applied regardless of who may be
person; and that is, Commissioner Sarmiento. Will the Chair recognize him please? affected, even if the law may be harsh or onerous.12
(emphasis, italics and underscoring supplied). In its third ratiocination, the ponencia faults Valenzuela for not according
764 weight and due consideration to the opinion of Justice Florenz Regalado. It accords
764 SUPREME COURT REPORTS ANNOTATED high regard to the opinion expressed by Justice Regalado as a former ConCom
Member, to the exception of the opinion of all others similarly situated.
De Castro vs. Judicial and Bar Council (JBC) It bears noting that the Court had spoken in one voice in Valenzuela.
appointing President is up for re-election in the peculiar situation The ponencia should not hastily reverse, on the sole basis of Justice Regalado’s
contemplated by Section 4, Article VII of the Constitution. opinion, the Court’s unani-
All rules of statutory construction re-
volt against the interpretation arrived _______________
at by the ponencia
It is simplistic and unreliable for the ponencia to contend that had the framers 10 The Iloilo City Zoning Board of Adjustment & Appeals v. Gegato-Abecia
intended to extend the ban in Article VII to appointments in the judiciary, they Funeral Homes, Inc., 462 Phil. 803, 815; 417 SCRA 337, 345 (2003).
would have easily and surely written so in Article VIII, for it backlashes the 11 Samson v. Court of Appeals, No. L-43182, November 25, 1986, 145 SCRA
question that had the framers intended to exclude judicial appointments in Article 654, 659.
VIII from the prohibition in Article VII, they would have easily and surely written 12 Pascual v. Pascual-Bautista, G.R. No. 84240, March 25, 1992, 207 SCRA
so in the excepting proviso in Article VII. 561, 568.
Taking into account how the framers painstakingly rummaged through
766
various sections of the Constitution and came up with only one exception with the
need to specify the executive department, it insults the collective intelligence and 766 SUPREME COURT REPORTS ANNOTATED
diligence of the ConCom to postulate that it intended to exclude the judiciary but De Castro vs. Judicial and Bar Council (JBC)
missed out on that one. mous en banc decision penned by Chief Justice Andres Narvasa, and concurred in
To hold that the ban on midnight appointments applies only to executive by, inter alia, Associate Justices who later became Chief Justices—Hilario Davide,
positions, and not to vacancies in the judiciary and independent constitutional Jr., Artemio Panganiban and Reynato Puno.
bodies, is to make the prohibition practically useless. It bears noting that Section The line of reasoning is specious. If that is the case and for accuracy’s sake, we
15, Article VII of the Constitution already allows the President, by way of might as well reconvene all ConCom members and put the matter to a vote among
exception, to make temporary appointments in the Executive Department during them.
the prohibited period. Under this view, there is virtually no restriction on the Providentially, jurisprudence is replete with guiding principles to
President’s power of appointment during the prohibited period. ascertain the true meaning of the Constitution when the provisions as
The general rule is clear since the prohibition applies to ALL kinds of midnight written appear unclear and the proceedings as recorded provide little
appointments. The Constitution made no distinction. Ubi lex non distinguit nec nos help:
distinguere debemos.765 “While it is permissible in this jurisdiction to consult the debates and
VOL. 615, March 17, 2010 765 proceedings of the constitutional convention in order to arrive at the reason and
purpose of the resulting Constitution, resort thereto may be had only when other
De Castro vs. Judicial and Bar Council (JBC)
guides fail as said proceedings are powerless to vary the terms of the Constitution
when the meaning is clear. Debates in the constitutional convention “are of value
The exception is likewise clear. Expressio unius et exclusio alterius. The express as showing the views of the individual members, and as indicating the reasons for
mention of one person, thing or consequence implies the exclusion of all their votes, but they give us no light as to the views of the large majority who did
others.10 There is no clear circumstance that would indicate that the enumeration not talk, much less of the mass of our fellow citizens whose votes at the polls gave
in the exception was not intended to be exclusive. Moreover, the fact that Section that instrument the force of fundamental law. We think it safer to construe the
15 was couched in negative language reinforces the exclusivity of the exception. constitution from what appears upon its face.” The proper interpretation therefore
“Under the rules of statutory construction, exceptions, as a general rule, should depends more on how it was understood by the people adopting it than in the
be strictly but reasonably construed; they extend only so far as their language framers’ understanding thereof.”13 (underscoring supplied)
fairly warrants, and all doubts should be resolved in favor of the general provisions The clear import of Section 15 of Article VII is readily apparent. The people
rather than the exception. Where a general rule is established by statute with may not be of the same caliber as Justice Regalado, but they simply could not read
exceptions, the court will not curtail the former nor add to the latter by
implication.”11 (italics in the original; underscoring supplied)
into Section 15 something that is not there. Casus omissus pro omisso habendus dent to do so will be a clear disobedience to the Constitution.”15
est. The ponencia quotes certain records of the ConCom deliberations which,
however, only support the view that the number of Justices should “not be reduced
_______________ for any appreciable length of time” and it is a “mandate to the executive to fill the
vacancy”. Notably, there is no citation of any debate on how the framers reckoned
13 Francisco, Jr. v. The House of Representatives, 460 Phil. 830, 887; 415 SCRA or determined an appreciable length of time of 90 days, in which case a delay of
44, 128-129 (2003), citing Civil Liberties Union v. Executive Secretary, G.R. No. one day could already bring about the evils it purports to avoid and spell a culpable
83896, February 22, 1991, 194 SCRA 317, 337-338. violation of the Constitution. On the contrary, that the addition of one month to
767 the original proposal of 60 days was approved without controversy16 ineluctably
shows that the intent was not to strictly impose an inflexible timeframe.
VOL. 615, March 17, 2010 767
Respecting the rationale for suspending the 90-day period, in cases where there
De Castro vs. Judicial and Bar Council (JBC) is physical or legal impossibility of compliance with the duty to fill the vacancy
within the said period, the fulfillment of the obligation is released because the law
What complicates the ponencia is its great preoccupation with Section 15 of cannot exact compliance with what is impossible.
Article VII, particularly its fixation with sentences or phrases that are neither In the present case, there can only arise a legal impossibilitywhen the JBC
written nor referred to therein. Verba legis non est recedendum, index animi sermo list is submitted or the vacancy occurred during the appointments ban and the 90-
est. There should be no departure from the words of the statute, for speech is the day period would expire before the end of the appointments ban, in which case the
index of intention. fresh 90-day period should start to run at noon of June 30. This was the factual
IN FINE, all rules of statutory construction virtually revolt against the antecedent respecting the trial court judges involved in Valenzuela. There also
interpretation arrived at by the ponencia. arises a legal impossibility when the list is submitted or the vacancy occurred prior
The 90-day period to fill a vacancy in to the ban and no appointment was made before the ban starts, rendering the lapse
the Supreme Court is suspended during of the 90-day period within the period of the ban, in which case the remaining
the ban on midnight appointments period should resume to run at noon of June 30. The outgoing President would be
Although practically there is no constitutional crisis or conflict involved upon released from non-fulfillment of the constitutional obligation, and the duty
the retirement of the incumbent Chief Justice, the ponencia illustrates the devolves upon the new President.
inapplicability of the 90-day mandate to every situation of vacancy in the Supreme
Court (i.e., the 19-day vacuum articulated in the sixth ratiocination) if only to _______________
buttress its thesis that judicial appointment is an exception to the midnight
appointments ban. The contemplated situation, however, supports the idea that 15 Decision, p. 37.
the 90-day period is suspended during the effectivity of the ban. 16 Infra note 18.
I submit that the more important and less complicated question iswhether the 769
90-day period in Section 4(1) of Article VIII14 runs during the period of prohibition
VOL. 615, March 17, 2010 769
in Section 15 of Article VII.
In response to that question, the ponencia declares that it is the President’s De Castro vs. Judicial and Bar Council (JBC)
“imperative duty to make an appointment of a Member of the Supreme Court
within 90 days from the occurrence of the vacancy [and that t]he failure by the Considering also that Section 15 of Article VII is an express limitation on the
Presi- President’s power of appointment, the running of the 90-day period is
deemed suspended during the period of the ban which takes effect only once
_______________ every six years.
This view differs from Valenzuela in that it does not implement Section 15 of
14 CONSTITUTION, Art. VIII, Sec. 4 (1). The Supreme Court shall be composed Article VII so as to breach Section 4(1) of Article VIII. Instead of disregarding the
of a Chief Justice and fourteen Associate Justices. It may sit en banc or in its 90-day period in the observance of the ban on midnight appointments, the more
discretion, in division of three, five, or seven members. Any vacancy shall be filled logical reconciliation of the two subject provisions is to consider the ban as having
within ninety days from the occurrence thereof. (emphasis and underscoring the effect of suspending the duty to make the appointment within 90 days from the
supplied) occurrence of the vacancy. Otherwise stated, since there is a ban, then there is no
768 duty to appoint as the power to appoint does not even exist. Accordingly, the 90-
day period is suspended once the ban sets in and begins or continues to run only
768 SUPREME COURT REPORTS ANNOTATED
upon the expiration of the ban.
De Castro vs. Judicial and Bar Council (JBC)
One situation which could result in physical impossibility is the inability of 18 Vide Decision, p. 45.
the JBC to constitute a quorum for some reasons beyond their control, as that 19 RECORD OF THE 1986 CONSTITUTIONAL COMMISSION, Vol. 1, July 14, 1986,
depicted by Justice Arturo Brion in his Separate Opinion, in which case the 90-day RCC No. 29 (CD Format. Commissioner Romulo stated that “[t]he sense of the
period could lapse without fulfilling the constitutional obligation. Committee is that 60 days is awfully short and that the
Another such circumstance which could frustrate the ponencia’s depiction of [Judicial and Bar] Council, as well as thePresident, may have difficulties with
the inflexibility of the period is a “no-takers” situation where, for some reason, that.”
there are no willing qualified nominees to become a Member of the Court. 17 Some 771
might find this possibility remote, but then again, the situa- VOL. 615, March 17, 2010 771

_______________ De Castro vs. Judicial and Bar Council (JBC)


and could never submit a list to the President before the 90-day period.
17 There is no problem in the case of lower courts since the 90-day period starts Sustaining the view means20 that in case the President appoints as Chief
from the submission of the list to the President. Parenthetically, over and above Justice a sitting member of the Court, from a JBC list which includes, for instance,
the alleged level of importance and urgency between the Court and the lower incumbent justices and “outsiders,” the JBC must forthwith submit a list of
courts, the lack of applicants for judicial posts in the province is a practical reason nominees for the post left vacant by the sitting member-now new Chief Justice.
why the 90-day period for lower courts is reckoned from the submission of the JBC This thus calls for the JBC, in anticipation, to also commence and conclude another
list. Otherwise, one could just imagine the countless constitutional violations nomination process to fill the vacancy, and simultaneously submit a list of
incurred by the President. nominees for such vacancy, together with the list of nominees for the position of
Chief Justice. If the President appoints an “outsider” like Sandiganbayan Justice
770
Edilberto Sandoval as Chief Justice, however, the JBC’s toil and time in the second
770 SUPREME COURT REPORTS ANNOTATED nomination process are put to waste.
De Castro vs. Judicial and Bar Council (JBC) It is ironic for the ponencia to state on the one hand that the President would
tion at hand or the “absurdity”18 of a 19-day overlapping vacuum may have also be deprived of ample time to reflect on the qualifications of the nominees, and to
been perceived to be rare. show on the other hand that the President has, in recent history, filled the vacancy
The seventh ratiocination is admittedly a non-issue. Suffice it to state that in the position of Chief Justice in one or two days.
the Constitution is clear that the appointment must come “from a list x x x It is ironic for the ponencia to recognize that the President may need as much
prepared by the Judicial and Bar Council.” as 90 days of reflection in appointing a member of the Court, and yet abhor the
The Supreme Court can function idea of an acting Chief Justice in the interregnum as provided for by
effectively during the midnight ap- law,21 confirmed by tradition,22 and settled by jurispru-
pointments ban without an ap-
pointed Chief Justice _______________
The ponencia also holds that the JBC has until May 17, 2010, at the latest,
within which to submit to the President the list of nominees for the position of 20 In which case the Court’s complement remains incomplete with still 14
Chief Justice. It declares that the JBC should start the process of selecting the members.
candidates to fill the vacancy in the Supreme Court before the occurrence of the 21 Republic Act No. 296 (Judiciary Act of 1948), Section 12 states that in case
vacancy, explaining that the 90-day period in the proviso, “Any vacancy shall be of a vacancy in the office of Chief Justice, the Associate Justice who is first in
filled within ninety days from the occurrence thereof,” is addressed to the precedence may act as Chief Justice until one is appointed and duly qualified.
President, not to the JBC. 22 Since the time of Chief Justice Cayetano Arellano, this rule of succession
Such interpretation is absurd as it takes the application and nomination stages has been observed throughout the Court’s history whenever the position of Chief
in isolation from the whole appointment process. For the ponencia, the filling of Justice is temporarily vacant for any reason. Vide Revised copy of Special Order
the vacancy only involves the President, and the JBC was not considered when the No. 826 (March 16, 2010)
period was increased from 60 days to 90 days. The sense of the Concom is the exact 772
opposite.19 772 SUPREME COURT REPORTS ANNOTATED
The flaw in the reasoning is made more evident when the vacancy occurs by
virtue of death of a member of the Court. In that instance, the JBC could never De Castro vs. Judicial and Bar Council (JBC)
anticipate the vacancy, dence23to be an internal matter.
The express allowance of a 90-day period of vacancy rebuts any policy
_______________ argument on the necessity to avoid a vacuum of even a single day in the position
of an appointed Chief Justice.
As a member of the Court, I strongly take exception to The Antecedents
the ponencia’s implication that the Court cannot function without a
sitting Chief Justice. In recent weeks, two potential scenarios have gripped the public mind. The
To begin with, judicial power is vested in one Supreme Court24and not in its first is the specter of the failure of our first ever automated election which has
individual members, much less in the Chief Justice alone. Notably, after Chief evoked numerous doomsday predictions. The second is the possibility of the
Justice Puno retires, the Court will have 14 members left, which is more than appointment by President Gloria Macapagal Arroyo of the Chief Justice of the
sufficient to constitute a quorum. Supreme Court—after the compulsory retirement of incumbent Chief Justice
The fundamental principle in the system of laws recognizes that there is only Reynato S. Puno on May 17, 2010.
one Supreme Court from whose decisions all other courts are required to take their
bearings. While most of the Court’s work is performed by its three divisions, the _______________
Court remains one court—single, unitary, complete and supreme. Flowing from
this is the fact that, while individual justices may dissent or only partially concur, 1 Chief Justice Reynato S. Puno in Atty. Oliver O. Lozano and Atty. Evangeline
when the Court states what the law is, it speaks with only one voice.25 J. Lozano-Endriano v. Speaker Prospero C. Nograles, Representative, Majority,
The Court, as a collegial body, operates on a “one member, one vote” basis, House of Representatives; Louis “Barok” C. Biraogo v. Speaker Prospero C.
whether it sits en banc or in divisions. The competence, probity and independence Nograles, Speaker of the House of Representatives, Congress of the Philippines, G.R.
of the Court en banc, or those of the Court’s Division to which the Chief Justice Nos. 187883 & 187910, June 16, 2009, 589 SCRA 356.
belongs, have never depended on whether the member voting 774
774 SUPREME COURT REPORTS ANNOTATED
_______________
De Castro vs. Judicial and Bar Council (JBC)
issued by Chief Justice Reynato S. Puno who goes on wellness and sabbatical leave This has generated frenzied debates in media, in various lawyers’ assemblies,
from March 18-30, 2010 designating Senior Associate Justice Antonio T. Carpio as in the academe, and in coffee shops. It has even spawned a number of rallies and
acting Chief Justice effective March 18, 2010 until Chief Justice Puno reports back demonstrations by civil society groups and by self-styled constitutional experts.
to work. It does not matter that these two situations are merely possibilities, that they
are conjectural and speculative at this moment in time. They have, nonetheless,
23 Cf. Brillantes, Jr. v. Yorac, G.R. No. 93867, December 18, 1990, 192 SCRA captured the public imagination, and have ushered an open season for unfettered
358. discussion and for dire prognostication.
24 CONSTITUTION, Art. VIII, Sec. 1. Not unexpectedly, the controversy posed by the second scenario—involving
25 Complaint of Mr. Aurelio Indencia Arrienda against Justice Puno, 499 Phil. concerns closest to home—has arrived in this Court through various petitions and
1, 14-15; 460 SCRA 1, 16 (2005). interventions.
773 The core issue is whether the sitting President of the Philippines, Gloria
Macapagal Arroyo, can validly appoint the Chief Justice of the Supreme Court
VOL. 615, March 17, 2010 773
when the incumbent Chief Justice, Reynato S. Puno, compulsorily retires on May
De Castro vs. Judicial and Bar Council (JBC) 17, 2010, in light of two apparently conflicting provisions of the Constitution.
as Chief Justice is merely an acting Chief Justice or a duly appointedone. Article VII, Section 15, provides a constitutional limitation on the President’s
IN LIGHT OF THE FOREGOING, I vote to hold, for the guidance of the power of appointment, viz.:
Judicial and Bar Council, that the incumbent President is constitutionally “Sec. 15. Two months immediately before the next presidential
proscribed from appointing the successor of Chief Justice Reynato S. Puno upon elections and up to the end of his term, a President or Acting President
his retirement on May 17, 2010 until the ban ends at 12:00 noon of June 30, 2010. shall not make appointments, except temporary appointments to executive
positions when continued vacancies therein will prejudice public service or
SEPARATE OPINION endanger public safety.2
On the other hand, Article VIII, Section 4(1) contains an express mandate for
NACHURA, J.: the President to appoint the Members of the Supreme Court within ninety days
“No amount of exigency can make this Court exercise a power where it from the occurrence of a vacancy, thus—
is not proper.”1 Sec. 4(1). The Supreme Court shall be composed of a Chief Justice and
I am deeply impressed by the very well written ponencia of Justice Lucas P. fourteen Associate Justices. It may sit en banc or, in its
Bersamin. However, I am unable to concur in all of his conclusions. Instead, I vote
to dismiss all the petitions because they have utterly failed to present a justiciable _______________
controversy.
2 Emphasis supplied. As to the time to submit this shortlist to the proper appointing authority, in
775 the light of the Constitution, existing laws and jurisprudence, the JBC welcomes
VOL. 615, March 17, 2010 775 and will consider all views on the matter.”5
On January 20, 2010, the JBC formally announced the opening, for application
De Castro vs. Judicial and Bar Council (JBC) or recommendation, of the position of Chief Justice of this Court, thus—
discretion, in divisions of three, five, or seven Members. Any vacancy shall be “The Judicial and Bar Council (JBC) announces the opening for application or
filled within ninety days from the occurrence thereof.3 recommendation, of the position of CHIEF JUSTICE OF THE SUPREME COURT,
in relation to Article VIII, Section 9, which states that— which will be vacated on 17 May 2010 upon the retirement of the incumbent Chief
“Sec. 9. The Members of the Supreme Court and judges of lower courts shall Justice, HON. REYNATO S. PUNO.
be appointed by the President from a list of at least three nominees prepared by Applications or recommendations for this position must be submitted not later
the Judicial and Bar Council for every vacancy. Such appointments need no than 4 February 2010 (Thursday) to the JBC Secretariat. x x x.”6
confirmation. Any vacancy shall be filled within ninety days from the occurrence In its February 8, 2010 meeting, the JBC decided to proceed with the process
thereof. of announcing to the public the names of the candidates for the position. Included
For the lower courts, the President shall issue the appointments within ninety in the list of applicants are: (1) Brion, Arturo D.; (2) Carpio, Antonio T.; (3) Corona,
days from the submission of the list. Renato C.; (4) Carpio Morales, Conchita; (5) Leonardo-de Castro, Teresita J.; and
The perceived conflict was resolved in administrative matter, In Re (6) Sandoval, Edilberto G.7
Appointments Dated March 30, 1998 of Hon. Mateo A. Valenzuela and Hon. Placido
B. Vallarta as Judges of the Regional Trial Court of Branch 62, Bago City and of _______________
Branch 24, Cabanatuan City, respectively.4 Therein, the Court was confronted with
the question of whether the appointments of the concerned RTC judges, issued 5 http://jbc.judiciary.gov.ph/announcements/JBCreCJ.pdf (visited: March 11,
within two months before the presidential election in 1998, were valid. The Court 2010).
answered that, in the given situation, Article VII, Section 15, has primacy over 6 http://jbc.judiciary.gov.ph/announcements/jbc_announce_2009/jan.22’10.pdf
Article VIII, Section 4(1), because the former was “couched in stronger negative (visited: March 11, 2010).
language.” Accordingly, the appointments were nullified. However, Valenzuela’s 7 Comment of the JBC, p. 6.
applicability to the present controversy is challenged by most of herein petitioners.
777
The petitions were filed following certain acts of the Judicial and Bar Council
(JBC) related to the constitutional procedure for the appointment of Supreme VOL. 615, March 17, 2010 777
Court justices, specifically in the matter of the appointment of Chief Justice Puno’s De Castro vs. Judicial and Bar Council (JBC)
successor. On January 18, 2010, the JBC passed a Resolution which relevantly These developments, having already engendered near-hysterical debates,
reads: impelled a number of petitioners to file suit. However, obviously hedging against
the possibility that the cases would be disallowed on the ground of prematurity,
_______________ petitioners came to Court using different procedural vehicles.
In G.R. No. 191002, petitioner Arturo de Castro entreats the Court to issue a
3 Emphasis supplied. writ of mandamus to compel the JBC to send the list of nominees for Chief Justice
4 A.M. No. 98-5-01-SC, November 9, 1998, 298 SCRA 408. to the incumbent President when the position becomes vacant upon the retirement
776 of Chief Justice Puno on May 17, 2010.
776 SUPREME COURT REPORTS ANNOTATED The Philippine Constitution Association (PHILCONSA) and John Peralta,
petitioners in G.R. Nos. 191057 and 191149, respectively, plead for the same relief.
De Castro vs. Judicial and Bar Council (JBC) In G.R. No. 191032, Jaime Soriano seeks the issuance by the Court of a writ
prohibiting the JBC from continuing with its proceedings, particularly the
“The JBC, in its en banc meeting of January 18, 2010, unanimously agreed to screening of applicants for Chief Justice, based on the hypothesis that the
start the process of filling up the position of Chief Justice to be vacated on May 17, authority to appoint the Chief Justice pertains exclusively to the Supreme Court.
2010 upon the retirement of the incumbent Chief Justice Honorable Reynato S. He posits that it is the Court that must commence its own internal proceeding to
Puno. select the successor of Chief Justice Puno.
It will publish the opening of the position for applications or recommendations; Amador Tolentino, Jr., in G.R. No. 191342, asks this Court to enjoin and
deliberate on the list of candidates; publish the names of candidates; accept restrain the JBC from submitting the list of nominees for judiciary positions,
comments on or opposition to the applications; conduct public interviews of including that of Chief Justice, to the incumbent President during the period
candidates; and prepare the shortlist of candidates. covered in Article VII, Section 15 of the Constitution.
In a cleverly crafted petition which he denominated an administrative matter, dismissed outright, because of the absence of an actual case or
former Solicitor General Estelito P. Mendoza filed A.M. No. 10-2-5-SC, imploring controversy ripe for judicial adjudication and because of petitioners’ lack
this Court to rule, for the guidance of the JBC, whether the constitutional of legal standing to institute the cases.
prohibition in Article VII, Section 15, applies to positions in the judiciary and Oppositor-Intervenor Mitchell John Boiser posits, among others, that the
whether the incumbent President may appoint the successor of Chief Justice Puno petitions for mandamus are premature because there is yet no final list of
upon the latter’s retirement.778 nominees and the position of Chief Justice is not yet vacant.
778 SUPREME COURT REPORTS ANNOTATED Oppositors-Intervenors Yolanda Quisumbing-Javellana, Belleza Alojado
Demaisip, Teresita Gandionco-Oledan, Ma. Verena Kasilag-Villanueva, Marilyn
De Castro vs. Judicial and Bar Council (JBC) Sta. Romana, Leonila de Jesus, and Guinevere de Leon contend, among others,
that the incumbent President is prohibited from making appointments within the
Notably, although the petitions sport different appellations (for mandamus, or period prescribed in Article VII, Section 15; that the next President will still have
prohibition, or even as an administrative matter), they (except the Soriano ample time to appoint a Chief Justice when Chief Justice Puno retires on May 17,
petition) share a common bottom line issue, i.e., a definitive ruling on whether, in 2010 before the 90-day period for appointment mandated in Article VIII, Section
light of the perceived conflict between Article VII, Section 15, and Article VIII, 4(1) expires; and that in the interim, the duties of the Chief Justice can be
Section 4(1), the incumbent President can validly appoint a Chief Justice after exercised by the most senior of the incumbent Supreme Court justices.
Chief Justice Puno retires on May 17, 2010.
Thus, the Court consolidated the petitions and required the JBC and the Office My Position
of the Solicitor General (OSG) to file their respective comments.
Significantly, the JBC, in its February 25, 2010 Comment, stated: After careful perusal of the pleadings and painstaking study of the applicable
11. The next stage of the process which will be the public interview of law and jurisprudence, I earnestly believe that the consolidated petitions should
the candidates, and the preparation of the shortlist of candidates have yet be dismissed, because they do not raise an actual case or controversy ripe
to be undertaken by the JBC as of this date, including the interview of the for judicial determination.780
constitutional experts, as may be needed.
Likewise, the JBC has yet to take a position on when to submit the 780 SUPREME COURT REPORTS ANNOTATED
shortlist to the proper appointing authority, in light of Section 4(1), Article De Castro vs. Judicial and Bar Council (JBC)
VIII of the Constitution, which provides that vacancy in the Supreme
Court shall be filled within ninety (90) days from the occurrence thereof, As an essential ingredient for the exercise of the power of judicial review,
Section 15, Article VIII of the Constitution concerning the ban on an actual case or controversy involves a conflict of legal rights, an assertion of
Presidential appointments “two (2) months immediately before the next opposite legal claims susceptible to judicial resolution.9 The controversy must
presidential elections and up to the end of his term” and Section 261(g), be justiciable—definite and concrete—touching on the legal relations of parties
Article XXII of the Omnibus Election Code of the Philippines.”8 having adverse legal interests. In other words, the pleadings must show an active
On the other hand, the OSG, in its Comment dated February 26, 2010, took antagonistic assertion of a legal right, on one hand, and a denial thereof, on the
the position that the incumbent President of the Philippines can appoint the other; that is, the case must concern a real and not a merely theoretical question
successor of Chief Justice Puno when he retires on May 17, 2010, because the or issue. There ought to be an actual and substantial controversy admitting of
prohibi- specific relief through a decree conclusive in nature, as distinguished from an
opinion advising what the law would be upon a hypothetical state of facts. 10 The
_______________ rationale for this requirement is to prevent the courts through avoidance of
premature adjudication from entangling themselves in abstract disagreements,
8 Italics supplied. and for us to be satisfied that the case does not present a hypothetical injury or a
779 claim contingent upon some event that has not and indeed may never transpire.11
VOL. 615, March 17, 2010 779 Thus, justiciability requires (1) that there be an actual controversy between
or among the parties to the dispute; (2) that the interests of the parties be adverse;
De Castro vs. Judicial and Bar Council (JBC) (3) that the matter in controversy be capable of being adjudicated by judicial power;
tion in Article VII, Section 15, of the Constitution does not apply to appointments and (4) that the determination of the controversy will result in practical relief to
in the Supreme Court. the complainant.12
Meanwhile, several motions for intervention with oppositions-in-intervention
were received by the Court. _______________
Oppositors-Intervenors Antonio Gregorio III, Peter Irving Corvera, Walden
Bello, Loretta Ann Rosales, and National Union of Peoples’ Lawyers uniformly
contend in their pleadings that the consolidated petitions should be
9 Congressman Enrique T. Garcia of the 2nd District of Bataan v. The De Castro vs. Judicial and Bar Council (JBC)
Executive Secretary, The Secretary of the Department of Energy, Caltex Philippines, submit its list, if it is already in existence, to the incumbent
Inc., Petron Corporation, and Pilipinas Shell Corporation, G.R. No. 157584, April President. Mandamus is proper only to compel the performance, when refused,
2, 2009, 583 SCRA 119. of a ministerial duty.15 The mandamus petition therefore has no leg to stand on as
10Information Technology Foundation of the Philippines v. Commission on it presents no actual case ripe for judicial determination.
Elections, G.R. No. 159139, June 15, 2005, 460 SCRA 291, 312-313. PHILCONSA, for its part, contends that two applicants for the post, Justices
11 Office of the Governor v. Select Committee of Inquiry, 271 Conn. 540, 570, Carpio and Carpio Morales, manifested their interest in their nomination on the
858 A.2d 709 (2004). condition that the same will be submitted to the next President. According to
12 Astoria Federal Mortgage Corporation v. Matschke, 111 Conn. App. 462, 959 PHILCONSA, this fact “has created a dilemma/quandary to respondent JBC
A.2d 652 (2008). whether to exclude [from] or include [in the list] the names of said two Senior
781 Justices.”16 It then prays for this Court to rule on the issue.
VOL. 615, March 17, 2010 781 PHILCONSA, like de Castro and Peralta, is not completely truthful. From its
De Castro vs. Judicial and Bar Council (JBC) comment, it appears that, as early as February 10, 2010, the JBC had already
By these standards, the consolidated petitions do not present a justiciable included the two justices, despite their conditional acceptance of their
nominations, in the list of applicants for the post. There is no quandary to speak
controversy because of the absence of clashing legal rights. The JBC has merely
started the selection process by accepting applications and nominations for the of.
position of Chief Justice. This is only the initial stage of the procedure for To justify their petitions for prohibition, Jaime Soriano and Amador Tolentino,
appointment of a Chief Justice. By the JBC’s own admission, it has yet to Jr. allege that the JBC has already started the screening process for Chief
undertake the public interview of the applicants; it has yet to prepare the shortlist Justice.17 Thus, they claim that the Court can now resolve the constitutional
and to decide whether it needs to interview constitutional experts. question and issue the writ prohibiting the JBC from submitting the list of
nominees to the incumbent President.
Arturo de Castro and John Peralta justify the propriety of the filing of their
respective petitions for certiorari and mandamus by a common thread: that the As earlier mentioned, absent a shortlist of nominees for Chief Justice prepared
JBC has deferred its decision as to whom to submit the list of by the JBC, there is yet nothing that the Court can prohibit the JBC
nominees.13 They are then asking the Court to compel the JBC to submit the from submitting to the incumbent President. The JBC has not even intimated
list to the incumbent President. concretely that it will perform the act sought to be prohibited—
De Castro’s and Peralta’s submission tends to mislead the Court. It is
clear from the narrated facts that there is yet no list to submit. The JBC is still in _______________
the process of screening applicants for the position. Since there is no list to be
submitted, there can be no deferment of its submission. De Castro and Peralta the damages sustained by the petitioner by reason of the wrongful acts of the
have not shown or even alleged that the JBC has refused or has been respondent.
unlawfully neglecting14 to
15 Pefianco v. Moral, 379 Phil. 468, 479; 322 SCRA 439 (2000).
_______________ 16 PHILCONSA petition, p. 5.
17 Soriano petition, p. 4; and Tolentino petition, p. 2.
13 De Castro petition, p. 5; and Peralta petition, p. 1. 783
14 Section 3 of Rule 65 pertinently provides that: VOL. 615, March 17, 2010 783
Sec. 3. Petition for mandamus.—When any tribunal, corporation, board, De Castro vs. Judicial and Bar Council (JBC)
officer or person unlawfully neglects the performance of an act which the law
submitting a list to the incumbent President. The JBC merely started the
specifically enjoins as a duty resulting from an office, trust, or station, or
screening process. Let it be noted that a writ of prohibition is issued to command
unlawfully excludes another from the use and enjoyment of a right or office to
a respondent to desist from further proceeding in the action or matter
which such other is entitled, and there is no other plain, speedy and adequate
specified.18 Likewise, without a shortlist, there is nothing that this Court can
remedy in the ordinary course of law, the person aggrieved thereby may file a
mandate the JBC to submit to the President.
verified petition in the proper court, alleging the facts with certainty and praying
As to the petition filed by Estelito Mendoza, while it is captioned as an
that judgment be rendered commanding the respondent, immediately or at some
administrative matter, the same is in the nature of a petition for declaratory
other time to be specified by the court, to do the act required to be done to protect
relief. Mendoza pleads that this Court interpret two apparently conflicting
the rights of the petitioner, and to pay
provisions of the Constitution—Article VII, Section 15 and Article VIII, Section
782
4(1). Petitioner Mendoza specifically prays for such a ruling “for the guidance of
782 SUPREME COURT REPORTS ANNOTATED the [JBC],” a relief evidently in the nature of a declaratory judgment.
Settled is the rule that petitions for declaratory relief are outside the _______________
jurisdiction of this Court.19 Moreover, the Court
thereof, bring an action in the appropriate Regional Trial Court to determine
_______________ any question of construction or validity arising, and for a declaration of his rights
or duties, thereunder.
18 Section 2 of Rule 65 provides that: 20 Albay Electric Cooperative, Inc., Edgardo A. San Pablo, and Evan Calleja v.
Sec. 2. Petition for prohibition.—When the proceedings of any tribunal, Hon. Rafael P. Santelices, in his capacity as the Presiding Judge of the Regional
corporation, board, officer or person, whether exercising judicial, quasi-judicial or Trial Court of Legazpi City, Branch No. 2, and Mayon International Hotel, Inc.,
ministerial functions, are without or in excess of its or his jurisdiction, or with G.R. No. 132540, April 16, 2009, 585 SCRA 103.
grave abuse of discretion amounting to lack or excess of jurisdiction, and there is 785
no appeal or any other plain, speedy, and adequate remedy in the ordinary course VOL. 615, March 17, 2010 785
of law, a person aggrieved thereby may file a verified petition in the proper court,
alleging the facts with certainty and praying that judgment be rendered De Castro vs. Judicial and Bar Council (JBC)
commanding the respondent to desist from further proceedings in the action or recently decided Lozano v. Nograles,21 which this Court dismissed through the pen
matter specified therein, or otherwise granting such incidental reliefs as law and of Chief Justice Puno. As no positive act has yet been committed by
justice may require. respondents, the Court must not intervene.Again, to borrow the words of
19 Article VIII, Section 5 of the Constitution does not include petitions for Chief Justice Puno in Lozano, “judicial review is effective largely because it is not
declaratory relief among those within the original jurisdiction of the Supreme available simply at the behest of a partisan faction, but is exercised only to remedy
Court. Section 1 of Rule 63 further provides that: a particular, concrete injury.”
Sec. 1. Who may file petition.—Any person interested under a deed, will, Further, the Mendoza petition cannot be likened to the administrative matter
contract or other written instrument, or whose rights are affected by a statute, in In Re Appointments of Hon. Valenzuela & Hon. Vallarta,22 over which the Court
executive order or regulation, ordinance, or any other governmental regulation assumed jurisdiction. In that case, the President appointed judges within the
may, before breach or violation constitutional ban and transmitted the appointments to the Chief Justice. Clearly,
784 an actual controversy ripe for judicial determination existed in that case because
a positive act had been performed by the President in violation of the Constitution.
784 SUPREME COURT REPORTS ANNOTATED Here, as shown above, no positive act has been performed by either the JBC or the
De Castro vs. Judicial and Bar Council (JBC) President to warrant judicial intervention.
does not sit to adjudicate mere academic questions to satisfy scholarly To repeat for emphasis, before this Court steps in to wield its awesome power
interest, however intellectually challenging.20 While Mendoza and the other of deciding cases, there must first be an actual controversy ripe for judicial
petitioners espouse worthy causes, they have presented before this Court issues adjudication. Here, the allegations in all the petitions are conjectural or
which are still subject to unforeseen possibilities. In other words, the issues they anticipatory. No actual controversy between real litigants exists.23 These
raised are hypothetical and unripe for judicial determination. consolidated petitions, in other words, are a “purely academic exercise.” Hence,
At this point, several contingent events are still about to unfold. The any resolution that this Court might make would constitute an attempt at
JBC, after it has screened the applicants, may decide to submit the shortlist of abstraction that
nominees either before or after the retirement of Chief Justice Puno. If it decides
to submit the list after May 17, 2010, it may opt to transmit said list of nominees _______________
to President Macapagal-Arroyo or to the next President. If the list is transmitted
to her, the incumbent President may either appoint or not appointthe 21 Supra note 1.
replacement of Chief Justice Puno. We cannot assume that the JBC will do one 22 358 Phil. 896; 298 SCRA 408 (1998).
thing or the other. Neither can we truly predict what the incumbent President will 23 See Confederation of Sugar Producers Association, Inc. v. Department of
do if such a shortlist is transmitted to her. For us to do so would be to engage in Agrarian Reform, G.R. No. 169514, March 30, 2007, 519 SCRA 582, 620; Board of
conjecture and to undertake a purely hypothetical exercise. Optometry v. Hon. Colet, 328 Phil. 1187, 1206 (1996); and Abbas v. Commission on
Thus, the situation calling for the application of either of the Elections, G.R. Nos. 89651 & 89965, November 10, 1989, 179 SCRA 287, 300.
conflicting constitutional provisions will arise only when still other 786
contingent events occur. What if the JBC does not finish the screening process
786 SUPREME COURT REPORTS ANNOTATED
during the subject period? What if the President does not make the appointment?
Verily, these consolidated petitions involve “uncertain contingent future events De Castro vs. Judicial and Bar Council (JBC)
that may not occur as anticipated, or indeed may not occur at all,” similar to the can only lead to barren legal dialectics and sterile conclusions unrelated to
actualities.24
Moreover, the function of the courts is to determine controversies between statute being challenged as unconstitutional, and they were about to exercise
litigants and not to give advisory opinions.25 Here, petitioners are asking this powers under the likewise challenged provisions of the statute. Thus, in those
Court to render an advisory opinion on what the JBC and the President should do. cases, there was the inevitability of the operation of a challenged statute against
To accede to it is tantamount to an incursion into the functions of the executive the appellants. No such situation exists in the cases before us.
department.26 This will further inappropriately make the Court an adviser of the Here, the factual and legal setting is entirely different. The JBC only started
President. Chief Justice Enrique Fernando, in his concurring opinion in Director the screening of the applicants. It has not yet transmitted a list to the President,
of Prisons v. Ang Cho Kio,27specifically counseled against this undue portrayal by as, in fact, it still has to make the list. The President has not yet made an
the Court of the alien role of adviser to the President, thus— appointment for there is yet no vacancy and no shortlist has yet been transmitted
“Moreover, I would assume that those of us entrusted with judicial to her. The constitutional provisions in question are not yet in operation; they may
responsibility could not be unaware that we may be laying ourselves open to the not even be called into operation. It is not time for the Court to intervene.
charge of presumptuousness. Considering that the exercise of judicial authority
does not embrace the alien role of a presidential adviser, an indictment of _______________
officiousness may be hard to repel. It is indefinitely worse if the advice thus
gratuitously offered is ignored or disregarded. The loss of judicial prestige may be 28 424 US 1 (1976).
incalculable. Thereafter, there may be less than full respect for court decisions. It 29 419 US 102 (1974).
would impair the confidence in its ability to live up to its trust not only on the part 788
of immediate parties to the litigation but of the general public as well. Even if the
788 SUPREME COURT REPORTS ANNOTATED
teaching of decided cases both here and in the Philippines is not as clear therefore,
there should be, to say the least, the utmost reluctance on the part of any court to De Castro vs. Judicial and Bar Council (JBC)
arrogate for itself such a prerogative, the exercise of which is fraught with
possibilities of such undesirable character.” A final note. If petitioners only want guidance from this Court, then, let it be
stated that enough guidance is already provided by the Constitution, the relevant
_______________ laws, and the prevailing jurisprudence on the matter. The Court must not be
unduly burdened with petitions raising abstract, hypothetical, or
24 Sec. Guingona, Jr. v. Court of Appeals, 354 Phil. 415, 429; 292 SCRA 402, contingent questions.As fittingly phrased by Chief Justice Puno in Lozano—
416-417 (1998); Angara v. Electoral Commission, 63 Phil. 139, 158 (1936). “Given the sparseness of our resources, the capacity of courts to render efficient
25 Automotive Industry Workers Alliance v. Romulo, G.R. No. 157509, January judicial service to our people is severely limited. For courts to indiscriminately
18, 2005, 449 SCRA 1, 10. open their doors to all types of suits and suitors is for them to unduly overburden
26 See Sec. Guingona, Jr. v. Court of Appeals, supra note 24. their dockets, and ultimately render themselves ineffective dispensers of justice.
27 33 Phil. 494, 510 (1970). To be sure, this is an evil that clearly confronts our judiciary today.”30
787 With the above disquisition, I find no compelling need to discuss the other
issues raised in the consolidated petitions.
VOL. 615, March 17, 2010 787
In light of the foregoing, I vote for the dismissal of the consolidated petitions.
De Castro vs. Judicial and Bar Council (JBC)
The ponencia holds that “we need not await the occurrence of the vacancy by SEPARATE OPINION
May 17, 2010 in order to have the principal issue be ripe for judicial
determination.” That may very well be desirable. But still, there must be the BRION, J.:
palpable presence of an actual controversy because, again, as discussed above, this I AGREE with the conclusion that the President can appoint the Chief Justice
Court does not issue advisory opinions. The Court only adjudicates actual cases and Members of the Supreme Court two months before a presidential election up
that present definite and concrete controversies touching on the legal relations of to the end of the President’s term, but DISAGREE with the conclusion that the
the parties having adverse legal interests. authority to appoint extends to the whole Judiciary.
The ponencia also sought refuge in the American cases of Buckley v. I. Prefatory Statement
Valeo28 and Regional Rail Reorganization Act Cases29 to support its position that The debate, in and out of this Court, on the issues these consolidated cases
“the reasonable certainty of the occurrence of the perceived threat to a pose, have been differently described to be at varying levels of severity and
constitutional interest is sufficient to afford a basis for bringing a challenge, intensity. What we in Court do know is the multiplicity of petitions and
provided the Court has sufficient facts before it to enable it to intelligently interventions
adjudicate the issues.” The cited American cases only considered the issue of
ripeness and did not confront the absence of an actual case or controversy. Further, _______________
in Buckley, the members of the Commission were already appointed under the
30 Supra note 1. (5) The Council shall have the principal functions of recommending
789 appointees to the Judiciary. It may exercise other functions and duties as the
VOL. 615, March 17, 2010 789 Supreme Court may assign to it.
Section 9. The Members of the Supreme Court and the judges of the lower
De Castro vs. Judicial and Bar Council (JBC) courts shall be appointed by the President from a list of at least three nominees
filed, generating arguments of varying shades of validity. Sad but true, what we prepared by the Judicial and Bar Council for every vacancy. Such appointment
need in considering all these submissions is simplification and focus on the critical needs no confirmation.
issues, not the mass of opinions that merely pile on top of one another. Based on For the lower courts, the President shall issue the appointment within ninety
this standard, this Opinion shall endeavor to be brief, succinct but clear, and may days from the submission of the list.”
not be the academic treatise lay readers and even lawyers customarily expect from These provisions are quoted together to stress the role the JBC plays in the
the Court. appointment process, and that it is effectively an adjunctof the Supreme Court:
The constitutional provisions whose interpretation and application are the JBC is under the supervision of the Court, but is fully independent in
disputed (the disputed provisions) are Section 15, Article VII (the Article on the undertaking its main function; the Chief Justice is the Chair, with the SC Clerk of
Executive Department) and Sections 4(1) and 9 of Article VIII (on the Judicial the Supreme Court as the Secretary; the emoluments of Council members are
Department). Not often mentioned but critical to the consideration of the disputed determined by the Supreme Court with the Council budget a part of the Supreme
provisions is Section 8, Article VIII on the Judicial and Bar Council (JBC)—the Court budget; and the Supreme Court may assign functions and duties to the
entity whose acts are under scrutiny in the dispute. Council.
Section 15 of Article VII provides: II. The Questions of Standing & Justiciability
“Section 15. Two months immediately before the next presidential elections I completely agree with the ponencia’s ruling on the parties’ standing,
and up to the end of his term, a President or Acting President shall not make their locus standi, to bring their petitions and interventions in their capacities as
appointments, except temporary appointments to executive positions when citizens and lawyers who
continued vacancies therein will prejudice public service or endanger public 791
safety.”
VOL. 615, March 17, 2010 791
On the other hand, the relevant Judicial Department provisions read:
“Section 4(1). The Supreme Court shall be composed of a Chief Justice and De Castro vs. Judicial and Bar Council (JBC)
fourteen Associate Justices. It may sit en banc or in its discretion, in division of stand to be affected by our ruling as lawyers or by the impact of our ruling on the
three, five, or seven Members. Any vacancy shall be filled within ninety days from nation and the all-important electoral exercise we shall hold in May 2010.
the occurrence thereof. Jurisprudence is replete with precedents on the liberal appreciation of the locus
xxx standi rule on issues that are of transcendental concern to the nation, 1 and the
Section 8. (1) A Judicial and Bar Council is hereby created under the petitioners very well qualify under these rulings. In this sense, locus standi is not
supervision of the Supreme Court composed of the Chief Justice as ex a critical issue in the present case. In fact, the concern voiced out during the
officio Chairman, the Secretary of Justice, and a representative of the Congress Court’s deliberations, was more on how participation can be limited to those who
as ex officio Members, a representative of the Integrated Bar, a professor of law, a have substantial contributions, through their submissions, to the resolution of the
retired Member of the Supreme Court, and a representative of the private grave issues before the Court.
sector.790 While the rule on locus standi can be relaxed, the rule on the need for an actual
790 SUPREME COURT REPORTS ANNOTATED justiciable case that is ripe for adjudication addresses a different concern and
cannot be similarly treated. I disagree with the ponencia’s ruling on
De Castro vs. Judicial and Bar Council (JBC) justiciability as I believe some of the petitions before us do not reach the
required level of justiciability; others, however, qualify as discussed
(2) The regular members of the Council shall be appointed by the President below so that my disagreement with the lack of justiciability of some of
for a term of four years with the consent of the Commission on Appointments. Of the petitions need not hinder the Court’s consideration of the main issue
the Members first appointed, the representative of the Integrated Bar shall serve at hand.
for four years, the professor of law for three years, the retired justice for two years,
and the representative of the private sector for one year. _______________
(3) The Clerk of the Supreme Court shall be the Secretary ex officio of the
Council and shall keep a record of its proceedings. 1 Roque v. Commission on Elections, G.R. No. 188456, September 10, 2009, 599
(4) The regular members of the Council shall receive such emoluments as SCRA 69; Garcillano v. House of Representatives, G.R. No. 170388, December 23,
may be determined by the Supreme Court. The Supreme Court shall provide in its 2008, 575 SCRA 170; David v. Macapagal-Arroyo, G.R. Nos. 171396, 171409,
annual budget the appropriations of the Council. 171485, 171483, 171400, 171489, 171424, May 3, 2006, 489 SCRA 160, 224; Agan
Jr. v. Philippine International Air Terminals Co., Inc., 450 Phil. 744; 803-804; 402 Separately from the above concept of claims involving demandable rights and
SCRA 612 (2003); Bayan v. Executive Secretary Zamora, 396 Phil. 623, 548-650; obligations (but no less real in the strict constitutional sense), is the authority of
342 SCRA 449 (2000); Kilosbayan, Incorporated v. Guingona, Jr., G.R. No. 113375, the Supreme Court to rule on matters arising in the exercise of its power of
May 5, 1994, 232 SCRA 110, 138; Association of Small Landowners in the supervision.
Philippines, Inc. v. Secretary of Agrariam Reform, G.R. Nos. 78742, 79310, 79744, Under Section 6 of Article VIII of the Constitution, the Supreme Court is
79777, July 14, 1989, 175 SCRA 343 365; and Araneta v. Dinglasan, 84 Phil. 368, granted the power of administrative supervision over all courts and the personnel
373 (1949). thereof. Pursuant to this power, the Court issues administrative circulars and
792 memoranda to promote the efficient and effective administration of justice, and
792 SUPREME COURT REPORTS ANNOTATED holds judges and court personnel administratively accountable for lapses they may
commit.6 Through these circulars, memoranda and administrative matters and
De Castro vs. Judicial and Bar Council (JBC) cases, the Court likewise interprets laws relevant to its power of supervision. 7 The
Court likewise issues rules concerning, among others, the protection and
The basic requisite before this Court can rule is the presence of an actual case enforcement of constitutional rights, pleading, practice, and procedure in all
calling for the exercise of judicial power. This is a requirement that the courts, the admission to the practice of law, and the Integrated Bar.8
Constitution itself expressly imposes; in granting the Court judicial power and in This aspect of the power of the Court—its power of supervision—is particularly
defining the grant, the Constitution expressly states that judicial power includes relevant in this case since the JBC was created “under the supervision of the
the duty to settle actual controversies involving rights which are legally Supreme Court,” with the “principal function of recommending appointees to the
demandable and enforceable.2 Thus, the Court does not issue advisory opinions, Judiciary.” In the same manner that the Court cannot dictate on the lower courts
nor do we pass upon hypothetical cases, feigned problems or friendly suits on how they should decide cases except through the appeal and review process
collusively arranged between parties without real adverse interests. Courts cannot provided by the Rules of Court, so also cannot the Court intervene in the JBC’s
adjudicate mere academic questions to satisfy scholarly interest, however
intellectually challenging they may be. As a condition precedent to the exercise of _______________
judicial power, an actual controversy between litigants must first exist.3
An actual case or controversy exists when a case involves a clash of legal rights 5 Salonga v. Ernani Cruz Pano, et al., 219 Phil. 402, 429-430; 134 SCRA 438,
or an assertion of opposite legal claims that the courts can resolve through the 463 (1985).
application of law and jurisprudence. The case cannot be abstract or hypothetical 6 See for example, In Re: List of Judges who failed to comply with
as it must be a concrete dispute touching on the legal relations of parties having Administrative Circular No. 10-94, dated June 29, 1994, 439 Phil. 118; 390 SCRA
adverse legal interests. A justiciable controversy admits of specific relief through 319 (2002).
a decree that is conclusive in character, whereas an opinion only advises what the 7 CONSTITUTION, Article VIII, Section, 6.
law would be upon a hypothetical state of facts. An actual case is ripe for 8 Id., Article VIII, Section 5(5).
adjudication when the act being challenged has a direct adverse effect on the
794
individual challenging it.4
In the justiciable cases this Court has passed upon, particularly in cases 794 SUPREME COURT REPORTS ANNOTATED
involving constitutional issues, we have held that the Court also has the duty to De Castro vs. Judicial and Bar Council (JBC)
formulate guiding and controlling constitutional principles, precepts, doctrines, or authority to discharge its principal function. In this sense, the JBC is fully
rules. The Court carries the symbolic function of educating independent as shown by A.M. No. 03-11-16-SC or Resolution Strengthening The
Role and Capacity of the Judicial and Bar Council and Establishing the Offices
_______________ Therein. In both cases, however and unless otherwise defined by the Court (as in
A.M. No. 03-11-16-SC), the Court can supervise by ensuring the legality and
2 Section 1, par. 2, Article VIII, CONSTITUTION. correctness of these entities’ exercise of their powers as to means and manner, and
3 See: Guingona, Jr., v. Court of Appeals, 354 Phil. 426; 292 SCRA 402 (1998); interpreting for them the constitutional provisions, laws and regulations affecting
see also: Director of Prisons v. Ang Cho Kio, 33 Phil. 494 (1970). the means and manner of the exercise of their powers as the Supreme Court is the
4 Id. final authority on the interpretation of these instruments. A prime example of the
793 exercise of the Court’s power of supervision is In Re: Appointments dated March
VOL. 615, March 17, 2010 793 30, 1998 of Hon. Mateo A. Valenzuela and Hon. Placido B. Vallarta as Judges of
the Regional Trial Court of Branch 62, Bago City, and of Branch 24, Cabanatuan
De Castro vs. Judicial and Bar Council (JBC) City, respectively, A.M. No. 98-5-01-SC, November 9, 1998 (hereinafter referred to
the bench and the bar on the extent of protection given by constitutional as Valenzuela) where the Court nullified the oath of office taken by Judge
guarantees.5 Valenzuela, while at the same time giving its interpretation of how the election
ban against appointment operates on the Judiciary, thereby setting the guidelines President following the May 11, 2010 national elections in view of Section 15,
on how Section 15, Article VII is to be read and interpreted. The Valenzuela case Article VII of the Constitution that bans appointments during the election
shall be discussed more fully below. period,”14 citing various newspaper clippings and the judicial notice of this Court.15
a. The De Castro Petition As suggested, we take judicial notice of the JBC action on the nomination
In his petition for certiorari and mandamus, Arturo De Castro (in G.R. process for the position of Chief Justice, as circulated in the media and as
191002) seeks the review of the action of the JBC deferring the sending to the evidenced by official JBC records, and we note that the JBC has taken preliminary
incumbent President of the list of nominees for the position of Chief Justice, and steps but not conclusive action on the submission of a list of nominees for the
seeks as well to compel the JBC to send this list to the incumbent President when position of Chief Justice.16So far, the JBC has announced the forthcoming vacancy,
the position of Chief Justice becomes vacant. He posits that the JBC’s decision to the opening of the position to applicants,17the announcement of nominees, and the
defer action on the list is both a grave abuse of discretion and a refusal to perform invitation for comments.18 These are confirmed in the JBC’s Comment dated
a February 25, 2010 which further states that “the next stage of the process will be
795 the public interview of the candidates, and the preparation of the shortlist of
VOL. 615, March 17, 2010 795 candidates have yet to be undertaken…including the interview of the
constitutional experts as may be needed.”19 Thus, this Court is fully aware,
De Castro vs. Judicial and Bar Council (JBC) based on its official knowledge that the petition cites, of the extent of JBC
constitutionally-mandated duty that may be compelled by mandamus.9 developments in the nomination process, and the petition cannot invoke our
On its face, this petition fails to present any justiciable controversy that can be judicial notice to validly allege that the JBC has deferred action on the matter. For
the subject of a ruling from this Court. As a petition for certiorari, it must first the petition to insist that a deferment has taken place is to mislead this Court on
show as a minimum requirement that the JBC is a tribunal, board or officer a matter that is within its official knowledge.
exercising judicial or quasi-judicial functions and is acting outside its jurisdiction
or with grave abuse of discretion amounting to lack or excess of jurisdiction. 10 A _______________
petition for mandamus, on the other hand, at the very least must show that a
tribunal, corporation, board or officer unlawfully neglects the performance of an 14 De Castro petition, p. 3.
act which the law specifically enjoins as a duty.”11 15 De Castro petition, p. 4.
The petition facially fails to characterize the JBC as a council exercising 16 Judicial notice is taken of the publications cited, as well as the records on
judicial or quasi-judicial functions, and in fact states that the JBC does not have which these publications are based.
any judicial function.12 It cannot so characterize the JBC because it really does not 17 JBC Announcement dated January 20, 2010, part of the record on file with
exercise judicial or quasi-judicial functions. It is not involved in the determination the JBC and with the Court, and published in the Phil. Daily Inquirer on January
of rights and obligations based on the constitution, laws and regulations; it is an 21, 2010.
administrative body under the supervision of the Supreme Court and was created 18 JBC Announcement dated 11 February, 2010, part of the record on file with
principally to nominate appointees to the Judiciary. 13 As such, it deals solely with the JBC and with the Court, and published in the Phil. Daily Inquirer on Feb. 13,
the screening of applicants who wish to have the privilege of applying for judicial 2010.
positions. 19 JBC Comment, dated Feb. 25, 2010, p. 6.
From the point of view of substance, the petition admits that the vacancy for
797
the position of Chief Justice will not occur until May 17, 2010, and alleges that the
JBC has resolved “to defer the decision to whom to send the list of 3 nominees, VOL. 615, March 17, 2010 797
whether to the incumbent President or to the next De Castro vs. Judicial and Bar Council (JBC)
Neither the Constitution nor the Rules of Procedure of the
_______________ JBC20 categorically states when a list of nominees for a vacant Supreme Court
position shall be submitted to the President, although the Constitution gives the
9 De Castro petition, p. 5. President 90 days within which to fill the vacancy.21 This presidential deadline
10 RULES OF COURT, Rule 65, Section 1. implies that the JBC should submit its list of nominees before, or at the latest, on
11 RULES OF COURT, Rule 65, Section 3. the day the vacancy materializes so as not to shorten the 90-day period given to
12 De Castro petition, par. 8, page 5. the President within which to act.
13 See: Constitutional Provision on the JBC, pp. 4-5 of this opinion. Given these timelines and the May 17, 2010 vacancy date—considered with
796 the allegations regarding the nature of the JBC’s functions and its actions that we
796 SUPREME COURT REPORTS ANNOTATED are asked to judicially notice—the De Castro petition filed on February 9, 2010
clearly does not present a justiciable case for the issuance of a writ of certiorari.
De Castro vs. Judicial and Bar Council (JBC) The petition cannot make an incorrect and misleading characterization of the JBC
action, citing our judicial notice as basis, and then proceed to claim that grave De Castro vs. Judicial and Bar Council (JBC)
abuse of discretion has been committed. The study of the question of submitting a
list to the President in the JBC’s step-by-step application and nomination process For the same absence of a justiciable case, the Peralta petition
is not a grave abuse of discretion simply because the petition calls it so for purposes for certiorari and mandamus and all related interventions should be dismissed
of securing a justiciable case for our consideration.22 outright.
Since the obligation to submit a list will not accrue until immediately before or c. The PHILCONSA Petition.
at the time the vacancy materializes (as the petition’s prayer in fact admits), no The petition of the Philippine Constitution Association (PHILCONSA, G.R.
duty can likewise be said to have as yet been neglected or violated to serve as basis 191057) is for mandamus under Rule 65 of the Rules of Court.
for the special civil action of mandamus. The JBC’s study of the applicable It seeks to compel the JBC to include the names of Senior Justices Antonio
constitutional issue, as part of the JBC’s nomination process, cannot be Carpio and Conchita Carpio-Morales, and Prosecutor Dennis Villa Ignacio, in the
“tantamount to a refusal to perform its constitutionally-mandated list of nominees for the position of Chief Justice although these nominees have
duty.” Presently, what exists is a purely potential controversy that has not rip- manifested that they want their names submitted to the incoming, not to the
incumbent, President of the Philippines.
_______________ The petition also seeks various declarations by this Court, among them, that
Section 15, Article VIII should apply only to the Executive Department and not to
20 JBC-009, October 18, 2000. the Judiciary; and that the Decision of this Court in Valenzuela should be set aside
21 CONSTITUTION, Article VIII, Section 4(1). and overruled.
22 See: allegation of grave abuse, De Castro petition, p.5. As basis, the petition alleges that the issues raised in the petition have
798 spawned “a frenzied inflammatory debate on the constitutional provisions” that
798 SUPREME COURT REPORTS ANNOTATED has “divided the bench and the bar and the general public as well.” It likewise
posits that due to the positions the nominees have taken, a “final authoritative
De Castro vs. Judicial and Bar Council (JBC)
pronouncement” from this Court on the meaning and construction of Sections 4(1),
ened into a concrete dispute where rights have been violated or can already be 8(5) and 9, Article VIII . . . in relation with Section 15, Article VII, is necessary.
asserted. The petition grounds itself, too, on the needs of public interest and public service.
In these lights, the Court should dismiss the De Castro petition On the whole, the PHILCONSA petition merely asks for a declaration from
outright. Similarly, the oppositions filed by way of intervening in and this Court of the meaning and interpretation of the constitutional provisions on
anchored on the De Castro petition should similarly be dismissed. the appointment of the Chief Justice, the Members of the Court, and the Judiciary
b. The Peralta Petition. in general during the election ban period.800
John G. Peralta’s petition (G.R. 191149) is likewise
for certiorariand mandamus. Like De Castro’s, he failed to allege that the JBC 800 SUPREME COURT REPORTS ANNOTATED
exercises judicial or quasi-judicial functions—a must in any petition De Castro vs. Judicial and Bar Council (JBC)
for certiorari. In fact the Peralta petition can be described as an imperfect carbon
copy of De Castro’s petition since it similarly asks for the “review of the JBC action As we did with the De Castro petition and based on the same standards we
in deferring to transmit to the incumbent President the list of nominees for discussed above, we hold that the PHILCONSA petition presents no justiciable
appointment of a new Chief Justice, and to compel the JBC to send the same to the controversy that can be the basis for its consideration as a petition
incumbent President for appointment of a Chief Justice, when the position for mandamus and for its adjudication on the merits. On its face, the petition
becomes vacant upon the mandatory retirement of the Honorable Chief Justice defines no specific duty that the JBC should exercise and has neglected to exercise,
Reynato S. Puno.” and presents no right that has been violated nor any basis to assert any legal
Peralta only differs from De Castro because it does not allege “deferment” on right.23 Like the De Castro petition, it only presents to the Court a potential
the basis of media reports and judicial notice; instead, it attaches the January 18, controversy that has not ripened.
2010 resolution of the JBC as Annex “A” and cites this as a basis. An examination Consequently, the Court should rule that the PHILCONSA petition should be
of Annex “A,” however, shows that the JBC did not in fact resolve to defer the dismissed outright together with any intervention supporting or opposing this
submission of the list of nominees; the JBC merely stated that—“As to the time to petition.
submit this shortlist to the proper appointing authority, in light of the Constitution, d. The Mendoza Petition.
existing laws and jurisprudence, the JBC welcomes and will consider all view on The Mendoza petition (A.M. 10-2-5-SC) is unique as even its docket case
the matter.” This is not a deferment, nor is it a refusal to perform a duty assigned number will show; it is presented as an administrative matter for the Court’s
by law as the duty to submit a list of nominees will not mature until a vacancy has consideration pursuant to its power of supervision over judges and over the
or is about to occur.799 JBC,24 following the lead taken in the Valenzuela case (an A.M. case).
VOL. 615, March 17, 2010 799
The cited Valenzuela case is rooted in a situation not far different from the 802
present case; a vacancy in the Court25 had occurred and a difference of opinion 802 SUPREME COURT REPORTS ANNOTATED
arose between the Executive and the Court on the application of Section 15, Article
VII, in relation with Section 4(1) and 9 of Article VIII, of the Constitution. An De Castro vs. Judicial and Bar Council (JBC)
exchange of letters took place between the Palace and the Court on their respective the Executive and the Judiciary on the matters; and the need to “avoid any possible
positions. In the meanwhile, the President appointed two RTC judges (Valenzuela polemics concerning the matter.”27 The petition mentions as well that the Court
and Vallarta) within the two-month period prior addressed the election ban issue in Valenzuela as an A.M. case, and apparently
takes the lead from this decided A.M. matter.
_______________ An undeniable feature of the Mendoza petition, compared to Valenzuela, is its
lack of any clear and specific point where an actual actionable case arose (the
23 Pursuant to Section 3, Rule 65 of the Rules of Court, a petition appointment of two RTC judges during the election ban period) calling for a
for mandamusmust allege the unlawful neglect to perform an act which the law determination of how the Chief Justice and the Court should act. The Mendoza
specifically enjoins as resulting from an office. petition, however, does not look up to the Court’s supervisory authority over lower
24 CONSTITUTION, Article VIII, Section 8(1). court personnel pursuant to Section 6 of Article VIII of the Constitution, in the
25 Upon the retirement of Associate Justice Ricardo J. Francisco. way the Court did in Valenzuela. Expressly, the Mendoza petition looks to
the Court’s supervisory authority over the JBC, an authority that the Court
801
in fact asserted in Valenzuela when, in the exercise of “its power of supervision over
VOL. 615, March 17, 2010 801 the Judicial and Bar Council,” it “INSTRUCTED” the JBC “to defer all actions on
De Castro vs. Judicial and Bar Council (JBC) the matter of nominations to fill up the lone vacancy in the Supreme Court or any
to the election. The Palace forwarded the judges’ appointments to the Court, thus other vacancy until further orders.”
confronting Chief Justice Narvasa with the question of whether—given the From the time of Valenzuela up to the present, the governing law and the
election ban under Section 15, Article VII that prima facie applies—he should relationships between the Court and the JBC have not changed; the supervisory
transmit the appointment papers to the appointed judges so they could take their relationship still exists full strength. The JBC is now in fact waiting for the Court’s
oaths in accordance with existing practice. At that point, the Court decided to treat action on how it regards the Valenzuela ruling—whether the Court will reiterate,
the matter as an “administrative matter” that was ripe for adjudication. modify or completely abandon it. The JBC expressly admitted its dilemna in its
An administrative matter that is entered in the Court’s docket is either an Comment when it said: “Since the Honorable Court is the final interpreter of the
administrative case (A.C.) or an administrative matter (A.M.) submitted to the Constitution, the JBC will be guided by its decision in these consolidated Petitions
Court for its consideration and action pursuant to its power of supervision. An A.C. and Administrative Matter.” Under these plain terms, the JBC recognizes that a
involves disciplinary and other actions over members of the Bar, based on the controversy exists on the issue of submitting a shortlist to the President and it will
Court’s supervision over them arising from the Supreme Court’s authority to not act except with guidance from this Court. This is a point
promulgate rules relating to the admission to the practice of law and to the
Integrated Bar. Closely related to A.C. cases are the Bar Matter (B.M.) cases _______________
particularly involving admission to the practice of law.26 An A.M. is a matter based
on the Supreme Court’s power of supervision: under Section 6, Article VIII, this 27 Mendoza petition, pp. 5 and 6.
refers to the Court’s administrative supervision over all courts and the personnel 803
thereof; under Section 8, it refers to its supervision over the JBC. VOL. 615, March 17, 2010 803
In using an administrative matter as its medium, the Mendoza petition cites
as basis the effect of a complete election ban on judicial appointments (in view of De Castro vs. Judicial and Bar Council (JBC)
the already high level of vacancies and the backlog of cases) and submits this as no less critical, from the point of view of supervision, than the
an administrative matter that the Court, in the exercise of its supervision over the appointment of the two judges during the election ban period
Judiciary, should act upon. At the same time, it cites the “public discourse and in Valenzuela.
controversy” now taking place because of the application of the election ban on the That the JBC has taken this stance is not surprising given the two petitions
appointment of the Chief Justice, citing in this regard the very same reasons for prohibition filed by Jaime N. Soriano (G.R. No. 191032) and Atty. Amador Z.
mentioned in Valenzuela about the need to resolve the issue and avoid the Tolentino, Jr., (G.R. No. 191342) that, on their face, show a cause of action ripe
recurrence of conflict between for adjudication.
d.1 The Soriano and Tolentino Petitions
_______________ Soriano seeks to bar the JBC from continuing the selection processes on the
ground that the Supreme Court, not the President, appoints the Chief Justice.
26 CONSTITUTION, Article VIII, Section 5(5). Tolentino, on the other hand, seeks the issuance of a writ of prohibition under Rule
65 of the 1997 Rules of Court, among others, to enjoin and restrain the JBC from
submitting a list of nominees for judiciary positions to the incumbent President, _______________
on the ground that an existing election ban against appointments is in place under
Section 15, Article VII of the Constitution. 30 Civil Liberties Union v. Executive Secretary, G.R. No. 83896, February 21,
In the simplest terms, the JBC—by its own admission in its Comment and by 1991, 194 SCRA 317, 330.
Soriano’s28 and Tolentino’s29 own admissions in their petitions—is now in the 805
process of preparing its submission of nominees for the vacancy to be created by
VOL. 615, March 17, 2010 805
the retirement of the incumbent Chief Justice, and has already completed the
initial phases of this preparation. Soriano and Tolentino want to stop this process De Castro vs. Judicial and Bar Council (JBC)
and compel the JBC to immediately discontinue its activities, apparently on the
theory that nomination is part of the appointment process The third reality, closely related to the second, is that in resolving the
While their cited grounds and the intrinsic merits of these grounds vary, the coverage of the election ban vis-à-vis the appointment of the Chief Justice and the
Soriano and Tolentino petitions, on their faces, present actual justiciable Members of the Court, provisions of the Constitution other than the disputed
controversies that are ripe for adjudication. Section 15, Article VII of the provisions must be taken into account. In considering when and how to act, the
Constitution embodies a ban against appointments by the incumbent Presi- JBC has to consider that:
1. The President has a term of six years which begins at noon of
_______________ June 30 following the election, which implies that the outgoing President
remains President up to that time. (Section 4, Article VII). The President
28 Soriano petition, p. 4. assumes office at the beginning of his or her term, with provision for the
29 Tolentino petition, p. 2. situations where the President fails to qualify or is unavailable at the
804 beginning of his term (Section 7, Article VII).
2. The Senators and the Congressmen begin their respective terms
804 SUPREME COURT REPORTS ANNOTATED also at midday of June 30 (Sections 4 and 7, Article VI). The Congress
De Castro vs. Judicial and Bar Council (JBC) convenes on the 4th Monday of July for its regular session, but the
dent two months before the election up to the end of her term. A ruling from this President may call a special session at any time. (Section 15, Article VI)
Court (Valenzuela) is likewise in place confirming the validity of this ban against 3. The Valenzuela case cited as authority for the position that the
the Judiciary, or at least against the appointment of lower court judges. A vacancy election ban provision applies to the whole Judiciary, only decided the issue
in the position of Chief Justice will occur on May 17, 2010, within the period of the with respect to lower court judges, specifically, those covered by Section 9,
ban, and the JBC is admittedly preparing the submission of its list of nominees for Article VIII of the Constitution. Any reference to the filling up of vacancies
the position of Chief Justice to the President. Under the terms of Section 15, Article in the Supreme Court pursuant to Section 4(1), Article VIII
VII and the obtaining facts, a prima facie case exists supporting the petition for constitutes obiter dictum as this issue was not directly in issue and was not
violation of the election ban. ruled upon.
d.2. Supervision over the JBC. These provisions and interpretation of the Valenzuela ruling—when read
That the JBC—now under a different membership—needs guidance on the together with disputed provisions, related with one another, and considered with
course of action it should take on the constitutional issues posed, can best be the May 17, 2010 retirement of the current Chief Justice—bring into focus certain
understood when the realities behind the constitutional provisions are examined. unavoidable realities, as follows:
A first reality is that the JBC cannot, on its own due to lack of the proper 1. If the election ban would apply fully to the Supreme Court, the
authority, determine the appropriate course of action to take under the incumbent President cannot appoint a
Constitution. Its principal function is to recommend appointees to the Judiciary 806
and it has no authority to interpret constitutional provisions, even those affecting 806 SUPREME COURT REPORTS ANNOTATED
its principal function; the authority to undertake constitutional interpretation
belongs to the courts alone. De Castro vs. Judicial and Bar Council (JBC)
A second reality is that the disputed constitutional provisions do not stand Member of the Court beginning March 10, 2010, all the way up to June 30,
alone and cannot be read independently of one another; the Constitution and its 2010.
various provisions have to be read and interpreted as one seamless whole, 30 giving 2. The retirement of the incumbent Chief Justice—May 17, 2010—
sufficient emphasis to every aspect in accordance with the hierarchy of our falls within the period of the election ban. (In an extreme example where the
constitutional values. The disputed provisions should be read together and, as retirement of a Member of the Court falls on or very close to the day the
reflections of the will of the people, should be given effect to the extent that they election ban starts, the Office of the Solicitor General calculates in its
should be reconciled. Comment that the whole 90 days given to the President to make appointment
would be covered by the election ban.)
3. Beginning May 17, 2010, the Chief Justice position would be _______________
vacant, giving rise to the question of whether an Acting Chief Justice can
act in his place. While this is essentially a Supreme Court concern, the 31 Under Section 1, Rule 63 of the Rules of Court, a petition for declaratory
Chief Justice is the ex officio Chair of the JBC; hence it must be concerned relief is available only before breach or violation of the deed or instrument whose
and be properly guided. terms are sought to be clarified.
4. The appointment of the new Chief Justice has to be made within 90 808
days from the time the vacancy occurs, which translates to a deadline of
808 SUPREME COURT REPORTS ANNOTATED
August 15, 2010.
5. The deadline for the appointment is fixed (as it is not reckoned from De Castro vs. Judicial and Bar Council (JBC)
the date of submission of the JBC list, as in the lower courts) which means To summarize the preliminary considerations of locus standi and
that the JBC ideally will have to make its list available at the start of the justiciability and the outstanding issues for resolution, the main issue in
90-day period so that its process will not eat up the 90-day period granted these consolidated cases continues to be whether Section 15, Article VII of the
the President. Constitution limiting the authority of the President of the Philippines to exercise
6. After noon of June 30, 2010, the JBC representation from Congress her power of appointment shall prevail over the mandate, provided under Section
would be vacant; the current representatives’ mandates to act for their 4(1) and 9, Article VIII, that appointments to the Supreme Court shall be within
principals extend only to the end of their present terms; thus, the JBC shall 90 days from the occurrence of the vacancy, and within 90 days from the JBC’s
be operating at that point at less than its full membership. submission of its list of nominees for the lower courts. A sub-issue is the continued
7. Congress will not convene until the 4th Monday of July, 2010, but effectiveness and strength of the Valenzuela case as guide and precedent in
would still need to organize before the two Houses of Congress can send resolving the above issue. All these should be read in the context of the
their representatives to the JBC—a process may extend well into August petitions for prohibition and the Mendoza A.M. petition, as the De Castro
2010.807 and the PHILCONSA petitions suffer from lack of justiciability and
VOL. 615, March 17, 2010 807 prematurity.
III. The Merits of the Petitions
De Castro vs. Judicial and Bar Council (JBC) a. The Soriano Petition.
The Soriano petition presents a very novel interpretation of Section 9, Article
8. In July 2010, one regular member of the JBC would vacate his post. VIII in its position that the authority to appoint the Chief Justice is lodged in the
Filling up this vacancy requires a presidential appointment and the Court, not in the President.
concurrence of the Commission on Appointments. The correctness of this reading of the law is contradicted by both history and
9. Last but not the least, the prohibition in Section 15, Article VII is by the law itself.
that “a President or Acting President shall not make appointments.” This History tells us that, without exception, the Chief Justice of the Supreme Court
prohibition is expressly addressed to the President and covers the act of has always been appointed by the head of the Executive Department. Thus, Chief
appointment; the prohibition is not against the JBC in the performance of Justices Cayetano Arellano, Victorino Mapa, Manuel Araullo, Ramon Avancena,
its function of “recommending appointees to the Judiciary”—an act that is Jose Abad Santos, Jose Yulo, Manuel Moran and all the Chief Justices after
one step away from the act of making appointments. Philippine independence were appointed by the Chief Executive. The only
d.3. Conclusion on the Mendoza Petition difference in their respective appointments is the sovereignty under which they
Given the justiciable Soriano and Tolentino petitions that directly address the were appointed.809
JBC and its activities, the impact of the above-outlined realities on the grant of a
VOL. 615, March 17, 2010 809
writ of prohibition, and the undeniable supervision that the Supreme Court
exercises over the JBC as well as its role as the interpreter of the Constitution— De Castro vs. Judicial and Bar Council (JBC)
sufficiently compelling reason exists to recognize the Mendoza petition
as a properly filed A.M. petition that should fully be heard in these The Chief Justices under the American regime were appointed by the
proceedings to fully ventilate the supervisory aspect of the Court’s President of the United States; one Chief Justice each was appointed under the
relationship with the JBC and to reflect, once again, how this Court views Commonwealth and under the Japanese Military Administration; and thereafter
the issues first considered in Valenzuela. The Court’s supervision over the all the Chief Justices were appointed by the Philippine President. In every case,
JBC, the latter’s need for guidance, and the existence of an actual controversy that the appointing authority was the Chief Executive.
Soriano and Tolentino cite, save the Mendoza petition from being one for The use of the generic term “Members of the Supreme Court” under Section 9,
declaratory relief, which petition is originally cognizable by the Regional Trial Article VIII in delineating the appointing authority under the 1987 Constitution,
Court, not by this Court.31 is not new. This was the term used in the present line of Philippine Constitutions,
from 1935 to 1987, and the inclusion of the Chief Justice with the general term
“Member of the Court” has never been in doubt.32 In fact, Section 4(1) of the
present Constitution itself confirms that the Chief Justice is a Member of the Court 9; in the latter the JBC appears free to determine when a submission is to be made,
when it provides that the Court “may sit en banc or, in its discretion, in divisions obligating the President to issue appointments within 90 days from the submission
of three, five, or seven Members.” The Chief Justice is a Member of the En Banc and of the JBC list. From this view, the appointment period under Section 9 is one that
of the First Division—in fact, he is the Chair of the En Banc and of the First is flexible and can move.811
Division—but even as Chair is counted in the total membership of the En Banc or VOL. 615, March 17, 2010 811
the Division for all purposes, particularly of quorum. Thus, at the same time that
Section 4(1) speaks of a “Supreme Court. . . composed of one Chief Justice and De Castro vs. Judicial and Bar Council (JBC)
fourteen Associate Justices,” it likewise calls all of them Members in defining how
they will sit in the Court. Thus, in terms of conflict, Sections 4(1) and Sections 15 can be said to be
Thus, both by law and history, the Chief Justice has always been a Member of directly in conflict with each other, while a conflict is much less evident from a
the Court—although, as a primus inter pares—appointed by the President comparison of Sections 9 and 15. This conclusion answers the verba legis argument
together with every other Associate Justice. For this reason, we should dismiss the of the Peralta petition that when the words or terms of a statute or provision is
Soriano petition for lack of merit. clear and unambiguous, then no interpretation is necessary as the words or terms
b. The Tolentino and Mendoza Petitions; shall be understood in their ordinary meaning. In this case, the individual
the OSG and JBC Comments provisions, in themselves, are clear; the conflict surfaces when they operate in
This is only a Separate Opinion, not a ponencia, and rather than recite or tandem or against one another.
tabulate the various positions taken in these b.2. The Valenzuela Ruling.
The Valenzuela decision gives the full flavor of how the election ban issue arose
_______________ because of Chief Justice Narvasa’s very candid treatment of the facts and the
issue. Valenzuela openly stated that at the root of the dispute was the then existing
32 See: Vargas v. Rilloraza, 80 Phil. 297, 342 (1948). vacancy in the Court and the difference of opinion on the matter between the
Executive and the Court on the application of Section 15, Article VII, in relation
810
with Section 4(1) and 9 of Article VIII, of the Constitution.
810 SUPREME COURT REPORTS ANNOTATED What appears very clear from the decision, however, is that the factual
De Castro vs. Judicial and Bar Council (JBC) situation the Court ruled upon, in the exercise of its supervision of court personnel,
submissions, I shall instead discuss the issues based on topicallyarranged was the appointment by the President of two RTC judges during the period of the
subdivisions and introduce the various positions as arguments, for or against, ban. It is clear from the decision, too, that no immediate appointment was ever
without always naming the source. This is solely for ease of presentation, clarity made to the Court for the replacement of retired Justice Ricardo Francisco as the
and continuity rather than for any devious reason. JBC failed to meet on the required nominations prior to the onset of the election
b.1. Does a conflict of provisions textually exist? ban.
No need exists to further recite Section 15, Article VII, on the one hand, and From this perspective, it appears clear to me that Valenzuelashould be read
Sections 4(1) and 9, Article VIII, on the other, as they are already quoted at the and appreciated for what it is—a ruling made on the basis of the Court’s
start of this Opinion. I do not believe any of the parties, though, will dispute that supervision over judicial personnel that upholds the election ban as against the
a conflict exists even from the text of these provisions alone. appointment of lower court judges appointed pursuant to the period provided by
Section 15 on its face disallows any appointment in clear negative terms (shall Section 9 of Article VIII. Thus, Valenzuela’s application to the filling up of a
not make) without specifying the appointments covered by the prohibition. From vacancy in the
this literal reading springs the argument that no exception is provided (except the 812
exception found in Section 15 itself) so that even the Judiciary is covered by the 812 SUPREME COURT REPORTS ANNOTATED
ban on appointments.
De Castro vs. Judicial and Bar Council (JBC)
On the other hand, Section 4(1) is likewise very clear and categorical in its
terms: any vacancy in the Court shall be filledwithin 90 days from its occurrence. Supreme Court is a mere obiter dictum as the Court is largely governed by Section
In the way of Section 15, Section 4(1) is also clear and categorical and provides no 4(1) with respect to the period of appointment. The Section 4(1) period, of course
exception; the appointment refers solely to the Members of the Supreme Court and and as already mentioned above, has an impact uniquely its own that is different
does not mention any period that would interrupt, hold or postpone the 90-day from that created by the period provided for the lower court under Section 9.
requirement. I find it interesting that Peralta largely justifies his position that the JBC
Section 9 may offer more flexibility in its application as the mandate for the should now be prohibited from proceeding with the nomination process based
President is to issue appointments within 90 days from submission of the list, on Valenzuela as the prevailing rule that should be followed under the principle
without specifying when the submission should be made. From their wordings, of stare decisis. Peralta apparently misappreciates the reach and real holding
urgency leaps up from Section 4(1) while no such message emanates from Section of Valenzuela, as explained and clarified above. A ruling involving the
appointment of lower court judges under Section 9, Article VIII cannot simply be
bodily lifted and applied in toto to the appointment of Members of the Supreme 33 163 U.S. 537 (1896).
Court under Section 4(1) of the same Article. 34 347 U.S. 483 (1954).
Because of his misappreciation, Peralta is likewise mistaken in his appeal to 814
the principle of stare decisis. The stability of judgments is indeed a glue that the 814 SUPREME COURT REPORTS ANNOTATED
Judiciary and the litigating public cannot do without if we are to have a working
and stable justice system. Because of this role, the principle is one that binds all De Castro vs. Judicial and Bar Council (JBC)
courts, including this Court, and the litigating public. The principle, however, is other, instead of saying that one provision should absolutely prevail over the other?
not open-ended and contains its own self-limitations; it applies only to actions in What Valenzuela failed to consider, because it was looking at the disputed
all future similar cases and to none other. Where ample room for distinction exists, provisions from the prism of two RTC judges, is that the reasons for the application
as in this case, then stare decisis does not apply. of Section 15, Article VII may not at all exist in appointments to the Supreme
Another aspect of stare decisis that must be appreciated is that Supreme Court Court.
rulings are not written in stone so that they will remain unerased and applicable In the first place, Section 4(1) covers only the appointment of 15 Members, not
for all times. The Supreme Court’s review of rulings and their binding effects is a in their totality, but singly and individually as Members disappear from the Court
continuing one so that a ruling in one era may be declared by the Court at some and are replaced. Thus, the evil that the Aytona case35 sought to remove—mass
future time to be no longer true and should thus be abandoned and changed. The midnight appointments—will not be present.
best and most unforgettable example of this kind of change happened in the United Secondly, partisanship is hardly a reason that would apply to the Supreme
States when the US Supreme Court overturned the Court except when the Members of the Court individually act in violation of their
813 oaths or directly transgress our graft and corruption laws. Let it be remembered
that the Constitution itself has entrusted to the Court the final and definitive
VOL. 615, March 17, 2010 813
recourse in election contest involving the President, the Vice President and
De Castro vs. Judicial and Bar Council (JBC) Members of Congress. Because of this reposed trust on the Supreme Court as a
ruling in Plessy v. Fergusson33 that upheld the constitutionality of racial body, reasons of partisanship can hardly be a reason to systemically place the
segregation under the “separate but equal” doctrine. After half a century, the US whole Supreme Court under a ban on appointments during the election period.
Court completely abandoned this ruling in the now famous Brown v. Board of Of course, partisanship is an objection that can apply to individual Members
Education when it ruled that “separate but equal” doctrine is inherently unequal of the Court and even to the applicants for the position of Chief Justice. But this is
in the context of public education.34 I mention this, if only as a reminder to one and a different question that should not result in placing the system of appointments
all, that the terms of the Valenzuela ruling, if truly applicable even to to the Court within the coverage of the election ban; objections personal to
appointments to this Court, is not written in stone and remains open for review by individual Members and to individual applicants are matters addressed to the
this Court. JBC and to the final appointing authority—the President. It is for reasons of these
Valenzuela rests on the reasoning that the evil that Section 15 seeks to possible individual objections that the JBC and even the Office of the President
remedy—vote buying, midnight appointments and partisan reasons to influence are open to comments and objections.
the results of the election—is so pervasive so that the Section 15 ban should prevail
over everything else. The Court, however, forgot in some statements in this case _______________
that hand in hand with Section 15 is Section 4(1) where the framers also
recognized, in clear and absolute terms, that a vacancy in the Court should be filled 35 Aytona v. Castillo, No. L-19313, January 19, 1962, 4 SCRA 1.
up because of the importance of having a Supreme Court with its full and complete 815
membership. Completeness has a heightened meaning when the missing Member
VOL. 615, March 17, 2010 815
is the head of the Judiciary and the Court in the person of the Chief Justice.
The separate realities that Section 15, Article VII and Section 4(1) bring to the De Castro vs. Judicial and Bar Council (JBC)
fore now confront us with the question of prioritizing our constitutional values in
terms of two provisions that effectively operate in their separate spheres, but Incidentally, the incumbent President is not up for re-election by operation of
which conflict when they directly confront one another. The direct question is: the Constitution so that a partisanship objection in the President’s favor has no
should we really implement Section 15 above everything else, even at the expense basis. If any, an objection personal to the Supreme Court applicant may be raised
of having an incomplete Supreme Court, or should we recognize that both because of perceived bias or partisanship in favor of the President’s choice in the
provisions should be allowed to operate within their own separate spheres with elections. This would be a meaningless objection, however, if it is considered that
one provision being an exception to the the same objection can be raised against a Supreme Court nominee appointed by
the incoming President; this new appointee will sit in judgment in the electoral
_______________ dispute that follows the presidential elections and can be chosen for bias towards
the new President and his party. In this sense, an objection on the basis of personal
bias is not at all an appropriate consideration when the issue is systemic in its
application—the application of the election ban on appointments to Supreme Court 36 400 Phil. 940; 347 SCRA 128 (2000).
appointments. 817
In any case, the comments made on this point in the petitions are conjectural VOL. 615, March 17, 2010 817
and speculative and can hardly be the bases for adjudication on the merits. If
records of the Court will matter, the duly proven facts on record about the De Castro vs. Judicial and Bar Council (JBC)
immediately past Chief Justices speak for themselves with respect to partisanship word less for the Court. One voice can be a big difference if the missing voice is
in favor of the sitting President. It is a matter of public record that Chief Justices that of the Chief Justice.
Davide, Panganiban and Puno did not try to please their respective incumbent Without meaning to demean the capability of an Acting Chief Justice, the
Presidents, and instead ruled in the way that the law, jurisprudence and the ascendancy in the Court of a permanent sitting Chief Justice cannot be equaled.
requirements of public interests dictated. He is the first among equals—a primus inter pares—who sets the tone for the
The Mendoza petition presents some very compelling reasons why the Court and the Judiciary, and who is looked up to on all matters, whether
Supreme Court, if not the whole Judiciary, should be exempt from the coverage of administrative or judicial. To the world outside the Judiciary, he is the
the election ban that Section 15, Article VII imposes. personification of the Court and the whole Judiciary. And this is not surprising
The Chief Justice is the head of the Judiciary in the same manner that the since, as Chief Justice, he not only chairs the Court en banc, but chairs as well the
President is the Chief Executive and the Senate President and the Speaker of the Presidential Electoral Tribunal that sits in judgment over election disputes
House head the two Houses of Congress. The Constitution ensures, through clear affecting the President and the Vice-President. Outside of his immediate Court
and precise provisions, that continuity will prevail in every branch by defining how duties, he sits as Chair of the Judicial and Bar Council, the Philippine Judicial
replacement and turnover of power Academy and, by constitutional command, presides over the impeachment of the
816 President.37 To be sure, the Acting Chief Justice may be the ablest, but he is not
the Chief Justice without the mantle and permanent title of the Office, and even
816 SUPREME COURT REPORTS ANNOTATED
his presence as Acting Chief Justice leaves the Court with one member less. Sadly,
De Castro vs. Judicial and Bar Council (JBC) this member is the Chief Justice; even with an Acting Chief Justice, the Judiciary
shall take place. Thus, after every election to be held in May, a turn over of power and the Court remain headless.
is mandated on the following 30th of June for all elective officials. The intent of the framers of the Constitution to extend to the Court a fixed
For the Supreme Court where continuity is by the appointment of a period that will assure the nation that the Court’s membership shall immediately
replacement, the Constitution requires that the replacement Member of the Court, be filled, is evidenced no less than by the Constitutional Commission’s own
including the Chief Justice, should be appointed within 90 days from the deliberations where the following exchange took place:
occurrence of the vacancy. This is the sense of urgency that the Constitution Mr. De Castro: I understand that our justices now in the Supreme Court,
imparts and is far different from the appointment of the justices and judges of the together with the Chief Justice, are only 11.
lower courts where the requirement is 90 days from the JBC’s submission of its Mr. Concepcion: Yes.
list. This constitutional arrangement is what the application of Section 15, Article
VII to the appointment of Members of the Supreme Court will displace. _______________
The Peralta petition argues that the appointment of a Chief Justice is not all
that important because the law anyway provides for an Acting Chief Justice. While 37 Constitution, Article XI, Section 2(6).
this is arguably true, Peralta misunderstands the true worth of a duly appointed 818
Chief Justice. He forgets, too, that a Supreme Court without a Chief Justice in
818 SUPREME COURT REPORTS ANNOTATED
place is not a whole Supreme Court; it will be a Court with only 14 members who
would act and vote on all critical matters before it. De Castro vs. Judicial and Bar Council (JBC)
The importance of the presence of one Member of the Court can and should Mr. De Castro: And the second sentence of this subsection reads: Any
never be underestimated, particularly on issues that may gravely affect the nation. vacancy shall be filled within ninety days from the occurrence
Many a case has been won or lost on the basis of one vote. On an issue of the thereof.”
constitutionality of a law, treaty or statute, a tie vote—which is possible in a 14 Mr. Concepcion: That is right.
member court—means that the constitutionality is upheld. This was our lesson Mr. De Castro: Is this a now a mandate to the executive to fill the
in Isagani Cruz v. DENR Secretary.36 vacancy.
More than the vote, Court deliberation is the core of the decision-making Mr. Concepcion: That is right. That is borne out of the fact that in the past
process and one voice less is not only a vote less but a contributed opinion, an 30 years, seldom has the Court had a complete complement.
observation, or a cautionary This exchange, to my mind, removes any remaining doubt about the framers’
recognition of the need to always have a full Court.
_______________ b.3. Construction of the Disputed Provisions
A notable aspect of the Valenzuela ruling in the context of constitutional _______________
interpretation, is its conclusion that in a conflict between two provisions—one in
the Article on the Executive Department and the other an Article in the Judicial 39 Mendoza petition, p. 3.
Department—one of them should completely give way and the other should 820
prevail. This is a very unusual approach in interpretation, particularly if the 820 SUPREME COURT REPORTS ANNOTATED
apparently conflicting provisions are from the Constitution—an instrument that
has painstakingly been deliberated upon by the best and the brightest minds in De Castro vs. Judicial and Bar Council (JBC)
the country. For, the rule in constitutional interpretation is that the constitution and intent of the Constitution and the public policy reasons behind them simply
must be appreciated and interpreted as one single instrument, with apparently speak against the interpretation that appointments of Members of the Court
conflicting provisions reconciled and harmonized in a manner that will give all of should be subject to the election ban. These are all discussed above and need not
them full force and effect.38 be repeated here.
Where, as in Valenzuela, the Chief Justice of the Supreme Court, no less, Principles of constitutional interpretation, too, militate against an
appeared to have given up the benefit of an immediate appointment of Members interpretation that would give primacy to one branch of government over another
of the Supreme Court, then extremely compelling reasons must have driven the in the absence of very compelling reasons. Each branch of government is in place
for a particular reason and each one should be given every opportunity to operate
_______________ to its fullest capacity and potential, again unless very compelling reasons exist for
the primacy of one over the other. No such compelling reason so far exists or has
38 See: Marcelino v. Cruz, No. L-42428, March 14, 1983, 121 SCRA 51. been cited.
819 Based on the values that the disputed provisions embody, what we need to
balance are the integrity of our electoral process and the protection needed to
VOL. 615, March 17, 2010 819 achieve this goal, as against the Judiciary’s need for independence and strength
De Castro vs. Judicial and Bar Council (JBC) enforced through a Supreme Court that is at its full strength. To be sure, the nation
Court to its conclusion. I fully understood though the former Chief Justice’s and our democracy need one as well as the other, for ultimately both contribute to
conclusion in this case when I realized that he was not effectively ruling on Section our overall national strength, resiliency, and stability. Thus, we must, to the extent
4(1) of Article VIII, and was in fact ruling on a case involving lower court judges. possible, give force and effect to both and avoid sacrificing one for the other.
For indeed, the reasons the former Chief Justice cited in Valenzuela justify the To do this and to achieve the policy of insulating our constitutional process
application of the Section 15, Article VII as against the rule on appointment of from the evils of vote-buying, influence peddling and other practices that affect the
lower court judges under Section 9, Article VIII. As I have shown above, Section 9 integrity of our elections, while at the same time recognizing the Judiciary’s and
does not impose a hard and fast rule on the period to be observed, apparently the nation’s need to have a full Supreme Court immediately after a vacancy occurs,
because the urgency of the appointment may not be as great as in the appointment Section 4(1) of Article VIII should be recognized as a narrow exception granted to
of Members of the Supreme Court. The period for appointment can move at the the Judiciary in recognition of its proven needs. This is a narrow exception as the
discretion of the JBC, although the exercise of this discretion also carries its own election ban of Section 15, Article VII, shall apply with full force and effect on the
butt-in and implicit limits. appointment of lower court justices and judges.821
The former Chief Justice’s weightier reason arose from the Aytona decision VOL. 615, March 17, 2010 821
where mass appointments were recognized as an evil that could affect the integrity
of our elections. Because of the number of appointments that may currently be De Castro vs. Judicial and Bar Council (JBC)
involved if appointments to lower courts are allowed before the May 2010 election
(around 537 vacancies at a 24.5% vacancy rate at the first and second level courts c. Guidelines for the Judicial and Bar Council
according to the figures of the Mendoza petition)39 and the power and influence The resolution of the present dispute can only be complete if clear guidelines
judges may exert over their local communities, an exemption from the election ban are given to the JBC on how it shall conduct itself under the present circumstances
may indeed bring about (or at least give the appearance of bringing about) the evils pursuant to this Court’s ruling. The Court should therefore direct the JBC to:
that the framers of the Constitution and this Court itself sought to remedy under A. forthwith proceed with its normal processes for the submission of
Section 15, Article VII and the Aytona decision, respectively. the list of nominees for the vacancy to be created by the retirement of Chief
For this reason, I do not disagree with Valenzuela for its ruling on lower court Justice Reynato S. Puno, to be submitted to the President on or before the
judges; Section 15, Article VII may indeed prevail over Section 9, Article VIII. day before the retirement of the Chief Justice;
In contrast with this conclusion, an interpretation that Section 15, Article VII B. in the course of preparing its list of nominees, determine with
will similarly prevail over Section 4(1), Article VIII is clearly misplaced. The certainty the nominees’ readiness to accept the nomination as well as the
structure, arrangement appointment they may receive from the President, deleting from the list the
nominees who will refuse to confirm their full readiness to accept without
conditions either their nomination or their appointment, if they will be De Castro vs. Judicial and Bar Council (JBC)
appointed; Ordinarily, the JBC would already be holding public interviews of
C. proceed with its normal processes for the preparation of the lists candidates to the office to be followed by a deliberation and the eventual
for the vacancies for the lower courts, to be submitted to the Office of the submission of a shortlist of nominees to the President. The Constitution provides
President as soon as the election ban on appointments is lifted; and that any vacancy in the Supreme Court “shall be filled within ninety days” from
D. in all other matters not otherwise falling under the above, conduct its occurrence.3 Since the position of Chief Justice will be vacant on May 17, 2010
itself in accordance with this Decision. when Chief Justice Puno shall have retired, the President has to fill up the vacancy
In light of all the foregoing, I vote to: during the period May 17 to August 15, 2010.
1. Dismiss the De Castro and Peralta petitions and for not being But by some unforeseen happenstance, that vacancy (May 18) will occur during
justiciability and for prematurity. the period of the midnight appointments ban (March 10 to June 30), a ban intended
2. Dismiss the Soriano and the Tolentino petitions for lack of merit. to prevent an outgoing president from buying votes using such appointments or
3. Dismiss all petitions and motions for interventions supporting or opposing robbing the incoming president of the opportunity to fill up important positions
the above petitions.822 with people he will be working with. Article VII, Section 15, of the Constitution
822 SUPREME COURT REPORTS ANNOTATED prohibits the outgoing President from making appointments “two months
De Castro vs. Judicial and Bar Council (JBC) immediately before the next presidential elections and up to the end of his term,”
except temporary appointments in the interest of public service or public
4. Grant the Mendoza petition and declare for the JBC’s guidance that: safety.4 The midnight appointments ban this year is in force from March 10 (two
a. Section 4(1), Article VIII is an exception to the coverage of Section months before the elections) to June 30 (the end of the incumbent President’s
term), a period of 112 days.
15, Article VII; appointments to the Supreme Court are not subject to the
election ban under Section 15, Article VII so that the JBC can submit its
list of nominees for the expected vacancy for the retirement of Chief Justice _______________
Reynato S. Puno, on or before the vacancy occurs, for the President’s
consideration and action pursuant to Section 4(1), Article VIII; 3 Article VIII, Section 4(1). The Supreme Court shall be composed of a Chief
b. Reiterate our ruling in In re: Valenzuela and Vallartathat no other Justice and fourteen Associate Justices. It may sit en banc or, in its discretion, in
appointments of judges of the lower courts can be made within the election divisions of three, five, or seven Members. Any vacancy shall be filled within
ban period, pursuant to Section 15, Article VII. ninety days from the occurrence thereof.
4 Article VII, Sec. 15. Two months immediately before the next presidential
CONCURRING OPINION elections and up to the end of his term, a President or Acting President shall not
make appointments, except temporary appointments to executive positions when
ABAD, J.: continued vacancies therein will prejudice public service or endanger public safety.
Chief Justice Reynato S. Puno will retire on May 17, 2010. Article VIII, Section 824
91of the 1987 Constitution requires the President to choose his successor from at 824 SUPREME COURT REPORTS ANNOTATED
least three nominees of the Judicial and Bar Council (JBC). On January 18, 2010 De Castro vs. Judicial and Bar Council (JBC)
the JBC passed a unanimous resolution2 to start the process of filling up the
anticipated vacancy. Indeed, it invited applications and nominations for the Issues to be addressed
position through newspapers, later announced the names of candidates to it, and Quite ably, the majority opinion already addressed the several issues raised by
finally received endorsements in favor of and oppositions against such candidates. the petitions and the oppositions to them. I join that opinion and would add a few
thoughts on what I believe to be the key issues in this case, namely:
_______________ 1. Whether or not the case presents an actual controversy that is ripe for this
Court’s adjudication; and
1 Article VIII, Sec. 9. The members of the Supreme Court and judges of lower 2. Whether or not the Constitutional ban on midnight appointments applies
courts shall be appointed by the President from a list of at least three nominees to the judiciary.
prepared by the Judicial and Bar Council for every vacancy. Such appointments
need no confirmation. For the lower courts, the President shall issue the Discussion
appointments within ninety days from the submission of the list.
2 http://jbc.judiciary.gov.ph/announcements/JBCreCJ.pdf. One. Invoking the fundamental rule that judicial power is the duty of the
823 courts of justice to settle “actual controversies involving rights which are legally
VOL. 615, March 17, 2010 823 demandable and enforceable,” the National Union of People’s Lawyers (NUPL)
claims that no actual controversy exists in this case as to warrant judicial 6 Id., Section 5.
determination of the issue of whether or not the Constitutional ban on midnight 7 Id., Section 9 in relation to Section 4(1).
appointment applies to the judiciary since the JBC has not as yet prepared a final 826
list of its nominees to current vacancies in the courts. BAYAN, COURAGE, 826 SUPREME COURT REPORTS ANNOTATED
KADAMAY, LFS, NUSTP, CEGP, SCMP, and BAYAN claim that what the
petitioners seek is a mere advisory opinion from the Court, something that it has De Castro vs. Judicial and Bar Council (JBC)
no power to give. that he has ordered 20 pigs to raise there, I will not wait till the pigs arrive and
The Constitution provides that judicial power is the duty of the courts of justice defecate before I bring an action to abate a nuisance.
to settle actual controversies involving rights which are legally demandable and As mandated by the Constitution, the incumbent President should be able to
enforceable.5 The court will not act on an action for damages for a slap on the fill up the vacancy within 90 days of its occurrence. This presupposes that the
plaintiff’s face if the defendant is still to deliver that slap. The law must have incumbent President should have the list on or before May 17, the day the vacancy
established a right which has in fact been violated. occurs, so she can comply with her duty under the Constitution to make the
Here, the Constitution imposes on the JBC the duty to recommend to the appointment within the 90-day period provided by it. Of course, the circumstances
President those whom he can appoint to the is such that the period for appointing the Chief Justice’s replacement will span the
tenure of the incumbent President (for 44 days) and her successor (for 46 days),
_______________ but it is the incumbent’s call whether to exercise the power or pass it on.
Again, assuming as correct petitioners’ view that the ban on midnight
5 Article VIII, Section 1, 1987 Constitution of the Philippines. appointments does not apply to the judiciary, the JBC’s suspension of its selection
process places it in default, given its above duty in regard to the submission of its
825
list of nominees to the President within a time constraint. Under the same
VOL. 615, March 17, 2010 825 assumption, moreover, the petitioner citizens and members of the bar would have
De Castro vs. Judicial and Bar Council (JBC) a demandable right or interest in having the JBC proceed with its selection process
judiciary when a vacancy occurs.6 In the case of a vacancy in the Supreme Court, and submit its list of nominees in time for the incumbent President or her
it is implicit that the JBC must submit a list of at least three nominees to the successor to fill up the vacancy within the period required by the Constitution.
President on time to enable him to fulfill his duty to fill up the vacancy within 90 Alternatively, assuming that an actual controversy has not yet developed as to
days after it occurs.7 Those who have an interest in the fulfillment of this duty has warrant action on the petitions filed in this case, the Court has the authority, as
the right to insist that it be done. an incident of its power of supervision over the JBC,8 to see to it that the JBC
But the JBC appears reluctant or unwilling to perform its above duty in the faithfully executes its duties as the Constitution requires of it.
case of the forthcoming May 17, 2010 vacancy in the office of the Chief Justice. It In its Resolution of January 18, 2010, the JBC confesses uncertainty regarding
expressed a desire to determine, initially, from views submitted to it by others and, when and to whom to submit its list of nominees for the May 17, 2010 vacancy in
later, from what the Court might provide it by way of guidance, whether it can the office of Chief
submit its list of nominees to the incumbent President during the ban on midnight
appointments that sets in on March 10. Indeed, the JBC said in its resolution of _______________
January 18, 2010 that, while it would start the selection process, it was yet to
determine when and to whom to submit its shortlist of nominees. It saw an 8 Id., Section 8(1).
apparent conflict between the provisions of Section 4(1) of Article VIII (the ban on 827
midnight appointments) and Section 15 of Article VII (the need to fill up the VOL. 615, March 17, 2010 827
vacancy within 90 days of its occurrence) of the 1987 Constitution.
Eventually, after taking some steps in the selection process, the JBC held the De Castro vs. Judicial and Bar Council (JBC)
process in abeyance, unable to decide as yet when and to whom it will submit its Justice in view of the apparently conflicting provisions of the Constitution.
list of nominees for the position that Chief Justice Puno will vacate on May 17, Further, in its comment in this case, the JBC declared that it “will be guided by
2010. Under the circumstances, the controversy is already ripe for adjudication for, [the Court’s] decision in these consolidated Petitions and Administrative Matter.”
assuming that the ban on midnight appointment does not apply to the judiciary as Consequently, as an incident of its Constitutional duty to supervise the JBC, the
the petitioners would have it, then the JBC’s suspension of its selection process Court can, to insure JBC’s faithful compliance with the Constitution, resolve the
would constitute a violation of its duty under the Constitution to carry on with issue of whether or not the ban on midnight appointments applies to the judiciary.
such process until it is able to submit the desired list to the incumbent President. Two. Citing “In Re: Appointments dated March 30, 1998 of Hon. Mateo A.
If my subdivision neighbor begins constructing a shed in his yard and tells me Valenzuela and Hon. Placido B. Vallarta as Judges of the Regional Trial Court of
Branch 62, Bago City and of Branch 24, Cabanatuan City,”9 the oppositors claim
_______________ that the ban on midnight appointments applies to the judiciary. After examining
the reasons for the two apparently conflicting provisions, the Court said that the
need to fill up vacancies in the judiciary within the period the Constitution _______________
provides must yield to the ban on Presidential midnight appointments. The Court
explained this ruling: 11 4 SCRA 1, 8 (1962).
Considering the respective reasons for the time frames for filling 12 Section 7(1) and (3), Article VIII, 1987 Constitution of the Philippines.
vacancies in the courts and the restriction on the President’s power of 829
appointment, it is this Court’s view that, as a general proposition, in case
VOL. 615, March 17, 2010 829
of conflict, the former should yield to the latter. Surely, the prevention of
vote-buying and similar evils outweighs the need for avoiding delays in De Castro vs. Judicial and Bar Council (JBC)
filling up of court vacancies or the disposition of some cases. Temporary dates, such as the pendency of administrative or criminal cases against them.
vacancies can abide the period of the ban which, incidentally and as Third. The JBC announces any vacancy in the judiciary in newspapers of large
earlier pointed out, comes to exist only once in every six years. Moreover, circulations. Secret recruitment and trading for votes in the coming elections is
those occurring in the lower courts can be filled temporarily by out.
designation. But prohibited appointments are long-lasting and Fourth. Anyone who has the basic qualifications can apply for a vacancy or be
permanent in their effects. They may, as earlier pointed out, in fact nominated to it. Thus, the opportunity to be recommended by the JBC for
influence the results of elections and, for that reason, their making is appointment is open or otherwise unrestricted. Political connection is not a
considered an election offense.10 consideration that the JBC entertains in short listing its nominees.
Fifth. The JBC invites the public to comment on or submit opposition to the
_______________ nomination of candidates to a vacancy. And it holds public hearings in which each
candidate is queried about his qualifications, affiliations, and other personal
9 358 Phil. 896; 298 SCRA 408 (1998). circumstances.
10 Id., at pp. 915-916; p. 426. Sixth. The names in the list submitted by the JBC to the President are not
828 negotiable. On July 24, 2009 the Executive Secretary returned to JBC its list of six
nominees for two vacancies in the Court, requesting additional names that the
828 SUPREME COURT REPORTS ANNOTATED incumbent President can choose from. Obviously, the President was unhappy with
De Castro vs. Judicial and Bar Council (JBC) the names on the list. But the JBC declined the request, the pertinent portion of
But the above assumes that the outgoing incumbent President can make which reads:
appointments in the judiciary during the period of the ban “to buy votes” and “We wish to inform you that the six (6) nominees of the JBC were
commit “similar evils” like denying the incoming President the opportunity to chosen after a long and thorough selection process. Among others, their
consider other appointees in the light of his new policies, a point former President public and private track record, experience and possession of the
Diosdado Macapagal made in Aytona v. Castillo.11 required qualities of competence, integrity, probity and independence
The fact, however, is that while the President can freely choose to appoint any were carefully studies and considered by the JBC. They are all highly
person who meets the basic qualifications for a position in the Executive qualified for the two (2) vacancies in the Supreme Court and indeed, your
Department, he does not have such freedom of choice when it comes to letter of July 26, 2009 does not assail and hence, concedes the
appointments in the judiciary. In the latter case, the Constitution provides in qualification of the six (6) nominees.
Section 9 of Article VIII that the President can choose his appointee only from a With due respect, the JBC cannot acquiesce to your request to expand
JBC short list of its nominees. the short list of nominees submitted to your office. The decision whether
Sec. 9. The Members of the Supreme Court and judges of lower to include three or more than three name in the short list of the nominees
courts shall be appointed by the President from a list of at least three exclusively belongs to the JBC. It is one of the important innovations in
nominees prepared by the Judicial and Bar Council for every vacancy. x the
xx 830
This restriction on the President’s appointing power is not a small matter. 830 SUPREME COURT REPORTS ANNOTATED
First. The JBC from whose list of nominees the President will make his
appointment is under the supervision of the Supreme Court itself. Indeed, it is De Castro vs. Judicial and Bar Council (JBC)
headed by the Chief Justice as its presiding officer. The JBC is not a subordinate 1987 Constitution designed to depoliticize appointments in the Judiciary
agency of the Executive Department; the President has neither control nor and promote its independence. This discretion given to the JBC is the
supervision over it. lynchpin of its autonomy and it cannot be compromised in the tiniest
Second. The JBC makes its own vetting rules and procedures. The Constitution degree without impairing the delicate check and balance in the
of course provides for the qualifications of members of the judiciary 12 but this has appointment of members of the Judiciary installed in our Constitution.
not prevented the JBC from establishing grounds for disqualifying candi- The JBC, voting unanimously, cannot therefore accede to your request in
light of the imperatives of the Constitution.”
Thus, the incumbent President was forced to choose from the few names on the
list that she had.
In reality, a President’s choice of Chief Justice is in fact first a choice of the
JBC before it is that of the President. Easily there should at least be 20,000 G.R. No. 191002. April 20, 2010.*
lawyers who are 40 years of age and have 15 years of law practice of some kind ARTURO M. DE CASTRO, petitioner, vs. JUDICIAL AND BAR COUNCIL (JBC)
who could qualify for Chief Justice. Yet, the President can choose only from a list and PRESIDENT GLORIA MACAPAGAL-ARROYO, respondents.
of three, four, or five lawyers that the JBC draws up for him. Consequently, the G.R. No. 191032. April 20, 2010.*
idea that the outgoing incumbent President can take advantage of her JAIME N. SORIANO, petitioner, vs. JUDICIAL AND BAR COUNCIL (JBC),
appointment of a Chief Justice to buy votes in the coming elections is utterly respondent.
ridiculous. She has no control over the JBC’s actions. G.R. No. 191057. April 20, 2010.*
Further, the idea that the incoming President should have the opportunity to PHILIPPINE CONSTITUTION ASSOCIATION (PHILCONSA), petitioner, vs.
choose a Chief Justice who will support his policies does not also make sense. The JUDICIAL AND BAR COUNCIL (JBC), respondent.
Supreme Court that the Chief Justice heads is not a support agency under the A.M. No. 10-2-5-SC. April 20, 2010.*
President. One of the functions of the Supreme Court is to provide a Constitutional IN RE APPLICABILITY OF SECTION 15, ARTICLE VII OF THE
check on abuses of the Executive Department. CONSTITUTION TO APPOINTMENTS TO THE JUDICIARY, ESTELITO P.
The proposition that a Chief Justice will always be beholden to the President MENDOZA, petitioner,
who appoints him is a myth. Former President Estrada appointed Chief Justice G.R. No. 191149. April 20, 2010.*
Hilario G. Davide, Jr. who presided over his impeachment and administered the JOHN G. PERALTA, petitioner, vs. JUDICIAL AND BAR COUNCIL (JBC),
oath to the incumbent President at the heels of EDSA II while President Estrada respondent.
still sat in Malacañang. Chief Justices Artemio V. Panganiban and Reynato S. PETER IRVING CORVERA;
Puno voted against positions taken by the administration of the incumbent CHRISTIAN ROBERT S. LIM;
President who appointed them both to their position. These Chief ALFONSO V. TAN, JR.;
831 NATIONAL UNION OF PEOPLE’S LAWYERS;
MARLOU B. UBANO;
VOL. 615, March 17, 2010 831
De Castro vs. Judicial and Bar Council (JBC) _______________
Justices like those before them were first choices of the JBC before they were those
of the Presidents concerned. * EN BANC.
I thus reiterate my concurrence with the main decision. 640
Petitions for certiorari, prohibition and mandamus dismissed, while petition in
AM No. 10-2-5-SC granted. 640 SUPREME COURT REPORTS ANNOTATED
Notes.—Not all “midnight” appointments are invalid—each appointment De Castro vs. Judicial and Bar Council (JBC)
must be judged on the basis of the nature, character, and merits of the individual INTEGRATED BAR OF THE PHILIPPINES-DAVAO DEL SUR CHAPTER,
appointment and the circumstances surrounding the same. It is only when the represented by its Immediate Past President, ATTY. ISRAELITO P. TORREON,
appointments were made en masse by the outgoing administration and shown to and the latter in his own personal capacity as a MEMBER of the PHILIPPINE
have been made through hurried maneuvers and under circumstances departing BAR;
from good faith, morality, and propriety that the Supreme Court has struck down MITCHELL JOHN L. BOISER;
“midnight” appointments. (Sales vs. Carreon, 515 SCRA 597 [2007]) BAGONG ALYANSANG BAYAN (BAYAN) CHAIRMAN DR. CAROLINA P.
The constitutional prohibition on so-called midnight appointments, ARAULLO; BAYAN SECRETARY GENERAL RENATO M. REYES, JR.;
specifically, those made within two (2) months immediately prior to the next CONFEDERATION FOR UNITY, RECOGNITION AND ADVANCEMENT OF
presidential elections, applies only to the President or Acting President. (Quirog GOVERNMENT EMPLOYEES (COURAGE) CHAIRMAN FERDINAND GAITE;
vs. Aumentado, 570 SCRA 582 [2008]) KALIPUNAN NG DAMAYANG MAHIHIRAP (KADAMAY) SECRETARY
Appointments are banned prior to the elections to ensure that partisan GENERAL GLORIA ARELLANO; ALYANSA NG NAGKAKAISANG KABATAAN
loyalties will not be a factor in the appointment process, and to prevent incumbents NG SAMBAYANAN PARA SA KAUNLARAN (ANAKBAYAN) CHAIRMAN KEN
from gaining any undue advantage during the elections. (Nazareno vs. City of LEONARD RAMOS; TAYO ANG PAG-ASA CONVENOR ALVIN PETERS;
Dumaguete, 602 SCRA 578 [2009]) LEAGUE OF FILIPINO STUDENTS (LFS) CHAIRMAN JAMES MARK TERRY
——o0o—— LACUANAN RIDON; NATIONAL UNION OF STUDENTS OF THE
© Copyright 2018 Central Book Supply, Inc. All rights reserved. PHILIPPINES (NUSP) CHAIRMAN EINSTEIN RECEDES; COLLEGE
EDITORS GUILD OF THE PHILIPPINES (CEGP) CHAIRMAN VIJAE
ALQUISOLA; and STUDENT CHRISTIAN MOVEMENT OF THE PHILIPPINES rigidly followed. A judicial pronouncement in an earlier decision may be followed
(SCMP) CHAIRMAN MA. CRISTINA ANGELA GUEVARRA; as a precedent in a subsequent case only when its reasoning and justification are
WALDEN F. BELLO and LORETTA ANN P. ROSALES; WOMEN TRIAL relevant, and the court in the latter case accepts such reasoning and justification
LAWYERS ORGANIZATION OF THE PHILIPPINES, represented by YOLANDA to be applicable to the case. The application of the precedent is for the sake of
QUISUMBING-JAVELLANA; BELLEZA ALOJADO DEMAISIP; TERESITA convenience and stability.
GANDIONCO-OLEDAN; MA. VERENA KASILAG-VILLANUEVA; MARILYN CARPIO-MORALES, J., Dissenting Opinion:
STA. ROMANA; LEONILA DE JESUS; and GUINEVERE DE LEON; AQUILINO Civil Procedure; Judgments; Stare Decisis; View that in reversing Valenzuela,
Q. PIMENTEL, JR.; Intervenors. the Decision held that the Valenzuela dictum did not firmly rest on Concom
641 deliberation, yet it did not offer to cite a material Concom deliberation—it instead
VOL. 618, APRIL 20, 2010 641 opted to rely on the memory of Justice Florenz Regalado which incidentally
mentioned only the “Court of Appeals.”—In reversing Valenzuela, the Decision held
De Castro vs. Judicial and Bar Council (JBC) that the Valenzuela dictum did not firmly rest on ConCom deliberations, yet it did
G.R. No. 191342. April 20, 2010.* not offer to cite a material ConCom deliberation. It instead opted to rely on
ATTY. AMADOR Z. TOLENTINO, JR., (IBP Governor-Southern Luzon), and the memory of Justice Florenz Regalado which incidentally mentioned only the
ATTY. ROLAND B. INTING (IBP Governor-Eastern Visayas), petitioners, vs. “Court of Appeals.” The Decision’s conclusion must rest on the strength of its own
JUDICIAL AND BAR COUNCIL (JBC), respondent. favorable Concom deliberation, none of which to date has been cited.
G.R. No. 191420. April 20, 2010.* Constitutional Law; Presidency; Power to Appoint; Appointments; View that
PHILIPPINE BAR ASSOCIATION, INC., petitioner, vs. JUDICIAL AND BAR because of the temporary nature of the circumstance causing the impossibility of
COUNCIL and HER EXCELLENCY GLORIA MACAPAGAL-ARROYO, performance, the outgoing President is released from non-fulfillment of the
respondents. obligation to appoint, and the duty devolved upon the new President.—In view of
Civil Procedure; Judgments; Stare Decisis; Words and Phrases; Stare decisis the temporary nature of the circumstance causing the impossibility of
means that a principle underlying the decision in one case is deemed of imperative performance, the outgoing President is released from non-fulfillment of the
authority, controlling the decisions of like cases in the same court and in lower obligation to appoint, and the duty devolves upon the new President. The delay in
courts within the same jurisdiction, unless and until the decision in question is the fulfillment of the obligation becomes excusable, since the law cannot exact
reversed or overruled by a court of competent authority.—Stare decisis derives its compliance with what is impossible. The 90-day period within which to appoint a
name from the Latin maxim stare decisis et non quieta movere, i.e., to adhere to member of the Court is thus suspended and the period could only start or resume
precedent and not to unsettle things that are settled. It simply means that a to run when the temporary obstacle disappears (i.e., after the period of the
principle underlying the decision in one case is deemed of imperative authority, appointments ban; when there is already a quorum in the JBC; or when there is
controlling the decisions of like cases in the same court and in lower courts within already at least three applicants).643
the same jurisdiction, unless and until the decision in question is reversed or
VOL. 618, APRIL 20, 2010 643
overruled by a court of competent authority. The decisions relied upon as
precedents are commonly those of appellate courts, because the decisions of the De Castro vs. Judicial and Bar Council (JBC)
trial courts may be appealed to higher courts and for that reason are probably not Same; Same; Same; View that to require the Judicial and Bar Council (JBC)
the best evidence of the rules of law laid down. to submit to the President a shortlist of nominees on or before the occurrence of
Same; Same; Same; The Court, as the highest court of the land, may be guided vacancy in the Supreme Court leads to preposterous results.—To require the JBC
but is not controlled by precedent; The adherence to precedents is strict and rigid in to submit to the President a shortlist of nominees on or before the occurrence of
a common-law setting like the United Kingdom, where judges make law as binding vacancy in the Court leads to preposterous results. It bears reiterating that the
as an Act of Parliament—but ours is not a common-law system; hence, judicial requirement is absurd when, inter alia, the vacancy is occasioned by the death of
precedents are not always strictly and rigidly followed.—The Court, as the highest a member of the Court, in which case the JBC could never anticipate the death of
court of the land, may be guided but is not controlled by precedent. Thus, the Court, a Justice, and could never submit a list to the President on or before the occurrence
especially with a new membership, is not obliged to follow blindly a particular of vacancy.
decision that it determines, after re-examination, to call for a rectification. The BRION, J., Concurring and Dissenting Opinion:
adherence to Constitutional Law; Presidency; Power to Appoint; Appointments; View that
642 the election appointment ban under Article VII, Section 15 of the Constitution
642 SUPREME COURT REPORTS ANNOTATED should not apply to the appointment of Members of the Supreme Court whose period
for appointment is separately provided for under Article VIII, Section 4(1).—The
De Castro vs. Judicial and Bar Council (JBC) election appointment ban under Article VII, Section 15 of the Constitution should
precedents is strict and rigid in a common-law setting like the United not apply to the appointment of Members of the Supreme Court whose period for
Kingdom, where judges make law as binding as an Act of Parliament. But ours is appointment is separately provided for under Article VIII, Section 4(1).
not a common-law system; hence, judicial precedents are not always strictly and
Same; Same; Same; Same; View that Valenzuela ruling is still good law; no appointment.—On the time element, the JBC list for the Supreme Court has to be
reason exists to touch the ruling as its main focus—the application of the election submitted on or before the vacancy occurs given the 90-day deadline that the
ban on the appointment of lower court judges under Article VIII, Section 9 of the appointing President is given in making the appointment. The list
Constitution—is not even an issue in the present case and was discussed only 645
because the petitions incorrectly cited the ruling as authority on the issue of the VOL. 618, APRIL 20, 2010 645
Chief Justice’s appointment.—I diverged fully from the Decision on the question of
whether we should maintain or reverse our ruling in Valenzuela. I maintained that De Castro vs. Judicial and Bar Council (JBC)
it is still good law; no reason exists to touch the ruling as its main focus—the will be submitted, not to the President as an outgoing President, nor
application of the election ban on the appointment of lower court judges under to the election winner as an incoming President, but to the President of the
Article VIII, Section 9 of the Constitution—is not even an issue in the present case Philippines whoever he or she may be. If the incumbent President does not act
and was discussed only because the petitions incorrectly cited the ruling as on the JBC list within the time left in her term, the same list shall be available to
authority on the issue of the Chief Justice’s appointment. The Decision proposed the new President for him to act upon. In all these, the Supreme Court bears the
to reverse Valenzuela but only secured the support of five (5) votes, while my burden of overseeing that the JBC’s duty is done, unerringly and with utmost
Separate Opinion in support of Valenzuela had four (4) votes. Thus, on the whole, dispatch; the Court cannot undertake this supervision in a manner consistent with
the Decision did not prevail in reversing Valenzuela, as it only had five (5) votes in the Constitution’s expectation from the JBC unless it adopts a pro-active stance
a field of 12 within the limits of its supervisory authority.
644 Same; Same; Same; Same; Same; View that the ponencia’s ruling is out of
place in the present case, since at issue here is the appointment of the Chief Justice
644 SUPREME COURT REPORTS ANNOTATED
during the period of the election ban, not the appointment of lower court judges that
De Castro vs. Judicial and Bar Council (JBC) Valenzuela resolved.—The ponencia’s ruling reversing Valenzuela, in my view, is
participating Members of the Court. Valenzuela should therefore remain, as out of place in the present case, since at issue here is the appointment of the Chief
of the filing of this Opinion, as a valid precedent. Justice during the period of the election ban, not the appointment of lower court
Same; Same; Same; Same; Judicial and Bar Council; View that the Supreme judges that Valenzuelaresolved. To be perfectly clear, the conflict in the
Court cannot dictate on the Judicial and Bar Council (JBC) the results of its constitutional provisions is not confined to Article VII, Section 15 and Article VIII,
assigned task, i.e., who to recommend or what standards to use to determine who Section 4(1) with respect to the appointment of Members of the Supreme Court;
to recommend—it cannot even direct the JBC on how and when to do its duty, but even before the Valenzuela ruling, the conflict already existed between Article VII,
it can, under its power of supervision, direct the JBC to “take such action or step as Section 15 and Article VIII, Section 9—the provision on the appointment of the
prescribed by law to make them perform their duties.”—The Court cannot dictate justices and judges of courts lower than the Supreme Court. After this Court’s
on the JBC the results of its assigned task, i.e., who to recommend or what ruling in Valenzuela, no amount of hairsplitting can result in the conclusion that
standards to use to determine who to recommend. It cannot even direct the JBC Article VII, Section 15 applied the election ban over the whole Judiciary, including
on how and when to do its duty, but it can, under its power of supervision, direct the Supreme Court, as the facts and the fallo of Valenzuela plainly spoke of the
the JBC to “take such action or step as prescribed by law to make them perform objectionable appointment of two Regional Trial Court judges. To
their duties,” if the duties are not being performed because of JBC’s fault or reiterate, Valenzuela only resolved the conflict between Article VII, Section 15 and
inaction, or because of extraneous factors affecting performance. Note in this appointments to the Judiciary under Article VIII, Section 9.
regard that, constitutionally, the Court can also assign the JBC other functions MOTIONS FOR RECONSIDERATION of a decision of the Supreme Court.
and duties—a power that suggests authority beyond what is purely supervisory. The facts are stated in the resolution of the Court.
Same; Same; Same; Same; Same; View that the process of preparing and Saklolo A. Leaño, Rita Linda V. Jimeno and Rico A. Limpengco for petitioner
submitting a list of nominees is an arduous and time-consuming task that cannot in G.R. No. 191420.646
be done overnight.—I hasten to add that the JBC’s constitutional task is not as 646 SUPREME COURT REPORTS ANNOTATED
simple as some people think it to be. The process of preparing and submitting a
list of nominees is an arduous and time-consuming task that cannot be done De Castro vs. Judicial and Bar Council (JBC)
overnight. It is a six-step process lined with standards requiring the JBC to attract Estelito P. Mendoza for petitioner in A.M. No. 10-2-5-SC.
the best available candidates, to examine and investigate them, to exhibit Amador Z. Tolentino, Jr. and Roland B. Inting for petitioner in G.R. No.
transparency in all its actions while ensuring that these actions conform to 191342.
constitutional and statutory standards (such as the election ban on appointments), Arturo M. De Castro for and on his own behalf in G.R. No. 191002.
to submit the required list of nominees on time, and to ensure as well that all these Jaime N. Soriano for himself in G.R. No. 191032.
acts are politically neutral. Manuel M. Lazaro, et al. for petitioner in G.R. No. 191057.
Same; Same; Same; Same; Same; View that the Judicial and Bar Council Benjamin P. Lozada III, et al. for movant-intervenor Atty. Marlon B. Urbano.
(JBC) list for the Supreme Court has to be submitted on or before the vacancy occurs Pitero M. Reig for oppositor-in-intervention Board of the Integrated Bar of the
given the 90-day deadline that the appointing President is given in making the Philippines-Pasay, Parañaque, Las Piñas and Muntinlupa Chapters.
Edre U. Olalia, et al. for oppositor-intervenor. 2. The Mendoza petition should have been dismissed, because it sought a
Al A. Parreño for movant oppositors-in-intervention. mere declaratory judgment and did not involve a justiciable controversy.
Ibarra M. Gutierrez for oppositor-intervenors Walden F. Bello and Loreta Ann 3. All Justices of the Court should participate in the next deliberations. The
P. Rosales. mere fact that the Chief Justice sits as ex officio head of the JBC should not
RESOLUTION prevail over the more compelling state interest for him to participate as a
BERSAMIN, J.: Member of the Court.
On March 17, 2010, the Court promulgated its decision, holding: Tolentino and Inting
“WHEREFORE, the Court: 1. A plain reading of Section 15, Article VII does not lead to an interpretation
1. Dismisses the petitions for certiorari and mandamus in G.R. No. 191002 that exempts judicial appointments from the express ban on midnight
and G.R. No. 191149, and the petition for mandamus in G.R. No. 191057 for being appointments.
premature; 2. In excluding the Judiciary from the ban, the Court has made distinctions
2. Dismisses the petitions for prohibition in G.R. No. 191032 and G.R. No. and has created exemptions when none exists.
191342 for lack of merit; and 3. The ban on midnight appointments is placed in Article VII, not in Article
3. Grants the petition in A.M. No. 10-2-5-SC and, accordingly, directs the VIII, because it limits an executive, not a judicial, power.
Judicial and Bar Council:647 4. Resort to the deliberations of the Constitutional Commission is
VOL. 618, APRIL 20, 2010 647 superfluous, and is powerless to vary the terms of the clear prohibition.
5. The Court has given too much credit to the position taken by Justice
De Castro vs. Judicial and Bar Council (JBC) Regalado. Thereby, the Court has raised the Constitution to the level of a
(a) To resume its proceedings for the nomination of candidates to fill venerated text whose intent can only be divined by its framers as to be
the vacancy to be created by the compulsory retirement of Chief Justice outside the realm of understanding by the sovereign people that ratified it.
Reynato S. Puno by May 17, 2010; 6. Valenzuela should not be reversed.
(b) To prepare the short list of nominees for the position of Chief 7. The petitioners, as taxpayers and lawyers, have the clear legal standing to
Justice; question the illegal composition of the JBC.
(c) To submit to the incumbent President the short list of nominees Philippine Bar Association
for the position of Chief Justice on or before May 17, 2010; and 1. The Court’s strained interpretation of the Constitution violates the basic
(d) To continue its proceedings for the nomination of candidates to principle that the Court should not formulate a rule of constitutional law
fill other vacancies in the Judiciary and submit to the President the short broader than what is required by the precise facts of the case.649
list of nominees corresponding thereto in accordance with this decision.
VOL. 618, APRIL 20, 2010 649
SO ORDERED.”
Motions for Reconsideration De Castro vs. Judicial and Bar Council (JBC)
Petitioners Jaime N. Soriano (G.R. No. 191032), Amador Z. Tolentino and 2. Considering that Section 15, Article VII is clear and straightforward, the
Roland B. Inting (G.R. No. 191342), and Philippine Bar Association (G.R. No. only duty of the Court is to apply it. The provision expressly and clearly
191420), as well as intervenors Integrated Bar of the Philippines-Davao del Sur provides a general limitation on the appointing power of the President in
(IBP-Davao del Sur, et al.); Christian Robert S. Lim; Peter Irving Corvera; Bagong prohibiting the appointment of any person to any position in the
Alyansang Bayan and others (BAYAN, et al.); Alfonso V. Tan, Jr.; the Women Trial Government without any qualification and distinction.
Lawyers Organization of the Philippines (WTLOP); Marlou B. Ubano; Mitchell 3. The Court gravely erred in unilaterally ignoring the constitutional
John L. Boiser; and Walden F. Bello and Loretta Ann P. Rosales (Bello, et al.), filed safeguard against midnight appointments.
their respective motions for reconsideration. Also filing a motion for 4. The Constitution has installed two constitutional safeguards:- the
reconsideration was Senator Aquilino Q. Pimentel, Jr., whose belated intervention prohibition against midnight appointments, and the creation of the JBC. It
was allowed. is not within the authority of the Court to prefer one over the other, for the
We summarize the arguments and submissions of the various motions for Court’s duty is to apply the safeguards as they are, not as the Court likes
reconsideration, in the aforegiven order: them to be.
Soriano 5. The Court has erred in failing to apply the basic principles of statutory
1. The Court has not squarely ruled upon or addressed the issue of whether construction in interpreting the Constitution.
or not the power to designate the Chief Justice belonged to the Supreme 6. The Court has erred in relying heavily on the title, chapter or section
Court en banc.648 headings, despite precedents on statutory construction holding that such
648 SUPREME COURT REPORTS ANNOTATED headings carried very little weight.
7. The Constitution has provided a general rule on midnight appointments,
De Castro vs. Judicial and Bar Council (JBC) and the only exception is that on temporary appointments to executive
positions.
8. The Court has erred in directing the JBC to resume the proceedings for the Corvera
nomination of the candidates to fill the vacancy to be created by the 1. The Court’s exclusion of appointments to the Judiciary from the
compulsory retirement of Chief Justice Puno with a view to submitting the Constitutional ban on midnight appointments is based on an interpretation
list of nominees for Chief Justice to President Arroyo on or before May 17, beyond the plain and unequivocal language of the Constitution.
2010. The Constitution grants the Court only the power of supervision over 2. The intent of the ban on midnight appointments is to cover appointments
the JBC; hence, the Court cannot tell the JBC what to do, how to do it, or in both the Executive and Judicial Departments. The application of the
when to do it, especially in the absence of a real and justiciable case principle of verba legis (ordinary meaning) would have obviated dwelling
assailing any specific action or inaction of the JBC. on the organization and arrangement of the provisions of the Constitution.
9. The Court has engaged in rendering an advisory opinion and has indulged If there is any ambiguity in Section 15, Article VII, the intent behind the
in speculations.650 provision, which is to prevent political partisanship in all branches of the
650 SUPREME COURT REPORTS ANNOTATED Government, should have controlled.
3. A plain reading is preferred to a contorted and strained interpretation
De Castro vs. Judicial and Bar Council (JBC) based on compartmentalization and physical arrangement, especially
10. The constitutional ban on appointments being already in effect, the considering that the Constitution must be interpreted as a whole.
Court’s directing the JBC to comply with the decision constitutes a culpable 4. Resort to the deliberations or to the personal interpretation of the framers
violation of the Constitution and the commission of an election offense. of the Constitution should yield to the plain and unequivocal language of
11. The Court cannot reverse on the basis of a secondary authority a doctrine the Constitution.
unanimously formulated by the Court en banc. 5. There is no sufficient reason for reversing Valenzuela, a ruling that is
12. The practice has been for the most senior Justice to act as Chief Justice reasonable and in accord with the Constitution.652
whenever the incumbent is indisposed. Thus, the appointment of the
652 SUPREME COURT REPORTS ANNOTATED
successor Chief Justice is not urgently necessary.
13. The principal purpose for the ban on midnight appointments is to arrest De Castro vs. Judicial and Bar Council (JBC)
any attempt to prolong the outgoing President’s powers by means of BAYAN, et al.
proxies. The attempt of the incumbent President to appoint the next Chief 1. The Court erred in granting the petition in A.M. No. 10-2-5-SC, because
Justice is undeniably intended to perpetuate her power beyond her term of the petition did not present a justiciable controversy. The issues it raised
office. were not yet ripe for adjudication, considering that the office of the Chief
IBP-Davao del Sur, et al. Justice was not yet vacant and that the JBC itself has yet to decide whether
1. Its language being unambiguous, Section 15, Article VII of the Constitution or not to submit a list of nominees to the President.
applies to appointments to the Judiciary. Hence, no cogent reason exists to 2. The collective wisdom of Valenzuela Court is more important and
warrant the reversal of the Valenzuela pronouncement. compelling than the opinion of Justice Regalado.
2. Section 16, Article VII of the Constitution provides for presidential 3. In ruling that Section 15, Article VII is in conflict with Section 4(1), Article
appointments to the Constitutional Commissions and the JBC with the VIII, the Court has violated the principle of ut magis valeat quam
consent of the Commission on Appointments. Its phrase “other officers pereat(which mandates that the Constitution should be interpreted as a
whose appointments are vested in him in this Constitution” is enough proof whole, such that any conflicting provisions are to be harmonized as to fully
that the limitation on the appointing power of the President extends to give effect to all). There is no conflict between the provisions; they
appointments to the Judiciary. Thus, Section 14, Section 15, and Section 16 complement each other.
of Article VII apply to all presidential appointments in the Executive and 4. The form and structure of the Constitution’s titles, chapters, sections, and
Judicial Branches of the Government. draftsmanship carry little weight in statutory construction. The clear and
3. There is no evidence that the framers of the Constitution abhorred the idea plain language of Section 15, Article VII precludes interpretation.
of an Acting Chief Justice in all cases.651 Tan, Jr.
VOL. 618, APRIL 20, 2010 651 1. The factual antecedents do not present an actual case or controversy. The
clash of legal rights and interests in the present case are merely
De Castro vs. Judicial and Bar Council (JBC) anticipated. Even if it is anticipated with certainty, no actual vacancy in
Lim the position of the Chief Justice has yet occurred.
1. There is no justiciable controversy that warrants the Court’s exercise of 2. The ruling that Section 15, Article VII does not apply to a vacancy in the
judicial review. Court and the Judiciary runs in conflict with long standing principles and
2. The election ban under Section 15, Article VII applies to appointments to doctrines of statutory construction. The provision admits only one
fill a vacancy in the Court and to other appointments to the Judiciary. exception, temporary appointments in the Executive Department. Thus,
3. The creation of the JBC does not justify the removal of the safeguard under the Court should not distinguish, because the law itself makes no
Section 15 of Article VII against midnight appointments in the Judiciary. distinction.
3. Valenzuela was erroneously reversed. The framers of the Constitution 3. The opinion of Justice Regalado is irrelevant, because Section 15, Article
clearly intended the ban on midnight appointments to cover the members VII and the pertinent records of the Constitutional Commission are clear
of the Judiciary. Hence, and unambiguous.
653 4. The Court has erred in ordering the JBC to submit the list of nominees to
VOL. 618, APRIL 20, 2010 653 the President by May 17, 2010 at the latest, because no specific law requires
the JBC to submit the list of nominees even before the vacancy has
De Castro vs. Judicial and Bar Council (JBC) occurred.
giving more weight to the opinion of Justice Regalado to reverse Boiser
the en banc decision in Valenzuela was unwarranted. 1. Under Section 15, Article VII, the only exemption from the ban on midnight
4. Section 15, Article VII is not incompatible with Section 4(1), Article VIII. appointments is the temporary appointment to an executive position. The
The 90-day mandate to fill any vacancy lasts until August 15, 2010, or a limitation is in keeping with the clear intent of the framers of the
month and a half after the end of the ban. The next President has roughly Constitution to place a restriction on the power of the outgoing Chief
the same time of 45 days as the incumbent President (i.e., 44 days) within Executive to make appointments.
which to scrutinize and study the qualifications of the next Chief Justice. 2. To exempt the appointment of the next Chief Justice from the ban on
Thus, the JBC has more than enough opportunity to examine the nominees midnight appointments makes the appointee beholden to the outgoing
without haste and political uncertainty. Chief Executive, and compromises the independence of the Chief Justice by
5. When the constitutional ban is in place, the 90-day period under Section having the outgoing President be continually influential.
4(1), Article VIII is suspended. 3. The Court’s reversal of Valenzuela without stating the sufficient reason
6. There is no basis to direct the JBC to submit the list of nominees on or violates the principle of stare decisis.655
before May 17, 2010. The directive to the JBC sanctions a culpable violation
VOL. 618, APRIL 20, 2010 655
of the Constitution and constitutes an election offense.
7. There is no pressing necessity for the appointment of a Chief Justice, De Castro vs. Judicial and Bar Council (JBC)
because the Court sits en banc, even when it acts as the sole judge of all Bello, et al.
contests relative to the election, returns and qualifications of the President 1. Section 15, Article VII does not distinguish as to the type of appointments
and Vice-President. Fourteen other Members of the Court can validly an outgoing President is prohibited from making within the prescribed
comprise the Presidential Electoral Tribunal. period. Plain textual reading and the records of the Constitutional
WTLOP Commission support the view that the ban on midnight appointments
1. The Court exceeded its jurisdiction in ordering the JBC to submit the list extends to judicial appointments.
of nominees for Chief Justice to the President on or before May 17, 2010, 2. Supervision of the JBC by the Court involves oversight. The subordinate
and to continue its proceedings for the nomination of the candidates, subject to oversight must first act not in accord with prescribed rules before
because it granted a relief not prayed for; imposed on the JBC a deadline the act can be redone to conform to the prescribed rules.
not provided by law or the Constitution; exercised control instead of mere 3. The Court erred in granting the petition in A.M. No. 10-2-5-SC, because
supervision over the JBC; and lacked sufficient votes to reverse Valenzuela. the petition did not present a justiciable controversy.
2. In interpreting Section 15, Article VII, the Court has ignored the basic Pimentel
principle of statutory construction to the effect that the literal meaning of 1. Any constitutional interpretative changes must be reasonable, rational,
the law must be applied when it is clear and unambiguous; and that we and conformable to the general intent of the Constitution as a limitation to
should not distinguish where the law does not distinguish.654 the powers of Government and as a bastion for the protection of the rights
654 SUPREME COURT REPORTS ANNOTATED of the people. Thus, in harmonizing seemingly conflicting provisions of the
Constitution, the interpretation should always be one that protects the
De Castro vs. Judicial and Bar Council (JBC) citizenry from an ever expanding grant of authority to its representatives.
3. There is no urgency to appoint the next Chief Justice, considering that the 2. The decision expands the constitutional powers of the President in a
Judiciary Act of 1948 already provides that the power and duties of the manner totally repugnant to republican constitutional democracy, and is
office devolve on the most senior Associate Justice in case of a vacancy in tantamount to a judicial amendment of the Constitution without proper
the office of the Chief Justice. authority.
Ubano COMMENTS
1. The language of Section 15, Article VII, being clear and unequivocal, needs The Office of the Solicitor General (OSG) and the JBC separately represent in
no interpretation their respective comments, thus:
2. The Constitution must be construed in its entirety, not by resort to the OSG
organization and arrangement of its provisions. 1. The JBC may be compelled to submit to the President a short list of its
nominees for the position of Chief Justice.656
656 SUPREME COURT REPORTS ANNOTATED We deny the motions for reconsideration for lack of merit, for all the matters
being thereby raised and argued, not being new, have all been resolved by the
De Castro vs. Judicial and Bar Council (JBC) decision of March 17, 2010.
2. The incumbent President has the power to appoint the next Chief Justice. Nonetheless, the Court opts to dwell on some matters only for the purpose of
3. Section 15, Article V11 does no apply to the Judiciary. clarification and emphasis.
4. The principles of constitutional construction favor the exemption of the First: Most of the movants contend that the principle of stare decisis is
Judiciary from the ban on midnight appointments. controlling, and accordingly insist that the Court has erred in disobeying or
5. The Court has the duty to consider and resolve all issues raised by the abandoning Valenzuela.1
parties as well as other related matters. The contention has no basis.
JBC Stare decisis derives its name from the Latin maxim stare decisis et non quieta
1. The consolidated petitions should have been dismissed for prematurity, movere, i.e., to adhere to precedent and not to unsettle things that are settled. It
because the JBC has not yet decided at the time the petitions were filed simply means that a
whether the incumbent President has the power to appoint the new Chief
Justice, and becausse the JBC, having yet to interview the candidates, has _______________
not submitted a short list to the President.
2. The statement in the decision that there is a doubt on whether a JBC short
1 In Re Appointments Dated March 30, 1998 of Hon. Mateo A. Valenzuela and
list is necessary for the President to appoint a Chief Justice should be Hon. Placido B. Vallarta as Judges of the Regional Trial Court of Branch 62, Bago
struck down as bereft of constitutional and legal basis. The statement
City and of Branch 24, Cabanatuan City, respectively, A.M. No. 98-5-01-SC,
undermines the independence of the JB.
November 9, 1998, 298 SCRA 408.
3. The JBC will abide by the final decision of the Court, but in accord with its
658
constitutional mandate and its implementing rules and regulations.
For his part, petitioner Estelito P. Mendoza (A.M. No. 10-2-5-SC) submits his 658 SUPREME COURT REPORTS ANNOTATED
comment even if the OSG and the JBC were the only ones the Court has required De Castro vs. Judicial and Bar Council (JBC)
to do so. He states that the motions for reconsideration were directed at the principle underlying the decision in one case is deemed of imperative authority,
administrative matter he initiated and which the Court resolved. His comment controlling the decisions of like cases in the same court and in lower courts within
asserts: the same jurisdiction, unless and until the decision in question is reversed or
1. The grounds of the motions for reconsideration were already resolved by overruled by a court of competent authority. The decisions relied upon as
the decision and the separate opinion. precedents are commonly those of appellate courts, because the decisions of the
2. The administrative matter he brought invoked the Court’s power of trial courts may be appealed to higher courts and for that reason are probably not
supervision over the JBC as provided by Section 8(1), Article VIII of the the best evidence of the rules of law laid down. 2
Constitution, as distin- Judicial decisions assume the same authority as a statute itself and, until
657 authoritatively abandoned, necessarily become, to the extent that they are
VOL. 618, APRIL 20, 2010 657 applicable, the criteria that must control the actuations, not only of those called
upon to abide by them, but also of those duty-bound to enforce obedience to
De Castro vs. Judicial and Bar Council (JBC)
them.3 In a hierarchical judicial system like ours, the decisions of the higher courts
guished from the Court’s adjudicatory power under Section 1, Article VIII. In the bind the lower courts, but the courts of co-ordinate authority do not bind each
former, the requisites for judicial review are not required, which was
other. The one highest court does not bind itself, being invested with the innate
why Valenzuela was docketed as an administrative matter. Considering that the
authority to rule according to its best lights.4
JBC itself has yet to take a position on when to submit the short list to the proper
The Court, as the highest court of the land, may be guided but is not controlled
appointing authority, it has effectively solicited the exercise by the Court of its
by precedent. Thus, the Court, especially with a new membership, is not obliged to
power of supervision over the JBC. follow blindly a particular decision that it determines, after re-examination, to call
3. To apply Section 15, Article VII to Section 4(1) and Section 9, Article VIII for a rectification.5The adherence to precedents is strict and rigid in a common-law
is to amend the Constitution.
setting like the United Kingdom, where judges make law as binding as an Act of
4. The portions of the deliberations of the Constitutional Commission quoted
in the dissent of Justice Carpio Morales, as well as in some of the motions
_______________
for reconsideration do not refer to either Section 15, Article VII or Section
4(1), Article VIII, but to Section 13, Article VII (on nepotism).
2 Price & Bitner, Effective Legal Research, Little, Brown & Co., New York
(1962), § 9.7.
RULING
3 Caltex (Phil.), Inc. v. Palomar, No. L-19650, September 29, 1966, 18 SCRA Second: Some intervenors are grossly misleading the public by their insistence
247 that the Constitutional Commission extended to the Judiciary the ban on
4 E.g., Dias, Jurisprudence, Butterworths, London, 1985, Fifth Edition, p. 127. presidential appointments during the period stated in Section 15, Article VII.
5 Limketkai Sons Milling, Inc. v. Court of Appeals, G.R. No. 118509, September The deliberations that the dissent of Justice Carpio Morales quoted from the
5, 1996, 261 SCRA 464. records of the Constitutional Commission did not concern either Section 15, Article
659 VII or Section 4(1), Article VIII, but only Section 13, Article VII, a provision on
VOL. 618, APRIL 20, 2010 659 nepotism. The records of the Constitutional Commission show that Commissioner
Hilario G. Davide, Jr. had proposed to include judges and justices related to the
De Castro vs. Judicial and Bar Council (JBC) President within the fourth civil degree of consanguinity or affinity among the
Parliament.6 But ours is not a common-law system; hence, judicial precedents are persons whom the President might not appoint during his or her tenure. In the
not always strictly and rigidly followed. A judicial pronouncement in an earlier end, however, Commissioner Davide, Jr. withdrew the proposal to include the
decision may be followed as a precedent in a subsequent case only when its Judiciary in Section 13, Article VII “(t)o avoid any further complication,”8 such that
reasoning and justification are relevant, and the court in the latter case accepts the final version of the second paragraph of Section 13, Article VII even completely
such reasoning and justification to be applicable to the case. The application of the omits any reference to the Judiciary, to wit:
precedent is for the sake of convenience and stability.
For the intervenors to insist that Valenzuela ought not to be disobeyed, or _______________
abandoned, or reversed, and that its wisdom should guide, if not control, the Court
in this case is, therefore, devoid of rationality and foundation. They seem to 7 Section 4 (2), Article VIII, provides:
conveniently forget that the Constitution itself recognizes the innate authority of xxx
the Court en banc to modify or reverse a (3) Cases or matters heard by a division shall be decided or resolved with the
concurrence of a majority of the Members who actually took part in the
_______________ deliberations on the issues in the case and voted thereon, and in no case, without
the concurrence of at least three of such Members. When the required number is
6 See Calabresi, A Common Law for the Age of Statutes, Harvard University not obtained, the case shall be decided en banc; Provided, that no doctrine or
Press, p. 4 (1982) and endnote 12 of the page, which essentially recounts that the principle of law laid down by the court in a decision rendered en banc or
strict application of the doctrine of stare decisis is true only in a common-law in division may be modified or reversed except by the court sitting en
jurisdiction like England (citing Wise, The Doctrine of Stare Decisis, 21 Wayne Law banc.
Review, 1043, 1046-1047 (1975). Calabresi recalls that the English House of Lords 8 Record of the 1986 Constitutional Commission, Vol. 2, July 31, 1986, RCC
decided in 1898 (London Tramways Co. v. London County Council, A.C. 375) that No. 44. pp. 542-543.
they could not alter precedents laid down by the House of Lords acting as the 661
supreme court in previous cases, but that such precedents could only be altered by
VOL. 618, APRIL 20, 2010 661
an Act of Parliament, for to do otherwise would mean that the courts would usurp
legislative function; he mentions that in 1966, Lord Chancellor Gardiner De Castro vs. Judicial and Bar Council (JBC)
announced in a Practice Statement a kind of general memorandum from the court “Section 13. xxx
that while: “Their Lordships regard the use of precedent as an indispensable The spouse and relatives by consanguinity or affinity within the fourth civil
foundation upon which to decide what is the law,” they “nevertheless recognize degree of the President shall not during his tenure be appointed as Members of the
that too rigid adherence to precedent may lead to injustice in a particular case and Constitutional Commissions, or the Office of the Ombudsman, or as Secretaries,
also unduly restrict the proper development of the law. They propose, therefore, to Undersecretaries, chairmen or heads of bureaus or offices, including government-
modify their present practice and, while treating former decisions of this House as owned or controlled corporations and their subsidiaries.”
normally binding, to depart from a previous decision when it appears right to do Last: The movants take the majority to task for holding that Section 15, Article
so.” (Calabresi cites Leach, Revisionism in the House of Lords: The Bastion of Rigid VII does not apply to appointments in the Judiciary. They aver that the Court
Stare Decisis Falls, 80 Harvard Law Review, 797 (1967). either ignored or refused to apply many principles of statutory construction.
660 The movants gravely err in their posture, and are themselves apparently
660 SUPREME COURT REPORTS ANNOTATED contravening their avowed reliance on the principles of statutory construction.
For one, the movants, disregarding the absence from Section 15, Article VII of
De Castro vs. Judicial and Bar Council (JBC) the express extension of the ban on appointments to the Judiciary, insist that the
doctrine or principle of law laid down in any decision rendered en banc or in ban applied to the Judiciary under the principle of verba legis. That is self-
division.7 contradiction at its worst.
Another instance is the movants’ unhesitating willingness to read into Section 10 State ex rel Everding v. Simon, 20 Ore. 365, 26 Pac. 170.
4(1) and Section 9, both of Article VIII, the express applicability of the ban under 663
Section 15, Article VII during the period provided therein, despite the silence of VOL. 618, APRIL 20, 2010 663
said provisions thereon. Yet, construction cannot supply the omission, for doing so
would generally constitute an encroachment upon the field of the Constitutional De Castro vs. Judicial and Bar Council (JBC)
Commission. Rather, Section 4(1) and Section 9 should be left as they are, given outgoing President nor the present Members of the Court had arranged the current
that their meaning is clear and explicit, and no words can be interpolated in situation to happen and to evolve as it has. None of the Members of the Court could
them.9 Interpolation of words is unnecessary, because the law is more than likely have prevented the Members composing the Court when she assumed the
to fail to express the legislative intent with the interpolation. In other words, the Presidency about a decade ago from retiring during her prolonged term and tenure,
addition of new words may alter the thought intended to be conveyed. And, even for their retirements were mandatory. Yet, she is now left with an imperative duty
where the meaning of the law is clear under the Constitution to fill up the vacancies created by such inexorable
retirements within 90 days from their occurrence. Her official duty she must
_______________ comply with. So must we ours who are tasked by the Constitution to settle the
controversy.
9 Smith v. State, 66 Md. 215, 7 Atl. 49. ACCORDINGLY, the motions for reconsideration are denied with finality.
SO ORDERED.
662
De Castro, Abad, Villarama, Jr. and Perez, JJ., concur.
662 SUPREME COURT REPORTS ANNOTATED Puno (C.J.), No Part. Chairman of JBC.
De Castro vs. Judicial and Bar Council (JBC) Carpio, J., No Part, prior inhibition.
and sensible, either with or without the omitted word or words, interpolation is Corona, J., No Part.
improper, because the primary source of the legislative intent is in the language of Carpio-Morales, J., Please see my Dissenting Opinion.
the law itself.10 Velasco, Jr., J., I join the dissent of J. Nachura.
Thus, the decision of March 17, 2010 has fittingly observed: Nachura, J., I maintain my position that there is no justiciable controversy.
“Had the framers intended to extend the prohibition contained in Section 15, Brion, J., See: Concurring & Dissenting Opinion.
Article VII to the appointment of Members of the Supreme Court, they could have Peralta, J., I join Justice Brion’s Opinion.
explicitly done so. They could not have ignored the meticulous ordering of the Del Castillo, J., I join Justice Brion in his concurring and dissenting opinion.
provisions. They would have easily and surely written the prohibition Mendoza, J., I join Justice Brion in his Concurring & Dissenting Opinion.
made explicit in Section 15, Article VII as being equally applicable to the 664
appointment of Members of the Supreme Court in Article VIII itself, most 664 SUPREME COURT REPORTS ANNOTATED
likely in Section 4 (1), Article VIII. That such specification was not done
De Castro vs. Judicial and Bar Council (JBC)
only reveals that the prohibition against the President or Acting
President making appointments within two months before the next DISSENTING OPINION
presidential elections and up to the end of the President’s or Acting CARPIO-MORALES, J.:
President’s term does not refer to the Members of the Supreme Court.” No compelling reason exists for the Court to deny a reconsideration of the
We cannot permit the meaning of the Constitution to be stretched to any assailed Decision. The various motions for reconsideration raise hollering
unintended point in order to suit the purposes of any quarter. substantial arguments and legitimately nagging questions which the Court must
meet head on.
If this Court is to deserve or preserve its revered place not just in the hierarchy
FINAL WORD
but also in history, passion for reason demands the issuance of an extended and
extensive resolution that confronts the ramifications and repercussions of its
It has been insinuated as part of the polemics attendant to the controversy we assailed Decision. Only then can it offer an illumination that any self-respecting
are resolving that because all the Members of the present Court were appointed student of the law clamors and any adherent of the law deserves. Otherwise, it
by the incumbent President, a majority of them are now granting to her the takes the risk of reeking of an objectionable air of supreme judicial arrogance.
authority to appoint the successor of the retiring Chief Justice. It is thus imperative to settle the following issues and concerns:
The insinuation is misguided and utterly unfair. Whether the incumbent President is constitutionally proscribed from
The Members of the Court vote on the sole basis of their conscience and the appointing the successor of Chief
merits of the issues. Any claim to the contrary proceeds from malice and Justice Reynato S. Puno upon his
condescension. Neither the retirement on May 17, 2010 until the
ban ends at 12:00 noon of June 30,
_______________ 2010
1. In interpreting the subject constitutional provisions, the reiterating that the requirement is absurd when, inter alia, the vacancy is
Decision disregarded established canons of statutory construction. Without occasioned by the death of a member of the Court, in which case the JBC could
explaining the inapplicability of each of the relevant rules, the Decision never anticipate the death of a Justice, and could never submit a list to the
immediately placed premium on the arrangement and ordering of provisions, one President on or before the occurrence of vacancy.
of the weakest tools of construction, to arrive at its conclusion. 3. The express allowance in the Constitution of a 90-day period of vacancy in
2. In reversing Valenzuela, the Decision held that the Valenzuela dictum did the membership of the Court rebuts any public policy argument on avoiding a
not firmly rest on ConCom deliberations, yet it did not offer to cite a material vacuum of even a single day without a duly appointed Chief Justice. Moreover, as
ConCom delibera- pointed out in my Dissenting Opinion, the practice of having an acting Chief
665 Justice in the interregnum is provided for by law, confirmed by tradition, and
VOL. 618, APRIL 20, 2010 665 settled by jurisprudence to be an internal matter.
The Resolution of the majority, in denying the present Motions for
De Castro vs. Judicial and Bar Council (JBC) Reconsideration, failed to rebut the foregoing crucial matters.
tion. It instead opted to rely on the memory of Justice Florenz Regalado which I, therefore, maintain my dissent and vote to GRANT the Motions for
incidentally mentioned only the “Court of Appeals.” The Decision’s conclusion must Reconsideration of the Decision of March 17, 2010 insofar as it holds that the
rest on the strength of its own favorable Concom deliberation, none of which to incumbent President is not constitutionally proscribed from appointing the
date has been cited. successor of Chief Justice Reynato S. Puno upon his retirement on May 17, 2010
3. Instead of choosing which constitutional provision carves out an exception until the ban ends at 12:00 noon of June 30, 2010 and that the Judicial and Bar
from the other provision, the most legally feasible interpretation (in the limited Council is obliged to submit to the President the shortlist of nominees for the
cases of temporary physical or legal impossibility of compliance, as expounded in position of Chief Justice on or before May 17, 2010.667
my Dissenting Opinion) is to consider the appointments ban or other substantial
VOL. 618, APRIL 20, 2010 667
obstacle as a temporary impossibility which excuses or releases the constitutional
obligation of the Office of the President for the duration of the ban or obstacle. De Castro vs. Judicial and Bar Council (JBC)
In view of the temporary nature of the circumstance causing the impossibility CONCURRING AND DISSENTING OPINION
of performance, the outgoing President is released from non-fulfillment of the BRION, J.:
obligation to appoint, and the duty devolves upon the new President. The delay in The Motions for Reconsideration
the fulfillment of the obligation becomes excusable, since the law cannot exact After sifting through the motions for reconsideration, I found that the
compliance with what is impossible. The 90-day period within which to appoint a arguments are largely the same arguments that we have passed upon, in one form
member of the Court is thus suspended and the period could only start or resume or another, in the various petitions. Essentially, the issues boil down
to run when the temporary obstacle disappears (i.e., after the period of the to justiciability; the conflict of constitutional provisions; the merits of the
appointments ban; when there is already a quorum in the JBC; or when there is cited constitutional deliberations; and the status and effect of
already at least three applicants). the Valenzuela1 ruling. Even the motion for reconsideration of the Philippine
Whether the Judicial and Bar Coun- Bar Association (G.R. No. 191420), whose petition I did not expressly touch upon
cil is obliged to submit to the Presi- in my Separate Opinion, basically dwells on these issues.
dent the shortlist of nominees for I have addressed most, if not all, of these issues and I submit my Separate
the position of Chief Justice (or Jus- Opinion2 as my basic response to the motions for reconsideration, supplemented
tice of this Court) on or before the by the discussions below.
occurrence of the vacancy. As I reflected in my Separate Opinion (which three other Justices joined),3 the
1. The ruling in the Decision that obligates the JBC to submit the shortlist to election appointment ban under Article VII, Section 15 of the
the President on or before the occurrence of the vacancy in the Court runs counter Constitution should not apply to the appointment of Members of the
to the Concom deliberations which explain that the 90-day period is allotted Supreme Court whose period for appointment is separately pro-
666
666 SUPREME COURT REPORTS ANNOTATED _______________
De Castro vs. Judicial and Bar Council (JBC)
1 A.M. No. 98-5-01-SC, November 9, 1998, 298 SCRA 408. This A.M. involves
for both the nomination by the JBC and the appointment by the President. In the
the constitutional validity of the appointment of two (2) RTC Judges on March 30,
move to increase the period to 90 days, Commissioner Romulo stated that “[t]he 1998—a date that falls within the supposed ban under Section 15, Article VII of
sense of the Committee is that 60 days is awfully short and that the [Judicial and the Constitution. We nullified the appointments.
Bar] Council, as well as the President, may have difficulties with that.” 2 G.R. No. 191002 and companion cases, promulgated on March 17, 2010.
2. To require the JBC to submit to the President a shortlist of nominees on or
before the occurrence of vacancy in the Court leads to preposterous results. It bears
3 Justices Diosdado M. Peralta, Mariano C. Del Castillo and Jose Catral because there was, and is, no JBC refusal to act.6 Thus, the mandamus aspects of
Mendoza. these petitions should have also been dismissed outright. The ponencia,
668 unfortunately, failed to fully discuss these legal infirmities.
668 SUPREME COURT REPORTS ANNOTATED The motions for reconsideration lay major emphasis on the alleged lack of an
actual case or controversy that made the Chief Justice’s appointment a justiciable
De Castro vs. Judicial and Bar Council (JBC) issue. They claim that the Court cannot exercise the power of judicial review where
vided for under Article VIII, Section 4(1). I shared this conclusion with the there is no clash of legal rights and interests or where this clash is merely
Court’s Decision although our reasons differed on some points. anticipated, although the anticipated event shall come with certainty.7
I diverged fully from the Decision on the question of whether we should What the movants apparently forgot, focused as they were on their respective
maintain or reverse our ruling in Valenzuela. I maintained that it is still good law; petitions, is that the present case is not a single-petition case that rises or falls on
no reason exists to touch the ruling as its main focus—the application of the the strength of that single petition. The present case involves various petitions
election ban on the appointment of lower court judges under Article VIII, Section 9
of the Constitution—is not even an issue in the present case and was discussed _______________
only because the petitions incorrectly cited the ruling as authority on the issue of
the Chief Justice’s appointment. The Decision proposed to reverse Valenzuela but 6 The JBC reiterates its position in its Comment (dated April 12, 2010) on the
only secured the support of five (5) votes, while my Separate Opinion in support of motions for reconsideration that it is still acting on the preparation of the list of
Valenzuela had four (4) votes. Thus, on the whole, the Decision did not prevail in nominees and is set to interview the nominees.
reversing Valenzuela, as it only had five (5) votes in a field of 12 participating 7 See, for instance, the motion for reconsideration of intervenor Alfonso Tan,
Members of the Court. Valenzuela should therefore remain, as of the filing of this Jr.
Opinion, as a valid precedent.
670
Acting on the present motions for reconsideration, I join the majority in
denying the motions with respect to the Chief Justice issue, although we differ in 670 SUPREME COURT REPORTS ANNOTATED
some respects on the reasons supporting the denial. I dissent from the conclusion De Castro vs. Judicial and Bar Council (JBC)
that the Valenzuela ruling should be reversed. My divergence from the majority’s and interventions,8 not necessarily pulling towards the same direction, although
reasons and conclusions compels me to write this Concurring and Dissenting each one is focused on the issue of whether the election appointment ban under
Opinion. Article VII, Section 15 of the Constitution should apply to the appointment of the
The Basic Requisites / Justiciability next Chief Justice of the Supreme Court.
One marked difference between the Decision and my Separate Opinion is our Among the petitions filed were those of Tolentino (G.R. No.
approach on the basic requisites/justiciability issues. The Decision apparently 191342), Soriano (G.R. No. 191032) and Mendoza (A.M. No. 10-2-5-SC). The first
glossed over this aspect of the case, while I fully explained why the De Castro4 and two are petitions for prohibition under Section 2 of Rule 65 of the Rules of
Peralta5 petitions should be dismissed outright. In my Court.9 While they commonly share this medium of review, they differ in their
supporting reasons. The Mendoza petition, on the other hand, is totally different—
_______________ it is a petition presented as an administrative matter (A.M.) in the manner that
the Valenzuela case was an A.M. case. As I pointed out in the Separate Opinion,
4 G.R. No. 191002, Petition for Certiorari and Mandamus. the Court uses the A.M. docket designation on matters relating to its exercise of
5 G.R. No. 191149, Petition for Certiorari and Mandamus. supervision over all courts and their personnel.10 I failed to note then, but I make
669 of record now, that court rules and regulations—the outputs in the Court’s
VOL. 618, APRIL 20, 2010 669 rulemaking function—are also docketed as A.M. cases.
That an actual case or controversy involving a clash of rights and interests
De Castro vs. Judicial and Bar Council (JBC) exists is immediately and patently obvious in the Tolentino and Soriano petitions.
view, these petitions violated the most basic requirements of their chosen medium At the time the petitions were filed, the JBC had started its six-phase nomination
for review—a petition for certiorari and mandamus under Rule 65 of the Rules of process that would culminate in the submission of a list of nominees to the
Court. President of the Philippines for appointive action. Tolentino and Soriano—lawyers
The petitions commonly failed to allege that the Judicial and Bar Council and citizens with interest in the strict observance of the election ban—sought to
(JBC) performs judicial or quasi-judicial functions, an allegation that the petitions prohibit the JBC from continuing with this process.
could not really make, since the JBC does not really undertake these functions
and, for this reason, cannot be the subject of a petition for certiorari; hence, the _______________
petitions should be dismissed outright. They likewise failed to facially show any
failure or refusal by the JBC to undertake a constitutional duty to justify the 8 The docketed petitions were seven; the petitions-in-intervention were ten.
issuance of a writ of mandamus; they invoked judicial notice that we could not give
9 A prohibition petition seeks to stop the proceedings of a tribunal, to secure a resolution of the election ban issue. The JBC, at that time, had
corporation, board, officer or person exercising judicial, quasi-judicial or indicated its intent to look up to the Court’s supervisory power and role as the final
ministerial functions if any of its act is without or in excess of jurisdiction or with interpreter of the Constitution to guide it in responding to the challenges it
grave abuse of discretion amounting to lack or excess of jurisdiction. confronts.14 To me, this was “a point no less critical, from the point of view of
10 Separate Opinion, p. 16. supervision, than the appointment of the two judges during the election ban period
671 in Valenzuela.”15
VOL. 618, APRIL 20, 2010 671 In making this conclusion, I pointed out in my Separate Opinion the
unavoidable surrounding realities evident from the confluence of events,
De Castro vs. Judicial and Bar Council (JBC) namely: (1) an election to be held on May 10, 2010; (2) the retirement of the Chief
The JBC had started to act, without any prodding from the Court, because of its Justice on May 17, 2010; (3) the lapse of the terms of the elective officials from the
duty to start the nomination process but was hampered by the petitions filed and President to the congressmen on June 30, 2010; (4) the delay before the Congress
the legal questions raised that only the Supreme Court can settle with can organize and send its JBC representatives; and (5) the expiration of the term
finality.11 Thus, a clash of interests based on law existed between the petitioners of a non-elective JBC member in July 2010.16 All these—juxtaposed
and the JBC. To state the obvious, a decision in favor of Tolentino or
Soriano would result in a writ of prohibition that would direct the JBC _______________
not to proceed with the nomination process.
The Mendoza petition cited the effect of a complete election ban on judicial 12 Mendoza Petition, pp. 5-6.
appointments (in view of the already high level of vacancies and the backlog of 13 Separate Opinion, pp. 16-17.
cases) as basis, and submitted the question as an administrative matter that the 14 Supra note 11.
Court, in the exercise of its supervisory authority over the Judiciary and the JBC 15 Id., at p. 17.
itself, should act upon. At the same time, it cited the “public discourse and 16 Separate Opinion, pp. 19-22:
controversy” now taking place because of the application of the election ban on the A first reality is that the JBC cannot, on its own due to lack of the proper
appointment of the Chief Justice, pointing in this regard to the very same reasons authority, determine the appropriate course of action to take under the
mentioned in Valenzuela about the Constitution. Its principal function is to recommend appointees to the Judiciary
and it has no authority to interpret constitutional provisions, even those affecting
_______________ its principal function;
673
11 The JBC position states:
VOL. 618, APRIL 20, 2010 673
xxxx
Likewise, the JBC has yet to take a position on when to submit the De Castro vs. Judicial and Bar Council (JBC)
shortlist to the proper appointing authority, in light of Section 4(1), with the Court’s supervision over the JBC, the latter’s need for guidance, and the
Article VIII of the Constitution, which provides that vacancy in the existence of an actual controversy on
Supreme Court shall be filled within ninety (90) days from the occurrence
thereof, Section 15, Article VII of the Constitution concerning the ban on _______________
Presidential appointments “two (2) months immediately before the next
presidential elections and up to the end of his term” and Section 261(g), the authority to undertake constitutional interpretation belongs to the courts
Article XXIII of the Omnibus Election Code of the Philippines. alone.
12. Since the Honorable Supreme Court is the final interpreter of the A second reality is that the disputed constitutional provisions do not stand
Constitution, the JBC will be guided by its decision in these consolidated alone and cannot be read independently of one another; the Constitution and its
Petitions and Administrative Matter. [Emphasis supplied.] various provisions have to be read and interpreted as one seamless whole, giving
672 sufficient emphasis to every aspect in accordance with the hierarchy of our
672 SUPREME COURT REPORTS ANNOTATED constitutional values. The disputed provisions should be read together and, as
reflections of the will of the people, should be given effect to the extent that they
De Castro vs. Judicial and Bar Council (JBC)
should be reconciled.
need to resolve the issue and avoid the recurrence of conflict between the Executive The third reality, closely related to the second, is that in resolving the
and the Judiciary, and the need to “avoid polemics concerning the matter.”12 coverage of the election ban vis-à-vis the appointment of the Chief Justice and the
I recognized in the Separate Opinion that, unlike in Valenzuelawhere an Members of the Court, provisions of the Constitution other than the disputed
outright defiance of the election ban took place, no such obvious triggering event provisions must be taken into account. In considering when and how to act, the
transpired in the Mendoza petition.13Rather, the Mendoza petition looked to the JBC has to consider that:
supervisory power of the Court over judicial personnel and over the JBC as basis
1. The President has a term of six years which begins at noon of means that the JBC ideally will have to make its list available at the start
June 30 following the election, which implies that the outgoing President of the 90-day period so that its process will not eat up the 90-day period
remains President up to that time. (Section 4, Article VII). The President granted the President.
assumes office at the beginning of his or her term, with provision for the 6. After noon of June 30, 2010, the JBC representation from Congress
situations where the President fails to qualify or is unavailable at the would be vacant; the current representatives’ mandates to act for their
beginning of his term (Section 7, Article VII). principals extend only to the end of their present terms; thus, the JBC shall
2. The Senators and the Congressmen begin their respective terms be operating at that point at less than its full membership.
also at midday of June 30 (Sections 4 and 7, Article VI). The Congress 675
convenes on the 4th Monday of July for its regular session, but the VOL. 618, APRIL 20, 2010 675
President may call a special session at any time. (Section 15, Article VI)
3. The Valenzuela case cited as authority for the position that the De Castro vs. Judicial and Bar Council (JBC)
election ban provision applies to the whole Judiciary, only decided the issue the jurisdiction of the lower court. This recognition is beyond the level of what this
with respect to lower court judges, specifically, those covered by Section 9, Court can do in handling a moot and academic case—usually, one that no longer
Article VIII of the Constitution. Any reference to the filling up of vacancies presents a judiciable controversy but one that can still be ruled upon at the
in the Supreme Court pursuant to Section 4(1), Article VIII constitutes discretion of the court when the constitutional issue is of paramount public interest
obiter dictum as this issue was not directly in issue and was not ruled upon. and controlling principles are needed to guide the bench, the bar and the public. 17
674 To be sure, this approach in recognizing when a petition is actionable is novel.
An overriding reason for this approach can be traced to the nature of the petition,
674 SUPREME COURT REPORTS ANNOTATED
as it rests on the Court’s supervisory authority and relates to the exercise of the
De Castro vs. Judicial and Bar Council (JBC) Court’s administrative rather than its judicial functions (other than these two
the same issues bedeviling the JBC—in my view, were sufficient to save the functions, the Court also has its rulemaking function under Article VIII, Section
Mendoza petition from being a mere request for opinion or a petition for 5(5) of the Constitution). Strictly speaking, the Mendoza petition calls for
declaratory relief that falls under directions from the Court in the exercise of its power of supervision over

_______________ _______________

These provisions and interpretation of the Valenzuela ruling—when read 7. Congress will not convene until the 4th Monday of July, 2010, but
together with disputed provisions, related with one another, and considered with would still need to organize before the two Houses of Congress can send
the May 17, 2010 retirement of the current Chief Justice—bring into focus certain their representatives to the JBC—a process may extend well into August,
unavoidable realities, as follows: 2010.
1. If the election ban would apply fully to the Supreme Court, the 8. By July 5, 2010, one regular member of the JBC would vacate his
incumbent President cannot appoint a Member of the Court beginning post. Filling up this vacancy requires a presidential appointment and the
March 10, 2010, all the way up to June 30, 2010. concurrence of the Commission on Appointments.
2. The retirement of the incumbent Chief Justice—May 17, 2010— 9. Last but not the least, the prohibition in Section 15, Article VII is
falls within the period of the election ban. (In an extreme example where the that “a President or Acting President shall not make appointments.” This
retirement of a Member of the Court falls on or very close to the day the prohibition is expressly addressed to the President and covers the act of
election ban starts, the Office of the Solicitor General calculates in its appointment; the prohibition is not against the JBC in the performance of
Comment that the whole 90 days given to the President to make appointment its function of “recommending appointees to the Judiciary”—an act that is
would be covered by the election ban.) one step away from the act of making appointments.
3. Beginning May 17, 2010, the Chief Justice position would be 17 The Province of North Cotabato v. Government of the Republic of the
vacant, giving rise to the question of whether an Acting Chief Justice can Philippines Peace Panel Ancestral Domain, G.R. Nos. 183591, 183791, 183752,
act in his place. While this is essentially a Supreme Court concern, the 183893, 183951 and 183962, October 14, 2008, 568 SCRA 402.
Chief Justice is the ex officio Chair of the JBC; hence it must be concerned 676
and be properly guided. 676 SUPREME COURT REPORTS ANNOTATED
4. The appointment of the new Chief Justice has to be made within
90 days from the time the vacancy occurs, which translates to a deadline of De Castro vs. Judicial and Bar Council (JBC)
August 15, 2010. the JBC,18 not on the basis of the power of judicial review.19 In this sense, it does
5. The deadline for the appointment is fixed (as it is not reckoned not need the actual clash of interests of the type that a judicial adjudication
from the date of submission of the JBC list, as in the lower courts) which requires. All that must be shown is the active need for supervision to justify the
Court’s intervention as supervising authority.
Under these circumstances, the Court’s recognition of the Mendoza petition assign the JBC other functions and duties—a power that suggests authority
was not an undue stretch of its constitutional powers. If the recognition is unusual beyond what is purely supervisory.
at all, it is so only because of its novelty; to my knowledge, this is the first time Where the JBC itself is at a loss on how to proceed in light of disputed
ever in Philippine jurisprudence that the supervisory authority of the constitutional provisions that require interpretation,22 the Court is not legally out
Court over an attached agency has been highlighted in this of line—as the final author-
manner. Novelty, per se, however, is not a ground for objection nor a mark of
infirmity for as long as the novel move is founded in law. In this case, as in the _______________
case of the writ of amparo and habeas data that were then novel and avowedly
activist in character, sufficient legal basis exists to actively invoke the Court’s 20 Control is the power of an officer to alter or modify or nullify or set aside
supervisory authority—granted under the Constitution, no less—as basis for what a subordinate officer had done in the performance of his duties and to
action. substitute the judgment of the former for that of the latter. It is distinguished from
To partly quote the wording of the Constitution, Article VIII, Section 8(1) and supervision in that the latter means overseeing, or the power or authority of an
(5) provide that “A Judicial and Bar Council is hereby created under the officer to see that subordinate officers perform their duties, and if the latter fail or
supervision of the Supreme Court… It may exercise such other functions and neglect to fulfill them, then the former may take such action or steps as prescribed
duties as the Supreme Court may assign to it.” Supervision, as a legal concept, by law to make them perform these duties. Nachura, J., Outline Reviewer in
more often than not, is defined in relation with Political Law, 2006 ed., p. 276.
21 G.R. No. 156052, February 13, 2008, 545 SCRA 92.
_______________ 22 Supra notes 11 and 14.
678
18 By virtue of its power of administrative supervision, the Supreme Court 678 SUPREME COURT REPORTS ANNOTATED
oversees the judges’ and court personnel’s compliance with the laws, rules and
regulations. It may take the proper administrative action against them if they De Castro vs. Judicial and Bar Council (JBC)
commit any violation. See Ampong v. CSC, G.R. No. 107910, August 26, 2008, 563 ity on the interpretation of the Constitution and as the entity constitutionally-
SCRA 293. The Constitution separately provides for the Supreme Court’s tasked to supervise the JBC—in exercising its oversight function by clarifying the
supervision over the JBC. See Article VIII, Section 8 of the Constitution. interpretation of the disputed constitutional provision to guide the JBC. In doing
19 Judicial Review is the power of the courts to test the validity of executive this, the Court is not simply rendering a general legal advisory; it is providing
and legislative acts for their conformity with the Constitution, Garcia v. Executive concrete and specific legal guidance to the JBC in the exercise of its supervisory
Secretary, G.R. No. 157584, April 2, 2009. authority, after the latter has asked for assistance in this regard. That the Court
677 does this while concretely resolving actual controversies (the Tolentino and
Soriano petitions) on the same issue immeasurably strengthens the intrinsic
VOL. 618, APRIL 20, 2010 677
correctness of the Court’s action.
De Castro vs. Judicial and Bar Council (JBC) It may be asked: why does the Court have to recognize the Mendoza petition
the concept of control.20 In Social Justice Society v. Atienza,21 we defined when it can resolve the conflict between Article VII, Section 15 and Article VIII,
“supervision” as follows: Section 4(1) through the Tolentino and Soriano petitions?
“[Supervision] means overseeing or the power or authority of an officer to see The answer is fairly simple and can be read between the lines of the above
that subordinate officers perform their duties. If the latter fail or neglect to fulfill explanation on the relationship between the Court and the JBC. First,
them, the former may take such action or step as prescribed by law to make them administrative is different from judicial function and providing guidance to the
perform their duties. Control, on the other hand, means the power of an officer to JBC can only be appropriate in the discharge of the Court’s administrative
alter or modify or nullify or set aside what a subordinate officer ha[s] done in the function. Second, the resolution of the Tolentino and Soriano petitions will lead to
performance of his duties and to substitute the judgment of the former for that of rulings directly related to the underlying facts of these petitions, without clear
the latter.” guidelines to the JBC on the proper parameters to observe vis-à-vis the
Under this definition, the Court cannot dictate on the JBC the results of its constitutional dispute along the lines the JBC needs. In fact, concrete guidelines
assigned task, i.e., who to recommend or what standards to use to determine who addressed to the JBC in the resolution of the Tolentino/Soriano petitions may even
to recommend. It cannot even direct the JBC on how and when to do its duty, but lead to accusations that the Court’s resolution is broader than is required by the
it can, under its power of supervision, direct the JBC to “take such action or step facts of the petitions. The Mendoza petition, because it pertains directly to the
as prescribed by law to make them perform their duties,” if the duties are not being performance of the JBC’s duty and the Court’s supervisory authority, allows the
performed because of JBC’s fault or inaction, or because of extraneous factors issuance of precise guidelines that will enable the JBC to fully and seasonably
affecting performance. Note in this regard that, constitutionally, the Court can also comply with its constitutional mandate.
I hasten to add that the JBC’s constitutional task is not as simple as some Constitution, particularly its structure and underlying intents, the conflict
people think it to be. The process of preparing however becomes obvious and unavoidable.
679 Section 15 on its face disallows any appointment in clear negative terms (“shall
VOL. 618, APRIL 20, 2010 679 not make”) without specifying the appointments covered by the prohibition.26 From
this literal and isolated reading springs the argument that no exception is provided
De Castro vs. Judicial and Bar Council (JBC) (except that found in Section 15 itself) so that even the Judiciary is covered by the
and submitting a list of nominees is an arduous and time-consuming task that ban on appointments.
cannot be done overnight. It is a six-step process lined with standards requiring On the other hand, Section 4(1) is likewise very clear and categorical in its
the JBC to attract the best available candidates, to examine and investigate them, terms: any vacancy in the Court shall be filledwithin 90 days from its
to exhibit transparency in all its actions while ensuring that these actions conform occurrence.27 In the way of Sec-
to constitutional and statutory standards (such as the election ban on
appointments), to submit the required list of nominees on time, and to ensure as _______________
well that all these acts are politically neutral. On the time element, the JBC list
for the Supreme Court has to be submitted on or before the vacancy occurs given 24 See PBA’s Motion for Reconsideration.
the 90-day deadline that the appointing President is given in making the 25 See the Motions for Reconsideration for PBA, WTLOP, Atty. Amador Z.
appointment. The list will be submitted, not to the President as an outgoing Tolentino, Atty. Roland B. Inting, Peter Irving Corvera and Alfonso V. Tan, Jr.
President, nor to the election winner as an incoming President, but to the 26 CONSTITUTION, Article VII, Section 15:
President of the Philippines whoever he or she may be. If the incumbent Two months immediately before the next presidential elections and up
President does not act on the JBC list within the time left in her term, the same to the end of his term, a President or Acting President shall not make
list shall be available to the new President for him to act upon. In all these, the appointments, except temporary appointments to executive positions
Supreme Court bears the burden of overseeing that the JBC’s duty is done, when continued vacancies therein will prejudice public service or endanger
unerringly and with utmost dispatch; the Court cannot undertake this supervision public safety.
in a manner consistent with the Constitution’s expectation from the JBC unless it 27 CONSTITUTION, Article VIII, Section 4(1):
adopts a pro-active stance within the limits of its supervisory authority.
681
The Disputed Provisions
The movants present their arguments on the main issue at several VOL. 618, APRIL 20, 2010 681
levels. Some argue that the disputed constitutional provisions—Article VII, De Castro vs. Judicial and Bar Council (JBC)
Section 15 and Article VIII, Section 4(1)—are clear and speak for themselves on tion 15, Section 4(1) is also clear and categorical and provides no exception; the
what the Constitution covers in banning appointments during the election appointment refers solely to the Members of the Supreme Court and does not
period.23 One even posits that there is no conflict because both mention any period that would interrupt, hold or postpone the 90-day requirement.
From this perspective, the view that no conflict exists cannot be seriously
_______________ made, unless with the mindset that one provision controls and the other should
yield. Many of the petitions in fact advocate this kind of reading, some of them
23 Philippine Bar Association (PBA), Women Trial Lawyers Organization of openly stating that the power of appointment should be reserved for the incoming
the Philippines (WTLOP), Atty. Amador Z. Tolentino, Atty. Roland B. Inting, Peter President.28 The question, however, is whether—from the viewpoint of strict law
Irving Corvera and Alfonso V. Tan, Jr. and devoid of the emotionalism and political partisanship that permeate the
680 present Philippine political environment—this kind of mindset can really be
680 SUPREME COURT REPORTS ANNOTATED adopted in reading and applying the Constitution.
In my view, this kind of mindset and the conclusion it inevitably leads to cannot
De Castro vs. Judicial and Bar Council (JBC) be adopted; the provisions of the Constitution cannot be read in isolation from what
provisions can be given effect without one detracting against the full effectiveness the whole contains. To be exact, the Constitution must be read and understood as
of the other,24 although the effect is to deny the sitting President the option to a whole, reconciling and harmonizing apparently conflicting provisions so that all
appoint in favor of a deferment for the incoming President’s action. Still others, of them can be given full force and effect,29 unless the Constitution itself expressly
repeating their original arguments, appeal to the principles of interpretation and states otherwise.30
latin maxims to prove their point.25
In my discussions in the Separate Opinion, I stated upfront my views on how _______________
the disputed provisions interact with each other. Read singly and in isolation, they
appear clear (this reading applies the “plain meaning rule” that Tolentino The Supreme Court shall be composed of a Chief Justice and fourteen
advocates in his motion for reconsideration, as explained below). Arrayed side by Associate Justices. It may sit en banc or, in its discretion, in divisions of
side with each other and considered in relation with the other provisions of the
three, five, or seven Members. Any vacancy shall be filled within ninety would have been all-encompassing and would, thus, have extended to all
days from the occurrence thereof. government positions the President can fill, had the Constitution not inserted a
xxxx provision, also on appointments, in the Article on the Judiciary with respect to
28 See Petition on Intervention of WTLOP, as cited in the decision in the above- appointments to the Supreme Court. This conflict gives rise to the
captioned cases; see also: PBA’s motion for reconsideration. questions: which provision should prevail, or should both be given effect?
29 Francisco v. House of Representatives, G.R. No. 160261, November 10, 2003, Or should both provisions yield to a higher concern—the need to
415 SCRA 44, citing Civil Liberties Union v. Executive Secretary, 194 SCRA 317 maintain the integrity of our elections?
(1994); Peralta v. Commission on Elections, G.R. No. 47771, March 11, 1978, 82 A holistic reading of the Constitution—a must in constitutional
SCRA 30 (1978); Ang- interpretation—dictates as a general rule that the tasks assigned to each
682 department and their limitations should be given full effect to fulfill the
682 SUPREME COURT REPORTS ANNOTATED constitutional purposes under the check and balance principle, unless the
Constitution itself expressly indicates its preference for one task, concern or
De Castro vs. Judicial and Bar Council (JBC) standard over the others,32 or unless this Court, in its role as interpreter of the
Not to be forgotten in reading and understanding the Constitution are the Constitution, has spoken on the appropriate interpretation that should be made.33
many established underlying constitutional principles that we have to observe and In considering the interests of the Executive and the Judiciary, a holistic
respect if we are to be true to the Constitution. These principles—among them the approach starts from the premise that the constitutional scheme is to grant the
principles of checks and balances and separation of powers—are not always President the power of appointment, subject to the limitation provided under
expressly stated in the Constitution, but no one who believes in and who has Article VII, Section 15. At the same time, the Judiciary is assured, without
studied the Constitution can deny that they are there and deserve utmost qualifications under Article VIII, Section 4(1), of the immediate appointment of
attention, respect, and even priority consideration. Members of the Supreme Court, i.e., within 90 days from the occurrence of the
In establishing the structures of government, the ideal that the Constitution vacancy. If both provisions would be allowed to take effect, as I believe they
seeks to achieve is one of balance among the three great departments of should, the limitation on the appointment power of the Presi-
government—the Executive, the Legislative and the Judiciary, with each
department undertaking its constitutionally-assigned task as a check against the _______________
exercise of power by the others, while all three departments move forward in
working for the progress of the nation. Thus, the Legislature makes the laws and 32 See Matibag v. Benipayo, G.R. No. 149036, April 2, 2002, 380 SCRA 49;
is supreme in this regard, in the way that the Executive is supreme in enforcing where the court resolved the clash between the power of the President to extend ad
and administering the law, while the Judiciary interprets both the Constitution interim appointments and the power of the Commission on Appointments to
and the law. Any provision in each of the Articles on these three confirm presidential appointments.
departments31 that intrudes into the other must be closely examined if the 33 Ibid.
provision affects and upsets the desired balance.
684
Under the division of powers, the President as Chief Executive is given the
prerogative of making appointments, subject only to the legal qualification 684 SUPREME COURT REPORTS ANNOTATED
standards, to the checks provided by the Legislature’s Commission on De Castro vs. Judicial and Bar Council (JBC)
Appointments (when applicable) and by the JBC for appointments in the Judiciary, dent under Article VII, Section 15 should itself be limited by the appointment of
and to the Constitution’s own limitations. Conflict Members of the Court pursuant to Article VIII, Section 4(1), so that the provision
applicable to the Judiciary can be given full effect without detriment to the
_______________ President’s appointing authority. This harmonization will result in restoring to the
President the full authority to appoint Members of the Supreme Court pursuant
Angco v. Castillo, G.R. No. 17169, November 30, 1963, 9 SCRA 619 (1963). to the combined operation of Article VII, Section 15 and Article VIII, Section 4(1).
30 Macalintal v. Commission on Elections, G.R. No. 157013, July 10, 2003, 310 Viewed in this light, there is essentially no conflict, in terms of the authority
SCRA 614, citing Chiongbian v. De Leon, 82 Phil 771 (1949). to appoint, between the Executive and Judiciary; the President would effectively
31 Article VI for the Legislature, Article VII for the Executive, and Article VIII be allowed to exercise the Executive’s traditional presidential power of
for the Judiciary. appointment while respecting the Judiciary’s own prerogative. In other words, the
683 President retains full powers to appoint Members of the Court during the election
VOL. 618, APRIL 20, 2010 683 period, and the Judiciary is assured of a full membership within the time frame
given.
De Castro vs. Judicial and Bar Council (JBC) Interestingly, the objection to the full application of Article VIII, Section 4(1)
comes in when the Constitution laid down Article VII, Section 15 limiting the comes, not from the current President, but mainly from petitioners echoing the
President’s appointing power during the election period. This limitation of power
present presidential candidates, one of whom shall soon be the incoming problems that the Court would resolve. These possibilities and the potential for
President. They do not, of course, cite reasons of power and the loss of the continuing influence in the Court, however, cannot be active considerations in
opportunity to appoint the Chief Justice; many of the petitioners/intervenors resolving the election ban issue as they are, in their present form and presentation,
oppose the full application of Article VIII, Section 4(1) based on the need to all speculative. If past record is to be the measure, the record of past Chief Justices
maintain the integrity of the elections through the avoidance of a “midnight and of this Court speaks for itself with respect to the Justices’ relationship with,
appointment.” and deferral to, the appointing authority in their decisions.
This “integrity” reason is a given in a democracy and can hardly be opposed on What should not be forgotten in examining the records of the Court, from the
the theoretical plane, as the integrity of the elections must indeed prevail in a true prism of problems an electoral exercise may bring, is the Court’s unique and proven
democracy. The statement, however, begs a lot of questions, among them the capacity to intervene and diffuse situations that are potentially explosive for the
question of whether the appointment of a full Court under the terms of Article nation. EDSA II particularly comes to mind in this regard (although it was an
VIII, Section 4(1) will adversely affect or enhance the integrity of the elections. event that was not rooted in election problems) as it is a perfect example of the
In my Separate Opinion, I concluded that the appointment of a Member of the potential for damage to the nation that the Court can address and has addressed.
Court even during the election period per When acting in this role, a vacancy in the Court is not only a vote less, but a
685 significant contribution less in the Court’s deliberations and capacity for action,
VOL. 618, APRIL 20, 2010 685 especially if the missing voice is the voice of the Chief Justice.
Be it remembered that if any EDSA-type situation arises in the coming
De Castro vs. Judicial and Bar Council (JBC) elections, it will be compounded by the lack of leaders because of the lapse of the
se implies no adverse effect on the integrity of the election; a full Court is ideal President’s term by June 30, 2010; by a possible failure of succession if for some
during this period in light of the Court’s unique role during elections. I maintain reason the election of the new leadership becomes problematic; and by the similar
this view and fully concur in this regard with the majority. absence of congressional leadership because Congress has not yet convened to
During the election period, the court is not only the interpreter of the organize itself.34 In this scenario, only the Judiciary of the three great departments
Constitution and the election laws; other than the Commission on Elections and of
the lower courts to a limited extent, the Court is likewise the highest impartial
recourse available to decisively address any problem or dispute arising from the _______________
election. It is the leader and the highest court in the Judiciary, the only one of the
three departments of government directly unaffected by the election. The Court is 34 Supra note 13.
likewise the entity entrusted by the Constitution, no less, with the gravest election-
687
related responsibilities. In particular, it is the sole judge of all contests in the
election of the President and the Vice-President, with leadership and participation VOL. 618, APRIL 20, 2010 687
as well in the election tribunals that directly address Senate and House of De Castro vs. Judicial and Bar Council (JBC)
Representatives electoral disputes. With this grant of responsibilities, the government stands unaffected by the election and should at least therefore be
Constitution itself has spoken on the trust it reposes on the Court on election complete to enable it to discharge its constitutional role to its fullest potential and
matters. This reposed trust, to my mind, renders academic any question of whether capacity. To state the obvious, leaving the Judiciary without any permanent leader
an appointment during the election period will adversely affect the integrity of the in this scenario may immeasurably complicate the problem, as all three
elections—it will not, as the maintenance of a full Court in fact contributes to the departments of government will then be leaderless.
enforcement of the constitutional scheme to foster a free and orderly election. To stress what I mentioned on this point in my Separate Opinion, the absence
In reading the motions for reconsideration against the backdrop of the partisan of a Chief Justice will make a lot of difference in the effectiveness of the Court as
political noise of the coming elections, one cannot avoid hearing echoes from some he or she heads the Judiciary, sits as Chair of the JBC and of the Presidential
of the arguments that the objection is related, more than anything else, to their Electoral Tribunal, presides over impeachment proceedings, and provides the
lack of trust in an appointment to be made by the incumbent President who will moral suasion and leadership that only the permanent mantle of the Chief Justice
soon be bowing out of office. They label the incumbent President’s act as a can bestow. EDSA II is just one of the many lessons from the past when the
“midnight appointment”—a term that has acquired a pejorative meaning in weightiest of issues were tackled and promptly resolved by the Court. Unseen by
contemporary society.686 the general public in all these was the leadership that was there to ensure that the
686 SUPREME COURT REPORTS ANNOTATED Court would act as one, in the spirit of harmony and stability although divergent
in their individual views, as the Justices individually make their contributions to
De Castro vs. Judicial and Bar Council (JBC)
the collegial result. To some, this leadership may only be symbolic, as the Court
As I intimated in my Separate Opinion, the imputation of distrust can be made has fully functioned in the past even with an incomplete membership or under an
against any appointing authority, whether outgoing or incoming. The incoming Acting Chief Justice. But as I said before, an incomplete Court “is not a whole
President himself will be before this Court if an election contest arises; any
President, past or future, would also naturally wish favorable outcomes in legal
Supreme Court; it will only be a Court with 14 members who would act and vote 689
on all matters before it.” To fully recall what I have said on this matter: VOL. 618, APRIL 20, 2010 689
“The importance of the presence of one Member of the Court can and should
never be underestimated, particularly on issues that may gravely affect the nation. De Castro vs. Judicial and Bar Council (JBC)
Many a case has been won or lost on the basis of one vote. On an issue of the the history of provisions, including the intents behind them, are primarily
constitutionality of a law, treaty or statute, a tie vote—which is possible in a 14 important to ascertain the purposes the provisions serve.
member court—means that the constitutionality is upheld. This was our lesson From these perspectives and without denigrating the framers’ historical
in Isagani Cruz v. DENR Secretary. contributions, I say that it is the Constitution that now primarily speaks to us in
More than the vote, Court deliberation is the core of the decision-making this case and what we hear are its direct words, not merely the recorded isolated
process and one voice is less is not only a vote less but a contributed opinion, an debates reflecting the personal intents of the constitutional commissioners as cited
observation, or a cautionary word less for the by the parties to fit their respective theories. The voice speaking the words of the
688 Constitution is our best guide, as these words will unalterably be there for us to
read in the context of their purposes and the nation’s needs and
688 SUPREME COURT REPORTS ANNOTATED
circumstances. This Concurring and Dissenting Opinion hears and listens to that
De Castro vs. Judicial and Bar Council (JBC) voice.
Court. One voice can be a big difference if the missing voice is that of the Chief The Valenzuela Decision
Justice. The ponencia’s ruling reversing Valenzuela, in my view, is out of place in the
Without meaning to demean the capability of an Acting Chief Justice, the present case, since at issue here is the appointment of the Chief Justice during the
ascendancy in the Court of a permanent sitting Chief Justice cannot be equaled. period of the election ban, not the appointment of lower court judges
He is the first among equals—a primus inter pares—who sets the tone for the that Valenzuela resolved. To be perfectly clear, the conflict in the constitutional
Court and the Judiciary, and who is looked up to on all matters, whether provisions is not confined to Article VII, Section 15 and Article VIII, Section 4(1)
administrative or judicial. To the world outside the Judiciary, he is the with respect to the appointment of Members of the Supreme Court; even before
personification of the Court and the whole Judiciary. And this is not surprising the Valenzuela ruling, the conflict already existed between Article VII, Section 15
since, as Chief Justice, he not only chairs the Court en banc, but chairs as well the and Article VIII, Section 9—the provision on the appointment of the justices and
Presidential Electoral Tribunal that sits in judgment over election disputes judges of courts lower than the Supreme Court. After this Court’s ruling in Valen-
affecting the President and the Vice-President. Outside of his immediate Court zuela, no amount of hairsplitting can result in the conclusion that Article VII,
duties, he sits as Chair of the Judicial and Bar Council, the Philippine Judicial Section 15 applied the election ban over the whole Judiciary, including the
Academy and, by constitutional command, presides over the impeachment of the Supreme Court, as the facts and the fallo of Valenzuela plainly spoke of the
President. To be sure, the Acting Chief Justice may be the ablest, but he is not the objectionable appointment of two Regional Trial Court judges. To
Chief Justice without the mantle and permanent title of the Office, and even his reiterate, Valenzuela only resolved the conflict between Article VII, Section 15 and
presence as Acting Chief Justice leaves the Court with one member less. Sadly, appointments to the Judiciary under Article VIII, Section 9.690
this member is the Chief Justice; even with an Acting Chief Justice, the Judiciary 690 SUPREME COURT REPORTS ANNOTATED
and the Court remains headless.”35
Given these views, I see no point in re-discussing the finer points of technical De Castro vs. Judicial and Bar Council (JBC)
interpretation and their supporting latin maxims that I have addressed in my If Valenzuela did prominently figure at all in the present case, the prominence
Separate Opinion and now feel need no further elaboration; maxims can be found can be attributed to the petitioners’ mistaken reading that this case is primary
to serve a pleader’s every need and in any case are the last interpretative tools in authority for the dictum that Article VII, Section 15 completely bans all
constitutional interpretation. Nor do I see any point in discussing arguments based appointments to the Judiciary, including appointments to the Supreme Court,
on the intent of the framers of the Constitution now cited by the parties in the during the election period up to the end of the incumbent President’s term.
contexts that would serve their own ends. As may be evident in these discussions, In reality, this mistaken reading is an obiter dictum in Valenzuela, and hence,
other than the texts of the disputed provisions, I prefer to examine their purposes cannot be cited for its primary precedential value. This legal situation still holds
and the consequences of their application, understood within the context of true as Valenzuela was not doctrinally reversed as its proposed reversal was
democratic values. Past precedents are equally invaluable for the lead, order, and supported only by five (5) out of the 12 participating Members of the Court. In
stability they contribute, but only if they are in point, certain, and still alive to other words, this ruling on how Article VII, Section 15 is to be interpreted in
current realities, while relation with Article VIII, Section 9, should continue to stand unless otherwise
expressly reversed by this Court.
_______________ But separately from the mistaken use of an obiter ruling as primary authority,
I believe that I should sound the alarm bell about the Valenzuela ruling in light of
35 Separate Opinion, p. 32. a recent vacancy in the position of Presiding Justice of the Sandiganbayan
resulting from Presiding Justice Norberto Geraldez’s death soon after we issued
the decision in the present case. Reversing the Valenzuela ruling now, in the De Castro vs. Judicial and Bar Council (JBC)
absence of a properly filed case addressing an appointment at this time to the tion, under the Constitution, of recommending nominees for the position of Chief
Sandiganbayan or to any other vacancy in the lower courts, will be an irregular Justice. Thus, I vote to deny with finality the Tolentino and Soriano motions for
ruling of the first magnitude by this Court, as it will effectively be a shortcut reconsideration.
that lifts the election ban on appointments to the lower courts without the benefit The other motions for reconsideration in so far as they challenge the conclusion
of a case whose facts and arguments would directly confront the continued validity that the President can appoint the Chief Justice even during the election period
of the Valenzuela ruling. This is especially so after we have placed the Court on are likewise denied with finality for lack of merit, but are granted in so far as they
notice that a reversal of Valenzuela is uncalled for because its ruling is not the support the continued validity of the ruling of this Court in In Re: Valenzuela and
litigated issue in this case. Vallarta, A.M. No. 98-5-01-SC, November 9, 1998.
In any case, let me repeat what I stressed in my Separate Opinion My opinion on the Mendoza petition stands.
about Valenzuela which rests on the reasoning that the evils Section 15 seeks to Motions for Reconsideration denied.
remedy—vote buying, midnight Notes.—Stare decisis simply means that for the sake of certainty, a conclusion
691 reached in one case should be applied to those that follow if the facts are
VOL. 618, APRIL 20, 2010 691 substantially the same, even though the parties may be different. Where the same
De Castro vs. Judicial and Bar Council (JBC) questions relating to the same event have been put forward by parties similarly
appointments and partisan reasons to influence the elections—exist, thus situated as in a previous case litigated and decided by a competent court, a rule
justifying an election appointment ban. In particular, the “midnight appointment” of stare decisis is a bar to any attempt to relitigate the same issue. (Grand
justification, while fully applicable to the more numerous vacancies at the lower Placement and General Services Corporation vs. Court of Appeals, 481 SCRA
echelons of the Judiciary (with an alleged current lower court vacancy level of 537 189; United Planters Sugar Milling Co., Inc. (UPSUMCO) vs. Court of
Appeals, 508 SCRA 310 [2006])
or a 24.5% vacancy rate), should not apply to the Supreme Court which has only a
total of 15 positions that are not even vacated at the same time. The most number The doctrine of adherence to precedents or stare decisis was applied by the
of vacancies for any one year occurred only last year (2009) when seven (7) English courts and was later adopted by the United States. (Ting vs. Velez-
positions were vacated by retirement, but this vacancy rate is not expected to be Ting, 582 SCRA 694 [2009])
replicated at any time within the next decade. Thus “midnight appointments” to ——o0o——
the extent that they were understood in Aytona36 will not occur in the vacancies of
this Court as nominations to its vacancies are all processed through the JBC under
the public’s close scrutiny. As already discussed above, the institutional integrity
of the Court is hardly an issue. If at all, only objections personal to the individual
Members of the Court or against the individual applicants can be made, but these
are matters addressed in the first place by the JBC before nominees are submitted.
There, too, are specific reasons, likewise discussed above, explaining why the G.R. No. 203372 June 16, 2015
election ban should not apply to the Supreme Court. These exempting reasons, of
course, have yet to be shown to apply to the lower courts. Thus, on the whole, the ATTY. CHELOY E. VELICARIA-GARAFIL, Petitioner,
reasons justifying the election ban in Valenzuela still obtain in so far as the lower vs.
courts are concerned, and have yet to be proven otherwise in a properly filed case. OFFICE OF THE PRESIDENT and HON. SOLICITOR GENERAL JOSE ANSELMO
Until then, Valenzuela, except to the extent that it mentioned Section 4(1), I. CADIZ, Respondents.
should remain an authoritative ruling of this Court.
x-----------------------x
CONCLUSION
G.R. No. 206290
In light of these considerations, a writ of prohibition cannot issue to prevent
the JBC from performing its principal func- ATTY. DINDO G. VENTURANZA, Petitioner,
vs.
_______________ OFFICE OF THE PRESIDENT, LEILA M. DE LIMA, in her capacity as the Secretary
of the Department of Justice, CLARO A. ARELLANO, in his capacity as the
36 Aytona v. Castillo, G.R. No. 19315, January 19, 1962, 4 SCRA 1. Prosecutor General, and RICHARD ANTHONY D. FADULLON, in his capacity as
692 the Officer-in-Charge of the Office of the City Prosecutor of Quezon
City,Respondents.
692 SUPREME COURT REPORTS ANNOTATED
x-----------------------x Facts of the Cases

G.R. No. 209138 Prior to the conduct of the May 2010 elections, then President Gloria Macapagal-
Arroyo (President Macapagal-Arroyo) issued more than 800 appointments to various
IRMA A. VILLANUEVA and FRANCISCA B. ROSQUITA, Petitioners, positions in several government offices.
vs.
COURT OF APPEALS and THE OFFICE OF THE PRESIDENT, Respondents. The ban on midnight appointments in Section 15, Article VII of the 1987 Constitution
reads:
x-----------------------x
Two months immediately before the next presidential elections and up to the end of
G.R. No. 212030 his term, a President or Acting President shall not make appointments, except
temporary appointments to executive positions when continued vacancies therein will
prejudice public service or endanger public safety.
EDDIE U. TAMONDONG, Petitioner,
vs.
EXECUTIVE SECRETARY PAQUITO N. OCHOA, JR., Respondent. Thus, for purposes of the 2010 elections, 10 March 2010 was the cutoff date for valid
appointments and the next day, 11 March 2010, was the start of the ban on midnight
appointments. Section 15, Article VII of the 1987 Constitution recognizes as an
DECISION exception to the ban on midnight appointments only "temporary appointments to
executive positions when continued vacancies therein will prejudice public service or
CARPIO, J.: endanger public safety." None of the petitioners claim that their appointments fall under
this exception.
The present consolidated cases involve four petitions: G.R. No. 203372 with Atty.
Cheloy E. Velicaria-Garafil (Atty. Velicaria-Garafil), who was appointed State Solicitor Appointments
II at the Office of the Solicitor General (OSG), as petitioner; G.R. No. 206290 with Atty.
Dindo G. Venturanza (Atty. Venturanza), who was appointed Prosecutor IV (City G.R. No. 203372
Prosecutor) of Quezon City, as petitioner; G.R. No. 209138 with Irma A. Villanueva
(Villanueva), who was appointed Administrator for Visayas of the Board of
Administrators of the Cooperative Development Authority (CDA), and Francisca B. The paper evidencing Atty. Velicaria-Garafil's appointment as State Solicitor II at the
Rosquita (Rosquita), who was appointed Commissioner of the National Commission OSG was dated 5 March 2010.13 There was a transmittal letter dated 8 March 2010 of
of Indigenous Peoples (NCIP), as petitioners; and G.R. No. 212030 with Atty. Eddie U. the appointment paper from the Office of the President (OP), but this transmittal letter
Tamondong (Atty. Tamondong), who was appointed member of the Board of Directors was received by the Malacañang Records Office (MRO) only on 13 May 2010. There
of the Subic Bay Metropolitan Authority (SBMA), as petitioner. All petitions question was no indication as to the OSG's date of receipt of the appointment paper. On 19
the constitutionality of Executive Order No. 2 (EO 2) for being inconsistent with Section March 2010, the OSG's Human Resources Department called up Atty. Velicaria-Garafil
15, Article VII of the 1987 Constitution. to schedule her oath-taking. Atty. Velicaria-Garafil took her oath of office as State
Solicitor II on 22 March 2010 and assumed her position on 6 April 2010.
Petitioners seek the :reversal of the separate Decisions of the Court of Appeals (CA)
that dismissed their petitions and upheld the constitutionality of EO 2. G.R. No. 203372 G.R. No. 206290
filed by Atty. Velicaria-Garafil is a Petition for Review on Certiorari,1 assailing the
Decision2 dated 31 August 2012 of the CA in CA-G.R. SP No. 123662. G.R. No. The paper evidencing Atty. Venturanza's appointment as Prosecutor IV (City
206290 filed by Atty. Venturanza is a Petition for Review on Certiorari, 3 assailing the Prosecutor) of Quezon City was dated 23 February 2010.14 It is apparent, however,
Decision4 dated 31 August 2012 and Resolution5 dated 12 March 2013 of the CA in that it was only on 12 March 2010 that the OP, in a letter dated 9 March 2010,
CA-G.R. SP No. 123659. G.R. No. 209138 filed by Villanueva and Rosquita is a transmitted Atty. Venturanza's appointment paper to then Department of Justice (DOJ)
Petition for Certiorari,6 seeking to nullify the Decision7 dated 28 August 2013 of the CA Secretary Alberto C. Agra.15 During the period between 23 February and 12 March
in CA-G.R. SP Nos. 123662, 123663, and 123664.8 Villanueva and Rosquita filed a 2010, Atty. Venturanza, upon verbal advice from Malacañang of his promotion but
Petition-in-Intervention in the consolidated cases before the CA. G.R. No. 212030 is a without an official copy of his appointment paper, secured clearances from the Civil
Petition for Review on Certiorari,9 assailing the Decision10 dated 31 August 2012 of the Service Commission (CSC),16 Sandiganbayan,17 and the DOJ.18 Atty. Venturanza
CA in CAG.R. SP No. 123664 and Resolution11 dated 7 April 2014 of the CA in CAG.R. took his oath of office on 15 March 2010, and assumed office on the same day.
SP Nos. 123662, 123663, and 123664.12
G.R. No. 209138
The paper evidencing Villanueva's appointment as Administrator for Visayas of the EXECUTIVE ORDER NO. 2
Board of Administrators of the CDA was dated 3 March 2010.19 There was no
transmittal letter of the appointment paper from the OP. Villanueva took her oath of RECALLING, WITHDRAWING, AND REVOKING APPOINTMENTS ISSUED BY
office on 13 April 2010. THE PREVIOUS ADMINISTRATION IN VIOLATION OF THE CONSTITUTIONAL
BAN ON MIDNIGHT APPOINTMENTS, AND FOR OTHER PURPOSES.
The paper evidencing Rosquita's appointment as Commissioner, representing Region
I and the Cordilleras, of the NCIP was dated 5 March 2010.20 Like Villanueva, there WHEREAS, Sec. 15, Article VII of the 1987 Constitution provides that "Two months
was no transmittal letter of the appointment paper from the OP. Rosquita took her oath immediately before the next presidential elections and up to the end of his term, a
of office on 18 March 2010. G.R. No. 212030 President or Acting President shall not make appointments, except temporary
appointments to executive positions when continued vacancies therein will prejudice
The paper evidencing Atty. Tamondong's appointment as member, representing the public service or endanger public safety."; WHEREAS, in the case of "In re:
private sector, of the SBMA Board of Directors was dated 1 March 2010. 21 Atty. Appointments dated March 30, 1998 of Hon. Mateo Valenzuela and Hon. Vallarta as
Tamondong admitted that the appointment paper was received by the Office of the Judges of the Regional Trial Court of Branch 62 of Bago City and Branch 24 of
SBMA Chair on 25 March 201022 and that he took his oath of office on the same Cabanatuan City, respectively" (A.M. No. 98-5-01-SC Nov. 9, 1998), the Supreme
day.23 He took another oath of office on 6 July 2010 as "an act of extra caution because Court interpreted this provision to mean that the President is neither required to make
of the rising crescendo of noise from the new political mandarins against the so-called appointments nor allowed to do so during the two months immediately before the next
'midnight appointments."'24 presidential elections and up to the end of her term. The only known exceptions to this
prohibition are (1) temporary appointments in the executive positions when continued
To summarize, the pertinent dates for each petitioner are as follows: vacancies will prejudice public service or endanger public safety and in the light of the
recent Supreme Court decision in the case of De Castro, et al. vs. JBC and PGMA,
G.R. No. 191002, 17 March 2010, (2) appointments to the Judiciary;
Date of Appointment Date of Transmittal Date of Receipt by Date of Oath of Assumption of
Letter Letter MRO Office Office
WHEREAS, Section 261 of the Omnibus Election Code provides that:

"Section 261. Prohibited Acts.-The following shall be guilty of an election offense:


- 5 March 2010 8 March 2010 13 May 2010 22 March 2010 6 April 2010

(g) Appointments of new employees, creation of new position, promotion, or


giving salary increases. - During the period of forty-five days before a regular
23 February 2010 9 March 2010 12 March 2010 15 March 2010 15 March 2010 and thirty days before a special election.
election

3 March 2010 4 May 2010 13 April 2010 (1) Any head, official or appointing officer of a government office, agency or
instrumentality, whether national or local, including government-owned or
controlled corporations, who appoints or hires any new employee, whether
5 March 2010 13 May 2010 18 March 2010 provisional, temporary or casual, or creates and fills any new position, except
upon prior authority to the Commission. The Commission shall not grant the
authority sought unless it is satisfied that the position to be filled is essential
25 March 2010 and to the proper functioning of the office or agency concerned, and that the
1 March 2010
6 July 2010 position shall not be filled in a manner that may influence the election.

Issuance of EO 2 As an exception to the foregoing provisions, a new employee may be


appointed in the case of urgent need:
On 30 June 2010, President Benigno S. Aquino III (President Aquino) took his oath of
office as President of the Republic of the Philippines. On 30 July 2010, President Provided, however, that notice of the appointment shall be given to the
Aquino issued EO 2 recalling, withdrawing, and revoking appointments issued by Commission within three days from the date of the appointment. Any
President Macapagal-Arroyo which violated the constitutional ban on midnight appointment or hiring in violation of this provision shall be null and void.
appointments.
(2) Any government official who promotes or gives any increase of salary or
The entirety of EO 2 reads: remuneration or privilege to any government official or employee, including
those in government-owned or controlled corporations.";
WHEREAS, it appears on record that a number of appointments were made on or SECTION 5. Separability Clause. - If any section or provision of this executive order
about 10 March 2010 in complete disregard of the intent and spirit of the constitutional shall be declared unconstitutional or invalid, the other sections or provision not affected
ban on midnight appointment and which deprives the new administration of the power thereby shall remain in full force and effect.
to make its own appointment;
SECTION 6. Effectivity. - This Executive order shall take effect immediately.
WHEREAS, based on established jurisprudence, an appointment is deemed complete
only upon acceptance of the appointee; DONE in the City of Manila, this 30th day of July, in the year Two Thousand and Ten.

WHEREAS, in order to strengthen the civil service system, it is necessary to uphold By the President:
the principle that appointments to the civil service must be made on the basis of merit
and fitness, it is imperative to recall, withdraw, and revoke all appointments made in
violation of the letter and spirit of the law; (Sgd.) PAQUITO N. OCHOA, JR.
Executive Secretary25
NOW, THEREFORE, I, BENIGNO S. AQUINO III, by virtue of the powers vested in me
by the Constitution as President of the Philippines, do hereby order and direct that: (Sgd.) BENIGNO S. AQUINO III

SECTION 1. Midnight Appointments Defined. - The following appointments made by Effect of the Issuance of EO 2
the former President and other appointing authorities in departments, agencies,
offices, and instrumentalities, including government-owned or controlled corporations, G.R. No. 203372
shall be considered as midnight appointments:
On 5 August 2010, Jose Anselmo Cadiz assumed office as Solicitor General (Sol. Gen.
(a) Those made on or after March 11, 2010, including all appointments Cadiz). On 6 August 2010, Sol. Gen. Cadiz instructed a Senior Assistant Solicitor
bearing dates prior to March 11, 2010 where the appointee has accepted, or General to inform the officers and employees affected by EO 2 that they were
taken his oath, or assumed public office on or after March 11, 2010, except terminated from service effective the next day.
temporary appointments in the executive positions when continued vacancies
will prejudice public service or endanger public safety as may be determined Atty. Velicaria-Garafil reported for work on 9 August 2010 without any knowledge of
by the appointing authority. her termination. She was made to return the office-issued laptop and cellphone, and
was told that her salary ceased as of 7 August 2010. On 12 August 2010, Atty.
(b) Those made prior to March 11, 2010, but to take effect after said date or Velicaria-Garafil was informed that her former secretary at the OSG received a copy
appointments to office that would be vacant only after March 11, 2010. of a memorandum on her behalf. The memorandum, dated 9 August 2010, bore the
subject "Implementation of Executive Order No. 2 dated 30 July 2010" and was
(c) Appointments and promotions made during the period of 45 days prior to addressed to the OSG's Director of Finance and Management Service.
the May 10, 2010 elections in violation of Section 261 of the Omnibus Election
Code. Atty. Velicaria-Garafil filed a petition for certiorari (G.R. No. 193327) before this Court
on 1 September 2010. The petition prayed for the nullification of EO 2, and for her
SECTION 2. Recall, Withdraw, and Revocation of Midnight Appointments. Midnight reinstatement as State Solicitor II without loss of seniority, rights and privileges, and
appointments, as defined under Section 1, are hereby recalled, withdrawn, and with full backwages from the time that her salary was withheld. 26
revoked. The positions covered or otherwise affected are hereby declared vacant.
G.R. No. 206290
SECTION 3. Temporary designations. - When necessary to maintain efficiency in
public service and ensure the continuity of government operations, the Executive On 1 September 2010, Atty. Venturanza received via facsimile transmission an
Secretary may designate an officer-in-charge (OIC) to perform the duties and undated copy of DOJ Order No. 556. DOJ Order No. 556, issued by DOJ Secretary
discharge the responsibilities of any of those whose appointment has been recalled, Leila M. De Lima (Sec. De Lima), designated Senior Deputy State Prosecutor Richard
until the replacement of the OIC has been appointed and qualified. Anthony D. Fadullon (Pros. Fadullon) as Officer-in-Charge of the Office of the City
Prosecutor in Quezon City. In a letter to Sec. De Lima dated 15 September 2010, Atty.
SECTION 4. Repealing Clause. - All executive issuances, orders, rules and regulations Venturanza asked for clarification of his status, duties, and functions since DOJ Order
or part thereof inconsistent with the provisions of this Executive Order are hereby No. 556 did not address the same. Atty. Venturanza also asked for a status quo ante
repealed or modified accordingly. order to prevent Pros. Fadullon ·from usurping the position and functions of the City
Prosecutor of Quezon City. Atty. Venturanza also wrote a letter to President Aquino 3 . Whether the appointments of the petitioners and intervenors were made
on the same day, and sought reaffirmation of his promotion as City Prosecutor of with undue haste, hurried maneuvers, for partisan reasons, and not in
Quezon City. accordance with good faith; and

On 6 October 2010, Atty. Venturanza received a letter dated 25 August 2010 from Sec. 4. Whether EO 2 violated the Civil Service Rules on Appointment.33
De Lima which directed him to relinquish the office to which he was appointed, and to
cease from performing its functions. This Court gave the CA the authority to resolve all pending matters and applications,
and to decide the issues as if these cases were originally filed with the CA.
Atty. Venturanza filed a Petition for Certiorari, Prohibition, Mandamus with Urgent
Prayer for Status Quo Ante Order, Temporary Restraining Order and/or Preliminary Rulings of the CA
Mandatory Injunction (G.R. No. 193 867) before this Court on 14 October 2010. 27
Even though the same issues were raised in the different petitions, the CA promulgated
G.R. No. 209138 separate Decisions for the petitions. The CA consistently ruled that EO 2 is
constitutional. The CA, however, issued different rulings as to the evaluation of the
The OP withheld the salaries of Villanueva and Rosquita on the basis of EO 2. On 3 circumstances of petitioners' appointments. In the cases of Attys. Velicaria-Garafil and
August 2010, Villanueva and Rosquita sought to intervene in G.R. No. 192991. 28 On 1 Venturanza, the CA stated that the OP should consider the circumstances of their
October 2010, Executive Secretary Paquito N. Ochoa, Jr. revoked Rosquita's appointments. In the cases of Villanueva, Rosquita, and Atty. Tamondong, the CA
appointment as NCIP Commissioner.29 On 13 October 2010, Villanueva and Rosquita explicitly stated that · the revocation of their appointments was proper because they
notified this Court that they wanted to intervene in Atty. Tamondong's petition (G.R. were midnight appointees.
No. 192987) instead.
G.R. No. 203372 (CA-G.R. SP No. 123662)
G.R. No. 212030
The CA promulgated its Decision in CA-G.R. SP No. 123662 on 31 August 2012. The
Atty. Tamondong was removed from the SBMA Board of Directors on 30 July 2010. CA ruled that EO 2 is not unconstitutional. However, the CA relied on Sales v.
He filed a petition for prohibition, declaratory relief and preliminary injunction with Carreon34 in ruling that the OP should evaluate whether Atty. Velicaria-Garafil's
prayer for temporary restraining order (G.R. No. 192987) before this Court on 9 August appointment had extenuating circumstances that might make it fall outside the ambit
2010. The petition prayed for the prohibition of the implementation of EO 2, the of EO 2.
declaration of his appointment as legal, and the declaration of EO 2 as
unconstitutional.30 The dispositive portion of the CA's Decision reads:

Referral to CA WHEREFORE, the petition for certiorari and mandamus [is] DENIED.

There were several petitions31 and motions for intervention32 that challenged the Executive Order No. 2, dated July 30, 2010, is NOT unconstitutional.
constitutionality of EO 2.
The issue on whether or not to uphold petitioner's appointment as State Solicitor II at
On 31 January 2012, this Court issued a Resolution referring the petitions, motions for the OSG is hereby referred to the Office of the President which has the sole authority
intervention, as well as various letters, to the CA for further proceedings, including the and discretion to pass upon the same.
reception and assessment of the evidence from all parties. We defined the issues as
follows:
SO ORDERED.35
1. Whether the appointments of the petitioners and intervenors were midnight
appointments within the coverage of EO 2; G.R. No. 212030 (CA-G.R. SP No. 123664)

2. Whether all midnight appointments, including those of petitioners and On 31 August 2012, the CA promulgated its Decision in CA-G.R. SP No. 123664. The
intervenors, were invalid; dispositive portion reads as follows:

WHEREFORE, premises considered, the instant Petition is hereby DISMISSED.


Executive Order No. 2 is hereby declared NOT UNCONSTITUTIONAL. Accordingly,
the revocation of Atty. Eddie Tamondong's appointment as Director of Subic Bay Administrative Order No. 2's cancellation of the "midnight or last minute" appointments.
Metropolitan Authority is VALID for being a midnight appointment. We wrote:

SO ORDERED.39 x x x But the issuance of 350 appointments in one night and the planned induction of
almost all of them a few hours before the inauguration of the new President may, with
The Issues for Resolution some reason, be regarded by the latter as an abuse of Presidential prerogatives, the
steps taken being apparently a mere partisan effort to fill all vacant positions
irrespective of fitness and other conditions, and thereby to deprive the new
We resolve the following issues in these petitions: (1) whether petitioners' administration of an opportunity to make the corresponding appointments.
appointments violate Section 15, Article VII of the 1987 Constitution, and (2) whether
EO 2 is constitutional. Ruling of the Court
x x x Now it is hard to believe that in signing 350 appointments in one night, President
Garcia exercised such "double care" which was required and expected of him; and
The petitions have no merit. All of petitioners' appointments are midnight appointments therefore, there seems to be force to the contention that these appointments fall
and are void for violation of Section 15, Article VII of the 1987 Constitution. EO 2 is beyond the intent and spirit of the constitutional provision granting to the Executive
constitutional. Villanueva and Rosquita, petitioners in G.R. No. 209138, did not appeal authority to issue ad interim appointments.
the CA's ruling under Rule 45, but instead filed a petition for certiorari under Rule 65.
This procedural error alone warrants an outright dismissal of G.R. No. 209138. Even if
it were correctly filed under Rule 45, the petition should still be dismissed for being filed Under the circumstances above described, what with the separation of powers, this
out of time.40 There was also no explanation as to why they did not file a motion for Court resolves that it must decline to disregard the Presidential .Administrative Order
reconsideration of the CA's Decision. Midnight Appointments No. 2, cancelling such "midnight" or "last-minute" appointments.

This ponencia and the dissent both agree that the facts in all these cases show that Of course the Court is . aware of many precedents to the effect that once an
"none of the petitioners have shown that their appointment papers (and transmittal appointment has been issued, it cannot be reconsidered, specially where the
letters) have been issued (and released) before the ban." 41The dates of receipt by the appointee has qualified. But none of them refer to mass ad interim appointments (three
MRO, which in these cases are the only reliable evidence of actual transmittal of the hundred and fifty), issued in the last hours of an outgoing Chief Executive, in a setting
appointment papers by President Macapagal-Arroyo, are dates clearly falling during similar to that outlined herein. On the other hand, the authorities admit of exceptional
the appointment ban. Thus, this ponencia and the dissent both agree that all the circumstances justifying revocation and if any circumstances justify revocation, those
appointments in these cases are midnight appointments in violation of Section 15, described herein should fit the exception.
Article VII of the 1987 Constitution.
Incidentally, it should be stated that the underlying reason for denying the power to
Constitutionality of EO 2 revoke after the appointee has qualified is the latter's equitable rights. Yet it is doubtful
if such equity might be successfully set up in the present situation, considering the rush
conditional appointments, hurried maneuvers and other happenings detracting from
Based on prevailing jurisprudence, appointment to a government post is a process that that degree of good faith, morality and propriety which form the basic foundation of
takes several steps to complete. Any valid appointment, including one made under the claims to equitable relief. The appointees, it might be argued, wittingly or unwittingly
exception provided in Section 15, Article VII of the 1987 Constitution, must consist of cooperated with the stratagem to beat the deadline, whatever the resultant
the President signing an appointee's appointment paper to a vacant office, the official consequences to the dignity and efficiency of the public service. Needless to say, there
transmittal of the appointment paper (preferably through the MRO), receipt of the are instances wherein not only strict legality, but also fairness, justice and
appointment paper by the appointee, and acceptance of the appointment by the righteousness should be taken into account.43
appointee evidenced by his or her oath of office or his or her assumption to office.
During the deliberations for the 1987 Constitution, then Constitutional Commissioner
Aytona v. Castillo (Aytona)42 is the basis for Section 15, Article VII of the 1987 (now retired Supreme Court Chief Justice) Hilario G. Davide, Jr. referred to this Court's
Constitution. Aytona defined "midnight or last minute" appointments for Philippine ruling in Aytona and stated that his proposal seeks to prevent a President, whose term
jurisprudence.1âwphi1 President Carlos P. Garcia submitted on 29 December 1961, is about to end, from preempting his successor by appointing his own people to
his last day in office, 350 appointments, including that of Dominador R. Aytona for sensitive positions.
Central Bank Governor. President Diosdado P. Macapagal assumed office on 30
December 1961, and issued on 31 December 1961 Administrative Order No. 2
recalling, withdrawing, and cancelling all appointments made by President Garcia after MR. DAVIDE: The idea of the proposal is that about the end of the term of the
13 December 1961 (President Macapagal's proclamation date). President Macapagal President, he may prolong his rule indirectly by appointing people to these sensitive
appointed Andres V. Castillo as Central Bank Governor on 1 January 1962. This Court positions, like the commissions, the Ombudsman, the judiciary, so he could perpetuate
dismissed Aytona's quo warranto proceeding against Castillo, and upheld himself in power even beyond his term of office; therefore foreclosing the right of his
successor to make appointments to these positions. We should realize that the term indirectly after the end of his term. The 1986 Constitutional Commission installed a
of the President is six years and under what we had voted on, there is no reelection definite cut-off date as an objective and unbiased marker against which this once-in-
for him. Yet he can continue to rule the country through appointments made about the every-six-years prohibition should be measured.
end of his term to these sensitive positions.44
The dissent's assertion that appointment should be viewed in its narrow sense (and is
The 1986 Constitutional Commission put a definite period, or an empirical value, on not a process) only during the prohibited period is selective and time-based, and
Aytona's intangible "stratagem to beat the deadline," and also on the act of "preempting ignores well-settled jurisprudence. For purposes of complying with the time limit
the President's successor," which shows a lack of "good faith, morality and propriety." imposed by the appointment ban, the dissent' s position cuts short the appointment
Subject to only one exception, appointments made during this period are thus process to the signing of the appointment paper and its transmittal, excluding the
automatically prohibited under the Constitution, regardless of the appointee's receipt of the appointment paper and acceptance of the appointment by the appointee.
qualifications or even of the President's motives. The period for prohibited
appointments covers two months before the elections until the end of the President's The President exercises only one kind of appointing power. There is no need to
term. The Constitution, with a specific exception, ended the President's power to differentiate the exercise of the President's appointing power outside, just before, or
appoint "two months immediately before the next presidential elections." For an during the appointment ban. The Constitution allows the President to exercise the
appointment to be valid, it must be made outside of the prohibited period or, failing power of appointment during the period not covered by the appointment ban, and
that, fall under the specified exception. disallows (subject to an exception) the President from exercising the power of
appointment during the period covered by the appointment ban. The concurrence of
The dissent insists that, during the prohibited period, an appointment should be viewed all steps in the appointment process is admittedly required for appointments outside
in its "narrow sense." In its narrow sense, an appointment is not a process, but is only the appointment ban. There is no justification whatsoever to remove acceptance as a
an "executive act that the President unequivocally exercises pursuant to his requirement in the appointment process for appointments just before the start of the
discretion."45 The dissent makes acceptance of the appointment inconsequential. The appointment ban, or during the appointment ban in appointments falling within the
dissent holds that an appointment is void if the appointment is made before the ban exception. The existence of the appointment ban makes no difference in the power of
but the transmittal and acceptance are made after the ban. However, the dissent holds the President to appoint; it is still the same power to appoint. In fact, considering the
that an appointment is valid, or "efficacious," if the appointment and transmittal are purpose of the appointment ban, the concurrence of all steps in the appointment
made before the ban even if the acceptance is made after the ban. In short, the dissent process must be strictly applied on appointments made just before or during the
allows an appointment to take effect during the ban, as long as the President signed appointment ban.
and transmitted the appointment before the ban, even if the appointee never received
the appointment paper before the ban and accepted the appointment only during the In attempting to extricate itself from the obvious consequences of its selective
ban. application, the dissent glaringly contradicts itself:

The dissent's view will lead to glaring absurdities. Allowing the dissent's proposal that Thus, an acceptance is still necessary in order for the appointee to validly assume his
an appointment is complete merely upon the signing of an appointment paper and its post and discharge the functions of his new office, and thus make the appointment
transmittal, excluding the appointee's acceptance from the appointment process, will effective. There can never be an instance where the appointment of an incumbent will
lead to the absurdity that, in case of non-acceptance, the position is considered automatically result in his resignation from his present post and his subsequent
occupied and nobody else may be appointed to it. Moreover, an incumbent public assumption of his new position; or where the President can simply remove an
official, appointed to another public office by the President, will automatically be incumbent from his current office by appointing him to another one. I stress that
deemed to occupy the new public office and to have automatically resigned from his acceptance through oath or any positive act is still indispensable before any
first office upon transmittal of his appointment paper, even if he refuses to accept the assumption of office may occur.46 (Emphasis added)
new appointment. This will result in chaos in public service.
The dissent proposes that this Court ignore well-settled jurisprudence during the
Even worse, a President who is unhappy with an incumbent public official can simply appointment ban, but apply the same jurisprudence outside of the appointment ban.
appoint him to another public office, effectively removing him from his first office without
due process. The mere transmittal of his appointment paper will remove the public
official from office without due process and even without cause, in violation of the [T]he well-settled rule in our jurisprudence, that an appointment is a process that
Constitution. begins with the selection by the appointing power and ends with acceptance of the
appointment by the appointee, stands. As early as the 1949 case of Lacson v. Romero,
this Court laid down the rule that acceptance by the appointee is the last act needed
The dissent's proferred excuse (that the appointee is not alluded to in Section 15, to make an appointment complete. The Court reiterated this rule in the 1989 case of
Article VII) for its rejection of "acceptance by the appointee" as an integral part of the Javier v. Reyes. In the 1996 case of Garces v. Court of Appeals, this Court emphasized
appointment process ignores the reason for the limitation of the President's power to that acceptance by the appointee is indispensable to complete an appointment. The
appoint, which is .to prevent the outgoing President from continuing to rule the country
1999 case of Bermudez v. Executive Secretary, cited in the ponencia, affirms this Indeed, the power of choice is the heart of the power to appoint. Appointment involves
standing rule in our jurisdiction, to wit: an exercise of discretion of whom to appoint; it is not a ministerial act of issuing
appointment papers to the appointee. In other words, the choice of the appointee is a
"The appointment is deemed complete once the last act required of the appointing fundamental component of the appointing power.
authority has been complied with and its acceptance thereafter by the appointee in
order to render it effective."47 Hence, when Congress clothes the President with the power to appoint an officer, it
(Congress) cannot at the same time limit the choice of the President to only one
The dissent's assertion creates a singular exception to the well-settled doctrine that candidate. Once the power of appointment is conferred on the President, such
appointment is a process that begins with the signing of the appointment paper, conferment necessarily carries the discretion of whom to appoint. Even on the pretext
followed by the transmittal and receipt of the appointment paper, and becomes of prescribing the qualifications of the officer, Congress may not abuse such power as
complete with the acceptance of the appointment. The dissent makes the singular to divest the appointing authority, directly or indirectly, of his discretion to pick his own
exception that during the constitutionally mandated ban on appointments, acceptance choice. Consequently, when the qualifications prescribed by Congress can only be met
is not necessary to complete the appointment. The dissent gives no reason why this by one individual, such enactment effectively eliminates the discretion of the appointing
Court should make such singular exception, which is contrary to the express provision power to choose and constitutes an irregular restriction on the power of appointment. 50
of the Constitution prohibiting the President from making appointments during the ban.
The dissent's singular exception will allow the President, during the ban on Transmittal
appointments, to remove from office incumbents without cause by simply appointing
them to another office and transmitting the appointment papers the day before the ban It is not enough that the President signs the appointment paper. There should be
begins, appointments that the incumbents cannot refuse because their acceptance is evidence that the President intended the appointment paper to be issued. It could
not required during the ban. Adoption by this Court of the dissent's singular exception happen that an appointment paper may be dated and signed by the President months
will certainly wreak havoc on the civil service. before the appointment ban, but never left his locked drawer for the entirety of his term.
Release of the appointment paper through the MRO is an unequivocal act that signifies
The following elements should always concur in the making of a valid (which should the President's intent of its issuance.
be understood as both complete and effective) appointment: (1) authority to appoint
and evidence of the exercise of the authority; (2) transmittal of the appointment paper The MRO was created by Memorandum Order No. 1, Series of 1958, Governing the
and evidence of the transmittal; (3) a vacant position at the time of appointment; and Organization and Functions of the Executive Office and General Matters of Procedure
(4) receipt of the appointment paper and acceptance of the appointment by the Therein. Initially called the Records Division, the MRO functioned as an administrative
appointee who possesses all the qualifications and none of the disqualifications. The unit of the Executive Office. Memorandum Order No. 1 assigned the following
concurrence of all these elements should always apply, regardless of when the functions:
appointment is made, whether outside, just before, or during the appointment ban.
These steps in the appointment process should always concur and operate as a single
process. There is no valid appointment if the process lacks even one step. And, unlike a. Receive, record and screen all incoming correspondence, telegrams,
the dissent's proposal, there is no need to further distinguish between an effective and documents and papers, and
an ineffective appointment when an appointment is valid.
(1) Forward those of a personal and unofficial nature to the
Appointing Authority President's Private Office; and

The President's exercise of his power to appoint officials is provided for in the (2) Distribute those requiring action within the Office or requiring
Constitution and laws.48 Discretion is an integral part in the exercise of the power of staff work prior to presentation to the President to the appropriate
appointment.49 Considering that appointment calls for a selection, the appointing units within the Office.
power necessarily exercises a discretion. According to Woodbury, J., "the choice of a
person to fill an office constitutes the essence of his appointment," and Mr. Justice b. Follow up on correspondence forwarded to entities outside the Office to
Malcolm adds that an "[a]ppointment to office is intrinsically an executive act involving assure that prompt replies are made and copies thereof furnished the Office.
the exercise of discretion." In Pamantasan ng Lungsod ng Maynila v. Intermediate
Appellate Court we held: c. Dispatch outgoing correspondence and telegrams.

The power to appoint is, in essence, discretionary. The appointing power has the right d. Have custody of records of the Office, except personal papers of the
of choice which he may exercise freely according to his judgment, deciding for himself President, and keep them in such condition as to meet the documentary and
who is best qualified among those who have the necessary qualifications and reference requirements of the Office.
eligibilities. It is a prerogative of the appointing power x x x x
e. Keep and maintain a filing and records system for acts, memoranda, 7. To be able to represent the OP and OP officials in response to
orders, circulars, correspondence and other documents affecting the Office Subpoena Duces Tecum and Testificandum served by courts and
for ready reference and use. other investigating bodies.52

f. Issue certified true copies of documents on file in the Division m accordance For purposes of verification of the appointment paper's existence and authenticity, the
with prevailing standard operating procedure. appointment paper must bear the security marks (i.e., handwritten signature of the
President, bar code, etc.) and must be accompanied by a transmittal letter from the
g. Keep a separate record of communications or documents of confidential MRO.
nature.
The testimony of Mr. Mariani to Dimaandal, Director IV of the MRO, underscores the
h. Have custody of the Great Seal of the Republic of the Philippines. purpose of the release of papers through his office.

i. Prepare and submit to the approving authority, periodic disposition Q: What are the functions of the MRO?
schedules of non-current records which have no historical, legal and/or claim
value. A: The MRO is mandated under Memorandum Order No. 1, series of 1958 to (1)
receive, record, and screen all incoming correspondence, telegrams, documents, and
j. With the approval of the Executive Secretary, assist other offices in the papers; (2) follow up on correspondence forwarded to entities outside the Office of the
installation or improvement of their records management system; and President ("OP") to assure that prompt replies are made and copies thereof furnished
the OP; (3) timely dispatch all outgoing documents and correspondence; (4) have
custody of records of the OP, except personal papers of the President, and keep them
k. Give instructions or deliver lectures and conduct practical training to in- in such condition as to meet the documentary and reference requirements of the Office;
service trainees from other offices and to students from educational (5) keep and maintain a filing and records system for Acts, Memoranda, Orders,
institutions on records management.51 Circulars, correspondence, and other pertinent documents for ready reference and
use; ( 6) issue certified copies of documents on file as requested and in accordance
The Records Division was elevated to an Office in 1975, with the addition of with prevailing standard operating procedures; (7) maintain and control vital
the following functions: documents and essential records to support the OP in its day-to-day activities; (8)
monitor the flow of communications from the time of receipt up to their dispatch; and
1. Maintain and control vital documents and essential records to (9) other related functions.
support the functions of the OP in its day to day activities;
xxxx
2. Monitor the flow of communications' from their time of receipt up
to their dispatch; Q: As you previously mentioned, the MRO is the custodian of all documents emanating
from Malacañang pursuant to its mandate under Memorandum Order No. 1, Series of
3. Service the documentary, information and reference 1958. Is the MRO required to follow a specific procedure in dispatching outgoing
requirements of top management and action officers of the OP, and documents?
the reference and research needs of other government agencies
and the general public; A: Yes.

4. Ensure the proper storage, maintenance, protection and Q: Is this procedure observed for the release of an appointment paper signed by the
preservation of vital and presidential documents, and the prompt President? A: Yes. It is observed for the release of the original copy of the appointment
disposal of obsolete and valueless records; paper signed by the President.

5. Effect the prompt publication/dissemination of laws, presidential Q: Can you briefly illustrate the procedure for the release of the original copy of the
issuances and classified documents; appointment paper signed by the President?

6. Provide computerized integrated records management support A: After an appointment paper is signed by the President, the Office of the Executive
services for easy reference and retrieval of data and information; Secretary (OES) forwards the appointment paper bearing the stamp mark, barcode,
and and hologram of the Office of the President, together with a transmittal letter, to the
MRO for official release. Within the same day, the MRO sends the original copy of the central repository of said documents. Not even the OES can issue a certified true copy
appointment paper together with the transmittal letter and a delivery receipt which of documents prepared by them.
contains appropriate spaces for the name of the addressee, the date released, and the
date received by the addressee. Only a photocopy of the appointment is retained for Q: Why do you say that, Mr. Witness?
the MRO's official file.
A: Because the MRO is the so-called "gatekeeper" of the Malacañang Palace. All
Q: What is the basis for the process you just discussed? incoming and outgoing documents and correspondence must pass through the MRO.
As the official custodian, the MRO is in charge of the official release of documents.
A: The Service Guide of the MRO.
Q: What if an appointment paper was faxed by the Office of the Executive Secretary
xxxx to the appointee, is that considered an official release by the MRO?

Q: What is the legal basis for the issuance of the MRO Service Guide, if any? A: No. It is still the MRO which will furnish the original copy of the appointment paper
to the appointee. That appointment paper is, at best, only an "advanced copy."
A: The MRO Service Guide was issued pursuant to Memorandum Circular No. 35,
Series of 2003 and Memorandum Circular No. 133, Series of 2007. Q: Assuming the MRO has already received the original appointment paper signed by
the President together with the transmittal letter prepared by the OES, you said that
xxxx the MRO is bound to transmit these documents immediately, that is, on the same day?

Q: Do you exercise any discretion in the release of documents forwarded to the MRO A: Yes.
for transmittal to various offices?
Q: Were there instances when the President, after the original appointment paper has
A: No. We are mandated to immediately release all documents and correspondence already been forwarded to the MRO, recalls the appointment and directs the MRO not
forwarded to us for transmittal. to transmit the documents?

Q: If a document is forwarded by the OES to the MRO today, when is it officially A: Yes, there were such instances.
released by the MRO to the department or agency concerned?
Q: How about if the document was already transmitted by the MRO, was there any
A: The document is released within the day by the MRO if the addressee is within instance when it was directed to recall the appointment and retrieve the documents
Metro Manila. For example, in the case of the appointment paper of Dindo Venturanza, already transmitted? A: Yes, but only in a few instances. Sometimes, when the MRO
the OES forwarded to the MRO on March 12, 2010 his original appointment paper messenger is already in transit or while he is already in the agency or office concerned,
dated February 23, 2010 and the transmittal letter dated March 9, 2010 prepared by we get a call to hold the delivery. Q: You previously outlined the procedure governing
the OES. The MRO released his appointment paper on the same day or on March 12, the transmittal of original copies of appointment papers to the agency or office
2010, and was also received by the DOJ on March 12, 2010 as shown by the delivery concerned. Would you know if this procedure was followed by previous
receipt. administrations?

Q: What is the effect if a document is released by an office or department within A: Yes. Since I started working in the MRO in 1976, the procedure has been followed.
Malacañan without going through the MRO? However, it was unusually disregarded when the appointments numbering more than
800 were made by then President Arroyo in March 2010. The MRO did not even know
about some of these appointments and we were surprised when we learned about
A: If a document does not pass through the MRO contrary to established procedure, them in the newspapers.
the MRO cannot issue a certified true copy of the same because as far as the MRO is
concerned, it does not exist in our official records, hence, not an official document from
the Malacañang. There is no way of verifying the document's existence and Q: You mentioned that then President Arroyo appointed more than 800 persons in the
authenticity unless the document is on file with the MRO even if the person who claims month of March alone. How were you able to determine this number?
to have in his possession a genuine document furnished to him personally by the
President. As a matter of fact, it is only the MRO which is authorized to issue certified A: My staff counted all the appointments made by then President An-oyo within the
true copies of documents emanating from Malacañan being the official custodian and period starting January 2009 until June 2010.
Q: What did you notice, if any, about these appointments? Q: So, insofar as these transmittal letters and appointment papers apparently released
by the OES are concerned, what is the actual date when the agency or the appointee
A: There was a steep rise in the number of appointments made by then President concerned received it?
Arroyo in the month of March 2010 compared to the other months.
A: I cannot answer. There is no way of knowing when they were actually received
Q: Do you have any evidence to show this steep rise? because the date and time were deliberately or inadvertently left blank.

A: Yes. I prepared a Certification showing these statistics and the graphical Q: Can we say that the date appearing on the face of the transmittal letters or the
representation thereof. appointment papers is the actual date when it was released by the OES?

Q: If those documents will be shown to you, will you be able to recognize them? A: We cannot say that for sure. That is why it is very unusual that the person who
received these documents did not indicate the date and time when it was received
because these details are very important.53
A: Yes.
The MRO's exercise of its mandate does not prohibit the President or the Executive
Q: I am showing you a Certification containing the number of presidential appointees Secretary from giving the appointment paper directly to the appointee. However, a
per month since January 2009 until June 2010, and a graphical representation thereof. problem may arise if an appointment paper is not coursed through the MRO and the
Can you go over these documents and tell us the relation of these documents to the appointment paper is lost or the appointment is questioned. The appointee would then
ones you previously mentioned? have to prove that the appointment paper was directly given to him.

A: These are [sic] the Certification with the table of statistics I prepared after we Dimaandal's counsel made this manifestation about petitioners' appointment papers
counted the appointments, as well as the graph thereof. and their transmittal:

xxxx Your Honors, we respectfully request for the following markings to be made:

Q: Out of the more than 800 appointees made in March 2010, how many appointment 1. A) The Transmittal Letter pertinent to the appointment of petitioner DINDO
papers and transmittal letters were released through the MRO? VENTURANZA dated March 9, 2010 as Exhibit "2-F" for the respondents;

A: Only 133 appointment papers were released through the MRO. B) The delivery receipt attached in front of the letter bearing the date
March 12, 2010 as Exhibit "2-F-l";
Q: In some of these transmittal letters and appointment papers which were not
released through the MRO but apparently through the OES, there were portions on the C) The Appointment Paper of DINDO VENTURANZA dated
stamp of the OES which supposedly indicated the date and time it was actually February 23, 2010 as Exhibit "2-G" for the respondents;
received by the agency or office concerned but were curiously left blank, is this regular
or irregular?
2. A) The Transmittal Letter pertinent to the appointment of CHELOY E.
VELICARIA-GARAFIL turned over to the MRO on May 13, 2010 consisting of
A: It is highly irregular. seven (7) pages as Exhibits "2-H," "2-H-l," "2-H-2," "2-H-3," "2-H-4," "2-H-5,"
and "2-H-6" respectively for the respondents;
Q: Why do you say so?
i. The portion with the name "CHELOY E.
A: Usually, if the document released by the MRO, the delivery receipt attached to the VELICARIAGARAFIL" as "State Solicitor II, Office of the
transmittal letter is filled out completely because the dates when the original Solicitor General" located on the first page of the letter as
appointment papers were actually received are very material. It is a standard operating Exhibit "2-H-7;"
procedure for the MRO personnel to ask the person receiving the documents to write
his/her name, his signature, and the date and time when he/she received it. ii. The portion rubber stamped by the Office of the
Executive Secretary located at the back of the last page of
the -letter showing receipt by the DOJ with blank spaces
for the date and time when it was actually received as Cooperative Development Authority, Department of Finance dated March 8,
Exhibit "2-H-8;" 2010 as Exhibit "2-X" for the respondents;

B) The Appointment Paper of CHELOY E. VELICARIA-GARAFIL (a) The portion rubber stamped by the Office of the Executive
dated March 5, 2010 as Exhibit "2-I" for the respondents; Secretary at the back thereof showing receipt by DOF with blank
spaces for the date and time when it was actually received as Exhibit
xxxx "2-X-1 ;"

4. A) The Transmittal Letter pertinent to the appointment of EDDIE U. B) The Appointment Paper of IRMA A. VILLANUEVA dated March 3, 2010 as
TAMONDONG dated 8 March 2010 but turned over to the MRO only on May Exhibit "2-Y" for the respondents.54
6, 2010 consisting of two (2) pages as Exhibits "2-L" and "2-L-l" respectively
for the respondents; The testimony of Ellenita G. Gatbunton, Division Chief of File Maintenance and
Retrieval Division of the MRO, supports Dimaandal's counsel's manifestation that the
(a) The portion with the name "EDDIE U. TAMONDONG" as transmittal of petitioners' appointment papers is questionable.
"Member, representing the Private Sector, Board of Directors" as
Exhibit "2-L-2"; Q: In the case of Cheloy E. Velicaria-Garafil, who was appointed as State Solicitor II
of the Office of the Solicitor General, was her appointment paper released through the
(b) The portion rubber stamped by the Office of the Executive MRO?
Secretary located at the back of the last page of the letter showing
receipt by Ma. Carissa O. Coscuella with blank spaces for the date A: No. Her appointment paper dated March 5, 2010, with its corresponding transmittal
and time when it was actually received as Exhibit "2-L-3"; letter, was merely turned over to the MRO on May 13, 2010. The transmittal letter that
was turned over to the MRO was already stamped "released" by the Office of the
xxxx Executive Secretary, but the date and time as to when it was actually received were
unusually left blank.
8. A) The Transmittal Letter pertinent to the appointments of x x x
FRANCISCA BESTOYONG-ROSQUITA dated March 8, 2010 but Q: What is your basis?
turned over to the MRO on May 13, 2010 as Exhibit "2-T" for the
respondents; A: The transmittal letter and appointment paper turned over to the MRO.

xxxx xxxx

(c) The portion with the name "FRANCISCA Q: In the case of Eddie U. Tamondong, who was appointed as member of the Board
BESTOYONGROSQUIT A" as "Commissioner, Representing of Directors of Subic Bay Metropolitan Authority, was her [sic] appointment paper
Region I and the Cordilleras" as Exhibit "2-T-3·" released through the MRO?

(d) The portion rubber stamped by the Office of the Executive A: No. His appointment paper dated March 1, 2010, with its corresponding transmittal
Secretary at the back thereof showing receipt by Masli A. Quilaman letter, was merely turned over to the MRO on May 6, 2010. The transmittal letter that
of NCIP-QC on March 15, 2010 as Exhibit "2-T-4;" was turned over to the MRO was already stamped "released" by the Office of the
Executive Secretary, but the date and time as to when it was actually received were
xxxx unusually left blank.

D) The Appointment Paper of FRANCISCA BESTOYONGROSQUIT A dated Q: What is your basis?


March 5, 2010 as Exhibit "2-W" for the respondents;
A: The transmittal letter and appointment paper turned over to the MRO.
9. A) The Transmittal Letter pertinent to the appointment of IRMA A.
VILLANUEVA as Administrator for Visayas, Board of Administrators, xxxx
Q: In the case of Francisca Bestoyong-Resquita who was appointed as Commissioner could be legally appointed; hence, Romero's appointment as provincial fiscal ofNegros
of the National Commission on Indigenous Peoples, representing Region 1 and the Oriental vice Lacson was invalid.
Cordilleras, was her appointment paper released thru the MRO?
The appointment to a government post like that of provincial fiscal to be complete
A: No. Her appointment paper dated March 5, 2010, with its corresponding transmittal involves several steps. First, comes the nomination by the President. Then to make
letter, was merely turned over to the MRO on May 13, 2010. The transmittal letter that that nomination valid and permanent, the Commission on Appointments of the
was turned over to the MRO was already stamped "released" by the Office of the Legislature has to confirm said nomination. The last step is the acceptance thereof by
Executive Secretary and received on March 15, 2010. the appointee by his assumption of office. The first two steps, nomination and
confirmation, constitute a mere offer of a post. They are acts of the Executive and
Q: What is your basis? Legislative departments of the Government. But the last necessary step to make the
appointment complete and effective rests solely with the appointee himself. He may or
he may not accept the appointment or nomination. As held in the case of Borromeo
A: The transmittal letter and appointment paper turned over to the MRO. vs. Mariano, 41 Phil. 327, "there is no power in this country which can compel a man
to accept an office." Consequently, since Lacson has declined to accept his
xxxx appointment as provincial fiscal of Tarlac and no one can compel him to do so, then
he continues as provincial fiscal of Negros Oriental and no vacancy in said office was
Q: In the case of Irma A. Villanueva who was appointed as Administrator for Visayas created, unless Lacson had been lawfully removed as such fiscal of Negros Oriental.59
of the Cooperative Development Authority, was her appointment paper released thru
the MRO? Paragraph (b ), Section 1 of EO 2 considered as midnight appointments those
appointments to offices that will only be vacant on or after 11 March 2010 even though
A: No. Her appointment paper dated March 3, 2010, with its corresponding transmittal the appointments are made prior to 11 March 2010. EO 2 remained faithful to the intent
letter, was merely turned over to the MRO on May 4, 2010. The transmittal letter that of Section 15, Article VII of the 1987 Constitution: the outgoing President is prevented
was turned over to the MRO was already stamped "released" by the Office of the from continuing to rule the country indirectly after the end of his term.
Executive Secretary, but the date and time as to when it was actually received were
unusually left blank. Acceptance by the Qualified Appointee

Q: What is your basis? Acceptance is indispensable to complete an appointment. Assuming office and taking
the oath amount to acceptance of the appointment. 60 An oath of office is a qualifying
A: The transmittal letter and appointment paper turned over to the MR0.55 requirement for a public office, a prerequisite to the full investiture of the office. 61

The possession of the original appointment paper is not indispensable to authorize an Javier v. Reyes62 is instructive in showing how acceptance is indispensable to
appointee to assume office. If it were indispensable, then a loss of the original complete an appointment. On 7 November 1967, petitioner Isidro M. Javier (Javier)
appointment paper, which could be brought about by negligence, accident, fraud, fire was appointed by then Mayor Victorino B. Aldaba as the Chief of Police of Malolos,
or theft, corresponds to a loss of the office.56 However, in case of loss of the original Bulacan. The Municipal Council confirmed and approved Javier's appointment on the
appointment paper, the appointment must be evidenced by a certified true copy issued same date. Javier took his oath of office on 8 November 1967, and subsequently
by the proper office, in this case the MRO. Vacant Position discharged the rights, prerogatives, and duties of the office. On 3 January 1968, while
the approval of Javier's appointment was pending with the CSC, respondent
Purificacion C. Reyes (Reyes), as the new mayor of Malolos, sent to the . CSC a letter
An appointment can be made only to a vacant office. An appointment cannot be made to recall Javier's appointment. Reyes also designated Police Lt. Romualdo F.
to an occupied office. The incumbent must first be legally removed, or his appointment Clemente as Officer-in-Charge of the police department. The CSC approved Javier's
validly terminated, before one could be validly installed to succeed him.57 appointment as permanent on 2 May 1968, and even directed Reyes to reinstate
Javier. Reyes, on the other hand, pointed to the appointment of Bayani Bernardo as
To illustrate: in Lacson v. Romero,58 Antonio Lacson (Lacson) occupied the post of Chief of Police of Malolos, Bulacan on 4 September 1967. This Court ruled that Javier's
provincial fiscal of Negros Oriental. He was later nominated and confirmed as appointment prevailed over that of Bernardo. It cannot be said that Bernardo accepted
provincial fiscal of Tarlac. The President nominated and the Commission on his appointment because he never assumed office or took his oath.
Appointments confirmed Honorio Romero (Romero) as provincial fiscal of Negros
Oriental as Lacson's replacement. Romero took his oath of office, but Lacson neither Excluding the act of acceptance from the appointment process leads us to the very evil
accepted the appointment nor assumed office as provincial fiscal of Tarlac. This Court which we seek to avoid (i.e., antedating of appointments). Excluding the act of
ruled that Lacson remained as provincial fiscal of Negros Oriental, having declined the acceptance will only provide more occasions to honor the Constitutional provision in
appointment as provincial fiscal of Tarlac. There was no vacancy to which Romero the breach. The inclusion of acceptance by the appointee as an integral part of the
entire appointment process prevents the abuse of the Presidential power to appoint. It
is relatively easy to antedate appointment papers and make it appear that they were
issued prior to the appointment ban, but it is more difficult to simulate the entire
appointment process up until acceptance by the appointee.

Petitioners have failed to show compliance with all four elements of a valid
appointment. They cannot prove with certainty that their appointment papers were
transmitted before the appointment ban took effect. On the other hand, petitioners
admit that they took their oaths of office during the appointment ban.

Petitioners have failed to raise any valid ground for the Court to declare EO 2, or any
part of it, unconstitutional. Consequently, EO 2 remains valid and constitutional.

WHEREFORE, the petitions in G.R. Nos. 203372, 206290, and 212030 are DENIED,
and the petition in G.R. No. 209138 is DISMISSED. The appointments of petitioners
Atty. Cheloy E. Velicaria-Garafil (G.R. No. 203372), Atty. Dindo G. Venturanza (G.R.
No. 206290), Irma A. Villanueva, and Francisca B. Rosquita (G.R. No. 209138), and
Atty. Eddie U. Tamondong (G.R. No. 212030) are declared VOID. We DECLARE that
Executive Order No. 2 dated 30 July 2010 is VALID and CONSTITUTIONAL.

SO ORDERED.

G.R. No. 180771 April 21, 2015


RESIDENT MARINE MAMMALS OF THE PROTECTED SEASCAPE TAÑON I
STRAIT, e.g., TOOTHED WHALES, DOLPHINS, PORPOISES, AND OTHER
CETACEAN SPECIES, Joined in and Represented herein by Human Beings In G.R. No. 180771, petitioners Resident Marine Mammals allegedly bring their case
Gloria Estenzo Ramos and Rose-Liza Eisma-Osorio, In Their Capacity as Legal in their personal capacity, alleging that they stand to benefit or be injured from the
Guardians of the Lesser Life-Forms and as Responsible Stewards of God's judgment on the issues. The human petitioners implead themselves in a representative
Creations, Petitioners, capacity "as legal guardians of the lesser life-forms and as responsible stewards of
vs. God's Creations."1 They use Oposa v. Factoran, Jr.2 as basis for their claim, asserting
SECRETARY ANGELO REYES, in his capacity as Secretary of the Department of their right to enforce international and domestic environmental laws enacted for their
Energy (DOE), SECRETARY JOSE L. ATIENZA, in his capacity as Secretary of benefit under the concept of stipulation pour autrui.3As the representatives of Resident
the Department of Environment and Natural Resources (DENR), LEONARDO R. Marine Mammals, the human petitioners assert that they have the obligation to build
SIBBALUCA, DENR Regional Director-Region VII and in his capacity as awareness among the affected residents of Tañon Strait as well as to protect the
Chairperson of the Tañon Strait Protected Seascape Management Board, Bureau environment, especially in light of the government's failure, as primary steward, to do
of Fisheries and Aquatic Resources (BFAR), DIRECTOR MALCOLM J. its duty under the doctrine of public trust.4
SARMIENTO, JR., BFAR Regional Director for Region VII ANDRES M. BOJOS,
JAPAN PETROLEUM EXPLORATION CO., LTD. (JAPEX), as represented by its
Philippine Agent, SUPPLY OILFIELD SERVICES, INC. Respondents. Resident Marine Mammals and the human petitioners also assert that through this
case, this court will have the opportunity to lower the threshold for locus standi as an
exercise of "epistolary jurisdiction."5
x-----------------------x
The zeal of the human petitioners to pursue their desire to protect the environment and
G.R. No. 181527 to continue to define environmental rights in the context of actual cases is
commendable. However, the space for legal creativity usually required for advocacy of
CENTRAL VISAYAS FISHERFOLK DEVELOPMENT CENTER (FIDEC), CERILO D. issues of the public interest is not so unlimited that it should be allowed to undermine
ENGARCIAL, RAMON YANONG, FRANCISCO LABID, in their personal capacity the other values protected by current substantive and procedural laws. Even rules of
and as representatives of the SUBSISTENCE FISHERFOLKS OF THE procedure as currently formulated set the balance between competing interests. We
MUNICIPALITIES OF ALOGUINSAN AND PINAMUNGAJAN, CEBU, AND THEIR cannot abandon these rules when the necessity is not clearly and convincingly
FAMILIES, AND THE PRESENT AND FUTURE GENERATIONS OF FILIPINOS presented.
WHOSE RIGHTS ARE SIMILARLY AFFECTED, Petitioners,
vs. The human petitioners, in G.R. No. 180771, want us to create substantive and
SECRETARY ANGELO REYES, in his capacity as Secretary of the Department of procedural rights for animals through their allegation that they can speak for them.
Energy (DOE), JOSE L. ATIENZA, in his capacity as Secretary of the Department Obviously, we are asked to accept the premises that (a) they were chosen by the
of Environment and Natural Resources (DENR), LEONARDO R. SIBBALUCA, in Resident Marine Mammals of Tañon Strait; (b) they were chosen by a representative
his capacity as DENR Regional Director-Region VII and as Chairperson of the group of all the species of the Resident Marine Mammals; (c) they were able to
Tañon Strait Protected Seascape Management Board, ALAN ARRANGUEZ, in his communicate with them; and (d) they received clear consent from their animal
capacity as Director - Environmental Management Bureau-Region VII, DOE principals that they would wish to use human legal institutions to pursue their interests.
Regional Director for Region VIII1 ANTONIO LABIOS, JAPAN PETROLEUM Alternatively, they ask us to acknowledge through judicial notice that the interests that
EXPLORATION CO., LTD. (JAPEX), as represented by its Philippine Agent, they, the human petitioners, assert are identical to what the Resident Marine Mammals
SUPPLY OILFIELD SERVICES, INC., Respondents. would assert had they been humans and the legal strategies that they invoked are the
strategies that they agree with.
CONCURRING OPINION
In the alternative, they want us to accept through judicial notice that there is a
"Until one has loved an animal, relationship of guardianship between them and all the resident mammals in the
a part of one 's soul remains unawakened." affected ecology.

Anatole France Fundamental judicial doctrines that may significantly change substantive and
procedural law cannot be founded on feigned representation.
LEONEN, J.:
Instead, I agree that the human petitioners should only speak for themselves and
I concur in the result, with the following additional reasons. already have legal standing to sue with respect to the issue raised in their pleading.
The rules on standing have already been liberalized to take into consideration the
difficulties in the assertion of environmental rights. When standing becomes too liberal, SEC. 2. Parties in interest.-A real party in interest is the party who stands to be
this can be the occasion for abuse. benefited or injured by the judgment in the suit, or the party entitled to the avails of the
suit. Unless otherwise authorized by law or these Rules, every action must be
II prosecuted or defended in the name of the real party in interest. (2a)6

Rule 3, Section 1 of the 1997 Rules of Civil Procedure, in part, provides: A litigant who stands to benefit or sustain an injury from the judgment of a case is a
real party in interest.7 When a case is brought to the courts, the real party in interest
must show that another party's act or omission has caused a direct injury, making his
SECTION 1. Who may be parties; plaintiff and defendant. - Only natural or juridical or her interest both material and based on an enforceable legal right.8
persons, or entities authorized by law may be parties in a civil action.
Representatives as parties, on the other hand, are parties acting in representation of
The Rules provide that parties may only be natural or juridical persons or entities that the real party in interest, as defined in Rule 3, Section 3 of the 1997 Rules of Civil
may be authorized by statute to be parties in a civil action. Procedure:

Basic is the concept of natural and juridical persons in our Civil Code: SEC. 3. Representatives as parties. - Where the action is allowed to be prosecuted or
defended by a representative or someone acting in a fiduciary capacity, the beneficiary
ARTICLE 37. Juridical capacity, which is the fitness to be the subject of legal relations, shall be included in the title of the case and shall be deemed to be the real party in
is inherent in every natural person and is lost only through death. Capacity to act, which interest. A representative may be a trustee of an express rust, a guardian, an executor
is the power to do acts with legal effect, is acquired and may be lost. or administrator, or a party authorized by law or these Rules. An agent acting in his
own name and for the benefit of an undisclosed principal may sue or be sued without
Article 40 further defines natural persons in the following manner: joining the principal except when the contract involves things belonging to the
principal.(3a)9

ARTICLE 40. Birth determines personality; but the conceived child shall be considered
born for all purposes that are favorable to it, provided it be born later with the conditions The rule is two-pronged. First, it defines .a representative as a party who is not bound
specified 'in the following article. to directly or actually benefit or suffer from the judgment, but instead brings a case in
favor of an identified real party in interest. 10 The representative is an outsider to the
cause of action. Second, the rule provides a list of who may be considered as
Article 44, on the other hand, enumerates the concept of a juridical person: "representatives." It is not an exhaustive list, but the rule limits the coverage only to
those authorized by law or the Rules of Court.11
ARTICLE 44. The following are juridical persons:
These requirements should apply even in cases involving the environment, which
(1) The State and its political subdivisions; means that for the Petition of the human petitioners to prosper, they must show that
(a) the Resident Marine Mammals are real parties in interest; and (b) that the human
petitioners are authorized by law or the Rules to act in a representative capacity.
(2) Other corporations, institutions and entities for public interest or purpose,
created by law; their personality begins as soon as they have been
constituted according to law; The Resident Marine Mammals are comprised of "toothed whales, dolphins, porpoises,
and other cetacean species inhabiting Tañon Strait." 12 While relatively new in
Philippine jurisdiction, the issue of whether animals have legal standing before courts
(3) Corporations, partnerships and associations for private interest or purpose
has been the subject of academic discourse in light of the emergence of animal and
to which the law grants a juridical personality, separate and distinct from that
environmental rights.
of each shareholder, partner or member.

In the United States, anim4l rights advocates have managed to establish a system
Petitioners in G.R. No. 180771 implicitly suggest that we amend, rather than simply
which Hogan explains as the "guardianship model for nonhuman animals":13
construe, the provisions of the Rules of Court as well as substantive law to
accommodate Resident Marine Mammals or animals. This we cannot do.
Despite Animal Lovers, there exists a well-established system by which nonhuman
animals may obtain judicial review to enforce their statutory rights and protections:
Rule 3, Section 2 of the 1997 Rules of Civil Procedure further defines real party in
guardianships. With court approval, animal advocacy organizations may bring suit on
interest:
behalf of nonhuman animals in the same way court-appointed guardians bring suit on
behalf of mentally-challenged humans who possess an enforceable right but lack the known example is found in the Endangered Species Act (ESA). Such provisions are
ability to enforce it themselves. evidence of legislative intent to encourage civic participation on behalf of nonhuman
animals. Our law of standing should reflect this intent and its implication that humans
In the controversial but pivotal Should Trees Have Standing?-Toward Legal Rights for are suitable representatives of the natural environment, which includes nonhuman
Natural Objects, Christopher D. Stone asserts that the environment should possess animals.14 (Emphasis supplied, citation omitted)
the right to seek judicial redress even though it is incapable of representing itself. While
asserting the rights of When a court allows guardianship as a basis of representation, animals are considered
as similarly situated as individuals who have enforceable rights but, for a legitimate
speechless entities such as the environment or nonhuman animals certainly poses reason (e.g., cognitive disability), are unable to bring suit for themselves. They are also
legitimate challenges - such as identifying the proper spokesman -the American legal similar to entities that by their very nature are incapable of speaking for themselves
system is already well-equipped with a reliable mechanism by which nonhumans may (e.g., corporations, states, and others).
obtain standing via a judicially established guardianship. Stone notes that other
speechless - and nonhuman - entities such as corporations, states, estates, and In our jurisdiction, persons and entities are recognized both in law and the Rules of
municipalities have standing to bring suit on their own behalf. There is little reason to Court as having standing to sue and, therefore, may be properly represented as real
fear abuses under this regime as procedures for removal and substitution, avoiding parties in interest. The same cannot be said about animals.
conflicts of interest, and termination of a guardianship are well established.
Animals play an important role in households, communities, and the environment.
In fact, the opinion in Animal Lovers suggests that such an arrangement is indeed While we, as humans, may feel the need to nurture and protect them, we cannot go as
possible. The court indicated that AL VA might have obtained standing in its own right far as saying we represent their best interests and can, therefore, speak for them
if it had an established history of dedication to the cause of the humane treatment of before the courts. As humans, we cannot be so arrogant as to argue that we know the
animals. It noted that the Fund for Animals had standing and indicated that another suffering of animals and that we know what remedy they need in the face of an injury.
more well-known advocacy organization might have had standing as well. The court
further concluded that an organization's standing is more than a derivative of its history, Even in Hogan's discussion, she points out that in a case before the United States
but history is a relevant consideration where organizations are not well-established District Court for the Central District of California, Animal Lovers Volunteer Ass'n v.
prior to commencing legal action. ALVA was not the proper plaintiff because it could Weinberger,15 the court held that an emotional response to what humans perceive to
not identify previous activities demonstrating its recognized activism for and be an injury inflicted on an animal is not within the "zone-of-interest" protected by
commitment to the dispute independent of its desire to pursue legal action. The court's law.16Such sympathy cannot stand independent of or as a substitute for an actual injury
analysis suggests that a qualified organization with a demonstrated commitment to a suffered by the claimant.17 The ability to represent animals was further limited in that
cause could indeed bring suit on behalf of the speechless in the form of a court- case by the need to prove "genuine dedication" to asserting and protecting animal
sanctioned guardianship. rights:

This Comment advocates a shift in contemporary standing doctrine to empower non- What ultimately proved fatal to ALVA 's claim, however, was the court's assertion that
profit organizations with an established history of dedication to the cause and relevant standing doctrine further required ALVA to differentiate its genuine dedication to the
expertise to serve as official guardians ad !item on behalf of nonhuman animals humane treatment of animals from the general disdain for animal cruelty shared by the
interests. The American legal system has numerous mechanisms for representing the public at large. In doing so, the court found ALVA 's asserted organizational injury to
rights and interests of nonhumans; any challenges inherent in extending these pre- be abstract and thus relegated ALVA to the ranks of the "concerned bystander. "
existing mechanisms to nonhuman animals are minimal compared to an interest in the
proper administration of justice. To adequately protect the statutory rights of nonhuman
animals, the legal system must recognize those statutory rights independent of ....
humans and provide a viable means of enforcement. Moreover, the idea of a
guardianship for speechless plaintiffs is not new and has been urged on behalf of the In fact, the opinion in Animal Lovers suggests that such an arrangement is indeed
natural environment. 'Such a model is even more compelling as applied to nonhuman possible. The court indicated that ALVA might have obtained standing in its own right
animals, because they are sentient beings with the ability to feel pain and exercise if it had an established history of dedication to the cause of the humane treatment of
rational thought. Thus, animals are qualitatively different from other legally protected animals. It noted that the Fund for Animals had standing and indicated that another
nonhumans and therefore have interests deserving direct legal protection. more well-known advocacy organization might have had standing as well. The court
further concluded that an organization's standing is more than a derivative of its history,
Furthermore, the difficulty of enforcing the statutory rights of nonhuman animals but history is a relevant consideration where organizations are not well-established
threatens the integrity of the federal statutes designed to protect them, essentially prior to commencing legal action. ALVA was not the proper plaintiff because it could
rendering them meaningless. Sensing that laws protecting nonhuman animals would not identify previous activities demonstrating its recognized activism for and
be difficult to enforce, Congress provided for citizen suit provisions: the most well- commitment to the dispute independent of its desire to pursue legal action. The court's
analysis suggests that a qualified organization with a demonstrated commitment to a assuming that it can speak best for those who will exist at a different time, under a
cause could indeed bring suit on behalf of the speechless in the form of a court- different set of circumstances. In essence, the unbridled resort to representative suit
sanctioned guardianship.18(Emphasis supplied, citation omitted) will inevitably result in preventing future generations from protecting their own rights
and pursuing their own interests and decisions. It reduces the autonomy of our children
What may be argued as being parallel to this concept of guardianship is the principle and our children 's children. Even before they are born, we again restricted their ability
of human stewardship over the environment in a citizen suit under the Rules of to make their own arguments.
Procedure for Environmental Cases. A citizen suit allows any Filipino to act as a
representative of a party who has enforceable rights under environmental laws before It is my opinion that, at best, the use of the Oposa doctrine in environmental cases
Philippine courts, and is defined in Section 5: . should be allowed only when a) there is a clear legal basis for the representative suit;
b) there are actual concerns based squarely upon an existing legal right; c) there is no
SEC. 5. Citizen suit. - Any Filipino citizen in representation of others, including minors possibility of any countervailing interests existing within the population represented or
or generations yet unborn, may file an action to enforce rights or obligations under those that are yet to be born; and d) there is an absolute necessity for such standing
environmental laws. Upon the filing of a citizen suit, the court shall issue an order which because there is a threat of catastrophe so imminent that an immediate protective
shall contain a brief description of the cause of action and the reliefs prayed for, measure is necessary. Better still, in the light of its costs and risks, we abandon the
requiring all interested parties to manifest their interest to intervene in the case within precedent all together.23 (Emphasis in the original)
fifteen (15) days from notice thereof. The plaintiff may publish the order once in a
newspaper of a general circulation in the Philippines or furnish all affected barangays Similarly, in Paje:
copies of said order.
A person cannot invoke the court's jurisdiction if he or she has no right or interest to
There is no valid reason in law or the practical requirements of this case to implead protect. He or she who invokes the court's jurisdiction must be the "owner of the right
and feign representation on behalf of animals. To have done so betrays a very sought to be enforced." In other words, he or she must have a cause of action. An
anthropocentric view of environmental advocacy. There is no way that we, humans, action may be dismissed on the ground of lack of cause of action if the person who
can claim to speak for animals let alone present that they would wish to use our court instituted it is not the real party in interest. 24 The term "interest" under the Rules of
system, which is designed to ensure that humans seriously carry their responsibility Court must refer to a material interest that is not merely a curiosity about or an "interest
including ensuring a viable ecology for themselves, which of course includes in the question involved." The interest must be present and substantial. It is not a mere
compassion for all living things. expectancy or a future, contingent interest.

Our rules on standing are sufficient and need not be further relaxed. A person who is not a real party in interest may institute an action if he or she is suing
as representative of a .real party in interest. When an action is prosecuted or defended
In Arigo v. Swift,19 I posed the possibility of further reviewing the broad interpretation by a representative, that representative is not and does not become the real party in
we have given to the rule on standing. While representatives are not required to interest. The person represented is deemed the real party in interest. The
establish direct injury on their part, they should only be allowed to represent after representative remains to be a third party to the action instituted on behalf of another.
complying with the following: [I]t is imperative for them to indicate with certainty the
injured parties on whose behalf they bring the suit. Furthermore, the interest of those ....
they represent must be based upon concrete legal rights. It is not sufficient to draw out
a perceived interest from a general, nebulous idea of a potential "injury."20 To sue under this rule, two elements must be present: "(a) the suit is brought on behalf
of an identified party whose right has been violated, resulting in some form of damage,
I reiterate my position in Arigo v. Swift and in Paje v. Casiño 21 regarding this rule and (b) the representative authorized by law or the Rules of Court to represent the
alongside the appreciation of legal standing in Oposa v. Factoran 22 for environmental victim."
cases. In Arigo, I opined that procedural liberality, especially in cases brought by
representatives, should be used with great caution: The Rules of Procedure for Environmental Cases allows filing of a citizen's suit. A
citizen's suit under this rule allows any Filipino citizen to file an action for the
Perhaps it is time to revisit the ruling in Oposa v. Factoran. enforcement of environmental law on behalf of minors or generations yet unborn. It is
essentially a representative suit that allows persons who are not real parties in interest
That case was significant in that, at that time, there was need to call attention to to institute actions on behalf of the real party in interest.
environmental concerns in light of emerging international legal principles. While
"intergenerational responsibility" is a noble principle, it should not be used to obtain The expansion of what constitutes "real party in interest" to include minors and
judgments that would preclude future generations from making their own assessment generations yet unborn is a recognition of this court's ruling in Oposa v. Factoran. This
based on their actual concerns. The present generation must restrain itself from court recognized the capacity of minors (represented by their parents) to file a class
suit on behalf of succeeding generations based on the concept of intergenerational III
responsibility to ensure the future generation's access to and enjoyment of [the]
country's natural resources. In our jurisdiction, there is neither reason nor any legal basis for the concept of implied
petitioners, most especially when the implied petitioner was a sitting President of the
To allow citizen's suits to enforce environmental rights of others, including future Republic of the Philippines. In G.R. No. 180771, apart from adjudicating unto
generations, is dangerous for three reasons: themselves the status of "legal guardians" of whales, dolphins, porpoises, and other
cetacean species, human petitioners also impleaded Former President Gloria
First, they run the risk of foreclosing arguments of others who are unable to take part Macapagal-Arroyo as "unwilling co-petitioner" for "her express declaration and
in the suit, putting into. question its representativeness. Second, varying interests may undertaking in the ASEAN Charter to protect Tañon Strait."28
potentially result in arguments that are bordering on political issues, the resolutions of
which do not fall upon this court. Third, automatically allowing a class or citizen's suit No person may implead any other person as a co-plaintiff or co-petitioner without his
on behalf of minors and generations yet unborn may result in the oversimplification of or her consent. In our jurisdiction, only when there is a party that should have been a
what may be a complex issue, especially in light of the impossibility of determining necessary party but was unwilling to join would there be an allegation as to why that
future generation's true interests on the matter. party has been omitted. In Rule 3, Section 9 of the 1997 Rules of Civil Procedure:

In citizen's suits, persons who may have no interest in the case may file suits for others. SEC. 9. Non-joinder of necessary parties to be pleaded. -Whenever in any pleading in
Uninterested persons will argue for the persons they represent, and the court will which a claim is asserted a necessary party is not joined, the pleader shall set forth his
decide based on their evidence and arguments. Any decision by the court will be name, if known, and shall state why he is omitted. Should the court find the reason for
binding upon the beneficiaries, which in this case are the minors and the future the omission unmeritorious, it may order the inclusion of the omitted necessary party
generations. The court's decision will be res judicata upon them and conclusive upon if jurisdiction over his person may be obtained.
the issues presented.25
The failure to comply with the order for his inclusion, without justifiable cause, shall be
The danger in invoking Oposa v. Factoran to justify all kinds of environmental claims deemed a waiver of the claim against such party.
lies in its potential to diminish the value of legitimate environmental rights. Extending
the application of "real party in interest" to the Resident Marine Mammals, or animals The non-inclusion of a necessary party does not prevent the court from proceeding in
in general, through a judicial pronouncement will potentially result in allowing petitions the action, and the judgment rendered therein shall be without prejudice to the rights
based on mere concern rather than an actual enforcement of a right. It is impossible of such necessary party.29
for animals to tell humans what their concerns are. At best, humans can only surmise
the extent of injury inflicted, if there be any. Petitions invoking a right and seeking legal
redress before this court cannot be a product of guesswork, and representatives have A party who should have been a plaintiff or petitioner but whose consent cannot be
the responsibility to ensure that they bring "reasonably cogent, rational, scientific, well- obtained should be impleaded as a defendant in the nature of an unwilling co-plaintiff
founded arguments"26 on behalf of those they represent. under Rule 3, Section 10 of the 1997 Rules of Civil Procedure:

Creative approaches to fundamental problems should be welcome. However, they SEC. 10. Unwilling co-plaintiff. - If the consent of any party who should be joined as
should be considered carefully so that no unintended or unwarranted consequences plaintiff can not be obtained, he may be made a defendant and the reason therefor
should follow. I concur with the approach of Madame Justice Teresita J. Leonardo-De shall be stated in the complaint.30
Castro in her brilliant ponencia as it carefully narrows down the doctrine in terms of
standing. Resident Marine Mammals and the human petitioners have no legal standing The reason for this rule is plain: Indispensable party plaintiffs who should be part of
to file any kind of petition. the action but who do not consent should be put within the jurisdiction of the court
through summons or other court processes. Petitioners. should not take it upon
However, I agree that petitioners in G.R. No. 181527, namely, Central Visayas themselves to simply imp lead any party who does not consent as a petitioner. This
Fisherfolk Development Center,. Engarcial, Yanong, and Labid, have standing both as places the unwilling co-petitioner at the risk of being denied due process.
real parties in interest and as representatives of subsistence fisherfolks of the
Municipalities of Aloguinsan and Pinamungahan, Cebu, and their families, and the Besides, Former President Gloria Macapagal-Arroyo cannot be a party to this suit. As
present and future generations of Filipinos whose rights are similarly affected. The a co-equal constitutional department, we cannot assume that the President needs to
activities undertaken under Service Contract 46 (SC-46) directly affected their source enforce policy directions by suing his or her alter-egos. The procedural situation
of livelihood, primarily felt through the significant reduction of their fish harvest. 27 The caused by petitioners may have gained public attention, but its legal absurdity borders
actual, direct, and material damage they suffered, which has potential long-term effects on the contemptuous. The Former President's name should be stricken out of the title
transcending generations, is a proper subject of a legal suit. of this case.
IV The President may enter into agreements with foreign-owned corporations involving
either technical or financial assistance for large-scale exploration, development, and
I also concur with the conclusion that SC-46 is both. illegal and unconstitutional. utilization of minerals, petroleum, and other mineral oils according to the general terms
and conditions provided by law, based on real contributions to the economic growth
and general welfare of the country. In such agreements, the State shall promote the
SC-46 is illegal because it violates Republic Act No. ·7586 or the National Integrated development and use of local scientific and technical resources.
Protected Areas System Act of 1992, and Presidential Decree No. 1234, 31 which
declared Tañon Strait as a protected seascape. It is unconstitutional because it violates
the fourth paragraph of Article XII, Section 2 of the Constitution. The President shall notify the Congress of every contract entered into in accordance
with this provision, within thirty days from its execution. (Emphasis supplied)
V
I agree that fully foreign-owned corporations may participate in the exploration,
development, and use of natural resources, but only through either financial
Petitioner Central Visayas Fisherfolk Development Center asserts that SC-46 violated agreements or technical ones. This is the clear import of the words "either financial or
Article XII, Section 2, paragraph 1 of the .1987 Constitution because Japan Petroleum technical assistance agreements." This is also
Exploration Co., Ltd. (JAPEX) is 100% Japanese-owned.32 It further asserts that SC-
46 cannot be validly classified as a technical and financial assistance agreement
executed under Article XII, Section 2, paragraph 4 of the 1987 Constitution. 33 Public the clear result if we compare the 1987 constitutional provision with the versions in the
respondents counter that SC-46 does not fall under the coverage of paragraph 1, but 1973 and 1935 Constitution:
is a validly executed contract under paragraph 4. 34· Public respondents further aver
that SC-46 neither granted exclusive fishing rights to JAPEX nor violated Central 1973 CONSTITUTION
Visayas Fisherfolk Development Center's right to preferential use of communal marine
and fishing resources.35 ARTICLE XIV
THE NATIONAL ECONOMY AND THE PATRIMONY OF THE NATION
VI
SEC. 9. The disposition, exploration, development, of exploitation, or utilization of any
Article XII, Section 2 of the 1987 Constitution states: of the natural resources of the Philippines shall be limited to citizens of the Philippines,
or to corporations or association at least sixty per centum of the capital of which is
Section 2. All lands of the public domain, waters, minerals, coal, petroleum, and other owned by such citizens. The Batasang Pambansa, in the national interest, may allow
mineral oils, all forces of potential energy, fisheries, forests or timber, wildlife, flora and such citizens, corporations, or associations to enter into service contracts for financial,
fauna, and other natural resources are owned by the State. With the exception. of technical, management, or other forms of assistance with any foreign person or entity
agricultural lands, all other natural resources shall not be alienated. The exploration, for the exploitation, development, exploitation, or utilization of any of the natural
development, and utilization of natural resources shall be under the full control and resources. Existing valid and binding service contracts for financial, the technical,
supervision of the State. The State may directly undertake such activities, or it may management, or other forms of assistance are hereby recognized as such. (Emphasis
enter into co-production, joint venture, or production-sharing agreements with Filipino supplied)
citizens, or corporations or associations at least sixty per centum of whose capital is
owned by such citizens. Such agreements may be for a period not exceeding twenty- 1935 CONSTITUTION
five years, renewable for not more than twenty-five years, and under such terms and
conditions as may be provided by law. In cases of water rights for irrigation, water ARTICLE XIII
supply fisheries, or industrial uses other than the development of water power, CONSERVATION AND UTILIZATION OF NATURAL RESOURCES
beneficial use may be the measure and limit of the grant.
SECTION 1. All agricultural timber, and mineral. lands of the public domain, waters,
The State shall protect the nation's marine wealth in its archipelagic waters, territorial minerals, coal, petroleum, and other mineral oils, all forces of potential energy, and
sea, and exclusive economic zone, and reserve its use and enjoyment exclusively to other natural resources of the Philippines belong to the State, and their disposition,
Filipino citizens. exploitation, development, or utilization shall be limited to citizens of the Philippines,
or to corporations or associations at least sixty per centum of the capital of which is
The Congress may, by law, allow small-scale utilization of natural resources by Filipino owned by such citizens, subject to any existing right, grant, lease, or concession at the
citizens, as well as cooperative fish farming, with priority to subsistence fishermen and time of the inauguration of the Government established under this Constitution. Natural
fish-workers in rivers, lakes, bays, and lagoons. resources, with the exception of public agricultural land, shall not be alienated, and no
license, concession, or lease for the exploitation, development, or utilization of any of
the natural resources shall be granted for a period exceeding twenty-five years,
renewable for another twenty-five years, except as to water rights for irrigation, water The general law referred to as a possible basis for SC-46's validity is Presidential
supply, fisheries, or industrial uses other than the development of water power, in Decree No. 87 or the Oil Exploration and Development Act of 1972.1âwphi1 It is my
which cases beneficial use may be the measure and the limit of the grant. opinion that this law is unconstitutional in that it allows service contracts, contrary to
Article XII, Section 2 of the 1987 Constitution:
The clear text of the Constitution in light of its history prevails over any attempt to infer
interpretation from the Constitutional Commission deliberations. The constitutional The President may enter into agreements with foreign-owned corporations involving
texts are the product of a full sovereign act: deliberations in a constituent assembly either technical or financial assistance for large-scale exploration, development, and
and ratification. Reliance on recorded discussion of Constitutional Commissions, on utilization of minerals, petroleum, and other mineral oils according to the general terms
the other hand, may result in dependence on incomplete authorship; Besides, it opens and conditions provided by law, based on real contributions to the economic growth
judicial review to further subjectivity from those who spoke during the Constitutional and general welfare of the country. In such agreements, the State shall promote the
Commission deliberations who may not have predicted how their words will be used. development and use of local scientific and technical resources. (Emphasis supplied)
It is safer that we use the words already in the Constitution. The Constitution was their
product. Its words were read by those who ratified it. The Constitution is what society The deletion of service contracts from the enumeration of the kind of agreements the
relies upon even at present. President may enter into with foreign-owned corporations for exploration and utilization
of resources means that service contracts are no longer allowed by the Constitution.
SC-46 is neither a financial assistance nor a technical assistance agreement. Pursuant to Article XVIII, Section 3 of the 1987 Constitution, 38 this inconsistency
renders the law invalid and ineffective.
Even supposing for the sake of argument that it is, it could not be declared valid in light
of the standards set forth in La Bugal-B'laan Tribal Association, Inc. v. Ramos:36 SC-46 suffers from the lack of a special law allowing its activities. The Main Opinion
emphasizes an important point, which is that SC-46 did not merely involve exploratory
Such service contracts may be entered into only with respect to minerals, petroleum activities, but also provided the rights and obligations of the parties should it be
and other mineral oils. The grant thereof is subject to several safeguards, among which discovered that there is oil in commercial quantities in the area. The Tañon Strait being
are these requirements: a protected seascape under Presidential Decree No. 1234 39 requires that the
exploitation and utilization of energy resources from that area are explicitly covered by
a law passed by Congress specifically for that purpose, pursuant to Section 14 of
(1) The service contract shall be crafted m accordance with a general law that Republic Act No. 7586 or the National Integrated Protected Areas System Act of 1992:
will set standard or uniform terms, conditions and requirements, presumably
to attain a certain uniformity in provisions and avoid the possible insertion of
terms disadvantageous to the country. SEC. 14. Survey for Energy R6'sources. - Consistent with the policies declared in
Section 2, hereof, protected areas, except strict nature reserves and natural parks,
may be subjected to exploration only for the purpose of gathering information on
(2) The President shall be the signatory for the government because, energy resources and only if such activity is carried out with the least damage to
supposedly before an agreement is presented to the President for signature, surrounding areas. Surveys shall be conducted only in accordance with a program
it will have been vetted several times over at different levels to ensure that it approved by the DENR, and the result of such surveys shall be made available to the
conforms to law and can withstand public scrutiny. public and submitted to the President for recommendation to Congress. Any
exploitation and utilization of energy resources found within NIP AS areas shall be
(3) Within thirty days of the executed agreement, the President shall report it allowed only through a law passed by Congress.40 (Emphasis supplied)
to Congress to give that branch of government an opportunity to look over the
agreement and interpose timely objections, if any.37 (Emphasis in the original, No law was passed by Congress specifically providing the standards, terms, and
citation omitted) conditions of an oil exploration, extraction, and/or utilization for Tañon Strait and,
therefore, no such activities could have been validly undertaken under SC-46. The
Based on the standards pronounced in La Bugal, SC-46' S validity must be tested National Integrated Protected Areas System Act of 1992 is clear that exploitation and
against three important points: (a) whether SC-46 was crafted in accordance with a utilization of energy resources in a protected seascape such as Tañon Strait shall only
general law that provides standards, terms, and conditions; (b) whether SC-46 was be allowed through a specific law.
signed by the President for and on behalf of the government; and (c) whether it was
reported by the President to Congress within 30 days of execution. VIII

VII Former President Gloria Macapagal-Arroyo was not the signatory to SC-46, contrary
to the requirement set by paragraph 4 of Article XII, Section 2 for service contracts
involving the exploration of petroleum. SC-46 was entered into by then Department of
Energy Secretary Vicente S. Perez, Jr., on behalf of the government. I agree with the SEC. 2. Declaration of Policy. Cognizant of the profound impact of man's activities on
Main Opinion that in cases where the Constitution or law requires the President to act all components of the natural environment particularly the effect of increasing
personally on the matter, the duty cannot be delegated to another public official. 41 La population, resource exploitation and industrial advancement and recognizing the
Bugal highlights the importance of the President's involvement, being one of the critical importance of protecting and maintaining the natural biological and physical
constitutional safeguards against abuse and corruption, as not mere formality: diversities of the environment notably on areas with biologically unique features to
sustain human life and development, as well as plant and animal life, it is hereby
At this point, we sum up the matters established, based on a careful reading of the declared the policy of the State to secure for the Filipino people of present and future
ConCom deliberations, as follows: generations the perpetual existence of all native plants and animals through the
establishment of a comprehensive system of integrated protected areas within the
classification of national park as provided for in the Constitution.
• In their deliberations on what was to become paragraph 4, the framers used
the term service contracts in referring to agreements x x x involving either
technical or financial assistance. • They spoke of service contracts as the It is hereby recognized that these areas, although distinct in features, possess common
concept was understood in the 1973 Constitution. ecological values that may be incorporated into a holistic plan representative of our
natural heritage; that effective administration of these areas is possible only through
cooperation among national government, local and concerned private organizations;
• It was obvious from their discussions that they were not about to ban or that the use and enjoyment of these protected areas must be consistent with the
eradicate service contracts. principles of biological diversity and sustainable development.

• Instead, they were plainly crafting provisions to. put in place safeguards that To this end, there is hereby established a National Integrated Protected Areas System
would eliminate or m minimize the abuses prevalent during the marital law (NIPAS), which shall encompass outstanding remarkable areas and biologically
regime.42 (Emphasis in the original) important public lands that are habitats of rare and endangered species of plants and
animals, biogeographic zones and related ecosystems, whether terrestrial, wetland or
Public respondents failed to show that. Former President Gloria Macapagal-Arroyo marine, all of which shall be designated as "protected areas." 44 (Emphasis supplied)
was involved in the signing or execution of SC-46. The failure to comply with this
constitutional requirement renders SC-46 null and void. Pursuant to this law, any proposed activity in Tañon Strait must undergo an
Environmental Impact Assessment:
IX
SEC. 12. Environmental Impact Assessment. - Proposals for activities which are
Public respondents also failed to show that Congress was subsequently informed of outside the scope of the management plan for protected areas shall be subject to an
the execution and existence of SC-46. The reporting requirement is an equally environmental impact assessment as required by law before they are adopted, and the
important requisite to the validity of any service contract involving the exploration, results thereof shall be taken into consideration in the decision-making
development, and utilization of Philippine petroleum. Public respondents' failure to process.45(Emphasis supplied)
report to Congress about SC-46 effectively took away any opportunity for the legislative
branch to scrutinize its terms and conditions. The same provision further requires that an Environmental Compliance Certificate be
secured under the Philippine Environmental Impact Assessment System before arty
In sum, SC-46 was executed and implemented absent all the requirements provided project is implemented:
under paragraph 4 of Article XII, Section 2. It is, therefore, null and void.
No actual implementation of such activities shall be allowed without the required
X Environmental Compliance Certificate (ECC) under the Philippine Environment Impact
Assessment (EIA) system. In instances where such activities are allowed to be
I am of the view that SC-46, aside from not having complied with the 1987 Constitution, undertaken, the proponent shall plan and carry them out in such manner as will
is also null and void for being violative of environmental laws protecting Tañon Strait. minimize any adverse effects and take preventive and remedial action when
In particular, SC-46 was implemented despite falling short of the requirements of the appropriate. The proponent shall be liable for any damage due to lack of caution or
National Integrated Protected Areas System Act of 1992. indiscretion.46 (Emphasis supplied)

As a protected seascape under Presidential Decree No. 1234, 43 Tañon Strait is In projects involving the exploration or utilization of energy resources, the National
covered by the National Integrated Protected Areas System Act of 1992. This law Integrated Protected Areas System Act of 1992 additionally requires that a program
declares as a matter of policy: be approved by the Department of Environment and Natural Resources, which shall
be publicly accessible. The program shall also be submitted to the President, who in
turn will recommend the program to Congress. Furthermore, Congress must enact a generation in filing any action in our courts of law to claim any of our fundamental rights
law specifically allowing the exploitation of energy resources found within a protected to a healthful ecology. In this way and with candor and courage, we fully shoulder the
area such as Tañon Strait: responsibility deserving of the grace and power endowed on our species.

SEC. 14. Survey for Energy Resources. - Consistent with the policies declared in ACCORDINGLY, I vote:
Section 2, hereof, protected areas, except strict nature reserves and natural parks,
may be subjected to exploration only for the purpose of gathering information on (a) to DISMISS G.R. No. 180771 for lack of standing and STRIKE OUT the
energy resources and only if such activity is carried out with the least damage to name of Former President Gloria Macapagal-Arroyo from the title of this case;
surrounding areas. Surveys shall be conducted only in accordance with a program
approved by the DENR, and the result of such surveys shall be made available to the
public and submitted to the President for recommendation to Congress. Any (b) to GRANT G.R. No. 181527; and
exploitation and utilization of energy resources found within NIPAS areas shall be
allowed only through a taw passed by Congress.47 (Emphasis supplied) (c) to DECLARE SERVICE CONTRACT 46 NULL AND VOID for violating the
1987 Constitution, Republic Act No. 7586, and Presidential Decree No. 1234.
Public respondents argue that SC-46 complied with the procedural requirements of
obtaining an Environmental Compliance Certificate.48 At any rate, they assert that the MARVIC M.V.F. LEONEN
activities covered by SC-46 fell under Section 14 of the National Integrated Protected Associate Justice
Areas System Act of 1992, which they interpret to be an exception to Section 12. They
argue that the Environmental Compliance Certificate is not a strict requirement for the
validity of SC-46 since (a) the Tañon Strait is not a nature' reserve or natural park; (b)
the exploration was merely for gathering information; and ( c) measures were in place
to ensure that the exploration caused the least possible damage to the area.49

Section 14 is not an exception to Section 12, but instead provides additional


requirements for cases involving Philippine energy resources. The National Integrated
Protected Areas System Act of 1992 was enacted to recognize the importance of
protecting the environment in light of resource exploitation, among others. 50 Systems
are put in place to secure for Filipinos local resources under the most favorable
conditions. With the status of Tañon Strait as a protected seascape, the institution of
additional legal safeguards is even more significant.

Public respondents did not validly obtain an Environmental Compliance Certificate for
SC-46. Based on the records, JAPEX commissioned an environmental impact
evaluation only in the second subphase of its project, with the Environmental
Management .Bureau of Region

VII granting the project an Environmental Compliance Certificate on March 6, 2007. 51

Despite its scale, the seismic surveys from May 9 to 18, 2005 were conducted without
any environmental assessment contrary to Section 12 of the National Integrated
Protected Areas System Act of 1992.

XI

Finally, we honor every living creature when we take care of our environment. As
sentient species, we do not lack in the wisdom or sensitivity to realize that we only
borrow the resources that we use to survive and to thrive. We are not incapable of
mitigating the greed that is slowly causing the demise of our planet. Thus, there is no
need for us to feign representation of any other species or some imagined unborn
G.R. No. 189028. July 16, 2013.* personal interest.—The parties who assail the constitutionality or legality of a
NATIONAL ARTIST FOR LITERATURE VIRGILIO ALMARIO, NATIONAL statute or an official act must have a direct and personal interest. They must show
ARTIST FOR LITERATURE BIENVENIDO LUMBERA, NATIONAL ARTIST not only that the law or any governmental act is invalid, but also that they
FOR VISUAL ARTS (PAINTING) BENEDICTO CABRERA, NATIONAL ARTIST sustained or are in immediate danger of sustaining some direct injury as a result
FOR VISUAL ARTS (SCULPTURE) NAPOLEON ABUEVA, NATIONAL ARTIST of its enforcement, and not merely that they suffer thereby in some indefinite way.
FOR VISUAL ARTS (PAINTING AND SCULPTURE) ARTURO LUZ, NATIONAL They must show that they have been or are about to be denied some right or
ARTIST FOR PRODUCTION DESIGN SALVADOR BERNAL, UNIVERSITY privilege to which they are lawfully entitled or that they are about to be subjected
PROFESSOR EMERITUS GEMINO ABAD, DEAN MARVIC M.V.F. LEONEN to some burdens or penalties by reason of the statute or act complained of.
(UP COLLEGE OF LAW), DEAN DANILO SILVESTRE (UP COLLEGE OF Constitutional Law; Equal Protection of the Law; No real and substantial
ARCHITECTURE), DEAN ROLAND TOLENTINO (UP COLLEGE OF MASS distinction between respondents and petitioner Abad has been shown that would
COMMUNICATION), PROF. JOSE DALISAY, DR. ANTON JUAN, DR. justify deviating from the laws, guidelines and established procedures, and placing
ALEXANDER CORTEZ, DR. JOSE NEIL GARCIA, DR. PEDRO JUN CRUZ respondents in an exceptional position. The undue classification was not germane
REYES, PROF. JOSE CLAUDIO GUERRERO, PROF. MICHAEL M. COROZA, to the purpose of the law. Instead, it contradicted the law and well-established
PROF. GERARD LICO, PROF. VERNE DE LA PENA, PROF. MARIAN ABUAN, guidelines, rules and regulations meant to carry the law into effect.—Among the
PROF. THEODORE O. TE, DR. CRISTINA PANTOJA-HIDALGO, PROF. JOSE other petitioners, Prof. Gemino Abad presents a unique valid personal and
WENDELL CAPILI, PROF. SIR ANRIAL TIATCO, PROF. NICOLO DEL substantial interest. Like respondents Caparas, Mañosa and Moreno, he was
CASTILLO, PROF. HORACIO DUMANLIG, PROF. DANTON REMOTO, PROF. among the 87 nominees for the 2009 Order of National Artists. Like respondent
PRISCELINA PATAJO-LEGASTO, PROF. BELEN CALINGACION, PROF. Moreno, he made it to the preliminary shortlist. As he did not make it to the second
AMIEL Y. LEONARDIA, PROF. VIM NADERA, PROF. MARILYN CANTA, shortlist, he was not considered by the Final Deliberation Panel, more so by the
PROF. CECILIA DELA PAZ, PROF. CHARLSON ONG, PROF. CLOD MARLON former President. It should be recalled too that respondent Guidote-Alvarez was
YAMBAO, PROF. KENNETH JAMANDRE, PROF. JETHRO JOAQUIN, ATTY. disqualified to be nominated for being the Executive Director of the NCCA at that
F.D. NICOLAS B. PICHAY, ATTY. ROSE BEATRIX ANGELES, MR. time while respondents Mañosa and Caparas did not make it to the preliminary
FERNANDO JOSEF, MS. SUSAN S. LARA, MR. ALFRED YUSON, MS. JING shortlist and respondent Moreno was not included in the second shortlist.
PANGANIBAN-MENDOZA, MR. ROMULO BAQUIRAN, JR., MR. CARLJOE
JAVIER, MS. REBECCA T. ANONUEVO, MR. JP ANTHONY D. CUNADA, MS. Yet, the four of them were treated differently and considered favorably when
LEAH NAVARRO, MR. MARK MEILLY, MR. VERGEL O. SANTOS, MR. GIL they were exempted from the rigorous screening process of the NCCA and the CCP
OLEA MENDOZA, MR. EDGAR C. SAMAR, MS. CHRISTINE BELLEN, MR. and conferred the Order of National Artists. The Committee on Honors and the
ANGELO R. LACUESTA, MS. ANNA MARIA KATIGBAK-LACUESTA, MR. LEX former President effectively treated respondents Guidote-Alvarez, Caparas,
LEDESMA, MS. KELLY PERIQUET, MS. CARLA PACIS, MR. J. ALBERT Mañosa and Moreno as a preferred class. The special treatment accorded to
GAMBOA, MR. CESAR EVANGELISTA BUENDIA, MR. PAOLO ALCAZAREN, respondents Guidote-Alvarez, Caparas, Mañosa and Moreno fails to pass rational
MR. ALWYN C. JAVIER, MR. RAYMOND MAGNO GARLITOS, MS. GANG scrutiny. No real and substantial distinction between respondents and petitioner
BADOY, MR. LESLIE BOCOBO, MS. FRANCES BRETANA, MS. JUDITH Abad has been shown that would justify deviating from the laws, guidelines and
TORRES, MS. JANNETTE PINZON, MS. JUNE POTICAR-DALISAY, MS. established procedures, and placing respondents in an exceptional position. The
CAMILLE DE LA ROSA, MR. JAMES LADIORAY, MR. RENATO undue classification was not germane to the purpose of the law. Instead, it
CONSTANTINO, JR., and CONCERNED ARTISTS OF THE PHILIPPINES contradicted the law and well-established guidelines, rules and regulations meant
(CAP), petitioners, vs. THE EXECUTIVE SECRETARY, THE SECRETARY OF to carry the law into effect. While petitioner Abad cannot claim entitlement to the
THE DEPARTMENT OF BUDGET AND MANAGEMENT, THE CULTURAL Order of National Artists, he is entitled to be given an equal opportunity to vie for
CENTER OF THE PHILIPPINES, THE NATIONAL COMMISSION ON that honor. In view of the foregoing, there was a violation of petitioner Abad’s right
CULTURE AND THE ARTS, MS. CECILE GUIDOTE-ALVAREZ, MR. CARLO to equal protection, an interest that is substantial enough to confer him standing
MAGNO JOSE CAPARAS,1 MR. JOSE MORENO, MR. FRANCISCO MAÑOSA, in this case.
AND ALL PERSONS, PUBLIC AND PRIVATE, ACTING UNDER THEIR Remedial Law; Civil Procedure; Taxpayer’s Suit; A taxpayer’s suit is proper
INSTRUCTIONS, DIRECTION, CONTROL AND SUPERVISION IN RELATION only when there is an exercise of the spending or taxing power of the Congress.—As
TO THE CONFERMENT OF THE ORDER OF THE NATIONAL ARTIST AND regards the other concerned artists and academics as well as the CAP, their claim
THE RELEASE OF FUNDS IN RELATION TO THE CONFERMENT OF THE of deep concern for the preservation of the country’s rich cultural and artistic
HONORS AND PRIVILEGES OF THE ORDER OF NATIONAL ARTISTS ON heritage, while laudable, falls short of the injury in fact requirement of standing.
RESPONDENTS GUIDOTE-ALVAREZ, CAPARAS, MORENO AND MAÑOSA, Their assertion constitutes a generalized grievance shared in a substantially equal
respondents. measure by all or a large class of citizens. Nor can they take refuge in their status
Remedial Law; Civil Procedure; Parties; The parties who assail the as taxpayers as the case does not involve any illegal appropriation or taxation. A
constitutionality or legality of a statute or an official act must have a direct and
taxpayer’s suit is proper only when there is an exercise of the spending or taxing the Order of National Artists is confined to the names submitted to him/her by the
power of the Congress. National Commission for Culture and Arts (NCCA) and the Cultural Center of the
Same; Prohibition; Injunction; It has been held that the remedies of Philippines (CCP) Boards.—In view of the various stages of deliberation in the
prohibition and injunction are preventive and, as such, cannot be availed of to selection process and as a consequence of his/her duty to faithfully enforce the
restrain an act that is already fait accompli.—The present action is a petition for relevant laws, the discretion of the President in the matter of the Order of National
prohibition, certiorari, injunction, restraining order and all other legal, just and Artists is confined to the names submitted to him/her by the NCCA and the CCP
equitable reliefs. It has been held that the remedies of prohibition and injunction Boards. This means that the President could not have considered conferment of
are preventive and, as such, cannot be availed of to restrain an act that is the Order of National Artists on any person not considered and recommended by
already fait accompli. Where the act sought to be prohibited or enjoined has the NCCA and the CCP Boards. That is the proper import of the provision of
already been accomplished or consummated, prohibition or injunction becomes Executive Order No. 435, s. 2005, that the NCCA and the CCP “shall advise the
moot. Nevertheless, even if the principal issue is already moot, this Court may still President on the conferment of the Order of National Artists.” Applying this to the
resolve its merits for the future guidance of both bench and bar. Courts will decide instant case, the former President could not have properly considered respondents
a question otherwise moot and academic if it is “capable of repetition, yet evading Guidote-Alvarez, Caparas, Mañosa and Moreno, as their names were not
review.” recommended by the NCCA and the CCP Boards. Otherwise, not only will the
Presidency; Order of National Artists; The President’s discretion in the stringent selection and meticulous screening process be rendered futile, the
conferment of the Order of National Artists should be exercised in accordance with respective mandates of the NCCA and the CCP Board of Trustees under relevant
the duty to faithfully execute the relevant laws.—In the matter of the conferment of laws to administer the conferment of Order of National Artists, draft the rules and
the Order of National Artists, the President may or may not adopt the regulations to guide its deliberations, formulate and implement policies and plans,
recommendation or advice of the NCCA and the CCP Boards. In other words, the and undertake any and all necessary measures in that regard will also become
advice of the NCCA and the CCP is subject to the President’s discretion. meaningless.
Nevertheless, the President’s discretion on the matter is not totally unfettered, nor Same; Same; Equal Protection of the Law; There was a violation of the equal
the role of the NCCA and the CCP Boards meaningless. Discretion is not a free- protection clause of the Constitution when the former President gave preferential
spirited stallion that runs and roams wherever it pleases but is reined in to keep treatment to respondents Guidote-Alvarez, Caparas, Mañosa and Moreno; The
it from straying. In its classic formulation, “discretion is not unconfined and conferment of the Order of National Artists on said respondents was therefore made
vagrant” but “canalized within banks that keep it from overflowing.” The with grave abuse of discretion and should be set aside.—There was a violation of
President’s power must be exercised in accordance with existing laws. Section 17, the equal protection clause of the Constitution when the former President gave
Article VII of the Constitution prescribes faithful execution of the laws by the preferential treatment to respondents Guidote Alvarez, Caparas, Mañosa and
President: Sec. 17. The President shall have control of all the executive Moreno. The former President’s constitutional duty to faithfully execute the laws
departments, bureaus and offices. He shall ensure that the laws be faithfully and observe the rules, guidelines and policies of the NCCA and the CCP as to the
executed. (Emphasis supplied.) The President’s discretion in the conferment of selection of the nominees for conferment of the Order of National Artists proscribed
the Order of National Artists should be exercised in accordance with the duty to her from having a free and uninhibited hand in the conferment of the said award.
faithfully execute the relevant laws. The faithful execution clause is best construed The manifest disregard of the rules, guidelines and processes of the NCCA and the
as an obligation imposed on the President, not a separate grant of power. It simply CCP was an arbitrary act that unduly favored respondents Guidote-Alvarez,
underscores the rule of law and, corollarily, the cardinal principle that the Caparas, Mañosa and Moreno. The conferment of the Order of National Artists on
President is not above the laws but is obliged to obey and execute them. This is said respondents was therefore made with grave abuse of discretion and should be
precisely why the law provides that “[a]dministrative or executive acts, orders and set aside.
regulations shall be valid only when they are not contrary to the laws or the SPECIAL CIVIL ACTION in the Supreme Court. Certiorari.
Constitution.” The facts are stated in the opinion of the Court.
Administrative Regulations; Statutes; An administrative regulation adopted Mary Grace C. Panganiban-Mendoza for petitioners.
pursuant to law has the force and effect of law.—We have held that an Kapunan, Lotilla, Garcia & Castillo Law Offices for respondent.
administrative regulation adopted pursuant to law has the force and effect of law. The Government Corporate Counsel for respondent Cultural Center of the
Thus, the rules, guidelines and policies regarding the Order of National Artists Philippines.
jointly issued by the CCP Board of Trustees and the NCCA pursuant to their M.M. Lazaro & Associates for private respondent Carlo Magno Jose Caparas.
respective statutory mandates have the force and effect of law. Until set aside, they Verano Law Firm for respondent Francisco T. Mañosa.
are binding upon executive and administrative agencies, including the President LEONARDO-DE CASTRO, J.:
himself/herself as chief executor of laws. Art has traditionally been viewed as the expression of everything that is true,
Presidency; Order of National Artists; In view of the various stages of good and beautiful. As such, it is perceived to evoke and produce a spirit of
deliberation in the selection process and as a consequence of his/her duty to harmony. Art is also considered as a civilizing force, a catalyst of nation-building.
faithfully enforce the relevant laws, the discretion of the President in the matter of The notion of art and artists as privileged expressions of national culture helped
shape the grand narratives of the nation and shared symbols of the people. The As both the CCP Board of Trustees and the NCCA have been mandated by law
artist does not simply express his/her own individual inspiration but articulates to promote, develop and protect the Philippine national culture and the arts, and
the deeper aspirations of history and the soul of the people.2 The law recognizes authorized to give awards to deserving Filipino artists, the two bodies decided to
this role and views art as something that “reflects and shapes values, beliefs, team up and jointly administer the National Artists Award. 9 Thereafter, they
aspirations, thereby defining a people’s national identity.” 3 If unduly politicized, reviewed the guidelines for the nomination, selection and administration of the
however, art and artists could stir controversy and may even cause discord, as National Artists Award. Pursuant to their respective powers to draft and
what happened in this case. promulgate rules, regulations and measures to guide them in their deliberations
The Antecedents in the choice of National Artists, the CCP and NCCA adopted the following revised
History of the Order of National Artists guidelines in September 2007:10
On April 27, 1972, former President Ferdinand E. Marcos issued Proclamation 4. ADMINISTRATION OF THE AWARD
No. 10014 and, upon recommendation of the Board of Trustees of the Cultural
Center of the Philippines (CCP), created the category of Award and Decoration of 4.1. The National Commission for Culture and the
National Artist to be awarded to Filipinos who have made distinct contributions to Arts (NCCA) shall plan, organize and
arts and letters. In the same issuance, Fernando Amorsolo was declared as the implement the Order of National
first National Artist. _______________
On May 15, 1973, Proclamation No. 11445 was issued. It amended 7 Republic Act No. 7356, Section 12(4).
Proclamation No. 1001 “by creating a National Artists Awards Committee” that 8 Id., Section 13(j).
would “administer the conferment of the category of National Artist” upon 9 Rollo, p. 82. This effort on coordination is consistent with the powers of the
deserving Filipino artists. The Committee, composed of members of the Board of NCCA to “set up a system of networking and coordination with and
Trustees of the CCP, was tasked to “draft the rules to guide its deliberations in the among all existing government cultural agencies for the effective
choice of National Artists, to the end that those who have created a body of work implementation of programs and activities” under Section 13(c) of
in the arts and letters capable of withstanding the test of time will be so Republic Act No. 7356. Section 18 in connection with Section 23(b) of
recognized.” the same law further provides that the NCCA “shall coordinate with
The authority of the National Artists Awards Committee to administer the the national cultural agencies including but not limited to the Cultural
conferment of the National Artist Award was again reiterated in Presidential Center of the Philippines” with the NCCA as “over all policy-making
Decree No. 2086issued on June 7, 1973. and coordinating body.”
On April 3, 1992, Republic Act No. 7356, otherwise known as the Law Creating 10 Id., at pp. 138-144.
the National Commission for Culture and the Arts, was signed into law. It 279
established the National Commission for Culture and the Arts (NCCA) and gave VOL. 701, JULY 16, 2013 279
it an extensive mandate over the development, promotion and preservation of the
National Artist for Literature Virgilio Almario, et al. vs. The Executive Secretary
Filipino national culture and arts and the Filipino cultural heritage. The NCCA
Artists in coordination with the Cultural Center of the
was tasked with the following:
Philippines (CCP).
Sec. 8. The Commission.—A National Commission for Culture and
Arts is hereby created to formulate policies for the development of culture 4.2. It shall enlist the support and cooperation of private
and arts; implement these policies in coordination with affiliated cultural sector experts from the various fields of art to ensure
agencies; coordinate the implementation of programs of these affiliated that the awards are implemented in a successful and
agencies; administer the National Endowment Fund for Culture and Arts impartial manner.
(NEFCA); encourage artistic creation within a climate of artistic freedom; 4.3. The National Artist Award Secretariat shall
develop and promote the Filipino national culture and arts; and preserve commission art experts to form a Special Research
Filipino cultural heritage. The Commission shall be an independent Group who shall verify information submitted on
agency. It shall render an annual report of its activities and achievements nominees and provide essential data. They shall be
to the President and to Congress. selected for their specialization and familiarity with
Among the specific mandates of the NCCA under Republic Act No. 7356 is to the works and accomplishments of nominated artists.
“extend recognition of artistic achievement through awards, grants and services to
artists and cultural groups which contribute significantly to the Filipino’s cultural 4.4. The Special Research Group shall be composed of ten
legacy.”7 In connection with this mandate, the NCCA is vested with the power to (10) to twenty (20) members who have expertise in
“advise the President on matters pertaining to culture and the arts, including the one or more fields or disciplines.
creation of a special decoration or award, for persons who have significantly 4.5. The National Artist Award Council of Experts shall
contributed to the development and promotion of Philippine culture and arts.”8 be created before or during the nomination period. It
is tasked to screen nominees and recommend to the National Artist for Literature Virgilio Almario, et al. vs. The Executive Secretary
NCCA and CCP Boards the candidates for the Order for purposes of adding new members or replacements.
of National Artists. It shall be composed of highly 4.10. The members of the National Artist Award Council of Experts shall
regarded peers, scholars, (including cultural serve for a fixed term of three (3) years.
philosophers and historians), academicians, 5. CRITERIA FOR SELECTION
researchers, art critics, and other knowledgeable The Order of National Artists shall be given to:
individuals. A wider age-range of experts who would 5.1 Living artists who are Filipino citizens at the time of
have first-hand knowledge of achievements of nomination, as well as those who died after the
nominees shall be considered. establishment of the award in 1972 but were Filipino
4.6. The selection of the members of the National Artist citizens at the time of their death.
Award Council of Experts shall be based on the 5.2 Artists who through the content and form of their works
following criteria: have contributed in building a Filipino sense of nationhood.
(a) should have achieved authority, credibility and track 5.3. Artists who have pioneered in a mode of creative expression
record in his field(s) of expertise; or style, thus, earning distinction and making an impact on
(b) should have extensive knowledge in his field(s) and his succeeding generations of artists.
views on Philippine art and culture must be national 5.4. Artists who have created a substantial and significant body
in perspective; of works and/or consistently displayed excellence in the
(c) should be a recognized authority in the study or practice of their art form thus enriching artistic expression
research of Philippine art and culture; or style.
(d) must be willing to devote sufficient time and effort to 5.5 Artists who enjoy broad acceptance through:
the work of the Council; 5.5.1. prestigious national and/or international
(e) must be willing to sign a non-disclosure statement in recognition, such as the Gawad CCP Para sa
order to safeguard the confidentiality of the Sining, CCP Thirteen Artists Award and NCCA
deliberations; Alab ng Haraya
(f) must not have been convicted with finality of any 5.5.2. critical acclaim and/or reviews of their works
crime by a court of justice or dismissed for cause by 5.5.3. respect and esteem from peers.
any organization, whether public or private.
4.7. The National Artist Award Council of Experts shall be composed of a 6. NOMINATION PROCEDURE
maximum of seven (7) members each of the seven (7) 6.1. The National Artist Award Secretariat shall announce the
areas/disciplines. The living National Artists will automatically opening of nominations through media releases and letters
become members in addition to the forty-nine (49) selected to qualified organizations.
members. These members will constitute the first deliberation panel 6.2. Candidates may be nominated under one or more of the
and will be invited to evaluate the nominations and materials following categories:
submitted by the Special Research Group. 6.2.1. Dance – choreography, direction and/or
4.8. Any member of the Council of Experts who is nominated or related to performance.
a nominee up to the fourth degree of consanguinity or affinity shall
inhibit himself/herself from the deliberation process. Likewise, any 6.2.2. Music – composition, direction, and/or
member may decline to participate in the deliberation for any reason performance.
or may be removed for just cause upon recommendation to the NCCA 6.2.3. Theater – direction, performance and/or
Board by at least two thirds (2/3) of the members; in which case, the production design.
National Artist Award Secretariat shall again select the 6.2.4. Visual Arts – painting, sculpture,
replacements for those who decline or resigned until the first printmaking, photography, installation art,
deliberation panel is completed. mixed media works, illustration,
4.9. The list of nominated members of the National Artist Award Council comics/komiks, graphic arts, performance
of Experts shall be reviewed by the National Artist Award Secretariat art and/or imaging.
as needed,
6.2.5. Literature – poetry, fiction (short story, novel
281 and play); non-fiction (essay, journalism,
VOL. 701, JULY 16, 2013 281 literary criticism and historical literature).
6.2.6. Film and Broadcast Arts – direction, writing, National Commission for Culture and the Arts
production design, cinematography, editing, 633 General Luna Street, Intramuros, Manila
camera work, and/or performance.
6.2.7. Architecture, Design and Allied Arts – 7. SCREENING AND SELECTION PROCESS
architecture design, interior design, 7.1. The National Artist Award Secretariat shall pre-
industrial arts design, landscape architecture screen the nominees based on technical guideline
and fashion design. items 5.1, 6.2, 6.3, 6.4, 6.5 and 6.6. The pre-screening
6.3. Nominations for the Order of National Artists may be shall not be based on the accomplishments and
submitted by government and non-government cultural merits of the nominee.
organizations and educational institutions, as well as 7.2. The Special Research Group shall accomplish its task
private foundations and councils. within six (6) months. The main objective is to verify the
6.4. Members of the Special Research Group, as well as validity of the data, and evaluate the quality, true value
agencies attached to the NCCA and CCP shall not submit and significance of works according to the criteria. It shall
nominations. come up with the updated and comprehensive profiles of
6.5. NCCA and CCP Board members and consultants nominees reflecting their most outstanding achievements.
and NCCA and CCP officers and staff are 7.3. The National Artist Award Secretariat will meet to review
automatically disqualified from being nominated. the list of nominees for oversights. Consequently, deserving
6.6. Nominations shall be accepted only when these are nominees shall be added to the list.
submitted in writing and with proper supporting 7.4. The first deliberation panel (Council of Experts) shall be
documentation, as follows: intradisciplinary. The panelists shall be grouped according
6.6.1. A cover letter signed by the head or designated to their respective fields of expertise or disciplines to
representative of the nominating organization. shortlist the nominees in their disciplines or categories for
The cover letter shall be accompanied by a Board presentation to the second deliberation panel.
Resolution approving the nominee concerned with 7.5. The second deliberation panel shall be composed of a
the said resolution signed by the organization different set of experts from the first deliberation panel
President and duly certified by the Board [three (3) experts each of the seven (7) areas/discipline] and
Secretary. may include members from varying backgrounds such as
6.6.2. A duly accomplished nomination form; critics and academicians. The achievements of each
6.6.3. A detailed curriculum vitae of the nominee; shortlisted nominee shall be presented by one designated
6.6.4. A list of the nominee’s significant works categorized member of Council of Experts. Then panel deliberates and
according to the criteria; ranks the shortlisted nominees according to the order of
6.6.5. The latest photograph (color or black and white) of precedence following the set criteria of the Order of
the nominee, either 5[”] x 7” or 8[”] x 11”; National Artists. In extreme cases, the Second Deliberation
6.6.6. Pertinent information materials on the nominee’s may add new names to the lists.
significant works (on CDs, VCDs and DVDs); 7.6. The second deliberation panel may recommend not to give
6.6.7. Copies of published reviews; award in any category if no nominee is found deserving.
6.6.8. Any other document that may be required. The number of awardees shall also depend on the
6.7. Nominations received beyond the announced deadline for availability of funds. All decisions and recommendations
the submission of nominations shall not be considered. shall be in writing.
6.8. The National Artist Award Secretariat shall announce the 7.7. The recommendations from the Second Deliberation Panel
opening of nominations through media releases. of the National Artist Award Council of Experts shall then
6.9. All inquiries and nominations shall be submitted to be presented to the joint boards of NCCA and CCP for final
The NATIONAL ARTIST AWARD SECRETARIAT selection. The presentors shall prepare their presentation
Office of the Artistic Director in writing together with an audio-visual presentation or
Cultural Center of the Philippines powerpoint presentation. Written interpellations/opinions
Roxas Boulevard, 1300 Pasay City will be accepted from selected critics. The review shall be
or based on the ranking done by the Second Deliberation. The
The NATIONAL ARTIST AWARD SECRETARIAT voting shall be across disciplines. The National Artists
Office of the Deputy Executive Director
will be given the option whether to vote on all of the Philippines and the National Commission for Culture and the
categories or on his/her particular discipline. Arts.”12Executive Order No. 236, s. 2003, further created a Committee on Honors
7.8. Proxy votes will not be allowed in the Selection Process. to “assist the President in evaluating nominations for recipients of
Designation of permanent representatives of agencies Honors,”13 including the Order of National Artists, and presidential awards. The
should be made at the outset to make them regular Board Committee on Honors has been allowed to “authorize relevant department or
members of NCCA and thus, may be allowed to cast votes. government agencies to maintain Honors and/or Awards Committees to process
7.9. The list of awardees shall be submitted to the nominations for Honors and/or Presidential Awards.14 In this connection, Section
President of the Republic of the Philippines for 2.4(A) of the Implementing Rules and Regulations15 of Executive Order No. 236, s.
confirmation, proclamation and conferral. 2003, states:

8. PRESENTATION OF THE AWARDS 2.4: Awards Committees


8.1. The Order of National Artists shall not be conferred more There shall be two types of awards committees: the Committee on Honors
frequently than every three (3) years. and the various awards committees in the various units of the government
8.2. The Order of National Artists shall be conferred by service.
the President of the Philippines on June 11 or any A. The Committee on Honors
appropriate date in fitting ceremonies to be organized The Committee on Honors serves as a National Awards Committee. It is
by the National Artist Secretariat. composed of the following:
8.3. The medallion of the Order of National Artists and The Executive Secretary, Chairman
citation shall be given to the honoree during the The Secretary of Foreign Affairs, Vice-Chairman
conferment ceremony. The cash award of Head, Presidential Management Staff, member
P100,000.00 in cheque shall be given immediately Presidential Assistant for Historical Affairs, member
after the ceremony or at another time and place as Chief of Presidential Protocol, member
requested by the honoree. Chief of Protocol, DFA, member
All nominations from the various awards committees must be submitted to
8.4. A posthumous conferral consisting of the medallion the Committee on Honors via the Chancellery of Philippine Orders and
and citation shall be given to the family or legal heir/s State Decorations. The Chancellery shall process nominations for the
of the honoree. The cash award of P75,000.00 in consideration of the Committee on Honors. The Committee on Honors shall
cheque shall be given to the honoree’s legal heir/s or screen and recommend these nominations to the President.
a representative designated by the family The Committee on Honors shall, as a general rule, serve as a
immediately after the ceremony or at another time screening committee to ensure that nominations received from the
and place as requested by the family. (Emphases various awards committees meet two tests: that there has not been
supplied.) an abuse of discretion in making the nomination, and that the
In 1996, the NCCA and the CCP created a National Artist Award Secretariat nominee is in good standing. Should a nomination meet these
composed of the NCCA Executive Director as Chairperson, the CCP President as criteria, a recommendation to the President for conferment shall
Vice-Chairperson, and the NCCA Deputy Executive Director, the CCP Vice- be made.
President/Artistic Director, the NCCA National Artist Award Officer and the CCP The President of the Philippines takes the recommendations of the
National Artist Award Officer as members. They also centralized with the NCCA Committee on Honors in the highest consideration when making the final
all financial resources and management for the administration of the National decision on the conferment of awards. (Emphasis supplied.)
Artists Award. They added another layer to the selection process to involve and
allow the participation of more members of the arts and culture sector of the Executive Order No. 435, s. 2005, entitled Amending Section 5(IV) of Executive
Philippines in the selection of who may be proclaimed a National Artist. Order No. 236 Entitled “Establishing the Honors Code of the Philippines to Create an
On September 19, 2003, Executive Order No. 236, s. 2003, Order of Precedence of Honors Conferred and for Other Purposes” was subsequently
entitled Establishing the Honors Code of the Philippines to Create an Order of issued on June 8, 2005. It amended the wording of Executive Order No. 236, s. 2003,
Precedence of Honors Conferred and for Other Purposes, was issued. The National on the Order of National Artists and clarified that the NCCA and the CCP “shall
Artists Award was renamed the Order of National Artists and raised to the level advise the President on the conferment of the Order of National Artists.”
of a Cultural Order, fourth in precedence among the orders and decorations that Controversy Surrounding the 2009
comprise the Honors of the Philippines.11 Executive Order No. 236, s. 2003, Order of National Artists
recognizes the vital role of the NCCA and the CCP in identifying Filipinos who
have made distinct contributions to arts and letters and states that the National Petitioners alleged that on January 30, 2007, a joint meeting of the NCCA
Artist recognition is conferred “upon the recommendation of the Cultural Center Board of Commissioners and the CCP Board of Trustees was held to discuss,
among others, the evaluation of the 2009 Order of National Artists and the 22 Id., at p. 43.
convening of the National Artist Award Secretariat. The nomination period was 23Id., at p. 22.
set for September 2007 to December 31, 2007, which was later extended to 293
February 28, 2008. The pre-screening of nominations was held from January to VOL. 701, JULY 16, 2013 293
March 200816
National Artist for Literature Virgilio Almario, et al. vs. The Executive Secretary
On April 3, 2009, the First Deliberation Panel met. 17 A total of 87
May 6, 2009
nominees18 were considered during the deliberation and
Her Excellency GLORIA MACAPAGAL-ARROYO
On April 23, 2009, the Second Deliberation Panel purportedly composed of an President of the Philippines
entirely new set of Council of Experts met and shortlisted 13 out of the 32 names Malacañan Palace, Manila
in the preliminary shortlist.20 On May 6, 2009, the final deliberation was
conducted by the 30-member Final Deliberation Panel comprised of the CCP Board Subject: 2009 Order of National Artist Awardees
of Trustees and the NCCA Board of Commissioners and the living National Dear President Arroyo:
Artists.21 From the 13 names in the second shortlist, a final list of four names was We are respectfully submitting a recommendation of the NCCA Board
agreed upon.22The final list, according to rank, follows: of Trustees and CCP Board of Trustees for the Proclamation of the following
Name Art Field/Category Number of Votes as 2009 Order of National Artists:
1. Mr. MANUEL CONDE+
Manuel Conde (+) Film and Broadcast Arts (Film) 26 (Posthumous) – Film and Broadcast Arts
2. Dr. RAMON SANTOS – Music
Ramon Santos Music 19 3. Mr. LAZARO FRANCISCO+
(Posthumous) – Literature
Lazaro Francisco (+) Literature 15 4. Mr. FEDERICO AGUILAR-ALCUAZ – Visual Arts
Federico Aguilar-Alcuaz Visual Arts 15 The above persons were identified by experts in the various fields of
arts and culture, including living National Artists. An intensive selection
process was observed following established practice. In the past, awards
On May 6, 2009, a letter, signed jointly by the Chairperson of the NCCA,
were presented by the President at a Ceremony held at the Malacañan
Undersecretary Vilma Labrador, and the President and Artistic Director of the
Palace followed by a program called “Parangal” at the Cultural Center of
CCP, Mr. Nestor Jardin, was sent to the President.23 The letter stated, thus:
the Philippines. We also propose to continue with past practice of
_______________
celebrating the life and works of the four (4) Order of National Artists
Music Santos, Ramon through an exhibit that will open and a commemorative publication that
will be released on the day of the proclamation.
Theater Bonifacio, Amelia Lapeña We respectfully suggest, subject to Her Excellency’s availability, that
Mabesa, Antonio the Proclamation be on June 11, 2009, if possible at the Malacañan Palace.
Thank you for your kind attention.
Visual Arts Alcuaz, Federico Aguilar Very respectfully yours,
(Sgd.)
Castrillo, Eduardo
VILMA L. LABRADOR
Literature Francisco, Lazaro+ Chairman
National Commission for Culture and the Arts
Jalandoni, Magdalena+ (Sgd.)
NESTOR O. JARDIN
Film and Broadcast Arts Conde, Manuel+ (Film) President and Artistic Director
Trinidad, Francisco+ (Broadcast Arts) Cultural Center of the Philippines24
According to respondents, the aforementioned letter was referred by the Office
Architecture, Design and Allied Arts Arguelles, Carlos+ (Architecture) of the President to the Committee on Honors. Meanwhile, the Office of the
President allegedly received nominations from various sectors, cultural groups and
Salazar, Joe+ (Fashion Design)
individuals strongly endorsing private respondents Cecile Guidote-Alvarez, Carlo
Magno Jose Caparas, Francisco Mañosa and Jose Moreno. The Committee on
21 Id., at p. 22.
Honors purportedly processed these nominations and invited resource persons to academics, and the Concerned Artists of the Philippines (CAP). The National
validate the qualifications and credentials of the nominees.25 Artists assert an “actual as well as legal interest in maintaining the reputation of
The Committee on Honors thereafter submitted a memorandum to then the Order of National Artists.”31 In particular, they invoke their right to due
President Gloria Macapagal-Arroyo recommending the conferment of the Order of process not to have the honor they have been conferred with diminished by the
National Artists on the four recommendees of the NCCA and the CCP Boards, as irregular and questionable conferment of the award on respondents Guidote-
well as on private respondents Guidote-Alvarez, Caparas, Mañosa and Moreno. Alvarez, Caparas, Mañosa and Moreno. For petitioners, this would adversely affect
Acting on this recommendation, Proclamation No. 1823 declaring Manuel Conde a their right to live a meaningful life as it detracts not only from their right to enjoy
National Artist was issued on June 30, 2009. Subsequently, on July 6, 2009, their honor as a fruit of their lifelong labor but also from the respect of their
Proclamation Nos. 1824 to 1829 were issued declaring Lazaro Francisco, Federico peers.32
AguilarAlcuaz and private respondents Guidote-Alvarez, Caparas, Mañosa and The cultural workers, academics and CAP claim to be Filipinos who are deeply
Moreno, respectively, as National Artists. This was subsequently announced to the concerned with the preservation of the country’s rich cultural and artistic heritage.
public by then Executive Secretary Eduardo Ermita on July 29, 2009.26 As taxpayers, they are concerned about the use of public monies for illegal
appointments or spurious acts of discretion.33
Convinced that, by law, it is the exclusive province of the NCCA Board of
Commissioners and the CCP Board of Trustees to select those who will be All of the petitioners claim that former President Macapagal-Arroyo gravely
conferred the Order of National Artists and to set the standard for entry into that abused her discretion in disregarding the results of the rigorous screening and
select group, petitioners instituted this petition for prohibition, certiorari and selection process for the Order of National Artists and in substituting her own
injunction (with prayer for restraining order) praying that the Order of National choice for those of the Deliberation Panels. According to petitioners, the President’s
Artists be conferred on Dr. Santos and that the conferment of the Order of National discretion to name National Artists is not absolute but limited. In particular, her
Artists on respondents Guidote-Alvarez, Caparas, Mañosa and Moreno be enjoined discretion on the matter cannot be exercised in the absence of or against the
and declared to have been rendered in grave abuse of discretion. 27 recommendation of the NCCA and the CCP. In adding the names of respondents
In a Resolution28 dated August 25, 2009, the Court issued a status Caparas, Guidote-Alvarez, Mañosa and Moreno while dropping Dr. Santos from
quo order29 enjoining “public respondents” “from conferring the rank and title of the list of conferees, the President’s own choices constituted the majority of the
the Order of National Artists on private respondents; from releasing the cash awardees in utter disregard of the choices of the NCCA and the CCP and the arts
awards that accompany such conferment and recognition; and from holding the and culture community which were arrived at after a long and rigorous process of
acknowledgment ceremonies for recognition of the private respondents as National screening and deliberation. Moreover, the name of Dr. Santos as National Artist
Artists.” for Music was deleted from the final list submitted by the NCCA and the CCP
What is the nature and scope of the power of the President to confer the Order Boards without clearly indicating the basis thereof. For petitioners, the President’s
of the National Artists and how should it be exercised? This is the essential issue discretion to name National Artists cannot be exercised to defeat the
presented in this case. It will determine whether the proclamation of respondents recommendations made by the CCP and NCCA Boards after a long and rigorous
as National Artists is valid. Preliminary procedural issues on the standing of the screening process and with the benefit of expertise and experience. The addition of
petitioners and the propriety of the remedies taken, 30however, call for resolution four names to the final list submitted by the Boards of the CCP and the NCCA and
as a prerequisite to the discussion of the main question. the deletion of one name from the said list constituted a substitution of judgment
_______________ by the President and a unilateral reconsideration without clear justification of the
27Id., at pp. 34-35. decision of the First, Second and Final Deliberation Panels composed of experts. 34
28 Id., at pp. 49-50. Petitioners further argue that the choice of respondent Guidote-Alvarez was
29 Id., at pp. 51-55. illegal and unethical because, as the then Executive Director of the NCCA and
30 Other procedural issues (such as violation of the hierarchy of courts and presidential adviser on culture and arts, she was disqualified from even being
lack of verification by some of the petitioners) have been raised by the public nominated.35 Moreover, such action on the part of the former President constituted
respondents and respondent Caparas. In view of the purely legal question, grave abuse of discretion as it gave preferential treatment to respondent Guidote-
substantial merit and paramount public interest involved in this case, however, Alvarez by naming the latter a National Artist despite her not having been
the said procedural infirmities have been brushed aside and strict technicalities nominated and, thus, not subjected to the screening process provided by the rules
relaxed. (Relevant to the relaxation of the rule on the hierachy of courts, see for selection to the Order of National Artists. Her inclusion in the list by the
296 President represented a clear and manifest favor given by the President in that
296 SUPREME COURT REPORTS ANNOTATED she was exempted from the process that all other artists have to undergo.
According to petitioners, it may be said that the President used a different
National Artist for Literature Virgilio Almario, et al. vs. The Executive Secretary procedure to qualify respondent Guidote-Alvarez. This was clearly grave abuse of
Contention of the Parties discretion for being manifest and undue bias violative of the equal protection
A perusal of the pleadings submitted by the petitioners reveals that they are clause.36
an aggrupation of at least three groups, the National Artists, cultural workers and
Respondent Caparas refutes the contention of the petitioning National Artists the NCCA and the CCP. The implementing rules and regulations of Executive
and insists that there could be no prejudice to the latter. They remain to be Order No. 236, s. 2003, recognized the binding character of the recommendation of
National Artists and continue to receive the emoluments, benefits and other the NCCA and the CCP Boards and limited the authority of the Committee on
privileges pertaining to them by virtue of that honor. On the other hand, all the Honors to the determination that (1) there has been no grave abuse of discretion
other petitioners failed to show any material and personal injury or harm caused on the part of the NCCA and the CCP Boards in making the nomination, and (2)
to them by the conferment of the Order of National Artists on respondents Guidote- the nominee is in good standing. Where a nomination meets the said two criteria,
Alvarez, Caparas, Mañosa and Moreno. The rule on standing may not be relaxed a recommendation to the President to confer the award shall be made.52
in favor of the petitioners as no question of constitutionality has been raised and
no issue of transcendental importance is involved.37 The OSG further argued that, while the President exercises control over the
Respondent Caparas further argues that the remedies of prohibition and NCCA and the CCP, the President has the duty to faithfully execute the laws,
injunction are improper as the act sought to be enjoined — the declaration of including the NCCA-CCP guidelines for selection of National Artists and the
respondents Guidote-Alvarez, Caparas, Mañosa and Moreno as National Artists implementing rules of Executive Order No. 236, s. 2003. Moreover, the laws
— had already been consummated. In particular, respondent Caparas was already recognize the expertise of the NCCA and the CCP in the arts and tasked them to
proclaimed National Artist through Proclamation No. 1827 issued on July 6, screen and select the artists to be conferred the Order of National Artists. Their
2009.38 mandate is clear and exclusive as no other agency possesses such expertise.53
The OSG also assailed the former President’s choice of respondent Guidote-
On the merits, respondent Caparas contends that no grave abuse of discretion Alvarez for being contrary to Republic Act No. 7356.54 Section 11 of the said law
attended his proclamation as National Artist. The former President considered the provides:
respective recommendations of the NCCA and the CCP Boards and of the Sec. 11. Membership Restrictions.—During his/ her term as member
Committee on Honors in eventually declaring him (Caparas) as National Artist. of the Commission, a Commissioner shall not be eligible for any grant, or
The function of the NCCA and the CCP Boards is simply to advise the President. such other financial aid from the Commission as an individual: Provided,
The award of the Order of National Artists is the exclusive prerogative of the however, That he/she may compete for grants and awards on the same level
President who is not bound in any way by the recommendation of the NCCA and as other artists one (1) year after his/her term shall have expired.
the CCP Boards. The implementing rules and regulations or guidelines of the The omission of the word “award” in the first portion of the above provision
NCCA cannot restrict or limit the exclusive power of the President to select the appears to be unintentional as shown by the proviso which states that a member
recipients of the Order of National Artists.39 may compete for grants and awards only one year after his or her term shall have
For her part, in a letter40 dated March 11, 2010, respondent Guidote-Alvarez expired. As such, respondent Guidote-Alvarez is restricted and disqualified from
manifested that she was waiving her right to file her comment on the petition and being conferred the 2009 Order of National Artists.55
submitted herself to the Court’s discretion and wisdom.
Respondent Mañosa manifested that his creations speak for themselves as his The Court’s Ruling
contribution to Filipino cultural heritage and his worthiness to receive the award. Standing of the Petitioners
Nonetheless, he expressed his conviction that the Order of National Artists is not Standing is the determination of whether a specific person is the proper party
a right but a privilege that he would willingly relinquish should he be found not to bring a matter to the court for adjudication.56 The gist of the question of standing
worthy of it.41 is whether a party alleges such personal stake in the outcome of the controversy
Respondent Moreno did not file any pleading despite being given several as to assure that concrete adverseness which sharpens the presentation of issues
opportunities to do so. Hence, the Court dispensed with his pleadings.42 upon which the court depends for illumination of difficult constitutional
In a Resolution dated July 12, 2011, this Court gave due course to the petition questions.57
and required the parties to file their respective memoranda.43 Respondent Caparas The parties who assail the constitutionality or legality of a statute or an official
filed his memorandum on September 8, 2011,44 the CCP filed its memorandum on act must have a direct and personal interest. They must show not only that the
September 19, 2011,45respondent Mañosa on September 20, 2011,46 and the Office law or any governmental act is invalid, but also that they sustained or are in
of the Solicitor General filed a manifestation stating that it is adopting its immediate danger of sustaining some direct injury as a result of its enforcement,
comment as its memorandum on September 21, 2011.47 Respondent Moreno failed and not merely that they suffer thereby in some indefinite way. They must show
to file a Memorandum, hence, the Court resolved to dispense with the that they have been or are about to be denied some right or privilege to which they
same.48 Petitioners filed their Memorandum on May 14, 2012.49 are lawfully entitled or that they are about to be subjected to some burdens or
On the other hand, the original position of the Office of the Solicitor General penalties by reason of the statute or act complained of.58
(OSG) was similar to that of respondent Caparas. 50 In a subsequent In this case, we find that the petitioning National Artists will be denied some
manifestation,51 however, the OSG stated that the current Board of right or privilege to which they are entitled as members of the Order of National
Commissioners of the NCCA agree with the petitioners that the President cannot Artists as a result of the conferment of the award on respondents Guidote-Alvarez,
honor as a National Artist one who was not recommended by the joint Boards of
Caparas, Mañosa and Moreno. In particular, they will be denied the privilege of necessary to serve that interest. It is used in classifications based on race, national
exclusive membership in the Order of National Artists. origin, religion alienage, denial of the right to vote, access to courts and other
In accordance with Section 2(a)59 of Executive Order No. 236, s. 2003, the rights recognized as fundamental. (Bernas, Joaquin S.J., THE 1987 CONSTITUTION
Order of National Artists is “an exclusive association of honored individuals.” To OF THE REPUBLIC OF THE PHILIPPINES: A COMMENTARY [2009 edition], pp. 139-140).
ensure the exclusivity of the membership in the Order, a rigid nomination and 305
screening process has been established with different sets of renowned artists and VOL. 701, JULY 16, 2013 305
respected art critics invited to sit as the Council of Experts for the First and Second
Deliberation Panels. Moreover, all living National Artists are given a voice on who National Artist for Literature Virgilio Almario, et al. vs. The Executive Secretary
should be included in their exclusive club as they automatically become members procedures, and placing respondents in an exceptional position. The undue
of the Final Deliberation Panel that will vote on who should be included in the classification was not germane to the purpose of the law. Instead, it contradicted
final list to be submitted to the President for conferment of the Order of National the law and well-established guidelines, rules and regulations meant to carry the
Artists. To allow the untrammeled discretion and authority of the President to law into effect. While petitioner Abad cannot claim entitlement to the Order of
confer the Order of National Artists without regard to the stringent screening and National Artists,61 he is entitled to be given an equal opportunity to vie for that
rigorous selection process established by the NCCA and the CCP will diminish, if honor. In view of the foregoing, there was a violation of petitioner Abad’s right to
not negate, the exclusive nature of the said Order. It will unduly subject the equal protection, an interest that is substantial enough to confer him standing in
selection and conferment of the Order of National Artists to politics rather than to this case.
principles and procedures. It will subvert the transparent and rigorous process and As regards the other concerned artists and academics as well as the CAP, their
allow entry to the exclusive Order of National Artists through a secret backdoor of claim of deep concern for the preservation of the country’s rich cultural and artistic
lobbying, back channeling and political accommodation. heritage, while laudable, falls short of the injury in fact requirement of standing.
Among the other petitioners, Prof. Gemino Abad presents a unique valid Their assertion constitutes a generalized grievance shared in a substantially equal
personal and substantial interest. Like respondents Caparas, Mañosa and Moreno, measure by all or a large class of citizens.62 Nor can they take refuge in their status
he was among the 87 nominees for the 2009 Order of National Artists. Like as taxpayers as the case does not involve any illegal appropriation or taxation. A
respondent Moreno, he made it to the preliminary shortlist. As he did not make it taxpayer’s suit is proper only when there is an exercise of the spending or taxing
to the second shortlist, he was not considered by the Final Deliberation Panel, power of the Congress.63
more so by the former President. Nonetheless, as a reading of the petition shows that it has advanced an issue
It should be recalled too that respondent Guidote-Alvarez was disqualified to which deserves the attention of this Court in view of its seriousness, novelty and
be nominated for being the Executive Director of the NCCA at that time while weight as precedent, it behooves the Court to relax the rules on standing and to
respondents Mañosa and Caparas did not make it to the preliminary shortlist and re-
respondent Moreno was not included in the second shortlist. Yet, the four of them _______________
were treated differently and considered favorably when they were exempted from 61 This is not to say that petitioner Abad is unworthy of the honor. It only
the rigorous screening process of the NCCA and the CCP and conferred the Order means that the Court is in no position to make that determination.
of National Artists. The Committee on Honors and the former President effectively 62 Warth v. Seldin, 422 U.S. 490, 499 (1975); see also David v. Macapagal-
treated respondents Guidote-Alvarez, Caparas, Mañosa and Moreno as a preferred Arroyo, 522 Phil. 705, 762; 489 SCRA 160, 223 (2006).
class. The special treatment accorded to respondents Guidote-Alvarez, Caparas, 63 Southern Hemisphere Engagement Network, Inc. v. Anti-Terrorism
Mañosa and Moreno fails to pass rational scrutiny.60 No real and substantial Council, supra note 58 at pp. 174-175; Automotive Industry Workers Alliance v.
distinction between respondents and petitioner Abad has been shown that would Romulo, 489 Phil. 710, 719; 449 SCRA 1, 11 (2005); Gonzales v. Narvasa, 392 Phil.
justify deviating from the laws, guidelines and established 518, 525; 337 SCRA 733, 741 (2000).
_______________ 306
60 The rational basis scrutiny is one of three tests used by the Court to test 306 SUPREME COURT REPORTS ANNOTATED
compliance with the equal protection clause. It is the minimal level of scrutiny
National Artist for Literature Virgilio Almario, et al. vs. The Executive Secretary
which requires that the challenged classification is rationally related to serving a
legitimate State interest. It is used when the government action is a type of solve the issue presented before it.64 Moreover, this issue is of paramount
discrimination that does not warrant the intermediate and strict levels of scrutiny. interest,65 which further justifies a liberal stance on standing.
The intermediate or middle-tier test requires the government to show that (1) the Propriety of the Remedies
challenged classification serves an important State interest, and (2) the The present action is a petition for prohibition, certiorari, injunction,
classification is at least substantially related to serving that interest. It is applied restraining order and all other legal, just and equitable reliefs.
to suspect classifications like gender or illegitimacy. The most demanding is the It has been held that the remedies of prohibition and injunction are preventive
strict scrutiny test which requires the government to show that (1) the challenged and, as such, cannot be availed of to restrain an act that is already fait
classification serves a compelling State interest, and (2) the classification is accompli.66Where the act sought to be prohibited or enjoined has already been
accomplished or consummated, prohibition or injunction becomes moot.67
Nevertheless, even if the principal issue is already moot, this Court may still “discretion is not unconfined and vagrant” but “canalized within banks that keep
resolve its merits for the future guidance of both bench and bar. Courts will decide it from overflowing.”78
a question otherwise moot and academic if it is “capable of repetition, yet evading The President’s power must be exercised in accordance with existing laws.
review.”68 Section 17, Article VII of the Constitution prescribes faithful execution of the laws
by the President:
It is an opportune time for the Court to assert its role as republican Sec. 17. The President shall have control of all the executive
schoolmaster,69 a teacher in a vital national seminar.70There are times when the departments, bureaus and offices. He shall ensure that the laws be
controversy is of such character that, to prevent its recurrence and to assure faithfully executed. (Emphasis supplied.)
respect for constitutional limitations, this Court must pass on the merits of a The President’s discretion in the conferment of the Order of National Artists
case.71 This is one such case. More than being a teaching moment, this is not the should be exercised in accordance with the duty to faithfully execute the relevant
first time that the Order of National Artists was conferred in the manner that is laws. The faithful execution clause is best construed as an obligation imposed on
being assailed in this case.72 If not addressed here and now, there is great the President, not a separate grant of power.79 It simply underscores the rule of
probability that the central question involved in this case will haunt us again in law and, corollarily, the cardinal principle that the President is not above the laws
the future. Every President may invoke absolute presidential prerogative and but is obliged to obey and execute them.80This is precisely why the law provides
thrust upon us National Artists after his or her own heart, in total disregard of the that “[a]dministrative or executive acts, orders and regulations shall be valid only
advise of the CCP and the NCCA and the voice of the community of artists, when they are not contrary to the laws or the Constitution.”81
resulting to repeated episodes of indignation and uproar from the artists and the In this connection, the powers granted to the NCCA and the CCP Boards in
public. connection with the conferment of the Order of National Artists by executive
Furthermore, if not corrected, such an act would give rise to mischief and issuances were institutionalized by two laws, namely, Presidential Decree No. 208
dangerous precedent whereby those in the corridors of power could avoid judicial dated June 7, 1973 and Republic Act No. 7356. In particular, Proclamation No.
intervention and review by merely speedily and stealthily completing the 1144 dated May 15, 1973 constituted the CCP Board as the National Artists
commission of an illegality.73 Awards Committee and tasked it to “administer the conferment of the category of
National Artist” upon deserving Filipino artists with the mandate to “draft the
In any event, the present petition is also for certiorari and there is no rules to guide its deliberations in the choice of National Artists”:
procedural bar for the Court to pass upon the question of whether the Proclamation No. 1001 dated April 27, 1972, creating the Award and
proclamations of respondents Guidote-Alvarez, Caparas, Mañosa and Moreno as Decoration of National Artist, is hereby amended by creating a National
National Artists were attended by grave abuse of presidential discretion. Artists Awards Committee, hereinafter to administer the
Limits of the President’s Discretion conferment of the category of National Artist upon those deserving
The respective powers of the CCP Board of Trustees and of the NCCA Board of thereof. The Committee, which shall be composed of members of the Board
Commissioners with respect to the conferment of the Order of National Artists are of Trustees of the Cultural Center of the Philippines, shall organize itself
clear. They jointly administer the said award and, upon their recommendation or immediately and shall draft the rules to guide its deliberations in the
advice, the President confers the Order of National Artists. choice of National Artists, to the end that those who have created a body
To “recommend” and to “advise” are synonymous. To “recommend” is “to advise of work in the arts and in letters capable of withstanding the test of time
or counsel.”74To “advise” is “to give an opinion or counsel, or recommend a plan or will be so recognized. (Emphases supplied.)
course of action; also to give notice. To encourage, inform or acquaint.” 75 “Advise” The authority of the CCP Board of Trustees as National Artists Awards
imports that it is discretionary or optional with the person addressed whether he Committee was reiterated in Presidential Decree No. 208 dated June 7, 1973.
will act on such advice or not.76 This has been clearly explained in Cojuangco, Jr. The function of the CCP Board of Trustees as National Artists Awards
v. Atty. Palma:77 Committee has been recognized under Republic Act No. 7356:
The “power to recommend” includes the power to give “advice, exhortation
or indorsement, which is essentially persuasive in character, not Sec. 18. The National Cultural Agencies.—The [NCCA] shall
binding upon the party to whom it is made.” (Emphasis supplied.) coordinate with the national cultural agencies including but not limited to
Thus, in the matter of the conferment of the Order of National Artists, the the Cultural Center of the Philippines, the Institute of Philippine
President may or may not adopt the recommendation or advice of the NCCA and Languages, the National Historical Institute, the National Library, the
the CCP Boards. In other words, the advice of the NCCA and the CCP is subject to National Museum, the Records Management and Archives Office.
the President’s discretion. However, they shall continue operating under their respective
Nevertheless, the President’s discretion on the matter is not totally unfettered, charters or as provided by law where provisions therein are not
nor the role of the NCCA and the CCP Boards meaningless. inconsistent with the provisions of this Act. They shall serve as the
Discretion is not a free-spirited stallion that runs and roams wherever it national repository and/or showcase, as the case may be, of the best of
pleases but is reined in to keep it from straying. In its classic formulation, Philippine culture and arts. For this purpose, these agencies shall submit
periodic reports, including recommendations to the [NCCA]. (Emphasis There shall be two types of awards committees: the Committee on Honors
supplied.) and the various awards committees in the various units of the government
On the other hand, the NCCA has been given the following mandate in service.
connection with the conferment of cultural or arts awards: A. The Committee on Honors
Sec. 12. Mandate.—The Commission is hereby mandated The Committee on Honors serves as a National Awards Committee. It is
to formulate and implement policies and plans in accordance with the composed of the following:
principles stated in Title 1 of this Act. The Executive Secretary, Chairman
(a) To encourage the continuing and balanced development of a The Secretary of Foreign Affairs, Vice-Chairman
pluralistic culture by the people themselves, it shall: Head, Presidential Management Staff, member
xxxx Presidential Assistant for Historical Affairs, member
(4) extend recognition of artistic achievement through awards, Chief of Presidential Protocol, member
grants and services to artists and cultural groups which Chief of Protocol, DFA, member
contribute significantly to the Filipino’s cultural legacy; All nominations from the various awards committees must be submitted to
xxxx the Committee on Honors via the Chancellery of Philippine Orders and
Sec. 13. Powers and Functions.—To carry out its mandate, the State Decorations. The Chancellery shall process nominations for the
Commission shall exercise the following powers and functions: consideration of the Committee on Honors. The Committee on Honors shall
xxx screen and recommend these nominations to the President.
(j) advise the President on matters pertaining to culture and The Committee on Honors shall, as a general rule, serve as a
the arts, including the creation of a special decoration or screening committee to ensure that nominations received from the
award, for persons who have significantly contributed to the various awards committees meet two tests: that there has not been
development and promotion of Philippine culture and arts; an abuse of discretion in making the nomination, and that the
(k) promulgate rules, regulations and undertake any and nominee is in good standing. Should a nomination meet these
all measures as may be necessary to implement this criteria, a recommendation to the President for conferment shall
Act[.] (Emphases supplied.) be made.
By virtue of their respective statutory mandates in connection with the The President of the Philippines takes the recommendations of the
conferment of the National Artist Award, the NCCA and the CCP decided to work Committee on Honors in the highest consideration when making the final
together and jointly administer the National Artist Award. They reviewed the decision on the conferment of awards. (Emphasis supplied.)
guidelines for the nomination, selection and administration of the National Artist Pursuant to the above provision of the implementing rules of Executive Order
Award, created a National Artist Award Secretariat, centralized all financial No. 236, s. 2003, the authority of the Committee on Honors is limited to
resources and management for the administration of the National Artist Award, determining whether the nominations submitted by a particular awards
and added another layer to the selection process so that more members of the arts committee, in this case, the joint NCCA and CCP Boards, have been tainted by
and culture sector of the Philippines may be involved and participate in the abuse of discretion, and whether the nominees are in good standing. Should the
selection of National Artists. nominations meet these two criteria, the Committee on Honors shall make a
We have held that an administrative regulation adopted pursuant to law has recommendation to the President for conferment of the Order of National Artists.
the force and effect of law.82 Thus, the rules, guidelines and policies regarding the In view of the various stages of deliberation in the selection process and as a
Order of National Artists jointly issued by the CCP Board of Trustees and the consequence of his/her duty to faithfully enforce the relevant laws, the discretion
NCCA pursuant to their respective statutory mandates have the force and effect of the President in the matter of the Order of National Artists is confined to the
of law. Until set aside, they are binding upon executive and administrative names submitted to him/her by the NCCA and the CCP Boards. This means that
agencies,83 including the President himself/herself as chief executor of laws. In this the President could not have considered conferment of the Order of National
connection, Section 2.5(A) of the Implementing Rules and Regulations84 of Artists on any person not considered and recommended by the NCCA and the CCP
Executive Order No. 236, s. 2003 provides: Boards. That is the proper import of the provision of Executive Order No. 435, s.
2.5: General Guidelines for Awards Committees 2005, that the NCCA and the CCP “shall advise the President on the conferment
A. National Orders of Cultural and Scientific Merit of the Order of National Artists.” Applying this to the instant case, the former
The existing modalities of the NCCA for selecting recipients for the President could not have properly considered respondents Guidote-Alvarez,
Order of National Artists, and the Gawad sa Manlilikha ng Bayan, and Caparas, Mañosa and Moreno, as their names were not recommended by the
of the NAST for selecting recipients of the Order of National NCCA and the CCP Boards. Otherwise, not only will the stringent selection and
Scientists, shall remain in force. (Emphases supplied.) meticulous screening process be rendered futile, the respective mandates of the
Section 2.4(A) of the same implementing rules further states: NCCA and the CCP Board of Trustees under relevant laws to administer the
2.4: Awards Committees conferment of Order of National Artists, draft the rules and regulations to guide
its deliberations, formulate and implement policies and plans, and undertake any WHEREFORE, the petition is hereby GRANTED in PART. Proclamation
and all necessary measures in that regard will also become meaningless. Nos. 1826 to 1829 dated July 6, 2009 proclaiming respondents Cecile Guidote-
Furthermore, with respect to respondent Guidote-Alvarez who was the Alvarez, Carlo Magno Jose Caparas, Francisco Mañosa, and Jose Moreno,
Executive Director of the NCCA at that time, the Guidelines expressly provides: respectively, as National Artists are declared INVALID and SET ASIDE for
6.5 NCCA and CCP Board members and consultants and NCCA and CCP having been issued with grave abuse of discretion.
officers and staff are automatically disqualified from being nominated.85 SO ORDERED.
Respondent Guidote-Alvarez could not have even been nominated, hence, she Petition granted in part, Proclamation Nos. 1826 to 1829 dated July 6, 2009
was not qualified to be considered and conferred the Order of National Artists at proclaiming respondents Cecile Guidote-Alvarez, Carlo Magno Jose Caparas,
that time. The President’s discretion on the matter does not extend to removing a Francisco Mañosa and Jose Moreno as National Artists declared invalid and set
legal impediment or overriding a legal restriction. aside.
From the foregoing, the advice or recommendation of the NCCA and the CCP
Boards as to the conferment of the Order of National Artists on Conde, Dr. Santos, Note.—Mere speculation, unsupported by convincing evidence, cannot establish
Francisco and Alcuaz was not binding on the former President but only discrimination on the part of the prosecution and the denial to the accused of the equal
discretionary or optional for her whether or not to act on such advice or protection of the laws. (People vs. Dumloa. 580 SCRA 409 [2009])
recommendation. Also, by virtue of the power of control, the President had the Quite often, the petitioner in a public action sues as a citizen or taxpayer to gain Lucos
authority to alter or modify or nullify or set aside such recommendation or advice. standi, which is not suprising, for even if the issue may appear to concern only the public in
It was well within the President’s power and discretion to proclaim all, or some or general, such capacities nonetheless equip the petitioner with adequate interest to sue. (De
even none of the recommendees of the CCP and the NCCA Boards, without having Castro vs. Judicial and Bar Council, 615 SCRA 666 [2010])
to justify his or her action. ――o0o――

Thus, the exclusion of Santos did not constitute grave abuse of discretion on the
part of the former President.
The conferment of the Order of National Artists on respondents Guidote-
Alvarez, Caparas, Mañosa and Moreno was an entirely different matter.
There is grave abuse of discretion when an act is (1) done contrary to the
Constitution, the law or jurisprudence or (2) executed whimsically, capriciously or
arbitrarily, out of malice, ill will or personal bias.86
There was a violation of the equal protection clause of the Constitution 87 when
the former President gave preferential treatment to respondents Guidote-Alvarez,
Caparas, Mañosa and Moreno. The former President’s constitutional duty to
faithfully execute the laws and observe the rules, guidelines and policies of the
NCCA and the CCP as to the selection of the nominees for conferment of the Order
of National Artists proscribed her from having a free and uninhibited hand in the
conferment of the said award. The manifest disregard of the rules, guidelines and
processes of the NCCA and the CCP was an arbitrary act that unduly favored
respondents Guidote-Alvarez, Caparas, Mañosa and Moreno. The conferment of
the Order of National Artists on said respondents was therefore made with grave
abuse of discretion and should be set aside.
While the Court invalidates today the proclamation of respondents Guidote-
Alvarez, Caparas, Mañosa and Moreno as National Artists, such action should not
be taken as a pronouncement on whether they are worthy to be conferred that
honor. Only the President, upon the advise of the NCCA and the CCP Boards, may
determine that. The Court simply declares that, as the former President
committed grave abuse of discretion in issuing Proclamation Nos. 1826 to 1829
dated July 6, 2009, the said proclamations are invalid. However, nothing in this
Decision should be read as a disqualification on the part of respondents Guidote-
Alvarez, Caparas, Mañosa and Moreno to be considered for the honor of National
Artist in the future, subject to compliance with the laws, rules and regulations
governing said award.
190 SUPREME COURT REPORTS ANNOTATED upon final conviction, implying that clemency could be given even before
conviction. Thus, petitioner’s unconditional pardon was granted even as her appeal
Monsanto vs. Factoran, Jr. was pending in the High Court. It is worth mentioning that under the 1987
G.R. No. 78239. February 9, 1989.* Constitution, the former limitation of final conviction was restored. But be that as
SALVACION A. MONSANTO, petitioner, vs. FULGENCIO S. FACTORAN, JR., it may, it is our view that in the present case, it is not material when the pardon
respondent. was bestowed, whether before or after conviction, for the result would still be the
Criminal Law; Pardon, Effects of; Administrative Law; Public same. Having accepted the pardon, petitioner is deemed to have abandoned her
Officers; Pardon does not ipso facto restore a convicted felon to public office appeal and her unreversed conviction by the Sandiganbayan assumed the
necessarily relinquished or forfeited by reason of such conviction.—Pardon granted character of finality.
after conviction frees the individual from all the penalties and legal disabilities
and restores him to all his civil rights. But unless expressly grounded on the PADILLA, J., Separate opinion:
person’s innocence (which is rare), it cannot bring back lost reputation for honesty,
integrity and fair dealing. This must be constantly kept in mind lest we lose track Criminal Law; Pardon, Effects of; Administrative Law; Public Officers; A
of the true character and purpose of the privilege. Thus, notwithstanding the public officer, like herein petitioner, who has been convicted of Estafa Through
expansive and effusive language of the Garland case, we are in full agreement Falsification of Public Documents, though subsequently pardoned, is deemed to
with the commonly-held opinion that pardon does not ipso facto restore a convicted have lost her right to public office, unless such right is expressly restored by the
felon to public office necessarily relinquished or forfeited by reason of the pardon.—An examination of the presidential pardon in question shows that, while
conviction although such pardon undoubtedly restores his eligibility for petitioner was granted “an absolute and unconditional pardon and restored to full
appointment to that office. civil and political rights”, yet, nothing therein expressly provides that the right to
Same; Same; Same; Same; Same; The pardon granted to petitioner resulted hold public office was thereby restored to the petitioner. In view of
in removing her disqualification from holding public employment, but to regain her the expressexclusion by Art. 36, R.P.C., of the right to hold public office,
former post, she must reapply and undergo the usual procedure required for a new notwithstanding a pardon unless the right is expressly restored by the pardon, it is
appointment.—For petitioner Monsanto, this is the bottom line: the absolute my considered opinion that, to the extent that the pardon granted to the petitioner
disqualification or ineligibility from public office forms part of the punishment did not expressly restore the right to hold public office as an effect of such pardon,
prescribed by the Revised Penal Code for estafa thru falsification of public that right must be kept away from the petitioner. It is a recognized principle in
documents. It is clear from the authorities referred to that when her guilt and public law—hopefully to be honored more in its compliance rather than in its
punishment were expunged by her pardon, this particular disability was likewise breach—that a “public office is a public trust.” The restoration of the right to hold
removed. Henceforth, petitioner may apply for reappointment to the office which public office to one who has lost such right by reason of conviction in a criminal
was forfeited by reason of her conviction. And in considering her qualifications and case, but subsequently pardoned, cannot be left to inference, no matter how
suitability for the public post, the facts constituting her offense must be and should intensely arguable, but must be stated in express, explicit, positive and specific
be evaluated and taken into account to determine ultimately whether she can once language. To require this would not be asking too much.
again be entrusted with public funds. Stated differently, the pardon granted to
petitioner has resulted in removing her disqualification from holding public PETITION to review the resolution of the Deputy Executive Secretary.
employment but it cannot go beyond that. To regain her former post as assistant
city treasurer, she must reapply and undergo the usual procedure required for a The facts are stated in the opinion of the Court.
new appointment.
Same; Same; Same; Civil Liability, Extinction Of; The pardon granted to FERNAN, C.J.:
herein petitioner did not extinguish the civil liability arising from the crime she has
been convicted of.—Finally, petitioner has sought exemption from the payment of
The principal question raised in this petition for review is whether or not a public
the civil indemnity imposed upon her by the sentence. The Court cannot oblige her.
officer, who has been granted an absolute pardon by the Chief Executive, is entitled
Civil liability arising from crime is governed by the Revised Penal Code. It subsists
to reinstatement to her former position without need of a new appointment.
notwithstanding service of sentence, or for any reason the sentence is not served
In a decision rendered on March 25, 1983, the Sandiganbayan convicted
by pardon, amnesty or commutation of sentence. Petitioner’s civil liability may
petitioner Salvacion A. Monsanto (then assistant treasurer of Calbayog City) and
only be extinguished by the same causes recognized in the Civil Code, namely:
three other accused, of the complex crime of estafa thru falsification of public
payment, loss of the thing due, remission of the debt, merger of the rights of
documents and sentenced them to imprisonment of four (4) years, two (2) months
creditor and debtor, compensation and novation.
and one (1) day of prision correccional as minimum, to ten (10) years and one (1)
Same; Same; Same; Acceptance of Pardon; Petitioner is deemed to have
day of prision mayor as maximum, and to pay a fine of P3,500. They were further
abandoned her appeal when she accepted the pardon granted to her.—The 1981
ordered to jointly and severally indemnify the government in the sum of P4,892.50
amendments had deleted the earlier rule that clemency could be extended only
representing the balance of the amount defrauded and to pay the costs
proportionately. Petitioner’s basic theory is that the general rules on pardon cannot apply to her
Petitioner Monsanto appealed her conviction to this Court which subsequently case by reason of the fact that she was extended executive clemency while her
affirmed the same. She then filed a motion for reconsideration but while said conviction was still pending appeal in this Court. There having been no final
motion was pending, she was extended on December 17, 1984 by then President judgment of conviction, her employment therefore as assistant city treasurer could
Marcos absolute pardon which she accepted on December 21, 1984. not be said to have been terminated or forfeited. In other words, without that final
By reason of said pardon, petitioner wrote the Calbayog City treasurer judgment of conviction, the accessory penalty of forfeiture of office did not attach
requesting that she be restored to her former post as assistant city treasurer since and the status of her employment remained “suspended.” More importantly, when
the same was still vacant. pardon was issued before the final verdict of guilt, it was an acquittal because there
Petitioner’s letter-request was referred to the Ministry of Finance for was no offense to speak of. In effect, the President has declared her not guilty of
resolution in view of the provision of the Local Government Code transferring the the crime charged and has accordingly dismissed the same.4
power of appointment of treasurers from the city governments to the said Ministry. It is well to remember that petitioner had been convicted of the complex crime
In its 4th Indorsement dated March 1, 1985, the Finance Ministry ruled that of estafa thru falsification of public documents and sentenced to imprisonment of
petitioner may be reinstated to her position without the necessity of a new four years, two months and one day of prision correccional as minimum, to ten
appointment not earlier than the date she was extended the absolute pardon. It years and one day of prision mayor as maximum. The penalty of prision
also directed the city treasurer to see to it that the amount of P4,892.50 which the mayor carries the accessory penalties of temporary absolute disqualification and
Sandiganbayan had required to be indemnified in favor of the government as well perpetual special disqualification from the right of suffrage, enforceable during the
as the costs of the litigation, be satisfied.1 term of the principal penalty.5 Temporary absolute disqualification bars the
Seeking reconsideration of the foregoing ruling, petitioner wrote the Ministry convict from public office or employment, such disqualification to last during the
on April 17, 1985 stressing that the full pardon bestowed on her has wiped out the term of the sentence.6 Even if the offender be pardoned, as to the principal penalty,
crime which implies that her service in the government has never been interrupted the accessory penalties remain unless the same have been expressly remitted by
and therefore the date of her reinstatement should correspond to the date of her the pardon.7 The penalty of prision correccional carries, as one of its accessory
preventive suspension which is August 1, 1982; that she is entitled to backpay for penalties, suspension from public office.8
the entire period of her suspension; and that she should not be required to pay the The propositions earlier advanced by petitioner reveal her inadequate
proportionate share of the amount of P4,892.50.2 understanding of the nature of pardon and its legal consequences. This is not
The Ministry of Finance, however, referred petitioner’s letter to the Office of totally unexpected considering that the authorities on the subject have not been
the President for further review and action. On April 15, 1986, said Office, through wholly consistent particularly in describing the effects of pardon.
Deputy Executive Secretary Fulgenio S. Factoran, Jr. held:
“We disagree with both the Ministry of Finance and the petitioner because, as The benign mercy of pardon is of British origin, conceived to temper the gravity of
borne out by the records, petitioner was convicted of the crime for which she was the King’s wrath. But Philippine jurisprudence on the subject has been largely
accused. In line with the government’s crusade to restore absolute honesty in influenced by American case law.
public service, this Office adopts, as a juridical guide (Miranda v. Imperial, 77 Pardon is defined as “an act of grace, proceeding from the power entrusted with
Phil. 1966), the Resolution of the Sandiganbayan, 2nd Division, in People v. the execution of the laws, which exempts the individual, on whom it is bestowed,
Lising, Crim. Case No. 6675, October 4, 1985, that acquittal, not absolute pardon, from the punishment the law inflicts for a crime he has committed. It is the private,
of a former public officer is the only ground for reinstatement to his former position though official act of the executive magistrate, delivered to the individual for whose
and entitlement to payment of his salaries, benefits and emoluments due to him benefit it is intended, and not communicated officially to the Court. x x x. A pardon
during the period of his suspension pendente lite. is a deed, to the validity of which delivery is essential, and delivery is not complete
“In fact, in such a situation, the former public official must secure a without acceptance.”8-a
reappointment before he can reassume his former position. xxx. At the time the antecedents of the present case took place, the pardoning power
“Anent the civil liability of Monsanto, the Revised Penal Code expressly was governed by the 1973 Constitution as amended in the April 7, 1981 plebiscite.
provides that ‘a pardon shall in no case exempt the culprit from payment of the The pertinent provision reads:
civil indemnity imposed upon him by the sentence.’ (Sec. 36, par. 2). “The President may, except in cases of impeachment, grant reprieves,
“IN VIEW OF THE FOREGOING, this Office holds that Salvacion A. commutations and pardons, remit fines and forfeitures, and with the concurrence
Monsanto is not entitled to an automatic reinstatement on the basis of the absolute of the Batasang Pambansa, grant amnesty.”9
pardon granted her but must secure an appointment to her former position and The 1981 amendments had deleted the earlier rule that clemency could be
that, notwithstanding said absolute pardon, she is liable for the civil liability extended only upon final conviction, implying that clemency could be given even
concomitant to her previous conviction.”3 before conviction. Thus, petitioner’s unconditional pardon was granted even as her
Her subsequent motion for reconsideration having been denied, petitioner filed the appeal was pending in the High Court. It is worth mentioning that under the 1987
present petition in her behalf. We gave due course on October 13, 1987. Constitution, the former limitation of final conviction was restored. But be that as
it may, it is our view that in the present case, it is not material when the pardon of guilt.17 But it relieves him from nothing more. “To say, however, that the
was bestowed, whether before or after conviction, for the result would still be the offender is a ‘new man’, and ‘as innocent as if he had never committed the offense;’
same. Having accepted the pardon, petitioner is deemed to have abandoned her is to ignore the difference between the crime and the criminal. A person adjudged
appeal and her unreversed conviction by the Sandiganbayan assumed the guilty of an offense is a convicted criminal, though pardoned; he may be deserving
character of finality. of punishment, though left unpunished; and the law may regard him as more
Having disposed of that preliminary point, we proceed to discuss the effects of dangerous to society than one never found guilty of crime, though it places no
a full and absolute pardon in relation to the decisive question of whether or not the restraints upon him following his conviction.”18
plenary pardon had the effect of removing the disqualifications prescribed by the
Revised Penal Code. A pardon looks to the future. It is not retrospective.19 It makes no amends for the
In Pelobello v. Palatino,10 we find a reiteration of the stand consistently past. It affords no relief for what has been suffered by the offender. It does not
adopted by the courts on the various consequences of pardon: “x x x we adopt the impose upon the government any obligation to make reparation for what has been
broad view expressed in Cristobal v. Labrador, G.R. No. 47941, December 7, 1940, suffered. “Since the offense has been established by judicial proceedings, that
that subject to the limitations imposed by the Constitution, the pardoning power which has been done or suffered while they were in force is presumed to have been
cannot be restricted or controlled by legislative action; that an absolute pardon not rightfully done and justly suffered, and no satisfaction for it can be
only blots out the crime committed but removes all disabilities resulting from the required.”20 This would explain why petitioner, though pardoned, cannot be
conviction. x x x. (W)e are of the opinion that the better view in the light of the entitled to receive backpay for lost earnings and benefits.
constitutional grant in this jurisdiction is not to unnecessarily restrict or impair Petitioner maintains that when she was issued absolute pardon, the Chief
the power of the Chief Executive who, after an inquiry into the environmental Executive declared her not guilty of the crime for which she was convicted. In the
facts, should be at liberty to atone the rigidity of the law to the extent of relieving case of State v. Hazzard,21 we find this strong observation: “To assume that all or
completely the party x x x concerned from the accessory and resultant disabilities even a major number of pardons are issued because of innocence of the recipients
of criminal conviction.” is not only to indict our judicial system, but requires us to assume that which we
The Pelobello v. Palatino and Cristobal v. Labrador cases,11 and several all know to be untrue. The very act of forgiveness implies the commission of wrong,
others12 show the unmistakable application of the doctrinal case of Ex Parte and that wrong has been established by the most complete method known to
Garland,13 whose sweeping generalizations to this day continue to hold sway in modern civilization. Pardons may relieve from the disability of fines and forfeitures
our jurisprudence despite the fact that much of its relevance has been downplayed attendant upon a conviction, but they cannot erase the stain of bad character,
by later American decisions. which has been definitely fixed.”22
Consider the following broad statements: In this ponencia, the Court wishes to stress one vital point: While we are
“A pardon reaches both the punishment prescribed for the offense and the guilt of prepared to concede that pardon may remit all the penal consequences of a
the offender; and when the pardon is full, it releases the punishment and blots out criminal indictment if only to give meaning to the fiat that a pardon, being a
of existence the guilt, so that in the eye of the law the offender is as innocent as if presidential prerogative, should not be circumscribed by legislative action, we do
he had never committed the offense. If granted before conviction, it prevents any not subscribe to the fictitious belief that pardon blots out the guilt of an individual
of the penalties and disabilities, consequent upon conviction, from attaching; if and that once he is absolved, he should be treated as if he were innocent. For
granted after conviction, it removes the penalties and disabilities and restores him whatever may have been the judicial dicta in the past, we cannot perceive how
to all his civil rights; it makes him, as it were, a new man, and gives him a new pardon can produce such “moral changes” as to equate a pardoned convict in
credit and capacity.”14 character and conduct with one who has constantly maintained the mark of a good,
Such generalities have not been universally accepted, recognized or law-abiding citizen.
approved.15 The modern trend of authorities now rejects the unduly broad Pardon cannot mask the acts constituting the crime. These are “historical”
language of the Garland case (reputed to be perhaps the most extreme statement facts which, despite the public manifestation of mercy and forgiveness implicit in
which has been made on the effects of a pardon). To our mind, this is the more pardon, “ordinary, prudent men will take into account in their subsequent dealings
realistic approach. While a pardon has generally been regarded as blotting out the with the actor.”23
existence of guilt so that in the eye of the law the offender is as innocent as though Pardon granted after conviction frees the individual from all the penalties and
he never committed the offense, it does not operate for all purposes. The very legal disabilities and restores him to all his civil rights. But unless expressly
essence of a pardon is forgiveness or remission of guilt. Pardon implies guilt. It grounded on the person’s innocence (which is rare), it cannot bring back lost
does not erase the fact of the commission of the crime and the conviction thereof. reputation for honesty, integrity and fair dealing.24 This must be constantly kept
It does not wash out the moral stain. It involves forgiveness and not in mind lest we lose track of the true character and purpose of the privilege.
forgetfulness.16 Thus, notwithstanding the expansive and effusive language of
The better considered cases regard full pardon (at least one not based on the the Garland case, we are in full agreement with the commonly-held opinion that
offender’s innocence) as relieving the party from all the punitive consequences of pardon does not ipso facto restore a convicted felon to public office necessarily
his criminal act, including the disqualifications or disabilities based on the finding
relinquished or forfeited by reason of the conviction 25 although such pardon SEPARATE OPINION
undoubtedly restores his eligibility for appointment to that office.26
The rationale is plainly evident. Public offices are intended primarily for the
PADILLA, J.:
collective protection, safety and benefit of the common good. They cannot be
compromised to favor private interests. To insist on automatic reinstatement
I concur in the result but on grounds different from those relied upon by the
because of a mistaken notion that the pardon virtually acquitted one from the
majority opinion.
offense of estafa would be grossly untenable. A pardon, albeit full and plenary,
Petitioner Salvacion A. Monsanto was Assistant Treasurer of Calbayog City.
cannot preclude the appointing power from refusing appointment to anyone
Together with three (3) other accused, she was charged before the Sandiganbayan
deemed to be of bad character, a poor moral risk, or who is unsuitable by reason of
with the complex crime of Estafa through falsification of public documents. After
the pardoned conviction.
trial, the accused were convicted and sentenced to imprisonment of four (4) years,
For petitioner Monsanto, this is the bottom line: the absolute disqualification
two (2) months and one (1) day of prision correccional, as minimum, to ten (10)
or ineligibility from public office forms part of the punishment prescribed by the
years and one (1) day of prision correccional, as maximum, and to pay a fine of
Revised Penal Code for estafa thru falsification of public documents. It is clear
P3,500.00. They were also ordered to jointly and severally indemnify the
from the authorities referred to that when her guilt and punishment were
government in the sum of P4,892.50 representing the balance of the amount
expunged by her pardon, this particular disability was likewise removed.
defrauded and to pay the costs proportionately.
Henceforth, petitioner may apply for reappointment to the office which was
Petitioner appealed the judgment of conviction to this Court which affirmed
forfeited by reason of her conviction. And in considering her qualifications and
the same. Petitioner then filed a motion for reconsideration but while said motion
suitability for the public post, the facts constituting her offense must be and should
was pending, President Ferdinand E. Marcos extended to her on 17 December 1984
be evaluated and taken into account to determine ultimately whether she can once
an absolute pardon which she accepted on 21 December 1984.
again be entrusted with public funds. Stated differently, the pardon granted to
By reason of said absolute pardon, petitioner in representations before the City
petitioner has resulted in removing her disqualification from holding public
Treasurer of Calbayog, the Ministry of Finance and the Office of the President,
employment but it cannot go beyond that. To regain her former post as assistant
asked that she be allowed to re-assume her former office, as of 1 August 1982 (the
city treasurer, she must reapply and undergo the usual procedure required for a
date of her preventive suspension), that she be paid her back salaries for the entire
new appointment.
period of her suspension, and that she be not required to pay her proportionate
Finally, petitioner has sought exemption from the payment of the civil
share of the amount of P4,892.50.
indemnity imposed upon her by the sentence. The Court cannot oblige her. Civil
Respondent Assistant Executive Secretary denied petitioner’s request for
liability arising from crime is governed by the Revised Penal Code. It subsists
automatic reinstatement as well as her other claims, because of which denial, this
notwithstanding service of sentence, or for any reason the sentence is not served
petition for review on certiorari
by pardon, amnesty or commutation of sentence. Petitioner’s civil liability may
203
only be extinguished by the same causes recognized in the Civil Code, namely:
payment, loss of the thing due, remission of the debt, merger of the rights of VOL. 170, FEBRUARY 9, 1989 203
creditor and debtor, compensation and novation.27 Monsanto vs. Factoran, Jr.
WHEREFORE, the assailed resolution of former Deputy Executive Secretary was filed before the Court seeking the setting aside and reversal of the decision of
Fulgencio S. Factoran, Jr., dated April 15, 1986, is AFFIRMED. No costs. the respondent Assistant Executive Secretary, on the main contention that, as a
So ordered. public officer who has been granted an absolute pardon by the President, she is
Narvasa, Paras, Gancayco, Bidin, Cortés, Griño- entitled to reinstatement to her former position without need of a new
Aquino,Medialdea and Regalado, JJ., concur. appointment, and to the other reliefs prayed for.
Melencio-Herrera, J., In the result and in the Separate Opinions of There can be no dispute that the pardon extinguished petitioner’s criminal
Justices Padilla and Feliciano. liability. At the same time, Art. 36 of the Revised Penal Code categorically covers
Gutierrez, Jr., J., I join Justice Feliciano in his concurring opinion. the effects of a pardon on the pardoned’s right to hold office, suffrage and on his
civil liability. It states:
Cruz, J., I concur subject to Mr. Justice Feliciano’s reservation in his “ART. 36. Pardon; its effects.—A pardon shall not work the restoration of the right
separate opinion. to hold public office, or the right of suffrage, unless such rights be expressly restored
Feliciano, J., Please see separate concurring opinion. by the terms of the pardon.
Padilla, J., Please see separate opinion. “A pardon shall in no case exempt the culprit from the payment of the civil
Sarmiento, J., I join the separate opinion of Justices Padilla and indemnity imposed upon him by the sentence.” (Italics supplied)
Feliciano. Applying Art. 36 of the Revised Penal Code to the case at bar, it is, to my mind,
clear that the pardon extended by the President to the petitioner did not per
se entitle her to again hold public office (including therefore the office of Assistant
Treasurer, Calbayog City) or to suffrage; nor did such pardon extinguish her civil “Article 40. Death—Its accessory penalties.—The death penalty, when it is not
liability for the criminal conviction, subject matter of the pardon. executed by reason of commutation or pardon shall carry with it that of perpetual
An examination of the presidential pardon in question shows that, while absolute disqualification and
petitioner was granted “an absolute and unconditional pardon and restored to full 205
civil and political rights”, yet, nothing therein expressly provides that the right to VOL. 170, FEBRUARY 9, 1989 205
hold public office was thereby restored to the petitioner. In view of
the express exclusion by Art. 36, R.P.C., of the right to hold public office, Monsanto vs. Factoran, Jr.
notwithstanding a pardon unless the right is expressly restored by the pardon, it is that of civil interdiction during thirty years following the date of sentence, unless
my considered opinion that, to the extent that the pardon granted to the petitioner such accessory penalties have been expressly remitted in the pardon.”
did not expressly restore the right to hold public office as an effect of such pardon, “Article 41. Reclusion perpetua and reclusion temporal.—Their accessory
that right must be kept away from the petitioner. penalties.—The penalties of reclusion perpetua and reclusion temporal shall carry
It is a recognized principle in public law—hopefully to be honored more in its with them that of civil interdiction for life or during the period of the sentence as
compliance rather than in its breach—that a “public office is a public trust.” The the case may be, and that of perpetual absolute disqualification which the offender
restoration of the right to shall suffer even though pardoned as to the principal penalty, unless the same shall
204 have been expressly remitted in the pardon.”
“Article 42. Prision mayor—Its accessory penalties.—The penalty of prision
204 SUPREME COURT REPORTS ANNOTATED
mayor shall carry with it that of temporary absolute disqualification and that of
Monsanto vs. Factoran, Jr. perpetual special disqualification from the right of suffrage which the offender
hold public office to one who has lost such right by reason of conviction in a criminal shall suffer although pardoned as to the principal penalty, unless the same shall
case, but subsequently pardoned, cannot be left to inference, no matter how have been expressly remitted in the pardon.”
intensely arguable, but must be stated in express, explicit, positive and specific “Article 43. Prision correccional—Its accessory penalties.—The penalty
language. To require this would not be asking too much. of prision correccional shall carry with it that of suspension from public office, from
I am aware that there are broad statement in Cristobal vs. Labrador, 71 Phil. the right to follow a profession or calling, and that of perpetual special
341 and Pelobello vs. Palatino, 72 Phil. 441which may be understood to mean that disqualification from the right of suffrage, if the duration of said imprisonment
an absolute pardon, without qualification, restores full civil rights which have been shall exceed eighteen months. The offender shall suffer the disqualification
construed, in turn, to include the right to hold public office (Versoza vs. provided in this article although pardoned as to the principal penalty, unless the
Fernandez, 55 Phil. 323). same shall have been expressly remitted in the pardon.” (Italics supplied)
If such be the message of said cases, then I submit that a modification is in The Chief Justice points out that the penalty imposed upon petitioner for the
order, so that an absolute pardon to work a restoration of the right to hold public complex crime of estafa through falsification of public documents, included the
office must expressly so state, in order to give substance and meaning to the sound accessory penalties of temporary absolute disqualification from public office or
provisions of Article 36 of the Revised Penal Code, particularly in the light of our employment and perpetual special disqualification from the right of suffrage. The
times and experience. 17 December 1984 pardon extended to petitioner in the instant case was written
ACCORDINGLY, I vote to DENY the petition. on a standard printed form which states in printed words that it was “an absolute
and unconditional pardon [which] restored [petitioner] to full civil and political
FELICIANO, J.: Concurring: rights.”1 While the right of suffrage and the right to hold public office or
employment are commonly regarded as “political rights,”2 it must be noted that
I concur in the result reached in the important and eloquent opinion of the Chief there are other “political rights”3 and that the pardon given to petitioner did not
Justice. I also join in the separate concurring opinion of Mr. Justice Padilla. At the
same time, I would add a few brief statements, basically for my own clarification. _______________
Article 36 of the Revised Penal Code states:
“Article 36. Pardon; its effects.—A pardon shall not work the restoration of the Rollo, p. 90.
1

right to hold public office, or the right of suffrage, unless such rights be expressly Vera v. Avelino, 77 Phil. 192 (1946).
2

restored by the terms of the pardon. 3 Malcolm and Laurel, The Constitutional Law of the Philippines, p. 378

A pardon shall in no case exempt the culprit from the payment of the civil (1936).
indemnity imposed upon him by the sentence.” (Italic supplied) 206
It is worthy of note that the rule embodied in Article 36 is reiterated four (4) times 206 SUPREME COURT REPORTS ANNOTATED
by the Revised Penal Code in its following provisions:
Monsanto vs. Factoran, Jr.
expressly and in printer’s ink restore to petitioner the particular right to hold
public office and the specific right to vote at elections and plebiscites.
I join in the basic point of Mr. Justice Padilla that because of the nature of a G.R. No. 206666. January 21, 2015.*
public office as a public trust, Articles 36 and 40-43 appropriately require a very
high degree of explicitness if a pardon is to work the restoration of such right to ATTY. ALICIA RISOS-VIDAL, petitioner,
petitioner. Exactly the same point may, of course, be made in respect of the ALFREDO S. LIM, petitioner-intervenor, vs. COMMISSION ON ELECTIONS
restoration of the right to vote. and JOSEPH EJERCITO ESTRADA, respondents.
Articles 36 and 40-43 of the Revised Penal Code, quoted above, have been in Constitutional Law; Criminal Law; Pardon; Former President Estrada was
our statute books since 1930. I believe that they have been left intact by the granted an absolute pardon that fully restored all his civil and political rights,
constitutional provisions on pardon, whether one refers to the 1935 Constitution which naturally includes the right to seek public elective office, the focal point of
or to the 1973 and 1987 Constitutions. I do not believe that Articles 36, et this controversy.—Former President Estrada was granted an absolute pardon
al. collided with any provision or principle embodied in either of our prior that fully restored all his civil and political rights, which naturally includes the
constitutions. The Chief Justice appears to agree with this position when he right to seek public elective office, the focal point of this controversy. The wording
referred to Article 36 of the Revised Penal Code (Opinion, p. 5). He goes on, of the pardon extended to former President Estrada is complete, unambiguous, and
however, to say (in page 13) that: “the pardon granted to petitioner has resulted in unqualified. It is likewise unfettered by Articles 36 and 41 of the Revised Penal
removing her disqualification from holding public employment but it cannot go Code. The only reasonable, objective, and constitutional interpretation of the
beyond that.” language of the pardon is that the same in fact conforms to Articles 36 and 41 of
It is submitted, with respect, that Articles 36, et al. of the Revised Penal Code the Revised Penal Code.
have not been shown to be an unconstitutional restriction on the pardoning power Same; Same; Same; The 1987 Constitution, specifically Section 19 of Article
of the President. The limitation on the President’s pardoning power, if limitation VII and Section 5 of Article IX-C, provides that the President of the Philippines
it be, does not appear to be an unreasonably onerous one. Articles 36, et al. merely possesses the power to grant pardons, along with other acts of executive clemency;
require the President to become completely explicit if the pardon he extends is The only instances in which the President may not extend pardon remain to be in:
intended to wipe out not merely the principal but also the accessory penalty of (1) impeachment cases; (2) cases that have not yet resulted in a final conviction; and
disqualification from holding public office and from voting and to restore the (3) cases involving violations of election laws, rules and regulations in which there
recipient of the pardon to the exercise of such fundamental political rights. Such was no favorable recommendation coming from the Commission on Elections
requirement of explicitness seems entirely in line with the fundamental point (COMELEC).—The 1987 Constitution, specifically Section 19 of Article VII and
made by the Chief Justice that a pardon does not blot out the factual guilt of the Section 5 of Article IX-C, provides that the President of the Philippines possesses
recipient of the pardon. In other words, the mere grant of a pardon to a public the power to grant pardons, along with other acts of executive clemency, to wit:
officer or employee who has been unfaithful to the public trust and sentenced to Section 19. Except in cases of impeachment, or as otherwise provided in this
disqualification from voting and from holding such office, does not create the Constitution, the President may grant reprieves, commutations, and pardons, and
presumption that the recipient of the pardon has thereby suddenly become morally remit fines and forfeitures, after
eligible once more to exercise the right to vote _______________
207
VOL. 170, FEBRUARY 9, 1989 207 * EN BANC.
211
Phil. National Construction Corp. vs. NLRC
and to hold public office. VOL. 747, JANUARY 21, 2015 211
In my view, the pardon extended to petitioner was ineffective to restore to her Risos-Vidal vs. Commission on Elections
the right to hold public office and on this ground, I vote to DENY the Petition for conviction by final judgment. He shall also have the power to grant amnesty
Review and to AFFIRM the assailed Resolution of the then Executive Secretary with the concurrence of a majority of all the Members of the Congress. x x x x
Fulgencio S. Factoran, Jr. Section 5. No pardon, amnesty, parole, or suspension of sentence for violation of
Resolution affirmed. election laws, rules, and regulations shall be granted by the President without the
Note.—Acquittal in a criminal case does not operate to dismiss a separate civil favorable recommendation of the Commission. It is apparent from the foregoing
action filed on the basis of the same facts alleged in the criminal case. (Salta vs. constitutional provisions that the only instances in which the President may not
De Veyra, 117 SCRA 212). extend pardon remain to be in: (1) impeachment cases; (2) cases that have not yet
resulted in a final conviction; and (3) cases involving violations of election laws,
——o0o—— rules and regulations in which there was no favorable recommendation coming
from the COMELEC. Therefore, it can be argued that any act of Congress by way
of statute cannot operate to delimit the pardoning power of the President.
Same; Same; Same; This doctrine of non-diminution or non-impairment of
the President’s power of pardon by acts of Congress, specifically through legislation,
was strongly adhered to by an overwhelming majority of the framers of the 1987 to grant executive clemency and, specifically, to decide to pardon the principal
Constitution when they flatly rejected a proposal to carve out an exception from the penalty while excluding its accessory penalties or to pardon both. Thus, Articles
pardoning power of the President in the form of “offenses involving graft and 36 and 41 only clarify the effect of the pardon so decided upon by the President on
corruption” that would be enumerated and defined by Congress through the the penalties imposed in accordance with law.
enactment of a law.—This doctrine of non-diminution or non-impairment of the Same; Criminal Law; Pardon; The pardon granted to former President
President’s power of pardon by acts of Congress, specifically through legislation, Estrada admits no other interpretation other than to mean that, upon acceptance
was strongly adhered to by an overwhelming majority of the framers of the 1987 of the pardon granted to him, he regained his FULL civil and political rights —
Constitution when they flatly rejected a proposal to carve out an exception from including the right to seek elective office.—From both law and jurisprudence, the
the pardoning power of the President in the form of “offenses involving graft and right to seek public
corruption” that would be enumerated and defined by Congress through the 213
enactment of a law. VOL. 747, JANUARY 21, 2015 213
Statutory Construction; It is well-entrenched in this jurisdiction that where
the words of a statute are clear, plain, and free from ambiguity, it must be given its Risos-Vidal vs. Commission on Elections
literal meaning and applied without attempted interpretation.—It is well- elective office is unequivocally considered as a political right. Hence, the
entrenched in this jurisdiction that where the words of a statute are clear, plain, Court reiterates its earlier statement that the pardon granted to former President
and free from ambiguity, it must be given its literal meaning and applied without Estrada admits no other interpretation other than to mean that, upon acceptance
attempted interpretation. Verba legis non est recedendum. From the words of a of the pardon granted to him, he regained his FULL civil and political rights —
statute there should be no departure. It is this Court’s firm view that the phrase including the right to seek elective office.
in the presidential pardon at issue which declares that former President Estrada Same; Preamble; Jurisprudence educates that a preamble is not an essential
“is hereby restored to his civil and political rights” substantially complies with the part of an act as it is an introductory or preparatory clause that explains the reasons
requirement of express restoration. for the enactment, usually introduced by the word “whereas.”—Jurisprudence
212 educates that a preamble is not an essential part of an act as it is an introductory
or preparatory clause that explains the reasons for the enactment, usually
212 SUPREME COURT REPORTS ANNOTATED
introduced by the word “whereas.” Whereas clauses do not form part of a statute
Risos-Vidal vs. Commission on Elections because, strictly speaking, they are not part of the operative language of the
Constitutional Law; Separation of Powers; The form or manner by which the statute. In this case, the whereas clause at issue is not an integral part of the
President, or Congress for that matter, should exercise their respective decree of the pardon, and therefore, does not by itself alone operate to make the
Constitutional powers or prerogatives cannot be interfered with unless it is so pardon conditional or to make its effectivity contingent upon the fulfilment of the
provided in the Constitution.—With due respect, I disagree with the overbroad aforementioned commitment nor to limit the scope of the pardon.
statement that Congress may dictate as to how the President may exercise his/her Same; Criminal Law; Pardon; The pardon granted to former President
power of executive clemency. The form or manner by which the President, or Estrada was absolute, meaning, it was not only unconditional, it was unrestricted
Congress for that matter, should exercise their respective Constitutional powers in scope, complete and plenary in character, as the term “political rights” adverted
or prerogatives cannot be interfered with unless it is so provided in the to has a settled meaning in law and jurisprudence.—The statement “[h]e is hereby
Constitution. This is the essence of the principle of separation of powers deeply restored to his civil and political rights,” to the mind of the Court, is crystal clear
ingrained in our system of government which “ordains that each of the three great — the pardon granted to former President Estrada was absolute, meaning, it was
branches of government has exclusive cognizance of and is supreme in matters not only unconditional, it was unrestricted in scope, complete and plenary in
falling within its own constitutionally allocated sphere.” More so, this fundamental character, as the term “political rights” adverted to has a settled meaning in law
principle must be observed if noncompliance with the form imposed by one branch and jurisprudence.
on a coequal and coordinate branch will result into the diminution of an exclusive BRION, J., Separate Opinion:
Constitutional prerogative. For this reason, Articles 36 and 41 of the Revised Penal Remedial Law; Special Civil Actions; Certiorari; View that Section 7, Article
Code should be construed in a way that will give full effect to the executive IX of the Constitution provides that “unless otherwise provided by this Constitution
clemency granted by the President, instead of indulging in an overly strict or by law, any decision, order or ruling of each Commission may be brought to the
interpretation that may serve to impair or diminish the import of the pardon which Supreme Court (SC) on certiorari by the aggrieved party.”—Section 7, Article IX of
emanated from the Office of the President and duly signed by the Chief Executive the Constitution provides that “unless otherwise provided by this Constitution or
himself/herself. The said codal provisions must be construed to harmonize the by law, any decision, order or ruling of each Commission may
power of Congress to define crimes and prescribe the penalties for such crimes and 214
the power of the President to grant executive clemency. All that the said provisions 214 SUPREME COURT REPORTS ANNOTATED
impart is that the pardon of the principal penalty does not carry with it the
remission of the accessory penalties unless the President expressly includes said Risos-Vidal vs. Commission on Elections
accessory penalties in the pardon. It still recognizes the Presidential prerogative
be brought to the Supreme Court on certiorari by the aggrieved party.” A proceedings, becomes a litigant in the case so that the intervenor could protect or
similar provision was found in the 1973 Constitution. In Aratuc v. COMELEC, 88 preserve a right or interest that may be affected by the proceedings.—Intervention
SCRA 251 (a 1979 case), the Court clarified that unlike in the 1935 Constitution is a remedy whereby a third party, not originally impleaded in the proceedings,
where the Court had the power of review over the decisions, orders and rulings of becomes a litigant in the case so that the intervenor could protect or preserve a
the COMELEC, the 1973 Constitution changed the nature of this remedy right or interest that may be affected by the proceedings. The intervenor’s interest
from appellate review to certiorari. Aratuc explained that under the then must be actual, substantial, material, direct and immediate, and not simply
existing Constitution and statutory provisions, the certiorari jurisdiction of the contingent or expectant. It must be of such direct and immediate character that
Court over orders, and decisions of the COMELEC was not as broad as it used to the intervenor will either gain or lose by the direct legal operation and effect of the
be and should be confined to instances of grave abuse of discretion amounting to judgment.
patent and substantial denial of due process. The Court further observed that Same; Same; Same; View that Section 2, Rule 19 of the Rules of Court
these constitutional, statutory and jurisprudential changes show the provides that the time to intervene is at any time before the rendition of judgment
definite intent to enhance and invigorate the role of the COMELEC as the by the trial court.—Since Lim intervened only in the present petition
independent constitutional body tasked to safeguard free, peaceful and for certiorari before this Court, the Rules of Court on intervention directly applies.
honest elections. In other words, the limited reach and scope of certiorari, Section 2, Rule 19 of the Rules of Court provides that the time to intervene is at
compared with appellate review, direct that utmost respect be given the any time before the rendition of judgment by the trial court. The Court explained
COMELEC as the constitutional body given the charge of elections. in Ongco v. Dalisay, 677 SCRA 232 (2012), that “the period within which a person
Same; Same; Same; Grave Abuse of Discretion; View that the grave abuse of may intervene is restricted and after the lapse of the period set in Section 2, Rule
discretion that justifies the grant of certiorari involves a defect of jurisdiction 19, intervention will no longer be warranted. This is because, basically, intervention
brought about, among others, by an indifferent disregard for the law, arbitrariness is not an independent action but is ancillary and supplemental to an existing
and caprice, an omission to weigh pertinent considerations, or a decision arrived at litigation.”
without rational deliberation — due process issues that rendered the decision or 216
ruling void.—The grave abuse of discretion that justifies the grant 216 SUPREME COURT REPORTS ANNOTATED
of certiorari involves a defect of jurisdiction brought about, among others, by an
indifferent disregard for the law, arbitrariness and caprice, an omission to weigh Risos-Vidal vs. Commission on Elections
pertinent considerations, or a decision arrived at without rational deliberation Same; Special Civil Actions; Certiorari; Intervention; Parties; View that as a
— due process issues that rendered the decision or ruling void. Our 1987 nonparty to the disqualification case before the Commission on Elections
Constitution maintained the same remedy of certiorari in the review of COMELEC (COMELEC), Lim cannot be deemed an “aggrieved party” who has earned the
decisions elevated to the Supreme Court as the Constitutional Convention rights under Rule 65 to file a certiorari petition or to intervene to assail the
deliberations show. This constitutional provision has since then been reflected COMELEC’s decision.—From the perspective of Rule 65 of the Rules of Court,
under Rules 64 and 65 of the Rules of Court. I add that because Lim was not a party before the COMELEC, he never had the
Constitutional Law; Criminal Law; Pardon; View that when the recipient of chance to file a motion for reconsideration before that body — a constitutional
pardon is likewise the people’s choice in an election held after the pardon, it is well and procedural requirement before a petition for certiorari may be filed
to remember that pardon is an act of clemency and grace exercised to mitigate the before the Court. As a nonparty to the disqualification case before the
harshness of the applica- COMELEC, he cannot be deemed an “aggrieved party” who has earned the rights
215 under Rule 65 to file a certiorari petition or to intervene to assail the COMELEC’s
decision. The Court, in particular, has no jurisdiction to grant the prayer
VOL. 747, JANUARY 21, 2015 215
of Lim to be declared as the winner, especially since the COMELEC never
Risos-Vidal vs. Commission on Elections had the chance to rule on this in its assailed decision.
tion of the law and should be understood in this spirit, i.e., in favor of the Election Disputes; Jurisdiction; View that the original jurisdiction to decide
grantee whom the people themselves have adjudged and found acceptable.—When election disputes lies with the Commission on Elections (COMELEC), not with the
the recipient of pardon is likewise the people’s choice in an election held after the Supreme Court (SC).—The original jurisdiction to decide election disputes lies with
pardon, it is well to remember that pardon is an act of clemency and grace the COMELEC, not with this Court. Thus, any ruling from us in the first instance
exercised to mitigate the harshness of the application of the law and should be on who should sit as mayor (in the event we grant the Risos-Vidal petition) will
understood in this spirit, i.e., in favor of the grantee whom the people themselves constitute grave abuse of discretion. Unfortunately, no recourse is available
have adjudged and found acceptable. It ought not be forgotten that in two high from our ruling. This character of finality renders it very important for us to
profile elections, the State had allowed Erap to offer himself as a candidate without settle the Lim intervention correctly.
any legal bar and without notice to the voting public that a vote for him could be Constitutional Law; Criminal Law; Pardon; Words and Phrases; View that
rendered useless and stray. pardon is defined as an act of grace, proceeding from the power entrusted with the
Remedial Law; Civil Procedure; Intervention; Words and Phrases; View that execution of the laws, which exempts the individual, on whom it is bestowed, from
intervention is a remedy whereby a third party, not originally impleaded in the the punishment that the law inflicts for a crime he has committed.—Section 19,
Article VII of the Constitution provides for the pardoning power of the President. punishment which the law inflicts for the offense he had committed resulting in the
It states that except in cases of impeachment, or as otherwise provided in this partial extinction of his criminal liability.”—Under the BPP’s Revised Rules and
Constitution, the President may grant reprieves, commutations, and pardons, and Regulations, “absolute pardon” refers “to the total extinction of the
remit fines and forfeitures, after conviction by final judgment. Pardon is defined criminal liability of the individual to whom it is granted without any
as an act of grace, proceeding from the power entrusted with the execution of the condition. It restores to the individual his civil and political rights and
217 remits the penalty imposed for the particular offense of which he was
VOL. 747, JANUARY 21, 2015 217 convicted.” Aside from absolute pardon, there is the conditional pardon which
is defined as “the exemption of an individual, within certain limits or conditions,
Risos-Vidal vs. Commission on Elections from the punishment which the law inflicts for the offense he had committed
laws, which exempts the individual, on whom it is bestowed, from the resulting in the partial extinction of his criminal liability.” These are the
punishment that the law inflicts for a crime he has committed. authoritative guidelines in determining the nature and extent of the pardon the
Same; Same; Same; View that the power to pardon, when exercised by the President grants, i.e., whether it is absolute or conditional. To stress, the BPP is
Chief Executive in favor of persons convicted of public crimes, is plenary, limited the body that investigates and recommends to the President whether or not a
only by the terms of the Constitution; its exercise within these limits is otherwise pardon should be granted to a convict, and that closely coordinates with the Office
absolute and fully discretionary.—The power to pardon, when exercised by the of the President on matters of pardons and parole.
Chief Executive in favor of persons convicted of public crimes, is plenary, limited Same; Same; Same; View that President Gloria Macapagal-Arroyo (PGMA)
only by the terms of the Constitution; its exercise within these limits is otherwise clearly intended the granted pardon to be absolute. Thus, the pardon granted totally
absolute and fully discretionary. The reasons for its exercise are not open to extinguished the criminal liability of Erap, including the accessory penalty of
judicial inquiry or review, and indeed it would appear that he may act without any perpetual absolute disqualification.—When PGMA (as President and Head of
reason, or at least without any expressed reason, in support of his action. the Executive Department to which the BPP belongs) granted Erap
Remedial Law; Civil Procedure; Judgments; Execution of Judgments; View executive clemency and used the words of the BPP rules and regulations,
that the order of execution should always follow the terms of the fallo or dispositive she raised the inference that her grant was in the spirit in which the
portion.—In judicial decisions, the Court’s resolution on a given issue before it is terms of the pardon are understood in the BPP rules. In other words, she
always embodied in the decision or order’s fallo or dispositive portion. It is the clearly intended the granted pardon to be absolute. Thus, the pardon granted
directive part of the decision or order which must be enforced or, in legal parlance, totally extinguished the criminal liability of Erap, including the accessory penalty
subjected to execution. A court that issues an order of execution contrary to the of perpetual absolute disqualification. It cannot be otherwise under the plain and
terms of its final judgment exceeds its jurisdiction, thus rendering its order invalid. unequivocal wording of the definition of absolute pardon, and the statement in the
Hence, the order of execution should always follow the terms of the fallo or pardon that Erap is restored to his civil and political rights.
dispositive portion. 219
Constitutional Law; Criminal Law; Pardon; View that a pardon, as an
VOL. 747, JANUARY 21, 2015 219
expression of an executive policy decision that must be enforced, hews closely to the
structure of a court decision.—A pardon, as an expression of an executive policy Risos-Vidal vs. Commission on Elections
decision that must be enforced, hews closely to the structure of a court decision. Same; Same; Same; View that irrespective of the nature of the pardon, the
Their structures run parallel with each other, with the Whereas Clauses briefly moment the convict avails of the clemency granted, with or without written
stating the considerations recognized and, possibly, the intents and purposes acceptance, then the pardon is already accepted.—Simply as an aside (as I feel
considered, in arriving at the directive to pardon and release a convicted prisoner. the topic does not deserve any extended consideration), I do not believe that
Thus, while a pardon’s introductory or Whereas Clauses may be considered in the “acceptance” of the pardon is important in the determination of whether
reading the pardon (in the manner that the opinion portion of a court decision is the pardon extended is absolute or conditional. Irrespective of the nature of the
read), these whereas clauses — as a rule — cannot also significantly affect the pardon, the moment the convict avails of the clemency granted, with or without
pardon’s dispositive portion. They can only do so and in fact may even prevail, but written acceptance, then the pardon is already accepted. If this is to be the
a clear and patent reason indicating a mistake in the grantor’s intent must be standard to determine the classification of the pardon, then there would hardly be
shown, as had happened in Cobarrubias any absolute pardon; upon his release, the pardon is deemed accepted and
218 therefore conditional. If an express acceptance would serve a useful purpose at all,
218 SUPREME COURT REPORTS ANNOTATED it is in the binding effect that this acceptance would put in place. As in the case of
an appointment, a pardon can be withdrawn at any time before it is accepted by
Risos-Vidal vs. Commission on Elections the grantor. Acceptance would thus be the means to tie the grantor to the grant.
v. People, 596 SCRA 77 (2009), where a mistake intervened in the fallo. What is important, to my mind, is proof of the communication of the pardon to the
Same; Same; Same; Conditional Pardon; Words and Phrases; View that aside convict, in the cases when terms and conditions are attached to the pardon.
from absolute pardon, there is the conditional pardon which is defined as “the Communications of these terms, and proof that the convict availed himself of the
exemption of an individual, within certain limits or conditions, from the
granted clemency, would suffice to conclude that the terms and conditions had would not have the effect of restoring these specific rights unless their specific
been accepted and should be observed. restoration is expressly mentioned in the pardon. The Erap’s pardon sought to
Same; Civil and Political Rights; View that in Simon v. Commission on comply with this RPC requirement by specifically stating that he was “restored to
Human Rights, 229 SCRA 117 (1994), the Supreme Court (SC) categorically his civil and political rights.” I take the view that this restoration already includes
explained the rights included under the term “civil and political rights,” in the the restoration of the right to vote and be voted for as these are rights subsumed
context of Section 18, Article XIII of the Constitution which provides for the within the “political rights” that the pardon mentions; in the absence of any ex-
Commission on Human Rights’ (CHR’s) power to investigate all forms of human 221
rights violations involving civil and political rights.”—In Simon v. Commission VOL. 747, JANUARY 21, 2015 221
on Human Rights, 229 SCRA 117 (1994), the Court categorically explained the
rights included under the term “civil and political rights,” in the context of Section Risos-Vidal vs. Commission on Elections
18, Article XIII of the Constitution which provides for the Commission on Human press accompanying reservation or contrary intent, this formulation grants
Rights’ power to investigate all forms of human rights violations involving civil a full restoration that is coterminous with the remitted principal penalty
and political rights. According to Simon, the term “civil rights,” has been defined of reclusion perpetua.
as referring (t)o those (rights) that belong to every citizen of the state or country, Same; Same; Same; View that to exclude the rights of suffrage and candidacy
or, in wider sense, to all its inhabitants, and are not connected with the from the restoration of civil and political rights shall likewise signify a diminution,
organization or administration of the government. They include the rights of other than what the Constitution allows, of the scope of pardon that the President
property, marriage, equal protection of the laws, freedom of contract, etc. or, as can extend under the 1987 Constitution.—In this age and time, “political rights”
otherwise cannot be understood meaningfully as rights with core values that our democratic
220 system protects, if these rights will not include the right to vote and be voted for.
To exclude the rights of suffrage and candidacy from the restoration of civil and
220 SUPREME COURT REPORTS ANNOTATED
political rights shall likewise signify a diminution, other than what the
Risos-Vidal vs. Commission on Elections Constitution allows, of the scope of pardon that the President can extend under
the 1987 Constitution. Significantly, this Constitution itself did not yet exist when
defined, civil rights are rights appertaining to a person by virtue of his the Revised Penal Code was passed so that this Code could not have taken into
citizenship in a state or community. Such term may also refer, in its general sense, account the intent of the framers of this Constitution to maintain the plenary
to rights capable of being enforced or redressed in a civil action. Also quite often nature of the pardoning power.
mentioned are the guarantees against involuntary servitude, religious Same; Same; Same; View that the Supreme Court (SC) still acknowledged
persecution, unreasonable searches and seizures, and imprisonment for that pardon may remove all the punitive consequences of a convict’s criminal act,
debt. Political rights, on the other hand, refer to the right to participate, directly including the disqualifications or disabilities based on the finding of guilt.—For
or indirectly, in the establishment or administration of government, the right of clarity, the inclusion phrase is part of the Court’s discussion in Monsanto v.
suffrage, the right to hold public office, the right of petition and, in general, Factoran, Jr., 170 SCRA 190 (1989), and was made in the context that although
the rights appurtenant to citizenship vis-à-vis the management of government. the Court repudiated the Ex ParteGarland, 71 U.S. 833 (1866), ruling (as cited
Same; Criminal Law; Pardon; View that the Erap’s pardon sought to comply in Pellobello v. Palatino, 72 Phil. 441 [1940], and Cristobal v. Labrador, 71 Phil.
with this Revised Penal Code (RPC) requirement by specifically stating that he was 34 [1940]) that pardon erases the guilt of the convict, the Court still acknowledged
“restored to his civil and political rights.” I take the view that this restoration that pardon may remove all the punitive consequences of a convict’s criminal
already includes the restoration of the right to vote and be voted for as these are act, including the disqualifications or disabilities based on the finding of
rights subsumed within the “political rights” that the pardon mentions; in the guilt.
absence of any express accompanying reservation or contrary intent, this Same; Same; Same; Civil and Political Rights; View that Erap’s pardon fully
formulation grants a full restoration that is coterminous with the remitted principal complied with the Revised Penal Code (RPC) requirements for the express remission
penalty of reclusion perpetua.—Reclusion perpetua, the penalty imposed on of the accessory penalty of perpetual absolute disqualification as the pardon in fact
Erap, carries with it the accessory penalty of civil interdiction for life or during the restored him to his civil and political rights.—In the present case, Erap’s pardon
period of the sentence and that of perpetual absolute disqualification which fully complied with the RPC requirements for the express remission of the
the offender shall suffer even though pardoned as to the principal penalty, unless accessory penalty of perpetual absolute disqualification as the pardon in fact
the same shall have been remitted in the pardon. The full understanding of the full restored him to his civil and political rights. In this
practical effects of pardon on the principal and the accessories penalties as 222
embodied in the RPC, requires the combined reading of Articles 36 and 41 of the 222 SUPREME COURT REPORTS ANNOTATED
RPC, with Article 41 giving full meaning to the requirement of Article 36 that the
restoration of the right to hold office be expressly made in a pardon if indeed this Risos-Vidal vs. Commission on Elections
is the grantor’s intent. An express mention has to be made of the restoration of the light, the Monsanto ruling still applies: while the PGMA pardon does
rights to vote and be voted for since a pardon with respect to the principal penalty not erase Erap’s guilt, it nonetheless remitted his disqualification to run
for public office and to vote as it expressly restored him to his civil and merits; and (d) that, between the first and the second actions, there is identity of
political rights. The Office of the Solicitor General succinctly expressed parties, subject matters, and causes of action.
the Monsanto ratio decidendi when it said that the Court, despite ruling against Same; Same; Judgments; Immutability of Judgments; View that once a
Monsanto, “nevertheless reaffirmed the well-settled doctrine that the grant of judgment attains finality, it becomes immutable and unalterable. It may not be
pardon also removes one’s absolute disqualification or ineligibility to hold public changed, altered or modified in any way even if the modification is for the purpose
office.” of correcting an erroneous conclusion of fact or law.—Once a judgment attains
Statutes; Statutory Construction; View that laws governing election contests finality, it becomes immutable and unalterable. It may not be changed, altered or
must be liberally construed to the end that the will of the people in the choice of modified in any way even if the modification is for the purpose of correcting an
public officials may not be defeated by mere technical objections.—Technicalities erroneous conclusion of fact or law. This is the “doctrine of finality of
and procedural niceties in election cases should not be made to stand in the way of judgments” which binds the immediate parties and their privies in
the true will of the electorate. Laws governing election contests must be liberally personal judgments; the whole world in judgments in rem; and even the
construed to the end that the will of the people in the choice of public officials may highest court of the land as to their binding effect.
not be defeated by mere technical objections. Election contests involve public Same; Same; Same; Same; Judgment on the Merits; View that a judgment is
interest, and technicalities and procedural barriers must yield if they constitute on the merits when it determines the rights and liabilities of the parties based on
an obstacle to the determination of the true will of the electorate in the choice of the disclosed facts, irrespective of formal, technical or dilatory objections.—A
their elective officials. The Court frowns upon any interpretation of the law that judgment is on the merits when it determines the rights and liabilities of the
would hinder in any way not only the free and intelligent casting of the votes in an parties based on the disclosed facts, irrespective of formal, technical or dilatory
election but also the correct ascertainment of the results. objections.
Remedial Law; Civil Procedure; Res Judicata; View that jurisprudence has 224
clarified that res judicata does not require absolute identity, but merely substantial 224 SUPREME COURT REPORTS ANNOTATED
identity.—Res judicata embraces two concepts: first, the bar by prior judgment
under Rule 39, Section 47(b) of the Rules of Court; and second, the preclusion of a Risos-Vidal vs. Commission on Elections
settled issue or conclusiveness of judgment under Rule 39, Section 47(c) of the Same; Same; Same; Res Judicata; View that when a right or fact has been
Rules of Court. The COMELEC’s 2010 decision resolving whether Erap’s pardon judicially tried and determined by a court of competent jurisdiction or an
allowed him to run for elections precludes further discussion of the very same issue opportunity for such trial has been given, the judgment of the court, as long as it
in the 2013 petition filed against his candidacy. Under our review in the present remains unreversed, should be conclusive upon the parties and those in privity with
case that is limited to the determination of grave abuse of discretion and not legal them.—At this juncture, I reiterate my disagreement with J. Leonen in strictly
error, I cannot agree with J. Leonen’s strict application of the requisites of bar by applying the requisites for the application of res judicata through bar by prior
prior judgment. Jurisprudence has clarified that res judicata does not require judgment. The Court itself, in numerous cases, did not strictly apply the
absolute identity, but merely substantial identity. This consideration, under a requirement that there must be absolute identity of causes of action. In fact, the
grave abuse standard of Court’s rulings on this particular element leaned towards substantial identity of
223 causes of action and its determination is arrived at not on the basis of the facial
value of the cases but after an in-depth analysis of each case. The reason why
VOL. 747, JANUARY 21, 2015 223
substantial identity of causes of action is permitted is to preclude a situation where
Risos-Vidal vs. Commission on Elections a party could easily escape the operation of res judicata by changing the form of
the action or the relief sought. The difference in form and nature of the two actions
review, leads me to the conclusion that we cannot reverse the COMELEC’s is also immaterial and is not a reason to exempt these cases from the effects of res
decision to apply res judicata, even if it meant the application of the concept of bar judicata. The philosophy behind this rule prohibits the parties from litigating the
by prior judgment. same issue more than once. When a right or fact has been judicially tried and
Same; Same; Same; View that res judicata, by way of bar by prior judgment, determined by a court of competent jurisdiction or an opportunity for such
binds the parties to a case, as well as their privies to its judgment, and prevents trial has been given, the judgment of the court, as long as it remains
them from re-litigating the same cause of action in another case.—Res judicata, by unreversed, should be conclusive upon the parties and those in privity with
way of bar by prior judgment, binds the parties to a case, as well as their privies them. In this way, there should be an end to litigation by the same parties and
to its judgment, and prevents them from re-litigating the same cause of action in their privies over a subject, once the issue involving the subject is fully and fairly
another case. Otherwise put, the judgment or decree of the court of competent adjudicated.
jurisdiction on the merits concludes the litigation between the parties, as well as Mendoza, J., Concurring Opinion:
their privies, and constitutes a bar to a new action or suit involving the same cause Constitutional Law; Criminal Law; Pardon; View that the acceptance confers
of action before the same or other tribunal. Res judicata through bar by prior effectivity in both absolute and conditional pardon.—I am of the view that the
judgment requires (a) that the former judgment be final; (b) that the judgment was acceptance confers effectivity in both absolute and conditional pardon. Pardon is
rendered by a court of competent jurisdiction; (c) that it is a judgment on the defined as “an act of grace, proceeding from the power entrusted with the execution
of the laws, which exempts the individual, on whom it is bestowed, from the reasons. That the executive clemency given to Estrada was unaccompanied by any
punishment the law inflicts for a crime he has committed. It is the private, though condition is clearly visible in the text of the pardon. The Court must simply read
official act of the executive magistrate, delivered to the individual for whose benefit the pardon as it is written. There is no necessity to resort to construction.
it is intended, and not communicated officially to the Court. ... A pardon is a deed, Same; Same; Same; View that a statement describing Estrada’s previous
to the validity of commitment not to seek any elective office cannot operate as a condition for his
225 pardon, sans any indication that it was intended to be so.—Suffice it to say, a
VOL. 747, JANUARY 21, 2015 225 statement describing Estrada’s previous commitment not to seek any elective office
cannot operate as a condition for his pardon, sans any indication that it was
Risos-Vidal vs. Commission on Elections intended to be so. In light of the clear absence of any condition in the pardon, no
which delivery is essential, and delivery is not complete without acceptance.” ambiguity warrants interpretation by the Court. At the most, the subject whereas
Same; Same; Same; View that the significance of “acceptance” is more clause depicts the state of affairs at the time when the pardon was granted. It
apparent in cases of “commutation,” which is the substitution of a lighter should not be considered as part and parcel of the entire act as it serves neither
punishment for a heavier one.—An “acceptance” does not classify a pardon as the ability to enlarge or confer powers nor the authority to control the words of the
conditional just by the mere reception and the placing of an inscription thereon. I act.
am not prepared to ignore the very intention and content of a pardon as standards Same; Same; Same; Separation of Powers; View that the pardoning power is
to determine its nature, as against the mere expediency of its delivery and granted exclusively to the President amidst the constitutional scheme of checks and
acceptance. I am much more amenable to the rule consistent with the benevolent balances; It would do the Court well to remember that neither the Congress nor the
nature of pardon: that it is an act of forgiveness predicated on an admission of courts can question the motives of the President in the use of the power.—The
guilt. To be effective, therefore, this admission of past wrongdoing must be pardoning power is granted exclusively to the President amidst the constitutional
manifested by the acceptance of a pardon, absolute or conditional. Further, the scheme of checks and balances. While it is most ideal that the executive strictly
significance of “acceptance” is more apparent in cases of “commutation,” which is adheres to this end, it is undeniable that the pardoning power is still dependent
the substitution of a lighter punishment for a heavier one. on the grantor’s measure of wisdom and sense of public policy. This reality invites,
Same; Same; Same; View that the “whereas clauses” in Estrada’s pardon if not bolsters, the application of the political question doctrine. The only weapon,
cannot adversely affect the ultimate command which it evokes, that is, executive which the Court has freedom to wield, is the exercise of judicial power against a
clemency is granted to Estrada absent any condition.—Primarily, rules on blatant violation of the Constitution. When unavailing, the Court is constrained to
statutory construction provide that whereas clauses, do not form part of a statute, curb its own rebuking power and to uphold the acumen of a coequal branch. It
strictly speaking; they are not part of the operative language of the statute. While would do the Court well to remember that neither the Congress nor the courts can
they may be helpful to the extent that they articulate the general purpose or reason question the motives of the President in the use of the power.
underlying a new enactment, reliance on whereas clauses as aids in construing 227
statutes is not justified when their interpretation “control the specific terms of the
VOL. 747, JANUARY 21, 2015 227
statute.” As applied in Estrada’s case, the subject whereas clause does not purport
to control or modify the unequivocal terms found in the pardon’s body. In this Risos-Vidal vs. Commission on Elections
sense, the “whereas clauses” in Estrada’s pardon cannot adversely affect the Same; Same; Same; View that a person adjudged guilty of an offense is a
ultimate command which it evokes, that is, executive clemency is granted to convicted criminal, though pardoned; he may be deserving of punishment, though
Estrada absent any condition. left unpunished; and the law may regard him as more dangerous to society than
Same; Same; Same; View that as no condition was patently evinced in the one never found guilty of crime, though it places no restraints upon him following
document, the Supreme Court (SC) is at no liberty to shape one, only because the his conviction; Estrada’s past conviction for plunder would forever form part of his
plain meaning of the pardon’s text is unacceptable for some waylaid and extraneous person, whether as a private individual or a public officer.—Lest it be
reasons. That the executive clemency given to Estrada was unaccompanied by any misunderstood, this conclusion does not degenerate from the doctrine that a
condition is clearly visible in the text of the pardon.—For a condition to be pardon only relieves a party from the punitive consequences of his past crimes,
operative, the condition must appear on the face of the docu- nothing more. Indeed, “a person adjudged guilty of an offense is a convicted
226 criminal, though pardoned; he may be deserving of punishment, though left
226 SUPREME COURT REPORTS ANNOTATED unpunished; and the law may regard him as more dangerous to society than one
never found guilty of crime, though it places no restraints upon him following his
Risos-Vidal vs. Commission on Elections conviction.” Estrada was not reborn into innocence by virtue of the forgiveness
ment. The conditions must be clear and specific. The reason is that the bestowed in by the pardon. The moral stain caused by his past crimes remains to
conditions attached to a pardon should be definite and specific as to inform the be part of his person, then as now. In no way did his pardon serve as a stamp of
person pardoned of what would be required. As no condition was patently evinced incorruptibility. It is not a magic spell that superimposes virtuousness over guilt.
in the document, the Court is at no liberty to shape one, only because the plain His past conviction for plunder would forever form part of his person, whether as
meaning of the pardon’s text is unacceptable for some waylaid and extraneous a private individual or a public officer.
Same; Same; Same; View that one thing is clear, in the exercise of her anchors itself on statutorily prescribed disqualifications — under Section 40 of the
exclusive power to grant executive clemency, President Gloria Macapagal-Arroyo Local Government Code, as well as Section 12 of the Omnibus Election Code —
(PGMA) pardoned Estrada, thereby wiping away the penalties of his crime and which jurisprudence has explicitly recognized as a valid basis for both a petition
entitling him the right to run for public office.—Without squabble, plunder is a for disqualification and a Section 78
crime involving moral turpitude. Nevertheless, this fact alone negates a 229
mechanical application of statutory provisions on disqualification. One thing is VOL. 747, JANUARY 21, 2015 229
clear, in the exercise of her exclusive power to grant executive clemency, PGMA
pardoned Estrada, thereby wiping away the penalties of his crime and entitling Risos-Vidal vs. Commission on Elections
him the right to run for public office. Corollary to this, Estrada’s fitness to hold petition. It follows that the petition was filed on time. The petition was filed
public office is an issue that should not concern the Court. All that the Court can on January 14, 2013, after the last day for filing of certificates of candidacy, and
rule on is the availability of Estrada’s right to seek public office. This ruling on his before the date of Estrada’s proclamation as Mayor on May 17, 2013. This is within
eligibility is not tantamount to a declaration that Estrada befits a person wholly the period permitted by Rule 25, Section 3 of COMELEC Resolution No. 9523.
deserving of the people’s trust. The Manileños’ decision alone can mould the city’s Remedial Law; Civil Procedure; Intervention; Legal Standing; View that in
journey to either development or decline. Indeed, election expresses the sovereign seeking to intervene, Lim has made no pretensions of acting as a representative of
will of the people consistent with the principle of vox populi est suprema lex. This the general public and, thus, advancing the public interest; Though what is
is the beauty of democracy which the Court must endeavour to protect at all cost. involved is a public office, what Lim seeks to enforce is, fundamentally, a (supposed)
As right accruing to him personally to assume an office.—In seeking to intervene, Lim
228 has made no pretensions of acting as a representative of the general public and,
thus, advancing the public interest. He merely prays that he be declared the
228 SUPREME COURT REPORTS ANNOTATED
elected Mayor of the City of Manila following a declaration that Estrada was
Risos-Vidal vs. Commission on Elections disqualified to run for the same post. Though what is involved is a public office,
Abraham Lincoln put it with both guile and eloquence: Elections belong to what Lim seeks to enforce is, fundamentally, a (supposed) right accruing to
the people. It’s their decision. If they decide to turn their back on the fire and burn him personally to assume an office. Lim has enough interest at stake in this case
their behinds, then they will just have to sit on their blisters. as would enable him to intervene. Rule 19, Section 1 of the 1997 Rules of Civil
Leonen, J., Dissenting Opinion: Procedure provides for who may intervene in a pending court action: Section
Election Law; Cancellation of Certificate of Candidacy; Disqualification of 1. Who may intervene.—A person who has a legal interest in the matter in litigation,
Candidates; View that it is clear that a false claim of eligibility made in a certificate or in the success of either of the parties, or an interest against both, or is so situated
of candidacy (CoC) despite a prior conviction which carries with it the accessory as to be adversely affected by a distribution or other disposition of property in the
penalty of disqualification is a ground for a Section 78 petition. Nevertheless, it is custody of the court or of an officer thereof may, with leave of court, be allowed to
also a ground for a petition for disqualification.—From these, it is clear that a false intervene in the action. The court shall consider whether or not the intervention
claim of eligibility made in a certificate of candidacy despite a prior conviction will unduly delay or prejudice the adjudication of the rights of the original parties,
which carries with it the accessory penalty of disqualification is a ground for a and whether or not the intervenor’s rights may be fully protected in a separate
Section 78 petition. Nevertheless, it is also a ground for a petition for proceeding.
disqualification. As explained in Dominador Jalosjos, Jr., 683 SCRA 1 (2012): Election Law; Cancellation of Certificate of Candidacy; Disqualification of
What is indisputably clear is that the false material representation of Jalosjos is a Candidates; View that it is true that the principal matter for resolution in this case
ground for a petition under Section 78. However, since the false material is whether Estrada, based on circumstances personally applying to him, was
representation arises from a crime penalized by prisión mayor, a petition under qualified to run for Mayor of the City of Manila. Nevertheless, the logical
Section 12 of the Omnibus Election Code or Section 40 of the Local Government consequence of a decision adverse to Estrada is the need to identify who shall,
Code can also be properly filed. The petitioner has a choice whether to anchor his henceforth, assume the position of Mayor.—It is true that the principal matter for
petition on Section 12 or Section 78 of the Omnibus Election Code, or on Section resolution in this case is whether Estrada, based on
40 of the Local Government Code. The law expressly provides multiple circumstances personally applying to him, was qualified to run for
remedies and the choice of which remedy to adopt belongs to the petitioner. 230
The concurrent availability of a Section 78 petition with a petition for 230 SUPREME COURT REPORTS ANNOTATED
disqualification should not be interpreted as diminishing the distinction between
the two (2) remedies. Risos-Vidal vs. Commission on Elections
Same; Same; Same; View that this petition unambiguously anchors itself on
statutorily prescribed disqualifications — under Section 40 of the Local Mayor of the City of Manila. Nevertheless, the logical consequence of a
Government Code (LGC), as well as Section 12 of the Omnibus Election Code (OEC) decision adverse to Estrada is the need to identify who shall, henceforth, assume
— which jurisprudence has explicitly recognized as a valid basis for both a petition the position of Mayor. Lim claims that he is entitled to replace Estrada. In support
for disqualification and a Section 78 petition.—This petition unambiguously of this, he cites a decision of this court and claims that, as a disqualified candidate,
the votes cast for Estrada should be deemed stray votes. This would result in Lim or has been sentenced by final judgment for subversion, insurrection,
being the qualified candidate obtaining the highest number of votes, which would, rebellion or for any offense for which he has been sentenced to a penalty
in turn, entitle him to being proclaimed the elected Mayor of the City of Manila. of more than eighteen months or for a crime involving moral turpitude,
Remedial Law; Civil Procedure; Intervention; Legal Standing; View that it is shall be disqualified to be a candidate and to hold any office, unless he
worth emphasizing that [t]he purpose of intervention is to enable a stranger to an has been given plenary pardon or granted amnesty. This [sic]
action to become a party in order for him to protect his interest and for the court to disqualifications to be a candidate herein provided shall be deemed removed upon
settle all conflicting claims.—It is worth emphasizing that “[t]he purpose of the declaration by competent authority that said insanity or incompetence had
intervention is to enable a stranger to an action to become a party in order for him been removed or after the expiration of a period of five years from his service of
to protect his interest and for the court to settle all conflicting claims. Intervention sentence, unless within the same period he again becomes disqualified. (Emphasis
is allowed to avoid multiplicity of suits more than on due process considerations.” supplied) Section 40 of the Local Government Code provides for disqualifications
Lim’s intervention serves this purpose. It enables the resolution of an issue which for local elective offices in particular: SECTION 40. Disqualifications.—The
is corollary to one of the two ways by which this court may decide on the issue of following persons are disqualified from running for any elective local position:
Estrada’s disqualification. (a) Those sentenced by final judgment for an offense involving moral
Election Law; Disqualification of Candidates; Res Judicata; View that the turpitude or for an offense punishable by one (1) year or more of
2010 disqualification cases filed against Estrada in connection with his 2010 bid imprisonment, within two (2) years
for the presidency do not bar the present case on account of res judicata.—The 2010 232
disqualification cases filed against Estrada in connection with his 2010 bid for the 232 SUPREME COURT REPORTS ANNOTATED
presidency do not bar the present case on account of res judicata. For one, the 2010
disqualification cases filed by Atty. Evilio C. Pormento and Mary Lou B. Estrada Risos-Vidal vs. Commission on Elections
involved issues and were anchored on causes of action that are markedly different after serving sentence; (b) Those removed from office as a result of an
from those in the present case. These cases were anchored on the constitutional administrative case; (c) Those convicted by final judgment for violating the oath of
prohibition against a President’s reelection, as provided by Article VII, Section 4 allegiance to the Republic; (d) Those with dual citizenship; (e) Fugitives from
of the 1987 Constitution, and the additional ground that Estrada was a nuisance justice in criminal or nonpolitical cases here or abroad; (f) Permanent residents in
candidate. To the contrary, the present case is anchored on Estrada’s conviction a foreign country or those who have acquired the right to reside abroad and
for plunder which carried with it the accessory penalty of perpetual absolute continue to avail of the same right after the effectivity of this Code; and (g) The
disqualification and invokes Section 40 of the Local Government Code, as well as insane or feeble-minded.
Section 12 of the Omnibus Election Code. Constitutional Law; Executive Clemency; View that jurisprudence as recent
231 as 2007 clarified that a court cannot preempt the grant of executive clemency.—The
present, the 1987 Constitution, requires prior conviction. Nevertheless, it retains
VOL. 747, JANUARY 21, 2015 231
the fundamental regard for the pardoning power as executive in nature.
Risos-Vidal vs. Commission on Elections Jurisprudence dating to 1991 noted how the 1986 Constitutional Commission
Same; Same; View that Estrada, though adjudged by the Commission on rejected a proposal to render the coverage of the pardoning power susceptible to
Elections (COMELEC) Second Division and COMELEC En Banc to be qualified legislative interference, particularly in matters relating to graft and corruption.
for a second bid at the presidency, was never conclusively adjudged by this court to Likewise, jurisprudence as recent as 2007 clarified that a court cannot preempt
be so qualified.—Estrada, though adjudged by the COMELEC Second Division and the grant of executive clemency.
COMELEC En Banc to be qualified for a second bid at the presidency, was never Same; Same; View that the 1987 Constitution, in Article VII, Section 19,
conclusively adjudged by this court to be so qualified. The 2010 disqualification enumerates the acts or means through which the President may extend clemency.—
cases reached their conclusion not because it was determined, once and for all, that The 1987 Constitution, in Article VII, Section 19, enumerates the acts or means
Estrada was not disqualified, but because — with Estrada’s loss in the elections — through which the President may extend clemency: (1) reprieve, or “the deferment
there was no longer a controversy to resolve. There was no “determin[ation of] the of the implementation of the sentence for an interval of time”; (2) commutation,
rights and liabilities of the parties based on the disclosed facts, irrespective of which “refers to the reduction of the duration of a prison sentence of a prisoner”;
formal, technical or dilatory objections”; neither was there “a determination of (3) remission of fines and forfeitures; (4) pardon; and (5) amnesty.
which party is right.” While the 2010 disqualification cases may have reached their Same; Pardon; View that Article VII, Section 19 of the 1987 Constitution
literal end or terminal point, there was no final judgment on the merits. provides two (2) limitations on the President’s exercise of the power to pardon: first,
Same; Same; View that Section 12 of the Omnibus Election Code (OEC) it can only be given after final conviction; and second, it cannot be exercised “in
provides for disqualifications for elective offices in general; Section 40 of the Local cases of impeachment, or as otherwise provided in this Constitution.”—Article VII,
Government Code (LGC) provides for disqualifications for local elective offices in Section 19 of the 1987 Constitution provides two (2) limitations on the President’s
particular.—Section 12 of the Omnibus Election Code provides for exercise of the power to pardon: first, it can only be given after final conviction;
disqualifications for elective offices in general: Section 12. Disqualifications.—Any and second, it cannot be exercised “in cases of impeachment, or as otherwise
person who has been declared by competent authority insane or incompetent,
provided in this Constitution.” Elsewhere in the Constitution, Article IX, C, Same; Same; Same; View that no grant of constitutional power is immune
Section 5 provides that: “No pardon, from review if it is done arbitrarily or without reason, capriciously, or on the basis
233 of whim.—Parenthetically, the Constitution also grants this court jurisdiction to
VOL. 747, JANUARY 21, 2015 233 determine “whether or not there has been a grave abuse of discretion amounting
to . . . excess of jurisdiction on the part of any branch or instrumentality of the
Risos-Vidal vs. Commission on Elections Government.” This means that no grant of constitutional power is immune from
amnesty, parole, or suspension of sentence for violation of election laws, review if it is done arbitrarily or without reason, capriciously, or on the basis of
rules, and regulations shall be granted by the President without the favorable whim. However, this court’s power of review in the present case is not raised by
recommendation of the Commission [on Elections].” Outside of the Constitution, any party and, thus, not an issue that this court must decide.
the Revised Penal Code contains provisions relating to pardon. Article 36 of the Same; Same; Same; View that from the plain text of the dispositive portion of
Revised Penal Code provides that: “A pardon shall in no case exempt the culprit the pardon extended by former President Gloria Macapagal-Arroyo (PGMA) to
from the payment of the civil indemnityimposed upon him.” The same Article 36 Estrada, it can be readily seen that there is no categorical statement actually saying
prescribes that for pardon to effect the restoration of the rights of suffrage and to that Estrada’s rights to vote and be voted for elective public office are restored, or
hold public office, “such rights [must] be expressly restored by the terms of the that the penalty of perpetual absolute disqualification is remitted.—The dispositive
pardon.” portion of the pardon extended by former President Gloria Macapagal-Arroyo to
Election Law; Criminal Law; Pardon; View that on suffrage and/or the rights Estrada reads: IN VIEW HEREOF and pursuant to the authority conferred upon
to vote for and be elected to public office, Articles 40 to 43 of the Revised Penal Code me by the Constitution, I hereby grant executive clemency to JOSEPH EJERCITO
(RPC) provide that the penalties of perpetual absolute disqualification, temporary ESTRADA, convicted by the Sandiganbayan of Plunder and imposed a penalty
absolute disqualification, perpetual special disqualification, and perpetual special of Reclusion Perpetua. He is hereby restored to his civil and political rights. The
disqualification on suffrage, which attach as accessory penalties to death, reclusion forfeitures imposed by the Sandiganbayan remain in force and in full, including
perpetua, reclusion temporal, prisión mayor and prisión correccional, as the case all writs and processes issued by the Sandiganbayan in pursuance hereof, except
may be, shall still be suffered by the offender even though pardoned as to the for the bank account(s) he owned before his tenure as President. Upon acceptance
principal penalty, “unless . . . expressly remitted in the pardon.”—Also on suffrage of this pardon by JOSEPH EJERCITO ESTRADA, this pardon shall take effect.
and/or the rights to vote for and be elected to public office, Articles 40 to 43 of the From the plain text of this disposition, it can be readily seen that there is
Revised Penal Code provide that the penalties of perpetual absolute no categorical statement actually saying that Estrada’s rights to vote and be voted
disqualification, temporary absolute disqualification, perpetual special for elective public office are restored, or that the penalty of perpetual absolute
disqualification, and perpetual special disqualification on suffrage, which attach disqualification is remitted.
as accessory penalties to death, reclusion perpetua, reclusion temporal, prisión 235
mayor and prisión correccional, as the case may be, shall still be suffered by the
VOL. 747, JANUARY 21, 2015 235
offender even though pardoned as to the principal penalty, “unless . . . expressly
remitted in the pardon.” Risos-Vidal vs. Commission on Elections
Same; Same; Same; View that Articles 36 and 41 of the Revised Penal Code Constitutional Law; Civil and Political Rights; View that Estrada capitalizes
(RPC) impress upon the President the significance of departing from the purely on the broad conception of civil and political rights as including in its scope the
private consequences of pardon should he or she stray into the public affair of rights of suffrage and the right to hold public office.—Estrada capitalizes on the
restoring a convict’s rights of suffrage and/or to hold public office.—Recall that the broad conception of civil and political rights as including in its scope the rights of
manner by which the 1987 Constitution phrases its investiture on the President of suffrage and the right to hold public office. That is precisely the handicap in his
the pardoning power now includes the phrase “as otherwise provided in this theory: It is broad; it fails to account for requirements relating to specific rights.
Constitution.” This phrase affirms the imperative of reading and interpreting the As against the broad concept of civil and political rights as an expansive composite
Constitution in its entirety, not taking a provision in isolation. The pardoning or a vast spectrum of rights having to do with liberty and membership in the
power of the President must, political community, Articles 36 and 41 of the Revised Penal Code specifically deal
234 with the rights of suffrage and to hold public office. Juxtaposed with the manifold
VOL. 747, JANUARY 21, 2015 234 category of civil and political rights, the effect of Articles 36 and 41 is that, in the
specific context of the President’s exercise of the power to grant pardon to a convict,
Risos-Vidal vs. Commission on Elections the rights of suffrage and to hold public office are segregated from all other similar
thus, not be divorced from the Constitution’s injunction that “[p]ublic office rights.
is a public trust.” Read in harmony with this injunction, Articles 36 and 41 of the Same; Pardon; View that it is revealing that former President Gloria
Revised Penal Code impress upon the President the significance of departing from Macapagal-Arroyo (PGMA) chose to deviate from many historical examples and
the purely private consequences of pardon should he or she stray into the public from what appears to be common practice.—The President must be presumed to be
affair of restoring a convict’s rights of suffrage and/or to hold public office. fully cognizant of the significance and consequences of the manner by which he or
she executes official acts, as well as the manner by which they are formally reduced
to writing. It is revealing that former President Gloria Macapagal-Arroyo chose to 237
deviate from many historical examples and from what appears to be common VOL. 747, JANUARY 21, 2015 237
practice. Aware of the significance of excluding the qualifier “full,” she chose to
grant pardon to Estrada under entirely generic and indistinct terms. Risos-Vidal vs. Commission on Elections
Statutory Construction; Preamble; View that jurisprudence and other official 682 (1997), unequivocally underscored the abhorrence that animates the
acts of this court are replete with instances in which reference to preambular clauses classification of plunder as a heinous crime punishable by death.
was resorted to in interpreting instruments other than statutes and official acts of Same; Same; View that plundering as a crime and by its scale, entails more
the President.—Jurisprudence and other official acts of this court are replete with than greed and covetousness.—Plundering as a crime and by its scale, therefore,
instances in which reference to preambular clauses was resorted to in interpreting entails more than greed and covetousness. It conjures the image of a public officer
instruments other than statutes and official acts of the President. In Licaros v. deluded in the thought that he or she is some overlord, free to ravage and entitled
Gatmaitan, 362 SCRA 548 (2001), this court sustained the Court of Appeals’ to seize all that his or her realm can provide. It entails more than ordinary moral
reference to a whereas clause in a contract between private parties (i.e., a turpitude (i.e., an inherently immoral act) as acts like theft, robbery, bribery,
memorandum of agreement) and thereby the conclusion that the parties “intended profiteering, estafa, extortion, and embezzlement have been categorized. It evinces
to treat such a degree of depravity and debasement so heinous that, were it not for the
236 subsequent enactment of a statute (i.e., Republic Act No. 9346), it would remain
punishable by death.
236 SUPREME COURT REPORTS ANNOTATED
Same; Same; View that Congress, in choosing to penalize plunder with
Risos-Vidal vs. Commission on Elections reclusion perpetua to death, must certainly have been cognizant of how these
their agreement as one of conventional subrogation.” In Kuwait penalties did not only entail the deprivation of the right to life and/or liberty, but
Airways Corporation v. Philippine Airlines, Inc., 587 SCRA 399 (2009), it was also of how, consistent with Articles 40 and 41 of the Revised Penal Code (RPC),
impliedly acknowledged that resort to a whereas clause is permissible in they carried the accessory penalty of perpetual absolute disqualification.—
interpreting a contract entered into by the government; except that, because the Recognition must be given to the legislative wisdom underlying the choice of
circumstances have changed, it was deemed unnecessary to proceed to an penalty. This is not only with respect to the severity of punishment chosen (i.e.,
interpretation in light of the relevant whereas clause. In Conte v. Commission on deprivation of life or deprivation of liberty for the longest duration contemplated
Audit, 264 SCRA 19 (1996), this court referred to whereas clauses in interpreting by the scale of penalties under the Revised Penal Code) but similarly with all other
a resolution issued by the Social Security System. Similarly, this court’s En accessories that the penalties of reclusion perpetua and/or death entail. Congress,
Banc resolution in A.M. No. 99-8-01-SC, issued by this court in the exercise of its in choosing to penalize plunder with reclusion perpetua to death, must certainly
rule-making power, cited a statute’s whereas clause. have been cognizant of how these penalties did not only entail the deprivation of
Constitutional Law; Pardon; View that the pardon extended to Estrada is the right to life and/or liberty, but also of how, consistent with Articles 40 and 41
definite by its omission: There is neither an express restoration of Estrada’s rights of the Revised Penal Code, they carried the accessory penalty of perpetual absolute
to vote and be voted for elective public office nor a remission of his perpetual disqualification.
absolute disqualification.—The pardon extended to Estrada is definite by its Constitutional Law; Pardon; View that the inclusion of the third preambular
omission: There is neither an express restoration of Estrada’s rights to vote and be clause is not empty rhetoric. It is an indispensable qualifier indicating that Estrada
voted for elective public office nor a remission of his perpetual absolute was pardoned precisely in view of his promise to no longer seek (elective) public
disqualification. To this extent, it is clear and unambiguous. This should suffice to office.—Consider the recognition made in the first and second preambular clauses
put an end to Estrada’s asseverations that he was qualified to run for Mayor of that Estrada was already more than 70 years old and had been in detention for
Manila. Nevertheless, even if the position that there remains room for about six and a half years. These preambular clauses provide
interpretation was to be indulged, a reading of the pardon as a whole, and an 238
illumination, through the preambular clauses, of the pardon’s supposed ambiguity, 238 SUPREME COURT REPORTS ANNOTATED
will lead to the same conclusion: Estrada was and remains to be disqualified.
Criminal Law; Plunder; View that in 2001, in Estrada v. Sandiganbayan, Risos-Vidal vs. Commission on Elections
369 SCRA 394, the Supreme Court (SC), against the asseverations of Estrada
himself, ruled that plunder is inherently immoral, i.e., malum in se.—In 2001, context to why President Gloria Macapagal-Arroyo saw wisdom in tempering
in Estrada v. Sandiganbayan, 369 SCRA 394, this court, against the asseverations Estrada’s suffering: Keeping in prison a septuagenarian — a man who could well
of Estrada himself, ruled that plunder is inherently immoral, i.e., malum in se. In be considered to be in the twilight years of his life — may be too severe; anyway,
so doing, this court, quoting the concurring opinion of Justice Vicente V. Mendoza, Estrada had already been deprived of liberty for a considerable length of time. The
emphasized that any doubt on the inherent immorality of plunder “must be third preambular clause is even more revealing. It unveils the undertaking made
deemed to have been resolved in the affirmative by the decision of Congress in by Estrada (acknowledged and unchallenged by him through his unqualified
1993 to include it among the heinous crimes punishable by reclusion perpetua to handwritten acceptance) that he would no longer embark on the very same affair,
death.” Estrada v. Sandiganbayan, quoting People v. Echegaray, 267 SCRA i.e., (elective) public office, that facilitated his commission of plunder. The inclusion
of the third preambular clause is not empty rhetoric. It is an indispensable qualifier serve as elective public officials, those qualifications must be met before one even
indicating that Estrada was pardoned precisely in view of his promise to no longer becomes a candidate. When a person who is not qualified is voted for and
seek (elective) public office. Similarly, it establishes that the grant of pardon eventually garners the highest number of votes, even the will of the electorate
notwithstanding, there is no betrayal of the fundamental policy of aversion against expressed through the ballot cannot cure the defect in the qualifications of the
plunder as an affront to “the larger socio-political and economic context.” candidate. To rule otherwise is to trample upon and rent asunder the very law that
Election Law; Disqualification of Candidates; View that Estrada was sets forth the qualifications and disqualifications of candidates. We might as well
disqualified to run for Mayor of the City of Manila in the May 13, 2013 elections. write off our election laws if the voice of the electorate is the sole determinant of
Moreover, his perpetual absolute disqualification not having been remitted, and his who should be proclaimed worthy to occupy elective positions in our republic.
rights to vote and be voted for elective public office not having been restored, Estrada 240
remains bound to suffer the effects of the penalty of perpetual absolute 240 SUPREME COURT REPORTS ANNOTATED
disqualification, as listed in Article 30 of the Revised Penal Code (RPC).—In sum,
Estrada was disqualified to run for Mayor of the City of Manila in the May 13, Risos-Vidal vs. Commission on Elections
2013 elections. Moreover, his perpetual absolute disqualification not having been SPECIAL CIVIL ACTION in the Supreme Court. Certiorari and Petition-in-
remitted, and his rights to vote and be voted for elective public office not having Intervention.
been restored, Estrada remains bound to suffer the effects of the penalty of The facts are stated in the opinion of the Court.
perpetual absolute disqualification, as listed in Article 30 of the Revised Penal Rodolfo G. Palattao for petitioner.
Code. Specifically, he remains disqualified from exercising the right to vote in any Renato G. Dela Cruz for petitioner-intervenor.
election for any popular elective office, and he remains barred from occupying any Pacifico A. Agabin for private respondent.
public office, elective, or otherwise.
Same; Same; View that Estrada did secure more votes than Lim, that much LEONARDO-DE CASTRO, J.:
can be conceded; but these votes were cast in favor of an ineligible candidate, i.e.,
one who was no candidate at all.—Estrada is very loosely invoking the concept of Before the Court are (1) a Petition for Certiorari filed under Rule 64, in relation
a “sovereign” as though a plurality of votes is the sole determinant of the “sovereign to Rule 65, both of the Revised Rules of Court, by Atty. Alicia Risos-Vidal (Risos-
will.” In the first place, what is involved here is merely an election Vidal), which essentially prays for the issuance of the writ of certiorari annulling
239 and setting aside the April 1, 20131 and April 23, 20132 Resolutions of the
Commission on Elections (COMELEC), Second Division and En Banc,
VOL. 747, JANUARY 21, 2015 239
respectively, in SPA No. 13-211 (DC), entitled “Atty. Alicia Risos-Vidal v. Joseph
Risos-Vidal vs. Commission on Elections Ejercito Estrada” for having been rendered with grave abuse of discretion
for a local elective position. Certainly, the voters of a single local government amounting to lack or excess of jurisdiction; and (2) a Petition-in-Intervention3 filed
unit ought not to be equated with the “sovereign Filipino people.” So blithely is by Alfredo S. Lim (Lim), wherein he prays to be declared the 2013 winning
Estrada celebrating his 349,770 votes, he seems to forget that Lim was not even candidate for Mayor of the City of Manila in view of private respondent former
too far off with 313,764 votes. Estrada celebrates the casting of votes in his favor President Joseph Ejercito Estrada’s (former President Estrada) disqualification to
as a seemingly indubitable expression of the sovereign will in trusting him with run for and hold public office.
elective public office. He forgets that a mere three years prior, the voters, not just
of the City of Manila, but of the entire Republic, repudiated him and rejected his The Facts
attempt to once again secure the Presidency. He placed a distant second, behind
by more than 5.72 million votes, to President Benigno Simeon Aquino III. Estrada The salient facts of the case are as follows:
did secure more votes than Lim, that much can be conceded; but these votes were On September 12, 2007, the Sandiganbayan convicted former President
cast in favor of an ineligible candidate, i.e., one who was no candidate at all. Estrada, a former President of the Republic of the Philippines, for the crime of
Same; Same; View that by definition, an ineligible individual is not even a plunder in Criminal Case No.
candidate in the first place.—By definition, an ineligible individual is not even a _______________
candidate in the first place. It is, therefore, erroneous to refer to him or her as a
“winner,” that is, as the “winning candidate,” should he or she obtain the plurality 1 Rollo (Vol. I), pp. 39-46.
of votes. Consequently, it is illogical to refer to the candidates who are trailing in 2 Id., at pp. 49-50.
the vote count as “losers,” which is what labels like “second-placer” entail. As his 3 Id., at pp. 395-414.
or her ineligibility as a candidate remains, the number of votes cast for him or her 241
is ultimately not decisive of who must be proclaimed as winner: The ballot cannot
VOL. 747, JANUARY 21, 2015 241
override the constitutional and statutory requirements for qualifications and
disqualifications of candidates. When the law requires certain qualifications to be Risos-Vidal vs. Commission on Elections
possessed or that certain disqualifications be not possessed by persons desiring to
26558, entitled “People of the Philippines v. Joseph Ejercito Estrada, et al.” The MALACAÑAN PALACE
dispositive part of the graft court’s decision reads: MANILA
WHEREFORE, in view of all the foregoing, judgment is hereby rendered in
Criminal Case No. 26558 finding the accused, Former President Joseph Ejercito
Estrada, GUILTY beyond reasonable doubt of the crime of PLUNDER, defined in
and penalized by Republic Act No. 7080, as amended. On the other hand, for failure
of the prosecution to prove and establish their guilt beyond reasonable doubt, the
Court finds the accused Jose “Jinggoy” Estrada and Atty. Edward S. Serapio NOT
GUILTY of the crime of plunder, and accordingly, the Court hereby orders
their ACQUITTAL. WHEREAS, this Administration has a policy of releasing inmates who have
The penalty imposable for the crime of plunder under Republic Act No. 7080, reached the age of seventy (70),
as amended by Republic Act No. 7659, is Reclusion Perpetua to Death. There being _______________
no aggravating or mitigating circumstances, however, the lesser penalty shall be
applied in accordance with Article 63 of the Revised Penal Code. Accordingly, the 4 Id., at pp. 260-262.
accused Former President Joseph Ejercito Estrada is hereby sentenced to suffer 243
the penalty of Reclusion Perpetua and the accessory penalties of civil VOL. 747, JANUARY 21, 2015 243
interdiction during the period of sentence and perpetual absolute disqualification. Risos-Vidal vs. Commission on Elections
The period within which accused Former President Joseph Ejercito Estrada
WHEREAS, Joseph Ejercito Estrada has been under detention for six and a
has been under detention shall be credited to him in full as long as he agrees
half years,
voluntarily in writing to abide by the same disciplinary rules imposed upon
WHEREAS, Joseph Ejercito Estrada has publicly committed to no longer seek
convicted prisoners.
any elective position or office,
Moreover, in accordance with Section 2 of Republic Act No. 7080, as amended
IN VIEW HEREOF and pursuant to the authority conferred upon me by the
by Republic Act No. 7659, the Court hereby declares the forfeiture in favor of the
Constitution, I hereby grant executive clemency to JOSEPH EJERCITO
government of the following:
ESTRADA, convicted by the Sandiganbayan of Plunder and imposed a penalty of
(1) The total amount of Five Hundred Forty[-]Two Million Seven Hundred
Reclusion Perpetua. He is hereby restored to his civil and political rights.
Ninety[-] One Thousand Pesos (P545,291,000.00), with interest and income
The forfeitures imposed by the Sandiganbayan remain in force and in full,
earned, inclusive of the amount of Two Hundred Million Pesos
including all writs and processes issued by the Sandiganbayan in pursuance hereof,
242
except for the bank account(s) he owned before his tenure as President.
242 SUPREME COURT REPORTS ANNOTATED Upon acceptance of this pardon by JOSEPH EJERCITO ESTRADA, this
Risos-Vidal vs. Commission on Elections pardon shall take effect.
(P200,000,000.00), deposited in the name and account of the Erap Muslim Given under my hand at the City of Manila, this 25th Day of October, in the
Youth Foundation. year of Our Lord, two thousand and seven.
(2) The amount of One Hundred Eighty[-] Gloria M. Arroyo (sgd.)
Nine Million Pesos (P189,000,000.00), inclusive of interests and income earned, By the President:
deposited in the Jose Velarde account. IGNACIO R. BUNYE (sgd.)
(3) The real property consisting of a house and lot dubbed as “Boracay Acting Executive Secretary5
Mansion” located at #100 11th Street, New Manila, Quezon City.
The cash bonds posted by accused Jose “Jinggoy” Estrada and Atty. Edward S. On October 26, 2007, at 3:35 p.m., former President Estrada “received and
Serapio are hereby ordered cancelled and released to the said accused or their duly accepted”6 the pardon by affixing his signature beside his handwritten notation
authorized representatives upon presentation of the original receipt evidencing thereon.
payment thereof and subject to the usual accounting and auditing procedures. On November 30, 2009, former President Estrada filed a Certificate of
Likewise, the hold departure orders issued against the said accused are hereby Candidacy7 for the position of President. During
recalled and declared functus oficio.4 _______________

On October 25, 2007, however, former President Gloria Macapagal-Arroyo 5 Id., at p. 265.
(former President Arroyo) extended executive clemency, by way of pardon, to 6 Id.
former President Estrada. The full text of said pardon states: 244
244 SUPREME COURT REPORTS ANNOTATED
Risos-Vidal vs. Commission on Elections (a) Those sentenced by final judgment for an offense involving moral
that time, his candidacy earned three oppositions in the COMELEC: (1) SPA turpitude or for an offense punishable by one (1) year or more of
No. 09-024 (DC), a “Petition to Deny Due Course and Cancel Certificate of imprisonment, within two (2) years after serving sentence;
Candidacy” filed by Rev. Elly Velez B. Lao Pamatong, ESQ; (2) SPA No. 09-028 (b) Those removed from office as a result of an administrative case;
(DC), a petition for “Disqualification as Presidential Candidate” filed by Evilio C. (c) Those convicted by final judgment for violating the oath of allegiance to
Pormento (Pormento); and (3) SPA No. 09-104 (DC), a “Petition to Disqualify the Republic;
Estrada Ejercito, Joseph M. from Running as President due to Constitutional (d) Those with dual citizenship;
Disqualification and Creating Confusion to the Prejudice of Estrada, Mary Lou B” (e) Fugitives from justice in criminal or nonpolitical cases here or abroad;
filed by Mary Lou Estrada. In separate Resolutions 8 dated January 20, 2010 by _______________
the COMELEC, Second Division, however, all three petitions were effectively
dismissed on the uniform grounds that: (i) the Constitutional proscription on 10 Rollo (Vol. I), p. 266.
reelection applies to a sitting president; and (ii) the pardon granted to former 11 Id., at p. 271.
President Estrada by former President Arroyo restored the former’s right to vote 246
and be voted for a public office. The subsequent motions for reconsideration thereto 246 SUPREME COURT REPORTS ANNOTATED
were denied by the COMELEC En Banc. Risos-Vidal vs. Commission on Elections
After the conduct of the May 10, 2010 synchronized elections, however, former
(f) Permanent residents in a foreign country or those who have acquired the
President Estrada only managed to garner the second highest number of votes.
right to reside abroad and continue to avail of the same right after the effectivity
Of the three petitioners above mentioned, only Pormento sought recourse to
of this Code; and
this Court and filed a petition for certiorari, which was docketed as G.R. No.
(g) The insane or feeble-minded. (Emphasis supplied)
191988, entitled “Atty. Evilio C. Pormento v. Joseph ‘ERAP’ Ejercito Estrada and
Sec. 12, Omnibus Election Code:
Commission on Elections.” But in a Resolution9 dated August 31, 2010, the Court
Section 12. Disqualifications.—Any person who has been declared by
dismissed the aforementioned petition on the ground of mootness considering that
competent authority insane or incompetent, or has been sentenced by final
former President Estrada lost his presidential bid.
judgment for subversion, insurrection, rebellion, or for any offense for which he
On October 2, 2012, former President Estrada once more ventured into the
has been sentenced to a penalty of more than eighteen months or for a crime
political arena, and filed a Certificate of
involving moral turpitude, shall be disqualified to be a candidate and to hold
_______________
any public office, unless he has been given plenary pardon or granted
amnesty. (Emphases supplied)
7 Rollo (Vol. II), p. 615.
8 Id., at pp. 509-533 and 534-572. In a Resolution dated April 1, 2013, the COMELEC, Second Division,
9 Pormento v. Estrada, G.R. No. 191988, August 31, 2010, 629 SCRA 530. dismissed the petition for disqualification, the fallo of which reads:
245 WHEREFORE, premises considered, the instant petition is
VOL. 747, JANUARY 21, 2015 245 hereby DISMISSED for utter lack of merit.12
Risos-Vidal vs. Commission on Elections
Candidacy,10 this time vying for a local elective post, that of the Mayor of the The COMELEC, Second Division, opined that “[h]aving taken judicial
City of Manila. cognizance of the consolidated resolution for SPA No. 09-028 (DC) and SPA No. 09-
On January 24, 2013, Risos-Vidal, the petitioner in this case, filed a Petition 104 (DC) and the 10 May 2010 En Banc resolution affirming it, this Commission
for Disqualification against former President Estrada before the COMELEC. The will not belabor the controversy further. More so, [Risos-Vidal] failed to present
petition was docketed as SPA No. 13-211 (DC). Risos-Vidal anchored her petition cogent proof sufficient to reverse the standing pronouncement of this Commission
on the theory that “[Former President Estrada] is Disqualified to Run for Public declaring categorically that [former President Estrada’s] right to seek public office
Office because of his Conviction for Plunder by the Sandiganbayan in Criminal has been effectively restored by the pardon vested upon him by former President
Case No. 26558 entitled ‘People of the Philippines v. Joseph Ejercito Estrada’ Gloria M. Arroyo. Since this Com-
Sentencing Him to Suffer the Penalty of Reclusion Perpetua with Perpetual _______________
Absolute Disqualification.”11 She relied on Section 40 of the Local Government
Code (LGC), in relation to Section 12 of the Omnibus Election Code (OEC), which 12 Id., at p. 43.
state respectively, that: 247
Sec. 40, Local Government Code: VOL. 747, JANUARY 21, 2015 247
SECTION 40. Disqualifications.—The following persons are disqualified
Risos-Vidal vs. Commission on Elections
from running for any elective local position:
mission has already spoken, it will no longer engage in disquisitions of a settled with 349,770 votes cast in his favor. The next day, the local board of canvassers
matter lest indulged in wastage of government resources.”13 proclaimed him as the duly elected Mayor of the City of Manila.
The subsequent motion for reconsideration filed by Risos-Vidal was denied in On June 7, 2013, Lim, one of former President Estrada’s opponents for the
a Resolution dated April 23, 2013. position of Mayor, moved for leave to intervene in this case. His motion was
On April 30, 2013, Risos-Vidal invoked the Court’s jurisdiction by filing the granted by the Court in a Resolution15dated June 25, 2013. Lim subscribed to
present petition. She presented five issues for the Court’s resolution, to wit: Risos-Vidal’s theory that former President Estrada is disqualified to run for and
I. RESPONDENT COMELEC COMMITTED GRAVE ABUSE OF hold public office as the pardon granted to the latter failed to expressly remit his
DISCRETION AMOUNTING TO LACK OR EXCESS OF JURISDICTION IN perpetual disqualification. Further, given that former President Estrada is
HOLDING THAT RESPONDENT ESTRADA’S PARDON WAS NOT disqualified to run for and hold public office, all the votes obtained by the latter
CONDITIONAL; should be declared stray, and, being the second placer with 313,764 votes to his
II. RESPONDENT COMELEC COMMITTED GRAVE ABUSE OF name, he (Lim) should be declared
DISCRETION AMOUNTING TO LACK OR EXCESS OF JURISDICTION IN _______________
NOT FINDING THAT RESPONDENT ESTRADA IS DISQUALIFIED TO RUN
AS MAYOR OF MANILA UNDER SEC. 40 OF THE LOCAL GOVERNMENT 14 Id., at pp. 10-11.
CODE OF 1991 FOR HAVING BEEN CONVICTED OF PLUNDER, AN 15 Id., at p. 438.
OFFENSE INVOLVING MORAL TURPITUDE; 249
III. RESPONDENT COMELEC COMMITTED GRAVE ABUSE OF VOL. 747, JANUARY 21, 2015 249
DISCRETION AMOUNTING TO LACK OR EXCESS OF JURISDICTION IN
DISMISSING THE PETITION FOR DISQUALIFICATION ON THE GROUND Risos-Vidal vs. Commission on Elections
THAT THE CASE INVOLVES THE SAME OR SIMILAR ISSUES IT ALREADY the rightful winning candidate for the position of Mayor of the City of Manila.
RESOLVED IN THE CASES OF “PORMENTO V. ESTRADA,” SPA NO. 09-028
(DC) AND IN “RE: PETITION TO DISQUALIFY ESTRADA EJERCITO, The Issue
JOSEPH M. FROM RUNNING AS PRESIDENT, ETC.,” SPA NO. 09-104 (DC);
IV. RESPONDENT COMELEC COMMITTED GRAVE ABUSE OF Though raising five seemingly separate issues for resolution, the petition filed
DISCRETION AMOUNTING TO by Risos-Vidal actually presents only one essential question for resolution by the
_______________ Court, that is, whether or not the COMELEC committed grave abuse of discretion
amounting to lack or excess of jurisdiction in ruling that former President Estrada
13 Id. is qualified to vote and be voted for in public office as a result of the pardon granted
to him by former President Arroyo.
248 In her petition, Risos-Vidal starts her discussion by pointing out that the
pardon granted to former President Estrada was conditional as evidenced by the
248 SUPREME COURT REPORTS ANNOTATED
latter’s express acceptance thereof. The “acceptance,” she claims, is an indication
Risos-Vidal vs. Commission on Elections of the conditional nature of the pardon, with the condition being embodied in the
LACK OR EXCESS OF JURISDICTION IN NOT RULING THAT third Whereas Clause of the pardon, i.e., “WHEREAS, Joseph Ejercito Estrada has
RESPONDENT ESTRADA’S PARDON NEITHER RESTORED HIS RIGHT OF publicly committed to no longer seek any elective position or office.” She explains
SUFFRAGE NOR REMITTED HIS PERPETUAL ABSOLUTE that the aforementioned commitment was what impelled former President Arroyo
DISQUALIFICATION FROM SEEKING PUBLIC OFFICE; and to pardon former President Estrada, without it, the clemency would not have been
V. RESPONDENT COMELEC COMMITTED GRAVE ABUSE OF extended. And any breach thereof, that is, when former President Estrada filed his
DISCRETION AMOUNTING TO LACK OR EXCESS OF JURISDICTION IN Certificate of Candidacy for President and Mayor of the City of Manila, he
NOT HAVING EXERCISED ITS POWER TO MOTU PROPRIO DISQUALIFY breached the condition of the pardon; hence, “he ought to be recommitted to prison
RESPONDENT ESTRADA IN THE FACE OF HIS PATENT to serve the unexpired portion of his sentence x x x and disqualifies him as a
DISQUALIFICATION TO RUN FOR PUBLIC OFFICE BECAUSE OF HIS candidate for the mayoralty [position] of Manila.”16
PERPETUAL AND ABSOLUTE DISQUALIFICATION TO SEEK PUBLIC Nonetheless, Risos-Vidal clarifies that the fundamental basis upon which
OFFICE AND TO VOTE RESULTING FROM HIS CRIMINAL CONVICTION former President Estrada must be disqualified from running for and holding public
FOR PLUNDER.14 elective office is actually the proscription found in Section 40 of the LGC, in
relation to Section 12 of the OEC. She argues that the crime of plunder is
While this case was pending before the Court, or on May 13, 2013, the elections _______________
were conducted as scheduled and former President Estrada was voted into office
16 Id., at pp. 12-15.
250 condition in the [third] [W]hereas [C]lause of the pardon x x x indubitably
250 SUPREME COURT REPORTS ANNOTATED indicating that the privilege to hold public office was not restored to him.”19
On the other hand, the Office of the Solicitor General (OSG) for public
Risos-Vidal vs. Commission on Elections respondent COMELEC, maintains that “the issue of whether or not the pardon
both an offense punishable by imprisonment of one year or more and involving extended to [former President Estrada] restored his right to run for public office
moral turpitude; such that former President Estrada must be disqualified to run had already been passed upon by public respondent COMELEC way back in
for and hold public elective office. 2010 via its rulings in SPA Nos. 09-024, 09-028 and 09-104, there is no cogent
Even with the pardon granted to former President Estrada, however, Risos- reason for it to reverse its standing pronouncement and declare [former President
Vidal insists that the same did not operate to make available to former President Estrada] disqualified to run and be voted as mayor of the City of Manila in the
Estrada the exception provided under Section 12 of the OEC, the pardon being absence of any new argument that would warrant its reversal. To be sure, public
merely conditional and not absolute or plenary. respondent COMELEC correctly exercised its discretion in taking judicial cogni-
Moreover, Risos-Vidal puts a premium on the ostensible requirements _______________
provided under Articles 36 and 41 of the Revised Penal Code, to wit:
ART. 36. Pardon; its effects.—A pardon shall not work the restoration of the 17 Id., at p. 25.
right to hold public office, or the right of suffrage, unless such rights be 18 252 Phil. 192, 207; 170 SCRA 190, 203-204 (1989).
expressly restored by the terms of the pardon. 19 Rollo (Vol. I), p. 29.
A pardon shall in no case exempt the culprit from the payment of the civil 252
indemnity imposed upon him by the sentence.
xxxx 252 SUPREME COURT REPORTS ANNOTATED
ART. 41. Reclusion perpetua and reclusion temporal — their accessory Risos-Vidal vs. Commission on Elections
penalties.—The penalties of reclusion perpetua and reclusion temporal shall carry zance of the aforesaid rulings which are known to it and which can be verified
with them that of civil interdiction for life or during the period of the sentence as from its own records, in accordance with Section 2, Rule 129 of the Rules of Court
the case may be, and that of perpetual absolute disqualification which the offender on the courts’ discretionary power to take judicial notice of matters which are of
shall suffer even though pardoned as to the principal penalty, unless the public knowledge, or are capable of unquestionable demonstration, or ought to be
same shall have been expressly remitted in the pardon. (Emphases known to them because of their judicial functions.”20
supplied) Further, the OSG contends that “[w]hile at first glance, it is apparent that
[former President Estrada’s] conviction for plunder disqualifies him from running
She avers that in view of the foregoing provisions of law, it is not enough that as mayor of Manila under Section 40 of the [LGC], the subsequent grant of pardon
a pardon makes a general statement that such pardon carries with it the to him, however, effectively restored his right to run for any public office.”21 The
restoration of civil and political rights. By virtue of Articles 36 and 41, a pardon restoration of his right to run for any public office is the exception to the prohibition
restoring civil and political rights without categorically making mention what under Section 40 of the LGC, as provided under Section 12 of the OEC. As to the
specific civil and political rights are restored “shall seeming requirement of Articles 36 and 41 of the Revised Penal Code, i.e., the
251 express restoration/remission of a particular right to be stated in the pardon, the
VOL. 747, JANUARY 21, 2015 251 OSG asserts that “an airtight and rigid interpretation of Article 36 and Article 41
of the [RPC] x x x would be stretching too much the clear and plain meaning of the
Risos-Vidal vs. Commission on Elections aforesaid provisions.”22 Lastly, taking into consideration the third Whereas
not work to restore the right to hold public office, or the right of suffrage; nor Clause of the pardon granted to former President Estrada, the OSG supports the
shall it remit the accessory penalties of civil interdiction and perpetual absolute position that it “is not an integral part of the decree of the pardon and cannot
disqualification for the principal penalties of reclusion perpetua and reclusion therefore serve to restrict its effectivity.”23
temporal.”17 In other words, she considers the above constraints as mandatory Thus, the OSG concludes that the “COMELEC did not commit grave abuse of
requirements that shun a general or implied restoration of civil and political rights discretion amounting to lack or excess of jurisdiction in issuing the assailed
in pardons. Resolutions.”24
Risos-Vidal cites the concurring opinions of Associate Justices Teodoro R. For his part, former President Estrada presents the following significant
Padilla and Florentino P. Feliciano in Monsanto v. Factoran, Jr.18 to endorse her arguments to defend his stay in office: that
position that “[t]he restoration of the right to hold public office to one who has lost _______________
such right by reason of conviction in a criminal case, but subsequently pardoned,
cannot be left to inference, no matter how intensely arguable, but must be stated 20 Rollo (Vol. II), p. 498.
in express, explicit, positive and specific language.” 21 Id., at pp. 498-499.
Applying Monsanto to former President Estrada’s case, Risos-Vidal reckons 22 Id., at p. 502.
that “such express restoration is further demanded by the existence of the
23 Id., at p. 503. It is likewise unfettered by Articles 36 and 41 of the Revised Penal Code. The only
24 Id., at p. 505. reasonable, objective, and constitutional interpretation of the language of the
253 pardon is that the same in fact conforms to Articles 36 and 41 of the Revised Penal
VOL. 747, JANUARY 21, 2015 253 Code.
Recall that the petition for disqualification filed by Risos-Vidal against former
Risos-Vidal vs. Commission on Elections President Estrada, docketed as SPA No. 13-211 (DC), was anchored on Section 40
“the factual findings of public respondent COMELEC, the Constitutional body of the LGC, in relation to Section 12 of the OEC, that is, having been convicted of
mandated to administer and enforce all laws relative to the conduct of the a crime punishable by imprisonment of one year or more, and involving moral
elections, [relative to the absoluteness of the pardon, the effects thereof, and the turpitude, former President Estrada must be disqualified to run for and hold public
eligibility of former President Estrada to seek public elective office] are binding elective office notwithstanding the fact that he is a grantee of a pardon that
[and conclusive] on this Honorable Supreme Court”; that he “was granted an includes a statement expressing “[h]e is hereby restored to his civil and political
absolute pardon and thereby restored to his full civil and political rights, including rights.”
the right to seek public elective office such as the mayoral (sic) position in the City Risos-Vidal theorizes that former President Estrada is disqualified from
of Manila”; that “the majority decision in the case of Salvacion A. Monsanto v. running for Mayor of Manila in the May 13, 2013 Elections, and remains
Fulgencio S. Factoran, Jr., which was erroneously cited by both Vidal and Lim as disqualified to hold any local elective post despite the presidential pardon extended
authority for their respective claims, x x x reveal that there was no discussion to him in 2007 by former President Arroyo for the reason that it (pardon) did not
whatsoever in the ratio decidendi of the Monsanto case as to the alleged necessity expressly provide for the remission of the penalty of perpetual absolute
for an expressed restoration of the ‘right to hold public office in the pardon’ as a disqualification, particularly the restoration of his (former President Estrada)
legal prerequisite to remove the subject perpetual special disqualification”; that right to vote and
moreover, the “principal question raised in this Monsanto case is whether or not a _______________
public officer, who has been granted an absolute pardon by the Chief Executive, is
entitled to reinstatement to her former position without need of a new 26 Id., at p. 607.
appointment”; that his “expressed acceptance [of the pardon] is not proof that the 255
pardon extended to [him] is conditional and not absolute”; that this case is a mere
rehash of the cases filed against him during his candidacy for President back in VOL. 747, JANUARY 21, 2015 255
2009-2010; that Articles 36 and 41 of the Revised Penal Code “cannot abridge or Risos-Vidal vs. Commission on Elections
diminish the pardoning power of the President expressly granted by the be voted upon for public office. She invokes Articles 36 and 41 of the Revised
Constitution”; that the text of the pardon granted to him substantially, if not fully, Penal Code as the foundations of her theory.
complied with the requirement posed by Article 36 of the Revised Penal Code as it It is insisted that, since a textual examination of the pardon given to and
was categorically stated in the said document that he was “restored to his civil and accepted by former President Estrada does not actually specify which political
political rights”; that since pardon is an act of grace, it must be construed favorably right is restored, it could be inferred that former President Arroyo did not
in favor of the grantee;25 and that his dis- deliberately intend to restore former President Estrada’s rights of suffrage and to
_______________ hold public office, or to otherwise remit the penalty of perpetual absolute
disqualification. Even if her intention was the contrary, the same cannot be upheld
25 Id., at pp. 582-596. based on the pardon’s text.
254
254 SUPREME COURT REPORTS ANNOTATED The pardoning power of the President cannot be limited by legislative
action.
Risos-Vidal vs. Commission on Elections
qualification will result in massive disenfranchisement of the hundreds of The 1987 Constitution, specifically Section 19 of Article VII and Section 5 of
thousands of Manileños who voted for him.26 Article IX-C, provides that the President of the Philippines possesses the power to
grant pardons, along with other acts of executive clemency, to wit:
Section 19. Except in cases of impeachment, or as otherwise provided in this
The Court’s Ruling Constitution, the President may grant reprieves, commutations, and pardons, and
remit fines and forfeitures, after conviction by final judgment.
The petition for certiorari lacks merit. He shall also have the power to grant amnesty with the concurrence of a
Former President Estrada was granted an absolute pardon that fully majority of all the Members of the Congress.
restored all his civil and political rights, which naturally includes the right to seek xxxx
public elective office, the focal point of this controversy. The wording of the pardon
extended to former President Estrada is complete, unambiguous, and unqualified.
Section 5. No pardon, amnesty, parole, or suspension of sentence for I suggest that this be deleted on the grounds that, first, violations of corrupt
violation of election laws, rules, and regulations shall be granted by the President practices may include a very little offense like stealing P10; second, which I think
without the favorable recommendation of the Commission. is more important, I get the impression, rightly or wrongly, that subconsciously we
are drafting a constitution on the premise that all our future Presidents will be
It is apparent from the foregoing constitutional provisions that the only bad and dishonest and, consequently, their acts will be lacking in wisdom.
instances in which the President may not extend Therefore, this Article seems to contribute towards the creation of an anti-
256 President Constitution or a President with vast responsibilities but no
256 SUPREME COURT REPORTS ANNOTATED corresponding power except to declare martial law. Therefore, I request that
these lines be deleted.
Risos-Vidal vs. Commission on Elections MR. REGALADO. Madam President, may the Committee react to that?
pardon remain to be in: (1) impeachment cases; (2) cases that have not yet THE PRESIDENT. Yes, please.
resulted in a final conviction; and (3) cases involving violations of election laws, MR. REGALADO. This was inserted here on the resolution of Commissioner
rules and regulations in which there was no favorable recommendation coming Davide because of the fact that similar to the provisions on the Commission on
from the COMELEC. Therefore, it can be argued that any act of Congress by way Elections, the recommendation of that Commission is required before executive
of statute cannot operate to delimit the pardoning power of the President. clemency is granted because violations of the election laws go into the very political
In Cristobal v. Labrador27 and Pelobello v. Palatino,28 which were decided life of the country.
under the 1935 Constitution, wherein the provision granting pardoning power to With respect to violations of our Corrupt Practices Law, we felt that it is also
the President shared similar phraseology with what is found in the present 1987 necessary to have that subjected to the same condition because violation of our
Constitution, the Court then unequivocally declared that “subject to the Corrupt Practices Law may be of such magnitude as to affect the very economic
limitations imposed by the Constitution, the pardoning power cannot be restricted system of the country. Nevertheless, as a compromise, we provided here that it will
or controlled by legislative action.” The Court reiterated this pronouncement be the Congress that will provide for the classification as to which
in Monsanto v. Factoran, Jr.29 thereby establishing that, under the present 258
Constitution, “a pardon, being a presidential prerogative, should not be
258 SUPREME COURT REPORTS ANNOTATED
circumscribed by legislative action.” Thus, it is unmistakably the long-standing
position of this Court that the exercise of the pardoning power is discretionary in Risos-Vidal vs. Commission on Elections
the President and may not be interfered with by Congress or the Court, except only
when it exceeds the limits provided for by the Constitution. convictions will still require prior recommendation; after all, the Congress
This doctrine of non-diminution or non-impairment of the President’s power of could take into account whether or not the violation of the Corrupt Practices Law
pardon by acts of Congress, specifically through legislation, was strongly adhered is of such magnitude as to affect the economic life of the country, if it is in the
to by an overwhelming majority of the framers of the 1987 Constitution when they millions or billions of dollars. But I assume the Congress in its collective wisdom
flatly rejected a proposal to carve out an exception from the pardoning power of will exclude those petty crimes of corruption as not to require any further stricture
the President in the form of “offenses involving graft and corruption” that would on the exercise of executive clemency because, of course, there is a whale of a
be enumerated and defined by Congress through the enactment of a law. The difference if we consider a lowly clerk committing malversation of government
following is the pertinent portion lifted from the Record of the Commission (Vol. property or funds involving one hundred pesos. But then, we also anticipate the
II): possibility that the corrupt practice of a public officer is of such magnitude as to
_______________ have virtually drained a substantial portion of the treasury, and then he goes
through all the judicial processes and later on, a President who may have close
27 71 Phil. 34, 38 (1940). connections with him or out of improvident compassion may grant clemency under
28 72 Phil. 441, 442 (1941). such conditions. That is why we left it to Congress to provide and make a
29 Monsanto v. Factoran, Jr., supra note 18 at p. 202; p. 199. classification based on substantial distinctions between a minor act of corruption
257 or an act of substantial proportions.
SR. TAN. So, why do we not just insert the word GROSS or GRAVE before the
VOL. 747, JANUARY 21, 2015 257
word “violations?”
Risos-Vidal vs. Commission on Elections MR. REGALADO. We feel that Congress can make a better distinction because
MR. ROMULO. I ask that Commissioner Tan be recognized to introduce an “GRAVE” or “GROSS” can be misconstrued by putting it purely as a policy.
amendment on the same section. MR. RODRIGO. Madam President.
THE PRESIDENT. Commissioner Tan is recognized. THE PRESIDENT. Commissioner Rodrigo is recognized.
SR. TAN. Madam President, lines 7 to 9 state: MR. RODRIGO. May I speak in favor of the proposed amendment?
However, the power to grant executive clemency for violations of THE PRESIDENT. Please proceed.
corrupt practices laws may be limited by legislation.
MR. RODRIGO. The power to grant executive clemency is essentially an is now the best time, since we have strengthened the Article on Accountability of
executive power, and that is precisely why it is called executive clemency. In this Public Officers, to accompany it with a mandate that the President’s right to grant
sentence, which the amendment seeks to delete, an exception is being executive clemency for offenders or violators of laws relating to the concept of a
made. Congress, which is the legislative arm, is allowed to intrude into public office may be limited by Congress itself.
this prerogative of the executive. Then it limits the power of MR. SARMIENTO. Madam President.
259 THE PRESIDENT. Commissioner Sarmiento is recognized.
VOL. 747, JANUARY 21, 2015 259 MR. SARMIENTO. May I briefly speak in favor of the amendment by deletion.
Madam President, over and over again, we have been saying and arguing
Risos-Vidal vs. Commission on Elections before this Constitutional Commission that we are emasculating the powers
Congress to subtract from this prerogative of the President to grant executive of the presidency, and this provision to me is another clear example of
clemency by limiting the power of Congress to only corrupt practices laws. There that. So, I speak against this provision. Even the 1935 and the 1973 Constitutions
are many other crimes more serious than these. Under this amendment, Congress do not provide for this kind of provision.
cannot limit the power of executive clemency in cases of drug addiction and drug I am supporting the amendment by deletion of Commissioner Tan.
pushing which are very, very serious crimes that can endanger the State; also, rape MR. ROMULO. Commissioner Tingson would like to be recognized.
with murder, kidnapping and treason. Aside from the fact that it is a THE PRESIDENT. Commissioner Tingson is recognized.
derogation of the power of the President to grant executive clemency, it MR. TINGSON. Madam President, I am also in favor of the amendment by
is also defective in that it singles out just one kind of crime. There are far deletion because I am in sympathy with the stand of Commissioner Francisco “Soc”
more serious crimes which are not included. Rodrigo. I do believe and we should remember that above all the elected or
MR. REGALADO. I will just make one observation on that. We admit that the appointed officers of our Republic, the
pardoning power is an executive power. But even in the provisions on the 261
COMELEC, one will notice that constitutionally, it is required that there be a
VOL. 747, JANUARY 21, 2015 261
favorable recommendation by the Commission on Elections for any violation of
election laws. Risos-Vidal vs. Commission on Elections
At any rate, Commissioner Davide, as the principal proponent of that and as a leader is the President. I believe that the country will be as the President
member of the Committee, has explained in the committee meetings we had why is, and if we systematically emasculate the power of this presidency, the
he sought the inclusion of this particular provision. May we call on Commissioner time may come when he will be also handcuffed that he will no longer be
Davide to state his position. able to act like he should be acting.
MR. DAVIDE. Madam President. So, Madam President, I am in favor of the deletion of this particular line.
THE PRESIDENT. Commissioner Davide is recognized. MR. ROMULO. Commissioner Colayco would like to be recognized.
MR. DAVIDE. I am constrained to rise to object to the proposal. We have just THE PRESIDENT. Commissioner Colayco is recognized.
approved the Article on Accountability of Public Officers. Under it, it is mandated MR. COLAYCO. Thank you very much, Madam President.
that a public office is a public trust, and all government officers are under I seldom rise here to object to or to commend or to recommend the approval of
obligation to observe the utmost of responsibility, integrity, loyalty and efficiency, proposals, but now I find that the proposal of Commissioner Tan is worthy of
to lead modest lives and to act with patriotism and justice. approval of this body.
In all cases, therefore, which would go into the very core of the concept that a Why are we singling out this particular offense? There are other crimes which
public office is a public trust, the violation is itself a violation not only of the cast a bigger blot on the moral character of the public officials.
economy but the moral fabric of public officials. And that is the reason we now Finally, this body should not be the first one to limit the almost
want that if there is any conviction for the absolute power of our Chief Executive in deciding whether to pardon, to
260 reprieve or to commute the sentence rendered by the court.
260 SUPREME COURT REPORTS ANNOTATED I thank you.
THE PRESIDENT. Are we ready to vote now?
Risos-Vidal vs. Commission on Elections MR. ROMULO. Commissioner Padilla would like to be recognized, and after
him will be Commissioner Natividad.
violation of the Anti-Graft and Corrupt Practices Act, which, in effect, is a THE PRESIDENT. Commissioner Padilla is recognized.
violation of the public trust character of the public office, no pardon shall be MR. PADILLA. Only one sentence, Madam President. The Sandiganbayan has
extended to the offender, unless some limitations are imposed. been called the Anti-Graft Court, so if this is allowed to stay, it would mean that
Originally, my limitation was, it should be with the concurrence of the the President’s power to grant pardon or reprieve will be limited to the cases
convicting court, but the Committee left it entirely to the legislature to formulate decided by the Anti-Graft Court, when as already stated, there are many
the mechanics at trying, probably, to distinguish between grave and less grave or provisions in the
serious cases of violation of the Anti-Graft and Corrupt Practices Act. Perhaps this 262
262 SUPREME COURT REPORTS ANNOTATED to free the beneficiary of presidential grace from the disqualifications specifically
prescribed by them.
Risos-Vidal vs. Commission on Elections Again, Articles 36 and 41 of the Revised Penal Code provides:
ART. 36. Pardon; its effects. – A pardon shall not work the restoration of the
Revised Penal Code that penalize more serious offenses. right to hold public office, or the right of suffrage, unless such rights be
Moreover, when there is a judgment of conviction and the case merits the expressly restored by the terms of the pardon.
consideration of the exercise of executive clemency, usually under Article V of the _______________
Revised Penal Code the judge will recommend such exercise of clemency. And so,
I am in favor of the amendment proposed by Commissioner Tan for the deletion of 30 Records of the Constitutional Commission of 1986 (Vol. II), July 31, 1986,
this last sentence in Section 17. pp. 524-526.
THE PRESIDENT. Are we ready to vote now, Mr. Floor Leader? 264
MR. NATIVIDAD. Just one more.
THE PRESIDENT. Commissioner Natividad is recognized. 264 SUPREME COURT REPORTS ANNOTATED
MR. NATIVIDAD. I am also against this provision which will again chip more Risos-Vidal vs. Commission on Elections
powers from the President. In case of other criminals convicted in our society, we A pardon shall in no case exempt the culprit from the payment of the civil
extend probation to them while in this case, they have already been convicted and indemnity imposed upon him by the sentence.
we offer mercy. The only way we can offer mercy to them is through this executive xxxx
clemency extended to them by the President. If we still close this avenue to ART. 41. Reclusion perpetua and reclusion temporal — their accessory
them, they would be prejudiced even worse than the murderers and the penalties.—The penalties of reclusion perpetua and reclusion temporal shall carry
more vicious killers in our society. I do not think they deserve this opprobrium with them that of civil interdiction for life or during the period of the sentence as
and punishment under the new Constitution. the case may be, and that of perpetual absolute disqualification which the offender
I am in favor of the proposed amendment of Commissioner Tan. shall suffer even though pardoned as to the principal penalty, unless the
MR. ROMULO. We are ready to vote, Madam President. same shall have been expressly remitted in the pardon. (Emphases
THE PRESIDENT. Is this accepted by the Committee? supplied)
MR. REGALADO. The Committee, Madam President, prefers to submit this to
the floor and also because of the objection of the main proponent, Commissioner A rigid and inflexible reading of the above provisions of law, as proposed by
Davide. So we feel that the Commissioners should vote on this question. Risos-Vidal, is unwarranted, especially so if it will defeat or unduly restrict the
263 power of the President to grant executive clemency.
VOL. 747, JANUARY 21, 2015 263 It is well-entrenched in this jurisdiction that where the words of a statute are
clear, plain, and free from ambiguity, it must be given its literal meaning and
Risos-Vidal vs. Commission on Elections applied without attempted interpretation. Verba legis non est recedendum. From
VOTING the words of a statute there should be no departure.31 It is this Court’s firm view
THE PRESIDENT. As many as are in favor of the proposed amendment of that the phrase in the presidential pardon at issue which declares that former
Commissioner Tan to delete the last sentence of Section 17 appearing on lines 7, 8 President Estrada “is hereby restored to his civil and political rights” substantially
and 9, please raise their hand. (Several Members raised their hand) complies with the requirement of express restoration.
As many as are against, please raise their hand. (Few Members raised their The Dissent of Justice Marvic M.V.F. Leonen agreed with Risos-Vidal that
hand) there was no express remission and/or restoration of the rights of suffrage and/or
The results show 34 votes in favor and 4 votes against; the amendment to hold public office in the pardon granted to former President Estrada, as required
is approved.30 (Emphases supplied) by Articles 36 and 41 of the Revised Penal Code.
Justice Leonen posits in his Dissent that the aforementioned codal provisions
The proper interpretation of Articles must be followed by the President, as
36 and 41 of the Revised Penal Code. _______________

The foregoing pronouncements solidify the thesis that Articles 36 and 41 of the
31 Republic v. Camacho, G.R. No. 185604, June 13, 2013, 698 SCRA 380, 398.
Revised Penal Code cannot, in any way, serve to abridge or diminish the exclusive
265
power and prerogative of the President to pardon persons convicted of violating
penal statutes. VOL. 747, JANUARY 21, 2015 265
The Court cannot subscribe to Risos-Vidal’s interpretation that the said Risos-Vidal vs. Commission on Elections
Articles contain specific textual commands which must be strictly followed in order
they do not abridge or diminish the President’s power to extend clemency. He and, specifically, to decide to pardon the principal penalty while excluding its
opines that they do not reduce the coverage of the President’s pardoning power. accessory penalties or to pardon both. Thus, Articles 36 and 41 only clarify the
Particularly, he states: effect of the pardon so decided upon by the President on the penalties imposed in
Articles 36 and 41 refer only to requirements of convention or form. They only accordance with law.
provide a procedural prescription. They are not concerned with areas where or the A close scrutiny of the text of the pardon extended to former President Estrada
instances when the President may grant pardon; they are only concerned shows that both the principal penalty of reclusion perpetua and its accessory
with how he or she is to exercise such power so that no other governmental penalties are included in the pardon. The first sentence refers to the executive
instrumentality needs to intervene to give it full effect. clemency extended to former President Estrada who was convicted by
All that Articles 36 and 41 do is prescribe that, if the President wishes to the Sandiganbayan of plunder and imposed a penalty of reclusion perpetua. The
include in the pardon the restoration of the rights of suffrage and to hold public latter is the principal penalty pardoned which relieved him of imprisonment. The
office, or the remission of the accessory penalty of perpetual absolute sentence that followed, which states that “(h)e is hereby restored to his civil and
disqualification, he or she should do so expressly. Articles 36 and 41 only ask that political rights,” expressly remitted the accessory penalties that attached to the
the President state his or her intentions clearly, directly, firmly, precisely, and principal penalty of reclusion perpetua. Hence, even if we apply Articles 36 and 41
unmistakably. To belabor the point, the President retains the power to make such of the Revised Penal Code, it is indubitable from the text of the pardon
restoration or remission, subject to a prescription on the manner by which he or 267
she is to state it.32 VOL. 747, JANUARY 21, 2015 267

With due respect, I disagree with the overbroad statement that Congress may Risos-Vidal vs. Commission on Elections
dictate as to how the President may exercise his/her power of executive clemency. that the accessory penalties of civil interdiction and perpetual absolute
The form or manner by which the President, or Congress for that matter, should disqualification were expressly remitted together with the principal penalty
exercise their respective Constitutional powers or prerogatives cannot be of reclusion perpetua.
interfered with unless it is so provided in the Constitution. This is the essence of In this jurisdiction, the right to seek public elective office is recognized by law
the principle of separation of powers deeply ingrained in our system of government as falling under the whole gamut of civil and political rights.
which “ordains that each of the three great branches of government has exclusive Section 5 of Republic Act No. 9225,34 otherwise known as the “Citizenship
cognizance of and is supreme in matters falling within its own constitutionally Retention and Reacquisition Act of 2003,” reads as follows:
allocated sphere.”33 Section 5. Civil and Political Rights and Liabilities.—Those who retain or
_______________ reacquire Philippine citizenship under this Act shall enjoy full civil and political
rights and be subject to all attendant liabilities and responsibilities under existing
32 Dissenting Opinion (Justice Marvic M.V.F. Leonen), pp. 440-441. laws of the Philippines and the following conditions:
33 Bureau of Customs Employees Association (BOCEA) v. Teves, G.R. No. (1) Those intending to exercise their right of suffrage must meet the
181704, December 6, 2011, 661 SCRA 589, 604. requirements under Section 1, Article V of the Constitution, Republic Act No. 9189,
266 otherwise known as “The Overseas Absentee Voting Act of 2003” and other existing
laws;
266 SUPREME COURT REPORTS ANNOTATED (2) Those seeking elective public office in the Philippines shall meet
Risos-Vidal vs. Commission on Elections the qualifications for holding such public office as required by the Constitution and
More so, this fundamental principle must be observed if noncompliance with existing laws and, at the time of the filing of the certificate of candidacy, make a
the form imposed by one branch on a coequal and coordinate branch will result personal and sworn renunciation of any and all foreign citizenship before any
into the diminution of an exclusive Constitutional prerogative. public officer authorized to administer an oath;
For this reason, Articles 36 and 41 of the Revised Penal Code should be (3) Those appointed to any public office shall subscribe and swear an oath of
construed in a way that will give full effect to the executive clemency granted by allegiance to the Republic of the Philippines and its duly constituted authorities
the President, instead of indulging in an overly strict interpretation that may serve prior to their assumption of office: Provided, That they re-
to impair or diminish the import of the pardon which emanated from the Office of _______________
the President and duly signed by the Chief Executive himself/herself. The said
codal provisions must be construed to harmonize the power of Congress to define 34 An Act Making the Citizenship of Philippine Citizens who Acquire Foreign
crimes and prescribe the penalties for such crimes and the power of the President Citizenship Permanent, Amending for the Purpose Commonwealth Act No. 63, as
to grant executive clemency. All that the said provisions impart is that the pardon Amended, and for Other Purposes.
of the principal penalty does not carry with it the remission of the accessory 268
penalties unless the President expressly includes said accessory penalties in the 268 SUPREME COURT REPORTS ANNOTATED
pardon. It still recognizes the Presidential prerogative to grant executive clemency
Risos-Vidal vs. Commission on Elections
nounce their oath of allegiance to the country where they took that oath; On the other hand, the theory of Risos-Vidal goes beyond the plain meaning of
(4) Those intending to practice their profession in the Philippines shall apply said penal provisions; and prescribes a formal requirement that is not only
with the proper authority for a license or permit to engage in such practice; and unnecessary but, if insisted upon, could be in derogation of the constitutional
(5) That right to vote or be elected or appointed to any public office in the prohibition relative to the principle that the exercise of presidential pardon cannot
Philippines cannot be exercised by, or extended to, those who: be affected by legislative action.
(a) are candidates for or are occupying any public office in the country of Risos-Vidal relied heavily on the separate concurring opinions in Monsanto v.
which they are naturalized citizens; and/or Factoran, Jr.36 to justify her argument that an absolute pardon must expressly
(b) are in active service as commissioned or noncommissioned officers in the state that the right to hold public office has been restored, and that the penalty of
armed forces of the country which they are naturalized citizens. (Emphases perpetual absolute disqualification has been remitted.
supplied) This is incorrect.
Her reliance on said opinions is utterly misplaced. Although the learned views
No less than the International Covenant on Civil and Political Rights, to which of Justices Teodoro R. Padilla and
the Philippines is a signatory, acknowledges the existence of said right. Article _______________
25(b) of the Convention states:
Article 25 36 Supra note 18.
Every citizen shall have the right and the opportunity, without any of the 270
distinctions mentioned in Article 2 and without unreasonable restrictions:
270 SUPREME COURT REPORTS ANNOTATED
xxxx
(b) To vote and to be elected at genuine periodic elections which shall be Risos-Vidal vs. Commission on Elections
by universal and equal suffrage and shall be held by secret ballot, guaranteeing Florentino P. Feliciano are to be respected, they do not form part of the
the free expression of the will of the electors[.] (Emphasis supplied) controlling doctrine nor to be considered part of the law of the land. On the
contrary, a careful reading of the majority opinion in Monsanto, penned by no less
Recently, in Sobejana-Condon v. Commission on Elections,35 the Court than Chief Justice Marcelo B. Fernan, reveals no statement that denotes
unequivocally referred to the right to seek public elective office as a political right, adherence to a stringent and overly nuanced application of Articles 36 and 41 of
to wit: the Revised Penal Code that will in effect require the President to use a statutorily
_______________ prescribed language in extending executive clemency, even if the intent of the
President can otherwise be deduced from the text or words used in the pardon.
35 G.R. No. 198742, August 10, 2012, 678 SCRA 267, 292. Furthermore, as explained above, the pardon here is consistent with, and not
269 contrary to, the provisions of Articles 36 and 41.
VOL. 747, JANUARY 21, 2015 269
The disqualification of former President Estrada under Section 40 of the
Risos-Vidal vs. Commission on Elections LGC in relation to Section 12 of the OEC was removed by his acceptance of
Stated differently, it is an additional qualification for elective office specific the absolute pardon granted to him.
only to Filipino citizens who reacquire their citizenship under Section 3 of R.A. No.
9225. It is the operative act that restores their right to run for public office. The Section 40 of the LGC identifies who are disqualified from running for any
petitioner’s failure to comply therewith in accordance with the exact tenor of the elective local position. Risos-Vidal argues that former President Estrada is
law, rendered ineffectual the Declaration of Renunciation of Australian disqualified under item (a), to wit:
Citizenship she executed on September 18, 2006. As such, she is yet to regain (a) Those sentenced by final judgment for an offense involving moral
her political right to seek elective office. Unless she executes a sworn turpitude or for an offense punishable by one (1) year or more of
renunciation of her Australian citizenship, she is ineligible to run for and hold any imprisonment, within two (2) years after serving sentence[.] (Emphasis
elective office in the Philippines. (Emphasis supplied) supplied)

Thus, from both law and jurisprudence, the right to seek public elective office Likewise, Section 12 of the OEC provides for similar prohibitions, but it
is unequivocally considered as a political right. Hence, the Court reiterates its provides for an exception, to wit:
earlier statement that the pardon granted to former President Estrada admits no Section 12. Disqualifications.—x x x unless he has been given plenary
other interpretation other than to mean that, upon acceptance of the pardon pardon or granted amnesty. (Emphasis supplied)
granted to him, he regained his FULL civil and political rights — including the 271
right to seek elective office. VOL. 747, JANUARY 21, 2015 271
Risos-Vidal vs. Commission on Elections
As earlier stated, Risos-Vidal maintains that former President Estrada’s Jurisprudence educates that a preamble is not an essential part of an act as it
conviction for plunder disqualifies him from running for the elective local position is an introductory or preparatory clause that explains the reasons for the
of Mayor of the City of Manila under Section 40(a) of the LGC. However, the enactment, usually introduced by the word “whereas.”40 Whereas clauses do not
subsequent absolute pardon granted to former President Estrada effectively form part of a statute because, strictly speaking, they are not part of the operative
restored his right to seek public elective office. This is made possible by reading language of the statute.41 In this case, the whereas clause at issue is not an
Section 40(a) of the LGC in relation to Section 12 of the OEC. integral part of the decree of the pardon, and therefore, does not by itself alone
While it may be apparent that the proscription in Section 40(a) of the LGC is operate to make the pardon conditional or to make its effectivity contingent upon
worded in absolute terms, Section 12 of the OEC provides a legal escape from the the fulfilment of the aforementioned commitment nor to limit the scope of the
prohibition — a plenary pardon or amnesty. In other words, the latter provision pardon.
allows any person who has been granted plenary pardon or amnesty after _______________
conviction by final judgment of an offense involving moral turpitude, inter alia, to
run for and hold any public office, whether local or national position. 39 Jalosjos, Jr. v. Commission on Elections, supra note 37 at pp. 30-31.
Take notice that the applicability of Section 12 of the OEC to candidates 40 People v. Balasa, 356 Phil. 362, 396; 295 SCRA 49, 85 (1998).
running for local elective positions is not unprecedented. In Jalosjos, Jr. v. 41 Llamado v. Court of Appeals, 256 Phil. 328, 339; 174 SCRA 566, 576 (1989).
Commission on Elections,37 the Court acknowledged the aforementioned provision 273
as one of the legal remedies that may be availed of to disqualify a candidate in a VOL. 747, JANUARY 21, 2015 273
local election filed any day after the last day for filing of certificates of candidacy,
but not later than the date of proclamation. 38 The pertinent ruling in Risos-Vidal vs. Commission on Elections
the Jalosjos case is quoted as follows: On this matter, the Court quotes with approval a relevant excerpt of
What is indisputably clear is that false material representation of Jalosjos is a COMELEC Commissioner Maria Gracia Padaca’s separate concurring opinion in
ground for a petition under Section 78. However, since the false material the assailed April 1, 2013 Resolution of the COMELEC in SPA No. 13-211 (DC),
representation arises from a crime penalized by prisión mayor, a petition under which captured the essence of the legal effect of preambular paragraphs/whereas
Section 12 of the Omnibus Election Code or Section 40 of the Local Government clauses, viz.:
Code can also be properly filed. The petitioner has a choice whether The present dispute does not raise anything which the 20 January 2010
_______________ Resolution did not conclude upon. Here, Petitioner Risos-Vidal raised the same
argument with respect to the 3rd “whereas clause” or preambular paragraph of the
37 G.R. Nos. 193237 and 193536, October 9, 2012, 683 SCRA 1. decree of pardon. It states that “Joseph Ejercito Estrada has publicly committed to
38 Commission on Elections Resolution No. 9523, Rule 25, Section 3. no longer seek any elective position or office.” On this contention, the undersigned
272 reiterates the ruling of the Commission that the 3rd preambular paragraph does
not have any legal or binding effect on the absolute nature of the pardon extended
272 SUPREME COURT REPORTS ANNOTATED
by former President Arroyo to herein Respondent.
Risos-Vidal vs. Commission on Elections This ruling is consistent with the traditional and customary usage of
preambular paragraphs. In the case of Echegaray v. Secretary of Justice, the
to anchor his petition on Section 12 or Section 78 of the Omnibus Supreme Court ruled on the legal effect of preambular paragraphs or whereas
Election Code, or on Section 40 of the Local Government Code. The law clauses on statutes. The Court stated, viz.:
expressly provides multiple remedies and the choice of which remedy to Besides, a preamble is really not an integral part of a law. It is merely an
adopt belongs to petitioner.39 (Emphasis supplied) introduction to show its intent or purposes. It cannot be the origin of rights and
obligations. Where the meaning of a statute is clear and unambiguous,
The third preambular clause of the pardon did not operate to make the the preamble can neither expand nor restrict its operation much less
pardon conditional. prevail over its text.
If former President Arroyo intended for the pardon to be conditional on
Contrary to Risos-Vidal’s declaration, the third preambular clause of the Respondent’s promise never to seek a public office again, the former ought to have
pardon, i.e., “[w]hereas, Joseph Ejercito Estrada has publicly committed to no explicitly stated the same in the text of the pardon itself. Since former President
longer seek any elective position or office,” neither makes the pardon conditional, Arroyo did not make this an integral part of the decree of pardon, the Commission
nor militate against the conclusion that former President Estrada’s rights to is constrained to rule that the 3rd preambular clause cannot be
suffrage and to seek public elective office have been restored. This is especially 274
true as the pardon itself does not explicitly impose a condition or limitation, 274 SUPREME COURT REPORTS ANNOTATED
considering the unqualified use of the term “civil and political rights” as being
restored. Risos-Vidal vs. Commission on Elections
interpreted as a condition to the pardon extended to former President include the rights of suffrage and to hold public office, she should have been more
Estrada.42 (Emphasis supplied) clear on her intentions.
However, the statement “[h]e is hereby restored to his civil and political rights,”
Absent any contrary evidence, former President Arroyo’s silence on former to the mind of the Court, is crystal clear — the pardon granted to former President
President Estrada’s decision to run for President in the May 2010 elections Estrada was absolute, meaning, it was not only unconditional, it was unrestricted
against, among others, the candidate of the political party of former President in scope, complete and plenary in character, as the term “political rights” adverted
Arroyo, after the latter’s receipt and acceptance of the pardon speaks volume of to has a settled meaning in law and jurisprudence.
her intention to restore him to his rights to suffrage and to hold public office. With due respect, I disagree too with Justice Leonen that the omission of the
Where the scope and import of the executive clemency extended by the qualifying word “full” can be construed as excluding the restoration of the rights
President is in issue, the Court must turn to the only evidence available to it, and of suffrage and to hold public office. There appears to be no distinction as to the
that is the pardon itself. From a detailed review of the four corners of said coverage of the term “full political rights” and the term “political rights” used alone
document, nothing therein gives an iota of intimation that the third Whereas without any qualification. How to ascribe to the latter term the meaning that it is
Clause is actually a limitation, proviso, stipulation or condition on the grant of the “partial” and not
pardon, such that the breach of the mentioned commitment not to seek public office 276
will result in a revocation or cancellation of said pardon. To the Court, what it is 276 SUPREME COURT REPORTS ANNOTATED
simply is a statement of fact or the prevailing situation at the time the executive
clemency was granted. It was not used as a condition to the efficacy or to delimit Risos-Vidal vs. Commission on Elections
the scope of the pardon. “full” defies one’s understanding. More so, it will be extremely difficult to
Even if the Court were to subscribe to the view that the third Whereas identify which of the political rights are restored by the pardon, when the text of
Clause was one of the reasons to grant the pardon, the pardon itself does not the latter is silent on this matter. Exceptions to the grant of pardon cannot be
provide for the attendant consequence of the breach thereof. This Court will be presumed from the absence of the qualifying word “full” when the pardon restored
hard put to discern the resultant effect of an eventual infringement. Just like it the “political rights” of former President Estrada without any exclusion or
will be hard put to determine which civil or political rights were restored if the reservation.
Court were to take the road suggested by Risos-Vidal that the statement “[h]e is Therefore, there can be no other conclusion but to say that the pardon granted
hereby restored to his civil and political rights” excludes the restoration of former to former President Estrada was absolute in the absence of a clear, unequivocal
President Estrada’s rights to suffrage and to hold public office. The aforequoted and concrete factual basis upon which to anchor or support the Presidential intent
text of the executive clem- to grant a limited pardon.
_______________ To reiterate, insofar as its coverage is concerned, the text of the pardon can
withstand close scrutiny even under the provisions of Articles 36 and 41 of the
42 Rollo (Vol. I), p. 46. Revised Penal Code.
275
The COMELEC did not commit grave abuse of discretion amounting to
VOL. 747, JANUARY 21, 2015 275 lack or excess of jurisdiction in issuing the assailed Resolutions.
Risos-Vidal vs. Commission on Elections
ency granted does not provide the Court with any guide as to how and where In light of the foregoing, contrary to the assertions of Risos-Vidal, the
to draw the line between the included and excluded political rights. COMELEC did not commit grave abuse of discretion amounting to lack or excess
Justice Leonen emphasizes the point that the ultimate issue for resolution is of jurisdiction in issuing the assailed Resolutions.
not whether the pardon is contingent on the condition that former President The Court has consistently held that a petition for certiorariagainst actions of
Estrada will not seek another elective public office, but it actually concerns the the COMELEC is confined only to instances of grave abuse of discretion amounting
coverage of the pardon — whether the pardon granted to former President Estrada to patent and substantial denial of due process, because the COMELEC is
was so expansive as to have restored all his political rights, inclusive of the rights presumed to be most competent in matters falling within its domain.43
of suffrage and to hold public office. Justice Leonen is of the view that the pardon As settled in jurisprudence, grave abuse of discretion is the arbitrary exercise
in question is not absolute nor plenary in scope despite the statement that former of power due to passion, prejudice or per-
President Estrada is “hereby restored to his civil and political rights,” that is, the _______________
foregoing statement restored to former President Estrada all his civil and political
rights except the rights denied to him by the unremitted penalty of perpetual 43 Naval v. Commission on Elections, G.R. No. 207851, July 8, 2014, 729
absolute disqualification made up of, among others, the rights of suffrage and to SCRA 299.
hold public office. He adds that had the President chosen to be so expansive as to 277
VOL. 747, JANUARY 21, 2015 277
Risos-Vidal vs. Commission on Elections below, these rights are subsumed under the phrase “civil and political rights” that
sonal hostility; or the whimsical, arbitrary, or capricious exercise of power that PGMA expressly restored in Erap’s pardon.
amounts to an evasion or refusal to perform a positive duty enjoined by law or to I add that aside from the points discussed by the ponencia, other material legal
act at all in contemplation of law. For an act to be condemned as having been done justifications exist that would support the same conclusion and address the
with grave abuse of discretion, such an abuse must be patent and gross. 44 vagueness that Risos-Vidal attributes to the textual language of Erap’s pardon.
The arguments forwarded by Risos-Vidal fail to adequately demonstrate any These legal justifications include an unbiased examination of the third preambular
factual or legal bases to prove that the assailed COMELEC Resolutions were clause of Erap’s pardon, the official definition of “absolute pardon,” and the
issued in a “whimsical, arbitrary or capricious exercise of power that amounts to pertinent rules on statutory construction that, in instances of doubt, give primacy
an evasion or refusal to perform a positive duty enjoined by law” or were so “patent to the interests of the voters in election cases such as the present case. I shall
and gross” as to constitute grave abuse of discretion. discuss all these below.
On the foregoing premises and conclusions, this Court finds it unnecessary to I maintain, too, that despite the ponencia’s resolution of the issue of Erap’s
separately discuss Lim’s petition-in-intervention, which substantially presented pardon and its effects on his perpetual absolute disqualification, an equally
the same arguments as Risos-Vidal’s petition. important issue lingers and remains unresolved — whether or not the
WHEREFORE, the petition for certiorari and petition-inintervention Commis-
are DISMISSED. The Resolution dated April 1, 2013 of the Commission on 279
Elections, Second Division, and the Resolution dated April 23, 2013 of the VOL. 747, JANUARY 21, 2015 279
Commission on Elections, En Banc, both in SPA No. 13-211 (DC), Risos-Vidal vs. Commission on Elections
are AFFIRMED. sion on Elections (COMELEC) gravely abused its discretion in relying
SO ORDERED. on its 2010 rulings that Erap’s pardon restored his rights to vote and to be
Velasco, Jr., Peralta, Bersamin, Del Castillo, Villarama, Jr., Perez, voted for a public office.
Reyes and Perlas-Bernabe, JJ., concur.
This issue is particularly important since the Court’s certiorari jurisdiction is
Sereno, CJ., I join the dissent of J. Leonen. being invoked and the assailed COMELEC rulings are not being questioned
Carpio, J., I join the dissent of J. Leonen. specifically on its ruling on the issue of Erap’s pardon but on the COMELEC’s
Brion,** J., On Official Leave. reliance on its 2010 ruling on this particular issue.
_______________ This 2010 disqualification ruling pertained to the consolidated COMELEC
Resolution in SPA No. 09-028 (DC) and SPA No. 09-104 (DC), entitled Atty. Evilio
44 Hayudini v. Commission on Elections, G.R. No. 207900, April 22, 2014, 723 C. Pormento v. Joseph Ejercito Estrada and In Re: Petition to Disqualify Estrada
SCRA 223. Ejercito, Joseph M. From Running As President Due to Constitutional
* * As per CJ. Sereno, J. Brion left his vote. See his Dissenting Opinion. Disqualification and Creating Confusion to the Prejudice of Estrada, Mary Lou
278 B. These cases were filed against Erap when he ran as President of the Philippines
278 SUPREME COURT REPORTS ANNOTATED in the 2010 elections.
Risos-Vidal vs. Commission on Elections For clarity, the COMELEC Second Division’s resolution dated April 1, 2013
that is being questioned in the present case states: “Today, this Commission is
Mendoza, J., See Concurring Opinion.
confronted with a controversy that is far from novelty. Albeit raised by another
Jardeleza, J., No part.
petitioner, the issue raised in the present case is glaringly similar to or intertwined
with the issues involved in the consolidated resolution for SPA No. 09-028 (DC) and
SEPARATE OPINION
SPA No. 09-104 (DC). Therefore, it cannot be gainsaid that the question of
whether or not the pardon granted to respondent has restored his right to
BRION, J.:
run for public office, which was curtailed by virtue of his conviction for
plunder that carries with it the penalty of perpetual absolute
I concur with the ponencia’s conclusion that the pardon granted to respondent
disqualification, has been passed upon and ruled out by this Commission
Joseph Ejercito Estrada (or Erap for brevity) by President Gloria Macapagal-
way back in 2010... Having taken judicial cognizance of the consolidated
Arroyo (or PGMA for brevity) restored his rights to run for and hold public office
resolution for SPA No. 09-028 (DC) and SPA No. 09-104 (DC) and the 10 May 2010
and to vote.
En Banc resolution affirming it, this Commission will not belabor the controversy
I likewise agree with the ponencia that Erap’s pardon complied with the
further. More so, petitioner failed to present cogent
requirements under Articles 36 and 41 of the Revised Penal Code
280
(RPC). Specifically, Erap’s pardon contained an express restoration of his rights to
vote and to hold public office and an express remission of Erap’s perpetual absolute 280 SUPREME COURT REPORTS ANNOTATED
disqualification brought about by his conviction for plunder. As I will discuss Risos-Vidal vs. Commission on Elections
proof sufficient to reverse the standing pronouncement of this Commission dispute in a political setting — the elections of 2013. This characterization of the
declaring categorically that respondent’s right to seek public office has been present case, however, should not change nor affect the Court’s mode of resolution:
effectively restored by the pardon vested upon him by former President Gloria M. the Constitution only allows us to adjudicate on the basis of the law, jurisprudence
Arroyo. Since this Commission has already spoken, it will no longer engage and established legal principles.
in disquisitions of a settled matter lest indulged in wastage of government Under this approach, the Court should also be aware that beyond the direct
resources.” parties, another party — the formally unnamed and unimpleaded electorate — has
This COMELEC Second Division ruling was upheld by the COMELEC En interests that the Court should take into account. The electorate has a continuing
Banc in its Resolution dated April 23, 2013, which is also being assailed in the stake in this case because they participated and expressed their choice in the 2013
present case. elections; in fact, not one of the entities that could have prevented them from voting
I stress that the above 2013 COMELEC rulings that are sought to be nullified — the COMELEC and this Court — acted to prevent Erap from being voted upon.
in the present case did not explicitly rule on the issue of Erap’s pardon but merely 282
relied on the 2010 COMELEC rulings on this particular issue. According to Risos- 282 SUPREME COURT REPORTS ANNOTATED
Vidal, this “reliance” constituted grave abuse of discretion.
To my mind, in the exercise of the Court’s certiorari jurisdiction, the issue of Risos-Vidal vs. Commission on Elections
whether or not the COMELEC gravely abused its discretion in relying on its 2010 Their participation, to my mind, brings into the picture the need to consider
rulings on Erap’s pardon should be squarely ruled upon on the merits, especially and apply deeper democratic principles: while the voters are generally the
because Risos-Vidal and the parties raised this particular issue in the present case. governed, they are at the same time the sovereign who decides how and by whom
Another crucial issue that must be resolved, in view of its jurisprudential they are to be governed. This step is particularly relevant in the present case
repercussions, is the legal propriety of Alfredo S. Lim’s (Lim) intervention in the since the electorate’s unquestioned preference was Erap, the recipient of
present case. the disputed pardon.
I discuss all these issues below. I recite all these as they are the underlying considerations I shall take into
account in this Separate Opinion.
I. Aside from points of law, I also take into account the interests of the voters.
These interests, in my view, should not only be considered but given weight and
Prefatory Statement even primacy, particularly in a situation of doubt.

Before this Court is an election disqualification case involving a candidate (and II.
subsequent winner) in the 2013 elections. By their nature, disqualification cases
are not unusual; in our political system they are given free rein because they affect The Roots of the Present Case
voters’ choice and governance.
281 A. The Early Roots: The Plunder and the Pardon.
VOL. 747, JANUARY 21, 2015 281
The present case traces its roots to respondent Erap’s term as President of the
Risos-Vidal vs. Commission on Elections Philippines which started at noon of June 30, 1998. He relinquished his post in the
What distinguishes this case is the basis for the objection — the executive middle of his term and was thereafter charged with the crime of
clemency (or as interchangeably used in this Opinion, the pardon) previously Plunder.1 The Sandiganbayan convicted him on September 12, 2007 and imposed
granted by the former President of the Republic Gloria Macapagal-Arroyo to her on him the penalty of reclusion perpetua and its accessory penalties.
immediate predecessor, respondent President Joseph Ejercito Estrada, whom the On October 25, 2007, former President Gloria Macapagal-Arroyo (PGMA)
former replaced under extraordinary circumstances. granted Erap executive clemency under terms that in part provides:
At issue is not the validity of the pardon as this issue has not been raised; at IN VIEW HEREOF and pursuant to the authority conferred upon me by the
issue (to be decided in the context of the presence or absence of grave abuse Constitution, I hereby grant executive clemency to JOSEPH EJERCITO
of discretion by the COMELEC) are the interpretation of the terms of the ESTRADA,
pardon and the grantor’s intent, a matter that — in the absence of direct _______________
evidence from grantor PGMA — the Court has to discern from the pardon’s written
terms. Intertwined with this issue is the question of whether or not the 1 Section 2, Republic Act No. 7080.
COMELEC gravely abused its discretion in dismissing the Risos-Vidal 283
petition based on its 2010 ruling that Erap’s pardon restored his rights to
VOL. 747, JANUARY 21, 2015 283
vote and to be voted for a public office.
Thus, we are largely left with the task of interpreting the terms of the pardon Risos-Vidal vs. Commission on Elections
that a politician granted to another politician, for the application of its terms to a
convicted by the Sandiganbayan of Plunder and imposed a penalty as SPA Nos. 09-028 (DC) and 09-104 (DC)] only involved Erap as the respondent.
of Reclusion Perpetua. He is hereby restored to his civil and political rights. Significantly, while three separate decisions were issued, they all commonly
[Emphasis supplied] discussed, practically using the same wording, the pardon extended to Erap
and concluded that the pardon restored Erap’s “right to vote and to be voted for
Erap accepted the pardon without qualifications on October 26, 2007. a public office.”5
_______________
B. Erap’s 2010 Presidential Candidacy & Disqualification Cases.
4 COMELEC, Second Division Resolution on SPA No. 09-028 (DC), attached
On November 30, 2009, Erap filed his Certificate of Candidacy (CoC) for the as Annex “O” to Memorandum of Intervenor Lim.
position of President of the Philippines. 5 A. At page 22 of the COMELEC Resolution dated January 20, 2010 in the
His candidacy immediately drew a trilogy of cases that were filed on or about Pamatong petition [SPA No. 09-024 (DC)], the COMELEC Second Division ruled
the same time, with the intent of disqualifying him from running as President and that:
from holding office if he would win. “Furthermore, there is absolutely no indication that the executive clemency
The first was a petition to cancel and deny due course to Estrada’s CoC [SPA exercised by President Arroyo to pardon Former President Estrada was a mere
09-024 (DC)]2 filed by Elly Velez B. Lao Pamatong (Pamatong). PGMA was also conditional pardon. It clearly stated that the former president is “restored to his
impleaded as a respondent. Pamatong alleged that Erap could not validly run for civil and political rights” and there is nothing in the same which limits the
the presidency because of the constitutional ban against reelection; he also claimed restoration. The
that PGMA was also prohibited from running for any elective public office, even as 285
a representative of the 2nd district of Pampanga. Pamatong also argued in his
VOL. 747, JANUARY 21, 2015 285
position paper that Erap’s pardon was not absolute as it was conditioned
on his promise not to run for any public office.3 Risos-Vidal vs. Commission on Elections
_______________
B.1. The Disqualification Rulings in the 2010 Election Cases.
2 Resolution of the COMELEC dated January 20, 2010 was attached as Annex
4 to Annex H of the Petitioner’s Memorandum. Thus, in clear and explicit terms, the Resolutions in all three cases uniformly
3 See page 8 of the COMELEC, Second Division Resolution dated January 20, ruled that Erap was not disqualified from running and from holding office, not only
2010 in SPA No. 09-024(DC) entitled Rev. Elly Velez B. Lao Pamatong, Esq v. because he was not running for reelection, but likewise because of the pardon that
Joseph Ejercito Estrada and Gloria Macapagal-Arroyo. This Resolution was had been extended to him.
attached as Exhibit “4” to Annex “E” of the Memorandum that Petitioner Risos- _______________
Vidal submitted to the Court.
284 only thing stated therein that may have some bearing on the supposed
284 SUPREME COURT REPORTS ANNOTATED conditions is that statement in the whereas clause that contained the following:
Whereas, Joseph Ejercito Estrada has publicly committed to no longer seek any
Risos-Vidal vs. Commission on Elections elective position or office, but that is not a condition but is merely part of a
The second formal objection to Erap’s presidential candidacy came preliminary statement. It cannot therefore serve to restrict the operation of or
from Evilio C. Pormento (Pormento) who filed his “Urgent Petition for prevail over the explicit statement in the executive clemency which restored all of
Disqualification as Presidential Candidate” on December 5, 2009 (docketed Estrada’s civil and political rights, including the “right to vote and to be voted for
as SPA 09-028). Pormento alleged that Erap was not eligible for reelection for the a public office,” including the position of the Presidency.
position of President pursuant to Article VII, Section 4 of the Constitution. In his This executive clemency granted to the former President being absolute and
answer to Pormento, Erap re-pleaded his defenses in the Pamatong case and added unconditional and having been accepted by him, the same can no longer be
that the grant of executive clemency in his favor removed all legal impediments revoked.”
that might bar his candidacy for the presidency.4 B. At pages 23-24 of the of the COMELEC Resolution dated January 20, 2010
The third objection was filed by Mary Lou Estrada, a presidential candidate, in the Pormento and Mary Lou petitions [SPA Nos. 09-028 (DC) and 09-104 (DC)],
who filed a petition for disqualification and cancellation of Erap’s CoC based on the COMELEC Second Division ruled that:
the grounds that he was not eligible for reelection and that Erap’s candidacy would Furthermore, there is absolutely no indication that the executive clemency
confuse the electorate, to her prejudice. This case was docketed as SPA 09-104. exercised by President Arroyo to pardon Former President Estrada was a mere
The COMELEC, Second Division, called the trilogy to a joint hearing but opted conditional pardon. It clearly stated that the former president is “restored to his
to issue separate but simultaneous decisions because the Pamatong case, SPA 09- civil and political rights” and there is nothing in the same which limits the
024, involved PGMA as a second respondent, while the two other cases [docketed restoration. The only thing stated therein that may have some bearing on the
supposed conditions is that statement in the whereas clause thereof that contained participated in the elections since Pormento did not pray for the issuance of a
the following: “Whereas, Joseph Ejercito Estrada has publicly committed to no TRO.
longer seek any elective position or office,” but that is not really a condition but is Pamatong and Mary Lou Estrada did not pursue further remedies after the
merely part of a preliminary statement, referring to what respondent Estrada had COMELEC En Banc denied their respective motions for reconsideration. This
said publicly. There is nothing stated in the dispositive part that it was conditioned Court, on the other hand, dismissed Pormento’s Rules 64/65 petition assailing the
upon said respondent’s purported public commitment. His public statement COMELEC ruling. Thus, the COMELEC ruling in the three cases
cannot, therefore, restrict the operation of, or pre- became final, executory, non-appealable and non-assailable.9
As I will discuss below, these final COMELEC decisions on Erap’s pardon and
286 his resulting qualification to run for elective public office preclude this same issue
286 SUPREME COURT REPORTS ANNOTATED of pardon from again being questioned because res judicata has already set in.
Significantly, when voting took place on May 10, 2010, no prohibition was in
Risos-Vidal vs. Commission on Elections place to prevent the voters from voting for Erap as a candidate. Neither the
The COMELEC specifically ruled that the statement in the pardon stating that COMELEC (because it had dismissed the petitions against Erap’s candidacy) nor
— “Whereas, Joseph Estrada has publicly committed to no longer seek any elective this Court (because it did not issue any temporary restraining order or
position or office” — was not really a condition but was merely a part of the injunction) prevented Erap from being voted upon. In a field of ten
pardon’s preliminary statement. The dispositive portion of the pardon did not state (10) candidates, Erap garnered 9,487,837 votes and landed in second place, as
that it was conditioned on this purported public commitment. Additionally, his against the winner’s 15,208,678 votes.10
public statement cannot serve to restrict the operation of, or prevail over, the _______________
explicit statement in the pardon that restored all his civil and political rights,
including the right to vote and to be voted for a public office.6 9 They are final and non-appealable pursuant to Section 3, Rule 37 of the
Petitioner Mary Lou Estrada pointedly questioned the COMELEC rulings in COMELEC Rules of Procedure; they are no longer assailable because the period to
her motion for reconsideration, including the terms of the pardon extended to question them before the Supreme Court had lapsed pursuant to Section A(7),
Erap.7 Before the 2010 elections took place, the COMELEC En Banc adopted the Article IX, 1987 Constitution
Second Division ruling and denied all the motions. 8 Only Pormento responded 10 Pursuant to the Congress’ Joint Public Session, Resolution of Both Houses
to the denial by filing a petition for certiorari before the Court, docketed No. 01 entitled, Resolution of Both Houses Approving the Report of the Joint
as G.R. No. 191988. Committee, Declaring the Results of the National Elections Held on May 10, 2010,
In resolving Pormento’s petition, the Court solely touched on the issue of For the Offices of Presi-
“reelection” and held that there was no longer 288
_______________
288 SUPREME COURT REPORTS ANNOTATED
vail over, the explicit statement in the executive clemency which restored all Risos-Vidal vs. Commission on Elections
of Estrada’s civil and political rights, including the “right to vote and to be voted
for a public office,” including to the position of the Presidency. This executive III.
clemency granted to the former President being absolute and unconditional and
having been accepted by him, the same can no longer be revoked or be made subject The Risos-Vidal Petition
to a condition.
6 Id. On October 2, 2012, Erap filed his Certificate of Candidacy (CoC) for the
7 The COMELEC En Banc denied the motions for reconsideration of position of City Mayor of Manila. As had happened in the past, this Erap move did
Pormento and Mary Lou Estrada in its Resolutions dated May 4, 2010 and April not go unchallenged.
27, 2010, respectively. These resolutions were attached as Exhibits “5” and “6,”
respectively, to Annex “E” of Petitioner Risos-Vidal’s Memorandum that she A. The COMELEC Petition.
submitted to the Court.
8 See Exhibits “5” and “6” attached to Annex “E” of Petitioner Risos-Vidal’s Petitioner Risos-Vidal filed on January 24, 2013 — or before the 2013 elections
Memorandum that she submitted to the Court. — a petition for disqualification against private respondent Erap based on Section
287 4011 of the Local Government Code (R.A. No. 7160, the LGC) in relation with
VOL. 747, JANUARY 21, 2015 287 Section 1212 of the Omnibus Election Code (B.P. No. 881, the OEC). Both the LGC
and the OEC commonly disqualify
Risos-Vidal vs. Commission on Elections _______________
any justiciable issue to be resolved because Erap had already lost the 2010
elections. Thus, the Court dismissed the whole petition, observing that Erap fully
dent and Vice President, and Proclaiming the Duly Elected President and Vice 13 See Exhibit “4” attached to Annex “E” of Petitioner Risos-Vidal’s
President of the Republic of the Philippines. Memorandum that she submitted to the Court.
11 Section 40. Disqualifications.—The following persons 14 See Exhibit “5” attached to Annex “E” of Petitioner Risos-Vidal’s
are disqualified from running for any elective local position: Memorandum that she submitted to the Court.
(a) Those sentenced by final judgment for an offense involving moral 15 See Exhibit “6” attached to Annex “E” of Petitioner Risos-Vidal’s
turpitude or for an offense punishable by one (1) year or more of imprisonment, Memorandum that she submitted to the Court.
within two (2) years after serving sentence. [Emphasis supplied] 16 April 23, 2013.
12 Sec. 12. Disqualifications.—Any person who has been declared by 290
competent authority insane or incompetent, or has been sentenced by final 290 SUPREME COURT REPORTS ANNOTATED
judgmentfor subversion, insurrection, rebellion or for any offense for which he
has been sentenced to a penalty of more than eighteen months or for a crime Risos-Vidal vs. Commission on Elections
involving moral turpitude, shall be disqualified to be a candidate and to hold for certiorari, where she alleged that the COMELEC gravely abused its
any office, unless he has been given plenary pardon or granted amnesty. discretion in issuing the assailed COMELEC resolutions.17
This disqualifications to be a candidate herein provided shall be deemed While the petition was pending before the Court, the 2013 elections took
removed upon the declaration by competent authority that said insanity or place. Neither the COMELEC nor this Court barred Erap from running and
incompetence had been removed or after the expiration of a period of five years being voted upon. He obtained 349,770 votes and was proclaimed as the “duly
from his service of sentence, unless within the same period he again becomes elected” Mayor on May 14, 2013. His opponent, Lim, obtained 313,764 votes and
disqualified. [Emphasis supplied] conceded that Erap had won.18
289
C. The Lim Intervention.
VOL. 747, JANUARY 21, 2015 289
Risos-Vidal vs. Commission on Elections On June 7, 2013 — i.e., after the 2013 elections; Erap’s proclamation as elected
any person convicted of an offense involving moral turpitude from running for Mayor; his concession of the elections to Erap; and while the present petition was
office. pending before the Court — Lim (Erap’s opponent in the mayoralty race) filed a
She sought to disqualify Erap from running for mayor for having been motion for leave to intervene, which motion the Court granted in a Resolution
convicted of a crime involving moral turpitude (plunder), an offense that carries dated June 25, 2013.
the penalty of reclusion perpetua and the accessory penalties of interdiction and
perpetual absolute disqualification. She alleged that Erap’s subsequent pardon IV.
was conditional and did not cover the accessory penalty of perpetual absolute
disqualification. The Issues for Resolution
Risos-Vidal and Erap fully argued the pardon aspect of the case before the
COMELEC and before the Court. In Risos-Vidal’s Memorandum that she The main issue in this case is whether the COMELEC committed GRAVE
submitted to the Court, she attached as Annex “E” the COMELEC Memorandum ABUSE OF DISCRETION in ruling that Erap had been extended a
of Erap with the attached Pamatong,13 Pormento14 and Mary Lou PARDON that qualified him to run for City Mayor of Manila in the 2013
Estrada15 COMELEC resolutions. elections.
Interrelated with this issue is the question of whether or not the
B. The COMELEC Ruling. COMELEC committed GRAVE ABUSE OF DISCRETION in dismissing the
Risos-Vidal petition based on the 2010 COMELEC rulings that
On April 1, 2013 or 42 days before the 2013 elections, the COMELEC Second _______________
Division dismissed the petition for disqualification, citing its 2010 rulings in the
cases filed against Erap after he filed his CoC for the position of President of the 17 Filed on April 30, 2013.
Philippines in 2010. According to the COMELEC, it had already ruled in these 18 See the COMELEC Provincial Canvass Report attached to the Petitioner’s
disqualification cases and had then held that the pardon granted to Erap was Memorandum as Annex “L.”
absolute and unconditional; hence, his previous conviction no longer barred him 291
from running for an elective public office. VOL. 747, JANUARY 21, 2015 291
The COMELEC En Banc denied Risos-Vidal’s motion for
reconsideration,16 prompting her to file the present petition Risos-Vidal vs. Commission on Elections
_______________ Erap’s pardon restored his rights to vote and to be voted for a public
office.
Closely related to these main issues is the question of whether — based the limited reach and scope of certiorari, compared with appellate review, direct
on the voting circumstances that surrounded the 2010 and 2013 elections that utmost respect be given the COMELEC as the constitutional body given the
— equitable reasons exist that should now prevent the Court from charge of elections.22
declaring Erap ineligible for the position to which he had been elected by
the majority of Manila voters. A.1(a) Certiorari v. Appeal.
Central to these issues is the determination of the nature and effects of the An appellate review includes the full consideration of the merits, demerits
pardon granted to Erap, as well as the effects of all the developments in the case and errors of judgment in the decision under review, while certiorari deals
on the electorate — the innocent third party whose exercise of the democratic exclusively with the presence
right to vote underlies the present dispute. _______________
A tangential side issue that should be settled for its jurisprudential value is
the legal propriety of the intervention of Alfredo S. Lim only at the Supreme Court 19 177 Phil. 205, 222; 88 SCRA 251, 269 (1979)
level. 20 Sec. 2, first paragraph, Article X.
Other subsidiary issues must necessarily be resolved to get at the main and 21 Aratuc v. COMELEC, supra at p. 223; p. 272.
side issues. They shall all be topically identified in the course of resolving the 22 Id.
leading issues. 293
VOL. 747, JANUARY 21, 2015 293
V.
Risos-Vidal vs. Commission on Elections
My Separate Opinion or absence of grave abuse of discretion amounting to lack of jurisdiction
that rendered the assailed decision or ruling a nullity; such kind of abuse is way
A. Preliminary Considerations. beyond mere error in the assailed judgment or ruling, and is not necessarily
present in a valid but erroneous decision.
A.1. The Standard of Review in Considering the present petition.
A.1(b) Grave Abuse of Discretion.
In the review of the COMELEC’s ruling on the Risos-Vidal petition, an issue
that we must settle at the outset is the nature and extent of the review we shall The grave abuse of discretion that justifies the grant of certiorari involves
undertake. This determination is important so that everyone — both the direct a defect of jurisdiction brought about, among others, by an indifferent disregard
parties as well as the voting public — will know and understand how this case was for the law, arbitrariness and caprice, an omission to weigh pertinent
decided and that the Court had not engaged in any kind of “overreach.” considerations, or a decision arrived at without rational deliberation 23 — due
292 process issues that rendered the decision or ruling void.
292 SUPREME COURT REPORTS ANNOTATED Our 1987 Constitution maintained the same remedy of certiorari in the review
of COMELEC decisions elevated to the Supreme Court as the Constitutional
Risos-Vidal vs. Commission on Elections Convention deliberations show.24 This constitutional provision has since then been
Section 7, Article IX of the Constitution provides that “unless otherwise reflected under Rules 64 and 65 of the Rules of Court.
provided by this Constitution or by law, any decision, order or ruling of each _______________
Commission may be brought to the Supreme Court on certiorari by the aggrieved
party.” A similar provision was found in the 1973 Constitution. 23 Id.
In Aratuc v. COMELEC (a 1979 case)19 the Court clarified that unlike in the 24 Fr. Bernas: The decision I cited was precisely an interpretation of the clause
1935 Constitution where the Court had the power of review over the decisions, in the provisions on the COMELEC which says: “Any decision, order, or ruling of
orders and rulings of the COMELEC,20the 1973 Constitution changed the the Commission may be brought to the Supreme Court on certiorari…” In
nature of this remedy from appellate review to certiorari. interpreting that provision in the case of Aratuc, the Supreme Court said:
Aratuc explained that under the then existing Constitution and statutory We hold therefore that under the existing constitutional and statutory
provisions, the certiorari jurisdiction of the Court over orders, and decisions of the provisions, the certiorari jurisdiction of the Court over orders, rulings and decision
COMELEC was not as broad as it used to be and should be confined to instances of the COMELEC is not as broad as it used to be and should be confined to
of grave abuse of discretion amounting to patent and substantial denial of due instances of grave abuse of discretion amounting to patent and substantial denial
process.21 of due process. Does that express the sense of the Committee?
The Court further observed that these constitutional, statutory and Mr. Regalado. That was the view of Justice Barredo in the Aratuc case while
jurisprudential changes show the definite intent to enhance and he was the ponente x x x In subsequent decisions wherein Chief Justice Teehankee
invigorate the role of the COMELEC as the independent constitutional
body tasked to safeguard free, peaceful and honest elections. In other words,
concurred, he believed that the mode of review on certiorari under Rule XLV
[should be LXV] is to be under- To assail a COMELEC ruling, the assailing party must show that the final
and inappealable ruling is void, not merely erroneous, because the
294 COMELEC acted with grave abuse of discretion in considering the case or in
294 SUPREME COURT REPORTS ANNOTATED issuing its ruling.
Under our established jurisprudence, this grave abuse of discretion has been
Risos-Vidal vs. Commission on Elections almost uniformly defined as a “capricious or whimsical exercise of judgment as is
equivalent to lack of jurisdiction.” The abuse of discretion, to be grave, must be so
Aside from the jurisdictional element involved, another basic and important patent and gross as to amount to an “evasion of a positive duty or to a virtual
element to fully understand the remedy of certiorari, is that it applies to rulings refusal to perform a duty enjoined by law, or to act at all in contemplation of law,
that are not, or are no longer, appealable. Thus, certiorari is not an appeal as where the power is exercised in an arbitrary and despotic manner by reason of
that opens up the whole case for review; it is limited to a consideration of a specific passion and hostility.”
aspect of the case, to determine if grave abuse of discretion had intervened. The present Erap case is an election case brought from a ruling of the
For example, it is a remedy that may be taken against an interlocutory order COMELEC En Banc to this Court as an independent action for
(or one that does not resolve the main disputed issue in the case and is thus not a certiorari under Rule 64 in relation with Rule 65 of the Rules of Court, and must
final order on the merits of the case) that was issued with grave abuse of discretion. perforce be judged under the above discussed standards.
This is the remedy to address a denial of a bill of particulars 25 or of the right to The question before us is not simply whether the COMELEC erred in
bail26 by the trial court in a criminal appreciating the nature of the pardon granted to Erap and in relying on its 2010
_______________ rulings on this matter; the question to ask is, even if the COMELEC did err,

stood as including acts of the Constitutional Commissions, without jurisdiction _______________


or acting in excess of jurisdiction.
Fr. Bernas. This seems to be the same thing. If it is without jurisdiction or in 26 Caballes v. Court of Appeals, 492 Phil. 410, 417-418; 452 SCRA 312, 328
excess of jurisdiction, there is grave abuse of discretion. (2005).
Mr. Regalado. No, Commissioner. Grave abuse of discretion may be equivalent 27 Section A(7), Article IX, 1987 Constitution; Section 3, Rule 37 of the
to lack of jurisdiction, if it was done in a capricious or whimsical manner. But COMELEC Rules of Procedure.
excess of jurisdiction is a little different, meaning, that the Supreme Court had 296
jurisdiction but it overstepped the bounds of jurisdiction in the exercise thereof.
296 SUPREME COURT REPORTS ANNOTATED
That is what Justice Teehankee also pointed out. Grave abuse of discretion, I
agree, results in lack of jurisdiction, but excess of jurisdiction presupposes that the Risos-Vidal vs. Commission on Elections
Court, while with jurisdiction just overstepped the permissible bounds in the whether its error is to the point of grave abuse of discretion.
exercise thereof.
Fr. Bernas: So, for purposes of the record now, what is the intention of the 1. The Interests of the Electorate.
Committee? What are the grounds for certiorari?
Mr. Regalado. The Committee which refers specifically to technical term of As I narrated above, the Erap story did not end with his crime and conviction.
review by certiorari would be relying on the provisions of Rule XLV [Should be While he had undeniably committed a crime involving betrayal of the public trust,
LXV] of the Rules of Court that laid down the three grounds. (The Intent of the he was subsequently and lawfully pardoned for his misdeed. While jurisprudence
1986 Constitution Writers, 1995 ed., Fr. Joaquin Bernas, SJ) may be divided on the effects of pardon (i.e., whether it erases both the guilt and
25 Virata v. Sandiganbayan, G.R. No. 106527, April 6, 1993, 221 SCRA 52, the penalty), the various cases giving rise to this jurisprudence do not appear to
60-61. have considered at all the election setting that presently confronts us.
295 Where the crime from which the guilt resulted is not unknown and was in fact
VOL. 747, JANUARY 21, 2015 295 a very widely publicized event in the country when it happened, the subsequent
electoral judgment of the people on the recipient of the executive clemency cannot
Risos-Vidal vs. Commission on Elections and should not be lightly disregarded. People participation is the essence of
case. It is also the sole remedy available against a COMELEC ruling on the democracy and we should be keenly aware of the people’s voice and heed it to the
merits of a case as this ruling on the main disputed issue is considered by the extent that the law does not bar this course of action. In case of doubt,
Constitution and by the law to be final and non-appealable.27 the sentiment that the people expressed should assume primacy.
When the recipient of pardon is likewise the people’s choice in an election held
A.1(c) Application of the Stardards of Review to the COMELEC after the pardon, it is well to remember that pardon is an act of clemency and grace
Ruling.
exercised to mitigate the harshness of the application of the law and should be interlocutory order that is subject to the Court’s final ruling on the merits of the
understood in this spirit, i.e., in favor of the grantee whom the people themselves case.
have adjudged and found acceptable. I have to discuss the intervention, too, for jurisprudential reasons: this
It ought not be forgotten that in two high profile elections, the State had intervention, apparently granted without in-depth consideration, may sow
allowed Erap to offer himself as a candidate without any legal bar and without confusion into the jurisprudence that those who came before us in this Court took
notice to the voting public that a vote for him could be rendered useless and stray. pains to put in order.
In the 2010 presidential elections, he had offered himself as a presidential
candidate and his candidacy was objected to, among others, because of the nature 2.a. Intervention in General.
of the pardon extended to
297 Intervention is a remedy whereby a third party, not originally impleaded in
VOL. 747, JANUARY 21, 2015 297 the proceedings, becomes a litigant in the case so that the intervenor could protect
or preserve a right or interest that may be affected by the proceedings.
Risos-Vidal vs. Commission on Elections The intervenor’s interest must be actual, substantial, material, direct and
him. The COMELEC resolved the objection and he was voted upon without any immediate, and not simply contingent or expectant. It must be of such direct
formal notice of any legal bar to his candidacy. It is now a matter of record and and immediate character that the intervenor will either gain or lose by the direct
history that he landed 2nd in these elections, in a field of ten (10) candidates, with legal operation and effect of the judgment.
9,487,837 voting for him as against the winner who garnered 15,208,678 votes. To As discussed below, there are also other equally important limitations and
Erap’s credit, he gracefully accepted his electoral defeat.28 restrictions to consider before an intervention can be allowed, among them, the
In 2013, he again ran for office. He won this time but a case was again filed need for the intervention to be timely filed.
against him with the COMELEC and the case eventually reached this Court. This
is the present case. 2.b. The context of Lim’s intervention.
The COMELEC cleared Erap by election day of 2013, dismissing the
disqualification case against him and ruling that the pardon granted to him The timing and incidents of Lim’s intervention are jurisprudentially
restored his right to vote and to be voted upon. Notably, even this Court did not interesting and, by themselves, speak loudly against his cause.
prevent Erap’s candidacy and did not prevent him from being voted upon after his The records of this case show that Lim never filed any petition to cancel
disqualification case was brought to this Court. Thus, the people went to the Erap’s CoC nor to disqualify him. Neither did he intervene in the COMELEC
polls and voted Erap into office with no expectation that their votes could proceedings in the Risos-Vidal petition. Instead, Lim allowed Erap to continue as
be rendered stray. his rival
Under these circumstances, we cannot and should not rashly rule on the basis 299
of black letter law and jurisprudence that address only the fact of pardon; we
VOL. 747, JANUARY 21, 2015 299
cannot forget the election setting and simply disregard the interests of the voters
in our ruling. While the people were not impleaded as direct parties to the case, Risos-Vidal vs. Commission on Elections
we cannot gloss over their interests as they are the sovereign who cannot be candidate in the 2013 elections for Mayor of the City of Manila.
disregarded in a democratic state like ours. It will be recalled that Risos-Vidal filed her petition for certiorari before this
Court on April 30, 2013 (or before the May 13, 2013 elections). Lim likewise did
2. The Intervention of former Mayor Alfredo S. Lim. not intervene at that point. Erap won in the elections and in fact, on May 14, 2013,
Lim publicly announced that he respected and acknowledged the COMELEC’s
I have included the intervention of former Mayor Alfredo S. Lim as a matter proclamation of Erap and wished him all the best.29
for Preliminary Consideration as it is an immaterial consideration under my On June 7, 2013 (25 days after the May 13, 2013 elections, or 24 days after
position that the COMELEC did not gravely abuse its discretion in its assailed Erap’s proclamation, and 24 days likewise after Lim conceded victory to Erap), Lim
_______________ then filed with this Court his motion for leave to intervene with the attached
petition-in-intervention. His arguments were: 1) Erap was disqualified to run for
28 Supra note 10. public office as his pardon did not restore his rights to vote and to hold public
298 office;30 and 2) his intervention was still timely.
Lim also argued that it would have been premature to intervene in the Risos-
298 SUPREME COURT REPORTS ANNOTATED
Vidal petition before the proclamation because had Erap’s votes not then been
Risos-Vidal vs. Commission on Elections counted, they would have been considered stray and intervention would have been
ruling. Despite its immateriality, I nevertheless discuss it in light of the unnecessary. Lim further argued that, in view of Erap’s disqualification, he should
Court’s prior action approving his intervention, which court approval was an be declared as the winner, having obtained the second highest number of votes.
Lim also additionally alleged that he never conceded defeat, and the COMELEC
committed grave abuse of discretion when it dismissed Risos-Vidal’s petition for which permits the filing of the motion to intervene at any time before the rendition
disqualification based on its 2010 rulings.31 of the judgment, in line with the ruling in Lichauco.39
_______________ The justification for this amendment is that before judgment is
rendered, the court, for good cause shown, may still allow the introduction
29 See page 45 of Memorandum for Intervenor. of additional evidence as this is still within a liberal interpretation of the
30 Id., at pp. 22-23. period for trial. Also, since no judgment has yet been rendered, the matter
31 Id., at pp. 46-55. subject of the intervention may still be readily resolved and integrated in
300 the judgment disposing of all claims in the case, without requiring an
overall reassessment of these claims as would be the case if the judgment
300 SUPREME COURT REPORTS ANNOTATED
had already been rendered.40
Risos-Vidal vs. Commission on Elections The Court held in Ongco that under the present rules, [t]he period within
2.c. Lim’s petition-in-intervention should be dismissed. which a person may intervene is also restricted… after the lapse of this period, it
will not be warranted anymore. This is because, basically, intervention is not an
Since Lim intervened only in the present petition for certiorari before this independent action but is ancillary and supplemental to an existing litigation.41
Court, the Rules of Court on intervention directly applies. Section 2, Rule 19 of the The Court further held in Ongco that “there is wisdom in strictly enforcing the
Rules of Court provides that the time to intervene is at any time before the period set by Rule 19 of the Rules of Court for the filing of a motion for intervention.
rendition of judgment by the trial court. Otherwise, undue delay would result from many belated filings of motions for
The Court explained in Ongco v. Dalisay32 that “the period within which a intervention after judgment has already been rendered, because a reassessment of
person may intervene is restricted and after the lapse of the period set in Section 2, claims would have to be done. Thus, those who slept on their lawfully granted
Rule 19, intervention will no longer be warranted. This is because, basically, privi-
intervention is not an independent action but is ancillary and supplemental to an _______________
existing litigation.”
In Ongco,33 the Court further traced the developments of the present rule on 38 Id., citing Director of Lands v. Court of Appeals, No. L-45168, September
the period to file a motion for intervention. The former rule was that intervention 25, 1979, 93 SCRA 238.
may be allowed “before or during a trial.” Thus, there were Court rulings that a 39 Id.
motion for leave to intervene may be filed “before or during a trial,” even on the 40 Id.
day when the case is submitted for decision as long as it will not unduly delay the 41 Id., at pp. 241-243.
disposition of the case.34 There were also rulings where the Court interpreted 302
“trial” in the restricted sense such that the Court upheld the denial of the motion
302 SUPREME COURT REPORTS ANNOTATED
for intervention when it was filed after the case had been submitted for
decision.35 In Lichauco v. CA,36 intervention was allowed at any time after the Risos-Vidal vs. Commission on Elections
rendition of the final judgment.37 In one exceptional case,38 the Court lege to intervene will be rewarded, while the original parties will be unduly
_______________ prejudiced.”42
While the Court may have liberally relaxed the rule on intervention in some
32 677 SCRA 232, 241 (2012). cases, a liberal approach cannot be made in the present case because of
33 Id., at pp. 240-241. jurisdictional restrictions, further explained below.
34 Id., at p. 241, citing Falcasantos v. Falcasantos, No. L-4627, May 13, 1952. Other than these reasons, I add that under COMELEC rules, only “a person
35 Id., citing Vigan Electric Light Co., Inc. v. Arciaga, Nos. L-29207 and L- allowed to initiate an action or proceeding may, before or during the trial of an
29222, July 31, 1974, 58 SCRA 211. action or proceeding, be permitted by the Commission, in its discretion, to intervene
36 Id., citing L-23842, March 13, 1975, 63 SCRA 723. in such action or proceeding, if he has legal interest in the matter in litigation, or
37 Id. in the success of either of the parties, or an interest against both, or when he is so
301 situated as to be adversely affected by such action or proceeding.” Thus, Lim could
have intervened at the COMELEC level before or during the hearing of the petition
VOL. 747, JANUARY 21, 2015 301
for disqualification that Risos-Vidal filed.
Risos-Vidal vs. Commission on Elections The records show that Lim intervened only after Risos-Vidal filed the present
allowed the intervention in a case pending before it on appeal in order to avoid petition for certiorari with the Court and not during the disqualification
injustice. proceedings before the COMELEC. He was therefore never a party in the
To cure these inconsistent rulings, the Court clarified in Ongco that [t]he disqualification proceeding before the COMELEC and, consequently, has not
uncertainty in these rulings has been eliminated by the present Section 2, Rule 19, presented any evidence to support his claims; nor was Erap ever given the chance
to controvert Lim’s claims before the COMELEC, the tribunal vested with the As my last two points on the requested intervention, I would deny the
jurisdiction to settle the issues that he raised in his petition-in- intervention even if it technically satisfies the rules by reason of the estoppel that
intervention before the Court. set in when Lim publicly announced that he was acknowledging and respecting
From the perspective of Rule 65 of the Rules of Court, I add that because Erap’s proclamation. This public announcement is an admission against his
Lim was not a party before the COMELEC, he never had the chance to file a motion interest that, in a proper case, would be admissible against Lim.
for reconsideration before that body — a constitutional and procedural I also disregard outright, for lack of relevance, the cases that Lim cited
requirement before a petition for certiorari may be regarding intervention. In his cited Maquiling v. COMELEC46and Aratea v.
_______________ COMELEC47 cases, the intervenors filed their intervention before the COMELEC
and not before the Court. Thus, any reliance on these cases would be misplaced.
42 Ongco v. Dalisay, supra note 32 at p. 242. In sum, I maintain that Lim should be barred from participating in the present
303 case as intervenor. Otherwise, the Court will effectively throw out of the window
the jurisprudence that has developed on intervention, while disregarding as well
VOL. 747, JANUARY 21, 2015 303
the sound and applicable COMELEC rules on the same topic.
Risos-Vidal vs. Commission on Elections
filed before the Court.43 As a nonparty to the disqualification case before the VI.
COMELEC, he cannot be deemed an “aggrieved party” who has earned the rights
under Rule 65 to file a certiorari petition or to intervene to assail the COMELEC’s The Merits of the Petition
decision. The Court, in particular, has no jurisdiction to grant the prayer A.
of Lim to be declared as the winner, especially since the COMELEC never On the Issue of Pardon and the COMELEC’s Grave Abuse of
had the chance to rule on this in its assailed decision. Discretion.
The original jurisdiction to decide election disputes lies with the COMELEC,
not with this Court.44 Thus, any ruling from us in the first instance on who should The COMELEC did not err at all and thus could not have committed
sit as mayor (in the event we grant the Risos-Vidal petition) will constitute grave grave abuse of discretion in its ruling that the terms of Erap’s pardon
abuse of discretion. Unfortunately, no recourse is available from our ruling. restored to him the
This character of finality renders it very important for us to settle the Lim _______________
intervention correctly.
At this juncture, I refer back to Ongco, where the Court held that the filing of 46 G.R. No. 195649, April 16, 2013, 696 SCRA 420.
a motion for intervention with the CA after the MTC had rendered judgment is an 47 G.R. No. 195229, October 9, 2012, 683 SCRA 1.
inexcusable delay and is a sufficient ground for denying a motion for 305
intervention.45
VOL. 747, JANUARY 21, 2015 305
Note that in Ongco, the Court still upheld the CA’s denial of the motion for
intervention and strictly applied the period to intervene even if what was involved Risos-Vidal vs. Commission on Elections
was an appeal or a continuation of the proceedings of the trial court. right to vote and to be voted upon. Too, the COMELEC did not gravely
In contrast, the present case is not a continuation of the COMELEC abuse its discretion in dismissing the petition of Risos-Vidal and in citing its 2010
proceedings and decision, but an original special civil action of certiorari. Thus, final and executory rulings that Erap’s pardon restored his right to vote and be
with more reason should the rules on intervention be more stringently applied, voted upon.
given too that the Court has no original jurisdiction over the issues
_______________ A.1. Pardoning Power and the Pardon Extended.

43 See Esteves v. Sarmiento, 591 Phil. 620, 625; 570 SCRA 656, 662 (2008). Section 19, Article VII of the Constitution provides for the pardoning power of
44 Section 12, Article I and Section 68, Article IX of the OEC; Section 6, RA the President. It states that except in cases of impeachment, or as otherwise
6646. provided in this Constitution, the President may grant reprieves, commutations,
45 Ongco v. Dalisay, supra note 32 at p. 240. and pardons, and remit fines and forfeitures, after conviction by final judgment.
304 Pardon is defined as an act of grace, proceeding from the power entrusted with
the execution of the laws, which exempts the individual, on whom it is bestowed,
304 SUPREME COURT REPORTS ANNOTATED
from the punishment that the law inflicts for a crime he has committed.48
Risos-Vidal vs. Commission on Elections The power to pardon, when exercised by the Chief Executive in favor of persons
involved in the requested intervention, in particular, over the issue of who convicted of public crimes, is plenary, limited only by the terms of the Constitution;
should sit as Mayor of the City of Manila if Risos-Vidal petition would be granted. its exercise within these limits is otherwise absolute and fully discretionary. The
reasons for its exercise are not open to judicial inquiry or review, and indeed it powers. (II RECORD of the Constitutional Commission, 392, 418-419, 524-
would appear that he may act without any reason, or at least without any 525)
expressed reason, in support of his action.49 It is evident from the intent of the Constitutional Commission, therefore, that
Where appropriate, however, his acts may be subject to the expanded the President’s executive clemency powers may not be limited in terms of coverage,
jurisdiction of the Court under Article VIII, Section 1, paragraph 2 of the except as already provided in the Constitution, that is, “no pardon, amnesty,
Constitution. This jurisdiction may be triggered, for example, if the President acts parole, or suspension of sentence for violation of election laws, rules and
outside, or in excess, of the limits of the pardoning power granted him, as regulations shall be granted by the President without the favorable
_______________ recommendation of the COMELEC.” (Article IX, C, Section 5, Constitution) If those
already adjudged guilty criminally in court may be pardoned, those adjudged
48 Monsanto v. Factoran, Jr., 252 Phil. 192, 198-199; 170 SCRA 190, 196 guilty administratively should likewise be extended the same benefit. [Emphasis
(1989). supplied]
49 The ruling in Guarin v. US, 30 Phil. 85, 87 (1915), accordingly adapted to
the terms of the 1987 Constitution. In considering and interpreting the terms of the pardon therefore, the
306 starting point for analysis is the position that the President’s power is full and
plenary, save only for the textual limits under the Constitution. In the exercise of
306 SUPREME COURT REPORTS ANNOTATED
this power, too, it is not unreasonable to conclude, in the absence of any plain and
Risos-Vidal vs. Commission on Elections expressed contrary intention, that the President exercised the full scope of his
when he extends a pardon for a crime as yet not committed or when he extends power.
a pardon before conviction.50
Llamas v. Orbos,51 a 1991 case, discussed the extent and scope of the A.2. Structural Examination of the Erap Pardon.
President’s pardoning power:
During the deliberations of the Constitutional Commission, a subject of The whole text of the pardon that PGMA granted states:
deliberations was the proposed amendment to Art. VII, Sec. 19 which reads as WHEREAS, this Administration has a policy of releasing inmates who have
follows: “However, the power to grant executive clemency for violation of corrupt reached the age of seventy (70),
practices laws may be limited by legislation.” The Constitutional Commission, WHEREAS, Joseph Ejercito Estrada has been under detention for six and half
however, voted to remove the amendment, since it was in derogation of the powers years,
of the President. As Mr. Natividad stated: WHEREAS, Joseph Ejercito Estrada has publicly committed to no longer seek
I am also against this provision which will again chip more powers from the any elective position or office,
President. In case of other criminals convicted in our society we extend probation IN VIEW HEREOF and pursuant to the authority conferred upon me by the
to them while in this case, they have already been convicted and we offer mercy. Constitution, I hereby grant executive clemency to JOSEPH EJERCITO
The only way we can offer mercy to them is through this executive clemency ESTRADA, convicted by the Sandiganbayanof Plunder and imposed
extended to them by the President. If we still close this avenue to them, they would 308
be prejudiced even worse than the murderers and the more vicious killers in our 308 SUPREME COURT REPORTS ANNOTATED
society x x x.
The proposal was primarily intended to prevent the President from Risos-Vidal vs. Commission on Elections
protecting his cronies. Manifestly, however, the Commission preferred to a penalty of Reclusion Perpetua. He is hereby restored to his civil and political
trust in the discretion of Presidents and refrained from putting rights.
additional limitations on his clemency The forfeitures imposed by the Sandiganbayan remain in force and in full,
_______________ including all writs and processes issued by the Sandiganbayan in pursuance
hereof, except for the bank account(s) he owned before his tenure as President.
50 Judicial power includes the duty of the courts of justice to settle actual Upon acceptance of this pardon by JOSEPH EJERCITO ESTRADA, this
controversies involving rights which are legally demandable and enforceable, and pardon shall take effect.
to determine whether or not there has been a grave abuse of discretion amounting
to lack or excess of jurisdiction on the part of any branch or instrumentality of the Structurally, this grant is composed of two parts, namely, the introductory
Government. Whereas Clauses consisting of three (3) paragraphs, and the Dispositive or
51 229 Phil. 920, 937-938; 202 SCRA 844, 858-859 (1991). Command portion which defines the clemency extended and commands its
307 implementation.
In issuing a pardon, the President not only exercises his full discretion but
VOL. 747, JANUARY 21, 2015 307
likewise directs and gives notice to all — the recipient, the officials and entities
Risos-Vidal vs. Commission on Elections concerned — that the recipient should now be released and his disqualification
lifted, pursuant to the terms of the pardon. In this sense, the structure of the fallo or dispositive portion of the decision ordains or decrees. 56[Emphasis
written pardon assumes importance as pardon has to be implemented in supplied]
accordance with its express terms and is no different in this sense from a judicial
decision that likewise must be implemented. Thus, the body of the decision (or opinion portion) carries
In judicial decisions, the Court’s resolution on a given issue before it is always no commanding effect; the fallo or dispositive portion carries the definite directive
embodied in the decision or order’s fallo or dispositive portion.52 It is the directive that prevails over whatever is written in the opinion of the court. The body
part of the decision or order which must be enforced or, in legal parlance, subjected contains the reasons or conclusions of the court, but orders nothing; execution
to execution. A court that issues an order of execution contrary to the terms of its springs from the fallo or dispositive portion, not from the decision’s body or opinion
final judgment exceeds its jurisdiction, thus rendering its order invalid.53 Hence, portion. In short, the fallo or dispositive portion prevails in case of conflict.
the order of I say all these, aware that in Cobarrubias v. People,57 the Court made an
_______________ exception to the general rule that the fallo or dispositive portion always prevails
over the decision or order’s body. The exception is when one can clearly and
52 Obra v. Badua, 556 Phil. 456, 458; 529 SCRA 621, 626 (2007). unquestionably conclude, based on the body of the decision and its discussions,
53 Id., at p. 461; p. 622. that a mistake had been committed in formulating the dispositive portion. In such
309 cases, reason dictates that the body of the decision should prevail.58
VOL. 747, JANUARY 21, 2015 309 This contrary Cobarrubias result, to be properly understood, must be read and
considered in its factual context. In this case, the court itself made a blatant
Risos-Vidal vs. Commission on Elections mistake in the dispositive portion as it mixed up the criminal docket case numbers,
execution should always follow the terms of the fallo or dispositive portion. thus resulting in the erroneous dismissal of the wrong criminal case. Since the
Other than the fallo, a decision or executory order contains a body — the decision’s body very clearly discussed which criminal case should be dismissed, the
court’s opinion — explaining and discussing the decision. This opinion serves Court then held that the body should prevail over the dispositive portion. In other
as the reason for the decision or order embodied in the fallo. In legalese, this words, when the decision’s intent is beyond doubt and is very clear but was simply
opinion embodies the decision’s ratio decidendi54 or the matter or issue directly beclouded by an intervening mistake, then the body of the decision must prevail.
ruled upon and the terms and reasons for the ruling. A pardon, as an expression of an executive policy decision that must be
The decision’s structure has given rise in certain instances to conflicts, or at enforced, hews closely to the structure of a court
the very least, to ambiguities that clouded the implementation of the decision. _______________
In Gonzales v. Solid Cement Corporation,55 this Court laid down the rule when
these instances occur: in a conflict between the body of the decision and its fallo or 56 Id.
dispositive portion, the rule is: 57 G.R. No. 160610, August 14, 2009, 596 SCRA 77, 89-90.
The resolution of the court in a given issue — embodied in the fallo or 58 Id.
dispositive part of a decision or order — is the controlling factor in resolving 311
the issues in a case. The fallo embodies the court’s decisive action on the issue/s
VOL. 747, JANUARY 21, 2015 311
posed, and is thus the part of the decision that must be enforced during execution.
The other parts of the decision only contain, and are aptly called, the ratio Risos-Vidal vs. Commission on Elections
decidendi (or reason for the decision) and, in this sense, assume a lesser role in decision. Their structures run parallel with each other, with the Whereas
carrying into effect the tribunal’s disposition of the case. Clauses briefly stating the considerations recognized and, possibly, the intents and
When a conflict exists between the dispositive portion and the opinion purposes considered, in arriving at the directive to pardon and release a convicted
of the court in the text or body of the decision, the former must prevail over prisoner.
the latter under the rule that the dispositive portion is the definitive order, Thus, while a pardon’s introductory or Whereas Clauses may be considered in
while the opinion is merely an explanatory statement without the effect of reading the pardon (in the manner that the opinion portion of a court decision is
a directive. Hence, the execution must conform with what the read), these whereas clauses — as a rule — cannot also significantly affect the
_______________ pardon’s dispositive portion. They can only do so and in fact may even prevail, but
a clear and patent reason indicating a mistake in the grantor’s intent must be
54 PH Credit Corporation v. Court of Appeals, 421 Phil. 821, 833; 370 SCRA shown, as had happened in Cobarrubias where a mistake intervened in the fallo.
155, 166 (2001).
55 G.R. No. 198423, October 23, 2012, 684 SCRA 344, 352. A.3. The Pardon Extended to Erap Examined.
310
A.3(a) The Decision Convicting Erap.
310 SUPREME COURT REPORTS ANNOTATED
Risos-Vidal vs. Commission on Elections
To fully understand the terms of the granted executive clemency, reference A.3(b) The Pardon in light of the Judgment of Conviction.
should be made to the September 12, 2007 decision of the Sandiganbayan which
states: This judgment has several components, namely: the finding of guilt; the
WHEREFORE, in view of all the foregoing, judgment is hereby rendered in principal penalty of imprisonment imposed; the inherent accessory penalties; the
Criminal Case No. 26558 finding the accused, Former President Joseph Ejercito confiscation and forfeitures; and the disposition of the cash bonds that the
Estrada, GUILTY beyond reasonable doubt of the crime of PLUNDER, defined in acquitted accused filed.
and penalized by Republic Act No. 7080, as amended. On the other hand, for failure Of these, actions on the forfeitures and the cash bonds have apparently been
of the prosecution to prove and establish their guilt beyond reasonable doubt, the recognized as completed pursuant to Article 45 of the RPC, and have been
Court finds the accused Jose “Jinggoy” Estrada and Atty. Edward S. Serapio NOT expressly excluded from the executive clemency.59 Thus, what remained for the
GUILTY of the crime of plunder and, accordingly, the Court hereby orders their executive clemency to touch upon were the principal and the accessory penalties
ACQUITTAL. that were outstanding, i.e., the remaining terms of the imprisonment; and the
The penalty imposable for the crime of plunder under Republic Act No. 7080, accessory penalties decreeing that Erap is “restored to his civil and political
as amended by Republic Act No. 7659, is Reclusion Perpetua to Death. There being rights.”
no aggravating or mitigating circumstances, however, the lesser penalty shall be
applied in accordance with Article B.
312 The Risos-Vidal’s
312 SUPREME COURT REPORTS ANNOTATED Objections Relating to Pardon.

Risos-Vidal vs. Commission on Elections The Risos-Vidal petition sows confusion into the plain terms of the executive
63 of the Revised Penal Code. Accordingly, the accused Former President clemency by arguing that: first, the Third Whereas Clause (referring to Erap’s
Joseph Ejercito Estrada is hereby sentenced to suffer the penalty of Reclusion public commitment that he would no longer seek public office) in fact embodies a
Perpetua and the accessory penalties of civil interdiction during the period of condition for the grant of the executive clemency; and second, no express
sentence and perpetual absolute disqualification. restoration of the right to hold public office and to suffrage was made as the
The period within which accused Former President Joseph Ejercito Estrada “restoration” was under general terms that did not cover these specific rights.
has been under detention shall be credited to him in full as long as he agrees 59 The pardon reads in part that “The forfeitures imposed by
voluntarily in writing to abide by the same disciplinary rules imposed upon the Sandiganbayanremain in force and in full, including all writs and processes
convicted prisoners. issued by the Sandiganbayan in pursuance hereof, except for the bank account(s)
Moreover, in accordance with Section 2 of Republic Act No. 7080, as amended he owned before his tenure as President.”
by Republic Act No. 7659, the Court hereby declares the forfeiture in favor of the 314
government of the following:
314 SUPREME COURT REPORTS ANNOTATED
(1) The total amount of Five Hundred Forty Two Million Seven Hundred
Ninety One Thousand Pesos (P545,291,000.00), with interest and income earned, Risos-Vidal vs. Commission on Elections
inclusive of the amount of Two Hundred Million Pesos (P200,000,000.00), B.1. Refutation of the Risos-Vidal Objections.
deposited in the name and account of the Erap Muslim Youth Foundation.
(2) The amount of One Hundred Eighty-Nine Million Pesos B.1(a) “Absolute Pardon” as Officially Defined.
(P189,000,000.00), inclusive of interests and income earned, deposited in the Jose
Velarde account. A ready reference to understand a pardon is its official definitionunder the
(3) The real property consisting of a house and lot dubbed as Boracay applicable law and applicable rules and regulations. The definition of absolute
Mansion located at #100 11th Street, New Manila, Quezon City. pardon appears in the rules and regulations of the Board of Pardons and Parole
The cash bonds posted by accused Jose Jinggoy Estrada and Atty. Edward S. (BPP).60 The BPP is the constituent office in the Executive
Serapio are hereby ordered cancelled and released to the said accused or their duly Department61 responsible for the handling of cases of pardon upon petition, or any
authorized representatives upon presentation of the original receipt evidencing referral by the Office of the President on pardons and parole, or motu propio.62 In
payment thereof and subject to the usual accounting and auditing procedures. other words, the BPP is the foremost authority on what its title plainly states —
Likewise, the hold departure orders issued against the said accused are hereby pardons and paroles.
recalled and declared functus officio. Under the BPP’s Revised Rules and Regulations, “absolute pardon”
SO ORDERED. refers “to the total extinction of the criminal liability of the individual to
313 whom it is granted without any condition. It restores to the individual his
VOL. 747, JANUARY 21, 2015 313 civil and political rights and remits the penalty imposed for the particular
offense of which he was convicted.”63
Risos-Vidal vs. Commission on Elections
Aside from absolute pardon, there is the conditional pardon64which is The pardon extended to Erap was very briefly worded. After three
defined as “the exemption of an individual, within certain limits or conditions, from short Whereas Clauses referring to: the Administration policy on the release of
the punishment which the law inflicts for the offense he had committed resulting in inmates;65 the period Erap had been under detention;66 and Erap’s attributed past
the partial extinction of his criminal liability.” statement publicly committing that he would no longer seek any elective
_______________ position,67 the pardon proceeds to its main directives touching on the principal
penalty of reclusion perpetua and the accessory penalties by expressly restoring
60 Rule 1, Section 2 paragraph (p) of the Revised Rules and Regulations of the Erap’s civil and political rights.
Board of Pardons and Parole; This definition is also found in the 2006 Revised Unlike in a court decision where the ratio decidendi fully expounds on the
Manual of the BPP. presented issues and leads up to the dispositive portion, the Whereas Clauses all
61 Under the Department of Justice pursuant to the Administrative Code, related to Erap but did not, singly or collectively, necessarily indicate that they are
Book IV, Title III, Chapter I, Section 4(6). conditions that Erap must comply with for the continued validity of his pardon.
62 2006 Revised Manual on Parole and Executive Clemency. Notably, the first two Whereas Clauses are pure statements of fact that the
63 Supra note 60. grantor recognized, referring as they do to an administration policy and to the age
64 Rule 1, Section 2 paragraph (q) of the Revised Rules and Regulations of the of Erap.
Board of Pardons and Parole; This definition is also found in the 2006 Revised The statement on the administration policy of releasing convicts who are 70
Manual of the BPP. years old, to be sure, could not have been intended to be conditional so that a future
315 change of policy or a mistake in Erap’s age would have led to the invalidity of the
pardon. Purely and simply, these two Whereas clauses were nothing more
VOL. 747, JANUARY 21, 2015 315
than statements of fact that the grantor recognized in the course of
Risos-Vidal vs. Commission on Elections considering the pardon and they were never intended to operate as conditions.
These are the authoritative guidelines in determining the nature and extent of _______________
the pardon the President grants, i.e., whether it is absolute or conditional. To
stress, the BPP is the body that investigates and recommends to the President 65 Under Section 3(e) of the 2006 Revised Manual on Parole and Executive
whether or not a pardon should be granted to a convict, and that closely Clemency, the BPP could recommend for pardon [p]risoners who are 70 years old
coordinates with the Office of the President on matters of pardons and parole. and above and who have served at least 5 years of their sentence or those whose
Even a cursory examination of the Erap pardon and the BPP Rules would show continued imprisonment is inimical to their health.
that the wordings of the pardon, particularly on civil and political rights, carried 66 Presumably from Court and Department of Justice records.
the wordings of the BPP Rules. Thus, Erap’s pardon states: 67 Source and circumstances unknown.
IN VIEW HEREOF, and pursuant to the authority conferred upon me by the 317
Constitution, I hereby grant executive clemency to JOSEPH EJERCITO
VOL. 747, JANUARY 21, 2015 317
ESTRADA, convicted by the Sandiganbayan of Plunder and imposed a penalty of
Reclusion Perpetua. He is hereby restored to his civil and political rights. Risos-Vidal vs. Commission on Elections
The third Whereas Clause, one of the three clauses that the pardon contains,
In these lights, when PGMA (as President and Head of the Executive is similarly a statement of fact — what Erap had publicly committed in the
Department to which the BPP belongs) granted Erap executive clemency past, i.e., that he would no longer seek public office. Such a statement would not
and used the words of the BPP rules and regulations, she raised the be strange coming from a 70-year-old man convicted of plunder and sentenced
inference that her grant was in the spirit in which the terms of the pardon to reclusion perpetua (literally, life imprisonment) and who, in the ordinary course,
are understood in the BPP rules. looks forward to an extended prison term. Under these conditions, he could easily
In other words, she clearly intended the granted pardon to be absolute. say he would not seek political office again.
Thus, the pardon granted totally extinguished the criminal liability of Erap, Of course, because the statement, standing by itself, can be equivocal, it can
including the accessory penalty of perpetual absolute disqualification. It cannot be also be read with a bias against Erap and be understood to be a promise or a
otherwise under the plain and unequivocal wording of the definition of absolute “commitment.” The plain reality, however, is that this clause does not bear the
pardon, and the statement in the pardon that Erap is restored to his civil and required context that would lead to this conclusion, and is totality lacking in any
political rights. indicator that would make it a condition for the pardon. In short, a clear link to
316 this kind of conclusion is plainly missing.
316 SUPREME COURT REPORTS ANNOTATED This link, for example, would have been there and would have radically
changed the meaning of this Whereas clause had it stated that Erap publicly
Risos-Vidal vs. Commission on Elections committed that, if pardoned, he would not seek public office. No such link,
B.2. The Third Whereas Clause as a Condition. however, appears in the body of the pardon, nor is any evidence available from the
records of the case, to show that a promissory commitment had been made and
adopted by PGMA, as grantor.
Thus, as matters stand, the third Whereas clause stands in the same footing interpretative aids, even those extraneous to the pardon, such as the events
and should be characterized in the same manner that the two other clauses are that transpired since the grant of the pardon. This case, in particular, the most
characterized: singly or collectively, they are simply declarations of what the relevant interpretative aids would be the two elections where Erap had been a
grantor recognized as facts at the time the pardon was granted. In the manner the candidate, the electorate’s choices, and the significant number who voted in good
Court spoke of preambles in the case of Kuwait Airways Corporation v. Philippine faith to elect Erap.
Airlines, Inc.,68 the Whereas clauses merely manifest considerations that cannot In 2010, this number was sizeable but Erap only landed in second place with a
be the vote of 9,487,837 in a field of ten (10) candidates. This result though cannot but be
_______________ given appropriate recognition since the elections were nationwide and Erap’s
conviction and pardon were issues used against him.
68 G.R. No. 156087, May 8, 2009, 587 SCRA 388, 410. In the 2013 elections (where Erap’s qualification is presently being contested),
318 the results were different; he garnered sufficient votes to win, beating the
incumbent in this electoral fight for the premiere post in the City of Manila.
318 SUPREME COURT REPORTS ANNOTATED
Under these circumstances, no reason exists to disregard the popular vote,
Risos-Vidal vs. Commission on Elections given that it is the only certain determinant under the uncertainty that
origin of rights and obligations69 and cannot make the Erap pardon petitioner Risos-Vidal NOW TRIES to introduce in the present case. If this
conditional. is done and the popular vote is considered together with the official definition of
Simply as an aside (as I feel the topic does not deserve any extended pardon under the BPP regulations, the conclusion cannot but be the recognition by
consideration), I do not believe that the “acceptance” of the pardon is this Court that Erap had been given back his right to vote and be voted upon.
important in the determination of whether the pardon extended is absolute or
conditional. B.3(a) The Express Restoration of the Right to Hold Office.
Irrespective of the nature of the pardon, the moment the convict avails of the
clemency granted, with or without written acceptance, then the pardon is already The petitioner Risos-Vidal in her second substantive objection posits that the
accepted. If this is to be the standard to determine the classification of the pardon, pardon did not expressly include the right to hold office, relying on Article 36 of
then there would hardly be any absolute pardon; upon his release, the pardon is the RPC that provides:
deemed accepted and therefore conditional. Pardon; its effects.—A pardon shall not work on the restoration of the right to
If an express acceptance would serve a useful purpose at all, it is in the binding hold public office or the right of suffrage, unless such rights be expressly restored
effect that this acceptance would put in place. As in the case of an appointment, a by the terms of the pardon.
pardon can be withdrawn at any time before it is accepted by the grantor. 320
Acceptance would thus be the means to tie the grantor to the grant. 320 SUPREME COURT REPORTS ANNOTATED
What is important, to my mind, is proof of the communication of the pardon to
the convict, in the cases when terms and conditions are attached to the pardon. Risos-Vidal vs. Commission on Elections
Communications of these terms, and proof that the convict availed himself of the To the petitioner, it was not sufficient that under the express terms of the
granted clemency, would suffice to conclude that the terms and conditions had pardon, Erap had been “restored to his civil and political rights.” Apparently, she
been accepted and should be observed. wanted to find the exact wording of the above quoted Article 36 or, as stated in her
various submissions, that Erap should be restored to his “full” civil and political
B.3. Any Doubt Should Take Popular Vote into Account. rights.
To set the records straight, what is before us is not a situation where a pardon
At most, I can grant in a very objective reading of the bare terms of the was granted without including in the terms of the pardon the restoration of civil
third Whereas clause that it can admit of various interpretations. Any and political rights. What is before us is a pardon that expressly and pointedly
interpretative exercise, however, in order to be meaningful and conclusive must restored these rights; only, the petitioner wants the restoration in her own terms.
bring into play relevant In raising this objection, the petitioner apparently refuses to accept the official
_______________ definition of “absolute pardon” pointed out above; she also fails or refuses to grasp
the full import of what the term “civil and political rights” connotes. The term
69 Id. traces its roots to the International Covenant on Civil and Political
319 Rights70 which in turn traces its genesis to the same process that led to
the Universal Declaration of Human Rights to which the Philippines is a
VOL. 747, JANUARY 21, 2015 319
signatory.71
Risos-Vidal vs. Commission on Elections _______________
70 The International Covenant on Civil and Political Rights (ICCPR) is a Commission; the Philippines voted in favor of this Declaration.
multilateral treaty adopted by the United Nations General Assembly on December (Source: http://en.wikipedia.org/wiki/Universal_Declaration_of_Human_Rights)
16, 1966, and in force from March 23, 1976. It commits its parties to respect the 72 G.R. No. 100150, January 5, 1994, 229 SCRA 117, 132-133.
civil and political rights of individuals, including the right to life, freedom of 322
religion, freedom of speech, freedom of assembly, electoral rights and rights to due 322 SUPREME COURT REPORTS ANNOTATED
process and a fair trial. As of April 2014, the Covenant has 74 signatories and 168
parties. The ICCPR is part of the Declaration on the Granting of Independence to Risos-Vidal vs. Commission on Elections
Colonial Countries and Peoples, International Bill of Human Rights, along with zen of the state or country, or, in wider sense, to all its inhabitants, and are not
the International Covenant on Economic, Social and Cultural Rights (ICESCR) connected with the organization or administration of the government. They include
and the Universal Declaration of Human Rights (UDHR). The Philippines signed the rights of property, marriage, equal protection of the laws, freedom of
this treaty on December 19, 1966 and ratified it on October 23, 1986. contract, etc. or, as otherwise defined, civil rights are rights appertaining to a
[Source: http://en.wikipedia.or/wiki/International_Covenant_ person by virtue of his citizenship in a state or community. Such term may also
on_Civil_and_Political_Rights] refer, in its general sense, to rights capable of being enforced or redressed in a civil
71 The Universal Declaration of Human Rights (UDHR) is a declaration action. Also quite often mentioned are the guarantees against involuntary
adopted by the United Nations General Assembly on 10 servitude, religious persecution, unreasonable searches and seizures, and
321 imprisonment for debt.73
Political rights, on the other hand, refer to the right to participate, directly
VOL. 747, JANUARY 21, 2015 321
or indirectly, in the establishment or administration of government, the right of
Risos-Vidal vs. Commission on Elections suffrage, the right to hold public office, the right of petition and, in general,
Closer to home, Republic Act No. 9225 (The Citizenship Retention and the rights appurtenant to citizenship vis-à-vis the management of government.74
Reacquisition Act of 2003) also speaks of “Civil and Political Rights and In my view, these distinctions and enumerations of the rights included in the
Liabilities” in its Section 5 by providing that “Those who retain or reacquire term “civil and political rights,”75 as accepted internationally and domestically, are
Philippine citizenship under this Act shall enjoy full civil and political rights and sufficiently clear and cannot be made the serious basis of the present
be subject to all the attendant liabilities and responsibilities under existing laws of _______________
the Philippines…” and in Section 5(5) mentions the “right to vote and be elected
or appointed to any public office in the Philippines x x x.” 73 Id.
In Simon v. Commission on Human Rights,72 the Court categorically 74 Id.
explained the rights included under the term “civil and political rights,” in the 75 Civil rights include the rights of property, marriage, equal protection of the
context of Section 18, Article XIII of the Constitution which provides for the laws, freedom of contract, etc. or, as otherwise defined, civil rights are rights
Commission on Human Rights’ power to investigate all forms of human rights appertaining to a person by virtue of his citizenship in a state or community. Such
violations involving civil and political rights. term may also refer, in its general sense, to rights capable of being enforced or
According to Simon, the term “civil rights,” has been defined as referring (t)o redressed in a civil action. Also quite often mentioned are the guarantees against
those (rights) that belong to every citi- involuntary servitude, religious persecution, unreasonable searches and seizures,
_______________ and imprisonment for debt.
Political rights refer to the right to participate, directly or indirectly, in the
December 1948 at the Palais de Chaillot, Paris. The Declaration arose directly establishment or administration of government, the right of suffrage, the right to
from the experience of the Second World War and represents the first global hold public office, the right of petition and, in general, the rights appurtenant to
expression of rights to which all human beings are inherently entitled. The citizenship vis-à-vis the management of government.
Declaration consists of thirty articles which have been elaborated in subsequent 323
international treaties, regional human rights instruments, national constitutions, VOL. 747, JANUARY 21, 2015 323
and other laws. The International Bill of Human Rights consists of the Universal
Declaration of Human Rights, the International Covenant on Economic, Social and Risos-Vidal vs. Commission on Elections
Cultural Rights, and the International Covenant on Civil and Political Rights and objection, i.e., that further specification should be made in light of Article 36 of
its two Optional Protocols. In 1966, the General Assembly adopted the two detailed the RPC that requires the restoration of the rights of the right to suffrage and to
Covenants, which complete the International Bill of Human Rights. In 1976, after hold office to be express. To insist on this argument is to require to be written into
the Covenants had been ratified by a sufficient number of individual nations, the the pardon what is already there, in the futile attempt to defeat the clear intent of
Bill took on the force of international law. the pardon by mere play of words.
The Declaration was commissioned in 1946 and was drafted over two years by
the Commission on Human Rights. The Philippine representative was part of the B.3(a)(i) The RPC Perspectives.
From the perspective of the RPC, it should be appreciated, as discussed above, 78 Pardon; its effect.—A pardon shall not work the restoration of the right to
that a conviction carries penalties with varying components. These are mainly the hold public office, or the right of suffrage, unless such rights be expressly restored
principal penalties and the accessory penalties.76 by the terms of the pardon.
Reclusion perpetua, the penalty imposed on Erap, carries with it the A pardon shall in no case exempt the culprit from the payment of the civil
accessory penalty of civil interdiction for life or during the period of the sentence indemnity imposed upon him by the sentence.
and that of perpetual absolute disqualification which the offender shall suffer 79 Reclusion perpetua and reclusion temporal; Their accessory penalties.—The
even though pardoned as to the principal penalty, unless the same shall have been penalties of reclusion perpetua and reclusion temporal shall carry with them that
remitted in the pardon.77 of civil interdiction for life or during the period of the sentence as the case may be,
The full understanding of the full practical effects of pardon on the and that of perpetual absolute disqualification which the offender shall suffer even
principal and the accessories penalties as embodied in the RPC, requires the though pardoned as to the principal penalty, unless the same shall have been
combined reading of Articles 36 and 41 of the RPC, with Article 41 giving full expressly remitted in the pardon.
meaning to the requirement of Article 36 that the restoration of the right to hold 80 The Revised Penal Code, Act No. 3815 was passed on December 8, 1930 and
office be expressly made in a pardon if indeed this is the grantor’s intent. An become effective on January 1, 1932. It has undergone a lot of amendments but
express mention has to be made of the restoration of the rights to vote and be voted Articles 36 and 41 are provisions that have largely been left intact.
for since a pardon with respect to the principal penalty would not have the effect 325
of restoring these specific rights unless their specific restoration is expressly VOL. 747, JANUARY 21, 2015 325
mentioned in the pardon.
The Erap’s pardon sought to comply with this RPC requirement by specifically Risos-Vidal vs. Commission on Elections
stating that he was “restored to his above, technical meanings have since then attached to the term “civil and
_______________ political rights,” which meanings cannot be disregarded without doing violence to
the safeguards that these rights have acquired over the years.
76 See Articles 40 to 45 of the Revised Penal Code on penalties in which In this age and time, “political rights” cannot be understood meaningfully as
accessory penalties are inherent. rights with core values that our democratic system protects, if these rights will not
77 Article 41, Revised Penal Code. include the right to vote and be voted for. To exclude the rights of suffrage and
324 candidacy from the restoration of civil and political rights shall likewise signify a
diminution, other than what the Constitution allows, of the scope of pardon that
324 SUPREME COURT REPORTS ANNOTATED the President can extend under the 1987 Constitution. Significantly, this
Risos-Vidal vs. Commission on Elections Constitution itself did not yet exist when the Revised Penal Code was passed so
civil and political rights.” I take the view that this restoration already includes that this Code could not have taken into account the intent of the framers of this
the restoration of the right to vote and be voted for as these are rights subsumed Constitution to maintain the plenary nature of the pardoning power.81
within the “political rights” that the pardon mentions; in the absence of any
express accompanying reservation or contrary intent, this formulation grants a B.3(a)(ii) Harmonization of Conflicting Provisions.
full restoration that is coterminous with the remitted principal penalty of reclusion
perpetua. Where seeming conflicts appear between or among provisions of law,
Risos-Vidal objects to this reading of Article 36 on the ground that Section particularly between a constitutional provision and a statute, the primary rule in
3678 and 4179 expressly require that the restoration be made specifically of the understanding these seeming conflicts is to harmonize them, giving effect to
right to vote and to be voted upon. J. Leonen supports Risos-Vidal’s arguments both provisions within the limits of the constitutional provision.82
and opines that civil and political rights collectively constitute a bundle of rights As posed in this case, this seeming conflict occurs between the terms and intent
and the rights to vote and to be voted upon are specific rights expressly singled out of the current Constitution to give the President the full power to grant executive
and required by these RPC articles and thus must be expressly restored. It posits clemency, limited only by the terms of the Constitution itself, on the one hand, and
too that these are requirements of form that do not diminish the pardoning power the collective application of the Articles 36 and 41 of the RPC, on the other.
of the President. In my view, harmonization occurs under the Erap pardon by giving due
I note in this juncture that J. Leonen’s position on the requirements of Articles recognition to the essentially plenary nature of
36 and 41, is a very literal reading of 80-year-old _______________
provisions124 whose interpretations have been overtaken by events and should now
be updated. As I discussed 81 See: discussions and footnotes at pp. 304-307 and 316-318.
_______________ 82 Teehankee v. Rovira, 75 Phil. 634, 643 (1945).
326
326 SUPREME COURT REPORTS ANNOTATED
Risos-Vidal vs. Commission on Elections Casido,88 and in 2000, in People v. Patriarca, Jr.89 — which cited with approval
the President’s pardoning power under Section 19, Article VII of the this Court’s statement in Barrioquinto v. Fernandez.90
Constitution, while giving effect to the RPC intent to make clear in the terms of J. Leonen added that the Monsanto inclusion must also be deemed
the pardon the intent to restore the convict’s rights to vote and to be voted upon, superseded by the Court’s ruling in Romeo Jalosjos v. COMELEC91 which
as a matter of form that is satisfied by reference to the restoration of political rights recognized that “one who is previously convicted of a crime punishable by reclusion
that, as now understood internationally and domestically, include the restoration perpetua or reclusion temporal continues to suffer the accessory penalty of
of the right to vote and to be voted upon. Understood in this manner, the RPC perpetual absolute disqualification even though pardoned as to the principal
provisions would not be constitutionally infirm as they would not diminish the penalty, unless the accessory penalty shall have been expressly remitted in the
pardoning power of the President. pardon.”
To address another concern that J. Leonen expressed, no need exists to require _______________
the President to grant the “full” restoration of Erap’s civil and political rights as
this kind of interpretation renders illusory the extent of the President’s pardoning 86 Id., at p. 41.
power by mere play of words. In the absence of any contrary intent, the use of the 87 Id.
modifier “full” is an unnecessary surplusage. 88 336 Phil. 344; 269 SCRA 360 (1997).
89 395 Phil. 690; 341 SCRA 464 (2000).
B.3(a)(iii) The Monsanto v. Factoran Case. 90 82 Phil. 642 (1949).
91 G.R. No. 205033, June 18, 2013, 698 SCRA 742.
I also address J. Leonen’s discussion of the Monsanto v. Factoran case. 328
Part and parcel of the topic “RPC Perspectives” is the position that J. Leonen 328 SUPREME COURT REPORTS ANNOTATED
took in Monsanto — in the course of repudiating Cristobal v. Labrador,83 Pelobello
Risos-Vidal vs. Commission on Elections
v. Palatino84 and Ex Parte Garland.85 J. Leonen took notice of the statement
I disagree with these positions, particularly with the statement that
in Monsanto that “[t]he better considered cases regard full pardon x x x as
the Monsanto inclusion was overturned by Casido, Patriarca (citing Barrioquinto)
relieving the party from all the punitive consequences of his criminal
and Romeo Jalosjos.
act, including the disqualification or disabilities based on finding of guilt.” J.
I maintain that the inclusion was the ratio decidendi of the case and was not
Leonen went on to state that this “including phrase or inclusion” is not an
just a passing statement of the Court. In Monsanto, the Court emphasized that a
authority in concluding that the grant of pardon ipso facto
pardon may remit all the penal consequences of a criminal indictment.92 The Court
_______________
even applied this statement by categorically ruling that the full pardon granted
to Monsanto “has resulted in removing her disqualification from holding
83 71 Phil. 34 (1940). public employment.”93 In fact, J. Leonen’s interpretation of Monsanto is
84 72 Phil. 441 (1940). misleading; his conclusion on the superiority
85 71 U.S. 833 (1866). of Casido, Patriarca and Jalosjos over Monsanto is likewise misplaced and
327 without basis.
VOL. 747, JANUARY 21, 2015 327 For clarity, the inclusion phrase is part of the Court’s discussion
Risos-Vidal vs. Commission on Elections in Monsanto and was made in the context that although the Court repudiated
remits the accessory disqualifications or disabilities imposed on a convict the Garland ruling (as cited in Pellobello and Cristobal) that pardon erases the
regardless of whether the remission was explicitly stated, 86citing the following guilt of the convict, the Court still acknowledged that pardon may remove all the
reasons: punitive consequences of a convict’s criminal act, including the
First, J. Leonen maintains that the inclusion was not a pronouncement of a disqualifications or disabilities based on the finding of guilt.94
prevailing rule but was merely a statement made in the course of a comparative The complete discussion of the Court in Monsanto where J.
survey of cases during which the Court manifested a preference for “authorities Leonen selectively lifted the inclusion for his own purposes is as follows:95
[that reject] the unduly broad language of the Garland case.”87 Having disposed of that preliminary point, we proceed to discuss the
Second, the footnote to the inclusion indicates that Monsanto relied on a case effects of a full and absolute pardon in relation to the decisive question
decided by a United States court. Thus, Monsantowas never meant as a of whether or not the plenary pardon had the effect of removing
summation of the controlling principles in this jurisdiction and did not consider _______________
Articles 36 and 41 of the RPC.
Lastly, J. Leonen argues that even granting that the inclusion articulated a 92 Supra note 48 at p. 202; p. 199.
rule, this inclusion, made in 1989, must be deemed to have been abandoned, in 93 Id., at p. 204; p. 201.
light of the Court’s more recent pronouncements — in 1997, in People v. 94 Id., at p. 201; p. 198.
95 Id., at pp. 199-204; pp. 197-201. blots out the guilt of an individual and that once he is absolved, he should
329 be treated as if he were innocent. For whatever may have been the judicial
VOL. 747, JANUARY 21, 2015 329 dicta in the past, we cannot perceive how pardon can produce such “moral changes”
as to equate a pardoned convict in character and conduct with one who has
Risos-Vidal vs. Commission on Elections constantly maintained the mark of a good, law-abiding citizen.
the disqualifications prescribed by the Revised Penal Code. xxxx
xxxx Pardon granted after conviction frees the individual from all the penalties and
The Pelobello v. Palatino and Cristobal v. Labrador cases, and several others legal disabilities and restores him to all his civil rights. But unless expressly
show the unmistakable application of the doctrinal case of Ex Parte Garland, grounded on the person’s innocence (which is rare), it cannot bring back lost
whose sweeping generalizations to this day continue to hold sway in our reputation for honesty, integrity and fair dealing. This must be constantly kept in
jurisprudence despite the fact that much of its relevance has been downplayed by mind lest we lose track of the true character and purpose of the privilege.
later American decisions. Consider the following broad statements: 331
A pardon reaches both the punishment prescribed for the offense and the guilt
VOL. 747, JANUARY 21, 2015 331
of the offender; and when the pardon is full, it releases the punishment and blots
out of existence the guilt, so that in the eye of the law the offender is as innocent Risos-Vidal vs. Commission on Elections
as if he had never committed the offense. If granted before conviction, it prevents Thus, notwithstanding the expansive and effusive language of
any of the penalties and disabilities, consequent upon conviction, from attaching; the Garland case, we are in full agreement with the commonly held
if granted after conviction, it removes the penalties and disabilities and restores opinion that pardon does not ipso facto restore a convicted felon to public
him to all his civil rights; it makes him, as it were, a new man, and gives him a office necessarily relinquished or forfeited by reason of the conviction
new credit and capacity. although such pardon undoubtedly restores his eligibility for
Such generalities have not been universally accepted, recognized or approved. appointment to that office.
The modern trend of authorities now rejects the unduly broad language of xxxx
the Garland case (reputed to be perhaps the most extreme statement which has For petitioner Monsanto, this is the bottom line: the absolute
been made on the effects of a pardon). To our mind, this is the more realistic disqualification or ineligibility from public office forms part of the
approach. While a pardon has generally been regarded as blotting out the existence punishment prescribed by the Revised Penal Code for estafa thru
of guilt so that in the eye of the law the offender is as innocent as though he never falsification of public documents. It is clear from the authorities referred to
committed the offense, it does not operate for all purposes. The very essence of a that when her guilt and punishment were expunged by her pardon, this
pardon is forgiveness or remission of guilt. Pardon implies guilt. It does not erase particular disability was likewise removed. Henceforth, petitioner may apply
the fact of the commission of the crime and the conviction thereof. It does not wash for reappointment to the office which was forfeited by reason of her conviction. And
out the moral stain. It involves forgiveness and not forgetfulness. in considering her qualifications and suitability for the public post, the facts
330 constituting her offense must be and should be evaluated and taken into account
330 SUPREME COURT REPORTS ANNOTATED to determine ultimately whether she can once again be entrusted with public
funds. Stated differently, the pardon granted to petitioner has resulted in
Risos-Vidal vs. Commission on Elections removing her disqualification from holding public employment but it
The better considered cases regard full pardon (at least one not based on the cannot go beyond that. To regain her former post as assistant city treasurer,
offender’s innocence) as relieving the party from all the punitive consequences of she must reapply and undergo the usual procedure required for a new
his criminal act, including the disqualifications or disabilities based on the appointment. [Emphasis and underscoring supplied; citations omitted]
finding of guilt. But it relieves him from nothing more. “To say, however, that
the offender is a ‘new man,’ and ‘as innocent as if he had never committed the As against J. Leonen’s interpretation of the Monsanto ruling above, I deduce
offense’; is to ignore the difference between the crime and the criminal. A person the following contrary points:
adjudged guilty of an offense is a convicted criminal, though pardoned; he may be First, contrary to J. Leonen’s statement, the Court took into consideration
deserving of punishment, though left unpunished; and the law may regard him as the provisions of the RPC in arriving at its ruling in Monsanto.
more dangerous to society than one never found guilty of crime, though it places To reiterate, Monsanto exhaustively discussed the effects of a full and absolute
no restraints upon him following his conviction.” pardon on the accessory penalty of dis-
xxxx 332
In this ponencia, the Court wishes to stress one vital point: While we are
332 SUPREME COURT REPORTS ANNOTATED
prepared to concede that pardon may remit all the penal consequences of
a criminal indictment if only to give meaning to the fiat that a pardon, Risos-Vidal vs. Commission on Elections
being a presidential prerogative, should not be circumscribed by qualification. Hence, the Court ruled that the full pardon granted to Monsanto
legislative action, we do not subscribe to the fictitious belief that pardon resulted in removing her disqualification from holding public employment under
the RPC but did not result in her automatic reinstatement as Assistant City concurrence of Congress, and it is a public act of which the courts should take
Treasurer due to the repudiation of the Garland ruling cited judicial notice. Pardon is granted to one after conviction; while amnesty is to classes
in Pelobello and Labrador. of persons or communities who may be guilty of political offenses, generally before
In contrast, the ruling of the Court in Casido96 and Patriarca,97which both or after the institution of the criminal prosecution and sometimes after
cited Barrioquinto,98 all related to amnesty conviction. Pardon looks forward and relieves the offender from the
_______________ consequences of an offense of which he has been convicted, that is, it
abolishes or forgives the punishment, and for that reason it does “nor
96 In the Court’s July 30, 1996 resolution, it ruled that the conditional pardons work the restoration of the rights to hold public office, or the right of
granted in this case to accused-appellants William Casido and Franklin Alcorin suffrage, unless such rights be expressly restored by the terms of the
are void for having been extended during the pendency of their instant appeal. pardon,” and it “in no case exempts the culprit
However, subsequent to this, the applications for amnesty of accused-appellants _______________
were granted by the National Amnesty Commission on February 22, 1996. Issue:
Whether or not Casido and Alcorin may now be released on the basis of the Jimenez have admitted having committed the offense, because Barrioquinto
amnesty granted to them. alleged that it was Hipolito Tolentino who shot and killed the victim, they cannot
97 Accused-appellant Jose Patriarca is a member of the New People’s Army. invoke the benefits of amnesty. Issue: Whether or not petitioners may not be
He was convicted of murder for killing persons in pursuit of his group’s political covered by the amnesty because they have not pleaded guilty to the offense
belief. Subsequently, accused-appellant applied for amnesty under Proclamation charged.
No. 724 amending Proclamation No. 347, dated March 25, 1994, entitled “Granting 99 People v. Casido, supra note 88 at pp. 351-352; p. 368.
Amnesty to Rebels, Insurgents, and All Other Persons Who Have or May Have 100 People v. Patriarca, Jr., supra note 89 at p. 699; p. 472.
Committed Crimes Against Public Order, Other Crimes Committed in 334
Furtherance of Political Ends, and Violations of the Article of War, and Creating a 334 SUPREME COURT REPORTS ANNOTATED
National Amnesty Commission.” His application was favorably granted by the
National Amnesty Board.Issue: Whether or not Patriarca is entitled to amnesty. Risos-Vidal vs. Commission on Elections
98 Petitioners Norberto Jimenez and Loreto Barrioquinto were charged with
the crime of murder. Subsequently, Proclamation No. 8, dated September 7, 1946, from the payment of the civil indemnity imposed upon him by the
which grants amnesty in favor of all persons who may be charged with an act sentence.” (article 36, Revised Penal Code) While amnesty looks
penalized under the Revised Penal Code in furtherance of the resistance to the backward and abolishes and puts into oblivion the offense itself, it so
Japanese forces or against persons aiding in the war efforts of the enemy. overlooks and obliterates the offense with which he is charged that the
After a preliminary hearing had started, the Amnesty Commission issued an person released by amnesty stands before the law precisely as though he
order returning the cases of the petitioners to the Court of First Instance of had committed no offense.101 [Emphasis supplied]
Zamboanga, without deciding whether or not they are entitled to the benefits of he
said Amnesty Proclamation, on the ground that inasmuch as neither Barrioquinto As between Monsanto, involving a full pardon, and the three amnesty cases
nor (Casido, Patriarca and Barrioquinto), Monsantoclearly applies to the pardon that
333 is involved in the present case where the dispositive portion made a restoration of
Erap’s civil and political rights. Note that the pardon described in the amnesty
VOL. 747, JANUARY 21, 2015 333
cases does not even identify whether the pardon being described was absolute or
Risos-Vidal vs. Commission on Elections conditional. In fact, the portion cited by the majority in the amnesty cases merely
and not to pardon. The paragraph in Casido and Patriarca that J. Leonen repeated what Article 36 of the RPC provides. Monsanto, on the other hand and to
quoted to contradict the Monsanto inclusion is part of the Court’s attempt the contrary, took into consideration these RPC provisions on disqualifications in
in Casido and Patriarca to distinguish amnesty from pardon. relation with the effects of a full pardon.
For clarity, below is the complete paragraph From this perspective, J. Leonen is thus careless and misleading in
in Casido99 and Patriarca100 where J. Leonen lifted the portion (highlighted in immediately concluding that the Monsanto ruling on “inclusion” was overturned
bold) that he used to contradict the Monsanto inclusion: by the amnesty cases.
The theory of the respondents, supported by the dissenting opinion, is Similarly, contrary to J. Leonen’s argument, the ruling in Romeo Jalosjos v.
predicated on a wrong contention of the nature or character of an amnesty. COMELEC (Jalosjos) did not supersede the Monsantoruling cited above.
Amnesty must be distinguished from pardon. In Jalosjos,102 the Court merely reconciled the apparent conflict between
Pardon is granted by the Chief Executive and as such it is a private act which Section 40(a)103 of the Local Government
must be pleaded and proved by the person pardoned, because the courts take no _______________
notice thereof; while amnesty by Proclamation of the Chief Executive with the
101 As cited in Barrioquinto v. Fernandez, supra note 90 at pp. 646-647. remitted his disqualification to run for public office and to vote as it
102 Jalosjos v. Comelec, supra note 91at pp. 759-760. expressly restored him to his civil and political rights.
103 Sec. 40. Disqualifications.—The following persons are disqualified from The Office of the Solicitor General succinctly expressed the Monsanto ratio
running for any elective local position: decidendi when it said that the Court, despite ruling against Monsanto,
(a) Those sentenced by final judgment for an offense involving moral “nevertheless reaffirmed the well-settled doctrine that the grant of pardon also
turpitude or for an offense punishable by one (1) year or removes one’s absolute disqualification or ineligibility to hold public office.”
335
VOL. 747, JANUARY 21, 2015 335 B.3(b) Arguments via the Interpretative Route.

Risos-Vidal vs. Commission on Elections Alternatively, if indeed the third Whereas clause had injected doubt in the
Code and Article 30104 of the RPC, which provides for the effects of perpetual express and unequivocal restoration made, then two interpretative recourses can
or temporary absolute disqualification. be made to determine how this doubt can be resolved.
The Court held in Jalosjos that Article 41 of the RPC expressly states that one
who was previously convicted of a crime punishable by reclusion B.3(b)(i) The Liberal Mode of Interpretation.
perpetua or reclusion temporal continues to suffer the accessory penalty of
perpetual absolute disqualification even though pardoned as to the principal The first approach is to use by analogy the ruling and reasoning in the case
penalty, unless this accessory penalty had been expressly remitted in the pardon. of Frank v. Wolfe106 which involved commutation of sentence, a lesser grant but
In Jalosjos, the accessory penalty had not been expressly remitted in the Order of which is an act of grace nevertheless.
Commutation or by any subsequent pardon; hence, Jalosjos’ disqualification to run The Court held in this case that “it is a principle universally recognized
for elective office was deemed to subsist.105 that all such grants are to the construed favorably to the grantee, and
Jalosjos could be harmonized with Monsanto in that the latter also recognized strictly as to the grantor, not only because they partake of the nature of a
the provisions of the RPC on the accessory penalty of disqualification but holds deed, and the general rule of interpretation that the terms of a written
that the full pardon remits this disqualification. instrument evidencing with especial force to grants or pardon and
In the present case, Erap’s pardon fully complied with the RPC requirements commutations, wherein the grantor executes the instrument with little or no
for the express remission of the accessory right on the part of the grantee to intervene in its execution or dictate
_______________ _______________

more of imprisonment, within two (2) years after serving sentence. (Emphasis 106 11 Phil. 466, 470-471, October 21, 1908.
and underscoring supplied) 337
104 Art. 30. Effects of the penalties of perpetual or temporary absolute
VOL. 747, JANUARY 21, 2015 337
disqualification.—The penalties of perpetual or temporary absolute
disqualification for public office shall produce the following effects: Risos-Vidal vs. Commission on Elections
1. The deprivation of the public offices and employments which the offender its terms, but because of the very nature of the grant itself as an act of grace
may have held, even if conferred by popular election. and clemency. (Bishop Crim. Law, Sec. 757, and cases cited: Osborn v. U.S., 91
2. The deprivation of the right to vote in any election for any popular office or U.S. 474; Lee v. Murphy, 22 Grat. Va., 789) Applying the rule we think that, if it
to be elected to such office. had been the intention of the commuting authority to deprive the prisoner of the
3. The disqualification for the offices or public employments and for the beneficent provisions of Act No. 1533,107 language should have been used and
exercise of any of the rights mentioned. In case of temporary disqualification, such would have been used which would leave no room for doubt as to its meaning, and
disqualification as is comprised in paragraphs 2 and 3 of this Article shall last would make clearly manifest the object intended.”
during the term of the sentence. This approach, read with the plain meaning rule of statutory interpretation
4. The loss of all rights to retirement pay or other pension for any office (i.e., that an instrument should, as a first rule, be read in accordance with the plain
formerly held. (Emphasis and underscoring supplied) meaning that its words import)108cannot but lead us to the conclusion that the
105 Jalosjos v. COMELEC, supra note 91 at pp. 762-763. Risos-Vidal’s “third Whereas Clause” objection should be thrown out for lack of
336 merit.
336 SUPREME COURT REPORTS ANNOTATED
B.3(b)(ii) The Vox Populi Line of Cases.
Risos-Vidal vs. Commission on Elections
penalty of perpetual absolute disqualification as the pardon in fact restored The second approach is to accept that such doubt cannot be resolved within
him to his civil and political rights. In this light, the Monsanto ruling still the four corners of the written pardon and resort should be taken to the external
applies: while the PGMA pardon does not erase Erap’s guilt, it nonetheless
surrounding circumstances that followed the grant and the interests involved (i.e., VOL. 747, JANUARY 21, 2015 339
protection of the interests of the electorate and the recognition of vox populi), as
already discussed above and supplemented by the rulings below. Risos-Vidal vs. Commission on Elections
In the Fernandez v. House of Representatives Electoral Tribunal109 line of cases of philosophy and perception of how to interpret and apply the laws relating to
involving the issue of ineligibility based on the residency requirements, that Court elections; literal or liberal; the letter or the spirit; the naked provision or the
declared that it must exercise utmost caution before disqualifying a ultimate purpose; legal syllogism or substantial justice; in isolation or in context
_______________ of social conditions; harshly against or gently in favor of the voter’s obvious
choice. In applying election laws, it would be far better to err in favor of
107 An Act Providing for the Diminution of Sentences Imposed Upon popular sovereignty than to be right in complex but little understood
Prisoners Convicted of Any Offense and Sentenced for a Definite Term of More legalisms.
than Thirty Days and Less than Life in Consideration of Good Conduct and In Rulloda v. COMELEC, et al.114 involving substitution of candidates, the
Diligence. Court ruled that the purpose of election laws is to give effect to, rather than
108 Bolos v. Bolos, G.R. No. 186400, October 20, 2010, 634 SCRA 429, 437. frustrate, the will of the voters. It is a solemn duty to uphold the clear and
109 G. R. No. 187478, December 21, 2009, 608 SCRA 733, 753. unmistakable mandate of the people. It is well-settled that in case of doubt,
338 political laws must be so construed as to give life and spirit to the popular mandate
freely expressed through the ballot.
338 SUPREME COURT REPORTS ANNOTATED Technicalities and procedural niceties in election cases should not be made to
Risos-Vidal vs. Commission on Elections stand in the way of the true will of the electorate. Laws governing election contests
winning candidate, shown to be the clear choice of the constituents to must be liberally construed to the end that the will of the people in the choice of
represent them in Congress. public officials may not be defeated by mere technical objections. 115
Citing Frivaldo v. COMELEC,110 the Court held that time and again it has Election contests involve public interest, and technicalities and procedural
liberally and equitably construed the electoral laws of our country to give barriers must yield if they constitute an obstacle to the determination of the true
fullest effect to the manifest will of our people, for in case of doubt, will of the electorate in the choice of their elective officials. The Court frowns upon
political laws must be interpreted to give life and spirit to the popular any interpretation of the law that would hinder in any way not only the free and
mandate freely expressed through the ballot. Otherwise stated, legal niceties intelligent casting of the votes in an election but also the correct ascertainment of
and technicalities cannot stand in the way of the sovereign will. the results.116
Furthermore, to successfully challenge a winning candidate’s These rulings, applicable in a situation of doubt yields the conclusion that the
qualifications, the petitioner must clearly demonstrate that the doubt, if any, in the present case should be resolved in Erap’s favor.
ineligibility is so patently antagonistic to constitutional and legal _______________
principles that overriding such ineligibility and thereby giving effect to
the apparent will of the people, would ultimately create greater prejudice 114 443 Phil. 649, 654-655; 395 SCRA 535, 540 (2003).
to the very democratic institutions and juristic traditions that our 115 Id.
Constitution and laws so zealously protect and promote. 116 Id.
Another significant ruling to consider is Malabaguio v. COMELEC, et 340
al.111 involving the appreciation of ballots, the Court, citing its ruling in Alberto v. 340 SUPREME COURT REPORTS ANNOTATED
COMELEC,112 declared that election cases involve public interest; thus, laws
governing election contests must be liberally construed to the end that the Risos-Vidal vs. Commission on Elections
will of the people in the choice of public officials may not be defeated by B.4. Conclusions on Pardon and Grave Abuse of Discretion.
mere technical objections.
The Court further reiterated in Maruhom v. COMELEC, et al.113its ruling that In the light of all the above arguments on pardon and the refutation of the
the question really boils down to a choice positions of the petitioner Risos-Vidal, I submit to the Court that under the Rule
_______________ 65 standard of review discussed above, no compelling reason exists to conclude
that the COMELEC committed grave abuse of discretion in ruling on the pardon
110 G.R. No. 120295, June 28, 1996, 257 SCRA 727, 770-771. aspect of the case.
111 400 Phil. 551, 567; 346 SCRA 699, 712 (2000). No grave abuse of discretion could have been committed as the COMELEC was
112 G.R. No. 132242, July 27, 1999, 311 SCRA 215, 222; See also Punzalan v. correct in its substantive considerations and conclusions. As outlined above, Erap
COMELEC, G.R. No. 126669, April 27, 1998, 289 SCRA 702, 720. indeed earned the right to vote and to be voted for from the pardon that PGMA
113 387 Phil. 491, 516; 331 SCRA 473, 494-495 (2000). granted him. It is the only reasonable and logical conclusion that can be reached
339 under the circumstances of the case.
C. 118 See page 8 of the COMELEC, Second Division Resolution dated January
20, 2010 in SPA No. 09-024(DC) entitled Rev. Elly
The Objections Relating to the 2010 COMELEC Rulings in the 342
Disqualification Trilogy. VOL. 747, JANUARY 21, 2015 342

As I previously discussed, despite the ponencia’s resolution that the Risos-Vidal vs. Commission on Elections
COMELEC did not gravely abuse its discretion in ruling on the issue of Erap’s On December 28, 2009, Petitioner Pamatong submitted his Position Paper on
pardon, another crucial issue to be resolved is whether or not the COMELEC Joseph E. Estrada and Gloria M. Arroyo, asking the questions: Are they above the
gravely abused its discretion in relying on its 2010 rulings in dismissing the Risos- law? The Petitioner Pamatong took the absolutist point of view that former
Vidal petition. President Joseph Ejercito Estrada is banned forever from seeking the same
This issue must be resolved in the present case as the assailed COMELEC position of President of the Republic having been previously elected as such
rulings did not rule specifically on the issue of Erap’s pardon but resolved instead President. He also espoused the idea that Respondent Gloria Macapagal Arroyo as
that the issue of Erap’s pardon is already a previously “settled matter,” referring the sitting President is forever banned from seeking any other elective office,
to the consolidated COMELEC Rulings in SPA No. 09-028 (DC) and SPA No. 09- including a post such as member of the House of Representatives.
104 (DC), entitled Atty. Evilio C. Pormento v. Joseph Ejercito Estrada and In Re: xxxx
Petition to Disqualify Estrada Ejercito, Joseph M. From Running As President Due Furthermore, Petitioner maintains that the pardon granted Estrada
341 was conditioned on his promise not to run for any public office again. It
was not a full pardon but was a conditional one. The exercise of executive
VOL. 747, JANUARY 21, 2015 341
clemency was premised on the condition that former President Estrada should not
Risos-Vidal vs. Commission on Elections run again for Office of the President of the Philippines or for any other public
to Constitutional Disqualification and Creating Confusion to the Prejudice of office.119
Estrada, Mary Lou B. xxxx
As I will discuss below, the COMELEC did not gravely abuse its discretion in Furthermore, there is absolutely no indication that the executive
relying on its 2010 disqualification rulings in dismissing Risos-Vidal’s petition. clemency exercised by President Gloria Arroyo to pardon Former President
Estrada was a mere conditional pardon. It clearly stated that the Former
C.1. The Trilogy of Disqualification Cases in 2010. President is “restored to his civil and political rights” and there is nothing
in the same which limits the restoration. The only thing stated therein that
As narrated above,117 Erap’s 2010 presidential candidacy gave rise to three may have some bearing on the supposed condition is that statement in the
cases — the Pamatong, Pormento and Mary Lou Estrada cases — all aimed at whereas clause that contained the following: Whereas, Jo-
disqualifying him. The COMELEC duly ruled in all these cases. If the effects of _______________
these rulings have been muddled at all in the understanding of some, the confusion
might have been due to the failure to look at the whole 2010 disqualification scene Velez B. Lao Pamatong, Esq v. Joseph Ejercito Estrada and Gloria Macapagal-
and to see how these trilogy of disqualification cases interacted with one another. Arroyo. This Resolution was attached as Exhibit “4” to Annex “E” of the
The three cases, appropriately given their respective docket numbers, were Memorandum that Petitioner Risos-Vidal submitted to the Court.
heard at the same time. While they were essentially based on the same grounds 119 Id.
(hence, the description trilogy or a series of three cases that are closely related 343
under a single theme — the disqualification of Erap), only the Pormento and Mary
VOL. 747, JANUARY 21, 2015 343
Lou Estradacases were formally consolidated; the Pamatong case, the first of the
cases, was not included because Pamatong also sought the disqualification from Risos-Vidal vs. Commission on Elections
public office of PGMA on the ground that she is also constitutionally barred from seph Estrada has publicly committed to no longer seek any elective
being reelected. position or office, but that is not a condition but is merely part of the
Petitioner Pamatong expressly put in issue Erap’s fitness to be a candidate preliminary statement. It cannot therefore serve to restrict the operation
based on his previous conviction for plunder and the terms of the pardon extended of or prevail over the explicit statement in the executive clemency which
him by PGMA; the COMELEC, for its part, directly ruled on the matter. To quote restored all of Estrada’s civil and political rights, including the “right to
the relevant portions of the COMELEC Resolution in Pamatong:118 vote and to be voted for public office” for the position of the Presidency.
_______________ This executive clemency granted to the former President being absolute
and unconditional and having been accepted by him, the same can no
117 See pp. 283-287. longer be revoked.120 [Emphasis supplied]
How the three cases exactly related to one another in terms of the issues posed suspend elections shall become final and executory after the lapse of five (5) days
is described by the COMELEC in its consolidated Resolution in the cases from their promulgation, unless restrained by the Supreme Court.
of Pormento and Mary Lou Estrada, as follows:121 345
However, as to the substantive aspect of the case, the Respondent’s Answer VOL. 747, JANUARY 21, 2015 345
basically raises and repleads the same defenses which were relied upon in SPA 09-
024, except for the additional ground that “the grant of executive clemency Risos-Vidal vs. Commission on Elections
removed all legal impediments that may bar his candidacy for the Of the three, petitioner Pormento went one step further to assail the final
Presidency.”122 These grounds consisted of: COMELEC ruling before this Court. His effort did not bear fruitful result as the
(a) The “President” being alluded to under section 4 of Article VII of the 1987 Court dismissed his petition for mootness — when the Court issued its ruling, Erap
Constitution refers to the incumbent President; had lost the 2013 presidential elections.
_______________ In the dismissal of the Pormento petition before this Court [G.R. No. 191188],
a nagging issue that has left some uncertainty is the effect of the dismissal on the
120 Id., at p. 22. COMELEC’s Pormento ruling. This assailed COMELEC resolution tackled two
121 See pp. 5-6 of the COMELEC, Second Division Resolution on SPA No. 09- issues: 1) the constitutional prohibition on reelection; and 2) the nature of Erap’s
028 (DC), attached as Annex “O” to Memorandum of Intervenor Lim. pardon and its effect on his qualification to run for an elective public office or as
122 The original grounds in SPA 09-024 as cited in Erap’s Answer in President.
Pamatong’s case did not include the issue of pardon which Pamatong later added The Court, however, in dismissing the case, focused its discussions solely on
in his Position Paper. the issue of the constitutional ban on reelection and ruled that this issue had been
344 rendered moot by the supervening event of Erap’s loss in the 2010 elections; the
Court did not discuss or even mention the issue of whether the COMELEC gravely
344 SUPREME COURT REPORTS ANNOTATED abused its discretion in ruling that Erap’s pardon was absolute and had restored
Risos-Vidal vs. Commission on Elections his right to run for the Presidency.
(b) The Prohibition does not apply to the person who merely serves a tenure In this situation, the assailed COMELEC ruling simply becomes, not only final
and not a complete term; and executory, but unassailable. No appeal is available as an appeal is barred by
(c) Joseph Estrada is not running for reelection but is “running again” for the the Constitution.125 No petition for certiorari is likewise available unless another
same position of President of the Philippines; petition had been filed within the period for filing allowed by the Rules of
(d) The Provisions of section 4 (1st par), Article VII of the 1987 Constitution is Court.126 Thus, the COMELEC rulings
clear, unequivocal and unambiguous; hence not subject to any interpretation; _______________
(e) The evil sought to be prevented is directed against the incumbent President;
(f) The sovereignty of the people should be paramount; and 125 Section A(7), Article IX, 1987 Constitution.
(g) The grant of executive clemency removed all legal impediments 126 Id., and Section 3, Rule 64 which provides that the petition
that may bar his candidacy for the presidency. [Emphasis supplied] for certiorari shall be filed within thirty (30) days from notice of the judgment or
final order or resolution sought to be reviewed. The filing of a motion for new trial
As arranged during the COMELEC’s common hearing on the trilogy, separate or reconsideration of said judgment or final order or resolution, if allowed under
decisions were rendered simultaneously.123 They all touched on the issue of the procedural rules of the Commission concerned, shall interrupt the period
pardon. herein fixed. If the motion is denied, the aggrieved party may file the petition
As likewise already explained above, all three cases became final, executory within the remaining period, but which shall not be less than five (5) days in any
and unappealable five (5) days after its promulgation, pursuant to Section 3, Rule event, reckoned from notice of denial.
37 of the COMELEC Rules of Procedure.124 Since all the petitioners filed their 346
respective motions for reconsideration, finality was reckoned from the denial of 346 SUPREME COURT REPORTS ANNOTATED
these motions.
_______________ Risos-Vidal vs. Commission on Elections
on the trilogy of disqualification cases fully stand, enforceable according to
123 Supra notes 2 at p. 7 and 4 at pp. 7-8. their terms. From the perspective of the Court, no enforceable ruling was made
124 Section 3, Rule 37 of the COMELEC Rules of Procedure states: nor any principle of law established. In other words, the final ruling to be reckoned
Decisions Final After Five Days.—Decisions in pre-proclamation cases and with in any future dispute is effectively the COMELEC ruling.
petitions to deny due course to or cancel certificates of candidacy, to declare a
candidate as nuisance candidate or to disqualify a candidate, and to postpone or C.2. The Risos-Vidal Petition and its Objections against Erap’s Status.

C.2(a) The Objections and its Fallacies.


I disagree with J. Leonen. As I earlier pointed out, we must review the
The Risos-Vidal petition, fully supported by J. Leonen, objects to the binding COMELEC’s decision using the standard of grave abuse of discretion: we nullify
effect of the 2010 disqualification trilogy decisions, on the claim that res the COMELEC ruling if it gravely abused its discretion in ruling on the present
judicata did not apply because pardon was not an issue ruled upon in 2010. case; if no grave abuse of discretion existed, the Risos-Vidal petition should be
This may have partly stemmed from the statement of issues in the 2010 dismissed instead of being granted.
COMELEC Resolution in Pormento defining the issues common to Pormento As I will proceed to discuss below, the COMELEC did not gravely abuse its
and Mary Lou Estrada, disregarding the incidents that transpired in the trilogy discretion when it ruled in the pre-
and the issues that Erap raised in his Answer.127 Another source of confusion 348
perhaps was the fact that the COMELEC, in ruling on the 2013 Risos-Vidal 348 SUPREME COURT REPORTS ANNOTATED
petition, only cited the Pormento and Mary Lou Estrada cases.
The objections, in my view, do not take into account the sequence of events in Risos-Vidal vs. Commission on Elections
2010 on the filing of the disqualification cases, the relationship of the sent case that Erap’s pardon qualified him to run for an elective public
disqualification cases with one another, the law on the finality and binding office and that this issue is a previously “settled matter.”128 I say this
effect of rulings, and the reason for the COMELEC’s citation of the because the principle of res judicata, under either of its two modes
Pormento and Mary Lou Estradarulings in the subsequent 2013 Risos-Vidal — conclusiveness of judgment or bar by prior judgment — applies in the
petition. present case.
In Pamatong, Pamatong raised this issue in his Position Paper. Thus, Res judicata embraces two concepts: first, the bar by prior judgment under
pardon was an issue raised and ruled Rule 39, Section 47(b) of the Rules of Court; and second, the preclusion of a settled
_______________ issue or conclusiveness of judgment under Rule 39, Section 47(c) of the Rules of
Court. The COMELEC’s 2010 decision resolving whether Erap’s pardon allowed
127 See pp. 5-6 of the COMELEC, Second Division Resolution on SPA No. 09- him to run for elections precludes further discussion of the very same issue in the
028 (DC), attached as Annex “O” to Memorandum of Intervenor Lim. 2013 petition filed against his candidacy.
347 Under our review in the present case that is limited to the determination of
grave abuse of discretion and not legal error, I cannot agree with J. Leonen’s strict
VOL. 747, JANUARY 21, 2015 347 application of the requisites of bar by prior judgment. Jurisprudence has clarified
Risos-Vidal vs. Commission on Elections that res judicatadoes not require absolute identity, but merely substantial
upon. The same process took place in the subsequent consolidated cases identity. This consideration, under a grave abuse standard of review, leads me to
of Pormento and Mary Lou Estrada, so that the COMELEC itself, in its resolution the conclusion that we cannot reverse the COMELEC’s decision to apply res
of these cases, recognized that pardon was one of the issues that Erap raised and judicata, even if it meant the application of the concept of bar by prior judgment.
accordingly ruled on the matter. Significantly, the COMELEC rulings on the
matter of pardon in all three cases practically carried the same wording, C.2(b)(i) Issue preclusion or res judicata by conclusiveness of
revealing the COMELEC’s view that the cases constituted a trilogy that posed judgment.
practically the same issues, one of which is the pardon of Erap.
Issue preclusion (or conclusiveness of judgment) prevents the same parties
C.2(b) Res Judicata and its Application to the Case. and their privies from reopening an issue that has already been decided in a prior
case. In other words, once a right, fact, or matter in issue has been directly
The COMELEC Second Division, in dismissing the Risos-Vidal disqualification _______________
petition against Erap, emphasized that the issue of whether Erap’s pardon allowed
him to run for office had already been fully discussed in previous cases, and no 128 See page 2 of the COMELEC’s Resolution dated April 1, 2013 in SPA 13-
longer needed reexamination. The COMELEC additionally pointed out that 211 (DC) entitled Atty. Alicia Risos-Vidal v. Joseph Ejercito Estrada.
petitioner Risos-Vidal failed to provide sufficient reason to reverse its prior 349
decision. VOL. 747, JANUARY 21, 2015 349
J. Leonen noted that this Court is not barred by res judicata from revisiting
the issue of Erap’s pardon; we can review the COMELEC’s decision because there Risos-Vidal vs. Commission on Elections
is neither identity of the parties, of subject matters, and of causes of action in the adjudicated or necessarily involved in the determination of an action, it is
previous disqualification cases. J. Leonen also pointed out that the Court had not conclusively settled and cannot again be litigated between the parties and their
ruled with finality on the issue of Erap’s pardon in Pormento, because supervening privies, regardless of whether or not the claim, demand, or subject matter of the
events had rendered the case moot. two actions are the same.
For conclusiveness of judgment to apply, the second case should
have identical parties as the first case, which must have been settled by final
judgment. It does not, unlike the bar by previous judgment, need identity of political parties may file with the Law Department of the Commission a petition
subject matter and causes of action. to disqualify a candidate on grounds provided by law.
Note at this point, that Rule 37, Section 3 of the COMELEC Rules of Procedure 130 See Layos v. Fil-Estate Golf and Development, Inc., 583 Phil. 72, 106; 561
renders the COMELEC’s decision final and executory within five days after its SCRA 75, 106-107 (2008); Valencia v. RTC Quezon City, Branch 90, 262 Phil. 938,
promulgation, unless otherwise restrained by the Court. Neither of the two 947-948; 184 SCRA 80, 91 (1990).
COMELEC decisions involving Erap’s disqualification in 2010 had been restrained 351
by the Court; suffice it to say that the five-day period after promulgation of the VOL. 747, JANUARY 21, 2015 351
decisions in these cases had long passed.
Thus, the COMELEC did not err in considering its decisions in these cases — Risos-Vidal vs. Commission on Elections
all of which resolved the character of Erap’s pardon on the merits — to be final
and executory. That the Court refused to give due course to Pormento’s petition The COMELEC had already decided this issue, not once, but twice when it
assailing the COMELEC decision on the ground that its issues had been rendered separately but simultaneously decided Pamatong’s petition and the consolidated
moot by the 2010 elections, did not make the COMELEC’s decision any less final. petitions of Pormento and Estrada. In these cases, it gave the petitioners
In fact, Pormento was already final when it reached the Court, subject to the Pamatong, Pormento and Estrada ample opportunity to present their arguments
Court’s authority to order its nullification if grave abuse of discretion had regarding the nature of Erap’s pardon, to which Erap had also been allowed to
intervened. reply. After considering their arguments, the COMELEC issued its resolutions
On the requirement of identity of parties, Erap was the defendant in all four that the absolute nature of Erap’s pardon restored both his right to vote and be
cases. While the petitioners in these cases were not the same persons, all of them voted for.
represented the same interest as citizens of voting age filing their petitions to
ensure that Erap, an election candidate, is declared not qualified to run and hold C.2(b)(ii) Res judicata through bar by prior judgment.
office. Notably, Rule 25, Section 2 of the
350 Res judicata, by way of bar by prior judgment, binds the parties to a case, as
well as their privies to its judgment, and prevents them from re-litigating the same
350 SUPREME COURT REPORTS ANNOTATED
cause of action in another case. Otherwise put, the judgment or decree of the court
Risos-Vidal vs. Commission on Elections of competent jurisdiction on the merits concludes the litigation between the
COMELEC Rules of Procedure129 requires a prospective petitioner to be a parties, as well as their privies, and constitutes a bar to a new action or suit
citizen of voting age, or a duly registered political party, to file a petition for involving the same cause of action before the same or other tribunal.
disqualification, regardless of the position the candidate sought to be disqualified Res judicata through bar by prior judgment requires (a) that the former
aspires for. judgment be final; (b) that the judgment was rendered by a court of competent
We have had, in several instances, applied res judicata to subsequent cases jurisdiction; (c) that it is a judgment on the merits; and (d) that, between the first
whose parties were not absolutely identical, but substantially identical in and the second actions, there is identity of parties, subject matters, and causes of
terms of the interests they represent.130The cases filed against Erap’s candidacy in action.
the 2010 elections and in the 2013 elections share substantially the common These requisites were complied with in the present case.
interest of disqualifying Erap as a candidate; these petitioners also all contended
that Erap was not qualified to be a candidate because of his previous conviction of C.2(b)(ii)(a) COMELEC as Tribunal of Competent Jurisdiction.
plunder.
That the 2010 cases involved Erap’s bid for reelection for presidency and the That the COMELEC is a tribunal of competent jurisdiction in cancellation of
2013 cases revolved around his mayoralty bid is not, in my view, relevant for CoC and candidate disqualification cases is
purposes of applying collateral estoppel because the identity of the causes of action 352
or the subject matters are not necessary to preclude an issue already litigated and 352 SUPREME COURT REPORTS ANNOTATED
decided on the merits in a prior case. What is crucial for collateral estoppel to apply
to the second case is the identity of the issues between the two cases, which Risos-Vidal vs. Commission on Elections
had already been decided on the merits in the first case. All the cases seeking to mandated by the Constitution no less. Section 2(2), Article IX(C) of the
disqualify Erap from running hinged on his previous conviction and on arguments Constitution provides that:
characterizing his subsequent pardon to be merely conditional. Section 2. The Commission on Elections shall exercise the following
_______________ powers and functions:
xxxx
129 Sec. 2. Who May File Petition for Disqualification.—Any citizen of 2. Exercise exclusive original jurisdiction over all contests relating
voting age, or duly registered political party, organization or coalition of tothe elections, returns, and qualifications of all elective regional,
provincial, and city officials, and appellate jurisdiction over all contests
involving elective municipal officials decided by trial courts of general 132 GSIS v. Group Management Corp., G.R. No. 167000, June 8, 2011, 651
jurisdiction, or involving elective barangay officials decided by trial SCRA 279, 305.
courts of limited jurisdiction. [Emphasis and underscoring supplied] 133 Id.
134 Celendro v. Court of Appeals, 369 Phil. 1102, 1111; 310 SCRA 835, 844
Thus, the competence of the COMELEC to rule on these cases at the first (1999).
instance needs no further elaboration. 135 Id.
354
C.2(b)(ii)(b) Finality of the 2010 Disqualification Rulings. 354 SUPREME COURT REPORTS ANNOTATED

Some aspects of finality of the disqualification trilogy rulings have been Risos-Vidal vs. Commission on Elections
discussed above131 in terms of when COMELEC judgments become final and the inclusion of his name as a candidate, but more importantly, the public, by
recourses available to assail these judgments. But separately from these questions allowing the electorate to vote for him as a presidential candidate in 2010 and as
is the question of the effects of the finality of judgments. a mayoralty candidate in 2013.
Once a judgment attains finality, it becomes immutable and unalterable. It The difference of this case from the usual disqualification cases is that the 2010
may not be changed, altered or modified in any way even if the modification is for unalterable COMELEC ruling on the Erap pardon involved the issue of his
the purpose of correcting an erroneous conclusion of fact or law. This is political status binding on the whole world and has made his candidacy in the 2013
the “doctrine of finality of judgments” which binds the immediate parties elections and other future elections valid and immune from another petition for
and their privies in personal judgments; the disqualification based on his conviction for plunder. This topic will be discussed at
_______________ length below.

131 See p. 284. C.2(b)(ii)(c) Judgment on the Merits.


353
A judgment is on the merits when it determines the rights and liabilities of the
VOL. 747, JANUARY 21, 2015 353 parties based on the disclosed facts, irrespective of formal, technical or dilatory
Risos-Vidal vs. Commission on Elections objections.136
whole world in judgments in rem; and even the highest court of the In Pamatong’s petition to cancel and deny due course to Estrada’s CoC 137 for
land as to their binding effect.132 the position of President in the 2010 elections, the issue of pardon was clearly
This doctrine is grounded on fundamental considerations of public policy and raised and argued by the parties, resulting in the COMELEC resolution
sound practice and that, at the risk of occasional errors, the judgments or orders quoted above, specifically ruling that the Erap pardon was absolute and
of courts must become final at some definite time fixed by law; otherwise, there not conditional, entitling him the right to vote and to be voted upon. Not
would be no end to litigations, thus setting to naught the main role of courts, which being conditional simply meant that it was not based on Erap’s promise
is, to assist in the enforcement of the rule of law and the maintenance of peace and not to run for any public office.138
order by settling justiciable controversies with finality.133 In Pormento (which was consolidated with Mary Lou Estrada), the petitioner
A final judgment vests in the prevailing party a right recognized and protected likewise sought to prevent Estrada
by law under the due process clause of the Constitution. A final judgment is a _______________
vested interest and it is only proper and equitable that the government should
recognize and protect this right. Furthermore, an individual cannot be deprived of 136 Meralco v. Philippine Consumers Foundation, Inc., 425 Phil. 65, 79; 374
this right arbitrarily without causing injustice.134 SCRA 262, 273-274 (2002).
Just as the losing party has the right to file an appeal within the prescribed 137 SPA 09-24-DC.
period, the winning party also has the correlative right to enjoy the finality of the 138 Resolution of the COMELEC, Second Division dated January 20, 2010 in
resolution of his case.135 SPA No. 09-024 (DC) [Pamatong petition]; p. 8 of the Resolution; attached as
In the present case, the COMELEC’s final rulings in Exhibit “4” to Annex “H” of the Petitioner’s Memorandum.
the Pamatong, Pormento and Mary Lou Estrada petitions had been made 355
executory through the inclusion of Erap as a candidate not only as a President in VOL. 747, JANUARY 21, 2015 355
the 2010 elections but as Mayor in the 2013 elections.
Thus, the COMELEC’s 2010 final ruling in Pamatong and Pormento had been Risos-Vidal vs. Commission on Elections
made executory twice not only with respect to the interest of Erap, the winning from running as President in the 2010 elections. Estrada re-pleaded in his
party, through the answer the defenses that he raised in Pamatong and added the argument that the
_______________ grant of executive clemency in his favor removed all legal impediments
that may bar his candidacy for the presidency.139
That pardon was not an issue specified by the COMELEC when it defined the This rule is embodied under Section 47, Rule 39 which provides the effect of a
issues common to petitioners Pormento and Mary Lou Estrada is of no moment judgment or final order rendered by a court of the Philippines, having jurisdiction
since COMELEC only outlined the issues that petitioners Pormento and Mary Lou to pronounce the judgment or final order. In paragraph 47(a), the rules provide
Estrada commonly shared. The matter of pardon was raised as a defense by that in case of a judgment or final order x x x in respect to the
Estrada and this was duly noted by the COMELEC in its resolution.140 Under personal, political, or legal condition or status of a particular person or
these circumstances, what assumes importance are the terms of the COMELEC his relationship to another, the judgment or final order is conclusive
resolution itself which expressly discussed and ruled that the Erap pardon was upon the title to the thing, the will or administration or the condition, status or
absolute and had the effect of restoring his right to vote and be voted upon. relationship of the person x x x.142
In fact, even if petitioners Pormento and Mary Lou Estrada did not fully argue _______________
the pardon issue that Erap raised, it must be appreciated that this issue was
indisputably fully argued, ruled upon and became final in Pamatong which 141 Feria and Noche, Civil Procedure Annotated, Vol. II, p. 270.
was one of the 2010 trilogy of disqualification cases. This finality could not but 142 PCI Leasing and Finance, Inc. v. Dai, 560 Phil. 84, 94-95; 533 SCRA 611,
have an effect on the Pormento and Mary Lou Estrada rulings which carried the 620 (2007).
same rulings on pardon as Pamatong. The Pormento and Mary Lou 357
Estrada rulings on pardon, which themselves lapsed to finality can, at the very VOL. 747, JANUARY 21, 2015 357
least, be read as a recognition of the final judgment on the pardon in issue
in Pamatong, as well as the official final stand of COMELEC on the issue of the Risos-Vidal vs. Commission on Elections
Erap pardon. In the present case, the 2010 COMELEC final rulings that Erap was qualified
_______________ to run for public office, after consideration of the issues of presidential reelection
and the effect of his pardon for the crime of plunder, constituted a judgment in
139 COMELEC, Second Division Resolution dated January 20, 2010 in SPA rem as it was a judgment or final order on the political status of Erap to
No. 09-028 (DC) [Pormento petition] and SPA No. 09-104 [Mary Lou Estrada run for and to hold public office.
petition]; pp. 5-6 of the Resolution; attached as Annex “O” to Memorandum of In other words, a declaration of the disqualification or qualification of a
Intervenor Lim. candidate binds the whole world as the final ruling of the COMELEC regarding
140 See pp. 5-6 of the COMELEC, Second Division Resolution on SPA No. 09- Erap’s perpetual absolute disqualification and pardon had already become
028 (DC), attached as Annex “O” to Memorandum of Intervenor Lim. conclusive. The 2010 final rulings of the COMELEC thus bar Risos-Vidal in 2013
356 from raising the same issue in view of the nature of the 2010 rulings as
judgments in rem.
356 SUPREME COURT REPORTS ANNOTATED
I also reiterate my previous discussion that in determining whether res
Risos-Vidal vs. Commission on Elections judicata exists, the Court had previously ruled that absolute identity of parties is
These antecedent proceedings, the parties’ arguments in their respective not required but substantial identity, such that the parties in the first and second
pleadings, and the COMELEC rulings in Pamatong [SPA 09-24 (DC)] and cases share the same or a community of interest. As discussed above, this requisite
in Pormento [SPA 09-28] clearly show that the COMELEC rulings in these cases is present in the 2010 disqualification cases and the present Risos-Vidal case.
on the issue of pardon were decisions on the merits that can be cited as authorities
in future cases. 2. Identity of causes of action and subject matters

C.2(b)(ii)(d) Identity of Parties, Subject Matter and Cause of Action. I discuss first the element of identity of causes of action because, in the process,
the element of identity of subject matters would be likewise covered. On the
1. Identity of parties element of identity of causes of action between the first and second cases, J.
Leonen asserts that the 2010 disqualification cases filed by Pormento and Mary
Two kinds of judgments exist with respect to the parties to the case. Lou Estrada were based on causes of action that were different from those in the
The first are the parties in proceedings in personam where the judgments are present case.
enforceable only between the parties and their successors in interests, but not According to J. Leonen, the 2010 cases were anchored on the constitutional
against strangers thereto. The second type are the judgments in proceedings prohibition against a president’s reelection and the additional ground that Erap
where the object of the suit is to bar indifferently all who might be minded to make was a nuisance candidate. The present case is anchored on Erap’s conviction for
an objection of any sort against the right sought to be established, and anyone in plunder which carried with it the accessory penalty of perpetual absolute
the world who has a right to be heard on the strength of alleged facts which, if true, disqualification. The present case is additionally
show an inconsistent interest; the proceeding is in rem and the judgment is a 358
judgment in rem.141 358 SUPREME COURT REPORTS ANNOTATED
Risos-Vidal vs. Commission on Elections and former causes of action?” If the answer is in the affirmative, then the prior
based on Section 40 of the LGC as well as Section 12 of the OEC. This is clear judgment is a bar to the subsequent action; conversely, it is not.146
from the COMELEC’s recital of issues.143 Applying these tests, it is readily apparent that there were identical causes of
I disagree with J. Leonen’s positions and short-sighted view of the issues and action in the 2010 disqualification cases against Erap and the present Risos-Vidal
I maintain that there are identical subject matters and causes of actions, especially case.
for purposes of complying with the requirements of res judicata by way of bar by Using the absence of inconsistency test, the 2010 final COMELEC rulings
prior judgment. that Erap was qualified to run for Presidency, an elective public office, would be
At this juncture, I reiterate my disagreement with J. Leonen in strictly inconsistent with the ruling being sought in the present case which is, essentially,
applying the requisites for the application of res judicata through bar by prior that Erap’s pardon did not remove his perpetual absolute disqualification to run
judgment. The Court itself, in numerous cases, did not strictly apply the for elective public office, this time as Mayor of the City of Manila.
requirement that there must be absolute identity of causes of action. In fact, the _______________
Court’s rulings on this particular element leaned towards substantial identity of
causes of action and its determination is arrived at not on the basis of the facial 145 Antonio v. Vda. de Monje, G.R. No. 149624, September 29, 2010, 631
value of the cases but after an in-depth analysis of each case. SCRA 471, 482.
The reason why substantial identity of causes of action is permitted is to 146 Id.
preclude a situation where a party could easily escape the operation of res 360
judicata by changing the form of the action or the relief sought. The difference in 360 SUPREME COURT REPORTS ANNOTATED
form and nature of the two actions is also immaterial and is not a reason to exempt
Risos-Vidal vs. Commission on Elections
these cases from the effects of res judicata.
In short, Erap’s pardon and its effects on his perpetual absolute
The philosophy behind this rule prohibits the parties from litigating the same
disqualification brought about by his conviction affect his qualification to run
issue more than once. When a right or fact has been judicially tried and
for all elective public offices. Thus the 2010 rulings cannot be limited or linked
determined by a court of competent jurisdiction or an opportunity for
only to the issue of his qualification to run as President of the Philippines but to
such trial has been given, the judgment of the court, as long as it remains
any elective public position that he may aspire for in the future.
unreversed, should be conclusive upon the parties and those in privity with
Applying the “same evidence test,” suffice it to say that the Risos-Vidal’s
them. In this way, there should be an end to litigation by the same parties and
petition rests and falls on Erap’s pardon and its effects on his qualification to run
their privies over a subject, once the issue involving the subject is fully and fairly
for elective public office. Erap’s pardon is the same evidence necessary for the
adjudicated.144
COMELEC to resolve in the 2010 disqualification cases the issue of whether or not
_______________
Erap’s pardon removed his disqualification to run for elective public office, thus
qualifying him to run for Presidency.
143 Id. It must be recalled that Risos-Vidal relies on Section 40147 of the LGC and
144 Pilar Development Corporation v. Court of Appeals, G.R. No. 155943, Section 12148 of the OEC, specifically relating
August 28, 2013, 704 SCRA 403. _______________
359
VOL. 747, JANUARY 21, 2015 359 147 Section 40. Disqualifications.—The following persons are disqualified
Risos-Vidal vs. Commission on Elections from running for any elective local position:
In light of the jurisprudence on res judicata by way of bar by prior judgment, (a) Those sentenced by final judgment for an offense involving moral
it is my view that the COMELEC did not gravely abuse its discretion in ruling that turpitude or for an offense punishable by one (1) year or more of
the issue of Erap’s pardon and its effects on his right to run for elective public office imprisonment, within two (2) years after serving sentence.
had already been settled in the 2010 disqualification cases. xxxx
In our jurisdiction, the Court uses various tests in determining whether or not 148 Sec. 12. Disqualifications.—Any person who has been declared by
there is identity of causes of action in the first and second cases. One of these tests competent authority insane or incompetent, or has been sentenced by final
is the “absence of inconsistency test” where it is determined whether or not the judgment for subversion, insurrection, rebellion or for any offense for which
judgment sought will be inconsistent with the prior judgment. If inconsistency is he has been sentenced to a penalty of more than eighteen months or for
not shown, the prior judgment shall not constitute a bar to subsequent actions.145 a crime involving moral turpitude, shall be disqualified to be a candidate
The second and more common approach in ascertaining identity of causes of and to hold any office, unless he has been given plenary pardon or
action is the “same evidence test,” where the criterion is determined by the granted amnesty.
question: “would the same evidence support and establish both the present This disqualifications to be a candidate herein provided shall be deemed
removed upon the declaration by competent authority that said insanity or
incompetence had been removed or after the expiration of a period of five 3. Grave Abuse of Discretion, the 2010 Disqualification Trilogy, and
years from his service of sen- COMELEC’s Risos-Vidal Ruling.
361
VOL. 747, JANUARY 21, 2015 361 In light of the above discussions, the COMELEC did not gravely abuse its
discretion in its Resolution of April 1, 2013 dismissing the Risos-Vidal petition for
Risos-Vidal vs. Commission on Elections lack of merit. In fact, the COMELEC would have gravely abused its discretion had
to the disqualification ground of a person’s conviction for a crime involving it granted the petition in light of the 2010 trilogy of disqualification cases and the
moral turpitude, in this case, plunder. However, if we are to look closely at these finality of its previous final rulings that the third Whereas Clause of Erap’s pardon
provisions,149 Erap would not have been disqualified under these provisions did not affect at all the restoration of his civil and political rights, including his
because he had already served the 2-year prohibitive period under Section 40 of right to vote and to be voted upon.
the LGC.150 The real main issue of the Risos-Vidal petition is the perpetual Whatever might be said of the trilogy of cases, the reality is that the issue of
absolute disqualification imposed on Erap as an accessory penalty for his pardon was brought to the forefront of the argued issues when the parties raised
conviction for a crime involving moral turpitude; and that his pardon did not remit it in all the disqualification cases against Erap and the COMELEC ruled on the
this disqualification. This issue was obviously directly ruled upon by the issue. That the pardon issue was overshadowed by the presidential reelection
COMELEC in the 2010 disqualification cases. Hence, applying the same evidence issue, not only in the COMELEC, but all the way to this Court, may be an
test, there is identity of causes of action between the 2010 and the Risos-Vidal adjudicatory defect, but certainly is not imperfection on the part of Erap for which
cases. There was likewise identity of subject matters, specifically the qualification he should suffer.
of Erap to run for public office in relation to his pardon. To be sure, the COMELEC resolution is not a model resolution that is free from
As a side note, I observe that in the 2010 cases, had the COMELEC ruled that imperfections; it cannot serve as a model for legal drafting or for legal reasoning.
Erap had been disqualified to run for elective public office despite his pardon, the But whatever these
issue of the constitutional ban against his reelection would have become moot and 363
academic as Erap would never be qualified in the first place to run for an elective
VOL. 747, JANUARY 21, 2015 363
office. Therefore, the ground for Erap’s disqualification based on his perpetual
absolute disqualification in relation to his pardon, which were raised by the parties Risos-Vidal vs. Commission on Elections
in 2010, were material and necessary for the resolution of the reelection issue. imperfections might be, they could not — as above explained — have gone
Otherwise, to simply disregard the pardon issue and proceed immediately to the beyond errors of law, into grave abuse of discretion. Having been rulings twice
issue on the constitutional ban on reelection is not only absurd but would have implemented in 2010 and 2013 elections, these past rulings cannot and should not
been the height of legal ignorance. Fortunately, the now be repudiated without committing fraud against the electorate who cast their
_______________ vote and showed their preference for Erap without any notice that their votes ran
the risk of being declared stray.
tence, unless within the same period he again becomes disqualified. For all the above reasons, I vote to dismiss the Risos-Vidal petition for lack of
149 Id. merit.
150 See Magno v. COMELEC, 439 Phil. 339, 347-348; 390 SCRA 495, 498
(2002) where the Court held that the 2-year prohibitive period under the LGC CONCURRING OPINION
prevails over the 5-year prohibitive period under Section 12 of the OEC.
362 MENDOZA, J.:
362 SUPREME COURT REPORTS ANNOTATED
At first glance, this case presents itself as an ordinary election case involving
Risos-Vidal vs. Commission on Elections the issue of who is the rightful winner in the 2013 mayoralty elections in the City
COMELEC correctly ruled on the pardon issue directly and did not gravely of Manila. The matter, however, is engrossed in a deeper constitutional conundrum
abuse its discretion in doing so. that affects the exercise of one of the most benevolent powers of the President —
Since the COMELEC had already decided the issue of Erap’s pardon in the power to extend executive clemency in the form of pardon. Undoubtedly, the
the past, it did not act with grave abuse of discretion when it chose not to Court’s ruling on this case would shape the parameters surrounding the future
reverse its prior rulings. Its past decisions, which became final and executory, exercise of the said power, thus, requiring a pragmatic stance that would equal the
addressed this issue on the merits. This, and the substantial causes of action, theoretical and practical purpose of the pardoning power, that is, the realization
subject matters, and substantial identity of the parties in the 2010 and 2013 cases, of checks and balances in government and the relief given to the pardonee.
sufficiently justified the COMELEC from keeping the discussion of the issue of The undisputed facts as culled from the records:
Erap’s pardon in the 2013 disqualification case. In its September 12, 2007 Decision, the Sandiganbayanconvicted respondent
former President Joseph Ejercito Estrada (Estrada) of plunder. The fallo of the
decision reads:
WHEREFORE, in view of all the foregoing, judgment is hereby rendered in On October 25, 2007, then President Gloria Macapagal-Arroyo (PGMA)
Criminal Case No. 26558 finding the accused, Former President Joseph Ejercito granted executive clemency to Estrada. The text of the said pardon is hereunder
Estrada, GUILTY beyond reasonable doubt of the crime replicated:
364 MALACAÑAN PALACE
364 SUPREME COURT REPORTS ANNOTATED MANILA

Risos-Vidal vs. Commission on Elections

of PLUNDER, defined in and penalized by Republic Act No. 7080, as amended.


On the other hand, for failure of the prosecution to prove and establish their guilt
beyond reasonable doubt, the Court finds the accused Jose “Jinggoy” Estrada and
Atty. Edward S. Serapio NOT GUILTY of the crime of plunder and, accordingly, Whereas, this Administration has a policy of releasing inmates who have
the Court hereby orders their ACQUITTAL. reached the age of seventy (70),
The penalty imposable for the crime of plunder under Republic Act No. 7080, Whereas, Joseph Ejercito Estrada has been under detention for six and a half
as amended by Republic Act No. 7659, is Reclusion Perpetua to Death. There being years,
no aggravating or mitigating circumstances, however, the lesser penalty shall be Whereas, Joseph Ejercito Estrada has publicly committed to no longer seek
applied in accordance with Article 63 of the Revised Penal Code. Accordingly, the any elective position or office,
accused Former President Joseph Ejercito Estrada is hereby sentenced to suffer In view hereof and pursuant to the authority conferred upon me by the
the penalty of Reclusion Perpetua and the accessory penalties of civil interdiction Constitution, I hereby grant executive clemency to Joseph Ejercito Estrada,
during the period of sentence and perpetual absolute disqualification. convicted by the Sandiganbayan of plunder and imposed a penalty of reclusion
The period within which accused Former President Joseph Ejercito Estrada perpetua. He is hereby restored to his civil and political rights.
has been under detention shall be credited to him in full as long as he agrees The forfeitures imposed by the Sandiganbayan remain in force and in full,
voluntarily in writing to abide by the same disciplinary rules imposed upon including all writs and processes issued by the Sandiganbayan in pursuance
convicted prisoners. hereof, except for the bank account(s) he owned before his tenure as President.
Moreover, in accordance with Section 2 of Republic Act No. 7080, as amended Upon acceptance of this pardon by JOSEPH EJERCITO ESTRADA, this
by Republic Act No. 7659, the Court hereby declares the forfeiture in favor of the pardon shall take effect.
government of the following:
366
(1) The total amount of Five Hundred Forty Two Million Seven Hundred
Ninety One Thousand Pesos (P545,291,000.00), with interest and income earned, 366 SUPREME COURT REPORTS ANNOTATED
inclusive of the amount of Two Hundred Million Pesos (P200,000,000.00), Risos-Vidal vs. Commission on Elections
deposited in the name and account of the Erap Muslim Youth Foundation. Given under my hand at the City of Manila, this 25th day of October, in the
(2) The amount of One Hundred Eighty-Nine Million Pesos year of Our Lord, two thousand and seven.
(P189,000,000.00), inclusive of interests and income earned, deposited in the Jose Gloria M. Arroyo (sgd.)
Velarde account. By the President:
(3) The real property consisting of a house and lot dubbed as Boracay IGNACIO R. BUNYE (sgd.)
Mansion located at #100 11th Street, New Manila, Quezon City. Acting Executive Secretary
365 [Emphasis supplied]
VOL. 747, JANUARY 21, 2015 365
The next day, Estrada accepted the pardon as evidenced by a handwritten
Risos-Vidal vs. Commission on Elections
notation in the same document.
The cash bonds posted by accused Jose Jinggoy Estrada and Atty. Edward S. Subsequently, Estrada undertook his second bid for the presidency during the
Serapio are hereby ordered cancelled and released to the said accused or their duly 2010 elections. This candidacy hurdled two (2) disqualification cases filed by Atty.
authorized representatives upon presentation of the original receipt evidencing Evilio C. Pormento and Mary Lou B. Estrada (2010 disqualification cases), when
payment thereof and subject to the usual accounting and auditing procedures. these were denied for lack of merit by the Commission on Elections (COMELEC),
Likewise, the hold departure orders issued against the said accused are hereby Second Division, and the COMELEC En Banc in its respective resolutions, dated
recalled and declared functus oficio. January 20, 20101 and April 27, 2010.2The COMELEC was of the position that
SO ORDERED. Estrada was eligible to run for president on the ground that the constitutional
prohibition on reelection3 applies to an incumbent president.
Upon elevation to the Court, however, the opportunity to resolve the said hold public office was thereby restored to the petitioner. In view of
constitutional issue was arrested by mootness, with Estrada having lost the the express exclusion by Art. 36, RPC of the right to hold public office,
elections to President Benigno Aquino.4 notwithstanding a pardon unless the right is expressly restored by the pardon, it is
Undaunted by his defeat in the race for national office, Estrada thereafter my considered opinion that, to the extent that the pardon granted to the petitioner
sought the position of mayor in no less did not expressly restore the right to hold public office as an effect of such pardon,
_______________ that right must be kept away from the petitioner.

1 Rollo, pp. 1009-1034. After an exchange of pleadings, the COMELEC Second Division issued its
2 Id., at pp. 1035-1054. April 1, 2013 Resolution dismissing the petition for lack of merit.8 The dismissal
3 Section 4, Article VII, 1987 Constitution. was grounded on its resolution of the 2010 disqualification cases where it found
4 Pormento v. Estrada, G.R. No. 191988, August 31, 2010, 629 SCRA 530. that the pardon granted to Estrada was absolute and unconditional, hence,
367 entitling him to run for public office. The dismissal was affirmed over petitioner’s
motion for reconsideration in the April 23, 2013 Resolution of the COMELEC En
VOL. 747, JANUARY 21, 2015 367
Banc.9
Risos-Vidal vs. Commission on Elections Impervious to her cause, the petitioner comes to this Court, ascribing grave
than the City of Manila. He filed his certificate of candidacy on October 2, 2012. abuse of discretion on the part of the COMELEC in declining to disqualify
Petitioner Atty. Alicia Risos-Vidal (petitioner) invoked Estrada’s Estrada motu propio, based on the following grounds cited by it: 1] the issues
disqualification from running for public office, this time on the ground that his raised in the petition have already been passed upon in the past; 2] Estrada’s
candidacy was a violation of the pardon extended by PGMA. She filed a petition pardon was not conditional; 3] Estrada is not disqualified to run as mayor despite
for disqualification with the COMELEC5 pursuant to Section 12 of Batas Section 40 of the Local Government Code (LGC); and 4] Estrada’s pardon restored
Pambansa Blg. 881 (Omnibus Election Code),6 grounded on a sole argument, viz.: his right to suffrage and remitted his perpetual disqualification from seeking
RESPONDENT IS DISQUALIFIED TO RUN FOR PUBLIC OFFICE public office.
BECAUSE OF HIS CONVICTION FOR PLUNDER BY During the pendency of the petition, local elections were conducted on May 13,
THE SANDIGANBAYAN IN CRIMINAL CASE NO. 26558 ENTITLED 2013, yielding a victory for Estrada
“PEOPLE OF THE PHILIPPINES V. JOSEPH EJERCITO ESTRADA” _______________
SENTENCING HIM TO SUFFER THE PENALTY OF RECLUSION
PERPETUA WITH PERPETUAL ABSOLUTE DISQUALIFICATION. 8 Rollo, pp. 39-46.
9 Id., at pp. 49-50.
In the main, the petitioner argued that Estrada was still suffering from the 369
accessory penalties of civil interdiction and perpetual disqualification because the
VOL. 747, JANUARY 21, 2015 369
pardon granted to him failed to expressly restore his right to suffrage and to run
for public office as provided under Articles 36 and 41 of the Revised Penal Code. Risos-Vidal vs. Commission on Elections
Furthermore, the “whereas clause” in the pardon which stated that, “Joseph over his opponents including then incumbent Mayor Alfredo S. Lim (Lim).
Ejercito Estrada has publicly committed to no longer seek any elective position or Consequently, the latter moved to intervene in the petition, which was granted by
office” would indicate a condition that Estrada must abide by under pain of the Court in its June 25, 2013 Resolution.10 Lim supports petitioner’s theory that
recommitment to prison in the event of violation thereof. The petitioner likewise Estrada remains to be disqualified to hold public office as his pardon did not
finds support in the concurring opinion of Justice Padilla in Monsanto v. Factoran, expressly remit his perpetual disqualification, and, pursuant to the Court’s ruling
Jr.,7 stated in this wise: in Jalosjos v. COMELEC,11 he must be declared as the rightful mayor of the City
_______________ of Manila.
After an exchange of pleadings,12 the parties were required to submit their
5 Rollo, pp. 267-285. respective memoranda. The parties complied on different dates.13
6 Docketed as SPA No. 13-211 (DC). To my mind, the following queries and premises, which are crafted in a clear-
7 252 Phil. 192, 206-207; 170 SCRA 190, 203 (1989). cut and logical sequence, serve as guideposts for the Court in order to arrive at
368 conclusions that are consonant with prevailing law and jurisprudence:
368 SUPREME COURT REPORTS ANNOTATED
I. Was the executive pardon extended to Estrada conditional or absolute?
Risos-Vidal vs. Commission on Elections II. What were the effects of the pardon, particularly the statement, “[h]e is
An examination of the presidential pardon in question shows that, while hereby restored to his civil and political rights?” Does this include the restoration
petitioner was granted “an absolute and unconditional pardon and restored to full of his right to suffrage and to run for public office?
civil and political rights,” yet, nothing therein expressly provides that the right to
_______________
delivery is essential, and delivery is not complete without acceptance.”15
10 Id., at p. 438.
11 G.R. No. 193237, October 9, 2012, 683 SCRA 1. The fact of Estrada’s acceptance of the pardon, by affixing his signature
12 Estrada filed his comment to Lim’s petition-in-intervention on July 15, therein, is an insufficient indication of its conditional nature. Petitioner’s reliance
2013; the COMELEC, through the Office of the Solicitor General (OSG) filed its on Cabantag v. Wolf,16 where the Court ruled that a conditional pardon has no
consolidated comment on July 29, 2013; Estrada filed his comment to the petition force until accepted by the condemned because the condition may be less acceptable
on August 6, 2013; Lim filed his reply to Estrada’s comment on August 23, 2013; to him than the original punishment and may in fact be more onerous, is
petitioner filed her reply to Estrada’s comment to the petition on August 27, 2013; misplaced. It merely stated that a conditional pardon must be accepted in the
petitioner filed her reply to the COMELEC’s consolidated comment on December exercise of the pardonee’s right to choose whether to accept or reject the terms of
13, 2013. the pardon. It does not operate in the manner suggested by petitioner. It does not
13 Lim on May 27, 2014; petitioner on June 2, 2014; Estrada on June 16, 2014 work the other way around.
and the COMELEC on June 26, 2014. An “acceptance” does not classify a pardon as conditional just by the mere
370 reception and the placing of an inscription thereon. I am not prepared to ignore
the very intention and content of a pardon as standards to determine its nature,
370 SUPREME COURT REPORTS ANNOTATED
as against the mere expediency of its delivery and acceptance. I am much more
Risos-Vidal vs. Commission on Elections amenable to the rule consistent with the benevolent nature of pardon: that it is an
III. Given that the nature of pardon, whether absolute or conditional, does not act of forgiveness predicated on an admission of guilt. To be effective, therefore,
imply the automatic obliteration of the pardonee’s guilt, is Estrada qualified to run this admission of past wrongdoing must be manifested by the acceptance of a
for and hold a mayoralty position? pardon, absolute or conditional.
Further, the significance of “acceptance” is more apparent in cases of
I. Estrada’s Pardon was Absolute “commutation,” which is the substitution of a lighter punishment for a heavier one.
William F. Duker elucidates:
After admittedly having failed to argue on this before the COMELEC, the Although for a pardon to be effective it usually must be accepted, commutation
petitioner expressly elevated this issue for the resolution of the Court. Her is effective without acceptance. In Chapman v. Scott, the President granted a
insistence on the conditional nature of Estrada’s pardon is anchored on the latter’s commutation to “time-served” to a convict so that he would be available for
expressed acceptance of the same. In her words, this acceptance became “the prosecution in a state court on a
fundamental basis and indicium of the conditional nature of the pardon.”14 She _______________
contends that had PGMA intended to issue an absolute pardon, she would have
not required Estrada’s acceptance thereof. Having accepted its terms with a 15 Monsanto v. Factoran, Jr., supra note 7 at pp. 198-199; p. 196.
commitment of strict compliance, Estrada should be deemed to have breached the 16 G.R. No. 3080, May 5, 1906.
“contract” when he ran for Mayor. 372
Amidst this argument, the primordial question continues to nag: was the
372 SUPREME COURT REPORTS ANNOTATED
pardon bestowed on Estrada conditional or absolute? For the following reasons, I
find that Estrada’s pardon was absolute in nature: Risos-Vidal vs. Commission on Elections
First. I am of the view that the acceptance confers effectivity in both absolute capital case. The convict refused the commutation and argued that it was not
and conditional pardon. effective until accepted, but the court held that a commutation did not require
Pardon is defined as “an act of grace, proceeding from the power entrusted with acceptance:
the execution of the laws, which exempts the individual, on whom it is bestowed, Although power to commute is logically derivable from power to pardon,
from the punishment the law inflicts for a crime he has committed. It is the private, commutation is essentially different from pardon. Pardon exempts from
though official act of the executive magistrate, delivered to the individual for whose punishment, bears no relation to term of punishment, and must be accepted, or it
benefit it is intended, and not communicated officially to the Court. ... A pardon is is nugatory. Commutation merely substitutes lighter for heavier punishment. It
a deed, to the validity of which removes no stain, restores no civil privileges, and may be effected without the
_______________ consent and against the will of the prisoner.17

14 Rollo, p. 12. As applied to Estrada’s case, his acceptance of the pardon does not necessarily
371 negate its absolute nature. The more appropriate test to apply in the
VOL. 747, JANUARY 21, 2015 371 determination of the subject pardon’s character is the grantor’s intention as
revealed in the four corners of the document.
Risos-Vidal vs. Commission on Elections
Second. The controversial perambulatory clause which states, “Whereas, 374
Joseph Ejercito Estrada has publicly committed to no longer seek any elective 374 SUPREME COURT REPORTS ANNOTATED
position or office,” should not be considered as a restriction on Estrada’s pardon.
Primarily, rules on statutory construction provide that whereas clauses, do not Risos-Vidal vs. Commission on Elections
form part of a statute, strictly speaking; they are not part of the operative language shape one, only because the plain meaning of the pardon’s text is unacceptable
of the statute.18 While they may be helpful to the extent that they articulate the for some waylaid and extraneous reasons. That the executive clemency given to
general purpose or reason underlying a new enactment, reliance on whereas Estrada was unaccompanied by any condition is clearly visible in the text of the
clauses as aids in construing statutes is not justified when their interpretation pardon. The Court must simply read the pardon as it is written. There is no
“control the specific terms of the statute.”19 necessity to resort to construction. I choose to heed the warning enunciated
_______________ in Yangco v. Court of First Instance of Manila:
. . . [w]here language is plain, subtle refinements which tinge words so as to
17 William and Mary Law Review, The President’s Power to Pardon: A give them the color of a particular judicial theory are not only unnecessary but
Constitutional History by William F. Duker, Volume 18, Issue 3, Article 3. decidedly harmful. That which has caused so much confusion in the law, which has
18 Llamado v. Court of Appeals, 256 Phil. 328, 339; 174 SCRA 566, 576 (1989), made it so difficult for the public to understand and know what the law is with
citing Yazoo & Mississippi Valley R. Co. v. Thomas, 132 US 174 (1889); 33 L Ed respect to a given matter, is in considerable measure the unwarranted interference
302. by judicial tribunals with the English language as found in statutes and contracts,
19 Llamado v. Court of Appeals, id. cutting the words here and inserting them there, making them fit personal ideas
373 of what the legislature ought to have done or what parties should have agreed
upon, giving them meanings which they do not ordinarily have cutting, trimming,
VOL. 747, JANUARY 21, 2015 373 fitting, changing and coloring until lawyers themselves are unable to advise their
Risos-Vidal vs. Commission on Elections clients as to the meaning of a given statute or contract until it has been submitted
As applied in Estrada’s case, the subject whereas clause does not purport to to some court for its interpretation and construction.21
control or modify the unequivocal terms found in the pardon’s body. In this sense,
the “whereas clauses” in Estrada’s pardon cannot adversely affect the ultimate Suffice it to say, a statement describing Estrada’s previous commitment not to
command which it evokes, that is, executive clemency is granted to Estrada absent seek any elective office cannot operate as a condition for his pardon, sans any
any condition. indication that it was intended to be so. In light of the clear absence of any
A conditional pardon basically imposes a condition. I take this to mean that it condition in the pardon, no ambiguity warrants interpretation by the Court. At the
must either stipulate a circumstance, a situation, or a requisite that must come most, the subject whereas clause depicts the state of affairs at the time when the
into pass or express a restriction that must not ensue. I find none in this case. The pardon was granted. It should not be considered as part and parcel of the entire
plain language of the pardon extended to Estrada does not set forth any of these. act
It was couched in a straightforward conferment of pardon, to wit: _______________
I hereby grant executive clemency to Joseph Ejercito Estrada, convicted by
the Sandiganbayan of plunder and imposed a penalty of reclusion perpetua. 21 29 Phil. 188 (1915).
375
Had PGMA intended to impress a condition on Estrada, the same would have VOL. 747, JANUARY 21, 2015 375
been clearly stated as a requirement of, or restriction to, the above conferment. I
am inclined to posit that the extension of a conditional pardon to her political rival Risos-Vidal vs. Commission on Elections
is a matter that PGMA would have regarded with solemnity and tact. After all, the as it serves neither the ability to enlarge or confer powers nor the authority to
pardoning power is a pervasive means to bluntly overrule the force and effect, not control the words of the act.
only of a court’s judgment of conviction, but the punitive aspect of criminal laws. Third. The pardoning power is granted exclusively to the President amidst the
As it turned out, no direct showing suggests that the pardon was conditional. constitutional scheme of checks and balances. While it is most ideal that the
For a condition to be operative, the condition must appear on the face of the executive strictly adheres to this end, it is undeniable that the pardoning power is
document. The conditions must be clear and specific. The reason is that the still dependent on the grantor’s measure of wisdom and sense of public policy. This
conditions attached to a pardon should be definite and specific as to inform the reality invites, if not bolsters, the application of the political question doctrine. The
person pardoned of what would be required.20 As no condition was patently evinced only weapon, which the Court has freedom to wield, is the exercise of judicial power
in the document, the Court is at no liberty to against a blatant violation of the Constitution. When unavailing, the Court is
_______________ constrained to curb its own rebuking power and to uphold the acumen of a coequal
branch. It would do the Court well to remember that neither the Congress nor the
20 Ex Parte Reno, 66 Mo. 266, 269 (Mo. 1877). courts can question the motives of the President in the use of the power.22
Hence, in determining the nature of Estrada’s pardon, the Court must Risos-Vidal vs. Commission on Elections
undertake a tempered disposition and avoid a strained analysis of the obvious. qualification or ineligibility to hold public office because this formed part of the
Where there is no ostensible condition stated in the body of the pardon, to envisage penalty against her. As in the foregoing discussion on the absolute nature of
one by way of statutory construction is an inexcusable judicial encroachment. Estrada’s pardon, there is no question that his pardon likewise remitted the
The absolute nature of Estrada’s pardon now begets a more astute query: what punishment previously imposed in his conviction for plunder. As such, he was
rights were restored in his favor? released from incarceration and thereafter regained his liberty of movement, albeit
ordered to abide by the forfeiture of his properties as listed in the judgment of
II. Estrada’s Civil and Political Rights Restored the Sandiganbayan. More significantly, there was no categorical statement
impressed in Monsanto that banned her from holding public office again. All that
In this particular issue, the ponencia deserves my full agreement in finding it withheld was an automatic reinstatement to her previous office and her
that the third preambular clause of Estrada’s pardon does not militate against the entitlement to backpay. In other words, Monsanto may hold public office provided
conclusion that Estrada’s rights to suffrage and to seek public office have been that there is favorable action on her application.
restored. Further, the subject pardon had substantially complied with the While I generally acquiesce with the scholarly opinions of Justices Padilla and
statutory requirements laid down in Articles 36 and 41 of the RPC. The authority Feliciano in Monsanto, I find it difficult to apply their respective observations (that
of the said provi- based on Article 36 of the RPC, it was clear that the pardon extended by the
_______________ President did not per seentitle Monsanto to again hold public office or to suffrage
because nothing therein expressly provided the restoration of the said rights with
22 Supra note 17. specifity) precisely because this was not adopted in the majority decision. There is
376 a stark difference between the positions taken by the concurring justices from the
376 SUPREME COURT REPORTS ANNOTATED very holding of the majority. The former entirely and perpetually denied Monsanto
of her right to hold public office, while the latter merely disallowed an automatic
Risos-Vidal vs. Commission on Elections
reinstatement but permitted her to undergo reapplication with the
sions of law was reinforced by the ruling of the Court in Monsanto v. Factoran,
only caveat that her pardon did not place her in a state of complete innocence. In
Jr. A deeper analysis of Monsanto, however, reveals that its repercussions actually
other words, her past conviction should be considered as forming part of her
favor Estrada.
credentials in her reapplication for public office. Between these two conclusions, I
Consider these points:
choose with steadfast belief that the holding pronounced in the majority decision
1. Monsanto involved an absolute pardon, from which, Estrada likewise
should prevail. The strict interpretation of Article 36 as advocated in the
benefits.
concurring opinion was not adopted in the main decision, hence, rendering the
2. The issue in Monsanto involved the propriety of an automatic reinstatement
same as mere obiter dictum which has no controlling effect.
to public office. In refutation of the Garland cases, the Court maintained that while
378
an absolute pardon remits all the penal consequences of a criminal indictment if
only to give meaning to the fiat that a pardon, being a presidential prerogative ... 378 SUPREME COURT REPORTS ANNOTATED
it, however, rejected the “fictitious belief that pardon blots out the guilt of an Risos-Vidal vs. Commission on Elections
individual and that once he is absolved, he should be treated as if he were While I do not subscribe to Estrada’s theory that Articles 36 and 41 of the RPC
innocent.” have the effect of abridging and diminishing the power of the President, I also
3. Monsanto’s absolute disqualification or ineligibility from public office was remain unconvinced that the said provisions of law should apply to his case
considered to have formed part of the punishment prescribed against because the strict interpretation of these provisions were not encapsulated in
her. Ultimately, when her guilt and punishment were expunged by her jurisprudence, particularly Monsanto. Therefore, the statement, “He is hereby
pardon, this particular disability was likewise removed. restored to his civil and political rights,” as found in the subject pardon does not
4. Noteworthy is the observation of the Court that she may apply for fall short of producing the effect of wiping away the penalties being suffered by the
reappointment to the office, but in the appraisal of her suitability to a public post, pardonee. As things stand now, an absolute and full pardon erases both the
the facts constituting her past offense should be taken into account to determine principal and accessory penalties meted against him, thereby allowing him to hold
whether she could once again serve in a public office. public office once again.
After serious reflection, I am convinced that the foregoing pronouncement Corollary to this, I am of the opinion that PGMA’s failure to use the term “full,”
parallels that which should apply to Estrada. apropos to the restoration of Estrada’s rights does not denigrate its coverage.
In Monsanto, the Court declared that the absolute pardon granted to her by PGMA’s omission to use such term in the case of Estrada may have been caused
the President effectively expunged her dis- by reasons unknown to the Court. The Court cannot discount the possibility that
377 this was borne out of plain inadvertence, considering the fact that the pardon was
VOL. 747, JANUARY 21, 2015 377 unaccompanied by a clear condition. Had it been PGMA’s intention to restrict the
rights restored to Estrada, she could have stated clear exceptions thereto, instead Court must endeavour to protect at all cost. As Abraham Lincoln put it with both
of employing a phrase, which, in its plain meaning, comprises the right to vote and guile and eloquence:
to run for public office. Besides, the deprivation of these rights is a dangerous Elections belong to the people. It’s their decision. If they decide to turn their
ground that the Court should not tread on, especially when the intention to restrict back on the fire and burn their behinds, then they will just have to sit on their
their exercise is impalpable. blisters.
Applying this to the case at bench, no ban from holding public office should be For the foregoing reasons, I vote to CONCUR with the majority opinion.
imposed on Estrada, because the absolute pardon given to him had effectively
extinguished both the principal and accessory penalties brought forth by his DISSENTING OPINION
conviction. Succinctly, Estrada’s civil and political rights had been restored in full.
379 LEONEN, J.:
VOL. 747, JANUARY 21, 2015 379
This case has distressing consequences on the Rule of Law. By reading an
Risos-Vidal vs. Commission on Elections ambiguity in favor of a convicted public officer, impunity is tolerated.
III. Estrada’s Right to Run for Public Office Restored I dissent.
Joseph Ejercito Estrada, former President of the Republic of the Philippines,
Consistent with my view that Monsanto reflects the obliteration of Estrada’s was found guilty beyond reasonable doubt of the crime of plunder. A heinous crime
perpetual disqualification, I conclude that he now possesses the right to vote and of the highest order, the law penalizing plunder — Republic Act No. 7080 — made
to run for public office. possible the imposition of the supreme penalty of death upon public officers who
Lest it be misunderstood, this conclusion does not degenerate from the doctrine amass ill-gotten wealth on a grand scale through a combination or series of
that a pardon only relieves a party from the punitive consequences of his past acts.1 Though an interven-
crimes, nothing more. Indeed, “a person adjudged guilty of an offense is a convicted _______________
criminal, though pardoned; he may be deserving of punishment, though left
unpunished; and the law may regard him as more dangerous to society than one 1 Rep. Act No. 7080 (1991), Sec. 2:
never found guilty of crime, though it places no restraints upon him following his Sec. 2. Definition of the Crime of Plunder; Penalties.—Any public officer who,
conviction.”23 Estrada was not reborn into innocence by virtue of the forgiveness by himself or in connivance with members of his family, relatives by affinity or
bestowed in by the pardon. The moral stain caused by his past crimes remains to consanguinity, business associates, subordinates or other persons, amasses,
be part of his person, then as now. In no way did his pardon serve as a stamp of accumulates or acquires ill-gotten wealth through a combination or series of overt
incorruptibility. It is not a magic spell that superimposes virtuousness over guilt. or criminal acts as described in Section 1(d) hereof in the aggregate amount or total
His past conviction for plunder would forever form part of his person, whether as value of at least Fifty million pesos (P50,000,000.00) shall be
a private individual or a public officer. 381
Without squabble, plunder is a crime involving moral turpitude. Nevertheless,
this fact alone negates a mechanical application of statutory provisions on VOL. 747, JANUARY 21, 2015 381
disqualification. One thing is clear, in the exercise of her exclusive power to grant Risos-Vidal vs. Commission on Elections
executive clemency, PGMA pardoned Estrada, thereby wiping away the penalties ing statute2 now prevents the imposition of the penalty of death, our laws have
of his crime and entitling him the right to run for public office. Corollary to this, no less abhorrence for this crime.
Estrada’s fitness to hold public office is an issue that should not concern the Court. Joseph Ejercito Estrada, former President of the Republic of the Philippines,
All that the Court can rule on is the availability of Estrada’s right to seek public was pardoned shortly after he had been convicted. This case presents to this court
office. This ruling on his eligibility is not tantamount to a declaration that Estrada a dilemma engendered by ambiguities in the pardon extended to him.
befits a person wholly deserving of the people’s trust. The Manileños’ The court must decide on whether these ambiguities shall be interpreted to
_______________ benefit a convicted former President, shown to have amassed ill-gotten wealth on
a grand scale and to have betrayed the trust given to him through the investiture
23 Monsanto v. Factoran, Jr., supra note 7 at p. 201; p. 198, citing State v. of the highest office in the land; or to benefit the public which reposes its trust on
Cullen, 127 P. 2d 257, cited in 67 C.J.S. 577, note 18. elected public officials. Many other public officials have been found liable for graft
380 and corrupt practices of far lesser scales than those for which Joseph Ejercito
380 SUPREME COURT REPORTS ANNOTATED Estrada had been convicted. They now languish in jails, deprived of liberties and
entitlements. This case is not about their pardon. They continue to suffer the
Risos-Vidal vs. Commission on Elections penalties that their convictions entail, unlike the former President of the Republic
decision alone can mould the city’s journey to either development or decline. of the Philippines.
Indeed, election expresses the sovereign will of the people consistent with the
principle of vox populi est suprema lex. This is the beauty of democracy which the
This case, in short, will affect the public’s attitude to the Rule of Law and the 7 Id., at p. 438.
possibilities for immunity for very influential public officials. 383
VOL. 747, JANUARY 21, 2015 383
_______________
Risos-Vidal vs. Commission on Elections
guilty of the crime of plunder and shall be punished by reclusion perpetua to II
death. Any person who participated with the said public officer in the commission
of an offense contributing to the crime of plunder shall likewise be punished for Statement of the antecedents
such offense. In the imposition of penalties, the degree of participation and the
attendance of mitigating and extenuating circumstances, as provided by the On April 4, 2001, the Office of the Ombudsman filed against
Revised Penal Code, shall be considered by the court. The court shall declare any private respondent, Joseph Ejercito Estrada, former President of the Republic of
and all ill-gotten wealth and their interests and other incomes and assets including the Philippines, and several other accused,8 an information for plunder, penalized
the properties and shares of stocks derived from the deposit or investment thereof by Republic Act No. 7080, as amended by Republic Act No. 7659. This case was
forfeited in favor of the State. (As amended by Rep. Act No. 7659, approved on filed before the Sandiganbayan and docketed as Criminal Case No. 26558.
December 13, 1993) In the decision9 dated September 12, 2007, the Sandiganbayan, Special
2 Rep. Act No. 9346 (2006), otherwise known as An Act Prohibiting the Division, convicted Estrada of the crime of plunder. He was sentenced to suffer
Imposition of Death Penalty in the Philippines. “the penalty of Reclusion Perpetua and the accessory penalties of civil interdiction
382 during the period of sentence and perpetual absolute disqualification.”10
The dispositive portion of this decision reads:
382 SUPREME COURT REPORTS ANNOTATED WHEREFORE, in view of all the foregoing, judgment is hereby rendered in
Risos-Vidal vs. Commission on Elections Criminal Case No. 26558 finding the accused, Former President Joseph Ejercito
Not having been unequivocally restored to a status worthy of being a repository Estrada, GUILTY beyond reasonable doubt of the crime of PLUNDER defined in
of the public trust, there is no reason to lavish Joseph Ejercito Estrada by and penalized by Republic Act No. 7080, as amended. On the other hand, for failure
facilitating his reversion to elective public office. Thus, I dissent from the majority of the prosecution to prove and establish their guilt beyond reasonable doubt, the
decision. Court finds the accused Jose “Jinggoy” Estrada and Atty. Edward S. Serapio NOT
GUILTY of the crime of plunder, and accordingly, the Court hereby orders their
I ACQUITTAL.
_______________
Through a petition for certiorari, Atty. Alicia Risos-Vidal (Risos-Vidal) prays
that the assailed resolutions3 dated April 1, 2013 of the Second Division of public 8 Jose “Jinggoy” Estrada, Charlie “Atong” Tiu Hay Sy Ang, Edward S. Serapio,
respondent Commission on Elections (COMELEC), and April 23, 2013 of Yolanda T. Ricaforte, Alma Alfaro, a John Doe (also known as Eleuterio Ramos
COMELEC, sitting En Banc, be annulled and set aside. In addition, she prays that Tan or Mr. Uy), a Jane Doe (also known as Delia Rajas), and several other John
a new judgment be entered disqualifying private respondent Joseph Ejercito and Jane Does.
Estrada (Estrada) from running as Mayor of the City of Manila, and cancelling the 9 Rollo, pp. 52-262.
certificate of candidacy he filed in connection with the May 13, 2013 election for 10 Id., at p. 261.
the position of Mayor of the City of Manila.4 384
The assailed April 1, 2013 resolution dismissed the petition for disqualification 384 SUPREME COURT REPORTS ANNOTATED
filed by Risos-Vidal and docketed as SPA No. 13-211 (DC). The assailed April 23,
2013 resolution denied her motion for reconsideration. Risos-Vidal vs. Commission on Elections
A motion for leave to intervene5 was filed by Estrada’s opponent in the The penalty imposable for the crime of plunder under Republic Act No.
mayoralty race, Alfredo S. Lim (Lim). Attached to Lim’s motion was his petition- 7080,11 as amended by Republic Act No. 7659,12 is Reclusion Perpetua to Death.
in-intervention.6 Lim’s motion was granted by the court in the resolution7 dated There being no aggravating or mitigating circumstances, however, the lesser
June 25, 2013. penalty shall be applied in accordance with Article 63 of the Revised Penal
______________ Code.13Accordingly, the accused Former President Joseph Ejercito Estrada
is hereby sentenced to suffer the penalty of Reclusion Perpetua and the
3 Rollo, pp. 39-43, 49-50. accessory penalties of civil interdiction during the period of sentence and
4 Id., at p. 34. perpetual absolute disqualification.
5 Id., at pp. 390-392. _______________
6 Id., at pp. 395-412.
11 Rep. Act No. 7080 (1991), otherwise known as An Act Defining and _______________
Penalizing the Crime of Plunder.
12 Rep. Act No. 7659 (1993), otherwise known as An Act to Impose the Death 14 In the decision dated September 12, 2007, Rollo, p. 261, the numbers in
Penalty on Certain Heinous Crimes, Amending for that Purpose the Revised Penal words and in figures do not match.
Laws, as Amended, Other Special Penal Laws, and for Other Purposes. 15 Rollo, pp. 260-262.
13 Art. 63. Rules for the application of indivisible penalties.— In all cases in 386
which the law prescribes a single indivisible penalty, it shall be applied by the
386 SUPREME COURT REPORTS ANNOTATED
courts regardless of any mitigating or aggravating circumstances that may have
attended the commission of the deed. Risos-Vidal vs. Commission on Elections
In all cases in which the law prescribes a penalty composed of two indivisible On October 25, 2007, then President Gloria Macapagal-Arroyo granted pardon
penalties, the following rules shall be observed in the application thereof: to Estrada. The complete text of this pardon reads:
1. When in the commission of the deed there is present only one aggravating MALACAÑAN PALACE
circumstance, the greater penalty shall be applied. MANILA
2. When there are neither mitigating nor aggravating circumstances and there
is no aggravating circumstance, the lesser penalty shall be applied.
3. When the commission of the act is attended by some mitigating
circumstances and there is no aggravating circumstance, the lesser penalty shall
be applied.
4. When both mitigating and aggravating circumstances attended the
commission of the act, the court shall reasonably allow them to offset one another
in consideration of their number and importance, for the purpose of applying the WHEREAS, this Administration has a policy of releasing inmates who have
penalty in accordance with the preceding rules, according to the result of such reached the age of seventy (70),
compensation. WHEREAS, Joseph Ejercito Estrada has been under detention for six and a
385 half years,
VOL. 747, JANUARY 21, 2015 385 WHEREAS, Joseph Ejercito Estrada has publicly committed to no longer seek
any elective position or office,
Risos-Vidal vs. Commission on Elections
IN VIEW HEREOF and pursuant to the authority conferred upon me by the
The period within which accused Former President Joseph Ejercito Estrada
Constitution, I hereby grant executive clemency to JOSEPH EJERCITO
has been under detention shall be credited to him in full as long as he agrees
ESTRADA, convicted by the Sandiganbayan of Plunder and imposed a penalty
voluntarily in writing to abide by the same disciplinary rules imposed upon
of Reclusion Perpetua. He is hereby restored to his civil and political rights.
convicted prisoners.
The forfeitures imposed by the Sandiganbayan remain in force and in full,
Moreover, in accordance with Section 2 of Republic Act No. 7080, as amended
including all writs and processes issued by the Sandiganbayan in pursuance
by Republic Act No. 7659, the Court hereby declares the forfeiture in favor of the
hereof, except for the bank account(s) he owned before his tenure as President.
government of the following:
Upon acceptance of this pardon by JOSEPH EJERCITO ESTRADA, this
(1) The total amount of Five Hundred Forty-Two Million Seven Ninety-One
pardon shall take effect.
Thousand Pesos (P545,291,000.00)14 with interest and income earned, inclusive of
the amount of Two Hundred Million Pesos (P200,000,000.00), deposited in the
Given under my hand at the City of Manila, this 25th Day of October, in the
name and account of the Erap Muslim Youth Foundation.
year of Our Lord, two thousand and seven.
(2) The amount of One Hundred Eighty-Nine Million Pesos
Gloria M. Arroyo (sgd.)
(P189,000,000.00), inclusive of interests and income earned, deposited in the Jose
Velarde account.
By the President:
(3) The real property consisting of a house and lot dubbed as “Boracay
IGNACIO R. BUNYE (sgd.)
Mansion” located at #100 11th Street, New Manila, Quezon City.
Acting Executive Secretary16
The cash bonds posted by accused Jose “Jinggoy” Estrada and Atty. Edward S.
Serapio are hereby ordered cancelled and released to the said accused or their duly
On October 26, 2007, Estrada accepted the entire pardon without
authorized representatives upon presentation of the original receipt evidencing
qualifications. This acceptance is evidenced by a handwritten notation on the
payment thereof and subject to the usual accounting and auditing procedures.
pardon, which reads:
Likewise, the hold departure orders issued against the said accused are hereby
Received & accepted
recalled and declared functus officio.
SO ORDERED.15 (Emphasis and citations supplied)
Joseph E. Estrada (sgd.) (f) Permanent residents in a foreign country or those who have acquired the
DATE: 26 Oct. ‘07 right to reside abroad and continue to avail of the same right after the effectivity
TIME: 3:35 PM17 of this Code; and
(g) The insane or feeble-minded.
On October 2, 2012, Estrada filed his certificate of candidacy18for the position 21 Sec. 12. Disqualifications.—Any person who has been declared by
of Mayor of the City of Manila. competent authority insane or incompetent, or has been sentenced by final
On January 14, 2013, Risos-Vidal, a resident and registered voter of the City judgment for subversion, insurrection, rebellion or for any offense for which he has
of Manila, filed before public respondent COMELEC a petition for been sentenced to a penalty of more than eighteen months or for a crime involving
disqualification19 against Estrada. This petition, docketed as SPA No. 13-211 (DC), moral turpitude, shall be disqualified to be a candidate and to hold any office,
was filed pursuant to Section 40 of Republic Act No. 7160, otherwise known as the unless he has been given plenary pardon or granted amnesty.
Local Government Code of 1991 (the Local Government Code), 20 in relation to This disqualifications to be a candidate herein provided shall be deemed
Section 12 of Batas Pam- removed upon the declaration by competent authority that said insanity or
_______________ incompetence had been removed or after the expiration of a period of five years
from his service of sentence, unless within the same period he again becomes
16 Id., at p. 265. disqualified.
17 Id. Certified true copy issued by Marianito M. Dimaandal, Director IV, 22 Rollo, p. 267.
Malacañan Records Office. 23 Id., at pp. 284-296.
18 Id., at p. 266. 389
19 Id., at pp. 267-275. VOL. 747, JANUARY 21, 2015 389
20 Sec. 40. Disqualifications.—The following persons are disqualified from
running for any elective local position: Risos-Vidal vs. Commission on Elections
(a) Those sentenced by final judgment for an offense involving moral turpitude January 20, 2010 of the COMELEC Second Division and second, through the
or for an offense punishable by one (1) year or more of imprisonment, within two resolution of the COMELEC En Banc dated May 4, 2010 — the Commission on
(2) years after serving sentence; Elections had already ruled that the pardon granted to Estrada was absolute and
(b) Those removed from office as a result of an administrative case; unconditional and, hence, did not prevent him from running for public office. Thus,
(c) Those convicted by final judgment for violating the oath of allegiance to the the matter of Estrada’s qualification, in relation to the efficacy of the penalties
Republic; imposed on him on account of his conviction for plunder, “ha[d] been passed upon
388 and ruled out by this Commission way back in 2010.”24
In the resolution dated April 23, 2013, the COMELEC En Bancdenied Risos-
388 SUPREME COURT REPORTS ANNOTATED Vidal’s motion for reconsideration.
Risos-Vidal vs. Commission on Elections On April 30, 2013, Risos-Vidal filed the present petition.25 Risos-Vidal ascribed
bansa Blg. 881, otherwise known as the Omnibus Election Code.21 It sought to grave abuse of discretion amounting to lack or excess of jurisdiction on COMELEC
disqualify Estrada from running for Mayor of the City of Manila on account of his in not disqualifying Estrada. She assailed COMELEC’s refusal to grant her
conviction for plunder and having been sentenced to suffer the penalty of reclusion petition on account of its having supposedly ruled on the same issues in the
perpetua, and the accessory penalties of civil interdiction and perpetual absolute disqualification cases filed in connection with Estrada’s 2010 bid for the
disqualification.22 presidency.26 She asserted that Estrada’s pardon was conditional and served
Estrada filed his answer23 on January 24, 2013. neither to restore his rights “to vote, be voted upon and to hold public office” 27 nor
On April 1, 2013, the COMELEC Second Division issued the first assailed to remit the accessory penalty of perpetual absolute disqualification.28 She added
resolution dismissing Risos-Vidal’s petition for lack of merit. that, for having been convicted of plunder, a crime involving moral turpitude,
In this resolution, the COMELEC Second Division noted that in 2010, Estrada was barred from running for Mayor by Section 40 of the Local Government
following Estrada’s filing of a certificate of candidacy for President of the Code.29 Insisting that the grounds for disqualifying Estrada were so manifest, she
Philippines, two disqualification cases — SPA No. 09-028 (DC) and SPA No. 09- faulted COMELEC for not having disqualified motu proprio.30
104 (DC) — were filed against him. It added that, in deciding these disqualification
cases — first, through the resolution dated In the meantime, elections were conducted on May 13, 2013. Per COMELEC’s
_______________ “Certificate of Canvass of Votes and Proclamation of Winning Candidates for
National Capital Region – Manila” dated May 17, 2013,31 Estrada was noted to
(d) Those with dual citizenship; have obtained 349,770 votes.32 His opponent in the mayoralty race, Lim, obtained
(e) Fugitives from justice in criminal or nonpolitical cases here or abroad; 313,764 votes,33 giving the lead to Estrada. Estrada was, thus, proclaimed as the
“duly elected”34 city mayor.
On June 7, 2013, Lim filed a motion for leave to intervene 35 to which was 42 Id., at p. 438.
attached his petition-in-intervention.36 He argued that, regardless of whether the 43 Id., at pp. 457-485.
pardon granted to Estrada was absolute or conditional, it did not expressly restore 44 Id., at p. 460.
his right of suffrage and his right to hold public office, and it did not remit his 45 Id., at pp. 464-467.
perpetual absolute disqualification as required by Articles 3637 and 4138 of the 46 Id., at pp. 468-481
Revised Penal Code. Thus, he remained ineligible for election into public 47 Sec. 94. Disqualifications.—The following persons shall not be qualified
office.39 He added that, per this court’s decision in Dominador Jalos- to vote:
_______________ (a) Any person who has been sentenced by final judgment suffer eighteen
months or more of imprisonment, such disability not having been removed by
31 Id., at p. 726. plenary pardon.
32 Id. (b) Any person who has been declared by final judgment guilty of any crime
33 Id., at p. 437. against property.
34 Id., at p. 726. (c) Any person who has violated his allegiance to the United States or to the
35 Id., at pp. 390-393. Commonwealth of the Philippines.
36 Id., at pp. 395-412. (d) Insane or feeble-minded persons.
37 ARTICLE 36. Pardon; Its Effects.—A pardon shall not work the (e) Persons who can not prepare their ballots themselves.
restoration of the right to hold public office, or the right of suffrage, unless such 48 Sec. 12. Disqualifications.—Any person who has been declared by
rights be expressly restored by the terms of the pardon. competent authority insane or incompetent, or has been
A pardon shall in no case exempt the culprit from the payment of the civil 392
indemnity imposed upon him by the sentence. 392 SUPREME COURT REPORTS ANNOTATED
38 ARTICLE 41. Reclusion Perpetua and Reclusion Temporal — Their
accessory penalties.—The penalties of reclusion perpetua and reclusion Risos-Vidal vs. Commission on Elections
temporalshall carry with them that of civil interdiction for life or during the period “plenary pardon[s].” He added that Risos-Vidal’s assertions that President
of the sentence as the case may be, and that of perpetual absolute disqualification Gloria Macapagal-Arroyo could not have intended for Estrada’s pardon to be
which the offender shall suffer even though pardoned as to the principal penalty, absolute as they were “political rivals”49 is a factual issue that required the
unless the same shall have been expressly remitted in the pardon. “remand”50 of the case to the Court of Appeals or the reception of evidence through
39 Rollo, pp. 401-409. oral arguments.51
391 On July 29, 2013, public respondent COMELEC, through the Office of the
Solicitor General (OSG) filed its consolidated comment.52 It noted that the effects
VOL. 747, JANUARY 21, 2015 391 of the pardon granted to Estrada had already been ruled upon by COMELEC in
Risos-Vidal vs. Commission on Elections connection with disqualification cases filed against him on the occasion of his 2010
jos, Jr. v. COMELEC,40 he had the “right to be declared and proclaimed mayor bid for the presidency.53 It added that Estrada’s rights to vote and be voted for had
of Manila upon the declaration of respondent Estrada’s disqualification.” 41 indeed been restored and his perpetual disqualification remitted by the pardon
In the resolution42 dated June 25, 2013, this court granted Lim’s motion for granted to him.
leave to intervene and required respondents to file their comments on Lim’s On August 6, 2013, Estrada filed his comment54 on Risos-Vidal’s petition. In
petition-in-intervention in addition to filing their comment on Risos-Vidal’s addition to arguing that he was granted an absolute pardon which rendered him
petition. eligible to run and be voted as mayor, Estrada argued that the present case
On July 15, 2013, Estrada filed his comment on Lim’s petition-in-- involves the same issues as those in the 2010 disqualification cases filed against
intervention.43 He argued that Lim lacked “legal standing to prosecute this him, that “the findings of fact of the public
case,”44 that the pardon granted to him restored his right to seek public _______________
office,45 and that Articles 36 and 41 of the Revised Penal Code are not only
unconstitutional, as they diminish the pardoning power of the President, 46 but sentenced by final judgment for subversion, insurrection, rebellion or for any
have also been repealed by subsequent election laws (e.g., Section 94 of offense for which he has been sentenced to a penalty of more than eighteen months
Commonwealth Act No. 35747 and Section 12 of the Omnibus Election or for a crime involving moral turpitude, shall be disqualified to be a candidate and
Code),48 which recognize to hold any office, unless he has been given plenary pardon or granted amnesty.
_______________ This [sic] disqualifications to be a candidate herein provided shall be deemed
removed upon the declaration by competent authority that said insanity or
40 G.R. No. 193237, October 9, 2012, 683 SCRA 1 [Per J. Carpio, En Banc]. incompetence had been removed or after the expiration of a period of five years
41 Id., at p. 409.
from his service of sentence, unless within the same period he again becomes 1. Whether private respondent Joseph Ejercito Estrada was qualified to run
disqualified. for Mayor of the City of Manila; and
49 Rollo, p. 483. 2. Assuming private respondent Joseph Ejercito Estrada was not qualified,
50 Id. whether petitioner-intervenor Alfredo S. Lim should be declared Mayor of the City
51 Id. of Manila.
52 Id., at pp. 489-507. At the core of this case is the issue of whether Estrada was qualified to run for
53 Id., at p. 498. Mayor of the City of Manila. Estrada, however, has invoked several procedural
54 Id., at pp. 574-610. issues that, if decided in his favor, would effectively impede this court’s having to
393 rule on the substantive issue of his qualification. All of these procedural obstacles
VOL. 747, JANUARY 21, 2015 393 lack merit and should not prevent this court from ruling on Estrada’s qualification.

Risos-Vidal vs. Commission on Elections IV


respondent COMELEC relative to the absoluteness of the pardon, the effects
thereof and the eligibility of the Private Respondent Estrada are binding and The petition filed by petitioner Atty.
conclusive”55 on this court, and that the allegations made by Risos-Vidal are Alicia Risos-Vidal with COMELEC
insufficient to disturb the assailed resolutions. 56 He added that Risos-Vidal’s was filed on time
petition before the COMELEC was filed out of time, it being, in reality, a petition
to deny due course to or to cancel his certificate of candidacy, and not a petition for Estrada argues that the petition filed by Risos-Vidal before the COMELEC
disqualification.57 He also asserted that Dominador Jalosjos, Jr. was inapplicable should be treated as a petition to deny due course to or to cancel a certificate of
to the present case.58 Finally, he claimed that his disqualification would mean the candidacy (CoC) under Section 78 of Batas Pambansa Blg. 881, otherwise known
disenfranchisement of the voters who elected him.59 as the Omnibus Election Code67 (Section 78 petition). He claims that the petition
On August 23, 2013, Lim filed his reply to Estrada’s comment on his petition- effectively assailed the falsity of a representation he made in his CoC — that is,
in-intervention and to COMELEC’s consolidated comment.60 On August 27, 2013, that he was eligible for the office he sought to be elected to — and, therefore,
Risos-Vidal filed her reply61 to Estrada’s comment on her petition. On December invoked a ground for a Section 78 petition, rather than a ground for a petition for
13, 2013, Risos-Vidal filed her reply62 to COMELEC’s consolidated comment. disqualification.
In the resolution dated April 22, 2014, the petition and petition-in-intervention Estrada adds that Rule 23, Section 2 of COMELEC Resolution No.
were given due course and the parties required to submit their memoranda. The 952368 provides that a Section 78 petition must be filed within five (5) days from
parties complied: Lim on May 27, 2014,63 Risos-Vidal on June 2, 2014,64 Estrada the last day for filing a CoC, but not later than 25 days from the time of the filing
on June 16, 2014,65and COMELEC on June 26, 2014.66 of the CoC specifically subject of the petition. He claims that, since Risos-
_______________
III
67 Sec. 78. Petition to deny due course to or cancel a certificate of
Statement of issues candidacy.—A verified petition seeking to deny due course or to cancel a certificate
of candidacy may be filed by the person exclusively on the ground that any material
For resolution are the following issues: representation contained therein as required under Section 74 hereof is false. The
petition may be filed at any time not later than twenty-five days from the time of
A. Procedural issues the filing of the certificate of candidacy and shall be decided, after due notice and
hearing, not later than fifteen days before the election.
1. Whether the petition filed by petitioner Atty. Alicia Risos-Vidal before the 68 Sec. 2. Period to File Petition.—The Petition must be filed within five (5)
COMELEC was filed on time; days from the last day for filing of certificate of candidacy; but not later than
2. Whether petitioner-intervenor Alfredo S. Lim may intervene in this case; twenty-five (25) days from the time of filing of the certificate of candidacy subject
and of the Petition. In case of a substitute candidate, the Petition must be filed within
3. Whether COMELEC’s rulings in the disqualification cases filed against five (5) days from the time the substitute candidate filed his certificate of
private respondent Joseph Ejercito Estrada in connection with his 2010 bid for the candidacy.
presidency bar the consideration of the petition filed by petitioner Atty. Alicia
Risos-Vidal before the COMELEC, as well as the present petition for certiorari. Vidal’s petition was all but a “camouflaged”69 petition for disqualification, Rule
25, Section 3 of COMELEC Resolution No. 9523,70 which allows for petitions for
B. Substantive issues disqualification to be “filed any day after the last day for filing of certificates of
candidacy, but not later than the date of proclamation” finds no application. As
Risos-Vidal’s petition was filed before the COMELEC on January 14, 2013 — one to influence, induce or corrupt the voters or public officials performing electoral
hundred and four (104) days removed from October 2, 2012, when he filed his CoC functions; (b) committed acts of terrorism to enhance his candidacy; (c) spent in his
— Estrada argues that Risos-Vidal’s petition was belatedly filed and, hence, should election campaign an amount in excess of that allowed by this Code; (d) solicited,
have been summarily dismissed by COMELEC. received or made any contribution prohibited under Sections 89, 95, 96, 97 and
Estrada’s assertion is erroneous. 104; or (e) violated any of Sections 80, 83, 85, 86 and 261, paragraphs d, e, k, v,
This court’s 2008 decision in Fermin v. COMELEC71 allowed for an and cc, subparagraph 6, shall be disqualified from continuing as a candidate, or if
opportunity “to dichotomize, once and for all, two popular remedies to prevent a he has been elected, from holding the office. Any person who is a permanent
candidate from running for an elective position which are indiscriminately resident of or an immigrant to a foreign country shall not be qualified to run for
interchanged by the Bench and the Bar”:72 on the one hand, a petition to deny due any elective office under this Code, unless said person has waived his status as a
course to or to cancel a certificate of candidacy under Section 78 of the Omnibus permanent resident or immigrant of a foreign country in accordance with the
Election Code and, on the other, a petition for disqualification under Section 68 of residence requirement provided for in the election laws.
the Omnibus Election Code (Section 68 petition). Likewise, the other provisions of law referring to “disqualification” do not
The two remedies, and their distinctions, were discussed in the course of this include the lack of the one-year residency qualification as a ground therefor, thus:
court’s characterization of the petition involved in Fermin — whether it was a Section 12 of the OEC
Section 78 petition or a Section 68 petition — considering that such petition was SEC. 12. Disqualifications.—Any person who has been declared by
anchored on an allegation that a candidate for Mayor was ineligible for failing to competent authority insane or incompetent, or has been sentenced by final
satisfy the requirement of residency of at least one (1) year immediately preceding judgment for subversion, insurrection, rebellion, or for any offense for which he
the election. The problem of characterization is the same issue facing us at this has been sentenced to a penalty of more than eighteen months or for a crime
juncture: involving moral turpitude, shall be disqualified to be a candidate and to hold any
Lest it be misunderstood, the denial of due course to or the cancellation of the office, unless he
CoC is not based on the lack of qualifications but on a finding that the candidate 399
made a material representation that is false, which may relate to the VOL. 747, JANUARY 21, 2015 399
qualifications required of the public office he/she is running for. It is noted that
the candidate states in his/her CoC that he/she is eligible for the office he/she Risos-Vidal vs. Commission on Elections
seeks. Section 78 of the OEC, therefore, is to be read in relation to the has been given plenary pardon or granted amnesty.
constitutional and statutory provisions on qualifications or eligibility for public The disqualifications to be a candidate herein provided shall be deemed
office. If the candidate subsequently states a material representation in the CoC removed upon the declaration by competent authority that said insanity or
that is false, the COMELEC, following the law, is empowered to deny due course incompetence had been removed or after the expiration of a period of five years
to or cancel such certificate. Indeed, the Court has already likened a proceeding from his service or sentence, unless within the same period he again becomes
under Section 78 to a quo warrantoproceeding under Section 253 of the OEC since disqualified.
they both deal with the eligibility or qualification of a candidate, with the Section 40 of the Local Government Code (LGC)
distinction mainly in the fact that a “Section 78” petition is filed before SECTION 40. Disqualifications.—The following persons are disqualified
proclamation, while a petition for quo warranto is filed after proclamation of the from running for any elective local position:
winning candidate. (a) Those sentence by final judgment for an offense involving moral turpitude
At this point, we must stress that a “Section 78” petition ought not to be or for an offense punishable by one (1) year or more of imprisonment, within two
interchanged or confused with a “Section 68” petition. They are different remedies, (2) years after serving sentence;
based on different grounds, and resulting in different eventualities. . . . (b) Those removed from office as a result of an administrative case;
The ground raised in the Dilangalen petition is that Fermin allegedly lacked (c) Those convicted by final judgment for violating the oath of allegiance to
one of the qualifications to be elected as mayor of Northern Kabuntalan, i.e., he the Republic;
had not established residence in the said locality for at least one year immediately (d) Those with dual citizenship;
preceding the election. Failure to meet the one-year residency requirement for the (e) Fugitive from justice in criminal or nonpolitical cases here or abroad;
public office is not a ground for the “disqualification” of a candidate under Section (f) Permanent residents in a foreign country or those who have acquired the
68. [Section 68] only refers to the commission of prohibited acts and the right to reside abroad and continue to avail of the same right after the effectivity
possession of a permanent resident status in a foreign country as grounds of this Code; and
for disqualification, thus: (g) The insane or feeble-minded.
Considering that the Dilangalen petition does not state any of these grounds
SEC. 68. Disqualifications.—Any candidate who, in an action or protest in for disqualification, it cannot be categorized as a “Section 68” petition.
which he is a party is declared by final decision of a competent court guilty of, or
found by the Commission of having (a) given money or other material consideration To emphasize, a petition for disqualification, on the one hand, can be premised
on Section 12 or 68 of the [Omnibus Election Code], or Section 40 of the [Local
Government Code]. On the other hand, a petition to deny due course to or cancel a Government Code vis-à-visresort to Section 78 petitions. Any standing ambiguity
CoC can only be grounded on a statement of a material representation in the said was settled by this court’s discussion in Dominador Jalosjos, Jr. v. Commission on
certificate that is false. . . .73 (Emphasis supplied, citations omitted) Elections.76
In Dominador Jalosjos, Jr., this court affirmed the COMELEC’s grant of a
The quoted discussion clearly establishes the distinction of when it is proper to Section 78 petition and sustained the cancellation of the certificate of candidacy
resort to a Section 78 petition as against a petition for disqualification under filed by Dominador Jalosjos, Jr. in his bid to be elected Mayor of Dapitan City,
Section 68 of the Omnibus Election Code: (1) a Section 78 petition is proper when Zamboanga del Norte in the May 10, 2010 elections. This cancellation was
a statement of a material representation in a certificate of candidacy is false; and premised on a finding that Jalosjos, Jr. made a ma-
(2) a Section 68 petition is proper when disqualification is sought on account of _______________
having committed electoral offenses and/or possession of status as a permanent
resident in a foreign country. 75 Id., at pp. 141-142.
Fermin, however, did not just touch on petitions for disqualification anchored 76 Supra note 40.
on Section 68 of the Omnibus Election Code, but also on petitions for 402
disqualification anchored on Section 12 of the Omnibus Election Code and on
402 SUPREME COURT REPORTS ANNOTATED
Section 40 of the Local Government Code. Fermin made the pronouncement that
Section 12 of the Omnibus Election Code and Section 40 of the Local Government Risos-Vidal vs. Commission on Elections
Code are equally valid grounds for a petition for disqualification. terial misrepresentation in his CoC in stating that he was eligible for election.
Nevertheless, Fermin was not categorical on when a petition for disqualification Jalosjos, Jr. had previously been convicted of robbery and sentenced to suffer the
anchored on these statutory provisions may be resorted to vis-à-vis a Section 78 accessory penalty of perpetual special disqualification. In sustaining the
petition. cancellation of his CoC, this court reasoned:
A subsequent case, Aratea v. COMELEC,74 affirms that petitions for The perpetual special disqualification against Jalosjos arising from his
disqualification may be anchored on Section 12 of the Omnibus Election Code, criminal conviction by final judgment is a material fact involving eligibility which
and/or Section 40 of the Local Government Code, much as they can be anchored on is a proper ground for a petition under Section 78 of the Omnibus Election Code.
Section ....
_______________ A false statement in a certificate of candidacy that a candidate is eligible to
run for public office is a false material representation which is a ground for a
73 Id., at pp. 465-469; pp. 792-796. petition under Section 78 of the same Code. . . .
74 G.R. No. 195229, October 9, 2012, 683 SCRA 105 [Per J. Carpio, En Banc]. ....
401 Section 74 requires the candidate to state under oath in his certificate of
candidacy “that he is eligible for said office.” A candidate is eligible if he has a right
VOL. 747, JANUARY 21, 2015 401
to run for the public office. If a candidate is not actually eligible because he is
Risos-Vidal vs. Commission on Elections barred by final judgment in a criminal case from running for public office, and he
68 of the Omnibus Election Code: “A petition for disqualification can only be still states under oath in his certificate of candidacy that he is eligible to run for
premised on a ground specified in Section 12 or 68 of the Omnibus Election Code public office, then the candidate clearly makes a false material representation that
or Section 40 of the Local Government Code.”75 is a ground for a petition under Section 78.77 (Citations omitted)
Likewise, Rule 25, Section 1 of COMELEC Resolution No. 9523 indicates that
a petition for disqualification is based on legally (i.e., by Constitution or by statute) From these, it is clear that a false claim of eligibility made in a certificate of
prescribed disqualifications. It provides: candidacy despite a prior conviction which carries with it the accessory penalty of
Section 1. Grounds.—Any candidate who, in an action or protest in which he disqualification is a ground for a Section 78 petition. Nevertheless, it is also a
is a party, is declared by final decision of a competent court, guilty of, or found by ground for a petition for disqualification. As explained in Dominador Jalosjos, Jr.:
the Commission to be suffering from any disqualification provided by law _______________
or the Constitution.
A Petition to Disqualify a Candidate invoking grounds for a Petition to Deny 77 Id., at pp. 20-21.
to or Cancel a Certificate of Candidacy or Petition to Declare a Candidate as a 403
Nuisance Candidate, or a combination thereof, shall be summarily dismissed.
VOL. 747, JANUARY 21, 2015 403
(Emphasis supplied)
However, Aratea and COMELEC Resolution No. 9523, like Fermin, are Risos-Vidal vs. Commission on Elections
uncategorical on the availability of petitions for disqualification anchored on What is indisputably clear is that the false material representation of Jalosjos
Section 12 of the Omnibus Election Code and/or Section 40 of the Local is a ground for a petition under Section 78. However, since the false material
representation arises from a crime penalized by prisión mayor, a petition under
Section 12 of the Omnibus Election Code or Section 40 of the Local Government recognized as a valid basis for both a petition for disqualification and a Section 78
Code can also be properly filed. The petitioner has a choice whether to anchor his petition.
petition on Section 12 or Section 78 of the Omnibus Election Code, or on Section It follows that the petition was filed on time. The petition was filed on January
40 of the Local Government Code. The law expressly provides multiple 14, 2013, after the last day for filing of certificates of candidacy, and before the
remedies and the choice of which remedy to adopt belongs to the date of Estrada’s proclamation as Mayor on May 17, 2013. This is within the period
petitioner.78 permitted by Rule 25, Section 3 of COMELEC Resolution No. 9523.

The concurrent availability of a Section 78 petition with a petition for V


disqualification should not be interpreted as diminishing the distinction between
the two (2) remedies. Alfredo S. Lim may intervene in the
The pivotal consideration in a Section 78 petition is material present petition for certiorari
misrepresentation relating to qualifications for elective public office. To
“misrepresent” is “to describe (someone or something) in a false way especially in Citing Section 44 of the Local Government Code83 — on succession in case of
order to deceive someone.”79 It, therefore, connotes malevolent intent or bad faith permanent vacancies in the Office of the
that impels one to adulterate information. A Section 78 petition thus, squarely _______________
applies to instances in which a candidate is fully aware of a matter of fact that
disqualifies him or her but conceals or otherwise falsely depicts that fact as to 82 Id., at p. 271.
make it appear that he or she is qualified. A petition for disqualification, on the 83 Sec. 44. Permanent Vacancies in the Offices of the Governor, Vice
other hand, may apply in cases where a disqualification exists but, because of an Governor, Mayor, and Vice Mayor.—
attendant ambiguity (such as an unsettled legal question), a candidate acts in good (a) If a permanent vacancy occurs in the office of the governor or mayor, the
faith and without any deliberate attempt to conceal or mislead. vice governor or vice mayor concerned shall become the governor or mayor. If a
Right at the onset, the petition filed by Risos-Vidal before the COMELEC on permanent vacancy occurs in the offices of the governor, vice governor, mayor, or
January 14, 2013 asserts that it was filed pursuant to Section 40 of the Local vice mayor, the highest ranking sanggunian member or, in case of his permanent
Government Code, “in relation to”80 Section 12 of the Omnibus Election Code: inability, the second highest ranking sanggunian member, shall become the
This is a petition pursuant to Sec. 40 of R.A. No. 7160, otherwise known as “The governor, vice governor, mayor or vice mayor, as the case may be. Subsequent
Local Government Code of 1991,” in relation to Sec. 12 of BP Blg. 881, otherwise vacancies in the said office shall be filled automatically by the
known as the “Omnibus Election Code of the Philippines,” seeking to disqualify other sanggunian members according to their ranking as defined herein.
former President Joseph Ejercito Estrada from running for the mayoralty position (b) If a permanent vacancy occurs in the office of the punong barangay, the
in Manila in the coming May 13, 2013 elections, on the ground of his prior highest ranking sanggunian barangay member
conviction of the crime of plunder by the Sandiganbayan and his having been 406
sentenced to reclusion perpetua with the accessory penalties of civil interdiction
406 SUPREME COURT REPORTS ANNOTATED
and perpetual absolute disqualification.81 (Emphasis supplied)
Risos-Vidal vs. Commission on Elections
This petition posits that Estrada is disqualified from running as Mayor of the
City of Manila, pursuant to Section 40 of the Local Government Code, as follows: Mayor — and jurisprudence to the effect that “the candidate who obtains the
Sec. 40 of the LGC provides that a person sentenced by final judgment for an second highest number of votes may not be proclaimed winner in case the winning
offense involving moral turpitude or for an offense punishable by imprisonment of candidate is disqualified,”84 Estrada claims that “the party who stands to benefit
one (1) year or more is disqualified from running for any elective local position. in the event of [his] disqualification is none other than the duly elected Vice Mayor
As earlier said, respondent was sentenced in Crim. Case No. 26558 to suffer of the City of Manila, Isko Moreno.”85 Thus, he asserts that “it is clear that Lim
the penalty of reclusion perpetua. has NO LEGAL STANDING to institute his Petition-In-Intervention.”86
He was, however, granted pardon by former Pres. Gloria Macapagal-Arroyo, In the first place, Estrada is erroneously invoking the concept of “legal
thus, did not serve his sentence in full. standing.” What Estrada is really questioning is whether Lim is a real party-in-
Nonetheless, while the pardon did restore to him his civil and political rights, interest.
it did not restore to him his right to run for or hold public office or the right of The distinction between the rule on standing and real party-in-interest was
suffrage because it was not expressly restored by the terms of the pardon. . . .82 extensively discussed by this court in Kilosbayan, Inc. v. Morato:87
_______________
This petition unambiguously anchors itself on statutorily prescribed
disqualifications — under Section 40 of the Local Government Code, as well as or, in case of his permanent inability, the second highest
Section 12 of the Omnibus Election Code — which jurisprudence has explicitly ranking sanggunianmember, shall become the punong barangay.
(c) A tie between or among the highest ranking sanggunian members shall be 408 SUPREME COURT REPORTS ANNOTATED
resolved by the drawing of lots.
(d) The successors as defined herein shall serve only the unexpired terms of Risos-Vidal vs. Commission on Elections
their predecessors. avails of the suit.’” (Salonga v. Warner Barnes & Co., Ltd., 88 Phil. 125, 131
For purposes of this Chapter, a permanent vacancy arises when an elective [1951])88 (Emphasis supplied)
local official fills a higher vacant office, refuses to assume office, fails to qualify,
dies, is removed from office, voluntarily resigns, or is otherwise permanently In seeking to intervene, Lim has made no pretensions of acting as a
incapacitated to discharge the functions of his office. representative of the general public and, thus, advancing the public interest. He
For purposes of succession as provided in this Chapter, ranking in merely prays that he be declared the elected Mayor of the City of Manila following
the sanggunianshall be determined on the basis of the proportion of votes obtained a declaration that Estrada was disqualified to run for the same post. Though what
by each winning candidate to the total number of registered voters in each district is involved is a public office, what Lim seeks to enforce is, fundamentally, a
in the immediately preceding local election. (supposed) right accruing to him personally to assume an office.
84 Codilla, Sr. v. De Venecia, 442 Phil. 139, 182; 393 SCRA 639, 675 (2002) Lim has enough interest at stake in this case as would enable him to intervene.
[Per J. Puno, En Banc]. Rule 19, Section 1 of the 1997 Rules of Civil Procedure provides for who may
85 Rollo, p. 1757, emphasis and capitalization in the original. intervene in a pending court action:
86 Id. Section 1. Who may intervene.—A person who has a legal interest in the
87 316 Phil. 652; 246 SCRA 540 (1995) [Per J. Mendoza, En Banc]. matter in litigation, or in the success of either of the parties, or an interest against
407 both, or is so situated as to be adversely affected by a distribution or other
disposition of property in the custody of the court or of an officer thereof may, with
VOL. 747, JANUARY 21, 2015 407
leave of court, be allowed to intervene in the action. The court shall consider
Risos-Vidal vs. Commission on Elections whether or not the intervention will unduly delay or prejudice the adjudication of
Not only is petitioners’ standing a legal issue that may be determined again in the rights of the original parties, and whether or not the intervenor’s rights may
this case. It is, strictly speaking, not even the issue in this case, since standing is be fully protected in a separate proceeding. (Emphasis supplied)
a concept in constitutional law and here no constitutional question is actually
involved. The issue in this case is whether petitioners are the “real parties-in- The requirement of “legal interest” was discussed in Magsaysay-Labrador v.
interest” within the meaning of Rule 3, §2 of the Rules of Court which requires Court of Appeals,89 as follows:
that “Every action must be prosecuted and defended in the name of the real party- The interest which entitles a person to intervene in a suit between other parties
in-interest.” must be in the matter in litiga-
The difference between the rule on standing and real party-in-interest has _______________
been noted by authorities thus: It is important to note . . . that standing because of
its constitutional and public policy underpinnings, is very different from questions 88 Id., at pp. 695-696; pp. 562-563.
relating to whether a particular plaintiff is the real party-in-interest or has capacity 89 259 Phil. 748; 180 SCRA 266 (1989) [Per CJ. Fernan, En Banc].
to sue. Although all three requirements are directed towards ensuring that only 409
certain parties can maintain an action, standing restrictions require a partial VOL. 747, JANUARY 21, 2015 409
consideration of the merits, as well as broader policy concerns relating to the proper
role of the judiciary in certain areas. (FRIEDENTHAL, KANE AND MILLER, Risos-Vidal vs. Commission on Elections
CIVIL PROCEDURE 328 [1985]) tion and of such direct and immediate character that the intervenor will either
Standing is a special concern in constitutional law because in some cases suits gain or lose by the direct legal operation and effect of the judgment. Otherwise, if
are brought not by parties who have been personally injured by the operation of a persons not parties of the action could be allowed to intervene, proceedings will
law or by official action taken, but by concerned citizens, taxpayers or voters who become unnecessarily complicated, expensive and interminable. And this is not the
actually sue in the public interest. Hence the question in standing is whether such policy of the law.
parties have “alleged such a personal stake in the outcome of the controversy as to The words “an interest in the subject” mean a direct interest in the cause of
assure that concrete adverseness which sharpens the presentation of issues upon action as pleaded, and which would put the intervenor in a legal position to litigate
which the court so largely depends for illumination of difficult constitutional a fact alleged in the complaint, without the establishment of which plaintiff could
questions.” (Baker v. Carr, 369 U.S. 186, 7 L.Ed. 2d 633 [1962]) not recover.90 (Emphasis supplied)
....
On the other hand, the question as to “real party-in-interest” is whether he is It is true that the principal matter for resolution in this case is whether
“the party who would be benefitted or injured by the judgment, or the ‘party entitled Estrada, based on circumstances personally applying to him, was qualified to run
to the for Mayor of the City of Manila. Nevertheless, the logical consequence of a decision
408
adverse to Estrada is the need to identify who shall, henceforth, assume the conditions is that statement in the whereas clause thereof that contained the
position of Mayor. following: “WHEREAS, Joseph Ejercito Estrada has publicly committed to no
Lim claims that he is entitled to replace Estrada. In support of this, he cites a longer seek any elective position or office,” but that is not really a condition but is
decision of this court91 and claims that, as a disqualified candidate, the votes cast merely part of a preliminary statement, referring to what respondent Estrada had
for Estrada should be deemed stray votes. This would result in Lim being said publicly. There is nothing stated in the dispositive part that it was conditioned
the qualified candidate obtaining the highest number of votes, which would, in upon said respondent’s purported public commitment. His public statement
turn, entitle him to being proclaimed the elected Mayor of the City of Manila. cannot, therefore, serve to restrict the operation of, or prevail over the explicit
It is worth emphasizing that “[t]he purpose of intervention is to enable a statement in the executive clemency which restored all of Estrada’s civil and
stranger to an action to become a party in order for him to protect his interest and political rights, including “the right to vote and to be voted for a public office,”
for the court to settle all conflicting claims. Intervention is allowed to avoid including to the position of the Presidency. This executive clemency granted to the
multiplicity of suits more than on due process considerations.”92 Lim’s intervention former President being absolute and unconditional and having been accepted by
serves this purpose. It enables the resolution of an issue which is corollary to one him, the same can no longer be revoked or be made subject to a condition. 98
of the two ways by which this court may decide on the issue of Estrada’s
disqualification. b. The 2010 disqualification cases and Risos-Vidal’s
petition are anchored on different causes of action and, hence, involve
VI different issues and subject matters

This case is not barred by Res judicata was discussed in Pryce Corporation v. China Banking
COMELEC’s rulings in the Corporation99 as follows:
disqualification cases filed against According to the doctrine of res judicata, “a final judgment or decree on the
Estrada in connection with his 2010 merits by a court of competent jurisdiction is conclusive of the rights of the parties
bid for the presidency or their privies in all later suits on all points and matters determined in the former
a. Estrada’s theory: suit.”
case is barred by res The elements for res judicata to apply are as follows: (a) the former judgment
judicata was final; (b) the court that rendered it had jurisdiction over the subject matter
and the parties; (c) the judgment was based on the merits; and (d) between the
Estrada avers that in 2010, in connection with what was then his second bid first and the second actions, there was an identity of parties, subject matters, and
for the presidency of the Republic, two (2) disqualification cases were filed against causes of action.
him: one, by a certain Atty. Evilio C. Pormento, docketed as SPA No. 09-028 (DC); Res judicata embraces two concepts: (1) bar by prior judgment and (2)
and two, by a certain Mary Lou B. Estrada, docketed as SPA No. 09-104 (DC). In conclusiveness of judgment.
the resolution dated January 20, 2010,93 the COMELEC Second Division denied Bar by prior judgment exists “when, as between the first case where the
these disqualification petitions for lack of merit and upheld Estrada’s qualification judgment was rendered and the second case that is sought to be barred, there is
to run for President. In the resolution dated April 27, 2010, 94 the COMELEC En identity of parties, subject matter, and causes of action.”
Banc denied Mary Lou B. Estrada’s motion for reconsideration. In another On the other hand, the concept of conclusiveness of judgment finds application
resolution dated May 4, 2010, the COMELEC En Banc denied Pormento’s motion “when a fact or question has been squarely put in issue, judicially passed upon,
for reconsideration.95 and adjudged in a former suit by a court of competent jurisdiction.” This principle
only needs identity of parties and issues to apply.100
Estrada claims that “[t]he issue surrounding the character of [his] pardon and
eligibility to seek public elective office was already extensively dealt with and The 2010 disqualification cases filed against Estrada in connection with his
passed upon”96 in these disqualification cases. He asserts that as these cases 2010 bid for the presidency do not bar the present case on account of res judicata.
involved and resolved “the same or identical issues,” 97 the present case is now For one, the 2010 disqualification cases filed by Atty. Evilio C. Pormento and
barred by res judicata. Mary Lou B. Estrada involved issues and were anchored on causes of action that
Estrada draws particular attention to the following pronouncement of the are markedly different from those in the present case. These cases were anchored
COMELEC Second Division in its January 20, 2010 resolution: on the constitutional prohibition against a President’s reelection, as provided by
Furthermore, there is absolutely no indication that the executive clemency Article VII, Section 4 of the 1987 Constitution,101 and the additional ground that
exercised by President Arroyo to pardon Former President Estrada was a mere Estrada was a nuisance
conditional pardon. It clearly stated that the former president is “restored to his _______________
civil and political rights” and there is nothing in the same which limits this
restoration. The only therein stated that may have some bearing on the supposed 100 Id.
101 Sec. 4. The President and the Vice President shall be elected by direct The Congress shall promulgate its rules for the canvassing of the certificates.
vote of the people for a term of six years which shall begin at noon on the thirtieth The Supreme Court, sitting En Banc, shall be the sole judge of all contests
day of June next following the day of the election and shall end at noon of the same relating to the election, returns, and qualifications of the President or Vice
date, six years thereafter. The President shall not be eligible for any reelection. No President, and may promulgate its rules for the purpose.
person who has succeeded as President and has served as such for more than four 102 Rollo, p. 619.
years shall be qualified for election to the same office at any time. 415
No Vice President shall serve for more than two successive terms. Voluntary VOL. 747, JANUARY 21, 2015 415
renunciation of the office for any length of time shall not be considered as an
interruption in the continuity of the service for the full term for which he was Risos-Vidal vs. Commission on Elections
elected.
Unless otherwise provided by law, the regular election for President and Vice Disqualify Estrada Ejercito Joseph M. from running as President due to
President shall be held on the second Monday of May. Constitutional Disqualification and Creating Confusion to the Prejudice of
The returns of every election for President and Vice President, duly certified Estrada, Mary Lou B.” and prayed for the disqualification of the Respondent and
by the board of canvassers of each province or city, shall be transmitted to the to have his Certificate of Candidacy (COC) cancelled. She also made reference to
Congress, directed to the President of the Senate. Upon receipt of the certificates the Respondent being a “Nuisance Candidate.”103 (Emphasis supplied)
of canvass, the President of the Senate shall, not later than thirty days after the
day of the election, open all the certificates in the presence of the Senate and the That these disqualification cases involved issues and invoked causes of action
House of Representatives in joint public session, and the Congress, upon that are different from those in this case is evident in the recital of issues in the
determination of the authenticity and due execution thereof in the manner COMELEC Second Division’s January 20, 2010 resolution:
provided by law, canvass the votes. THE ISSUES IN THE TWO CASES
414 (a) Whether or not Respondent Joseph Ejercito Estrada is qualified to be a
candidate for the position of President of the Philippines in the forthcoming
414 SUPREME COURT REPORTS ANNOTATED
elections on May 10, 2010, despite the fact that he had previously been elected to,
Risos-Vidal vs. Commission on Elections assumed and discharged the duties of, the same position;
candidate. To the contrary, the present case is anchored on Estrada’s (b) Whether or not, former President Estrada may be considered a nuisance
conviction for plunder which carried with it the accessory penalty of perpetual candidate in view of the Constitutional prohibition against any reelection of a
absolute disqualification and invokes Section 40 of the Local Government Code, as former President who has previously elected and had assumed the same
well as Section 12 of the Omnibus Election Code. position.104
The COMELEC Second Division, summarizing the circumstances of the (Emphasis supplied)
petition for disqualification subject of SPA No. 09-028 (DC), filed by Atty. Evilio C.
Pormento, stated: This, too, is evident, in the resolution’s introductory paragraphs:
Petitioner Evilio C. Pormento filed the first case against Respondent Joseph At the very core of the controversy involved in these two cases which stands
Ejercito Estrada on December 05, 2009. It was properly titled an “Urgent Petition like a stratospheric totem pole is the specific provision under Sec. 4 of Article VII
for Disqualification as Presidential Candidate.” This Petition is premised on the of the 1987 Constitution which states:
specific provision of Article VII, Section 4 of the 1987 Constitution a portion of ....
which stated that: x x x the President shall not be eligible for any
reelection.”102 (Emphasis in the original) This Commission (Second Division) is confronted with the dilemma of deciding
a brewing controversy considering the above Constitutional provision which
On the other hand, summarizing the circumstances of the petition filed by prohibits reelection of “the President”; that is, whether former President Joseph
Mary Lou B. Estrada, the COMELEC Second Division stated: Ejercito “Erap” Estrada may or may not be allowed to run in the coming May 2010
The second of the above entitled cases was filed on December 12, 2009, by elections for the same position of the President of the Republic of the
Petitioner Mary Lou Estrada alleging that the name of Joseph M. Ejercito Estrada Philippines?105 (Emphasis supplied)
might cause confusion to her prejudice. She filed a “Petition to
_______________ Whatever pronouncement the COMELEC Second Division made on the matter
of Estrada’s conviction for plunder and subsequent pardon was thus a superfluity.
The person having the highest number of votes shall be proclaimed elected, but Ultimately, it was unnecessary to the resolution of the issues involved in the
in case two or more shall have an equal and highest number of votes, one of them disqualification cases filed by Atty. Evilio C. Pormento and Mary Lou B. Estrada.
shall forthwith be chosen by the vote of a majority of all the Members of both It was nothing more than obiter dictum.
Houses of the Congress, voting separately. Another disqualification case filed in connection with Estrada’s 2010 bid for
the presidency, which, however, Estrada did not cite in his averments was Rev.
Elly Velez B. Lao Pamatong, ESQ, petitioner, v. Joseph Ejercito Estrada and Gloria sought to be reviewed[; moreover,] petitioner did not even pray for the issuance of
Macapagal-Arroyo, SPA No. 09-024 (DC). This case was similarly focused on the a temporary restraining order or writ of preliminary injunction.”111 Thus, Estrada
constitutional prohibition against a President’s reelection and on the allegation was able to participate in the May 10, 2010 presidential elections. He, however,
that Estrada was a nuisance candidate: only obtained the second highest number of votes and was, thus, not proclaimed
The bone of contention of this controversy revolves around the interpretation winner.
of the specific provisions of Sec. 4 of Article VII of the 1987 Constitution. . . .106 Not having been elected President for a second time, this court ruled that Atty.
Its recital of issues reads: Evilio C. Pormento’s petition had become moot and academic. Thus, it was denied
From the foregoing, the Commission (Second Division) hereby rules on the due course and dismissed:
following issues: Private respondent was not elected President the second time he ran. Since the
(a) Can a former elected President be qualified to become a Presidential issue on the proper interpretation of the phrase “any reelection” will be premised
Candidate and be elected again to the same position he or she previously occupied? on a person’s second (whether immediate or not) election as President, there is no
(b) May President Arroyo being a sitting President be allowed to run for any case or controversy to be resolved in this case. No live conflict of legal rights exists.
elected position such as a member of the House of Representatives? There is in this case no definite, concrete, real or substantial controversy that
(c) Are President Arroyo and Former President Estrada nuisance touches on the legal relations of parties having adverse legal interests. No specific
candidates?107 relief may conclusively be decreed upon by this Court in this case that will benefit
any of the parties herein. As such, one of the essential requisites for the exercise
That the 2010 disqualification cases were anchored on a constitutional of the power of judicial review, the existence of an actual case or controversy, is
provision relating to the executive branch of government, while the present case is sorely lacking in this case.
anchored on the provisions of the Local Government Code on the disqualification As a rule, this Court may only adjudicate actual, ongoing controversies. The
of candidates for local elective offices, makes evident that the former entailed a Court is not empowered to decide moot questions or abstract propositions, or to
different subject matter. While the 2010 disqualification cases relate to Estrada’s declare principles or rules of law which cannot affect the result as to the thing in
bid for the presidency, the present case relates to his bid to become Mayor of the issue in the case before it. In other words, when a case is moot, it becomes
City of Manila. nonjusticiable.
An action is considered “moot” when it no longer presents a justiciable
c. There was no final judgment on the merits arising from the 2010 controversy because the issues involved have become academic or dead or when
disqualification cases the matter in dispute has already been resolved and hence, one is not entitled to
judicial intervention unless the issue is likely to be raised again between the
Not only do the 2010 disqualification cases involve different issues, causes of parties. There is nothing for the court to resolve as the determination thereof has
action, and subject matters, but these disqualification cases do not even have a been overtaken by subsequent events.
final judgment on the merits to speak of. Assuming an actual case or controversy existed prior to the proclamation of a
Cabreza, Jr. v. Cabreza108 explains the concept of a “judgment on the merits” President who has been duly elected in the May 10, 2010 elections, the same is no
as follows: longer true today. Following the results of that elections, private respondent was
A judgment may be considered as one rendered on the merits “when it not elected President for the second time. Thus, any discussion of his “reelection”
determines the rights and liabilities of the parties based on the disclosed facts, will simply be hypothetical and speculative. It will serve no useful or practical
irrespective of formal, technical or dilatory objections”; or when the judgment is purpose.
rendered “after a determination of which party is right, as distinguished from a
judgment rendered upon some preliminary or formal or merely technical point.”109 Accordingly, the petition is denied due course and is hereby DISMISSED.
SO ORDERED.112 (Citations omitted)
Following the denial of his motion for reconsideration by the COMELEC En
Banc, Atty. Evilio C. Pormento sought relief from this court via a petition From these, it is plain to see that the substance of Estrada’s qualification (vis-
for certiorari, insisting that Estrada was barred by Article VII, Section 4 of the à-vis Article VII, Section 4 of the 1987 Constitution) was not at all discussed. This
Constitution from making a second bid for the presidency. This petition was court even explicitly stated that were it to make a pronouncement on that matter,
docketed as G.R. No. 191988 and entitled Atty. Evilio C. Pormento, petitioner, v. this pronouncement would amount to nothing more than a nonbinding opinion:
Joseph “Erap” Ejercito Estrada and Commission on Elections, respondents. What is the proper interpretation of the following provision of Section 4, Article
As noted by this court in its August 31, 2010 resolution in Pormento v. VII of the Constitution: “[t]he President shall not be eligible for any reelection?”
Estrada,110 the May 10, 2010 elections proceeded without Estrada having been The novelty and complexity of the constitutional issue involved in this case
removed from the list of candidates or otherwise being restricted in his candidacy present a temptation that magistrates, lawyers, legal scholars and law students
as “under the Rules of Court, the filing of such petition would not stay the alike would find hard to resist. However, prudence dictates that this Court exercise
execution of the judgment, final order or resolution of the COMELEC that is judicial restraint where the issue before it has already been mooted by subsequent
events. More importantly, the constitutional requirement of the existence of a and to hold any office, unless he has been given plenary pardon or granted
“case” or an “actual controversy” for the proper exercise of the power of judicial amnesty.
review constrains us to refuse the allure of making a grand pronouncement that, This [sic] disqualifications to be a candidate herein provided shall be deemed
in the end, will amount to nothing but a nonbinding opinion.113 removed upon the declaration by competent authority that said insanity or
incompetence had been removed or after the expiration of a period of five years
Estrada, though adjudged by the COMELEC Second Division and from his service of sentence, unless within the same period he again becomes
COMELEC En Banc to be qualified for a second bid at the presidency, was never disqualified. (Emphasis supplied)
conclusively adjudged by this court to be so qualified. The 2010 disqualification _______________
cases reached their conclusion not because it was determined, once and for all, that
Estrada was not disqualified, but because — with Estrada’s loss in the elections — 117 See Teves v. Commission on Elections, 604 Phil. 717, 728-729; 587 SCRA
there was no longer a controversy to resolve. There was no “determin[ation of] the 1, 12-13 (2009) [Per J. Ynares-Santiago, En Banc], citing Dela Torre v.
rights and liabilities of the parties based on the disclosed facts, irrespective of Commission on Elections, 327 Phil. 1144, 1150-1151; 258 SCRA 483, 487-488
formal, technical or dilatory objections”;114 neither was there “a determination of (1996) [Per J.Francisco, En Banc].
which party is right.” 115 While the 2010 disqualification cases may have reached “It (moral turpitude) implies something immoral in itself, regardless of the fact
their literal end or terminal point, there was no final judgment on the merits. that it is punishable by law or not. It must not be merely mala prohibita, but the
act itself must be inherently immoral. The doing of the act itself, and not its
VII prohibition by statute fixes the moral turpitude. Moral turpitude does not,
however, include such acts as are not of themselves immoral but whose illegality
Estrada was disqualified from lies in their being positively prohibited.”
running for Mayor of the City of 423
Manila in the May 13, 2013
VOL. 747, JANUARY 21, 2015 423
elections and remains disqualified
from running for any elective post Risos-Vidal vs. Commission on Elections
Section 40 of the Local Government Code provides for disqualifications for local
a. Joseph Ejercito Estrada: convicted, disqualified, and pardoned elective offices in particular:
SECTION 40. Disqualifications.—The following persons are disqualified
We now come to the core of this case, that is, whether Estrada was qualified to from running for any elective local position:
run for Mayor of the City of Manila. (a) Those sentenced by final judgment for an offense involving moral
It is not disputed that Estrada was found guilty beyond reasonable doubt and turpitude or for an offense punishable by one (1) year or more of
convicted for plunder by the Sandiganbayan. This conviction stands unreversed imprisonment, within two (2) years after serving sentence;
and unmodified, whether by the Sandiganbayan, on reconsideration, or by this (b) Those removed from office as a result of an administrative case;
court, on appeal. By this conviction, Estrada was sentenced to suffer the accessory (c) Those convicted by final judgment for violating the oath of allegiance to the
penalty of perpetual absolute disqualification. Per Article 30 of the Revised Penal Republic;
Code, this accessory penalty produces the effect of, among others, “[t]he (d) Those with dual citizenship;
deprivation of the right to vote in any election for any popular elective office or to (e) Fugitives from justice in criminal or nonpolitical cases here or abroad;
be elected to such office.”116 (f) Permanent residents in a foreign country or those who have acquired the
Apart from the specific penalty of perpetual absolute disqualification meted on right to reside abroad and continue to avail of the same right after the effectivity
Estrada on account of his conviction, statutory provisions provide for the of this Code; and
disqualification from elective public office of individuals who have been convicted (g) The insane or feeble-minded. (Emphasis supplied)
for criminal offenses involving moral turpitude 117 and/or entailing a sentence of a
defined duration of imprisonment. It is with this backdrop of, on the one hand, Estrada’s conviction for plunder
Section 12 of the Omnibus Election Code provides for disqualifications for (with its concomitant penalty of absolute perpetual disqualification), as well as the
elective offices in general: cited statutory disqualifications, and, on the other, the pardon granted to Estrada,
Section 12. Disqualifications.—Any person who has been declared by that this court must rule on whether Estrada was qualified to run for Mayor of
competent authority insane or incompetent, or has been sentenced by final Manila in the May 13, 2013 elections.
judgment for subversion, insurrection, rebellion or for any offense for which
he has been sentenced to a penalty of more than eighteen months or for b. The power to grant clemency: an executive function
a crime involving moral turpitude, shall be disqualified to be a candidate
The power to grant pardons, along with other acts of executive clemency, is
vested in the President of the Philippines by Article VII, Section 19 of the 1987 Contrasting the provisions of the Malolos Constitution with the present
Constitution: iteration of the pardoning power, it is particularly notable that the power, as
Section 19. Except in cases of impeachment, or as otherwise provided in this provided for in 1899, is deferential to the legislative branch of government. While
Constitution, the President may grant reprieves, commutations, and pardons, and recognizing the pardoning power as ultimately one for the President to wield, it
remit fines and forfeitures, after conviction by final judgment. remained subject to legislative imprimatur.
He shall also have the power to grant amnesty with the concurrence of a Aided by the lens of history, this is most effectively understood in the context
majority of all the Members of the Congress. of a “conflict between people, on one hand, who were determined to secure the kind
of freedom and economic benefits never enjoyed by them before, and groups, on the
The recognition that the power to grant clemency is lodged in the executive has other, who wanted to maintain a social status and economic privilege inherited
been made since the earliest days of the Philippines as a republic. It “is founded from way back or recently acquired by the displacement of elements formerly
on the recognition that human institutions are imperfect and that there are controlling the destiny of the colony.”119 The latter — ilustrados — were the
infirmities, deficiencies or flaws in the administration of justice. The power exists driving force behind the adoption of a constitution, and they endeavored “to make
as an instrument or means for correcting these infirmities and also for mitigating the legislature the most powerful unit in the government.”120
whatever harshness might be generated by a too strict an application of the law.”118 The adoption of organic acts under the auspices of American rule enabled the
Our constitutional history is a cumulative affirmation of the fundamental assimilation of some American constitutional principles. Not least of these is the
conception of the power to pardon as an executive power. grant to the executive of the power to pardon. The Constitution of the United
Provisions from Title VIII of the Malolos Constitution of 1899 read: States of America includes the grant of the pardoning power in the recital of the
Article 67. Apart from the powers necessary to execute laws, it is the duty of President’s powers:
the President of the Republic to: Article II, Section 2.
_______________ The President shall be Commander-in-Chief of the Army and Navy of the
United States, and of the Militia of the several States, when called into the actual
118 J. Padilla, Dissenting Opinion in Llamas v. Orbos, 279 Phil. 920, 946; 202 Service of the United States; he may require the Opinion, in writing, of the
SCRA 844, 866 (1991) [Per J. Paras, En Banc], citing the comment by Joaquin G. principal Officer in each of the executive Departments, upon any Subject relating
Bernas, S.J., Revised 1973 Philippine Constitution, part 1, p. 228 (1983). to the Duties of their respective Offices, and he shall have Power to grant Reprieves
425 and Pardons for Offences against the United States, except in Cases of
Impeachment.
VOL. 747, JANUARY 21, 2015 425
....
Risos-Vidal vs. Commission on Elections
1. Confer civil and military employment in accordance to the law; Thus, the Jones Law of 1916 provides:
2. Appoint Secretaries of Government; _______________
3. Direct diplomatic and commercial relations with other powers;
4. Ensure the swift and complete administration of justice in the entire 119 Cesar Adib Majul, Mabini and the Philippine Revolution,
territory; p. 165 (1960).
5. Pardon lawbreakers in accordance to the law, subject to the provisions 120 Id., at p. 171.
relating to the Secretaries of Government; 427
6. Preside over national solemnities, and welcome accredited envoys and
VOL. 747, JANUARY 21, 2015 427
representatives of foreign powers.
Article 68. The President of the Republic needs to be authorized by a Risos-Vidal vs. Commission on Elections
special law: Section 21. The Governor-General
1. To transfer, cede or exchange any part of Philippine territory; (b) Powers and duties.—. . . . He is hereby vested with the exclusive power
2. To incorporate any other territory into the Philippines; to grant pardons and reprieves and remit fines and forfeitures, and may
3. To allow foreign troops in Philippine territory; veto any legislation enacted as herein provided. . . .
4. To ratify treaties of offensive and defensive alliance, special commercial
treaties, treaties that stipulate subsidies to a foreign power, and any other treaty As against the Malolos Constitution, the Jones Law makes no reference to the
that compels Filipinos to perform any individual obligation; need for legislative consent, whether a priori or a posteriori, for the exercise of the
In no case can the confidential articles of a treaty nullify those that are public. pardoning power. Equally notable, the pardoning power is mentioned in the same
5. To grant general amnesties and pardons; breath (i.e., the same sentence) as the veto power — a power that delineates the
6. To mint money. (Emphasis supplied) relation of the executive branch with the legislative branch.
With the onset of the Commonwealth and en route to independence, the 1935 _______________
Constitution affirmed that the power to pardon is executive in nature. Article VII,
Section 11(6) of the 1935 Constitution reads: 122 Id., at pp. 436-437.
Section 11. . . . 429
(6) The President shall have the power to grant reprieves, commutations, and
VOL. 747, JANUARY 21, 2015 429
pardons, and remit fines and forfeitures, after conviction, for all offenses, except in
cases of impeachment, upon such conditions and with such restrictions and Risos-Vidal vs. Commission on Elections
limitations as he may deem proper to impose. He shall have the power to grant guage of the 1935 Constitution on the executive nature of the pardoning power.
amnesty with the concurrence of the National Assembly. The text of Article II, Section 13 of the 1943 Constitution is substantially similar
with its counterpart in the 1935 Constitution except for the non-mention of
A recollection of the proceedings of the Constitutional Convention reveals impeachment as beyond the coverage of pardoning power:
attempts “to limit the absolute character of the pardoning power of the Section 13. The President shall have the power to grant reprieves,
Executive”:121 commutations and pardons, and remit fines and forfeitures, after conviction, for
It was also generally held that, as it was under the Jones Law and in other all offenses, upon such conditions and with such restrictions and limitations as he
countries, the pardoning power should be vested in the Executive, although there may deem proper to impose. He shall have the power to grant amnesty with the
was a concurrence of the National Assembly.
_______________
Like the Jones Law, but unlike the 1935 and 1943 Constitutions, the 1973
121 Jose M. Aruego, The Framing of the Philippine Constitution (1949). Constitution (as amended) dispensed with the requirement of prior conviction. The
428 1973 Constitution, adopted during the rule of President Ferdinand E. Marcos, is
characteristic of a strong executive. Article VII, Section 11 of the 1973 Constitution
428 SUPREME COURT REPORTS ANNOTATED
provides:
Risos-Vidal vs. Commission on Elections Section 11. The President may, except in cases of impeachment, grant
difference of opinion with respect to the authority to exercise the power to grant reprieves, commutations and pardons, remit fines and forfeitures and, with the
amnesty. There were many proposals, however, intended to limit the absolute concurrence of the Batasang Pambansa, grant amnesty.
character of the pardoning power of the Executive. Of them were the proposal in
the report of the committee on executive power and in the first draft of the From the grant of the power made by Section 21(b) of the Jones Law of 1916 to
Constitution to the effect that pardon should be granted to a person only after his the present, the 1987 Constitution, the shifts in the grant to the executive of the
conviction; the Galang amendment embodying a proposal in the report of the power to extend clemency has mainly been in the matter of requiring or dispensing
committee on executive power to the effect that the Chief Executive could grant with conviction as a condition precedent for the exercise of executive clemency.
pardon to a person only after the latter had served part of the sentence imposed The present, the 1987 Constitution, requires prior conviction. Nevertheless, it
upon him, except in cases where the convicting court should recommend executive retains the fundamental regard for the pardoning power as executive in nature.
clemency, when the same could be exercised even prior to the service of the Jurisprudence dating to 1991123 noted how the 1986 Constitutional Commission
sentence; and the Sanvictores amendment providing that no pardon should, _______________
without the recommendation of the Supreme Court, be granted until the prisoner
should have served at least one-half of the minimum sentence imposed. 123 Llamas v. Orbos, supra note 118.
.... 430
The Galang amendment and the Sanvictores amendment would go further by
430 SUPREME COURT REPORTS ANNOTATED
requiring that no person, even if already convicted, should be pardoned unless he
had served partially his sentence. The Galang amendment would permit executive Risos-Vidal vs. Commission on Elections
clemency even before the commencement of the service of the sentence, upon the rejected a proposal to render the coverage of the pardoning power susceptible
recommendation of the convicting court; and the Sanvictores amendment, upon to legislative interference, particularly in matters relating to graft and corruption.
the recommendation of the Supreme Court. . . .122 Likewise, jurisprudence as recent as 2007124 clarified that a court cannot preempt
the grant of executive clemency.
As will be gleaned from the final text of the 1935 Constitution, the Galang and In addition to restoring the requirement of prior conviction, the 1987
Sanvictores amendments were both defeated. Thus was affirmed the executive Constitution now includes the phrase “as otherwise provided in this Constitution.”
nature of the power to pardon. The 1987 Constitution, in Article VII, Section 19, enumerates the acts or
The 1943 Constitution, adopted in the interlude of the Second World War and means through which the President may extend clemency: (1) reprieve, or “the
the Japanese occupation, echoed the lan- deferment of the implementation of the sentence for an interval of time”;125 (2)
commutation, which “refers to the reduction of the duration of a prison sentence of 128 Id., at pp. 198-199; p. 196, citing United States v. Wilson, 7 Pet. 160, 160-
a prisoner”;126 (3) remission of fines and forfeitures; (4) pardon; and (5) amnesty. 1, cited in Joaquin G. Bernas, The 1973 Philippine Constitution, Notes and Cases,
“[P]ardon is of British origin, conceived to temper the gravity of the King’s part I, p. 355 (1974).
wrath.”127 It is “an act of grace, proceeding from the power entrusted with the 129 Barrioquinto v. Fernandez, 82 Phil. 642, 646-647 (1949) [Per J. Feria, En
execution of the laws, which exempts the individual, on whom it is bestowed, from Banc].
the punishment the law inflicts for a crime he has committed. It is the private, 432
though official act of the executive magistrate, delivered to the individual for whose 432 SUPREME COURT REPORTS ANNOTATED
benefit it is intended, and not communicated officially to the Court. . . . A pardon
is a deed, to the validity of which delivery is essential, Risos-Vidal vs. Commission on Elections
_______________ Estrada argues that pardon is characterized by what he refers to as the
“forgive-and-forget rule.”130 He cites several decisions rendered in the United
124 People v. Rocha, 558 Phil. 521, 538-539; 531 SCRA 761, 778 (2007) States131 (chiefly, the 1866, post-Civil War decision in Ex parte Garland) and
[Per J.Chico-Nazario, Third Division], citing Joaquin G. Bernas, The 1987 insists that “pardon not merely releases the offender from the punishment . . . but
Constitution of the Republic of the Philippines, A Commentary, p. 935 (2003). that it obliterates in legal contemplation the offense itself”132 and that it “forever
125 Implementing Rules and Regulations of Act No. 4103, the Indeterminate closes the eyes of the court.”133 Citing this court’s decisions in Cristobal v.
Sentence Law, Sec. 2(n). Labrador134 and in Pelobello v. Palatino,135 Estrada asserts that pardon “blots out
126 Implementing Rules and Regulations of Act No. 4103, the Indeterminate of existence the guilt, so that in the eye of the law the offender is as innocent as if
Sentence Law, Sec. 2(o). he had never committed the offence . . . it makes him, as it were, a new man, and
127 Monsanto v. Factoran, Jr., 252 Phil. 192, 198; 170 SCRA 190, 196 (1989) gives him new credit and capacity.”136
[Per CJ. Fernan, En Banc]. Estrada is in grave error for insisting on what he has dubbed as the “forgive-
431 and-forget rule.”
In Monsanto v. Factoran,137 this court repudiated the pronouncements made
VOL. 747, JANUARY 21, 2015 431 by Cristobal and Pelobello, as well as reliance on Garland, on the nature and
Risos-Vidal vs. Commission on Elections effects of pardon:
and delivery is not complete without acceptance.”128 (Emphasis supplied) In Pelobello v. Palatino, we find a reiteration of the stand consistently adopted
Pardon and amnesty have been distinguished as follows: by the courts on the various consequences of pardon: “x x x we adopt the broad
Pardon is granted by the Chief Executive and as such it is a private actwhich view expressed in Cristobal v. Labrador, G.R. No. 47941, December 7, 1940, that
must be pleaded and proved by the person pardoned, because the courts take no subject to the limitations imposed by the Constitution, the pardoning power cannot
notice thereof; while amnesty by Proclamation of the Chief Executive with the be restricted or controlled by legislative action; that an absolute pardon not only
concurrence of Congress, and it is a public act of which the courts should take blots out the crime committed but
judicial notice. Pardon is granted to one after conviction; while amnesty is granted _______________
to classes of persons or communities who may be guilty of political offenses,
generally before or after the institution of the criminal prosecution and sometimes 130 Rollo, p. 1793.
after conviction. Pardon looks forward and relieves the offender from the 131 Ex parte Garland, 71 U.S. 833 (1866); Biddle v. Perovich, 274 U.S. 480
consequences of an offense of which he has been convicted, that is, it abolishes or (1927); Ex parte Grossman, 267 U.S. 87 (1925); Carlisle v. U.S., 83 U.S. 147 (1872).
forgives the punishment, and for that reason it does “not work the restoration 132 Rollo, p. 1794, citing Carlisle v. United States, id., at p. 151.
of the rights to hold public office, or the right of suffrage, unless such 133 Id.
rights be expressly restored by the terms of the pardon,” and it “in no case 134 71 Phil. 34 (1940) [Per J. Laurel, En Banc].
exempts the culprit from the payment of the civil indemnity imposed upon him by 135 72 Phil. 441 (1940) [Per J. Laurel, En Banc].
the sentence.” While amnesty looks backward and abolishes and puts into oblivion 136 Rollo, pp. 1738-1739.
the offense with which he is charged that the person released by amnesty stands 137 Supra note 127.
before the law precisely as though he had committed no offense.129 (Emphasis 433
supplied, citations omitted) VOL. 747, JANUARY 21, 2015 433

c. Pardon and its effects: forgiveness but not forgetfulness Risos-Vidal vs. Commission on Elections
_______________ removes all disabilities resulting from the conviction. x x x (W)e are of the
opinion that the better view in the light of the constitutional grant in this
jurisdiction is not to unnecessarily restrict or impair the power of the Chief
Executive who, after an inquiry into the environmental facts, should be at liberty
to atone the rigidity of the law to the extent of relieving completely the party x x x ultimate resolution of the dispute and was indispensable to the conclusions this
concerned from the accessory and resultant disabilities of criminal conviction.” court reached. As against Monsanto, Estrada would have this court rely on a
The Pelobello v. Palatino and Cristobal v. Labrador cases, and several others decision, which was rendered nearly a century and a half ago by a court outside of
show the unmistakable application of the doctrinal case of Ex Parte Garland, this jurisdiction (i.e., Ex parte Garland), and which, this court
whose sweeping generalizations to this day continue to hold sway in our _______________
jurisprudence despite the fact that much of its relevance has been downplayed by
later American decisions. 138 Id., at pp. 199-201.
Consider the following broad statements: 139 Rollo, p. 1771.
“A pardon reaches both the punishment prescribed for the offense and the guilt 435
of the offendor; and when the pardon is full, it releases the punishment and blots VOL. 747, JANUARY 21, 2015 435
out of existence the guilt, so that in the eye of the law the offender is as innocent
as if he had never committed the offense. If granted before conviction, it prevents Risos-Vidal vs. Commission on Elections
any of the penalties and disabilities, consequent upon conviction, from attaching; has observed to be against the grain of contemporary authorities. In addition,
if granted after conviction, it removes the penalties and disabilities and restores Estrada would have us rely on jurisprudence which themselves depend on the
him to all his civil rights; it makes him, as it were, a new man, and gives him a same archaic and foreign decision. To do, as Estrada suggests, would be to indulge
new credit and capacity.” an absurdity. Estrada effectively invites this court to irrationality and to arrive at
Such generalities have not been universally accepted, recognized or a conclusion resting on premises that have been roundly renounced.
approved. The modern trend of authorities now rejects the unduly broad language In any case, from the preceding discussions, two points are worthy of particular
of the Garland case (reputed to be perhaps the most extreme statement which has emphasis:
been made on the effects of a pardon). To our mind, this [i.e., the rejection of I. Pardon is a private, though official, act of the executive. Proceeding from the
Garland] is the more realistic approach. While a pardon has generally been power to execute laws, it merely evinces the executive’s choice to decline from
regarded as blotting out the existence of guilt so that in the eye of the law the enforcing punishment so as to mollify penal misery.
offender is as innocent as though he never committed the offense, it does not oper- II. Pardon does not erase the moral stain and the fact of conviction. It retains
434 the law’s regard for a convict “as more dangerous to society than one never found
434 SUPREME COURT REPORTS ANNOTATED guilty of a crime”;140 the convict remains “deserving of punishment” though left
unpunished.141
Risos-Vidal vs. Commission on Elections It is with the illumination of this fundamental notion of pardon as a ‘private
ate for all purposes. The very essence of a pardon is forgiveness or remission of act that does not erase the moral stain and the fact of conviction’ that this court
guilt. Pardon implies guilt. It does not erase the fact of the commission of the crime must proceed to make a determination of Estrada’s qualification.
and the conviction thereof. It does not wash out the moral stain. It involves
forgiveness and not forgetfulness. VIII
The better considered cases regard full pardon (at least one not based on the
offender’s innocence) as relieving the party from all the punitive consequences of The exercise of pardon:
his criminal act, including the disqualifications or disabilities based on the finding limitations and prescriptions
of guilt. But it relieves him from nothing more. “To say, however, that the offender
is a ‘new man,’ and ‘as innocent as if he had never committed the offense’; is to a. Articles 36 and 41 of the Revised Penal
ignore the difference between the crime and the criminal. A person adjudged guilty
of an offense is a convicted criminal, though pardoned; he may be deserving of _______________
punishment, though left unpunished; and the law may regard him as more
dangerous to society than one never found guilty of crime, though it places no 140 Monsanto v. Factoran, Jr., supra note 127 at p. 201; p. 198, citing State v.
restraints upon him following his conviction.”138 (Emphasis and underscoring Cullen, 127 P. 2d 257, cited in 67 C.J.S. 577, note 18.
supplied, citations omitted) 141 Id., citing State v. Cullen, id.
436
Estrada has made much of how Monsanto centered on the issue of the need for
436 SUPREME COURT REPORTS ANNOTATED
a new appointment of a pardoned officer seeking to be reinstated to her former
position. He posits that Monsanto could not be controlling in this case, as what is Risos-Vidal vs. Commission on Elections
at issue here is qualification for elective public office.139 Code do not abridge or diminish the pardoning power of the President
This is but a vain attempt to split hairs. It is clear from the previously quoted
discussion in Monsanto that there was an unequivocal consideration by this court Article VII, Section 19 of the 1987 Constitution provides two (2) limitations on
of the nature and effects of pardon. This discussion laid the premises for the the President’s exercise of the power to pardon: first, it can only be given after final
conviction; and second, it cannot be exercised “in cases of impeachment, or as Citing the same cases of Cristobal, Pelobello, and Garland, Estrada argues
otherwise provided in this Constitution.” Elsewhere in the Constitution, Article that Articles 36 and 41 of the Revised Penal Code violate the Constitution in
IX, C, Section 5 provides that: “No pardon, amnesty, parole, or suspension of requiring that the restoration of the rights of suffrage or to otherwise vote for and
sentence for violation of election laws, rules, and regulations shall be granted by be elected to public office must be made expressly. Specifically, he
the President without the favorable recommendation of the Commission [on 438
Elections].” 438 SUPREME COURT REPORTS ANNOTATED
Outside of the Constitution, the Revised Penal Code contains provisions
relating to pardon. Risos-Vidal vs. Commission on Elections
Article 36 of the Revised Penal Code provides that: “A pardon shall in no case claims that these provisions “abridge or diminish the pardoning power of the
exempt the culprit from the payment of the civil indemnity imposed upon him.” President.”142
The same Article 36 prescribes that for pardon to effect the restoration of This court has previously acknowledged, in Llamas v. Orbos,143that the 1986
the rights of suffrage and to hold public office, “such rights [must] be expressly Constitutional Commission rejected a proposal to include in Article VII, Section
restored by the terms of the pardon.” 19, a statement to the effect that “the power to grant executive clemency for
Also on suffrage and/or the rights to vote for and be elected to public office, violation of corrupt practices laws may be limited by legislation.” Thus, this court
Articles 40 to 43 of the Revised Penal Code provide that the penalties of perpetual concluded that “the President’s executive clemency powers may not be limited in
absolute disqualification, temporary absolute disqualification, perpetual special terms of coverage, except as already provided in the Constitution”:
disqualification, and perpetual special disqualification on suffrage, which attach During the deliberations of the Constitutional Commission, a subject of
as accessory penalties to death, reclusion perpetua, reclusion temporal, prisión deliberations was the proposed amendment to Art. VII, Sec. 19 which reads as
mayor and prisión correccional, as the case may be, shall still be suffered by the follows: “However, the power to grant executive clemency for violation of corrupt
offender even though pardoned as to the principal penalty, “unless . . . expressly practices laws may be limited by legislation.” The Constitutional Commission,
remitted in the pardon”: however, voted to remove the amendment, since it was in derogation of the powers
437 of the President. As Mr. Natividad stated:
“I am also against this provision which will again chip more powers from the
VOL. 747, JANUARY 21, 2015 437
President. In case of other criminals convicted in our society we extend probation
Risos-Vidal vs. Commission on Elections to them while in this case, they have already been convicted and we offer mercy.
ARTICLE 40. Death — Its Accessory Penalties.—The death penalty, when it The only way we can offer mercy to them is through this executive clemency
is not executed by reason of commutation or pardon shall carry with it that of extended to them by the President. If we still close this avenue to them, they would
perpetual absolute disqualification and that of civil interdiction during thirty years be prejudiced even worse than the murderers and the more vicious killers in our
following the date of sentence, unless such accessory penalties have been expressly society. . . .”
remitted in the pardon. The proposal was primarily intended to prevent the President from protecting
ARTICLE 41. Reclusion Perpetua and Reclusion Temporal — Their his cronies. Manifestly, however, the Commission preferred to trust in the
accessory penalties.—The penalties of reclusion perpetua and reclusion discretion of Presidents and refrained from putting additional limi-
temporal shall carry with them that of civil interdiction for life or during the period 439
of the sentence as the case may be, and that of perpetual absolute disqualification VOL. 747, JANUARY 21, 2015 439
which the offender shall suffer even though pardoned as to the principal
penalty, unless the same shall have been expressly remitted in the pardon. Risos-Vidal vs. Commission on Elections
ARTICLE 42. Prisión Mayor — Its Accessory Penalties.—The penalty tations on his clemency powers. (II RECORD of the Constitutional
of prisión mayor shall carry with it that of temporary absolute disqualification and Commission, pp. 392, 418-419, 524-525)
that of perpetual special disqualification from the right of suffrage which the It is evident from the intent of the Constitutional Commission, therefore, that
offender shall suffer although pardoned as to the principal penalty, unless the same the President’s executive clemency powers may not be limited in terms of coverage,
shall have been expressly remitted in the pardon. except as already provided in the Constitution, that is, “no pardon, amnesty, parole,
ARTICLE 43. Prisión Correccional — Its Accessory Penalties.—The penalty or suspension of sentence for violation of election laws, rules and regulations shall
of prisión correccional shall carry with it that of suspension from public office, from be granted by the President without the favorable recommendation of the
the right to follow a profession or calling, and that of perpetual special COMELEC.” (Article IX, C, Section 5, Constitution) If those already adjudged
disqualification from the right of suffrage, if the duration of said imprisonment guilty criminally in court may be pardoned, those adjudged guilty administratively
shall exceed eighteen months. The offender shall suffer the disqualification should likewise be extended the same benefit.144
provided in this article although pardoned as to the principal penalty, unless the
same shall have been expressly remitted in the pardon. (Emphasis supplied) Not only has the coverage of executive clemency been recognized to be beyond
the reach of legislative action, this court has also noted that the matter of whether
the President should actually choose to extend executive clemency to a convict
cannot be preempted by judicial action. Thus, the determination of whether a 441
convict shall be extended clemency is a decision that is solely for the President to VOL. 747, JANUARY 21, 2015 441
make:
This Court cannot review, much less preempt, the exercise of executive Risos-Vidal vs. Commission on Elections
clemency under the pretext of preventing the accused from evading the penalty All that Articles 36 and 41 do is prescribe that, if the President wishes to
of reclusion perpetua or from trifling with our judicial system. Clemency is not a include in the pardon the restoration of the rights of suffrage and to hold public
function of the judiciary; it is an executive function. . . .145 office, or the remission of the accessory penalty of perpetual absolute
disqualification, he or she should do so expressly. Articles 36 and 41 only ask that
The 1987 Constitution’s recital of the instances when pardon may or may not the President state his or her intentions clearly, directly, firmly, precisely, and
be exercised and this court’s prior recognition of clemency as an executive function unmistakably. To belabor the point, the President retains the power to make such
notwithstanding, Articles 36 and 41 of the Revised Penal Code could not be restoration or remission, subject to a prescription on the manner by which he or
144 Id., at pp. 937-938; pp. 858-859. she is to state it.
145 People of the Philippines v. Rocha, supra note 124, citing Joaquin G. This interpretation is consistent with the clear constitutional intention to
Bernas, The 1987 Constitution of the Republic of the Philippines, A Commentary, grant exclusive prerogative to the President to decide when to exercise such power.
p. 935 (2003). As in this case, any ambiguity invites judicial intervention.
440 Also, it is a basic precept that “public office is a public trust.”149In contrast,
pardon is a “private, though official act of the executive magistrate, delivered to
440 SUPREME COURT REPORTS ANNOTATED
the individual for whose benefit it is intended.”150 Given the contrasting natures
Risos-Vidal vs. Commission on Elections of, on the one hand, elective office as a public trust, and, on the other, pardon as a
considered as abridging or diminishing the President’s right to extend private act, it “would not be asking too much”151 of the President to be unequivocal
clemency. with his or her intentions on restoring a convict’s right not just to vote, but more
To “abridge” or to “diminish” is to shorten, reduce, or lessen.146Further, so, to be voted for elective public office.
“coverage” pertains to scope,147 it refers to “[t]he extent to which something deals Doing so serves not only a practical purpose but, more importantly, the greater
with or applies to something else.”148 public interest in not leaving to inference the qualification of a person who is
Articles 36 and 41 do not reduce the coverage of the President’s pardoning regarded “as more
power. At no point do they say that the President may not grant pardon. They do _______________
not recite instances or areas in which the President’s power to pardon is rendered
nonexistent, or in which the President is otherwise incapable of granting pardon. 149 Const. (1987), Art. XI, Sec. 1.
Articles 36 and 41 notwithstanding, the only instances in which the President may 150 Monsanto v. Factoran, Jr., supra note 127 at pp. 198-199; p. 196,
not extend pardon remain to be: (1) impeachment cases; (2) cases that have not yet citing United States v. Wilson, supra note 128, cited in Joaquin G. Bernas, The
resulted in a final conviction; and (3) cases involving violations of election laws, 1973 Philippine Constitution, Notes and Cases, part I, p. 355 (1974). See
rules, and regulations in which there was no favorable recommendation coming also Barrioquinto v. Fernandez, supra note 129 at pp. 646-647.
from the COMELEC. Stated otherwise, the President remains capacitated to grant 151 J. Padilla, Dissenting Opinion in Monsanto v. Factoran, Jr., id., at p. 206;
a pardon that works to restore the rights of suffrage and/or to hold public office, or p. 204 [Per CJ. Fernan, En Banc].
to otherwise remit the penalty of perpetual absolute disqualification. 442
Articles 36 and 41 refer only to requirements of convention or form. They only
442 SUPREME COURT REPORTS ANNOTATED
provide a procedural prescription. They are not concerned with areas where or the
instances when the President may grant pardon; they are only concerned Risos-Vidal vs. Commission on Elections
with how he or she is to exercise such power so that no other governmental dangerous to society”152 but stands to gain from the reposition of public
instrumentality needs to intervene to give it full effect. trust.153 It addresses the “presumptive rule that one who is rendered infamous by
_______________ conviction of a felony, or other base offense indicative of moral turpitude, is unfit
to hold public office, as the same partakes of a privilege which the State grants only
146 Definition available at <http://www.merriam-webster.com/ to such classes of persons which are most likely to exercise it for the common
dictionary/abridge> and <http://www.merriam-webster.com/dictionary/ good.”154
diminish>. Pronouncing in express and unmistakable language the restoration of the right
147 Definition available at <http://www.merriam-webster.com/ to vote and be voted, therefore, complementsthe private act of pardoning such that
dictionary/coverage>. it enables the inclusion of public effects in the private act. It desegregates the
148 Definition available at <http://www.oxforddictionaries.com/us/ public consequence of enabling the convict with the opportunity to lead the
definition/american_english/coverage>. community by being the occupant of a public office.
Recall that the manner by which the 1987 Constitution phrases its investiture _______________
on the President of the pardoning power now includes the phrase “as otherwise
provided in this Constitution.” This phrase affirms the imperative of reading and 156 Const. (1987), Art. VIII, Sec. 1(2).
interpreting the Constitution in its entirety, not taking a provision in isolation. 157 Monsanto v. Factoran, Jr., supra note 127 at p. 201; p. 198.
The pardoning power of the President must, thus, not be divorced from the 158 Id.
Constitution’s injunction that “[p]ublic office is a public trust.”155 Read in harmony 159 203 N.E. 2d 95.
with this injunction, Articles 36 and 41 of the Revised Penal Code impress upon 444
the President the significance of departing from the purely private consequences
444 SUPREME COURT REPORTS ANNOTATED
of pardon should he or she stray into the public affair of restoring a convict’s rights
of suffrage and/or to hold public office. Risos-Vidal vs. Commission on Elections
Parenthetically, the Constitution also grants this court jurisdiction to Lastly, even if it were to be granted that this statement articulated a rule, this
determine “whether or not there has been a grave statement, made in 1989, must be deemed to have been abandoned, in light of this
_______________ court’s more recent pronouncements — in 1997, in People v. Casido,160 and in 2000,
in People v. Patriarca, Jr.161 — which cited with approval this court’s statement
152 Monsanto v. Factoran, Jr., id., at p. 201; p. 198, citing State v. in Barrioquinto v. Fernandez162 that:
Cullen, supranote 140. [p]ardon looks forward and relieves the offender from the consequences of an
153 Id. offense of which he has been convicted, that is, it abolishes or forgives the
154 Romeo Jalosjos v. COMELEC, G.R. No. 205033, June 18, 2013, 698 SCRA punishment, and for that reason it does ‘not work the restoration of the rights to
742 [Per J. Perlas-Bernabe, En Banc]. hold public office, or the right of suffrage, unless such rights be expressly restored
155 Const. (1987), Art. XI, Sec. 1. by the terms of the pardon,’ and it ‘in no case exempts the culprit from the payment
443 of the civil indemnity imposed upon him by the sentence.’163 (Emphasis supplied)
VOL. 747, JANUARY 21, 2015 443
So, too, this statement indicating “inclusion” must be deemed superseded by
Risos-Vidal vs. Commission on Elections this court’s 2013 pronouncement in Romeo Jalosjos v. COMELEC164 which
abuse of discretion amounting to . . . excess of jurisdiction on the part of any recognizes that “one who is previously convicted of a crime punishable by reclusion
branch or instrumentality of the Government.”156 This means that no grant of perpetua or reclusion temporal continues to suffer the accessory penalty of
constitutional power is immune from review if it is done arbitrarily or without perpetual absolute disqualification even though pardoned as to the principal
reason, capriciously, or on the basis of whim. However, this court’s power of review penalty, unless the said accessory penalty shall have been expressly remitted in the
in the present case is not raised by any party and, thus, not an issue that this court pardon.”165
must decide. _______________

(b) Clarifying Monsanto 160 336 Phil. 344; 269 SCRA 360 (1997) [Per J. Davide, Jr., Third Division].
161 395 Phil. 690; 341 SCRA 464 (2000) [Per J. Buena, Second Division].
Monsanto, in the course of repudiating Cristobal, Pelobello, and Garland, 162 Barrioquinto v. Fernandez, supra note 129.
declared that “[t]he better considered cases regard full pardon . . . as relieving the 163 Id., at p. 647, citing Rev. Pen. Code, Art. 36.
party from all the punitive consequences of his criminal act, including the 164 Supra note 154.
disqualifications or disabilities based on the finding of guilt.”157 165 Id., at p. 763.
This “inclusion” should not be taken as authority for concluding that the grant 445
of pardon ipso facto remits the accessory disqualifications or disabilities imposed
VOL. 747, JANUARY 21, 2015 445
on a convict regardless of whether the remission was explicitly stated.
For one, this “inclusion” was not a categorical articulation by this court of a Risos-Vidal vs. Commission on Elections
prevailing rule. It was a statement made only in the course of a comparative survey IX
of cases during which the court manifested a preference for “authorities [that
reject] the unduly broad language of the Garland case.”158 No remission of the penalty of perpetual absolute disqualification
Second, the footnote to this statement indicates that it relied on a case decided and restoration of the rights to vote and be voted for elective public office
by a United States court: Comm. of Met. Dist. Com. v. Director of Civil in Estrada’s pardon
Service.159 Thus, it was never meant as a summation of the controlling principles
in this jurisdiction. It did not account for Articles 36 and 41 of the Revised Penal Having established that the challenge to the validity of Articles 36 and 41 of
Code. the Revised Penal Code must fail, we turn to the pivotal issue of whether, in light
of these statutory provisions, the pardon granted to Estrada effectively restored
his rights to vote and be voted for elective public office, or otherwise remitted his Estrada’s use of tentative and indefinite language — such as “deemed
perpetual absolute disqualification. subsumed” and “substantial compliance” — reveals his own acknowledgement that
It did not. the restoration and/or remission, if any, in the pardon are not as unequivocal or
as absolutely clear as they could otherwise have been had the pardon simply
(a) No express remission and/or restoration; reliance on inference is stated, for instance, that “the penalty of perpetual absolute disqualification is
improper hereby removed.”
Estrada is noticeably compelled to resort to syllogism in order to arrive at the
The dispositive portion of the pardon extended by former President Gloria deductive conclusion that he is qualified to run. He rests his position on an
Macapagal-Arroyo to Estrada reads: inference.
IN VIEW HEREOF and pursuant to the authority conferred upon me by the This reliance on inference is precisely what the requirement
Constitution, I hereby grant executive clemency to JOSEPH EJERCITO of expressly stating the restoration or remission seeks to avoid. To be “express” is
ESTRADA, convicted by the Sandiganbayan of Plunder and imposed a penalty to state “directly, firmly, and explicitly.”169 It is synonymous with being
of Reclusion Perpetua. He is hereby restored to his civil and political rights. precise.170 On the contrary, to “infer” is to rely on what is implied; it is to
The forfeitures imposed by the Sandiganbayan remain in force and in full, “surmise.”171 Inference is exactly what relying on an express pronouncement does
including all writs and processes issued by the Sandiganbayan in pursuance not entail.
hereof, except for the bank account(s) he owned before his tenure as President.
446 (b) Even the inference that Estrada proffers is laden with fallacies
446 SUPREME COURT REPORTS ANNOTATED
In any case, even if Estrada’s inferences and reliance on the characterization
Risos-Vidal vs. Commission on Elections of the rights to vote and be voted for elective public office as political rights is to be
Upon acceptance of this pardon by JOSEPH EJERCITO ESTRADA, this indulged, it does not follow that these specific rights have been restored by the
pardon shall take effect.166 pardon’s genericrestoration of civil and political rights.
The concept of “civil and political rights” both as its own collectivity and in
From the plain text of this disposition, it can be readily seen that there is contrast with other classes of human rights
no categorical statement actually saying that Estrada’s rights to vote and be voted _______________
for elective public office are restored, or that the penalty of perpetual absolute
disqualification is remitted. 169 Definition available at <http://www.merriam-webster.com/
The disposition contains three (3) clauses that delimit the effects of the pardon: dictionary/express>.
1. The general grant of executive clemency to Estrada (i.e., “I hereby grant 170 Id.
executive clemency to JOSEPH EJERCITO ESTRADA”); 171 Definition available at <http://www.merriam-webster.com/
2. The restoration of Estrada’s civil and political rights (i.e., “He is hereby dictionary/infer>.
restored to his civil and political rights”); and 448
3. The continuing validity of the forfeitures imposed by the Sandiganbayan.
As a cure for the lack of a categorical statement restoring his rights to vote and 448 SUPREME COURT REPORTS ANNOTATED
be voted for elective public office, or otherwise remitting the penalty of perpetual Risos-Vidal vs. Commission on Elections
absolute disqualification, Estrada argues that the rights to vote and be voted for emerged in the aftermath of the Second World War. Its conceptual
elective public office are political rights; hence, “the restoration of Estrada’s right development is more effectively understood in the context of the emergence of the
to seek public office is deemed subsumed when the pardon extended by GMA contemporary human rights regime and the efforts at enabling the then nascent
expressly restored the civil and political rights of the Public (sic) United Nations to “assum[e] the role of guarantor of human rights on a universal
Respondent.”167 He asserts that “[s]uch statement is already a substantial if not scale”172 consistent with the perceived need that “the individual human being be
full compliance with the requirements of Article 36 of the Revised Penal Code.”168 placed under the protection of the international community.”173
_______________ As Professor Christian Tomuschat discussed in an introductory note to the
International Convention on Civil and Political Rights (ICCPR), the Second World
166 Rollo, p. 265. War revealed that “national governments could gravely fail in their duty to ensure
167 Id., at p. 1779. the life and the liberty of their citizens.”174 Worse, some of these national
168 Id. governments have themselves “become murderous institutions.”175It was,
447 therefore, evident “that protective mechanisms at the domestic level alone did not
VOL. 747, JANUARY 21, 2015 447 provide sufficiently stable safeguards.”176

Risos-Vidal vs. Commission on Elections


The historical milieu of the efforts taken to enable the United Nations to 1954. Yet it took many years before eventually the political climate was ripe for
assume the previously mentioned “role of guarantor of human rights on a universal the adoption of these two ambitious texts. While both the
scale”177 reveals how “civil and political rights” as a concept of distinct rights — 450
embodied in its own instrument — came to be: 450 SUPREME COURT REPORTS ANNOTATED
At the San Francisco Conference in 1945, some Latin American countries
requested that a full code of human rights be included in the Charter of the United Risos-Vidal vs. Commission on Elections
Nations itself. Since such an initiative required careful preparation, their motions Western and the Socialist States were still not fully convinced of their
could not be successful at that stage. Nonetheless, human rights were embraced usefulness, it was eventually pressure brought to bear upon them from Third
as a matter of principle. The Charter contains references to human World countries which prompted them to approve the outcome of the protracted
_______________ negotiating process. Accordingly, on 16 December 1966, the two Covenants were
adopted by the General Assembly by consensus, without any abstentions
172 Available at <http://legal.un.org/avl/pdf/ha/iccpr/iccpr_e.pdf>. (resolution 2200 [XXI]). Since that time, the two comprehensive human rights
173 Id. instruments of the United Nations have sailed on different courses.178
174 Id.
175 Id. Professor Tomuschat further summarizes the provisions of the ICCPR, its
176 Id. manner of recital of civil and political rights, and the common thread binding the
177 Id. rights recited in it:
449 The ICCPR comprises all of the traditional human rights as they are
known from historic documents such as the First Ten Amendments to the
VOL. 747, JANUARY 21, 2015 449 Constitution of the United States (1789/1791) and the French Déclaration
Risos-Vidal vs. Commission on Elections des droits de l’homme et du citoyen (1789). However, in perfect harmony with
rights in the Preamble, among the purposes of the Organization (Article 1) and its sister instrument, Part I starts out with the right of self-determination which
in several other provisions (Articles 13, 55, 62 and 68). Immediately after the is considered to be the foundational stone of all human rights (article 1). Part II
actual setting up of the institutional machinery provided for by the Charter, the (articles 2 to 5) contains a number of general principles that apply across the board,
new Commission on Human Rights began its work for the creation of an among them in particular the prohibition on discrimination. Part III enunciates
International Bill of Rights. In a first step, the Universal Declaration of Human an extended list of rights, the first of which being the right to life (article 6). Article
Rights was drafted, which the General Assembly adopted on 10 December 1948. 7 establishes a ban on torture or other cruel, inhuman or degrading treatment or
In order to make human rights an instrument effectively shaping the lives of punishment, and article 8 declares slavery and forced or compulsory labour
individuals and nations, more than just a political proclamation was needed. unlawful. Well-balanced guarantees of habeas corpus are set forth in article 9, and
Hence, from the very outset there was general agreement to the effect that the article 10 establishes the complementary proviso that all persons deprived of their
substance of the Universal Declaration should be translated into the hard legal liberty shall be treated with humanity.
form of an international treaty. The General Assembly reaffirmed the necessity of Freedom of movement, including the freedom to leave any country, has found
complementing, as had already been done in the Universal its regulation in article 12. Aliens, who do not enjoy a stable right of sojourn, must
Declaration, traditional civil and political rights with economic, social and _______________
cultural rights, since both classes of rights were “interconnected and
interdependent.” (see section E of resolution 421 (V) of 4 December 1950) The only 178 Id.
question was whether, following the concept of unity of all human rights, the new
conventional rights should be encompassed in one international instrument or 451
whether, on account of their different specificities, they should be arranged VOL. 747, JANUARY 21, 2015 451
according to those specificities. Western nations in particular claimed that
the implementation process could not be identical, economic and social Risos-Vidal vs. Commission on Elections
rights partaking more of the nature of goals to be attained whereas civil as a minimum be granted due process in case their expulsion is envisaged
and political rights had to be respected strictly and without any (article 13). Fair trial, the scope ratione materiae of which is confined to criminal
reservations. It is this latter view that eventually prevailed. By resolution prosecution and to civil suits at law, has its seat in articles 14 and 15. Privacy, the
543 (VI) of 4 February 1952, the General Assembly directed the Commission on family, the home or the correspondence of a person are placed under the protection
Human Rights to prepare, instead of just one Covenant, two draft treaties; a of article 17, and the social activities of human beings enjoy the safeguards of
Covenant setting forth civil and political rights and a parallel Covenant providing article 18 (freedom of thought, conscience and religion), article 19 (freedom of
for economic, social and cultural rights. The Commission completed its work in expression), article 21 (freedom of assembly), and article 22 (freedom of
association). Going beyond the classic dimension of protection against interference
by State authorities, articles 23 and 24 proclaim that the family and the child are 182 Available at <http://www.globalization101.org/three-generations-of-
entitled to protection by society and the State. rights/>.
Article 25 establishes the right for everyone to take part in the running of the 183 G.R. No. 100150, January 5, 1994, 229 SCRA 117 [Per J. Vitug, En Banc].
public affairs of his/her country. With this provision, the ICCPR makes clear that 184 Id., at p. 126.
State authorities require some sort of democratic legitimacy. Finally, article 27 453
recognizes an individual right of members of ethnic, religious or linguistic VOL. 747, JANUARY 21, 2015 453
minorities to engage in the cultural activities characteristic of such minorities. No
political rights are provided for. Minorities as such have not been endowed with Risos-Vidal vs. Commission on Elections
any rights of political autonomy.179 Political rights, on the other hand, are said to refer to the right to participate,
directly or indirectly, in the establishment or administration of government, the
Consistent with this concept of civil and political rights as a collectivity of right of suffrage, the right to hold public office, the right of petition and, in general,
“traditional human rights as they are known from historic documents” 180 is Karal the rights appurtenant to citizenship vis-à-vis the management of
Vasak’s conception181 of civil and political rights as “first-generation human government.185 (Citations omitted)
rights.” This is in contrast with economic, social and cultural rights as “second-
generation human rights” and collective-developmen- The recurring refrain of these discussions — historical, academic and
_______________ jurisprudential — is the understanding that “civil and political rights” is a
collectivity. It is a figurative basket of “rights directly possessed by individuals
179 Id. [that are correlatively] positive duties upon the government to respect and fulfil
180 Id. them.”186 Understood in this context, it is clear that the rights of suffrage and to
181 See Karel Vasak, “Human Rights: A Thirty-Year Struggle: The Sustained hold public (elective) office, are but two of a manifold category of rights “deal[ing]
Efforts to give Force of law to the Universal Declaration of Human Rights,” with liberty and participation in political life”187 and encompassing the entire
UNESCO Courier 30:11, Paris: United Nations Educational, Scientific, and spectrum of all such “rights appurtenant to citizenship vis-à-vis the management
Cultural Organization, November 1977. of government.”188
452 In light of the circumstances of this case, to speak of “restor[ing] civil and
political rights”189 is to refer to an entire composite of rights. Estrada theorizes
452 SUPREME COURT REPORTS ANNOTATED that because there was a sweeping reference to this collectivity, then everything
Risos-Vidal vs. Commission on Elections in the ‘basket’ has been restored.
tal rights as “third-generation human rights.” Vasak’s conception of three Estrada’s theory fails on two points. First, it fails to consider the consequences
generations of human rights is a deliberate effort to parallel the French Revolution of statutory requirements which specifically refer to the rights of suffrage and to
ideals of liberty, equality, and fraternity, with each generation ordinally reflecting hold public office. Second, it fails to recognize that the language used in the pardon
the three ideals. Thus, “[f]irst-generation, ‘civil-political’ rights deal with liberty is equivocal at best, and, worse, the conclusion he derives from this equivocal
and participation in political life.”182 language is even contradicted by
In our jurisprudence, Simon, Jr. v. Commission on Human Rights183 discussed _______________
the concept of human rights as “so generic a term that any attempt to define it . . .
could at best be described as inconclusive.”184 Further, it attempted to define civil 185 Id., at pp. 132-133.
rights and political rights as follows: 186 Available at <http://www.globalization101.org/three-generations-of-
The term “civil rights,” has been defined as referring — rights/>.
“(to) those (rights) that belong to every citizen of the state or country, or, in a 187 Id.
wider sense, to all its inhabitants, and are not connected with the organization or 188 Id.
administration of government. They include the rights of property, marriage, equal 189 Rollo, p. 265.
protection of the laws, freedom of contract, etc. or, as otherwise defined civil rights 454
are rights appertaining to a person by virtue of his citizenship in a state or 454 SUPREME COURT REPORTS ANNOTATED
community. Such term may also refer, in its general sense, to rights capable of
being enforced or redressed in a civil action.” Risos-Vidal vs. Commission on Elections
Also quite often mentioned are the guarantees against involuntary servitude, other examples previously considered in jurisprudence. Thus, he insists on a
religious persecution, unreasonable searches and seizures, and imprisonment for conclusion that does not logically follow from his premises.
debt. Estrada capitalizes on the broad conception of civil and political rights as
_______________ including in its scope the rights of suffrage and the right to hold public office. That
is precisely the handicap in his theory: It is broad; it fails to account for
requirements relating to specific rights.
As against the broad concept of civil and political rights as an expansive 552 (1968) [Per J. J.B.L. Reyes, En Banc]; Lacuna v. Abes, 133 Phil. 770; 24 SCRA
composite or a vast spectrum of rights having to do with liberty and membership 780 (1968) [Per J. J.B.L. Reyes, En Banc]; In Re: Atty. Saturnino Parcasio, 161
in the political community, Articles 36 and 41 of the Revised Penal Phil. 437; 69 SCRA 336 (1976) [Per J.Aquino, Second Division]; In Re: Atty.
Code specifically deal with the rights of suffrage and to hold public office. Tranquilino Rovero, 189 Phil. 605; 101 SCRA 799 (1980) [Per J. Concepcion,
Juxtaposed with the manifold category of civil and political rights, the effect of Jr., En Banc]; Sabello v. Department of Education, Culture and Sports,
Articles 36 and 41 is that, in the specific context of the President’s exercise of the 456
power to grant pardon to a convict, the rights of suffrage and to hold public office 456 SUPREME COURT REPORTS ANNOTATED
are segregated from all other similar rights.
This segregation is not grounded on whim. It hearkens to the fundamental Risos-Vidal vs. Commission on Elections
distinction between public office as a public trust, on the one hand, and pardon as indicate that the inclusion of the qualifier “full” is common practice. In that
a private act, on the other. The special requirement of express restoration or case, the phrase “full civil and political rights” was “written on a standard printed
remission affirms what was earlier discussed to be the need to desegregate, or to form.”193
bridge the disjunct between the private gesture of pardoning — originally intended This is not the occasion to rule on the sufficiency of adding the qualifier “full”
only to relieve an individual’s misery over the harshness of punishment — and the for purposes of restoring even the rights of suffrage and to hold public office.
public consequence (no longer connected with the basic purpose of mollifying penal However, burdened with the task of interpretation, particular note should be taken
misery) of not only enabling a convict to participate in the selection of public by this court of President Gloria Macapagal-Arroyo’s deviation from previous,
officials, but to himself or herself be a repository of public trust should he or she standard practice.
become a public officer. To reiterate, public office “partakes of a privilege which the The President must be presumed to be fully cognizant of the significance and
State grants only to such classes consequences of the manner by which he or she executes official acts, as well as
455 the manner by which they are formally reduced to writing. It is revealing that
former President Gloria Macapagal-Arroyo chose to deviate from many historical
VOL. 747, JANUARY 21, 2015 455
examples and from what appears to be common practice. Aware of the significance
Risos-Vidal vs. Commission on Elections of excluding the qualifier “full,” she chose to grant pardon to Estrada under
of persons which are most likely to exercise it for the common good.”190 entirely generic and indistinct terms.
Consistent with the public interest inherent in the rights of suffrage and Similarly, the President must be presumed to be cognizant of statutes and
holding public office, thus, if the President is to not actually say that the rights of what they require. In granting pardon to Estrada, former President Gloria
suffrage and to hold public office are restored, there is plainly no basis for Macapagal-Arroyo must have been fully informed of the requirements of Articles
concluding that they have, in fact, been restored. 36 and 41 of the Revised Penal Code if it was ever her intent to restore Estrada’s
Such is the situation in this case. At no point does the pardon actually, rights to vote and be voted for elective public office or to otherwise remit the
expressly, categorically, and unmistakably say that Estrada’s rights to suffrage penalty of perpetual absolute disqualification.
and to hold public office have been restored. That this court — the Supreme Court Not only did former President Arroyo choose to shy away from qualifying the
of the Republic — has been asked to step in and settle the controversy is the best restoration of Estrada’s civil and political rights as “full.” She also chose, contrary
proof of this. to Articles 36 and 41, to be totally silent on the restoration of the rights to vote and
Apart from these, a meticulous consideration of how the restoration of be voted for elective public office and on the remission of the
Estrada’s civil and political rights is worded, especially in contrast with other _______________
examples previously considered in jurisprudence, casts serious doubt on whether
the restoration was as expansive as Estrada asserts. 259 Phil. 1109; 180 SCRA 623 (1989) [Per J. Gancayco, First Division].
The exact words of the pardon granted to Estrada are: “He is hereby restored 193 Monsanto v. Factoran, Jr., supra note 127.
to his civil and political rights.”191 457
In contrast, jurisprudence is replete with pardon, working to restore civil and VOL. 747, JANUARY 21, 2015 457
political rights in this wise: “full civil and political rights.”192 A fact noted in one
case even seems to Risos-Vidal vs. Commission on Elections
_______________ penalty of absolute disqualification. These twin circumstances — first, of her
exclusion of a qualifier and second, her silence on restoration and remission — can
190 Romeo Jalosjos v. COMELEC, supra note 154. only mean that contrary to Estrada’s contention, his rights to vote and be voted for
191 Rollo, p. 265. elective public office have not been restored, and his perpetual absolute
192 Cristobal v. Labrador, supra note 134; see also Pelobello v. disqualification not remitted.
Palatino, supranote 135; National Shipyards and Steel Corporation v. Lest misinterpretation ensue, I am not here giving rise to a false dilemma and
National Shipyards Employees and Workers Association, 132 Phil. 59; 23 SCRA rendering inutile the restoration of Estrada’s civil and political rights. Indeed, they
have been restored, all but the rights denied to him on account of the unremitted
penalty of perpetual absolute disqualification, among these being the rights to vote 197 Id., at pp. 487-488; pp. 409-410, citing West’s Encyclopedia of American
and be voted for elective public office. That entire spectrum of rights “deal[ing] with Law (2nd ed., 2008); Echegaray v. Secretary of Justice, G.R. No. 132601, January
liberty and participation in political life”194 — to mention but a few such as his right 19, 1999, 297 SCRA 654 [Per Curiam, En Banc]; Ruben E. Agpalo, Statutory
to liberty; freedom of abode and movement; privacy rights; rights of expresion, Construction (2nd ed., 1990) and Martin, Statutory Construction (6th ed., 1984).
association, assembly; his right to petition the government and to a redress of 198 See People v. Purisima, 176 Phil. 186, 204; 86 SCRA 542, 559 (1978)
grievances — are his to enjoy except for the select class of rights denied to him on [Per J.Muñoz-Palma, En Banc], citing Words and
account of the omissions in his pardon. 459
Similarly, my pronouncements should not be taken as rendering illusory the VOL. 747, JANUARY 21, 2015 459
concept of “plenary pardon” — a concept that, as Estrada pointed out, is recognized
in Section 12 of the Omnibus Election Code. The President remains free to grant Risos-Vidal vs. Commission on Elections
pardon that works to restore all of a convict’s civil and political rights, even those In People v. Judge Purisima,199 this court had occasion to interpret an act of
of suffrage and to hold public office. What I have however emphasized is that, the President (who then held the power to legislate) through a reading of whereas
should the President choose to be so expansive in making such a restoration, he or clauses.200 People v. Judge Purisima concluded, referring to “the presence of events
she should be clear with his or her intentions. which led to or precipitated the enactment of P.D. 9… [as] clearly spelled out in
_______________ the ‘Whereas’ clauses,”201 that Presidential Decree No. 9 excluded instances where
a defendant carried bladed, pointed, or blunt weapons in situations which were not
194 Available at <http://www.globalization101.org/three-generations-of- related to the purposes of Proclamation No. 1081 and General Orders Nos. 6 and
rights/>. 7. Further identifying the purposes for the issuance of Proclamation No. 1081, this
458 court also read two of Proclamation No. 1081’s own whereas clauses 202 and
concluded that it was aimed at putting an end
458 SUPREME COURT REPORTS ANNOTATED _______________
Risos-Vidal vs. Commission on Elections
X Phrases, “Preamble,” citing James v. Du Bois, 16 N.J.L. (1 Har.) 285, 294.
199 People v. Purisima, id.
The pardon’s preambular clauses 200 WHEREAS, pursuant to Proclamation No. 1081 dated September 21,
militate against Estrada’s position 1972, the Philippines has been placed under a state of martial law;
WHEREAS, by virtue of said Proclamation No. 1081, General Order No. 6
Apart from the pardon’s absolute silence on the matters of restoration and dated September 22, 1972 and General Order No. 7 dated September 23, 1972,
remission, its preambular or whereas clauses militate against the conclusion that have been promulgated by me;
Estrada’s rights to suffrage and to hold public office have been restored. WHEREAS, subversion, rebellion, insurrection, lawless violence, criminally,
The pardon’s three preambular clauses read: chaos and public disorder mentioned in the aforesaid Proclamation No. 1081 are
WHEREAS, this Administration has a policy of releasing inmates who have committed and abetted by the use of firearms, explosives and other deadly
reached the age of seventy (70), weapons[.]
WHEREAS, Joseph Ejercito Estrada has been under detention for six and a 201 People v. Purisima, supra note 198 at p. 203; p. 558.
half years, 202 WHEREAS, these lawless elements having taken up arms against our
WHEREAS, Joseph Ejercito Estrada has publicly committed to no longer seek duly constituted government and against our people, and having committed and
any elective position or office[.]195 are still committing acts of armed insurrection and rebellion consisting of armed
raids, forays, sorties, ambushes, wanton acts of murders, spoilage, plunder,
A preamble is “not an essential part of an act.”196 It is only an introduction looting, arsons, destruction of public and private buildings, and attacks against
which indicates intent or purpose. In and of itself, it cannot be the source of rights innocent and defenseless civilian lives and property, all of which activities have
and obligations. Thus, “[w]here the meaning of [an instrument] is clear and seriously endangered and continue to endanger public order and safety and the
unambiguous, the preamble can neither expand nor restrict its operation, much security of the nation. . . .
less prevail over its text.”197 Stated otherwise, it may be resorted to only when the ....
instrument is “ambiguous and difficult of interpretation.”198 460
_______________ 460 SUPREME COURT REPORTS ANNOTATED

195 Rollo, p. 265. Risos-Vidal vs. Commission on Elections


196 Kuwait Airways Corporation v. Philippine Airlines, Inc., 605 Phil. 474; to subversive activities. Thus, this court concluded that the act of carrying
587 SCRA 399 (2009) [Per J. Tinga, Second Division]. bladed, pointed, or blunt weapons was only punishable to the extent that it was
done in the context of subversive activities.
Jurisprudence and other official acts of this court are replete with instances in accentuates their intention to treat the agreement as one of conventional
which reference to preambular clauses was resorted to in interpreting instruments subrogation. And it is basic in the interpretation of contracts that the intention of
other than statutes and official acts of the President. In Licaros v. the parties must be the one pursued (Rule 130, Section 12, Rules of Court).
Gatmaitan,203 this court sustained the Court of Appeals’ reference to a whereas ....
clause in a contract between private parties (i.e., a memorandum of agreement) As previously discussed, the intention of the parties to treat the
and thereby the conclusion that the parties “intended to treat their agreement as Memorandum of Agreement as embodying a conventional subrogation is
one of conventional subrogation.”204 In Kuwait Airways Corporation v. shown not only by the “whereas clause” but also by the signature space
_______________ captioned “WITH OUR CONFORME” reserved for the signature of a
representative of Anglo-Asean Bank. These provisions in the aforementioned
WHEREAS, it is evident that there is throughout the land a state of anarchy Memorandum of Agreement may not simply be disregarded or dismissed as
and lawlessness, chaos and disorder, turmoil and destruction of a magnitude superfluous.
equivalent to an actual war between the forces of our duly constituted government It is a basic rule in the interpretation of contracts that “(t)he various
and the New People’s Army and their satellite organizations because of the stipulations of a contract shall be interpreted together, attributing to the doubtful
unmitigated forays, raids, ambuscades, assaults, violence, murders, ones that sense which may result from all
assassinations, acts of terror, deceits, coercions, threats, intimidations, treachery, 462
machinations, arsons, plunders and depredations committed and being committed 462 SUPREME COURT REPORTS ANNOTATED
by the aforesaid lawless elements who have pledged to the whole nation that they
will not stop their dastardly effort and scheme until and unless they have fully Risos-Vidal vs. Commission on Elections
attained their primary and ultimate purpose of forcibly seizing political and state Philippine Airlines, Inc.,205 it was impliedly acknowledged that resort to a
power in this country by overthrowing our present duly constituted government. . whereas clause is permissible in interpreting a contract entered into by the
.. government; except that, because the circumstances have changed, it was deemed
203 414 Phil. 857; 362 SCRA 548 (2001) [Per J. Gonzaga-Reyes, Third unnecessary to proceed to an interpretation in light of the relevant whereas
Division]. clause.206 In Conte v. Commission on Audit,207 this
204 Id., at pp. 868-872; p. 558: _______________
We agree with the finding of the Court of Appeals that the Memorandum of
Agreement dated July 29, 1988 was in the nature of a conventional subrogation of them taken jointly.” Moreover, under our Rules of Court, it is mandated that
which requires the consent of the debtor, Anglo-Asean Bank, for its validity. We “(i)n the construction of an instrument where there are several provisions or
note with approval the following pronouncement of the Court of Appeals: particulars, such a construction is, if possible, to be adopted as will give effect to
“Immediately discernible from above is the common feature of contracts all.” Further, jurisprudence has laid down the rule that contracts should be so
involving conventional subrogation, namely, the approval of the debtor to the construed as to harmonize and give effect to the different provisions thereof.
subroga- (Emphasis and underscoring supplied)
461 205 Supra note 196.
206 Id., at pp. 487-488; pp. 409-410:
VOL. 747, JANUARY 21, 2015 461
One line of argument raised by Kuwait Airways can be dismissed outright.
Risos-Vidal vs. Commission on Elections Kuwait Airways points out that the third Whereas clause of the 1981 Commercial
Agreement stated: “NOW, it is hereby agreed, subject to and without prejudice to
_______________ any existing or future agreements between the Government Authorities of the
Contracting Parties hereto. . . .” That clause, it is argued, evinces
tion of a third person in place of the creditor. That Gatmaitan and Licaros had acknowledgement that from the beginning Philippine Airlines had known fully
intended to treat their agreement as one of conventional subrogation is plainly well that its rights under the Commercial Agreement would be limited by whatever
borne by a stipulation in their Memorandum of Agreement, to wit: agreements the Philippine and Kuwait governments may enter into later.
“WHEREAS, the parties herein have come to an agreement on the nature, form But can a perambulatory clause, which is what the adverted “Whereas” clause
and extent of their mutual prestations which they now record herein with the is, impose a binding obligation or limitation on the contracting parties? In the case
express conformity of the third parties concerned” (emphasis supplied), of statutes, while a preamble manifests the reasons for the passage of the statute
which third party is admittedly Anglo-Asean Bank. and aids in the interpretation of any ambiguities within the statute to which it is
Had the intention been merely to confer on appellant the status of a prefixed, it nonetheless is not an essential part of an act, and it neither enlarges
mere “assignee” of appellee’s credit, there is simply no sense for them to nor confers powers. Philippine Airlines submits that the same holds true as to the
have stipulated in their agreement that the same is conditioned on the preambular whereas clauses of a contract.
“express conformity” thereto of Anglo-Asean Bank. That they did so only
What was the intention of the parties in forging the “Whereas” clause and unambiguous. This should suffice to put an end to Estrada’s asseverations
and the contexts the parties understood it in 1981? In order to judge the that he was qualified to run for Mayor of Manila.
intention of the contracting parties, their contemporaneous and subsequent acts Nevertheless, even if the position that there remains room for interpretation
shall be principally considered, and in doing so, the courts may was to be indulged, a reading of the pardon as a whole, and an illumination,
463 through the preambular clauses, of the pardon’s supposed ambiguity, will lead to
VOL. 747, JANUARY 21, 2015 463 the same conclusion: Estrada was and remains to be disqualified.
As in Purisima, the pardon’s whereas clauses indicate events and
Risos-Vidal vs. Commission on Elections considerations that precipitated or led to the grant of pardon. More specifically,
court referred to whereas clauses in interpreting a resolution issued by the the third whereas clause reveals that the pardon was premised on Estrada’s prior,
Social Security System.208 Similarly, this court’s public commitment of disabling himself from being a candidate in an
_______________ _______________

consider the relations existing between the parties and the purpose of the preted to mean that the benefit being granted is none other than a kind of
contract. In 1981, Philippine Airlines was still owned by the Philippine amelioration to enable the retiring employee to enjoy (or survive) his retirement
government. In that context, it is evident that the Philippine government, years and a reward for his loyalty and service. Moreover, it is plain to see that the
as owner Philippine Airlines, could enter into agreements with the grant of said financial assistance is inextricably linked with and inseparable from
Kuwait government that would supersede the Commercial Agreement the application for and approval of retirement benefits under RA 660, i.e., that
entered into by one of its GOCCs, a scenario that changed once Philippine availment of said financial assistance under Res. 56 may not be done
Airlines fell to private ownership. Philippine Airlines argues before us independently of but only in conjunction with the availment of retirement benefits
that the cited preambular stipulation is in fact superfluous, and we can under RA 660, and that the former is in augmentation or supplementation of the
agree in the sense that as of the time of the execution of the Commercial latter benefits.
Agreement, it was evident, without need of stipulation, that the 209 En Banc Resolution Providing for Other Sources of the Judiciary
Philippine government could enter into an agreement with the Kuwait Development Fund dated September 14, 1999.
government that would prejudice the terms of the commercial 210 Pres. Decree No. 1949 (1984), otherwise known as Establishing a
arrangements between the two airlines. After all, Philippine Airlines then Judiciary Development Fund and for Other Purposes.
would not have been in a position to challenge the wishes of its then majority 465
stockholder — the Philippine government. (Emphasis and underscoring supplied)
VOL. 747, JANUARY 21, 2015 465
207 332 Phil. 20; 264 SCRA 19 (1996) [Per J. Panganiban, En Banc].
208 Id., at pp. 32-33; pp. 27-28: Risos-Vidal vs. Commission on Elections
Petitioners’ contentions are not supported by law. We hold that Res. 56 election (i.e., “to no longer seek any elective position or office”).211
constitutes a supplementary retirement plan. The preceding discussions underscored the nature of the power to pardon (in
A cursory examination of the preambular clauses and provisions of Res. 56 particular, and to extend clemency, in general) as being fundamentally a matter of
provides a number of clear indications that its financial assistance plan constitutes executive discretion. However, that this is a matter resting on the President’s
a supplemental retirement/pension benefits plan. In particular, the fifth prerogative is no license for the President to heedlessly brandish it. As with all
preambular clause which provides that “it is the policy of the Social Security other powers vested in the executive, it is a power that is not to be abused. It cannot
Commission to promote and to protect the interest of all SSS employees, with a be exercised arbitrarily, whimsically, or capriciously. The President may well be a
view to providing for their well-being during both their working and retirement despot, otherwise.
years,” and the wording of the resolution itself which states “Resolved, further, Thus, if the power to pardon were ever to be invoked, it must remain true to
that SSS employees who availed themselves of the said life annuity (under RA its reason for existence: to correct “infirmities, deficiencies or flaws in the
660), in appreciation and recognition of their long and faithful service, be granted administration of justice”;212 to “mitigat[e] whatever harshness might be
financial assistance x x x” can only be inter- generated by a too strict an application of the law[;]”213 or to otherwise “temper the
464 gravity of [a punishment’s] wrath.”214 To the extent, therefore, that the power to
464 SUPREME COURT REPORTS ANNOTATED pardon is exercised in a manner that evinces nothing more than the indulgence of
caprices, an issue that may properly be taken cognizance of by this court arises:
Risos-Vidal vs. Commission on Elections grave abuse of discretion amounting to lack or excess of jurisdiction.
En Banc resolution in A.M. No. 99-8-01-SC,209 issued by this court in the In stating this, I remain mindful of this court’s pronouncement in 2007
exercise of its rule-making power, cited a statute’s210whereas clause. in People v. Rocha,215 which I have cited earlier. At initial glance, Rocha appears
The pardon extended to Estrada is definite by its omission: There is neither an to totally erode the power of judicial review in relation to the grant of executive
express restoration of Estrada’s rights to vote and be voted for elective public office clemency:
nor a remission of his perpetual absolute disqualification. To this extent, it is clear
_______________ While it behooves this court to extend to the President the presumption that
the grant is attended with good reason, so, too, this court should not indulge a
211 Rollo, p. 265. patently frivolous exercise of presidential discretion.
212 J. Padilla, Dissenting Opinion in Llamas v. Orbos, supra note 118 at p. Presently, this court finds itself grappling with pardon extended to a deposed
946; p. 866, citing Joaquin G. Bernas, S.J., On The Revised 1973 Philippine President of the Republic who was convicted for the crime of plunder.
Constitution, part 1, p. 228 (1983). Joseph Ejercito Estrada is no common convict. In him was reposed the trust of
213 Id. an overwhelming number of Filipinos. He was elected to nothing less than the
214 Monsanto v. Factoran, Jr., supra note 127 at pp. 198-199; p. 196, highest office of the land. Assuming the presidency, he swore, invoking the name
citing United States v. Wilson, supra note 128, cited in Joaquin G. Bernas, of God, to “faithfully and conscientiously fulfil [his] duties as President[; to]
S.J., The 1973 Philippine Constitution, Notes and Cases, part 1, p. 355 (1974). preserve and defend [the] Constitution[;] and [to] consecrate [himself] to the
215 Supra note 124. service of the Nation.”217 This notwithstanding, he is a man, who, tormented with
466 recriminations of massive corruption and failing to exculpate himself in the eyes
of the Filipino people, was left with no recourse but to leave the Presidency. He
466 SUPREME COURT REPORTS ANNOTATED
stood trial for and was convicted of plunder: a conviction that endures and stands
Risos-Vidal vs. Commission on Elections unreversed.
This Court cannot review, much less preempt, the exercise of executive A ruling on this petition cannot be bereft of context, both of the present and of
clemency under the pretext of preventing the accused from evading the penalty our history. Similarly, this court cannot turn a blind eye on its own recognition of
of reclusion perpetua or from trifling with our judicial system. Clemency is not a the gravity and grievousness that Estrada’s conviction for plunder entails.
function of the judiciary; it is an executive function. Thus, it is the President, not _______________
the judiciary, who should exercise caution and utmost circumspection in the
exercise of executive clemency in order to prevent a derision of the criminal justice 217 Const. (1987), Art. VII, Sec. 5:
system. We cannot and shall not deny accused-appellants’ Motions to Withdraw Section 5. Before they enter on the execution of their office, the President,
Appeal just because of their intention of applying for executive clemency. With the the Vice President, or the Acting President shall take the following oath or
Constitution bestowing upon the Executive the power to grant clemency, it affirmation:
behooves the Court to pass the ball to the President and let her determine the fate “I do solemnly swear (or affirm) that I will faithfully and conscientiously fulfill
of accused-appellants.216 my duties as President (or Vice President or Acting President) of the Philippines,
preserve and defend its Constitution, execute its laws, do justice to every man, and
However, a meticulous reading of Rocha reveals that its pronouncements were consecrate myself to the service of the Nation. So help me God.” (In case of
made in a very specific context, i.e., the issue of whether this court should allow affirmation, last sentence will be omitted.)
the withdrawal of the appeals of accused-appellants in order that they may avail 468
themselves of executive clemency. In making the quoted pronouncement, this court
468 SUPREME COURT REPORTS ANNOTATED
merely affirmed the basic precept that the power to extend clemency is a choice for
the President — and not for any other institution, such as this court — to make. Risos-Vidal vs. Commission on Elections
Thus, it would be improper for this court to take any action that would effectively In 2001, in Estrada v. Sandiganbayan,218 this court, against the asseverations
prevent the President from even making that choice. of Estrada himself, ruled that plunder is inherently immoral, i.e., malum in se. In
Rocha was a deferential statement that recognized where the power to extend so doing, this court, quoting the concurring opinion of Justice Vicente V. Mendoza,
clemency was lodged. It was a recognition that this court could not preempt the emphasized that any doubt on the inherent immorality of plunder “must be
grant of clemency. At no point, however, did Rocha sanction the fanciful exercise deemed to have been resolved in the affirmative by the decision of Congress in
of the power. Nowhere did it say that the power granted to the President may be 1993 to include it among the heinous crimes punishable by reclusion perpetua to
divorced from its raison d’être. death.”219 Estrada v. Sandiganbayan, quoting People v.
_______________ Echegaray,220 unequivocally underscored the abhorrence that animates the
classification of plunder as a heinous crime punishable by death. This court did
216 Id., at pp. 538-539; p. 778, citing Joaquin G. Bernas, The 1987 not mince words:
Constitution of the Republic of the Philippines, A Commentary, p. 935 (2003). There are crimes, however, in which the abomination lies in the significance
467 and implications of the subject criminal acts in the scheme of the larger socio-
political and economic context in which the state finds itself to be struggling to
VOL. 747, JANUARY 21, 2015 467
develop and provide for its poor and underprivileged masses. Reeling from decades
Risos-Vidal vs. Commission on Elections of corrupt tyrannical rule that bankrupted the government and impoverished the
population, the Philippine Government must muster the political will to dismantle
the culture of corruption, dishonesty, greed and syndicated criminality that so 222 Id., at pp. 366-367; p. 454.
deeply entrenched itself in the structures of society and the psyche of the populace. 470
[With the government] terribly lacking the money to provide even the most basic 470 SUPREME COURT REPORTS ANNOTATED
services to its people, any form of misappropriation or misapplication of
government funds translates to an actual threat to the very existence of Risos-Vidal vs. Commission on Elections
government, and in turn, the very survival of the people it governs over. Viewed in and shall be punished by reclusion perpetua to death. Any person who
this context, no less heinous are the effects and repercussions of crimes like participated with the said public officer in the commission of an offense
qualified bribery, destructive arson resulting in death, and drug offenses involving contributing to the crime of plunder shall likewise be punished for such offense. In
government officials, employees or officers, that the imposition of penalties, the degree of participation and the attendance of
_______________ mitigating and extenuating circumstances, as provided by the Revised Penal Code,
shall be considered by the court. The court shall declare any and all ill-gotten
218 421 Phil. 290; 369 SCRA 394 (2001) [Per J. Bellosillo, En Banc]. wealth and their interests and other incomes and assets including the properties
219 Id., at p. 365; p. 452. and shares of stocks derived from the deposit or investment thereof forfeited in
220 335 Phil. 343; 267 SCRA 682 (1997) [Per Curiam, En Banc]. favor of the State.
469
This technical-legal definition notwithstanding, in common understanding, to
VOL. 747, JANUARY 21, 2015 469 plunder is to pillage or to ransack. It denotes more than wrongful taking as to
Risos-Vidal vs. Commission on Elections amount to common larceny. Synonymous with despoiling and marauding,
their perpetrators must not be allowed to cause further destruction and plundering evokes the devastation wrought by hordes laying waste to an
damage to society.221 (Emphasis supplied) enemy.223 By plundering, a subjugator impresses the fact of its having vanquished
another by arrogating unto itself the spoils of conquest and rendering more
Turning its attention specifically to Republic Act No. 7080, the Anti-Plunder ignominious an otherwise simple defeat.
Law, Estrada v. Sandiganbayan stated: Plundering as a crime and by its scale, therefore, entails more than greed and
Our nation has been racked by scandals of corruption and obscene profligacy of covetousness. It conjures the image of a public officer deluded in the thought that
officials in high places which have shaken its very foundation. The anatomy of he or she is some overlord, free to ravage and entitled to seize all that his or her
graft and corruption has become more elaborate in the corridors of time as realm can provide. It entails more than ordinary moral turpitude (i.e., an
unscrupulous people relentlessly contrive more and more ingenious ways to bilk inherently immoral act)224 as acts like theft, rob-
the coffers of the government. Drastic and radical measures are imperative to fight _______________
the increasingly sophisticated, extraordinarily methodical and economically
catastrophic looting of the national treasury. Such is the Plunder Law, especially 223 Definition available at <http://www.merriam-webster.com/
designed to disentangle those ghastly tissues of grand-scale corruption which, if dictionary/plunder>.
left unchecked, will spread like a malignant tumor and ultimately consume the 224 See Teves v. Commission on Elections, supra note 117, citing Dela Torre v.
moral and institutional fiber of our nation. The Plunder Law, indeed, is a living Commission on Elections, supra note 117.
testament to the will of the legislature to ultimately eradicate this scourge and thus “It (moral turpitude) implies something immoral in itself, regardless of the fact
secure society against the avarice and other venalities in public office.222 (Emphasis that it is punishable by law or not. It must not be merely mala prohibita, but the
supplied) act itself must be inherently immoral. The doing of the act itself, and not its
prohibition by statute fixes the moral turpitude. Moral turpitude does not,
Section 2 of Republic Act No. 7080, as amended, provides for the definition of however, in-
and penalties for plunder, as follows: 471
Section 2. Definition of the Crime of Plunder; Penalties.—Any public officer VOL. 747, JANUARY 21, 2015 471
who, by himself or in connivance with members of his family, relatives by affinity
or consanguinity, business associates, subordinates or other persons, amasses, Risos-Vidal vs. Commission on Elections
accumulates or acquires ill-gotten wealth through a combination or series of overt bery, bribery, profiteering, estafa, extortion, and embezzlement have been
or criminal acts as described in Section 1(d) hereof in the aggregate amount or total categorized.225 It evinces such a degree of depravity and debasement so heinous
value of at least Fifty million pesos (P50,000,000.00) shall be guilty of the crime of that, were it not for the subsequent enactment of a statute (i.e., Republic Act No.
plunder 9346), it would remain punishable by death.
_______________ Recognition must be given to the legislative wisdom underlying the choice of
penalty. This is not only with respect to the severity of punishment chosen (i.e.,
221 Estrada v. Sandiganbayan, supra note 218 at pp. 365-366; p. 453. deprivation of life or deprivation of liberty for the longest duration contemplated
by the scale of penalties under the Revised Penal Code) but similarly with all other
accessories that the penalties of reclusion perpetua and/or death entail. Congress, of the third preambular clause is not empty rhetoric. It is an indispensable qualifier
in choosing to penalize plunder with reclusion perpetua to death, must certainly indicating that Estrada was pardoned precisely in view of his promise to no longer
have been cognizant of how these penalties did not only entail the deprivation of seek (elective) public office. Similarly, it estab-
the right to life and/or liberty, but also of how, consistent with Articles 40 and 41
of the Revised Penal Code, they carried the accessory penalty of perpetual absolute _______________
disqualification.
To recognize this legislative wisdom is, thus, to recognize that penalizing 227 Id.
plunder inherently entails the exclusion of a convict from elective exercises for 228 Id., at p. 366; p. 454.
public office, both as a candidate and as a voter, as well as from offices and public 473
employments. This is consistent with the recognition that plunder is an VOL. 747, JANUARY 21, 2015 473
“abomination . . . in the scheme of the larger socio-political and economic
context.”226Through the penalty of perpetual absolute disqualification, it is, thus, Risos-Vidal vs. Commission on Elections
ensured that a person convicted of plunder will no longer find himself or herself in lishes that the grant of pardon notwithstanding, there is no betrayal of the
the same setting, i.e., holding (elective) public office, which, in the first place, fundamental policy of aversion against plunder as an affront to “the larger socio-
enabled the commission of plunder. political and economic context.”229
_______________ Accordingly, any reading of the phrase on which Estrada capitalizes — “[h]e is
hereby restored to his civil and political rights” — must be made in accordance
clude such acts as are not of themselves immoral but whose illegality lies in with the qualifier evinced by an undertaking Estrada himself made “to no longer
their being positively prohibited.” seek any elective position or office.”230 Read as such, the pardon could not have
225 See J. Brion’s Concurring Opinion in Teves v. Commission on possibly worked to reverse the effects of the penalty of perpetual absolute
Elections, supra note 117 at pp. 740-742; pp. 24-26. disqualification or to otherwise restore his right to vote in any election for any
226 Estrada v. Sandiganbayan, supra note 118 at p. 356; p. 453. popular elective office or to be elected to such office.
472
XI
472 SUPREME COURT REPORTS ANNOTATED
Estrada’s reincarceration is not a
Risos-Vidal vs. Commission on Elections proper issue in this case.
It is against this backdrop of plunder as a social “abomination” 227as well as
“corruption and obscene profligacy of officials in high places”228 that Estrada Drawing attention to Estrada’s undertaking, Risos-Vidal theorizes that
insists on a pardon that worked to restore his rights to vote and be voted for Estrada was granted a conditional pardon, i.e, that it was laden with a resolutory
elective public office. Bereft of any clue as to the intent behind the grant of pardon, condition and that, as Estrada reneged on his undertaking, the rights vested by
such grant is mind-boggling. It, and its statement that Estrada is restored to his the pardon must be deemed extinguished. Citing Article 159 of the Revised Penal
civil and political rights, appear to defy the disdain which animates the policy Code, Risos-Vidal, thus, suggests that Estrada should once again be incarcerated:
against plunder. Thus, clearly, when Joseph Estrada himself intentionally and wilfully
To reiterate, however, a President’s grant of pardon must be presumed to be breached his pardon when he filed his certificate of candidacy for the position of
grounded on the basic nature of pardon as a means for tempering the harshness of Mayor of the City of Manila, he is guilty of breach of the conditions of the pardon
punishment. A reading of the preamble or whereas clauses of the pardon granted which puts and [sic] end to the pardon itself and thereby immediately restoring
to Estrada will reveal that, indeed, the pardon was animated by nothing more than the terms of conviction imposed by the Sandiganbayan. He should therefore be
a desire to salve Estrada’s suffering. recommitted to prisión consistent with Article 159 of the Revised Penal Code
Consider the recognition made in the first and second preambular clauses that which provides:
Estrada was already more than 70 years old and had been in detention for about _______________
six and a half years. These preambular clauses provide context to why President
Gloria Macapagal-Arroyo saw wisdom in tempering Estrada’s suffering: Keeping 229 Id., at p. 365; p. 453.
in prison a septuagenarian — a man who could well be considered to be in the 230 Rollo, p. 265.
twilight years of his life — may be too severe; anyway, Estrada had already been 474
deprived of liberty for a considerable length of time.
474 SUPREME COURT REPORTS ANNOTATED
The third preambular clause is even more revealing. It unveils the undertaking
made by Estrada (acknowledged and unchallenged by him through his unqualified Risos-Vidal vs. Commission on Elections
handwritten acceptance) that he would no longer embark on the very same affair, ART. 159. Other Cases of Evasion of Service of Sentence.—The penalty
i.e., (elective) public office, that facilitated his commission of plunder. The inclusion of prisión correccional in its minimum period shall be imposed upon the convict
who, having been granted conditional pardon by the Chief Executive, shall violate
any of the conditions of such pardon. However, if the penalty remitted by the disqualification is not negated by the statement in Section 40(a) of the Local
granting of such pardon be higher than six years, the convict shall then suffer the Government Code that the disqualification relating to “[t]hose sentenced by final
unexpired portion of his original sentence.231 judgment for an offense involving moral turpitude or for an offense punishable by
one (1) year or more of imprisonment” shall last for “two (2) years after serving
Estrada counters that he was “granted an absolute pardon and thereby sentence.”
restored to his full civil and political rights, including the right to seek public _______________
elective [sic] office.”232 Estrada, therefore, construes an “absolute pardon” as one
with sweeping, all-encompassing effects. 234 Monsanto v. Factoran, Jr., supra note 127 at p. 198; p. 196, citing United
As against the pardon’s premise of Estrada’s commitment to no longer seek States v. Wilson, supra note 128, cited in Joaquin G. Bernas, The 1973 Philippine
any elective position or office is Estrada’s acceptance: Constitution, Notes and Cases, part I, p. 355 (1974).
Received & accepted 235 Rollo, p. 265.
Joseph E. Estrada (sgd.) 476
DATE: 26 Oct. ‘07
476 SUPREME COURT REPORTS ANNOTATED
TIME: 3:35 PM233
Risos-Vidal vs. Commission on Elections
Made in Estrada’s own handwriting, the acceptance articulates no This, even if Section 40 of the Local Government Code is the specific ground relied
qualification or reservation. Hence, it is an acceptance that is inclusive of his upon by Risos-Vidal in seeking to disqualify Estrada.
promise to no longer seek elective public office. The relation between Article 30 of the Revised Penal Code — on the effects of
Nevertheless, the matter of Estrada’s reincarceration as a possible perpetual absolute disqualification — and Section 40(a) of the Local Government
consequence of the occurrence of a resolutory condi- Code was extensively discussed in Romeo Jalosjos v. COMELEC:236
_______________ Well-established is the rule that every new statute should be construed in
connection with those already existing in relation to the same subject matter and
231 Id., at p. 1521. all should be made to harmonize and stand together, if they can be done by any
232 Id., at pp. 1765-1766. fair and reasonable interpretation.
233 Id. Certified true copy issued by Marianito M. Dimaandal, Director IV, ....
Malacañan Records Office. Keeping with the above mentioned statutory construction principle, the Court
475 observes that the conflict between these provisions of law may be properly
reconciled. In particular, while Section 40(a) of the LGC allows a prior convict to
VOL. 747, JANUARY 21, 2015 475
run for local elective office after the lapse of two (2) years from the time he serves
Risos-Vidal vs. Commission on Elections his sentence, the said provision should not be deemed to cover cases wherein
tion is no longer essential to the disposition of this case. After all, this case the law imposes a penalty, either as principal or accessory, which has the
pertains to a petition for disqualification. What this court is called upon to rule on effect of disqualifying the convict to run for elective office. An example of
is Estrada’s qualification to run for Mayor of Manila. this would be Article 41 of the RPC, which imposes the penalty of perpetual
In the limited context that excludes the question of Estrada’s possible absolute disqualification as an accessory to the principal penalties of reclusion
reincarceration, the materiality of his acceptance is in how such acceptance was perpetua and reclusion temporal[.]
imperative in order to bring the pardon to effect. As noted in Monsanto, “[a] pardon ....
is a deed, to the validity of which delivery is essential, and delivery is not complete Pertinently, it is observed that the import of Article 41 in relation to Article 30
without acceptance.”234 This, too, is reflected in the pardon’s text, the last of the RPC is more direct and specific in nature — insofar as it deprives the
paragraph of which reads: candidate to run for elective office due to his conviction — as compared to Section
Upon acceptance of this pardon by JOSEPH EJERCITO ESTRADA, this 40(a) of the LGC which broadly speaks
pardon shall take effect.235 _______________
XII
Estrada’s disqualification not 236 Supra note 154.
affected by the lapse of more than 477
two years since his release from
VOL. 747, JANUARY 21, 2015 477
prison
Risos-Vidal vs. Commission on Elections
Having settled on Estrada’s disqualification, it is worth emphasizing (in the of offenses involving moral turpitude and those punishable by one (1) year or
interest of settling whatever lingering doubts there may be) that his more of imprisonment without any consideration of certain disqualifying effects to
one’s right to suffrage. Accordingly, Section 40(a) of the LGC should be considered
as a law of general application and therefore, must yield to the more definitive RPC Estrada warns against the “massive disenfranchisement of votes [sic]”238 and
provisions in line with the principle of lex specialis derogat generali — general cautions against disrespecting “the sovereign will of the people as expressed
legislation must give way to special legislation on the same subject, and generally through the ballot.”239 In doing so, he makes much of the margin of more than
is so interpreted as to embrace only cases in which the special provisions are not 35,000 votes by which he edged out Lim.240
applicable. In other words, where two statutes are of equal theoretical application _______________
to a particular case, the one specially designed therefor should prevail.
In the present case, petitioner was sentenced to suffer the principal penalties 237 Id., at pp. 757-763.
of reclusion perpetua and reclusion temporal which, pursuant to Article 41 of the 238 Rollo, p. 1764.
RPC, carried with it the accessory penalty of perpetual absolute disqualification 239 Id., at p. 1735.
and in turn, pursuant to Article 30 of the RPC, disqualified him to run for elective 240 Id., at p. 1748.
office. As discussed, Section 40(a) of the LGC would not apply to cases wherein a 479
penal provision — such as Article 41 in this case — directly and specifically
VOL. 747, JANUARY 21, 2015 479
prohibits the convict from running for elective office. Hence, despite the lapse of
two (2) years from petitioner’s service of his commuted prison term, he Risos-Vidal vs. Commission on Elections
remains bound to suffer the accessory penalty of perpetual absolute Estrada is very loosely invoking the concept of a “sovereign” as though a
disqualification which consequently, disqualifies him to run as mayor for plurality of votes is the sole determinant of the “sovereign will.”
Zamboanga City. In the first place, what is involved here is merely an election for a local elective
Notably, Article 41 of the RPC expressly states that one who is previously position. Certainly, the voters of a single local government unit ought not to be
convicted of a crime punishable by reclusion perpetua or reclusion equated with the “sovereign Filipino people.” So blithely is Estrada celebrating his
temporal continues to suffer the accessory penalty of perpetual absolute 349,770 votes, he seems to forget that Lim was not even too far off with 313,764
disqualification even though pardoned as to the principal penalty, unless the said votes.
accessory penalty shall have been expressly remitted in the pardon. In this case, Estrada celebrates the casting of votes in his favor as a seemingly indubitable
the same accessory penalty had not been expressly remitted in the Order of expression of the sovereign will in trusting him with elective public office. He
Commutation or by any subsequent pardon and as such, petitioner’s forgets that a mere three years prior, the voters, not just of the City of Manila, but
disqualification to run for elective of- of the entire Republic, repudiated him and rejected his attempt to once again
478 secure the Presidency. He placed a distant second, behind by more than 5.72
478 SUPREME COURT REPORTS ANNOTATED million votes, to President Benigno Simeon Aquino III.
Estrada did secure more votes than Lim, that much can be conceded; but these
Risos-Vidal vs. Commission on Elections votes were cast in favor of an ineligible candidate, i.e., one who was no candidate
fice is deemed to subsist.237 (Emphasis supplied, citations omitted) at all.
The matter of eligibility relates to circumstances personally pertaining to a
Similarly, in this case, it is of no consequence that, by the time Estrada filed candidate, e.g., citizenship, residency, age, lack of a prior conviction, and literacy.
his candidacy and sought election as Mayor of the City of Manila, more than (2) No amount of votes can cure a candidate’s ineligibility. It could not, for instance,
years had lapsed since he was released from incarceration following President turn a 34-year-old person who filed a certificate of candidacy for Senator into a 35-
Gloria Macapagal-Arroyo’s grant, and his acceptance, of pardon. year-old and suddenly qualify that person for election as a Senator. The matter of
In sum, Estrada was disqualified to run for Mayor of the City of Manila in the qualification is entirely beyond the mere plurality of votes.
May 13, 2013 elections. Moreover, his perpetual absolute disqualification not In the context of constitutional democracy, the sovereign will is as effectively
having been remitted, and his rights to vote and be voted for elective public office expressed in the official acts of public institutions. The Filipino people speak as
not having been restored, Estrada remains bound to suffer the effects of the much through the laws enacted by their elected representatives as they do through
penalty of perpetual absolute disqualification, as listed in Article 30 of the Revised the ballot. Among these laws are those which prescribe the qualifications for elective
Penal Code. Specifically, he remains disqualified from exercising the right to vote public offices. Thus, by these requirements, the sovereign Filipino people delimit
in any election for any popular elective office, and he remains barred from those
occupying any public office, elective, or otherwise. 480
480 SUPREME COURT REPORTS ANNOTATED
XIII
On the supposed Risos-Vidal vs. Commission on Elections
disenfranchisement of voters and who may be elected to public office. Among these, too, is the Revised Penal Code,
disregard of the sovereign will Articles 36 and 41 of which require the express restoration of the rights of suffrage
and to hold public office, or otherwise the express remission of the penalty of
perpetual absolute disqualification. So too, the Filipino people speak through the
Constitution they have adopted, a basic precept of which is that public office is a produced by declaring a person ineligible to hold such an office. In the former case
public trust. Thus, matters relating to public office cannot be expediently dispensed the court, after an examination of the ballots may find that some other person than
with through the private act of granting pardon unless such grant be in compliance the candidate declared to have
with legally established requisites. _______________
The plurality of voters in Manila may appear to have decided contrary to what
is expressed in our laws, but this cannot trump the sovereign will as expressed in 243 Id., at p. 240.
our Constitution and laws. 244 Maquiling v. COMELEC, supra note 241 at p. 443.
“[T]he COMELEC First Division and the COMELEC En Banc correctly treated
XIV the petition as one for disqualification.”
Petitioner-intervenor Alfredo S. 245 Id., at p. 464. “[Arnado] was a dual citizen disqualified to run for public
Lim is the qualified candidate who office based on Section 40(d) of the Local Government Code.”
obtained the highest number of 482
votes in the election for Mayor of
482 SUPREME COURT REPORTS ANNOTATED
the City of Manila
Risos-Vidal vs. Commission on Elections
Having settled that Estrada suffered and continues to suffer from perpetual received a plura[l]ity by the board of canvassers actually received the greater
absolute disqualification, it is proper to resolve the resultant issue of who must be number of votes, in which case the court issues its mandamus to the board of
named Mayor of the City of Manila in lieu of Estrada. canvassers to correct the returns accordingly; or it may find that the manner of
In this court’s April 16, 2013 decision in Maquiling v. COMELEC,241 we holding the election and the returns are so tainted with fraud or illegality that it
revisited the 1912 case of Topacio v. Paredes242from which originated the often- cannot be determined who received a [plurality] of the legally cast ballots. In the
quoted phrase “the wreath of victory cannot be transferred from an ineligible latter case, no question as to the correctness of the returns or the manner of casting
candidate to any other candidate when the sole question is and counting the ballots is before the deciding power, and generally the only result
_______________ can be that the election fails entirely. In the former, we have a contest in the strict
sense of the word, because of the opposing parties are striving for supremacy. If it
241 G.R. No. 195649, April 16, 2013, 696 SCRA 420 [Per CJ. Sereno, En be found that the successful candidate (according to the board of canvassers)
Banc]. obtained a plurality in an illegal manner, and that another candidate was the real
242 23 Phil. 238 (1912) [Per J. Trent, En Banc]. victor, the former must retire in favor of the latter. In the other case, there is not,
481 strictly speaking, a contest, as the wreath of victory cannot be transferred from an
ineligible candidate to any other candidate when the sole question is the eligibility
VOL. 747, JANUARY 21, 2015 481
of the one receiving a plurality of the legally cast ballots. In the one case the
Risos-Vidal vs. Commission on Elections question is as to who received a plurality of the legally cast ballots; in the other,
the eligibility of the one receiving a plurality of the legally cast ballots.”243 This the question is confined to the personal character and circumstances of a single
was the progenitor of the principle that a supposed second placer cannot be individual.
proclaimed the winner in an election contest. Note that the sentence where the phrase is found starts with “In the other case,
As in the present case, Maquiling involved a petition for there is not, strictly speaking, a contest” in contrast to the earlier statement, “In
disqualification244 anchored on Section 40 of the Local Government Code.245 Thus, the former, we have a contest in the strict sense of the word, because of the
the principles laid down by Maquiling as to who must occupy an elective position opposing parties are striving for supremacy.”
following the determination that a candidate was disqualified are squarely The Court in Topacio v. Paredes cannot be said to have held that “the wreath
applicable in this case. of victory cannot be trans-
As explained in Maquiling, the ‘often-quoted phrase’ from Topacio was a 483
mere obiter dictum: VOL. 747, JANUARY 21, 2015 483
This phrase is not even the ratio decidendi; it is a mere obiter dictum. The
Court was comparing “the effect of a decision that a candidate is not entitled to the Risos-Vidal vs. Commission on Elections
office because of fraud or irregularities in the elections x x x [with] that produced ferred from an ineligible candidate to any other candidate when the sole
by declaring a person ineligible to hold such an office.” question is the eligibility of the one receiving a plurality of the legally cast ballots.”
The complete sentence where the phrase is found is part of a comparison and A proper reading of the case reveals that the ruling therein is that since the
contrast between the two situations, thus: Court of First Instance is without jurisdiction to try a disqualification case based
Again, the effect of a decision that a candidate is not entitled to the office on the eligibility of the person who obtained the highest number of votes in the
because of fraud or irregularities in the elections is quite different from that election, its jurisdiction being confined “to determine which of the contestants has
been duly elected” the judge exceeded his jurisdiction when he “declared that no
one had been legally elected president of the municipality of Imus at the general This court’s 2012 decisions in Aratea v. COMELEC252 and Dominador Jalosjos,
election held in that town on 4 June 1912” where “the only question raised was Jr. v. COMELEC253 ruled that a certifi-
whether or not Topacio was eligible to be elected and to hold the office of municipal _______________
president.”
The Court did not rule that Topacio was disqualified and that Abad as the 249 Id., at p. 459.
second placer cannot be proclaimed in his stead. . . .246 (Citations omitted) 250 G.R. No. 207900, April 22, 2014, 723 SCRA 223 [Per J. Peralta, En Banc].
251 Id., citing Maquiling v. COMELEC, supra note 241 at pp. 456-457.
By definition, an ineligible individual is not even a candidate in the first 252 Supra note 74.
place.247 It is, therefore, erroneous to refer to him or her as a “winner,” that is, as 485
the “winning candidate,” should he or she obtain the plurality of votes.
VOL. 747, JANUARY 21, 2015 485
Consequently, it is illogical to refer to the candidates who are trailing in the vote
count as “losers,” which is what labels like “second-placer” entail. As his or her Risos-Vidal vs. Commission on Elections
ineligibility as a candidate remains, the number of votes cast for him or her is cate of candidacy that was cancelled for being void ab initio, it having been
ultimately not decisive of who must be proclaimed as winner:248 filed by a candidate who falsely claimed that he was eligible, produces no effect, it
The ballot cannot override the constitutional and statutory requirements for “cannot give rise to a valid candidacy, and much less to valid votes.”254 Thus, the
qualifications and disqualifications of candidates. When the law requires certain votes cast for the ineligible candidate should be considered “stray votes and should
qualifications to be possessed or that certain disqualifications be not possessed by not be counted.”255
persons desiring to serve as This court’s June 25, 2013 resolution in Svetlana Jalosjos v.
_______________ COMELEC256 expounded on the reasons for enabling the qualifiedcandidate (the
erstwhile “second placer, unless of course, he is himself ineligible) who obtained
246 Id., at pp. 456-457. the highest number of votes to assume the contested office. It has also clarified the
247 Id., at p. 458. proper operation of Section 44 of the Local Government Code on the rules on
248 Id. succession in case of a permanent vacancy in the Office of the Mayor:
484 There is another more compelling reason why the eligible candidate who
garnered the highest number of votes must assume the office. The ineligible
484 SUPREME COURT REPORTS ANNOTATED
candidate who was proclaimed and who already assumed office is a de factoofficer
Risos-Vidal vs. Commission on Elections by virtue of the ineligibility.
The rule on succession in Section 44 of the Local Government Code cannot
elective public officials, those qualifications must be met before one even apply in instances when a de facto officer is ousted from office and the de
becomes a candidate. When a person who is not qualified is voted for and jure officer takes over. The ouster of a de facto officer cannot create a permanent
eventually garners the highest number of votes, even the will of the electorate vacancy as contemplated in the Local Government Code. There is no vacancy to
expressed through the ballot cannot cure the defect in the qualifications of the speak of as the de jure officer, the rightful winner in the elections, has the legal
candidate. To rule otherwise is to trample upon and rent asunder the very law that right to assume the position.257
sets forth the qualifications and disqualifications of candidates. We might as well
write off our election laws if the voice of the electorate is the sole determinant of Dominador Jalosjos, Jr. has not only ruled that the votes for an ineligible
who should be proclaimed worthy to occupy elective positions in our republic.249 candidate are stray votes. It has also im-
_______________
To rule as such is not tantamount to disrespecting the will of the electorate. As
was very recently said in Hayudini v. COMELEC:250 253 Supra note 40.
[T]he will of the electorate is still actually respected even when the votes for 254 Aratea v. COMELEC, supra note 74 at p. 145.
the ineligible candidate are disregarded. The votes cast in favor of the ineligible 255 Jalosjos, Jr. v. COMELEC, supra note 40.
candidate are not considered at all in determining the winner of an election for 256 G.R. No. 193314, June 25, 2013, 699 SCRA 507 [Per CJ. Sereno, En Banc].
these do not constitute the sole and total expression of the sovereign voice. On the 257 Id., at pp. 519-520.
other hand, those votes for the eligible and legitimate candidates form an integral 486
part of said voice, which must equally be given due respect, if not more.251
486 SUPREME COURT REPORTS ANNOTATED
Contemporary jurisprudence has seen the repudiation of the position that a Risos-Vidal vs. Commission on Elections
“second placer” cannot be proclaimed a winner in lieu of an ineligible candidate. pressed upon the COMELEC that it is duty-bound to “motu proprio bar from
running for public office those suffering from perpetual special disqualification by
virtue of a final judgment.”258
Even without a petition under either Section 12 or Section 78 of the Omnibus way into the halls of the House of Representatives and the Senate in a historic
Election Code, or under Section 40 of the Local Government Code, the COMELEC impeachment proceeding. Events unravelled, which caused the offending
is under a legal duty to cancel the certificate of candidacy of anyone suffering from President to vacate Malacañan, to be considered resigned, and to finally be
the accessory penalty of perpetual special disqualification to run for public office replaced.
by virtue of a final judgment of conviction. The final judgment of conviction is His prosecution subsequently ensued. A first in our history,
notice to the COMELEC of the disqualification of the convict from running for the Sandiganbayan found him guilty of committing the highest possible crime
public office. The law itself bars the convict from running for public office, and the attended by graft and corruption. This betrayal of the public trust is called plunder.
disqualification is part of the final judgment of conviction. The final judgment of It is statutorily punished by a penalty of reclusion perpetua and permanent
the court is addressed not only to the Executive branch, but also to other disqualification from public office.
government agencies tasked to implement the final judgment under the law. _______________
Whether or not the COMELEC is expressly mentioned in the judgment to
implement the disqualification, it is assumed that the portion of the final judgment 259 Id., at pp. 23-24, citing Const. (1987), Art. IX-C, Sec. 2(1).
on disqualification to run for elective public office is addressed to the COMELEC 488
because under the Constitution the COMELEC is duty bound to “[e]nforce and
488 SUPREME COURT REPORTS ANNOTATED
administer all laws and regulations relative to the conduct of an election.” The
disqualification of a convict to run for public office under the Revised Penal Code, Risos-Vidal vs. Commission on Elections
as affirmed by final judgment of a competent court, is part of the enforcement and The person convicted of plunder now walks free among us. He did not spend a
administration of “all laws” relating to the conduct of elections. single day in an ordinary jail. There is no question that he was pardoned. Today,
To allow the COMELEC to wait for a person to file a petition to cancel the the majority completes the circle by reading an ambiguous pardon allowing him
certificate of candidacy of one suffering from perpetual special disqualification will yet again to run for public office. The majority uses the equivocal silence of the
result in the anomaly that these cases so grotesquely exemplify. Despite a prior succeeding President who devised the ambiguous pardon as one of the bases to say
perpetual special disqualification, Jalos- that the convicted former President can again seek public office.
_______________ This is template for our political elite at the expense of the masses who toil and
suffer from the consequences of corruption. It is hope for those who occupy high
258 Jalosjos, Jr. v. COMELEC, supra note 40 at p. 24. government offices who commit crimes as they await a next political term when
487 the people’s vigilance would have waned. It is the denouement in a narrative that
will explain why there is no effective deterrent to corruption in high places. The
VOL. 747, JANUARY 21, 2015 487
pragmatism of politics takes over the highest notion that public office should be of
Risos-Vidal vs. Commission on Elections effective public trust. The rule of law should unravel to meet this expectation.
jos was elected and served twice as mayor. The COMELEC will be grossly The pardon was ambiguous. By our laws and constitutional fiat, it should have
remiss in its constitutional duty to “enforce and administer all laws” relating to been read as perpetually prohibiting he who was convicted of plunder from again
the conduct of elections if it does not motu proprio bar from running for public occupying any public office. This is my reading of what the values in our laws
office those suffering from perpetual special disqualification by virtue of a final require.
judgment.259 I do not judge respondent for who he is as a person. That is not within our
constitutional competence. But as a leader, the respondent will best show that the
Applying these principles, the votes cast for private respondent Joseph Ejercito way forward for the country he loves should be for him to repent and for him to
Estrada, a disqualified and ineligible candidate, must be held as stray votes. suffer courageously the consequences of his past acts. There are things which are
Petitioner-intervenor Alfredo S. Lim is the qualified candidate who obtained clearly right. There are things which are clearly wrong. For in our hearts we know
the highest number of votes in the contest to be elected Mayor of the City of Manila that impunity, in any form, should be abhorred especially when it gives advantage
in the May 13, 2013 elections. Accordingly, he must be proclaimed the duly elected to the privileged and the powerful.
Mayor of the City of Manila, lest there be grounds, not contemplated in this Thus, I dissent.
opinion, barring his proclamation. ACCORDINGLY, contrary to the majority, I vote to GRANT the petition and
the petition-in-intervention. The
Final note 489
VOL. 747, JANUARY 21, 2015 489
Not so long ago, our people were moved by revelations of wrongdoing
committed by one who temporarily occupied one of the most important public Risos-Vidal vs. Commission on Elections
offices of our society — the Presidency. Our people’s collective voices uttered in assailed resolutions dated April 1, 2013 of the Second Division of public
private conversations avalanched into a people’s movement. This voice found its respondent Commission on Elections (COMELEC), and April 23, 2013 of public
respondent COMELEC, sitting En Banc, must be ANNULLED and SET ASIDE.
Private respondent Joseph Ejercito Estrada continues to suffer the penalty of convened under the leadership of respondent Abdusakur Mahail Tan, the Provincial
perpetual absolute disqualification and is thereby DISQUALIFIED from Governor of Sulu. Its armed forces component was headed by respondents General
exercising the right to vote in any election for any popular elective office or to be Juancho Saban, and his deputy, Colonel Eugenio Clemen. The PNP component was
elected to such office. headed by respondent Police Superintendent Bienvenido G. Latag, the Police Deputy
Petitions dismissed, Resolution of Commission on Elections, Second Division Director for Operations of the Autonomous Region of Muslim Mindanao (ARMM). 4
dated April 1, 2013 and Resolution of Commission on Elections En Banc dated
April 23, 2013 affirmed. Governor Tan organized the Civilian Emergency Force (CEF), a group of armed male
Notes.—Commutation of sentence is a prerogative of the Chief Executive — civilians coming from different municipalities, who were redeployed to surrounding
the recommendation of the Bureau of Pardons and Parole is just a mere areas of Patikul.5 The organization of the CEF was embodied in a "Memorandum of
recommendation, and until and unless approved by the President, there is no Understanding"6 entered into
commutation to speak of. (Barredo vs. Vinarao, 529 SCRA 120 [2007])
A forfeiture case under Republic Act No. 1379 arises out of a cause of action between three parties: the provincial government of Sulu, represented by Governor
separate and different from a plunder case, thus negating the notion that the crime Tan; the Armed Forces of the Philippines, represented by Gen. Saban; and the
of plunder absorbs the forfeiture case. (Garcia vs. Sandiganbayan, 603 SCRA 348 Philippine National Police, represented by P/SUPT. Latag. The Whereas clauses of
[2009]) the Memorandum alluded to the extraordinary situation in Sulu, and the willingness of
——o0o—— civilian supporters of the municipal mayors to offer their services in order that "the early
and safe rescue of the hostages may be achieved."7

This Memorandum, which was labeled ‘secret’ on its all pages, also outlined the
responsibilities of each of the party signatories, as follows:

G.R. No. 187298 July 03, 2012 Responsibilities of the Provincial Government:

JAMAR M. KULAYAN, TEMOGEN S. TULAWIE, HJI. MOH. YUSOP ISMI, 1) The Provincial Government shall source the funds and logistics needed for
JULHAJAN AWADI, and SPO1 SATTAL H. JADJULI, Petitioners, the activation of the CEF;
vs.
GOV. ABDUSAKUR M. TAN, in his capacity as Governor of Sulu; GEN. JUANCHO
SABAN, COL. EUGENIO CLEMEN PN, P/SUPT. JULASIRIM KASIM and P/SUPT. 2) The Provincial Government shall identify the Local Government Units
BIENVENIDO G. LATAG, in their capacity as officers of the Phil. Marines and which shall participate in the operations and to propose them for the approval
Phil. National Police, respectively, Respondents. of the parties to this agreement;

DECISION 3) The Provincial Government shall ensure that there will be no unilateral
action(s) by the CEF without the knowledge and approval by both parties.
SERENO, J.:
Responsibilities of AFP/PNP/ TF ICRC (Task Force ICRC):
On 15 January 2009, three members from the International Committee of the Red
Cross (ICRC) were kidnapped in the vicinity of the Provincial Capitol in Patikul, 1) The AFP/PNP shall remain the authority as prescribed by law in military
Sulu.1 Andres Notter, a Swiss national and head of the ICRC in Zamboanga City, operations and law enforcement;
Eugenio Vagni, an Italian national and ICRC delegate, and Marie Jean Lacaba, a
Filipino engineer, were purportedly inspecting a water and sanitation project for the 2) The AFP/PNP shall ensure the orderly deployment of the CEF in the
Sulu Provincial Jail when inspecting a water and sanitation project for the Sulu performance of their assigned task(s);
Provincial Jail when they were seized by three armed men who were later confirmed
to be members of the Abu Sayyaf Group (ASG).2 The leader of the alleged kidnappers 3) The AFP/PNP shall ensure the safe movements of the CEF in identified
was identified as Raden Abu, a former guard at the Sulu Provincial Jail. News reports areas of operation(s);
linked Abu to Albader Parad, one of the known leaders of the Abu Sayyaf.

4) The AFP/PNP shall provide the necessary support and/or assistance as


On 21 January 2009, a task force was created by the ICRC and the Philippine National called for in the course of operation(s)/movements of the CEF. 8
Police (PNP), which then organized a parallel local group known as the Local Crisis
Committee.3 The local group, later renamed Sulu Crisis Management Committee,
Meanwhile, Ronaldo Puno, then Secretary of the Department of Interior and Local 31STDAY OF MARCH 2009. Sgd. Abdusakur M. Tan Governor.14
Government, announced to the media that government troops had cornered some one
hundred and twenty (120) Abu Sayyaf members along with the three (3) On 1 April 2009, SPO1 Sattal Jadjuli was instructed by his superior to report to
hostages.9 However, the ASG made respondent P/SUPT. Julasirim Kasim.15 Upon arriving at the police station, he was
booked, and interviewed about his relationship to Musin, Jaiton, and Julamin, who
contact with the authorities and demanded that the military pull its troops back from were all his deceased relatives. Upon admitting that he was indeed related to the three,
the jungle area.10 The government troops yielded and went back to their barracks; the he was detained. After a few hours, former Punong Barangay Juljahan Awadi, Hadji
Philippine Marines withdrew to their camp, while police and civilian forces pulled back Hadjirul Bambra, Abdugajir Hadjirul, as well as PO2 Marcial Hajan, SPO3 Muhilmi
from the terrorists’ stronghold by ten (10) to fifteen (15) kilometers. Threatening that Ismula, Punong Barangay Alano Mohammad and jeepney driver Abduhadi Sabdani,
one of the hostages will be beheaded, the ASG further demanded the evacuation of were also arrested.16 The affidavit17 of the apprehending officer alleged that they were
the military camps and bases in the different barangays in Jolo. 11 The authorities were suspected ASG supporters and were being arrested under Proclamation 1-09. The
given no later than 2:00 o’clock in the afternoon of 31 March 2009 to comply.12 following day, 2 April 2009, the hostage Mary Jane Lacaba was released by the ASG.

On 31 March 2009, Governor Tan issued Proclamation No. 1, Series of 2009 On 4 April 2009, the office of Governor Tan distributed to civic organizations, copies of
(Proclamation 1-09), declaring a state of emergency in the province of Sulu.13 It cited the "Guidelines for the Implementation of Proclamation No. 1, Series of 2009 Declaring
the kidnapping incident as a ground for the said declaration, describing it as a terrorist a State of Emergency in the Province of Sulu." 18These Guidelines suspended all
act pursuant to the Human Security Permits to Carry

Act (R.A. 9372). It also invoked Section 465 of the Local Government Code of 1991 Firearms Outside of Residence (PTCFORs) issued by the Chief of the PNP, and
(R.A. 7160), which bestows on the Provincial Governor the power to carry out allowed civilians to seek exemption from the gun ban only by applying to the Office of
emergency measures during man-made and natural disasters and calamities, and to the Governor and obtaining the appropriate identification cards. The said guidelines
call upon the appropriate national law enforcement agencies to suppress disorder and also allowed general searches and seizures in designated checkpoints and
lawless violence. chokepoints.

In the same Proclamation, respondent Tan called upon the PNP and the CEF to set On 16 April 2009, Jamar M. Kulayan, Temogen S. Tulawie, Hadji Mohammad Yusop
up checkpoints and chokepoints, conduct general search and seizures including Ismi, Ahajan Awadi, and SPO1 Sattal H. Jadjuli, residents of Patikul, Sulu, filed the
arrests, and other actions necessary to ensure public safety. The pertinent portion of present Petition for Certiorari and Prohibition,19 claiming that Proclamation 1-09 was
the proclamation states: issued with grave abuse of discretion amounting to lack or excess of jurisdiction, as it
threatened fundamental freedoms guaranteed under Article III of the 1987
NOW, THEREFORE, BY VIRTUE OF THE POWERS VESTED IN ME BY LAW, I, Constitution.
ABDUSAKUR MAHAIL TAN, GOVERNOR OF THE PROVINCE OF SULU, DO
HEREBY DECLARE A STATE OF EMERGENCY IN THE PROVINCE OF SULU, AND Petitioners contend that Proclamation No. 1 and its Implementing Guidelines were
CALL ON THE PHILIPPINE NATIONAL POLICE WITH THE ASSISTANCE OF THE issued ultra vires, and thus null and void, for violating Sections 1 and 18, Article VII of
ARMED FORCES OF THE PHILIPPINES AND THE CIVILIAN EMERGENCY FORCE the Constitution, which grants the President sole authority to exercise emergency
TO IMPLEMENT THE FOLLOWING: powers and calling-out powers as the chief executive of the Republic and commander-
in-chief of the armed forces.20 Additionally, petitioners claim that the Provincial
1. The setting-up of checkpoints and chokepoints in the province; Governor is not authorized by any law to create civilian armed forces under his
command, nor regulate and limit the issuances of PTCFORs to his own private army.
2. The imposition of curfew for the entire province subject to such Guidelines
as may be issued by proper authorities; In his Comment, Governor Tan contended that petitioners violated the doctrine on
hierarchy of courts when they filed the instant petition directly in the court of last resort,
even if both the Court of Appeals (CA) and the Regional Trial Courts (RTC) possessed
3. The conduct of General Search and Seizure including arrests in the pursuit concurrent jurisdiction with the
of the kidnappers and their supporters; and
Supreme Court under Rule 65.21 This is the only procedural defense raised by
4. To conduct such other actions or police operations as may be necessary respondent Tan. Respondents Gen. Juancho Saban, Col. Eugenio Clemen, P/SUPT.
to ensure public safety. Julasirim Kasim, and P/SUPT. Bienvenido Latag did not file their respective
Comments.1âwphi1
DONE AT THE PROVINCIAL CAPITOL, PROVINCE OF SULU THIS
On the substantive issues, respondents deny that Proclamation 1-09 was issued ultra of a public official which pertain to restrictive custody, and is thus impressed with
vires, as Governor Tan allegedly acted pursuant to Sections 16 and 465 of the Local transcendental public importance that would warrant the relaxation of the general rule.
Government Code, which empowers the Provincial Governor to carry out emergency The Court would be remiss in its constitutional duties were it to dismiss the present
measures during calamities and disasters, and to call upon the appropriate national petition solely due to claims of judicial hierarchy.
law enforcement agencies to suppress disorder, riot, lawless violence, rebellion or
sedition.22 Furthermore, the Sangguniang Panlalawigan of Sulu authorized the In David v. Macapagal-Arroyo,29 the Court highlighted the transcendental public
declaration of a state of emergency as evidenced by Resolution No. 4, Series of 2009 importance involved in cases that concern restrictive custody, because judicial review
issued on 31 March 2009 during its regular session.23 in these cases serves as "a manifestation of the crucial defense of civilians ‘in police
power’ cases due to the diminution of their basic liberties under the guise of a state of
The threshold issue in the present case is whether or not Section 465, in relation to emergency."30 Otherwise, the importance of the high tribunal as the court of last resort
Section 16, of the Local Government Code authorizes the respondent governor to would be put to naught, considering the nature of "emergency" cases, wherein the
declare a state of emergency, and exercise the powers enumerated under proclamations and issuances are inherently short-lived. In finally disposing of the claim
Proclamation 1-09, specifically the conduct of general searches and seizures. that the issue had become moot and academic, the Court also cited transcendental
Subsumed herein is the secondary question of whether or not the provincial governor public importance as an exception, stating:
is similarly clothed with authority to convene the CEF under the said provisions.
Sa kabila ng pagiging akademiko na lamang ng mga isyu tungkol sa mahigpit na
We grant the petition. pangangalaga (restrictive custody) at pagmonitor ng galaw (monitoring of movements)
ng nagpepetisyon, dedesisyunan namin ito (a) dahil sa nangingibabaw na interes ng
I. Transcendental public Importance warrants a relaxation of the Doctrine of Hierarchy madla na nakapaloob dito,
of Courts
(b) dahil sa posibilidad na maaaring maulit ang pangyayari at (c) dahil kailangang
We first dispose of respondents’ invocation of the doctrine of hierarchy of courts which maturuan ang kapulisan tungkol dito.
allegedly prevents judicial review by this Court in the present case, citing for this
specific purpose, Montes v. Court of Appeals and Purok Bagong Silang Association, The moot and academic principle is not a magical formula that can automatically
Inc. v. Yuipco.24 Simply put, the dissuade the courts in resolving a case. Courts will decide cases, otherwise moot and
academic, if: first, there is a grave violation of the Constitution; second, the exceptional
doctrine provides that where the issuance of an extraordinary writ is also within the character of the situation and the paramount public interest is involved; third, when
competence of the CA or the RTC, it is in either of these courts and not in the Supreme [the] constitutional issue raised requires formulation of controlling principles to guide
Court, that the specific action for the issuance of such writ must be sought unless the bench, the bar, and the public; and fourth, the case is capable of repetition yet
special and important laws are clearly and specifically set forth in the petition. The evading review.
reason for this is that this Court is a court of last resort and must so remain if it is to
perform the functions assigned to it by the Constitution and immemorial tradition. It …There is no question that the issues being raised affect the public interest, involving
cannot be burdened with deciding cases in the first instance. 25 as they do the people’s basic rights to freedom of expression, of assembly and of the
press. Moreover, the
The said rule, however, is not without exception. In Chavez v. PEA-Amari,26 the Court
stated: Court has the duty to formulate guiding and controlling constitutional precepts,
doctrines or rules. It has the symbolic function of educating the bench and the bar, and
PEA and AMARI claim petitioner ignored the judicial hierarchy by seeking relief directly in the present petitions, the military and the police, on the extent of the protection given
from the Court. The principle of hierarchy of courts applies generally to cases involving by constitutional guarantees. And lastly, respondents contested actions are capable of
factual questions. As it is not a trier of facts, the Court cannot entertain cases involving repetition. Certainly, the petitions are subject to judicial review.
factual issues. The instant case, however, raises constitutional questions of
transcendental importance to the public. The Court can resolve this case without Evidently, the triple reasons We advanced at the start of Our ruling are justified under
determining any factual issue related to the case. Also, the instant case is a petition the foregoing exceptions. Every bad, unusual incident where police officers figure in
for mandamus which falls under the original jurisdiction of the Court under Section 5, generates public interest and people watch what will be done or not done to them. Lack
Article VIII of the Constitution. We resolve to exercise primary jurisdiction over the of disciplinary steps taken against them erode public confidence in the police
instant case.27 institution. As petitioners themselves assert, the restrictive custody of policemen under
investigation is an existing practice, hence, the issue is bound to crop up every now
The instant case stems from a petition for certiorari and prohibition, over which the and then. The matter is capable of repetition or susceptible of recurrence. It better be
Supreme Court possesses original jurisdiction.28 More crucially, this case involves acts resolved now for the education and guidance of all concerned. 31 (Emphasis supplied)
Hence, the instant petition is given due course, impressed as it is with transcendental Philippines or any part thereof under martial law. Within forty-eight hours from the
public importance. proclamation of martial law or the suspension of the privilege of the writ of habeas
corpus, the President shall submit a report in person or in writing to the Congress. The
II. Only the President is vested with calling-out powers, as the commander-in-chief of Congress, voting jointly, by a vote of at least a majority of all its Members in regular or
the Republic special session, may revoke such proclamation or suspension, which revocation shall
not be set aside by the President. Upon the initiative of the President, the Congress
may, in the same manner, extend such proclamation or suspension for a period to be
i. One executive, one commander-in-chief determined by the Congress, if the invasion or rebellion shall persist and public safety
requires it.
As early as Villena v. Secretary of Interior,32 it has already been established that there
is one repository of executive powers, and that is the President of the Republic. This The Congress, if not in session, shall, within twenty-four hours following such
means that when Section 1, Article VII of the Constitution speaks of executive power, proclamation or suspension, convene in accordance with its rules without need of a
it is granted to the President and no one else. 33 As emphasized by Justice Jose P. call.36
Laurel, in his ponencia in Villena:
The power to declare a state of martial law is subject to the Supreme Court’s authority
With reference to the Executive Department of the government, there is one purpose to review the factual basis thereof. 37 By constitutional fiat, the calling-out powers,
which is crystal-clear and is readily visible without the projection of judicial searchlight, which is of lesser gravity than the power to declare martial law, is bestowed upon the
and that is the establishment of a single, not plural, Executive. The first section of President alone. As noted in Villena, "(t)here are certain constitutional powers and
Article VII of the Constitution, dealing with the Executive Department, begins with the prerogatives of the Chief Executive of the Nation which must be exercised by him in
enunciation of the principle that "The executive power shall be vested in a President person and no amount of approval or ratification will validate the exercise of any of
of the Philippines." This means that the President of the Philippines is the Executive of those powers by any other person. Such, for instance, is his power to suspend the writ
the Government of the Philippines, and no other.34 of habeas corpus and proclaim martial law x x x.38

Corollarily, it is only the President, as Executive, who is authorized to exercise Indeed, while the President is still a civilian, Article II, Section 339 of the Constitution
emergency powers as provided under Section 23, Article VI, of the Constitution, as mandates that civilian authority is, at all times, supreme over the military, making the
well as what became known as the calling-out powers under Section 7, Article VII civilian president the nation’s supreme military leader. The net effect of Article II,
thereof. Section 3, when read with Article VII,

ii. The exceptional character of Commander-in-Chief powers dictate that they are Section 18, is that a civilian President is the ceremonial, legal and administrative head
exercised by one president of the armed forces. The Constitution does not require that the President must be
possessed of military training and talents, but as Commander-in-Chief, he has the
Springing from the well-entrenched constitutional precept of One President is the power to direct military operations and to determine military strategy. Normally, he
notion that there are certain acts which, by their very nature, may only be performed would be expected to delegate the actual command of the armed forces to military
by the president as the Head of the State. One of these acts or prerogatives is the experts; but the ultimate power is his.40 As Commander-in-Chief, he is authorized to
bundle of Commander-in-Chief powers to which the "calling-out" powers constitutes a direct the movements of the naval and military forces placed by law at his command,
portion. The President’s Emergency Powers, on the other hand, is balanced only by and to employ them in the manner he may deem most effectual. 41
the legislative act of Congress, as embodied in the second paragraph of Section 23,
Article 6 of the Constitution: In the case of Integrated Bar of the Philippines v. Zamora,42 the Court had occasion to
rule that the calling-out powers belong solely to the President as commander-in-chief:
Article 6, Sec 23(2). In times of war or other national emergency, the Congress may,
by law, authorize the President, for a limited period and subject to such restrictions as When the President calls the armed forces to prevent or suppress lawless violence,
it may prescribe, to exercise powers necessary and proper to carry out a declared invasion or rebellion, he necessarily exercises a discretionary power solely vested in
national policy. Unless sooner withdrawn by resolution of the Congress, such powers his wisdom. This is clear from the intent of the framers and from the text of the
shall cease upon the next adjournment thereof.35 Constitution itself. The Court, thus, cannot be called upon to overrule the President’s
wisdom or substitute its own. However, this does not prevent an examination of
Article 7, Sec 18. The President shall be the Commander-in-Chief of all armed forces whether such power was exercised within permissible constitutional limits or whether
of the Philippines and whenever it becomes necessary, he may call out such armed it was exercised in a manner constituting grave abuse of discretion. In view of the
forces to prevent or suppress lawless violence, invasion or rebellion. In case of constitutional intent to give the President full discretionary power to determine the
invasion or rebellion, when the public safety requires it, he may, for a period not necessity of calling out the armed forces, it is incumbent upon the petitioner to show
exceeding sixty days, suspend the privilege of the writ of habeas corpus or place the that the President’s decision is totally bereft of factual basis.
There is a clear textual commitment under the Constitution to bestow on the President These distinctions hold true to this day as they remain embodied in our fundamental
full discretionary power to call out the armed forces and to determine the necessity for law. There are certain presidential powers which arise out of exceptional
the exercise of such power.43(Emphasis supplied) circumstances, and if exercised, would involve the suspension of fundamental
freedoms, or at least call for the supersedence of executive prerogatives over those
Under the foregoing provisions, Congress may revoke such proclamation or exercised by co-equal branches of government. The declaration of martial law, the
suspension and the Court may review the sufficiency of the factual basis thereof. suspension of the writ of habeas corpus, and the exercise of the pardoning power,
However, there is no such equivalent provision dealing with the revocation or review notwithstanding the judicial determination of guilt of the accused, all fall within this
of the President’s action to call out the armed forces. The distinction places the calling special class that demands the exclusive exercise by the President of the
out power in a different category from the power to declare martial law and the power constitutionally vested power. The list is by no means exclusive, but there must be a
to suspend the privilege of the writ of habeas corpus, otherwise, the framers of the showing that the executive power in question is of similar gravitas and exceptional
Constitution would have simply lumped together the three powers and provided for import.47
their revocation and review without any qualification.44
In addition to being the commander-in-chief of the armed forces, the President also
That the power to call upon the armed forces is discretionary on the president is clear acts as the leader of the country’s police forces, under the mandate of Section 17,
from the deliberation of the Constitutional Commission: Article VII of the Constitution, which provides that, "The President shall have control of
all the executive departments, bureaus, and offices. He shall ensure that the laws be
faithfully executed." During the deliberations of the Constitutional Commission on the
FR. BERNAS. It will not make any difference. I may add that there is a graduated framing of this provision, Fr. Bernas defended the retention of the word "control,"
power of the President as Commander-in-Chief. First, he can call out such Armed employing the same rationale of singularity of the office of the president, as the only
Forces as may be necessary to suppress lawless violence; then he can suspend the Executive under the presidential form of government. 48
privilege of the writ of habeas corpus, then he can impose martial law. This is a
graduated sequence.
Regarding the country’s police force, Section 6, Article XVI of the Constitution states
that: "The State shall establish and maintain one police force, which shall be national
When he judges that it is necessary to impose martial law or suspend the privilege of in scope and civilian in character, to be administered and controlled by a national police
the writ of habeas corpus, his judgment is subject to review. We are making it subject commission. The authority of local executives over the police units in their jurisdiction
to review by the Supreme Court and subject to concurrence by the National Assembly. shall be provided by law."49
But when he exercises this lesser power of calling on the Armed Forces, when he says
it is necessary, it is my opinion that his judgment cannot be reviewed by anybody.
A local chief executive, such as the provincial governor, exercises operational
supervision over the police,50 and may exercise control only in day-to-day
xxx xxx xxx operations, viz:

MR. REGALADO. That does not require any concurrence by the legislature nor is it Mr. Natividad: By experience, it is not advisable to provide either in our Constitution or
subject to judicial review. by law full control of the police by the local chief executive and local executives, the
mayors. By our experience, this has spawned warlordism, bossism and sanctuaries
The reason for the difference in the treatment of the aforementioned powers highlights for vices and abuses. If the national government does not have a mechanism to
the intent to grant the President the widest leeway and broadest discretion in using the supervise these 1,500 legally, technically separate police forces, plus 61 city police
power to call out because it is considered as the lesser and more benign power forces, fragmented police system, we will have a lot of difficulty in presenting a modern
compared to the power to suspend the privilege of the writ of habeas corpus and the professional police force. So that a certain amount of supervision and control will have
power to impose martial law, both of which involve the curtailment and suppression of to be exercised by the national government.
certain basic civil rights and individual freedoms, and thus necessitating safeguards by
Congress and review by this Court. For example, if a local government, a town cannot handle its peace and order problems
or police problems, such as riots, conflagrations or organized crime, the national
x x x Thus, it is the unclouded intent of the Constitution to vest upon the President, as government may come in, especially if requested by the local executives. Under that
Commander-in-Chief of the Armed Forces, full discretion to call forth the military when situation, if they come in under such an extraordinary situation, they will be in control.
in his judgment it is necessary to do so in order to prevent or suppress lawless violence, But if the day-to-day business of police investigation of crime, crime prevention,
invasion or rebellion.45 (Emphasis Supplied) activities, traffic control, is all lodged in the mayors, and if they are in complete
operational control of the day-to-day business of police service, what the national
In the more recent case of Constantino, Jr. v. Cuisia,46 the Court characterized these government would control would be the administrative aspect.
powers as exclusive to the President, precisely because they are of exceptional import:
xxx xxx xxx
Mr. de los Reyes: so the operational control on a day-to-day basis, meaning, the usual Mr. Rodrigo: Control and supervision.
duties being performed by the ordinary policemen, will be under the supervision of the
local executives? Mr. Natividad: Yes, in fact, the National Police Commission is under the Office of the
President.52
Mr. Natividad: Yes, Madam President.
In the discussions of the Constitutional Commission regarding the above provision it is
xxx xxx xxx clear that the framers never intended for local chief executives to exercise unbridled
control over the police in emergency situations. This is without prejudice to their
Mr. de los Reyes: But in exceptional cases, even the operational control can be taken authority over police units in their jurisdiction as provided by law, and their prerogative
over by the National Police Commission? to seek assistance from the police in day to day situations, as contemplated by the
Constitutional Commission. But as a civilian agency of the government, the police,
through the NAPOLCOM, properly comes within, and is subject to, the exercise by the
Mr. Natividad: If the situation is beyond the capacity of the local President of the power of executive control.53
governments.51 (Emphases supplied)
iii. The provincial governor does not possess the same calling-out powers as the
Furthermore according to the framers, it is still the President who is authorized to President
exercise supervision and control over the police, through the National Police
Commission:
Given the foregoing, respondent provincial governor is not endowed with the power to
call upon the armed forces at his own bidding. In issuing the assailed proclamation,
Mr. Rodrigo: Just a few questions. The President of the Philippines is the Commander- Governor Tan exceeded his authority when he declared a state of emergency and
in-Chief of all the armed forces. called upon the Armed Forces, the police, and his own Civilian Emergency Force. The
calling-out powers contemplated under the Constitution is exclusive to the President.
Mr. Natividad: Yes, Madam President. An exercise by another official, even if he is the local chief executive, is ultra vires, and
may not be justified by the invocation of Section 465 of the Local Government Code,
Mr. Rodrigo: Since the national police is not integrated with the armed forces, I do not as will be discussed subsequently.
suppose they come under the Commander-in-Chief powers of the President of the
Philippines. Respondents, however, justify this stance by stating that nowhere in the seminal case
of David v. Arroyo, which dealt squarely with the issue of the declaration of a state of
Mr. Natividad: They do, Madam President. By law, they are under the supervision and emergency, does it limit the said authority to the President alone. Respondents
control of the President of the Philippines. contend that the ruling in David expressly limits the authority to declare a national
emergency, a condition which covers the entire country, and does not include
emergency situations in local government units. 54 This claim is belied by the clear
Mr. Rodrigo: Yes, but the President is not the Commander-in-Chief of the national intent of the framers that in all situations involving threats to security, such as lawless
police. violence, invasion or rebellion, even in localized areas, it is still the President who
possesses the sole authority to exercise calling-out powers. As reflected in the Journal
Mr. Natividad: He is the President. of the Constitutional Commission:

Mr. Rodrigo: Yes, the Executive. But they do not come under that specific provision Thereafter, Mr. Padilla proposed on line 29 to insert the phrase OR PUBLIC
that the President is the Commander-in-Chief of all the armed forces. DISORDER in lieu of "invasion or rebellion." Mr. Sumulong stated that the committee
could not accept the amendment because under the first section of Section 15, the
President may call out and make use of the armed forces to prevent or suppress not
Mr. Natividad: No, not under the Commander-in-Chief provision.
only lawless violence but even invasion or rebellion without declaring martial law. He
observed that by deleting "invasion or rebellion" and substituting PUBLIC DISORDER,
Mr. Rodrigo: There are two other powers of the President. The the President would have to declare martial law before he can make use of the armed
forces to prevent or suppress lawless invasion or rebellion.
President has control over ministries, bureaus and offices, and supervision over local
governments. Under which does the police fall, under control or under supervision? Mr. Padilla, in reply thereto, stated that the first sentence contemplates a lighter
situation where there is some lawless violence in a small portion of the country or public
Mr. Natividad: Both, Madam President. disorder in another at which times, the armed forces can be called to prevent or
suppress these incidents. He noted that the Commander-in-Chief can do so in a minor (b) For efficient, effective and economical governance the purpose of which is the
degree but he can also exercise such powers should the situation worsen. The words general welfare of the province and its inhabitants pursuant to Section 16 of this Code,
"invasion or rebellion" to be eliminated on line 14 are covered by the following sentence the provincial governor shall:
which provides for "invasion or rebellion." He maintained that the proposed
amendment does not mean that under such circumstances, the President cannot call (1) Exercise general supervision and control over all programs, projects, services, and
on the armed forces to prevent or suppress the same.55 (Emphasis supplied) activities of the provincial government, and in this connection, shall:

III. Section 465 of the Local xxx xxx xxx

Government Code cannot be invoked to justify the powers enumerated under (vii) Carry out such emergency measures as may be necessary during and in the
Proclamation 1-09 aftermath of man-made and natural disasters and calamities;

Respondent governor characterized the kidnapping of the three ICRC workers as a (2) Enforce all laws and ordinances relative to the governance of the province and the
terroristic act, and used this incident to justify the exercise of the powers enumerated exercise of the appropriate corporate powers provided for under Section 22 of this
under Proclamation 1-09.56 He invokes Section 465, in relation to Section 16, of the Code, implement all approved policies, programs, projects, services and activities of
Local Government Code, which purportedly allows the governor to carry out the province and, in addition to the foregoing, shall:
emergency measures and call upon the appropriate national law enforcement
agencies for assistance. But a closer look at the said proclamation shows that there is
no provision in the Local Government Code nor in any law on which the broad and xxx xxx xxx
unwarranted powers granted to the Governor may be based.
(vi) Call upon the appropriate national law enforcement agencies to suppress disorder,
Petitioners cite the implementation of "General Search and Seizure including arrests riot, lawless violence, rebellion or sedition or to apprehend violators of the law when
in the pursuit of the kidnappers and their supporters,"57 as being violative of the public interest so requires and the police forces of the component city or municipality
constitutional proscription on general search warrants and general seizures. where the disorder or violation is happening are inadequate to cope with the situation
Petitioners rightly assert that this alone would be sufficient to render the proclamation or the violators.
void, as general searches and seizures are proscribed, for being violative of the rights
enshrined in the Bill of Rights, particularly: Section 16. General Welfare. - Every local government unit shall exercise the powers
expressly granted, those necessarily implied therefrom, as well as powers necessary,
The right of the people to be secure in their persons, houses, papers, and effects appropriate, or incidental for its efficient and effective governance, and those which
against unreasonable searches and seizures of whatever nature and for any purpose are essential to the promotion of the general welfare. Within their respective territorial
shall be inviolable, and no search warrant or warrant of arrest shall issue except upon jurisdictions, local government units shall ensure and support, among other things, the
probable cause to be determined personally by the judge after examination under oath preservation and enrichment of culture, promote health and safety, enhance the right
or affirmation of the complainant and the witnesses he may produce, and particularly of the people to a balanced ecology, encourage and support the development of
describing the place to be searched and the persons or things to be seized. 58 appropriate and self-reliant scientific and technological capabilities, improve public
morals, enhance economic prosperity and social justice, promote full employment
among their residents, maintain peace and order, and preserve the comfort and
In fact, respondent governor has arrogated unto himself powers exceeding even the convenience of their inhabitants. (Emphases supplied)
martial law powers of the President, because as the Constitution itself declares, "A
state of martial law does not suspend the operation of the Constitution, nor supplant
the functioning of the civil courts or legislative assemblies, nor authorize the Respondents cannot rely on paragraph 1, subparagraph (vii) of Article 465 above, as
conferment of the jurisdiction on military courts and agencies over civilians where civil the said provision expressly refers to calamities and disasters, whether man-made or
courts are able to function, nor automatically suspend the privilege of the writ."59 natural. The governor, as local chief executive of the province, is certainly empowered
to enact and implement emergency measures during these occurrences. But the
kidnapping incident in the case at bar cannot be considered as a calamity or a disaster.
We find, and so hold, that there is nothing in the Local Government Code which justifies Respondents cannot find any legal mooring under this provision to justify their actions.
the acts sanctioned under the said Proclamation. Not even Section 465 of the said
Code, in relation to Section 16, which states:
Paragraph 2, subparagraph (vi) of the same provision is equally inapplicable for two
reasons. First, the Armed Forces of the Philippines does not fall under the category of
Section 465. The Chief Executive: Powers, Duties, Functions, and Compensation. a "national law enforcement agency," to which the National Police Commission
(NAPOLCOM) and its departments belong.
xxx xxx xxx
Its mandate is to uphold the sovereignty of the Philippines, support the Constitution, Taken in conjunction with each other, it becomes clear that the Constitution does not
and defend the Republic against all enemies, foreign and domestic. Its aim is also to authorize the organization of private armed groups similar to the CEF convened by the
secure the integrity of the national territory.60 respondent Governor. The framers of the Constitution were themselves wary of armed
citizens’ groups, as shown in the following proceedings:
Second, there was no evidence or even an allegation on record that the local police
forces were inadequate to cope with the situation or apprehend the violators. If they MR. GARCIA: I think it is very clear that the problem we have here is a paramilitary
were inadequate, the recourse of the provincial governor was to ask the assistance of force operating under the cloak, under the mantle of legality is creating a lot of
the Secretary of Interior and Local Government, or such other authorized officials, for problems precisely by being able to operate as an independent private army for many
the assistance of national law enforcement agencies. regional warlords. And at the same time, this I think has been the thrust, the intent of
many of the discussions and objections to the paramilitary units and the armed groups.
The Local Government Code does not involve the diminution of central powers
inherently vested in the National Government, especially not the prerogatives solely MR. PADILLA: My proposal covers two parts: the private armies of political warlords
granted by the Constitution to the President in matters of security and defense. and other armed torces not recognized by constituted authority which shall be
dismantled and dissolved. In my trips to the provinces, I heard of many abuses
The intent behind the powers granted to local government units is fiscal, economic, committed by the CHDF (Civilian Home Defense Forces), specially in Escalante,
and administrative in nature.1âwphi1 The Code is concerned only with powers that Negros Occidental. But I do not know whether a particular CHDF is approved or
would make the delivery of basic services more effective to the constituents, 61 and authorized by competent authority. If it is not authorized, then the CHDF will have to
should not be unduly stretched to confer calling-out powers on local executives. be dismantled. If some CHDFs, say in other provinces, are authorized by constituted
authority, by the Armed Forces of the Philippines, through the Chief of Staff or the
Minister of National Defense, if they are recognized and authorized, then they will not
In the sponsorship remarks for Republic Act 7160, it was stated that the devolution of be dismantled. But I cannot give a categorical answer to any specific CHDF unit, only
powers is a step towards the autonomy of local government units (LGUs), and is the principle that if they are armed forces which are not authorized, then they should
actually an experiment whose success heavily relies on the power of taxation of the be dismantled. 64 (Emphasis supplied)
LGUs. The underpinnings of the Code can be found in Section 5, Article II of the 1973
Constitution, which allowed LGUs to create their own sources of revenue. 62 During the
interpellation made by Mr. Tirol addressed to Mr. de Pedro, the latter emphasized that Thus, with the discussions in the Constitutional Commission as guide, the creation of
"Decentralization is an administrative concept and the process of shifting and the Civilian Emergency Force (CEF) in the present case, is also invalid.
delegating power from a central point to subordinate levels to promote independence,
responsibility, and quicker decision-making. … (I)t does not involve any transfer of final WHEREFORE, the instant petition is GRANTED. Judgment is rendered commanding
authority from the national to field levels, nor diminution of central office powers and respondents to desist from further proceedings m implementing Proclamation No. 1,
responsibilities. Certain government agencies, including the police force, are Series of 2009, and its Implementing Guidelines. The said proclamation and guidelines
exempted from the decentralization process because their functions are not inherent are hereby declared NULL and VOID for having been issued in grave abuse of
in local government units."63 discretion, amounting to lack or excess of jurisdiction.

IV. Provincial governor is not authorized to convene CEF SO ORDERED.

Pursuant to the national policy to establish one police force, the organization of private
citizen armies is proscribed. Section 24 of Article XVIII of the Constitution mandates
that:

Private armies and other armed groups not recognized by duly constituted authority
shall be dismantled. All paramilitary forces including Civilian Home Defense Forces
(CHDF) not consistent with the citizen armed force established in this Constitution,
shall be dissolved or, where appropriate, converted into the regular force. G.R. No. 190259. June 7, 2011.*
DATU ZALDY UY AMPATUAN, ANSARUDDIN ADIONG, REGIE SAHALI-
GENERALE, petitioners, vs. HON. RONALDO PUNO, in his capacity as
Additionally, Section 21of Article XI states that, "The preservation of peace and order
Secretary of the Department of Interior and Local Government and alter-ego of
within the regions shall be the responsibility of the local police agencies which shall be
organized, maintained, supervised, and utilized in accordance with applicable laws. President Gloria Macapagal-Arroyo, and anyone acting in his stead and on behalf
The defense and security of the regions shall be the responsibility of the National of the President of the Philippines, ARMED FORCES OF THE PHILIPPINES
Government." (AFP), or any of their units operating in the Autonomous Region in Muslim
Mindanao (ARMM), and PHILIPPINE NATIONAL POLICE, or any of their units On November 24, 2009, the day after the gruesome massacre of 57 men and
operating in ARMM, respondents. women, including some news reporters, then President Gloria Macapagal-Arroyo
Administrative Law; State of Emergency; The Department of Interior and issued Proclamation 1946,1 placing “the Provinces of Maguindanao and Sultan
Local Government (DILG) Secretary did not take over the administration or Kudarat and the City of Cotabato under a state of emergency.” She directed the
operations of the Autonomous Region of Muslim Mindanao (ARMM).—The DILG Armed Forces of the Philippines (AFP) and the Philippine National Police (PNP)
Secretary did not take over control of the powers of the ARMM. After law “to undertake
enforcement agents took respondent Governor of ARMM into custody for alleged
complicity in the Maguindanao massacre, the ARMM Vice-Governor, petitioner _______________
Ansaruddin Adiong, assumed the vacated post on December 10, 2009 pursuant to
the rule on succession found in Article VII, Section 12, of RA 9054. In turn, Acting 1 Rollo, p. 34.
Governor Adiong named the then Speaker of the ARMM Regional Assembly, 230
petitioner Sahali-Generale, Acting ARMM Vice-Governor. In short, the DILG
230 SUPREME COURT REPORTS ANNOTATED
Secretary did not take over the administration or operations of the ARMM.
Same; Same; The President did not proclaim a national emergency, only a Ampatuan vs. Puno
state of emergency in the three places mentioned; the calling out of the armed forces such measures as may be allowed by the Constitution and by law to prevent and
to prevent or suppress lawless violence in such places is a power that the suppress all incidents of lawless violence” in the named places.
Constitution directly vests in the President; she did not need a congressional Three days later or on November 27, President Arroyo also issued
authority to exercise the same.—The President did not proclaim a national Administrative Order 273 (AO 273)2 “transferring” supervision of the Autonomous
emergency, only a state of emergency in the three places mentioned. And she did Region of Muslim Mindanao (ARMM) from the Office of the President to the
not act pursuant to any law enacted by Congress that authorized her to Department of Interior and Local Government (DILG). But, due to issues raised
over the terminology used in AO 273, the President issued Administrative Order
_______________ 273-A (AO 273-A) amending the former, by “delegating” instead of “transferring”
supervision of the ARMM to the DILG.3
* EN BANC. Claiming that the President’s issuances encroached on the ARMM’s autonomy,
229 petitioners Datu Zaldy Uy Ampatuan, Ansaruddin Adiong, and Regie Sahali-
Generale, all ARMM officials,4 filed this petition for prohibition under Rule 65.
VOL. 651, JUNE 7, 2011 229
They alleged that the proclamation and the orders empowered the DILG Secretary
Ampatuan vs. Puno to take over ARMM’s operations and seize the regional government’s powers, in
exercise extraordinary powers. The calling out of the armed forces to prevent violation of the principle of local autonomy under Republic Act 9054 (also known
or suppress lawless violence in such places is a power that the Constitution directly as the Expanded ARMM Act) and the Constitution. The President gave the DILG
vests in the President. She did not need a congressional authority to exercise the Secretary the power to exercise, not merely administrative supervision, but control
same. over the ARMM since the latter could suspend ARMM officials and replace them.5
Same; Same; It is clearly to the President that the Constitution entrusts the Petitioner ARMM officials claimed that the President had no factual basis for
determination of the need for calling out the armed forces to prevent and suppress declaring a state of emergency, especially in the Province of Sultan Kudarat and
lawless violence; Unless it is shown that such determination was attended by grave the City of Cotabato, where no critical violent incidents occurred. The deployment
abuse of discretion, the court will accord respect to the President’s judgment.—
While it is true that the Court may inquire into the factual bases for the President’s _______________
exercise of the above power, it would generally defer to her judgment on the matter.
As the Court acknowledged in Integrated Bar of the Philippines v. Hon. Zamora, 2 Id., at p. 36.
338 SCRA 81 (2000), it is clearly to the President that the Constitution entrusts 3 Id., at p. 80.
the determination of the need for calling out the armed forces to prevent and 4 Ampatuan, Adiong and Sahali-Generale were, respectively, the Governor,
suppress lawless violence. Unless it is shown that such determination was Vice-Governor and Speaker of the Legislative Assembly of the ARMM at that time.
attended by grave abuse of discretion, the Court will accord respect to the 5 Rollo, pp. 14-17.
President’s judgment. 231
SPECIAL CIVIL ACTION in the Supreme Court. Prohibition.
VOL. 651, JUNE 7, 2011 231
The facts are stated in the opinion of the Court.
Fortun, Narvasa & Salazar for petitioners. Ampatuan vs. Puno
The Solicitor General for respondents. of troops and the taking over of the ARMM constitutes an invalid exercise of the
ABAD, J.: President’s emergency powers.6 Petitioners asked that Proclamation 1946 as well
as AOs 273 and 273-A be declared unconstitutional and that respondents DILG One. The claim of petitioners that the subject proclamation and
Secretary, the AFP, and the PNP be enjoined from implementing them. administrative orders violate the principle of local autonomy is anchored on the
In its comment for the respondents,7 the Office of the Solicitor General (OSG) allegation that, through them, the President authorized the DILG Secretary to
insisted that the President issued Proclamation 1946, not to deprive the ARMM of take over the operations of the ARMM and assume direct governmental powers
its autonomy, but to restore peace and order in subject places. 8 She issued the over the region.
proclamation pursuant to her “calling out” power9 as Commander-in-Chief under But, in the first place, the DILG Secretary did not take over control of the
the first sentence of Section 18, Article VII of the Constitution. The determination powers of the ARMM. After law enforcement agents took respondent Governor of
of the need to exercise this power rests solely on her wisdom. 10 She must use her ARMM into custody for alleged complicity in the Maguindanao massacre, the
judgment based on intelligence reports and such best information as are available 233
to her to call out the armed forces to suppress and prevent lawless violence VOL. 651, JUNE 7, 2011 233
wherever and whenever these reared their ugly heads.
On the other hand, the President merely delegated through AOs 273 and 273- Ampatuan vs. Puno
A her supervisory powers over the ARMM to the DILG Secretary who was her alter ARMM Vice-Governor, petitioner Ansaruddin Adiong, assumed the vacated post
ego any way. These orders did not authorize a take over of the ARMM. They did on December 10, 2009 pursuant to the rule on succession found in Article VII,
not give him blanket authority to suspend or replace ARMM officials. 11 The Section 12,14 of RA 9054. In turn, Acting Governor Adiong named the then Speaker
delegation was necessary to facilitate the investigation of the mass of the ARMM Regional Assembly, petitioner Sahali-Generale, Acting ARMM Vice-
killings.12 Further, the assailed proclamation and administrative orders did not Governor.15 In short, the DILG Secretary did not take over the administration or
provide for the exercise of emergency powers.13 operations of the ARMM.
Two. Petitioners contend that the President unlawfully exercised emergency
_______________ powers when she ordered the deployment of AFP and PNP personnel in the places
mentioned in the proclamation.16 But such deployment is not by itself an exercise
6 Id., at pp. 20-22. of emergency powers as understood under Section 23 (2), Article VI of the
7 Id., at p. 63. Constitution, which provides:
8 Id., at pp. 85, 87, 95. “SECTION 23. x x x (2) In times of war or other national emergency,
9 Id., at p. 98. the Congress may, by law, authorize the President, for a limited period
10 Id., at p. 76. and subject to such restrictions as it may prescribe, to exercise powers
11 Id., at p. 95. necessary and proper to carry out a declared national policy. Unless
12 Id., at p. 78. sooner withdrawn by resolution of the Congress, such powers shall cease
13 Id., at p. 110. upon the next adjournment thereof.”
232
232 SUPREME COURT REPORTS ANNOTATED _______________
Ampatuan vs. Puno
Although normalcy has in the meantime returned to the places subject of this 14 SEC. 12. Succession to Regional Governorship in Cases of Temporary
petition, it might be relevant to rule on the issues raised in this petition since some Incapacity.—In case of temporary incapacity of the regional Governor to perform
acts done pursuant to Proclamation 1946 and AOs 273 and 273-A could impact on his duties on account of physical or legal causes, or when he is on official leave of
the administrative and criminal cases that the government subsequently filed absence or on travel outside the territorial jurisdiction of the Republic of the
against those believed affected by such proclamation and orders. Philippines, the Regional Vice-Governor, or if there be none or in case of his
The Issues Presented permanent or temporary incapacity or refusal to assume office, the Speaker of the
The issues presented in this case are: Regional Assembly shall exercise the powers, duties and functions of the Regional
1. Whether or not Proclamation 1946 and AOs 273 and 273-A violate the Governor as prescribed by law enacted by the Regional Assembly or in the absence
principle of local autonomy under Section 16, Article X of the Constitution, and thereof, by the pertinent provisions of Republic Act 7160 or the Local Government
Section 1, Article V of the Expanded ARMM Organic Act; Code of 1991.
2. Whether or not President Arroyo invalidly exercised emergency powers 15 http://services.inquirer.net/print/print.php?article_id=20100707-279759.
when she called out the AFP and the PNP to prevent and suppress all incidents of 16 Rollo, p. 22.
lawless violence in Maguindanao, Sultan Kudarat, and Cotabato City; and 234
3. Whether or not the President had factual bases for her actions. 234 SUPREME COURT REPORTS ANNOTATED
The Rulings of the Court
We dismiss the petition. Ampatuan vs. Puno
The President did not proclaim a national emergency, only a state of emergency imperatively necessary in emergency situations to avert great loss of
in the three places mentioned. And she did not act pursuant to any law enacted by human lives and mass destruction of property. Indeed, the decision to call
Congress that authorized her to exercise extraordinary powers. The calling out of out the military to prevent or suppress lawless violence must be done
the armed forces to prevent or suppress lawless violence in such places is a power swiftly and decisively if it were to have any effect at all. x x x.”20
that the Constitution directly vests in the President. She did not need a Here, petitioners failed to show that the declaration of a state of emergency in
congressional authority to exercise the same. the Provinces of Maguindanao, Sultan Kudarat and Cotabato City, as well as the
Three. The President’s call on the armed forces to prevent or suppress President’s exercise of the “calling out” power had no factual basis. They simply
lawless violence springs from the power vested in her under Section 18, Article VII alleged that, since not all areas under the ARMM were placed under a state of
of the Constitution, which provides.17 emergency, it follows that the take over of the entire ARMM by the DILG Secretary
“SECTION 18. The President shall be the Commander-in-Chief of all had no basis too.21
armed forces of the Philippines and whenever it becomes necessary, he
may call out such armed forces to prevent or suppress lawless violence, _______________
invasion or rebellion. x x x”
While it is true that the Court may inquire into the factual bases for the 20 Id., at pp. 643-644; pp. 110-111.
President’s exercise of the above power,18 it would generally defer to her judgment 21 Rollo, pp. 20-21.
on the matter. As the Court acknowledged in Integrated Bar of the Philippines v. 236
Hon. Zamora,19it is clearly to the President that the Constitution entrusts the
236 SUPREME COURT REPORTS ANNOTATED
determination of the need for calling out the armed forces to prevent and suppress
lawless violence. Unless it is shown that such determination was attended by grave Ampatuan vs. Puno
abuse of discretion, the Court will accord respect to the President’s judgment. But, apart from the fact that there was no such take over to begin with, the
Thus, the Court said: OSG also clearly explained the factual bases for the President’s decision to call out
the armed forces, as follows:
_______________ “The Ampatuan and Mangudadatu clans are prominent families
engaged in the political control of Maguindanao. It is also a known fact
17 See SANLAKAS v. Executive Secretary Reyes, 466 Phil. 482, 509-510; 421 that both families have an arsenal of armed followers who hold elective
SCRA 656, 668 (2004). positions in various parts of the ARMM and the rest of Mindanao.
18 Lacson v. Sec. Perez, 410 Phil. 78, 93; 357 SCRA 756, 772 (2001). Considering the fact that the principal victims of the brutal bloodshed
19 392 Phil. 618, 635; 338 SCRA 81, 102-103 (2000). are members of the Mangudadatu family and the main perpetrators of the
235 brutal killings are members and followers of the Ampatuan family, both
the military and police had to prepare for and prevent reported
VOL. 651, JUNE 7, 2011 235
retaliatory actions from the Mangudadatu clan and additional offensive
Ampatuan vs. Puno measures from the Ampatuan clan.
“If the petitioner fails, by way of proof, to support the assertion that xxxx
the President acted without factual basis, then this Court cannot The Ampatuan forces are estimated to be approximately two thousand
undertake an independent investigation beyond the pleadings. The four hundred (2,400) persons, equipped with about two thousand (2,000)
factual necessity of calling out the armed forces is not easily quantifiable firearms, about four hundred (400) of which have been accounted for.
and cannot be objectively established since matters considered for xxx
satisfying the same is a combination of several factors which are not As for the Mangudadatus, they have an estimated one thousand eight
always accessible to the courts. Besides the absence of textual standards hundred (1,800) personnel, with about two hundred (200) firearms. x x x
that the court may use to judge necessity, information necessary to arrive Apart from their own personal forces, both clans have Special Civilian
at such judgment might also prove unmanageable for the courts. Certain Auxiliary Army (SCAA) personnel who support them: about five hundred
pertinent information might be difficult to verify, or wholly unavailable (500) for the Ampatuans and three hundred (300) for the Mangudadatus.
to the courts. In many instances, the evidence upon which the President What could be worse than the armed clash of two warring clans and
might decide that there is a need to call out the armed forces may be of a their armed supporters, especially in light of intelligence reports on the
nature not constituting technical proof. potential involvement of rebel armed groups (RAGs).
On the other hand, the President, as Commander-in-Chief has a vast One RAG was reported to have planned an attack on the forces of Datu
intelligence network to gather information, some of which may be Andal Ampatuan, Sr. to show support and sympathy for the victims. The
classified as highly confidential or affecting the security of the state. In said attack shall worsen the
the exercise of the power to call, on-the-spot decisions may be 237
VOL. 651, JUNE 7, 2011 237 pressly granted to the Executive under the Constitution. (Neri vs. Senate
Committee on Accountability of Public Officers and Investigations, 549 SCRA 77
Ampatuan vs. Puno [2008])
age-old territorial dispute between the said RAG and the Ampatuan ——o0o——
family.
xxxx
On the other hand, RAG faction which is based in Sultan Kudarat was
reported to have received three million pesos (P3,000,000.00) from Datu
Andal Ampatuan, Sr. for the procurement of ammunition. The said
faction is a force to reckon with because the group is well capable of
launching a series of violent activities to divert the attention of the people
and the authorities away from the multiple murder case. x x x
In addition, two other factions of a RAG are likely to support the G.R. No. 190293. March 20, 2012.*
Mangudadatu family. The Cotabato-based faction has the strength of PHILIP SIGFRID A. FORTUN and ALBERT LEE G. ANGELES,
about five hundred (500) persons and three hundred seventy-two (372) petitioners, vs. GLORIA MACAPAGAL-ARROYO, as Commander-in-Chief and
firearms while the Sultan Kudarat-based faction has the strength of President of the Republic of the Philippines, EDUARDO ERMITA, Executive
about four hundred (400) persons and three hundred (300) firearms and Secretary, ARMED FORCES OF THE PHILIPPINES (AFP), or any of their units,
was reported to be moving towards Maguindanao to support the PHILIPPINE NATIONAL POLICE (PNP), or any of their units, JOHN DOES and
Mangudadatu clan in its armed fight against the Ampatuans.”22 JANE DOES acting under their direction and control, respondents.
In other words, the imminence of violence and anarchy at the time the G.R. No. 190294. March 20, 2012.*
President issued Proclamation 1946 was too grave to ignore and she had to act to DIDAGEN P. DILANGALEN, petitioner, vs. EDUARDO R. ERMITA in his
prevent further bloodshed and hostilities in the places mentioned. Progress reports capacity as Executive Secretary, NORBERTO GONZALES in his capacity as
also indicated that there was movement in these places of both high-powered Secretary of National Defense, RONALDO PUNO in his capacity as Secretary of
firearms and armed men sympathetic to the two clans. 23 Thus, to pacify the Interior and Local Government, respondents.
people’s fears and stabilize the situation, the President had to take preventive G.R. No. 190301. March 20, 2012.*
action. She called out the armed forces to control the proliferation of loose firearms NATIONAL UNION OF PEOPLES’ LAWYERS (NUPL) SECRETARY GENERAL
and dismantle the armed groups that continuously threatened the peace and NERI JAVIER COLMENARES, BAYAN MUNA REPRESENTATIVE SATUR C.
security in the affected places. OCAMPO, GABRIELA WOMEN’S PARTY REPRESENTATIVE LIZA L. MAZA,
Notably, the present administration of President Benigno Aquino III has not ATTY. JULIUS GARCIA MATIBAG, ATTY. EPHRAIM B. CORTEZ, ATTY.
withdrawn the declaration of a state of emergency under Proclamation 1946. It JOBERT ILARDE PAHILGA, ATTY. VOLTAIRE B. AFRICA, BAGONG
has been reported24 that the declaration would not be lifted soon because there is ALYANSANG MAKABAYAN (BAYAN) SECRETARY GENERAL RENATO M.
still a need to disband private armies and confiscate loose firearms. Apparently, REYES, JR. and ANTHONY IAN CRUZ, petitioners, vs. PRESIDENT GLORIA
the presence of troops in those places is still necessary to ease fear and tension MACAPAGAL-ARROYO, EXECUTIVE SECRETARY EDUARDO R. ERMITA,
among the citizenry and prevent and suppress any violence that may still erupt, ARMED FORCES OF THE PHILIPPINES CHIEF OF STAFF GENERAL
despite the passage of more than a year from the time of the Maguindanao VICTOR S. IBRADO, PHILIPPINE NATIONAL POLICE DIRECTOR GENERAL
massacre. JESUS A. VERZOSA, DE-
Since petitioners are not able to demonstrate that the proclamation of state of _______________
emergency in the subject places and the calling out of the armed forces to prevent * EN BANC.
or suppress lawless violence there have clearly no factual bases, the Court must 505
respect the President’s actions.
VOL. 668, MARCH 20, 2012 505
WHEREFORE, the petition is DISMISSED for lack of merit.
SO ORDERED. Fortun vs. Macapagal-Arroyo
Corona (C.J.), Carpio, Carpio-Morales, Velasco, Jr., Nachura, Leonardo-De PARTMENT OF JUSTICE SECRETARY AGNES VST DEVANADERA, ARMED
Castro, Brion, Peralta, Bersamin, Del Castillo, Villarama, Jr., Perez, FORCES OF THE PHILIPPINES EASTERN MINDANAO COMMAND CHIEF
Mendoza and Sereno, JJ., concur. LIEUTENANT GENERAL RAYMUNDO B. FERRER, respondents.
Petition dismissed. G.R. No. 190302. March 20, 2012.*
Note.—View that the Constitution does not expressly grant executive privilege JOSEPH NELSON Q. LOYOLA, petitioner, vs. HER EXCELLENCY PRESIDENT
power to the President but courts have long recognized implied Presidential GLORIA MACAPAGAL-ARROYO, ARMED FORCES CHIEF OF STAFF
powers if “necessary and proper” in carrying out powers and functions ex- GENERAL VICTOR IBRADO, PHILIPPINE NATIONAL POLICE (PNP),
DIRECTOR GENERAL JESUS VERZOSA, EXECUTIVE SECRETARY may, in the same manner, extend such proclamation or suspension for a period to
EDUARDO ERMITA, respondents. be determined by the Congress, if the invasion or rebellion shall persist and public
G.R. No. 190307. March 20, 2012.* safety requires it. The Congress, if not in session, shall, within twenty-four hours
JOVITO R. SALONGA, RAUL C. PANGALANGAN, H. HARRY L. ROQUE, JR., following such proclamation or suspension, convene in accordance with its rules
JOEL R. BUTUYAN, EMILIO CAPULONG, FLORIN T. HILBAY, ROMEL R. without
BAGARES, DEXTER DONNE B. DIZON, ALLAN JONES F. LARDIZABAL and 507
GILBERT T. ANDRES, suing as taxpayers and as CONCERNED Filipino citizens, VOL. 668, MARCH 20, 2012 507
petitioners, vs. GLORIA MACAPAGAL-ARROYO, in his (sic) capacity as
President of the Republic of the Philippines, HON. EDUARDO ERMITA, JR., in Fortun vs. Macapagal-Arroyo
his capacity as Executive Secretary, and HON. ROLANDO ANDAYA in his any need of a call. x x x x Although the above vests in the President the power
capacity as Secretary of the Department of Budget and Management, GENERAL to proclaim martial law or suspend the privilege of the writ of habeas corpus, he
VICTOR IBRADO, in his capacity as Armed Forces of the Philippines Chief of shares such power with the Congress. Thus: 1. The President’s proclamation or
Staff, DIRECTOR JESUS VERZOSA, in his capacity as Chief of the Philippine suspension is temporary, good for only 60 days; 2. He must, within 48 hours of the
National Police, respondents. proclamation or suspension, report his action in person or in writing to Congress;
G.R. No. 190356. March 20, 2012.* 3. Both houses of Congress, if not in session must jointly convene within 24 hours
BAILENG S. MANTAWIL, DENGCO SABAN, Engr. OCTOBER CHIO, of the proclamation or suspension for the purpose of reviewing its validity; and 4.
AKBAYAN PARTY LIST REPRESENTATIVES WALDEN F. BELLO and ANA The Congress, voting jointly, may revoke or affirm the President’s proclamation or
THERESIA HONTIVEROS-BARAQUEL, LORETTA ANN P. ROSALES, suspension, allow their limited effectivity to lapse, or extend the same if Congress
MARVIC M.V.F. LEONEN, THEODORE O. TE and IBARRA M. GUTIERREZ deems warranted. It is evident that under the 1987 Constitution the President and
506 the Congress act in tandem in exercising the power to proclaim martial law or
suspend the privilege of the writ of habeas corpus. They exercise the power, not
506 SUPREME COURT REPORTS ANNOTATED
only sequentially, but in a sense jointly since, after the President has initiated the
Fortun vs. Macapagal-Arroyo proclamation or the suspension, only the Congress can maintain the same based
III, petitioners, vs. THE EXECUTIVE SECRETARY, THE SECRETARY OF on its own evaluation of the situation on the ground, a power that the President
NATIONAL DEFENSE, THE SECRETARY OF JUSTICE, THE SECRETARY OF does not have.
INTERIOR AND LOCAL GOVERNMENT, THE SECRETARY OF BUDGET AND Same; Courts; Judicial Review; The Court’s duty is to steer clear of declaring
MANAGEMENT, and THE CHIEF OF STAFF OF THE ARMED FORCES OF unconstitutional the acts of the Executive or the Legislative department, given the
THE PHILIPPINES, THE DIRECTOR GENERAL OF THE PHILIPPINE assumption that it carefully studied those acts and found them consistent with the
NATIONAL POLICE, respondents. fundamental law before taking them.—The Court does not resolve purely academic
G.R. No. 190380. March 20, 2012.* questions to satisfy scholarly interest, however intellectually challenging these
CHRISTIAN MONSOD and CARLOS P. MEDINA, JR., are. This is especially true, said the Court in Philippine Association of Colleges and
petitioners, vs. EDUARDO R. ERMITA, in his capacity as Executive Secretary, Universities v. Secretary of Education, where the issues “reach constitutional
respondent. dimensions, for then there comes into play regard for the court’s duty to avoid
Constitutional Law; Executive Department; Congress; Martial Law; Writ of decision of constitutional issues unless avoidance becomes evasion.” The Court’s
Habeas Corpus; It is evident that under the 1986 Constitution the President and duty is to steer clear of declaring unconstitutional the acts of the Executive or the
the Congress act in tandem in exercising the power to proclaim martial law or Legislative department, given the assumption that it carefully studied those acts
suspend the privilege of the writ of habeas corpus. They exercise the power, not only and found them consistent with the fundamental law before taking them. “To
sequentially, but in a sense jointly.—The pertinent provisions of Section 18, Article doubt is to sustain.”
VII of the 1987 Constitution state: Sec. 18. The President shall be the Same; Same; Executive Department; Congress; Martial Law; Writ of Habeas
Commander-in-Chief of all armed forces of the Philippines and whenever it Corpus; If the Congress procrastinates or altogether fails to fulfill its duty respecting
becomes necessary, he may call out such armed forces to prevent or suppress the proclamation or suspension within the short time expected of it, then the Court
lawless violence, invasion or rebellion. In case of invasion or rebellion, when the can step in, hear the petitions challenging the President’s action, and ascertain if it
public safety requires it, he may, for a period not exceeding sixty days, suspend the has
privilege of the writ of habeas corpus or place the Philippines or any part thereof 508
under martial law. Within forty-eight hours from the proclamation of martial law 508 SUPREME COURT REPORTS ANNOTATED
or the suspension of the privilege of writ of habeas corpus, the President shall
submit a report in person or in writing to the Congress. The Congress, voting Fortun vs. Macapagal-Arroyo
jointly, by a vote of at least a majority of all its Members in regular or special a factual basis.—If the Congress procrastinates or altogether fails to fulfill
session, may revoke such proclamation or suspension, which revocation shall not its duty respecting the proclamation or suspension within the short time expected
be set aside by the President. Upon the initiative of the President, the Congress of it, then the Court can step in, hear the petitions challenging the President’s
action, and ascertain if it has a factual basis. If the Court finds none, then it can 160 (2006), this Court held that the “moot and academic” principle is not a magical
annul the proclamation or the suspension. But what if the 30 days given it by the formula that automatically dissuades courts in resolving a case. Courts are not
Constitution proves inadequate? Justice Carpio himself offers the answer in his prevented from deciding cases, otherwise moot and academic, if (1) there is a grave
dissent: that 30-day period does not operate to divest this Court of its jurisdiction violation of the Constitution; (2) the situation is of exceptional character and of
over the case. The settled rule is that jurisdiction once acquired is not lost until paramount public interest; (3) the constitutional issue raised requires formulation
the case has been terminated. of controlling principles to guide the bench, the bar, and the public; and (4) the
CARPIO, J., Dissenting Opinion: case is capable of repetition yet evading review.
Remedial Law; Civil Procedure; Locus Standi; Words and Phrases; View that Same; Criminal Law; Rebellion; View that the term “rebellion” in Section 18,
“legal standing” or locus standi has been defined as a personal and substantial Article VII of the 1987 Constitution must be understood as having the same
interest in the case such that the party has sustained or will sustain direct injury meaning as the crime of “rebellion” that is defined in Article 134 of the Revised
as a result of the governmental act that is being challenged.—“Legal standing” Penal Code, as amended.—The term “rebellion” in Section 18, Article VII of the
or locus standi has been defined as a personal and substantial interest in the case 1987 Constitution must be understood as having the same meaning as the crime
such that the party has sustained or will sustain direct injury as a result of the of “rebellion” that is defined in Article 134 of the Revised Penal Code, as
governmental act that is being challenged. In case of a suit questioning the amended. First, this is the clear import of the last two paragraphs of Section 18,
sufficiency of the factual basis of the proclamation of martial law or suspension of Article VII of the Constitution, which explicitly state: The suspension of the
the writ, such as here, Section 18, Article VII of the Constitution expressly privilege of the writ of habeas corpus shall apply only to persons judicially
provides: The Supreme Court may review, in an appropriate proceeding filed charged for rebellion or offenses inherent in, or directly connected with,
by any citizen, the sufficiency of the factual basis of the proclamation of martial invasion. During the
law or the suspension of the privilege of the writ of habeas corpus or the extension 510
thereof, and must promulgate its decision thereon within thirty days from its 510 SUPREME COURT REPORTS ANNOTATED
filing. (Emphasis supplied) It is clear that the Constitution explicitly clothes “any
citizen” with the legal standing to challenge the constitutionality of the Fortun vs. Macapagal-Arroyo
declaration of martial law or suspension of the writ. The Constitution does not suspension of the privilege of the writ of habeas corpus, any person thus
make any distinction as to who can bring such an action. As discussed in the arrested or detained shall be judicially charged within three days, otherwise
deliberations of the Constitutional Commission, the “citizen” who can challenge he shall be released. (Emphasis supplied) For a person to be judicially charged for
the declaration of martial law or suspension of the writ need not even be a rebellion, there must necessarily be a statute defining rebellion. There is no statute
taxpayer. This was deliberately designed to arrest, without further delay, the defining rebellion other than the Revised Penal Code. Hence, “one can be ‘judicially
grave effects of an illegal declaration of martial law or suspension of the writ, and charged’ with rebellion only if one is suspected of having committed acts defined
to provide immediate relief to those aggrieved by the same. Accordingly, as rebellion in Article 134 of the Revised Penal Code.” Second, the Revised Penal
petitioners, being Filipino citizens, possess legal standing to Code definition of rebellion is the only legal definition of rebellion known and
509 understood by the Filipino people when they ratified the 1987 Constitution.
x x x Third, one of the Whereas clauses of Proclamation No. 1959 expressly cites
VOL. 668, MARCH 20, 2012 509
the Revised Penal Code definition of rebellion, belying the government’s claim that
Fortun vs. Macapagal-Arroyo the Revised Penal Code definition of rebellion merely guided the President in
file the present petitions assailing the sufficiency of the factual basis of issuing Proclamation No. 1959.
Proclamation No. 1959. Same; Same; Same; Commander-in-Chief Powers; View that in exercising the
Constitutional Law; Judicial Review; Courts; Moot and Academic; View that Commander-in-Chief powers under the Constitution, every President must insure
courts may exercise their review power only when there is an actual case or the existence of the elements of the crime of rebellion.—In exercising the
controversy, which involves a conflict of legal claims susceptible of judicial Commander-in-Chief powers under the Constitution, every President must insure
resolution; Corollarily, courts generally decline jurisdiction over a moot and the existence of the elements of the crime of rebellion, which are: (1) there is a (a)
academic case or outrightly dismiss it on the ground of mootness.—As a rule, courts public uprising and (b) taking arms against the Government; and (2) the purpose
may exercise their review power only when there is an actual case or controversy, of the uprising or movement is either (a) to remove from the allegiance to the
which involves a conflict of legal claims susceptible of judicial resolution. Such a Government or its laws: (1) the territory of the Philippines or any part thereof; or
case must be “definite and concrete, touching the legal relations of parties having (2) any body of land, naval, or other armed forces; or (b) to deprive the Chief
conflicting legal interests;” a real, as opposed to an imagined, controversy calling Executive or Congress, wholly or partially, of any of their powers and prerogatives.
for a specific relief. Corollarily, courts generally decline jurisdiction over a moot Same; Same; View that probable cause is the same amount of proof required
and academic case or outrightly dismiss it on the ground of mootness. A moot and for the filing of a criminal information by the prosecutor and for the issuance of an
academic case is one that ceases to present a justiciable controversy by virtue of arrest warrant by a judge.—I am of the view that probable cause of the existence
supervening events, so that assuming jurisdiction over the same, and eventually of either invasion or rebellion suffices and satisfies the standard of proof for a valid
deciding it, would be of no practical use or value. In David v. Arroyo, 489 SCRA declaration of martial law and suspension of the writ. Probable cause is the same
amount of proof required for the filing of a criminal information by the prosecutor Bernas points out, “Since the powers of Congress and the Court are independent
and for the issuance of an arrest warrant by a judge. Probable cause has been of each other, there is nothing to prevent Congress and the Court from
defined as a “set of facts and circumstances as would lead a reasonably discreet simultaneously exercising their separate powers.”
and prudent man to believe that the offense charged in the Information or any Same; Same; Same; Same; View that whatever the Court’s decision will be on
offense the sufficiency of the factual basis of the President’s declaration or suspension does
511 not preclude those aggrieved by such illegal acts from pursuing any course of legal
VOL. 668, MARCH 20, 2012 511 action available to them.—Indisputably, unlawful acts may be committed during
martial law or suspension of the writ, not only by the rebels, but also by
Fortun vs. Macapagal-Arroyo government forces who are duty bound to enforce the declaration or suspension
included therein has been committed by the person sought to be arrested.” and immediately put an end to the root cause of the emergency. Various acts
Same; Martial Law; Writ of Habeas Corpus; View that the constitutional carried out by government forces during martial law or suspension of the writ in
guarantees under the Bill of Rights remain fully operative and continue to accord the guise of protecting public safety may in reality amount to serious abuses of
the people its mantle of protection during a state of martial law.—The Constitution power and authority. Whatever the Court’s decision will be on the sufficiency of
now expressly declares, “A state of martial law does not suspend the operation of the factual basis of the President’s declaration or suspension does not preclude
the Constitution.” Neither does a state of martial law supplant the functioning of those aggrieved by such illegal acts from pursuing any course of legal action
the civil courts or legislative assemblies. Nor does it authorize the conferment of available to them. Therefore, the determination by this Court of the sufficiency of
jurisdiction on military courts and agencies over civilians where civil courts are the factual basis of the declaration or suspension is not essential to the resolution
able to function, or automatically suspend the writ. There is therefore no dispute of issues concerning the validity of related acts that government forces may have
that the constitutional guarantees under the Bill of Rights remain fully operative committed during the emergency.
and continue to accord the people its mantle of protection during a state of martial Same; Same; Same; View that the Constitution no longer allows imminent
law. In case the writ is also suspended, the suspension applies only to those danger of rebellion or invasion as a ground for the declaration or suspension, which
judicially charged for rebellion or offenses directly connected with invasion. the 1935 and 1973 Constitutions expressly permitted.—Consistent with the
Considering the non-suspension of the operation of the Constitution during a state framers’ intent to reformulate the Commander-in-Chief powers of the President,
of martial law, a declaration of martial law does not authorize warrantless arrests, the 1987 Constitution requires the concurrence of two conditions in declaring
searches and seizures, in derogation of Section 2, Article III of the Constitution. martial law or suspending the writ, namely, (1) an actual invasion or rebellion,
Same; Same; Same; View that even if Congress has not acted upon the and (2) public safety requires the exercise of such power. The Constitution no
President’s declaration or suspension, the Court may review the declaration or longer allows imminent danger of rebellion or invasion as a ground for the
suspension in an appropriate proceeding filed by any citizen.—It is clear, therefore, declaration or suspension, which the 1935 and 1973 Constitutions expressly
that the President’s power to declare martial law or suspend the writ is permitted.513
independent, separate, and distinct from any constitutionally mandated act to be
VOL. 668, MARCH 20, 2012 513
performed by either the Legislature or the Judiciary. It is neither joint nor
sequential with Congress’ power to revoke the declaration or suspension or to Fortun vs. Macapagal-Arroyo
extend it upon the initiative of the President. Accordingly, even if Congress has Same; Same; Same; View that only in case of actual invasion or rebellion,
not acted upon the President’s declaration or suspension, the Court may review when public safety requires it, may the President declare martial law or suspend
the declaration or suspension in an appropriate proceeding filed by any citizen. the writ.—Only in case of actual invasion or rebellion, when public safety requires
Otherwise stated, Congress’ inaction on the declaration or suspension is not it, may the President declare martial law or suspend the writ. In declaring martial
determinative of the Court’s exercise of its review power under Section 18, Article law and suspending the writ in Maguindanao in the absence of an actual rebellion,
VII of the Constitution. President Arroyo indisputably violated the explicit provisions of Section 18, Article
Same; Same; Same; Executive Department; View that the President has the VII of the Constitution.
sole and exclusive power to declare martial law or suspend the writ subject to review Same; Same; Same; Proclamation No. 1959; View that Proclamation No.
separately by Congress and the Supreme Court.—The President has the sole and 1959 was anchored on a non-existent rebellion; It cannot be justified on the basis of
exclusive power to a threatened, imminent, or looming rebellion, which ground was intentionally
512 deleted by the framers of the 1987 Constitution.—In sum, Proclamation No. 1959
512 SUPREME COURT REPORTS ANNOTATED was anchored on a non-existent rebellion. Based on the events before, during and
after the Maguindanao massacre, there was obviously no rebellion justifying the
Fortun vs. Macapagal-Arroyo declaration of martial law and suspension of the writ. The discovery of the
declare martial law or suspend the writ. This power of the President is Ampatuans’ private army and massive weaponry does not establish an armed
subject to review separately by Congress and the Supreme Court. Justice Mendoza public uprising aimed at overthrowing the government. Neither do the closure of
stresses, “Thus, Congress and this Court have separate spheres of competence. government offices and the reluctance of the local government officials and
They do not act ‘jointly and sequentially’ but independently of each other.” Father employees to report for work indicate a rebellion. The Constitution is clear. Only
in case of actual invasion or rebellion, when public safety requires it, can a state public safety is alleged to exist as a condition to make such declaration or
of martial law be declared or the privilege of the writ of habeas corpus be suspension.
suspended. Proclamation No. 1959 cannot be justified on the basis of a threatened, Same; Same; Same; View that it is one thing to declare a decree issued by the
imminent, or looming rebellion, which ground was intentionally deleted by the President as unconstitutional, and it is another to pronounce that she indisputably
framers of the 1987 Constitution. Considering the non-existence of an actual violated the Constitution.—To a certain
rebellion in Maguindanao, Proclamation No. 1959 is unconstitutional for lack of 515
factual basis as required under Section 18, Article VII of the Constitution for the VOL. 668, MARCH 20, 2012 515
declaration of martial law and suspension of the privilege of the writ of habeas
corpus. Fortun vs. Macapagal-Arroyo
VELASCO, JR., J., Dissenting Opinion: extent, I conform to Justice Carpio’s dissent as to the unconstitutionality of
Constitutional Law; Martial Law; Writ of Habeas Corpus; View that despite Proclamation No. 1959. To my mind, however, it is one thing to declare a decree
the lifting of the martial law and restoration of the privilege of the writ, the Court issued by the President as unconstitutional, and it is another to pronounce that
must take the bull by the horn to guide, explain and elucidate to the executive she indisputably violated the Constitution. Notably, the power to issue the
branch, the legislative branch, the bar, and more importantly the public on the subject decree is expressly granted the President. There is also compliance with
parameters of a declaration of martial law.—It is my view that, despite the lifting the report required after the issuance of said decree. However, the issuance of the
of the martial law and restoration of the privilege of the writ, the Court subject decree may not be sustained after due consideration of the circumstances
514 which may or may not support such decree.
PEREZ, J., Separate Opinion:
514 SUPREME COURT REPORTS ANNOTATED
Constitutional Law; Martial Law; Writ of Habeas Corpus; View that the
Fortun vs. Macapagal-Arroyo dismissal of the petition was more than by reason of their mootness but because the
must take the bull by the horn to guide, explain and elucidate to the Court action was already overdue.—I concur in the resulting dismissal of these
executive branch, the legislative branch, the bar, and more importantly the public petitions, more than by reason of their mootness but because I find our action
on the parameters of a declaration of martial law. Indeed, it is a well-settled rule overdue, it being my well-thought-out position that the constitutional authority of
that this Court may only adjudicate actual and current controversies. This is the Supreme Court to review the sufficiency of the factual basis of Proclamation
because the Court is “not empowered to decide moot questions or abstract No. 1959 has expired and is no more. Proclamation No. 1959 declaring martial law
propositions, or to declare principles or rules of law which cannot affect the result and suspending the privilege of the writ of habeas corpus in the Province of
as to the thing in issue in the case before it.” Nonetheless, this “moot and academic” Maguindanao was issued by then President Gloria Macapagal Arroyo on 4
rule admits of exceptions. December 2009. In compliance with the mandate of Section 18, Article VII of the
Same; Same; Same; View that there is neither any allegation nor proof that present Constitution, she submitted her Report to Congress on 6 December 2009
President Arroyo acted in bad faith when she declared martial law and suspended or “within forty-eight hours from the proclamation.”
the writ of habeas corpus in Maguindanao.—In the case at bar, there is neither Same; Same; Same; Commander-in-Chief Power; Judicial Review; View that
any allegation nor proof that President Arroyo acted in bad faith when she declared each and every exercise by the President of his commander-in-chiefship must, if
martial law and suspended the writ of habeas corpus in Maguindanao. There was review by the Supreme Court be asked and called for, be examined under the current
also no showing that there was a deliberate or intentional attempt on the part of events and the present affairs that determine the presence of the necessity of such
President Arroyo to break or dishonor the Constitution by issuing the assailed exercise.—Respectfully, I submit that each and every exercise by the President of
proclamation. On the contrary, what is extant from the records is that President his commander-in-chiefship must, if review by this Court be asked and called for,
Arroyo made such declaration and suspension on the basis of intelligence reports be examined under the current events and the present affairs that determine the
that lawless elements have taken up arms and committed public uprising against presence of the necessity of such exercise. All the decisions of the actors covered by
the government and the people of Maguindanao for the purpose of depriving the Section 18 of Article VII must be done within the tight and narrow time frames in
Chief Executive of her powers and prerogatives to enforce the laws of the land and the provision. These framed periods, I submit, emphasize the imperative for
to maintain public order and safety, to the great damage, prejudice and detriment currency of the decision that each must make, as indeed,
of the people in Maguindanao and the nation as a whole. 516
Same; Same; Same; Executive Department; View that the President has the 516 SUPREME COURT REPORTS ANNOTATED
discretion to make a declaration of martial law or suspension of the writ of habeas
corpus based on information or facts available or gathered by the President’s Fortun vs. Macapagal-Arroyo
office.—Significantly, the President has the discretion to make a declaration of the presidential proclamation, aside from having been subjected to
martial law or suspension of the writ of habeas corpus based on information or constitutional checks, has been given limited life.
facts available or gathered by the President’s office. It would be preposterous to Same; Same; Same; Judicial Review; View that the Supreme Court cannot
impose upon the President to be physically present at the place where a threat to now define for the future the “sufficiency of the factual basis” of the possibly coming
proclamations of martial law.—I respectfully submit that the Court cannot now
define for the future the “sufficiency of the factual basis” of the possibly coming In her report, President Arroyo said that she acted based on her finding that
proclamations of martial law. I cannot see how such a pre-determination can lawless men have taken up arms in Maguindanao and risen against the
prevent an unconstitutional imposition of martial law better than the requirement, government. The President described the scope of the uprising, the nature,
already constitutionalized, that the President must within forty-eight hours, quantity,
submit a report in person or in writing to Congress which can, by a majority of all 517
its members revoke, the imposition. VOL. 668, MARCH 20, 2012 517
PETITIONS to challenge the constitutionality of Proclamation No. 1959.
The facts are stated in the opinion of the Court. Fortun vs. Macapagal-Arroyo
Philip Sigfrid A. Fortun and John Michael S. Galauran for petitioners in G.R. and quality of the rebels’ weaponry, the movement of their heavily armed units in
No. 190293. strategic positions, the closure of the Maguindanao Provincial Capitol, Ampatuan
Teddy Esteban F. Rigoroso and Roderick R. Rabino for petitioner in G.R. No. Municipal Hall, Datu Unsay Municipal Hall, and 14 other municipal halls, and the
190294. use of armored vehicles, tanks, and patrol cars with unauthorized “PNP/Police”
Julius Garcia Matibag, Jobert Ilarde Pahilga, Ephraim B. markings.
Cortez and Voltaire B. Africa for petitioners in G.R. No. 190301. On December 9, 2009 Congress, in joint session, convened pursuant to Section
Nelson H. Loyola for petitioner in G.R. No. 190302. 18, Article VII of the 1987 Constitution to review the validity of the President’s
H. Harry L. Roque, Jr. and Joel Ruiz Butuyan for petitioners in G.R. No. action. But, two days later or on December 12 before Congress could act, the
190307. President issued Presidential Proclamation 1963, lifting martial law and restoring
Carlos P. Medina, Jr. for petitioners in G.R. No. 190380. the privilege of the writ of habeas corpus in Maguindanao.
Marvic M.V.F. Leonen for petitioners in G.R. No. 190356. Petitioners Philip Sigfrid A. Fortun and the other petitioners in G.R. 190293,
R.A.V. Saguisag lead counsel for N. Loyola. 190294, 190301,190302, 190307, 190356, and 190380 brought the present actions
517 to challenge the constitutionality of President Arroyo’s Proclamation 1959
affecting Maguindanao. But, given the prompt lifting of that proclamation before
VOL. 668, MARCH 20, 2012 517
Congress could review it and before any serious question affecting the rights and
Fortun vs. Macapagal-Arroyo liberties of Maguindanao’s inhabitants could arise, the Court deems any review of
ABAD, J.: its constitutionality the equivalent of beating a dead horse.
These cases concern the constitutionality of a presidential proclamation of Prudence and respect for the co-equal departments of the government dictate
martial law and suspension of the privilege of habeas corpus in 2009 in a province that the Court should be cautious in entertaining actions that assail the
in Mindanao which were withdrawn after just eight days. constitutionality of the acts of the Executive or the Legislative department. The
issue of constitutionality, said the Court in Biraogo v. Philippine Truth
The Facts and the Case Commission of 2010,1 must be the very issue of the case, that the resolution of such
issue is unavoidable.
The essential background facts are not in dispute. On November 23, 2009 The issue of the constitutionality of Proclamation 1959 is not unavoidable for
heavily armed men, believed led by the ruling Ampatuan family, gunned down and two reasons:
buried under shoveled dirt 57 innocent civilians on a highway in Maguindanao. In _______________
response to this carnage, on November 24 President Arroyo issued Presidential 1 G.R. Nos. 192935 & 193036, December 7, 2010, 637 SCRA 78, 147-148.
Proclamation 1946, declaring a state of emergency in Maguindanao, Sultan 519
Kudarat, and Cotabato City to prevent and suppress similar lawless violence in VOL. 668, MARCH 20, 2012 519
Central Mindanao.
Fortun vs. Macapagal-Arroyo
Believing that she needed greater authority to put order in Maguindanao and
secure it from large groups of persons that have taken up arms against the One. President Arroyo withdrew her proclamation of martial law and
constituted authorities in the province, on December 4, 2009 President Arroyo suspension of the privilege of the writ of habeas corpusbefore the joint houses of
issued Presidential Proclamation 1959 declaring martial law and suspending the Congress could fulfill their automatic duty to review and validate or invalidate the
privilege of the writ of habeas corpus in that province except for identified areas of same. The pertinent provisions of Section 18, Article VII of the 1987 Constitution
the Moro Islamic Liberation Front. state:
Two days later or on December 6, 2009 President Arroyo submitted her report “Sec. 18. The President shall be the Commander-in-Chief of all armed forces
to Congress in accordance with Section 18, Article VII of the 1987 Constitution of the Philippines and whenever it becomes necessary, he may call out such armed
which required her, within 48 hours from the proclamation of martial law or the forces to prevent or suppress lawless violence, invasion or rebellion. In case of
suspension of the privilege of the writ of habeas corpus, to submit to that body a invasion or rebellion, when the public safety requires it, he may, for a period not
report in person or in writing of her action. exceeding sixty days, suspend the privilege of the writ of habeas corpus or place
the Philippines or any part thereof under martial law. Within forty-eight hours
from the proclamation of martial law or the suspension of the privilege of writ _______________
of habeas corpus, the President shall submit a report in person or in writing to the 2 See Funa v. Ermita, G.R. No. 184740, February 11, 2010, 612 SCRA 308, 319.
Congress. The Congress, voting jointly, by a vote of at least a majority of all its 521
Members in regular or special session, may revoke such proclamation or VOL. 668, MARCH 20, 2012 521
suspension, which revocation shall not be set aside by the President. Upon the
initiative of the President, the Congress may, in the same manner, extend such Fortun vs. Macapagal-Arroyo
proclamation or suspension for a period to be determined by the Congress, if the Two. Since President Arroyo withdrew her proclamation of martial law and
invasion or rebellion shall persist and public safety requires it. suspension of the privilege of the writ of habeas corpus in just eight days, they
The Congress, if not in session, shall, within twenty-four hours following such have not been meaningfully implemented. The military did not take over the
proclamation or suspension, convene in accordance with its rules without any need operation and control of local government units in Maguindanao. The President
of a call. did not issue any law or decree affecting Maguindanao that should ordinarily be
x x x x” enacted by Congress. No indiscriminate mass arrest had been reported. Those who
Although the above vests in the President the power to proclaim martial law were arrested during the period were either released or promptly charged in court.
or suspend the privilege of the writ of habeas corpus, he shares such power with Indeed, no petition for habeas corpus had been filed with the Court respecting
the Congress. Thus: arrests made in those eight days. The point is that the President intended by her
1. The President’s proclamation or suspension is temporary, good for only 60 action to address an uprising in a relatively small and sparsely populated province.
days; In her judgment, the rebellion was localized and swiftly disintegrated in the face
2. He must, within 48 hours of the proclamation or suspension, report his of a determined and amply armed government presence.
action in person or in writing to Congress;520 In Lansang v. Garcia,3 the Court received evidence in executive session to
determine if President Marcos’ suspension of the privilege of the writ of habeas
520 SUPREME COURT REPORTS ANNOTATED
corpus in 1971 had sufficient factual basis. In Aquino, Jr. v. Enrile,4 while the
Fortun vs. Macapagal-Arroyo Court took judicial notice of the factual bases for President Marcos’ proclamation
3. Both houses of Congress, if not in session must jointly convene within 24 of martial law in 1972, it still held hearings on the petitions for habeas corpus to
hours of the proclamation or suspension for the purpose of reviewing its validity; determine the constitutionality of the arrest and detention of the petitioners. Here,
and however, the Court has not bothered to examine the evidence upon which
4. The Congress, voting jointly, may revoke or affirm the President’s President Arroyo acted in issuing Proclamation 1959, precisely because it felt no
proclamation or suspension, allow their limited effectivity to lapse, or extend the need to, the proclamation having been withdrawn within a few days of its issuance.
same if Congress deems warranted. Justice Antonio T. Carpio points out in his dissenting opinion the finding of the
It is evident that under the 1987 Constitution the President and the Congress Regional Trial Court (RTC) of Quezon City that no probable cause exist that the
act in tandem in exercising the power to proclaim martial law or suspend the accused before it committed rebellion in Maguindanao since the prosecution failed
privilege of the writ of habeas corpus. They exercise the power, not only to establish the elements of the crime. But the Court
sequentially, but in a sense jointly since, after the President has initiated the _______________
proclamation or the suspension, only the Congress can maintain the same based 3 149 Phil. 547; 42 SCRA 448 (1971).
on its own evaluation of the situation on the ground, a power that the President 4 158-A Phil. 1; 59 SCRA 183 (1974).
does not have. 522
Consequently, although the Constitution reserves to the Supreme Court the 522 SUPREME COURT REPORTS ANNOTATED
power to review the sufficiency of the factual basis of the proclamation or
suspension in a proper suit, it is implicit that the Court must allow Congress to Fortun vs. Macapagal-Arroyo
exercise its own review powers, which is automatic rather than initiated. Only cannot use such finding as basis for striking down the President’s proclamation
when Congress defaults in its express duty to defend the Constitution through and suspension. For, firstly, the Court did not delegate and could not delegate to
such review should the Supreme Court step in as its final rampart. The the RTC of Quezon City its power to determine the factual basis for the
constitutional validity of the President’s proclamation of martial law or suspension presidential proclamation and suspension. Secondly, there is no showing that the
of the writ of habeas corpus is first a political question in the hands of Congress RTC of Quezon City passed upon the same evidence that the President, as
before it becomes a justiciable one in the hands of the Court. Commander-in-Chief of the Armed Forces, had in her possession when she issued
Here, President Arroyo withdrew Proclamation 1959 before the joint houses of the proclamation and suspension.
Congress, which had in fact convened, could act on the same. Consequently, the The Court does not resolve purely academic questions to satisfy scholarly
petitions in these cases have become moot and the Court has nothing to review. interest, however intellectually challenging these are.5 This is especially true, said
The lifting of martial law and restoration of the privilege of the writ of habeas the Court in Philippine Association of Colleges and Universities v. Secretary of
corpus in Maguindanao was a supervening event that obliterated any justiciable Education,6 where the issues “reach constitutional dimensions, for then there
controversy.2 comes into play regard for the court’s duty to avoid decision of constitutional issues
unless avoidance becomes evasion.” The Court’s duty is to steer clear of declaring 524 SUPREME COURT REPORTS ANNOTATED
unconstitutional the acts of the Executive or the Legislative department, given the
assumption that it carefully studied those acts and found them consistent with the Fortun vs. Macapagal-Arroyo
fundamental law before taking them. “To doubt is to sustain.”7 over the case. The settled rule is that jurisdiction once acquired is not lost until
Notably, under Section 18, Article VII of the 1987 Constitution, the Court has the case has been terminated.
only 30 days from the filing of an appropriate proceeding to review the sufficiency The problem in this case is that the President aborted the proclamation of
of the factual basis of the proclamation of martial law or the suspension of the martial law and the suspension of the privilege of the writ of habeas corpus in
privilege of the writ of habeas corpus. Thus— Maguindanao in just eight days. In a real sense, the proclamation and the
“The Supreme Court may review, in an appropriate proceeding filed by any suspension never took off. The Congress itself adjourned without touching the
citizen, the sufficiency of the factual basis of the procla- matter, it having become moot and academic.
_______________ Of course, the Court has in exceptional cases passed upon issues that
5 Sec. Guingona, Jr. v. Court of Appeals, 354 Phil. 415, 426; 292 SCRA 402, ordinarily would have been regarded as moot. But the present cases do not present
413 (1998). sufficient basis for the exercise of the power of judicial review. The proclamation
6 97 Phil. 806, 811 (1955), citing Rice v. Sioux City, U.S. Sup. Ct. Adv. Rep., of martial law and the suspension of the privilege of the writ of habeas corpus in
May 23, 1955, Law Ed., Vol. 99, p. 511. this case, unlike similar Presidential acts in the late 60s and early 70s, appear
7 Board of Optometry v. Colet, 328 Phil. 1187, 1207; 260 SCRA 88, 105 (1996), more like saber-rattling than an actual deployment and arbitrary use of political
citing Drilon v. Lim, G.R. No. 112497, August 4, 1994, 235 SCRA 135, 140. power.
523 WHEREFORE, the Court DISMISSES the consolidated petitions on the
ground that the same have become moot and academic.
VOL. 668, MARCH 20, 2012 523
SO ORDERED.
Fortun vs. Macapagal-Arroyo Leonardo-De Castro, Brion, Peralta, Bersamin, Villarama,
mation of martial law or the suspension of the privilege of the writ of habeas Jr. and Mendoza, JJ., concur.
corpus or the extension thereof, and must promulgate its decision thereon Corona (C.J.), I join the dissent of Mr. Justice Velasco.
within thirty days from its filing.” (Emphasis supplied) Carpio, J., See Dissenting Opinion.
More than two years have passed since petitioners filed the present actions to Velasco, Jr., J., Please see Dissenting Opinion.
annul Proclamation 1959. When the Court did not decide it then, it actually opted Del Castillo, J., On Leave.
for a default as was its duty, the question having become moot and academic. Perez, J., Please see separate opinion.
Justice Carpio of course points out that should the Court regard the powers of Sereno, J., I join the dissent of J. Carpio.
the President and Congress respecting the proclamation of martial law or the Reyes, J., I join the dissent of J. A.T. Carpio.
suspension of the privilege of the writ of habeas corpus as sequential or joint, it Perlas-Bernabe, J., I join the dissent of J. Carpio.
would be impossible for the Court to exercise its power of review within the 30 days 525
given it. VOL. 668, MARCH 20, 2012 525
But those 30 days, fixed by the Constitution, should be enough for the Court to
fulfill its duty without pre-empting congressional action. Section 18, Article VII, Fortun vs. Macapagal-Arroyo
requires the President to report his actions to Congress, in person or in writing,
within 48 hours of such proclamation or suspension. In turn, the Congress is DISSENTING OPINION
required to convene without need of a call within 24 hours following the President’s
proclamation or suspension. Clearly, the Constitution calls for quick action on the CARPIO, J.:
part of the Congress. Whatever form that action takes, therefore, should give the I dissent.
Court sufficient time to fulfill its own mandate to review the factual basis of the
proclamation or suspension within 30 days of its issuance. The Cases
If the Congress procrastinates or altogether fails to fulfill its duty respecting
the proclamation or suspension within the short time expected of it, then the Court These are consolidated petitions for the writs of certiorari and prohibition
can step in, hear the petitions challenging the President’s action, and ascertain if challenging the constitutionality of Presidential Proclamation No. 1959, which
it has a factual basis. If the Court finds none, then it can annul the proclamation declared a state of martial law and suspended the privilege of the writ of habeas
or the suspension. But what if the 30 days given it by the Constitution proves corpus in the Province of Maguindanao, except for identified areas of the Moro
inadequate? Justice Carpio himself offers the answer in his dissent: that 30-day Islamic Liberation Front.
period does not operate to divest this Court of its jurisdiction
524 The Antecedents
In the morning of 23 November 2009, fifty-seven (57) innocent civilians met 12. Walida Ali Kalim – Relative
their tragic and untimely death in a gruesome massacre unequaled in recent 13. Atty. Concepcion Brizuela – Lawyer
history,1 considered to be 14. Atty. Cynthia Oquendo Ogano – Lawyer
_______________ 15. Cataleno Oquendo – Father of Atty. Cynthia Oquendo
1 Presidential Adviser for Mindanao Jesus Dureza’s statement reported in 16. Marife Montano – Saksi News, Gensan
Philippine Daily Inquirer on 23 November 2009 17. Alejandro Bong Reblando – Manila Bulletin, Gensan
(http://newsinfo.inquirer.net/breakingnews/nation/view/20091123-237934/Wife- 18. Mc Delbert “Mac Mac” Areola – UNTV Gensan
of-gubernatorial-bet-35-killed-in-Maguindanao [accessed on 4 November 2011], 19. Rey Marisco – Periodico Ini, Koronadal City
Wife of gubernatorial bet, 35 killed in Maguindanao Palace adviser calls for state 20. Bienvenido Jun Lagarta – Prontierra News, Koronadal City
of emergency) and in Philippine Star on 24 November 2009 21. Napoleon Salaysay – Mindanao Gazette
(http://www.philstar.com/ 22. Eugene Depillano – UNTV Gen San
article.aspx?articleid=526314 [accessed on 4 November 2011]; 23. Rosell Morales – News Focus
Maguindanaomassacre). 24. Arturo Betia – Periodico Ini, Gen San
The mass murder of the journalists was tagged “as the darkest point of 528
democracy and free press in this recent time.” (Statement of NUJP Cebu Chapter 528 SUPREME COURT REPORTS ANNOTATED
President Rico Lucena reported in philstar.com with title Maguindanao death toll
now 46: Emergency rule in two provinces Fortun vs. Macapagal-Arroyo
(http://www.philstar.com/article.aspx?articleid=526616 [accessed on 4 November In its Consolidated Comment dated 14 December 2009, the Office of the
2011]). Solicitor General (OSG), representing public respondents, narrated the harrowing
The massacre was considered “one of the deadliest single events for the press events which unfolded on that fateful day of 23 November 2009, to wit:
in memory" and the Philippines the world’s worst place to be a journalist, according xxxx
to international press freedom watchdog Committee to Protect Journalists (CPJ). 3. Vice Mayor Mangudadatu confirmed having received reports that his political
(http://www. rivals (Ampatuans) were planning to kill him upon
526 _______________
25. Noel Decena – Periodico Ini
526 SUPREME COURT REPORTS ANNOTATED
26. John Caniba – Periodico Ini
Fortun vs. Macapagal-Arroyo 27. Junpee Gatchalian – DXGO, Davao City
the Philippines’ worst case of election-related violence. Brutally killed were female 28. Victor Nunez – UNTV Gen San
family members of then Buluan Vice Mayor Esmael “Toto” Mangudadatu 29. Andres Teodoro –Central Mindanao Inquirer
(Mangudadatu), including his wife and sisters, and members of the press who were 30. Romeo Capelo – Midland Review, Tacurong City
part of a convoy on the way to Shariff Aguak in Maguindanao. Mangudadatu’s wife 31. Joy Duhay – Gold Star Daily
was bringing with her Mangudadatu’s certificate of candidacy for Governor of 32. Ronnie Perante – Gold Star Daily, Koronadal City
Maguindanao for filing with the Provincial Office of the Commission on Elections 33. Benjie Adolfo – Gold Star Daily, Koronadal City
in Shariff Aguak. Five of the victims were not part of the convoy but happened to 34. Ian Subang – Socsargen Today, Gen San
be traveling on the same highway.2 35. Joel Parcon – Prontiera News, Koronadal City
_______________ 36. Robello Bataluna – Gold Star Daily, Koronadal City
gmanews.tv/story/177821/the-ampatuan-massacre-a-map-and-timeline 37. Lindo Lipugan – Mindanao Daily Gazette, Davao City
[accessed on 4 November 2011]) 38. Ernesto Maravilla – Bombo Radyo, Koronadal City
2 Fifty-five of the casualties were identified as follows: 39. Henry Araneta – Radio DZRH, Gen San
1. Bai Genelyn T. Mangudadatu – Wife of Mangudadatu 40. Fernando Razon – Periodico Ini, Gen San
2. Bai Eden Mangudadatu – Sister/Vice Mayor, Mangudadatu, Maguindanao 41. Hannibal Cachuela – Punto News, Koronadal City
3. Pinky Balaiman – Cousin of Mangudadatu 42. Lea Dalmacio – Socsargen News, Gensan
4. Mamotavia Mangudadatu – Aunt 43. Marites Cablitas – News Focus, Gensan
5. Bai Farida Mangudadatu – Youngest sister 44. Gina Dela Cruz – Saksi News, Gensan
6. Rowena Ante Mangudadatu – Relative 45. Anthony Ridao – Government employee
7. Faridah Sabdula – Sister 46. Mariam Calimbol – Civilian
8. Soraida Vernan – Cousin 47. Norton “Sidic” Edza – Driver
9. Raida Sapalon Abdul – Cousin 48. Jephon C. Cadagdagon – Civilian
10. Rahima Puto Palawan – Relative 49. Abdillah Ayada – Driver
11. Lailan “Ella” Balayman – Relative 50. Joselito Evardo – Civilian
51. Cecille Lechonsito – Civilian the Maguindanao Provincial Government parked nearby with its engine still
52. Wilhelm Palabrica – Government Employee running were found at the site. When the graves were dug up by the soldiers,
53. Mercy Palabrica – Government Employee twenty four (24) dead bodies were found in the first grave; six (6) dead bodies with
54. Daryll Vincent Delos Reyes – Government Employee three (3) vehicles, particularly a Toyota Vios with the seal of the Tacurong City
55. Eduardo “Nonie” Lechonsito – Government Employee Government, a Tamaraw FX and an L300 owned by the media outfit UNTV were
528 found in the second grave; and five (5) more dead bodies were recovered from the
528 SUPREME COURT REPORTS ANNOTATED third grave, yielding 35 buried dead bodies and, together with other cadavers,
resulted in a total of fifty seven (57) fatalities.
Fortun vs. Macapagal-Arroyo 8. x x x
his filing of a certificate of candidacy (COC) for the gubernatorial seat in 9. Examination of the bodies revealed that most, if not all, of the female victims’
Maguindanao. Believing that the presence of women and media personalities pants were found unzipped, and their sexual organs mutilated and mangled. Five
would deter any violent assault, he asked his wife and female relatives to file his (5) of them were tested positive for traces of semen, indicative of sexual abuse while
COC and invited several media reporters to cover the event. some of the victims were shot in the genital area. The genitalia of Genelyn
4. At around 10 a.m., the convoy stopped at a designated PNP checkpoint along Mangudadatu was lacerated four (4) times, and blown off by a gun fire, and her
the highway of Ampatuan, Maguindanao manned by the Maguindanao 1508th body horrifyingly mutilated. Two of the women killed were pregnant, while
Provincial Mobile Group, particularly, Eshmail Canapia and Takpan Dilon. While another two were lawyers. Twenty-nine (29) of the casualties were media
at a stop, they were approached by about one hundred (100) armed men. The personnel. Almost all gun shot injuries were on the heads of the victims, rendering
armed men pointed their weapons at the members of the 1508th Provincial Mobile them unrecognizable albeit two (2) bodies remain unidentified. Those found in the
Group manning the check point, and threatened them to refrain from interfering. graves were coarsely lumped like trash, and some of the victims were found
The members of the convoy were then ordered to alight from their vehicles and to hogtied. All the dead bodies bear marks of despicable torture, contempt and
lie face down on the ground, as the armed men forcibly took their personal outrageous torment.3
belongings. Subsequently, all members of the convoy were ordered to board their A day after the carnage, on 24 November 2009, former President Gloria
vehicles. They were eventually brought by the armed men to the hills in Barangay Macapagal-Arroyo (President Arroyo) issued Proclamation No. 1946, declaring a
Masalay, Ampatuan, about 2.5 kilometers from the checkpoint. state of emergency in the provinces of Maguindanao and Sultan Kudarat, and in
5. At about the same time, Vice Mayor Mangudadatu received a call from his wife the City of Cotabato, “to prevent and suppress the occurrence of
Genelyn who, in a trembling voice, told him that a group of more or less 100 armed _______________
men stopped their convoy, and that Datu Unsay Mayor Andal Ampatuan, Jr. was 3 Rollo (G.R. No. 190293), pp. 105-109.
walking towards her, and was about to slap her face. After those last words were 530
uttered, the phone line went dead and her cellphone could not be contacted any
530 SUPREME COURT REPORTS ANNOTATED
longer. Alarmed that his wife and relatives, as well as the media personalities were
in grave danger, Vice Mayor Mangudadatu immediately reported the incident to Fortun vs. Macapagal-Arroyo
the Armed Forces of the Philippines. similar other incidents of lawless violence in Central Mindanao.” The full text of
6. In the afternoon of the same day, soldiers—aboard two army trucks led by Lt. Proclamation No. 1946 reads:
Col. Rolando Nerona, Head of the Philippine Army’s 64th Infantry Battalion— DECLARING A STATE OF EMERGENCY IN CENTRAL MINDANAO
went to the town of Ampatuan to confirm the report. At around 3 p.m., they passed WHEREAS, on November 23, 2009, several persons, including women and
by the checkpoint along the highway in Ampatuan manned by the 1508th members of media were killed in a violent incident which took place in Central
Provincial Mobile Group and asked whether they were aware of the reported Mindanao;
abduction. Members of the 1508th Provincial Mobile Group denied having WHEREAS, there is an urgent need to prevent and suppress the occurrence of
knowledge of what they have witnessed at around 10 in the morning purportedly similar other incidents of lawless violence in Central Mindanao;
out of fear of retaliation from the powerful Ampatuan clan. Nevertheless, P/CI NOW, THEREFORE, I, GLORIA MACAPAGAL-ARROYO, President of the
Sukarno Adil Dicay, the head of the Mobile Group, instructed P/INSP Diongon to Republic of the Philippines, by virtue of the powers vested in me by the
accompany the military on foot Constitution and by law, do hereby proclaim, as follows:
529 SECTION 1. The Provinces of Maguindanao and Sultan Kudarat and the
VOL. 668, MARCH 20, 2012 529 City of Cotabato are hereby placed under a state of emergency for the purpose of
preventing and suppressing lawless violence in the aforesaid jurisdiction.
Fortun vs. Macapagal-Arroyo SECTION 2. The Armed Forces of the Philippines (AFP) and the Philippine
patrol as they conduct their operation relative to the reported abduction. National Police (PNP) are hereby ordered to undertake such measures as may be
7. Upon reaching Barangay Masalay, Ampatuan, the soldiers on foot patrol found allowed by the Constitution and by law to prevent and suppress all incidents of
dead bodies, bloodied and scattered on the ground and inside the four (4) vehicles lawless violence in the said jurisdiction.
used by the convoy. Three (3) newly covered graves and a back hoe belonging to
SECTION 3. The state of emergency covering the Provinces of Maguindanao system and other government mechanisms in the province are not functioning,
and Sultan Kudarat and the City of Cotabato shall remain in force and effect until thus endangering public safety;
lifted or withdrawn by the President.4 WHEREAS, the Implementing Operational Guidelines of the GRP-MILF
On 4 December 2009, President Arroyo issued Proclamation No. 1959, Agreement on the General Cessation of Hostilities dated 14 November 1997
declaring martial law and suspending the privilege of the writ of habeas provides that the following is considered a prohibited hostile act: “x x x
corpus (writ) in the Province of Maguindanao, except for the identified areas of the establishment of checkpoints except those necessary for the GRP’s enforcement
Moro Islamic Liberation Front (MILF). The full text of Proclama- and maintenance of peace and order; and, for the defense and security of the MILF
_______________ in their identified areas, as jointly determined by the GRP and MILF. x x x”
4 Id., at p. 185. NOW, THEREFORE, I, GLORIA MACAPAGAL-ARROYO, President of the
531 Republic of the Philippines, by virtue of the powers vested in me by the
VOL. 668, MARCH 20, 2012 531 Constitution and by law, do hereby proclaim, as follows:
SECTION 1. There is hereby declared a state of martial law in the province
Fortun vs. Macapagal-Arroyo of Maguindanao, except for the identified areas of the Moro Islamic Liberation
tion No. 1959, signed by President Arroyo and attested by Executive Secretary Front as referred to in the Implementing Operational Guidelines of the GRP-MILF
Eduardo Ermita, reads: Agreement on the General Cessation of Hostilities.
PROCLAMATION NO. 1959 SECTION 2. The privilege of the writ of habeas corpus shall likewise be
PROCLAIMING A STATE OF MARTIAL LAW AND SUSPENDING THE suspended in the aforesaid area for the duration of the state of martial law. 6
PRIVILEGE OF THE WRIT OF HABEAS CORPUS IN THE PROVINCE OF On 6 December 2009, President Arroyo submitted her Report to Congress in
MAGUINDANAO, EXCEPT FOR CERTAIN AREAS accordance with the provision in Section 18, Article VII of the 1987 Constitution,
WHEREAS, Proclamation No. 1946 was issued on 24 November 2009 declaring which states that “within forty-eight hours from the proclamation of martial law
a state of emergency in the provinces of Maguindanao, Sultan Kudarat and the or the suspension of the privilege of the writ of habeas corpus, the President shall
City of Cotabato for the purpose of preventing and suppressing lawless violence in submit a report in person or in writing to the Congress.” In her Report, President
the aforesaid areas; Arroyo presented the following justifications for imposing martial law and
WHEREAS, Section 18, Art.VII of the Constitution provides that “x x x In case suspending the writ in Maguindanao, to wit:
of invasion or rebellion, when the public safety requires it, (the President) may, for Pursuant to the provision of Section 18, Article VII of the 1987 Constitution, the
a period not exceeding sixty days, suspend the privilege of the writ of habeas President of the Republic of the Philippines is submitting the hereunder Report
corpus or place the Philippines or any part thereof under martial law. x x x” relative to Proclamation No. 1959 “Proclaiming a State of Martial Law and
WHEREAS, R.A. No. 69865 provides that the crime of rebellion or insurrection Suspending the Privilege of the
is committed by rising publicly and taking arms against the Government for the _______________
purpose of x x x depriving the Chief Executive or the Legislature, wholly or 6 Rollo (G.R. No. 190293), pp. 186-187.
partially, of any of their powers or prerogatives.” 533
WHEREAS, heavily armed groups in the province of Maguindanao have
VOL. 668, MARCH 20, 2012 533
established positions to resist government troops, thereby depriving the Executive
of its powers and prerogatives to enforce the laws of the land and to maintain Fortun vs. Macapagal-Arroyo
public order and safety; Writ of Habeas Corpus in the Province of Maguindanao, except for Certain Areas,”
WHEREAS, the condition of peace and order in the province of Maguindanao which she issued on 04 December 2009, as required by public safety, after finding
has deteriorated to the extent that the local judicial that lawless elements have taken up arms and committed public uprising
_______________ against the duly constituted government and against the people of
5 Should be Republic Act No. 6968, which is “An Act Punishing the Crime of Maguindanao, for the purpose of removing from the allegiance to the
Coup D’état by Amending Articles 134, 135 and 136 of Chapter One, Title Three of Government or its laws, the Province of Maguindanao, and likewise
Act Numbered Thirty-Eight Hundred and Fifteen, Otherwise Known as the depriving the Chief Executive of her powers and prerogatives to enforce
Revised Penal Code, and for Other Purposes.” Republic Act No. 6986 is titled “An the laws of the land and to maintain public order and safety, to the great
Act Establishing a High School in Barangay Dulop, Municipality of Dumingag, damage, prejudice and detriment of the people in Maguindanao and the
Province of Zamboanga Del Sur, to be Known as the Dulop High School, and nation as a whole.
Appropriating Funds Therefor.” xxx
532 The capture of identified leader Mayor Andal Ampatuan, Jr. would have resulted
532 SUPREME COURT REPORTS ANNOTATED in the expeditious apprehension and prosecution of all others involved in the
gruesome massacre, but the situation proved the contrary. The Ampatuan group
Fortun vs. Macapagal-Arroyo backed by formidable group of armed followers, have since used their strength and
political position to deprive the Chief Executive of her power to enforce the law and
to maintain public order and safety. More importantly, a separatist group based in Fortun vs. Macapagal-Arroyo
Maguindanao has joined forces with the Ampatuans for this purpose. These are VIII. More or less 100 armed rebels led by one of the identified leaders in the
the facts: massacre have been sighted at the quarry of Barangay Lagpan, boundary of Rajah
1. Local government offices in the province of Maguindanao were closed and Buayan and Sultan Sa Barongis. The group is armed with one (1) 90RR, one (1)
ranking local government officials refused to discharge their functions, which cal 50 LMG, two (2) cal 30 LMG, two (2) 60 mm mortar and assorted rifles.
hindered the investigation and prosecution team from performing their tasks; The strength of the rebels is itself estimated to be around 800 with about 2,000
2. The Local Civil Registrar of Maguindanao refused to accept the registration of firearms (Fas). These forces are concentrated in the following areas in
the death certificates of the victims purportedly upon the orders of Andal Maguindanao which are apparently also their political stronghold:
Ampatuan Sr.; xxx
3. The local judicial system has been crippled by the absence or non-appearance The existence of this armed rebellion is further highlighted by the recent recovery
of judges of local courts, thereby depriving the government of legal remedies in of high powered firearms and ammunitions from the 400 security escorts of Datu
their prosecutorial responsibilities (i.e. issuance of warrants of searches, seizure Andal Ampatuan Sr.
and arrest). While the Supreme Court has designated an Acting Presiding Judge xxx
from another province, the normal judicial proceedings could not be carried out in Indeed, the nature, quantity and quality of their weaponry, the movement
view of threats to their lives or safety, prompting government to seek a change of of heavily armed rebels in strategic positions, the closure of the
venue of the criminal cases after informations have been filed.534 Maguindanao Provincial Capitol, Ampatuan Municipal Hall, Datu Unsay
534 SUPREME COURT REPORTS ANNOTATED Municipal Hall, and fourteen other municipal halls, and the use of
Fortun vs. Macapagal-Arroyo armored vehicles, tanks and patrol cars with unauthorized “PNP/Police”
markings, all together confirm the existence of armed public uprising for
Duly verified information disclosed that the Ampatuan group is behind the closing
the political purpose of:
down of government offices, the refusal of local officials to discharge their functions
(1) removing allegiance from the national government of the Province
and the simultaneous absence or non-appearance of judges in local courts.
of Maguindanao; and,
Detailed accounts pertaining to the rebel armed groups and their active
movements in Maguindanao have been confirmed: (2) depriving the Chief Executive of her powers and prerogatives to
(I) As of November 29, 2009, it is estimated that there are about 2,413 armed enforce the laws of the land and to maintain public order and safety.
While the government is at present conducting legitimate operations to
combatants coming from the municipalities of Shariff Aguak, Datu Unsay, Datu
address the on-going rebellion, public safety still requires the continued
Salibo, Mamasapano, Datu Saudi Ampatuan (Dikalungan), Sultan Sa Barungis,
implementation of martial law and the suspension of the privilege of the
Datu Piang, Guindulungan, and Talayan, who are in possession of around 2,000
writ of habeas corpus in the Province of Maguindanao until the time that
firearms/armaments.
(II) The Ampatuan group has consolidated a group of rebels consisting of 2,413 such rebellion is completely quelled.7 (Emphasis supplied)
heavily armed men, with 1,160 of them having been strategically deployed in _______________
Maguindanao. Validated information on the deployment of rebels are as follows: 7 Id., at pp. 163-164, 173-177, 182.
I. Around five hundred (500) armed rebels with 2 “Sanguko” armored vehicles 536
are in offensive position in the vicinity of Kakal, Ampatuan, Dimampao, 536 SUPREME COURT REPORTS ANNOTATED
Mamasapano and Sampao Ampatuan. Fortun vs. Macapagal-Arroyo
II. A group with more or less 200 armed rebels has moved from Old Maganoy into In the meantime, the present petitions were filed impugning the
an offensive position. constitutionality of Proclamation No. 1959.
III. More or less 80 fully armed rebels remain in Tuka, Mamasapano. 1. G.R. No. 190293 is a petition “for the issuance of a temporary restraining
IV. More or less 50 fully armed rebels led by a former MNLF Commander are in order and writs of prohibition and preliminary prohibitory injunction (1) to
offensive position in Barangay Baital, Rajah Buayan. declare Proclamation No. 1959 or any act, directive or order arising from or
V. More or less 70 fully armed rebels with two (2) M60 LMG remain in offensive connected to it as unconstitutional, and (2) to enjoin public respondents
position in the vicinity of Barangay Kagwaran, Barangay Iginampong, Datu Unsay from further enforcing the same.”
(right side of Salvo-General Santos City national highway). 2. G.R. No. 190294 is a petition for certiorari assailing the constitutionality of
VI. More or less 60 fully armed rebels with four (4) M60 LMG remain in offensive Proclamation No. 1959 “for gross insufficiency of the factual basis in
position in the vicinity of Kinugitan, the upper portion of Barangay Maitumaig, proclaiming a state of martial law and suspending the [writ] in the Province
Datu Unsay. of Maguindanao.” It prayed for the issuance of a writ of prohibition under
VII. Kagui Akmad Ampatuan was sighted in Sultan Sa Barongis with 400 armed Section 2 of Rule 65 to enjoin and prohibit respondents from enforcing
rebels. Locals heard him uttered “PATAYAN NA KUNG PATAYAN.”535 Proclamation No. 1959.
VOL. 668, MARCH 20, 2012 535
3. G.R. No. 190301 is a petition seeking “the nullification of Proclamation No. PROCLAIMING THE TERMINATION OF THE STATE OF MARTIAL LAW AND
1959, proclaiming a state of martial law and suspending the [writ] in the THE RESTORATION OF THE PRIVILEGE OF THE WRIT OF HABEAS
province of Maguindanao, except for certain areas, as it is patently illegal CORPUS IN THE PROVINCE OF MAGUINDANAO
and unconstitutional for lack of any factual basis.” WHEREAS, Proclamation No. 1946 was issued on 24 November 2009 declaring
4. G.R. No. 190302 is a petition for certiorari to declare Proclamation No. a state of emergency in the provinces of Maguindanao, Sultan Kudarat and the
1959 as null and void for being unconstitutional, and for prohibition to City of Cotabato for the purpose of preventing and suppressing lawless violence in
enjoin respondents from further actions or proceedings in enforcing or the aforesaid areas;
implementing Proclamation No. 1959. WHEREAS, by virtue of the powers granted under Section 18, Article VII of
5. G.R. No. 190307 is a petition for certiorari, prohibition, and mandamus the Constitution, the President of the Philippines promulgated Proclamation No.
with a prayer for a preliminary prohibitory injunction and/or a temporary 1959 on December 4, 2009, proclaiming a state of martial law and suspending the
restraining order, and/or a petition for review pursuant to Article VII, privilege of the writ of Habeas Corpus in the province of Maguindanao, except for
Section 18, paragraph 3 of the 1987 Constitution, asking the Court to certain areas;
declare that then Executive Secretary Eduardo Ermita committed grave WHEREAS, the Armed Forces of the Philippines and the Philippine National
abuse of discre- Police have reported that over six hundred (600) persons who allegedly rose
537 publicly and took up arms against the Government have surrendered or have been
VOL. 668, MARCH 20, 2012 537 arrested or detained;
WHEREAS, the Armed Forces of the Philippines and the Philippine National
Fortun vs. Macapagal-Arroyo Police have reported that the areas where heavily armed groups in the province of
tion amounting to lack or excess of jurisdiction when he signed, in the name Maguindanao established positions to resist government troops have been cleared;
of President Arroyo, Proclamation No. 1959. The petition also prayed for WHEREAS, the court and prosecutors’ offices of Cotabato City have resumed
the issuance of a Temporary Restraining Order and/or preliminary normal working hours, paving the way for the criminal justice system in
prohibitory injunction, prohibiting respondents, and anyone acting under Maguindanao to be restored to normalcy;
their authority, stead, or behalf, from implementing Proclamation No. 1959 WHEREAS, the Vice-Governor of the Autonomous Region of Muslim Mindanao
during the pendency of the case. has assumed as Acting Governor, paving the way for the restoration of the
6. G.R. No. 190356 is a petition for prohibition, with an application for the functioning of government mechanisms in the province of Maguindanao;
issuance of a temporary restraining order and/or a writ of preliminary NOW, THEREFORE, I, GLORIA MACAPAGAL-ARROYO, President of the
injunction, assailing the constitutionality and the sufficiency of the factual Republic of the Philippines, by virtue of the powers vested in me by the
basis of Proclamation No. 1959, declaring a state of martial law in the Constitution and by law, do hereby revoke Proclamation No. 1959 and proclaim
province of Maguindanao (except for identified areas of the MILF) and the termination of the state of martial law and the restoration of the privilege of
suspending the writ in the same area. the writ of habeas
7. G.R. No. 190380 is a petition for certiorari assailing the validity of 539
Proclamation No. 1959, declaring a state of martial law in the province of
VOL. 668, MARCH 20, 2012 539
Maguindanao, except for the identified areas of the MILF, and suspending
the writ in the same area. Fortun vs. Macapagal-Arroyo
On 9 December 2009, Congress convened in joint session pursuant to Section corpus in the province of Maguindanao; provided that Proclamation No. 1946 shall
18, Article VII of the 1987 Constitution, which provides, “The Congress, if not in continue to be in force and effect.8
session, shall, within twenty-four hours following such proclamation [of martial In the Resolutions dated 8 and 15 December 2009,9 the Court consolidated the
law] or suspension [of the writ], convene in accordance with its rules without need petitions and required the Office of the Solicitor General and the respondents to
of a call.” comment on the petitions.
Meanwhile, eight days after the declaration of martial law, on 12 December In a Resolution dated 12 January 2010, the Court resolved “to appoint as amici
2009, President Arroyo issued Proclamation No. 1963 lifting martial law and curiae Justice Vicente Mendoza, Senator Joker Arroyo, and Father Joaquin
restoring the writ in Maguindanao. The full text of Proclamation No. 1963, signed Bernas, [S.J.] and request them to submit their respective Amicus Brief on the
by President Arroyo and attested by Executive Secretary Eduardo Ermita, questions to be addressed by the parties.”10
reads:539 Meanwhile, on 9 December 2009, an Information for rebellion was filed before
VOL. 668, MARCH 20, 2012 539 the Regional Trial Court, Branch 15, Cotabato City (RTC-Cotabato), against
Ampatuan, et al.11 The information reads:
Fortun vs. Macapagal-Arroyo “That on or about 27th day of November, 2009, and continuously thereafter,
PROCLAMATION NO. 1963 until the present time, in Maguindanao Province
_______________
8 Rollo (G.R. No. 190293), pp. 190-191.
9 Rollo (G.R. No. 190293), pp. 83-84; Rollo (G.R. No. 190356), p. 55. 541
10 Rollo (G.R. No. 190293), p. 407. VOL. 668, MARCH 20, 2012 541
11 The accused are: Datu Andal Ampatuan, Sr., Datu Zaldy Uy Ampatuan,
Datu Akmad Tato Ampatuan, Datu Anwar Ampatuan, and Datu Sajid Islam Uy Fortun vs. Macapagal-Arroyo
Ampatuan, as persons who allegedly promoted, maintained or headed the time to go over the resolution finding probable cause against the accused Datu
rebellion; and Kusain Akmad Sakilan, Jovel Vista Lopez, Rommy Gimba Mamay, Andal Ampatuan, Sr., [et al.].”
Sammy Duyo Villanueva, Ibrahim Tukya Abdulkadir, Samil Manalo Mindo, Goldo On 1 February 2010, the Regional Trial Court of Quezon City received the
B. Ampatuan, Amaikugao Obab Dalgan, Billy Cabaya Gabriel, Jr., Abdulla records of the case, pursuant to the Supreme Court’s En Banc Resolution, dated
Kaliangat Ampatuan, Moneb Smair Ibrahim, Umpa Ugka Yarka, Manding 12 January 2010, which ordered the transfer of venue of the rebellion case to
Abdulkadir, Dekay Idra Ulama, Kapid Gabriel Cabay, Koka Batong Managilid, Quezon City. The case, docketed as Criminal Case No. Q-10-162667 and
Sammy Ganda Macabuat, Duca Lendungan Amban, Akmad Abdullah Ulilisen and entitled People of the Philippines v. Datu Andal Ampatuan, Sr., et al., was raffled
several John Does, as participants or the persons executing the commands of to Branch 77 of the Regional Trial Court of Quezon City (RTC-Quezon City) on 2
others in a rebellion or insurrection. (RTC-Quezon City Order dated 26 March February 2010.
2010, pp. 3-4). On 3 February 2010, the accused filed an Urgent Motion praying for the
540 issuance of an order suspending the transfer of custody of all the accused pending
the resolution of their motion for judicial determination of probable cause.
540 SUPREME COURT REPORTS ANNOTATED
On 26 March 2010, the RTC-Quezon City dismissed the charge of rebellion for
Fortun vs. Macapagal-Arroyo lack of probable cause, to wit:
and within the jurisdiction of this Honorable Court, accused Datu Andal “After a careful and judicious scrutiny of the evidence forming part of the
Ampatuan, Sr., Datu Zaldy Uy Ampatuan, Datu Akmad Tato Ampatuan, Datu records and those adduced by the prosecution during the hearing on the motion for
Anwar Ampatuan and Datu Sajid Islam Uy Ampatuan as heads of the rebellion, judicial determination of probable cause, the Court is convinced that there exist[s]
conspiring, confederating and cooperating with each other, as well as with the no probable cause to hold under detention and to indict the accused for rebellion.
other accused as participants or executing the commands of others in the rebellion xxxx
and also with other John Does whose whereabouts and identities are still Rebellion under Article 134 of the Revised Penal Code is committed—
unknown, the said accused, who are heads of the rebellion, did then and there [B]y rising publicly and taking arms against the Government for the purpose
willfully, unlawfully and feloniously help, support, promote, maintain, cause, of removing from the allegiance to said Government or its laws, the territory of the
direct and/or command their co-accused who are their followers to rise publicly and Republic of the Philippines or any part thereof, or any body of land, naval, or other
take arms against the Republic of the Philippines, or otherwise participate in such armed forces or depriving the Chief Executive or the Legislature, wholly or
armed public uprising, for the purpose of removing allegiance to the government partially, of any of their powers or prerogatives.
or its laws, the territory of the Republic of the Philippines or any part thereof or The elements of the offense are:
depriving the Chief Executive of any of her powers or prerogatives as in fact they 1. That there be a (a) public uprising and (b) taking arms against the
have been massing up armed men and organizing assemblies, as a necessary Government; and
means to commit the crime of rebellion, and in furtherance thereof, have then and 2. That the purpose of the uprising or movement is either—542
there committed acts preventing public prosecutors from being available to 542 SUPREME COURT REPORTS ANNOTATED
conduct inquest and preliminary investigations. There were massive formations of
numerous armed civilians supported by armored vehicles and under the command Fortun vs. Macapagal-Arroyo
of the Ampatuans who have formed a private army to resist government troops; (a)to remove from the allegiance to said Government or its laws:
that the local provincial government of Maguindanao could not function with their (1) the territory of the Philippines or any part thereof; or
employees going on mass leave and their respective offices were closed and not (2) any body of land, naval, or other armed forces; or
functioning. The Regional Trial Courts of the area are not functioning, refused to (b) to deprive the Chief Executive or Congress, wholly or partially, of any of
accept the application for search warrants for violation of PD 1866 to authorize the their powers and prerogatives.
search of the properties of the heads of the rebellion; and that there was undue xxxx
delay in the issuance of court processes despite the exigency of the situation. The essential element of public armed uprising against the government is
CONTRARY TO LAW.”12 lacking. There were no masses or multitudes involving crowd action done in
On the next day, 10 December 2009, accused Ampatuan, et al. filed an Urgent furtherance of a political end. So, even assuming that there was uprising, there is
Omnibus Motion, which included a motion for judicial determination of probable no showing that the purpose of the uprising is political, that is, to overthrow the
cause for the offense charged. On the same day, the Acting Presiding Judge of RTC- duly constituted government in order to establish another form of government. In
Cotabato issued an Order, stating that “the Court needs other words, the second element is also absent.
_______________ xxxx
12 RTC-Quezon City Order dated 26 March 2010, p. 4.
x x x It is quite interesting that the prosecution failed to present any particular Let this Order be served personally upon the accused-movants, through the
instance where the accused had directly or indirectly prevented government responsible officers of the law having custody over them, who are hereby directed
prosecutors from performing their job relative to the prosecution of the suspects in to release the accused from detention immediately upon receipt hereof.544
the infamous Maguindanao massacre. 544 SUPREME COURT REPORTS ANNOTATED
On the contrary, documentary evidence on record shows that the alleged
principal suspect in the mass killings, Datu Andal Ampatuan, Jr., was made to Fortun vs. Macapagal-Arroyo
undergo inquest proceedings at General Santos City, immediately after he was SO ORDERED.”13
taken into custody by law enforcement authorities. This alone belies the In an Order dated 28 May 2010, the RTC-Quezon City denied the prosecution’s
prosecution’s theory that the prosecutors were not available to conduct inquest and motion for reconsideration of the Order dated 26 March 2010.
preliminary investigations relative to the mass killings in the Municipality of The DOJ filed a petition for certiorari14 before the Court of Appeals assailing
Ampatuan, Province of Maguindanao. the dismissal of the rebellion charges against accused Ampatuan, et al.
xxxx In a Decision promulgated on 15 December 2011,15 the Court of Appeals denied
x x x [T]he intelligence reports presented by the military and police are the petition for certiorari. Quoting the findings of the RTC-Quezon City, the Court
unfounded. The reports do not suggest that the alleged armed groups loyal to the of Appeals held that there is no probable cause as there is no showing that all the
accused are initiating violent and hostile actions, whether directly or indirectly, elements of the crime of rebellion are present. The Court of Appeals stated that “a
against government security forces. Even the discovery and confiscation of large review of its own narration of events only lends to the belief that the rebellion
cache of firearm and ammunitions, allegedly belonging to the Ampatuans, cannot existed only in the minds of the complainants.” The Court of Appeals ruled that
be considered as an act of rebellion. In fact, the firearms and ammunitions were there was no armed public uprising, finding “no proof that armed groups were
subsequently unearthed, recovered and confiscated from massing up and were planning to instigate civil disobedience and to challenge the
543 government authorities for political ends.”
VOL. 668, MARCH 20, 2012 543
The Issues
Fortun vs. Macapagal-Arroyo
different places. The government security forces should have been able to engage The crux of the present controversy is the constitutionality of Proclamation No.
and neutralize the reported armed groups on the basis of its intelligence reports 1959, declaring martial law and suspending the writ in Maguindanao. The
confirming their size, strength and whereabouts. threshold issue before this Court is whether there is sufficient factual basis for the
xxxx issuance of Proclamation No. 1959 based on the stringent re-
The statements of prosecution witnesses Mangacop and Dingcong are general _______________
allegations. Their statements do not show that the accused were responsible for 13 RTC-Quezon City Order dated 26 March 2010, pp. 10-13, 15-16, 18. Penned
the mass leave of officials and employees of the local government units. There is by Presiding Judge Vivencio S. Baclig.
no evidence to show that the accused actually prevented the local officials and 14 Under Rule 65 of the 1997 Rules of Procedure. Docketed as CA-G.R. SP No.
employees from reporting to their offices. 115168.
The evidence will show that the Department of Interior and Local Government 15 Penned by Associate Justice Elihu A. Ybañez, and concurred in by Associate
and the Philippine National Police closed down these offices, without any Justices Fernanda Lampas Peralta and Francisco P. Acosta.
justifiable reasons. In fact, there were news footages which showed that many 545
employees were caught by surprise on the unexpected closure of their offices.
VOL. 668, MARCH 20, 2012 545
xxxx
It is alleged in the Information that the courts were no longer functioning in Fortun vs. Macapagal-Arroyo
Cotabato City and in Maguindanao province, which have jurisdiction over the quirements set forth in Section 18, Article VII of the 1987 Constitution.
place of the commission of the massacre. The factual circumstances, however, belie In its 15 December 2009 Resolution, the Court additionally posed the following
said allegation. This Court takes judicial notice of the fact that no less than the questions for resolution:
Supreme Court of the Republic of the Philippines had denied the allegation that 1. Whether the issuance of Proclamation No. 1963, lifting martial law and
civilian courts were or are no longer functioning in Maguindanao. restoring the writ in Maguindanao, rendered the issues raised in the present
xxxx petitions moot and academic;
WHEREFORE, premises considered, the Court finds that there exists no 2. Whether the term “rebellion” in Section 18, Article VII of the 1987
probable cause to indict and hold under detention the accused for rebellion. Constitution has the same meaning as the term “rebellion” that is defined in
Accordingly, the instant case is hereby dismissed and the accused-movants are Article 134 of the Revised Penal Code;
hereby ordered released from further detention, unless they are held by a court of 3. Whether the declaration of martial law or the suspension of the writ
law for other lawful cause/s. authorizes warrantless arrests, searches and seizures;
4. Whether the declaration of martial law or the suspension of the writ is a 547
joint and sequential function of the President and Congress such that, without VOL. 668, MARCH 20, 2012 547
Congressional action on the proclamation either affirming or revoking it, the
President having in the meantime lifted the declaration and restored the writ, this Fortun vs. Macapagal-Arroyo
Court has nothing to review; governmental act that is being challenged.16 In case of a suit questioning the
5. If the constitutional power of this Court to review the factual basis of the sufficiency of the factual basis of the proclamation of martial law or suspension of
declaration of martial law or suspension of the writ can be exercised the writ, such as here, Section 18, Article VII of the Constitution expressly
simultaneously with the constitutional power of Congress to revoke the declaration provides:
or suspension, and the decision of this Court conflicts with the decision of “The Supreme Court may review, in an appropriate proceeding filed by any
Congress, which decision shall prevail; and citizen, the sufficiency of the factual basis of the proclamation of martial law or
6. Whether this Court’s determination of the sufficiency of the factual basis the suspension of the privilege of the writ of habeas corpusor the extension thereof,
of the declaration of martial law or suspension of the writ, which in the meantime and must promulgate its decision thereon within thirty days from its filing.”
has been lifted and restored, respectively, would be essential to the resolution of (Emphasis supplied)
issues concerning the validity of related acts that the government committed It is clear that the Constitution explicitly clothes “any citizen” with the legal
during the time martial law was in force. standing to challenge the constitutionality of the declaration of martial law or
In its Comment Re: Resolution dated 15 December 2009, the OSG raised the suspension of the writ. The Constitution does not make any distinction as to who
issue of whether petitioners possess legal can bring such an action. As discussed in the deliberations of the Constitutional
546 Commission, the “citizen” who can challenge the declaration of martial law or
suspension of the writ need not even be a taxpayer.17This was deliberately
546 SUPREME COURT REPORTS ANNOTATED
designed to arrest, without further delay, the grave effects of an illegal declaration
Fortun vs. Macapagal-Arroyo of martial law or suspension of the writ, and to provide immediate relief to those
standing to challenge the constitutionality of Proclamation No. 1959. aggrieved by the same. Accordingly, petitioners, being Filipino citizens, possess
legal standing to file the present petitions assailing the sufficiency of the factual
Discussion basis of Proclamation No. 1959.
Moreover, given the transcendental importance of the issues raised in the
I dissent from the majority’s dismissal of the petitions as moot. I find present petitions, the Court may relax the standing requirement and allow a suit
Proclamation No. 1959 unconstitutional for lack of factual basisas required in to prosper even where
Section 18, Article VII of the 1987 Constitution for the declaration of martial law _______________
and suspension of the writ. The majority in effect refuses to exercise this Court’s 16 SANLAKAS v. Executive Secretary Reyes, 466 Phil. 482, 507; 421 SCRA 656,
constitutional power in Section 18 of Article VII, to “review, in an appropriate 665-666 (2004), citing Integrated Bar of the Philippines v. Zamora, 392 Phil. 618;
proceeding filed by any citizen, the sufficiency of the factual basis of the 338 SCRA 81 (2000).
proclamation of martial law or the suspension of the privilege of the writ or the 17 BERNAS, THE INTENT OF THE 1986 CONSTITUTION WRITERS, 1995 Edition, p.
extension thereof.” 474.
Before proceeding to the substantive issues, I shall first discuss the issue 548
on locus standi. 548 SUPREME COURT REPORTS ANNOTATED
In its Comment Re: Resolution dated 15 December 2009, the OSG questioned
Fortun vs. Macapagal-Arroyo
the legal standing of petitioners in challenging the constitutionality of
Proclamation No. 1959. The OSG argued that the phrase “any citizen” in Section there is no direct injury to the party claiming the right of judicial review. 18 The
18, Article VII of the 1987 Constitution must be read in conjunction with the Court has held:
phrase “appropriate proceeding.” Since petitioners deemed the original actions “Notwithstanding, in view of the paramount importance and the constitutional
for certiorari and prohibition as the appropriate proceeding referred to in Section significance of the issues raised in the petitions, this Court, in the exercise of its
18, Article VII of the Constitution, petitioners must satisfy the requirements under sound discretion, brushes aside the procedural barrier and takes cognizance of the
Rule 65 of the Rules of Court, one of which is the institution of the action by the petitions, as we have done in the early Emergency Powers Cases, where we had
aggrieved party. The OSG pointed out that none of the petitioners qualify as an occasion to rule:
aggrieved party. ‘x x x ordinary citizens and taxpayers were allowed to question the
This is error. constitutionality of several executive orders issued by President Quirino
“Legal standing” or locus standi has been defined as a personal and substantial although they [involved] only an indirect and general interest shared in
interest in the case such that the party has sustained or will sustain direct injury common with the public. The Court dismissed the objection that they were
as a result of the not proper parties and ruled that ‘transcendental importance to the
public of these cases demands that they be settled promptly and
definitely, brushing aside, if we must, technicalities of procedure.’ 23 Id., citing Province of Batangas v. Romulo, 473 Phil. 806; 429 SCRA 736
We have since then applied the exception in many other (2004).
cases.”19 (Emphasis supplied) 24 Id., citing Lacson v. Perez, 410 Phil. 78; 357 SCRA 756 (2001).
550
I. 550 SUPREME COURT REPORTS ANNOTATED
Fortun vs. Macapagal-Arroyo
Whether the issuance of Proclamation No. 1963, lifting martial law and
restoring the writ in the province of Maguindanao, rendered the issues raised in the public;25 and (4) the case is capable of repetition yet evading review.26
the petitions moot and academic. In Province of North Cotabato v. Government of the Republic of the Philippines
The majority dismisses the petitions on mootness, agreeing with respondents’ Peace Panel on Ancestral Domain (GRP),27 the Court ruled that once a suit is filed,
contention that the issuance of Proclamation No. 1963, lifting martial law and the Court cannot automatically be deprived of its jurisdiction over a case by the
restoring the writ in the province of Maguindanao, rendered the issues raised in mere expedient of the doer voluntarily ceasing to perform the challenged conduct.
the present petitions moot and academic. Respondents maintain Otherwise, the doer would be dictating when this Court should relinquish its
_______________ jurisdiction over a case. Further, a case is not mooted when the plaintiff seeks
18 David v. Arroyo, 522 Phil. 705, 757-759; 489 SCRA 160, 220 (2006). damages or prays for injunctive relief against the possible recurrence of the
See Chavez v. Public Estates Authority, 433 Phil. 506; 403 SCRA 1 (2002), Bagong violation.28
Alyansang Makabayan v. Zamora, 396 Phil. 623; 342 SCRA 449 (2000). Contrary to the majority opinion, the present petitions fall squarely under
19 Lim v. Executive Secretary, 430 Phil. 555, 570-571; 380 SCRA 739, 751 these exceptions, justifying this Court’s exercise of its review power.
(2002) citing Bagong Alyansang Makabayan v. Zamora, supra. First, whether Proclamation No. 1959 complied with the requirements under
Section 18, Article VII of the Constitution is without doubt an extremely serious
549
constitutional question. In order to forestall any form of abuse in the exercise of
VOL. 668, MARCH 20, 2012 549 the President’s extraordinary emergency powers, as what happened during the
Fortun vs. Macapagal-Arroyo Martial Law regime under former President Ferdinand Marcos (President
that the petitions have ceased to present an “actual case or controversy” with the Marcos), the 1987 Constitution has carefully put in place specific safeguards,
lifting of martial law and the restoration of the writ, the sufficiency of the factual which the President must strictly observe. Any declaration of martial law or
basis of which is the subject of these petitions. Proclamation No. 1963 is allegedly suspension of the writ falling short of the constitutional requirements must be
a “supervening event” that rendered of no practical use or value the consolidated stricken down as a matter of constitutional duty by this Court.
petitions. _______________
As a rule, courts may exercise their review power only when there is an actual 25 Id., citing Province of Batangas v. Romulo, supra.
case or controversy, which involves a conflict of legal claims susceptible of judicial 26 Id., citing Albaña v. Commission on Elections, 478 Phil. 941; 435 SCRA 98
resolution. Such a case must be “definite and concrete, touching the legal relations (2004); Acop v. Guingona, Jr., 433 Phil. 62; 383 SCRA 577 (2002); SANLAKAS v.
of parties having conflicting legal interests;” a real, as opposed to an imagined, Executive Secretary Reyes, supra note 16.
controversy calling for a specific relief.20 27 G.R. Nos. 183591, 183752, 183893, 183951, 183962, 14 October 2008, 568
Corollarily, courts generally decline jurisdiction over a moot and academic case SCRA 402.
or outrightly dismiss it on the ground of mootness. A moot and academic case is 28 Id.
one that ceases to present a justiciable controversy by virtue of supervening events, 551
so that assuming jurisdiction over the same, and eventually deciding it, would be VOL. 668, MARCH 20, 2012 551
of no practical use or value.21
Fortun vs. Macapagal-Arroyo
In David v. Arroyo,22 this Court held that the “moot and academic” principle is
not a magical formula that automatically dissuades courts in resolving a case. Second, whether the President exercised her Commander-in-Chief powers in
Courts are not prevented from deciding cases, otherwise moot and academic, if (1) accordance with the Constitution indisputably presents a transcendental issue
there is a grave violation of the Constitution;23 (2) the situation is of exceptional fully imbued with public interest. I agree with amicus curiae Father Joaquin
character and of paramount public interest;24 (3) the constitutional issue raised Bernas’ opinion: “The practice of martial rule can have a profoundly disturbing
requires formulation of controlling principles to guide the bench, the bar, and effect on the life, liberty and fortunes of people. Likewise, the actions taken by the
_______________ police and military during the period when martial law is in effect can have serious
20 David v. Arroyo, supra note 18 at p. 753; p. 213. consequences on fundamental rights.”29
21 Id., at p. 753; pp. 213-214. Third, the issue on the constitutionality of Proclamation No. 1959
22 Id., at p. 754; pp. 214-215. unquestionably requires formulation of controlling principles to guide the
Executive, Legislature, and the public.
The President’s issuance of Proclamation No. 1959 generated strong reactions (http://opinion.inquirer.net/inquireropinion/columns/view/20091220-
from various sectors of society. This, of course, is an expected response from a 243027/Uphold-the-Constitution [accessed on 4 November 2011], With Due
nation whose painful memory of the dark past remains fresh. The nation Respect: Uphold the Constitution)
remembers that martial law was the vehicle of President Marcos to seize unlimited 31 Rollo (G.R. No. 190293), p. 509; Brief of Amicus Curiae Father Joaquin
State power, which resulted in gross and wanton violations of fundamental human Bernas, S.J.
rights of the people. That era saw the collapse of the rule of law and what reigned 553
supreme was a one man-rule for the dictator’s own personal benefit. VOL. 668, MARCH 20, 2012 553
The present controversy, being the first case under the 1987 Constitution
involving the President’s exercise of the power to declare martial law and suspend Fortun vs. Macapagal-Arroyo
the writ, provides this Court with a rare opportunity,30 which it must forthwith Moreover, the fact that every declaration of martial law or suspension of the
_______________ writ will involve its own set of circumstances peculiar to the necessity of time,
29 Rollo (G.R. No. 190293), p. 508; Brief of Amicus Curiae Father Joaquin events or participants should not preclude this Court from reviewing the
Bernas, S.J. President’s use of such emergency powers. Whatever are the circumstances
30 Retired Chief Justice Panganiban called this a historic moment and surrounding each declaration of martial law or suspension of the writ, the
reminded the Court of its duty to uphold the Constitution. He writes: declaration or suspension will always be governed by the same safeguards and
The Court faces a historic moment. It cannot cower or cop-out or hide behind limitations prescribed in the same provisions of the Constitution. Failing to
legalisms. Worse, in a false sense of gratitude, it should not invent legal excuses determine the constitutionality of Proclamation No. 1959 by dismissing the cases
to justify or cover plainly unconstitutional acts. Rare is the opportunity for on the ground of mootness sets a very dangerous precedent to the leaders of this
greatness. Let the Court not country that they could easily impose martial law or suspend the writ without any
552 factual or legal basis at all, and before this Court could review such declaration,
they would simply lift the same and escape possible judicial rebuke.
552 SUPREME COURT REPORTS ANNOTATED
II.
Fortun vs. Macapagal-Arroyo Whether the term “rebellion” in Section 18, Article VII of the 1987 Constitution
seize, to formulate controlling principles for the guidance of all sectors concerned, has the same meaning as the term “rebellion” that is defined in Article 134
most specially the Executive which is in charge of enforcing the emergency of the Revised Penal Code.
measures. Dismissing the petitions on the ground of mootness will most certainly Article 134 of the Revised Penal Code, as amended by Republic Act No.
deprive the entire nation of instructive and valuable principles on this extremely 6968,32 defines the crime of rebellion, thus:
crucial national issue. “Art. 134. Rebellion or insurrection; How committed.—The crime of rebellion
Fourth, the present case is capable of repetition yet evading review. I agree or insurrection is committed by rising publicly and taking arms against the
with Father Bernas’ view: “[H]istory clearly attests that the events that can lead Government for the purpose of removing from the allegiance to said Government
to martial law, as well as the imposition of martial law itself, and the suspension or its laws, the territory of the Philippine Islands or any part thereof, of any body
of the privilege together with actions taken by military and police during a period of land, naval or other armed forces, depriving the Chief Executive or the
of martial law are capable of repetition and are too important to allow to escape Legislature, wholly or partially, of any of their powers or prerogatives.”
review through the simple expedient of the President lifting a challenged _______________
proclamation.”31 32 An Act Punishing the Crime of Coup D’état By Amending Articles 134, 135
Fifth, the respondent’s or doer’s voluntary cessation of the questioned act does and 136 of Chapter One, Title Three of Act Numbered Thirty-Eight Hundred and
not by itself deprive the Court of its jurisdiction once the suit is filed. In this case, Fifteen, Otherwise Known as The Revised Penal Code, and for Other Purposes.
President Arroyo, after eight days from the issuance of Proclamation No. 1959, 554
issued Proclamation No. 1963 revoking Proclamation No. 1959. President Arroyo’s 554 SUPREME COURT REPORTS ANNOTATED
lifting of martial law and restoration of the writ translate to a voluntary cessation
of the very acts complained of in the present petitions. However, the present Fortun vs. Macapagal-Arroyo
petitions were filed with this Court while Proclamation No. 1959 was still in effect The Constitution, however, does not provide any definition of the term
and before Proclamation No. 1963 was issued, thus foreclosing any legal strategy “rebellion.” Portions of the first paragraph of Section 18, Article VII of the
to divest this Court of its jurisdiction by the mere cessation or withdrawal of the Constitution, where the term “rebellion” appears, read:
challenged act. “Section 18. The President shall be the Commander-in-Chief of all armed forces
_______________ of the Philippines and whenever it becomes necessary, he may call out such armed
squander the moment. Let it perform its duty forthrightly and uphold the forces to prevent or suppress lawless violence, invasion or rebellion. In case of
Constitution. invasion or rebellion, when the public safety requires it, he may, for a period not
exceeding sixty days, suspend the privilege of the writ of habeas corpus or place
the Philippines or any part thereof under martial law.”
Respondents submit that the term “rebellion” must, “for constitutional law 556
purposes, be applied in such manner as to be amply responsive to the call of the 556 SUPREME COURT REPORTS ANNOTATED
times.” Respondents point out that the deliberations of the 1986 Constitutional
Commission reveal that the concept of the term “rebellion” depends much on its Fortun vs. Macapagal-Arroyo
magnitude and scope, as determined by the President based on prevailing in Proclamation No. 427 a superfluity,37 the term “rebellion” in said proclamation
circumstances.33 referred to the crime of rebellion as defined in Article 134 of the Revised Penal
I disagree. The term “rebellion” in Section 18, Article VII of the 1987 Code. Proclamation No. 427 pertinently reads:
Constitution must be understood as having the same meaning as the crime of DECLARING A STATE OF REBELLION
“rebellion” that is defined in Article 134 of the Revised Penal Code, as amended. “WHEREAS, certain elements of the Armed Forces of the Philippines, armed with
First, this is the clear import of the last two paragraphs of Section 18, Article high-powered firearms and explosives, acting upon the instigation and command
VII of the Constitution, which explicitly state: and direction of known and unknown leaders, have seized a building in Makati
“The suspension of the privilege of the writ of habeas corpus shall apply only City, put bombs in the area, publicly declared withdrawal of support for, and took
to persons judicially charged for rebellion or offenses inherent in, or directly arms against the duly constituted Government, and continue to rise publicly and
connected with, invasion. show open hostility, for the purpose of removing allegiance to the
During the suspension of the privilege of the writ of habeas corpus, any Government certain bodies of the Armed Forces of the Philippines and
person thus arrested or detained shall be judicially charged within three the Philippine National Police, and depriving the President of the
days, otherwise he shall be released.” (Emphasis supplied) Republic of the Philippines, wholly or partially, of her powers and
_______________ prerogatives which constitute the crime of rebellion punishable under
33 Rollo (G.R. No. 190293), p. 138. Article 134 of the Revised Penal Code, as amended; x x x” (Emphasis
555 supplied)
In issuing Proclamation No. 427, President Arroyo relied on the Revised Penal
VOL. 668, MARCH 20, 2012 555
Code definition of rebellion in declaring a state of rebellion. In other words,
Fortun vs. Macapagal-Arroyo President Arroyo understood that, for purposes of declaring a state of rebellion,
For a person to be judicially charged for rebellion, there must necessarily be a the term “rebellion” found in the Constitution refers to the crime of rebellion
statute defining rebellion. There is no statute defining rebellion other than the defined in Article 134 of the Revised Penal Code.
Revised Penal Code. Hence, “one can be ‘judicially charged’ with rebellion only if In exercising the Commander-in-Chief powers under the Constitution, every
one is suspected of having committed acts defined as rebellion in Article 134 of the President must insure the existence of the elements of the crime of rebellion, which
Revised Penal Code.”34 are: (1) there is a (a) public uprising and (b) taking arms against the Government;
Second, the Revised Penal Code definition of rebellion is the only legal and (2) the purpose of the uprising or movement is either (a) to remove from the
definition of rebellion known and understood by the Filipino people when they allegiance to the Government or its laws:
ratified the 1987 Constitution. Indisputably, the Filipino people recognize and are _______________
familiar with only one meaning of rebellion, that is, the definition provided in 37 Id., at p. 520. The Court stated that “[a] declaration of a state of rebellion is
Article 134 of the Revised Penal Code. To depart from such meaning is to betray an utter superfluity. At most, it only gives notice to the nation that such a state
the Filipino people’s understanding of the term “rebellion” when they ratified the exists and that the armed forces may be called to prevent or suppress it.”
Constitution. There can be no question that “the Constitution does not derive its 557
force from the convention which framed it, but from the people who ratified it.”35 VOL. 668, MARCH 20, 2012 557
Third, one of the Whereas clauses of Proclamation No. 1959 expressly cites the
Revised Penal Code definition of rebellion, belying the government’s claim that the Fortun vs. Macapagal-Arroyo
Revised Penal Code definition of rebellion merely guided the President in issuing (1) the territory of the Philippines or any part thereof; or (2) any body of land,
Proclamation No. 1959. naval, or other armed forces; or (b) to deprive the Chief Executive or Congress,
In SANLAKAS v. Executive Secretary,36 where the Court regarded President wholly or partially, of any of their powers and prerogatives.38
Arroyo’s declaration of a state of rebellion To repeat, the term “rebellion” in Section 18, Article VII of the Constitution
_______________ must be understood to have the same meaning as the crime of rebellion defined in
34 Id., at p. 493, Amicus Memorandum of Justice Vicente V. Mendoza. Article 134 of the Revised Penal Code. Ascribing another meaning to the term
35 See retired Chief Justice Puno’s separate concurring opinion in United “rebellion” for constitutional law purposes, more specifically in imposing martial
Pepsi-Cola Supervisory Union v. Judge Laguesma, 351 Phil. 244, 292; 288 SCRA law and suspending the writ, different from the definition in Article 134 of the
15, 58 (1998), citing Cooley, Treatise on Constitutional Limitations, Vol. 1, pp. 142- Revised Penal Code, overstretches its definition without any standards, invites
143 [1927]; also cited in Willoughby, The Constitutional Law of the United States, unnecessary confusion, and undeniably defeats the intention of the Constitution
Sec. 32, pp. 54-55, Vol. 1 [1929]. to restrain the extraordinary Commander-in-Chief powers of the President.
36 Supra note 16.
Since the term “rebellion” in Section 18, Article VII of the Constitution pertains SECTION 1. Preponderance of evidence, how determined.—In civil cases, the
to the crime of rebellion as defined in Article 134 of the Revised Penal Code, the party having the burden of proof must establish his case by a preponderance of
next question turns on the kind of proof required for a valid declaration of martial evidence. In determining where the
law and suspension of the writ. 559
While the Constitution expressly provides strict safeguards against any VOL. 668, MARCH 20, 2012 559
potential abuse of the President’s emergency powers, the Constitution does not
compel the President to produce such amount of proof as to unduly burden and Fortun vs. Macapagal-Arroyo
effectively incapacitate her from exercising such powers. By preponderance of evidence is meant that the evidence as a whole adduced by
Definitely, the President need not gather proof beyond reasonable doubt, which one side is superior to that of the other. It refers to the weight, credit and value of
is the standard of proof required for convicting an accused charged with a criminal the aggregate evidence on either side and is usually considered to be synonymous
offense. Section 2, Rule 133 of the Rules of Court defines proof beyond reasonable with the term “greater weight of evidence” or “greater weight of the credible
doubt as follows: evidence”. It is evidence which is more convincing to the court as worthy of belief
Proof beyond reasonable doubt does not mean such a degree of proof as, excluding than that which is offered in opposition thereto.42
possibility of error, produces absolute certainty. Moral Weighing the superiority of the evidence on hand, from at least two opposing
_______________ sides, before she can act and impose martial law or suspend the writ unreasonably
38 See Ladlad v. Velasco, G.R. Nos. 172070-72, 1 June 2007, 523 SCRA 318, curtails the President’s emergency powers.
336. Similarly, substantial evidence constitutes an unnecessary restriction on the
558 President’s use of her emergency powers. Substantial evidence is the amount of
proof required in administrative or quasi-judicial cases, or that amount of relevant
558 SUPREME COURT REPORTS ANNOTATED
evidence which a reasonable mind might accept as adequate to justify a
Fortun vs. Macapagal-Arroyo conclusion.43
certainty only is required, or that degree of proof which produces conviction in an _______________
unprejudiced mind. preponderance or superior weight of evidence on the issues involved lies, the
Proof beyond reasonable doubt is the highest quantum of evidence, and to court may consider all the facts and circumstances of the case, the witnesses’
require the President to establish the existence of rebellion or invasion with such manner of testifying, their intelligence, their means and opportunity of knowing
amount of proof before declaring martial law or suspending the writ amounts to the facts to which they are testifying, the nature of the facts to which they testify,
an excessive restriction on “the President’s power to act as to practically tie her the probability or improbability of their testimony, their interest or want of
hands and disable her from effectively protecting the nation against threats to interest, and also their personal credibility so far as the same may legitimately
public safety.”39 appear upon the trial. The court may also consider the number of witnesses,
Neither clear and convincing evidence, which is employed in either criminal or though the preponderance is not necessarily with the greater number.
civil cases, is indispensable for a lawful declaration of martial law or suspension 42 Raymundo v. Lunaria, G.R. No. 171036, 17 October 2008, 569 SCRA 526.
of the writ. This amount of proof likewise unduly restrains the President in 43 Section 5, Rule 133 of the Rules of Court provides:
exercising her emergency powers, as it requires proof greater than preponderance SECTION 5. Substantial evidence.—In cases filed before administrative or
of evidence although not beyond reasonable doubt.40 quasi-judicial bodies, a fact may be deemed established if it is supported by
Not even preponderance of evidence,41 which is the degree of proof necessary substantial evidence, or that amount of relevant evidence which a reasonable mind
in civil cases, is demanded for a lawful declaration of martial law. might accept as adequate to justify a conclusion.
_______________ 560
39 Rollo (G.R. No. 190293), p. 512, Brief of Amicus Curiae Father Joaquin 560 SUPREME COURT REPORTS ANNOTATED
Bernas, S.J.
40 Manalo v. Roldan-Confesor, G.R. No. 102358, 19 November 1992, 215 SCRA Fortun vs. Macapagal-Arroyo
808, 819. The Court held therein: I am of the view that probable cause of the existence of either invasion or
Clear and convincing proof is “x x x more than mere preponderance, but not to rebellion suffices and satisfies the standard of proof for a valid declaration of
extent of such certainty as is required beyond reasonable doubt as in criminal cases martial law and suspension of the writ.
x x x” while substantial evidence “x x x consists of more than a mere scintilla of Probable cause is the same amount of proof required for the filing of a criminal
evidence but may be somewhat less than a preponderance x x x” Consequently, in information by the prosecutor and for the issuance of an arrest warrant by a judge.
the hierarchy of evidentiary values, We find proof beyond reasonable doubt at the Probable cause has been defined as a “set of facts and circumstances as would lead
highest level, followed by clear and convincing evidence, preponderance of a reasonably discreet and prudent man to believe that the offense charged in the
evidence, and substantial evidence, in that order. (Citations omitted) Information or any offense included therein has been committed by the person
41 Section 1, Rule 133 of the Rules of Court provides: sought to be arrested.”44
“In determining probable cause, the average man weighs the facts and Constitution to serve as an essential safeguard against potential abuses in the
circumstances without resorting to the calibrations of the rules of exercise of the President’s emergency powers.
evidence of which he has no technical knowledge. He relies on common The Constitution now expressly declares, “A state of martial law does not
sense. A finding of probable cause needs only to rest on evidence showing that, suspend the operation of the Constitution.” Neither does a state of martial law
more likely than not, a crime has been committed and that it was committed by supplant the functioning of the civil courts or legislative assemblies. Nor does it
the accused. Probable cause demands more than suspicion; it requires less than au-
evidence that would justify conviction.”45 (Emphasis supplied) 562
Probable cause, basically premised on common sense, is the most reasonable, 562 SUPREME COURT REPORTS ANNOTATED
most practical, and most expedient standard by which the President can fully
ascertain the existence or non-existence of rebellion, necessary for a declaration of Fortun vs. Macapagal-Arroyo
martial law or suspension of the writ. Therefore, lacking probable cause of the thorize the conferment of jurisdiction on military courts and agencies over civilians
existence of rebellion, a declaration of martial law or suspension of the writ is where civil courts are able to function, or automatically suspend the writ. There is
without any basis and thus, unconstitutional. therefore no dispute that the constitutional guarantees under the Bill of Rights
The requirement of probable cause for the declaration of martial law or remain fully operative and continue to accord the people its mantle of protection
suspension of the writ is consistent with Sec- during a state of martial law. In case the writ is also suspended, the suspension
_______________ applies only to those judicially charged for rebellion or offenses directly connected
44 Santos v. Orda, Jr., G.R. No. 189402, 6 May 2010, 620 SCRA 375, 384. with invasion.
45 Viudez II v. Court of Appeals, G.R. No. 152889, 5 June 2009, 588 SCRA 345, Considering the non-suspension of the operation of the Constitution during a
357. state of martial law, a declaration of martial law does not authorize warrantless
561 arrests, searches and seizures, in derogation of Section 2, Article III of the
Constitution, which provides:
VOL. 668, MARCH 20, 2012 561
“Section 2. The right of the people to be secure in their persons, houses, papers,
Fortun vs. Macapagal-Arroyo and effects against unreasonable searches and seizures of whatever nature and for
tion 18, Article VII of the Constitution. It is only upon the existence of probable any purpose shall be inviolable, and no search warrant or warrant of arrest shall
cause that a person can be “judicially charged” under the last two paragraphs of issue except upon probable cause to be determined personally by the judge after
Section 18, Article VII, to wit: examination under oath or affirmation of the complainant and the witnesses he
“The suspension of the privilege of the writ of habeas corpus shall apply only may produce, and particularly describing the place to be searched and the persons
to persons judicially charged for rebellion or offenses inherent in, or directly or things to be seized.”
connected with, invasion. Warrantless arrests, search and seizure are valid only in instances where such
During the suspension of the privilege of the writ of habeas corpus, any person acts are justified, i.e., those enumerated in Section 5, Rule 113 of the Rules of
thus arrested or detained shall be judicially charged within three days, Court.46
otherwise he shall be released.” (Emphasis supplied) _______________
46 Sec. 5. Arrest without warrant; when lawful.—A peace officer or a private
III. person may, without a warrant, arrest a person:
Whether the declaration of martial law or the suspension of the writ authorizes (a) When, in his presence, the person to be arrested has committed, is
warrantless arrests, searches and seizures. actually committing, or is attempting to commit an offense;
(b) When an offense has just been committed and he has probable cause to
Section 18, Article VII of the Constitution partially states: believe based on personal knowledge of facts or circumstances that the person to
“A state of martial law does not suspend the operation of the Constitution, nor be arrested has committed it; and
supplant the functioning of the civil courts or legislative assemblies, nor authorize (c) When the person to be arrested is a prisoner who has escaped from a penal
the conferment of jurisdiction on military courts and agencies over civilians where establishment or place where he is serving final
civil courts are able to function, nor automatically suspend the privilege of the writ 563
of habeas corpus. VOL. 668, MARCH 20, 2012 563
The suspension of the privilege of the writ of habeas corpus shall apply only to
Fortun vs. Macapagal-Arroyo
persons judicially charged for rebellion or offenses inherent in, or directly
connected with, invasion.” In Pequet v. Tangonan,47 decided during the Martial Law regime under former
The 1935 and 1973 Constitutions did not contain a similar provision. Obviously, President Marcos, the Court stressed that military personnel, in effecting arrests,
this new provision in the 1987 Constitution was envisioned by the framers of the must strictly observe the applicable Rules of Court and settled jurisprudence, thus:
“Martial law has precisely been provided in both the 1935 Charter and the
present Constitution to assure that the State is not powerless to cope with
invasion, insurrection or rebellion or any imminent danger of its occurrence. When Teehankee — to the law of force rather than the force of law, it is necessary to
resort to it is therefore justified, it is precisely in accordance with and not in remind ourselves that certain basic rights and liberties are immutable and cannot
defiance of the fundamental law. There is all the more reason then for the rule of be sacrificed to the transient needs or imperious demands of the ruling power. The
law to be followed. For as was so eloquently proclaimed in Ex parte Milligan: “The rule of law must prevail, or else liberty will perish. x x x
Constitution is a “law for rulers and for people equally in war and in peace and xxxx
covers with the shield of its protection all classes of men at all times and under all It may be that the respondents, as members of the Armed Forces of the
circumstances.” It is Philippines, were merely responding to their duty, as they claim, “to prevent or
_______________ suppress lawless violence, insurrection, rebellion and subversion” in accordance
judgment or is temporarily confined while his case is pending, or has escaped with Proclamation No. 2054 of President Marcos, despite the lifting of martial law
while being transferred from one confinement to another. on January 27,
In cases falling under paragraphs (a) and (b) above, the person arrested _______________
without a warrant shall be forthwith delivered to the nearest police station or jail 48 243 Phil. 735; 160 SCRA 590 (1988).
and shall be proceeded against in accordance with section 7 of Rule 112. 565
In addition, jurisprudence tells us that in the following instances, a VOL. 668, MARCH 20, 2012 565
warrantless search and seizure is valid.
(1) search incidental to a lawful arrest, Fortun vs. Macapagal-Arroyo
(2) search of moving vehicles, 1981, and in pursuance of such objective, to launch pre-emptive strikes against
(3) seizure in plain view, alleged communist terrorist underground houses. But this cannot be construed
(4) customs search, and as a blanket license or a roving commission untramelled by any
(5) waiver by the accused themselves of their right against unreasonable constitutional restraint, to disregard or transgress upon the rights and
search and seizure. liberties of the individual citizen enshrined in and protected by the
See Manalili v. Court of Appeals, 345 Phil. 632, 645-646; 280 SCRA 400, 413 Constitution. The Constitution remains the supreme law of the land to
(1997), citing People v. Lacerna, 344 Phil. 100; 278 SCRA 561 (1997). which all officials, high or low, civilian or military, owe obedience and
Stop-and-frisk is also another exception to the general rule against a search allegiance at all times.
without a warrant (Posadas v. Court of Appeals, G.R. No. 89139, 2 August 1990, xxxx
188 SCRA 288, 292-293, cited in Manalili). This is not to say that military authorities are restrained from pursuing their
47 160 Phil. 906, 909; 66 SCRA 216, 219-220 (1975); citations omitted. assigned task or carrying out their mission with vigor. We have no quarrel with
564 their duty to protect the Republic from its enemies, whether of the left or of the
right, or from within or without, seeking to destroy or subvert our democratic
564 SUPREME COURT REPORTS ANNOTATED
institutions and imperil their very existence. What we are merely trying to say
Fortun vs. Macapagal-Arroyo is that in carrying out this task and mission, constitutional and legal
true, of course, as admitted by Willoughby, who would limit the scope of martial safeguards must be observed, otherwise, the very fabric of our faith will
law power, that the military personnel are called upon to assist in the maintenance start to unravel. x x x
of peace and order and the enforcement of legal norms. They can therefore act like We do not agree. We find merit in petitioners’ contention that the suspension
ordinary peace officers. In effecting arrests, however, they are not free to of the privilege of the writ of habeas corpus does not destroy petitioners’
ignore, but are precisely bound by, the applicable Rules of Court and right and cause of action for damages for illegal arrest and detention and
doctrinal pronouncements.” (Emphasis supplied) other violations of their constitutional rights. The suspension does not
In Aberca v. Ver,48 the Court emphasized that the suspension of the writ does render valid an otherwise illegal arrest or detention. What is suspended
not give imprimatur to warrantless arrests in violation of the Constitution. In that is merely the right of the individual to seek release from detention
case, which involved the issue of whether the suspension of the writ bars a civil through the writ of habeas corpus as a speedy means of obtaining his
action for damages for illegal searches and for other human rights violations liberty.”48 (Emphasis supplied)
committed by the military, the Court held:
“At the heart of petitioners’ complaint is Article 32 of the Civil Code which IV.
provides: Whether the declaration of martial law or suspension of the writ is a joint and
xxxx sequential function of the President and Congress such that, without
It is obvious that the purpose of the above codal provision is to provide a Congressional action on the proclamation or suspension either affirming or
sanction to the deeply cherished rights and freedoms enshrined in the revoking it, the President having in the meantime lifted the same, this Court has
Constitution. Its message is clear; no man may seek to violate those sacred rights nothing to review.
with impunity. In times of great upheaval or of social and political stress, when
the temptation is strongest to yield — borrowing the words of Chief Justice Claudio
_______________ mandated act to be performed by either the Legislature or the Judiciary. It is
49 Id., at pp. 743-745, 748-749; pp. 600-605. neither joint nor sequential with Congress’ power to revoke the declaration or
566 suspension or to extend it upon the initiative of the President. Accordingly, even if
566 SUPREME COURT REPORTS ANNOTATED Congress has not acted upon the President’s declaration or suspension, the Court
may review the declaration or suspension in an appropriate proceeding filed by
Fortun vs. Macapagal-Arroyo any citizen. Otherwise stated, Congress’ inaction on the declaration or suspension
Section 18, Article VII of the 1987 Constitution provides: is not determinative of the Court’s exercise of its review power under Section 18,
“Section 18. The President shall be the Commander-in-Chief of all armed forces Article VII of the Constitution.
of the Philippines and whenever it becomes necessary, he may call out such armed To hold that the power of this Court to review the President’s declaration of
forces to prevent or suppress lawless violence, invasion or rebellion. In case of martial law or suspension of the writ is sequential, or joint, with the review
invasion or rebellion, when the public safety requires it, he may, for a period not power of Congress is to make it impossible for this Court to decide a case
exceeding sixty days, suspend the privilege of the writ of habeas corpus or place challenging the declaration or suspension “within thirty days from its filing,”
the Philippines or any part thereof under martial law. Within forty-eight hours as mandated by the Constitution. Congress has no deadline when to revoke the
from the proclamation of martial law or the suspension of the privilege of the writ President’s declaration or suspension. Congress may not even do anything with
of habeas corpus, the President shall submit a report in person or in writing to the the President’s declaration or suspension and merely allow it to lapse after 60 days.
Congress. The Congress, voting jointly, by a vote of at least a majority of all its On the other hand, the Constitution mandates that this Court “must promulgate
Members in regular or special session, may revoke such proclamation or its decision thereon within thirty days from [the] filing” of the case. Clearly,
suspension, which revocation shall not be set aside by the President. Upon the the Court’s review power is neither sequential nor joint with the review power of
initiative of the President, the Congress may, in the same manner, extend such Congress.568
proclamation or suspension for a period to be determined by the Congress, if the
568 SUPREME COURT REPORTS ANNOTATED
invasion or rebellion shall persist and public safety requires it.
The Congress, if not in session, shall, within twenty-four hours following such Fortun vs. Macapagal-Arroyo
proclamation or suspension, convene in accordance with its rules without need of Moreover, the President’s lifting of the declaration or suspension before this
a call. Court could decide the case within the 30-day period does not operate to divest this
The Supreme Court may review, in an appropriate proceeding filed by any Court of its jurisdiction over the case. A party cannot simply oust the Court’s
citizen, the sufficiency of the factual basis of the proclamation of martial law or the jurisdiction, already acquired, by a party’s own unilateral act. The President’s
suspension of the privilege of the writ of habeas corpusor the extension thereof, lifting of the declaration or suspension merely means that this Court does not have
and must promulgate its decision thereon within thirty days from its filing.” to decide the case within the 30-day period, as the urgency of deciding has ceased.
The Constitution vests exclusively in the President, as Commander-in-Chief, Certainly, the Court is not divested of its jurisdiction simply because the urgency
the emergency powers to declare martial law or suspend the writ in cases of of deciding a case has ceased.
rebellion or invasion, when the public safety requires it. The imposition of martial
law or suspension of the writ takes effect the moment it is declared by the V.
President. No other act is needed for the perfection of the declaration of martial If the constitutional power of this Court to review the factual basis of the
law or the suspension of the writ. As amicus curiae retired Justice Mendoza states: declaration of martial law or suspension of the writ can be exercised
“A declaration of martial law by the President alone is complete by itself and simultaneously with the constitutional power of Congress to revoke the declaration
does not require for its validity the approval or concurrence of Congress. It is a or suspension, and the decision of this Court conflicts with the decision of
power placed solely in the keeping of Congress, which decision shall prevail.
567
VOL. 668, MARCH 20, 2012 567 The President has the sole and exclusive power to declare martial law or
suspend the writ. This power of the President is subject to review separately by
Fortun vs. Macapagal-Arroyo
Congress and the Supreme Court. Justice Mendoza stresses, “Thus, Congress and
the President to enable him to secure the people from harm and restore the public this Court have separate spheres of competence. They do not act ‘jointly and
order so that they can enjoy their freedoms. Because it is liable to abuse, it is made sequentially’ but independently of each other.”50 Father Bernas points out, “Since
subject to check by Congress and/or the [Supreme Court]. the powers of Congress and the Court are independent of each other, there is
The power of Congress is to revoke—not to confirm or ratify, much less to nothing to prevent Congress and the Court from simultaneously exercising their
approve,—the President’s action declaring martial law or suspending the privilege
separate powers.”51
of the writ of habeas corpus. It is a veto power, just as the power of the judiciary to In the exercise by the Court and Congress of their separate “review powers”
review the President’s action is a veto power on the Executive’s action.” under Section 18, Article VII of the Constitution, three possible scenarios may
It is clear, therefore, that the President’s power to declare martial law or arise.
suspend the writ is independent, separate, and distinct from any constitutionally
_______________
50 Id., at p. 497, Brief of Amicus Curiae Retired Associate Justice Vicente V. Whether this Court’s determination of the sufficiency of the factual basis of the
Mendoza. declaration of martial law and suspension of the writ, which in the meantime have
51 Id., at p. 523, Brief of Amicus Curiae Father Joaquin Bernas, S.J. been lifted, would be essential to the resolution of issues concerning the validity of
569 related acts that the government committed during the time that martial law and
VOL. 668, MARCH 20, 2012 569 the suspension of the writ were in force.
Indisputably, unlawful acts may be committed during martial law or
Fortun vs. Macapagal-Arroyo suspension of the writ, not only by the rebels, but also by government forces who
First, the President’s martial law declaration or suspension of the writ is are duty bound to enforce the declaration or suspension and immediately put an
questioned in the Supreme Court without Congress acting on the same. Such a end to the root cause of the emergency. Various acts carried out by government
situation generates no conflict between the Supreme Court and Congress. There is forces during martial law or suspension of the writ in the guise of protecting public
no question that the Supreme Court can annul such declaration or suspension if it safety may in reality amount to serious abuses of power and authority. Whatever
lacks factual basis. Congress, whose only power under Section 18, Article VII of the Court’s decision will be on the sufficiency of the factual basis of the President’s
the Constitution is to revoke the declaration or suspension on any ground, is left declaration or suspension does not preclude those aggrieved by such illegal acts
with nothing to revoke if the Court has already annulled the declaration or from pursuing any course of legal action available to them. Therefore, the
suspension. determination by this Court of the sufficiency of the factual basis of the declaration
Second, Congress decides first to revoke the martial law declaration or or suspension is not essential to the resolution of issues concerning the validity of
suspension of the writ. Since the Constitution does not limit the grounds for related acts that government forces may have committed during the
congressional revocation, Congress can revoke the declaration or suspension for emergency.571
policy reasons, or plainly for being insignificant, as for instance it involves only
VOL. 668, MARCH 20, 2012 571
one barangay rebelling, or if it finds no actual rebellion. In this case, the Supreme
Court is left with nothing to act on as the revocation by Congress takes effect Fortun vs. Macapagal-Arroyo
immediately. The Supreme Court must respect the revocation by Congress even if
the Court believes a rebellion exists because Congress has the unlimited power to VII.
revoke the declaration or suspension. Whether Proclamation No. 1959 has sufficient factual basis.
Third, the Supreme Court decides first and rules that there is factual basis for
the declaration of martial law or suspension of the writ. In such a situation, The full text of Section 18, Article VII of the 1987 Constitution reads:
Congress can still revoke the declaration or suspension as its power under the “Section 18. The President shall be the Commander-in-Chief of all armed forces
Constitution is broader insofar as the declaration or suspension is concerned. of the Philippines and whenever it becomes necessary, he may call out such armed
“Congress cannot be prevented by the Court from revoking the President’s decision forces to prevent or suppress lawless violence, invasion or rebellion. In case of
because it is not for the Court to determine what to do with an existing factual invasion or rebellion, when the public safety requires it, he may, for a period not
situation. x x x Congress has been given unlimited power to revoke the President’s exceeding sixty days, suspend the privilege of the writ of habeas corpus or place
decision.”52 In short, even if there is an actual rebellion, whether affirmed or not the Philippines or any part thereof under martial law. Within forty-eight hours
by the Supreme Court, from the proclamation of martial law or the suspension of the privilege of the writ
_______________ of habeas corpus, the President shall submit a report in person or in writing to the
52 Id., at p. 524, Brief of Amicus Curiae Father Joaquin Bernas, S.J. Congress. The Congress, voting jointly, by a vote of at least a majority of all its
570 Members in regular or special session, may revoke such proclamation or
570 SUPREME COURT REPORTS ANNOTATED suspension, which revocation shall not be set aside by the President. Upon the
initiative of the President, the Congress may, in the same manner, extend such
Fortun vs. Macapagal-Arroyo
proclamation or suspension for a period to be determined by the Congress, if the
Congress has the power to revoke the President’s declaration or suspension. invasion or rebellion shall persist and public safety requires it.
In the present controversy, Congress failed to act on Proclamation No. 1959 The Congress, if not in session, shall, within twenty-four hours following such
when it commenced its Joint Session on 9 December 2009 until the lifting of the proclamation or suspension, convene in accordance with its rules without need of
martial law declaration and restoration of the writ on 12 December 2009. Congress’ a call.
non-revocation of Proclamation No. 1959 categorizes the present case under the The Supreme Court may review, in an appropriate proceeding filed by any
first scenario. In such a situation, where no conflict ensues, Congress’ inaction on citizen, the sufficiency of the factual basis of the proclamation of martial law or the
Proclamation No. 1959 does not preclude this Court from ruling on the sufficiency suspension of the privilege of the writ of habeas corpusor the extension thereof,
of the factual basis of the declaration of martial law and suspension of the writ. and must promulgate its decision thereon within thirty days from its filing.

VI.
A state of martial law does not suspend the operation of the Constitution, nor The mechanism and limitations laid down in Section 18, Article VII of the
supplant the functioning of the civil courts or legislative assemblies, nor authorize Constitution in declaring martial law or suspending the writ were introduced
the conferment of jurisdiction on military courts and agencies over civilians where precisely to preclude a repetition of the kind of martial law imposed by President
civil courts are able to function, nor automatically suspend the privilege of the writ Marcos, which ushered in a permanent authoritarian regime. As Father Bernas
of habeas corpus.572 wrote in his book:
572 SUPREME COURT REPORTS ANNOTATED “The Commander-in-Chief provisions of the 1935 Constitution had enabled
President Ferdinand Marcos to impose authoritarian rule on the Philippines from
Fortun vs. Macapagal-Arroyo 1972 to 1986. Supreme Court decisions during that period upholding the actions
The suspension of the privilege of the writ of habeas corpus shall apply only to taken by Mr. Marcos made authoritarian rule part of Philippine constitutional
persons judicially charged for rebellion or offenses inherent in, or directly jurisprudence. The members of the Constitutional Commission, very much aware
connected with, invasion. of these facts, went about reformulating the Commander-in-Chief powers with a
During the suspension of the privilege of the writ of habeas corpus, any person view to dismantling what had been constructed during the authoritarian years.
thus arrested or detained shall be judicially charged within three days, otherwise The new formula included revised grounds for the activation of emergency powers,
he shall be released.” the manner of activating them, the scope of the powers, and review of presidential
The Commander-in-Chief provisions of the 1935 and 1973 Constitutions, on action.”54
the other hand, respectively state: Consistent with the framers’ intent to reformulate the Commander-in-Chief
Section 10(2), Article VII of the 1935 Constitution powers of the President, the 1987 Constitution requires the concurrence of two
(2) The President shall be commander-in-chief of all armed forces of the conditions in declaring martial law or suspending the writ, namely, (1) an actual
Philippines, and, whenever it becomes necessary, he may call out such armed invasion or rebellion, and (2) public safety requires the exer-
forces to prevent or suppress lawless violence, invasion, insurrection, or rebellion. _______________
In case of invasion, insurrection, or rebellion or imminent danger thereof, when 53 See Senate P.S. Resolution No. 1522.
the public safety requires it, he may suspend the privilege of the writ of habeas 54 Bernas, The Intent of the 1986 Constitution Writers, 1995 Edition, p. 456.
corpus, or place the Philippines or any part thereof under Martial Law. 574
Section 12, Article IX of the 1973 Constitution
574 SUPREME COURT REPORTS ANNOTATED
SEC. 12. The Prime Minister shall be commander-in-chief of all armed forces of
the Philippines, and, whenever it becomes necessary, he may call out such armed Fortun vs. Macapagal-Arroyo
forces to prevent or suppress lawless violence, invasion, insurrection, or rebellion cise of such power.55 The Constitution no longer allows imminent danger of
or imminent danger thereof, when the public safety requires it, he may suspend rebellion or invasion as a ground for the declaration or suspension, which the 1935
the privilege of the writ of habeas corpus, or place the Philippines or any part and 1973 Constitutions expressly permitted.
thereof under Martial Law. In the present case, President Arroyo grounded the declaration of martial law
Notably, the 1935 and 1973 Constitutions only specify the instances when martial and suspension of the writ on the existence of rebellion in Maguindanao. In her
law may be declared or when the writ may be suspended. Report submitted to Congress, President Arroyo cited the following instances as
The 1987 Constitution, on the other hand, not only explicitly includes the constitutive of rebellion:
specific grounds for the activation of such emergency powers, but also imposes 1. Local government offices in the province of Maguindanao were closed and
express limitations on the exercise of such powers. Upon the President’s ranking local government officials refused to discharge their functions, which
declaration of martial law or suspension of the writ, the following safeguards are hindered the investigation and prosecution team from performing their tasks;
automatically set into motion: (1) the duration of martial law or suspension of the 2. The Local Civil Registrar of Maguindanao refused to accept the registration of
writ is limited to a period not exceeding sixty days; (2) the President is mandated the death certificates of the victims purportedly upon the orders of Andal
to submit Ampatuan Sr.;
573 3. The local judicial system has been crippled by the absence or non-appearance
VOL. 668, MARCH 20, 2012 573 of judges of local courts, thereby depriving the government of legal remedies in
their prosecutorial responsibilities (i.e. issuance of warrants of searches, seizure
Fortun vs. Macapagal-Arroyo and arrest). While the Supreme Court has designated an Acting Presiding Judge
a report to Congress within forty-eight hours from the declaration or suspension; from another province, the normal judicial proceedings could not be carried out in
and (3) the declaration or suspension is subject to review by Congress, which may view of threats to their lives or safety, prompting government to seek a change of
revoke such declaration or suspension. If Congress is not in session, it shall venue of the criminal cases after informations have been filed.
convene within 24 hours without need for call.53 In addition, the sufficiency of the xxxx
factual basis of the declaration, suspension, or their extension is subject to review Indeed, the nature, quantity and quality of their weaponry, the movement
by the Supreme Court in an appropriate proceeding. of heavily armed rebels in strategic positions, the closure of the
Maguindanao Provincial Capitol, Ampatuan Municipal Hall, Datu Unsay groups that prevent authorities from being able to do its duty of even effecting the
Municipal Hall, and four- arrest of those who should be arrested in spite of the testimonies of witnesses.
_______________ REP. LAGMAN. Well, we are happy to note that there is an admission
55 SANLAKAS v. Executive Secretary, supra note 16. See Section 15, Article that there was no actual rebellion in Maguindanao. But the presence of
III of the 1987 Constitution. In Velasco v. Court of Appeals, 315 Phil. 757; 245 armed groups would be indicative of lawless violence which is not synonymous to
SCRA 677 (1995), the Court declared that the privilege of the writ of habeas rebellion. As a matter of fact, the Maguindanao situationer which was made by
corpus cannot be suspended except in cases of invasion or rebellion when the public Police Director Andres Caro was premised on a statement that this was the worst
safety requires it. election-related violence—an act of gross lawlessness but definitely not related to
575 rebellion.
VOL. 668, MARCH 20, 2012 575 x x x x57 (Emphasis supplied)
_______________
Fortun vs. Macapagal-Arroyo 57 Transcript of Plenary Proceedings, Joint Session of the Congress of the
teen other municipal halls, and the use of armored vehicles, tanks and Republic of the Philippines, 9 December 2009. See also “Ermita: ML proclaimed
patrol cars with unauthorized “PNP/Police” markings, all together without actual rebellion,” The Philippine Star, 11 December 2009
confirm the existence of armed public uprising for the political purpose (http://www.philstar.com/Article.aspx?
of: articleId=531416&publicationSubCategoryId=63 [accessed on 4 November 2011],
(1) removing allegiance from the national government of the Province where the following report appeared:
of Maguindanao; and, Executive Secretary Eduardo Ermita admitted Wednesday night that
(2) depriving the Chief Executive of her powers and prerogatives to President Arroyo proclaimed martial law in Maguindanao without an
enforce the laws of the land and to maintain public order and safety. “actual” rebellion taking place in the province as required by the
While the government is at present conducting legitimate operations to Constitution.
address the on-going rebellion, public safety still requires the continued But in response to questions raised by Albay Rep. Edcel Lagman,
implementation of martial law and the suspension of the privilege of the Ermita pointed to the presence of armed groups supporting the Ampatuan
writ of habeas corpus in the Province of Maguindanao until the time that family that were preventing the authorities from enforcing the law, which,
such rebellion is completely quelled.56 (Emphasis supplied) he added, was frustrating the ends of justice.
The question now is whether there was probable cause, which is the required Ermita said the government considered the “presence” or “massing” of
quantum of proof, to declare the existence of rebellion justifying the President’s the Ampatuans’ armed followers as “rebellion,” one of only two grounds
declaration of martial law and suspension of the writ. under the Constitution, aside from invasion, for the imposition of martial
The answer is in the negative. law.
The contemporaneous public statements made by the President’s alter egos 577
explaining the grounds for the issuance of Proclamation No. 1959 negate rather
VOL. 668, MARCH 20, 2012 577
than establish the existence of an actual rebellion in Maguindanao.
During the interpellations in the Joint Session of Congress, convened pursuant Fortun vs. Macapagal-Arroyo
to the provisions of Section 18, Article VII of the Constitution, then Executive Also, during the Joint Session, then Senator (now President) Benigno S.
Secretary Eduardo Ermita admitted the absence of an actual rebellion in Aquino III pointed out the public statements made by former Department of
Maguindanao, to wit: Interior and Local Government Secretary Ronaldo V. Puno, then Armed Forces of
REP. LAGMAN. Mr. Speaker, Mr. President, a perusal of the text of the Philippines spokesperson Lt. Col. Romeo Brawner, and former Defense
Proclamation No. 1959 would show the absence of a clear and categorical finding Secretary Norberto Gonzales admitting there was no need for martial law:
or determination that actual rebellion is occur- THE SENATE PRESIDENT. With the indulgence of the Chamber and the
_______________ Speaker, may we request now to allow the distinguished Gentleman from Tarlac,
56 Rollo (G.R. No. 190293), pp. 163-164, 173-177, 182. Senator Benigno “Noynoy” Aquino III the floor.
576 SEN. AQUINO. Thank you, Mr. President. May I direct my first question to
576 SUPREME COURT REPORTS ANNOTATED Secretary Puno. And this is to lay the proper predicate for our first question. The
newspaper has been quoting Secretary Puno as not having recommended the
Fortun vs. Macapagal-Arroyo imposition of martial law prior to its imposition in Maguindanao. May we know if
ring in Maguindanao. Would that be an accurate observation of a reading of the this was a correct attribution to the Honorable Secretary.
text of Proclamation No. 1959? _______________
MR. ERMITA. Your Honor, you may be correct that there was no actual Ermita though conceded there was no actual rebellion taking place, in
rebellion going on. However, all the indicators that rebellion is, indeed, being the sense of people taking up arms to withdraw allegiance from the central
committed and happening on the ground is because of the presence of the armed government or prevent it from enforcing the law.
Lagman said that Ermita’s answers to his questions and Justice 579
Secretary Agnes Devanadera’s statement that there was rebellion in VOL. 668, MARCH 20, 2012 579
Maguindanao was only “looming” prove that Mrs. Arroyo received “bad
legal advice” in imposing martial rule in the province. Fortun vs. Macapagal-Arroyo
“The President has enough powers under the commander-in-chief Significantly, at a press conference, then Secretary of Justice Agnes
provision of the Constitution to quell a ‘looming’ rebellion or neutralize the Devanadera declared, “We noticed and observed there was a rebellion in the
‘presence’ or ‘massing’ of armed loyalists of the Ampatuans. She is offing.” In another press briefing, Devanadera stated that “rebellion which does
authorized to call on the Armed Forces to accomplish that objective,” not necessarily involve a physical takeover by armed elements as argued by some
Lagman said. critics of the President’s order, was “looming in Maguindanao.”60 In short, the
Lagman pointed out the absence of rebellion in Maguindanao as defined Department of Justice Secretary, who is the principal legal officer of the Arroyo
under the Revised Penal Code. administration, publicly admitted that there was only a “looming” rebellion, a
“What happened there was lawlessness. It was just a partisan conflict “rebellion in the offing,” in Maguindanao.
that did not require the imposition of martial law,” he said. Likewise, in a press conference, “the AFP Chief of Staff claimed that armed
578 groups, numbering between 40 to 400 men and spread out in the province, planned
to prevent the arrest of members of the Ampatuan family, the prime suspects in
578 SUPREME COURT REPORTS ANNOTATED
the Maguindanao massacre. He stated, “Based on the reports we received, there
Fortun vs. Macapagal-Arroyo were a lot of groupings of armed groups in different places. We also received reports
MR. PUNO. Until, Your Honor, Mr. Speaker, Mr. Senate President, until the that they have plans to undertake hostile action if ever government officials, the
situation developed where police officers went absent on leave and joined the rebel Ampatuans particularly, were taken in custody. We felt this was very
forces, and a significant segment of the civilian armed volunteers of the local imminent threat, that’s why we recommended this proclamation.”61
governments constituted themselves into a rebel group, until that time I did not Then Defense Secretary Norberto Gonzales was quoted as stating that the
believe that it was necessary that martial law be declared. But upon receipt of a “recommendation to declare martial law in Maguindanao is a sensitive matter that
report from the Armed Forces of the Philippines and the briefing conducted with needs to be studied.”62 In an interview, Gonzales said, “titingnan natin (we will
the National Security Council, where it was made clear that a separate rebel see) how the situation develops there.”63 He further stated, “As of
armed group had already been organized, we concurred, Your Honor, with the _______________
recommendation on martial law. law-maguindanao [accessed on 10 November 2011], Arroyo proclaims martial
SEN. AQUINO. For the record, Mr. Senate President and Mr. Speaker, the AFP, law in Maguindanao.
we understand, through the spokesperson, Lt. Col. Romeo Brawner, declared on 60 http://www.philstar.com/Article.aspx?articleid=529869 [accessed on 4
13 November 2009 that there is no need for the declaration of martial law in November 2011], DOJ: Rebellion was looming.
Maguindanao or elsewhere in the country because the AFP and PNP are on top of 61 Mantawil Petition (G.R. No. 190356), pp. 8-9.
the situation. He was quoted as saying, and we quote: “We now have a level of 62 http://www.mb.com.ph/node/231907/martial-law-idea-need [accessed on 4
normalcy in the Province of Maguindanao, primarily because of the occupation by November 2011], Martial law idea needs study — Gonzales.
our government forces and our law enforcement agencies of the seats of 63 Id.
government.” Secretary Norberto Gonzales, who unfortunately is not present, 580
declared on December 1, 2009 that the government’s effort to contain the tension 580 SUPREME COURT REPORTS ANNOTATED
in the province is holding ground. We also have now the admission by the
honorable Secretary Puno that prior to the undated national security briefing, he Fortun vs. Macapagal-Arroyo
was also of the opinion that martial law was not necessary in Maguindanao. x x now, I think whatever the government is doing so far is really effective. We will
x58 wait for the results of the work of Secretary Devanadera of Justice and also
Even before the interpellations in Congress, then Executive Secretary Ermita Secretary Puno of DILG. So, so far maganda naman yun takbo ng ating operation
publicly confirmed the inadequacies of Proclamation No. 1959: doon.”64 Gonzales added, “Yung tungkol sa martial law, alam mo sensitive na
“We’ll have to get the report from the field from the AFP and PNP that the bagay yan kaya pag-aaralan natin.”65
conditions that prompted the President to issue the proclamation, have improved, The admissions and public statements made by members of the Cabinet, who
and therefore, the threat of further lawlessness and probability of are the President’s alter egos, as well as the public assessments made by the
rebellion is already down.”59 (Emphasis supplied) highest ranking military officials, clearly demonstrate that instead of being
_______________ anchored on the existence of an actual rebellion, Proclamation No. 1959 was based
58 Transcript of Plenary Proceedings, Joint Session of the Congress of the on a mere threat, or at best an imminent threat of rebellion, or a rebellion “in the
Republic of the Philippines, 9 December 2009. offing.”66 This undeniably runs counter to the letter and intent of the Constitution.
59 Quoted in the Petition in G.R. No. 190307, p. 15, citing http://www.abs- A looming rebellion is analogous to imminent danger of rebellion, which was
cbnnews.com/nation/12/04/09/arroyo-orders-martial- deliberately eliminated by the framers of the 1987 Constitution as a ground for the
declaration of martial law precisely to avoid a repetition of the misguided and 67 Proclamation No. 1081 (PROCLAIMING A STATE OF MARTIAL LAW IN THE
oppressive martial law imposed by former President Marcos. PHILIPPINES), 21 September 1972.
There is absolutely nothing which shows that the Ampatuans and their 582
armed followers, at any point in time, intended to overthrow the government. On 582 SUPREME COURT REPORTS ANNOTATED
the contrary, the Ampatuans were publicly known as very close political allies of
President Arroyo. There is not a single instance where the Ampatuans denounced, Fortun vs. Macapagal-Arroyo
expressly or impliedly, the government, or attempted to remove allegiance to the ian rule in the country. Expectedly, President Arroyo’s Proclamation No. 1959
government or its laws or to deprive the President or Congress of any of their refreshed the nation’s bitter memories of the tyranny during the Martial Law
powers. Based on the records, what the government clearly established, among regime of President Marcos, and sparked the public’s vigilance to prevent a
others, were (1) the existence of the Am- possible recurrence of that horrible past.
_______________ In issuing Proclamation No. 1959, President Arroyo exercised the most
64 Id. awesome and powerful among her graduated Commander-in-Chief powers to
65 Id. suppress a supposed rebellion in Maguindanao, following the massacre of 57
66 http://newsinfo.inquirer.net/breakingnews/nation/view/ civilians in the worst election-related violence in the country’s history. Since then,
20091205-240273/A-rebellion-was-in-the-offingjustice-chief [accessed on 4 the government branded the Ampatuans, the alleged masterminds of the
November 2011], ‘A rebellion was in the offing’—justice chief. massacre, as rebels orchestrating the overthrow of the Arroyo administration.
581 However, the events before, during, and after the massacre negate the existence
of an armed uprising aimed at bringing down the government, but rather point to
VOL. 668, MARCH 20, 2012 581
a surfeit of impunity and abuse of power of a political clan closely allied with the
Fortun vs. Macapagal-Arroyo Arroyo administration. In short, Proclamation No. 1959 was issued without an
patuans’ private army; and (2) the Ampatuans’ vast collection of high powered actual rebellion justifying the same.
firearms and ammunitions. Apparently, President Arroyo resorted to martial law and suspension of the
These shocking discoveries, however, do not amount to rebellion as defined in writ, not to quell a purported rebellion because there was absolutely none, but to
Article 134 of the Revised Penal Code. Based on the statements made by ranking show her indignation over the gruesome massacre and her swift response in
government and military officials, and as clearly found by the RTC-Quezon City in addressing the difficult situation involving her close political allies. She was
Criminal Case No. Q-10-162667 and affirmed by the Court of Appeals, there was reported to be “under pressure to deliver, amid rising public outrage and
no public uprising and taking arms against the government for the international condemnation of the massacre.”68 However, mounting pressure to
purpose of removing from the allegiance to the government or its laws bring the murderers to justice, without any invasion or rebellion in Maguindanao,
the territory of the Philippines or any part thereof, or depriving the Chief does not warrant the imposition of martial law or suspension of the writ. Rather,
Executive or Congress, wholly or partially, of any of their powers and what the nation expects, and what the victims and their families truly deserve, is
prerogatives. The Ampatuans’ amassing of weaponry, including their collection the
of armored cars, tanks and patrol cars, merely highlights this political clan’s _______________
unbelievably excessive power and influence under the Arroyo administration. 68http://www.time.com/time/world/article/0,8599,1943191,00.html
To repeat, only in case of actual invasion or rebellion, when public safety [accessed on 4 November 2011], Behind the Philippines’ Maguindanao Massacre,
requires it, may the President declare martial law or suspend the writ. In declaring by Alastair McIndoe.
martial law and suspending the writ in Maguindanao in the absence of an actual 583
rebellion, President Arroyo indisputably violated the explicit provisions of Section VOL. 668, MARCH 20, 2012 583
18, Article VII of the Constitution.
Fortun vs. Macapagal-Arroyo
Conclusion speedy and credible investigation and prosecution, and eventually the conviction,
of the merciless killers.
Thirty-seven years after President Marcos’ Proclamation No. 1081, President In sum, Proclamation No. 1959 was anchored on a non-existent rebellion.
Arroyo issued Proclamation No. 1959 declaring martial law and suspending the Based on the events before, during and after the Maguindanao massacre, there
privilege of the writ of habeas corpus in the province of Maguindanao, except in was obviously no rebellion justifying the declaration of martial law and suspension
MILF identified areas. President Marcos’ martial law, justified to counteract the of the writ. The discovery of the Ampatuans’ private army and massive weaponry
Communist insurgency in the country,67 turned out to be a vehicle to establish a does not establish an armed public uprising aimed at overthrowing the
one-man authoritar- government. Neither do the closure of government offices and the reluctance of the
_______________ local government officials and employees to report for work indicate a rebellion.
The Constitution is clear. Only in case of actual invasion or rebellion, when
public safety requires it, can a state of martial law be declared or the privilege of
the writ of habeas corpus be suspended. Proclamation No. 1959 cannot be justified VOL. 668, MARCH 20, 2012 585
on the basis of a threatened, imminent, or looming rebellion, which ground was
intentionally deleted by the framers of the 1987 Constitution. Considering the non- Fortun vs. Macapagal-Arroyo
existence of an actual rebellion in Maguindanao, Proclamation No. 1959 is Subsequently, on December 6, 2009, President Arroyo submitted her Report3 to
unconstitutional for lack of factual basis as required under Section 18, Article VII Congress in compliance with Section 18, Article VII of the 1987 Constitution.
of the Constitution for the declaration of martial law and suspension of the Meanwhile, the instant petitions were filed challenging the constitutionality of
privilege of the writ of habeas corpus. Proclamation No. 1959.
Accordingly, I vote to GRANT the petitions and DECLARE Proclamation No. Also consonant with Sec. 18, Art. VII of the 1987 Constitution, Congress
1959 UNCONSTITUTIONAL for failure to comply with Section 18, Article VII of convened in joint session on December 9, 2009.
the Constitution. Eventually, on December 12, 2009, President Arroyo lifted martial law and
DISSENTING OPINION restored the privilege of the writ of habeas corpus in Maguindanao with the
VELASCO, JR., J.: issuance of Proclamation No. 1963.4
The martial law era has left the country with harrowing memories of a dark Justiciability of the instant petitions
past, thus invoking passionate sentiments from the people and bringing forth In the majority opinion, the Court declined to rule on the constitutionality of
remarkable vigilance as a lesson learned, and only rightfully so. Nonetheless, legal Proclamation No. 1959, racionating that “given the prompt lifting of the
dis- proclamation before Congress could review it and before any serious question
584 affecting the rights and liberties of Maguindanao’s inhabitants could arise, the
Court deems any review of its constitutionality the equivalent of beating a dead
584 SUPREME COURT REPORTS ANNOTATED
horse.”
Fortun vs. Macapagal-Arroyo It is my view that, despite the lifting of the martial law and restoration of the
course must be made within bounds, as must always be the case in a civilized privilege of the writ, the Court must take the bull by the horn to guide, explain
society governed by the rule of law and not of men. It is on the basis of the foregoing and elucidate to the executive branch, the legislative branch, the bar, and more
precept that I am constrained to register my dissent in the instant case. importantly the public on the parameters of a declaration of martial law.
As can be gathered from the ponencia, the controversy in the instant case Indeed, it is a well-settled rule that this Court may only adjudicate actual and
revolves around the issuance by then President Gloria Macapagal-Arroyo current controversies.5 This is because the Court is “not empowered to decide moot
(President Arroyo) of Proclamation No. 1959,1which declared a state of martial law questions or abstract
and suspended the privilege of the writ of habeas corpus in the province of _______________
Maguindanao, except for certain identified areas of the Moro Islamic Liberation 3 Id., at pp. 163-182.
Front. 4 Id., at pp. 190-191.
To recall, the issuance of Proclamation No. 1959 was precipitated by the 5 Pormento v. Estrada, G.R. No. 191988, August 31, 2010, 629 SCRA 530, 533;
chilling and loathsome killing, on November 23, 2009, of 57 innocent civilians, citing Honig v. Doe, 484 U.S. 305 (1988).
including the wife of then Buluan Vice-Mayor Esmail “Toto” Mangudadatu 586
(Mangudadatu), who was supposed to file the latter’s certificate of candidacy for 586 SUPREME COURT REPORTS ANNOTATED
Governor of Maguindanao with the Provincial Office of the Commission on
Elections in Shariff Aguak, accompanied by Mangudadatu’s relatives, lawyers and Fortun vs. Macapagal-Arroyo
members of the press, among others. The victims included five others who only propositions, or to declare principles or rules of law which cannot affect the result
happened to be travelling on the same highway traversed by the Mangudadatu as to the thing in issue in the case before it.”6Nonetheless, this “moot and
convoy. academic” rule admits of exceptions. As We wrote in David v. Arroyo:
As a consequence of the detestable killings tagged by media as the “The “moot and academic” principle is not a magical formula that can
“Maguindanao massacre,” President Arroyo immediately issued Proclamation No. automatically dissuade the courts in resolving a case. Courts will decide cases,
19462 on the following day, November 24, 2009, by which a state of emergency was otherwise moot and academic, if: first, there is a grave violation of the
declared in the provinces of Maguindanao and Sultan Kudarat, and in the City of Constitution; second, the exceptional character of the situation and the
Cotabato, “to prevent and suppress the occurrence of similar other incidents of paramount public interest is involved; third, when constitutional issue
lawless violence in Central Mindanao.” This was followed with the issuance of the raised requires formulation of controlling principles to guide the bench,
assailed Proclamation No. 1959 on December 4, 2009. the bar, and the public; and fourth, the case is capable of repetition yet
_______________ evading review.”7(Emphasis supplied.)
1 Rollo (G.R. No. 190293), pp. 186-187. All the aforementioned exceptions are present in this case. First, in the instant
2 Id., at p. 185. petitions, it was alleged that the issuance of Proclamation No. 1959 is violative of
585 the Constitution. Second, it is indubitable that the issues raised affect the public’s
interest as they may have an unsettling effect on the fundamental rights of the
people. Third, the Court has the duty to formulate controlling principles Fortun vs. Macapagal-Arroyo
concerning issues which involve the declaration of martial law and suspension of immediately. The Supreme Court must respect the revocation by Congress even if
the privilege of the writ of habeas corpus to guide the bench, the bar, and the the Court believes a rebellion exists because Congress has the unlimited power to
public. And fourth, the assailed proclamation is capable of repetition yet evading revoke the declaration or suspension.
review. Considerably, the instant petitions are subject to judicial review. Third, the Supreme Court decides first and rules that there is factual basis for
While I disagree with the majority, I wish, however, to take exception to certain the declaration of martial law or suspension of the writ. In such a situation,
suppositions and discourse made in the dissent of Justice Carpio. In particular, I Congress can still revoke the declaration or suspension as its power under the
refer to his discussion on hypothetical situations concerning the simultaneous Constitution is broader insofar as the declaration or suspension is concerned.
exercise of the power to review by this Court and by the Congress, as well as to the “Congress cannot be prevented by the Court from revoking the President’s decision
proposition that “[i]n declaring martial law and suspending the writ in because it is not for the Court to determine what to do with an existing factual
Maguindanao in the absence situation. x x x Congress has been given unlimited power to revoke the President’s
_______________ decision.” In short, even if there is an actual rebellion, whether affirmed or not by
6 Id. the Supreme Court, Congress has the power to revoke the President’s declaration
7 G.R. Nos. 171396, 171409, 171485, 171483, 171400, 171489 & 171424, May or suspension” (Italics in the original; citations omitted.)
3, 2006, 489 SCRA 160. With the exception of the first, the two other possible scenarios adverted to
587 that may arise from the action or inaction of the two co-equal branches of the
VOL. 668, MARCH 20, 2012 587 government upon the declaration by the President of martial law or suspension of
Fortun vs. Macapagal-Arroyo the writ cannot be resolved in the present case. Otherwise, this Court would, in
of an actual rebellion, President Arroyo indisputably violated the explicit effect, be making a ruling on a hypothetical state of facts which the Court is
provisions of Section 18, Article VII of the Constitution.” proscribed from doing.
As We have mentioned in Albay Electric Cooperative, Inc. v. Santelices, “[i]t is
Simultaneous exercise by the Court
and the Congress of their constitu- a rule almost unanimously observed that courts of justice will take cognizance only
tional power to review of justiciable controversies wherein actual and not merely hypothetical
One of the matters traversed by the dissent of Justice Carpio is “[i]f the issues are involved.”9 The reason behind this requisite is “to prevent the courts
constitutional power of this Court to review the factual basis of the declaration of through avoidance of premature adjudication from entangling themselves in
martial law or suspension of the writ can be exercised simultaneously with the abstract disagreements, and for us to be satisfied that the case does not present a
_______________
constitutional power of the Congress to revoke the declaration of martial law or
suspension of the writ, and if the decision of this Court conflicts with the decision 9 G.R. No. 132540, April 16, 2009, 585 SCRA 103, 118-119; citing Jaafar v.
of Congress, which decision shall prevail[?]”8 Commission on Elections, 364 Phil 322, 327-328; 304 SCRA 672, 677-678 (1999);
In addressing this issue, Justice Carpio, in his dissent, considered three emphasis supplied.
scenarios, to wit: 589
“First, the President’s martial law declaration or suspension of the writ is VOL. 668, MARCH 20, 2012 589
questioned in the Supreme Court without Congress acting on the same. Such a Fortun vs. Macapagal-Arroyo
situation generates no conflict between the Supreme Court and Congress. There is hypothetical injury or a claim contingent upon some event that has not and indeed
no question that the Supreme Court can annul such declaration or suspension if it may never transpire.”10
lacks factual basis. Congress, whose only power under Section 18, Article VII of Further, the discussions made in Justice Carpio’s dissent, and curiously, even
the Constitution is to revoke the initial declaration or suspension on any ground, in the majority opinion itself, fail to take into consideration the powers of review
is left with nothing to revoke if the Court has already annulled the declaration. by this Court under its expanded jurisdiction as conferred by Sec. 1, Art. VIII of
Second, Congress decides first to revoke the martial law declaration or the Constitution, “which includes the authority to determine whether grave abuse
suspension of the writ. Since the Constitution does not limit the grounds for of discretion amounting to excess or lack of jurisdiction has been committed by any
congressional revocation, Congress can revoke the declaration or suspension for branch or instrumentality of the government.”11
policy reasons, or plainly for being insignificant, as for instance it involves only In his dissent, Justice Carpio explicitly declares that “Congress has the
one barangay rebelling, or if it finds no actual rebellion. In this case, the Supreme unlimited power to revoke the declaration or suspension.” Similarly, the majority,
Court is left with nothing to act on as the revocation by Congress takes effect in justifying the Court’s refusal to exercise its judicial power of review, states that
_______________ “[o]nly when Congress defaults in its express duty to defend the Constitution
8 Rollo (G.R. No. 190293), pp. 192-194. Resolution dated December 15, 2009. through such review should the Supreme Court step in as its final rampart.”
588 Irresistibly implied in these statements is that once Congress acts and reviews the
588 SUPREME COURT REPORTS ANNOTATED declaration of martial law and suspension of the privilege of the writ, this Court
becomes powerless to make further inquiry on the sufficiency of the factual basis There is indeed a plethora of cases in which this Court exercised the power of
of the proclamation in an appropriate proceeding filed by any citizen as mandated judicial review over congressional action. Thus, in
under Sec. 18, Art. VII of the Constitution. 591
The categorical statements made in both the majority opinion and in Justice VOL. 668, MARCH 20, 2012 591
Carpio’s dissent minimize, if not totally disregard, the power of this Court to pass
upon the constitutionality of acts of Congress under its expanded jurisdiction Fortun vs. Macapagal-Arroyo
under the Constitution. The significance of this Court’s power to review under its Santiago v. Guingona, Jr., this Court ruled that it is well within the power and
“expanded certiorari jurisdiction” was jurisdiction of the Court to inquire whether the Senate or its officials committed a
_______________ violation of the Constitution or grave abuse of discretion in the exercise of their
10 Separate Opinion of Justice Nachura in De Castro v. Judicial and Bar functions and prerogatives. In Tanada v. Angara, in seeking to nullify an act of the
Council, G.R. Nos. 191002, 191032, 191057, A.M. No. 10-2-5-SC, G.R. Nos. 191149, Philippine Senate on the ground that it contravened the Constitution, it held that
191342 & 191420, March 17, 2010, 615 SCRA 666, 780; citing Office of the the petition raises a justiciable controversy and that when an action of the
Governor v. Select Committee of Inquiry, 271 Conn. 540, 570, 858 A. 2d 709 (2004). legislative branch is seriously alleged to have infringed the Constitution, it
11 Coseteng v. Mitra, G.R. No. 86649, July 12, 1990, 187 SCRA 377, 383. becomes not only the right but in fact the duty of the judiciary to settle the dispute.
590 In Bondoc v. Pineda, this Court declared null and void a resolution of the House of
Representatives withdrawing the nomination, and rescinding the election, of a
590 SUPREME COURT REPORTS ANNOTATED
congressman as a member of the House Electoral Tribunal for being violative of
Fortun vs. Macapagal-Arroyo Section 17, Article VI of the Constitution. In Coseteng v. Mitra, it held that the
extensively discussed in Francisco, Jr. v. Nagmamalasakit na mga Manananggol resolution of whether the House representation in the Commission on
ng mga Manggagawang Pilipino, Inc.: Appointments was based on proportional representation of the political parties as
“As indicated in Angara v. Electoral Commission, judicial review is indeed an provided in Section 18, Article VI of the Constitution is subject to judicial review.
integral component of the delicate system of checks and balances which, together In Daza v. Singson, it held that the act of the House of Representatives in removing
with the corollary principle of separation of powers, forms the bedrock of our the petitioner from the Commission on Appointments is subject to judicial review.
republican form of government x x x. In Tanada v. Cuenco, it held that although under the Constitution, the legislative
The separation of powers is a fundamental principle in our system of power is vested exclusively in Congress, this does not detract from the power of the
government. It obtains not through express provision but by actual division in courts to pass upon the constitutionality of acts of Congress. In Angara v. Electoral
our Constitution. Each department of the government has exclusive cognizance of Commission, it ruled that confirmation by the National Assembly of the election of
matters within its jurisdiction, and is supreme within its own sphere. But it does any member, irrespective of whether his election is contested, is not essential
not follow from the fact that the three powers are to be kept separate and distinct before such member-elect may discharge the duties and enjoy the privileges of a
that the Constitution intended them to be absolutely unrestrained and member of the National Assembly.
independent of each other. The Constitution has provided for an elaborate Finally, there exists no constitutional basis for the contention that the exercise
system of checks and balances to secure coordination in the workings of of judicial review over impeachment proceedings would upset the system of checks
the various departments of the government. x x x And the judiciary in and balances. Verily, the Constitution is to be interpreted as a whole and “one
turn, with the Supreme Court as the final arbiter, effectively checks the section is not to be allowed to defeat another.” Both are integral components of the
other departments in the exercise of its power to determine the law, and calibrated system of independence and interdependence that insures that no
hence to declare executive and legislative acts void if violative of the branch of government act beyond the powers assigned to it by the
Constitution. Constitution.”12 (Emphasis in the original; citations omitted.)
In the scholarly estimation of former Supreme Court Justice Florentino _______________
Feliciano, “x x x judicial review is essential for the maintenance and enforcement 12 G.R. No. 160261, November 10, 2003, 415 SCRA 44, 123-124, 132-133.
of the separation of powers and the balancing of powers among the three great 592
departments of government through the definition and maintenance of the 592 SUPREME COURT REPORTS ANNOTATED
boundaries of authority and control between them.” To him, “[j]udicial review is
the chief, indeed the only, medium of participation—or instrument of Fortun vs. Macapagal-Arroyo
intervention—of the judiciary in that balancing operation.” Indeed, the Court does not have the authority to pass upon the wisdom behind
To ensure the potency of the power of judicial review to curb grave abuse of the acts of the Congress. Nonetheless, the Court is not powerless to review the
discretion by “any branch or instrumentalities of government,” the afore- legality of the manner by which such acts have been arrived at in order to
quoted Section 1, Article VIII of the Constitution engraves, for the first time into determine whether Congress has transgressed the reasonable bounds of its
its history, into block letter law the so-called “expanded certiorari jurisdiction” of power.13 This is an obligation which the Court cannot, and should not, abdicate.
this Court x x x. Moreover, by indicating that Congress, if it so decides to act, has an unlimited
xxxx power to revoke the declaration of a state of martial law or suspension of the
privilege of the writ unfettered by this Court’s power to review, We are treading The following excerpts from the Brief of Amicus Curiae of Fr. Joaquin Bernas,
on treacherous grounds by handing over such an unbridled discretion to Congress. S.J. is illuminating:594
Such statement, to me, partakes of an obiter without precedential value, being 594 SUPREME COURT REPORTS ANNOTATED
unnecessary to resolve the issues and arrive at a proper decision in the present
case. This matter should instead be addressed at the proper case and at the proper Fortun vs. Macapagal-Arroyo
time. “From all these it is submitted that the focus on public safety adds a nuance to
President Arroyo’s alleged indisputable the meaning of rebellion in the Constitution which is not found in the meaning of
violation of the explicit provisions of the the same word in Article 134 of the Penal Code. The concern of the Penal Code,
Constitution after all, is to punish acts of the past. But the concern of the Constitution is to
With due respect to Justice Carpio, I cannot join him in his contention that counter threat to public safety both in the present and in the future arising from
“President Arroyo indisputably violated the explicit provisions of Section 18, present and past acts. Such nuance, it is submitted, gives to the President a degree
Article VII of the Constitution” for declaring martial law and suspending the writ of flexibility for determining whether rebellion constitutionally exists as basis for
in Maguindanao in the absence of an actual rebellion. The magnification is martial law even if facts cannot obviously satisfy the requirements of the Penal
uncalled for. Code whose concern is about past acts. To require that the President must first
When We speak of “violation” in reference to a law, it pertains to an act of convince herself that there can be proof beyond reasonable doubt of the existence
breaking or dishonoring the law.14 The use of said word, coupled with the of rebellion as defined in the Penal Code and jurisprudence can severely restrict
ascription of the term “indisputable,” somehow implies that an act was done the President’s capacity to safeguard public safety for the present and the future
intentionally or and can defeat the purpose of the Constitution.
_______________ What all these point to are that the twin requirements of “actual rebellion or
13 See Coseteng v. Mitra, supra note 11. invasion” and the demand of public safety are inseparably entwined. But whether
14 BLACK’S LAW DICTIONARY (9th ed., 2010). there exists a need to take action in favour of public safety is a factual issue
593 different in nature from trying to determine whether rebellion exists. The need of
public safety is an issue whose existence, unlike the existence of rebellion, is not
VOL. 668, MARCH 20, 2012 593
verifiable through the visual or tactile sense. Its existence can only be determined
Fortun vs. Macapagal-Arroyo through the application of prudential estimation of what the consequences might
wilfully. At worst, its use can even be suggestive of bad faith on the part of the be of existing armed movements. Thus, in deciding whether the President acted
doer. rightly or wrongly in finding that public safety called for the imposition of martial
In the case at bar, there is neither any allegation nor proof that President law, the Court cannot avoid asking whether the President acted wisely and
Arroyo acted in bad faith when she declared martial law and suspended the writ prudently and not in grave abuse of discretion amounting to lack or excess of
of habeas corpus in Maguindanao. There was also no showing that there was a jurisdiction. Such decision involves the verification of factors not as easily
deliberate or intentional attempt on the part of President Arroyo to break or measurable as the demands of Article 134 of the Penal Code and can lead to a
dishonor the Constitution by issuing the assailed proclamation. On the contrary, prudential judgment in favour of the necessity of imposing martial law to ensure
what is extant from the records is that President Arroyo made such declaration public safety even in the face of uncertainty whether the Penal Code has been
and suspension on the basis of intelligence reports that lawless elements have violated. This is the reason why courts in earlier jurisprudence were reluctant to
taken up arms and committed public uprising against the government and the override the executive’s judgment.
people of Maguindanao for the purpose of depriving the Chief Executive of her In sum, since the President should not be bound to search for proof beyond
powers and prerogatives to enforce the laws of the land and to maintain public reasonable doubt of the existence of rebellion and since deciding whether public
order and safety, to the great damage, prejudice and detriment of the people in safety demands action is a prudential matter, the function of the President is far
Maguindanao and the nation as a whole. from different from the func-
President Arroyo cannot be blamed for relying upon the information given to 595
her by the Armed Forces of the Philippines and the Philippine National Police, VOL. 668, MARCH 20, 2012 595
considering that the matter of the supposed armed uprising was within their realm
of competence, and that a state of emergency has also been declared in Central Fortun vs. Macapagal-Arroyo
Mindanao to prevent lawless violence similar to the “Maguindanao massacre,” tion of a judge trying to decide whether to convict a person for rebellion or not. Put
which may be an indication that there is a threat to the public safety warranting differently, looking for rebellion under the Penal Code is different from looking for
a declaration of martial law or suspension of the writ. rebellion under the Constitution.”15
Certainly, the President cannot be expected to risk being too late before Significantly, the President has the discretion to make a declaration of martial
declaring martial law or suspending the writ of habeas corpus. The Constitution, law or suspension of the writ of habeas corpusbased on information or facts
as couched, does not require precision in establishing the fact of rebellion. The available or gathered by the President’s office. It would be preposterous to impose
President is called to act as public safety requires.
upon the President to be physically present at the place where a threat to public Seven petitions, now before the Court, were filed disputing the
safety is alleged to exist as a condition to make such declaration or suspension. constitutionality of the Proclamation. In the Resolutions of 8 and 15 December
In the present case, it should not escape the attention of the Court that 2009, the Court consolidated the petitions and required the Office of the Solicitor
President Arroyo complied with the reportorial requirement in Sec. 18, Art. VII of General to com-
the Constitution, which states that “within forty-eight hours from the 597
proclamation of martial law or the suspension of the privilege of the writ of habeas VOL. 668, MARCH 20, 2012 597
corpus, the President shall submit a report in person or in writing to the Congress.”
Further, it appearing thereafter that when President Arroyo subsequently Fortun vs. Macapagal-Arroyo
received intelligence reports on the advisability of lifting martial law or restoring ment on the petitions. By that time, 15 December 2009, President Arroyo has, on
the writ of habeas corpus in Maguindanao, she immediately issued the 12 December 2009, already issued Proclamation No. 1963 lifting martial law and
corresponding proclamation. restoring the privilege of the writ of habeas corpus in Maguindanao.1
To a certain extent, I conform to Justice Carpio’s dissent as to the The authority of this Court to act on the petitions is embodied in the third
unconstitutionality of Proclamation No. 1959. To my mind, however, it is one thing paragraph of Section 18, Article VII of the 1987 Constitution which states:
to declare a decree issued by the President as unconstitutional, and it is another “The Supreme Court may review in an appropriate proceeding filed by any
to pronounce that she indisputably violated the Constitution. Notably, the power citizen, the sufficiency of the factual basis of the proclamation of martial law or the
to issue the subject decree is expressly granted the President. There is also suspension of the privilege of the writ or the extension thereof, and must
compliance with the report required after the issuance of said decree. However, promulgate its decision thereon within thirty days from its filing.”
the issuance of the subject decree may not be sustained after due consideration of Clearly, the mandate is both grant and limitation of authority. For while the
the circumstances which may or may not support such decree. Court, upon a proceeding filed by any citizen, may review the sufficiency of the
_______________ factual basis of the proclamation of martial law by the President, or even its
15 Rollo (G.R. No. 190293), pp. 516-518. extension by Congress, it can only do so within thirty days from filing of the
596 proceeding, the period within which it MUST PROMULGATE its decision.
Over two (2) years have passed since the seven petitions at bar were filed.
596 SUPREME COURT REPORTS ANNOTATED
Today, unquestionably, the Constitutional authority granted to the Court to decide
Fortun vs. Macapagal-Arroyo the petitions had lapsed.
This dissent fears that overbearing declarations may later create an It must be made clear that I do not rely, for my position, on the act of the
unwarranted limitation on the power of a President to respond to exigencies and doer2 “voluntarily ceasing to perform the challenged conduct” or, precisely, on the
requirements of public safety. We must recognize that as society progresses, then lifting of martial law by Proclamation No. 1963. Indeed, from the time of lifting on
so may the manner and means of endangering the very existence of our society 12 December 2009 up to the thirtieth day following the filing of the instant
develop. This Court is fortunate for having the benefit of hindsight. This benefit petitions, Proclamation No. 1959 may be reviewed
may not be equally shared by the President, who is tasked to act with a sense of _______________
urgency based on best judgment as facts develop and events unfold. We may only 1 This and the immediately preceding paragraph were taken from
be judges of the past. But history will be harsh on a President who is not up to the the ponencia of Justice Antonio T. Carpio.
challenge and declines, or worse, fails to act when so required. 2 Province of North Cotabato v. Government of the Republic of the Philippines
I, therefore, vote to declare Proclamation No. 1959 unconstitutional, but as Peace Panel on Ancestral Domain (GRP), 568 SCRA 402, cited by Justice Antonio
heretofore qualified. T. Carpio, p. 27 of ponencia.
SEPARATE OPINION 598
PEREZ, J.: 598 SUPREME COURT REPORTS ANNOTATED
I concur in the resulting dismissal of these petitions, more than by reason of
their mootness but because I find our action overdue, it being my well-thought-out Fortun vs. Macapagal-Arroyo
position that the constitutional authority of the Supreme Court to review the for all the reasons mentioned in the ponencia against which I do not now dissent.
sufficiency of the factual basis of Proclamation No. 1959 has expired and is no The Court did not say during the permitted time of pronouncement what the
more. majority now deems needed saying. Thereafter, and today, no opinion as judgment
Proclamation No. 1959 declaring martial law and suspending the privilege of is constitutionally permissible.
the writ of habeas corpus in the Province of Maguindanao was issued by then Neither can I join the submission that the question of constitutionality of
President Gloria Macapagal Arroyo on 4 December 2009. In compliance with the Proclamation No. 1959 requires formulation of controlling principles to guide the
mandate of Section 18, Article VII of the present Constitution, she submitted her Executive, Legislative and the public.
Report to Congress on 6 December 2009 or “within forty-eight hours from the Respectfully, I submit that each and every exercise by the President of his
proclamation.” commander-in-chiefship3 must, if review by this Court be asked and called for, be
examined under the current events and the present affairs that determine the ISABELITA C. VINUYA, VICTORIA C. DELA PEÑA, HERMINIHILDA
presence of the necessity of such exercise. MANIMBO, LEONOR H. SUMAWANG, CANDELARIA L. SOLIMAN, MARIA L.
All the decisions of the actors covered by Section 18 of Article VII must be done QUILANTANG, MARIA L. MAGISA, NATALIA M. ALONZO, LOURDES M.
within the tight and narrow time frames in the provision. These framed periods, I NAVARO, FRANCISCA M. ATENCIO, ERLINDA MANALASTAS, TARCILA M.
submit, emphasize the imperative for currency of the decision that each must SAMPANG, ESTER M. PALACIO, MAXIMA R. DELA CRUZ, BELEN A. SAGUM,
make, as indeed, the presidential proclamation, aside from having been subjected FELICIDAD TURLA, FLORENCIA M. DELA PEÑA, EUGENIA M. LALU,
to constitutional checks, has been given limited life. JULIANA G. MAGAT, CECILIA SANGUYO, ANA ALONZO, RUFINA P.
The present limitations of the power to declare martial law, including the MALLARI, ROSARIO M. ALARCON, RUFINA C. GULAPA, ZOILA B.
consequent circumscription of the legislative and judicial participation in the MANALUS, CORAZON C. CALMA, MARTA A. GULAPA, TEODORA M.
exercise of the power, themselves limit the occasion and need for “formulation of HERNANDEZ, FERMIN B. DELA PEÑA, MARIA DELA PAZ B. CULALA,
controlling principles to guide the Executive, Legislative and the public.” The way ESPERANZA MANAPOL, JUANITA M. BRIONES, VERGINIA M. GUEVARRA,
and manner by which the Constitution provided for the commander-in-chief clause MAXIMA ANGULO, EMILIA SANGIL, TEOFILA R. PUNZALAN, JANUARIA G.
require decisions for the present, not guidelines for the future. I respectfully GARCIA, PERLA B. BALINGIT, BELEN A. CULALA, PILAR Q. GALANG,
submit that the Court cannot now define for the future the “sufficiency of the ROSARIO C. BUCO, GAUDENCIA C. DELA PEÑA, RUFINA Q. CATACUTAN,
factual basis” of the possibly coming proclamations of martial law. I cannot see FRANCIA A. BUCO, PASTORA C. GUEVARRA, VICTORIA M. DELA CRUZ,
how such a pre-determination can prevent an unconstitutional imposition of PETRONILA O. DELA CRUZ, ZENAIDA P. DELA CRUZ, CORAZON M. SUBA,
_______________ EMERINCIANA A. VINUYA, LYDIA A. SANCHEZ, ROSALINA M. BUCO,
3 Term used by Bernas, SJ., The 1987 Constitution of the Philippines and PATRICIA A. BERNARDO, LUCILA H. PAYAWAL, MAGDALENA LIWAG,
Commentary, 2003 Ed., p. 865. ESTER C. BALINGIT, JOVITA A. DAVID, EMILIA C. MANGILIT, VERGINIA
599 M. BANGIT, GUILERMA S. BALINGIT, TERECITA PANGILINAN, MAMERTA
VOL. 668, MARCH 20, 2012 599 C. PUNO, CRISENCIANA C. GULAPA, SEFERINA S. TURLA, MAXIMA B.
TURLA, LEONICIA G. GUEVARRA, ROSALINA M. CULALA, CATALINA Y.
Fortun vs. Macapagal-Arroyo MANIO, MAMERTA T. SAGUM, CARIDAD L. TURLA, et al., in their capacity
martial law better than the requirement, already constitutionalized, that the and as members of the “Malaya Lolas Organizations,” petitioners, vs.
President must within forty-eight hours, submit a report in person or in writing to _______________
Congress which can, by a majority of all its members revoke, the imposition. * EN BANC.
WHEREFORE, the cases are declared closed and terminated by constitutional 596THE HONORABLE EXECUTIVE SECRETARY ALBERTO G. ROMULO,
rescript. THE HONORABLE SECRETARY OF FOREIGN AFFAIRS DELIA DOMINGO-
Petitions dismissed for being moot and academic. ALBERT, THE HONORABLE SECRETARY OF JUSTICE MERCEDITAS N.
Notes.—Section 20, Title I of Book III of Executive Order 292 speaks of other GUTIERREZ, and THE HONORABLE SOLICITOR GENERAL ALFREDO L.
powers vested in the President under the law—the exercise of the President’s BENIPAYO, respondents.
residual powers under this provision requires legislation, as the provision clearly Remedial Law; Special Civil Actions; Certiorari; To establish the timeliness
states that the exercise of the President’s other powers and functions has to be of the petition for certiorari, the date of receipt of the assailed judgment, final order
“provided for under the law.” (Review Center Association of the Philippines vs. or resolution or the denial of the motion for reconsideration or new trial must be
Ermita, 583 SCRA 428 [2009]) stated in the petition; otherwise, the petition for certiorari must be dismissed.—
The Court finds it well-within its power to determine whether public Petitioners did not show that their bringing of the special civil action
respondent committed a violation of the Constitution or gravely abused its for certiorari was timely, i.e., within the 60-day period provided in Section 4, Rule
discretion in the exercise of its functions and prerogatives that could translate as 65 of the Rules of Court, to wit: Section 4. When and where position filed.—The
lack or excess of jurisdiction, which would require corrective measures from the petition shall be filed not later than sixty (60) days from notice of judgment, order
Court. (Gutierrez vs. The House of Representatives Committee on Justice, 643 or resolution. In case a motion for reconsideration or new trial is timely filed,
SCRA 198 [2011]) whether such motion is required or not, the sixty (60)-day period shall be counted
——o0o—— from notice of the denial of said motion. As the rule indicates, the 60-day period
© Copyright 2018 Central Book Supply, Inc. All rights reserved. starts to run from the date petitioner receives the assailed judgment, final order
or resolution, or the denial of the motion for reconsideration or new trial timely
filed, whether such motion is required or not. To establish the timeliness of the
petition for certiorari, the date of receipt of the assailed judgment, final order or
resolution or the denial of the motion for reconsideration or new trial must be
stated in the petition; otherwise, the petition for certiorari must be dismissed. The
G.R. No. 162230. August 12, 2014.*
importance of the dates cannot be understated, for such dates determine the body or health, unlawful deportation or transfer or unlawful confinement of
timeliness of the filing of the petition for certiorari. a protected person, compelling a protected person to serve in the forces of a
Same; Provisional Remedies; Preliminary Injunction; Preliminary injunction hostile Power, or wilfully depriving a protected person of the rights of fair and
is provisional because it constitutes a temporary measure availed of during the regular trial prescribed in the present Convention, taking of hostages and
pendency of the action; and it is ancillary because it is a mere incident in and is extensive destruction and appropriation of property, not justified by military
dependent upon the result of the main action.—Preliminary injunction is merely a necessity and carried out unlawfully and wantonly. ARTICLE 148 No High
provisional remedy that is adjunct to the main case, and is subject to the latter’s Contracting Party shall be allowed to absolve itself or any other High
outcome. It is not a cause of action itself. It is provisional because it constitutes a Contracting Party of any liability incurred by itself or by another High
temporary measure availed of during the pendency of the action; and it is ancillary Contracting Party in respect of breaches referred to in the preceding
because it is a mere incident in and is dependent upon the result of the main Article.
action.597Following the dismissal of the petition for certiorari, there is no more Same; Same; Same; View that a state is precluded from absolving other states
legal basis to issue the writ of injunction sought. As an auxiliary remedy, the writ from liability on the ground that the individual persons who actually perpetrated
of preliminary mandatory injunction cannot be issued independently of the the grave breach of the convention have already been punished; Pursuant to the
principal action. 1949 Geneva Conventions, a state remains responsible — and continues to be liable
Same; Same; Mandatory Injunction; A mandatory injunction requires the to pay compensation — for the grave breaches committed against protected
performance of a particular act.—In any event, a mandatory injunction requires persons.—It is said that the non-absolution clause under Article 148 is a logical
the performance of a particular act. Hence, it is an extreme remedy, to be granted consequence of the grave breaches under Article 147 of Geneva Convention IV. A
only if the following requisites are attendant, namely: (a) The applicant has a clear state is precluded from absolving other states from liability on the ground that the
and unmistakable right, that is, a right in esse; (b) There is a material and individual persons who actually perpetrated the grave breach of the convention
substantial invasion of such right; and (c) There is an urgent need for the writ to have already been punished. Indeed, it would seem unjust for individuals to be
prevent irreparable injury to the applicant; and no other ordinary, speedy, and punished while the state in whose name or on whose instructions they acted is
adequate remedy exists to prevent the infliction of irreparable injury. released from all liability. Article 148 was meant to prevent the defeated state from
Constitutional Law; Foreign Relations; The Constitution has entrusted to the being forced into entering into an armistice or peace treaty in which it would
Executive Department the conduct of foreign relations for the Philippines; The renounce all reparations for grave breaches committed by persons in the service of
Supreme Court (SC) cannot interfere with or question the wisdom of the conduct of the victorious state. Thus, pursuant to the 1949 Geneva Conventions, a state
foreign relations by the Executive Department.—The Constitution has entrusted to remains responsible — and continues to be liable to pay compensation — for the
the Executive Department the conduct of foreign relations for the Philippines. grave breaches committed against protected persons.
Whether or not to espouse petitioners’ claim against the Government of Japan is 599
left to the exclusive determination and judgment of the Executive Department. Same; Same; Same; View that I vote to dismiss the petition for failure to
The Court cannot interfere with or question the wisdom of the conduct of foreign establish that respondents committed grave abuse of discretion in declining to
relations by the Executive Department. Accordingly, we cannot direct the espouse the claims of petitioners.—In the light of the foregoing context, I vote to
Executive Department, either by writ of certiorari or injunction, to conduct our dismiss the petition for failure to establish that respondents committed grave
foreign relations with Japan in a certain manner. abuse of discretion in declining to espouse the claims of petitioners. The dismissal
Sereno, CJ., Concurring Opinion: thereof should not, however, be taken as a definitive ruling on the merits of the
Constitutional Law; International Law; Reparations; View that the statement claims of petitioners, in the event that they bring the same to an appropriate forum
in the 2010 ponencia — that the “wisdom of such decision [to waive all claims for or through a proper recourse. Neither should it be taken to mean that we should
reparations] is not for the courts to question” — must be qualified.—I am of the forget the suffering that our people, especially petitioners, bore in the Second
opinion, however, that the statement in the 2010 ponencia — that the World War, or the unfortunate story of our attempts to get the reparation that was
“wisdom of such decision [to waive all claims for reparations] is not for due us, and learn. From such understanding, we must forge the elements that will
the courts to question” — must be qualified. As party to the 1949 Geneva make the Philippine state strong, able to protect its people and safeguard their
Conventions, the Philippines has limited discretion to waive another state’s well-being under the aegis of the Constitution. Justice demands no less.
reparations obligation arising from the commission of grave breaches of the MOTION FOR RECONSIDERATION and SUPPLEMENTAL MOTION FOR
convention. The 1949 Geneva RECONSIDERATION of a decision of the Supreme Court.
598Convention Relative to the Protection of Civilian Persons in Time of War The facts are stated in the resolution of the Court.
(Geneva Convention IV) expressly states the following: ARTICLE 147 Grave Roque & Butuyan Law Offices for petitioners.
breaches to which the preceding Article relates shall be those involving any of The Solicitor General for respondents.
the following acts, if committed against persons or property protected by the RESOLUTION
present Convention: wilful killing, torture or inhuman treatment, including BERSAMIN, J.:
biological experiments, wilfully causing great suffering or serious injury to
Petitioners filed a Motion for Reconsideration1 and a Supplemental Motion for the laws of humanity has long become jus cogens norms, and that international
Reconsideration,2 praying that the Court reverse its decision of April 28, 2010, and legal obligations prevail over national legal norms; that the Court’s invocation of
grant their petition for certiorari. the political doctrine in the instant case is misplaced; and that the Chief Executive
In their Motion for Reconsideration, petitioners argue that our constitutional has the constitutional duty to afford redress and to give justice to the victims of
and jurisprudential histories have rejected the comfort women system in the Philippines.8
_______________ Petitioners further argue that the Court has confused diplomatic protection
1 Rollo, pp. 419-429. with the broader responsibility of states to protect the human rights of their
2 Id., at pp. 435-529. citizens, especially where the rights asserted are subject of erga omnes obligations
600the Court’s ruling that the foreign policy prerogatives of the Executive Branch and pertain to jus cogensnorms; that the claims raised by petitioners are not
are unlimited; that under the relevant jurisprudence and constitutional provisions, simple private claims that are the usual subject of diplomatic protection; that the
such prerogatives are proscribed by international human rights and international crimes committed against petitioners are shocking to the conscience of humanity;
conventions of which the Philippines is a party; that the Court, in holding that the and that the atrocities committed by the Japanese
Chief Executive has the prerogative whether to bring petitioners’ claims against _______________
Japan, has read the foreign policy powers of the Office of the President in isolation 5 G.R. No. 101949, December 1, 1994, 238 SCRA 524.
from the rest of the constitutional protections that expressly textualize 6 G.R. No. 76607, February 26, 1990, 182 SCRA 644.
international human rights; that the foreign policy prerogatives are subject to 7 No. L-49112, February 2, 1979, 88 SCRA 195.
obligations to promote international humanitarian law as incorporated into the 8 Supra note 1.
laws of the land through the Incorporation Clause; that the Court must revisit its 602soldiers against petitioners are not subject to the statute of limitations under
decisions in Yamashita v. Styer3 and Kuroda v. Jalandoni4 which have been noted international law.9
for their prescient articulation of the import of laws of humanity; that in said Petitioners pray that the Court reconsider its April 28, 2010 decision, and
decision, the Court ruled that the State was bound to observe the laws of war and declare: (1) that the rapes, sexual slavery, torture and other forms of sexual
humanity; that in Yamashita, the Court expressly recognized rape as an violence committed against the Filipina comfort women are crimes against
international crime under international humanitarian law, and in Jalandoni, the humanity and war crimes under customary international law; (2) that the
Court declared that even if the Philippines had not acceded or signed the Hague Philippines is not bound by the Treaty of Peace with Japan, insofar as the waiver
Convention on Rules and Regulations covering Land Warfare, the Rules and of the claims of the Filipina comfort women against Japan is concerned; (3) that
Regulations formed part of the law of the nation by virtue of the Incorporation the Secretary of Foreign Affairs and the Executive Secretary committed grave
Clause; that such commitment to the laws of war and humanity has been abuse of discretion in refusing to espouse the claims of Filipina comfort women;
enshrined in Section 2, Article II of the 1987 Constitution, which provides “that and (4) that petitioners are entitled to the issuance of a writ of preliminary
the Philippines…adopts the generally accepted principles of international law as injunction against the respondents.
part of the law of the land and adheres to the policy of peace, equality, justice, Petitioners also pray that the Court order the Secretary of Foreign Affairs and
freedom, cooperation, and amity with all nations.” the Executive Secretary to espouse the claims of Filipina comfort women for an
The petitioners added that the status and applicability of the generally official apology, legal compensation and other forms of reparation from Japan. 10
accepted principles of international law within the Philippine jurisdiction would In their Supplemental Motion for Reconsideration, petitioners stress that it
be uncertain without the was highly improper for the April 28, 2010 decision to lift commentaries from at
_______________ least three sources without proper attribution — an article published in 2009 in
3 75 Phil. 563 (1945). the Yale Law Journal of International Law; a book published by the Cambridge
4 83 Phil. 171 (1949). University Press in 2005; and an article published in 2006 in the Western Reserve
601Incorporation Clause, and that the clause implied that the general Journal of International Law — and make it appear that such commentaries
international law forms part of Philippine law only insofar as they are expressly supported its arguments for dismissing the petition, when in truth the plagiarized
adopted; that in its rulings in The Holy See, v. Rosario, Jr.5 and U.S. v. Guinto6 the sources even made a strong case in favour of petitioners’ claims.11
Court has said that international law is deemed part of the Philippine law as a _______________
consequence of Statehood; that in Agustin v. Edu,7 the Court has declared that a 9 Id., at pp. 426-427.
treaty, though not yet ratified by the Philippines, was part of the law of the land 10 Id., at pp. 427-428.
through the Incorporation Clause; that by virtue of the Incorporation Clause, the 11 Id., at p. 436.
Philippines is bound to abide by the erga omnes obligations arising from the jus 603 In their Comment,12 respondents disagree with petitioners,
cogens norms embodied in the laws of war and humanity that include the principle maintaining that aside from the statements on plagiarism, the arguments raised
of the imprescriptibility of war crimes; that the crimes committed against by petitioners merely rehashed those made in their June 7, 2005 Memorandum;
petitioners are proscribed under international human rights law as there were that they already refuted such arguments in their Memorandum of June 6, 2005
undeniable violations of jus cogens norms; that the need to punish crimes against that the Court resolved through its April 28, 2010 decision, specifically as follows:
1. The contentions pertaining to the alleged plagiarism were then already motion for reconsideration or new trial is timely filed, whether such motion is
lodged with the Committee on Ethics and Ethical Standards of the Court; hence, required or not, the sixty (60)-day period shall be counted from notice of the denial
the matter of alleged plagiarism should not be discussed or resolved herein. 13 of said motion.
2. A writ of certiorari did not lie in the absence of grave abuse of discretion As the rule indicates, the 60-day period starts to run from the date petitioner
amounting to lack or excess of jurisdiction. Hence, in view of the failure of receives the assailed judgment, final order or resolution, or the denial of the motion
petitioners to show any arbitrary or despotic act on the part of respondents, the for reconsideration or new trial timely filed, whether such motion is required or
relief of the writ of certiorari was not warranted.14 not. To establish the timeliness of the petition for certiorari, the date of receipt of
3. Respondents hold that the Waiver Clause in the Treaty of Peace with the assailed judgment, final order or resolution or the denial of the motion for
Japan, being valid, bound the Republic of the Philippines pursuant to the reconsideration or new trial must be stated in the petition; otherwise, the petition
international law principle of pacta sunt servanda. The validity of the Treaty of for certiorari must be dismissed. The importance of the dates cannot be
Peace was the result of the ratification by two mutually consenting parties. understated, for such dates determine the timeliness of the filing of the petition
Consequently, the obligations embodied in the Treaty of Peace must be carried out for certiorari. As the Court has emphasized in Tambong v. R. Jorge Development
in accordance with the common and real intention of the parties at the time the Corporation:17
treaty was concluded.15 There are three essential dates that must be stated in a petition
4. Respondents assert that individuals did not have direct international for certiorari brought under Rule 65. First, the date when notice of the judgment
remedies against any State that violated their human rights except or final order or resolution was received; second, when a motion for new trial or
_______________ reconsideration was filed; and third, when notice of the denial thereof was
12 Id., at pp. 665-709. received. Failure of petitioner to comply with this requirement shall be
13 Id., at pp. 684-685. sufficient ground for the dismissal of the petition. Substantial compliance
14 Id., at pp. 686-690. will not suffice in a matter involving strict observance with the Rules.
15 Id., at pp. 690-702. (Emphasis supplied)
604where such remedies are provided by an international agreement. Herein, _______________
neither of the Treaty of Peace and the Reparations Agreement, the relevant 17 G.R. No. 146068, August 31, 2006, 500 SCRA 399, 403-404.
agreements affecting herein petitioners, provided for the reparation of petitioners’ 606
claims. Respondents aver that the formal apology by the Government of Japan and The Court has further said in Santos v. Court of Appeals:18
the reparation the Government of Japan has provided through the Asian Women’s The requirement of setting forth the three (3) dates in a petition
Fund (AWF) are sufficient to recompense petitioners on their claims, specifically: for certiorari under Rule 65 is for the purpose of determining its timeliness. Such
a. About 700 million yen would be paid from the national treasury over the a petition is required to be filed not later than sixty (60) days from notice of the
next 10 years as welfare and medical services; judgment, order or Resolution sought to be assailed. Therefore, that the petition
b. Instead of paying the money directly to the former comfort women, the for certiorari was filed forty-one (41) days from receipt of the denial of the motion
services would be provided through organizations delegated by governmental for reconsideration is hardly relevant. The Court of Appeals was not in any position
bodies in the recipient countries (i.e., the Philippines, the Republic of Korea, and to determine when this period commenced to run and whether the motion for
Taiwan); and reconsideration itself was filed on time since the material dates were not stated. It
c. Compensation would consist of assistance for nursing services (like home should not be assumed that in no event would the motion be filed later than fifteen
helpers), housing, environmental development, medical expenses, and medical (15) days. Technical rules of procedure are not designed to frustrate the ends of
goods.16 justice. These are provided to effect the proper and orderly disposition of cases and
Ruling thus effectively prevent the clogging of court dockets. Utter disregard of the Rules
The Court DENIES the Motion for Reconsideration and Supplemental Motion cannot justly be rationalized by harking on the policy of liberal construction.19
for Reconsideration for being devoid of merit. The petition for certiorari contains the following averments, viz.:
1. 82. Since 1998, petitioners and other victims of the “comfort women system,”
Petitioners did not show that their resort approached the Executive Department through the Department of Justice in order
was timely under the Rules of Court. to request for assistance to file a claim against the Japanese officials and military
Petitioners did not show that their bringing of the special civil action officers who ordered the establishment of the “comfort women” stations in the
for certiorari was timely, i.e., within the 60-day Philippines;
_______________ 83. Officials of the Executive Department ignored their request and refused
16 Id., at pp. 703-706. to file a claim against the said Japanese officials and military officers;
605period provided in Section 4, Rule 65 of the Rules of Court, to wit: _______________
Section 4. When and where position filed.—The petition shall be filed not 18 G.R. No. 141947, July 5, 2001, 360 SCRA 521, 527-528.
later than sixty (60) days from notice of judgment, order or resolution. In case a 19 Id.
607 in the ordinary course of law, a person aggrieved thereby may file a verified
84. Undaunted, the Petitioners in turn approached the Department of petition in the proper court, alleging the facts with certainty and praying that
Foreign Affairs, Department of Justice and Office of the of the Solicitor General to judgment be rendered annulling or modifying the proceedings of such tribunal,
file their claim against the responsible Japanese officials and military officers, but board or officer, and granting such incidental reliefs as law and justice may
their efforts were similarly and carelessly disregarded.20 require.
The petition thus mentions the year 1998 only as the time when petitioners The petition shall be accompanied by a certified true copy of the judgment,
approached the Department of Justice for assistance, but does not specifically state order, or resolution subject thereof, copies of all pleadings and documents relevant
when they received the denial of their request for assistance by the Executive and pertinent thereto, and a sworn certification of nonforum shopping as provided
Department of the Government. This alone warranted the outright dismissal of in the third paragraph of Section 3, Rule 46.
the petition. 609
Even assuming that petitioners received the notice of the denial of their However, petitioners did not make such a showing.
request for assistance in 1998, their filing of the petition only on March 8, 2004 3.
was still way beyond the 60-day period. Only the most compelling reasons could Petitioners were not entitled
justify the Court’s acts of disregarding and lifting the strictures of the rule on the to the injunction.
period. As we pointed out in MTM Garment Mfg., Inc. v. Court of Appeals:21 The Court cannot grant petitioners’ prayer for the writ of preliminary
All these do not mean, however, that procedural rules are to be ignored or mandatory injunction.
disdained at will to suit the convenience of a party. Procedural law has its own Preliminary injunction is merely a provisional remedy that is adjunct to the
rationale in the orderly administration of justice, namely: to ensure the effective main case, and is subject to the latter’s outcome. It is not a cause of action itself. 22 It
enforcement of substantive rights by providing for a system that obviates is provisional because it constitutes a temporary measure availed of during the
arbitrariness, caprice, despotism, or whimsicality in the settlement of disputes. pendency of the action; and it is ancillary because it is a mere incident in and is
Hence, it is a mistake to suppose that substantive law and procedural law are dependent upon the result of the main action.23 Following the dismissal of the
contradictory to each other, or as often suggested, that enforcement of procedural petition for certiorari, there is no more legal basis to issue the writ of injunction
rules should never be permitted if it would result in prejudice to the substantive sought. As an auxiliary remedy, the writ of preliminary mandatory injunction
rights of the litigants. cannot be issued independently of the principal action.24
As we have repeatedly stressed, the right to file a special civil action In any event, a mandatory injunction requires the performance of a particular
of certiorari is neither a natural right act. Hence, it is an extreme remedy,25 to be granted only if the following requisites
_______________ are attendant, namely:
20 Rollo, p. 18. (a) The applicant has a clear and unmistakable right, that is, a right in esse;
21 G.R. No. 152336, June 9, 2005, 460 SCRA 55, 66. (b) There is a material and substantial invasion of such right; and
608nor an essential element of due process; a writ of certiorari is a _______________
prerogative writ, never demandable as a matter of right, and never issued 22 Buyco v. Baraquia, G.R. No. 177486, December 21, 2009, 608 SCRA 699,
except in the exercise of judicial discretion. Hence, he who seeks a writ 703-704.
of certiorari must apply for it only in the manner and strictly in 23 Id., at p. 704.
accordance with the provisions of the law and the Rules. 24 Bangko Sentral ng Pilipinas Monetary Board v. Antonio-Valenzuela, G.R.
Herein petitioners have not shown any compelling reason for us to relax the No. 184778, October 2, 2009, 602 SCRA 698, 715, citing Lim v. Court of Appeals,
rule and the requirements under current jurisprudence. x x x. (Emphasis supplied) G.R. No. 134617, February 13, 2006, 482 SCRA 326, 331.
25 Regalado, Remedial Law Compendium, Vol. I, p. 638, Seventh Revised
2. Edition.
Petitioners did not show that the assailed act 610
was either judicial or quasi-judicial (c) There is an urgent need for the writ to prevent irreparable injury to the
on the part of respondents. applicant; and no other ordinary, speedy, and adequate remedy exists to prevent
the infliction of irreparable injury.26
Petitioners were required to show in their petition for certiorarithat the In Marquez v. The Presiding Judge (Hon. Ismael B. Sanchez), RTC Br. 58,
assailed act was either judicial or quasi-judicial in character. Section 1, Rule 65 of Lucena City,27 we expounded as follows:
the Rules of Court requires such showing, to wit: It is basic that the issuance of a writ of preliminary injunction is addressed to
Section 1. Petition for certiorari.—When any tribunal, board or officer the sound discretion of the trial court, conditioned on the existence of a clear and
exercising judicial or quasi-judicial functions has acted without or in excess of its positive right of the applicant which should be protected. It is an extraordinary,
or his jurisdiction, or with grave abuse of discretion amounting to lack or excess of peremptory remedy available only on the grounds expressly provided by law,
jurisdiction, and there is no appeal, nor any plain, speedy, and adequate remedy specifically Section 3, Rule 58 of the Rules of Court. Moreover, extreme caution
must be observed in the exercise of such discretion. It should be granted only when authoritarians have impelled us to ensure, through the Constitution, that every
the court is fully satisfied that the law permits it and the emergency demands it. Filipino will attain justice and will be guaranteed full respect for human
The very foundation of the jurisdiction to issue a writ of injunction rests in the rights.3 This is one of the core duties imposed by our organic law on public officials.
existence of a cause of action and in the probability of irreparable injury, Nevertheless, I concur with the Resolution holding that there is basis to
inadequacy of pecuniary compensation, and the prevention of multiplicity of suits. dismiss the petition on various technical grounds. I also fully agree with this
Where facts are not shown to bring the case within these conditions, the relief of Court’s Decision4 dated 28 April 2010, which ruled that the matter of exercising
injunction should be refused.28 diplomatic protection is within the sphere of discretion of the executive
Here, the Constitution has entrusted to the Executive Department the department.
conduct of foreign relations for the Philippines. Whether or not to espouse I am of the opinion, however, that the statement in the 2010 ponencia —
petitioners’ claim against the Government of Japan is left to the exclusive that the “wisdom of such decision [to waive all claims for reparations] is
determination and judgment of the Executive Department. The Court cannot not for the courts to question”5 — must be qualified. As party to the 1949
interfere with or question the wisdom of the conduct of foreign relations by the Geneva Conventions,6 the Philippines has limited discretion
Executive Department. Accordingly, _______________
26 Philippine Leisure and Retirement Authority v. Court of Appeals, G.R. No. 2 See: E.O. 292–Administrative Code of the Philippines, Book IV, Title III,
156303, December 19, 2007, 541 SCRA 85, 99-100. Chap. 12, Sec. 34, pars. 10 & 11.
27 G.R. No. 141849, February 13, 2007, 515 SCRA 577. 3 Constitution, Art. II, Sec. 11; Art. XIII, Secs. 1 & 18(3).
28 Id., at p. 589. 4 Vinuya v. Romulo, G.R. No. 162230, 28 April 2010, 619 SCRA 533.
611we cannot direct the Executive Department, either by writ of certiorari or 5 Id., at p. 560.
injunction, to conduct our foreign relations with Japan in a certain manner. 6 Geneva Convention for the Amelioration of the Condition of the Wounded
WHEREFORE, the Court DENIES the Motion for and Sick in Armed Forces in the Field, 12 August 1949, 75 U.N.T.S. 31 (hereinafter
Reconsideration and Supplemental Motion for Reconsideration for their lack of “Geneva Convention I”); Geneva Convention for the Amelioration of the Condition
merit. of the Wounded, Sick and Shipwrecked Members of Armed Forces at Sea, 12
SO ORDERED. August 1949, 75 U.N.T.S. 85 (hereinafter “Geneva Convention II”); Geneva
Sereno (CJ.), Carpio, Velasco, Jr., Leonardo-De Castro, Brion, Peralta, Convention Relative to the Treatment of Prisoners of War, 12 August 1949, 75
Villarama, Jr., Perez, Mendoza, Reyes and Perlas-Bernabe, JJ., concur. U.N.T.S. 135 (hereinafter “Geneva Convention III”); and Geneva Convention
Del Castillo, J., No part. Relative to the Protection of Civilian
Leonen, J., No part. 613to waive another state’s reparations obligation arising from the commission of
CONCURRING OPINION grave breaches of the convention. The 1949 Geneva Convention Relative to the
SERENO, CJ.: Protection of Civilian Persons in Time of War (Geneva Convention IV) expressly
[T]he phrase “comfort women” does not in the least reflect the suffering, states the following:
such as multiple rapes on an everyday basis and severe physical abuse, that ARTICLE 147
women victims had to endure during their forced prostitution and sexual Grave breaches to which the preceding Article relates shall be
subjugation and abuse in wartime. The Special Rapporteur, therefore, those involving any of the following acts, if committed against persons or
considers with conviction that the phrase “military sexual slaves” represents property protected by the present Convention: wilful killing, torture or
a much more accurate and appropriate terminology.1 inhuman treatment, including biological experiments, wilfully causing great
Ms. Radhika Coomaraswamy suffering or serious injury to body or health, unlawful deportation or transfer
Special Rapporteur on or unlawful confinement of a protected person, compelling a protected person
Violence Against Women to serve in the forces of a hostile Power, or wilfully depriving a protected person of
_______________ the rights of fair and regular trial prescribed in the present Convention, taking of
1 Special Rapporteur on Violence Against Women, its Causes and hostages and extensive destruction and appropriation of property, not justified by
Consequences, Rep. on the Mission to the Democratic People’s Republic of Korea, military necessity and carried out unlawfully and wantonly.
the Republic of Korea and Japan on the Issue of Military Sexual Slavery in ARTICLE 148
Wartime, Comm’n. on Human Rights, UN Doc. E/CN.4/1996/53/Add.l, at 4 (4 No High Contracting Party shall be allowed to absolve itself or any
January 1996)(by Radhika Coomaraswamy). other High Contracting Party of any liability incurred by itself or by
612 another High Contracting Party in respect of breaches referred to in the
This Petition is a reminder to all public officials of the trust the Filipino people preceding Article. (Emphases supplied)
have reposed in them to ensure their well-being, address their sufferings, and _______________
promote the rule of law within the national and international sphere. 2 Our history Persons in Time of War, 12 August 1949, 75 U.N.T.S. 287 (hereinafter Geneva
as a nation and our brutal experiences at the hands of colonialists and “Convention IV”).
614 _______________
Legal commentators stress that Article 148 should be related to Article 3 of the 13 Takushi Ohno, War Reparations & Peace Settlement: Philippines-Japan
1907 Hague Convention IV,7 which reads: Relations 1945-1956, p. 8 (1986); Yang Zhihui, From War Reparation to Postwar
A belligerent Party which violates the provisions of the said Regulations Reparation (Louisa Rubinfien trans.), in Toward a History Beyond Borders:
shall, if the case demands, be liable to pay compensation. It shall Contentious Issues in Sino-Japanese Relations, pp. 374-375 (Daqing Yang, Jie Liu,
be responsible for all acts committed by persons forming part of its armed Hiroshi Mitani & Andrew Gordon eds., 2012).
forces. (Emphases supplied) 14 OHNO, id.
It is said that the non-absolution clause under Article 148 is a logical 15 OHNO, id., at p. 19.
consequence of the grave breaches under Article 147 of Geneva Convention IV. 8 A 16 OHNO, id., at p. 11.
state is precluded from absolving other states from liability on the ground that the 17 OHNO, id.; Yang Zhihui, supra.
individual persons who actually perpetrated the grave breach of the convention 616However, due to the opposition of the former Soviet Union, the proposals did
have already been punished.9 Indeed, it would seem unjust for individuals to be not materialize.18
punished while the state in whose name or on whose instructions they acted is American reparations policy shifted in 1947.19 As the Cold War developed, the
released from all liability.10 Article 148 was meant to prevent the defeated state U.S. initiated the drafting of a peace treaty with Japan. 20 On the issue of
from being forced into entering into an armistice or peace treaty in which it would reparations, the U.S. negotiated for a complete waiver of all claims arising from
renounce all reparations for grave breaches committed by persons in the service of Japan’s war acts.21 The alleged rationale for this “peace formula” rested on the
the victorious state.11 Thus, pursuant to the 1949 Geneva Conventions, a state U.S. assumption that if Japan were to be lured into the communist influence, the
remains responsible — and continues to be liable to pay compensation — for the strength of the Sino-Soviet camp would significantly increase, and the resulting
grave breaches committed against protected persons.12 change in the power balance in Asia would be “dangerously
Nevertheless, I am of the opinion that respondents cannot be found entirely formidable.”22 According to the U.S., Japan must be given a chance to recover full
guilty of a whimsical or capricious exercise of judgment, or a patent and gross economic self-sufficiency “by not placing upon her any heavy economic or financial
abuse of discretion. Their burdens or major commercial liabilities.”23 The peace treaty was to be “brief,
_______________ liberal, and nonpunitive.”24
7 Oscar M. Uhler & Henri Coursier, Commentary: Geneva Convention Relative to the Projection of Civilian Persons _______________
in Time of War IV, pp. 602-603 (Jean S. Pictet ed., 1958). 18 OHNO, id., at p. 13.
8 Id. 19 OHNO, id., at pp. 18-26; John F. Dulles, a Peace Treaty in the Making
9 Id. (Addresses and Remarks Regarding the Making of the Japanese Peace Treaty and
10 Id. the Cause of World freedom) pp. 3-7 (1951); Yang Zhihui, supra note 13 at pp. 375-
11 Id. 377.
12 Id. 20 OHNO, id., at p. 36.
615reliance on the Treaty of Peace with Japan (1951 Peace Treaty) and the 21 OHNO, id., at pp. 37-38 (citing United States Memorandum to the
Reparations Agreement Between the Republic of the Philippines and Japan (1956 Government on the Far Eastern Commission, in Royal Institute of International
Reparations Agreement) as bases for declining to espouse petitioners’ claims Affairs, Documents on International Affairs, 1947-1949, pp. 615-616 [1952]); Yang
against Japan was not without reason, especially if the treaties are analyzed in Zhihui,id., at p. 376.
the light of the events leading to their conclusion. 22 OHNO, id., at p. 38; See also Dulles, supra at pp. 40-42; Yang Zhihui, id.
The subject of reparations for damages suffered during the war was discussed 23 OHNO, id., at p. 37 (citing John Foster Dulles, “Peace May Be Won,” U.S.A.
during the occupation of Japan by the Allied Forces. At the time, the initial Department of State, DSB, Vol. 24 No. 605, at p. 255 [1951]); See Dulles, id., at pp.
reparations policy that had been put forward by the U.S. was to utilize reparations 19-21.
for rehabilitating the war-devastated countries, particularly countries in Asia and 24 OHNO, id. (citing U.S.A., Department of State, “An Estimate of Conditions
the Pacific.13The plan was also envisioned as a “vital integral means” for Japan’s in Asia and the Pacific at the Close of the War in the Far East and the Objectives
economic demilitarization.14 The U.S. supported an “early and just share of and Policies of the United States,” Diplomatic Papers, Vol. VI, pp. 556-580, 1945
reparations” in favor of the Philippines and pledged that “Japan will be reduced to [1969]); See Dulles, id.
the level of a small power and her people will not be permitted to have in the future 617 The Philippines rejected the U.S. proposal of total waiver of reparations
a living standard higher than those nations which she has overrun x x x.”15 It is claims against Japan.25 While the Philippine government had full appreciation of
said that an interim reparations removal policy was also adopted with the objective the international political reality of the spread of communism in Asia, 26 it
of transferring the industrial capacity of Japan to the Philippines.16 Under this remained firm that Japan should “sufficiently x x x repair the injuries they
policy, Japan’s industrial plants and facilities would be handed over to the inflicted in a war of aggression x x x.”27 The U.S. tried to persuade the Philippine
Philippines as war reparations in order to aid its rehabilitation and agricultural government. It pointed out that the problem of reparations was “not merely a
development needs.17 matter of justice,” but also a “matter of economics.”28 It argued that they could not
see “any effective way” of demanding reparations from an economically depressed 38 Id.
Japan.29 In a last effort to convince the Philippines to accept a no-reparations 39 Id., at pp. 54-55 (citing Acheson’s Speech, delivered on September 8, 1951,
peace arrangement, the U.S. emphasized the usually “intimate” relations between in U.S. Dep’t of State Publications, Record of Proceedings of the Conference for the
both countries.30 Conclusion and Signature of the Treaty of Peace with Japan, pp. 175-177 [1951]).
The U.S. was unsuccessful; the Philippines maintained an irrevocable stance 40 Id., at pp. 55-56.
on the matter of reparations.31 Former Undersecretary of Foreign Affairs Felino 619most aspect of the country’s peace settlement with Japan.41 They also
Neri criticized the American peace policy and remarked: “Reparations is first a demanded the inclusion in the peace treaty of a more categorical statement of
matter of justice and the realities of economics are, in our view, a secondary Japan’s guilt and reparations obligation.42The government defended its decision to
consideration. In our case, reparations from Japan is a matter of absolute sign the peace treaty on the basis of its “security first policy.” 43 It explained that
necessity.”32 The Philippines’ condemnation of the American peace formula security threats of the aggressive communist expansion impelled it to act swiftly
intensified when the U.S. government made available its draft of the in the ratification of the peace treaty.44 The opposition countered that the
_______________ Philippines was sufficiently safeguarded by its Mutual Defense Treaty with the
25 OHNO, id., at p. 40; Yang Zhihui, supra note 13 at p. 376. U.S.45
26 OHNO, id., at p. 39. The negotiations for reparations dragged on for almost five years after the
27 OHNO, id., at p. 40 (citing The President’s Inaugural Address, December signing of the 1951 Peace Treaty.46 From the initial demand of USD8 billion, the
30, 1949, Official Gazette, Vol. 45, No. 12, at p. 5384 [1949]). final reparations agreed upon amounted to a mere USD550 million,47 which was
28 OHNO, id., at p. 42 (citing Truman’s Envoy has long conference on Jap pact to be paid in the form of capital goods, cash, and services. 48 A note sent by
with EQ, the Manila Times, February 12, 1951, pp. 1-2). President Magsaysay to the Senate reads as follows:
29 OHNO, id. Considering the losses and suffering the Philippines sustained as a result of
30 Id. the Pacific War, these terms do not come up to the generally-accepted
31 Id., at pp. 42-43; See also Dulles, supra note 19 at p. 48; Yang concept of reparations as compensation for damage done and injury
Zhihui, supra at p. 376. suffered.
32 OHNO, id., at p. 43 (citing Neri assails Dulles stand, the Manila Times, 3 Judged, however, from the point of view of the requirements of our national
March 1951, pp. 1, 12). interest and viewed in the light of the practical realities posed by the political and
618treaty, which provided for the absolute abandonment of reparations claims on _______________
the ground that Japan lacked the “capacity to make payments” in any form.33 The 41 Id., at pp. 58, 80.
Philippines refused such claims.34 It repeatedly declared that Japan was solvent, 42 Id., at p. 58.
and that the Philippines would never withdraw its claims.35 43 Id., at pp. 75-80.
At the height of the developing Cold War, the U.S. and the U.K. initiated a 44 Id.
conference on the Japanese peace treaty.36 During the discussions, the Philippines, 45 Id., at p. 80.
through then Foreign Affairs Secretary Carlos P. Romulo, expressed that it had 46 Id., at pp. 64-134.
yet to be satisfied with the reparations provisions of the proposed 47 Id., at p. 121.
treaty.37 Secretary Romulo voiced out a reservation on an “inflexible restriction” 48 President’s Letter of Transmittal, reproduced in Philippine Senate,
on the form of reparations payment, asserting that the Philippines could not accept Reparations Agreement, the Annex Thereto, the Exchange Notes and the Other
that reparations be made only through the “services” of the Japanese people in the Supporting Documents: Treaty of Peace with Japan, Understanding of the Senate
processing of raw materials that would be supplied by the injured on Certain Provisions of the Reparations Agreement, p. 3 (1956).
countries.38 However, the reservation was neither accepted nor recorded.39 Faced 620economic situation obtaining in both countries as well as in their part of the
with the dilemma of supporting its allies in winning over Japan at the expense of world, I subscribe to the conclusion reached by the Philippine Panel of Negotiators
fully satisfying security and reparations claims, the Philippines reluctantly signed that this settlement is the best that can be obtained under the
the 1951 Peace Treaty.40 circumstances x x x.49
The decision of the Philippine government to eventually sign the peace treaty After a heated debate, the 1951 Peace Treaty and the 1956 Reparations
was met with strong resistance. The opposition insisted that the reparations issue Agreement were ratified on 16 July 1956. The pertinent provision of the 1951
was the fore- Peace Treaty is reproduced below:
_______________ ARTICLE 14
33 Id.; See also Dulles, supra note 19 at p. 48. (a) It is recognized that Japan should pay reparations to the Allied
34 OHNO, id. Powers for the damage and suffering caused by it during the war.
35 Id. Nevertheless it is also recognized that the resources of Japan are not
36 Id., at p. 52. presently sufficient if it is to maintain a viable economy, to make
37 Id., at p. 54.
complete reparations for all such damage and suffering and at the same time products referred to in Article 5, paragraph 4 of the present Agreement. These
meet its other obligations. payments shall be made in Japanese yen.
Therefore, 2. By and upon making a payment in yen under the preceding
1. Japan will promptly enter into negotiations with Allied Powers so paragraph, Japan shall be deemed to have supplied the Republic of the
desiring, whose present territories were occupied by Japanese forces and Philippines with the services and products thus paid for and shall be
damaged by Japan, with a view to assisting to compensate those released from its reparations obligation to the extent of the equivalent value
countries for the cost of repairing the damage done, by making in United States dollars of such yen payment in accordance with Articles 1 and 2
available the services of the Japanese people in production, salvaging of the present Agreement. (Emphases supplied)
and other work for the Allied Powers in question. Such arrangements In the light of the foregoing context, I vote to dismiss the petition for failure
shall avoid the imposition of additional liabilities on other Allied Powers, and, to establish that respondents committed grave abuse of discretion in declining to
where the manufacturing of raw materials is called for, they shall be supplied espouse the claims of petitioners. The dismissal thereof should not, however, be
by the taken as a definitive ruling on the merits of the claims of petitioners, in the event
_______________ that they bring the same to an appropriate forum or through a proper recourse.
49 Id., at p. 4. Neither should it be taken to mean that we should forget the suffering that our
621Allied Powers in question, so as not to throw any foreign exchange burden people, especially petitioners, bore in the Second World War, or the unfortunate
upon Japan. story of our attempts to get the reparation that was due us, and learn. From such
2. x x x x understanding, we must forge the elements that will make the Philippine state
(b) Except as otherwise provided in the present Treaty, the Allied Powers strong, able to protect its people and safeguard their well-being under the aegis of
waive all reparation claims of the Allied Powers, other claims of the Allied the Constitution. Justice demands no less.
Powers and their nationals arising out of any actions taken by Japan and 623
nationals in the course of the prosecution of the war, and claims of the Motion for Reconsideration and Supplemental Motion for Reconsideration
Allied Powers for direct military costs of occupation. (Emphases supplied) denied.
On the other hand, the relevant provisions of the 1956 Reparations Notes.—It is worth stressing that the assessment and evaluation of evidence
Agreement are quoted as follows: in the issuance of the writ of preliminary injunction involves findings of facts
ARTICLE 1 ordinarily left to the trial court for its conclusive determination. (Dela Rosa vs.
Japan, by way of reparations, shall supply the Republic of Heirs of Juan Valdez, 654 SCRA 467 [2011])
the Philippines with the services of the Japanese people and the products A preliminary mandatory injunction is more cautiously regarded than a mere
of Japan in the form of capital goods, the total value of which will be so much prohibitive injunction since, more than its function of preserving the status
in yen as shall be equivalent to five hundred fifty million United States dollars quo between the parties, it also commands the performance of an act. (Id.)
($550,000,000) at present computed at one hundred ninety-eight billion yen ——o0o——
(Y198,000,000,000), within the period and in the manner hereinafter prescribed.
ARTICLE 2
The supply of the services and products referred to in the preceding Article
shall be made on an annual average of so much in yen as shall be equivalent to
twenty-five million United States dollars ($25,000,000) at present computed at
nine billion yen (Y9,000,000,000), during the ten-year period from the date of
coming into force of the present Agreement; and on an annual average of so much
in yen as shall be equivalent to thirty million United States dollars ($30,000,000)
at present computed at ten billion eight hundred million yen (Y10,800,000,000),
G.R. No. 212426. January 12, 2016.*
during the succeeding ten-year period. However, by
622agreement between the two Governments, this latter period may be reduced
RENE A.V. SAGUISAG, WIGBERTO E. TAÑADA, FRANCISCO “DODONG”
to a period shorter than ten years, provided the outstanding balance is settled in
NEMENZO, JR., SR. MARY JOHN MANANZAN, PACIFICO A. AGABIN,
full within the remainder of the reduced period.
ESTEBAN “STEVE” SALONGA, H. HARRY L. ROQUE, JR., EVALYN G.
ARTICLE 6
URSUA, EDRE U. OLALIA, DR. CAROL PAGADUAN-ARAULLO, DR. ROLAND
1. In the discharge of the reparations obligation under Article 1 of the present
SIMBULAN, and TEDDY CASIÑO, petitioners, vs. EXECUTIVE SECRETARY
Agreement, the Government of Japan shall, through procedures to be determined
PAQUITO N. OCHOA, JR., DEPARTMENT OF NATIONAL DEFENSE
under Article 11, make payments to cover the obligations incurred by the Mission
SECRETARY VOLTAIRE GAZMIN, DEPARTMENT OF FOREIGN AFFAIRS
under Reparations Contracts and the expenses for the supply of services and
SECRETARY ALBERT DEL ROSARIO, JR., DEPARTMENT OF BUDGET AND
MANAGEMENT SECRETARY FLORENCIO ABAD, and ARMED FORCES OF
THE PHILIPPINES CHIEF OF STAFF GENERAL EMMANUEL T. BAUTISTA, government has acted beyond the scope of the latter’s constitutional powers.—
respondents. Distinguished from the general notion of judicial power, the power of judicial
review specially refers to both the authority and the duty of this Court to determine
G.R. No. 212444. January 12, 2016.* whether a branch or an instrumentality of government has acted beyond the scope
of the latter’s constitutional powers. As articulated in Section 1, Article VIII of the
BAGONG ALYANSANG MAKABAYAN (BAYAN), represented by its Constitution, the power of judicial review involves the power to resolve cases in
SECRETARY GENERAL RENATO M. REYES, JR., BAYAN MUNA PARTY-LIST which the questions concern the constitutionality or validity of any treaty,
REPRESENTATIVES NERI J. COLMENARES and CARLOS ZARATE,
GABRIELA WOMEN’S PARTY-LIST REPRESENTATIVES LUZ ILAGAN and
EMERENCIANA DE JESUS, ACT TEACHERS PARTY-LIST 243
REPRESENTATIVE ANTONIO L. TINIO, ANAKPAWIS PARTY-LIST VOL. 779, JANUARY 12, 2016 243
REPRESENTATIVE FERNANDO HICAP, KABATAAN PARTY-LIST
REPRESENTATIVE TERRY RIDON, MAKABAYANG KOALISYON NG Saguisag vs. Ochoa, Jr.
MAMAMAYAN (MAKABAYAN), represented by SATURNINO OCAMPO and
LIZA MAZA, BIENVENIDO LUMBERA, JOEL C. LAMANGAN, RAFAEL international or executive agreement, law, presidential decree,
MARIANO, SALVADOR FRANCE, ROGELIO M. SOLUTA, and CLEMENTE G. proclamation, order, instruction, ordinance, or regulation. In Angara v. Electoral
BAUTISTA, petitioners, vs. DEPARTMENT OF NATIONAL DEFENSE Commission, 63 Phil. 139 (1936), this Court exhaustively discussed this
_______________ “moderating power” as part of the system of checks and balances under the
Constitution. In our fundamental law, the role of the Court is to determine whether
* EN BANC. a branch of government has adhered to the specific restrictions and limitations of
the latter’s power.
Same; Demetria v. Alba, 148 SCRA 208 (1987) and Francisco, Jr. v.
242 Nagmamalasakit na mga Manananggol ng mga Manggagawang Pilipino, Inc., 415
SCRA 44 (2003), cite the “pillars” of the limitations on the power of judicial review
242 SUPREME COURT REPORTS ANNOTATED as enunciated in the concurring opinion of United States (U.S.) Supreme Court
Saguisag vs. Ochoa, Jr. Justice Brandeis in Ashwander v. Tennessee Valley Authority, 297 U.S. 288, 346-
348 (1936).—Even as we are left with no recourse but to bare our power to check
an act of a coequal branch of government — in this case the executive — we must
(DND) SECRETARY VOLTAIRE GAZMIN, DEPARTMENT OF FOREIGN abide by the stringent requirements for the exercise of that power under the
AFFAIRS SECRETARY ALBERT DEL ROSARIO, EXECUTIVE SECRETARY Constitution. Demetria v. Alba, 148 SCRA 208 (1987) and Francisco, Jr. v.
PAQUITO N. OCHOA, JR., ARMED FORCES OF THE PHILIPPINES CHIEF OF Nagmamalasakit na mga Manananggol ng mga Manggagawang Pilipino, Inc., 415
STAFF GENERAL EMMANUEL T. BAUTISTA, DEFENSE SCRA 44 (2003), cite the “pillars” of the limitations on the power of judicial review
UNDERSECRETARY PIO LORENZO BATINO, AMBASSADOR LOURDES as enunciated in the concurring opinion of U.S. Supreme Court Justice Brandeis
YPARRAGUIRRE, AMBASSADOR J. EDUARDO MALAYA, DEPARTMENT OF in Ashwander v. Tennessee Valley Authority, 297 U.S. 288
JUSTICE UNDERSECRETARY FRANCISCO BARAAN III, and DND (1936). Francisco redressed these “pillars” under the following categories: 1. That
ASSISTANT SECRETARY FOR STRATEGIC ASSESSMENTS RAYMUND JOSE there be absolute necessity of deciding a case; 2. That rules of constitutional
QUILOP AS CHAIRPERSON AND MEMBERS, respectively, of the law shall be formulated only as required by the facts of the case; 3. That
NEGOTIATING PANEL FOR THE PHILIPPINES ON EDCA, respondents. judgment may not be sustained on some other ground; 4. That there
KILUSANG MAYO UNO, represented by its CHAIRPERSON, ELMER LABOG, be actual injury sustained by the party by reason of the operation of the
CONFEDERATION FOR UNITY, RECOGNITION AND ADVANCEMENT OF statute; 5. That the parties are not in estoppel; 6. That the Court upholds
GOVERNMENT EMPLOYEES (COURAGE), represented by its NATIONAL the presumption of constitutionality. (Emphases supplied) These are the
PRESIDENT FERDINAND GAITE, NATIONAL FEDERATION OF LABOR specific safeguards laid down by the Court when it exercises its power of judicial
UNIONS-KILUSANG MAYO UNO, represented by its NATIONAL PRESIDENT review. Guided by these pillars, it may invoke the power only when the following
JOSELITO USTAREZ, NENITA GONZAGA, VIOLETA ESPIRITU, VIRGINIA four stringent requirements are satisfied: (a) there is an actual case or controversy;
FLORES, and ARMANDO TEODORO, JR., petitioners-in-intervention. (b) petitioners possess locus standi; (c) the question of constitutionality is raised at
RENE A.Q. SAGUISAG, JR., petitioner-in-intervention. the earliest opportunity; and (d) the issue of constitutionality is the lis mota of the
Judicial Review; Distinguished from the general notion of judicial power, the case. Of these four, the first two conditions will be the focus of our discussion.
power of judicial review specially refers to both the authority and the duty of the
Supreme Court (SC) to determine whether a branch or an instrumentality of
244
244 SUPREME COURT REPORTS ANNOTATED Same; Same; Taxpayers’ Suit; The Supreme Court (SC) emphasizes that a
taxpayers’ suit contemplates a situation in which there is already an appropriation
Saguisag vs. Ochoa, Jr. or a disbursement of public funds. A reading of Article X(1) of Enhanced Defense
Cooperation Agreement (EDCA) would show that there has been neither an
Same; Actual Case or Controversy; The performance of an official act by the appropriation nor an authorization of disbursement of funds.—A taxpayer’s suit
Executive Department that led to the entry into force of an executive agreement was concerns a case in which the official act complained of directly involves the illegal
sufficient to satisfy the actual case or controversy requirement.—We find that the disbursement of public funds derived from taxation. Here, those challenging the
matter before us involves an actual case or controversy that is already ripe for act must specifically show that they have sufficient interest in preventing the
adjudication. The Executive Department has already sent an official confirmation illegal expenditure of public money, and that they will sustain a direct injury as a
to the U.S. Embassy that “all internal requirements of the Philippines x x x have result of the enforcement of the assailed act. Applying that principle to this case,
already been complied with.” By this exchange of diplomatic notes, the Executive they must establish that EDCA involves the exercise by Congress of its taxing or
Department effectively performed the last act required under Article XII(1) of spending powers. We agree with the OSG that the petitions cannot qualify as
EDCA before the agreement entered into force. Section 25, Article XVIII of the taxpayers’ suits. We emphasize that a taxpayers’ suit contemplates a situation in
Constitution, is clear that the presence of foreign military forces in the country which there is already an appropriation or a disbursement of public funds. A
shall only be allowed by virtue of a treaty concurred in by the Senate. Hence, the reading of Article X(1) of EDCA would show that there has been neither an
performance of an official act by the Executive Department that led to the entry appropriation nor an authorization of disbursement of funds.
into force of an executive agreement was sufficient to satisfy the actual case or Same; Same; Legislators’ Suit; In a legislators’ suit, those Members of
controversy requirement. Congress who are challenging the official act have standing only to the extent that
Same; Locus Standi; The question of locus standi or legal standing focuses on the alleged violation impinges on their right to participate in the exercise of the
the determination of whether those assailing the governmental act have the right of powers of the institution of which they are members.—We emphasize that in a
appearance to bring the matter to the court for adjudication. They must show that legislators’ suit, those Members of Congress who are challenging the official act
they have a personal and substantial interest in the case, such that they have have standing only to the extent that the alleged violation impinges on their right
sustained or are in immediate danger of sustaining, some direct injury as a to participate in the exercise of the powers of the institution of which they are
consequence of the enforcement of the challenged governmental act.—The question members. Legislators have the standing “to maintain inviolate the prerogatives,
of locus standi or legal standing focuses on the determination of whether those powers, and privileges vested by the Constitution in their office and are allowed to
assailing the governmental act have the right of appearance to bring the matter to sue to question the validity of any official action, which they claim infringes their
the court for adjudication. They must show that they have a personal and prerogatives as legislators.” As legislators, they must clearly show that there was
substantial interest in the case, such that they have sustained or are in immediate a direct injury to their persons or the institution to which they belong.
danger of sustaining, some direct injury as a consequence of the enforcement of the
challenged governmental act. Here, “interest” in the question involved must be
material — an interest that is in issue and will be affected by the official act — as 246
distinguished from being merely incidental or general. Clearly, it would be
246 SUPREME COURT REPORTS ANNOTATED
insufficient to show that the law or any governmental act is invalid, and that
petitioners stand to suffer in some indefinite way. They must show that they have Saguisag vs. Ochoa, Jr.
a particular interest in bringing the suit, and that they have been or are about to
be denied some right or privilege to which they are lawfully entitled, or that they Constitutional Law; Treaties; Power to Concur in a Treaty; The power to
are about to be subjected to some burden or penalty by reason of the act complained concur in a treaty or an international agreement is an institutional prerogative
of. The reason why those who challenge the validity of a law or an international granted by the Constitution to the Senate, not to the entire Legislature.—As
correctly argued by respondent, the power to concur in a treaty or an international
agreement is an institutional prerogative granted by the Constitution to the
245 Senate, not to the entire Legislature. In Pimentel, Jr. v. Office of the Executive
VOL. 779, JANUARY 12, 2016 245 Secretary, 462 SCRA 622 (2005), this Court did not recognize the standing of one
of the petitioners therein who was a member of the House of Representatives. The
Saguisag vs. Ochoa, Jr. petition in that case sought to compel the transmission to the Senate for
concurrence of the signed text of the Statute of the International Criminal Court.
agreement are required to allege the existence of a personal stake in the Since that petition invoked the power of the Senate to grant or withhold its
outcome of the controversy is “to assure the concrete adverseness which sharpens concurrence in a treaty entered into by the Executive Department, only then
the presentation of issues upon which the court so largely depends for illumination incumbent Senator Pimentel was allowed to assert that authority of the Senate of
of difficult constitutional questions.” which he was a member.
Same; Judicial Review; When those who challenge the official act are able to “gain entry” Philippine territory. Note that the provision “shall not be allowed” is
craft an issue of transcendental significance to the people, the Supreme Court (SC) a negative injunction. This wording signifies that the President is not authorized
may exercise its sound discretion and take cognizance of the suit.—In a number of by law to allow foreign military bases, troops, or facilities to enter the Philippines,
cases, this Court has indeed taken a liberal stance towards the requirement of except under a treaty concurred in by the Senate. Hence, the constitutionally
legal standing, especially when paramount interest is involved. Indeed, when restricted authority pertains to the entry of the bases, troops, or facilities, and not
those who challenge the official act are able to craft an issue of transcendental to the activities to be done after entry.
significance to the people, the Court may exercise its sound discretion and take Verba Legis; Under the principles of constitutional construction, of
cognizance of the suit. It may do so in spite of the inability of the petitioners to paramount consideration is the plain meaning of the language expressed in the
show that they have been personally injured by the operation of a law or any other Constitution, or the verba legis rule.—Under the principles of constitutional
government act. construction, of paramount consideration is the plain meaning of the language
Executive Power; The duty to faithfully execute the laws of the land is inherent expressed in the Constitu-
in executive power and is intimately related to the other executive functions.—The
duty to faithfully execute the laws of the land is inherent in executive power and
is intimately related to the other executive functions. These functions include the 248
faithful execution of the law in autonomous regions; the right to prosecute crimes; 248 SUPREME COURT REPORTS ANNOTATED
the implementation of transportation projects; the duty to ensure compliance with
treaties, executive agreements and executive orders; the authority to deport Saguisag vs. Ochoa, Jr.
undesirable aliens; the conferment of national awards under the President’s
jurisdiction; and the overall administration and control of the executive tion, or the verba legis rule. It is presumed that the provisions have been
department. carefully crafted in order to express the objective it seeks to attain. It is incumbent
upon the Court to refrain from going beyond the plain meaning of the words used
in the Constitution. It is presumed that the framers and the people meant what
247 they said when they said it, and that this understanding was reflected in the
Constitution and understood by the people in the way it was meant to be
VOL. 779, JANUARY 12, 2016 247
understood when the fundamental law was ordained and promulgated.
Saguisag vs. Ochoa, Jr. Foreign Military Bases; It is evident that the constitutional restriction refers
solely to the initial entry of the foreign military bases, troops, or facilities. Once
Same; Presidency; Foreign Military Bases; Despite the President’s roles as entry is authorized, the subsequent acts are thereafter subject only to the limitations
defender of the State and sole authority in foreign relations, the 1987 Constitution provided by the rest of the Constitution and Philippine law, and not to the Section
expressly limits his ability in instances when it involves the entry of foreign military 25 requirement of validity through a treaty.—It is evident that the constitutional
bases, troops or facilities.—Despite the President’s roles as defender of the State restriction refers solely to the initial entry of the foreign military bases, troops, or
and sole authority in foreign relations, the 1987 Constitution expressly limits his facilities. Once entry is authorized, the subsequent acts are thereafter subject only
ability in instances when it involves the entry of foreign military bases, troops or to the limitations provided by the rest of the Constitution and Philippine law, and
facilities. The initial limitation is found in Section 21 of the provisions on the not to the Section 25 requirement of validity through a treaty.
Executive Department: “No treaty or international agreement shall be valid and Presidency; The President has the inherent power to enter into agreements
effective unless concurred in by at least two-thirds of all the Members of the with other states, including the prerogative to conclude binding executive
Senate.” The specific limitation is given by Section 25 of the Transitory Provisions, agreements that do not require further Senate concurrence.—As the sole organ of
the full text of which reads as follows: SECTION 25. After the expiration in 1991 our foreign relations and the constitutionally assigned chief architect of our foreign
of the Agreement between the Republic of the Philippines and the United States policy, the President is vested with the exclusive power to conduct and manage the
of America concerning Military Bases, foreign military bases, troops, or facilities country’s interface with other states and governments. Being the principal
shall not be allowed in the Philippines except under a treaty duly concurred in by representative of the Philippines, the Chief Executive speaks and listens for the
the Senate and, when the Congress so requires, ratified by a majority of the votes nation; initiates, maintains, and develops diplomatic relations with other states
cast by the people in a national referendum held for that purpose, and recognized and governments; negotiates and enters into international agreements; promotes
as a treaty by the other contracting State. trade, investments, tourism and other economic relations; and settles
Same; Same; Same; The President is not authorized by law to allow foreign international disputes with other states. As previously discussed, this
military bases, troops, or facilities to enter the Philippines, except under a treaty constitutional mandate emanates from the inherent power of the President to
concurred in by the Senate.—To this Court, a plain textual reading of Article XIII, enter into agreements with other states, including the prerogative to
Section 25, inevitably leads to the conclusion that it applies only to a proposed conclude binding executive agreements that do not require further Senate
agreement between our government and a foreign government, whereby military concurrence. The existence of this presidential power is so well-entrenched that
bases, troops, or facilities of such foreign government would be “allowed” or would
Section 5(2)(a), Article VIII of the Constitution, even provides for a check on its
exercise. As expressed firmed by long usage. From the earliest days of our history we have
entered into executive agreements covering such subjects as commercial and
consular relations, most-favored-nation rights, patent rights, trademark and
249 copyright protection, postal and navigation arrangements and the settlement of
VOL. 779, JANUARY 12, 2016 249 claims. The validity of these has never been seriously questioned by our
courts. (Emphases supplied) That notion was carried over to the present
Saguisag vs. Ochoa, Jr. Constitution. In fact, the framers specifically deliberated on whether the general
term “international agreement” included executive agreements, and whether it
below, executive agreements are among those official governmental acts that was necessary to include an express proviso that would exclude executive
can be the subject of this Court’s power of judicial review: (2) Review, revise, agreements from the requirement of Senate concurrence. After noted
reverse, modify, or affirm on appeal or certiorari, as the law or the Rules of constitutionalist Fr. Joaquin Bernas quoted the Court’s ruling in Eastern Sea
Court may provide, final judgments and orders of lower courts in: (a) Trading, the Constitutional Commission members ultimately decided that the
All cases in which the constitutionality or term “international agreements” as contemplated in Section 21, Article VII, does
validity of any treaty, international or executive agreement, law, not include executive agreements, and that a proviso is no longer needed.
presidential decree, proclamation, order, instruction, ordinance, or regulation is in Same; Same; International practice has accepted the use of various forms and
question. designations of international agreements, ranging from the traditional notion of a
Executive Agreements; Words and Phrases; In Commissioner of Customs v. treaty — which connotes a formal, solemn instrument — to engagements concluded
Eastern Sea Trading, 3 SCRA 351 (1961), executive agreements are defined as in modern, simplified forms that no longer necessitate ratification.—The special
“international agreements embodying adjustments of detail carrying out well- nature of an executive agreement is not just a domestic variation in international
established national policies and traditions and those involving arrangements of a agreements. International practice has accepted the use of various forms and
more or less temporary nature.”—In Commissioner of Customs v. Eastern Sea designations of international agreements, ranging from the traditional notion of a
Trading, 3 SCRA 351 (1961), executive agreements are defined as “international treaty — which connotes a formal, solemn instrument — to engagements
agreements embodying adjustments of detail carrying out well-established concluded in modern, simplified forms that no longer necessitate ratification. An
national policies and traditions and those involving arrangements of a more or less international agreement may take different forms: treaty, act, protocol,
temporary nature.” In Bayan Muna v. Romulo, 641 SCRA 244 (2011), this Court agreement, concordat, compromis d’arbitrage, convention, covenant, declaration,
further clarified that executive agreements can cover a wide array of subjects that exchange of notes, statute, pact, charter, agreed minute, memorandum of
have various scopes and purposes. They are no longer limited to the traditional agreement, modus vivendi, or some other form. Consequently, under international
subjects that are usually covered by executive agreements as identified in Eastern law, the distinction between a treaty and an international agreement or even an
Sea Trading. executive agreement is irrelevant for purposes of determining international rights
Same; International Agreements; After noted constitutionalist Fr. Joaquin and obligations.
Bernas quoted the Supreme Court’s (SC’s) ruling in Commissioner of Customs Same; Section 9 of Executive Order (EO) No. 459, or the Guidelines in the
v. Eastern Sea Trading, 3 SCRA 351 (1961), the Constitutional Commission Negotiation of International Agreements and its Ratification, thus, correctly
members ultimately decided that the term “international agreements” as reflected the inherent powers of the President
contemplated in Section 21, Article VII, does not include executive agreements, and
that a proviso is no longer needed.—One of the distinguishing features of executive
agreements is that their validity and effectivity are not affected by a lack of Senate 251
concurrence. This distinctive feature was recognized as early as in Eastern Sea
VOL. 779, JANUARY 12, 2016 251
Trading (1961), viz.: Treaties are formal documents which require
ratification with the approval of two-thirds of the Senate. Executive Saguisag vs. Ochoa, Jr.
agreements become binding through executive action without the need of
a vote by the Senate or by Congress. x x x x [T]he right of the when it stated that the Department of Foreign Affairs (DFA) “shall determine
Executive to enter into binding agreements without the necessity of whether an agreement is an executive agreement or a treaty.”—Indeed, in the field
subsequent Congressional approval has been con- of external affairs, the President must be given a larger measure of authority and
wider discretion, subject only to the least amount of checks and restrictions under
the Constitution. The rationale behind this power and discretion was recognized
250 by the Court in Vinuya v. Romulo, 619 SCRA 533 (2010), cited earlier. Section 9 of
250 SUPREME COURT REPORTS ANNOTATED Executive Order No. 459, or the Guidelines in the Negotiation of International
Agreements and its Ratification, thus, correctly reflected the inherent powers of
Saguisag vs. Ochoa, Jr. the President when it stated that the DFA “shall determine whether an agreement
is an executive agreement or a treaty.” Accordingly, in the exercise of its power of by national security, public safety, public health, public morals, and national
judicial review, the Court does not look into whether an international agreement interest; In contrast, Article 111(5) of the Visiting Forces Agreement (VFA) requires
should be in the form of a treaty or an executive agreement, save in cases in which a request for removal from the Philippine government before a member of the U.S.
the Constitution or a statute requires otherwise. Rather, in view of the vast personnel may be “dispos[ed] x x x outside of the Philippines.”—In the same vein,
constitutional powers and prerogatives granted to the President in the field of the President may exercise the plenary power to expel or deport U.S. contractors
foreign affairs, the task of the Court is to determine whether the international as may be necessitated by national security, public safety, public health, public
agreement is consistent with the applicable limitations. morals, and national interest. They may also be deported if they are found to be
Foreign Military Bases; Enhanced Defense Cooperation Agreement; Visiting illegal or undesirable aliens pursuant to the Philippine Immigration Act and the
Forces Agreement; The admission and presence of United States (U.S.) military and Data Privacy Act. In contrast, Article 111(5) of the VFA requires a request for
civilian personnel in Philippine territory are already allowed under the Visiting removal from the Philippine government before a member of the U.S. personnel
Forces Agreement (VFA), the treaty supposedly being implemented by Enhanced may be “dispos[ed] x x x outside of the Philippines.”
Defense Cooperation Agreement (EDCA). What EDCA has effectively done, in fact, Same; Same; Same; Certain privileges denied to aliens are likewise denied to
is merely provide the mechanism to identify the locations in which U.S. personnel foreign military contractors.—EDCA requires that all activities within Philippine
may perform allowed activities pursuant to the VFA.—The admission and territory be in accordance with Phil-
presence of U.S. military and civilian personnel in Philippine territory
are already allowed under the VFA, the treaty supposedly being
implemented by EDCA. What EDCA has effectively done, in fact, is merely 253
provide the mechanism to identify the locations in which U.S. personnel may VOL. 779, JANUARY 12, 2016 253
perform allowed activities pursuant to the VFA. As the implementing agreement,
it regulates and limits the presence of U.S. personnel in the country. Saguisag vs. Ochoa, Jr.
Same; Same; Nowhere in Enhanced Defense Cooperation
Agreement(EDCA) are United States (U.S.) contractors guaranteed immediate ippine law. This means that certain privileges denied to aliens are likewise
admission into the Philippines; It is neither mandatory denied to foreign military contractors. Relevantly, providing security and carrying,
owning, and possessing firearms are illegal for foreign civilians. The laws in place
already address issues regarding the regulation of contractors. In the 2015 Foreign
252 Investment Negative list, the Executive Department has already identified
corporations that have equity restrictions in Philippine jurisdiction. Of note is No.
252 SUPREME COURT REPORTS ANNOTATED
5 on the list — private security agencies that cannot have any foreign equity by
Saguisag vs. Ochoa, Jr. virtue of Section 4 of Republic Act No. 5487; and No. 15, which regulates contracts
for the construction of defense-related structures based on Commonwealth Act No.
nor obligatory on the part of the Philippines to admit United States (U.S.) 541.
contractors into the country.—Nowhere in EDCA are U.S. contractors guaranteed Same; Same; Same; United States (U.S.) contractors are explicitly excluded
immediate admission into the Philippines. Articles III and IV, in fact, merely grant from the coverage of the Visiting Forces Agreement (VFA). As visiting aliens, their
them the right of access to, and the authority to conduct certain activities within entry, presence, and activities are subject to all laws and treaties applicable within
the Agreed Locations. Since Article II(3) of EDCA specifically leaves out U.S. the Philippine territory.—We emphasize that U.S. contractors are explicitly
contractors from the coverage of the VFA, they shall not be granted the same entry excluded from the coverage of the VFA. As visiting aliens, their entry, presence,
accommodations and privileges as those enjoyed by U.S. military and civilian and activities are subject to all laws and treaties applicable within the Philippine
personnel under the VFA. Consequently, it is neither mandatory nor obligatory on territory. They may be refused entry or expelled from the country if they engage
the part of the Philippines to admit U.S. contractors into the country. We in illegal or undesirable activities. There is nothing that prevents them from being
emphasize that the admission of aliens into Philippine territory is “a matter of detained in the country or being subject to the jurisdiction of our courts. Our penal
pure permission and simple tolerance which creates no obligation on the part of laws, labor laws, and immigrations laws apply to them and therefore limit their
the government to permit them to stay.” Unlike U.S. personnel who are accorded activities here. Until and unless there is another law or treaty that specifically
entry accommodations, U.S. contractors are subject to Philippine immigration deals with their entry and activities, their presence in the country is subject to
laws. The latter must comply with our visa and passport regulations and prove unqualified Philippine jurisdiction.
that they are not subject to exclusion under any provision of Philippine Same; Same; Agreed Locations; The latest agreement is Enhanced Defense
immigration laws. The President may also deny them entry pursuant to his Cooperation Agreement (EDCA), which proposes a novel concept termed “Agreed
absolute and unqualified power to prohibit or prevent the admission of aliens Locations.”—The latest agreement is EDCA, which proposes a novel concept
whose presence in the country would be inimical to public interest. termed “Agreed Locations.” By definition, Agreed Locations are facilities and areas
Same; Same; Visiting Forces Agreement; Presidency; The President may that are provided by the Government of the Philippines through the AFP and that
exercise the plenary power to expel or deport U.S. contractors as may be necessitated United States forces, United States contractors, and others as mutually agreed,
shall have the right to access and use pursuant to this Agreement. Such Agreed Same; Same; Same; For actual operations, Enhanced Defense Cooperation
Locations may be listed in an annex to be appended to this Agreement, and may Agreement (EDCA) is clear that any activity must be planned and preapproved by
be further described in implementing arrangements. Preliminarily, respondent the Mutual Defense Board-Security Engagement Board (MDB-SEB).—The legal
already claims that the proviso that the Philippines shall retain ownership of concept of operational control involves authority over personnel in a commander-
subordinate relationship and does not include control over the Agreed Locations in
this particular case. Though not necessarily stated in EDCA provisions, this
254 interpretation is readily implied by the reference to the taking of “appropriate
254 SUPREME COURT REPORTS ANNOTATED measures to protect United States forces and United States contractors.” It is but
logical, even necessary, for the U.S. to have operational control over its own forces,
Saguisag vs. Ochoa, Jr. in much the same way that the Philippines exercises operational control over its
own units. For actual operations, EDCA is clear that any activity must be planned
and title to the Agreed Locations means that EDCA is “consistent with and preapproved by the MDB-SEB. This provision evinces the partnership aspect
Article II of the VFA which recognizes Philippine sovereignty and jurisdiction over of EDCA, such that both stakeholders have a say on how its provisions should be
locations within Philippine territory.” put into effect.
Same; Same; Same; Enhanced Defense Cooperation Same; Same; Same; The Philippines retains primary responsibility for
Agreement(EDCA) explicitly provides that ownership of the Agreed Locations security with respect to the Agreed Locations.—From the text of EDCA itself,
remains with the Philippine government. What United States (U.S.) personnel have Agreed Locations are territories of the Philippines that the U.S. forces are allowed
a right to, pending mutual agreement, is access to and use of these locations.—Once to access and use. By withholding ownership of these areas and retaining
ownership is established, then the rights of ownership flow freely. Article 428 of unrestricted access to them, the government asserts sovereignty over its territory.
the Civil Code provides that “[t]he owner has the right to enjoy and dispose of a That sovereignty exists so long as the Filipino people exist. Significantly, the
thing, without other limitations than those established by law.” Moreover, the Philippines retains primary responsibility for security with respect to the Agreed
owner “has also a right of action against the holder and possessor of the thing in Locations. Hence, Philippine law remains in force therein, and it cannot be said
order to recover it.” Philippine civil law therefore accords very strong rights to the that jurisdiction has been transferred to the U.S. Even the previously discussed
owner of property, even against those who hold the property. Possession, after all, necessary measures for operational control and defense over U.S. forces must be
merely raises a disputable presumption of ownership, which can be contested coordinated with Philippine authorities. Jurisprudence bears out the fact that even
through normal judicial processes. In this case, EDCA explicitly provides that under the former legal regime of the MBA, Philippine laws continue to be in force
ownership of the Agreed Locations remains with the Philippine government. What within the bases. The difference between then and now is that EDCA retains the
U.S. personnel have a right to, pending mutual agreement, is access to and use of primary jurisdiction of the Philippines over the security of the Agreed Locations,
these locations. an important provision that gives it actual control over those locations. Previously,
Same; Same; Same; So long as the right of ownership itself is not transferred, it was the provost marshal of the U.S. who kept the peace and enforced Philippine
then whatever rights are transmitted by agreement does not completely divest the law in the bases. In this instance, Philippine forces act as peace officers, in stark
owner of the rights over the property, but may only limit them in accordance with contrast to the 1947 MBA provisions on jurisdiction.
law.—The right of the owner of the property to allow access and use is consistent
with the Civil Code, since the owner may dispose of the property in whatever way
deemed fit, subject to the limits of the law. So long as the right of ownership itself 256
is not transferred, then whatever rights are transmitted by agreement does not
256 SUPREME COURT REPORTS ANNOTATED
completely divest the owner of the rights over the property, but may only limit
them in accordance with law. Hence, even control over the property is something Saguisag vs. Ochoa, Jr.
that an owner may transmit freely. This act does not translate into the full transfer
of ownership, but only of certain rights. In Roman Catholic Apostolic Same; Same; Same; Even if the lawfulness of the attack were not in question,
Administrator of Davao, Inc. v. Land Registration Commission, 102 Phil. 596 international humanitarian law standards prevent participants in an armed
(1957), we stated that the constitutional proscription on property ownership is not conflict from targeting nonparticipants.—Even if the lawfulness of the attack were
violated despite the foreign national’s control over the property. not in question, international humanitarian law standards prevent participants in
an armed conflict from targeting nonparticipants. International humanitarian
law, which is the branch of international law applicable to armed conflict,
255 expressly limits allowable military conduct exhibited by forces of a participant in
VOL. 779, JANUARY 12, 2016 255 an armed conflict. Under this legal regime, participants to an armed conflict are
held to specific standards of conduct that require them to distinguish between
Saguisag vs. Ochoa, Jr. combatants and non-combatants, as embodied by the Geneva Conventions and
their Additional Protocols.
Same; Same; Same; There is ample legal protection for the Philippines under absurd or unreasonable.” This is particularly true to a mutual defense treaty the
international law that would ensure its territorial integrity and national security purpose of which is mutual self-defense against sudden armed attack by a third
in the event an Agreed Location is subjected to attack.—Any armed attack by forces state.
of a third state against an Agreed Location can only be legitimate under Foreign Military Bases; Enhanced Defense Cooperation Agreement; View that
international humanitarian law if it is against a bona fide U.S. military base, the Philippines agreed with the United States (U.S.) to use the Mutual Defense
facility, or installation that directly contributes to the military effort of the U.S. Treaty (MDT) to preposition U.S. war materials in strategic locations in the
Moreover, the third state’s forces must take all measures to ensure that they have Philippines, particularly in the islands of Palawan and Luzon facing the West
complied with the principle of distinction (between combatants and non- Philippine Sea.—In 2012, China seized Scarborough Shoal from the Philippines,
combatants). There is, then, ample legal protection for the Philippines under which could offer no armed resistance to Chinese naval forces. The Scarborough
international law that would ensure its territorial integrity and national security Shoal seizure finally made the Philippine Government realize that there was an
in the event an Agreed Location is subjected to attack. As EDCA stands, it does absolute need to deter China’s creeping invasion of Philippine islands, rocks and
not create the situation so feared by petitioners — one in which the Philippines, reefs in the West Philippine Sea. Thus, the Philippines rushed the modernization
while not participating in an armed conflict, would be legitimately targeted by an of its navy and
enemy of the U.S.
Same; Same; Visiting Forces Agreement; The Visiting Forces
Agreement (VFA) already authorizes the presence of United States (U.S.) military 258
equipment in the country.—The provisions in EDCA dealing with Agreed Locations 258 SUPREME COURT REPORTS ANNOTATED
are analogous to those in the aforementioned executive agreements. Instead of
authorizing the building of temporary structures as previous agreements have Saguisag vs. Ochoa, Jr.
done, EDCA authorizes the U.S. to build permanent structures or alter or improve
existing ones for, and to be owned by, the Philippines. EDCA is clear that the air force. The Philippines also agreed with the U.S. to use the MDT to
Philippines retains ownership of altered or improved facilities and newly preposition U.S. war materials in strategic locations in the Philippines,
constructed permanent or non-relocatable struc- particularly in the islands of Palawan and Luzon facing the West Philippine Sea.
Same; Same; View that this is what the Enhanced Defense Cooperation
Agreement (EDCA) is all about — the prepositioning in strategic locations of war
257 materials to successfully resist any armed aggression.—In modern warfare, the
successful implementation of a mutual defense treaty requires the strategic
VOL. 779, JANUARY 12, 2016 257
prepositioning of war materials. Before the advent of guided missiles and drones,
Saguisag vs. Ochoa, Jr. wars could take months or even years to prosecute. There was plenty of time to
conscript and train soldiers, manufacture guns and artillery, and ship war
tures. Under EDCA, U.S. forces will also be allowed to use facilities and areas materials to strategic locations even after the war had started. Today, wars could
for “training; x x x; support and related activities; x x x; temporary accommodation be won or lost in the first few weeks or even first few days after the initial outbreak
of personnel; communications” and agreed activities. Concerns on national security of war. In modern warfare, the prepositioning of war materials, like mobile anti-
problems that arise from foreign military equipment being present in the ship and anti-aircraft missiles, is absolutely necessary and essential to a successful
Philippines must likewise be contextualized. Most significantly, the VFA already defense against armed aggression, particularly for a coastal state like the
authorizes the presence of U.S. military equipment in the country. Article Philippines. This is what the EDCA is all about — the prepositioning in strategic
VII of the VFA already authorizes the U.S. to import into or acquire in the locations of war materials to successfully resist any armed aggression. Such
Philippines “equipment, materials, supplies, and other property” that will be used prepositioning will also publicly telegraph to the enemy that any armed aggression
“in connection with activities” contemplated therein. The same section also would be repelled. The enemy must know that we possess the capability, that is,
recognizes that “[t]itle to such property shall remain” with the US and that they the war materials, to defend the country against armed aggression. Otherwise,
have the discretion to “remove such property from the Philippines at any time.” without such capability, we telegraph to the enemy that further seizure of
CARPIO, J., Separate Concurring Opinion: Philippine islands, rocks and reefs in the South China Sea would be a walk in the
Constitutional Law; Treaties; View that a ratified treaty cannot be interpreted park, just like China’s seizure of Mischief Reef and Scarborough Shoal. Without
to require a second ratified treaty to implement the first ratified treaty, as a such capability, we would practically be inviting the enemy to seize whatever
fundamental rule is that a treaty must be interpreted to avoid a “result which is Philippine island, rock or reef it desires to seize in the West Philippine Sea.
manifestly absurd or unreasonable.”—A ratified treaty like the MDT must be Same; Same; View that the Philippines will lose 381,000 square kilometers of
interpreted to allow the Executive to take all necessary measures to insure that its Exclusive Economic Zone (EEZ) in the West Philippine Sea, a maritime space
the treaty’s purpose is attained. A ratified treaty cannot be interpreted to require larger than the total Philippine land area of 300,000 square kilometers.—It is
a second ratified treaty to implement the first ratified treaty, as a fundamental obvious that China will use the three air and naval bases in its artificial islands to
rule is that a treaty must be interpreted to avoid a “result which is manifestly prevent Philippine ships and planes from re-supplying Philippine-occupied islands
in the Spratlys, forcing the Philippines to abandon its occupied islands. Already,
Chinese coast guard vessels are preventing medium-sized Philippine ships from prises the Philippine archipelago, with all the islands and waters embraced
re-supplying the BRP Sierra Madre, the therein, and all other territories over which the Philippines has sovereignty
or jurisdiction, consisting of its terrestrial, fluvial, and aerial domains, including
its territorial sea, the seabed, the subsoil, the insular shelves, and other submarine
259 areas. The waters around, between, and connecting the islands of the archipelago,
VOL. 779, JANUARY 12, 2016 259 regardless of their breadth and dimensions, form part of the internal waters of the
Philippines. (Emphasis supplied) Thus, the Philippine “national territory” refers
Saguisag vs. Ochoa, Jr. to areas over which the Philippines has “sovereignty or jurisdiction.” The
Constitution mandates: “The State shall protect the nation’s marine wealth in its
dilapidated Philippine landing ship beached in Ayungin Shoal, just 20 NM archipelagic waters, territorial sea, and exclusive economic zone, and reserve
from Mischief Reef. Only the Philippines’ use of small watercrafts enables the re- its use and enjoyment exclusively to Filipino citizens.”
supply to the BRP Sierra Madre, which is manned by about a dozen Philippine Same; Same; International Law; View that under international law and in
marine soldiers. The Philippines’ small watercrafts can navigate the shallow particular under the United Nations Convention on the Law of the Sea (UNCLOS),
waters of Ayungin Shoal while China’s large coast guard vessels cannot. With the the Philippines has jurisdiction over its Exclusive Economic Zone (EEZ) and
anticipated installation by China of military facilities and war materials in its Extended Continental Shelf (ECS).—Under international law and in particular
three air and naval bases in the Spratlys, expected to be completed before the end under the UNCLOS, the Philippines has jurisdiction over its EEZ and ECS.
of 2016, China will begin to aggressively enforce its 9-dashed lines claim over the Thus, under domestic law, the Philippines’ EEZ and ECS form part of Philippine
South China Sea. Under this claim, China asserts sovereignty not only to all the “national territory” since the Constitution defines “national territory” to include
islands, rocks and reefs in the Spratlys, but also to 85.7% of the South China Sea, areas over which the Philippines has “jurisdiction,” a term which means less
comprising all the waters, fisheries, mineral resources, seabed and submarine than sovereignty. However, under international law, the Philippine “national
areas enclosed by the 9-dashed lines. Under this claim, the Philippines will lose territory” refers to the areas over which the Philippines has sovereignty,
381,000 square kilometers of its EEZ in the West Philippine Sea, a maritime space referring to the Philippines’ land territory, archipelagic waters and territorial sea,
larger than the total Philippine land area of 300,000 square kilometers. China’s 9- excluding areas over which the Philippines exercises only jurisdiction like its EEZ
dashed lines claim encroaches on all the traditional fishing grounds of Filipino and ECS. China has already invaded repeatedly Philippine “national territory”
fishermen in the South China Sea: Scarborough Shoal, Macclesfield Bank and the in two separate areas, one in the Kalayaan Island Group in the Spratlys and the
Spratlys. other in Scarborough Shoal. When China seized in 1988 Subi Reef, a submerged
Same; Same; View that unless the United States (U.S.) and the Philippines area within the Philippines’ ECS and beyond the territorial sea of any high tide
have prepositioned anti-ship missiles in Palawan, there will be no deterrence to feature, China invaded Philippine national territory as defined in the Constitution.
China, and no swift response from U.S. and Philippine forces.—If China’s navy When China seized in 1995 Mischief Reef, a submerged area within the
ships attack a Philippine military ship re-supplying Philippine-occupied islands in Philippines’ EEZ and beyond the territorial sea of any high tide feature, China
the Spratlys, that will be covered by the MDT. However, unless the U.S. and the invaded Philippine national territory as defined in the Constitution. When China
Philippines have prepositioned anti-ship missiles in Palawan, there will be no seized in 2012 Scarborough Shoal, a rock above water at high tide and constituting
deterrence to China, and no swift response from U.S. and Philippine forces. The land territory under international law, China invaded Philippine national
absence of any deterrence will likely invite Chinese harassment, or even armed territory as defined in the Constitution and as
attack, on Philippine resupply ships. That will lead to the loss of all Philippine-
occupied islands in the Spratlys, as well as the loss of the gas-rich Reed Bank.
Constitutional Law; National Territory; View that the Philippine “national 261
territory” refers to areas over which the Philippines has
VOL. 779, JANUARY 12, 2016 261
“sovereignty or jurisdiction.”—The 1987 Constitution defines the “national
territory” to include not only islands or rocks above water at high tide but also the Saguisag vs. Ochoa, Jr.
seabed, subsoil and other submarine areas “over which the Philippines has
sovereignty or jurisdiction.” Article 1 of the 1987 Constitution provides: The understood in international law. Republic Act No. 9522, amending the
national territory com- Philippine Baselines Law, expressly declares that Scarborough Shoal is part of
Philippine territory over which the Philippines exercises “sovereignty and
jurisdiction.”
260 Same; Same; Enhanced Defense Cooperation Agreement; View that the
260 SUPREME COURT REPORTS ANNOTATED Enhanced Defense Cooperation Agreement (EDCA) was born, to give much needed
teeth to the Mutual Defense Treaty (MDT) as a deterrent to further Chinese
Saguisag vs. Ochoa, Jr. aggression in the West Philippine Sea. Without the EDCA, the MDT remains a
toothless paper tiger.—After China’s seizure of Scarborough Shoal in 2012, the Section 25, Article XVIII bans foreign military bases, troops, or facilities in
Philippines finally woke up and summoned the political will to address the serial Philippine territory, unless the following requisites are complied with: (1) the
and creeping Chinese invasion of Philippine national territory. Thus, the EDCA presence of foreign military bases, troops, or facilities should be allowed by
was born, to give much needed teeth to the MDT as a deterrent to further Chinese a treaty; (2) the treaty must be duly concurred in by the Philippine
aggression in the West Philippine Sea. Without the EDCA, the MDT remains a Senate and, when Congress so requires, such treaty should be ratified by a
toothless paper tiger. With the EDCA, the MDT acquires a real and ready majority of the votes cast by the Filipino people in a national referendum held for
firepower to deter any armed aggression against Philippine public vessels or that purpose; and (3) such treaty should be recognized as a treaty by the other
aircrafts operating in the West Philippine Sea. With the EDCA, China will think contracting party. Couched in negative terms, Section 25, Article XVIII
twice before attacking Philippine military resupply ships to Philippine-occupied embodies a prohibition: “foreign military bases, troops, or facilities shall not be
islands in the Spratlys. With the EDCA, the Philippines will have a fighting chance allowed in the Philippines,” unless the requisites in the said section are met.
to hold on to Philippine-occupied islands in the Spratlys. With the EDCA, China Foreign Military Bases; Visiting Forces Agreement; View that the Visiting
will think twice before attacking Philippine navy and coast guard vessels Forces Agreement (VFA) expressly allowed the importation into the Philippines of
patrolling the West Philippine Sea. This will give the Philippines a fighting chance reasonable quantities of personal baggage, personal effects, and other property for
to ward off China’s impending enforcement of its 9-dashed lines as China’s the personal use of United States (U.S.) personnel.—The VFA expressly allowed the
“national boundaries” as shown in its 2013 official vertical map. importation into the Philippines of reasonable quantities of personal baggage,
Same; Same; Same; View that to hold that the Enhanced Defense Cooperation personal effects, and other property for the personal use of United
Agreement (EDCA) cannot take effect without Senate ratification is to render the
Mutual Defense Treaty (MDT), our sole mutual self-defense treaty, totally inutile to
meet the grave, even existentialist,national security threat that the Philippines is 263
now facing in the West Philippine Sea.—The EDCA is absolutely necessary and VOL. 779, JANUARY 12, 2016 263
essential to implement the purpose of the MDT, which on the part of the
Philippines, given the existing situation in the West Philippine Sea, is to deter or Saguisag vs. Ochoa, Jr.
repel any armed attack on Philippine territory or on any Philippine public vessel
or aircraft operating in the West Philippine Sea. To hold that the EDCA cannot States personnel. The VFA likewise expressly allowed the entry into the
take effect without Senate ratification is to render the MDT, our sole mutual Philippines of (1) aircraft operated by or for the United States armed forces upon
approval of the Government of the Philippines in accordance with procedures
stipulated in implementing arrangements; and (2) vessels operated by or for the
262 United States armed forces upon approval of the Government of the Philippines,
in accordance with international custom and practice and such agreed
262 SUPREME COURT REPORTS ANNOTATED
implementing arrangements as necessary. The VFA also provided for the
Saguisag vs. Ochoa, Jr. jurisdiction over criminal and disciplinary cases over United States personnel with
respect to offences committed within the Philippines. The VFA further stated that
self-defense treaty, totally inutile to meet the grave, even existentialist, the same shall remain in force until the expiration of 180 days from the date on
national security threat that the Philippines is now facing in the West Philippine which either party gives the other party notice in writing that it desires to
Sea. China has already invaded several geologic features comprising part terminate the agreement.
of Philippine “national territory” as defined in the Constitution. The Same; Enhanced Defense Cooperation Agreement; View that under the
territorial integrity of the Philippines has been violated openly and repeatedly. Enhanced Defense Cooperation Agreement (EDCA), the Philippines by mutual
The President, as Commander-in-Chief of the Armed Forces, “chief architect” of agreement with the United States (U.S.), shall provide the U.S. forces the access
foreign policy and implementer of the MDT, has decided on the urgent need to and use of portions of Philippine territory; These portions of Philippine territory
fortify Philippine military defenses by prepositioning war materials of our treaty that will be made available to the U.S. are called “Agreed Locations,” which is a
ally on Philippine soil. This Court should not erect roadblocks to the President’s new concept defined under Article II(4) of the EDCA.—Under the EDCA, the
implementation of the MDT, particularly since time is of the essence and the Philippines by mutual agreement with the United States, shall provide the United
President’s act of entering into the EDCA on his own does not violate any provision States forces the access and use of portions of Philippine territory. United States
of the Constitution. forces are “the entity comprising United States personnel and all property,
equipment, and materiel of the United States Armed Forces present in the
LEONARDO-DE CASTRO, J., Concurring and Dissenting Opinion: territory of the Philippines.” These portions of Philippine territory that will be
made available to the US are called “Agreed Locations,” which is a new concept
Constitutional Law; Foreign Military Bases; View that Section 25, Article defined under Article II(4) of the EDCA as: 4. “Agreed Locations” means facilities
XVIII embodies a prohibition: “foreign military bases, troops, or facilities shall not and areas that are provided by the Government of the Philippines through the AFP
be allowed in the Philippines,” unless the requisites in the said section are met.— and that the United States forces, United States contractors, and others as
mutually agreed, shall have the right to access and usepursuant to this VOL. 779, JANUARY 12, 2016 265
Agreement. Such Agreed Locations may be listed in an annex to be appended to
this Agreement, and may further be described in implementing arrangements. Saguisag vs. Ochoa, Jr.
Same; Same; View that aside from the right to access and to use the Agreed
Locations, the United States (U.S.) may undertake the following types of activities base under Section 2, Presidential Decree (PD) No. 1227, otherwise known
within the Agreed Locations: security as “Punishing Unlawful Entry into Any Military Base in the Philippines.”—The
United States is authorized to preposition and store defense equipment, supplies,
and materiel (“prepositioned materiel”), including but not limited to, humanitarian
264 assistance and disaster relief equipment, supplies and material, at Agreed
Locations. The prepositioned materiel of the United States forces shall be for the
264 SUPREME COURT REPORTS ANNOTATED
exclusive use of United States forces, and full title to all such equipment, supplies
Saguisag vs. Ochoa, Jr. and materiel remains with the United States. United States forces and United
States contractors shall have unimpeded access to Agreed Locations for all matters
cooperation exercises; joint and combined training activities; humanitarian relating to the prepositioning and storage of defense equipment, supplies, and
and disaster relief activities; and such other activities that as may be agreed upon materiel, including delivery, management, inspection, use, maintenance, and
by the Parties.—Aside from the right to access and to use the Agreed Locations, removal of such equipment, supplies and materiel. The United States forces and
the United States may undertake the following types of activities within the United States contractors shall retain title to all equipment, materiel, supplies,
Agreed Locations: security cooperation exercises; joint and combined training relocatable structures, and other movable property that have been imported into
activities; humanitarian and disaster relief activities; and such other activities or acquired within the territory of the Philippines by or on behalf of United States
that as may be agreed upon by the Parties.” Article III(1) of the EDCA further forces. Considering the presence of US armed forces: military personnel, vehicles,
states in detail the activities that the United States may conduct inside the Agreed vessels, and aircrafts and other defensive equipment, supplies, and materiel in the
Locations: 1. With consideration of the views of the Parties, the Philippines hereby Philippines, for obvious military purposes and with the obvious intention of
authorizes and agrees· that United States forces, United States contractors, and assigning or stationing them within the Agreed Locations, said Agreed Locations,
vehicles, vessels, and aircrafts operated by or for United States forces may conduct for all intents and purposes, are considered military bases and fall squarely under
the following activities with respect to Agreed Locations: training; transit; the definition of a military base under Section 2, Presidential Decree No. 1227,
support and related activities; refueling of aircraft; bunkering of vessels; otherwise known as “Punishing Unlawful Entry into Any Military Base in the
temporary maintenance of vehicles, vessels, and aircraft; temporary Philippines,” which states: SECTION 2. The term “military base” as used in this
accommodation of personnel; communications; prepositioning of decree means any military, air, naval, or coast guard reservation, base,
equipment, supplies, and materiel; deploying forces and materiel; and fort, camp, arsenal, yard, station, or installation in the Philippines.
such other activities as the Parties may agree. Same; Same; View that the Enhanced Defense Cooperation Agreement
Same; Same; View that the United States (U.S.) is also granted operational (EDCA) provided for an initial term of ten (10) years, which thereafter shall
control of Agreed Locations to do construction activities, make alterations or continue in force automatically, unless terminated by either party by giving one
improvements of the Agreed Locations.—The United States is also granted year’s written notice through diplomatic channels of its intention to terminate the
operational control of Agreed Locations to do construction activities, make agreement.—The EDCA provided for an initial term of ten years, which thereafter
alterations or improvements of the Agreed Locations. All buildings, non- shall continue in force automatically, unless terminated by either party by giving
relocatable structures, and assemblies affixed to the land in the Agreed Locations, one year’s written notice through diplomatic channels of its intention to terminate
including [those] altered or improved by United States forces, remain the property the agreement.
of the Philippines. Permanent buildings constructed by the United States forces
become the property of the Philippines, once constructed, but shall be used by the
United States forces until no longer required. 266
Same; Same; View that considering the presence of United States (U.S.) 266 SUPREME COURT REPORTS ANNOTATED
armed forces: military personnel, vehicles, vessels, and aircrafts and other defensive
equipment, supplies, and materiel in the Philippines, for obvious military purposes Saguisag vs. Ochoa, Jr.
and with the obvious intention of assigning or stationing them within the Agreed
Locations, said Agreed Locations, for all intents and purposes, are considered Same; Same; International Agreements; View that the Enhanced Defense
military bases and fall squarely under the definition of a military Cooperation Agreement (EDCA) is an international agreement that allows the
presence in the Philippines of foreign military bases, troops and facilities, and thus
requires that the three requisites under Section 25, Article XVIII be complied with.
265 The EDCA must be submitted to the Senate for concurrence.—These EDCA
provisions establishes military areas similar to that in the Military Bases
Agreement, and for that reason alone, the EDCA is far greater in scope than both Philippines and the United States. Furthermore, as to the nature of the presence
the Mutual Defense Treaty and the Visiting Forces Agreement. The EDCA is not of foreign military troops in this country, the VFA is explicit in its characterization
a mere implementing agreement of either the MDT or the VFA. The EDCA is an that it is an agreement between the governments of the Philippines and the United
international agreement that allows the presence in the Philippines of States regarding the treatment of United States Armed Forces visiting the
foreign military bases, troops and facilities, and thus requires that the three Philippines. The Preamble of the VFA likewise expressly provides that, “noting
requisites under Section 25, Article XVIII be complied with. The EDCA must be that from time to time elements of the United States armed forces may visit the
submitted to the Senate for concurrence. Republic of the Philippines” and “recognizing the desirability of defining the
Same; Same; Same; View that international agreements may be in the form treatment of United States personnel visiting the Republic of the Philippines” the
of: (1) treaties, which require legislative concurrence after executive ratification; or parties to the VFA agreed to enter into the said treaty. The use of the word visit is
(2) executive agreements, which are similar to treaties, except that they do not very telling. In its ordinary usage, to visit is to “stay temporarily with (someone)
require legislative concurrence and are usually less formal and deal with a or at (a place) as a guest or tourist” or to “go to see (someone or something) for a
narrower range of subject matters than treaties.—Generally, the parties to an specific purpose.” Thus, the word visit implies the temporariness or impermanence
international agreement are given the freedom to choose the form of their of the presence at a specific location. On the other hand, under the EDCA, United
agreement. International agreements may be in the form of: (1) treaties, which States forces and United States contractors are permitted to stay in the Agreed
require legislative concurrence after executive ratification; or (2) executive Locations to undertake military activities therein without any clear limitation
agreements, which are similar to treaties, except that they do not require as to the duration of their stay. Moreover, they are given unimpeded access to
legislative concurrence and are usually less formal and deal with a narrower range Agreed Locations to
of subject matters than treaties. Under Article 2 of the Vienna Convention on the
Law of Treaties, a treaty is defined as an international agreement concluded
between states in written form and governed by international law, whether 268
embodied in a single instrument or in two or more related instruments and 268 SUPREME COURT REPORTS ANNOTATED
whatever its particular designation.
Same; Same; View that contrary to the posturing of the majority, the presence Saguisag vs. Ochoa, Jr.
of foreign military bases, troops, or facilities provided under the Enhanced Defense
Cooperation Agreement (EDCA) cannot be traced to the Mutual Defense Treaty conduct different activities that definitely were not contemplated under the
(MDT).—The very clear-cut focal point of the EDCA is the authority granted to the VFA.
United States forces and contractors to have unimpeded access to so- Same; Same; View that in the instant case, some of the activities that the
United States (U.S.) forces will undertake within the Agreed Locations such as
prepositioning of defense equipment, supplies and materiel, and deploying of forces
267 and materiel are actual military measures supposedly put into place in anticipation
of battle.—In the instant case, some of the activities that the United States forces
VOL. 779, JANUARY 12, 2016 267
will undertake within the Agreed Locations such as prepositioning of defense
Saguisag vs. Ochoa, Jr. equipment, supplies and materiel, and deploying of forces and materiel are actual
military measures supposedly put into place in anticipation of battle. To
called Agreed Locations — which can be anywhere in the Philippines — and preposition means “to place military units, equipment, or supplies at or near the
to build there military facilities and use the same to undertake various military point of planned use or at a designated location to reduce reaction time, and to
activities. The very wording of the EDCA shows that it undoubtedly deals with the ensure timely support of a specific force during initial phases of an operation.” On
presence of foreign military bases, troops, and facilities in Philippine territory. the other hand, materiel is defined as “all items necessary to equip, operate,
Thus, contrary to the posturing of the majority, the presence of foreign military maintain, and support military activities without distinction as to its application
bases, troops, or facilities provided under the EDCA cannot be traced to the MDT. for administrative or combat purposes.” Also, to deploy means “to place or arrange
Moreover, the general provisions of the MDT cannot prevail over the categorical (armed forces) in battle disposition or formation or in locations appropriate for
and specific provision of Section 25, Article XVIII of the Constitution. their future employment.” Deployment also means “the rotation of forces into and
Same; Same; Agreed Locations; View that under the Enhanced Defense out of an operational area.”
Cooperation Agreement (EDCA), United States (U.S.) forces and U.S. contractors Same; Same; View that this grant of authority to construct new buildings and
are permitted to stay in the Agreed Locations to undertake military activities the improvement of existing buildings inside the Agreed Locations — which
therein without any clear limitation as to the duration of their stay.—Initially, what buildings are to be used indefinitely — further evinces the permanent nature of the
is abundantly clear with the foregoing enumeration is that the EDCA is an entirely stay of United States (U.S.) forces and contractors in this country under the
new creation. The provisions of the EDCA are not found in or have no Enhanced Defense Cooperation Agreement (EDCA).—The EDCA likewise allows
corresponding provisions in the VFA. They cover entirely different subject matters the construction of permanent buildings, which the United States forces can utilize
and they create new and distinct rights and obligations on the part of the until such time that they no longer need the use thereof. The construction of
permanent buildings, including the alteration or improvement by the United
States of existing buildings, structures and assemblies affixed to the land, are 270
certainly necessary not only for the accommodation of its troops, bunkering of 270 SUPREME COURT REPORTS ANNOTATED
vessels, maintenance of its vehicles, but also the creation of the proper facilities
for the storage and prepositioning of its defense materiel. This grant of authority Saguisag vs. Ochoa, Jr.
to construct new buildings and the improvement of existing buildings inside the
Agreed Locations — which buildings are to be used indefinitely — further evinces Constitution. In the same vein, the initial entry of United
the perma- States troops under the VFA cannot, as postulated by the ponencia, justify a
“treaty-authorized” presence under the EDCA, since the presence contemplated in
the EDCA also pertains to the establishment of foreign military bases or facilities,
269 and not merely visiting troops.
Same; Same; View that the argument that the entry of the United States (U.S.)
VOL. 779, JANUARY 12, 2016 269
bases, troops and facilities under the Enhanced Defense Cooperation Agreement
Saguisag vs. Ochoa, Jr. (EDCA) is already allowed in view of the “initial entry” of U.S. troops under the
Visiting Forces Agreement (VFA) glaringly ignores that the entry of visiting foreign
nent nature of the stay of United States forces and contractors in this country military troops is distinct and separate from the presence or establishment of
under the EDCA. This is a far cry from the temporary visits of United States armed foreign military bases or facilities in the country under Section 25, Article XVIII of
forces contemplated in the VFA. the Constitution.—The argument that the entry of the United States bases, troops
Same; Same; View that the intervention of the Senate through the and facilities under the EDCA is already allowed in view of the “initial entry” of
constitutionally ordained treaty-making process in defining the new national policy United States troops under the VFA glaringly ignores that the entry of visiting
concerning United States (U.S.) access to Agreed Locations enunciated in the foreign military troops is distinct and separate from the presence or establishment
Enhanced Defense Cooperation Agreement (EDCA), which has never been before of foreign military bases or facilities in the country under Section 25, Article XVIII
expressly or impliedly authorized, is imperative and indispensible for the validity of the Constitution. To reiterate, the EDCA is entirely a new treaty, separate and
and effectivity of the EDCA.—Article II(4) of the EDCA states that the Agreed distinct from the VFA and the MDT. Hence, it must satisfy the requirements under
Locations shall be provided by the Philippine Government through the Section 25, Article XVIII of the Constitution. The Senate itself issued Resolution
AFP. What is readily apparent from said article is that the AFP is given a broad No. 105 on November 10, 2015, whereby it expressed its “definite stand on the non-
discretion to enter into agreements with the United States with respect to the negotiable power of the Senate to decide whether a treaty will be valid and effective
Agreed Locations. The grant of such discretion to the AFP is without any guideline, depending on the Senate concurrence” and resolved “that the RP-US EDCA [is a]
limitation, or standard as to the size, area, location, boundaries and even the treaty [that] requires Senate concurrence in order to be valid and effective.”
number of Agreed Locations to be provided to the United States forces. As there is
no sufficient standard in the EDCA itself, and no means to determine the limits of BRION, J., Dissenting Opinion:
authority granted, the AFP can exercise unfettered power that may have grave
implications on national security. The intervention of the Senate through the Constitutional Law; Executive Agreements; Enhanced Defense Cooperation
constitutionally ordained treaty-making process in defining the new national Agreement; View that the Enhanced Defense Cooperation Agreement (EDCA) is an
policy concerning United States access to Agreed Locations enunciated in the agreement that, on deeper examination, violates the letter and spirit of Article
EDCA, which has never been before expressly or impliedly authorized, XVIII, Section 25 and Article VII, Section 21, both of the Constitution.—I
is imperative and indispensible for the validity and effectivity of the EDCA. dissent, as I disagree that an executive agreement is the proper medium
Same; Same; View that the initial entry of United States (U.S.) troops under for the matters covered by the EDCA. The EDCA is an agreement that, on
the Visiting Forces Agreement (VFA) cannot, as postulated by the ponencia, justify deeper examination, violates the letter and spirit of Article XVIII, Section 25 and
a “treaty-authorized” presence under the Enhanced Defense Cooperation Agreement Article VII, Section 21, both of the Constitution. The EDCA should be in the
(EDCA), since the presence contemplated in the EDCA also pertains to the form of a treaty as it brings back to
establishment of foreign military bases or facilities, and not merely visiting
troops.—The VFA, which allows the presence of the units of the United States
military troops, cannot by any stretch of the imagination include any arrangement 271
that practically allows the establishment of United States VOL. 779, JANUARY 12, 2016 271
military bases or facilities in the so-called Agreed Locations under the EDCA.
Thus, the EDCA goes far-beyond the arrangement contemplated by the VFA and Saguisag vs. Ochoa, Jr.
therefore it necessarily requires Senate concurrence as mandated by Section 25,
Article XVIII of the the Philippines: the modern equivalent of the foreign military bases
whose term expired in 1991 and which Article XVIII, Section 25 of the
Constitution directly addresses; foreign troops under arrangements case are so important that a plain citizen sufficiently knowledgeable of the
outside of the contemplation of the visiting forces that the 1998 VFA outstanding issues, should be allowed to sue. The petitioners — some of whom are
allows; and military facilities that, under modern military strategy, recognized legal luminaries or are noted for their activism on constitutional
likewise can be brought in only through a treaty. matters — should thus be recognized as parties with proper standing to file and
Same; Judicial Review; View that judicial review is part of the exercise of pursue their petitions before this Court.
judicial power under Article VIII, Section 1 of the Constitution, particularly when Same; Foreign Military Bases; View that in the present cases, Article VIII,
it is exercised under the judiciary’s expanded power (i.e., when courts pass upon the Section 25 of the Constitution lays down in no uncertain terms the conditions under
actions of other agencies of government for the grave abuse of discretion they which foreign military bases, troops, and facilities may be allowed into the country:
committed), or when the Supreme Court (SC) reviews, on appeal or certiorari, the there should at least be the concurrence of the Senate.—In the present cases, Article
constitutionality or validity of any law or other governmental instruments under VIII, Section 25 of the Constitution lays down in no uncertain terms the conditions
Section 5(2)(a) and (b) of Article VIII of the Constitution.—I agree with under which foreign military bases, troops, and facilities may be allowed into the
the ponencia’s ultimate conclusions on the threshold issues raised. I agree as well country: there should at least be the concurrence of the Senate. Under these terms,
that a justiciable issue exists that the Court can pass upon, although on both the refusal to allow entry of foreign military bases, troops, and facilities into the
counts I differ from the ponencia’s line of reasoning. Let me point out at the outset, country without the required Senate concurrence is a prerogative that the people
too, that judicial review is only an exercise of the wider judicial power that Article of this country adopted for themselves under their Constitution: they want
VIII, Section 1 of the Constitution grants and defines. One should not be confused participation in this decision, however indirect this participation might be. This
with the other. Judicial review is part of the exercise of judicial power under Article prerogative is exercised through the Senate; thus, a violation of this constitutional
VIII, Section 1 of the Constitution, particularly when it is exercised under the prerogative is not only a transgression against the Senate but one against the
judiciary’s expanded power (i.e., when courts pass upon the actions of other people who the Senate represents. The violation in this case occurred when the
agencies of government for the grave abuse of discretion they committed), or when President ratified the EDCA as an executive agreement and certified to the other
the Supreme Court reviews, on appeal or certiorari, the constitutionality or contracting party (the U.S.) that all the internal processes have been complied
validity of any law or other governmental instruments under Section 5(2)(a) and with, leading the latter to believe that the
(b) of Article VIII of the Constitution. A basic requirement is the existence of
an actual case or controversy that, viewed correctly, is a limit on the exercise of
judicial power or the more specific power of judicial review. 273
Same; Locus Standi; View that the standing of even a plain citizen sufficiently VOL. 779, JANUARY 12, 2016 273
able to bring and support a suit, should be recognized as he or she can then be
deemed to be acting in representation of the general public.—Locus standi is a Saguisag vs. Ochoa, Jr.
requirement for the exercise of judicial review and is in fact an aspect of the actual
case or controversy requirement viewed from the prism of the complaining party agreement is already valid and enforceable. Upon such violation, the dispute
whose right has been violated. When a violation of a private right is between the President and the Filipino people ripened.
Same; Same; Enhanced Defense Cooperation Agreement; View that if indeed
a referral to the Senate is required and no referral has been made, then the
272 Enhanced Defense Cooperation Agreement (EDCA) is constitutionally deficient so
that its terms cannot be enforced.—The main issue the petitioners pose — the
272 SUPREME COURT REPORTS ANNOTATED
constitutional status of the EDCA as an executive agreement in light of the
Saguisag vs. Ochoa, Jr. mandate of Article XVIII, Section 25 of the Constitution — is not a political
question outside the judiciary’s competence and authority to resolve. The
asserted, the locus standi requirement is sharp and narrow because the respondents’ argument on this point is therefore erroneous. If indeed a referral to
claim of violation accrues only to the complainant or the petitioner whose right is the Senate is required and no referral has been made, then the EDCA
alleged to have been violated. On the other hand, when a violation of a public is constitutionally deficient so that its terms cannot be enforced. This finding
right is asserted — i.e., a right that belongs to the public in general and whose renders further proceedings on the merits of the substantive issues raised,
violation ultimately affects every member of the public — the locus pointless and unwarranted. There is likewise no point in determining whether the
standi requirement cannot be sharp or narrow; it must correspond in width to the substantive issues raised call for the application of the political question doctrine.
right violated. Thus, the standing of even a plain citizen sufficiently able to bring Same; Same; Same; View that the Filipino nation cannot attain self-respect
and support a suit, should be recognized as he or she can then be deemed to be unless it shows its respect for its own Constitution — the only instrument that binds
acting in representation of the general public. the whole nation.—EDCA and Article XVIII, Section 25 of the Constitution,
Same; Same; View that the issues involved in the present case are so in their larger regional signification, mean that the Philippines would
important that a plain citizen sufficiently knowledgeable of the outstanding thereafter, not only be bound as an American ally under the 1951 MDT, but
issues, should be allowed to sue.—I believe that the issues involved in the present as an active participant as “pivot” and projection points in the grand
American strategy in Asia. How the Philippines will react to all these Same; Same; View that while the President’s role as the country’s lead official
developments is largely for the Executive and the people (through the Legislature) in the conduct of foreign affairs is beyond question, his authority is not without
to determine. In making its decisions, they must — at the very least — show one limit.—While the President’s role as the country’s lead official in the conduct of
and all that our country is entitled to respect as an independent and sovereign foreign affairs is beyond
nation. This respect must come primarily from within the Philippines and
the Filipinos themselves, from the nation’s own sense of self-respect: in
negative terms, the Filipino nation cannot attain self-respect unless it 275
shows its respect for its own Constitution — the only instrument that binds VOL. 779, JANUARY 12, 2016 275
the whole nation.
Same; Presidency; View that the supremacy of the Constitution means that in Saguisag vs. Ochoa, Jr.
the performance of his duties, the President should always be guided and kept in
check by the safeguards that were crafted by the framers of the Constitution and question, his authority is not without limit. When examined within the
ratified by the people.—The supremacy of the Constitution means that in the larger context of how our tripartite system of government works (where each
performance of his duties, the President should always be guided and kept in check branch of government is supreme within its sphere but coordinate with the others),
by the safeguards that were we can see that the conduct of foreign affairs, particularly when it comes to
international agreements, is a shared function among all three branches of
government. The President is undeniably the chief architect of foreign policy and
274 is the country’s representative in international affairs. He is vested with the
authority to preside over the nation’s foreign relations which involve, among
274 SUPREME COURT REPORTS ANNOTATED
others, dealing with foreign states and governments, extending or withholding
Saguisag vs. Ochoa, Jr. recognition, maintaining diplomatic relations, and entering into treaties. In the
realm of treaty-making, the President has the sole authority to negotiate with
crafted by the framers of the Constitution and ratified by the people. The other States.
Constitution prescribes the limitations to the otherwise awesome powers of the Same; Same; Treaties; View that by requiring the concurrence of the
Executive who wields the power of the sword and shares in the power of the purse. Legislature in the treaties entered into by the President, the Constitution ensures a
I also do not agree that constitutional limitations, such as the need for Senate healthy system of checks and balances necessary in the nation’s pursuit of political
concurrence in treaties, can be disregarded if they unduly “tie the hands” of the maturity and growth.—That we have consistently included the participation of the
President. These limitations are democratic safeguards that place the legislative branch in the treaty-making process is not without an important
responsibility over national policy beyond the hands of a single official. Their reason: it provides a check on the Executive in the field of foreign relations. By
existence is the hallmark of a strong and healthy democracy. In treaty-making, requiring the concurrence of the Legislature in the treaties entered into by the
this is how the people participate — through their duly elected Senate — or President, the Constitution ensures a healthy system of checks and balances
directly when the Congress so requires. When the Constitution so dictates, the necessary in the nation’s pursuit of political maturity and growth. Under this
President must act through the medium of a treaty and is left with no discretion on system, the functions of government are divided among three branches of
the matter. This is the situation under Article XVIII, Section 25 of the Constitution, government, each one supreme within its own sphere: the executive administers
whose application is currently in dispute. Let it be noted that noble objectives do and enforces laws; the legislature formulates and enacts laws; and the judiciary
not authorize the President to bypass constitutional safeguards and limits to his settles cases arising out of the enforcement of these laws. The requirement of
powers. To emphasize this point, we only need to refer to Article VI, Section 23(2) Senate concurrence to the executive’s treaty-making powers is a check on the
of the Constitution: (2) In times of war or other national emergency, prerogative of the Executive, in the same manner that the Executive’s veto on laws
the Congress may by law authorize the President, for a limited period and passed by Congress is a check on the latter’s legislative powers.
subject to such restrictions as it may prescribe, to exercise powers necessary Same; Same; Same; Judicial Review; View that entry into international
and proper to carry out a declared national policy. Unless sooner withdrawn agreements is a shared function among the three (3) branches of government.—The
by resolution of the Congress, such power shall cease upon the next adjournment judicial branch of government’s participation in international agreements is
thereof. [Emphasis supplied] Thus, the President cannot, by himself, usurp the largely passive, and is only triggered when cases reach the courts. The courts, in
prerogatives of a coequal branch to carry out what he believes is necessary for the the exercise of their judicial power, have the duty to ensure that the Executive and
country’s defense interests. His position as the Commander-in-Chief of the Armed Legislature stay within their spheres of competence; they ensure as
Forces of the Philippines (AFP) does not give him the sole discretion to increase
our military’s defensive capabilities; his role as commander-in-chief only gives him
control of the military’s chain of command. It grants him the power to call out the 276
armed forces to prevent/suppress lawless violence, invasion, insurrection, or 276 SUPREME COURT REPORTS ANNOTATED
rebellion.
Saguisag vs. Ochoa, Jr. They cannot contravene statutory enactments and treaties and would be invalid if
they do so. Again, this difference in impact is traceable to the source of their
well that constitutional standards and limitations set by the Constitution for authority; since a treaty has the approval of both the President and the Senate, it
the Executive and the Congress to follow are not violated. Article VIII, Section 5 has the same impact as a statute. In contrast, since an executive agreement
of the Constitution is even more explicit, as it gives the Supreme Court the springs from the President’s power to execute laws, it cannot amend or violate
jurisdiction “to review by appeal or certiorari all cases in which the existing treaties, and must be in accord with and be made pursuant to existing
constitutionality or validity of any treaty, international or executive agreement, laws and treaties.
law x x x is in question.” Thus, entry into international agreements is a shared Same; Same; Same; International Agreements; View that when an
function among the three branches of government. In this light and in the context international agreement is made merely to implement an existing law or
that the President’s actions should be viewed under our tripartite system of treaty, then it can properly take the form of an executive agreement. In contrast,
government, I cannot agree with the ponencia’s assertion that the when an international agreement involves the introduction of a new subject
case should be examined solely and strictly through the constitutional matter or the amendment of existing agreements or laws and has not passed the
limitation found in Article XVIII, Section 25 of the Constitution. required executive and legislative processes, then it should properly be in the form
Same; Treaties; Executive Agreements; View that Bayan Muna v. of a treaty.—Accordingly, the terms and objectives of the presidential entry into an
Romulo, 641 SCRA 244 (2011), did not distinguish between treaties and executive international agreement dictates the form the agreement must take. When an
agreements in terms of their binding effects on the contracting States concerned.— international agreement is made merely to implement an existing law or
Jurisprudential definitions of treaties and executive agreements are conceptually treaty, then it can properly take the form of an executive agreement. In contrast,
drawn from these distinctions although in Bayan Muna v. Romulo, 641 SCRA 244 when an international agreement involves the introduction of a new subject
(2011), we simply differentiated treaties from executive agreements in this wise: matter or the amendment of existing agreements or laws and has not passed
Article 2 of the Vienna Convention on the Law of Treaties: An international the required executive and legislative processes, then it should properly be in the
agreement concluded between states in written form and governed by form of a treaty.
international law, whether embodied in a single instrument or in two or more Same; Foreign Military Bases; View that the dynamics that Article VII,
related instruments and whatever its particular designation. International Section 21 embody, should be read into Article XVIII, Section 25 of the 1987
agreements may be in the form of (1) treaties that require legislative Constitution, which specifically covers and applies to the entry of foreign military
concurrence after executive ratification; or (2) executive agreements that bases, troops, or facilities into the country.—The dynamics that Article VII, Section
are similar to treaties, except that they do not require legislative 21 embody, should be read into Article XVIII, Section 25 of the 1987 Constitution,
concurrence and are usually less formal and deal with a narrower range of which specifically covers and applies to the entry of foreign military bases, troops,
subject matters than treaties. [Emphases supplied] Bayan Muna likewise did not or facilities into the country. It is on the basis of this joint
distinguish between treaties and executive agreements in terms of their binding
effects on the contracting States concerned. But neither one can contravene the
Constitution. This ambiguity perhaps might have been the root of the general 278
statement that the Executive generally has the discretion to determine whether 278 SUPREME COURT REPORTS ANNOTATED
an international obligation should be in the form of a treaty or an executive Saguisag vs. Ochoa, Jr.
agreement. This general statement, however, is far from complete and should be
qualified because the Executive’s exercise of discretion is affected and should be reading that the ponencia’s conclusion — that Article XVIII, Section 25
dictated by the demands of the enforceability of the obligations the international applies only to the initial entry of foreign military bases, troops, and facilities in
the country — is essentially incorrect. Article XVIII, Section 25 does not provide
for any such limitation in its applicability. Neither is there anything in the
277 language of the provision that remotely implies this consequence. What it simply
VOL. 779, JANUARY 12, 2016 277 states is that foreign military bases, troops, and facilities may only be present in
Saguisag vs. Ochoa, Jr. Philippine soil in accordance with a treatyconcurred in by the Senate.
Same; Same; International Agreements; View that any subsequent
agreement creates in the domestic sphere. Between a treaty and an executive international agreement referring to military bases, troops or facilities should
agreement, a treaty exists on a higher plane as it carries the authority of the be examined based on whether it creates a new obligation or implements an existing
President and the Senate. Treaties have the status, effect, and impact of statutory one.—After the initial entry of foreign military bases, troops, or facilities in the
law in the Philippines; they can amend or prevail over prior statutory enactments. Philippines under a duly ratified treaty, subsequent arrangements relating to
Executive agreements — which exist at the level of implementing rules and foreign military bases, troops or facilities that are claimed to be based on the same
regulations or administrative orders in the domestic sphere — carry no such effect. treaty, should be examined based on the treaty-executive agreement distinctions
recognized by jurisprudence under Article VII, Section 21 of the Constitution. In
other words, any subsequent international agreement referring to military bases, EDCA imposed new or amended existing obligations involving foreign
troops or facilities should be examined based on whether it creates a new military bases, troops, and facilities in the Philippines.
obligation or implements an existing one. The determination of this question Same; Foreign Military Bases; View that within its territory, a nation reigns
rests with the Executive but the treaty-executive agreement distinctions should supreme. If it will allow interference at all, such interference should be under the
limit the Executive’s discretion when the new international agreement relates to terms the nation allows and has accepted; beyond those terms, the primacy of
a new obligation (or a change in an existing obligation) as the presence of foreign sovereignty is the rule.—I believe that the ponencia’s approach and
military bases, troops, or facilities in the Philippines should then be effected interpretation are incorrect because they are overly simplistic. The proper
through another treaty. understanding of Article XVIII, Section 25 must take into account the many
Same; Same; Enhanced Defense Cooperation Agreement; View that if the considera-
Enhanced Defense Cooperation Agreement (EDCA) introduces foreign military
bases, troops, or facilities in the Philippines within the contemplation of Article
XVIII, Section 25 of the 1987 Constitution, and if these obligations are different 280
from those found in our existing treaty obligations with the United States (U.S.), 280 SUPREME COURT REPORTS ANNOTATED
then the EDCA cannot be enforced in the Philippines without the Senate’s
concurrence.—If the EDCA introduces foreign military bases, troops, or facilities Saguisag vs. Ochoa, Jr.
in the Philippines within the contemplation of Article XVIII, Section 25 of the 1987
Constitution, and if these obligations are different from those found in our existing tions that bear upon its plain terms, among them, the treaty-executive
treaty obligations with agreement distinctions under Article VII, Section 21 that I discussed above; the
history of Article XVIII, Section 25; the motivations that drove the framers to adopt
the provision; and the current and contemporaneous developments and usages
279 that give full and effective meaning to the provision. Separately from textual
interpretation considerations and as part of the history of Article XVIII, Section
VOL. 779, JANUARY 12, 2016 279
25, the basic concept of sovereignty that underlies it should not be forgotten.
Saguisag vs. Ochoa, Jr. Sovereignty means the full right and power of the nation to govern itself, its people,
and its territory without any interference from outside sources or entities. Within
the U.S., then the EDCA cannot be enforced in the Philippines without the its territory, a nation reigns supreme. If it will allow interference at all, such
Senate’s concurrence. The ponencia is then incorrect and the Dissent must interference should be under the terms the nation allows and has accepted; beyond
prevail. Conversely, if the EDCA merely implements present treaty obligations those terms, the primacy of sovereignty is the rule.
— particularly those under the 1951 MDT and the 1998 VFA — then the President Same; Treaties; View that Article XVIII, Section 25 imposed three (3)
was well within his powers in the execution of our present treaty requirements that must be complied with for an agreement to be considered valid
obligations. The ponencia is correct and the Dissent therefore fails. insofar as the Philippines is concerned. These three requirements are: (1) the
Same; International Agreements; Treaties; View that when the subject of an agreement must be embodied in a treaty; (2) the treaty must be duly concurred in
international agreement falls under Article XVIII, Section 25 of the Constitution, by 2/3 votes of all the members of the Senate; recognized as a treaty by the other
the President — by constitutional command — must enter into a treaty subject to State; and (3) the agreement must be recognized as a treaty by the other State.—
the concurrence of the Senate and, when Congress so desires, of the people through Article XVIII, Section 25 imposed three requirements that must be complied with
a national referendum.—When the subject of an international agreement falls for an agreement to be considered valid insofar as the Philippines is concerned.
under Article XVIII, Section 25 of the Constitution, the President — by These three requirements are: (1) the agreement must be embodied in a treaty; (2)
constitutional command — must enter into a treaty subject to the concurrence of the treaty must be duly concurred in by 2/3 votes of all the members of the Senate;
the Senate and, when Congress so desires, of the people through a national and (3) the agreement must be recognized as a treaty by the other State. On the
referendum. This rule opens the door for Court intervention pursuant to its duty second requirement, the two-thirds concurrence of all the members of the Senate,
to uphold the Constitution and its further duty (under its power of judicial review) the people’s representative, may be viewed as the people’s “voluntary submission”
to pass upon any grave abuse of discretion committed by any official or agency of of their sovereignty so they can reap the greater benefits of the agreement that the
government. It is under this constitutionally-mandated terms that this Court President, as policymaker, entered into. When the Congress so requires, the
invokes its power to review the constitutionality of the President’s actions in agreement should be ratified by a majority of the votes cast by the people in a
handling the EDCA. Within this framework, the issue these cases present is clear. national referendum held for that purpose. This additional requirement evinces
The bottom line question is whether the President gravely abused his the framers’ intent to emphasize the people’s direct participation in treaty-making.
discretion in executing the EDCA as an executive agreement; the alleged Same; Same; Mutual Defense Treaty; View that an attack on either party will
existence of grave abuse of discretion constitutes the actual case or be acted upon in accordance with their constitutional processes and any armed
controversy that allows the exercise of judicial power. Whether grave abuse attack on either party will be brought to the
exists, in turn, depends on the determination of whether the terms of the
official state visit by U.S. President Barack Obama. The 10-year accord is the
281 second military agreement between the U.S. and the Philippines (the first being
VOL. 779, JANUARY 12, 2016 281 the 1998 VFA) since American troops withdrew from its Philippines naval base in
1992. The agreement allows the U.S. to station troops and operations on Philippine
Saguisag vs. Ochoa, Jr. territory without establishing a permanent base and with the stipulation that the
U.S. is not allowed to store or position any nuclear weapons on Philippine territory.
attention of the United Nations (UN) for immediate action.—The 1951 MDT Same; View that the Enhanced Defense Cooperation Agreement (EDCA) is
provides for an alliance — that both nations would support one another if either effective for ten (10) years, unless both the United States (U.S.) and the Philippines
the Philippines or the U.S. would be attacked by an external party. It states that formally agree to alter it.—The EDCA is effective for 10 years, unless both the U.S.
each party shall either, separately or jointly, through mutual aid, acquire, develop and the Philippines formally agree to alter it. The U.S. is bound to hand over any
and maintain their capacity to resist armed attack. It provides for a mode of and all facilities in the “Agreed Locations” to the Philippine government upon the
consultations to determine the 1951 MDT’s appropriate implementation measures termination of the Agreement.
and when either of the parties determines that their territorial integrity, political Military Bases; View that a military base connotes the presence, in a relatively
independence, or national security is threatened by armed attack in the Pacific. An permanent degree, of troops and facilities in a particular area.—There exists no
attack on either party will be acted upon in accordance with their constitutional rigid definition of a military base. However, it is a term used in the field of military
processes and any armed attack on either party will be brought to the attention of operations and thus has a generally accepted connotation. The U.S. Department of
the United Nations for immediate action. The accord defines the meaning of an Defense (DoD) Dictionary of Military and Associated Terms defines a base as “an
armed attack as including armed attacks by a hostile power on a metropolitan area area or locality containing installations which provide logistic or other support”;
of either party, on the island territories under their jurisdiction in the Pacific, or home carrier. Under our laws, we find the definition of a military base in
on their armed forces, public vessels, or aircrafts in the Pacific. The U.S. Presidential Decree No. 1227 which states that a military base is “any military,
government guaranteed to defend the security of the Philippines against external air, naval, coast guard reservation, base, fort, camp, arsenal, yard, station, or
aggression but not necessarily against internal subversion. The treaty expressly installation in the Philippines.” A military base connotes the presence, in a
stipulates that its terms are indefinite and would last until one or both parties relatively permanent degree, of troops and facilities in a particular area.
terminate the agreement by a one-year advance notice. The treaty subsequently Enhanced Defense Cooperation Agreement; View that the Enhanced Defense
became the basis for an annual joint exercise, known as Balikatan, between the Cooperation Agreement (EDCA) allows United States (U.S.) military personnel to
Philippines and the U.S. enter and remain in Philippine territory. It grants the U.S. the right to construct
Same; Same; Visiting Forces Agreement; View that the 1998 Visiting Forces structures and assemblies. It also allows the U.S. to preposition defense equipment,
Agreement (VFA) allows the rotational presence of United States (U.S.) military supplies and materiel. The U.S. personnel may also use the Agreed Locations to
forces and their operations anywhere in the Philippines for a temporary but refuel aircraft and bunker vessels.—To go back to the EDCA, it nota-
undefined length of time to train and inter-operate with the Philippine armed forces
and to use their facilities.—In line with the American approach, the 1998 VFA
allows the rotational presence of U.S. military forces and their operations 283
anywhere in the Philippines for a temporary but undefined length of time to train
VOL. 779, JANUARY 12, 2016 283
and inter-operate with the Philippine armed forces and to use their facilities. The
Philippines retains jurisdiction over criminal cases, including capital offenses, Saguisag vs. Ochoa, Jr.
involving U.S. troops.
Enhanced Defense Cooperation Agreement; View that the agreement allows bly allows the U.S. to use the Agreed Locations for the following activities:
the United States (U.S.) to station troops and operations on Philippine territory “training, transit, support and related activities, refueling of aircraft; bunkering of
without establishing a permanent base and with the stipulation that the U.S. is not vessels; temporary maintenance of vehicles, vessels, and aircraft; temporary
allowed to store or accommodation of personnel; communications; prepositioning of equipment,
supplies, and materiel; deploying forces and materiel and such other activities as
the Parties may agree.” In order to carry out these activities, the EDCA allows U.S.
282 military personnel to enter and remain in Philippine territory. It grants the U.S.
282 SUPREME COURT REPORTS ANNOTATED the right to construct structures and assemblies. It also allows the
U.S. to preposition defense equipment, supplies and materiel. The U.S. personnel
Saguisag vs. Ochoa, Jr. may also use the Agreed Locations to refuel aircraft and bunker vessels.
Mutual Defense Treaty; Visiting Forces Agreement; View that neither the
position any nuclear weapons on Philippine territory.—The EDCA was 1951 Mutual Defense Treaty (MDT) nor the 1998 Visiting Forces Agreement
signed on April 28, 2014, in Manila, by Philippine Defense Secretary Voltaire (VFA) authorized stockpiling. The 1951 MDT focused on developing the Philippines
Gazmin, and U.S. Ambassador to the Philippines Philip Goldberg, in time for the and the United States’ (U.S.’) capacity to resist an armed attack while 1998 VFA
focused on the entry and exit of U.S. troops in the country. No provision in either Same; Same; View that the introduction of military bases or their equivalent,
treaty specifically allows stockpiling of military materiel.—Notably, neither the of troops, and of military facilities into the Philippines can now only take place by
1951 MDT nor the 1998 VFA authorized stockpiling. The 1951 MDT focused way of a treaty concurred in by the Senate.—The detailed arrangements that
on developing the Philippines and the U.S.’s capacity to resist an armed attack expired with the 1947 MBA were not carried over to the 1951 MDT as this treaty
while 1998 VFA focused on the entry and exit of US troops in the country. No only generally provided for the defense and alliance relationship between the U.S.
provision in either treaty specifically allows stockpiling of military materiel. In and the Philippines. Thus, there were no specific policies on military bases, troops,
sum, the Agreed Locations mentioned in the EDCA are areas where the U.S. can and facilities that could be implemented and operationalized by subsequent
perform military activities in structures built by its personnel. The extent of the executive agreements on the basis of the MDT. In particular, the terms of the
U.S.’ right to use of the Agreed Locations is broad enough to include even 1947 MBA that had expired and had not been renewed cannot be deemed
the stockpiling of weapons and the shelter and repair of vessels over which the U.S. carried
personnel has exclusive control. Clearly, this is a military base as this term is
ordinarily understood. Further, as we held in BAYAN (Bagong Alyansang
Makabayan) v. Zamora, 342 SCRA 449 (2000), Article XVIII, Section 25 refers to 285
three different situations: the presence of foreign military bases, troops, or VOL. 779, JANUARY 12, 2016 285
facilities. Even assuming that the EDCA is not a basing agreement, it
nevertheless involves the deployment of troops and facilities in Philippine Saguisag vs. Ochoa, Jr.
soil. As I have already stated, the EDCA allows U.S. forces to enter and remain in
the Philippines. It defines U.S. forces to include U.S. military and civilian over to the 1951 MDT. If any such future agreements would be made
personnel and U.S. Armed Forces property, after the effectivity of the 1987 Constitution, then such agreements would
be governed by Article XVIII, Section 25 of the new
Constitution.Significantly, when the 1987 Constitution and its Article XVIII,
284 Section 25 took effect, no absolute prohibition against the introduction of new U.S.
bases, troops, and facilities was put in place. In fact the 1951 MDT then still
284 SUPREME COURT REPORTS ANNOTATED
existed as a general defense alliance of the Philippines and the U.S. against armed
Saguisag vs. Ochoa, Jr. attack by third parties. But the introduction of military bases or their equivalent,
of troops, and of military facilities into the Philippines can now only take place by
equipment, and materiel. The EDCA itself provides that the U.S. can deploy way of a treaty concurred in by the Senate.
forces and materiel in the Agreed Locations. Same; Same; View that the Enhanced Defense Cooperation Agreement
Foreign Military Bases; Enhanced Defense Cooperation Agreement; View that (EDCA) introduces new arrangements and obligations to those existing under the
that the Enhanced Defense Cooperation Agreement (EDCA) allows this 1998 Visiting Forces Agreement (VFA); hence, the EDCA should be in the form of a
arrangement for an initial period of ten (10) years, to continue automatically treaty.—Is the EDCA merely an agreement implementing the 1998 VFA which
unless terminated, is further proof that it pertains to the presence in Philippine soil already allows the limited entry of U.S. military troops and the construction of
of foreign military bases, troops, and facilities on a more or less permanent basis.— facilities? The quick and short answer to the above question is — No, the EDCA
That the EDCA allows this arrangement for an initial period of 10 years, to does not implement the 1998 VFA as the EDCA in fact provides a wider
continue automatically unless terminated, is further proof that it pertains to the arrangement than the 1998 VFA with respect to the entry of military bases, troops,
presence in Philippine soil of foreign military bases, troops, and facilities on a more and facilities into the Philippines. A naughty view is that the 1998 VFA should
or less permanent basis.Note, at this point, that the Senators, during the form part of the EDCA and not the other way around. Another reality, based on
ratification of the 1998 VFA, observed that it only covers temporary visits of U.S. the treaty-executive agreement distinctions discussed above, is that the EDCA
troops and personnel in the country. These Senators gave their consent to the introduces new arrangements and obligations to those existing under the
1998 VFA on the knowledge that the U.S. forces’ stay in the country may 1998 VFA; hence, the EDCA should be in the form of a treaty.
last only up to three weeks to six months per batch. This temporary stay of Visiting Forces Agreement; View that the Visiting Forces
U.S. forces in the Philippines under the 1998 VFA means that it does not cover, or Agreement (VFA) itself does not authorize United States (U.S.) troops to
approve of, a more permanent stay of US forces and their equipment in the permanently stay in the Philippines, nor authorize any activity related to the
Philippines. Significantly, this is the key characteristic of the Agreed Locations in establishment and the operation of bases, as these activities had been defined under
the EDCA. For, if the EDCA had not envisioned the stay of U.S. forces and the 1947 Military Bases Agreement (MBA).—Harking back to the 1947 MBA and
equipment in the Agreed Locations in the Philippines for a period longer than its clear and certain terms, what comes out boldly is that the 1998 VFA is not an
envisioned in the 1998 VFA, it would not have added obligations regarding agreement that covers “activities” in the way that the 1947 MBA did; it is
the storage of their equipment and materiel. The more permanent nature of the simply an agreement regulating the status of and the treatment to be
EDCA, in contrast to the 1998 VFA, indicates a change in the tenor of the accorded to U.S. armed forces personnel and their aircraft and vehicles
agreement in the EDCA, one that does not merely implement the 1998 VFA.
while visiting the Philippines. The agreement itself does not authorize U.S. 1998 Visiting Forces Agreement (VFA). Rather, it is a continuation of the 1998
troops to permanently stay in the Philippines nor authorize any activity related VFA under new dimensions. These dimensions should not and cannot be hidden
behind reaffirmations of existing 1998 VFA obligations.—While the EDCA affirms
the treatment of U.S. personnel and U.S. forces in the Philippines, it at the same
286 time introduces the Philippines’ obligation to recognize the authority
286 SUPREME COURT REPORTS ANNOTATED of U.S. Forces in the “Agreed Locations.” Under the EDCA, U.S. forces can
now preposition and store defense equipment, supplies, and materiel at Agreed
Saguisag vs. Ochoa, Jr. Locations. They shall have unimpeded access to Agreed Locations for all matters
relating to the prepositioning and storage of defense equipment, supplies, and
to the establishment and the operation of bases, as these activities had been materiel. Lastly, the EDCA authorizes the U.S. forces to exercise all rights
defined under the 1947 MBA. and authorities within the Agreed Locations that are necessary for their
Enhanced Defense Cooperation Agreement; Agreed Locations; View that operational control or defense. In contrast, the 1998 VFA only refers to the tax and
within the Agreed Locations, the United States (U.S.) can also preposition defense duty-free entry of U.S. Government equipment in connection with the activities
equipment, supplies, and materiel over which the U.S. forces shall have exclusive during their visit. In the same manner, and despite being in a different class as
use and control. Clearly, the right to deploy weapons can be undertaken even if it is U.S. personnel and U.S. forces, U.S. contractors are also allowed “unimpeded
not in the pursuit of joint activities for common security interests.—A reading of the access” to the Agreed Locations when it comes to all matters relating to
EDCA’s provisions shows that the rights and privileges granted to the U.S. do not the prepositioning and storage of defense equipment, supplies and
always carry a concomitant right on the part of the Philippines nor do they involve materiel.Thus, these groups of people (U.S. personnel, U.S. forces and U.S.
joint exercises. While the EDCA mentions that the Agreed Locations may be used contractors) have been referred to in the EDCA not merely to implement the 1998
for “security cooperation exercises” and “joint and combined training activities,” VFA, but to further their roles in the Agreed Locations that the EDCA authorizes.
the provisions of the EDCA also provide for the conduct of other activities beyond From these perspectives, the EDCA cannot be considered to be a simple
the 1951 MDT and the 1998 VFA. Within the Agreed Locations, the U.S. may implementation of the 1998 VFA. Rather, it is a continuation of the 1998 VFA
conduct trainings for its troops, transit, support and related activities. The EDCA under new dimensions. These dimensions should not and cannot be hidden behind
also allows the U.S. to use the Agreed Locations to refuel aircraft, bunker vessels, reaffirmations of existing 1998 VFA obligations. These added dimensions reinforce
temporarily maintain vehicles, vessels and aircraft. Significantly, it does not the idea of military bases, as it allows them access to the Agreed Locations that,
provide for any qualification on the purpose for the entry of these vessels, vehicles, as I had earlier mentioned, is the cornerstone of the EDCA. From the legal end, the
and aircraft into Philippine jurisdiction. The EDCA also permits the temporary obligations under the EDCA, not its policy declarations and
accommodation of personnel, again without any qualification as to the purpose of characterization, should be decisive in determining whether Section 25, Article
their visit. The U.S. forces may also engage in communications activities including XVIII applies.
the use of its own radio spectrum, similarly without any limitation as to the Same; View that the Enhanced Defense Cooperation Agreement (EDCA) is
purpose by which such communications shall be carried out. Further, within the thus a new agreement that touches on military bases, troops, and facilities beyond
Agreed Locations, the U.S. can also preposition defense equipment, supplies, and the scope of the 1951 Mutual Defense Treaty (MDT) and the 1998 Visiting Forces
materiel over which the U.S. forces shall have exclusive use and control. Clearly, Agreement (VFA), and should be covered by a treaty pursuant to Article XVIII,
the right to deploy weapons can be undertaken even if it is not in the pursuit of joint Section 25 and Article VII, Section 21, both of the 1987 Constitution.—The EDCA
activities for common security interests. These rights, granted to the U.S. under the is
EDCA, do not contain an element of mutuality in the sense that mutuality is
reflected in the 1951 MDT and the 1998 VFA. As these rights go beyond the earlier
treaties and are, in fact, independent sources of rights and obligations between the 288
U.S. and the Philippines, they cannot be mere details of implementation of both
288 SUPREME COURT REPORTS ANNOTATED
the 1951 MDT and the 1998 VFA.
Same; View that the Enhanced Defense Cooperation Agreement (EDCA) Saguisag vs. Ochoa, Jr.
cannot be considered to be a simple implementation of the
thus a new agreement that touches on military bases, troops, and facilities
beyond the scope of the 1951 MDT and the 1998 VFA, and should be covered by a
287 treaty pursuant to Article XVIII, Section 25 and Article VII, Section 21, both of the
VOL. 779, JANUARY 12, 2016 287 1987 Constitution. Without the referral and concurrence by the Senate, the EDCA
is constitutionally deficient and, hence, cannot be enforced in our country.
Saguisag vs. Ochoa, Jr.
LEONEN, J., Dissenting Opinion:
Foreign Policy; Security Alliance; Defense Cooperation; View that foreign and scope of the presence of foreign military bases, troops, or facilities, thereby
policy indeed includes security alliances and defense cooperation among states.— justifying a distinction between their initial entry and subsequent activities. Its
Foreign policy indeed includes security alliances and defense cooperation among very structure shows that Article XVIII, Section 25 is not a mere gateway for the
states. In the conduct of negotiations and in the implementation of any valid and entry of foreign troops or facilities into the Philippines for them to carry out any
binding international agreement, Article II of the Constitution requires: Section 2. activity later on. The provision contains measures designed to protect our country
The Philippines renounces war as an instrument of national policy, adopts the in the broader scheme of international relations. Military presence shapes both
generally accepted principles of international law as part of the law of the land and foreign policy and political relations. War — or the threat thereof through the
adheres to the policy of peace, equality, justice, freedom, cooperation, and amity position of troops, basing, and provision of military facilities — is an extension of
with all nations. . . . . Section 7. The State shall pursue an independent foreign politic, thus: The use of military force is a means to a higher end — the political
policy. In its relations with other states the paramount consideration shall be object. War is a tool that policy uses to achieve its objectives and, as such, has a
national sovereignty, territorial integrity, national interest, and the right to self- measure of rational utility. So, the purpose for which the use of force is intended
determination. will be the major determinant of the course and character of a war. As Clausewitz
Presidency; Judicial Review; View that the power of the President as explains, war “is controlled by its political object,” which will set its course,
Commander-in-Chief and head of state is limited by the sovereign through prescribe the scale of means and effort which is required, and makes its influence
judicially determinable constitutional parameters.—Generally, the President’s felt throughout down to the smallest operational detail.
discretion is plenary in matters falling within executive functions. He is the chief Enhanced Defense Cooperation Agreement; View that the Enhanced Defense
executive, having the power of control over all executive departments, bureaus, Cooperation Agreement (EDCA) was negotiated in
and offices. Further, “by constitutional fiat and by the intrinsic nature of his office,
the President, as head of State, is the sole organ and authority in the external
affairs of the country [and] [i]n many ways, the President is the chief architect of 290
the nation’s foreign policy.” The President is also the Commander-in-Chief of all 290 SUPREME COURT REPORTS ANNOTATED
armed forces of the Philippines. He has the power to “call out such armed forces to
prevent or suppress lawless violence, invasion or rebellion . . . suspend the Saguisag vs. Ochoa, Jr.
privilege of the writ of habeas corpus or place the Philippines or any part thereof
under martial law” subject to the conditions and requisites under the provision. private between representatives of the President and the United States
However, the President’s discretion to allow our participation in the use of force — (U.S.).—The EDCA was negotiated in private between representatives of the
whether by commit- President and the United States. The complete text of the negotiations was
presented to the public in time for the visit of the President of the United States.
During its presentation, the President’s representatives took the position that no
289 further public discussion would be held that might affect the terms of the EDCA.
The President presented the EDCA as a final product withdrawn from Senate or
VOL. 779, JANUARY 12, 2016 289
Congressional input. The President curtailed even the possibility of full public
Saguisag vs. Ochoa, Jr. participation through a Congressional Resolution calling for a referendum on this
matter.
ting our own military assets and personnel or by allowing our territory to be Same; Foreign Policy; View that by allowing the entry of United States (U.S.)
used as waypoints, refueling or staging areas — is also constrained by the military personnel, their deployment into undefined missions here and abroad, and
Constitution. In this sense, the power of the President as Commander-in-Chief and their use of military assets staged from our territory against their present and future
head of state is limited by the sovereign through judicially determinable enemies based on a general provision in the Visiting Forces Agreement (VFA), the
constitutional parameters. majority now undermines the measures built into our present Constitution to allow
Foreign Military Bases; International Agreements; View that the prohibition the Senate, Congress and our People to participate in the shaping of foreign
in Article XVIII, Section 25 relates only to international agreements involving policy.—By allowing the entry of United States military personnel, their
foreign military bases, troops, or facilities.—The prohibition in Article XVIII, deployment into undefined missions here and abroad, and their use of military
Section 25 relates only to international agreements involving foreign military assets staged from our territory against their present and future enemies based on
bases, troops, or facilities. It does not prohibit the President from entering into a general provision in the VFA, the majority now undermines the measures built
other types of agreements that relate to other aspects of his powers as Commander- into our present Constitution to allow the Senate, Congress and our People to
in-Chief. participate in the shaping of foreign policy. The EDCA may be an agreement that
Same; View that there is nothing in Article XVIII, Section 25 that defines the “deepens defense cooperation” between the Philippines and the United States.
extent and scope of the presence of foreign military bases, troops, or facilities, However, like the 1947 Military Bases Agreement, it is the agreement more than
thereby justifying a distinction between their initial entry and subsequent any other that will extensively shape our foreign policy.
activities.—There is nothing in Article XVIII, Section 25 that defines the extent
Constitutional Law; Treaties; Executive Agreements; View that whether an
international agreement is to be regarded as a treaty or as an executive agreement
depends on the subject matter covered by and the temporal nature of the 292
agreement.—That the President may enter into international agreements as chief 292 SUPREME COURT REPORTS ANNOTATED
architect of the Philippines’ foreign policy has long been acknowledged. However,
whether an international agreement is to be regarded as a treaty or as an executive Saguisag vs. Ochoa, Jr.
agreement depends on the subject matter covered by and the temporal nature of
the agreement. Commissioner of Customs v. Eastern Sea Trading, 3 SCRA 351 Agreement in 1991. We could reasonably assume that those who drafted and
(1961), differentiated interna- ratified the 1987 Constitution were aware of this legal situation and of the broad
terms of the 1951 treaty yet did not expressly mention the 1951 Mutual Defense
Treaty in Article XVIII, Section 25. We can conclude, with sturdy and unassailable
291 logic, that the 1951 treaty is not the treaty contemplated in Article XVIII, Section
25.
VOL. 779, JANUARY 12, 2016 291
Treaties; Rebus Sic Stantibus; View that Article 62 of the Vienna Convention
Saguisag vs. Ochoa, Jr. on the Law of Treaties provides for the principle of “rebus sic stantibus,” in that a
fundamental change of circumstances may be a ground to terminate or withdraw
tional agreements that require Senate concurrence from those that do not: from a treaty.—Parenthetically, Article 62 of the Vienna Convention on the Law of
International agreements involving political issues or changes of national policy Treaties provides for the principle of “rebus sic stantibus,” in that a fundamental
and those involving international arrangements of a permanent character usually change of circumstances may be a ground to terminate or withdraw from a treaty.
take the form of treaties. But international agreements embodying adjustments of Dean Merlin M. Magallona is of the view that there has been a fundamental
detail carrying out well-established national policies and traditions and those change in circumstances that allows the Philippines to terminate the 1951 Mutual
involving arrangements of a more or less temporary nature usually take the form Defense Treaty. Although we should acknowledge this suggestion during the oral
of executive agreements. arguments by petitioners, we do not need to go into such an issue and at this time
Foreign Military Bases; View that the prohibition against “foreign military to be able to resolve the controversies in this case. We await a case that will provide
bases, troops, or facilities,” unless covered by treaty or allowed through a a clearer factual backdrop properly pleaded by the parties. In addition, the Mutual
referendum, becomes effective “after the expiration in 1991 of the Agreement . . . Defense Treaty is not the treaty contemplated by Article XVIII, Section 25 on
concerning Military Bases.”—While these provisions in the 1951 Mutual Defense account of its subject matter. In paragraph 5 of its Preamble, the Mutual Defense
Treaty could reasonably be interpreted to include activities done jointly by the Treaty articulates the parties’ desire “to strengthen their present efforts to
Philippines and the United States, nothing in International Law nor in the collective defense for the preservation of peace and security pending the
Constitution can be reasonably read as referring to this treaty for the development of a more comprehensive system of regional security in the Pacific
authorization for “foreign military bases, troops, or facilities” after the ratification Area.”
of the 1987 Constitution. Again, the constitutional provision reads: Section Same; Visiting Forces Agreement; View that the Visiting Forces Agreement
25. After the expiration in 1991 of the Agreement between the Republic of (VFA) cannot also be said to be the treaty required in Article XVIII, Section 25. This
the Philippines and the United States of America concerning Military is because the United States (U.S.), as the other contracting party, has never treated
Bases, foreign military bases, troops or facilities shall not be allowed in the it as such under its own domestic laws.—The VFA cannot also be said to be the
Philippines except under a treaty duly concurred in by the Senate and, when the treaty required in Article XVIII, Section 25. This is because the United States, as
Congress so requires, ratified by a majority of the votes cast by the people in a the other contracting party, has never treated it as such under its own domestic
national referendum held for that purpose, and recognized as a treaty by the other laws. The VFA has the same status as that of the 1947 Military Bases Agreement
contracting State. (Emphasis supplied) There is a time stamp to the obligation in that it is merely an executive agreement on the part of United States: As
under this provision. The prohibition against “foreign military bases, troops, or articulated by Constitutional Commissioner Blas F. Ople in the 1986
facilities,” unless covered by treaty or allowed through a referendum, becomes Constitutional Com-
effective “after the expiration in 1991 of the Agreement . . . concerning Military
Bases.” The treaty about to expire refers to the 1947 Military Bases Agreement as
amended. This was still in effect at the time of the drafting, submission, and 293
ratification of the 1987 Constitution. VOL. 779, JANUARY 12, 2016 293
Constitutional Law; Foreign Military Bases; Mutual Defense Treaty; View
that the 1951 treaty is not the treaty contemplated in Article XVIII, Section 25.— Saguisag vs. Ochoa, Jr.
The 1951 Mutual Defense Treaty was in effect at the time of the ratification of the
Constitution in 1987. It was also in effect even after the expiration of the Military mission deliberations on this provision, the 1947 RP-US Military Bases
Bases Agreement was ratified by the Philippine Senate, but not by the United
States Senate. In the eyes of Philippine law, therefore, the Military Bases areas in the Southeast Asian region. This can be in the form of landing rights given
Agreement was a treaty, but by the laws of the United States, it was a mere to their fighter jets and stealth bombers or way stations for SEALS or other special
executive agreement. This asymmetry in the legal treatment of the Military units entering foreign territory in states not officially at war with the
Bases Agreement by the two countries was believed to be a slur to our Philippines. Two: Parts of Philippine territory may be used to supplement overt
sovereignty. communication systems of the United States forces. For instance, cyberwarfare
Same; View that this court’s interpretation of a treaty under Article XVIII, targeting a state hostile to the United States can be launched from any of the
Section 25 in BAYAN (Bagong Alyansang Makabayan) v. Zamora, 342 SCRA 449 Agreed Locations to pursue their interests even if this will not augur well to
(2000), which did away with the requirement that the agreement be recognized as a Philippine foreign policy. Three: Parts of Philippine territory may be used to plan,
treaty by the other contracting party, has resulted in an absurd situation of political deploy, and supply covert operations done by United States contractors such as
asymmetry between the United States (U.S.) and the Philippines.—This court’s Blackwater and other mercenary groups that have been used by the United States
interpretation of a treaty under Article XVIII, Section 25 in BAYAN (Bagong in other parts of the world. The EDCA covers these types of operations within and
Alyansang Makabayan) v. Zamora, 342 SCRA 449 (2000), which did away with the outside Philippine territory. Again, the consequences to Philippine foreign policy
requirement that the agreement be recognized as a treaty by the other contracting in cases where targets are found in neighboring countries would be immeasurable.
party, has resulted in an absurd situation of political asymmetry between the The Visiting Forces Agreement does not cover these sample activities. Nor does it
United States and the Philippines. A relationship where both parties are on equal cover United States contractors.
footing must be demanded, and from one state to another. The Philippine Same; Same; Agreed Locations; View that blanket authority over Agreed
government must be firm in requiring that the United States establish stability in Locations is granted under Article VI, Section 3 of the Enhanced Defense
its international commitment, both by legislation and jurisprudence. The doctrine Cooperation Agreement (EDCA). The United States (U.S.) forces are given a broad
laid down in Bayan, insofar as the VFA is concerned, should now be revisited in range of powers with regard to the Agreed Locations that are “necessary for their
light of new circumstances and challenges in foreign policy and international operational control or defense.”—Blanket authority over Agreed Locations is
relations. granted under Article VI, Section 3 of the EDCA. The United States forces are
Executive Agreements; Enhanced Defense Cooperation Agreement; View that given a broad range of powers with regard to the Agreed Loca-
an executive agreement merely provides for the detailed adjustments of national
policies or principles already existing in other treaties, statutes, or the Constitution;
The Enhanced Defense Cooperation Agreement (EDCA) contains significant and 295
material obligations not contemplated by the Visiting Forces Agreement (VFA). As VOL. 779, JANUARY 12, 2016 295
an executive agreement, it cannot be given any legal effect.—As discussed earlier,
an executive agreement merely provides for the detailed adjustments of national Saguisag vs. Ochoa, Jr.
policies or principles already existing in other treaties, statutes, or the
Constitution. It involves only the enforcement of clear and specific provisions of tions that are “necessary for their operational control or defense.” This
the Constitution, law, or treaty. It cannot amend nor invalidate an existing statute, authority extends to the protection of United States forces and contractors. In
treaty, or provision in the Constitution. It includes agreements that addition, the United States is merely obligated to coordinate with Philippine
authorities the measures they will take in case they deem it necessary to take
action.
294 Same; Same; Same; Visiting Forces Agreement; View that what is clear is that
the Agreed Locations become a platform for the United States (U.S.) to execute its
294 SUPREME COURT REPORTS ANNOTATED
new military strategy and strengthen its presence in the Asia-Pacific, which is
Saguisag vs. Ochoa, Jr. clearly outside the coverage of the Visiting Forces Agreement (VFA).—If, indeed, the
goal is only to enhance mutual defense capabilities under the Mutual Defense
are of a temporary nature. This is not the case with the EDCA. The EDCA Treaty through conduct of joint military exercises authorized by the VFA, then it
contains significant and material obligations not contemplated by the VFA. As an behooves this court to ask the purpose of providing control and authority over
executive agreement, it cannot be given any legal effect. Agreed Locations here in the Philippines when it is outside the coverage of both
Enhanced Defense Cooperation Agreement; Foreign Military Bases; View that the Mutual Defense Treaty and the VFA. Through a vague reference to the VFA,
United States (U.S.) military forces will not only be allowed to “visit” Philippine respondents fail to establish how the EDCA merely implements the VFA. They
territory to do a transient military training exercise with their Philippine cannot claim that the provisions of the EDCA merely make use of the authority
counterparts.—United States military forces will not only be allowed to “visit” previously granted under the VFA. What is clear is that the Agreed Locations
Philippine territory to do a transient military training exercise with their become a platform for the United States to execute its new military strategy and
Philippine counterparts. They are also allowed to execute, among others, the strengthen its presence in the Asia-Pacific, which is clearly outside the coverage of
following scenarios: One: Parts of Philippine territory may be used as staging areas the VFA.
for special or regular United States military personnel for intervention in conflict
Same; Same; Same; View that the Enhanced Defense Cooperation Agreement same status as that of municipal law, may be modified either by another statute
(EDCA) authorizes the use of Philippine territory as bases of operations. Although or by the Constitution itself. Treaties such as the VFA cannot be amended by an
not as permanent as those set up pursuant to the 1947 Military Bases Agreement executive agreement.
(MBA), they are still foreign military bases within the contemplation of Article Remedial Law; Civil Procedure; Judgments; View that ruling on abstract
XVIII, Section 25 of the Constitution; Ownership of the Agreed Locations under the cases presents the danger of foreclosing litigation between
EDCA is a diluted concept, with the Philippine government devoid of any authority
to set the parameters for what may and may not be conducted within the confines
of these areas.—The EDCA authorizes the use of Philippine territory as bases of 297
operations. Although not as permanent as those set up pursuant to the 1947 VOL. 779, JANUARY 12, 2016 297
Military Bases Agreement, they are still foreign military bases within the
contemplation of Article XVIII, Section 25 of the Constitution. The development Saguisag vs. Ochoa, Jr.
and use of these Agreed Locations are clearly within the discretion of the United
States. The retention of ownership by the Philippines under Article V(1) of the real parties, and rendering advisory opinions presents the danger of a court
EDCA does not temper the wide latitude accorded to the other contracting party. that substitutes its own imagination and predicts facts, acts, or events that may or
may not happen.—It is not this court’s duty to “rule on abstract and speculative
issues barren of actual facts.” Ruling on abstract cases presents the danger of
296 foreclosing litigation between real parties, and rendering advisory opinions
presents the danger of a court that substitutes its own imagination and predicts
296 SUPREME COURT REPORTS ANNOTATED
facts, acts, or events that may or may not happen. Facts based on judicial proof
Saguisag vs. Ochoa, Jr. must frame the court’s discretion, as “[r]igor in determining whether controversies
brought before us are justiciable avoids the counter majoritarian difficulties
At best, the United States’ only obligation is to consult and coordinate with attributed to the judiciary.” Abstract cases include those where another political
our government. Under the EDCA, the consent of the Philippine government does department has yet to act. In other words, a case not ripe for adjudication is not
not extend to the operations and activities to be conducted by the United States yet a concrete case.
forces and contractors. Operational control remains solely with the United States Political Law; Political Question Doctrine; View that in Diocese of Bacolod v.
government. The agreement did not create a distinction between domestic and COMELEC, 747 SCRA 1 (2015), the Supreme Court (SC) held that the political
international operations. Ownership of the Agreed Locations under the EDCA is a question doctrine never precludes this court’s exercise of its power of judicial review
diluted concept, with the Philippine government devoid of any authority to set the when the act of a constitutional body infringes upon a fundamental individual or
parameters for what may and may not be conducted within the confines of these collective right.—In Diocese of Bacolod v. COMELEC, 747 SCRA 1 (2015), this
areas. court held that the political question doctrine never precludes this court’s exercise
Same; Same; View that the Enhanced Defense Cooperation Agreement of its power of judicial review when the act of a constitutional body infringes upon
(EDCA) has an open-ended duration. Despite having an initial term of ten (10) a fundamental individual or collective right. However, this will only be true if there
years, Article XII(4) specifically provides for the automatic continuation of the is no other constitutional body to whom the discretion to make inquiry is
agreement’s effectivity until a party communicates its intent to terminate.—The preliminarily granted by the sovereign. Ruling on the challenge to the content of
EDCA has an open-ended duration. Despite having an initial term of 10 years, the EDCA will preclude and interfere with any future action on the part of the
Article XII(4) specifically provides for the automatic continuation of the Senate as it inquires into and deliberates as to whether it should give its
agreement’s effectivity until a party communicates its intent to terminate. The concurrence to the agreement or whether it should advise the President to reopen
purpose of the Agreed Locations is also open-ended. At best, its definition and negotiations to amend some of its provisions. It is the Senate, through Article VII,
description of rights provide that the areas shall be for the use of United States Section 21 in relation to Article XVIII, Section 25, that was given the discretion to
forces and contractors. However, short of referring to Agreed Locations as bases, make this initial inquiry exclusive of all other constitutional bodies, including this
the EDCA enumerates activities that tend to be military in nature, such as court. A policy of deference and respect for the allocation of such power by the
bunkering of vessels, pre-positioning of equipment, supplies, and materiel, and sovereign to a legislative chamber requires that we refrain from making clear and
deploying forces and materiel. The United States is also allowed to undertake the categorical rulings on the constitutional challenges to the content of the EDCA.
construction of permanent facilities, as well as to use utilities and its own Constitutional Law; Enhanced Defense Cooperation Agreement; View that
telecommunications systems. while the Enhanced Defense Cooperation Agreement (EDCA) is a formal and official
Same; Visiting Forces Agreement; Treaties; View that the Enhanced Defense memorial of the results of negotia-
Cooperation Agreement (EDCA) amends the Visiting Forces Agreement (VFA).
Since the VFA is a treaty, the EDCA cannot be implemented; Treaties such as the
VFA cannot be amended by an executive agreement.—The EDCA amends the VFA. 298
Since the VFA is a treaty, the EDCA cannot be implemented. Treaties, being of the
298 SUPREME COURT REPORTS ANNOTATED SPECIAL CIVIL ACTIONS in the Supreme Court. Certiorari.
The facts are stated in the opinion of the Court.
Saguisag vs. Ochoa, Jr. Harry L. Roque, Jr., Gilbert Teruel Andres, Romel R. Bagares, Ethel C.
Avisado and Evalyn Ursua for petitioners in G.R. No. 212426.
tions between the Philippines and the United States (U.S.), it is not yet Rachel F. Pastores, Amylyn B. Sato, Francis Anthony P. Principe, Sandra Jill
effective until the Senate concurs or there is compliance with Congressional action S. Santos, Carlos A. Montemayor, Maria Kristina C. Conti and Maneeka Asistol
to submit the agreement to a national referendum in accordance with Article XVIII, Sarza for petitioners in G.R. No. 212444.
Section 25 of the Constitution.—The judiciary has the duty to ensure that the acts Remigio D. Saladero, Jr., Noel V. Neri and Vicente Jaime M. Topacio for
of all branches of government comply with the fundamental nature of the petitioners-in-intervention Kilusang Mayo Uno, et al.
Constitution. While the EDCA is a formal and official memorial of the results of Rene A.V. Saguisag petitioner in G.R. No. 212426 and for petitioner-
negotiations between the Philippines and the United States, it is not yet effective intervenor Rene A.Q. Saguisag, Jr.
until the Senate concurs or there is compliance with Congressional action to SERENO, CJ.:
submit the agreement to a national referendum in accordance with Article XVIII,
Section 25 of the Constitution. The petitions1 before this Court question the constitutionality of the Enhanced
Same; View that by abbreviating the constitutional process, the Supreme Defense Cooperation Agreement (EDCA) between the Republic of the Philippines
Court (SC) makes itself vulnerable to a reasonable impression that we do not have and the United States of America (U.S.). Petitioners allege that respondents
the courage to enforce every word, phrase, and punctuation in the Constitution committed grave abuse of discretion amounting to lack or excess of jurisdiction
promulgated by our People.—By abbreviating the constitutional process, this court when they entered into EDCA with the U.S.,2 claiming that the instrument
makes itself vulnerable to a reasonable impression that we do not have the courage violated multiple consti-
to enforce every word, phrase, and punctuation in the Constitution promulgated _______________
by our People. We will stand weak, as an institution and by implication as a state,
in the community of nations. In clear unequivocal words, the basic instrument 1 Petition of Saguisag, et al., Rollo (G.R. No. 212426, Vol. I),
through which we exist requires that we interpret its words to make real an pp. 3-66; Petition of Bayan, et al., Rollo (G.R. No. 212444, Vol. I), pp. 3-101.
independent foreign policy. It requires measures be fully publicly discussed before 2 Petition of Saguisag, et al., p. 5, Rollo (G.R. No. 212426, Vol. I), p. 7; Petition
any foreign resource capable of making war with our neighbors and at the of Bayan, et al., p. 5, Rollo (G.R. No. 212444, Vol. I), p. 7.
command of a foreign sovereign — foreign military bases, troops and facilities —
becomes effective.
Mutual Defense Treaty; Visiting Forces Agreement; View that the 1951 300
Mutual Defense Treaty (MDT) and the Visiting Forces Agreement (VFA) was in
effect when the Chinese invaded certain features within our Exclusive Economic 300 SUPREME COURT REPORTS ANNOTATED
Zone (EEZ) in the West Philippine Sea. The Americans did not come to our aid; The Saguisag vs. Ochoa, Jr.
commitment of the United States (U.S.) remains ambiguous. The U.S.’ statement is
that it will not interfere in those types of differences we have with China, among tutional provisions.3 In reply, respondents argue that petitioners lack standing
others.—The 1951 Mutual Defense Treaty and the Visiting Forces Agreement was to bring the suit. To support the legality of their actions, respondents invoke the
in effect when the Chinese invaded certain features within our Exclusive Economic 1987 Constitution, treaties, and judicial precedents.4
Zone in the West Philippine Sea. The Americans did not come to our aid. The A proper analysis of the issues requires this Court to lay down at the outset
President of the United States visited and, on the occasion of that visit, our own the basic parameters of the constitutional powers and roles of the President and
President announced the completion of the EDCA. No clear, the Senate in respect of the above issues. A more detailed discussion of these
powers and roles will be made in the latter portions.
I. Broad Constitutional Context of the Powers of the President: Defense,
299 Foreign Relations, and EDCA
VOL. 779, JANUARY 12, 2016 299
A. The Prime Duty of the
Saguisag vs. Ochoa, Jr. State and the Consolida-
tion of Executive Power
unequivocal, and binding commitment was given with respect to the in the President
applicability of the Mutual Defense Treaty to the entirety of our valid legal claims
in the West Philippine Sea. The commitment of the United States remains Mataimtim kong pinanunumpaan (o pinatotohanan) na tutuparin ko nang
ambiguous. The United States’ statement is that it will not interfere in those types buong katapatan at sigasig ang aking mga tungkulin bilang Pangulo (o
of differences we have with China, among others.
Pangalawang Pangulo o Nanunungkulang Pangulo) ng Pilipinas, Philippine archipelago, including all the islands and waters embraced therein and
pangangalagaan at ipagtatanggol ang kanyang Konstitusyon, ipatutupad ang mga all other territories over which it has sovereignty or jurisdiction. These territories
batas nito, magiging makatarungan sa bawat tao, at itatalaga ang aking sarili sa consist of its terrestrial, fluvial, and aerial domains; including its territorial sea,
paglilingkod sa Bansa. Kasihan nawa aka ng Diyos. the seabed, the subsoil, the insular shelves, and other submarine areas; and the
– Panunumpa sa Katungkulan ng Pangulo ng Pilipinas ayon sa Saligang waters around, between,
Batas5 _______________
_______________
history-and-symbolism-of-the-presidential-inauguration> (last visited 5 Nov.
3 Principally the following provisions under the Constitution: Art. VII, Sec. 2015).
21; Art. XVIII, Sec. 25; Art. I; Art. II, Secs. 2, 7, & 8; Art. VI, Sec. 28(4); and Art. 6 Constitution, Art. VII, Sec. 1.
VIII, Sec. 1. See Petition of Saguisag, et al., pp. 23-59, Rollo (G.R. No. 212426, Vol. 7 Id., Art. II, Sec. 4.
I), pp. 25-61; Petition of Bayan, et al., Rollo pp. 23-93 (G.R. No. 212444, Vol. I), pp.
25-95.
4 Memorandum of the OSG, pp. 8-38, Rollo (G.R. No. 212426, Vol. I), pp. 438- 302
468. 302 SUPREME COURT REPORTS ANNOTATED
5 The Protocol, Ceremony, History, and Symbolism of the Presidential
Inauguration, The Presidential Museum and Library, available at Saguisag vs. Ochoa, Jr.
<http://malacanang.gov.ph/1608-the-protocol-ceremony-
and connecting the islands of the archipelago, regardless of their breadth and
dimensions.8
301
To carry out this important duty, the President is equipped with authority over
VOL. 779, JANUARY 12, 2016 301
the Armed Forces of the Philippines (AFP),9 which is the protector of the people
Saguisag vs. Ochoa, Jr. and the state. The AFP’s role is to secure the sovereignty of the State and the
integrity of the national territory.10 In addition, the Executive is constitutionally
The 1987 Constitution has “vested the executive power in the President of the empowered to maintain peace and order; protect life, liberty, and property; and
Republic of the Philippines.”6 While the vastness of the executive power that has promote the general welfare.11 In recognition of these powers, Congress has
been consolidated in the person of the President cannot be expressed fully in one specified that the President must oversee, ensure, and reinforce our defensive
provision, the Constitution has stated the prime duty of the government, of which capabilities against external and internal threats12 and, in the same vein, ensure
the President is the head: that the country is adequately prepared for all national and local emergencies
arising from natural and man-made disasters.13
The prime duty of the Government is to serve and protect the To be sure, this power is limited by the Constitution itself. To illustrate, the
people. The Government may call upon the people to defend the State and, President may call out the AFP to prevent or suppress instances of lawless
in the fulfillment thereof, all citizens may be required, under conditions violence, invasion or rebellion,14 but not suspend the privilege of the writ of habeas
provided by law, to render personal military or civil service.7 (Emphases corpus for a period exceeding 60 days, or place the Philippines or any part thereof
supplied) under martial law exceeding that same span. In the exercise of these powers, the
President is also duty-bound to submit a report to Congress, in person or in
B. The duty to protect the writing, within 48 hours from the proclamation of martial law or the suspension of
territory and the citizens the privilege of the writ of habeas corpus; and
of the Philippines, the _______________
power to call upon the
people to defend the 8 Id., Art. I.
State, and the President 9 Id., Art. II, Sec. 3.
as Commander-in-Chief 10 Id.
11 Id., Art. II, Sec. 5.
The duty to protect the State and its people must be carried out earnestly and 12 Id., Art. VII, Sec. 18, in relation to Art. II, Secs. 3, 4 & 7; Executive Order
effectively throughout the whole territory of the Philippines in accordance with the No. 292 (Administrative Code of 1987), Book IV (Executive Branch), Title VIII
constitutional provision on national territory. Hence, the President of the (National Defense), Secs. 1, 15, 26 & 33 [hereinafter Administrative Code of 1987].
Philippines, as the sole repository of executive power, is the guardian of the
13 Administrative Code of 1987, Book IV (Executive Branch), Title XII (Local 304 SUPREME COURT REPORTS ANNOTATED
Government), Sec. 3(5).
14 Constitution, Art. VII, Sec. 18. Saguisag vs. Ochoa, Jr.

our diplomatic and consular officials regularly brief him on meaningful


303 events all over the world. He has also unlimited access to ultra-sensitive
military intelligence data. In fine, the presidential role in foreign affairs is
VOL. 779, JANUARY 12, 2016 303
dominant and the President is traditionally accorded a wider degree of
Saguisag vs. Ochoa, Jr. discretion in the conduct of foreign affairs. The regularity, nay, validity of
his actions are adjudged under less stringent standards, lest their judicial
Congress may in turn revoke the proclamation or suspension. The same repudiation lead to breach of an international obligation, rupture of state
provision provides for the Supreme Court’s review of the factual basis for the relations, forfeiture of confidence, national embarrassment and a plethora
proclamation or suspension, as well as the promulgation of the decision within 30 of other problems with equally undesirable consequences.17
days from filing.
The role of the President in foreign affairs is qualified by the Constitution in
C. The power and duty to that the Chief Executive must give paramount importance to the sovereignty of
conduct foreign relations the nation, the integrity of its territory, its interest, and the right of the sovereign
Filipino people to self-determination.18 In specific provisions, the President’s
The President also carries the mandate of being the sole organ in the conduct power is also limited, or at least shared, as in Section 2 of Article II on the conduct
of foreign relations.15 Since every state has the capacity to interact with and of war; Sections 20 and 21 of Article VII on foreign loans, treaties, and
engage in relations with other sovereign states,16it is but logical that every state international agreements; Sections 4(2) and 5(2)(a) of Article VIII on the judicial
must vest in an agent the authority to represent its interests to those other review of executive acts; Sections 4 and 25 of Article XVIII on treaties and
sovereign states. international agreements entered into prior to the Constitution and on the
presence of foreign military troops, bases, or facilities.
The conduct of foreign relations is full of complexities and
consequences, sometimes with life and death significance to the nation D. The relationship between
especially in times of war. It can only be entrusted to that department of the two major presiden-
government which can act on the basis of the best available information tial functions and the
and can decide with decisiveness. x x x It is also the President who role of the Senate
possesses the most comprehensive and the most confidential information _______________
about foreign countries for
_______________ 17 Vinuya v. Romulo, 633 Phil. 538, 570; 619 SCRA 533, 561 (2010) (quoting
the Dissenting Opinion of then Assoc. Justice Reynato S. Puno in Secretary of
15 Id., Art. VII, Sec. 1, in relation to Administrative Code of 1987, Book IV Justice v. Lantion, 379 Phil. 165, 233-234; 322 SCRA 160, 221-222 [2000]).
(Executive Branch), Title I (Foreign Affairs), Secs. 3(1) and 20; Akbayan Citizens 18 Constitution, Art. II, Sec. 7.
Action Party (“AKBAYAN”) v. Aquino, 580 Phil. 422; 558 SCRA 468
(2008); Pimentel, Jr. v. Office of the Executive Secretary, 501 Phil. 303; 462 SCRA
622 (2005); People’s Movement for Press Freedom v. Manglapus, G.R. No. 84642, 305
13 September 1988 (unreported) (citing United States v. Curtiss-Wright Export VOL. 779, JANUARY 12, 2016 305
Corp.,299 U.S. 304 [1936]); Bernas, Joaquin, Foreign Relations in Constitutional
Law, p. 101 (1995); Cortes, Irene R., the Philippine Presidency: A Study of Saguisag vs. Ochoa, Jr.
Executive Power, p. 187 (1966); Sinco, Vicente G., Philippine Political Law:
Principles and Concepts, p. 297 (10th ed., 1954). Clearly, the power to defend the State and to act as its representative in the
16 See 1933 Montevideo Convention on the Rights and Duties of States, Art. international sphere inheres in the person of the President. This power, however,
1, 165 LNTS 19; Crawford, James, The Creation of States in International Law, does not crystallize into absolute discretion to craft whatever instrument the Chief
p. 61 (2nd ed. 2007). Executive so desires. As previously mentioned, the Senate has a role in ensuring
that treaties or international agreements the President enters into, as
contemplated in Section 21 of Article VII of the Constitution, obtain the approval
304 of two-thirds of its members.
Previously, treaties under the 1973 Constitution required ratification by a When it became clear that the American forces intended to impose colonial
majority of the Batasang Pambansa,19 except in instances wherein the President control over the Philippine Islands, General Emilio Aguinaldo immediately led the
“may enter into international treaties or agreements as the national welfare and Filipinos into an all-out war against the U.S.27 The Filipinos were ultimately
interest may require.”20This left a large margin of discretion that the President defeated in the Philippine-American War, which lasted until 1902 and
could use to bypass the Legislature altogether. This was a departure from the 1935 _______________
Constitution, which explicitly gave the President the power to enter into treaties
only with the concurrence of two-thirds of all the Members of the Senate.21 The pensable toward our nation’s pursuit of political maturity and
1987 Constitution returned the Senate’s power22 and, with it, the legislative’s growth.” Bayan(Bagong Alyansang Makabayan) v. Zamora, 396 Phil. 623; 342
traditional role in foreign affairs.23 SCRA 449 (2000).
_______________ 24 Foreign Service Institute, Agreements on United States Military Facilities
in Philippine Military Bases, 1947-1985, p. ix (Pacifico A. Castro, revised ed.,
19 Id. (1973, as amended), Art. VIII, Sec. 14(1). 1985).
20 Id. (1973, as amended), Art. VIII, Sec. 16. 25 Treaty of Peace Between the United States of America and the Kingdom of
21 Id. (1935), Art. VII, Sec. 10(7). Spain, 10 Dec. 1898, 30 US Stat. 1754, T.S. No. 343 (1898) (entered into force 11
22 Id., Art. VII, Sec. 21. Apr. 1899).
23 Quoth the Court: “For the role of the Senate in relation to treaties is 26 Foreign Service Institute, supra.
essentially legislative in character; the Senate, as an independent body possessed 27 Id.
of its own erudite mind, has the prerogative to either accept or reject the proposed
agreement, and whatever action it takes in the exercise of its wide latitude of
discretion, pertains to the wisdom rather than the legality of the act. In this sense, 307
the Senate partakes a principal, yet delicate, role in keeping the principles of VOL. 779, JANUARY 12, 2016 307
separation of powers and of checks and balances alive and vigilantly ensures that
these cherished rudiments remain true to their form in a democratic government Saguisag vs. Ochoa, Jr.
such as ours. The Constitution thus animates, through this treaty-concurring
power of the Senate, a healthy system of checks and balances indis- led to the downfall of the first Philippine Republic.28 The Americans henceforth
began to strengthen their foothold in the country.29 They took over and expanded
the former Spanish Naval Base in Subic Bay, Zambales, and put up a cavalry post
306 called Fort Stotsenberg in Pampanga, now known as Clark Air Base.30
When talks of the eventual independence of the Philippine Islands gained
306 SUPREME COURT REPORTS ANNOTATED
ground, the U.S. manifested the desire to maintain military bases and armed
Saguisag vs. Ochoa, Jr. forces in the country.31 The U.S. Congress later enacted the Hare-Hawes-Cutting
Act of 1933, which required that the proposed constitution of an independent
The responsibility of the President when it comes to treaties and international Philippines recognize the right of the U.S. to maintain the latter’s armed forces
agreements under the present Constitution is therefore shared with the Senate. and military bases.32 The Philippine Legislature rejected that law, as it also gave
This shared role, petitioners claim, is bypassed by EDCA. the U.S. the power to unilaterally designate any part of Philippine territory as a
permanent military or naval base of the U.S. within two years from complete
II. Historical Antecedents of EDCA independence.33
A. U.S. takeover of Spanish _______________
colonization and its mili-
tary bases, and the tran- 28 Id.
sition to Philippine inde- 29 Id.
pendence 30 Id.
31 Id.; Simbulan, Roland G., The Bases of our Insecurity: A Study of the US
The presence of the U.S. military forces in the country can be traced to their Military Bases in the Philippines, p. 13 (2nd ed. 1985).
pivotal victory in the 1898 Battle of Manila Bay during the Spanish-American 32 Hare-Hawes-Cutting Act, ch. 11, Sec. 2(1), 47 US Stat. 761 (1933).
War.24 Spain relinquished its sovereignty over the Philippine Islands in favor of According to the law: “Sec. 2. The constitution formulated and drafted shall
the U.S. upon its formal surrender a few months later. 25 By 1899, the Americans be republican in form, shall contain a bill of rights, and shall, either as a part
had consolidated a military administration in the archipelago.26 thereof or in an ordinance appended thereto, contain provisions to the
effect that, pending the final and complete withdrawal of the
sovereignty of the United States over the Philippine Islands — (1) The Philippine reservations and fueling stations as are reservedunder Section 5) x x x.”
Islands recognizes the right of the United States x x x to maintain military See Foreign Service Institute, id.
and other reservations and armed forces in the Philippinesx x x.”
33 Hare-Hawes-Cutting Act, Secs. 5 & 10. According to the law: “Sec. 5. All
the property and rights which may have been acquired in the Philippine 309
Islands by the United States under the treaties mentioned in the first section 309 SUPREME COURT REPORTS ANNOTATED
of this Act, except such land or other property as has heretofore been
designated by Saguisag vs. Ochoa, Jr.

thorized the U.S. President to enter into negotiations for the adjustment and
308 settlement of all questions relating to naval reservations and fueling stations
within two years after the Philippines would have gained independence.35 Under
308 SUPREME COURT REPORTS ANNOTATED
the Tydings-McDuffie Act, the U.S. President would proclaim the American
Saguisag vs. Ochoa, Jr. withdrawal and surrender of sovereignty over the islands 10 years after the
inauguration of the new government in the Philippines.36This law eventually led
The U.S. Legislature subsequently crafted another law called the Tydings- to the promulgation of the 1935 Philippine Constitution.
McDuffie Act or the Philippine Independence Act of 1934. Compared to the old The original plan to surrender the military bases changed. 37 At the height of
Hare-Hawes-Cutting Act, the new law provided for the surrender to the the Second World War, the Philippine and the U.S. Legislatures each passed
Commonwealth Government of “all military and other reservations” of the U.S. resolutions authorizing their respective Presidents to negotiate the matter of
government in the Philippines, except “naval reservations and refueling retaining military bases in the country after the planned withdrawal of the
stations.”34 Furthermore, the law au- U.S.38 Subsequently, in 1946, the countries entered into the Treaty of General
_______________ Relations, in which the U.S. relinquished all control and sovereignty over the
Philippine Islands, except the areas that would be covered by the American
the President of the United States for military and other reservations military bases in the country.39 This treaty eventually led
of the Government of the United States x x x are hereby granted to the
government of the Commonwealth of the Philippine Islands when constituted. _______________
x x x x.” “Sec. 10. On the 4th day of July, immediately following the expiration
of a period of ten years from the date of the inauguration of the new 35 Philippine Independence Act, Secs. 5 & 10; Foreign Service Institute, id.
government under the constitution provided for in this Act, the President of
the United States shall by proclamation withdraw and surrender all right 36 Philippine Independence Act, Sec. 10.
of possession, supervision, jurisdiction, control, or sovereignty then existing and 37 Foreign Service Institute, supra note 24 at p. ix; Simbulan, supra note 31 at
exercised by the United States in and over the territory and people of the pp. 13-14.
Philippine Islands, including all military and other reservations of the 38 See Agreement Between the Republic of the Philippines and the United
Government of the United States in the Philippines (except such land or States of America Concerning Military Bases, preamble, 14 Mar. 1947, 43 UNTS
property reserved under Section 5 as may be redesignated by the 271 (entered into force 26 Mar. 1947) [hereinafter 1947 Military Bases
President of the United States not later than two years after the date of such Agreement]; Foreign Service Institute, id.
proclamation).” See Foreign Service Institute, supra note 24; Simbulan, supra note 39 Treaty of General Relations between the Republic of the Philippines and
31. the United States of America, Art. I, 4 Jul. 1946, 7 UNTS 3 (1946) (entered into
34 Philippine Independence Act, US Pub. L. No. 73-127, Secs. 5 & 10, 48 US force 22 Oct. 1946) [hereinafter 1946 Treaty of General Relations]. According to
Stat. 456 (1934) [hereinafter Philippine Independence Act]. According to the law: the treaty: “The United States of America agrees to withdraw and surrender,
“SEC. 10. (a) On the 4th day of July immediately following the expiration and does hereby withdraw and surrender, all rights of possession,
of a period of ten years from the date of the inauguration of the new supervision, jurisdiction, control or sovereignty existing and exercised by the
governmentunder the constitution provided for in this Act the President of the United States of America in and over the territory and the
United States shall by proclamation withdraw and surrender all right of
possession, supervision, jurisdiction, control, or sovereignty then existing and
exercised by the United States in and over the territory and people of the 310
Philippine Islands, including all military and other reservations of the 310 SUPREME COURT REPORTS ANNOTATED
Government of the United States in the Philippines (except such naval
Saguisag vs. Ochoa, Jr.
to the creation of the post-colonial legal regime on which would hinge the hectares.47 Twelve years later, the U.S. returned Sangley Point in Cavite City
continued presence of U.S. military forces until 1991: the Military Bases through an exchange of notes.48 Then, through the Romulo-Murphy Exchange of
Agreement (MBA) of 1947, the Military Assistance Agreement of 1947, and the Notes of 1979, the parties agreed to the recognition of Philippine sovereignty over
Mutual Defense Treaty (MDT) of 1951.40 Clark and Subic Bases and the reduction of the areas that could be used by the
U.S. military.49 The agreement also provided for the mandatory review of the
B. Former legal regime on treaty every five years.50 In 1983, the parties revised the 1947 MBA through the
the presence of U.S. armed Romualdez-Armacost Agreement.51 The revision pertained to the operational use
forces in the territory of of the military bases by the U.S. government within the context of Philippine
an independent Philip- sovereignty,52 including the need for prior consultation with the Philippine
pines (1946-1991) government on the former’s use of the bases for military combat operations or the
establishment of long-range missiles.53
Soon after the Philippines was granted independence, the two countries _______________
entered into their first military arrangement pursuant to the Treaty of General
Relations — the 1947 MBA.41 The Senate concurred on the premise of “mutuality 44 1947 Military Bases Agreement, Art. 1(3); Foreign Service Institute, id., at
of security interest,”42 which provided for the presence and operation of 23 U.S. p. xii; Simbulan, id., at pp. 78-79.
military bases in the Philippines for 99 years or until the year 2046. 43 The treaty 45 Foreign Service Institute, id., at pp. xii-xv.
also obliged the Philippines to negotiate with the U.S. to allow the latter to 46 Id., at p. xiii.
_______________ 47 Id., at p. xii.
48 Id., at p. xiii.
people of the Philippine Islands, except the use of such bases, necessary 49 Id.
appurtenances to such bases, and the rights incident thereto, as the United 50 Id.
States of America, by agreement with the Republic of the Philippines may 51 Id., at pp. xiii-xiv.
deem necessary to retain for the mutual protection of the Republic of the 52 Id.
Philippines and of the United States of America. x x x.” The Philippine Senate 53 Id.
concurred in this treaty (S. Res. 11, 1st Cong. [1946]). See also: Nicolas v.
Romulo, 598 Phil. 262; 578 SCRA 438 (2009).
40 Foreign Service Institute, supra note 24 at pp. x-xi; Bayan (Bagong 312
Alyansang Makabayan) v. Zamora, supra note 23. 312 SUPREME COURT REPORTS ANNOTATED
41 1947 Military Bases Agreement.
42 S. Res. 29, 1st Cong. (1946); Philippine instrument of ratification was Saguisag vs. Ochoa, Jr.
signed by the President on 21 Jan. 1948 and the treaty entered into force on 26
Mar. 1947; Nicolas v. Romulo, supra note 39. Pursuant to the legislative authorization granted under Republic Act No.
43 Foreign Service Institute, supra note 24 at pp. x-xi; Simbulan, supra note 9,54 the President also entered into the 1947 Military Assistance Agreement55 with
31 at pp. 76-79. the U.S. This executive agreement established the conditions under which U.S.
military assistance would be granted to the Philippines,56 particularly the
provision of military arms, ammunitions, supplies, equipment, vessels, services,
311 and training for the latter’s defense forces.57 An exchange of notes in 1953 made it
clear that the agreement would remain in force until terminated by any of the
VOL. 779, JANUARY 12, 2016 311
parties.58
Saguisag vs. Ochoa, Jr. _______________

expand the existing bases or to acquire new ones as military necessity might 54 Republic Act No. 9 — Authority of President to Enter into Agreement with
require.44 US under Republic of the Phil. Military Assistance Act (1946). According to Section
A number of significant amendments to the 1947 MBA were made. 45 With 1 thereof: “The President of the Philippines is hereby authorized to enter into
respect to its duration, the parties entered into the Ramos-Rusk Agreement of agreement or agreements with the President of the United States, or with any
1966, which reduced the term of the treaty from 99 years to a total of 44 years or of the agencies or instrumentalities of the Government of the United
until 1991.46Concerning the number of U.S. military bases in the country, the States, regarding military assistance to the armed forces of the
Bohlen-Serrano Memorandum of Agreement provided for the return to the Republic of the Philippines, in the form of transfer of property and
Philippines of 17 U.S. military bases covering a total area of 117,075 information, giving of technical advice and lending of personnel to
instruct and train them, pursuant to the provisions of United States Public Act 64 Padua, Colonel Paterno C., Republic of the Philippines United States
Numbered Four hundred and fifty-four, commonly called the ‘Republic of the Defense Cooperation: Opportunities and Challenges, A Filipino Perspective, p. 6
Philippines Military Assistance Act,’ under the terms and conditions (2010).
provided in this Act.”
55 Agreement Between the Government of the Republic of the Philippines and
the Government of the United States of America on Military Assistance to the 314
Philippines, 45 UNTS 47 (entered into force 21 Mar. 1947) [hereinafter 1947 314 SUPREME COURT REPORTS ANNOTATED
Military Assistance Agreement].
56 Foreign Service Institute, supra note 24 at p. xi; Simbulan, supra note 31, Saguisag vs. Ochoa, Jr.
at pp. 79-89.
57 1947 Military Assistance Agreement, Sec. 6. C. Current legal regime on
58 Exchange of Notes Constituting an Agreement Extending the Agreement the presence of U.S. armed
Between the Government of the Republic of the Philippines and the Government forces in the country
of the United States of America on Military Assistance to the Philippines, 26 Jun.
1953, 213 UNTS 77 (entered into force 5 Jul. 1953) reproduced in Foreign Service In view of the impending expiration of the 1947 MBA in 1991, the Philippines
Institute, supra note 24 at pp. 197-203. See Mutual Logistics Support Agree- and the U.S. negotiated for a possible renewal of their defense and security
relationship.65 Termed as the Treaty of Friendship, Cooperation and Security, the
countries sought to recast their military ties by providing a new framework for
313 their defense cooperation and the use of Philippine installations.66 One of the
To further strengthen their defense and security relationship,59 the proposed provisions included an arrangement in which U.S. forces would be
Philippines and the U.S. next entered into the MDT in 1951. Concurred in by both granted the use of certain installations within the Philippine naval base in
the Philippine60 and the U.S.61 Senates, the treaty has two main features: first, it Subic.67 On 16 September 1991, the Senate rejected the proposed treaty.68
allowed for mutual assistance in maintaining and developing their individual and The consequent expiration of the 194 7 MBA and the resulting paucity of any
collective capacities to resist an armed attack;62 and second, it provided for their formal agreement dealing with the treatment of U.S. personnel in the Philippines
mutual self-defense in the event of an armed attack against the territory of either led to the suspension in 1995 of large-scale joint military exercises.69 In the
party.63 The treaty was premised on their recognition that an armed attack on meantime, the respective governments of the two countries
either of them would equally be a threat to the security of the other.64 _______________
_______________
65 Bayan (Bagong Alyansang Makabayan) v. Zamora, supra note 23; People’s
ment (21 Nov. 2007). See generally: People v. Nazareno, 612 Phil. 753; 595 Movement for Press Freedom v. Manglapus, supra note 15.
SCRA 438 (2009) (on the continued effectivity of the agreement). 66 See Treaty of Friendship, Cooperation and Security Between the
59 See Mutual Defense Treaty between the Republic of the Philippines and Government of the Republic of the Philippines and the Government of the United
the United States of America, 30 Aug. 1951, 177 UNTS 133 (entered into force 27 States of America, 27 Aug. 1991 (rejected by the Senate on 16 Sept. 1991).
Aug. 1952) [hereinafter 1951 MDT]. According to its preamble: “The Parties to this 67 Id., Art. VII; Supplementary Agreement Two to the Treaty of Friendship,
Treaty x x x Desiring further to strengthen their present efforts to collective Cooperation and Security, Arts. I & II(9).
defense for the preservation of peace and security pending the development of a 68 Bayan (Bagong Alyansang Makabayan) v. Zamora, supra note 23.
more comprehensive system of regional security in the Pacific Area x x x hereby 69 Id.; Joint Report of the Committee on Foreign Relations and the Committee
agreed as follows[.]” See: Bayan (Bagong Alyansang Makabayan) v. on National Defense and Security reproduced in Senate of the Philippines, The
Zamora, supra note 23. Visiting Forces Agreement: The Senate Decision, p. 206 (1999); Lim v. Executive
60 S. Res. 84, 2nd Cong. (1952); Foreign Service Institute, supra note 24 at pp. Secretary,430 Phil. 555; 380 SCRA 739 (2002).
193-194; The Philippine instrument of ratification was signed by the President on
27 August 1952 and it entered into force on the same date upon the exchange of
ratification between the Parties (Philippines and U.S.), and was proclaimed by the 315
President by virtue of Proc. No. 341, S. 1952. VOL. 779, JANUARY 12, 2016 315
61 Nicolas v. Romulo, supra note 39 (citing U.S. Congressional Record,
Saguisag vs. Ochoa, Jr.
82ndCongress, Second Session, Vol. 98, Part 2, pp. 2594-2595).
62 1951 MDT, Art. II.
63 Id., Arts. IV-V. agreed70 to hold joint exercises at a substantially reduced level. 71The military
arrangements between them were revived in 1999 when they concluded the first
Visiting Forces Agreement (VFA).72
As a “reaffirm[ation] [of the] obligations under the MDT,” 73 the VFA has laid In the same year, the Philippines and the U.S. entered into the Mutual
down the regulatory mechanism for the treatment of U.S. military and civilian Logistics Support Agreement to “further the interoperability, readiness, and
personnel visiting the country.74 It contains provisions on the entry and departure effectiveness of their respective military forces”80 in accordance with the MDT, the
of U.S. personnel; the purpose, extent, and limitations of their activities; criminal Military
and disciplinary jurisdiction; the waiver of _______________
_______________
75 VFA I; Lim v. Executive Secretary, supra note 69.
70 Agreement regarding the status of U.S. military and civilian personnel, 76 Agreement between the Government of the United States of America and
Exchange of notes between the DFA and the U.S. Embassy in Manila on Apr. 2, the Government of the Republic of the Philippines Regarding the Treatment of
and June 11 and 21, 1993, Hein’s No. KAV 3594 (entered into force 21 June 1993) Republic of the Philippines Personnel Visiting the United States of America, Phil.-
[hereinafter Status of Forces Agreement of 1993]. The agreement was extended on U.S., 9 Oct. 1998, TIAS No. 12931 [hereinafter VFA II].
19 September 1994; on 28 April 1995 (See Hein’s No. KAV 4245); and 8 December 77 Senate Resolution No. 18, 27 May 1999 reproduced in Senate of the
1995 (See Hein’s No. KAV 4493). See also Mason, R. Chuck, Status of Forces Philippines, supra note 69 at pp. 185-190; Bayan (Bagong Alyansang Makabayan)
Agreement (Sofa): What Is It, and How Has It Been Utilized?, p. 14 (2012). v. Zamora, supra note 23.
71 Joint Report of the Committee on Foreign Relations and the Committee on 78 Lim v. Executive Secretary, supra note 69.
National Defense and Security reproduced in Senate of the Philippines, supra note 79 Id.
69; Lim v. Executive Secretary, supra note 69; Bayan (Bagong Alyansang 80 Mutual Logistics Support Agreement Between the Department of Defense
Makabayan) v. Zamora, supra note 23. of the United States of America and the Department of National Defense of the
72 Agreement Between the Government of the Republic of the Philippines and Republic of the Philippines, Preamble, 21 Nov. 2002 [hereinafter 2002 MLSA].
the Government of the United States of America Regarding the Treatment of According to the preamble thereof, the parties “have resolved to conclude” the
United States Armed Forces Visiting the Philippines, Phil.-U.S., 10 Feb. 1998, agreement in light of their “desir[e] to further the interoperability, readiness, and
TIAS No. 12931 (entered into force 1 Jun. 1999) [hereinafter VFA I], reproduced effectiveness of their respective military forces through increased logistic
in Senate of the Philippines, id., at pp. 257-266 (1999); Lim v. Executive cooperation in accordance with the RP-US Mutual Defense
Secretary, id.
73 VFA I, preamble. See: Lim v. Executive Secretary, id. In Lim, we explained
that “It is the VFA which gives continued relevance to the MDT despite the 317
passage of years. Its primary goal is to facilitate the promotion of optimal VOL. 779, JANUARY 12, 2016 317
cooperation between American and Philippine military forces in the event of an
attack by a common foe.” Saguisag vs. Ochoa, Jr.
74 Bayan (Bagong Alyansang Makabayan) v. Zamora, supra note 23 at p. 637;
p. 469. Assistance Agreement of 1953, and the VFA.81 The new agreement outlined
the basic terms, conditions, and procedures for facilitating the reciprocal provision
of logistics support, supplies, and services between the military forces of the two
316 countries.82 The phrase “logistics support and services” includes billeting,
operations support, construction and use of temporary structures, and storage
316 SUPREME COURT REPORTS ANNOTATED
services during an approved activity under the existing military
Saguisag vs. Ochoa, Jr. arrangements.83 Already extended twice, the agreement will last until 2017.84

certain claims; the importation and exportation of equipment, materials, D. The Enhanced Defense
supplies, and other pieces of property owned by the U.S. government; and the Cooperation Agreement
movement of U.S. military vehicles, vessels, and aircraft into and within the
country.75 The Philippines and the U.S. also entered into a second counterpart EDCA authorizes the U.S. military forces to have access to and conduct
agreement (VFA II), which in turn regulated the treatment of Philippine military activities within certain “Agreed Locations” in the country. It was not transmitted
and civilian personnel visiting the U.S.76 The Philippine Senate concurred in the to the Senate on the executive’s understanding that to do so was no longer neces-
first VFA on 27 May 1999.77 _______________
Beginning in January 2002, U.S. military and civilian personnel started
arriving in Mindanao to take part in joint military exercises with their Filipino Treaty, RP-US Visiting Forces Agreement or the RP-US Military Assistance
counterparts.78 Called Balikatan, these exercises involved trainings aimed at Agreement.” Consequently, Article II of the agreement provides that: “[it] shall be
simulating joint military maneuvers pursuant to the MDT.79 implemented, applied and interpreted by the Parties in accordance with the
provisions of the Mutual Defense Treaty, the Visiting Forces Agreement or the 88 EDCA; Memorandum of OSG, p. 3, Rollo (G.R. No. 212426, Vol. I), p. 433.
Military Assistance Agreement and their respective constitutions, national laws 89 Instrument of Ratification, Annex of A of the Memorandum of OSG, Rollo,
and regulations.” p. 476.
81 2002 MLSA, Preamble. 90 Oral Arguments, TSN, 25 November 2014, pp. 119-120.
82 Id., Art. I.
83 Id., Art. IV(1)(a); Padua, supra note 64 at pp. 1-2.
84 See Mutual Logistics Support Agreement Between the Department of 319
Defense of the United States of America and the Department of National Defense VOL. 779, JANUARY 12, 2016 319
of the Republic of the Philippines, Art. IX, 8 Nov. 2007 (applied provisionally on 8
Nov. 2007; entered into force 14 Jan. 2009) [hereinafter 2007 MLSA]; Extension of Saguisag vs. Ochoa, Jr.
the Mutual Logistics Support Agreement (RP-US-01) Between the Department of
Defense of the United States of America and the Department of National Defense Two petitions for certiorari were thereafter filed before us assailing the
of the Republic of the Philippines (entered into force 6 Nov. 2012). constitutionality of EDCA. They primarily argue that it should have been in the
form of a treaty concurred in by the Senate, not an executive agreement.
On 10 November 2015, months after the oral arguments were concluded and
318 the parties ordered to file their respective memoranda, the Senators adopted
Senate Resolution No. (SR) 105.91The resolution expresses the “strong sense”92 of
318 SUPREME COURT REPORTS ANNOTATED
the Senators that for EDCA to become valid and effective, it must first be
Saguisag vs. Ochoa, Jr. transmitted to the Senate for deliberation and concurrence.

sary.85 Accordingly, in June 2014, the Department of Foreign Affairs (DFA) III. Issues
and the U.S. Embassy exchanged diplomatic notes confirming the completion
of all necessary internal requirements for the agreement to enter into force in the Petitioners mainly seek a declaration that the Executive Department
two countries.86 committed grave abuse of discretion in entering into EDCA in the form of an
According to the Philippine government, the conclusion of EDCA was the result executive agreement. For this reason, we cull the issues before us:
of intensive and comprehensive negotiations in the course of almost two
years.87 After eight rounds of negotiations, the Secretary of National Defense and A. Whether the essential requisites for judicial review are present.
the U.S. Ambassador to the Philippines signed the agreement on 28 April B. Whether the President may enter into an executive agreement on
2014.88 President Benigno S. Aquino III ratified EDCA on 6 June 2014.89 The OSG foreign military bases, troops, or facilities.
clarified during the oral arguments90 that the Philippine and the U.S. C. Whether the provisions under EDCA are consistent with the
governments had yet to agree formally on the specific sites of the Agreed Locations Constitution, as well as with existing laws and treaties.
mentioned in the agreement.
_______________ _______________

85 Memorandum of the OSG, pp. 8, 24 Rollo (G.R. No. 212426, Vol. I), pp. 438, 91 Rollo (G.R. No. 212444), pp. 865-867.
454. 92 According to the Resolution: “Be it further resolved that this resolution
86 See Note No. 1082 of the U.S. Embassy to the DFA dated 25 June 2014, expressing the strong sense of the Senate be formally submitted to the Supreme
Annex B of the Memorandum of the OSG, Rollo (G.R. No. 212426, Vol. I), p. 477; Court through the Chief Justice.” Id., at p. 867.
Memorandum of the OSG, p. 8, Rollo (G.R. No. 212426, Vol. I), p. 438.
87 Statement of Secretary Albert F. del Rosario on the signing of the PH-U.S.
EDCA, Department of Foreign Affairs (28 Apr. 2014) available at 320
<https://www.dfa.gov.ph/index.php/newsroom/dfa-releases/2694-statement-of- 320 SUPREME COURT REPORTS ANNOTATED
secretary-albert-f-del-rosario-on-the-signing-of-the-philippines-us-enhanced--
defense-cooperation-agreement> (last visited 5 Nov. 2015); Frequently Asked Saguisag vs. Ochoa, Jr.
Questions (FAQ) on the Enhanced Defense Cooperation Agreement, Department of
Foreign Affairs (28 Apr. 2014) available at IV. Discussion
<https://www.dfa.gov.ph/index.php/newsroom/dfa-releases/2693-frequently-
asked-questions-faqs-on-the-enhanced-defense-cooperation-agreement> (last A. Whether the essential
visited 5 Nov. 2015). requisites for judicial
review have been satisfied
with the Supreme Court as the final arbiter, effectively checks the other
Petitioners are hailing this Court’s power of judicial review in order to strike departments in the exercise of its power to determine the law, and
down EDCA for violating the Constitution. They stress that our fundamental law hence to declare executive and legislative acts void if violative of
is explicit in prohibiting the presence of foreign military forces in the country, the Constitution.
except under a treaty concurred in by the Senate. Before this Court may begin to
analyze the constitutionality or validity of an official act of a coequal branch of xxxx
government, however, petitioners must show that they have satisfied all the
essential requisites for judicial review.93 As any human production, our Constitution is of course lacking
Distinguished from the general notion of judicial power, the power of judicial perfection and perfectibility, but as much as it was within the power of our
review specially refers to both the authority and the duty of this Court to determine people, acting through
whether a branch or an instrumentality of government has acted beyond the scope _______________
of the latter’s constitutional powers.94 As articulated in Section 1, Article VIII of
the Constitution, the power of judicial review involves the power to resolve cases 95 The Constitution provides: “SECTION 1. The judicial power shall be
in which the questions concern the constitutionality or validity of any treaty, vested in one Supreme Court and in such lower courts as may be established by
international or executive agreement, law, presidential decree, proclamation, law. Judicial power includes the duty of the courts of justice to settle actual
order, instruction, ordinance, or regula- controversies involving rights which are legally demandable and enforceable, and
_______________ to determine whether or not there has been a grave abuse of discretion
amounting to lack or excess of jurisdiction on the part of any branch or
93 Francisco, Jr. v. Nagmamalasakit na mga Manananggol ng mga instrumentality of the Government.”
Manggagawang Pilipino, Inc., 460 Phil. 830, 914; 415 SCRA 44, 133 (2003). 96 Angara v. Electoral Commission, 63 Phil. 139, 156-158 (1936).
94 See: Chavez v. Judicial and Bar Council, G.R. No. 202242, 17 July 2012,
676 SCRA 579; Tagolino v. House of Representatives Electoral Tribunal, G.R. No.
202202, 19 March 2013, 693 SCRA 574; Gutierrez v. House of Representatives 322
Committee on Justice, 658 Phil. 322; 643 SCRA 198 (2011); Francisco, Jr. 322 SUPREME COURT REPORTS ANNOTATED
v. Nagmamalasakit na mga Manananggol ng mga Manggagawang Pilipino,
Inc., id.; Demetria v. Alba, 232 Phil. 222; 148 SCRA 208 (1987). Saguisag vs. Ochoa, Jr.

their delegates to so provide, that instrument which is the


321 expression of their sovereignty however limited, has established a
republican government intended to operate and function as a
VOL. 779, JANUARY 12, 2016 321
harmonious whole, under a system of checks and balances, and
Saguisag vs. Ochoa, Jr. subject to specific limitations and restrictions provided in the said
instrument. The Constitution sets forth in no uncertain language
tion.95 In Angara v. Electoral Commission, this Court exhaustively discussed the restrictions and limitations upon governmental powers and
this “moderating power” as part of the system of checks and balances under the agencies. If these restrictions and limitations are transcended it
Constitution. In our fundamental law, the role of the Court is to determine whether would be inconceivable if the Constitution had not provided for a
a branch of government has adhered to the specific restrictions and limitations of mechanism by which to direct the course of government along
the latter’s power:96 constitutional channels, for then the distribution of powers would
be mere verbiage, the bill of rights mere expressions of sentiment,
The separation of powers is a fundamental principle in our system of and the principles of good government mere political apothegms.
government. It obtains not through express provision but by actual division Certainly, the limitations and restrictions embodied in our Constitution are
in our Constitution. Each department of the government has real as they should be in any living constitution. x x x. In our case, this
exclusive cognizance of matters within its jurisdiction, and is moderating power is granted, if not expressly, by clear implication from
supreme within its own sphere. But it does not follow from the fact that Section 2 of Article VIII of [the 1935] Constitution.
the three powers are to be kept separate and distinct that the Constitution The Constitution is a definition of the powers of government. Who is to
intended them to be absolutely unrestrained and independent of each determine the nature, scope and extent of such powers? The Constitution
other. The Constitution has provided for an elaborate system of itself has provided for the instrumentality of the judiciary as the rational
checks and balances to secure coordination in the workings of the way. And when the judiciary mediates to allocate constitutional
various departments of the government. x x x. And the judiciary in turn, boundaries, it does not assert any superiority over the other
departments; it does not in reality nullify or invalidate an act of Saguisag vs. Ochoa, Jr.
the legislature, but only asserts the solemn and sacred obligation
assigned to it by the Constitution to determine conflicting claims the risk of supplanting the wisdom of the constitutionally appointed actor with
of authority under the Constitution and to establish for the parties that of its own.100
in an actual controversy the rights which that instrument secures Even as we are left with no recourse but to bare our power to check an act of a
and guarantees to them. This is in truth all that is involved in what is coequal branch of government — in this case the executive — we must abide by
termed “judicial supremacy” which properly is the power of judicial the stringent requirements for the exercise of that power under the
review under the Constitution. x x x x. (Emphases supplied) Constitution. Demetria v. Alba101and Francisco, Jr. v. Nagmamalasakit na mga
Manananggol ng mga Manggagawang Pilipino, Inc.,102 cite the “pillars” of the
limitations on the power of judicial review as enunciated in the concurring opinion
323 of U.S. Supreme Court Justice Brandeis in Ashwander v. Tennessee Valley
VOL. 779, JANUARY 12, 2016 323 Authority.103 Francisco104 redressed these “pillars” under the following categories:
Saguisag vs. Ochoa, Jr.
1. That there be absolute necessity of deciding a case;
The power of judicial review has since been strengthened in the 1987 2. That rules of constitutional law shall be formulated only as
Constitution. The scope of that power has been extended to the determination of required by the facts of the case;
whether in matters traditionally considered to be within the sphere of appreciation 3. That judgment may not be sustained on some other ground;
of another branch of government, an exercise of discretion has been attended with 4. That there be actual injury sustained by the party by reason of
grave abuse.97 The expansion of this power has made the political question the operation of the statute;
doctrine “no longer the insurmountable obstacle to the exercise of judicial power 5. That the parties are not in estoppel;
or the impenetrable shield that protects executive and legislative actions from 6. That the Court upholds the presumption of constitutionality.
judicial inquiry or review.”98 (Emphases supplied)
This moderating power, however, must be exercised carefully and only if it _______________
cannot be completely avoided. We stress that our Constitution is so incisively
designed that it identifies the spheres of expertise within which the different 100 See: Francisco, Jr. v. Nagmamalasakit na mga Manananggol ng mga
branches of government shall function and the questions of policy that they shall Manggagawang Pilipino, Inc., supra note 93; United States v. Raines, 362 U.S. 17
resolve.99 Since the power of judicial review involves the delicate exercise of (1960); and Angara v. Electoral Commission, id.
examining the validity or constitutionality of an act of a coequal branch of 101 Demetria v. Alba, supra note 94 at p. 226.
government, this Court must continually exercise restraint to avoid 102 Francisco, Jr. v. Nagmamalasakit na mga Manananggol ng mga
_______________ Manggagawang Pilipino, Inc., supra at pp. 922-923; p. 161.
103 Ashwander v. Tennessee Valley Authority, 297 U.S. 288, 346-348 (1936).
97 Gutierrez v. House of Representatives Committee on Justice, supra note 104 Francisco, Jr. v. Nagmamalasakit na mga Manananggol ng mga
94; Francisco, Jr. v. Nagmamalasakit na mga Manananggol ng mga Manggagawang Pilipino, Inc., supra at p. 923; p. 161.
Manggagawang Pilipino, Inc., supra note 93; Tañada v. Angara, 338 Phil. 546; 272
SCRA 18 (1997); Oposa v. Factoran, Jr., G.R. No. 101083, 30 July 1993, 224 SCRA
792, 809-810 (citing Llamas v. Orbos, 279 Phil. 920; 202 SCRA 844 325
[1991]; Bengzon, Jr. v. Senate Blue Ribbon Committee, G.R. No. 89914, 20 VOL. 779, JANUARY 12, 2016 325
November 1991, 203 SCRA 767; Gonzales v. Macaraig, Jr., G.R. No. 87636, 19 Saguisag vs. Ochoa, Jr.
November 1990, 191 SCRA 452; Coseteng v. Mitra, Jr., G.R. No. 86649, 12 July
1990, 187 SCRA 377; Daza v. Singson, 259 Phil. 980; 180 SCRA 496 [1989]; and I These are the specific safeguards laid down by the Court when it exercises its
Record, Constitutional Commission, pp. 434-436 [1986]). power of judicial review.105 Guided by these pillars, it may invoke the power only
98 Oposa v. Factoran, Jr., id., at p. 97. when the following four stringent requirements are satisfied: (a) there is an actual
99 Morfe v. Mutuc, 130 Phil. 415, 442; 22 SCRA 424, 442 (1968); Angara v. case or controversy; (b) petitioners possess locus standi; (c) the question of
Electoral Commission, supra note 96 at p. 178. constitutionality is raised at the earliest opportunity; and (d) the issue of
constitutionality is the lis mota of the case.106 Of these four, the first two conditions
will be the focus of our discussion.
324
324 SUPREME COURT REPORTS ANNOTATED 1. Petitioners have shown
the presence of an actual than that, courts accord the presumption of constitutionality to legislative
case or controversy. enactments, not only because the legislature is
_______________
The OSG maintains107 that there is no actual case or controversy that exists,
since the Senators have not been deprived of the opportunity to invoke the 109 Southern Hemisphere Engagement Network, Inc. v. Anti-Terrorism
privileges of the institution they are representing. It contends that the Council,supra note 106 at p. 479.
nonparticipation of the Senators in the present petitions only confirms that even 110 Information Technology Foundation of the Philippines v. Commission on
they believe that EDCA is a binding executive agreement that does not require Elections, 499 Phil. 281, 304-305; 460 SCRA 291, 313 (2005) (citing Aetna Life
their concurrence. Insurance Co. v. Hayworth, 300 U.S. 227 [1937]); Southern Hemisphere
It must be emphasized that the Senate has already expressed its position Engagement Network, Inc. v. Anti-Terrorism Council, id., at p. 480; David v.
through SR 105.108 Through the Resolution, the Senate has taken a position Macapagal-Arroyo,supra note 106 at p. 753; p. 213; Francisco, Jr. v.
contrary to that of the OSG. As the body tasked to participate in foreign affairs by Nagmamalasakit na mga Manananggol ng mga Manggagawang Pilipino,
ratifying treaties, its belief that EDCA infringes upon its constitutional role Inc., supra note 93 at pp. 879-880; p. 121; Angara v. Electoral
indicates that an actual controversy — albeit brought to the Court by non- Commission, supra note 96 at p. 158.
Senators, exists. 111 Id. (citing Aetna Life Insurance Co. v. Hayworth, id.; Southern
_______________ Hemisphere Engagement Network, Inc. v. Anti-Terrorism Council, id.; Lozano v.
Nograles, 607 Phil. 334, 340; 589 SCRA 354, 358 [2009]).
105 Id., at p. 922; p. 160. 112 Angara v. Electoral Commission, supra note 96 at pp. 158-159.
106 Southern Hemisphere Engagement Network, Inc. v. Anti-Terrorism
Council, 646 Phil. 452, 471; 632 SCRA 146, 166-167 (2010); David v. Macapagal-
Arroyo, 522 Phil. 705, 753; 489 SCRA 160, 213 (2006); Francisco, Jr. v. 327
Nagmamalasakit na mga Manananggol ng mga Manggagawang Pilipino, Inc., id., VOL. 779, JANUARY 12, 2016 327
at p. 892;p. 133; Angara v. Electoral Commission, supra note 96 at p. 158.
107 Memorandum of OSG, p. 6; Rollo, p. 436. Saguisag vs. Ochoa, Jr.
108 Rollo (G.R. No. 212444), pp. 865-867.
presumed to abide by the Constitution but also because the judiciary
in the determination of actual cases and controversies must reflect
326 the wisdom and justice of the people as expressed through their
representatives in the executive and legislative departments of the
326 SUPREME COURT REPORTS ANNOTATED
government. (Emphases supplied)
Saguisag vs. Ochoa, Jr.
We find that the matter before us involves an actual case or controversy that
Moreover, we cannot consider the sheer abstention of the Senators from the is already ripe for adjudication. The Executive Department has already sent an
present proceedings as basis for finding that there is no actual case or controversy official confirmation to the U.S. Embassy that “all internal requirements of the
before us. We point out that the focus of this requirement is the ripeness for Philippines x x x have already been complied with.”113 By this exchange of
adjudication of the matter at hand, as opposed to its being merely conjectural or diplomatic notes, the Executive Department effectively performed the last act
anticipatory.109 The case must involve a definite and concrete issue involving real required under Article XII(1) of EDCA before the agreement entered into force.
parties with conflicting legal rights and legal claims admitting of specific relief Section 25, Article XVIII of the Constitution, is clear that the presence of foreign
through a decree conclusive in nature.110It should not equate with a mere request military forces in the country shall only be allowed by virtue of a treaty concurred
for an opinion or advice on what the law would be upon an abstract, hypothetical, in by the Senate. Hence, the performance of an official act by the Executive
or contingent state of facts.111 As explained in Angara v. Electoral Commission:112 Department that led to the entry into force of an executive agreement was
sufficient to satisfy the actual case or controversy requirement.
[The] power of judicial review is limited to actual cases and
controversies to be exercised after full opportunity of argument by 2. While petitioners Saguisag
the parties, and limited further to the constitutional question raised or et al., do not have legal
the very lis mota presented. Any attempt at abstraction could only standing, they nonetheless
lead to dialectics and barren legal questions and to sterile raise issues involving mat-
conclusions of wisdom, justice or expediency of legislation. More ters of transcendental im-
portance.
The question of locus standi or legal standing focuses on the determination of 329
whether those assailing the governmental act have the right of appearance to bring VOL. 779, JANUARY 12, 2016 329
the matter to the
_______________ Saguisag vs. Ochoa, Jr.

113 Memorandum of OSG, supra note 80. See also Note No. 1082, supra note the concrete adverseness which sharpens the presentation of issues upon
86. which the court so largely depends for illumination of difficult constitutional
questions.”119

328 The present petitions cannot


qualify as citizens’, taxpayers’,
328 SUPREME COURT REPORTS ANNOTATED or legislators’ suits; the Senate
Saguisag vs. Ochoa, Jr. as a body has the requisite
standing, but considering that it
court for adjudication.114 They must show that they have a personal and has not formally filed a plead-
substantial interest in the case, such that they have sustained or are in immediate ing to join the suit, as it merely
danger of sustaining, some direct injury as a consequence of the enforcement of the conveyed to the Supreme Court
challenged governmental act.115 Here, “interest” in the question involved must be its sense that EDCA needs the
material — an interest that is in issue and will be affected by the official act — as Senate’s concurrence to be valid,
distinguished from being merely incidental or general. 116 Clearly, it would be petitioners continue to suffer
insufficient to show that the law or any governmental act is invalid, and that from lack of standing.
petitioners stand to suffer in some indefinite way.117 They must show that they
have a particular interest in bringing the suit, and that they have been or are about In assailing the constitutionality of a governmental act, petitioners suing as
to be denied some right or privilege to which they are lawfully entitled, or that they citizens may dodge the requirement of having to establish a direct and personal
are about to be subjected to some burden or penalty by reason of the act complained interest if they show that the act affects a public right.120 In arguing that they have
of.118 The reason why those who challenge the validity of a law or an international legal standing, they claim121 that the case they have filed is a concerned citizen’s
agreement are required to allege the existence of a personal stake in the outcome suit. But aside from general statements that the petitions involve the protection of
of the controversy is “to assure a public right, and that their constitutional rights as citizens would be violated,
_______________ _______________

114 Almario v. Executive Secretary, G.R. No. 189028, 16 July 2013, 701 SCRA 120 Id., at pp. 266-267; p. 256; Akbayan Citizens Action Party (“AKBAYAN”)
269, 302; Bayan Muna v. Romulo, 656 Phil. 246; 641 SCRA 244 (2011). v. Aquino, supra note 15; Francisco, Jr. v. Nagmamalasakit na mga Manananggol
115 Funa v. Duque III, G.R. No. 191672, 25 November 2014, 742 SCRA ng mga Manggagawang Pilipino, Inc., id.; Tañada v. Tuvera, 220 Phil. 422; 136
166; Almario v. Executive Secretary, id.; Bayan Muna v. Romulo, id., at p. 265; p. SCRA 27 (1985).
255; Bayan (Bagong Alyansang Makabayan) v. Zamora, supra note 23; Francisco 121 Petition of Saguisag, et al., p. 20, Rollo (G.R. No. 212426, Vol. I), p. 22;
v. Nagmamalasakit na mga Manananggol ng mga Manggagawang Pilipino, Memorandum of Saguisag, et al., p. 15, Rollo (G.R. No. 212426, Vol. II), p. 985;
Inc., supranote 93 at pp. 895-896; p. 136. Petition of Bayan, et al., p. 9, Rollo (G.R. No. 212444, Vol. I), p. 11; Memorandum
116 Bayan Muna v. Romulo, id., at p. 265; p. 254; Pimentel, Jr. v. Office of the of Bayan, et al., pp. 19, 23, Rollo (G.R. No. 212444, Vol. I), pp. 583, 587.
Executive Secretary, supra note 15; Joya v. Presidential Commission on Good
Government, G.R. No. 96541, 24 August 1993, 225 SCRA 568.
117 Funa v. Duque III, supra; Almario v. Executive Secretary, supra note 114 330
at p. 302; Southern Hemisphere Engagement Network, Inc. v. Anti-Terrorism 330 SUPREME COURT REPORTS ANNOTATED
Council, supra note 106 at p. 472; p. 167; Francisco v. Nagmamalasakit na mga
Manananggol ng mga Manggagawang Pilipino, Inc., supra note 93 at pp. 895-896; Saguisag vs. Ochoa, Jr.
p. 136.
118 Southern Hemisphere Engagement Network, Inc. v. Anti-Terrorism they fail to make any specific assertion of a particular public right that would
Council,id. be violated by the enforcement of EDCA. For their failure to do so, the present
petitions cannot be considered by the Court as citizens’ suits that would
justify a disregard of the aforementioned requirements.
action. Until and unless the Legislature appropriates funds for EDCA, or
In claiming that they have legal standing as taxpayers, petitioners122 aver that unless petitioners can pinpoint a specific item in the current budget that
the implementation of EDCA would result in the unlawful use of public funds. allows expenditure under the agreement, we cannot at this time rule that
They emphasize that Article X(1) refers to an appropriation of funds; and that the there is in fact an appropriation or a disbursement of funds that would
agreement entails a waiver of the payment of taxes, fees, and rentals. During the justify the filing of a taxpayers’ suit.
oral arguments, however, they admitted that the government had not yet _______________
appropriated or actually disbursed public funds for the purpose of implementing
the agreement.123 The OSG, on the other hand, maintains that petitioners cannot 126 Id., (citing Pascual v. Secretary of Public Works, 110 Phil. 331
sue as taxpayers.124 Respondent explains that EDCA is neither meant to be a tax [1960]; Maceda v. Macaraig, Jr., G.R. No. 88291, 31 May 1991, 197 SCRA
measure, nor is it directed at the disbursement of public funds. 771; Lozada v. Commission on Elections, 205 Phil. 283; 120 SCRA 337
A taxpayer’s suit concerns a case in which the official act complained of directly [1983]; Dumlao v. COMELEC, 184 Phil. 369; 95 SCRA 392 [1980]; Gonzales v.
involves the illegal disbursement of public funds derived from taxation.125 Here, Marcos, 160 Phil. 637; 65 SCRA 624 [1975]).
those challenging the act must specifically show that they have sufficient interest 127 Id., (citing Bugnay Construction and Development Corporation v.
in preventing the illegal expenditure of public money, and that they will sustain a Laron, 257 Phil. 245; 176 SCRA 240 [1989]).
direct injury as a result of the en- 128 Lozano v. Nograles, supra note 111 at pp. 342-343; p. 361.
_______________

122 Petition of Saguisag, et al., p. 10, Rollo (G.R. No. 212426, Vol. I), p. 12; 332
Petition of Bayan, et al., pp. 9-10, Rollo (G.R. No. 212444, Vol. I), pp. 11-12; 332 SUPREME COURT REPORTS ANNOTATED
Memorandum of Bayan, et al., pp. 19, 23, Rollo (G.R. No. 212444, Vol. I), pp. 583,
587. Saguisag vs. Ochoa, Jr.
123 Oral Arguments, TSN, 18 November 2014, pp. 19-20.
124 Consolidated Comment of the OSG, p. 4, Rollo (G.R. No. 212426, Vol. I), Petitioners Bayan, et al. also claim129 that their co-petitioners who are party-
p. 241; Memorandum of OSG, p. 7, Rollo (G.R. No. 212426, Vol. I), p. 437. list representatives have the standing to challenge the act of the Executive
125 Bayan (Bagong Alyansang Makabayan) v. Zamora, supra note 23. Department, especially if it impairs the constitutional prerogatives, powers, and
privileges of their office. While they admit that there is no incumbent Senator who
has taken part in the present petition, they nonetheless assert that they also stand
331 to sustain a derivative but substantial injury as legislators. They argue that under
the Constitution, legislative power is vested in both the Senate and the House of
VOL. 779, JANUARY 12, 2016 331
Representatives; consequently, it is the entire Legislative Department that has a
Saguisag vs. Ochoa, Jr. voice in determining whether or not the presence of foreign military should be
allowed. They maintain that as members of the Legislature, they have the
forcement of the assailed act.126 Applying that principle to this case, they must requisite personality to bring a suit, especially when a constitutional issue is
establish that EDCA involves the exercise by Congress of its taxing or spending raised.
powers.127 The OSG counters130 that petitioners do not have any legal standing to file the
We agree with the OSG that the petitions cannot qualify as taxpayers’ suits. suits concerning the lack of Senate concurrence in EDCA. Respondent emphasizes
We emphasize that a taxpayers’ suit contemplates a situation in which there is that the power to concur in treaties and international agreements is an
already an appropriation or a disbursement of public funds.128 A reading of Article “institutional prerogative” granted by the Constitution to the Senate. Accordingly,
X(1) of EDCA would show that there has been neither an appropriation nor an the OSG argues that in case of an allegation of impairment of that power, the
authorization of disbursement of funds. The cited provision reads: injured party would be the Senate as an institution or any of its incumbent
members, as it is the Senate’s constitutional function that is allegedly being
All obligations under this Agreement are subject to the violated.
availability of appropriated funds authorized for these purposes. The legal standing of an institution of the Legislature or of any of its Members
(Emphases supplied) has already been recognized by this Court in a number of cases.131 What is in
question here is the alleged
This provision means that if the implementation of EDCA would require the _______________
disbursement of public funds, the money must come from appropriated funds that
are specifically authorized for this purpose. Under the agreement, before there can
even be a disbursement of public funds, there must first be a legislative
129 Petition of Bayan, et al., p. 10, Rollo (G.R. No. 212444, Vol. I), p. 12; 135 Pimentel, Jr. v. Office of the Executive Secretary, supra note
Memorandum of Bayan, et al., pp. 19-20, Rollo (G.R. No. 212444, Vol. I), pp. 583- 15; Bayan(Bagong Alyansang Makabayan) v. Zamora, supra note 23; Philippine
584. Constitution Association v. Enriquez, supra note 131.
130 Consolidated Comment of the OSG, pp. 3-4, Rollo (G.R. No. 212444, Vol.
I), pp. 240-241; Memorandum of the OSG, pp. 4-7, Rollo (G.R. No. 212444, Vol. I),
pp. 434-437. 334
131 Pimentel, Jr. v. Office of the Executive Secretary, supra note 334 SUPREME COURT REPORTS ANNOTATED
15; Bayan(Bagong Alyansang Makabayan) v. Zamora, supra note 23; Philippine
Constitution Association v. Enriquez, G.R. Nos. 113105, 113174, 113766, 113888, Saguisag vs. Ochoa, Jr.
19 August 1994, 235 SCRA 506; Gonzales v. Macaraig, supra note 97; Mabanag v.
Lopez Vito, 78 Phil. 1 (1947). which they claim infringes their prerogatives as legislators.” 136As legislators,
they must clearly show that there was a direct injury to their persons or the
institution to which they belong.137
333 As correctly argued by respondent, the power to concur in a treaty or an
international agreement is an institutional prerogative granted by the
VOL. 779, JANUARY 12, 2016 333
Constitution to the Senate, not to the entire Legislature. In Pimentel, Jr. v. Office
Saguisag vs. Ochoa, Jr. of the Executive Secretary, this Court did not recognize the standing of one of the
petitioners therein who was a member of the House of Representatives. The
impairment of the constitutional duties and powers granted to, or the petition in that case sought to compel the transmission to the Senate for
impermissible intrusion upon the domain of, the Legislature or an institution concurrence of the signed text of the Statute of the International Criminal Court.
thereof.132 In the case of suits initiated by the legislators themselves, this Court Since that petition invoked the power of the Senate to grant or withhold its
has recognized their standing to question the validity of any official action that concurrence in a treaty entered into by the Executive Department, only then
they claim infringes the prerogatives, powers, and privileges vested by the incumbent Senator Pimentel was allowed to assert that authority of the Senate of
Constitution in their office.133 As aptly explained by Justice Perfecto in Mabanag which he was a member.
v. Lopez Vito:134 Therefore, none of the initial petitioners in the present controversy has
the standing to maintain the suits as legislators.
Being members of Congress, they are even duty bound to see that Nevertheless, this Court finds that there is basis for it to review the act of the
the latter act within the bounds of the Constitution which, as Executive for the following reasons.
representatives of the people, they should uphold, unless they are to
commit a flagrant betrayal of public trust. They are representatives of the In any case, petitioners raise
sovereign people and it is their sacred duty to see to it that the issues involving matters of
fundamental law embodying the will of the sovereign people is not transcendental importance.
trampled upon. (Emphases supplied)
Petitioners138 argue that the Court may set aside procedural technicalities, as
We emphasize that in a legislators’ suit, those Members of Congress who are the present petition tackles issues that
challenging the official act have standing only to the extent that the alleged _______________
violation impinges on their right to participate in the exercise of the powers of the
institution of which they are members.135 Legislators have the standing “to 136 Pimentel, Jr. v. Office of the Executive Secretary, id.
maintain inviolate the prerogatives, powers, and privileges vested by the 137 Bayan (Bagong Alyansang Makabayan) v. Zamora, supra note 23.
Constitution in their office and are allowed to sue to question the validity of any 138 Petition of Saguisag, et al., pp. 21-22, Rollo (G.R. No. 212426, Vol. I), pp.
official action, 23-24; Memorandum of Saguisag, et al., pp. 15-17, Rollo (G.R. No. 212426, Vol. II),
_______________ pp. 985-987; Petition of Bayan, et al., p. 6, Rollo (G.R. No. 212444, Vol. I), p. 8;
Memorandum of Bayan, et al., pp. 19, 23, Rollo (G.R. No. 212444, Vol. I), pp. 583,
132 Philippine Constitution Association v. Enriquez, id. 587.
133 Pimentel, Jr. v. Office of the Executive Secretary, supra note 15; Philippine
Constitution Association v. Enriquez, id.
134 Bayan (Bagong Alyansang Makabayan) v. Zamora, supra note 23. 335
VOL. 779, JANUARY 12, 2016 335
Saguisag vs. Ochoa, Jr.
An exhaustive evaluation of the memoranda of the parties, together with the
are of transcendental importance. They point out that the matter before us is oral arguments, shows that petitioners have presented serious constitutional
about the proper exercise of the Executive Department’s power to enter into issues that provide ample justification for the Court to set aside the rule on
international agreements in relation to that of the Senate to concur in those standing. The transcendental importance of the issues presented here is rooted in
agreements. They also assert that EDCA would cause grave injustice, as well as the Constitution itself. Section 25, Article XVIII thereof, cannot be any clearer:
irreparable violation of the Constitution and of the Filipino people’s rights. there is a much stricter mechanism required before foreign military troops,
The OSG, on the other hand, insists139 that petitioners cannot raise the mere facilities, or bases may be allowed in the country. The DFA has already confirmed
fact that the present petitions involve matters of transcendental importance in to the U.S. Embassy that “all internal requirements of the Philippines x x x have
order to cure their inability to comply with the constitutional requirement of already been complied with.”142 It behooves the Court in this instance to take a
standing. Respondent bewails the overuse of “transcendental importance” as an liberal stance towards the rule on standing and to determine
exception to the traditional requirements of constitutional litigation. It stresses _______________
that one of the purposes of these requirements is to protect the Supreme Court
from unnecessary litigation of constitutional questions. 141 Kilosbayan, Incorporated v. Guingona, Jr. [Con. Op., J. Feliciano], id., at
In a number of cases,140 this Court has indeed taken a liberal stance towards pp. 155-156 (cited in Magallona v. Ermita, 671 Phil. 243; 655 SCRA 476
the requirement of legal standing, especially when paramount interest is involved. [2011]); Paguia v. Office of the President, 635 Phil. 568; 621 SCRA 600
Indeed, when those who challenge the official act are able to craft an issue of [2010]; Francisco, Jr. v. Nagmamalasakit na mga Manananggol ng mga
transcendental significance to the people, the Court may exercise its sound Manggagawang Pilipino, Inc.,supra note 93 at p. 899; p. 139).
discretion and take cognizance of the suit. 142 Memorandum of OSG, supra note 80. See also Note No. 1082, supra note
_______________ 86.

139 Consolidated Comment of the OSG, pp. 4-5, Rollo (G.R. No. 212444, Vol.
I), pp. 241-242; Memorandum of the OSG, pp. 7-8, Rollo (G.R. No. 212444, Vol. I), 337
pp. 437-438. VOL. 779, JANUARY 12, 2016 337
140 Bayan Muna v. Romulo, supra note 114 at p. 265; p. 255
(citing Constantino, Jr. v. Cuisia, 509 Phil. 486; 472 SCRA 505 [2005]; Agan, Jr. v. Saguisag vs. Ochoa, Jr.
Philippine International Air Terminals Co., Inc., 450 Phil. 744; 402 SCRA 612
[2003]; Del Mar v. Philippine Amusement and Gaming Corporation, 400 Phil. 307; forthwith whether there was grave abuse of discretion on the part of the
346 SCRA 485 [2000]; Tatad v. Garcia, Jr., 313 Phil. 296; 243 SCRA 436 Executive Department.
[1995]; Kilosbayan, Incorporated v. Guingona, Jr., G.R. No. 113375, 5 May 1994, We therefore rule that this case is a proper subject for judicial review.
232 SCRA 110); Integrated Bar of the Philippines v. Zamora, 392 Phil. 618; 338 B. Whether the President may enter into an executive agreement on
SCRA 81 (2000). foreign military bases, troops, or facilities.
C. Whether the provisions under EDCA are consistent with the
Constitution, as well as with existing laws and treaties.
336 Issues B and C shall be discussed together infra.
336 SUPREME COURT REPORTS ANNOTATED
1. The role of the President
Saguisag vs. Ochoa, Jr. as the executor of the law
includes the duty to de-
It may do so in spite of the inability of the petitioners to show that they have fend the State, for which
been personally injured by the operation of a law or any other government act. purpose he may use that
While this Court has yet to thoroughly delineate the outer limits of this power in the conduct of
doctrine, we emphasize that not every other case, however strong public interest foreign relations.
may be, can qualify as an issue of transcendental importance. Before it can be
impelled to brush aside the essential requisites for exercising its power of judicial Historically, the Philippines has mirrored the division of powers in the U.S.
review, it must at the very least consider a number of factors: (1) the character of government. When the Philippine government was still an agency of the Congress
the funds or other assets involved in the case; (2) the presence of a clear case of of the U.S., it was as an agent entrusted with powers categorized as executive,
disregard of a constitutional or statutory prohibition by the public respondent legislative, and judicial, and divided among these three great branches.143 By this
agency or instrumentality of the government; and (3) the lack of any other party division, the law implied that the divided powers cannot be exercised except by the
that has a more direct and specific interest in raising the present questions. 141 department given the power.144
This divide continued throughout the different versions of the Philippine
Constitution and specifically vested the supreme executive power in the Governor- 339
General of the Philip- VOL. 779, JANUARY 12, 2016 339
_______________
Saguisag vs. Ochoa, Jr.
143 Government of the Philippine Islands v. Springer, 50 Phil. 259 (1927).
144 Id. guage, not in the form of the President’s oath, was present in the 1935
Constitution, particularly in the enumeration of executive functions.150 By 1987,
executive power was codified not only in the Constitution, but also in the
338 Administrative Code:151
338 SUPREME COURT REPORTS ANNOTATED
Saguisag vs. Ochoa, Jr. SECTION 1. Power of Control.—The President shall have control of
all the executive departments, bureaus, and offices. He shall ensure that
pines,145 a position inherited by the President of the Philippines when the the laws be faithfully executed. (Emphasis supplied)
country attained independence. One of the principal functions of the supreme
executive is the responsibility for the faithful execution of the laws as embodied by Hence, the duty to faithfully execute the laws of the land is inherent in
the oath of office.146The oath of the President prescribed by the 1987 Constitution executive power and is intimately related to the other executive functions. These
reads thus: functions include the faithful execution of the law in autonomous regions;152 the
right to prosecute crimes;153 the implementation of transportation projects;154 the
I do solemnly swear (or affirm) that I will faithfully and duty to ensure compliance with treaties, executive agreements and executive
conscientiously fulfill my duties as President (or Vice President or orders;155 the authority to deport undesirable aliens;156 the conferment of national
Acting President) of the Philippines, preserve and defend its awards under the President’s jurisdiction;157and the overall administration and
Constitution, execute its laws, do justice to every man, and consecrate control of the executive department.158
myself to the service of the Nation. So help me God. (In case of affirmation, _______________
last sentence will be omitted)147 (Emphases supplied)
150 Constitution (1935, as amended), Art. VII, Sec. 10(1): “The President shall
This Court has interpreted the faithful execution clause as an obligation have control of all executive departments, bureaus or offices, exercise general
imposed on the President, and not a separate grant of power.148 Section 17, Article supervision over all local governments as may be provided by law, and take care
VII of the Constitution, expresses this duty in no uncertain terms and includes it that the laws be faithfully executed.”
in the provision regarding the President’s power of control over the executive 151 Administrative Code of 1987, Book III, Title I, Sec. 1.
department, viz.: 152 Constitution, Art. X, Sec. 16: “The President shall exercise general
supervision over autonomous regions to ensure that the laws are faithfully
The President shall have control of all the executive departments, executed.”
bureaus, and offices. He shall ensure that the laws be faithfully executed. 153 Ilusorio v. Ilusorio, 564 Phil. 746; 540 SCRA 182 (2007); Gonzalez v.
Hongkong & Shanghai Banking Corp., 562 Phil. 841; 537 SCRA 255 (2007).
The equivalent provisions in the next preceding Constitution did not explicitly 154 Metropolitan Manila Development Authority v. Viron Transportation Co.,
require this oath from the President. In the 1973 Constitution, for instance, the Inc., 557 Phil. 121; 530 SCRA 341 (2007).
provision simply gives the President control over the ministries. 149 A similar lan- 155 La Perla Cigar & Cigarette Factory v. Capapas, 139 Phil. 451; 28 SCRA
_______________ 1085 (1969).
156 In re: R. McCulloch Dick, 38 Phil. 211 (1918).
145 Id. 157 Almario v. Executive Secretary, supra note 114.
146 Constitution, Art. VII, Sec. 5; Constitution (1973, as amended), Art. VII, 158 Administrative Code of 1987, Book IV, Sec. 38.
Sec. 7; Constitution (1935, as amended), Art. VII, Sec. 7.
147 Id.
148 Almario v. Executive Secretary, supra note 114. 340
149 Constitution (1973, as amended), Art. VII, Sec. 10: “The President shall 340 SUPREME COURT REPORTS ANNOTATED
have control of the ministries.”
Saguisag vs. Ochoa, Jr.
These obligations are as broad as they sound, for a President cannot function Every other consideration to one side, this remains certain — The
with crippled hands, but must be capable of securing the rule of law within all Congress of the United States clearly intended that the Governor General’s
territories of the Philippine Islands and be empowered to do so within power should be commensurate with his responsibility. The Congress never
constitutional limits. Congress cannot, for instance, limit or take over the intended that the Governor-General should be saddled with the
President’s power to adopt implementing rules and regulations for a law it has responsibility of administering the government and of executing the laws
enacted.159 but shorn of the power to do so. The interests of the Philippines will be best
More important, this mandate is self-executory by virtue of its being inherently served by strict adherence to the basic principles of constitutional
executive in nature.160 As Justice Antonio T. Carpio previously wrote,161 government.

[i]f the rules are issued by the President in implementation or execution In light of this constitutional duty, it is the President’s prerogative to do
of self-executory constitutional powers vested in the President, the rule- whatever is legal and necessary for Philippine defense interests. It is no
making power of the President is not a delegated legislative power. The coincidence that the constitutional provision on the faithful execution clause was
most important self-executory constitutional power of the President is the followed by that on the President’s commander-in-chief powers,164 which are
President’s constitutional duty and mandate specifically granted during extraordinary events of lawless violence, invasion, or
to “ensure that the laws be faithfully executed.” The rule is that the rebellion. And this duty of defending the country is unceasing, even in times when
President can execute the law without any delegation of power from the there is no state of lawless violence, invasion, or rebellion. At such times, the
legislature. President has full powers to ensure the faithful execution of the laws.
It would therefore be remiss for the President and repugnant to the faithful-
The import of this characteristic is that the manner of the President’s execution clause of the Constitution to do nothing when the call of the moment
execution of the law, even if not expressly granted by the law, is justified requires increasing the military’s defensive capabilities, which could include
by necessity and limited only by law, since the President must “take forging alliances with states that hold a common interest with the Philippines or
necessary and proper steps to carry into execution the law.”162 Justice bringing an international suit against an offending state.
George Malcolm states this principle in a grand manner:163 _______________

The executive should be clothed with sufficient power to administer 164 See Constitution, Art. VII, Secs. 17 & 18.
efficiently the affairs of state. He should have complete control of the
instrumentalities
_______________ 342
342 SUPREME COURT REPORTS ANNOTATED
159 Concurring Opinion of J. Carpio, Abakada Guro Party-List v.
Purisima, 584 Phil. 246; 562 SCRA 251 (2008). Saguisag vs. Ochoa, Jr.
160 Id.
161 Id., at p. 297; p. 304. The context drawn in the analysis above has been termed by Justice Arturo D.
162 Philippine Constitution Association v. Enriquez, supra note 131. Brion’s Dissenting Opinion as the beginning of a “patent misconception.” 165 His
163 Government of the Philippine Islands v. Springer, supra note 143. dissent argues that this approach taken in analyzing the President’s role as
executor of the laws is preceded by the duty to preserve and defend the
Constitution, which was allegedly overlooked.166
341 In arguing against the approach, however, the dissent grossly failed to
appreciate the nuances of the analysis, if read holistically and in context. The
VOL. 779, JANUARY 12, 2016 341
concept that the President cannot function with crippled hands and therefore can
Saguisag vs. Ochoa, Jr. disregard the need for Senate concurrence in treaties167 was never expressed or
implied. Rather, the appropriate reading of the preceding analysis shows that the
through whom his responsibility is discharged. It is still true, as said point being elucidated is the reality that the President’s duty to execute the laws
by Hamilton, that “A feeble executive implies a feeble execution of the and protect the Philippines is inextricably interwoven with his foreign affairs
government. A feeble execution is but another phrase for a bad execution; powers, such that he must resolve issues imbued with both concerns to the full
and a government ill executed, whatever it may be in theory, must be in extent of his powers, subject only to the limits supplied by law. In other words,
practice a bad government.” The mistakes of State governments need not apart from an expressly mandated limit, or an implied limit by virtue of
be repeated here. incompatibility, the manner of execution by the President must be given utmost
xxxx
deference. This approach is not different from that taken by the Court in situations Neither could petitioners herein assail the said determination by the
with fairly similar contexts. Executive Department via the instant petition for certiorari.
Thus, the analysis portrayed by the dissent does not give the President
authority to bypass constitutional safeguards and limits. In fact, it specifies what
these limitations are, how these limitations are triggered, how these limitations 344
function, and what can be done within the sphere of constitutional duties and 344 SUPREME COURT REPORTS ANNOTATED
limitations of the President.
Justice Brion’s dissent likewise misinterprets the analysis proffered when it Saguisag vs. Ochoa, Jr.
claims that the foreign relations power of the President should not be interpreted
in isolation.168 The In the seminal case of US v. Curtiss-Wright Export Corp.,the US
_______________ Supreme Court held that “[t]he President is the sole organ of the nation in
its external relations, and its sole representative with foreign relations.”
165 Dissenting Opinion of Justice Arturo D. Brion, p. 556. It is quite apparent that if, in the maintenance of our
166 Id., at p. 557. international relations, embarrassment — perhaps serious
167 Id., at pp. 556-559. embarrassment — is to be avoided and success for our aims
168 Id., at pp. 559-560. achieved, congressional legislation which is to be made effective
through negotiation and inquiry within the international field must
often accord to the President a degree of discretion and
343 freedom from statutory restriction which would not be
admissible where domestic affairs alone involved. Moreover,
VOL. 779, JANUARY 12, 2016 343 he, not Congress, has the better opportunity of knowing the
Saguisag vs. Ochoa, Jr. conditions which prevail in foreign countries, and especially is this
true in time of war. He has his confidential sources of information.
analysis itself demonstrates how the foreign affairs function, while mostly the He has his agents in the form of diplomatic, consular and other
President’s, is shared in several instances, namely in Section 2 of Article II on the officials....
conduct of war; Sections 20 and 21 of Article VII on foreign loans, treaties, and This ruling has been incorporated in our jurisprudence
international agreements; Sections 4(2) and 5(2)(a) of Article VIII on the judicial through Bayan v. Executive Secretary and Pimentel v. Executive
review of executive acts; Sections 4 and 25 of Article XVIII on treaties and Secretary; its overreaching principle was, perhaps, best articulated in
international agreements entered into prior to the Constitution and on the (now Chief) Justice Puno’s dissent in Secretary of Justice v. Lantion:
presence of foreign military troops, bases, or facilities. . . . The conduct of foreign relations is full of complexities and
In fact, the analysis devotes a whole subheading to the relationship between consequences, sometimes with life and death significance to the
the two major presidential functions and the role of the Senate in it. nation especially in times of war. It can only be entrusted to that
This approach of giving utmost deference to presidential initiatives in respect department of government which can act on the basis of the best
of foreign affairs is not novel to the Court. The President’s act of treating EDCA as available information and can decide with decisiveness. . . It is also
an executive agreement is not the principal power being analyzed as the the President who possesses the most comprehensive and the most
Dissenting Opinion seems to suggest. Rather, the preliminary analysis is in confidential information about foreign countries for our diplomatic
reference to the expansive power of foreign affairs. We have long treated this power and consular officials regularly brief him on meaningful events all
as something the Courts must not unduly restrict. As we stated recently in Vinuya over the world. He has also unlimited access
v. Romulo:
To be sure, not all cases implicating foreign relations present political
questions, and courts certainly possess the authority to construe or 345
invalidate treaties and executive agreements. However, the question VOL. 779, JANUARY 12, 2016 345
whether the Philippine government should espouse claims of its nationals
against a foreign government is a foreign relations matter, the authority Saguisag vs. Ochoa, Jr.
for which is demonstrably committed by our Constitution not to the courts
but to the political branches. In this case, the Executive Department has to ultra-sensitive military intelligence data. In fine, the
already decided that it is to the best interest of the country to waive all presidential role in foreign affairs is dominant and the
claims of its nationals for reparations against Japan in the Treaty of Peace President is traditionally accorded a wider degree of
of 1951. The wisdom of such decision is not for the courts to question. discretion in the conduct of foreign affairs. The regularity,
nay, validity of his actions are adjudged under less stringent
standards, lest their judicial repudiation lead to breach of It is quite plain that the Transitory Provisions of the 1987 Constitution
an international obligation, rupture of state relations, intended to add to the basic requirements of a treaty under Section 21 of Article
forfeiture of confidence, national embarrassment and a VII. This means that both provisions must be read as additional limitations to the
plethora of other problems with equally undesirable President’s overarching executive function in matters of defense and foreign
consequences.169 (Emphases supplied) relations.

Understandably, this Court must view the instant case with the same 3. The President, however, may
perspective and understanding, knowing full well the constitutional and legal enter into an executive
repercussions of any judicial overreach. agreement on foreign military
bases, troops, or facilities, if
2. The plain meaning of the (a) it is not the instrument
Constitution prohibits the en- that allows the presence of
try of foreign military bases, foreign military bases, troops,
troops or facilities, except by or facilities; or (b) it merely
way of a treaty concurred in aims to implement an existing
by the Senate — a clear limi- law or treaty.
tation on the President’s dual
role as defender of the State Again we refer to Section 25, Article XVIII of the Constitution:
and as sole authority in for-
eign relations. SECTION 25. After the expiration in 1991 of the Agreement between
the Republic of the Philippines and the United States of America
Despite the President’s roles as defender of the State and sole authority in concerning Military Bases,
foreign relations, the 1987 Constitution expressly limits his ability in instances
when it involves the entry of foreign military bases, troops or facilities. The initial
limitation is found in Section 21 of the provisions on the Executive Department: 347
“No treaty or international agreement VOL. 779, JANUARY 12, 2016 347
_______________
Saguisag vs. Ochoa, Jr.
169 Vinuya v. Romulo, supra note 17.
foreign military bases, troops, or facilities shall not be
allowed in the Philippines except under a treaty duly concurred
346 in by the Senate and, when the Congress so requires, ratified by a
majority of the votes cast by the people in a national referendum held for
346 SUPREME COURT REPORTS ANNOTATED that purpose, and recognized as a treaty by the other contracting State.
Saguisag vs. Ochoa, Jr. (Emphases supplied)

shall be valid and effective unless concurred in by at least two-thirds of all the In view of this provision, petitioners argue170 that EDCA must be in the form
Members of the Senate.” The specific limitation is given by Section 25 of the of a “treaty” duly concurred in by the Senate. They stress that the Constitution is
Transitory Provisions, the full text of which reads as follows: unambiguous in mandating the transmission to the Senate of all international
agreements concluded after the expiration of the MBA in 1991 — agreements that
SECTION 25. After the expiration in 1991 of the Agreement between concern the presence of foreign military bases, troops, or facilities in the country.
the Republic of the Philippines and the United States of America Accordingly, petitioners maintain that the Executive Department is not given the
concerning Military Bases, foreign military bases, troops, or facilities shall choice to conclude agreements like EDCA in the form of an executive agreement.
not be allowed in the Philippines except under a treaty duly concurred in This is also the view of the Senate, which, through a majority vote of 15 of its
by the Senate and, when the Congress so requires, ratified by a majority of members — with 1 against and 2 abstaining — says in SR 105171 that EDCA must
the votes cast by the people in a national referendum held for that purpose, be submitted to the
and recognized as a treaty by the other contracting State. _______________
170 Memorandum of Bayan, et al., pp. 29-32, Rollo (G.R. No. 212444), pp. 593- WHEREAS, under the rules of constitutional and statutory construction, the
596; Memorandum of Saguisag, et al., pp. 17-29, 35-37, Rollo (G.R. No. 212426, two constitutional provisions on Senate concurrence are specific provisions, while
Vol. II), pp. 987-999, 1005-1007. the lone constitutional provision merely mentioning an “executive agreement” is a
171 The pertinent text of SR 105 is reproduced below: general provision, and therefore, the specific provisions on Senate concurrence
WHEREAS, the treaty known as RP-US EDCA (Enhanced Defense prevail over the general provision on “executive agreement”;
Cooperation Agreement) is at present subject of Supreme Court proceedings on the
question of whether this treaty is valid and effective, considering that the Senate
has not concurred with the treaty; 349
WHEREAS, the Office of the President argues that the document is not a VOL. 779, JANUARY 12, 2016 349
treaty but is instead an executive agreement that allegedly does not require Senate
concurrence; Saguisag vs. Ochoa, Jr.
WHEREAS, the only constitutional ground for the position taken by the
Executive is the mere inclusion of the term “executive agreement” in the Senate in the form of a treaty for concurrence by at least two-thirds of all its
Constitution which provides: “All cases involving the constitutionality of an ... members.
executive agree- The Senate cites two constitutional provisions (Article VI, Section 21 and
Article XVIII, Section 25) to support its position. Compared with the lone
constitutional provision that the Office of the Solicitor General (OSG) cites, which
349 is Article XVIII, Section 4(2), which includes the constitutionality of “executive
agreement(s)” among the cases subject to the Supreme Court’s power of judicial
VOL. 779, JANUARY 12, 2016 349
review, the Constitution clearly requires submission of EDCA to the Senate. Two
Saguisag vs. Ochoa, Jr. specific provisions versus one general provision means that the specific provisions
prevail. The term “executive agreement” is “a term wandering alone in the
_______________ Constitution, bereft of provenance and an unidentified constitutional mystery.”
The author of SR 105, Senator Miriam Defensor Santiago, upon interpellation
ment ...” (Article VIII, Section 4, paragraph 2) as one of items included in the even added that the MDT, which the Executive claims to be partly implemented
list of cases which the Supreme Court has power to decide; through EDCA, is already obsolete.
WHEREAS, there is no other provision in the Constitution concerning a so-- _______________
called executive agreement, and there is no mention at all of its definition, its
requirements, the role of the Senate, or any other characteristic of, or protocol for, WHEREAS, the Senate is aware of and obeys the ruling of the Supreme Court
any such so-called “executive agreement”; in Pimentel, Jr. v. Office of the Executive Secretary, 462 SCRA 622 (2005);
WHEREAS, “executive agreement” is a term wandering alone in the WHEREAS, the ruling cited above does not apply to the EDCA case, because
Constitution, bereft of provenance and an unidentified constitutional mystery; the Senate makes no attempt to force the President of the Philippines to submit
WHEREAS, in stark contrast to the lone mention of the term “executive the EDCA treaty for concurrence by the Senate, by this Resolution, the Senate
agreement,” the Constitution provides categorically: merely takes a definitive stand on the nonnegotiable power of the Senate to decide
(a) “No treaty or international agreement shall be valid and effective unless whether a treaty will be valid and effective, depending on the Senate concurrence[;]
concurred in by at least two-thirds of all the Members of the Senate” (Article VII, WHEREFORE, be it hereby resolved by the Senate that the RP-US EDCA
Section 21); treaty requires Senate concurrence, in order to be valid and effective;
(b) “After the expiration in 1991 of the Agreement between the Republic of Be it further resolved, That this Resolution expressing the strong sense of the
the Philippines and the United States of America concerning Military Bases, Senate be formally submitted to the Supreme Court through the Chief Justice.
foreign military bases, troops, or facilities shall not be allowed in the Philippines
except under a treaty duly concurred in the Senate and, when the Congress so
requires, ratified by a majority of the votes cast by the people in a national 350
referendum held for that purpose, and recognized as a treaty by the other 350 SUPREME COURT REPORTS ANNOTATED
contracting State” (Article XVIII, Section 25);
WHEREAS, on the one hand, the Constitution is clear and categorical that Saguisag vs. Ochoa, Jr.
Senate concurrence is absolutely necessary for the validity and effectivity of any
treaty, particularly any treaty that promotes for foreign military bases, troops and There are two insurmountable obstacles to this Court’s agreement with SR
facilities, such as the EDCA; 105, as well as with the comment on interpellation made by Senator Santiago.
First, the concept of “executive agreement” is so well-entrenched in this Court’s
pronouncements on the powers of the President. When the Court validated the
concept of “executive agreement,” it did so with full knowledge of the Senate’s role Under the principles of constitutional construction, of paramount
in concurring in treaties. It was aware of the problematique of distinguishing when consideration is the plain meaning of the language expressed in the Constitution,
an international agreement needed Senate concurrence for validity, and when it or the verba legis rule.175 It is presumed that the provisions have been carefully
did not; and the Court continued to validate the existence of “executive crafted in order to express the objective it seeks to attain. 176 It is incumbent upon
agreements” even after the 1987 Constitution.172 This follows a long line of similar the Court to refrain from going beyond the plain
decisions upholding the power of the President to enter into an executive _______________
agreement.173
_______________ 174 Nicolas v. Romulo, supra note 39.
175 Chavez v. Judicial and Bar Council, supra note 94; Francisco, Jr. v.
172 Arigo v. Swift, G.R. No. 206510, 16 September 2014, 735 SCRA 102; Land Nagmamalasakit na mga Manananggol ng mga Manggagawang Pilipino,
Bank of the Philippines v. Atlanta Industries, Inc., G.R. No. 193796, 2 July 2014, Inc., supranote 93 (quoting J.M. Tuason & Co., Inc. v. Land Tenure
729 SCRA 12; Roxas v. Ermita, G.R. No. 180030, June 10, 2014; Bayan Muna v. Administration, 142 Phil. 719; 31 SCRA 413 [1970], citing Baranda v. Gustilo, 248
Romulo,supra note 114; Vinuya v. Romulo, supra note 17; Nicolas v. Phil. 205; 165 SCRA 757 [1988]; Luz Farms v. Secretary of the Department of
Romulo, supra note 39; Akbayan Citizens Action Party v. Aquino, supra note Agrarian Reform, 270 Phil. 151; 192 SCRA 51 [1990]; Ordillo v. Commission on
15; Suplico v. NEDA, 580 Phil. 301; 558 SCRA 329 (2008); Neri v. Senate Elections, 270 Phil. 183; 192 SCRA 100 [1990]).
Committee on Accountability of Public Officers and Investigations, 572 Phil. 554; 176 Id.; Ang Bagong Bayani-OFW Labor Party v. Commission on Elections,
549 SCRA 77 (2008); Abaya v. Ebdane, Jr., 544 Phil. 645; 515 SCRA 720 412 Phil. 308; 359 SCRA 698 (2001) (citing J.M. Tuason & Co., Inc. v. Land Tenure
(2007); Senate of the Philippines v. Ermita, 522 Phil. 1; 488 SCRA 1 Administration, id.; Gold Creek Mining Corp. v. Rodriguez, 66 Phil. 259, 264
(2006); Pimentel, Jr. v. Office of the Executive Secretary, supranote [1938]; Ruben C. Agpalo, Statutory Construction, p. 311 [1990]).
15; Bayan (Bagong Alyansang Makabayan) v. Zamora, supra note note 23; Chavez
v. Presidential Commission on Good Government, 360 Phil. 133; 299 SCRA 744
(1998). 352
173 Republic v. Quasha, 150-B Phil. 140; 46 SCRA 160 (1972); Adolfo v. Court 352 SUPREME COURT REPORTS ANNOTATED
of First Instance of Zambales, 145 Phil. 264; 34 SCRA 166 (1970); Commissioner of
Internal Revenue v. Guerrero, 128 Phil. 197; 19 SCRA 205 (1967); Gonzales v. Saguisag vs. Ochoa, Jr.
Hechanova, 118 Phil. 1065; 9 SCRA 230 (1963); Commissioner of Customs v.
Eastern Sea Trading, 113 Phil. 333; 3 SCRA 351 (1961); USAFFE Veterans Ass’n., meaning of the words used in the Constitution. It is presumed that the framers
Inc. v. Treasurer of the Phil., 105 Phil. 1030 (1959); Uy Matiao & Co., Inc. v. City and the people meant what they said when they said it, and that this
of Cebu, 93 Phil. 300 (1953); Abbot Laboratories v. Agrava, 91 Phil. 328 (1952). understanding was reflected in the Constitution and understood by the people in
the way it was meant to be understood when the fundamental law was ordained
and promulgated.177 As this Court has often said:
351
We look to the language of the document itself in our search for its
VOL. 779, JANUARY 12, 2016 351
meaning. We do not of course stop there, but that is where we begin. It is
Saguisag vs. Ochoa, Jr. to be assumed that the words in which constitutional provisions are
couched express the objective sought to be attained. They are to be given
Second, the MDT has not been rendered obsolescent, considering that as late their ordinary meaning except where technical terms are
as 2009,174 this Court continued to recognize its validity. employed in which case the significance thus attached to them prevails.
Third, to this Court, a plain textual reading of Article XIII, Section 25, As the Constitution is not primarily a lawyer’s document, it being
inevitably leads to the conclusion that it applies only to a proposed agreement essential for the rule of law to obtain that it should ever be present in the
between our government and a foreign government, whereby military bases, people’s consciousness, its language as much as possible should be
troops, or facilities of such foreign government would be “allowed” or would “gain understood in the sense they have in common use. What it says
entry” Philippine territory. according to the text of the provision to be construed compels acceptance
Note that the provision “shall not be allowed” is a negative injunction. This and negates the power of the courts to alter it, based on the postulate
wording signifies that the President is not authorized by law to allow foreign that the framers and the people mean what they say. Thus, these
military bases, troops, or facilities to enter the Philippines, except under a treaty are the cases where the need for construction is reduced to a
concurred in by the Senate. Hence, the constitutionally restricted authority minimum.178 (Emphases supplied)
pertains to the entry of the bases, troops, or facilities, and not to the activities to _______________
be done after entry.
177 Id.; Francisco, Jr. v. Nagmamalasakit na mga Manananggol ng mga
Manggagawang Pilipino, Inc., supra note 93 (quoting J.M. Tuason & Co., Inc. v.
Land Tenure Administration, id.; citing Baranda v. Gustila, supra note 175 at p. 354
770; Luz Farms v. Secretary of the Department of Agrarian Reform, supra note 354 SUPREME COURT REPORTS ANNOTATED
175; Ordillo v. Commission on Elections, supra note 175); Sarmiento III v. Mison,
240 Phil. 505; 156 SCRA 549 (1987); Gold Creek Mining Corp. v. Saguisag vs. Ochoa, Jr.
Rodriguez, supra note 176.
178 Francisco, Jr. v. Nagmamalasakit na mga Manananggol ng mga of, within (any place or thing).”183 That something is the Philippines, which is
Manggagawang Pilipino, Inc., id. (quoting J.M. Tuason & Co., Inc. v. Land the noun that follows.
Tenure Administration, id.). It is evident that the constitutional restriction refers solely to the initial entry
of the foreign military bases, troops, or facilities. Once entry is authorized, the
subsequent acts are thereafter subject only to the limitations provided by the rest
353 of the Constitution and Philippine law, and not to the Section 25 requirement of
validity through a treaty.
VOL. 779, JANUARY 12, 2016 353
The VFA has already allowed the entry of troops in the Philippines. This Court
Saguisag vs. Ochoa, Jr. stated in Lim v. Executive Secretary:

It is only in those instances in which the constitutional provision is unclear, After studied reflection, it appeared farfetched that the ambiguity
ambiguous, or silent that further construction must be done to elicit its surrounding the meaning of the word “activities” arose from accident. In
meaning.179 In Ang Bagong Bayani-OFW Labor Party v. Commission on our view, it was deliberately made that way to give both parties a certain
Elections,180 we reiterated this guiding principle: leeway in negotiation. In this manner, visiting US forces may sojourn
in Philippine territory for purposes other than military. As
it [is] safer to construe the Constitution from what appears conceived, the joint exercises may include training on new techniques of
upon its face. The proper interpretation therefore depends more on how patrol and surveillance to protect the nation’s marine resources, sea search-
it was understood by the people adopting it than in the framers’ and-rescue operations to assist vessels in distress, disaster relief
understanding thereof. (Emphases supplied) operations, civic action projects such as the building of school houses,
medical and humanitarian missions, and the like.
The effect of this statement is surprisingly profound, for, if taken literally, the Under these auspices, the VFA gives legitimacy to the current
phrase “shall not be allowed in the Philippines” plainly refers to the entry of bases, Balikatan exercises. It is only logical to assume that “Balikatan 02-1,” a
troops, or facilities in the country. The Oxford English Dictionary defines the word “mutual anti-terrorism advising, assisting and training exercise,” falls
“allow” as a transitive verb that means “to permit, enable”; “to give consent to the under the umbrella of sanctioned or allowable activities in the context of
occurrence of or relax restraint on (an action, event, or activity)”; “to consent to the the agreement. Both the history and intent of the Mutual Defense Treaty
presence or attendance of (a person)”; and when with an adverbial of place, “to and the VFA support the conclusion that combat-related activities — as
permit (a person or animal) to go, come, or be in, out, near, etc.” 181 Black’s Law opposed to com-
Dictionary defines the term as one that means “[t]o grant, approve, or permit.”182 _______________
The verb “allow” is followed by the word “in,” which is a preposition used to
indicate “place or position in space or anything having material extension: Within 183 OED Online, available at
the limits or bounds <http://www.oed.com/view/Entry/92970?rskey=JDaO1Y&result=6>, accessed on
_______________ 28 October 2015; See also Merriam-Webster Online Dictionary, available at
<http://www.merriamwebster.com/dictionary/in>, accessed on 28 October 2015.
179 Ang Bagong Bayani-OFW Labor Party v. Commission on
Elections, supranote 176.
180 Id. (quoting the Separate Opinion of Justice Mendoza in Civil Liberties 355
Union v. Executive Secretary, 272 Phil. 147; 194 SCRA 317 [1991]). VOL. 779, JANUARY 12, 2016 355
181 OED Online, available at <http://www.oed.com/view/Entry/5460>,
accessed on 28 October 2015; See also Merriam-Webster Online Dictionary, Saguisag vs. Ochoa, Jr.
“allow,” available at <http://www.merriamwebster.com/dictionary/allow>,
accessed on 28 October 2015. bat itself — such as the one subject of the instant petition, are indeed
182 Black’s Law Dictionary (2nd ed). authorized.184 (Emphasis supplied)
Moreover, the Court indicated that the Constitution continues to govern the for the rule of law to obtain that it should ever be present in the people’s
conduct of foreign military troops in the Philippines,185readily implying the legality consciousness, its language as much as possible should be
of their initial entry into the country. understood in the sense they have in common use. What it says
The OSG emphasizes that EDCA can be in the form of an executive agreement, according to the text of the provision to be construed compels acceptance
since it merely involves “adjustments in detail” in the implementation of the MDT and negates the power of the courts to alter it, based on the postulate
and the VFA.186 It points out that there are existing treaties between the that the framers and the people mean what they say. Thus, these
Philippines and the U.S. that have already been concurred in by the Philippine are the cases where the need for construction is reduced to a
Senate and have thereby met the requirements of the Constitution under Section minimum.190 (Emphases supplied)
25. Because of the status of these prior agreements, respondent emphasizes that
EDCA need not be transmitted to the Senate. As applied, verba legis aids in construing the ordinary meaning of terms. In
The aforecited Dissenting Opinion of Justice Brion disagrees with this case, the phrase being construed is “shall not be allowed in the Philippines”
the ponencia’s application of verba legis construction to the words of Article XVIII, and not the preceding one referring to “the expiration in 1991 of the Agreement
Section 25.187 It claims that the provision is “neither plain, nor that simple.”188 To between the Republic of the Philippines and the United States of America
buttress its disagreement, the dissent states that the provision refers to a concerning Military Bases, foreign military bases, troops, or facilities.” It is explicit
historical incident, which is the expiration of the 1947 MBA. 189 Accordingly, this in the wording of the provision itself that any interpretation goes beyond the text
position requires questioning the circumstances that led to the historical event, _______________
and the meaning of the terms under Article XVIII, Section 25.
This objection is quite strange. The construction technique of verba legis is not 190 Francisco, Jr. v. Nagmamalasakit na mga Manananggol ng mga
inapplicable just because a provision has Manggagawang Pilipino, Inc., supra note 93 (quoting J.M. Tuason & Co., Inc. v.
_______________ Land Tenure Administration, supra note 175).

184 Supra note 69.


185 In the words of the Court: “The present Constitution contains key 357
provisions useful in determining the extent to which foreign military troops are VOL. 779, JANUARY 12, 2016 357
allowed in Philippine territory.” Lim v. Executive Secretary, id.
186 Memorandum of OSG, pp. 14-27, Rollo, pp. 444-457. Saguisag vs. Ochoa, Jr.
187 Dissenting Opinion of Justice Arturo D. Brion, p. 574.
188 Id., at p. 576. itself and into the discussion of the framers, the context of the Constitutional
189 Id. Commission’s time of drafting, and the history of the 1947 MBA. Without reference
to these factors, a reader would not understand those terms. However, for the
phrase “shall not be allowed in the Philippines,” there is no need for such reference.
356 The law is clear. No less than the Senate understood this when it ratified the VFA.
356 SUPREME COURT REPORTS ANNOTATED
4. The President may gener-
Saguisag vs. Ochoa, Jr. ally enter into executive
agreements subject to limi-
a specific historical context. In fact, every provision of the Constitution has a tations defined by the Con-
specific historical context. The purpose of constitutional and statutory construction stitution and may be in fur-
is to set tiers of interpretation to guide the Court as to how a particular provision therance of a treaty already
functions. Verba legis is of paramount consideration, but it is not the only concurred in by the Senate.
consideration. As this Court has often said:
We discuss in this section why the President can enter into executive
We look to the language of the document itself in our search for its agreements.
meaning. We do not of course stop there, but that is where we begin. It would be helpful to put into context the contested language found in Article
It is to be assumed that the words in which constitutional provisions are XVIII, Section 25. Its more exacting requirement was introduced because of the
couched express the objective sought to be attained. They are to be given previous experience of the country when its representatives felt compelled to
their ordinary meaning except where technical terms are consent to the old MBA.191 They felt constrained to agree to the MBA in fulfillment
employed in which case the significance thus attached to them prevails. of one of the major conditions for the country to gain independence from the
As the Constitution is not primarily a lawyer’s document, it being essential U.S.192 As a result of that experience, a second layer of consent for agreements that
_______________ 195 Republic Act No. 9184 (Government Procurement Reform Act) (2003), Sec.
4; Administrative Code of 1987, Book II, Sec. 18(2)(a); Presidential Decree No.
191 See IV Record, Constitutional Commission, p. 759 (18 Sep. 1986): “By 1464, as amended (Tariff and Customs Code of 1978), Sec. 402(f); Republic Act No.
inequalities, is the Commissioner referring to the one-sided provisions, the onerous 1789 (Reparations Law) (1957), Sec. 18; Commonwealth Act No. 733 (Acceptance
conditions of the RP-US Bases Agreement?” Nicolas v. Romulo, supra note 39 at p. of
280; p. 458.
192 See Treaty of General Relations between the Republic of the Philippines
and the United States of America, October 22, 1946, 359
Art. 1 (1946); Philippine Independence Act (Tydings-McDuffie Act), Pub.L. 73-127, VOL. 779, JANUARY 12, 2016 359
48 Stat. 456, (24 March 1934), Secs. 5 and 10; Foreign Service Institute, supra note
24 at pp. ix-x. Saguisag vs. Ochoa, Jr.

deliberations of the Constitutional Commission.197 They cover a wide array of


358 subjects with varying scopes and purposes,198including those that involve the
presence of foreign military forces in the country.199
358 SUPREME COURT REPORTS ANNOTATED As the sole organ of our foreign relations200 and the constitutionally assigned
Saguisag vs. Ochoa, Jr. chief architect of our foreign policy,201 the President is vested with the exclusive
power to conduct and manage the country’s interface with other states and
allow military bases, troops and facilities in the country is now articulated in governments. Being the principal representative of the Philippines, the Chief
Article XVIII of our present Constitution. Executive speaks and listens for the nation; initiates, maintains, and develops
This second layer of consent, however, cannot be interpreted in such a way that diplomatic relations with other states and governments; negotiates and enters into
we completely ignore the intent of our constitutional framers when they provided international agreements; promotes trade, investments, tourism and
for that additional layer, nor the vigorous statements of this Court that affirm the _______________
continued existence of that class of international agreements called “executive
agreements.” Executive Agreement Under Title IV of [United States] Public Law 371-
The power of the President to enter into binding executive agreements without 79thCongress) (1946).
Senate concurrence is already well-established in this jurisdiction.193 That power 196 Neri v. Senate Committee on Accountability of Public Officers and
has been alluded to in our present and past Constitutions,194 in various Investigations, supra note 172; Republic v. Quasha, supra note 173; Commissioner
statutes,195 in Supreme Court decisions,196 and during the of Internal Revenue v. Guerrero, supra note 173; Gonzales v.
_______________ Hechanova, supra note 173; Commissioner of Customs v. Eastern Sea
Trading, supra note 173; USAFFE Veterans Ass’n., Inc. v. Treasurer of the
193 Land Bank of the Philippines v. Atlanta Industries, Inc., supra note Phil., supra note 173; Abbot Laboratories v. Agrava, supra note 173.
172; Bayan Muna v. Romulo, supra note 114; Nicolas v. Romulo, supra note 197 II Record, Constitutional Commission, supra note 193.
39; Neri v. Senate Committee on Accountability of Public Officers and 198 Bayan Muna v. Romulo, supra note 114. See also Sinco,supra note 15.
Investigations, supra note 172; Department of Budget and Management 199 See generally: Nicolas v. Romulo, supra note 39; Lim v. Executive
Procurement Service v. Kolonwel Trading, 551 Phil. 1030; 524 SCRA 591 Secretary, supra note 69.
(2007); Abaya v. Ebdane, Jr., supra note 172; Republic v. Quasha, supra note 200 See: Akbayan Citizens Action Party v. Aquino, supra note 15; Pimentel,
173; Adolfo v. Court of First Instance of Zambales, supra note 173; Commissioner Jr. v. Office of the Executive Secretary, supra note 15. See Constitution, Art. VII,
of Internal Revenue v. Guerrero, supra note 173; Gonzales v. Sec. 1, in relation to Administrative Code of 1987, Book IV (Executive Branch),
Hechanova, supra note 173; Commissioner of Customs v. Eastern Sea Title I (Foreign Affairs), Secs. 3(1) and 20; Sinco, supra note 15 at p. 297.
Trading, supra note 173; USAFFE Veterans Ass’n., Inc. v. Treasurer of the 201 Pimentel, Jr. v. Office of the Executive Secretary, id. See Constitution, Art.
Phil.,supra note 173; Uy Matiao & Co., Inc. v. City of Cebu, supra note 173; Abbot VII, Sec. 1, in relation to Administrative Code of 1987, Book IV (Executive Branch),
Laboratories v. Agrava, supra note 173; II Record, Constitutional Commission, pp. Title I (Foreign Affairs), Secs. 3(1) and 20; Sinco, id., at p. 298.
544-546 (31 July 1986); Cortes, supra note 15 at p. 190; Sinco, supra note 15 at pp.
303-305.
194 Constitution, Art. VIII (Judicial Department), Secs. 4(2) & 5(2)(a); 360
Constitution (1973, as amended), Art. X (The Judiciary), Secs. 2(2) & 5(2)(a), Art. 360 SUPREME COURT REPORTS ANNOTATED
XVII (Transitory Provisions), Sec. 12; Constitution (1935), Ordinance Appended to
the Constitution or “Parity Amendment.” Saguisag vs. Ochoa, Jr.
other economic relations; and settles international disputes with other The categorization of subject matters that may be covered by
states.202 international agreements mentioned in Eastern Sea Trading is not
As previously discussed, this constitutional mandate emanates from the cast in stone. x x x.
inherent power of the President to enter into agreements with other states, As may be noted, almost half a century has elapsed sincethe Court
including the prerogative to conclude binding executive agreements that do not rendered its decision in Eastern Sea Trading. Since then, the conduct
require further Senate concurrence. The existence of this presidential power 203 is of foreign affairs has become more complex and the domain of
so well-entrenched that Section 5(2)(a), Article VIII of the Constitution, even international law wider, as to include such subjects as human rights,
provides for a check on its exercise. As expressed below, executive agreements are the environment, and the sea. In fact, in the US alone, the executive
among those official governmental acts that can be the subject of this Court’s power agreements executed by its President from 1980 to 2000 covered subjects
of judicial review: such as defense, trade, scientific cooperation, aviation, atomic
energy, environmental cooperation, peace corps, arms limitation,
(2) Review, revise, reverse, modify, or affirm on appeal and nuclear safety, among others. Surely, the enumeration
or certiorari, as the law or the Rules of Court may provide, final in Eastern Sea Trading cannot circumscribe the option of each
judgments and orders of lower courts in: state on the matter of which the international agreement format
(a) All cases in which the constitutionality or would be convenient to serve its best interest. As Francis Sayre said
validity of any treaty, international or executive agreement, in his work referred to earlier:
law, presidential decree, proclamation, order, instruction, . . . It would be useless to undertake to discuss here the
ordinance, or regulation is in question. (Emphases supplied) large variety of executive agreements as such concluded

In Commissioner of Customs v. Eastern Sea Trading, executive agreements are _______________


defined as “international agreements
_______________ 204 Commissioner of Customs v. Eastern Sea Trading, supra note 173.
205 Bayan Muna v. Romulo, supra note 114. See also Sinco, supra note 15.
202 See: Constitution, Art. VII, Sec. 1, in relation to Administrative Code of
1987, Book III (Office of the President), Title I (Powers of the President), Sec. 1
and Book IV (Executive Branch), Title I (Foreign Affairs), Secs. 3(1) and 20 and 362
Title III (Justice), Sec. 35(10); Pimentel, Jr. v. Office of the Executive 362 SUPREME COURT REPORTS ANNOTATED
Secretary, supra note 15 (on ratification of treaties); Vinuya v. Executive
Secretary, supra note 17 (on espousing claims against foreign Saguisag vs. Ochoa, Jr.
governments); Abaya v. Ebdane, Jr., supra note 172 (on contracting foreign
loans); People’s Movement for Press Freedom v. Manglapus, supra note 15 (on from time to time. Hundreds of executive agreements, other
treaty negotiations with foreign states); Sinco, id., at p. 298. than those entered into under the trade-agreement act, have been
203 See Sinco, id., at pp. 297-298. negotiated with foreign governments. . . . They cover such subjects
as the inspection of vessels, navigation dues, income tax on shipping
profits, the admission of civil air craft, custom matters and
361 commercial relations generally, international claims, postal
matters, the registration of trademarks and copyrights, etc. . . .
VOL. 779, JANUARY 12, 2016 361
(Emphases supplied)
Saguisag vs. Ochoa, Jr.
One of the distinguishing features of executive agreements is that their
embodying adjustments of detail carrying out well-established national validity and effectivity are not affected by a lack of Senate concurrence. 206 This
policies and traditions and those involving arrangements of a more or less distinctive feature was recognized as early as in Eastern Sea Trading (1961), viz.:
temporary nature.”204 In Bayan Muna v. Romulo, this Court further clarified that
executive agreements can cover a wide array of subjects that have various scopes Treaties are formal documents which require ratification with
and purposes.205 They are no longer limited to the traditional subjects that are the approval of two-thirds of the Senate. Executive
usually covered by executive agreements as identified in Eastern Sea Trading. The agreements become binding through executive action without the
Court thoroughly discussed this matter in the following manner: need of a vote by the Senate or by Congress.
xxxx
[T]he right of the Executive to enter into binding
agreements without the necessity of subsequent Congressional 364
approval has been confirmed by long usage. From the earliest days of 364 SUPREME COURT REPORTS ANNOTATED
our history we have entered into executive agreements covering such
subjects as commercial and consular relations, most-favored-nation rights, Saguisag vs. Ochoa, Jr.
patent rights, trademark and copyright protection, postal and navigation
arrangements and the settlement of claims. The validity of these has ceeding from an executive act which affects external relations
never been seriously questioned by our courts. (Emphases supplied) independent of the legislative and the other is anexecutive act in
_______________ pursuance of legislative authorization. Thefirst kind might take the
form of just conventions or exchanges of notes or protocol while the
206 Commissioner of Customs v. Eastern Sea Trading, supra note 173. other, which would be pursuant to the legislative authorization, may
be in the nature of commercial agreements.
MR. CONCEPCION: Executive agreements are generally made
363 to implement a treaty already enforced or to determine the details
for the implementation of the treaty. We are speaking of executive
VOL. 779, JANUARY 12, 2016 363 agreements, not international agreements.
Saguisag vs. Ochoa, Jr. MS. AQUINO: I am in full agreement with that, except that it does
not cover the first kind of executive agreement which is just protocol or an
That notion was carried over to the present Constitution. In fact, the framers exchange of notes and this would be in the nature of reinforcement of claims
specifically deliberated on whether the general term “international agreement” of a citizen against a country, for example.
included executive agreements, and whether it was necessary to include an MR. CONCEPCION: The Commissioner is free to require ratification
express proviso that would exclude executive agreements from the requirement of for validity insofar as the Philippines is concerned.
Senate concurrence. After noted constitutionalist Fr. Joaquin Bernas quoted the MS. AQUINO: It is my humble submission that we
Court’s ruling in Eastern Sea Trading, the Constitutional Commission members should provide, unless the Committee explains to us otherwise, an
ultimately decided that the term “international agreements” as contemplated in explicit proviso which would except executive agreements from the
Section 21, Article VII, does not include executive agreements, and that requirement of concurrence of two-thirds of the Members of the
a proviso is no longer needed. Their discussion is reproduced below:207 Senate. Unless I am enlightened by the Committee I propose that
tentatively, the sentence should read. “No treaty or international
MS. AQUINO: Madam President, first I would like a clarification agreement EXCEPT EXECUTIVE AGREEMENTS shall be valid and
from the Committee. We have retained the words “international effective.”
agreement” which I think is the correct judgment on the matter because an FR. BERNAS: I wonder if a quotation from the Supreme Court
international agreement is different from a treaty. A treaty is a contract decision [in Eastern Sea Trading] might help clarify this:
between parties which is in the nature of international agreement and also The right of the executive to enter into binding
a municipal law in the sense that the people are bound. So there is a agreements without the necessity of subsequent
conceptual difference. However, I would like to be clarified if the Congressional approval has been confirmed by long usage.
international agreements include executive agreements. From the earliest days of our history, we have
MR. CONCEPCION: That depends upon the parties. All parties to
these international negotiations stipulate the conditions which are
necessary for the agreement or whatever it may be to become valid or 365
effective as regards the parties. VOL. 779, JANUARY 12, 2016 365
MS. AQUINO: Would that depend on the parties or would that depend
on the nature of the executive agreement? According to common usage, Saguisag vs. Ochoa, Jr.
there are two types of executive agreement: one is purely pro-
_______________ entered into executive agreements covering such subjects as
commercial and consular relations, most favored nation rights, patent
207 II Record, Constitutional Commission, pp. 544-546 (31 July 1986). See rights, trademark and copyright protection, postal and navigation
also Defensor-Santiago, Miriam, International Agreements in Constitutional Law: arrangements and the settlement of claims. The validity of this has never
The Suspended RP-China (ZTE) Loan Agreement, 53 Ateneo L.J. 537, 539 (2008). been seriously questioned by our Courts.
Agreements with respect to the registration of trademarks have been
concluded by the executive of various countries under the Act of Congress
of March 3, 1881 (21 Stat. 502) . . . International agreements involving MR. CONCEPCION: Whether it partakes or not of the nature of a
political issues or changes of national policy and those involving treaty, it is within the power of the Constitutional Commission to require
international agreements of a permanent character usually take the that.
form of treaties. But international agreements embodying adjustments MR. GUINGONA: Yes. That is why I am trying to clarify
of detail, carrying out well-established national policies whether the words “international agreements” would include
and traditions and those involving arrangements of a more or less executive agreements.
temporary nature usually take the form of executive agreements. MR. CONCEPCION: No, not necessarily; generally no.
MR. ROMULO: Is the Commissioner, therefore, excluding the xxx
executive agreements?
FR. BERNAS: What we are referring to, therefore, when we say
international agreements which need concurrenceby at least two- 367
thirds are those which are permanent in nature. VOL. 779, JANUARY 12, 2016 367
MS. AQUINO: And it may include commercial agreements which are
executive agreements essentially but which are proceeding from the Saguisag vs. Ochoa, Jr.
authorization of Congress. If that is our understanding, then I am willing
to withdraw that amendment. MR. ROMULO: I wish to be recognized first. I have only one question.
FR. BERNAS: If it is with prior authorization of Congress, then Do we take it, therefore, that as far as the Committee is concerned, the
it does not need subsequent concurrence by Congress. term “international agreements” does not include the term
MS. AQUINO: In that case, I am withdrawing my amendment. “executive agreements” as read by the Commissioner in that text?
MR. TINGSON: Madam President. FR. BERNAS: Yes. (Emphases Supplied)

The inapplicability to executive agreements of the requirements under Section


366 21 was again recognized in BAYAN v. Zamora and in Bayan Muna v.
Romulo. These cases, both decided under the aegis of the present Constitution,
366 SUPREME COURT REPORTS ANNOTATED
quoted Eastern Sea Trading in reiterating that executive agreements are valid
Saguisag vs. Ochoa, Jr. and binding even without the concurrence of the Senate.
Executive agreements may dispense with the requirement of Senate
THE PRESIDENT: Is Commissioner Aquino satisfied? concurrence because of the legal mandate with which they are concluded. As culled
MS. AQUINO: Yes. There is already an agreement among us on from the aforequoted deliberations of the Constitutional Commission, past
the definition of “executive agreements” and that would make Supreme Court Decisions, and works of noted scholars,208 executive agreements
unnecessary any explicit proviso on the matter. merely involve arrangements on the implementation of existing policies, rules,
xxx laws, or agreements. They are concluded (1) to adjust the details of a treaty;209 (2)
MR. GUINGONA: I am not clear as to the meaning of “executive pursuant
agreements” because I heard that these executive agreements must rely on _______________
treaties. In other words, there must first be treaties.
MR. CONCEPCION: No, I was speaking about the common use, as 208 Bayan Muna v. Romulo, supra note 114 at p. 261; p. 253; Gonzales v.
executive agreements being the implementation of treaties, details of which Hechanova, supra note 173; Commissioner of Customs v. Eastern Sea
do not affect the sovereignty of the State. Trading, supranote 173; II Record, Constitutional Commission, pp. 544-546 (31
MR. GUINGONA: But what about the matter of permanence, Madam July 1986); Cortes, supra note 15; Sinco, supra note 15.
President? Would 99 years be considered permanent? What would be the 209 See, e.g.: Bayan Muna v. Romulo, id. (on the transfer or surrender of US
measure of permanency? I do not conceive of a treaty that is going to be nationals in the Philippines who may be sued before international
forever, so there must be some kind of a time limit. tribunals); Nicolas v. Romulo, supra note 39 (on agreement concerning the
MR. CONCEPCION: I suppose the Commissioner’s question is detention of a member of the U.S. Armed Forces, who was accused of committing
whether this type of agreement should be included in a provision of the a crime in the Philippines); Adolfo v. Court of First Instance of
Constitution requiring the concurrence of Congress. Zambales, supra note 173 (on exchange of notes pursuant to the 1947 MBA);
MR. GUINGONA: It depends on the concept of the executive Treaty of General Relations Between the Republic of the Philippines and the
agreement of which I am not clear. If the executive agreement United States of America (1946).
partakes of the nature of a treaty, then it should also be included.
368 However, this principle does not mean that the domestic law
368 SUPREME COURT REPORTS ANNOTATED distinguishing treaties, international agreements, and executive agreements is
relegated to a mere variation in form, or that the constitutional requirement of
Saguisag vs. Ochoa, Jr. Senate concurrence is demoted to an optional constitutional directive. There
remain two very important features that distinguish treaties from executive
to or upon confirmation by an act of the Legislature;210 or (3) in the exercise of agreements and translate them into terms of art in the domestic setting.
the President’s independent powers under the Constitution.211 The raison d’être of First, executive agreements must remain traceable to an express or implied
executive agreements hinges on prior constitutional or legislative authorizations. authorization under the Constitution, statutes, or treaties. The absence of these
The special nature of an executive agreement is not just a domestic variation precedents puts the validity and effectivity of executive agreements under serious
in international agreements. International practice has accepted the use of various question for the main function of the Executive is to enforce the Constitution and
forms and designations of international agreements, ranging from the traditional the laws enacted by the Legislature, not to defeat or interfere in the performance
notion of a treaty — which connotes a formal, solemn instrument — to of these rules.214 In turn, executive agreements cannot create new international
engagements concluded in modern, simplified forms that no longer necessitate obligations that are not expressly allowed or reasonably implied in the law they
ratification.212 An international agreement may take different forms: treaty, act, purport to implement.
protocol, agreement, concordat, compromis d’arbitrage, convention, covenant, Second, treaties are, by their very nature, considered superior to executive
declaration, exchange of notes, statute, pact, charter, agreed minute, agreements. Treaties are products of the acts of the Executive and the
memorandum of agreement, Senate215 unlike executive agreements, which are solely executive
_______________ actions.216 Because of legislative participation through the Senate, a treaty is
regarded
210 See, e.g.: Republic v. Quasha, supra note 173; Commissioner of Internal _______________
Revenue v. Guerrero, supra note 173; Abbot Laboratories v. Agrava, supra note 173
(on the interpretation of the provision in the Philippine Patent Law of 1947 213 See generally: Bayan (Bagong Alyansang Makabayan) v. Zamora, id.;
concerning the reciprocity measure on priority rights to be granted to U.S. Gautier, id., at p. 37 (quoting Customs régime between Germany and Austria,
nationals); Uy Matiao & Co., Inc. v. City of Cebu, supra note 173; Republic Act No. Advisory Opinion, 1931 PCIJ, Ser. A/B no. 41, p. 47).
9 —Authority of President to Enter into Agreement with US under Republic of the 214 Gonzales v. Hechanova, supra note 173.
Phil. Military Assistance Act (1946). 215 Bayan Muna v. Romulo, supra note 114 (affirming Adolfo v. Court of First
211 See, e.g.: Land Bank v. Atlanta Industries, Inc., supra note 172 (on foreign Instance of Zambales, supra note 173).
loan agreement); Bayan Muna v. Romulo, supra note 114; Department of Budget 216 See: Bayan Muna v. Romulo, id.
& Management Procurement Service v. Kolonwel Trading, supra note 193 (on
foreign loan agreement); Abaya v. Ebdane, Jr., supra note 172 (on foreign loan
agreement); Commissioner of Customs v. Eastern Sea Trading, supra note 173 (on 370
foreign trade and financial agreements); USAFFE Veterans Ass’n., Inc. v.
370 SUPREME COURT REPORTS ANNOTATED
Treasurer of the Phil., supra note 173 (on conversion of unspent fund as a foreign
loan). But see on limitations: Gonzales v. Hechanova, supra note 173. Saguisag vs. Ochoa, Jr.
212 See generally: Bayan (Bagong Alyansang Makabayan) v.
Zamora, supra note 23; Gautier, Philippe, 1969 Vienna Convention, Article 2 Use as being on the same level as a statute.217 If there is an irreconcilable conflict,
of Terms, in The Vienna Conventions on the Law of Treaties: A Commentary, Vol. a later law or treaty takes precedence over one that is prior. 218 An executive
I, pp. 35-36 (Olivier Corten & Pierre Klein eds., 2011). agreement is treated differently. Executive agreements that are inconsistent with
either a law or a treaty are considered ineffective.219 Both types of international
agreement are nevertheless subject to the supremacy of the Constitution.220
369 This rule does not imply, though, that the President is given carte blanche to
VOL. 779, JANUARY 12, 2016 369 exercise this discretion. Although the Chief Executive wields the exclusive
authority to conduct our foreign relations, this power must still be exercised within
Saguisag vs. Ochoa, Jr. the context and the parameters set by the Constitution, as well as by existing
domestic and international laws. There are constitutional provisions that restrict
modus vivendi, or some other form.213 Consequently, under international law, or limit the President’s prerogative in concluding international agreements, such
the distinction between a treaty and an international agreement or even an as those that involve the following:
executive agreement is irrelevant for purposes of determining international rights
and obligations.
a. The policy of freedom from nuclear weapons within Philippine is clear that they were also keen to preserve the concept of “executive agreements”
territory.221 and the right of the President to enter into such agreements.
b. The fixing of tariff rates, import and export quotas, tonnage and _______________
wharfage dues, and other duties or imposts, which must be pursuant to the
authority granted by Congress.222 223 Id., Art. VI, Sec. 28(4).
_______________ 224 Id., Art. VII, Sec. 20.
225 Id., Art. XVIII, Sec. 25.
217 Pharmaceutical and Health Care Association of the Philippines v. Duque
III, 561 Phil. 386; 535 SCRA 265 (2007); Lim v. Executive Secretary, supra note
69; Secretary of Justice v. Lantion, supra note 17; Philip Morris, Inc. v. Court of 372
Appeals, G.R. No. 91332, 16 July 1993, 224 SCRA 576. 372 SUPREME COURT REPORTS ANNOTATED
218 See: Bayan Muna v. Romulo, supra note 114 (affirming Adolfo v. Court of
First Instance of Zambales, supra note 173); Civil Code, Art. 7. Saguisag vs. Ochoa, Jr.
219 Id.; Nicolas v. Romulo, supra note 39; Gonzales v. Hechanova, supra note
173; Civil Code, Art. 7. What we can glean from the discussions of the Constitutional Commissioners
220 See Constitution, Art. VIII, Sec. 5(2); Civil Code, Art. 7. is that they understood the following realities:
221 Id., Art. II, Sec. 8. 1. Treaties, international agreements, and executive agreements are all
222 Id., Art. VI, Sec. 28(2). constitutional manifestations of the conduct of foreign affairs with their distinct
legal characteristics.
a. Treaties are formal contracts between the Philippines and other
371 States-parties, which are in the nature of international agreements, and
also of municipal laws in the sense of their binding nature.226
VOL. 779, JANUARY 12, 2016 371
b. International agreements are similar instruments, the provisions of
Saguisag vs. Ochoa, Jr. which may require the ratification of a designated number of parties
thereto. These agreements involving political issues or changes in national
c. The grant of any tax exemption, which must be pursuant to a law policy, as well as those involving international agreements of a permanent
concurred in by a majority of all the Members of Congress.223 character, usually take the form of treaties. They may also include
d. The contracting or guaranteeing, on behalf of the Philippines, of commercial agreements, which are executive agreements essentially, but
foreign loans that must be previously concurred in by the Monetary which proceed from previous authorization by Congress, thus dispensing
Board.224 with the requirement of concurrence by the Senate.227
e. The authorization of the presence of foreign military bases, troops, or c. Executive agreements are generally intended to implement a treaty
facilities in the country must be in the form of a treaty duly concurred in already enforced or to determine the details of the implementation thereof
by the Senate.225 that do not affect the sovereignty of the State.228
f. For agreements that do not fall under paragraph 5, the concurrence 2. Treaties and international agreements that cannot be mere executive
of the Senate is required, should the form of the government chosen be a agreements must, by constitutional
treaty. _______________

5. The President had the 226 II Record, Constitutional Commission, p. 544 (31 July 1986).
choice to enter into 227 Id., at p. 545.
EDCA by way of an ex- 228 Id.
ecutive agreement or a
treaty.
373
No court can tell the President to desist from choosing an executive agreement
VOL. 779, JANUARY 12, 2016 373
over a treaty to embody an international agreement, unless the case falls squarely
within Article VIII, Section 25. Saguisag vs. Ochoa, Jr.
As can be gleaned from the debates among the members of the Constitutional
Commission, they were aware that legally binding international agreements were decree, be concurred in by at least two-thirds of the Senate.
being entered into by countries in forms other than a treaty. At the same time, it 3. However, an agreement — the subject of which is the entry of foreign
military troops, bases, or facilities — is particularly restricted. The requirements
are that it be in the form of a treaty concurred in by the Senate; that when x x x. As the President wields vast powers and influence, her conduct in
Congress so requires, it be ratified by a majority of the votes cast by the people in the external affairs of the nation is, as Bayan would put it, “executive
a national referendum held for that purpose; and that it be recognized as a treaty altogether.” The right of the President to enter into or ratify binding
by the other contracting State. executive agreements has been confirmed by long practice.
4. Thus, executive agreements can continue to exist as a species of In thus agreeing to conclude the Agreement thru E/N BF0-028-
international agreements. 03, then President Gloria Macapagal-Arroyo, represented by the Secretary
of Foreign Affairs, acted within the scope of the authority and
That is why our Court has ruled the way it has in several cases. discretion vested in her by the Constitution. At the end of the day, the
In Bayan Muna v. Romulo, we ruled that the President acted within the scope President — by ratifying, thru her deputies, the non-surrender
of her constitutional authority and discretion when she chose to enter into the RP- agreement — did nothing more than discharge a constitutional
U.S. Non-Surrender Agreement in the form of an executive agreement, instead of duty and exercise a prerogative that pertains to her office.
a treaty, and in ratifying the agreement without Senate concurrence. The Court En (Emphases supplied)
Banc discussed this intrinsic presidential prerogative as follows:
Petitioner parlays the notion that the Agreement is of dubious validity,
partaking as it does of the nature of a treaty; hence, it must be duly 375
concurred in by the Senate. x x x x. Pressing its point, petitioner submits VOL. 779, JANUARY 12, 2016 375
that the subject of the Agreement does not fall under any of the subject-
categories that x x x may be covered by an executive agreement, such as Saguisag vs. Ochoa, Jr.
commercial/consular relations, most-favored nation rights, patent rights,
trademark and copyright protection, postal and navigation arrangements Indeed, in the field of external affairs, the President must be given a larger
and settlement of claims. measure of authority and wider discretion, subject only to the least amount of
The categorization of subject matters that may be covered by checks and restrictions under the Constitution.229 The rationale behind this power
international agreements mentioned in Eastern Sea Trading is not cast in and discretion was recognized by the Court in Vinuya v. Romulo, cited earlier.230
stone. There are no hard and fast rules on the propriety of entering, Section 9 of Executive Order No. 459, or the Guidelines in the Negotiation of
on a International Agreements and its Ratification, thus, correctly reflected the
inherent powers of the President when it stated that the DFA “shall determine
whether an agreement is an executive agreement or a treaty.”
374 Accordingly, in the exercise of its power of judicial review, the Court does not
look into whether an international agreement should be in the form of a treaty or
374 SUPREME COURT REPORTS ANNOTATED
an executive agreement, save in cases in which the Constitution or a statute
Saguisag vs. Ochoa, Jr. requires otherwise. Rather, in view of the vast constitutional powers and
prerogatives granted to the President in the field of foreign affairs, the task of the
given subject, into a treaty or an executive agreement as an Court is to determine whether the international agreement is consistent with the
instrument of international relations. The primary consideration in applicable limitations.
the choice of the form of agreement is the parties’ intent and desire to
craft an international agreement in the form they so wish to 6. Executive agreements may
further their respective interests. Verily, the matter of form takes a cover the matter of foreign
back seat when it comes to effectiveness and binding effect of the military forces if it merely
enforcement of a treaty or an executive agreement, as the parties in either involves detail adjustments.
international agreement each labor under the pacta sunt _______________
servanda principle.
xxxx 229 Sinco, supra note 15 at p. 297. See: Vinuya v. Romulo, supra note 17 (on
But over and above the foregoing considerations is the fact that — espousal of the claims of Philippine nationals against a foreign
save for the situation and matters contemplated in Sec. 25, Art. XVIII of government); Pimentel, Jr. v. Office of the Executive Secretary, supra note 15 (on
the Constitution —when a treaty is required, the Constitution does not ratification of international agreements); Secretary of Justice v.
classify any subject, like that involving political issues, to be in the Lantion, supra note 17 (on temporarily withholding of the right to notice and
form of, and ratified as, a treaty. What the Constitution merely hearing during the evaluation stage of the extradition process); People’s Movement
prescribes is that treaties need the concurrence of the Senate by a vote for Press Freedom v. Manglapus, supra note 15 (on the imposition of secrecy in
defined therein to complete the ratification process. treaty negotiations with foreign countries).
xxxx
230 Vinuya v. Romulo, id. Culling from the foregoing discussions, we reiterate the following
pronouncements to guide us in resolving the present controversy:
1. Section 25, Article XVIII of the Constitution, contains stringent
376 requirements that must be fulfilled by the international agreement allowing the
376 SUPREME COURT REPORTS ANNOTATED presence of foreign military bases, troops, or facilities in the Philippines: (a) the
agreement must be in the form of a treaty, and (b) it must be duly concurred in by
Saguisag vs. Ochoa, Jr. the Senate.
2. If the agreement is not covered by the above situation, then the President
The practice of resorting to executive agreements in adjusting the details of a may choose the form of the agreement (i.e., either an executive agreement or a
law or a treaty that already deals with the presence of foreign military forces is treaty), provided that the agreement dealing with foreign military bases, troops,
not at all unusual in this jurisdiction. In fact, the Court has already implicitly or facilities is not the principal agreement that first allows their entry or presence
acknowledged this practice in Lim v. Executive Secretary.231 In that case, the Court in the Philippines.
was asked to scrutinize the constitutionality of the Terms of Reference of 3. The executive agreement must not go beyond the parameters, limitations,
the Balikatan 02-1 joint military exercises, which sought to implement the VFA. and standards set by the law and/or treaty that the former purports to implement;
Concluded in the form of an executive agreement, the Terms of Reference detailed and must not unduly expand the international obligation expressly mentioned or
the coverage of the term “activities” mentioned in the treaty and settled the necessarily implied in the law or treaty.
matters pertaining to the construction of temporary structures for the U.S. troops 4. The executive agreement must be consistent with the Constitution, as well
during the activities; the duration and location of the exercises; the number of as with existing laws and treaties.
participants; and the extent of and limitations on the activities of the U.S. forces. _______________
The Court upheld the Terms of Reference as being consistent with the VFA. It no
longer took issue with the fact that the Balikatan Terms of Reference was not in 233 Id., at p. 291; p. 468.
the form of a treaty concurred in by the Senate, even if it dealt with the regulation
of the activities of foreign military forces on Philippine territory.
In Nicolas v. Romulo,232 the Court again impliedly affirmed the use of an 378
executive agreement in an attempt to adjust the details of a provision of the VFA.
The Philippines and the U.S. entered into the Romulo-Kenney Agreement, which 378 SUPREME COURT REPORTS ANNOTATED
undertook to clarify the detention of a U.S. Armed Forces member, whose case was Saguisag vs. Ochoa, Jr.
pending appeal after his conviction by a trial court for the crime of rape. In testing
the validity of the latter agreement, the Court precisely alluded to one of the In light of the President’s choice to enter into EDCA in the form of an executive
inherent limitations of an executive agreement: it cannot go beyond the terms of agreement, respondents carry the burden of proving that it is a mere
the treaty it purports to implement. It was eventually ruled that the Romulo- implementation of existing laws and treaties concurred in by the Senate. EDCA
Kenney Agreement was “not in accord” with the VFA, since the former was must thus be carefully dissected to ascertain if it remains within the legal
squarely inconsistent with a provision in the treaty requiring that the detention parameters of a valid executive agreement.
be “by Philippine authorities.” Consequently, the
_______________ 7. EDCA is consistent with
the content, purpose, and
231 Lim v. Executive Secretary, supra note 69. framework of the MDT
232 Nicolas v. Romulo, supra note 39. and the VFA.

The starting point of our analysis is the rule that “an executive agreement x x x
377 may not be used to amend a treaty.”234 In Lim v. Executive Secretary and
VOL. 779, JANUARY 12, 2016 377 in Nicolas v. Romulo, the Court approached the question of the validity of
executive agreements by comparing them with the general framework and the
Saguisag vs. Ochoa, Jr. specific provisions of the treaties they seek to implement.
In Lim, the Terms of Reference of the joint military exercises was scrutinized
Court ordered the Secretary of Foreign Affairs to comply with the VFA and by studying “the framework of the treaty antecedents to which the Philippines
“forthwith negotiate with the United States representatives for the appropriate bound itself,”235 i.e., the MDT and the VFA. The Court proceeded to examine the
agreement on detention facilities under Philippine authorities as provided in Art. extent of the term “activities” as contemplated in Articles 1236 and II237 of the VFA.
V, Sec. 10 of the VFA.”233 It later on found that the term “ac-
_______________ place of detention to ensure the United States is in compliance with the terms of
the VFA.”
234 Bayan Muna v. Romulo, supra note 114 at p. 273; p. 263. See also: Nicolas
v. Romulo, id.; Adolfo v. Court of First Instance of Zambales, supra note 173; Abbot
Laboratories v. Agrava, supra note 173. Senate Resolution No. 18, dated 27 May 380
1999, which embodies the concurrence of the Senate in the VFA, stresses in its 380 SUPREME COURT REPORTS ANNOTATED
preamble that “nothing in this Resolution or in the VFA shall be construed
as authorizing the President of the Philippines alone to bind the Philippines Saguisag vs. Ochoa, Jr.
to any amendment of any provision of the VFA.” (Emphases supplied)
235 Lim v. Executive Secretary, supra note 69 at p. 571; p. 752. custody during the trial and detention after conviction.”241Pursuant to Article
236 The provision states: “As used in this Agreement, ‘United States V(6) of the VFA, the custody of a U.S. military personnel resides with U.S. military
personnel’ means United States military and civilian per- authorities during trial. Once there is a finding of guilt, Article V(10) requires that
the confinement or detention be “by Philippine authorities.”
Justice Marvic M.V.F. Leonen’s Dissenting Opinion posits that EDCA
379 “substantially modifies or amends the VFA”242 and follows with an enumeration of
the differences between EDCA and the VFA. While these arguments will be
VOL. 779, JANUARY 12, 2016 379 rebutted more fully further on, an initial answer can already be given to each of
Saguisag vs. Ochoa, Jr. the concerns raised by his dissent.
The first difference emphasized is that EDCA does not only regulate visits as
tivities” was deliberately left undefined and ambiguous in order to permit “a the VFA does, but allows temporary stationing on a rotational basis of U.S.
wide scope of undertakings subject only to the approval of the Philippine military personnel and their contractors in physical locations with permanent
government”238 and thereby allow the parties “a certain leeway in facilities and pre-positioned military materiel.
negotiation.”239 The Court eventually ruled that the Terms of Reference fell within This argument does not take into account that these permanent facilities,
the sanctioned or allowable activities, especially in the context of the VFA and the while built by U.S. forces, are to be owned by the Philippines once
MDT. constructed.243 Even the VFA allowed construction for the benefit of U.S. forces
The Court applied the same approach to Nicolas v. Romulo. It studied the during their temporary visits.
provisions of the VFA on custody and detention to ascertain the validity of the The second difference stated by the dissent is that EDCA allows the
Romulo-Kenney Agreement.240 It eventually found that the two international prepositioning of military materiel, which can include various types of warships,
agreements were not in accord, since the Romulo-Kenney Agreement had fighter planes, bombers, and vessels, as well as land and amphibious vehicles and
stipulated that U.S. military personnel shall be detained at the U.S. Embassy their corresponding ammunition.244
Compound and guarded by U.S. military personnel, instead of by Philippine However, the VFA clearly allows the same kind of equipment, vehicles, vessels,
authorities. According to the Court, the parties “recognized the difference between and aircraft to be brought into the country. Articles VII and VIII of the VFA
_______________ contemplates that
_______________
sonnel temporarily in the Philippines in connection with activities
approvedby the Philippine Government. x x x.” (Emphases supplied) 241 Nicolas v. Romulo, supra note 39 at p. 287; p. 464.
237 The provision states: “It is the duty of United States personnel to respect 242 Dissenting Opinion of Justice Marvic M.V.F. Leonen, p. 630.
the laws of the Republic of the Philippines and to abstain from any activity 243 EDCA, Art. V(1) and (4).
inconsistent with the spirit of this agreement, and, in particular, from any 244 Dissenting Opinion of Justice Leonen, p. 631.
political activity in the Philippines. The Government of the United States
shall take all measures within its authority to ensure that this is done.” (Emphases
supplied) 381
238 Lim v. Executive Secretary, supra note 69 at p. 572; pp. 752-753. VOL. 779, JANUARY 12, 2016 381
239 Id., at p. 575; p. 755.
240 According to the agreement: “[H]e will be detained at the first floor, Rowe Saguisag vs. Ochoa, Jr.
(JUSMAG) Building, U.S. Embassy Compound in a room of approximately 10 x 12
square feet. He will be guarded round the clock by U.S. military personnel. The U.S. equipment, materials, supplies, and other property are imported into or
Philippine police and jail authorities, under the direct supervision of the Philippine acquired in the Philippines by or on behalf of the U.S. Armed Forces; as are
Department of the Interior and Local Government (DILG) will have access to the vehicles, vessels, and aircraft operated by or for U.S. forces in connection with
activities under the VFA. These provisions likewise provide for the waiver of the treaty, but are not necessarily expressed word-for-word in the mother treaty. This
specific duties, taxes, charges, and fees that correspond to these equipment. concern shall be further elucidated in this Decision.
The third difference adverted to by the Justice Leonen’s dissent is that the VFA The fifth difference highlighted by the Dissenting Opinion is that the VFA does
contemplates the entry of troops for training exercises, whereas EDCA allows the not have provisions that may be construed as a restriction on or modification of
use of territory for launching military and paramilitary operations conducted in obligations found in existing statues, including the jurisdiction of courts, local
other states.245 The dissent of Justice Teresita J. Leonardo-De Castro also notes autonomy, and taxation. Implied in this argument is that EDCA contains such
that VFA was intended for non-combat activities only, whereas the entry and restrictions or modifications.249
activities of U.S. forces into Agreed Locations were borne of military necessity or This last argument cannot be accepted in view of the clear provisions of EDCA.
had a martial character, and were therefore not contemplated by the VFA.246 Both the VFA and EDCA ensure Philippine jurisdiction in all instances
This Court’s jurisprudence however established in no uncertain terms that contemplated by both agreements, with the exception of those outlined by the VFA
combat-related activities, as opposed to actual combat, were allowed under the in
MDT and VFA, viz.: _______________

Both the history and intent of the Mutual Defense Treaty and the VFA 248 Dissenting Opinion of Justice Leonen, p. 631.
support the conclusion that combat-related activities as opposed to combat 249 Id.
itself such as the one subject of the instant petition, are indeed
authorized.247
383
Hence, even if EDCA was borne of military necessity, it cannot be said to have
VOL. 779, JANUARY 12, 2016 383
strayed from the intent of the VFA since EDCA’s combat-related components are
allowed under the treaty. Saguisag vs. Ochoa, Jr.
_______________
Articles III-VI. In the VFA, taxes are clearly waived whereas in EDCA, taxes
245 Id. are assumed by the government as will be discussed later on. This fact does not,
246 Concurring and Dissenting Opinion of Justice Teresita J. Leonardo-De therefore, produce a diminution of jurisdiction on the part of the Philippines, but
Castro, p. 528. rather a recognition of sovereignty and the rights that attend it, some of which
247 Lim v. Executive Secretary, supra note 69 at p. 575; p. 755. may be waived as in the cases under Articles III-VI of the VFA.
Taking off from these concerns, the provisions of EDCA must be compared with
those of the MDT and the VFA, which are the two treaties from which EDCA
382 allegedly draws its validity.
382 SUPREME COURT REPORTS ANNOTATED
“Authorized presence” under
Saguisag vs. Ochoa, Jr. the VFA versus “authorized
activities” under EDCA: (1)
Moreover, both the VFA and EDCA are silent on what these activities actually U.S. personnel and (2) U.S.
are. Both the VFA and EDCA deal with the presence of U.S. forces within the contractors.
Philippines, but make no mention of being platforms for activity beyond Philippine
territory. While it may be that, as applied, military operations under either the The OSG argues250 that EDCA merely details existing policies under the MDT
VFA or EDCA would be carried out in the future, the scope of judicial review does and the VFA. It explains that EDCA articulates the principle of defensive
not cover potential breaches of discretion but only actual occurrences or blatantly preparation embodied in Article II of the MDT; and seeks to enhance the defensive,
illegal provisions. Hence, we cannot invalidate EDCA on the basis of the strategic, and technological capabilities of both parties pursuant to the objective of
potentially abusive use of its provisions. the treaty to strengthen those capabilities to prevent or resist a possible armed
The fourth difference is that EDCA supposedly introduces a new concept not attack. Respondent also points out that EDCA simply implements Article I of the
contemplated in the VFA or the MDT: Agreed Locations, Contractors, VFA, which already allows the entry of U.S. troops and personnel into the country.
Prepositioning, and Operational Control.248 Respondent stresses this Court’s recognition in Lim v. Executive Secretary that
As previously mentioned, these points shall be addressed fully and individually U.S. troops and personnel are authorized to conduct activities that promote the
in the latter analysis of EDCA’s provisions. However, it must already be clarified goal of maintaining and developing their defense capability.
that the terms and details used by an implementing agreement need not be found _______________
in the mother treaty. They must be sourced from the authority derived from the
250 Memorandum of OSG, pp. 14-27, Rollo (G.R. No. 212426), pp. 444-457. “United States personnel” means United States military and civilian
personnel temporarily in the territory of the Philippinesin connection with
activities approved by the Philippines, as those terms are defined in
384 the VFA.252
384 SUPREME COURT REPORTS ANNOTATED
“United States forces” means the entity comprising United States
Saguisag vs. Ochoa, Jr. personnel and all property, equipment, and materiel of the United
States Armed Forces present in the territory of the Philippines.253
Petitioners contest251 the assertion that the provisions of EDCA merely “United States contractors” means companies and firms,
implement the MDT. According to them, the treaty does not specifically authorize and their employees, under contract or subcontractto or on behalf of
the entry of U.S. troops in the country in order to maintain and develop the the United States Department of Defense. United States contractors
individual and collective capacities of both the Philippines and the U.S. to resist are not included as part of the definition of United States personnel in
an armed attack. They emphasize that the treaty was concluded at a time when this Agreement, including within the context of the VFA.254
there was as yet no specific constitutional prohibition on the presence of foreign United States forces may contract for any materiel, supplies,
military forces in the country. equipment, and services (including construction) to be furnished or
Petitioners also challenge the argument that EDCA simply implements the undertaken in the territory of the Philippines without restriction as to
VFA. They assert that the agreement covers only short-term or temporary visits of choice of contractor, supplier, or person who provides such materiel,
U.S. troops “from time to time” for the specific purpose of combined military supplies, equipment, or services. Such contracts shall be solicited,
exercises with their Filipino counterparts. They stress that, in contrast, U.S. awarded, and administered in accordance with the laws and regulations of
troops are allowed under EDCA to perform activities beyond combined military the United States.255 (Emphases Supplied)
exercises, such as those enumerated in Articles III(1) and IV(4) thereof.
Furthermore, there is some degree of permanence in the presence of U.S. troops in A thorough evaluation of how EDCA is phrased clarities that the
the country, since the effectivity of EDCA is continuous until terminated. They agreement does not deal with the entry into the country of U.S. personnel
proceed to argue that while troops have a “rotational” presence, this scheme in fact and contractors per se.
fosters their permanent presence. _______________

a. Admission of U.S. mili- 252 EDCA, Art. II(1).


tary and civilian per- 253 Id., Art. II(2).
sonnel into Philippine 254 Id., Art. II(3).
territory is already al- 255 Id., Art. VIII(1).
lowed under the VFA.

We shall first deal with the recognition under EDCA of the presence in the 386
country of three distinct classes of individuals who will be conducting different
types of activities within the Agreed Locations: (1) U.S. military personnel; (2) U.S. 386 SUPREME COURT REPORTS ANNOTATED
civilian Saguisag vs. Ochoa, Jr.
_______________
While Articles I(1)(b)256 and II(4)257 speak of “the right to access and use” the
251 Memorandum of Saguisag, et al., pp. 22-23, 38-49, Rollo (G.R. No. 212426, Agreed Locations, their wordings indicate the presumption that these groups have
Vol. II), pp. 992-993, 1008-1019; Memorandum of Bayan, et al., pp. 35- already been allowed entry into Philippine territory, for which, unlike the VFA,
41, Rollo (G.R. No. 212444), pp. 599-605. EDCA has no specific provision. Instead, Article II of the latter simply alludes to
the VFA in describing U.S. personnel, a term defined under Article I of the treaty
as follows:
385
VOL. 779, JANUARY 12, 2016 385 As used in this Agreement, “United States personnel” means United
States military and civilian personnel temporarily in the Philippines in
Saguisag vs. Ochoa, Jr. connection with activities approved by the Philippine Government.
Within this definition:
personnel; and (3) U.S. contractors. The agreement refers to them as follows:
1. The term “military personnel” refers to military members of 4. United States civilian personnel shall be exempt from visa
the United States Army, Navy, Marine Corps, Air Force, and Coast requirements but shall present, upon demand, valid passports upon
Guard. entry and departure of the Philippines. (Emphases supplied)
2. The term “civilian personnel” refers to individuals who By virtue of Articles I and III of the VFA, the Philippines already allows U.S.
are neither nationals of nor ordinarily resident in the military and civilian personnel to be “temporarily in the Philippines,” so long as
Philippines and who are employed by the United States armed their presence is “in connection with activities approved by the Philippine
forces or Government.” The Philippines, through Article III, even guarantees that it shall
facilitate the admission of U.S. personnel into the country and grant exemptions
_______________ from passport and visa regulations. The VFA does not even limit their temporary
presence to specific locations.
256 According to this provision: 1. This Agreement deepens defense Based on the above provisions, the admission and presence
cooperation between the Parties and maintains and develops their individual and of U.S. military and civilian personnel in Philippine territory are already
collective capacities, in furtherance of Article II of the MDT, which states that ‘the allowed under the VFA, the
Parties separately and jointly by self-help and mutual aid will maintain and _______________
develop their individual and collective capacity to resist armed attack,’ and within
the context of the VFA. This includes: x x x x (b) Authorizing access to Agreed 258 VFA I, Art. I.
Locations in the territory of the Philippines by United States forces on a
rotational basis, as mutually determined by the Parties.
257 According to this provision: “Agreed Locations” means facilities and 388
areasthat are provided by the Government of the Philippines through the AFP 388 SUPREME COURT REPORTS ANNOTATED
and that United States forces, United States contractors, and others as
mutually agreed, shall have the right to access and use pursuant to this Saguisag vs. Ochoa, Jr.
agreement. Such Agreed Locations may be listed in an annex to be appended to
this Agreement, and may be further described in implementing arrangements. treaty supposedly being implemented by EDCA. What EDCA has
effectively done, in fact, is merely provide the mechanism to identify the locations
in which U.S. personnel may perform allowed activities pursuant to the VFA. As
387 the implementing agreement, it regulates and limits the presence of U.S.
personnel in the country.
VOL. 779, JANUARY 12, 2016 387
Saguisag vs. Ochoa, Jr. b. EDCA does not provide t
he legal basis for ad-
who are accompanying the United States armed forces, such as mission of U.S. contrac-
employees of the American Red Cross and the United Services tors into Philippine
Organization.258 territory; their entry
must be sourced from
Article II of EDCA must then be read with Article III of the VFA, which extraneous Philippine
provides for the entry accommodations to be accorded to U.S. military and civilian statutes and regula-
personnel: tions for the admission of
1. The Government of the Philippines shall alien employees or
facilitatethe admission of United States personnel and their business persons.
departure from the Philippines in connection with activities covered by this
agreement. Of the three aforementioned classes of individuals who will be conducting
2. United States military personnel shall be exempt from certain activities within the Agreed Locations, we note that only U.S.
passport and visa regulations upon entering and departing the contractors are not explicitly mentioned in the VFA. This does not mean, though,
Philippines. that the recognition of their presence under EDCA is ipso facto an amendment of
3. The following documents only, which shall be required in respect of the treaty, and that there must be Senate concurrence before they are allowed to
United States military personnel who enter the Philippines; x x x x. enter the country.
Nowhere in EDCA are U.S. contractors guaranteed immediate admission into
the Philippines. Articles III and IV, in fact, merely grant them the right of access
to, and the authority to conduct certain activities within the Agreed Locations.
Since Article II(3) of EDCA specifically leaves out U.S. contractors from the
coverage of the VFA, they shall not be granted the same entry accommodations 390
and privileges as those enjoyed by U.S. military and civilian personnel under the 390 SUPREME COURT REPORTS ANNOTATED
VFA.
Consequently, it is neither mandatory nor obligatory on the part of the Saguisag vs. Ochoa, Jr.
Philippines to admit U.S. contractors into the
them entry pursuant to his absolute and unqualified power to prohibit or
prevent the admission of aliens whose presence in the country would be inimical
389 to public interest.264
In the same vein, the President may exercise the plenary power to expel or
VOL. 779, JANUARY 12, 2016 389
deport U.S. contractors265 as may be necessi-
Saguisag vs. Ochoa, Jr.
_______________
country.259 We emphasize that the admission of aliens into Philippine territory
is “a matter of pure permission and simple tolerance which creates no obligation opinion of the Commissioner of Immigration no hardship would result from
on the part of the government to permit them to stay.”260 Unlike U.S. personnel their admission; (11) Persons accompanying an excluded person who is helpless
who are accorded entry accommodations, U.S. contractors are subject to Philippine from mental or physical disability or infancy, when the protection or guardianship
immigration laws.261 The latter must comply with our visa and passport of such accompanying person or persons is required by the excluded person, as
regulations262 and prove that they are not subject to exclusion under any provision shall be determined by the Commissioner of Immigration; (12) Children under
of Philippine immigration laws.263The President may also deny fifteen years of age, unaccompanied by or not coming to a parent, except that any
_______________ such children may be admitted in the discretion of the Commissioner of
Immigration, if otherwise admissible; (13) Stowaways, except that any stowaway
259 See: Djumantan v. Domingo, 310 Phil. 848; 240 SCRA 746 (1995). may be admitted in the discretion of the Commissioner of Immigration, if
260 Id., at p. 854; p. 752. otherwise admissible; (14) Persons coming to perform unskilled manual labor in
261 Commonwealth Act No. 613 (The Philippine Immigration Act of 1940, as pursuance of a promise or offer of employment, express or implied, but this
amended). provision shall not apply to persons bearing passport visas authorized by Section
262 Id., Secs. 10 & 11. Twenty of this Act; (15) Persons who have been excluded or deported from
263 Id., Secs. 29 & 30. Under Section 29, the following classes of aliens shall the Philippines, but this provision may be waived in the discretion of the
be excluded from entry into the Philippines: (1) Idiots or insane persons and Commissioner of Immigration: Provided, however, That the Commissioner of
persons who have been insane; (2) Persons afflicted with a loathsome or dangerous Immigration shall not exercise his discretion in favor of aliens excluded or deported
contagious disease, or epilepsy; (3) Persons who have been convicted of a crime on the ground of conviction for any crime involving moral turpitude or for any
involving moral turpitude; (4) Prostitutes, or procurers, or persons coming for any crime penalized under Sections [45] and [46] of this Act or on the ground of having
immoral purposes; (5) Persons likely to become, public charge; (6) Paupers, engaged in hoarding, black-marketing or profiteering unless such aliens have
vagrants, and beggars; (7) Persons who practice polygamy or who believe in or previously resided in the Philippines immediately before his exclusion or
advocate the practice of polygamy; (8) Persons who believe in or deportation for a period of ten years or more or are married to native Filipino
advocate the overthrow by force and violence of the Government of the women; (16) Persons who have been removed from the Philippines at the expense
Philippines, or of constituted lawful authority, or who disbelieve in or are opposed of the Government of the Philippines, as indigent aliens, under the provisions of
to organized government, or who advocate the assault or assassination of Section [43] of this Act, and who have not obtained the consent of the Board of
public officials because of their office, or who advocate or teach principles, Commissioners to apply for readmission; and (17) Persons not properly
theories, or ideas contrary to the Constitution of the Philippines or documented for admission as may be required under the provisions of this Act.
advocate or teach the unlawful destruction of property, or who are members of or (Emphasis supplied)
affiliated with any organization entertaining or teaching such doctrines; (9) 264 Supra note 259.
Persons over fifteen years of age, physically capable of reading, who cannot read 265 Administrative Code of 1987, Book III (Office of the President), Title I
printed matter in ordinary use in any language selected by the alien, but this (Powers of the President), Secs. 8 & 11, in relation to
provision shall not apply to the grandfather, grandmother, father, mother, wife,
husband or child of a Philippine citizen or of an alien lawfully resident in the
Philippines; (10) Persons who are members of a family accompanying an excluded 391
alien, unless in the VOL. 779, JANUARY 12, 2016 391
Saguisag vs. Ochoa, Jr. c. Authorized activities of
U.S. military and civil-
tated by national security, public safety, public health, public morals, and ian personnel within
national interest.266 They may also be deported if they are found to be illegal or Philippine territory are
undesirable aliens pursuant to the Philippine Immigration Act267 and the Data in furtherance of the
Privacy Act.268 MDT and the VFA.
_______________
We begin our analysis by quoting the relevant sections of the MDT and the
VFA that pertain to the activities in which U.S. military and civilian personnel
Commonwealth Act No. 613 (The Philippine Immigration Act of 1940), Sec. 52
may engage:
and Act. No. 2711 (Revised Administrative Code of 1917), Sec. 69. See: Djumantan
_______________
v. Domingo, id.; Teo Tung v. Machlan, 60 Phil. 916 (1934).
266 See: Commonwealth Act No. 613 (The Philippine Immigration Act of 1940,
as amended), Secs. 6, 12, 28 & 29; Djumantan v. Domingo, id.; Salazar v. ner whatsoever lends assistance, financial or otherwise, to the dissemination
Achacoso, 262 Phil. 160; 183 SCRA 145 (1990); Ledesma, Ronaldo P., Deportation of such doctrines; (9) Any alien who commits any of the acts described in Sections
Proceedings: Practice, Precedents, and Procedures, p. 96 (2013). [45] and [46] of this Act, independent of criminal action which may be brought
267 Id., Sec. 37. The provision enumerates as follows: (1) Any alien who against him: x x x; (10) Any alien who, at any time within five years after entry,
enters the Philippines x x x by means of false and misleading shall have been convicted of violating the provisions of the Philippine
statements or without inspection and admission by the immigration Commonwealth Act [653], otherwise known as the Philippine Alien
authorities x x x; (2) (Any alien who enters the Philippines x x x, who was not Registration Act of 1941, or who, at any time after entry, shall have been
lawfully admissible at the time of entry; (3) Any alien who, x x x, is convicted convicted more than once of violating the provisions of the same Act; (11) Any alien
in the Philippines and sentenced for a term of one year or more for a crime who engages in profiteering, hoarding, or blackmarketing, independent of
involving moral turpitude committed within five years after his entry to the any criminal action which may be brought against him; (12) Any alien who
Philippines, or who, at any time after such entry, is so convicted and sentenced is convicted of any offense penalized under Commonwealth Act [473],
more than once; (4) Any alien who is convicted and sentenced for a violation of the otherwise known as the Revised Naturalization Laws of the Philippines, or
law governing prohibited drugs; (5) Any alien who practices prostitution or is an any law relating to acquisition of Philippine citizenship; (13) Any alien who
inmate of a house of prostitution or is connected with the management of a house defrauds his creditor by absconding or alienating properties to prevent them from,
of prostitution, or is a procurer; (6) Any alien who becomes a public charge within being attached or executed. (Emphasis supplied)
five years after entry from causes not affirmatively shown to have arisen 268 Republic Act No. 10173, Sec. 34. According to the provision, “[i]f the
subsequent to entry; (7) Any alien who remains in the Philippines in violation of offender is an alien, he or she shall, in addition to the penalties herein
any limitation or condition under which he was admitted as a prescribed, be deported without further proceedings after serving the
nonimmigrant; (8) Any alien who believes in, advises, advocates or teaches penalties prescribed.”
the overthrow by force and violence of the Government of the
Philippines, or of constituted law and authority, or who disbelieves in or is
opposed to organized government or who advises, advocates, or teaches the assault 393
or assassination of public officials because of their office, or who advises, advocates, VOL. 779, JANUARY 12, 2016 393
or teaches the unlawful destruction of property, or who is a member of or affiliated Saguisag vs. Ochoa, Jr.
with any organization entertaining, advocating or teaching such doctrines, or who
in any man- Mutual Defense Treaty
Article II
392 In order more effectively to achieve the objective of this Treaty, the
392 SUPREME COURT REPORTS ANNOTATED Parties separately and jointly by self-help and mutual
Saguisag vs. Ochoa, Jr. aid will maintain and develop their individual and collective
capacity to resist armed attack.
In contrast, Article III(5) of the VFA requires a request for removal from the
Philippine government before a member of the U.S. personnel may be “dispos[ed] Article III
x x x outside of the Philippines.”
The Parties, through their Foreign Ministers or their deputies, the Philippines at any time, free from export duties, taxes, and other
will consult together from time to time regarding the implementation similar charges. x x x.
of this Treaty and whenever in the opinion of either of them the territorial
integrity, political independence or security of either of the Parties is Article VIII – Movement of Vessels and Aircraft
threatened by external armed attack in the Pacific.
1. Aircraft operated by or for the United States armed forces
Visiting Forces Agreement may enter the Philippines upon approval of the Government of the
Preamble Philippines in accordance with procedures stipulated in implementing
arrangements.
xxx 2. Vessels operated by or for the United States armed forces
Reaffirming their obligations under the Mutual Defense may enter the Philippines upon approval of the Government of the
Treaty of August 30, 1951; Philippines. The movement of vessels shall be in accordance with
Noting that from time to time elements of the United States armed international custom and practice governing such vessels,
forces may visit the Republic of the Philippines; and such agreed implementing arrangements as necessary. x x x
Considering that cooperation between the United States and the (Emphases supplied)
Republic of the Philippines promotes their common security interests.
xxx
395
Article I – Definitions VOL. 779, JANUARY 12, 2016 395

As used in this Agreement, “United States personnel” means United Saguisag vs. Ochoa, Jr.
States military and civilian personnel temporarily in the Philippines in
connection with activi- Manifest in these provisions is the abundance of references to the creation of
further “implementing arrangements” including the identification of “activities [to
be] approved by the Philippine Government.” To determine the parameters of
394 these implementing arrangements and activities, we referred to the content,
purpose, and framework of the MDT and the VFA.
394 SUPREME COURT REPORTS ANNOTATED
By its very language, the MDT contemplates a situation in which both
Saguisag vs. Ochoa, Jr. countries shall engage in joint activities, so that they can maintain and develop
their defense capabilities. The wording itself evidently invites a reasonable
ties approved by the Philippine Government. Within this construction that the joint activities shall involve joint military trainings,
definition: x x x. maneuvers, and exercises. Both the interpretation269 and the subsequent
practice270 of the parties show that the MDT inde-
Article II – Respect for Law _______________

It is the duty of United States personnel to respect the laws of 269 See: Secretary of Justice v. Lantion, supra note 17. According to the Court:
the Republic of the Philippines and to abstain from any activity “An equally compelling factor to consider is the understanding of the
inconsistent with the spirit of this agreement, and, in particular, parties themselves to the RP-US Extradition Treaty x x x. The rule is recognized
from any political activity in the Philippines. The Government of the that while courts have the power to interpret treaties, the meaning given them
United States shall take all measures within its authority to ensure that by the departments of government particularly charged with their
this is done. negotiation and enforcement is accorded great weight. x x x This
interpretation by the two governments cannot be given scant significance. It will
Article VII – Importation and Exportation be presumptuous for the Court to assume that both governments did not
understand the terms of the treaty they concluded.” (Emphasis supplied)
1. United States Government equipment, materials, supplies, 270 See Status of Forces Agreement of 1993, supra note 70. The International
and other property imported into or acquired in the Philippines by Law Commission explains that the subsequent practice of states in the application
or on behalf of the United States armed forces in connection with of the treaty may be taken into account in ascertaining the parties’ agreement in
activities to which this agreement applies, shall be free of all the interpretation of that treaty. This is “well-established in the jurisprudence of
Philippine duties, taxes and other similar charges. Title to such property international tribunals” even before the Vienna Convention on the Law of Treaties
shall remain with the United States, which may remove such property from
was concluded. See International Law Commission, Draft Articles on the Law of VOL. 779, JANUARY 12, 2016 397
Treaties with Commentaries, 1966 (II) Y.B.I.L.C. 187 at pp. 221-222
(citing Russian Claim for Indemnities [Russia/Turkey], XI R.I.A.A. 421, 433 [1912] Saguisag vs. Ochoa, Jr.
[Nov. 11]; Competence of the ILO to Regulate Agricultural Labour, 1922 P.C.I.J.
[ser. B] No. 2, 39 [Aug. 12]; Interpretation of Article 3, paragraph 2, of the Treaty term armed attack includes “an armed attack on the metropolitan
of Lausanne,1925 P.C.I.J. [ser. B] No. 12, 24 [Nov. 21]; Brazilian territory of either of the Parties, or on the island territories under its
jurisdiction in the Pacific or on its armed forces, public vessels or aircraft in
the Pacific.” Surely, in maintaining and developing our defense capabilities, an
396 assessment or training will need to be performed, separately and jointly by self-
help and mutual aid, in the territories of the contracting parties. It is reasonable
396 SUPREME COURT REPORTS ANNOTATED
to conclude that the assessment of defense capabilities would entail understanding
Saguisag vs. Ochoa, Jr. the terrain, wind flow patterns, and other environmental factors unique to the
Philippines.
pendently allows joint military exercises in the country. Lim v. Executive It would also be reasonable to conclude that a simulation of how to respond to
Secretary271 and Nicolas v. Romulo272 recognized that Balikatan exercises, which attacks in vulnerable areas would be part of the training of the parties to maintain
are activities that seek to enhance and develop the strategic and technological and develop their capacity to resist an actual armed attack and to test and validate
capabilities of the parties to resist an armed attack, “fall squarely under the the defense plan of the Philippines. It is likewise reasonable to imagine that part
provisions of the RP-US MDT.”273 In Lim, the Court especially noted that the of the training would involve an analysis of the effect of the weapons that may be
Philippines and the U.S. continued to conduct joint military exercises even after used and how to be prepared for the eventuality. This Court recognizes that all of
the expiration of the MBA and even before the conclusion of the VFA. 274 These this may require training in the area where an armed attack might be directed at
activities presumably related to the Status of Forces Agreement, in which the the Philippine territory.
parties agreed on the status to be accorded to U.S. military and civilian personnel The provisions of the MDT must then be read in conjunction with those of the
while conducting activities in the Philippines in relation to the MDT. 275 VFA.
Further, it can be logically inferred from Article V of the MDT that Article I of the VFA indicates that the presence of U.S. military and civilian
these joint activities may be conducted on Philippine or on U.S. soil. The article personnel in the Philippines is “in connection with activities approved by the
expressly provides that the Philippine Government.” While the treaty does not expressly enumerate or detail
_______________ the nature of activities of U.S. troops in the country, its Preamble makes explicit
references to the reaffirmation of the obligations of both countries under the MDT.
Loans, 1929 P.C.I.J. [ser. A] No. 21, 119 [Jul. 12]; and Corfu Channel [U.K. v. These obligations include the strengthening of international and regional security
Albania], 1949 I.C.J. 4, 25 [Apr. 9]). in the Pacific area and the promotion of common security interests.
271 Lim v. Executive Secretary, supra note 69 at pp. 571-572; p. 752. The Court has already settled in Lim v. Executive Secretary that the phrase
272 Nicolas v. Romulo, supra note 39 at p. 284; p. 461. “activities approved by the Philippine Government” under Article I of the VFA was
273 Id. intended to be am-
274 Lim v. Executive Secretary, supra note 69 at p. 575; p. 755; Joint Report of
the Committee on Foreign Relations and the Committee on National Defense and
Security reproduced in Senate of the Philippines, supra note 69 at p. 206. 398
275 Status of Forces Agreement of 1993, supra note 70. According to Note No. 398 SUPREME COURT REPORTS ANNOTATED
93-2301 dated 11 June 1993 of the DFA to the U.S. Embassy, “The [DFA] x x x has
Saguisag vs. Ochoa, Jr.
the honor to reaffirm its position that all U.S. military and civilian personnel
present in the Philippines participating in activities undertaken in relation to the
Mutual Defense Treaty will be accorded the same status as the U.S. Embassy’s biguous in order to afford the parties flexibility to adjust the details of the
technical and administrative personnel who are qualified to enter the Philippines purpose of the visit of U.S. personnel.276 In ruling that the Terms of Reference for
under existing Philippine laws. The Department further proposes that the the Balikatan Exercises in 2002 fell within the context of the treaty, this Court
procedures as well as the arrangements for these MDT-related activities are to be explained:
mutually agreed upon by the MDB, subject to the guidelines of the Council of
Ministers.” After studied reflection, it appeared farfetched that the ambiguity
surrounding the meaning of the word “activities” arose from
accident. In our view, it was deliberately made that way to give both
397 parties a certain leeway in negotiation. In this manner, visiting US
forces may sojourn in Philippine territory for purposes other than
military. As conceived, the joint exercises may include training on new Siazon clarified that it is not the VFA by itself that determines
techniques of patrol and surveillance to protect the nation’s marine what activities will be conducted between the armed forces of the U.S.
resources, sea search-and-rescue operations to assist vessels in distress, and the Philippines. The VFA regulates and provides the legal
disaster relief operations, civic action projects such as the building of school framework for the presence, conduct and legal status of U.S.
houses, medical and humanitarian missions, and the like. personnel while they are in the country for visits, joint exercises and other
Under these auspices, the VFA gives legitimacy to the current related activities. (Emphases supplied)
Balikatan exercises. It is only logical to assume that “Balikatan 02- _______________
1,” a “mutual anti-terrorism advising, assisting and training
exercise,” falls under the umbrella of sanctioned or allowable 277 Joint Report of the Committee on Foreign Relations and the Committee
activities in the context of the agreement. Both the history and intent on National Defense and Security reproduced in Senate of the Philippines, id., at
of the Mutual Defense Treaty and the VFA support the conclusion that pp. 205-206, 231.
combat-related activities — as opposed to combat itself — such as the one
subject of the instant petition, are indeed authorized. (Emphases supplied)
400
The joint report of the Senate committees on foreign relations and on national
400 SUPREME COURT REPORTS ANNOTATED
defense and security further explains
_______________ Saguisag vs. Ochoa, Jr.

276 Lim v. Executive Secretary, supra note 69. See also Joint Report of the What can be gleaned from the provisions of the VFA, the joint report
Committee on Foreign Relations and the Committee on National Defense and of the Senate committees on foreign relations and on national defense
Security reproduced in Senate of the Philippines, supra note 69 at pp. 230-231. and security, and the ruling of this Court in Lim is that the “activities”
referred to in the treaty are meant to be specified
and identified in further agreements. EDCA is one such agreement.
399 EDCA seeks to be an instrument that enumerates the Philippine approved
activities of U.S. personnel referred to in the VFA. EDCA allows U.S. military and
VOL. 779, JANUARY 12, 2016 399
civilian personnel to perform “activities approved by the Philippines, as those
Saguisag vs. Ochoa, Jr. terms are defined in the VFA”278 and clarifies that these activities include those
conducted within the Agreed Locations:
the wide range and variety of activities contemplated in the VFA, and how
these activities shall be identified:277 1. Security cooperation exercises; joint and combined training activities;
humanitarian assistance and disaster relief activities; and such other
These joint exercises envisioned in the VFA are not limited to activities as may be agreed upon by the Parties.279
combat-related activities; they have a wide range and variety. They 2. Training; transit; support and related activities; refueling of aircraft;
include exercises that will reinforce the AFP’s ability to acquire new bunkering of vessels; temporary maintenance of vehicles, vessels, and
techniques of patrol and surveillance to protect the country’s maritime aircraft; temporary accommodation of personnel; communications;
resources; sea-search and rescue operations to assist ships in distress; prepositioning of equipment, supplies, and materiel; deployment of forces
and disaster-relief operations to aid the civilian victims of natural and materiel; and such other activities as the Parties may agree.280
calamities, such as earthquakes, typhoons and tidal waves. 3. Exercise of operational control over the Agreed Locations for
xxxx construction activities and other types of activity, including alterations and
Joint activities under the VFA will include combat maneuvers; training improvements thereof.281
in aircraft maintenance and equipment repair; civic-action projects; and _______________
consultations and meetings of the Philippine-U.S. Mutual Defense
Board. It is at the level of the Mutual Defense Board — which is 278 EDCA, Art. II(1).
headed jointly by the Chief of Staff of the AFP and the Commander in Chief 279 Id., Art. I(3).
of the U.S. Pacific Command — that the VFA exercises are planned. 280 Id., Art. III(1).
Final approval of any activityinvolving U.S. forces is, 281 Id., Art. III(4) & (6).
however, invariably given by the Philippine Government.
xxxx
401
VOL. 779, JANUARY 12, 2016 401 US Forces operate independently during field training exercises
(FTX). AFP and US Unit Commanders will retain command over
Saguisag vs. Ochoa, Jr. their respective forces under the overall authority of the Exercise
Co-Directors. RP and US participants shall comply with operational
4. Exercise of all rights and authorities within the Agreed Locations instructions of the AFP during the FTX.
that are necessary for their operational control or defense, including the The exercise shall be conducted and completed within a period of not
adoption of appropriate measures to protect U.S. forces and contractors. 282 more than six months, with the projected participation of 660 US personnel
5. Use of water, electricity, and other public utilities.283 and 3,800 RP Forces. The Chief of Staff, AFP shall direct the Exercise Co-
6. Operation of their own telecommunication systems, including the Directors to wind up and terminate the Exercise and other activities within
utilization of such means and services as are required to ensure the full the six month Exercise period.
ability to operate telecommunication systems, as well as the use of the The Exercise is a mutual counter-terrorism advising, assisting
necessary radio spectrum allocated for this purpose.284 and training Exercise relative to Philippine efforts against the ASG, and
will be conducted on the Island of Basilan. Further advising, assisting and
According to Article I of EDCA, one of the purposes of these activities is to training exercises shall be conducted in Malagutay and the Zamboanga
maintain and develop, jointly and by mutual aid, the individual and collective area. Related activities in Cebu will be for support of the Exercise.
capacities of both countries to resist an armed attack. It further states that the _______________
activities are in furtherance of the MDT and within the context of the VFA.
We note that these planned activities are very similar to those under the Terms 286 Lim v. Executive Secretary, supra note 69 at pp. 565-566; pp. 746-747.
of Reference285 mentioned in Lim. Both EDCA and the Terms of Reference
authorize the U.S. to perform the following: (a) participate in training exercises;
(b) retain command over their forces; (c) establish temporary structures in the 403
country; (d) share in the use of their respec-
_______________ VOL. 779, JANUARY 12, 2016 403
Saguisag vs. Ochoa, Jr.
282 Id., Art. VI(3).
283 Id., Art. VII(1). x x x x.
284 Id., Art. VII(2). US exercise participants shall not engage in combat, without
285 According to the Agreed Minutes of the Discussion between the former prejudice to their right of self-defense.
Philippine Vice President/Secretary of Foreign Affairs Teofisto T. Guingona, Jr. These terms of Reference are for purposes of this Exercise only and do
and U.S. Assistant Secretary of State for East Asian and Pacific Affairs James not create additional legal obligations between the US Government and the
Kelly, both countries approved the Terms of Agreement of the Balikatan exercises. Republic of the Philippines.
See: Rollo (G.R. No. 151445), pp. 99-100.
II. EXERCISE LEVEL
1. TRAINING
402 a. The Exercise shall involve the conduct of mutual military
402 SUPREME COURT REPORTS ANNOTATED assisting, advising and training of RP and US Forces with the primary
objective of enhancing the operational capabilities of both forces to combat
Saguisag vs. Ochoa, Jr. terrorism.
b. At no time shall US Forces operate independently within RP
tive resources, equipment and other assets; and (e) exercise their right to self- territory.
defense. We quote the relevant portion of the Terms and Conditions as follows:286 c. Flight plans of all aircraft involved in the exercise will comply with
the local air traffic regulations.
I. POLICY LEVEL 2. ADMINISTRATION & LOGISTICS
xxxx xxxx
No permanent US basing and support facilities shall be a. RP and US participating forces may share, in accordance with their
established. Temporary structures such as those for troop billeting, respective laws and regulations, in the use of their resources,
classroom instruction and messing may be set up for use by RP and equipment and other assets. They will use their respective logistics
US Forces during the Exercise. channels. x x x. (Emphases supplied)
The Exercise shall be implemented jointly by RP and US Exercise Co-
Directors under the authority of the Chief of Staff, AFP. In no instance will
After a thorough examination of the content, purpose, and framework of the Justice Brion’s dissent argues that the presence of U.S. forces under EDCA is
MDT and the VFA, we find that EDCA has remained within the parameters set in “more permanent” in nature.289 However, this argument has not taken root by
these two treaties. Just like the Terms of Reference mentioned in Lim, mere virtue of a simple glance at its provisions on the effectivity period. EDCA does not
adjustments in detail to implement the MDT and the VFA can be in the form of grant permanent bases, but rather temporary rotational access to facilities for
executive agreements. efficiency. As Professor Aileen S.P. Baviera notes:

The new EDCA would grant American troops, ships and planes
404 rotational access to facilities of the Armed Forces of the Philippines — but
404 SUPREME COURT REPORTS ANNOTATED not permanent bases which are prohibited under the Philippine
Constitution — with the result of reducing response time should an
Saguisag vs. Ochoa, Jr. external threat from a common adversary crystallize.290

Petitioners assert287 that the duration of the activities mentioned in EDCA is EDCA is far from being permanent in nature compared to the practice of states
no longer consistent with the temporary nature of the visits as contemplated in the as shown in other defense cooperation agreements. For example, Article XIV(1) of
VFA. They point out that Article XII(4) of EDCA has an initial term of 10 years, a the U.S.-Romania defense agreement provides the following:
term automatically renewed unless the Philippines or the U.S. terminates the
agreement. According to petitioners, such length of time already has a badge of This Agreement is concluded for an indefinite period and shall
permanency. enter into force in accordance with the internal laws of each Party x x x.
In connection with this, Justice Teresita J. Leonardo-De Castro likewise (emphasis supplied)
argues in her Concurring and Dissenting Opinion that the VFA contemplated mere
temporary visits from U.S. forces, whereas EDCA allows an unlimited period for Likewise, Article 36(2) of the US-Poland Status of Forces Agreement reads:
U.S. forces to stay in the Philippines.288
However, the provisions of EDCA directly contradict this argument by limiting This Agreement has been concluded for an indefinite period of
itself to 10 years of effectivity. Although this term is automatically renewed, the time. It may be terminated by written notification by either Party and in
process for terminating the agreement is unilateral and the right to do so that event it terminates 2 years after the receipt of the notification.
automatically accrues at the end of the 10-year period. Clearly, this method does (emphasis supplied)
not create a permanent obligation. _______________
Drawing on the reasoning in Lim, we also believe that it could not have been
by chance that the VFA does not include a maximum time limit with respect to the 289 Dissenting Opinion of Justice Brion, pp. 607-608.
presence of U.S. personnel in the country. We construe this lack of specificity as a 290 Baviera, Aileen S.P., Implications of the US-Philippines Enhanced
deliberate effort on the part of the Philippine and the U.S. governments to leave Defense Cooperation Agreement, Asia Pacific Bulletin No. 292, 9 May 2014.
out this aspect and reserve it for the “adjustment in detail” stage of the
implementation of the treaty. We interpret the subsequent, unconditional
concurrence of the Senate in the entire text of the VFA as an implicit grant to the 406
President of a margin of appreciation in determining the duration of the
“temporary” presence of U.S. personnel in the country. 406 SUPREME COURT REPORTS ANNOTATED
_______________ Saguisag vs. Ochoa, Jr.

287 Memorandum of Saguisag, et al., pp. 43-46, Rollo (G.R. No. 212426, Vol. Section VIII of US-Denmark Mutual Support Agreementsimilarly provides:
II), pp. 1013-1016.
288 Concurring and Dissenting Opinion of Justice Teresita J. Leonardo-De 8.1 This Agreement, which consists of a Preamble, SECTIONS I-VIII,
Castro, p. 526. and Annexes A and B, shall become effective on the date of the last
signature affixed below and shall remain in force until terminated by
the Parties, provided that it may be terminated by either Party upon 180
405 days written notice of its intention to do so to the other Party. (emphasis
VOL. 779, JANUARY 12, 2016 405 supplied)

Saguisag vs. Ochoa, Jr. On the other hand, Article XXI(3) of the US-Australia Force Posture
Agreement provides a longer initial term:
291 See Constitution, Art. VII, Sec. 18, in relation to Art. II, Sec. 3.
3. This Agreement shall have an initial term of 25 years and 292 See Administrative Code of 1987, Book IV (Executive Branch), Title I
thereafter shall continue in force, but may be terminated by either (Foreign Affairs), Sec. 3(1), in relation to Constitution, Art. VII, Sec. 1 and Art. II,
Party at any time upon one year’s written notice to the other Party through Sec. 3; Akbayan Citizens Action Party v. Aquino, supra note 15; Pimentel, Jr. v.
diplomatic channels. (emphasis supplied) Office of the Executive Secretary, supra note 15; Bayan (Bagong Alyansang
Makabayan) v. Zamora, supra note 23.
The phrasing in EDCA is similar to that in the U.S.-Australia treaty but with 293 Vinuya v. Romulo, supra note 17.
a term less than half of that is provided in the latter agreement. This means that 294 Id.
EDCA merely follows the practice of other states in not specifying a non-extendible
maximum term. This practice, however, does not automatically grant a badge of
permanency to its terms. Article XII(4) of EDCA provides very clearly, in fact, that 408
its effectivity is for an initial term of 10 years, which is far shorter than the terms 408 SUPREME COURT REPORTS ANNOTATED
of effectivity between the U.S. and other states. It is simply illogical to conclude
that the initial, extendible term of 10 years somehow gives EDCA provisions a Saguisag vs. Ochoa, Jr.
permanent character.
The reasoning behind this interpretation is rooted in the constitutional role of Evidently, the fact that the VFA does not provide specificity in regard to the
the President who, as Commander-in-Chief of our armed forces, is the principal extent of the “temporary” nature of the visits of U.S. personnel does not suggest
strategist of the nation and, as such, duty-bound to defend our national sover- that the duration to which the President may agree is unlimited. Instead, the
boundaries of the meaning of the term temporary in Article I of the treaty must be
measured depending on the purpose of each visit or activity.295 That purpose must
407 be analyzed on a case-by-case basis depending on the factual circumstances
surrounding the conclusion of the implementing agreement. While the validity of
VOL. 779, JANUARY 12, 2016 407
the President’s actions will be judged under less stringent standards, the power of
Saguisag vs. Ochoa, Jr. this Court to determine whether there was grave abuse of discretion remains
unimpaired.
eignty and territorial integrity;291 who, as chief architect of our foreign
relations, is the head policymaker tasked to assess, ensure, and protect our d. Authorized activities per-
national security and interests;292 who holds the most comprehensive and most formed by U.S. contractors
confidential information about foreign countries293 that may affect how we conduct within Philippine territory
our external affairs; and who has unrestricted access to highly classified military — who were legitimately
intelligence data294 that may threaten the life of the nation. Thus, if after a permitted to enter the coun-
geopolitical prognosis of situations affecting the country, a belief is engendered try independent of EDCA —
that a much longer period of military training is needed, the President must be are subject to relevant Phil-
given ample discretion to adopt necessary measures including the flexibility to set ippine statutes and regula-
an extended timetable. tions and must be consistent
Due to the sensitivity and often strict confidentiality of these concerns, we with the MDT and the VFA.
acknowledge that the President may not always be able to candidly and openly
discuss the complete situation being faced by the nation. The Chief Executive’s Petitioners also raise296 concerns about the U.S. government’s purported
hands must not be unduly tied, especially if the situation calls for crafting practice of hiring private security contractors in other countries. They claim that
programs and setting timelines for approved activities. These activities may be these contractors —
necessary for maintaining and developing our capacity to resist an armed attack, _______________
ensuring our national sovereignty and territorial integrity, and securing our
national interests. If the Senate decides that the President is in the best position 295 See generally Joint Report of the Committee on Foreign Relations and the
to define in operational terms the meaning of temporary in relation to the visits, Committee on National Defense and Security reproduced in Senate of the
considered individually or in their totality, the Court must respect that policy Philippines, supra note 69 at p. 206. According to the report: “The Mutual Defense
decision. If the Senate feels that there is no need to set a time limit to these visits, Board programs an average of 10 to 12 exercises annually. Participating U.S.
neither should we. personnel, numbering from 10 to more than 1,000, stay in Philippine territory from
_______________ four days to four weeks, depending on the nature of the exercise.”
296 Memorandum of Bayan, pp. 47-51, Rollo (G.R. No. 212444), pp. 611-615.
Hence, any other entity brought into the Philippines by virtue of EDCA must
subscribe to corporate and civil requirements imposed by the law, depending on
409 the entity’s corporate structure and the nature of its business.
VOL. 779, JANUARY 12, 2016 409 That Philippine laws extraneous to EDCA shall govern the regulation of the
activities of U.S. contractors has been clear even to some of the present members
Saguisag vs. Ochoa, Jr. of the Senate.
For instance, in 2012, a U.S. Navy contractor, the Glenn Marine, was accused
one of which has already been operating in Mindanao since 2004 — have been of spilling fuel in the waters off Manila Bay.304 The Senate Committee on Foreign
implicated in incidents or scandals in other parts of the globe involving rendition, Relations and the Senate Committee on Environment and Natural Resources
torture and other human rights violations. They also assert that these contractors chairperson claimed environmental and procedural violations by the
employ paramilitary forces in other countries where they are operating. contractor.305 The U.S. Navy investigated the contractor
Under Articles III and IV of EDCA, U.S. contractors are authorized to perform _______________
only the following activities:
1. Training; transit; support and related activities; refueling of aircraft; 302 Executive Order No. 184 (2015).
bunkering of vessels; temporary maintenance of vehicles, vessels, and 303 Republic Act No. 5487 — The Private Security Agency Law, as amended
aircraft; temporary accommodation of personnel; communications; by P.D. No. 11.
prepositioning of equipment, supplies, and materiel; deployment of forces 304 Glenn Defense: SBMA suspension doesn’t cover all our functions, Rappler,
and materiel; and such other activities as the Parties may agree. 297 available at <http://www.rappler.com/nation/16688-glenn-defense-sbma-
2. Prepositioning and storage of defense equipment, supplies, and suspension-does-not-cover-all-functions> (last visited 3 December 2015).
materiel, including delivery, management, inspection, use, maintenance, 305 Id.; Bordadora, Norman, US Navy contractor liable for Subic waste
and removal of such equipment, supplies and materiel.298 dumping,Philippine Daily Inquirer, available at
3. Carrying out of matters in accordance with, and to the extent <http://globalnation.inquirer.net/63765/us-navy-contractor-liable-for-subic-waste-
permissible under, U.S. laws, regulations, and policies.299 dumping> (last visited 3 December 2015); Santos, Matikas, US navy contractor
dumped millions of liters of wastes in Subic,Philippine Daily Inquirer, available at
EDCA requires that all activities within Philippine territory be in accordance <http://globalnation.inquirer.
with Philippine law. This means that certain privileges denied to aliens are
likewise denied to foreign military contractors. Relevantly, providing
security300 and carrying, owning, and possessing firearms301 are illegal for foreign 411
civilians.
_______________ VOL. 779, JANUARY 12, 2016 411
Saguisag vs. Ochoa, Jr.
297 EDCA, Art. III(1).
298 Id., Art. IV(4). and promised stricter guidelines to be imposed upon its contractors. 306 The
299 Id., Art. IV(5). statement attributed to Commander Ron Steiner of the public affairs office of the
300 Commonwealth Act No. 541. U.S. Navy’s 7th Fleet — that U.S. Navy contractors are bound by Philippine laws
301 Republic Act No. 10951. —is of particular relevance. The statement acknowledges not just the presence of
the contractors, but also the U.S. position that these contractors are bound by the
local laws of their host state. This stance was echoed by other U.S. Navy
410 representatives.307
410 SUPREME COURT REPORTS ANNOTATED This incident simply shows that the Senate was well aware of the presence of U.S.
contractors for the purpose of fulfilling the terms of the VFA. That they are bound
Saguisag vs. Ochoa, Jr. by Philippine law is clear to all, even to the U.S.
As applied to EDCA, even when U.S. contractors are granted access to the
The laws in place already address issues regarding the regulation of Agreed Locations, all their activities must be consistent with Philippine laws and
contractors. In the 2015 Foreign Investment Negative list,302 the Executive regulations and pursuant to the MDT and the VFA.
Department has already identified corporations that have equity restrictions in While we recognize the concerns of petitioners, they do not give the Court
Philippine jurisdiction. Of note is No. 5 on the list — private security agencies that enough justification to strike down EDCA. In Lim v. Executive Secretary, we have
cannot have any foreign equity by virtue of Section 4 of Republic Act No. already explained that we cannot take judicial notice of claims aired in news
5487;303 and No. 15, which regulates contracts for the construction of defense- reports, “not because of any issue as to their truth, accuracy, or impartiality, but
related structures based on Commonwealth Act No. 541.
for the simple reason that facts must be established in accordance with the rules all its reportorial requirements to the SEC; and (d) It has paid all its income taxes
of evidence.”308 What is for the year, as duly certified by the Bureau of Internal Revenue. x x x x. Security
_______________ agencies and LGUs shall be included in this category of licensed holders but shall
be subject to additional requirements as may be required by the Chief of the PNP.”
net/63649/us-navy-contractor-dumped-millions-of-liters-of-wastes-in-subic> Finally, Section 22 expresses: “A person
(last visited 3 December 2015).
306 Cabreza, Vincent, US Embassy says dumping of untreated waste in Subic
not condoned, Philippine Daily Inquirer, available at 413
<http://globalnation.inquirer.net/60255/us-embassy-says-dumping-of-untreated- VOL. 779, JANUARY 12, 2016 413
waste-in-subic-not-condoned> (last visited 3 December 2015).
307 Gonzaga, Robert, Contractor could face sanctions from US navy for Saguisag vs. Ochoa, Jr.
violations, Philippine Daily Inquirer, available at
<http://globalnation.inquirer.net/56622/contractor-could-face-sanctions-from-us- and therefore limit their activities here. Until and unless there is another law
navy-for-violations> (last visited 3 December 2015). or treaty that specifically deals with their entry and activities, their presence in
308 Lim v. Executive Secretary, supra note 69 at p. 580; pp. 759-760. the country is subject to unqualified Philippine jurisdiction.
_______________

412 arriving in the Philippines who is legally in possession of any firearm


or ammunition in his/her country of origin and who has declared the
412 SUPREME COURT REPORTS ANNOTATED
existence of the firearm upon embarkation and disembarkation but
Saguisag vs. Ochoa, Jr. whose firearm is not registered in the Philippines in accordance with this
Act shall deposit the same upon written receipt with the Collector of
more, we cannot move one step ahead and speculate that the alleged illegal Customs for delivery to the FEO of the PNP for safekeeping, or for the issuance
activities of these contractors in other countries would take place in the Philippines of a permit to transport if the person is a competitor in a sports shooting
with certainty. As can be seen from the above discussion, making sure that U.S. competition. If the importation of the same is allowed and the party in
contractors comply with Philippine laws is a function of law enforcement. EDCA question desires to obtain a domestic firearm license, the same should be
does not stand in the way of law enforcement. undertaken in accordance with the provisions of this Act. If no license is
Nevertheless, we emphasize that U.S. contractors are explicitly excluded from desired or leave to import is not granted, the firearm or ammunition in question
the coverage of the VFA. As visiting aliens, their entry, presence, and activities are shall remain in the custody of the FEO of the PNP until otherwise disposed of in
subject to all laws and treaties applicable within the Philippine territory. They accordance with law.” (Emphasis supplied)
may be refused entry or expelled from the country if they engage in illegal or 310 Article 40 of the Labor Code, as amended, provides: “Employment permit
undesirable activities. There is nothing that prevents them from being detained in of nonresident aliens.—Any alien seeking admission to the Philippines for
the country or being subject to the jurisdiction of our courts. Our penal employment purposes and any domestic or foreign employer who desires to
laws,309 labor laws,310 and immigrations laws311 apply to them engage an alien for employment in the Philippines shall obtain an employment
_______________ permit from the Department of Labor. The employment permit may be issued
to a nonresident alien or to the applicant employer after a determination of the
309 See R.A. No. 10591 or the Comprehensive Firearms and Ammunition nonavailability of a person in the Philippines who is competent, able and
Regulation Act. According to Section 4, Article II thereof: In order to qualify and willing at the time of application to perform the services for which the
acquire a license to own and possess a firearm or firearms and alien is desired. For an enterprise registered in preferred areas of
ammunition, the applicant must be a Filipino citizen, at least twenty-one (21) investments, said employment permit may be issued upon recommendation
years old and has gainful work, occupation or business or has filed an Income Tax of the government agency charged with the supervision of said registered
Return (ITR) for the preceding year as proof of income, profession, business or enterprise.” (Emphasis supplied)
occupation. In addition, the applicant shall submit the following certification 311 Supra notes 263 and 267.
issued by appropriate authorities attesting the following: x x x x. On the other
hand, Section 5 states: “A juridical person maintaining its own security
force may be issued a regular license to own and possess firearms and 414
ammunition under the following conditions: (a) It must be Filipino-owned and 414 SUPREME COURT REPORTS ANNOTATED
duly registered with the Securities and Exchange Commission (SEC); (b) It is
current, operational and a continuing concern; (c) It has completed and submitted Saguisag vs. Ochoa, Jr.
The context of use is “required by United States forces for activities under this
EDCA does not allow the presence Agreement.” Therefore, the return of an Agreed Location would be within the
of U.S.-owned or -controlled mili- parameters of an activity that the Mutual Defense Board (MDB) and the Security
tary facilities and bases in the Engagement Board (SEB) would authorize. Thus, possession by the U.S. prior to
Philippines its return of the Agreed Location would be based on the authority given to it by a
joint body co-chaired by the “AFP Chief of Staff and Commander, U.S. PACOM
Petitioners Saguisag, et al. claim that EDCA permits the establishment of U.S. with representatives from the Philippines’ Department of National Defense and
military bases through the “euphemistically” termed “Agreed Department of Foreign Affairs sitting as members.”313 The terms shall be
Locations.”312 Alluding to the definition of this term in Article II(4) of EDCA, they negotiated by both the Philippines and the U.S., or through their Designated
point out that these locations are actually military bases, as the definition refers Authorities. This provision, seen as a whole, contradicts petitioners’ interpretation
to facilities and areas to which U.S. military forces have access for a variety of of the return as a “badge of exclusivity.” In fact, it shows the cooperation and
purposes. Petitioners claim that there are several badges of exclusivity in the use partnership aspect of EDCA in full bloom.
of the Agreed Locations by U.S. forces. First, Article V(2) of EDCA alludes to a Second, the term “unimpeded access” must likewise be viewed from a
“return” of these areas once they are no longer needed by U.S. forces, indicating contextual perspective. Article IV(4) states that U.S. forces and U.S. contractors
that there would be some transfer of use. Second, Article IV(4) of EDCA talks shall have “unimpeded access to Agreed Locations for all matters relating to the
about American forces’ unimpeded access to the Agreed Locations for all matters prepositioning and storage of defense equipment, supplies, and materiel, including
relating to the prepositioning and storage of U.S. military equipment, supplies, delivery, management, inspection, use, maintenance, and removal of such
and materiel. Third, Article VII of EDCA authorizes U.S. forces to use public equipment, supplies and materiel.”
utilities and to operate their own telecommunications system. _______________

a. Preliminary point on 313 PH-US MDB and SEB Convenes, Department of National Defense,
badges of exclusivity. available at <http://www.dndph.org/press-releases/ph-us-mdb-and-seb-convenes>
(last visited 3 December 2015).
As a preliminary observation, petitioners have cherry-picked provisions of
EDCA by presenting so-called “badges of exclusivity,” despite the presence of
contrary provisions within the text of the agreement itself. 416
First, they clarify the word “return” in Article V(2) of EDCA. However, the use
416 SUPREME COURT REPORTS ANNOTATED
of the word “return” is within the context of a lengthy provision. The provision as
a whole reads as follows: Saguisag vs. Ochoa, Jr.
_______________
At the beginning of Article IV, EDCA states that the Philippines gives the U.S.
312 Memorandum of Saguisag, et al., pp. 25-29, Rollo (G.R. No. 212426, Vol. the authority to bring in these equipment, supplies, and materiel through the MDB
II), pp. 995-999. and SEB security mechanism. These items are owned by the U.S., 314 are
exclusively for the use of the U.S.315 and, after going through the joint consent
mechanisms of the MDB and the SEB, are within the control of the U.S. 316 More
415 importantly, before these items are considered prepositioned, they must have gone
through the process of prior authorization by the MDB and the SEB and given
VOL. 779, JANUARY 12, 2016 415
proper notification to the AFP.317
Saguisag vs. Ochoa, Jr. Therefore, this “unimpeded access” to the Agreed Locations is a necessary
adjunct to the ownership, use, and control of the U.S. over its own equipment,
The United States shall return to the Philippines any Agreed Locations, supplies, and materiel and must have first been allowed by the joint mechanisms
or any portion thereof, including non-relocatable structures and assemblies in play between the two states since the time of the MDT and the VFA. It is not
constructed, modified, or improved by the United States, once no longer the use of the Agreed Locations that is exclusive per se; it is mere access to items
required by United States forces for activities under this Agreement. The in order to exercise the rights of ownership granted by virtue of the Philippine Civil
Parties or the Designated Authorities shall consult regarding the terms of Code.318
return of any Agreed Locations, including possible compensation for As for the view that EDCA authorizes U.S. forces to use public utilities and to
improvements or construction. operate their own telecommunications system, it will be met and answered in
part D, infra.
Petitioners also point out319 that EDCA is strongly reminiscent of and in fact 322 Id., at p. 1000. EDCA, Arts. I(1)(b), I(2), I(3), & III(4).
bears a one-to-one correspondence with the 323 Id., at p. 1002.
_______________

314 EDCA, Art. IV(3). 418


315 Id. 418 SUPREME COURT REPORTS ANNOTATED
316 Id.
317 Id. Saguisag vs. Ochoa, Jr.
318 Such rights gleaned from Title II, Chapter I of the Civil Code are
(Cojuangco v. Sandiganbayan, 604 Phil. 670; 586 SCRA 790 [2009]) the right to that the U.S. retains ownership over immovable and movable properties.
possess, to use and enjoy, to abuse or consume, to accessories, to dispose or To our mind, both EDCA and the MBA simply incorporate what is already the
alienate, to recover or vindicate, and to the fruits. law of the land in the Philippines. The Civil Code’s provisions on ownership, as
319 Memorandum of Saguisag, et al., pp. 29-33, Rollo (G.R. No. 212426, Vol. applied, grant the owner of a movable property full rights over that property, even
II), pp. 999-1003; Memorandum of Bayan, et al., pp. 41-71, Rollo (G.R. No. if located in another person’s property.324
212444), pp. 605-635. The parallelism, however, ends when the situation involves facilities that can
be considered immovable. Under the MBA, the U.S. retains ownership if it paid for
the facility.325 Under EDCA, an immovable is owned by the Philippines, even if
417 built completely on the back of U.S. funding.326 This is consistent with the
constitutional prohibition on foreign land ownership.327
VOL. 779, JANUARY 12, 2016 417 Despite the apparent similarity, the ownership of property is but a part of a
Saguisag vs. Ochoa, Jr. larger whole that must be considered before the constitutional restriction is
violated. Thus, petitioners’ points on operational control will be given more
provisions of the 1947 MBA. They assert that both agreements (a) allow similar attention in the discussion below. The arguments on policy are, however, outside
activities within the area; (b) provide for the same “species of ownership” over the scope of judicial review and will not be discussed.
facilities; and (c) grant operational control over the entire area. Finally, they Moreover, a direct comparison of the MBA and EDCA will result in several
argue320 that EDCA is in fact an implementation of the new defense policy of the important distinctions that would allay suspicion that EDCA is but a disguised
U.S. According to them, this policy was not what was originally intended either by version of the MBA.
the MDT or by the VFA.
On these points, the Court is not persuaded. b. There are substantial
The similar activities cited by petitioners321 simply show that under the MBA, matters that the U.S. can-
the U.S. had the right to construct, operate, maintain, utilize, occupy, garrison, not do under EDCA, but
and control the bases. The so-called parallel provisions of EDCA allow only which it was authorized to
operational control over the Agreed Locations specifically for construction do under the 1947 MBA.
activities. They do not allow the overarching power to operate, maintain, utilize, _______________
occupy, garrison, and control a base with full discretion. EDCA in fact limits the
rights of the U.S. in respect of every activity, including construction, by giving the 324 See generally Civil Code, Arts. 427-429.
MDB and the SEB the power to determine the details of all activities such as, but 325 Memorandum of Saguisag, et al., pp. 33-35, Rollo (G.R. No. 212426, Vol.
not limited to, operation, maintenance, utility, occupancy, garrisoning, and II), pp. 1001-1002.
control.322 326 Id.
The “species of ownership” on the other hand, is distinguished by the nature of 327 Constitution, Art. XII, Sec. 7.
the property. For immovable property constructed or developed by the U.S., EDCA
expresses that ownership will automatically be vested to the Philippines. 323 On the
other hand, for movable properties brought into the Philippines by the U.S., EDCA 419
provides that ownership is retained by the latter. In contrast, the MBA dictates VOL. 779, JANUARY 12, 2016 419
_______________
Saguisag vs. Ochoa, Jr.
320 Memorandum of Saguisag, et al., pp. 33-35, Rollo (G.R. No. 212426, Vol.
II), pp. 1003-1005. The Philippine experience with U.S. military bases under the 1947 MBA is
321 Id., at pp. 1000-1001. simply not possible under EDCA for a number of important reasons.
First, in the 1947 MBA, the U.S. retained all rights of jurisdiction in and over
Philippine territory occupied by American bases. In contrast, the U.S. under EDCA
does not enjoy any such right over any part of the Philippines in which its forces
or equipment may be found. Below is a comparative table between the old treaty
and EDCA: 426
426 SUPREME COURT REPORTS ANNOTATED
Saguisag vs. Ochoa, Jr.
420
420 SUPREME COURT REPORTS ANNOTATED
Saguisag vs. Ochoa, Jr.
427
VOL. 779, JANUARY 12, 2016 427

421 Saguisag vs. Ochoa, Jr.


VOL. 779, JANUARY 12, 2016 421
Saguisag vs. Ochoa, Jr.

428
428 SUPREME COURT REPORTS ANNOTATED

422 Saguisag vs. Ochoa, Jr.


422 SUPREME COURT REPORTS ANNOTATED
Saguisag vs. Ochoa, Jr.

429
VOL. 779, JANUARY 12, 2016 429

423 Saguisag vs. Ochoa, Jr.


VOL. 779, JANUARY 12, 2016 423
Saguisag vs. Ochoa, Jr.

430
430 SUPREME COURT REPORTS ANNOTATED

424 Saguisag vs. Ochoa, Jr.


424 SUPREME COURT REPORTS ANNOTATED
Saguisag vs. Ochoa, Jr.

431
VOL. 779, JANUARY 12, 2016 431

425 Saguisag vs. Ochoa, Jr.


VOL. 779, JANUARY 12, 2016 425
Saguisag vs. Ochoa, Jr.
In sum, EDCA is a far cry from a basing agreement as was understood by the _______________
people at the time that the 1987 Constitution was adopted.
Nevertheless, a comprehensive review of what the Constitution means by 328 P.D. No. 1227 – Punishing Unlawful Entry into Any Military Base in the
“foreign military bases” and “facilities” is required before EDCA can be deemed to Philippines, Sec. 2.
have passed judicial scrutiny. 329 IV Record, Constitutional Commission, p. 648 (15 September 1986).

c. The meaning of military


facilities and bases. 433
VOL. 779, JANUARY 12, 2016 433
An appreciation of what a military base is, as understood by the Filipino people
in 1987, would be vital in determining whether EDCA breached the constitutional Saguisag vs. Ochoa, Jr.
restriction.
Prior to the drafting of the 1987 Constitution, the last definition of “military raise funds by the sale of portions of Metro Manila military camps, and
base” was provided under Presidential to apply said funds as provided herein for the development and conversion
to productive civilian use of the lands covered under the 1947 Military
Bases Agreement between the Philippines and the United States of
432 America, as amended.330
432 SUPREME COURT REPORTS ANNOTATED
The result of the debates and subsequent voting is Section 25, Article XVIII of
Saguisag vs. Ochoa, Jr. the Constitution, which specifically restricts, among others, foreign military
facilities or bases. At the time of its crafting of the Constitution, the 1986
Decree No. (PD) 1227.328 Unlawful entry into a military base is punishable Constitutional Commission had a clear idea of what exactly it was restricting.
under the decree as supported by Article 281 of the Revised Penal Code, which While the term “facilities and bases” was left undefined, its point of reference was
itself prohibits the act of trespass. clearly those areas covered by the 1947 MBA as amended.
Section 2 of the law defines the term in this manner: “‘[M]ilitary base’ as used Notably, nearly 30 years have passed since then, and the ever-evolving world
in this decree means any military, air, naval, or coast guard reservation, base, fort, of military technology and geopolitics has surpassed the understanding of the
camp, arsenal, yard, station, or installation in the Philippines.” Philippine people in 1986. The last direct military action of the U.S. in the region
Commissioner Tadeo, in presenting his objections to U.S. presence in the was the use of Subic base as the staging ground for Desert Shield and Desert Storm
Philippines before the 1986 Constitutional Commission, listed the areas that he during the Gulf War.331 In 1991, the Philippine Senate rejected the successor
considered as military bases: treaty of the 1947 MBA that would have allowed the continuation of U.S. bases in
the Philippines.
1,000 hectares Camp O’Donnel Henceforth, any proposed entry of U.S. forces into the Philippines had to evolve
20,000 hectares Crow Valley Weapon’s Range likewise, taking into consideration the subsisting agreements between both
55,000 hectares Clark Air Base parties, the rejection of the 1991 proposal, and a concrete understanding of what
150 hectares Wallace Air Station was constitutionally restricted. This trend birthed the VFA which, as discussed,
400 hectares John Hay Air Station has already been upheld by this Court.
15,000 hectares Subic Naval Base The latest agreement is EDCA, which proposes a novel concept termed “Agreed
1,000 hectares San Miguel Naval Communication Locations.”
750 hectares Radio Transmitter in Capas, Tarlac By definition, Agreed Locations are:
900 hectares Radio Bigot Annex at Bamban, Tarlac329 _______________

The Bases Conversion and Development Act of 1992 described its coverage in 330 R.A. No. 7227.
its Declaration of Policies: 331 Padua, supra note 64.

Sec. 2. Declaration of Policies.—It is hereby declared the policy of the


Government to accelerate the sound and balanced conversion into 434
alternative productive uses of the Clark and Subic military reservations
434 SUPREME COURT REPORTS ANNOTATED
and their extensions (John Hay Station, Wallace Air Station, O’Donnell
Transmitter Station, San Miguel Naval Communications Station and Saguisag vs. Ochoa, Jr.
Capas Relay Station), to
facilities and areas that are provided by the Government of the standard for the application of its text, given the particular historical events
Philippines through the AFP and that United States forces, United States preceding the agreement.
contractors, and others as mutually agreed, shall have the right to access Once more, we must look to the 1986 Constitutional Commissioners to glean,
and use pursuant to this Agreement. Such Agreed Locations may be listed from their collective wisdom, the intent of Section 25. Their speeches are rich with
in an annex to be appended to this Agreement, and may be further history and wisdom and present a clear picture of what they considered in the
described in implementing arrangements.332 crafting the provision.

Preliminarily, respondent already claims that the proviso that the Philippines SPEECH OF COMMISSIONER REGALADO334
shall retain ownership of and title to the Agreed Locations means that EDCA is xxxx
“consistent with Article II of the VFA which recognizes Philippine sovereignty and We have been regaled here by those who favor the adoption of the anti-
jurisdiction over locations within Philippine territory.”333 bases provisions with what purports to be an objective presentation of the
By this interpretation, respondent acknowledges that the contention of historical background of the military bases in the Philippines. Care
petitioners springs from an understanding that the Agreed Locations merely appears, however, to have been taken to underscore the inequity in their
circumvent the constitutional restrictions. Framed differently, the bone of inception as well as their implementation, as to seriously reflect on
contention is whether the Agreed Locations are, from a legal perspective, foreign the supposed objectivity of the report. Pronouncements of military and
military facilities or bases. This legal framework triggers Section 25, Article XVIII, civilian officials shortly after World War II are quoted in support of the
and makes Senate concurrence a sine qua non. proposition on neutrality; regrettably, the implication is that the same
Article III of EDCA provides for Agreed Locations, in which the U.S. is remains valid today, as if
authorized by the Philippines to “conduct the following activities: “training; _______________
transit; support and related activities; refueling of aircraft; bunkering of vessels;
temporary maintenance of vehicles, vessels and aircraft; temporary 334 IV Record, Constitutional Commission, pp. 628-630 (15 September 1986).
accommodation of personnel; communications; prepositioning of equipment,
supplies and materiel; deploying forces and materiel; and such other activities as
the Parties may agree.” 436
This creation of EDCA must then be tested against a proper interpretation of
436 SUPREME COURT REPORTS ANNOTATED
the Section 25 restriction.
_______________ Saguisag vs. Ochoa, Jr.

332 EDCA, Art. II(4). the world and international activity stood still for the last 40 years.
333 Memorandum of OSG, p. 23, Rollo (G.R. No. 212426), p. 453. We have been given inspired lectures on the effect of the
presence of the military bases on our sovereignty — whether in its
legal or political sense is not clear — and the theory that any
435 country with foreign bases in its territory cannot claim to be fully
sovereign or completely independent.
VOL. 779, JANUARY 12, 2016 435
I was not aware that the concepts of sovereignty and independence have
Saguisag vs. Ochoa, Jr. now assumed the totality principle, such that a willing assumption of some
delimitations in the exercise of some aspects thereof would put that State
d. Reasons for the constitu- in a lower bracket of nationhood.
tional requirements and le- xxxx
gal standards for constitu- We have been receiving a continuous influx of materials on the pros and
tionally compatible mili- cons on the advisability of having military bases within our shores. Most of
tary bases and facilities. us who, only about three months ago, were just mulling the prospects of
these varying contentions are now expected, like armchair generals, to
Section 25 does not define what is meant by a “foreign military facility or base.” decide not only on the geopolitical aspects and contingent implications of
While it specifically alludes to U.S. military facilities and bases that existed during the military bases but also on their political, social, economic and cultural
the framing of the Constitution, the provision was clearly meant to apply to those impact on our national life. We are asked to answer a plethora of questions,
bases existing at the time and to any future facility or base. The basis for the such as: 1) whether the bases are magnets of nuclear attack or are
restriction must first be deduced from the spirit of the law, in order to set a deterrents to such attack; 2) whether an alliance or mutual defense treaty
is a derogation of our national sovereignty; 3) whether criticism of us by
Russia, Vietnam and North Korea is outweighed by the support for us of 337 Id., at pp. 630-631.
the ASEAN countries, the United States, South Korea, Taiwan, Australia
and New Zealand; and 4) whether the social, moral and legal problems
spawned by the military bases and their operations can be compensated by 438
the economic benefits outlined in papers which have been furnished 438 SUPREME COURT REPORTS ANNOTATED
recently to all of us.335
xxxx Saguisag vs. Ochoa, Jr.
_______________
4. That these foreign military bases unnecessarily expose our
335 Id., at p. 628. country to devastating nuclear attacks;
5. That these foreign military bases create social problems and are
designed to perpetuate the strangle-hold of United States interests in our
437 national economy and development;
6. That the extraterritorial rights enjoyed by these foreign bases
VOL. 779, JANUARY 12, 2016 437 operate to deprive our country of jurisdiction over civil and
Saguisag vs. Ochoa, Jr. criminal offenses committed within our own national territory and
against Filipinos;
Of course, one side of persuasion has submitted categorical, 7. That the bases agreements are colonial impositionsand
unequivocal and forceful assertions of their positions. They are entitled to dictations upon our helpless country; and
the luxury of the absolutes. We are urged now to adopt the proposed 8. That on the legal viewpoint and in the ultimate analysis, all the
declaration as a “golden,” “unique” and “last” opportunity for bases agreements are null and void ab initio, especially because they did
Filipinos to assert their sovereign rights. Unfortunately, I have never not count the sovereign consent and will of the Filipino people.338
been enchanted by superlatives, much less for the applause of the moment
or the ovation of the hour. Nor do I look forward to any glorious summer xxxx
after a winter of political discontent. Hence, if I may join Commissioner In the real sense, Madam President, if we in the Commission could
Laurel, I also invoke a caveat not only against the tyranny of labels but also accommodate the provisions I have cited, what is our objection to include
the tyranny of slogans.336 in our Constitution a matter as priceless as the nationalist values we
cherish? A matter of the gravest concern for the safety and survival
xxxx of this nationindeed deserves a place in our Constitution.
xxxx
SPEECH OF COMMISSIONER SUAREZ337 x x x Why should we bargain away our dignity and our self-
respect as a nation and the future of generations to come with thirty pieces
MR. SUAREZ: Thank you, Madam President. of silver?339
I am quite satisfied that the crucial issues involved in the resolution of _______________
the problem of the removal of foreign bases from the Philippines have been
adequately treated by previous speakers. Let me, therefore, just 338 Id., at p. 630.
recapitulate the arguments adduced in favor of a foreign bases-free 339 Id., at p. 631.
Philippines:
1. That every nation should be free to shape its own destiny
without outside interference; 439
2. That no lasting peace and no true sovereignty would ever be VOL. 779, JANUARY 12, 2016 439
achieved so long as there are foreign military forces in our country;
3. That the presence of foreign military bases deprives us of the Saguisag vs. Ochoa, Jr.
very substance of national sovereignty and this is a constant source of
national embarrassment and an insult to our national dignity and self- SPEECH OF COMMISSIONER BENNAGEN340
respect as a nation; xxxx
_______________ The underlying principle of military bases and nuclear weapons
wherever they are found and whoever owns them is that those are for
336 Id., at p. 629. killing people or for terrorizing humanity. This objective by itself at
any point in history is morally repugnant. This alone is reason enough for
us to constitutionalize the ban on foreign military bases and on nuclear an incidental and secondary way, the security interest of the Republic of
weapons.341 the Philippines and the region. Yes, of course, Mr. Enrile also echoes the
sentiments of most of us in this Commission, namely: It is ideal for us as
SPEECH OF COMMISSIONER BACANI342 an independent and sovereign nation to ultimately abrogate the
xxxx RP-US military treaty and, at the right time, build our own air and
x x x Hence, the remedy to prostitution does not seem to be naval might.347
primarily to remove the bases because even if the bases are removed, xxxx
the girls mired in poverty will look for their clientele elsewhere. The remedy Allow me to say in summation that I am for the retention of
to the problem of prostitution lies primarily elsewhere — in an alert and American military bases in the Philippines provided that such an
concerned citizenry, a healthy economy and a sound education in values.343 extension from one period to another shall be concluded upon
concurrence of the parties, and such extension shall be based on
SPEECH OF COMMISSIONER JAMIR344 justice, the historical amity of
xxxx
One of the reasons advanced against the maintenance of _______________
foreign military bases here is that they impair portions of our
sovereignty. While I agree that our country’s sovereignty should not be 345 Id., at p. 636.
impaired, I also hold the view that there are times when it is necessary to 346 Id., at pp. 637-639.
do so according to the imperatives of national interest. There are 347 Id., at p. 638.
precedents to this effect. Thus, during World War II, England leased its
bases in the West Indies and in Bermuda for 99 years to the United
_______________ 441
VOL. 779, JANUARY 12, 2016 441
340 Id., at pp. 632-634.
341 Id., at p. 632. Saguisag vs. Ochoa, Jr.
342 Id., at pp. 634-635.
343 Id., at p. 634. the people of the Philippines and the United States and their
344 Id., at pp. 635-636. common defense interest.348

SPEECH OF COMMISSIONER ALONTO349


440 xxxx
Madam President, sometime ago after this Commission started with
440 SUPREME COURT REPORTS ANNOTATED
this task of framing a constitution, I read a statement of President Aquino
Saguisag vs. Ochoa, Jr. to the effect that she is for the removal of the U.S. military bases in this
country but that the removal of the U.S. military bases should not be done
States for its use as naval and air bases. It was done in consideration of just to give way to other foreign bases. Today, there are two world
50 overaged destroyers which the United States gave to England for its use superpowers, both vying to control any and all countries which have
in the Battle of the Atlantic. importance to their strategy for world domination. The Philippines is one
A few years ago, England gave the Island of Diego Garcia to the United such country.
States for the latter’s use as a naval base in the Indian Ocean. About the Madam President, I submit that I am one of those ready to
same time, the United States obtained bases in Spain, Egypt and Israel. In completely remove any vestiges of the days of enslavement, but not
doing so, these countries, in effect, contributed to the launching of a prepared to erase them if to do so would merely leave a vacuum to be
preventive defense posture against possible trouble in the Middle East and occupied by a far worse type.350
in the Indian Ocean for their own protection.345
SPEECH OF COMMISSIONER GASCON351
SPEECH OF COMMISSIONER TINGSON346 xxxx
xxxx Let us consider the situation of peace in our world today. Consider our
In the case of the Philippines and the other Southeast Asian nations, brethren in the Middle East, in Indo-China, Central America, in South
the presence of American troops in the country is a projection of America’s Africa — there has been escalation of war in some of these areas because of
security interest. Enrile said that nonetheless, they also serve, although in foreign intervention which views these conflicts through the narrow prism
of the East-West conflict. The United States bases have been used as 353 Id., at p. 644.
springboards for intervention in some of these conflicts. We should 354 Id., at pp. 645-649.
not allow ourselves to be party to the warlike mentality of these
foreign interventionists. We must al-
_______________ 443
VOL. 779, JANUARY 12, 2016 443
348 Id., at p. 639.
349 Id., at pp. 640-641. Saguisag vs. Ochoa, Jr.
350 Id., at p. 640.
351 Id., at pp. 641-645. litically and culturally. Para sa sambayanang magbubukid ang U.S.
military bases ay kasingkahulugan ng nuclear weapon — ang kahulugan
ay magneto ng isang nuclear war. Para sa sambayanang magbubukid, ang
442 kahulugan ng U.S. military basesay isang salot.355
442 SUPREME COURT REPORTS ANNOTATED SPEECH OF COMMISSIONER QUESADA356
Saguisag vs. Ochoa, Jr. xxxx
The drift in the voting on issues related to freeing ourselves from the
ways be on the side of peace — this means that we should not always instruments of domination and subservience has clearly been defined these
rely on military solution.352 past weeks.
xxxx xxxx
x x x The United States bases, therefore, are springboards for So for the record, Mr. Presiding Officer, I would like to declare my support for
intervention in our own internal affairs and in the affairs of other the committee’s position to enshrine in the Constitution a fundamental principle
nations in this region. forbidding foreign military bases, troops or facilities in any part of the Philippine
xxxx territory as a clear and concrete manifestation of our inherent right to
Thus, I firmly believe that a self-respecting nation should safeguard its national self-determination, independence and sovereignty.
fundamental freedoms which should logically be declared in black and Mr. Presiding Officer, I would like to relate now these attributes of genuine
white in our fundamental law of the land — the Constitution. Let us nationhood to the social cost of allowing foreign countries to maintain military
express our desire for national sovereignty so we may be able to bases in our country. Previous speakers have dwelt on this subject, either to
achieve national self-determination. Let us express our desire for highlight its importance in relation to the other issues or to gloss over its
neutrality so that we may be able to follow active nonaligned independent significance and make this a part of future negotiations.357
foreign policies. Let us express our desire for peace and a nuclear-free zone xxxx
so we may be able to pursue a healthy and tranquil existence, to have peace Mr. Presiding Officer, I feel that banning foreign military bases is one of the
that is autonomous and not imposed.353 solutions and is the response of the Filipino people against this condition and other
conditions that have already been clearly and emphati-
xxxx _______________
SPEECH OF COMMISSIONER TADEO354
Para sa magbubukid, ano ba ang kahulugan ng U.S. military 355 Id., at p. 645.
bases? Para sa magbubukid, ang kahulugan nito ay pagkaalipin. 356 Id., at pp. 649-652.
Para sa magbubukid, ang pananatili ng U.S. military bases ay tinik sa 357 Id., at p. 650.
dibdib ng sambayanang Pilipinong patuloy na nakabaon. Para sa
sambayanang magbubukid, ang ibig sabihin ng U.S. military
bases ay batong pabigat na patuloy na pinapasan ng sambayanang 444
Pilipino. Para sa sambayanang magbubukid, ang pananatili ng U.S. 444 SUPREME COURT REPORTS ANNOTATED
military bases ay isang nagdudumilat na katotohanan ng patuloy na
paggahasa ng imperyalistang Estados Unidos sa ating Inang Saguisag vs. Ochoa, Jr.
Bayan — economically, po-
_______________ cally discussed in past deliberations. The deletion, therefore, of Section
3 in the Constitution we are drafting will have the following implications:
352 Id., at p. 643. First, the failure of the Constitutional Commission to decisively
respond to the continuing violation of our territorial
integrity via the military bases agreement which permits the Mr. Presiding Officer, in advocating the majority committee report,
retention of U.S. facilities within the Philippine soil over which our specifically Sections 3 and 4 on neutrality, nuclear and bases-free country,
authorities have no exclusive jurisdiction contrary to the accepted some views stress sovereignty of the Republic and even invoke
definition of the exercise of sovereignty. survival of the Filipino nation and people.361
Second, consent by this forum, this Constitutional Commission, to
an exception in the application of a provision in the Bill of REBUTTAL OF COMMISSIONER NOLLEDO362
Rights that we have just drafted regarding equal application of the laws of xxxx
the land to all inhabitants, permanent or otherwise, within its territorial The anachronistic and ephemeral arguments against the provisions of
boundaries. the committee report to dismantle the American bases after 1991 only show
Third, the continued exercise by the United States of the urgent need to free our country from the entangling alliance with
extraterritoriality despite the condemnations of such practice by the any power bloc.363
world community of nations in the light of overwhelming international xxxx
approval of eradicating all vestiges of colonialism.358 x x x x Mr. Presiding Officer, it is not necessary for us to possess
xxxx expertise to know that the so-called RP-US Bases
Sixth, the deification of a new concept called pragmatic _______________
sovereignty, in the hope that such can be wielded to force the United
States government to concede to better terms and conditions concerning the 359 Id., at p. 652.
military bases agreement, including the transfer of complete control to 360 Id., at pp. 652-653.
the Philippine government of the U.S. facilities, while in the 361 Id.
meantime we have to suffer all existing indignities and disrespect towards 362 Id., at pp. 653-654.
our rights as a sovereign nation. 363 Id., at p. 653.
xxxx
Eighth, the utter failure of this forum to view the issue of foreign
military bases as essentially a question of sovereignty which does 446
not require in-
446 SUPREME COURT REPORTS ANNOTATED
_______________
Saguisag vs. Ochoa, Jr.
358 Id., at p. 651.
Agreement will expire in 1991, that it infringes on our sovereignty
and jurisdiction as well as national dignity and honor, that it goes
445 against the UN policy of disarmament and that it
constitutes unjust intervention in our internal affairs.364 (Emphases
VOL. 779, JANUARY 12, 2016 445
supplied)
Saguisag vs. Ochoa, Jr.
The Constitutional Commission eventually agreed to allow foreign military
depth studies or analyses and which this forum has, as a constituent bases, troops, or facilities, subject to the provisions of Section 25. It is thus
assembly drafting a constitution, the expertise and capacity to decide on important to read its discussions carefully. From these discussions, we can deduce
except that it lacks the political will that brought it to existence and now three legal standards that were articulated by the Constitutional Commission
engages in an elaborate scheme of buck-passing. Members. These are characteristics of any agreement that the country, and by
xxxx extension this Court, must ensure are observed. We can thereby determine
Without any doubt we can establish a new social order in our country, whether a military base or facility in the Philippines, which houses or is accessed
if we reclaim, restore, uphold and defend our national by foreign military troops, is foreign or remains a Philippine military base or
sovereignty. National sovereignty is what the military bases issue is facility. The legal standards we find applicable are: independence from foreign
all about. It is only the sovereign people exercising their national control, sovereignty and applicable law, and national security and territorial
sovereignty who can design an independent course and take full control of integrity.
their national destiny.359
i. First standard: independence
SPEECH OF COMMISSIONER PADILLA360 from foreign control
xxxx
Very clearly, much of the opposition to the U.S. bases at the time of the
Constitution’s drafting was aimed at asserting Philippine independence from the 448
U.S., as well as control over our country’s territory and military. 448 SUPREME COURT REPORTS ANNOTATED
Under the Civil Code, there are several aspects of control exercised over
property. Saguisag vs. Ochoa, Jr.
Property is classified as private or public.365 It is public if “intended for public
use, such as roads, canals, rivers, torrents, ports and bridges constructed by the Hence, even control over the property is something that an owner may
State, banks, shores, roadsteads, and others of similar character[,]” or transmit freely. This act does not translate into the full transfer of ownership, but
_______________ only of certain rights. In Roman Catholic Apostolic Administrator of Davao, Inc. v.
Land Registration Commission, we stated that the constitutional proscription on
364 Id., at p. 654. property ownership is not violated despite the foreign national’s control over the
365 Civil Code, Art. 419. property.370
EDCA, in respect of its provisions on Agreed Locations, is essentially a contract
of use and access. Under its pertinent provisions, it is the Designated Authority of
447 the Philippines that shall, when requested, assist in facilitating transit or access
to public land and facilities.371 The activities carried out within these locations are
VOL. 779, JANUARY 12, 2016 447 subject to agreement as authorized by the Philippine government.372 Granting the
Saguisag vs. Ochoa, Jr. U.S. operational control over these locations is likewise subject to EDCA’s security
mechanisms, which are bilateral procedures involving Philippine consent and
“[t]hose which belong to the State, without being for public use, and are cooperation.373 Finally, the Philippine Designated Authority or a duly designated
intended for some public service or for the development of the national wealth.”366 representative is given access to the Agreed Locations.374
Quite clearly, the Agreed Locations are contained within a property for public To our mind, these provisions do not raise the spectre of U.S. control, which
use, be it within a government military camp or property that belongs to the was so feared by the Constitutional Commission. In fact, they seem to have been
Philippines. the product of deliberate negotiation from the point of view of the Philippine
Once ownership is established, then the rights of ownership flow freely. Article government, which balanced constitutional restrictions on foreign military bases
428 of the Civil Code provides that “[t]he owner has the right to enjoy and dispose and facilities against the security needs of the country. In the 1947 MBA, the U.S.
of a thing, without other limitations than those established by law.” Moreover, the forces had “the right, power and authority x x x to construct (including dredging
owner “has also a right of action against the holder and possessor of the thing in and filling), operate, maintain, utilize, occupy, garrison
order to recover it.” _______________
Philippine civil law therefore accords very strong rights to the owner of
property, even against those who hold the property. Possession, after all, merely 370 Roman Catholic Apostolic Administrator of Davao, Inc. v. Land
raises a disputable presumption of ownership, which can be contested through Registration Commission, 102 Phil. 596 (1957).
normal judicial processes.367 371 EDCA, Art. III(2).
In this case, EDCA explicitly provides that ownership of the Agreed Locations 372 Id., Art. III(1).
remains with the Philippine government.368 What U.S. personnel have a right to, 373 Id., Art. III(4).
pending mutual agreement, is access to and use of these locations.369 374 Id., Art. III(5).
The right of the owner of the property to allow access and use is consistent with
the Civil Code, since the owner may dispose of the property in whatever way
deemed fit, subject to the limits of the law. So long as the right of ownership itself 449
is not transferred, then whatever rights are transmitted by agreement does not VOL. 779, JANUARY 12, 2016 449
completely divest the owner of the rights over the property, but may only limit
them in accordance with law. Saguisag vs. Ochoa, Jr.
_______________
and control the bases.”375 No similarly explicit provision is present in EDCA.
366 Id., Art. 420. Nevertheless, the threshold for allowing the presence of foreign military
367 Id., Art. 433. facilities and bases has been raised by the present Constitution. Section 25 is
368 EDCA, Art. V. explicit that foreign military bases, troops, or facilities shall not be allowed in the
369 Id., Art. II(4). Philippines, except under a treaty duly concurred in by the Senate. Merely stating
that the Philippines would retain ownership would do violence to the
constitutional requirement if the Agreed Locations were simply to become a less Rather, he claimed that the proposal was in compliance with the treaties between
obvious manifestation of the U.S. bases that were rejected in 1991. the two states.
When debates took place over the military provisions of the Constitution, the
committee rejected a specific provision proposed by Commissioner Sarmiento. The MR. DE CASTRO: If the Commissioner will take note of my speech on
discussion illuminates and provides context to the 1986 Constitutional U.S. military bases on 12 September 1986, I spoke on the self-reliance
Commission’s vision of control and independence from the U.S., to wit: policy of the armed forces. However, due to very limited resources, the only
thing we could do is manufacture small arms ammunition. We cannot
MR. SARMIENTO: Madam President, my proposed amendment blame the armed forces. We have to blame the whole Republic of the
reads as follows: “THE STATE SHALL ESTABLISH AND MAINTAIN AN Philippines for failure to provide the necessary funds to make the
INDEPENDENT AND SELF-RELIANT ARMED FORCES OF THE Philippine
PHILIPPINES.” Allow me to briefly explain, Madam President. The Armed _______________
Forces of the Philippines is a vital component of Philippine society
depending upon its training, orientation and support. It will either be the 376 V Record, Constitutional Commission, p. 240 (30 September 1986).
people’s protector or a staunch supporter of a usurper or tyrant, local and
foreign interest. The Armed Forces of the Philippines’ past and
recent experience shows it has never been independent and self- 451
reliant. Facts, data and statistics will show that it has been substantially VOL. 779, JANUARY 12, 2016 451
dependent upon a foreign power. In March 1968, Congressman Barbero,
himself a member of the Armed Forces of the Philippines, revealed top Saguisag vs. Ochoa, Jr.
secret documents showing what he described as U.S. dictation over the af-
_______________ Armed Forces self-reliant. Indeed that is a beautiful dream. And I
would like it that way. But as of this time, fighting an insurgency case, a
375 1947 MBA, III(2)(a). rebellion in our country —insurgency — and with very limited funds and
very limited number of men, it will be quite impossible for the Philippines
to appropriate the necessary funds therefor. However, if we say that the
450 U.S. government is furnishing us the military hardware, it is not
control of our armed forces or of our government. It is in
450 SUPREME COURT REPORTS ANNOTATED
compliance with the Mutual Defense Treaty. It is under the military
Saguisag vs. Ochoa, Jr. assistance program that it becomes the responsibility of the United States
to furnish us the necessary hardware in connection with the military bases
fairs of the Armed Forces of the Philippines. He showed that under agreement. Please be informed that there are three (3) treaties connected
existing arrangements, the United States unilaterally determines with the military bases agreement; namely: the RP-US Military Bases
not only the types and quantity of arms and equipments that our Agreement, the Mutual Defense Treaty and the Military Assistance
armed forces would have, but also the time when these items are Program.
to be made available to us. It is clear, as he pointed out, that the My dear Commissioner, when we enter into a treaty and we are
composition, capability and schedule of development of the Armed furnished the military hardware pursuant to that treaty, it is not
Forces of the Philippines is under the effective control of the U.S. in control of our armed forces nor control of our government. True
government.376 (Emphases supplied) indeed, we have military officers trained in the U.S. armed forces school.
This is part of our Military Assistance Program, but it does not mean that
Commissioner Sarmiento proposed a motherhood statement in the 1987 the minds of our military officers are for the U.S. government, no. I am one
Constitution that would assert “independent” and “self-reliant” armed forces. This of those who took four courses in the United States schools, but I assure
proposal was rejected by the committee, however. As Commissioner De you, my mind is for the Filipino people. Also, while we are sending military
Castro asserted, the involvement of the Philippine military with the U.S. officers to train or to study in U.S. military schools, we are also sending our
did not, by itself, rob the Philippines of its real independence. He made officers to study in other military schools such as in Australia, England and
reference to the context of the times: that the limited resources of the Philippines in Paris. So, it does not mean that when we send military officers to United
and the current insurgency at that time necessitated a strong military relationship States schools or to other military schools, we will be under the control of
with the U.S. He said that the U.S. would not in any way control the Philippine that country. We also have foreign officers in our schools, we in the
military despite this relationship and the fact that the former would furnish Command and General Staff College in Fort Bonifacio and in our National
military hardware or extend military assistance and training to our military. De-
Under Article VI(3) of EDCA, U.S. forces are authorized to act as necessary for
“operational control and defense.” The term “operational control” has led
452 petitioners to regard U.S. control over the Agreed Locations as unqualified and,
452 SUPREME COURT REPORTS ANNOTATED therefore, total.382Petitioners contend that the word “their” refers to the subject
“Agreed Locations.”
Saguisag vs. Ochoa, Jr. This argument misreads the text, which is quoted below:

fense College, also in Fort Bonifacio.377 (Emphases supplied) United States forces are authorized to exercise all rights and
authorities within Agreed Locations that are necessary for their
This logic was accepted in Tañada v. Angara, in which the Court ruled that operational control or defense, including taking appropriate measure to
independence does not mean the absence of foreign participation: protect United States forces and United States contractors. The United
States should coordinate such measures with appropriate authorities of the
Furthermore, the constitutional policy of a “self-reliant and Philippines.
independent national economy” does not necessarily rule out the entry
of foreign investments, goods and services. It contemplates neither A basic textual construction would show that the word “their,” as understood
“economic seclusion” nor “mendicancy in the international community.” As above, is a possessive pronoun for the subject “they,” a third-person personal
explained by Constitutional Commissioner Bernardo Villegas, sponsor of pronoun in plural form. Thus, “their” cannot be used for a nonpersonal subject such
this constitutional policy: as “Agreed Locations.” The simple grammatical conclusion is that “their” refers to
Economic self-reliance is a primary objective of a developing the previous third-person plural noun, which is “United States forces.” This
country that is keenly aware of overdependence on external conclusion is in line with the definition of operational control.
assistance for even its most basic needs. It does not mean autarky _______________
or economic seclusion; rather, it means avoiding mendicancy in the
international community. Independence refers to the freedom 380 Black’s Law Dictionary, p. 770 (6th ed., 1990). See also J. Carpio’s
from undue foreign control of the national economy, especially Dissenting Opinion in Liban v. Gordon, 654 Phil. 680; 639 SCRA 709 (2011).
in such strategic industries as in the development of natural 381 Memorandum of Saguisag, p. 56, Rollo (G.R. No. 212426), p. 594.
resources and public utilities.378(Emphases supplied) 382 Id., at p. 596.

The heart of the constitutional restriction on foreign military facilities and


bases is therefore the assertion of independence from the U.S. and other foreign 454
powers, as independence is exhibited by the degree of foreign control exerted over
these areas. The essence of that independence is selfgovernance and self- 454 SUPREME COURT REPORTS ANNOTATED
control.379Independence itself is “[t]he Saguisag vs. Ochoa, Jr.
_______________
a. U.S. operational control as
377 Id., at pp. 240-241. the exercise of authority
378 Tañada v. Angara, supra note 97. over U.S. personnel, and not
379 Tydings-McDuffie Act, Section 10(a), Pub.L. 73-127, 48 Stat. 456 (enacted over the Agreed Locations.
24 March 1934).
Operational control, as cited by both petitioner and respondents, is a military
term referring to:
453
VOL. 779, JANUARY 12, 2016 453 [t]he authority to perform those functions of command over subordinate
forces involving organizing and employing commands and forces, assigning
Saguisag vs. Ochoa, Jr. tasks, designating objective, and giving authoritative direction necessary
to accomplish the mission.383
state or condition of being free from dependence, subjection, or control.”380
Petitioners assert that EDCA provides the U.S. extensive control and authority At times, though, operational control can mean something slightly different.
over Philippine facilities and locations, such that the agreement effectively violates In JUSMAG Philippines v. National Labor Relations Commission, the
Section 25 of the 1987 Constitution.381
Memorandum of Agreement between the AFP and JUSMAG Philippines defined Authority with the phrase “consistent with operational safety and security
the term as follows:384 requirements in accordance with agreed procedures developed by the Parties”
leads to the conclusion that the U.S. exercises effective control over the Agreed
The term “Operational Control” includes, but is not limited to, all Locations.389 They claim that if the Philippines exercises possession of and control
personnel administrative actions, such as: hiring recommendations; firing over a given
recommendations; position classification; discipline; nomination and _______________
approval of incentive awards; and payroll computation.
386 Local Government Code of 1991, Sec. 444.
Clearly, traditional standards define “operational control” as personnel 387 Id., Sec. 455.
control. Philippine law, for instance, deems operational control as one exercised by 388 Rollo (G.R. No. 212426), pp. 515-525.
police officers and civilian authorities over their subordinates and is distinct from 389 Id., at p. 597.
the administrative control that they also exercise over police
subordinates.385 Similarly, a municipal mayor exercises operational control over
the police within the municipal govern- 456
_______________
456 SUPREME COURT REPORTS ANNOTATED

383 Id., at p. 460. Saguisag vs. Ochoa, Jr.


384 G.R. No. 108813, 15 December 1994, 239 SCRA 224, 229.
385 R.A. No. 6975 — Department of the Interior and Local Government Act of area, its representative should not have to be authorized by a special
1990, Sec. 86; P.D. No. 531, Secs. 4, 5, and 6. provision.390
For these reasons, petitioners argue that the “operational control” in EDCA is
the “effective command and control” in the 1947 MBA.391 In their Memorandum,
455 they distinguish effective command and control from operational control in U.S.
parlance.392 Citing the Doctrine for the Armed Forces of the United States, Joint
VOL. 779, JANUARY 12, 2016 455
Publication 1, “command and control (C2)” is defined as “the exercise of authority
Saguisag vs. Ochoa, Jr. and direction by a properly designated commander over assigned and attached
forces in the accomplishment of the mission x x x.”393 Operational control, on the
ment,386 just as city mayor possesses the same power over the police within the other hand, refers to “[t]hose functions of command over assigned forces involving
city government.387 the composition of subordinate forces, the assignment of tasks, the designation of
Thus, the legal concept of operational control involves authority over personnel objectives, the overall control of assigned resources, and the full authoritative
in a commander-subordinate relationship and does not include control over the direction necessary to accomplish the mission.”394
Agreed Locations in this particular case. Though not necessarily stated in EDCA Two things demonstrate the errors in petitioners’ line of argument.
provisions, this interpretation is readily implied by the reference to the taking of Firstly, the phrase “consistent with operational safety and security
“appropriate measures to protect United States forces and United States requirements in accordance with agreed procedures developed by the Parties” does
contractors.” not add any qualification beyond that which is already imposed by existing
It is but logical, even necessary, for the U.S. to have operational control over treaties. To recall, EDCA is based upon prior treaties, namely the VFA and the
its own forces, in much the same way that the Philippines exercises operational MDT.395 Treaties are in themselves contracts from which rights and obligations
control over its own units. may be claimed or waived.396
For actual operations, EDCA is clear that any activity must be planned and _______________
preapproved by the MDB-SEB.388 This provision evinces the partnership aspect of
EDCA, such that both stakeholders have a say on how its provisions should be put 390 Id.
into effect. 391 Id., at p. 598.
392 Id., at p. 599.
b. Operational control 393 Id., footnote 76.
vis-à-vis effective com- 394 Id., footnote 77.
mand and control. 395 EDCA, preamble.
396 See: Bayan Muna v. Romulo, supra note 114; Bayan (Bagong Alyansang
Petitioners assert that beyond the concept of operational control over Makabayan) v. Zamora, supra note 23; USAFFE Veterans Ass’n., Inc. v. Treasurer
personnel, qualifying access to the Agreed Locations by the Philippine Designated
of the Phil., supra note 173; Vienna Convention on the Law of the Treaties, Art. 27 forces involving organizing and employing commands and forces, assigning
(on internal law and observance of tasks, designating objectives, and giving authoritative direction over all
aspects of military operations and joint training necessary to accomplish
the mission. It should be delegated to and exercised by the commanders of
457 subordinate organizations; normally, this authority is exercised through
VOL. 779, JANUARY 12, 2016 457 subordinate JFCs, Service, and/or functional component commanders.
OPCON provides authority to organize and employ commands and forces
Saguisag vs. Ochoa, Jr. as the commander considers necessary to accomplish assigned missions. It
does not include authoritative direction for logistics or matters of
In this particular case, the Philippines has already agreed to abide by the administration, discipline, internal organization, or unit training. These
security mechanisms that have long been in place between the U.S. and the elements of COCOM must be specifically delegated by the CCDR. OPCON
Philippines based on the implementation of their treaty relations. 397 does include the authority to delineate functional responsibilities and
Secondly, the full document cited by petitioners contradicts the equation of operational areas of subordinate JFCs.
“operational control” with “effective command and control,” since it defines the Operational control is therefore the delegable aspect of combatant command,
terms quite differently, viz.:398 while command and control is the overall power and responsibility exercised by
the commander with reference to a mission. Operational control is a narrower
Command and control encompasses the exercise of authority, power and must be given, while command and control is plenary and vested in a
responsibility, and direction by a commander over assigned and attached commander. Operational control does not include the planning, programming,
forces to accomplish the mission. Command at all levels is the art of budgeting, and execution process input; the assignment of subordinate
motivating and directing people and organizations into action to accomplish commanders; the building of relationships with Department of Defense agencies;
missions. Control is inherent in command. To control is to manage and or the directive authority for logistics, whereas these factors are included in the
direct forces and functions consistent with a commander’s command concept of command and control.400
authority. Control of forces and functions helps commanders and staffs _______________
compute requirements, allocate means, and integrate efforts. Mission
command is the preferred method of exercising C2. A complete discussion 399 Id., at Chap. V-6.
of tenets, organization, and processes for effective C2 is provided in Section 400 Id., at Chap. V-2.
B, “Command and Control of Joint Forces,” of Chapter V “Joint Command
and Control.”
treaties) in relation to Art. 46 (on provisions of internal law regarding 459
competence to conclude treaties).
397 “Under EDCA, before constructions and other activities can be VOL. 779, JANUARY 12, 2016 459
undertaken, prior consent of the Philippines will have to be secured through the Saguisag vs. Ochoa, Jr.
Mutual Defense Board (MDB) and Security Engagement Board (SEB) which were
established under the MDT and the VFA.” See Q&A on the Enhanced Defense This distinction, found in the same document cited by petitioners, destroys the
Cooperation Agreement, Official Gazette, available at very foundation of the arguments they have built: that EDCA is the same as the
<http://www.gov.ph/2014/04/28/qna-on-the-enhanced-defense-cooperation- MBA.
agreement> (last accessed 3 December 2015).
398 United States Department of Defense, Doctrine for the Armed Forces of c. Limited operational control over the Agreed Locations only for
the United States: Joint Publication 1, Chaps. 1-18 (2013). construction activities.

As petitioners assert, EDCA indeed contains a specific provision that gives to


458 the U.S. operational control within the Agreed Locations during construction
458 SUPREME COURT REPORTS ANNOTATED activities.401 This exercise of operational control is premised upon the approval by
the MDB and the SEB of the construction activity through consultation and
Saguisag vs. Ochoa, Jr. mutual agreement on the requirements and standards of the construction,
alteration, or improvement.402
Operational control is defined thus:399 Despite this grant of operational control to the U.S., it must be emphasized
that the grant is only for construction activities. The narrow and limited instance
OPCON is able to be delegated from a lesser authority than COCOM.
It is the authority to perform those functions of command over subordinate
wherein the U.S. is given operational control within an Agreed Location cannot be 404 Rebuttal of Commissioner Nolledo, supra note 362.
equated with foreign military control, which is so abhorred by the Constitution. 405 Vienna Convention on Diplomatic Relations, Arts. 31-40, 500 U.N.T.S. 95
The clear import of the provision is that in the absence of construction (1961).
activities, operational control over the Agreed Location is vested in the Philippine
authorities. This meaning is implicit in the specific grant of operational control
only during construction activities. The principle of constitutional construction, 461
“expressio unius est exclusio alterius,” means the failure to mention the thing VOL. 779, JANUARY 12, 2016 461
becomes the ground for inferring that it was deliberately excluded.403 Following
this construc- Saguisag vs. Ochoa, Jr.
_______________
Sovereignty is the possession of sovereign power,406 while jurisdiction is the
401 EDCA, Art. III(4). conferment by law of power and authority to apply the law.407 Article I of the 1987
402 Id. Constitution states:
403 Sarmiento III v. Mison, supra note 177. The case also formulated this
principle as follows: “an express enumeration of subjects excludes others not The national territory comprises the Philippine archipelago, with all
enumerated.” the islands and waters embraced therein, and all other territories over
which the Philippines has sovereignty or jurisdiction, consisting of its
terrestrial, fluvial, and aerial domains, including its territorial sea, the
460 seabed, the subsoil, the insular shelves, and other submarine areas. The
waters around, between, and connecting the islands of the archipelago,
460 SUPREME COURT REPORTS ANNOTATED regardless of their breadth and dimensions, form part of the internal waters
Saguisag vs. Ochoa, Jr. of the Philippines. (Emphasis supplied)

tion, since EDCA mentions the existence of U.S. operational control over the From the text of EDCA itself, Agreed Locations are territories of the
Agreed Locations for construction activities, then it is quite logical to conclude that Philippines that the U.S. forces are allowed to access and use.408By withholding
it is not exercised over other activities. ownership of these areas and retaining unrestricted access to them, the
Limited control does not violate the Constitution. The fear of the government asserts sovereignty over its territory. That sovereignty exists so long
commissioners was total control, to the point that the foreign military forces might as the Filipino people exist.409
dictate the terms of their acts within the Philippines. 404 More important, limited Significantly, the Philippines retains primary responsibility for security with
control does not mean an abdication or derogation of Philippine sovereignty and respect to the Agreed Locations.410 Hence, Philippine law remains in force therein,
legal jurisdiction over the Agreed Locations. It is more akin to the extension of and it cannot be said that jurisdiction has been transferred to the U.S. Even the
diplomatic courtesies and rights to diplomatic agents, 405which is a waiver of previously discussed necessary measures for operational control and defense over
control on a limited scale and subject to the terms of the treaty. U.S. forces must be coordinated with Philippine authorities.411
This point leads us to the second standard envisioned by the framers of the Jurisprudence bears out the fact that even under the former legal regime of
Constitution: that the Philippines must retain sovereignty and jurisdiction over its the MBA, Philippine laws continue to be
territory. _______________

ii. Second standard: Philippine 406 See Black’s Law Dictionary, p. 1523 (9th ed., 2009).
sovereignty and applicable law 407 Id., at p. 927.
408 EDCA, Article I(1)(b).
EDCA states in its Preamble the “understanding for the United States not to 409 Laurel v. Misa, 77 Phil. 856 (1947).
establish a permanent military presence or base in the territory of the 410 EDCA, Art. VI(2).
Philippines.” Further on, it likewise states the recognition that “all United States 411 Id., Art. VI(3).
access to and use of facilities and areas will be at the invitation of the Philippines
and with full respect for the Philippine Constitution and Philippine laws.”
The sensitivity of EDCA provisions to the laws of the Philippines must be seen 462
in light of Philippine sovereignty and jurisdiction over the Agreed Locations. 462 SUPREME COURT REPORTS ANNOTATED
_______________
Saguisag vs. Ochoa, Jr.
in force within the bases.412 The difference between then and now is that EDCA international law, EDCA does not provide a legal basis for a justified attack on the
retains the primary jurisdiction of the Philippines over the security of the Agreed Philippines.
Locations, an important provision that gives it actual control over those locations. In the first place, international law disallows any attack on the Agreed
Previously, it was the provost marshal of the U.S. who kept the peace and enforced Locations simply because of the presence of U.S. personnel. Article 2(4) of the
Philippine law in the bases. In this instance, Philippine forces act as peace officers, United Nations Charter states that “All Members shall refrain in their
in stark contrast to the 1947 MBA provisions on jurisdiction.413 international relations from the threat or use of force against the territorial
integrity or political independence of any state, or in any other manner
iii. Third standard: must re- inconsistent with the Purposes of the United Nations.”418 Any unlawful attack on
spect national security and the Philippines breaches the treaty, and triggers Article 51 of the same charter,
territorial integrity which
_______________
The last standard this Court must set is that the EDCA provisions on the
Agreed Locations must not impair or threaten the national security and territorial 415 Memorandum of Saguisag, p. 72, Rollo (G.R. No. 212426), p. 610.
integrity of the Philippines. 416 Id.
This Court acknowledged in Bayan v. Zamora that the evolution of technology 417 Id.
has essentially rendered the prior notion of permanent military bases obsolete. 418 Charter of the United Nations, 24 October 1945, 1 U.N.T.S. XVI.
Moreover, military bases established within the territory of another state is no
longer viable because of the alternatives offered by new means and weapons of
warfare such as nuclear weapons, guided missiles as well as huge sea vessels that 464
can stay afloat in the sea even for months and years without returning to their
464 SUPREME COURT REPORTS ANNOTATED
home country. These military warships are actually used as substitutes for a land-
home base not only of military aircraft but also of military personnel and facilities. Saguisag vs. Ochoa, Jr.
Besides, vessels are mobile as compared to a land-based military headquarters.414
_______________ guarantees the inherent right of individual or collective self-defence.
Moreover, even if the lawfulness of the attack were not in question,
412 Liwanag v. Hamill, 98 Phil. 437 (1956). international humanitarian law standards prevent participants in an armed
413 1947 MBA, Art. XIII. conflict from targeting nonparticipants. International humanitarian law, which is
414 Bayan (Bagong Alyansang Makabayan) v. Zamora, supra note 23. the branch of international law applicable to armed conflict, expressly limits
allowable military conduct exhibited by forces of a participant in an armed
conflict.419Under this legal regime, participants to an armed conflict are held to
463 specific standards of conduct that require them to distinguish between combatants
and noncombatants,420 as embodied by the Geneva Conventions and their
VOL. 779, JANUARY 12, 2016 463
Additional Protocols.421
Saguisag vs. Ochoa, Jr. Corollary to this point, Professor John Woodcliffe, professor of international
law at the University of Leicester, noted that there is no legal consensus for what
The VFA serves as the basis for the entry of U.S. troops in a limited scope. It constitutes a base, as opposed to other terms such as “facilities” or
does not allow, for instance, the reestablishment of the Subic military base or the “installation.”422 In strategic literature, “base” is defined as an installation
Clark Air Field as U.S. military reservations. In this context, therefore, this Court _______________
has interpreted the restrictions on foreign bases, troops, or facilities as three
independent restrictions. In accord with this interpretation, each restriction must 419 Protocol Additional to the Geneva Conventions of 12 August 1949, and
have its own qualification. relating to the Protection of Victims of International Armed Conflicts (Protocol I),
Petitioners quote from the website http://en.wikipedia.org to define what a 1125 U.N.T.S. 3 (1977) [hereinafter Geneva Convention Additional Protocol I];
military base is.415 While the source is not authoritative, petitioners make the Protocol Additional to the Geneva Conventions of 12 August 1949, and relating to
point that the Agreed Locations, by granting access and use to U.S. forces and the Protection of Victims of Non-International Armed Conflicts (Protocol II), 1125
contractors, are U.S. bases under a different name.416 More important, they claim U.N.T.S. 609 (1977).
that the Agreed Locations invite instances of attack on the Philippines from 420 Articles 48, 51(2) and 52(2), Protocol I, id.
enemies of the U.S.417 421 1949 Geneva Convention (I) for the Amelioration of the Condition of the
We believe that the raised fear of an attack on the Philippines is not in the Wounded and Sick in Armed Forces in the Field, 12 August 1949, 75 U.N.T.S. 31;
realm of law, but of politics and policy. At the very least, we can say that under 1949 Geneva Convention (II) for the Amelioration of the Condition of Wounded,
Sick and Shipwrecked Members of Armed Forces at Sea, 12 August 1949, 75 Saguisag vs. Ochoa, Jr.
U.N.T.S. 85; 1949 Geneva Convention (III) Relative to the Treatment of Prisoners
of War, 12 August 1949, 75 U.N.T.S. 135; 1949 Geneva Convention (IV) Relative tional humanitarian law if it is against a bona fide U.S. military base, facility,
to the Protection of Civilian Persons in Time of War, 12 August 1949, 75 U.N.T.S. or installation that directly contributes to the military effort of the U.S. Moreover,
287; id. the third state’s forces must take all measures to ensure that they have complied
422 Woodcliffe, John, The Peacetime Use of Foreign Military Installations with the principle of distinction (between combatants and non-combatants).
Under Modern International Law, p. 30 (1992). There is, then, ample legal protection for the Philippines under international
law that would ensure its territorial integrity and national security in the event
an Agreed Location is subjected to attack. As EDCA stands, it does not create the
465 situation so feared by petitioners — one in which the Philippines, while not
VOL. 779, JANUARY 12, 2016 465 participating in an armed conflict, would be legitimately targeted by an enemy of
Saguisag vs. Ochoa, Jr. the U.S.431
In the second place, this is a policy question about the wisdom of allowing the
“over which the user State has a right to exclusive control in an extraterritorial presence of U.S. personnel within our territory and is therefore outside the scope
of judicial review.
sense.”423 Since this definition would exclude most foreign military installations, a
more important distinction must be made. Evidently, the concept of giving foreign troops access to “agreed” locations,
For Woodcliffe, a type of installation excluded from the definition of “base” is areas, or facilities within the military base of another sovereign state is nothing
one that does not fulfill a combat role. He cites an example of the use of the new on the international plane. In fact, this arrangement has been used as the
territory of a state for training purposes, such as to obtain experience in local framework for several defense cooperation agreements, such as in the following:
geography and climactic conditions or to carry out joint exercises. 424 Another 1. 2006 U.S.-Bulgaria Defense Cooperation Agreement432
example given is an advanced communications technology installation for 2. 2009 U.S.-Colombia Defense Cooperation Agreement433
purposes of information gathering and communication.425 Unsurprisingly, he _______________
deems these noncombat uses as borderline situations that would be excluded from
the functional understanding of military bases and installations.426 431 Memorandum of Saguisag, pp. 66-70, Rollo (G.R. No. 212426), pp. 604-
By virtue of this ambiguity, the laws of war dictate that the status of a building 608.
or person is presumed to be protected, unless proven otherwise. 427 Moreover, the 432 Article 11(6) thereof provides: “Agreed facilities and areas” means
principle of distinction requires combatants in an armed conflict to distinguish the state owned facilities and areas in the territory of the Republic of Bulgaria
between lawful targets428 and protected targets.429 In an actual armed conflict listed in Annex A, and such other state owned facilities and areas, as may be
between the U.S. and a third state, the Agreed Locations cannot be considered U.S. mutually agreed by the Parties.
territory, since ownership of territory even in times of armed conflict does not 433 Article I(g) thereof provides: “Agreed facilities and locations” means
change.430 those sites, installations, and infrastructure to which the United States is
Hence, any armed attack by forces of a third state against an Agreed Location authorized access and use by Colombia in
can only be legitimate under interna-
_______________
467
423 Id. VOL. 779, JANUARY 12, 2016 467
424 Id., at p. 32. Saguisag vs. Ochoa, Jr.
425 Id.
426 Id. 3. 2009 U.S.-Poland Status of Forces Agreement434
427 Henckaerts, Jean-Marie and Ooswald-Beck, Louise, Customary 4. 2014 U.S.-Australia Force Posture Agreement435
International Humanitarian Law-Volume I: Rules, pp. 34-36 (2005) 5. 2014 U.S.-Afghanistan Security and Defense Cooperation Agreement436
428 Art. 52, Protocol I, supra note 419.
429 Art. 48, id. In all of these arrangements, the host state grants U.S. forces access to their
430 Art. 4, id. military bases.437 That access is without rental or similar costs to the
U.S.438 Further, U.S. forces are
_______________
466
466 SUPREME COURT REPORTS ANNOTATED
connection with activities carried out within the framework of this Status of Forces Agreement, Art. 3(1); US-Australia Force Posture Agreement,
Agreement. Art. IV(7).
434 Article 2(i) thereof provides: “Agreed facilities and areas” shall mean 439 US-Bulgaria Defense Cooperation Agreement, Art. IV(7); US-Colombia
areas, facilities, buildings or structures in the territory of the Republic of Defense Cooperation Agreement, Arts. IV(7), XI; US-Poland Status of Forces
Poland, owned by the Republic of Poland, and used by United States forces Agreement, Art. 3(6); US-Australia Force Posture Agreement, Art. IV(8).
with the consent of the Republic of Poland. 440 US-Bulgaria Defense Cooperation Agreement, Arts. II(6), IV(1) & VI(1);
435 Article I thereof provides: “Agreed Facilities and Areas” means US-Colombia Defense Cooperation Agreement, Art. IV(6); US-Poland Status of
the facilities and areas in the territory of Australia provided by Australia Forces Agreement, Art. 4(1); US-Australia Force Posture Agreement, Art. XIV(1).
which may be listed in Annex A appended to this Agreement, and such other 441 US-Bulgaria Defense Cooperation Agreement, Art. IV(8); US-Colombia
facilities and areas in the territory of Australia as may be provided by Australia Defense Cooperation Agreement, Art. IV(4); US-Poland Status of Forces
in the future, to which United States Forces, United States Contractors, Agreement, Art. 3(10); US-Australia Force Posture Agreement, Art. X(2).
dependants, and other United States Government personnel as mutually agreed,
shall have the right to access and use pursuant to this Agreement.
436 Article I(7) thereof provides: “Agreed facilities and areas” means 469
the facilities and areas in the territory of Afghanistan provided by VOL. 779, JANUARY 12, 2016 469
Afghanistan at the locations listed in Annex A, and such other facilities and
areas in the territory of Afghanistan as may be provided by Afghanistan in the Saguisag vs. Ochoa, Jr.
future, to which United States forces, United States contractors, United States
contractor employees, and others as mutually agreed, shall have the right to 2007, which are essentially executive agreements that implement the VFA, the
access and use pursuant to this Agreement. MDT, and the 1953 Military Assistance Agreement. These executive agreements
437 US-Bulgaria Defense Cooperation Agreement, Arts. II(6) & IV(1); US- similarly tackle the “reciprocal provision of logistic support, supplies, and
Colombia Defense Cooperation Agreement, Art. IV; US-Poland Status of Forces services,”442 which include “[b]illeting, x x x operations support (and construction
Agreement, Art. 3(2); US-Australia Force Posture Agreement, Arts. I, IV. and use of temporary structures incident to operations support), training services,
438 US-Bulgaria Defense Cooperation Agreement, Art. IV(5); US-Colombia x x x storage services, x x x during an approved activity.”443 These logistic supplies,
Defense Cooperation Agreement, Art. IV; US-Poland support, and services include temporary use of “nonlethal items of military
equipment which are not designated as significant military equipment on the U.S.
Munitions List, during an approved activity.”444 The first Mutual Logistics
468 Support Agreement has lapsed, while the second one has been extended until 2017
without any formal objection before this Court from the Senate or any of its
468 SUPREME COURT REPORTS ANNOTATED
members.
Saguisag vs. Ochoa, Jr. The provisions in EDCA dealing with Agreed Locations are analogous to those
in the aforementioned executive agreements. Instead of authorizing the building
allowed to undertake construction activities in, and make alterations and of temporary structures as previous agreements have done, EDCA authorizes the
improvements to, the agreed locations, facilities, or areas.439 As in EDCA, the host U.S. to build permanent structures or alter or improve existing ones for, and to be
states retain ownership and jurisdiction over the said bases.440 owned by, the Philippines.445 EDCA is clear that the Philippines retains ownership
In fact, some of the host states in these agreements give specific military- of altered or improved facilities and newly constructed permanent or non-
related rights to the U.S. For example, under Article IV(1) of the U.S.-Bulgaria relocatable structures.446 Under EDCA, U.S. forces will also be allowed to use
Defense Cooperation Agreement, “the United States forces x x x are authorized facilities and areas for “training; x x x; support and related activities; x x x;
access to and may use agreed facilities and areas x x x for staging and deploying temporary accommodation of personnel; communications” and agreed activities.447
of forces and materiel, with the purpose of conducting x x x contingency operations Concerns on national security problems that arise from foreign military
and other missions, including those undertaken in the framework of the North equipment being present in the Philippines must likewise be contextualized. Most
Atlantic Treaty.” In some of these agreements, host countries allow U.S. forces to significantly, the VFA
construct facilities for the latter’s exclusive use.441 _______________
Troop billeting, including construction of temporary structures, is nothing new.
In Lim v. Executive Secretary, the Court already upheld the Terms of Reference 442 2002 MLSA, Art. III(2); 2007 MLSA, Art. III(2).
of Balikatan 02-1, which authorized U.S. forces to set up “[t]emporary structures 443 Id., Art. IV(1)(a)(2); id., Art. IV(1)(a)(2).
such as those for troop billeting, classroom instruction and messing x x x during 444 Id., Art. IV(1)(a)(3); id., Art. IV(1)(a)(3).
the Exercise.” Similar provisions are also in the Mutual Logistics Support 445 EDCA, Art. V(1).
Agreement of 2002 and 446 Id., Art. V(2).
_______________
447 Id., Art. III(1). Therefore, there is no basis to invalidate EDCA on fears that it increases the
threat to our national security. If anything, EDCA increases the likelihood that, in
an event requiring a defensive response, the Philippines will be prepared alongside
470 the U.S. to defend its islands and insure its territorial integrity pursuant to a
470 SUPREME COURT REPORTS ANNOTATED relationship built on the MDT and VFA.

Saguisag vs. Ochoa, Jr. 8. Others issues and


concerns raised
already authorizes the presence of U.S. military equipment in the
country. Article VII of the VFA already authorizes the U.S. to import into or A point was raised during the oral arguments that the language of the MDT
acquire in the Philippines “equipment, materials, supplies, and other property” only refers to mutual help and defense in the Pacific area.453 We believe that any
that will be used “in connection with activities” contemplated therein. The same discussion of the activities to be undertaken under EDCA vis-à-vis the defense of
section also recognizes that “[t]itle to such property shall remain” with the US and areas beyond the Pacific is premature. We note that a proper petition on that issue
that they have the discretion to “remove such property from the Philippines at any must be filed before we rule thereon. We also note that none of the petitions or
time.” memoranda has attempted to discuss this issue, except only to theorize that the
There is nothing novel, either, in the EDCA provision on the prepositioning U.S. will not come to our aid in the event of an attack outside of the Pacific. This
and storing of “defense equipment, supplies, and materiel,”448 since these are is a matter of policy and is beyond the scope of this judicial review.
sanctioned in the VFA. In fact, the two countries have already entered into various In reference to the issue on telecommunications, suffice it to say that the initial
implementing agreements in the past that are comparable to the present one. impression of the facility adverted to does appear to be one of those that require a
The Balikatan 02-1 Terms of Reference mentioned in Lim v. Executive public franchise by way of congressional action under Section 11, Article XII of the
Secretary specifically recognizes that Philippine and U.S. forces “may share x x x Constitution. As respondents submit, however, the system referred to in the
in the use of their resources, equipment and other assets.” Both the 2002 and 2007 agreement does not provide telecommunica-
Mutual Logistics Support Agreements speak of the provision of support and _______________
services, including the “construction and use of temporary structures incident to
operations support” and “storage services” during approved activities.449 These 452 EDCA, Art. IV(1).
logistic supplies, support, and services include the “temporary use of x x x 453 MDT, Arts. III, IV, and V.
nonlethal items of military equipment which are not designated as significant
military equipment on the U.S. Munitions List, during an approved
activity.”450 Those activities include “combined exercises and training, operations 472
and other deployments” and “cooperative efforts, such as humanitarian assistance,
disaster relief and rescue operations, and maritime anti-pollution operations” 472 SUPREME COURT REPORTS ANNOTATED
within or outside Philippine territory.451 Under EDCA, the equipment, supplies, Saguisag vs. Ochoa, Jr.
and materiel that will be prepositioned at Agreed
_______________ tions services to the public for compensation.454 It is clear from Article VII(2)
of EDCA that the telecommunication system is solely for the use of the U.S. and
448 Id., Art. IV(1). not the public in general, and that this system will not interfere with that which
449 2002 MLSA, Art. IV(1)(a)(2); 2007 MLSA, Art. IV(1)(a)(2). local operators use. Consequently, a public franchise is no longer necessary.
450 Id., Art. IV(1)(a)(3); id., Art. IV(1)(a)(3). Additionally, the charge that EDCA allows nuclear weapons within Philippine
451 Id., Art. III(1); id., Art. III(1). territory is entirely speculative. It is noteworthy that the agreement in fact
specifies that the prepositioned materiel shall not include nuclear
weapons.455 Petitioners argue that only prepositioned nuclear weapons are
471 prohibited by EDCA; and that, therefore, the U.S. would insidiously bring nuclear
VOL. 779, JANUARY 12, 2016 471 weapons to Philippine territory.456 The general prohibition on nuclear weapons,
whether prepositioned or not, is already expressed in the 1987 Constitution. 457 It
Saguisag vs. Ochoa, Jr. would be unnecessary or superfluous to include all prohibitions already in the
Constitution or in the law through a document like EDCA.
Locations include “humanitarian assistance and disaster relief equipment, Finally, petitioners allege that EDCA creates a tax exemption, which under
supplies, and materiel.”452 Nuclear weapons are specifically excluded from the the law must originate from Congress. This allegation ignores jurisprudence on
materiel that will be prepositioned. the government’s assumption of tax liability. EDCA simply states that the taxes
on the use of water, electricity, and public utilities are for the account of the 460 Id.
Philippine Government.458 This provision creates a situation in which a 461 EDCA, Art. III(6); Art. IV(2); Art. V(1, 4); Art. VIII(2).
contracting party assumes the tax liability of the other. 459 In National Power
Corporation v. Province of Quezon, we distinguished between enforceable and
unenforceable stipulations on the assumption of tax liability. Afterwards, we 474
concluded that an enforceable assumption of 474 SUPREME COURT REPORTS ANNOTATED
_______________
Saguisag vs. Ochoa, Jr.
454 Rollo, p. 464.
455 EDCA, Art. IV(6). the West Philippine Sea;462 initiated arbitration against that country under
456 Rollo, pp. 34-35. the United Nations Convention on the Law of the Sea;463 is in the process of
457 Article II, Sec. 8. negotiations with the Moro Islamic Liberation Front for peace in Southern
458 EDCA, Art. VII(1). Philippines,464 which is the subject of a current case before this Court; and faces
459 National Power Corporation v. Province of Quezon, 610 Phil. 456; 593 increasing incidents of kidnappings of Filipinos and foreigners allegedly by the
SCRA 47 (2009). Abu Sayyaf or the New People’s Army.465 The Philippine military is conducting
reforms that seek to ensure the security and safety of the nation in the years to
come.466 In the future, the Philippines must navigate a world in which armed
473 forces fight with increasing sophistication in both strategy and technology, while
employing asymmetric warfare and remote weapons.
VOL. 779, JANUARY 12, 2016 473 _______________
Saguisag vs. Ochoa, Jr.
462 Statement of Secretary Albert del Rosario before the Permanent Court of
tax liability requires the party assuming the liability to have actual interest in Arbitration, Peace Palace, The Hague, Netherlands, 7 July 2015, Official Gazette,
the property taxed.460 This rule applies to EDCA, since the Philippine Government available at <http://www.gov.ph/2015/07/07/statement-of-secretary-albert-del-
stands to benefit not only from the structures to be built thereon or improved, but rosario-before-the-permanent-court-of-arbitration-peace-palace-the-hague-
also from the joint training with U.S. forces, disaster preparation, and the netherlands/> (last visited 3 December 2015); Statement on Recent Incidents in the
preferential use of Philippine suppliers.461 Hence, the provision on the assumption Philippines’ Baja de Masinloc, 4 February 2015, Department of Foreign Affairs,
of tax liability does not constitute a tax exemption as petitioners have posited. available at <http://www.dfa.gov.ph/index.php/newsroom/dfa-releases/5337-
Additional issues were raised by petitioners, all relating principally to statement-on-recent-incidents-in-the-philippines-bajo-de-masinloc> (last visited
provisions already sufficiently addressed above. This Court takes this occasion to 21 October 2015).
emphasize that the agreement has been construed herein as to absolutely 463 The Republic of the Philippines v. The People’s Republic ofChina, Case No.
disauthorize the violation of the Constitution or any applicable statute. On the 2013-19 (Perm Ct. Arb.) <http://www.pcacases.com/web/view/7> (last visited 13
contrary, the applicability of Philippine law is explicit in EDCA. October 2015).
464 Comprehensive Agreement on the Bangsamoro, Official Gazette, available
EPILOGUE at <http://www.gov.ph/2014/03/27/document-cab> (last visited 21 October 2015).
465 Lim, Frinston, Authorities believe Abu Sayyaf behind abduction of
The fear that EDCA is a reincarnation of the U.S. bases so zealously protested Filipina, 3 foreigners, 22 September 2015, Philippine Daily Inquirer, available at
by noted personalities in Philippine history arises not so much from xenophobia, <http://globalnation.inquirer.net/128739/authorities-believe-npa-behind-
but from a genuine desire for self-determination, nationalism, and above all a abduction-of-filipina-foreigners> (last visited 3 December 2015).
commitment to ensure the independence of the Philippine Republic from any 466 Republic Act No. 10349 (2012); The Philippine Navy, Picture of the Future:
foreign domination. The Philippine Navy Briefer, available at
Mere fears, however, cannot curtail the exercise by the President of the <http://www.navy.mil.ph/downloads/THE%20PHILIPPINE%20NAVY%20BRIEF
Philippines of his Constitutional prerogatives in respect of foreign affairs. They ER.pdf> (last visited 3 December 2015).
cannot cripple him when he deems that additional security measures are made
necessary by the times. As it stands, the Philippines through the Department of
Foreign Affairs has filed several diplomatic protests against the actions of the 475
People’s Republic of China in VOL. 779, JANUARY 12, 2016 475
_______________
Saguisag vs. Ochoa, Jr.
Additionally, our country is fighting a most terrifying enemy: the backlash of Jardeleza, J., No part.
Mother Nature. The Philippines is one of the countries most directly affected and
damaged by climate change. It is no coincidence that the record-setting tropical SEPARATE CONCURRING OPINION
cyclone Yolanda (internationally named Haiyan), one of the most devastating
forces of nature the world has ever seen hit the Philippines on 8 November 2013 CARPIO, J.:
and killed at least 6,000 people.467 This necessitated a massive rehabilitation
project.468 In the aftermath, the U.S. military was among the first to extend help The threshold issue in this case is whether the Enhanced Defense Cooperation
and support to the Philippines. Agreement (EDCA) merely implements the existing and ratified 1951 Mutual
That calamity brought out the best in the Filipinos as thousands upon Defense Treaty1 (MDT), or whether the EDCA is a new treaty requiring Senate
thousands volunteered their help, their wealth, and their prayers to those affected. ratification to take effect.
It also brought to the fore the value of having friends in the international The answer to this question turns on whether, under present circumstances,
community. the attainment of the purpose of the MDT requires the EDCA. The fundamental
In order to keep the peace in its archipelago in this region of the world, and to rule in treaty interpretation is that a treaty must be interpreted “in the light of its
sustain itself at the same time against the destructive forces of nature, the object and purpose.”2
Philippines will need friends. Who they are, and what form the friendships will _______________
take, are for the President to decide. The only restriction is what the Constitution
itself expressly prohibits. It appears that this overarching concern for balancing 1 The Philippine Senate ratified the MDT on 12 May 1952 under Senate
constitutional requirements against the dictates of necessity was what led to Resolution No. 84.
EDCA. 2 Article 31 of the 1969 Vienna Convention on the Law of Treaties (Vienna
As it is, EDCA is not constitutionally infirm. As an executive agreement, it Convention) provides:
remains consistent with existing laws and treaties that it purports to implement.
WHEREFORE, we hereby DISMISS the petitions.
SO ORDERED. 477
_______________
VOL. 779, JANUARY 12, 2016 477

467 Locsin, Joel, NDRRMC: Yolanda death toll hits 6,300 mark nearly 6 Saguisag vs. Ochoa, Jr.
months after typhoon, 17 April 2014, GMA News Online
<http://www.gmanetwork.com/news/story/357322/news/nation/ndrrmc-yolanda- As stated in the MDT, the purpose of the United States (U.S.) and the
death-toll-hits-6-300-mark-nearly-6-months-after-typhoon> (last accessed 3 Philippines in forging the MDT is to “declare publicly and formally their sense of
December 2015). unity and their common determination to defend themselves against external
468 Typhoon Yolanda, Official Gazette, available at armed attack.” If the MDT cannot attain this purpose without the EDCA, then the
<http://www.gov.ph/crisis-response/updates-typhoon-yolanda/> (last visited 3 EDCA merely implements the MDT and Executive action is sufficient to make the
December 2015). EDCA valid.
A ratified treaty like the MDT must be interpreted to allow the Executive to
take all necessary measures to insure that the treaty’s purpose is attained. A
476 ratified treaty cannot be interpreted to require a second ratified treaty to
implement the first ratified treaty, as a fundamental rule is that a treaty must be
476 SUPREME COURT REPORTS ANNOTATED
interpreted to avoid a “result which is manifestly absurd or unreasonable.” 3 This
Saguisag vs. Ochoa, Jr. is particularly true to a mu-
_______________
Velasco, Jr., Del Castillo, Villarama, Jr., Perez, Mendoza and Reyes, JJ.,
concur. Article 31
Carpio, J., See Separate Concurring Opinion. General rule of interpretation
Leonardo-De Castro, J., I dissent: See my Dissenting Opinion. 1. A treaty shall be interpreted in good faith in accordance with the ordinary
Brion, J., I dissent: See my Dissenting Opinion. meaning to be given to the terms of the treaty in their context and in the light of
Peralta, J., I join J. Carpio’s Opinion. its object and purpose.
Bersamin, J., I join the Separate Concurring Opinion of J. Carpio. 2. The context for the purpose of the interpretation of a treaty shall comprise,
Perlas-Bernabe, J., I join the Dissenting Opinions. in addition to the text, including its preamble and annexes:
Leonen, J., I dissent. See Separate Opinion. x x x. (Emphasis supplied)
The Philippines acceded to the Vienna Convention on 15 November 1972. Saguisag vs. Ochoa, Jr.
3 Article 32 of the 1969 Vienna Convention on the Law of Treaties provides:
Article 32 pines, particularly in the islands of Palawan and Luzon facing the West
Supplementary means of interpretation Philippine Sea.
Recourse may be had to supplementary means of interpretation, including the In modern warfare, the successful implementation of a mutual defense treaty
preparatory work of the treaty and the circumstances of its conclusion, in order to requires the strategic prepositioning of war materials. Before the advent of guided
confirm the meaning resulting from the application of Article 31, or to determine missiles and drones, wars could take months or even years to prosecute. There was
the meaning when the interpretation according to Article 31: (a) leaves the plenty of time to conscript and train soldiers, manufacture guns and artillery, and
meaning ambiguous or obscure; or (b) leads to a result which is manifestly ship war materials to strategic locations even after the war had started. Today,
absurd or unreasonable. (Emphasis supplied) wars could be won or lost in the first few weeks or even first few days after the
initial outbreak of war.
In modern warfare, the prepositioning of war materials, like mobile anti-ship
478 and anti-aircraft missiles, is absolutely necessary and essential to a successful
478 SUPREME COURT REPORTS ANNOTATED defense against armed aggression, particularly for a coastal state like the
Saguisag vs. Ochoa, Jr. Philippines. This is what the EDCA is all about — the prepositioning in strategic
locations of war materials to successfully resist any armed aggression. Such
tual defense treaty the purpose of which is mutual self-defense against sudden prepositioning will also publicly telegraph to the enemy that any armed aggression
armed attack by a third state. would be repelled. The enemy must know that we possess the capability, that is,
However, if the MDT can attain its purpose without the EDCA, then the EDCA the war materials, to defend the country against armed aggression. Otherwise,
is a separate treaty that requires Senate ratification. I shall discuss why, under without such capability, we telegraph to the enemy that further seizure of
present circumstances, the EDCA is absolutely necessary and essential to attain Philippine islands, rocks and reefs in the South China Sea would be a walk in the
park, just like China’s seizure of Mischief Reef and Scarborough Shoal. Without
the purpose of the MDT.
With the departure in 1992 of U.S. military forces from Subic Naval Base and such capability, we would practically be inviting the enemy to seize whatever
Clark Air Base in Luzon, a power vacuum resulted in the South China Sea. As in Philippine island, rock or reef it desires to seize in the West Philippine Sea.
any power vacuum, the next power would rush in to fill the vacuum. Thus, China, Since 2014, China has started building artificial islands in the Spratlys out of
the next power after the U.S., filled the power vacuum in the South China Sea, submerged areas like Mischief Reef and Subi Reef, or out of rocks that barely
which includes the West Philippine Sea.4 protrude above water at high tide like Fiery Cross Reef. China has so far created
a 590-hectare artificial island in Mischief Reef which is only 125 nautical miles
In early 1995, barely three years after the departure of U.S. military forces
from the Philippines, China seized Mischief Reef from the Philippines. There was (NM) from Palawan, well within the Philippines’ Exclusive Economic Zone (EEZ).
no power to deter China as the U.S. forces had left. The Philippines did not In comparison, San Juan City is 595 hectares in area. China has built a 390-
anticipate that China would rush in to fill the power vacuum, or if the Philippines
anticipated this, it did not upgrade its military to deter any Chinese aggression.
After China seized Mischief Reef in 1995, the Philippines still did not upgrade its 480
military, particularly its navy. 480 SUPREME COURT REPORTS ANNOTATED
In 2012, China seized Scarborough Shoal from the Philippines, which could Saguisag vs. Ochoa, Jr.
offer no armed resistance to Chinese naval forces. The Scarborough Shoal seizure
finally made the Philippine Government realize that there was an absolute need hectare artificial island in Subi Reef, outside the Philippines’ EEZ but within
to deter China’s creeping invasion of Philippine islands, rocks and reefs in the West its Extended Continental Shelf (ECS). China has created a 265-hectare artificial
Philippine Sea. Thus, the Philippines rushed the modernization of its navy and air island in Fiery Cross Reef, outside the Philippines’ EEZ but within its ECS.
force. The Philippines also agreed with the U.S. to use the MDT to preposition U.S. China claims that its island-building activities are for civilian purposes but the
war materials in strategic locations in the Philip- configuration of these artificial islands shows otherwise. The configuration of
_______________ China’s Mischief Reef island, which is China’s largest artificial island in the
Spratlys, is that of a combined air and naval base, with a 3,000meter airstrip.5 The
4 See Administrative Order No. 29, 5 September 2012. configuration of China’s Subi Reef island is that of a naval base with a 3,000-meter
airstrip. The configuration of China’s Fiery Cross Reef island is that of an airbase
with a 3,000-meter airstrip and a harbor for warships. These three air and naval
479 bases form a triangle in the Spratlys, surrounding the islands occupied by the
VOL. 779, JANUARY 12, 2016 479 Philippines.
Mischief Reef, located mid-way between Palawan and Pagasa, is ideally an anti-ship missile system on its own. Military and security analysts are
situated to block Philippine ships resupplying Pagasa, the largest Philippine- unanimous that there is only one
occupied island in the Spratlys. Mischief Reef is also close to the gas-rich Reed _______________
Bank, the gas field that should replace Malampaya once Malampaya runs out of
gas in 10 to 12 years. Malampaya supplies 40% of the energy requirement of Luzon. 6 Final Transcript Day 1 — Merits Hearing, page 58, line 11, Philippines-
The Reed Bank and Malampaya are well within the Philippines’ EEZ. However, China Arbitration, http://www.pcacases.com/web/sendAttach/15487.
China’s 9-dashed lines enclose entirely the Reed Bank and encroach partly on
Malampaya.
It is obvious that China will use the three air and naval bases in its artificial 482
islands to prevent Philippine ships and planes from resupplying Philippine-
482 SUPREME COURT REPORTS ANNOTATED
occupied islands in the Spratlys, forcing the Philippines to abandon its occupied
islands. Already, Chinese coast guard vessels are preventing medium-sized Saguisag vs. Ochoa, Jr.
Philippine ships from resupplying the BRP Sierra Madre, the dilapidated
Philippine landing ship beached power on earth that can deter militarily China from enforcing its 9-dashed
_______________ lines claim, and that power is the United States. This is why the MDT is utterly
crucial to the Philippines’ defense of its EEZ in the West Philippine Sea.
5 A 3,000-meter airstrip is long enough for any military aircraft of China to Of course, the United States has repeatedly stated that the MDT does not cover
land and take off. A Boeing 747 airliner, or a B52 bomber, can easily land and take the disputed islands, rocks and reefs in the South China Sea. We understand this
off on a 3,000-meter airstrip. because at the time the MDT was signed the Philippine territory recognized by the
United States did not include the Kalayaan Island Group in the Spratlys. However,
the MDT provides that an armed attack on “public vessels or aircraft” (military or
481 coast guard ship or aircraft) of either the United States or the Philippines in the
Pacific area is one of the grounds for a party to invoke mutual defense under the
VOL. 779, JANUARY 12, 2016 481
MDT.7 The United States has officially clarified that the Pacific area includes the
Saguisag vs. Ochoa, Jr. South China Sea.8
If China’s navy ships attack a Philippine military ship re-supplying Philippine-
in Ayungin Shoal, just 20 NM from Mischief Reef. Only the Philippines’ use of occupied islands in the Spratlys, that will be covered by the MDT. However, unless
small watercrafts enables the resupply to the BRP Sierra Madre, which is manned the U.S. and the Philippines have prepositioned anti-ship missiles in Palawan,
by about a dozen Philippine marine soldiers. The Philippines’ small watercrafts there will be no deterrence to China, and no swift response from U.S. and
can navigate the shallow waters of Ayungin Shoal while China’s large coast guard Philippine forces. The absence of any deterrence will likely invite Chinese
vessels cannot. harassment, or even armed attack, on Philippine resupply ships. That will lead to
With the anticipated installation by China of military facilities and war the loss of all Philippine-occupied islands in the Spratlys, as well as the loss of the
materials in its three air and naval bases in the Spratlys, expected to be completed gas-rich Reed Bank.
before the end of 2016, China will begin to aggressively enforce its 9-dashed lines _______________
claim over the South China Sea. Under this claim, China asserts sovereignty not
only to all the islands, rocks and reefs in the Spratlys, but also to 85.7% of the 7 Article IV of the MDT provides: “Each Party recognizes that an armed attack
South China Sea, comprising all the waters, fisheries, mineral resources, seabed in the Pacific area on either of the Parties would be dangerous to its own peace and
and submarine areas enclosed by the 9-dashed lines. Under this claim, the safety and declares that it would act to meet the common dangers in accordance
Philippines will lose 381,000 square kilometers6 of its EEZ in the West Philippine with its constitutional processes. x x x.”
Sea, a maritime space larger than the total Philippine land area of 300,000 square 8 Letter of U.S. Secretary of State Cyrus Vance to Philippine Secretary of
kilometers. China’s 9-dashed lines claim encroaches on all the traditional fishing Foreign Affairs Carlos P. Romulo dated 6 January 1979; Letter of U.S. Ambassador
grounds of Filipino fishermen in the South China Sea: Scarborough Shoal, to the Philippines Thomas C. Hubbard to Foreign Secretary Domingo L. Siazon
Macclesfield Bank and the Spratlys. dated 24 May 1999.
The Philippines, acting by itself, cannot hope to deter militarily China from
enforcing its 9-dashed lines claim in the West Philippine Sea. The Philippines
cannot acquire war materials like anti-ship and anti-aircraft missiles off the shelf. 483
The operation of anti-ship missiles requires communications with airborne radar
VOL. 779, JANUARY 12, 2016 483
or satellite guidance systems. With the completion of China’s air and naval bases
before the end of 2016, the Philippines has no time to acquire, install and operate Saguisag vs. Ochoa, Jr.
The prepositioning of war materials is a necessary and essential element to 10 Article 56 of UNCLOS provides:
achieve the purpose of the MDT. Article II of the MDT expressly provides: Article 56
Rights, jurisdiction and duties of the coastal State
In order more effectively to achieve the objective of this Treaty, the in the exclusive economic zone
Parties separately and jointly by self-help and mutual aid will maintain 1. In the exclusive economic zone, the coastal State has:
and develop their individual and collective capacity to resist (a) sovereign rights for the purpose of exploring and exploiting, conserving
armed attack. (Emphasis supplied) and managing the natural resources, whether living or nonliving, of the waters
superjacent to the seabed and of the seabed and its subsoil, and with regard to
The prepositioning of war materials is the very essence of the phrase to other activities for the economic exploitation and exploration of the zone, such as
“maintain and develop (the Parties’) individual and collective capacity to the production of energy from the water, currents and winds;
resist armed attack.” Without the prepositioning of war materials, a Party to the (b) jurisdiction as provided for in the relevant provisions of this Convention
MDT cannot maintain and develop the capacity to resist armed attack. Without with regard to:
the prepositioning of war materials, a Party is simply and totally unprepared for (i) the establishment and use of artificial islands, installations and
armed attack. structures;
The 1987 Constitution defines the “national territory” to include not only (ii) marine scientific research;
islands or rocks above water at high tide but also the seabed, subsoil and other (iii) the protection and preservation of the marine environment;
submarine areas “over which the Philippines has sovereignty or jurisdiction.” (c) other rights and duties provided for in this Convention.
Article 1 of the 1987 Constitution provides: 2. In exercising its rights and performing its duties under this Convention in
the exclusive economic zone, the coastal State shall have due regard to the rights
The national territory comprises the Philippine archipelago, with all and duties of other States and shall act in a manner compatible with the provisions
the islands and waters embraced therein, and all other territories over of this Convention.
which the Philippines has sovereignty or jurisdiction, consisting of its x x x x (Emphasis supplied)
terrestrial, fluvial, and aerial domains, including its territorial sea, the
seabed, the subsoil, the insular shelves, and other submarine areas. The
waters around, between, and connecting the islands of the archipelago, 485
regardless of their breadth and dimensions, form part of the internal waters VOL. 779, JANUARY 12, 2016 485
of the Philippines. (Emphasis supplied)
Saguisag vs. Ochoa, Jr.
Thus, the Philippine “national territory” refers to areas over which the
Philippines has “sovereignty or jurisdiction.” The Constitution mandates: “The within its ECS.11 Under the UNCLOS, the Philippines also has sole
State shall protect the nation’s marine wealth in its archipelagic waters, territorial “jurisdiction” to create artificial islands or install structures within its EEZ12 and
sea, ECS.13
_______________

484 11 Article 77 of the UNCLOS provides:


484 SUPREME COURT REPORTS ANNOTATED Article 77
Rights of the coastal State over the continental shelf
Saguisag vs. Ochoa, Jr. 1. The coastal State exercises over the continental shelf sovereign rights
for the purpose of exploring it and exploiting its natural resources.
and exclusive economic zone, and reserve its use and enjoyment exclusively 2. The rights referred to in paragraph 1 are exclusive in the sense that if the
to Filipino citizens.”9 coastal State does not explore the continental shelf or exploit its natural resources,
Under both customary international law and the 1982 United Nations no one may undertake these activities without the express consent of the coastal
Convention on the Law of the Sea (UNCLOS), the Philippines has “sovereign State.
rights” and “jurisdiction”10 to exploit exclusively all the living and nonliving 3. The rights of the coastal State over the continental shelf do not depend on
resources within its EEZ. Under the UNCLOS, the Philippines has the sovereign occupation, effective or notional, or on any express proclamation.
rights to exploit exclusively the mineral resources 4. The natural resources referred to in this Part consist of the mineral and
_______________ other nonliving resources of the seabed and subsoil together with living organisms
belonging to sedentary species, that is to say, organisms which, at the harvestable
9 Section 2, Article XII of the 1987 Constitution. (Emphasis supplied)
stage, either are immobile on or under the seabed or are unable to move except in Article 80
constant physical contact with the seabed or the subsoil. (Emphasis supplied) Artificial islands, installations and structures
12 Article 60 of the UNCLOS provides: on the continental shelf
Article 60 Article 60 applies mutatis mutandis to artificial islands, installations and
Artificial islands, installations and structures structures on the continental shelf.
in the exclusive economic zone 14 Final Transcript Day 2 — Merits Hearing, page 23, lines 7, 8 and 9,
1. In the exclusive economic zone, the coastal State shall have the Philippines-China Arbitration, http://www.pcacases.com/web/
exclusive right to construct and to authorize and regulate the sendAttach/1548.
construction, operation and use of: 15 Id.
(a) artificial islands;
(b) installations and structures for the purposes provided for in Article 56 and
other economic purposes; 487
(c) installations and structures which may interfere with the exercise of the VOL. 779, JANUARY 12, 2016 487
rights of the coastal State in the zone.
2. The coastal State shall have exclusive jurisdiction over such Saguisag vs. Ochoa, Jr.
artificial islands, installations and structures, in-
rock above water at high tide and constituting land territory under
international law, China invaded Philippine national territory as defined in the
485 Constitution and as understood in international law. Republic Act No. 9522,
amending the Philippine Baselines Law, expressly declares that Scarborough
VOL. 779, JANUARY 12, 2016 485
Shoal is part of Philippine territory over which the Philippines exercises
Saguisag vs. Ochoa, Jr. “sovereignty and jurisdiction.”16
After China’s seizure of Scarborough Shoal in 2012, the Philippines finally
In short, under international law and in particular under the UNCLOS, the woke up and summoned the political will to address the serial and creeping
Philippines has jurisdiction over its EEZ and ECS. Thus, under domestic law, Chinese invasion of Philippine national territory. Thus, the EDCA was born, to
the Philippines’ EEZ and ECS form part of Philippine “national territory” since the give much needed teeth to the MDT as a deterrent to further Chinese aggression
Constitution defines “national territory” to include areas over which the in the West Philippine Sea. Without the EDCA, the MDT remains a toothless paper
Philippines has “jurisdiction,” a term which means less than sovereignty. tiger. With the EDCA, the MDT acquires a real and ready firepower to deter any
However, under international law, the Philippine “national territory” refers to the armed aggression against Philippine public vessels or aircrafts operating in the
areas over which the Philippines has sovereignty, referring to the Philippines’ West Philippine Sea.
land territory, archipelagic waters and territorial sea, excluding areas over which With the EDCA, China will think twice before attacking Philippine military
the Philippines exercises only jurisdiction like its EEZ and ECS. resupply ships to Philippine-occupied islands in the Spratlys. With the EDCA, the
China has already invaded repeatedly Philippine “national territory” in two Philippines will have a fighting chance to hold on to Philippine-occupied islands in
separate areas, one in the Kalayaan Island Group in the Spratlys and the other in the Spratlys. With the EDCA, China will think twice before attacking Philippine
Scarborough Shoal. When China seized in 1988 Subi Reef, a submerged area navy and coast guard vessels patrolling the West Philippine Sea. This will give the
within the Philippines’ ECS and beyond the territorial sea of any high tide Philippines a fighting chance to ward off China’s impending en-
feature,14 China invaded Philippine national territory as defined in the _______________
Constitution. When China seized in 1995 Mischief Reef, a submerged area within
the Philippines’ EEZ and beyond the territorial sea of any high tide 16 Section 2 of RA No. 9522 provides: “The baseline in the following areas over
feature,15 China invaded Philippine national territory as defined in the which the Philippines likewise exercises sovereignty and jurisdiction shall be
Constitution. When China seized in 2012 Scarborough Shoal, a determined as ‘Regime of Islands’ under the Republic of the Philippines consistent
_______________ with Article 121 of the United Nations Convention on the Law of the Sea
(UNCLOS):
cluding jurisdiction with regard to customs, fiscal, health, safety and a) The Kalayaan Island Group as constituted under Presidential Decree No.
immigration laws and regulations. 1596; and
x x x x (Emphasis supplied) b) Bajo de Masinloc, also known as Scarborough Shoal.” (Emphasis
13 Article 80 of the UNCLOS provides: supplied)
488 Moreover, the Senate ratification of the MDT complies with the requirement
488 SUPREME COURT REPORTS ANNOTATED of Section 25, Article XVII19 of the 1987 Constitution that any agreement allowing
foreign military facilities in the Philippines, like the prepositioning of U.S. war
Saguisag vs. Ochoa, Jr. materials, must be embodied in a treaty and ratified by two-thirds vote20 of the
Senate. That treaty is the MDT which the Philippine Senate ratified by two-thirds
forcement of its 9-dashed lines as China’s “national boundaries” as shown in vote on 12 May 195221 and which the U.S. Senate ratified on 20 March 1952.22
its 2013 official vertical map.17 _______________
The number and sites of the “agreed locations” to place the prepositioned war
materials must necessarily remain numerous and anonymous. The “agreed 19 Section 25, Article XVIII of the 1987 Constitution provides: “After the
locations” must be numerous enough to survive repeated or surprise armed expiration in 1991 of the Agreement between the Republic of the Philippines and
attacks. There must not only be redundant “agreed locations” but also dummy the United States of America concerning Military Bases, foreign military bases,
“agreed locations” to mislead the enemy. The sites of many of the “agreed locations” troops, or facilities shall not be allowed in the Philippines except under a treaty
cannot be disclosed publicly because that will give the enemy the fixed coordinates duly concurred in by the Senate and, when the Congress so requires, ratified
of the “agreed locations,” making them easy targets of long-range enemy cruise by a majority of the votes cast by the people in a national referendum held for that
missiles. The number and sites of the “agreed locations” are matters best left to purpose, and recognized as a treaty by the other contracting State.” (Emphasis
the sound discretion of the Executive, who is the implementing authority of the supplied)
MDT for the Philippines. 20 Section 21, Article VII of the 1987 Constitution provides: “No treaty or
The implementation of the MDT is a purely Executive function since the international agreement shall be valid and effective unless concurred in by at
Senate has already ratified the MDT. The implementation of the MDT is also part least two-thirds of all the Members of the Senate.” (Emphasis supplied)
of the purely Executive function of the President as Commander-in-Chief of the 21 The 1935 Constitution, under which the MDT was ratified, also required
Armed Forces. As executor and “chief architect”18 of the country’s relations with ratification of treaties by two-thirds vote of the Senate. Section 10(7), Article VII
foreign countries, including our treaty ally the United States, the President is of the 1935 Constitution provides: “The President shall have the power, with the
constitutionally vested with ample discretion in the implementation of the MDT. concurrence of two-thirds of all the Members of the Senate, to make
EDCA, being essentially and entirely an implementation of the MDT, is within the treaties, and with the consent of the Commission on Appointments, he shall
sole authority of the President to enter into as an executive agreement with the appoint ambassadors, other public ministers; and consuls. He shall receive
U.S. ambassadors and other public ministers duly accredited to the Government of the
Article VIII of the MDT provides: “This Treaty shall remain in force Philippines.” (Emphasis supplied)
indefinitely. Either party may terminate it one year after notice is given to the 22 See footnote 7, Nicolas v. Romulo, 598 Phil. 262; 578 SCRA 438 (2009).
other Party.” Neither the Philippines nor the United States has terminated the
MDT. On the
_______________ 490
490 SUPREME COURT REPORTS ANNOTATED
17 In its Note Verbale of 7 June 2013 to China, the Philippines stated it
“strongly objects to the indication that the nine-dash lines are China’s Saguisag vs. Ochoa, Jr.
national boundaries in the West Philippine Sea/South China Sea.”
(Emphasis supplied) In summary, the EDCA is absolutely necessary and essential to implement the
18 Pimentel, Jr. v. Office of the Executive Secretary, 501 Phil. 303; 462 SCRA purpose of the MDT, which on the part of the Philippines, given the existing
622 (2005). situation in the West Philippine Sea, is to deter or repel any armed attack on
Philippine territory or on any Philippine public vessel or aircraft operating in the
West Philippine Sea. To hold that the EDCA cannot take effect without Senate
489 ratification is to render the MDT, our sole mutual self-defense treaty, totally
VOL. 779, JANUARY 12, 2016 489 inutile to meet the grave, even existentialist,23 national security threat that the
Philippines is now facing in the West Philippine Sea.
Saguisag vs. Ochoa, Jr. China has already invaded several geologic features comprising part
of Philippine “national territory” as defined in the Constitution. The
contrary, the 1998 Visiting Forces Agreement between the Philippines and the territorial integrity of the Philippines has been violated openly and repeatedly.
United States, which the Philippine Senate has ratified, expressly states that the The President, as Commander-in-Chief of the Armed Forces, “chief architect” of
parties are “[r]eaffirming their obligations under the Mutual Defense Treaty of foreign policy and implementer of the MDT, has decided on the urgent need to
August 30, 1951.” Thus, the continued validity and relevance of the MDT cannot fortify Philippine military defenses by prepositioning war materials of our treaty
be denied.
ally on Philippine soil. This Court should not erect roadblocks to the President’s ARTICLE XVIII
implementation of the MDT, particularly since time is of the essence and the TRANSITORY PROVISIONS
President’s act of entering into the EDCA on his own does not violate any provision SEC. 25. After the expiration in 1991 of the Agreement between the
of the Constitution. Republic of the Philippines and
A final word. The EDCA does not detract from the legal arbitration case that _______________
the Philippines has filed against China under UNCLOS. The EDCA brings into
the Philippine strategy the element of credible self-defense. Having refused to 1 The Schooner Exchange v. McFaddon and Others, 3 Law. ed., 287, 293; cited
participate in the legal arbitration despite being obligated to do so under UNCLOS, in Dizon v. Commanding General of the Phil. Ryukus Command, U.S. Army, 81
China is now using brute force to assert its claim to almost the entire South China Phil. 286, 292 (1948).
Sea. Given
_______________
492
23 China’s successful control of the South China Sea will force the Philippines 492 SUPREME COURT REPORTS ANNOTATED
to share a 1,300-kilometer sea border with China, from Balabac Island in Palawan
to Yamin Island in Batanes, very close to the Philippine coastline facing the South Saguisag vs. Ochoa, Jr.
China Sea. This will bring the Philippines into China’s orbit, with the Philippines
adhering to China’s positions on matters involving foreign policy. the United States of America concerning Military Bases, foreign
military bases, troops, or facilities shall not be allowed in the Philippines
except under a treaty duly concurred in by the Senate and, when the
491 Congress so requires, ratified by a majority of the votes cast by the people
in a national referendum held for that purpose, and recognized as a treaty
VOL. 779, JANUARY 12, 2016 491
by the other contracting State.
Saguisag vs. Ochoa, Jr.
Section 25, Article XVIII bans foreign military bases, troops, or facilities in
this situation, the proper equation in defending the Philippines’ maritime Philippine territory, unless the following requisites are complied with: (1) the
zones in the West Philippine Sea is “legal right plus credible self-defense equals presence of foreign military bases, troops, or facilities should be allowed by
might.” a treaty; (2) the treaty must be duly concurred in by the Philippine
Accordingly, I vote to DISMISS the petitions on the ground that the EDCA Senate and, when Congress so requires, such treaty should be ratified by a
merely implements, and in fact is absolutely necessary and essential to the majority of the votes cast by the Filipino people in a national referendum held for
implementation of, the MDT, an existing treaty that has been ratified by the that purpose; and (3) such treaty should be recognized as a treaty by the other
Senate. contracting party.2
Couched in negative terms, Section 25, Article XVIII embodies a prohibition:
CONCURRING AND DISSENTING OPINION “foreign military bases, troops, or facilities shall not be allowed in the
Philippines,” unless the requisites in the said section are met.
LEONARDO-DE CASTRO, J.: In BAYAN v. Zamora,3 the Court held that Section 25, Article XVIII covers
three different situations: a treaty allowing the presence within the Philippines of
“The jurisdiction of the nation within its own territory is necessarily (a) foreign military bases, or (b) foreign military troops, or (c) foreign military
exclusive and absolute. It is susceptible of no limitation not imposed by facilities, such that a treaty that involves any of these three standing alone falls
itself. Any restriction upon it, deriving validity from an external source, within the coverage of the said provision.
would imply a diminution of its sovereignty to the extent of the restriction, BAYAN v. Zamora likewise expounded on the coverage of the two provisions of
and an investment of that sovereignty to the same extent in that power the Constitution — Section 21, Article VII and Section 25, Article XVIII — which
which could impose such restriction.” x x x.1 both require Senate concurrence in treaties and international agreements. The
Court stated:
I concur with the disposition of the procedural issues but not with the _______________
arguments and conclusions reached as to the substantive issues.
The focus of the present controversy, as mentioned by the Honorable Chief 2 BAYAN (Bagong Alyansang Makabayan) v. Zamora, 396 Phil. 623, 654-655;
Justice is the application of Section 25, Article XVIII of the Constitution which 342 SCRA 449, 486 (2000).
reads: 3 Id., at p. 653; p. 482.
guish — Ubi lex non distinguit nec nos distinguire debemos.
493 In like manner, we do not subscribe to the argument that Section 25,
VOL. 779, JANUARY 12, 2016 493 Article XVIII is not controlling since no foreign military bases, but merely
foreign troops and facilities, are involved in the VFA. Notably, a perusal of
Saguisag vs. Ochoa, Jr. said constitutional provision reveals that the proscription covers ‘‘foreign
military bases, troops, or facilities.” Stated differently, this prohibition is
Section 21, Article VII deals with treaties or international agreements not limited to the entry of troops and facilities without any foreign bases
in general, in which case, the concurrence of at least two-thirds (2/3) of all being established. The clause does not refer to ‘‘foreign military bases,
the Members of the Senate is required to make the subject treaty, or troops, or facilities”collectively but treats them as separate and
international agreement, valid and binding on the part of the Philippines. independent subjects. The use of comma and the disjunctive word “or”
This provision lays down the general rule on treaties or international clearly signifies disassociation and independence of one thing from the
agreements and applies to any form of treaty with a wide variety of subject others included in the enumeration, such that, the provision contemplates
matter, such as, but not limited to, extradition or tax treaties or those three different situations — a military treaty the subject of which could be
economic in nature. All treaties or international agreements entered into either (a) foreign bases, (b) foreign troops, or (c) foreign facilities — any of
by the Philippines, regardless of subject matter, coverage, or particular the three standing alone places it under the coverage of Section 25, Article
designation or appellation, requires the concurrence of the Senate to be XVIII.
valid and effective. To this end, the intention of the framers of the Charter, as manifested
In contrast, Section 25, Article XVIII is a special provision that applies during the deliberations of the 1986 Constitutional Commission, is
to treaties which involve the presence of foreign military bases, troops or consistent with this interpretation:
facilities in the Philippines. Under this provision, the concurrence of the MR. MAAMBONG. I just want to address a question or two to
Senate is only one of the requisites to render compliance with the Commissioner Bernas. This formulation speaks of three things: foreign
constitutional requirements and to consider the agreement binding on the military bases, troops or facilities. My first question is: If the country does
Philippines. Section 25, Article XVIII further requires that “foreign enter into such kind of a treaty, must it cover the three-bases, troops or
military bases, troops, or facilities” may be allowed in the Philippines only facilities or could the treaty entered into cover only one or two?
by virtue of a treaty duly concurred in by the Senate, ratified by a majority FR. BERNAS. Definitely, it can cover only one. Whether it covers only
of the votes cast in a national referendum held for that purpose if so one or it covers three, the requirement will be the same.
required by Congress, and recognized as such by the other contracting MR. MAAMBONG. In other words, the Philippine government can enter
state. into a treaty covering not bases but merely troops?
xxxx FR. BERNAS. Yes.
Moreover, it is specious to argue that Section 25, Article XVIII is
inapplicable to mere transient agreements for the reason that there is no
permanent placing of structure for the establishment of a military base. On 495
this score, the Constitution makes no distinction between “transient” and
VOL. 779, JANUARY 12, 2016 495
“permanent.” Certainly, we find nothing in Section 25, Article XVIII that
requires foreign troops or facilities to be stationed or placed permanently in Saguisag vs. Ochoa, Jr.
the Philippines.
It is a rudiment in legal hermeneutics that when no distinction is made MR. MAAMBONG. I cannot find any reason why the government can
by law the Court should not distin- enter into a treaty covering only troops.
_______________ FR. BERNAS. Why not? Probably if we stretch our imagination a little
bit more, we will find some. We just want to cover everything.4 (Citations
2 BAYAN (Bagong Alyansang Makabayan) v. Zamora, 396 Phil. 623, 654-655; omitted)
342 SCRA 449, 486 (2000).
3 Id., at p. 653; p. 482. Furthermore, the wording of Section 25, Article XVIII also provides an
indubitable implication: foreign military bases, troops and facilities have
ceased to be allowed in the Philippines after the expiration in 1991 of the
494 Military Bases Agreement; thereafter, the same can only be reallowed
494 SUPREME COURT REPORTS ANNOTATED upon the satisfaction of all the three requirements set forth in the Section
25, Article XVIII.
Saguisag vs. Ochoa, Jr. The legal consequence of the above provision with respect to the Military Bases
Agreement (March 14, 1947), the Mutual Defense Treaty (August 30, 1951), the
Visiting Forces Agreement (February 10, 1998), and the Enhanced Defense 1. The Government of the Republic of the Philippines (hereinafter referred to
Cooperation Agreement ([EDCA] April 28, 2014) can be appreciated by an as the Philippines) grants to the Government of the United States of America
examination of the respective rights and obligations of the parties in these (hereinafter referred to as the United States) the right to retain the use of the
agreements. bases in the Philippines listed in Annex A attached hereto.
2. The Philippines agrees to permit the United States, upon notice to the
Effect of Section 25, Article XVIII Philippines, to use such of those bases listed in Annex B as the United States
of the Constitution on the Military determines to be required by military necessity.
Bases Agreement, the Mutual De- 3. The Philippines agrees to enter into negotiations with the United States at
fense Treaty, the Visiting Forces the latter’s request, to permit the United States to expand such bases, to exchange
Agreement, and the Enhanced such bases for other bases, to acquire additional bases, or relinquish rights to
Defense Cooperation Agreement bases, as any of such exigencies may be required by military necessity.

On July 4, 1946, the United States recognized the independence of the Republic
of the Philippines, thereby apparently relinquishing any claim of sovereignty 497
thereto. However, on March 14, 1947, the Philippines and the United VOL. 779, JANUARY 12, 2016 497
_______________
Saguisag vs. Ochoa, Jr.
4 Id., at pp. 650-654; pp. 482-485.
“the rights, power and authority within the bases which are necessary for the
establishment, use, operation and defense thereof or appropriate for the control
496 thereof and all the rights, power and authority within the limits of territorial
waters and air space adjacent to, or in the vicinity of, the bases which are necessary
496 SUPREME COURT REPORTS ANNOTATED to provide access to them, or appropriate for their control.” 7 The term of the
Saguisag vs. Ochoa, Jr. original agreement was “for a period of ninety-nine years subject to extension
thereafter as agreed by the two Governments.”8 In 1966, the parties entered into
States entered into a Military Bases Agreement (MBA) which granted to the the Ramos-Rusk Agreement, which reduced the term of the Military Bases
United States government the right to retain5 the use of the bases listed in the Agreement to 25 years from 1966, or until 1991.
Annexes of said agreement.6 Within said bases, the United States was granted On August 30, 1951, the Philippines and the United States entered into
_______________ the Mutual Defense Treaty (MDT), whereby the parties recognized that “an
armed attack in the Pacific area on either of the Parties would be dangerous to its
5 The Court explained in Nicolas v. Romulo (598 Phil. 262, 279-280; 578 SCRA own peace and safety and declares that it would act to meet the common dangers
438, 456 [2009]) that: in accordance with its constitutional process.”9 The treaty provided that it “shall
“[U]nder the Philippine Bill of 1902, which laid the basis for the Philippine remain in force indefi-
Commonwealth and, eventually, for the recognition of independence, the United _______________
States agreed to cede to the Philippines all the territory it acquired from Spain
under the Treaty of Paris, plus a few islands later added to its realm, except certain 4. A narrative description of the boundaries of the bases to which this
naval ports and/or military bases and facilities, which the United States retained Agreement relates is given in Annex A and Annex B. An exact description of the
for itself. bases listed in Annex A, with metes and bounds, in conformity with the narrative
This is noteworthy, because what this means is that Clark and Subic and the descriptions, will be agreed upon between the appropriate authorities of the two
other places in the Philippines covered by the RP-US Military Bases Agreement of Governments as soon as possible. With respect to any of the bases listed in Annex
1947 were not Philippine territory, as they were excluded from the cession and B, an exact description with metes and bounds, in conformity with the narrative
retained by the US. description of such bases, will be agreed upon if and when such bases are acquired
xxxx by the United States.
Subsequently, the United States agreed to turn over these bases to the 7 Id., Article III(1).
Philippines; and with the expiration of the RP-US Military Bases Agreement in 8 Id., Article XXIX.
1991, the territory covered by these bases were finally ceded to the Philippines.” 9 Articles IV and V of the Mutual Defense Treaty (August 30, 1951) provides:
6 Military Bases Agreement (March 14, 1947), Article I, which provides: ARTICLE IV
Article I
GRANT OF BASES
Each Party recognizes that an armed attack in the Pacific Area on either of the eignty is being derogated. Our situation is different from theirs because
Parties would be dangerous to its own peace and safety and declares that it would we did not lease or rent these bases to the U.S. The U.S. retained them from
act to meet the common dangers in accordance with its constitutional process. us as a colonial power.
FR. BERNAS. So, the second sentence, Madam President, has specific
reference to what obtains now.
498 MR. AZCUNA. Yes. It is really determined by the present situation.
498 SUPREME COURT REPORTS ANNOTATED FR. BERNAS. Does the first sentence tolerate a situation radically
different from what obtains now? In other words, if we understand
Saguisag vs. Ochoa, Jr. sovereignty as auto-limitation, as a people’s power to give up certain goods
in order to obtain something which may be more valuable, would it be
nitely,” although either party “may terminate it one year after notice has been possible under this first sentence for the nation to negotiate some kind of a
given to the other Party.”10 It bears pointing out that there is no explicit provision treaty agreement that would not derogate against sovereignty?
in the MDT which authorized the presence in the Philippines of military bases, MR. AZCUNA. Yes. For example, Madam President, if it is negotiated
troops, or facilities of the United States. on a basis of true sovereign equality, such as a mutual ASEAN defense
In 1986, during the early stages of the deliberations of the Constitutional agreement wherein an ASEAN force is created and this ASEAN force is a
Commission, and in view of the impending expiration of the MBA in 1991, the foreign military force and may have a basis in the member ASEAN
members of the Commission expressed their concern that the continued presence countries, this kind of a situation, I think, would not derogate from
of foreign military bases in the country would amount to a derogation of national sovereignty.
sovereignty. The pertinent portion of the deliberations leading to the adoption of MR. NOLLEDO. Madam President, may I be permitted to make a
the present Section 25, Article XVIII is quoted as follows: comment on that beautiful question. I think there will be no derogation of
FR. BERNAS. My question is: Is it the position of the committee that sovereignty if the existence of the military bases as stated by Commissioner
the presence of foreign military bases in the country under any Azcuna is on the basis of a treaty which was not only ratified by the
circumstances is a derogation of national sovereignty? appropriate body, like the Congress, but also by the people.
MR. AZCUNA. It is difficult to imagine a situation based on existing I would like also to refer to the situation in Turkey where the Turkish
facts where it would not. However, in the abstract, it is possible that it government has control over the bases in Turkey, where the jurisdiction of
would not be that much of a derogation. I have in mind, Madam President, Turkey is not impaired in anyway, and Turkey retains the right to
the argument that has been presented. Is that the reason why there are terminate the treaty under circumstances determined by the host
U.S. bases in England, in Spain and in Turkey? And it is not being claimed government. I think under such circumstances, the existence of the military
that their sover- bases may not be considered a derogation of sovereignty, Madam President.
_______________ FR. BERNAS. Let me be concrete, Madam President, in our
circumstances. Suppose they were to
Any such armed attack and all measures taken as a result thereof shall be
immediately reported to the Security Council of the United Nations. Such
measures shall be terminated when the Security Council has taken the measures 500
necessary to restore and maintain international peace and security. 500 SUPREME COURT REPORTS ANNOTATED
ARTICLE V
For the purpose of Article IV, an armed attack on either of the Parties is Saguisag vs. Ochoa, Jr.
deemed to include an armed attack on the metropolitan territory of either of the
Parties, or on the island territories under its jurisdiction in the Pacific or on its have this situation where our government were to negotiate a
armed forces, public vessels or aircraft in the Pacific. treaty with the United States, and then the two executive
10 Id., Article VIII. departments in the ordinary course of negotiation come to an
agreement. As our Constitution is taking shape now, if this is to be
a treaty at all, it will have to be submitted to our Senate for its
499 ratification. Suppose, therefore, that what was agreed upon
between the United States and the executive department of the
VOL. 779, JANUARY 12, 2016 499
Philippines is submitted and ratified by the Senate, then it is
Saguisag vs. Ochoa, Jr. further submitted to the people for its ratification and
subsequently, we ask the United States: “Complete the process by
accepting it as a treaty through ratification by your Senate as the
United States Constitution requires,” would such an arrangement or for the United States armed forces upon approval of the Government of the
be in derogation of sovereignty? Philippines, in accordance with international
MR. NOLLEDO. Under the circumstances the Commissioner just _______________
mentioned, Madam President, on the basis of the provision of
Section 1 that “sovereignty resides in the Filipino people,” then we 13 Visiting Forces Agreement (February 10, 1998), Article III.
would not consider that a derogation of our sovereignty on the 14 Id., Article I.
basis and expectation that there was a plebiscite.11 (Emphasis 15 Id., Article VII.
supplied) 16 Id.

As a safeguard against the derogation of national sovereignty, the present form


of Section 25, Article XVIII was finalized by the Commission and ratified by the 502
Filipino people in 1987.
502 SUPREME COURT REPORTS ANNOTATED
On September 16, 1991, the Senate rejected the proposed Treaty of Friendship,
Cooperation and Security, which would have extended the presence of US military Saguisag vs. Ochoa, Jr.
bases in the Philippines. Nevertheless, the defense and security relationship
between the Philippines and the United States continued in accordance with the custom and practice and such agreed implementing arrangements as
MDT.12 necessary.17
_______________ The VFA also provided for the jurisdiction over criminal and disciplinary cases
over United States personnel with respect to offences committed within the
11 IV Record of the Constitutional Commission, pp. 661-662. Philippines.18
12 Supra note 2. The VFA further stated that the same shall remain in force until the expiration
of 180 days from the date on which either party gives the other party notice in
writing that it desires to terminate the agreement.19
501 Subsequently, the constitutionality of the VFA was questioned before the
Court in the aforementioned October 10, 2000 case of BAYAN v. Zamora,20 and
VOL. 779, JANUARY 12, 2016 501
again in the case of Nicolas v. Romulo.21 In both cases, the Court held that Section
Saguisag vs. Ochoa, Jr. 25, Article XVIII of the Constitution is applicable, but the requirements thereof
were nevertheless complied with. In Nicolas, however, the implementing Romulo-
Upon the expiration of the MBA in 1991, Section 25, Article XVIII came into Kenney Agreements of December 19 and 22, 2006 concerning the custody of Lance
effect. The presence of foreign military bases, troops or facilities in the country can Corporal Daniel J. Smith, who was charged with the crime of rape, were declared
only be allowed upon the satisfaction of all three requirements set forth in Section not in accordance with the VFA.
25, Article XVIII. Thereafter, on April 28, 2014, the governments of the Philippines and the
On February 10, 1998, the Philippines and the United States entered into the United States entered into the assailed EDCA.
Visiting Forces Agreement (VFA), which required the Philippines to facilitate the
admission of United States personnel,13 a term defined in the same treaty as The EDCA
“United States military and civilian personnel temporarily in the Philippines in
connection with activities approved by the Philippine Government.”14 Under the EDCA, the Philippines by mutual agreement with the United
United States Government equipment, materials, supplies, and other property States, shall provide the United States forces the access and use of portions of
imported into the Philippines in connection with activities to which the VFA Philippine territory. United States forces are “the entity comprising United States
applies, while not expressly stated to be allowed into the Philippines by the personnel and all property, equipment, and materiel of the United States Armed
provisions of the VFA, were nevertheless declared to be free from Philippine duties, Forces present in the territory of the
taxes and similar charges. Title thereto was also declared to remain with the _______________
United States.15
The VFA expressly allowed the importation into the Philippines of reasonable 17 Id., Article VIII.
quantities of personal baggage, personal effects, and other property for the 18 Id., Article V.
personal use of United States personnel.16 The VFA likewise expressly allowed the 19 Id., Article IX.
entry into the Philippines of (1) aircraft operated by or for the United States armed 20 Supra note 2.
forces upon approval of the Government of the Philippines in accordance with 21 Supra note 5.
procedures stipulated in implementing arrangements; and (2) vessels operated by
The United States may access and use the Agreed Locations without any
503 obligation on its part to pay any rent or similar costs.24
VOL. 779, JANUARY 12, 2016 503 In addition to the right to access and to use the Agreed Locations and to
conduct various activities therein, the United States, upon request to the
Saguisag vs. Ochoa, Jr. Philippines’ Designated Authorities,25 can further temporarily access public land
and facilities (including roads, ports, and airfields), including those owned or
Philippines.” These portions of Philippine territory that will be made available controlled by local governments, and to other land and facilities (including roads,
to the US are called “Agreed Locations,” which is a new concept defined under ports, and airfields).26
Article II(4) of the EDCA as: The United States is also granted operational control of Agreed Locations to do
construction activities, make alterations or improvements of the Agreed
4. “Agreed Locations” means facilities and areas that are provided by Locations.27 All buildings, non-relocatable structures, and assemblies affixed to
the Government of the Philippines through the AFP and that the United the land in the Agreed Locations, including [those] altered or improved by United
States forces,22 United States contractors, and others as mutually agreed, States forces, remain the property of the Philippines. Permanent buildings
shall have the right to access and use pursuant to this Agreement. Such constructed by the United States forces become the property of the Philippines,
Agreed Locations may be listed in an annex to be appended to this once constructed, but shall be used by the United States forces until no longer
Agreement, and may further be described in implementing arrangements. required.28
(Emphasis supplied) Incidental to the access and use of the Agreed Locations, the US is granted the
use of water, electricity and other pub-
Aside from the right to access and to use the Agreed Locations, the United _______________
States may undertake the following types of activities within the Agreed Locations:
security cooperation exercises; joint and combined training activities; 24 Id., Article III(3).
humanitarian and disaster relief activities; and such other activities that as may 25 Id., Article II(5) states:
be agreed upon by the Parties.”23 Article III(1) of the EDCA further states in detail 5. “Designated Authorities” means, respectively, the Philippine Department
the activities that the United States may conduct inside the Agreed Locations: of National Defense, unless the Philippines otherwise provides written notice to
the United States, and the United States Department of Defense, unless the
1. With consideration of the views of the Parties, the Philippines United States otherwise provides written notice to the Philippines.
hereby authorizes and agrees that United States forces, United States 26 Id., Article III(2).
contractors, and vehicles, vessels, and aircrafts operated by or for United 27 Id., Article III(4).
States forces may conduct the following activities with respect to Agreed 28 Id., Article V(4).
Locations: training; transit; support and related activities; refueling
of aircraft; bunkering of vessels; temporary maintenance of
vehicles, vessels, and aircraft; temporary accommodation of 505
personnel; communications; prepositioning of
VOL. 779, JANUARY 12, 2016 505
_______________ Saguisag vs. Ochoa, Jr.

22 “United States forces” means the entity comprising United States lic utilities,29 as well as the use of the radio spectrum in relation to the
personnel and all property, equipment and materiel of the United States Armed operation of its own telecommunications system.30
Forces present in the territory of the Philippines. [Enhanced Defense Cooperation As to the management of the Agreed Locations, the United States forces are
Agreement, Article II(2)] authorized to exercise an rights and authorities within the Agreed Locations that
23 Enhanced Defense Cooperation Agreement, Article I(3). are necessary for their operational control or defense, including taking appropriate
measures to protect United States forces and United States contractors. The
United States should coordinate such measures with appropriate authorities of the
504 Philippines.31
504 SUPREME COURT REPORTS ANNOTATED The United States is authorized to preposition and store defense equipment,
supplies, and materiel (“prepositioned materiel”), including but not limited to,
Saguisag vs. Ochoa, Jr. humanitarian assistance and disaster relief equipment, supplies and material, at
Agreed Locations.32 The prepositioned materiel of the United States forces shall be
equipment, supplies, and materiel; deploying forces and materiel; and for the exclusive use of United States forces, and full title to all such equipment,
such other activities as the Parties may agree. (Emphasis supplied)
supplies and materiel remains with the United States. 33United States forces and _______________
United States contractors34 shall have unimpeded access to Agreed Locations for
all matters relating to the prepositioning and storage of defense equipment, 36 Id., Article V(3).
supplies, and materiel, including delivery, management, inspection, use, 37 Annexes A and B referred to under the MBA included the following military
maintenance, and removal of such equipment, supplies and materiel. 35 The United bases in the Philippines, namely: Clark Field Air Base, Pampanga; Mariveles
States forces and United States contractors shall retain title to all equipment, Military Reservation, POL Terminal and Training Area, Bataan; Camp John Hay
materiel, supplies, relocat- Leave and Recreation Center, Baguio; Subic Bay, Northwest Shore Naval Base,
_______________ Zambales Province, and the existing Naval reservation at Olongapo and the
existing Baguio Naval Reservation; Cañacao-Sangley Point Navy Base, Cavite
29 Id., Article VII(1). Province; Mactan Island Army and Navy Air Base; Florida Blanca Air Base,
30 Id., Article VII(2). Pampanga; Camp Wallace, San Fernando, La Union; and Aparri Naval Air Base,
31 Id., Article VI(3). among others. (Military Bases Agreement [March 14, 1947])
32 Id., Article IV(1).
33 Id., Article IV(3).
34 Id., Article II defines United States contractors as: 507
3. “United States contractors” means companies and firms, and their VOL. 779, JANUARY 12, 2016 507
employees, under contract or subcontract to or on behalf of the United States
Department of Defense. United States contractors are not included as part of the Saguisag vs. Ochoa, Jr.
definition of United States personnel in this Agreement, including within the
context of the VFA. Considering further that the United States armed forces stationed in the
35 Id., Article IV(4). Philippines, as well as their relocatable structures, equipment and materiel are
owned, maintained, controlled, and operated by the United States within
Philippine territory, these Agreed Locations are clearly overseas military bases of
506 the US with RP as its host country.
The EDCA provided for an initial term of ten years, which thereafter shall
506 SUPREME COURT REPORTS ANNOTATED
continue in force automatically, unless terminated by either party by giving one
Saguisag vs. Ochoa, Jr. year’s written notice through diplomatic channels of its intention to terminate the
agreement.38
able structures, and other movable property that have been imported into or Interestingly, the EDCA has similar provisions found in the 1947 MBA:
acquired within the territory of the Philippines by or on behalf of United States
forces.36 _______________
Considering the presence of US armed forces: military personnel, vehicles,
vessels, and aircrafts and other defensive equipment, supplies, and materiel in the 38 Enhanced Defense Cooperation Agreement, Article XII(4).
Philippines, for obvious military purposes and with the obvious intention of
assigning or stationing them within the Agreed Locations, said Agreed Locations,
for all intents and purposes, are considered military bases and fall squarely under 508
the definition of a military base under Section 2, Presidential Decree No. 1227,
508 SUPREME COURT REPORTS ANNOTATED
otherwise known as “Punishing Unlawful Entry into Any Military Base in the
Philippines,” which states: Saguisag vs. Ochoa, Jr.

SECTION 2. The term “military base” as used in this decree


means any military, air, naval, or coast guard reservation, base,
fort, camp, arsenal, yard, station, or installation in the Philippines.
(Emphasis supplied) 509
VOL. 779, JANUARY 12, 2016 509
In the same vein, Article XXVI of the 1947 RP-US Military Bases Agreement
(MBA) defined a military base as “areas named in Annex A and Annex B and such Saguisag vs. Ochoa, Jr.
additional areas as may be acquired for military purposes pursuant to the terms
of this Agreement.”37
Treaties, a treaty is defined as an international agreement concluded between
510 states in written form and governed by international law, whether
510 SUPREME COURT REPORTS ANNOTATED _______________

Saguisag vs. Ochoa, Jr. 39 Supra note 2 at p. 653; p. 484.

512
The EDCA is not a mere
implementing agreement 512 SUPREME COURT REPORTS ANNOTATED
of the MDT or the VFA Saguisag vs. Ochoa, Jr.

As can be seen in the above table of comparison, these EDCA provisions embodied in a single instrument or in two or more related instruments and
establishes military areas similar to that in the Military Bases whatever its particular designation.40
Agreement, and for that reason alone, the EDCA is far greater in scope than both In the 1961 case of Commissioner of Customs v. Eastern Sea Trading,41 the
the Mutual Defense Treaty and the Visiting Forces Agreement. The EDCA is not Court had occasion to state that “[i]nternational agreements involving political
a mere implementing agreement of either the MDT or the VFA. issues or changes of national policy and those involving international
The EDCA is an international agreement that allows the presence in arrangements of a permanent character usually take the form of treaties. But
the Philippines of foreign military international agreements embodying adjustments of detail carrying out well-
established national policies and traditions and those involving arrangements of a
more or less temporary nature usually take the form of executive agreements.
511 In the more recent case of Bayan Muna v. Romulo,42 the Court expounded on
VOL. 779, JANUARY 12, 2016 511 the above pronouncement in this wise:
Saguisag vs. Ochoa, Jr. The categorization of subject matters that may be covered by
international agreements mentioned in Eastern Sea Trading is not cast in
bases, troops and facilities, and thus requires that the three requisites stone. There are no hard and fast rules on the propriety of entering,
under Section 25, Article XVIII be complied with. The EDCA must be submitted to on a given subject, into a treaty or an executive agreement as an
the Senate for concurrence. instrument of international relations. The primary consideration
The majority opinion posits, inter alia, that the President may enter into in the choice of the form of agreement is the parties’ intent and
an executive agreement on foreign military bases, troops, or facilities if: (a) it desire to craft an international agreement in the form they so wish
“is not the principal agreement that first allowed their entry or presence to further their respective interests. Verily, the matter of form takes a
in the Philippines,” or (b) it merely aims to implement an existing law or treaty. back seat when it comes to effectiveness and binding effect of the
Likewise, the President alone had the choice to enter into the EDCA by way of an enforcement of a treaty or an executive agreement, as the parties in either
executive agreement or a treaty. Also, the majority suggests that executive international agreement each labor under the pacta sunt
agreements may cover the matter of foreign military forces if it involves detail servanda principle.
adjustments of previously existing international agreements. As may be noted, almost half a century has elapsed since the Court
The above arguments fail to consider that Section 25, Article XVIII of the rendered its decision in Eastern Sea Trading. Since then, the conduct of
Constitution covers three distinct and mutually independent situations: the foreign affairs has become more complex and the domain of international
presence of foreign military bases or troops or facilities. The grant of entry to law wider, as to include such subjects as human rights,
foreign military troops does not necessarily allow the establishment of military _______________
bases or facilities.39
Generally, the parties to an international agreement are given the freedom to 40 Id., at p. 657; pp. 488-489.
choose the form of their agreement. 41 113 Phil. 333, 338; 3 SCRA 351, 356 (1961).
International agreements may be in the form of: (1) treaties, which require 42 656 Phil. 246, 271-272; 641 SCRA 244, 260-262 (2011).
legislative concurrence after executive ratification; or (2) executive agreements,
which are similar to treaties, except that they do not require legislative
concurrence and are usually less formal and deal with a narrower range of subject
513
matters than treaties. Under Article 2 of the Vienna Convention on the Law of
VOL. 779, JANUARY 12, 2016 513
Saguisag vs. Ochoa, Jr. What the majority did is to carve out exceptions to Section 25, Article XVIII
when none is called for.
the environment, and the sea. x x x Surely, the enumeration in Eastern As previously discussed, the language of Section 25, Article XVIII is clear and
Sea Trading cannot circumscribe the option of each state on the matter of unambiguous. The cardinal rule is that the plain, clear and unambiguous language
which the international agreement format would be convenient to serve its of the Constitution should be construed as such and should not be given a
best interest. As Francis Sayre said in his work referred to earlier: construction that changes its meaning.45 The Court also enunciated in Chavez v.
x x x It would be useless to undertake to discuss here the large Judicial and Bar Council46 that:
variety of executive agreements as such concluded from time to
time. Hundreds of executive agreements, other than those entered The Constitution evinces the direct action of the Filipino people by
into under the trade agreement act, have been negotiated with which the fundamental powers of government are established, limited and
foreign governments. x x x. They cover such subjects as the defined and by which those powers are distributed among the several
inspection of vessels, navigation dues, income tax on shipping departments for their safe and useful exercise for the benefit of the body
profits, the admission of civil air craft, custom matters and politic. The Framers reposed their wisdom and vision on one suprema lex to
commercial relations generally, international claims, postal be the ultimate expression of the principles and the framework upon
matters, the registration of trademarks and copyrights, etc. x x x. _______________
(Citations omitted)
43 Id., at p. 273; pp. 262-263.
However, it must be emphasized that while in the above case, the Court called 44 Supra note 2.
attention to “one type of executive agreement which is a treaty-authorized or 45 Soriano III v. Lista, 447 Phil. 566, 570; 399 SCRA 437, 440-441 (2003).
a treaty-implementing executive agreement, which necessarily would cover the 46 G.R. No. 202242, April 16, 2013, 696 SCRA 496, 507-508.
same matter subject of the underlying treaty,” still, the Court cited the special
situation covered by Section 25, Article XVIII of the Constitution which explicitly
prescribes the form of the international agreement. The Court stated: 515
VOL. 779, JANUARY 12, 2016 515
But over and above the foregoing considerations is the fact that — save
Saguisag vs. Ochoa, Jr.
for the situation and matters contemplated in Sec. 25, Art. XVIII of
the Constitution — when a treaty is required, the Constitution does
which government and society were to operate. Thus, in the
not classify any subject, like that involving political issues, to be in the form
interpretation of the constitutional provisions, the Court firmly relies on
of, and ratified as, a treaty. What the Constitution merely prescribes is that
the basic postulate that the Framers mean what they say. The language
treaties need the concurrence of the Senate by a vote defined
used in the Constitution must be taken to have been deliberately
chosen for a definite purpose. Every word employed in the
Constitution must be interpreted to exude its deliberate intent
514
which must be maintained inviolate against disobedience and
514 SUPREME COURT REPORTS ANNOTATED defiance. What the Constitution clearly says, according to its text,
Saguisag vs. Ochoa, Jr. compels acceptance and bars modification even by the branch
tasked to interpret it. (Emphasis supplied; citation omitted)
therein to complete the ratification process.43 (Emphasis supplied,
citation omitted) The majority opinion posits that the EDCA is consistent with the content,
purpose and framework of the MDT and the VFA. As such, the majority argues
Clearly, the Court had since ruled that when the situation and matters that the EDCA may be in the form of an executive agreement as it merely
contemplated in Sec. 25, Article XVIII obtains, i.e., when the subject matter of an implements the provisions of the MDT and the VFA.
international agreement involves the presence of foreign military bases, troops or I disagree. Compared closely with the provisions of the MDT and the VFA, the
facilities, a treaty is required and that the same must be submitted to the Senate EDCA transcends in scope and substance the subject matters covered by the
for the latter’s concurrence. In BAYAN v. Zamora,44 the Court held that Section aforementioned treaties. Otherwise stated, the EDCA is an entirely new
25, Article XVIII, like Section 21, Article VII, embodies a phrase in the agreement unto itself.
negative, i.e., “shall not be allowed” and therefore, the concurrence of the Senate
is indispensable to render the treaty or international agreement valid and The MDT in relation to the EDCA
effective.
We noted in Lim v. Executive Secretary47 that the MDT has been described as
the “core” of the defense relationship between the Philippines and its traditional
ally, the United States. The aim of the treaty is to enhance the strategic and 517
technological capabilities of our armed forces through joint training with its VOL. 779, JANUARY 12, 2016 517
American counterparts.
As explicitly pronounced in its declaration of policies, the MDT was entered Saguisag vs. Ochoa, Jr.
into between the Philippines and the
United Nations the occurrence of any such armed attack and all the measures
_______________ taken as result thereof. Said measures shall be terminated when the Security
Council has taken the measures necessary to restore and maintain international
47 430 Phil. 555, 571-572; 380 SCRA 739, 752 (2002). peace and security.53Article V of the treaty explained that “an armed attack on
either of the Parties is deemed to include an armed attack on the metropolitan
territory of either of the Parties, or on the island territories under its
516 jurisdiction in the Pacific or on its armed forces, public vessels or
aircraft in the Pacific.”54
516 SUPREME COURT REPORTS ANNOTATED Under Article VIII of the treaty, the parties agreed that the treaty shall remain
Saguisag vs. Ochoa, Jr. in force indefinitely and that either party may terminate it one year after notice
has been given to the other party.55
United States in order to actualize their desire “to declare publicly and Clear from the foregoing provisions is that the thrust of the MDT pertains to
formally their sense of unity and their common determination to defend the furtherance of the avowed purpose of the parties thereto of maintaining and
themselves against external armed attack”48 and “further to strengthen their developing their individual and collective capacity to resist external armed
present efforts to collective defense for the preservation of peace and security attack only in the metropolitan territory of either party or in their island
pending the development of a more comprehensive system of regional security in territories in the Pacific Ocean. Accordingly, the territories of the parties
the Pacific area.”49 other than those mentioned are not covered by the MDT.
Under Article II of the MDT, the parties undertook “separately and jointly by Conspicuously absent from the MDT are specific provisions regarding the
self-help and mutual aid” to “maintain and develop their individual and collective presence in Philippine territory — whether permanent or temporary — of foreign
capacity to resist armed attack.”50Article III thereof states that the parties to the military bases, troops, or facilities. The MDT did not contemplate the presence of
treaty shall “consult together from time to time regarding the implementation of foreign military bases, troops or facilities in our country in view of the fact that it
[the] Treaty and whenever in the opinion of either of them the territorial integrity, was already expressly covered by the MBA that was earlier entered into by the
political independence or security of either of the Parties is threatened by external Philippines and the United States in 1947. Moreover, the MDT contains no
armed attack in the Pacific.”51 delegation of power to the President to enter into an agreement relative to the
Moreover, Article IV states that the individual parties to the treaty “recognizes establishment of foreign military bases, troops,
that an armed attack in the Pacific area on either of the Parties would be _______________
dangerous to its own peace and safety and declares that it would act to meet the
common dangers in accordance with its constitutional process.”52 This 53 Id., Article IV, second paragraph.
provision highlights the need for each party to follow their respective 54 Id., Article V.
constitutional processes and, therefore, the MDT is not a self-executing agreement. 55 Id., Article VII.
It follows that if the Philippines aims to implement the MDT in the manner that
the majority opinion suggests, such implementation must adhere to the mandate
of Section 25, Article XVIII of the Constitution. 518
Also, under the above article, the parties are thereafter obligated to 518 SUPREME COURT REPORTS ANNOTATED
immediately report to the Security Council of the
_______________ Saguisag vs. Ochoa, Jr.

48 Mutual Defense Treaty, Preamble, paragraph 3. or facilities in our country. The MDT cannot also be treated as allowing an
49 Id., Preamble, paragraph 4. exception to the requirements of Section 25, Article XVIII of the Constitution,
50 Id., Article II. which took effect in 1987. As explained above, the reference to constitutional
51 Id., Article III. processes of either party in the MDT renders it obligatory that the Philippines
52 Id., Article IV, first paragraph. follow Section 25, Article XVIII of the Constitution.
Indeed, the MDT covers defensive measures to counter an armed attack between American and Philippine military forces in the event of an attack
against either of the parties’ territories or armed forces but there is nothing in the by a common foe.
MDT that specifically authorizes the presence, whether temporary or permanent,
of a party’s bases, troops, or facilities in the other party’s territory even during To a certain degree, the VFA is already an amplification of the MDT in that it
peace time or in mere anticipation of an armed attack. allows the presence of visiting foreign troops for cooperative activities in peace
On the other hand, the very clear-cut focal point of the EDCA is the authority time. Thus, in line with the mandate of Section 25, Article XVIII of the Constitu-
granted to the United States forces and contractors to have unimpeded access to _______________
so-called Agreed Locations — which can be anywhere in the Philippines — and to
build there military facilities and use the same to undertake various military 56 Supra note 2 at p. 652; p. 483.
activities. The very wording of the EDCA shows that it undoubtedly deals with the 57 Lim v. Executive Secretary, supra note 47 at p. 572; p. 752.
presence of foreign military bases, troops, and facilities in Philippine territory.
Thus, contrary to the posturing of the majority, the presence of foreign military
bases, troops, or facilities provided under the EDCA cannot be traced to the MDT. 520
Moreover, the general provisions of the MDT cannot prevail over the categorical
520 SUPREME COURT REPORTS ANNOTATED
and specific provision of Section 25, Article XVIII of the Constitution.
As will be further highlighted in the succeeding discussion, the EDCA creates Saguisag vs. Ochoa, Jr.
new rights, privileges and obligations between the parties thereto.
tion, the VFA is embodied in a treaty concurred in by the Senate.
The VFA in relation to the EDCA In particular, the coverage of the VFA is as follows:
1) The admission of United States personnel and their departure from
With respect to the VFA, the EDCA likewise surpasses the provisions of the Philippines in connection with activities covered by the agreement, and the grant
said former treaty. of exemption to United States personnel from passport and visa regulations upon
entering and departing from the Philippines;58
2) The validity of the driver’s license or permit issued by the United States,
519 thus giving United States personnel the authority to operate military or official
VOL. 779, JANUARY 12, 2016 519 vehicles within the Philippines;59
3) The rights of the Philippines and the United States in matters of criminal
Saguisag vs. Ochoa, Jr. jurisdiction over United States personnel who commit offenses within the
Philippine territory and punishable under Philippine laws;60
The VFA primarily deals with the subject of allowing elements of the United 4) The importation and exportation of equipment, materials, supplies and
States armed forces to visit the Philippines from time to time for the purpose of other property, by United States personnel free from Philippine duties, taxes and
conducting activities, approved by the Philippine government, in line with the similar charges;61
promotion and protection of the common security interests of both countries. 5) The movement of United States aircrafts, vessels and vehicles within
In the case of BAYAN v. Zamora,56 the Court ruled that the VFA “defines the Philippine territory;62 and
treatment of United States troops and personnel visiting the Philippines,” 6) The duration and termination of the agreement.63
“provides for the guidelines to govern such visits of military personnel,” and
“defines the rights of the United States and the Philippine government in the In contrast, the EDCA specifically deals with the following matters:
matter of criminal jurisdiction, movement of vessel and aircraft, importation and _______________
exportation of equipment, materials and supplies.”
We likewise reiterated in Lim v. Executive Secretary,57 that: 58 Visiting Forces Agreement, Article III.
59 Id., Article IV.
The VFA provides the “regulatory mechanism” by which “United States 60 Id., Article V.
military and civilian personnel [may visit] temporarily in the 61 Id., Article VII.
Philippines in connection with activities approved by the Philippine 62 Id., Article VIII.
Government.” It contains provisions relative to entry and departure of 63 Id., Article IX.
American personnel, driving and vehicle registration, criminal jurisdiction,
claims, importation and exportation, movement of vessels and aircraft, as
well as the duration of the agreement and its termination. It is the VFA 521
which gives continued relevance to the MDT despite the passage of years.
VOL. 779, JANUARY 12, 2016 521
Its primary goal is to facilitate the promotion of optimal cooperation
Saguisag vs. Ochoa, Jr. against the intentional release of hazardous waste by the United States and the
containment of thereof in case a spill occurs;73
1) The authority of the United States forces to access facilities and areas, 11) The need to execute implementing arrangements to address details
termed as “Agreed Locations,” and the activities that may be allowed therein; 64 concerning the presence of United States forces at the Agreed Locations and the
2) The grant to the United States of operational control of Agreed Locations to functional relations between the United States forces and the AFP with respect to
do construction activities and make alterations or improvements thereon; 65 the Agreed Locations;74 and
3) The conditional access to the Agreed Locations of the Philippine Designated 12) The resolution of disputes arising from the EDCA through consultation
Authority and its authorized representative;66 between the parties.75
4) The storage and prepositioning of defense equipment, supplies and materiel, _______________
as well as the unimpeded access granted to the United States contractors to the
Agreed Locations in matters regarding the prepositioning, storage, delivery, 69 Id., Article VI.
management, inspection, use, maintenance and removal of the defense equipment, 70 Id., Article VII(1).
supplies, and materiel; and the prohibition that the preposition materiel shall not 71 Id., Article VII(2).
include nuclear weapons;67 72 Id., Article VIII.
5) a) The ownership of the Agreed Locations by the Philippines, b) the 73 Id., Article IX.
ownership of the equipment, materiel, supplies, relocatable structures and other 74 Id., Article X.
moveable property imported or acquired by the United States, c) the ownership 75 Id., Article XI.
and use of the buildings, non-relocatable structures, and assemblies affixed to the
land inside the Agreed Locations;68
6) The cooperation between the parties in taking measures to ensure 523
protection, safety and security of United States forces, contractors and information VOL. 779, JANUARY 12, 2016 523
in Philippine territory; the primary responsibility of the
Saguisag vs. Ochoa, Jr.
_______________
Initially, what is abundantly clear with the foregoing enumeration is that the
64 Enhanced Defense Cooperation Agreement, Article II. EDCA is an entirely new creation. The provisions of the EDCA are not found in or
65 Id., Article III(4). have no corresponding provisions in the VFA. They cover entirely different subject
66 Id., Article III(5). matters and they create new and distinct rights and obligations on the part of the
67 Id., Article IV.
Philippines and the United States.
68 Id., Article V. Furthermore, as to the nature of the presence of foreign military troops in this
country, the VFA is explicit in its characterization that it is an agreement between
the governments of the Philippines and the United States regarding the treatment
522 of United States Armed Forces visiting the Philippines. The Preamble of the VFA
522 SUPREME COURT REPORTS ANNOTATED likewise expressly provides that, “noting that from time to time elements of the
Saguisag vs. Ochoa, Jr. United States armed forces may visit the Republic of the Philippines”76 and
“recognizing the desirability of defining the treatment of United States
Philippines to secure the Agreed Locations, and the right of the United States personnel visiting the Republic of the Philippines”77 the parties to the VFA
to exercise all rights and authorities within the Agreed Locations that are agreed to enter into the said treaty. The use of the word visit is very telling. In its
necessary for their operational control or defense;69 ordinary usage, to visit is to “stay temporarily with (someone) or at (a place) as a
7) The use of water, electricity and other public utilities;70 guest or tourist” or to “go to see (someone or something) for a specific
8) The use of the radio spectrum in connection with the operation of a purpose.”78 Thus, the word visit implies the temporariness or impermanence of the
telecommunications system by the United States;71 presence at a specific location.
9) The authority granted to the of the United States to contract for any On the other hand, under the EDCA, United States forces and United States
materiel, supplies, equipment, and services (including construction) to be contractors are permitted to stay in the Agreed Locations to undertake military
furnished or undertaken inside Philippine territory;72 activities therein without any clear limitation as to the duration of their
stay. Moreover, they are given unimpeded access to Agreed Locations to conduct
10) The protection of the environment and human health and safety, and the different activities that definitely were not contemplated under the VFA.
observance of Philippine laws on environment and health, and the prohibition _______________
76 Visiting Forces Agreement, Preamble, third paragraph. vising, assisting and training exercise,” falls under the umbrella of
77 Id., fifth paragraph. sanctioned or allowable activities in the context of the agreement. Both the
78 http://www.oxforddictionaries.com/us/definition/american_english/visit. history and intent of the Mutual Defense Treaty and the VFA
Accessed on December 14, 2015, 5:30 PM. support the conclusion that combat-related activities — as opposed
to combat itself — such as the one subject of the instant petition,
are indeed authorized. (Emphases supplied, citations omitted)
524
524 SUPREME COURT REPORTS ANNOTATED The above discussion clearly shows that the VFA was intended for noncombat
activities only.
Saguisag vs. Ochoa, Jr. In the instant case, the OSG averred that the entry of the United States forces
into the Agreed Location is borne out of “military necessity.”80 Military necessity
The Court’s ruling in Lim v. Executive Secretary79 provides some insights as to means the necessity attending belligerent military operations that is held to justify
the scope of activities germane to the intention of the VFA. Thus: all measures necessary to bring an enemy to complete submission excluding those
(as cruelty, torture, poison, perfidy, wanton destruction) that are forbidden by
The first question that should be addressed is whether “Balikatan 02- modern laws and customs of war.81
1” is covered by the Visiting Forces Agreement. To resolve this, it is In the instant case, some of the activities that the United States forces will
necessary to refer to the VFA itself. Not much help can be had therefrom, undertake within the Agreed Locations such as prepositioning of defense
unfortunately, since the terminology employed is itself the source of the equipment, supplies and materiel, and deploying of forces and materiel are actual
problem. The VFA permits United States personnel to engage, on an military measures supposedly put into place in anticipation of battle. To
impermanent basis, in “activities,” the exact meaning of which was left preposition means “to place military units, equipment, or supplies at or near the
undefined. The expression is ambiguous, permitting a wide scope of point of planned use or at a designated location to reduce reaction time, and to
undertakings subject only to the approval of the Philippine government. ensure timely support of a specific force during initial phases of an operation.”82 On
The sole encumbrance placed on its definition is couched in the negative, in the other hand, materiel is defined as “all items necessary to equip, operate,
that United States personnel must “abstain from any activity inconsistent maintain, and support military activities without distinction as to its application
with the spirit of this agreement, and in particular, from any political for administrative or
activity.” All other activities, in other words, are fair game. _______________
xxxx
After studied reflection, it appeared farfetched that the ambiguity 80 Rollo (G.R. No. 212444), p. 481.
surrounding the meaning of the word “activities” arose from accident. In 81 Webster’s Third New International Dictionary [1993].
our view, it was deliberately made that way to give both parties a certain 82 http://www.dtic.mil/doctrine/new_pubs/jp4_0.pdf. Accessed on December
leeway in negotiation. In this manner, visiting US forces may sojourn in 11, 2015, 11:48 AM.
Philippine territory for purposes other than military. As conceived, the joint
exercises may include training on new techniques of patrol and surveillance
to protect the nation’s marine resources, sea search-and-rescue operations 526
to assist vessels in distress, disaster relief operations, civic action projects
such as the building of school houses, medical and humanitarian missions, 526 SUPREME COURT REPORTS ANNOTATED
and the like. Saguisag vs. Ochoa, Jr.
Under these auspices, the VFA gives legitimacy to the
current Balikatan exercises. It is only logical to assume that “Balikatan 02- combat purposes.”83 Also, to deploy means “to place or arrange (armed forces)
1,” a “mutual anti-terrorism ad- in battle disposition or formation or in locations appropriate for their future
_______________ employment.”84 Deployment also means “the rotation of forces into and out of an
operational area.”85
79 Lim v. Executive Secretary, supra note 47 at pp. 572-575; pp. 752-755. The EDCA likewise allows the construction of permanent buildings, which the
United States forces can utilize until such time that they no longer need the use
thereof. The construction of permanent buildings, including the alteration or
525 improvement by the United States of existing buildings, structures and assemblies
VOL. 779, JANUARY 12, 2016 525 affixed to the land, are certainly necessary not only for the accommodation of its
troops, bunkering of vessels, maintenance of its vehicles, but also the creation of
Saguisag vs. Ochoa, Jr. the proper facilities for the storage and prepositioning of its defense materiel. This
grant of authority to construct new buildings and the improvement of existing the Agreed Locations by the United States forces and contractors rather than an
buildings inside the Agreed Locations — which buildings are to be used implementing instrument of both the MDT and the VFA.
indefinitely — further evinces the permanent nature of the stay of United States _______________
forces and contractors in this country under the EDCA. This is a far cry from the
temporary visits of United States armed forces contemplated in the VFA. 88 Id., Article II(4).
Moreover, aside from agreements that the Philippines and the United States 89 Id., Article III(6).
may subsequently enter into with respect to the access of the United States forces 90 Id., Article X(3).
in the Agreed Locations on a “rotational basis,”86 and other activities that the
United States may conduct therein,87 the EDCA also contains provisions requiring
the execution of further “implementing arrangements” with regard to description 528
of the Agreed Loca-
528 SUPREME COURT REPORTS ANNOTATED
_______________
Saguisag vs. Ochoa, Jr.
83 http://www.dtic.mil/doctrine/new_pubs/jp4_0.pdf. Accessed on December
11, 2015, 11:48 AM. As stated above, Section 25, Article XVIII contemplates three different
84 Webster’s Third New International Dictionary [1993]. situations: a treaty concerning the allowance within the Philippines of (a) foreign
85 http://www.dtic.mil/doctrine/new_pubs/jp1_02.pdf. Accessed on December military bases, (b) foreign military troops, or (c) foreign military facilities, such
11, 2015, 12:36 PM. that a treaty that involves any of these three standing alone would fall within the
86 Enhanced Defense Cooperation Agreement, Article I(1)(b). coverage of the said provision. The VFA clearly contemplates only visits of foreign
87 Id., Article III(1). military troops.
The VFA, which allows the presence of the units of the United States military
troops, cannot by any stretch of the imagination include any arrangement that
527 practically allows the establishment of United States military bases or facilities in
the so-called Agreed Locations under the EDCA. Thus, the EDCA goes far-beyond
VOL. 779, JANUARY 12, 2016 527
the arrangement contemplated by the VFA and therefore it necessarily requires
Saguisag vs. Ochoa, Jr. Senate concurrence as mandated by Section 25, Article XVIII of the Constitution.
In the same vein, the initial entry of United States troops under the VFA cannot,
tions,88 “[funding] for construction, development, operation and maintenance as postulated by the ponencia, justify a “treaty-authorized” presence under the
costs at the Agreed Locations,”89 and “additional details concerning the presence EDCA, since the presence contemplated in the EDCA also pertains to the
of the United States forces at the Agreed Locations and the functional relations establishment of foreign military bases or facilities, and not merely visiting troops.
between the United States forces and the AFP with respect to Agreed Locations.”90 The argument that the entry of the United States bases, troops and facilities
Article II(4) of the EDCA states that the Agreed Locations shall be provided by under the EDCA is already allowed in view of the “initial entry” of United States
the Philippine Government through the AFP. What is readily apparent from troops under the VFA glaringly ignores that the entry of visiting foreign military
said article is that the AFP is given a broad discretion to enter into agreements troops is distinct and separate from the presence or establishment of foreign
with the United States with respect to the Agreed Locations. The grant of such military bases or facilities in the country under Section 25, Article XVIII of the
discretion to the AFP is without any guideline, limitation, or standard as to the Constitution.
size, area, location, boundaries and even the number of Agreed Locations to be To reiterate, the EDCA is entirely a new treaty, separate and distinct from the
provided to the United States forces. As there is no sufficient standard in the VFA and the MDT. Hence, it must satisfy the requirements under Section 25,
EDCA itself, and no means to determine the limits of authority granted, the AFP Article XVIII of the Constitution. The Senate itself issued Resolution No. 105 on
can exercise unfettered power that may have grave implications on national November 10, 2015, whereby it expressed its “definite stand on the non-negotiable
security. The intervention of the Senate through the constitutionally ordained power of the Senate to decide whether a treaty will be valid and effective depending
treaty-making process in defining the new national policy concerning United on the Senate concurrence” and resolved “that the RP-US EDCA [is a] treaty
States access to Agreed Locations enunciated in the EDCA, which has never been
before expressly or impliedly authorized, is imperative and indispensible for
the validity and effectivity of the EDCA. 529
The above distinctions between the EDCA and the VFA, therefore, negate the VOL. 779, JANUARY 12, 2016 529
OSG’s argument that the EDCA merely involves “adjustments in detail” of the
VFA. To my mind, the EDCA is the general framework for the access and use of Saguisag vs. Ochoa, Jr.

[that] requires Senate concurrence in order to be valid and effective.”


Incidentally, with respect to the VFA, there is a difference of opinion whether
or not the same is an implementing agreement of the MDT, as the latter does not DISSENTING OPINION
confer authority upon the United States President (or the Philippine President) to
enter into an executive agreement to implement said treaties. Still, in Nicolas v. BRION, J.:
Romulo,91 the Court noted that even if the VFA was treated as an implementing
agreement of the MDT, the VFA was submitted to the Senate for concurrence. Before this Court is the constitutionality of the Enhanced Defense Cooperation
By no means should this opinion be construed as one questioning the Agreement (EDCA), an executive agreement with the United States of America
President’s intention and effort to protect our national territory and security. (U.S.) that the Executive Department entered into and ratified on June 6, 2014.1
However, in the case of Tawang Multi-Purpose Cooperative v. La Trinidad Water This case is not an easy one to resolve for many reasons — the stakes involved
District,92 the Court said: in light of contemporary history, the limited reach of judicial inquiry, the limits of
the Court’s own legal competence in fully acting on petitions before it, and the plain
There is no “reasonable and legitimate” ground to violate the and clear terms of our Constitution. While the petitions, the comments, and
Constitution. The Constitution should never be violated by anyone. Right the ponencia all extensively dwell on constitutional, statutory, and international
or wrong, the President, Congress, the Court, x x x have no choice but to law, the constitutional challenge cannot be resolved based solely on our con-
follow the Constitution. Any act, however noble its intentions, is void _______________
if it violates the Constitution. This rule is basic.
In Social Justice Society, the Court held that, “In the discharge of their 1 Instrument of Ratification, Annex A of the Memorandum of OSG, Rollo, p.
defined functions, the three departments of government have no choice but 476. [Per p. 318 of ponencia, to verify from Rollo]
to yield obedience to the commands of the Constitution. Whatever limits it
imposes must be observed.” In Sabio, the Court held that, “the Constitution
is the highest law of the land. It is ‘the basic and paramount law to which 531
x x x all persons, including the highest officials of the land, must defer. No VOL. 779, JANUARY 12, 2016 531
act shall be valid, however noble its intentions,
_______________ Saguisag vs. Ochoa, Jr.

91 Chief Justice Reynato S. Puno and Justice Carpio submitted stirring sideration of the Constitution nor through the prism of Philippine national
dissenting opinions which assail the constitutionality of the VFA on its being interest considerations, both expressed and those left unspoken in these cases. In
unenforceable due to the absence of ratification by the US Senate. our globalized world where Philippine interests have long been intersecting with
92 661 Phil. 390, 406; 646 SCRA 21, 39-40 (2011). those of others in the world, the country’s externalities — the international and
regional situations and conditions — must as well be considered as operating
background from where the Philippines must determine where its national
530 interests lie.
From the practical point of view of these externalities and the violation of
530 SUPREME COURT REPORTS ANNOTATED
Philippine territorial sovereignty that some of us have expressed, a quick decision
Saguisag vs. Ochoa, Jr. may immediately suggest itself — let us do away with all stops and do what
we must to protect our sovereignty and national integrity.
if it conflicts with the Constitution.’” In Bengzon v. Drilon,the Court What renders this kind of resolution difficult to undertake is the violation of
held that, “the three branches of government must discharge their our own Constitution — the express manifestation of the collective will of the
respective functions within the limits of authority conferred by the Filipino people — that may transpire if we simply embrace the proffered easy
Constitution.” In Mutuc v. Commission on Elections, the Court held that, solutions. Our history tells us that we cannot simply turn a blind eye to our
“The three departments of government in the discharge of the Constitution without compromising the very same interests that we as a nation
functions with which it is [sic] entrusted have no choice but to want to protect through a decision that looks only at the immediate practical view.
yield obedience to [the Constitution’s] commands. Whatever limits To lightly regard our Constitution now as we did in the past, is to open the way to
it imposes must be observed.” (Emphases supplied, citations omitted) future weightier transgressions that may ultimately be at the expense of the
Filipino people.
A final word. While it is true that the Philippines cannot stand alone and will It is with these thoughts that this Opinion has been written: I hope to consider
need friends within and beyond this region of the world, still we cannot offend our all the interests involved and thereby achieve a result that balances the immediate
Constitution and bargain away our sovereignty. with the long view of the concerns besetting the nation.
Accordingly, I vote to grant the consolidated petitions.
I am mindful, of course, that the required actions that would actively serve our Saguisag vs. Ochoa, Jr.
national interests depend, to a large extent, on the political departments of
government — the Executive and, to some extent, the Legislature.2 The Judiciary tional command to pursue an independent foreign policy;9 the violation of the
has only one assigned role — to ensure that the Constitution constitutional provision on the autonomy of local government units10 and of
_______________ National Building Code;11 and, last but not the least, they question the EDCA for
being a one-sided agreement in favor of the Americans.12
2 Constitution, Article VII, Section 21; Article XVIII, Section 25.
I.B. The Respondents’ Positions

532 The respondents, through the Office of the Solicitor General (OSG), respond by
532 SUPREME COURT REPORTS ANNOTATED questioning the petitioners on the threshold issues of justiciability, prematurity
and standing, and by invoking the application of the political question doctrine. 13
Saguisag vs. Ochoa, Jr.
The OSG claims as well that the EDCA is properly embodied in an executive
agreement as it is an exercise of the President’s power and duty to serve and
is followed and, in this manner, ensure that the Filipino people’s larger
protect the people, and of his commander-in-chief powers;14 that the practical
interests, as expressed in the Constitution, are protected. 3Small though this
considerations of the case requires a deferential review of executive decisions over
contribution may be, let those of us from the Judiciary do our part and be counted.
national security;15that the EDCA is merely in implementation of two previous
treaties — the Mutual Defense Treaty of 1951 (1951 MDT) and the Visiting Forces
I. The Case
Agreement of 1998 (1998 VFA);16 that the President may choose the form of the
agreement, provided that the agreement dealing with foreign military bases,
I.A. The Petitions
troops, or facilities is not the principal agreement that first allowed their entry or
presence in the Philippines.
The challenges to the EDCA come from several petitions that uniformly
_______________
question — based on Article XVIII, Section 25 of the 1987 Constitution — the use
of an executive agreement as the medium for the agreement with the
U.S. The petitioners posit that the EDCA involves foreign military bases, troops, 9 Id., at pp. 23-27; Saguisag, et al. Petition (G.R. No. 212444), pp. 36-38.
and facilities whose entry into the country should be covered by a treaty 10 Id., at pp. 87-89.
concurred in by the Senate. 11 Id., at pp. 90-91.
They question substantive EDCA provisions as well, particularly the grant of 12 Id., at pp. 44-45, 58-59; Saguisag, et al. Petition (G.R. No. 212426), pp. 39-
telecommunication and tax privileges to the U.S. armed forces and its 49.
personnel;4 the constitutional ban against the presence and storage of nuclear 13 OSG Consolidated Comment, pp. 3-8.
weapons within the Philippines;5 the violation of the constitutional mandate to 14 Id., at pp. 10-13.
protect the environment;6 the deprivation by the EDCA of the exercise by the 15 Id., at pp. 13-14.
Supreme Court of its power of judicial review;7 the violation of the constitutional 16 Id., at pp. 14-21.
policy on the preferential use of Filipino labor and materials; 8 the violation of the
constitu-
_______________ 534
534 SUPREME COURT REPORTS ANNOTATED
3 Derived from the Supreme Court’s powers under Article VIII, Section 5(2)(a) Saguisag vs. Ochoa, Jr.
of the Constitution.
4 Bayan Muna, et al. Petition (G.R. No. 212444), pp. 46-47, 79-81. I.C. The Ponencia
5 Id., at pp. 52-57; Saguisag, et al. Petition (G.R. No. 212444), pp. 32-34.
6 Bayan Muna, et al. Petition (G.R. No. 212444), pp. 84-87. The ponencia exhaustively discusses many aspects of the challenges in its
7 Id., at pp. 40-43; Saguisag, et al. Petition (G.R. No. 212444), pp. 34-36. support of the OSG positions. It holds that the President is the chief implementor
8 Id., at pp. 82-84. of the law and has the duty to defend the State, and for these purposes, he may
use these powers in the conduct of foreign relations;17 even if these powers are not
expressly granted by the law in this regard, he is justified by necessity and is
533 limited only by the law since he must take the necessary and proper steps to carry
VOL. 779, JANUARY 12, 2016 533 the law into execution.
The ponencia further asserts that the President may enter into an executive - foreign troops under arrangements outside of the contemplation of
agreement on foreign military bases, troops, or facilities, if: the visiting forces that the 1998 VFA allows; and
(a) it is not the instrument that allows the presence of foreign military bases, - military facilities that, under modern military strategy, likewise can
troops, or facilities; or be brought in only through a treaty.
(b) it merely aims to implement an existing law or treaty.18 _______________
It adds that the 1951 MDT is not an obsolete treaty; 19 that the 1998 VFA has
already allowed the entry of U.S. troops and civilian personnel and is the treaty 24 Id., at p. 375.
being implemented by the EDCA;20 that the President may generally enter into 25 Id.
executive agreements subject to the limitations defined by the Constitution, in 26 Id., at pp. 375-378.
furtherance of a treaty already concurred in by the Senate;21 that the President
can choose to agree to the EDCA either by way of an executive agreement or by
treaty.22 While it compares the EDCA with the 1951 MDT and the 1998 VFA, it 536
claims at the same time it merely implements these treaties.23
536 SUPREME COURT REPORTS ANNOTATED
_______________
Saguisag vs. Ochoa, Jr.
17 Ponencia, pp. 299-304, 337-340.
18 Id., at pp. 342-367. As the ponencia does, I shall discuss the background facts and the threshold
19 Id., at p. 349. issues that will enable the Court and the reading public to fully appreciate the
20 Id., at pp. 384-388. constitutional issues before us, as well as my reasons for the conclusion that the
21 Id., at pp. 357-371. EDCA, as an executive agreement, is constitutionally deficient.
22 Id., at pp. 371-375. I purposely confine myself to the term “constitutionally deficient”
23 Id., at pp. 375-413. (instead of saying “unconstitutional”) in light of my view that the
procedural deficiency that plagues the EDCA as an executive agreement
is remediable and can still be addressed. Also on purpose, I refrain from
535 commenting on the substantive objections on the contents of the EDCA for the
reasons explained below.
VOL. 779, JANUARY 12, 2016 535
Saguisag vs. Ochoa, Jr. II. The Threshold Issues

On the exercise of its power of judicial review, the ponencia posits that the The petitioners bring their challenges before this Court on the basis of their
Court does not look into whether an international agreement should be in the form standing as citizens, taxpayers, and former legislators. The respondents, on the
of a treaty or an executive agreement, save in the cases in which the Constitution other hand, question the justiciability of the issues raised and invoke as well
or a statute requires otherwise;24 that the task of the Court is to determine the political question doctrine to secure the prompt dismissal of the petitions. I
whether the international agreement is consistent with applicable shall deal with these preliminary issues below, singly and in relation with one
limitations;25 and that executive agreements may cover the matter of foreign another, in light of the commonality that these threshold issues carry.
military forces if these merely involve adjustments of details.26 The petitioners posit that the use of an executive agreement as the medium to
carry EDCA into effect, violates Article XVIII, Section 25 of the 1987 Constitution
I.D. The Dissent and is an issue of transcendental importance that they, as citizens, can raise before
the Supreme Court.27 (Significantly, the incumbent Senators are not direct
I dissent, as I disagree that an executive agreement is the proper participants in this case and only belatedly reflected their institutional sentiments
medium for the matters covered by the EDCA. The EDCA is an agreement through a Resolution.)28 The petitioners in G.R. No. 212444 also claim that the
that, on deeper examination, violates the letter and spirit of Article XVIII, Section constitutionality of the EDCA involves the assertion and
25 and Article VII, Section 21, both of the Constitution. _______________
The EDCA should be in the form of a treaty as it brings back to the
Philippines: 27 Saguisag, et al. Petition (G.R. No. 212426), pp. 19-22; Bayan Muna, et
- the modern equivalent of the foreign military bases whose term al. Petition (G.R. No. 212444), p. 6.
expired in 1991 and which Article XVIII, Section 25 of the Constitution 28 Senate Resolution No. 105 dated November 10, 2015.
directly addresses;
537 although on both counts I differ from the ponencia’s line of reasoning. Let me point
VOL. 779, JANUARY 12, 2016 537 out at the outset, too, that judicial review is only an exercise of the wider judicial
power that Article VIII, Section 1 of the Constitution grants and defines. One
Saguisag vs. Ochoa, Jr. should not be confused with the other.
Judicial review is part of the exercise of judicial power under Article VIII,
protection of a public right, in which they have a personal interest as affected Section 1 of the Constitution, particularly when it is exercised under the judiciary’s
members of the general public.29 expanded power (i.e., when courts pass upon the actions of other agencies of
The petitioners likewise claim that the EDCA requires the disbursement of government for the grave abuse of discretion they committed), or when the
public funds and the waiver of the payment of taxes, fees and rentals; thus, the Supreme Court reviews, on appeal or certiorari, the constitutionality or validity of
petitioners have the standing to sue as taxpayers.30 any law or other governmental instruments under Section 5(2)(a) and (b) of Article
They lastly claim that the exchange of notes between the Philippines’ VIII of the Constitution.
Department of National Defense Secretary Voltaire Gazmin and U.S. Ambassador A basic requirement is the existence of an actual case or controversy that,
Philip Goldberg31 — the final step towards the implementation of the EDCA — viewed correctly, is a limit on the exercise of judicial power or the more specific
rendered the presented issues ripe for adjudication. power of judicial review.36
The respondents, in response, assert that the petitioners lack standing,32 and Whether such case or controversy exists depends on the existence of a legal
that the petitions raise political questions that are outside the Court’s jurisdiction right and the violation of this right, giving rise to a dispute between or among
to resolve.33 adverse parties.37 Under the expanded power of judicial review, the actual case or
They also argue that the issues the petitions raise are premature.34The EDCA controversy arises when an official or agency of government is
requires the creation of separate agreements to carry out separate activities such _______________
as joint exercises, the prepositioning of materiel, or construction activities. At
present, these separate agreements do not exist. Thus, the respondents state that 35 Ponencia, pp. 327-337.
the petitioners are only speculating that the agreements to be forged under the 36 Imbong v. Ochoa, Jr., G.R. No. 204819, April 8, 2014, 721 SCRA 146, 278-
EDCA would violate our laws. These speculations cannot be the basis for a 279.
constitutional challenge. 37 Id., at pp. 279-280.

II.A. Locus Standi


539
The ponencia holds that the petitioners do not have the requisite standing to
question the constitutionality of the EDCA, but chooses to give due course to the VOL. 779, JANUARY 12, 2016 539
petitions because Saguisag vs. Ochoa, Jr.
_______________
alleged to have committed grave abuse of discretion in the exercise of its
29 Bayan Muna, et al. Petition (G.R. No. 212444), pp. 9-10. functions.38
30 Saguisag, et al. Petition (G.R. No. 212426), pp. 19-22. Locus standi is a requirement for the exercise of judicial review39and is in fact
31 Id., at p. 19. an aspect of the actual case or controversy requirement viewed from the prism of
32 OSG Consolidated Comment, pp. 3-5. the complaining party whose right has been violated.40
33 Id., at pp. 5-7. When a violation of a private right is asserted, the locus standi requirement is
34 Id., at pp. 7-8. sharp and narrow because the claim of violation accrues only to the complainant
or the petitioner whose right is alleged to have been violated.41
On the other hand, when a violation of a public right is asserted — i.e., a right
538 that belongs to the public in general and whose violation ultimately affects every
538 SUPREME COURT REPORTS ANNOTATED member of the public — the locus standi requirement cannot be sharp or narrow;
it must correspond in width to the right violated. Thus, the standing of even a plain
Saguisag vs. Ochoa, Jr. citizen sufficiently able to bring and support a suit, should be recognized as he or
she can then be deemed to be acting in representation of the general public. 42
of the transcendental importance of the issues these petitions raise.35 In effect, _______________
the ponencia takes a liberal approach in appreciating the threshold issue of locus
standi. 38 See Separate Opinion of J. Brion in Imbong v. Ochoa, Jr., id., at pp. 489-
I agree with the ponencia’s ultimate conclusions on the threshold issues raised. 491.
I agree as well that a justiciable issue exists that the Court can pass upon,
39 Galicto v. Aquino III, 683 Phil. 141, 170; 667 SCRA 150, 187 (2012). 46 See Chamber of Real Estate and Builders’ Association, Inc. (CREBA) v.
40 Id. Energy Regulatory Commission (ERC), 638 Phil. 542, 556-557; 624 SCRA 556, 570
41 See David v. Macapagal-Arroyo, 552 Phil. 705; 489 SCRA 160 (2006), where (2010), where the Court provided “instructive guides” as determinants in
the Court held that in private suits, standing is governed by the “real parties-in- determining whether a matter is of transcendental importance, namely: (1) the
interest” rule as contained in Section 2, Rule 3 of the 1997 Rules of Civil Procedure, character of the funds or other assets involved in the case; (2) the presence of a
as amended. It provides that “every action must be prosecuted or defended in the clear case of disregard of a constitutional or statutory prohibition by the
name of the real party-in-interest.” Accordingly, the “real party-in-interest” is “the
party who stands to be benefited or injured by the judgment in the suit or the party
entitled to the avails of the suit.” Succinctly put, the plaintiff’s standing is based 541
on his own right to the relief sought. VOL. 779, JANUARY 12, 2016 541
42 De Castro v. Judicial and Bar Council, 629 Phil. 629, 680; 615 SCRA 666,
725 (2010). Saguisag vs. Ochoa, Jr.

In the present cases, a violation of the Constitution, no less, is alleged by the


540 petitioners through the commission of grave abuse of discretion. The violation
potentially affects our national sovereignty, security, and defense, and the
540 SUPREME COURT REPORTS ANNOTATED
integrity of the Constitution — concerns that touch on the lives of the citizens as
Saguisag vs. Ochoa, Jr. well as on the integrity and survival of the nation. In particular, they involve the
nation’s capability for self-defense; the potential hazards the nation may face
Transcendental importance is a concept (a much abused one) that has been because of our officials’ decisions on defense and national security matters; and
applied in considering the requirements for the exercise of judicial power.43 To be our sovereignty as a nation as well as the integrity of the Constitution that all
sure, it may find application when a public right is involved because a right that citizens, including the highest officials, must protect.
belongs to the general public cannot but be important.44 Whether the importance In these lights, I believe that the issues involved in the present case are so
rises to the level of being transcendental is a subjective element that depends on important that a plain citizen sufficiently knowledgeable of the outstanding issues,
the user’s appreciation of the descriptive word “transcendental” or on his or her should be allowed to sue. The petitioners — some of whom are recognized legal
calibration of the disputed issues’ level of importance. luminaries or are noted for their activism on constitutional matters — should thus
In either case, the use of transcendental importance as a justification is replete be recognized as parties with proper standing to file and pursue their petitions
with risks of abuse as subjective evaluation is involved. 45 To be sure, this level of before this Court.
importance can be used as justification in considering locus standi with
liberality,46 but it can never be an excuse to find an actual controversy when there II.B. Ripeness of the Issues Raised for Adjudication
is none. To hold otherwise is to give the courts an unlimited opportunity for the
exercise of judicial power — a situation that is outside the Constitution’s intent in I agree with the ponencia’s conclusion that the cases before this Court, to the
the grant of judicial power. extent they are anchored on the need for Senate concurrence, are ripe for
_______________ adjudication. My own reasons for this conclusion are outlined below.
Like locus standi, ripeness for adjudication is an aspect of the actual case or
43 See Integrated Bar of the Philippines v. Zamora, 392 Phil. 618, 634; 338 controversy requirement in the exercise of
SCRA 81, 101 (2000), citing Tatad v. Secretary of the Department of Energy, G.R. _______________
No. 124360, November 5, 1997, 281 SCRA 330, 349, citing Garcia v. Executive
Secretary, G.R. No. 101273, July 3, 1992, 211 SCRA 219; Osmeña v. COMELEC, public respondent agency or instrumentality of the government; and (3) the
G.R. No. 100318, July 30, 1991, 199 SCRA 750; Basco v. PAGCOR, G.R. No. 91649, lack of any other party with a more direct and specific interest in the questions
May 14, 1991, 197 SCRA 52; and Araneta v. Dinglasan, 84 Phil. 368 (1949). being raised.
44 Initiatives for Dialogue and Empowerment through Alternative Legal
Services, Inc. (IDEALS, INC.) v. Power Sector Assets and Liabilities Management
Corporation (PSALM), G.R. No. 192088, October 9, 2012, 682 SCRA 602, 633-634. 542
45 See Separate Opinion of J. Brion in Cawad v. Abad, G.R. No. 207145, July 542 SUPREME COURT REPORTS ANNOTATED
28, 2015, 764 SCRA 1, citing Quinto v. COMELEC, G.R. No. 189698, December 1,
2009, 606 SCRA 258, 276 and GMA Network, Inc. v. COMELEC, G.R. No. 205357, Saguisag vs. Ochoa, Jr.
September 2, 2014, 734 SCRA 88, 125-126.
judicial power.47 The two concepts differ because ripeness is considered from Constitution, rendered the agreement’s constitutional status questionable. Thus,
the prism, not of the party whose right has been violated, but from the prism of the when the exchange of notes that signaled the implementation of the EDCA took
actual violation itself. place, the issue of its compliance with the constitutional requirements became ripe
Of the two basic components of actual case or controversy, namely, for judicial intervention under our expanded jurisdiction.
the existence of a right and the violation of that right, ripeness essentially
addresses the latter component.48 That a right exists is not sufficient to support II.C. The Political Question Doctrine
the existence of an actual case or controversy; the right must be alleged to have
been violated to give rise to a justiciable dispute. In other words, it is the fact of Another threshold issue that this Court must settle at the outset, relates to the
violation that renders a case ripe,49 assuming of course the undisputed existence political question doctrine that, as a rule, bars any judicial inquiry on any matter
of the right violated. that the Constitution and the laws have left to the discretion of a coordinate branch
In the present cases, Article VIII, Section 25 of the Constitution lays down in of government for action or determination.50
no uncertain terms the conditions under which foreign military bases, troops, and The respondents raise the political question issue as part of their defense,
facilities may be allowed into the country: there should at least be the concurrence arguing that the issues the petitioners raise are
of the Senate. _______________
Under these terms, the refusal to allow entry of foreign military bases, troops,
and facilities into the country without the required Senate concurrence is a 50 Bondoc v. Pineda, 278 Phil. 784; 201 SCRA 792 (1991).
prerogative that the people of this country adopted for themselves under their
Constitution: they want participation in this decision, however indirect this
participation might be. This prerogative is exercised through the Senate; thus, a 544
violation of this constitutional prerogative is not only a transgression against the
544 SUPREME COURT REPORTS ANNOTATED
Senate but one against the people who the Senate represents.
The violation in this case occurred when the President ratified the EDCA as Saguisag vs. Ochoa, Jr.
an executive agreement and certified to the other contracting party (the U.S.) that
all the internal processes have been complied with, leading the latter to believe policy matters that lie outside the Court’s competence or are matters where
that the agreement is already valid and enforceable. Upon the Court should defer to the Executive.51
47 Supra note 36 at p. 280. The political question bar essentially rests on the separation of powers doctrine
48 Id. that underlies the Constitution.52 The courts cannot interfere with questions that
49 Province of North Cotabato v. Government of the Republic of the Philippines involve policy determination exclusively assigned to the political departments of
Peace Panel on Ancestral Domain (GRP), 589 Phil. 387, 481; 568 SCRA 402, 451 the government.53The American case of Baker v. Carr54 best describes the
(2008). standards that must be observed in determining whether an issue involves a
political question, as follows:

543 Prominent on the surface of any case held to involve a political question
VOL. 779, JANUARY 12, 2016 543 is found a textually demonstrable constitutional commitment of the issue
to a coordinate political department; or a lack of judicially discoverable and
Saguisag vs. Ochoa, Jr. manageable standards for resolving it; or the impossibility of deciding
without an initial policy determination of a kind clearly for nonjudicial
such violation, the dispute between the President and the Filipino people discretion; or the impossibility of a court’s undertaking independent
ripened. resolution without expressing lack of the respect due coordinate branches
The same conclusion obtains even under the respondents’ argument that the of government; or an unusual need for unquestioning adherence to a
constitutionality of the EDCA is not yet ripe for adjudication, since it requires the political decision already made; or the potentiality of embarrassment from
creation of separate agreements to carry out separate activities such as joint multifarious pronouncements by various departments on one question.55
exercises, the prepositioning of materiel, or construction activities. To the
respondents, the petitioners are merely speculating on their claim of From among these tests, the presence or absence of constitutional standards is
unconstitutionality since these separate agreements do not yet exist. the most relevant under the circumstances of the present consolidated cases.
Indeed, issues relating to agreements yet to be made are not, and cannot be, _______________
ripe for adjudication for the obvious reason that they do not yet exist. The question
of the EDCA’s constitutionality, however, does not depend solely on the separate 51 Javellana v. Executive Secretary, 151-A Phil. 36, 131; 50 SCRA 30, 140
agreements that will implement it. The fact that an executive agreement had been (1973), citing In Re McConaughy, 119 N.W. 408, 417.
entered into, not a treaty as required by Article XVIII, Section 25 of the
52 See Garcia v. Executive Secretary, 602 Phil. 64, 73-77; 583 SCRA 119, 133- Thus, the main issue the petitioners pose — the constitutional status of the
134 (2009). EDCA as an executive agreement in light of the mandate of Article XVIII, Section
53 Id. 25 of the Constitution — is not a political question outside the judiciary’s
54 369 U.S. 186 (1962). competence and authority to resolve. The respondents’ argument on this point is
55 Id., at p. 217. therefore erroneous.
If indeed a referral to the Senate is required and no referral has been made,
then the EDCA is constitutionally deficient so that its terms cannot be
545 enforced. This finding renders further proceedings on the merits of the substantive
VOL. 779, JANUARY 12, 2016 545 issues raised, pointless and unwarranted. There is likewise no point in
determining whether the substantive issues raised call for the application of the
Saguisag vs. Ochoa, Jr. political question doctrine.57
On the other hand, the examination of the EDCA’s substantive contents may
After analyzing the issues raised, I find the respondents’ position partly be ripe and proper for resolution if indeed the EDCA can properly be the subject of
erroneous and partly premature for a political question doctrine ruling. an executive agreement. It is at that point when the respondents may claim that
This conclusion proceeds from my recognition that a distinction should be the substantive contents of the EDCA involve policy matters that are solely for the
drawn in recognizing the constitutional issues before us, some of which are President to determine and that the courts may not inquire into under the
procedural in character while others are substantive ones that require the separation of powers principle.58 It is only at that point when the application of the
application of different constitutional provisions. political question doctrine is called for.
The petitioners primarily question the constitutional validity of the EDCA for In these lights (particularly, my position on the merits of the procedural issue
violation of Article XVIII, Section 25 of the 1987 Constitution. They challenge, as raised), I find a ruling on the application of the political question doctrine to the
well, substantive provisions of the EDCA, among them, those relating to the grant substantive issues raised premature and unripe for adjudication; any ruling or
of telecommunication privileges and tax exemptions to American visiting forces, discussion I may make may only confuse the issues when a proper petition on the
and the EDCA provisions that would allegedly allow the entry of nuclear weapons constitutionality of the substantive contents of EDCA is filed.
into the country.
That the EDCA is an agreement that requires concurrence by the Senate before III. The Facts
it can be considered valid and enforceable, is an issue that is essentially procedural
as it requires that steps be taken before an international agreement can be III.A. Historical, International and Regional Contexts
considered fully valid and enforceable. It is an issue extrinsic to the terms of _______________
the EDCA and is properly a threshold issue that must be resolved before the
substantive challenges to the EDCA’s validity can be addressed. 57 See Constitution, Article VII, Section 21.
Aside from being procedural, the issue relates as well to the standard set by 58 Supra note 50.
the Constitution that delineates when an international agreement should be a
treaty subject to Senate concurrence. The presence of this standard renders the
determination of the medium to be used in forging an international agreement — 547
whether as a treaty or as an executive agreement an issue within the competence
and authority of the courts to resolve in their role as guardians of the VOL. 779, JANUARY 12, 2016 547
Constitution.56 Saguisag vs. Ochoa, Jr.
_______________
III.A(1) The Early Years of
56 Dueñas, Jr. v. House of Representatives Electoral Tribunal, 610 Phil. 730, Philippines-U.S. Relationship
742; 593 SCRA 316, 345 (2009); Lambino v. Commission on Elections, 536 Phil. 1,
111; 505 SCRA 160, 264 (2006). Active Philippine-American relations started in 1898, more than a century ago,
when Commodore George Dewey and his armada of warships defeated the Spanish
navy in the Philippines in the Battle of Manila Bay.59 The sea battle was
546 complemented by land assaults by Philippine forces who were then in open
546 SUPREME COURT REPORTS ANNOTATED rebellion against Spain under the leadership of General Emilio Aguinaldo. 60
The complementary effort started a relationship that, from the Philippine end,
Saguisag vs. Ochoa, Jr. was characterized by hope of collaboration and assistance in the then colony’s
quest for independence from Spain.61But the fulfillment of this hope did not come _______________
to pass and was in fact shattered when America, with its own
_______________ 62 Treaty of Peace Between the United States and Spain (December 10, 1898),
Article III:
59 On order of then U.S. Secretary of the Navy, Theodore Roosevelt, “Spain cedes to the United States the archipelago known as the Philippine
Commodore Dewey attacked the Spanish fleet in the Philippines. At noon of May Islands x x x.” See Yale Law School, The Avalon Project, Treaty of Peace between
1, 1898, Commodore Dewey’s ships had destroyed the Spanish fleet at the Battle the United States and Spain, available
of Manila Bay. See Bayan Muna, et al. Petition (G.R. No. 212444), p. at http://avalon.law.yale.edu/19th_century/sp1898.asp.
11, citinghttp://www.history.com/this-day-in-history/battle-of-manila-bay. 63 Constantino, Renato, The Philippines: A Past Revisited,pp. 228-229 (1975).
See Brzesinski, Zbigniew, The Grand Chessboard – American Primacy and its 64 See Brzesinski, supra note 59 at pp. 3-29.
Geostrategic Imperatives (1997). According to Brzesinski, America stands supreme in the four decisive domains
See also Weir, Fraser, A Centennial History of Philippine Independence, 1898- of global power: (1) militarily, it has an unmatched global reach;
1998: Spanish-American War – War of Philippine Independence 1898- (2) economically, it remains the main locomotive of global growth;
1901. University of Alberta, available (3) technologically, it retains the overall lead in the
at https://www.ualberta.ca/~vmitchel/fw4.html; The Spanish-American War,
1898, United States Department of State, available
at https://history.state.gov/milestones/1866-1898/spanish-american-war; and The 549
Spanish-American War in the Philippines (1989), American Experience, available VOL. 779, JANUARY 12, 2016 549
at http://www.pbs.org/wgbh/amex/macarthur/peopleevents/pandeAMEX87.html.
60 In the early part of 1898, the relations between the U.S. and Spain Saguisag vs. Ochoa, Jr.
deteriorated. As the war became imminent, Commodore George Dewey, the III.A(2) The Post-W.W.II Years
commander of the U.S. Asiatic Squadron, had discussion with Emilio Aguinaldo’s
government in exile in Singapore and Hong Kong. See Weir, id. It was soon after Philippine independence, as the U.S. superpower status was
61 Id. rising, that the U.S. and the Philippines forged the Military Bases Agreement
of 1947 (1947 MBA) and the 1951 MDT. The 1947 MBA was the agreement
specific to the U.S. bases, troops, and facilities in the Philippines,65 while the 1951
548 MDT was the overarching document,
_______________
548 SUPREME COURT REPORTS ANNOTATED
Saguisag vs. Ochoa, Jr. cutting-edge areas of innovation; and (4) culturally, despite some crassness,
it enjoys an appeal that is unrivaled. The combination of all four makes
geopolitical interests in mind, decided to fight the Philippine forces and to America the only comprehensive superpower.
keep the Philippines for itself as a colony. The American objective was fully Brzesinski traced the trajectory of the US’s rise to global supremacy beginning
realized under the Treaty of Paris between Spain and the U.S., when the from World War I (WWI) to the end of the Cold War, noting that the U.S.’s
Philippines was handed by Spain to the U.S. as a colony.62 participation in WWI introduced it as a new major player in the international
The result, of course, was inevitable as the Philippine forces were not then arena. While WWI was predominantly a European war, not a global one, its self-
fighting for a change of masters but for independence. The Philippine forces fought destructive power marked the beginning of the end of Europe’s political, economic
the Americans in the Philippine-American war, and lost.63 and cultural preponderance over the rest of the world. The European era in world
Thus, a new colonizer took Spain’s place. Unlike the Spanish colonial rule, politics ended in the course of World War II (WWII), the first truly global war.
however, one redeeming feature of the American colonial rule was the introduction Since the European (i.e., Germany) and the Asian (i.e., Japan) were defeated, the
of the concepts of democracy and governance. US and the Soviet Union, two extra-European victors, became the successors to
As a colony, the Philippines, played a distinct role as the American outpost in Europe’s unfulfilled quest for global supremacy.
the Far East as the American geopolitical interests slowly grew from the First The contest between the Soviet Union and the US for global supremacy
World War years. By the end of the Second World War, the U.S.’ international dominated the next fifty years following WWII. The outcome of this contest, the
primacy was confirmed as the leader of the victor-nations. This international author believes, was eventually decided by nonmilitary means: political vitality,
leadership role became sole leadership when the Soviet Union collapsed in the late ideological flexibility, economic dynamism, and cultural appeal. The protracted
1980s. Thus, the U.S. now stands as the only global superpower whose military, competition, in the end, eventually tip the scales in America’s favor simply because
economic, cultural, and technological reach and influence extend over all it was much richer, technologically much more advanced, militarily more resilient
continents.64 and innovative, socially more creative and appealing.
65 See Bayan Muna, et al. Petition (G.R. No. 212444), pp. 13-14; and Kilusang 14; Saguisag v. Executive Secretary Ochoa petition (G.R. No. 212426), p. 8;
Mayo Uno, et al. Petition-in-Intervention, p. 7. and Kilusang Mayo Uno, et al. Petition-in-
See also Shalom, Stephen, Securing the U.S.-Philippine Military Bases
Agreement of 1947, William Paterson University, available
at http://www.wpunj.edu/dotAsset/209673.pdf; Paterno, Robert, American 551
Military Bases in the Philippines: The Brownell Opinion, available VOL. 779, JANUARY 12, 2016 551
at http://philippinestudies.net/ojs/index.php/ps/article/view
Saguisag vs. Ochoa, Jr.

550 The solidity of the R.P.-U.S. relationship that started in the colonizer-colony
mode, shifted to defense/military alliance (through the MBA, MDT, and their
550 SUPREME COURT REPORTS ANNOTATED
supplementary agreements) after Philippine independence, and began to
Saguisag vs. Ochoa, Jr. progressively loosen as the Philippines tracked its own independent path as a
nation. Through various agreements,67 the American
entered into and ratified by the two countries as a treaty, to define the _______________
Philippine-American defense relationship in case of an armed attack by a third
country on either of them.66 As its title directly suggests, it is a defense agreement. Intervention, p. 7. It was concurred in by the Philippine Senate on May 12,
_______________ 1952; and was advised and consented to by the U.S. Senate on March 20, 1952, as
reflected in the U.S. Congressional Record, 82nd Congress, Second Session, Vol.
File/2602/5224; Gregor, James, The Key Role of U.S. Bases in the 98, Part 2, pp. 2594-2595. See Nicolas v. Romulo, 598 Phil. 262; 578 SCRA 438
Philippines, The Heritage Foundation, available (2009).
at http://www.heritage.org/research/reports/1984/01/the-key-role-of-us-bases-in- 67 1956: The Garcia-Bendetsen conference resolved the issue of jurisdiction in
the-philippines; Lim, Maria Teresa, “Removal Provisions of the Philippine-United the American bases. The US began to recognize sovereignty of the Philippine
States Military Bases Agreement: Can the United States Take it All” 20 Loyola of government over the base lands. See Exchange of Notes, U.S.-Philippines,
Los Angeles Law Review 421, 421-422. See Greene, Fred, The Philippine Bases: December 6, 1956, available
Negotiating For the Future, p. 4 (1988). at http://elibrary.judiciary.gov.ph/thebookshelf/docmonth/Dec/1956/35.
The 1947 Military Bases Agreement was signed by the Philippines and the 1959: Olongapo, which was then an American territory, was officially turned
U.S. on March 14, 1947; it entered into force on March 26, 1947 and was ratified over by the US to the Philippines. Over the years, 17 of the 23 military installations
by the Philippine President on January 21, 1948. See Bevans, Charles, Treaties were also turned over to the Philippines. See Memorandum of Agreement, U.S.-
and Other International Agreements of the United States of America (1776- Philippines, October 12, 1959, available
1949), available at United States Department of at http://elibrary.judiciary.gov.ph/thebookshelf/showdocs/35/11192.
State, https://books.google.com.ph/books?id=MUU6AQAA1AAJ&pg=PA55&1pg= 1965: An agreement was signed revising Article XIII of the treaty wherein the
PA55&dq=17+UST+1212;+T1AS+6084&source=bl&ots=VBt1V34ntR&sig=X2yY US will renounce exclusive jurisdiction over the on-base offenses and the creation
CbWVfJqF_o69-CcyiP88zw0&hl=en&sa=X&ved=0ahUKEwiKg- of a joint criminal jurisdiction committee. See Exchange of Notes, U.S.-Philippines,
jXq8LJAhXRBY4K August 10, 1965, available
HSicDeAQ6AE1GzAA#v=onepage&q=17%20UST%201212%3B%20TIAS%206084 at http://elibrary.judiciary.gov.ph/thebookshelf/showdocs/35/10934.
&f=false. 1966: The Ramos-Rusk Agreement reduced the term of the MBA to 25 years
The Philippine government also agreed to enter into negotiations with the starting from that year. See Exchange of Notes, U.S.-Philippines, September 16,
U.S., on the latter’s request, to: expand or reduce such bases, exchange those bases 1966, available at http://elibrary.jud
for others, or acquire additional base areas. The agreement allowed the U.S. full iciary.gov.ph/thebookshelf/showdocs/35/10859.
discretionary use of the bases’ facilities; gave criminal jurisdiction over U.S. base 1979: The US reaffirmed Philippine sovereignty over the basis and placed each
personnel and their dependents to the U.S. authorities irrespective of whether the base under command of a Philippine base commander. See Office of the President
alleged offenses were committed on or off the base areas. See Gregor, id. of the Philippines, Official Week in Review, Official Gazette of the Republic of the
66 The Philippines and the U.S. signed the MDT on August 30, 1951. It came Philippines, 75(1), iii-iv (1979), available at http://www.gov.ph/1979/01/08/official-
into force on August 27, 1952 by the exchange of instruments of ratification week-in-review-january-1-january-7-1979/.
between the parties. See Mutual Defense Treaty, U.S.-Philippines, August 30,
1951, 177 U.N.T.S. 134, available
at https://treaties.un.org/doc/Publication/UNTS/Volume%20177 /volume-177-1- 552
2315-English.pdf. See also Bayan v. Gazmin petition (G.R. No. 212444), p. 552 SUPREME COURT REPORTS ANNOTATED
Saguisag vs. Ochoa, Jr. activity (including helping Asian countries resolve disputes that they cannot resolve
themselves); and promoting an agenda of political reform where it is
hold and the length of stay of American military bases in the Philippines appropriate.”73This meant, among others, the strengthening of American military
progressively shrunk. alliance with Asian countries, including the Philippines.
The death knell for the U.S. military bases started sounding when a new The “pivot” has a direct relevance to Philippine concerns since it was prompted,
Philippine Constitution was ratified in 1987. The new Constitution provides among others, by “China’s growing military capabilities and its increasing
that after the expiration of the agreement on military bases, no foreign assertiveness of claims to disputed maritime territory, with implications for
military bases, troops or facilities shall be allowed except through a treaty freedom of navigation and the United States’ ability to project power in the
concurred in by the Senate or with the direct consent of the Filipino region.”74 The opening of new areas for military coop-
people if Congress would require this mode of approval.68 _______________
The actual end of the military bases came in 1991 when the 1947 MBA expired
with no replacement formal arrangement in place except the 1951 MDT.69 For 71 Hemmings, John, Understanding the U.S. Pivot: Past, Present, and Future,
some years, R.P.-U.S. relationship on defense/military matters practically froze. 34(6) Royal United Services Institute Newsbrief (November 26, 2014), available
The thaw only came when the 1998 VFA was negotiated and agreed upon as a at https://hemmingsjohn.wordpress.com/2014/11/27/understanding-the-us-pivot-
treaty that the Philippine Senate concurred in. past-present-and-future/.
72 Id.
III.A(3) The U.S.’s “Pivot to Asia” Strategy 73 Bush, Richard, No rebalance necessary: The essential continuity of U.S.
policy in the Asia-Pacific, Brookings Institution (March 18, 2015), available
During the latter part of the first term of the Obama Administration, the U.S. at http://www.brookings.edu/blogs/order-from-chaos/posts/2015/18-value-of-
announced a shift in its global strategy in favor of a military and diplomatic “pivot” continuity-us-policy-in-asia-pacific.
or “rebalance” toward Asia.70 The strategy involved a shift of the U.S.’s diplomatic, 74 US Congressional Research Service, Pivot to the Pacific? The Obama
economic, and defense resources to Asia, made ur- Administration’s “Rebalancing” Toward Asia, p. 2, March 28, 2012, available
_______________ at http://www.fas.org/sgp/crs/natsec/R42448.pdf.

68 Constitution, Article XVIII, Section 25.


69 On September 16, 1991, the Philippine Senate voted to reject a new treaty 554
that would have extended the presence of U.S. military bases in the Philippines. 554 SUPREME COURT REPORTS ANNOTATED
See Bayan (Bagong Alyansang Makabayan) v. Zamora, 396 Phil. 623, 632; 342 Saguisag vs. Ochoa, Jr.
SCRA 449, 464 (2000), citing the Joint Report of the Senate Committee on Foreign
Relation and the Committee on National Defense and Security on the Visiting eration with the Philippines is among the announced features of the “pivot.”75
Forces Agreement.
70 United States Department of Defense, Sustaining U.S. Global Leadership: III.A(4) The EDCA
Priorities for 21st Century Defense, p. 2 (January 2012), available
at http://archive.defense.gov/news/Defense_Strategic_Guidance.pdf. It was soon after the launch of the “pivot” strategy that the initiatives
for the EDCA came. The EDCA, of course, did not introduce troops into the
country for the first time, as the 1998 VFA already ushered in the presence of U.S.
553 military troops on a rotational but temporary basis.
VOL. 779, JANUARY 12, 2016 553 What the EDCA brought with it was the concept of “agreed locations”
Saguisag vs. Ochoa, Jr. to which the U.S. has “unimpeded access” for the refueling of aircraft;
bunkering of ships; pre-positioning and storage of equipment, supplies
gent by “the rise of Chinese regional power and influence, and China’s and materials; the introduction of military contractors into the agreed
apparent inclination to exercise its burgeoning military power in territorial locations; and the stationing and deployment point for troops.76
disputes with its neighbors.”71 These disputes affected sea lanes that are vital to In these lights, the confirmed and valid adoption of the EDCA would make the
the U.S. and its allies; hence, the U.S. was particularly concerned with their Philippines an active ally participating either as a forward operating
peaceful resolution.72Critical to the strategy is the projection of American power site (FOS) or Cooperative Security Location (CSL) in the American “pivot”
and influence worldwide. strategy or, in blunter terms, in the projection and protection of American
The key to the new strategy in the military-political area is “presence: forward worldwide power. FOS and CSL shall be explained under the proper topic below.
deployment of U.S. military forces; a significant tempo of regional diplomatic
All these facts are recited to place our reading of the EDCA in proper context
— historically, geopolitically, and with a proper appreciation of the interests
involved, both for the Philippines and the U.S. 556
_______________ 556 SUPREME COURT REPORTS ANNOTATED

75 United States Department of Defense, The Asia-Pacific Maritime Security Saguisag vs. Ochoa, Jr.
Strategy: Achieving U.S. National Security Objectives in a Changing Environment,
p. 23 (2015), available Constitution — the only instrument that binds the whole nation.
at http://www.defense.gov/Portals/1/Documents/pubs/NDAA%20A-
P_Maritime_SecuritY_Strategy-08142015-1300-FlNALFORMAT.PDF. IV. The President’s Role
76 EDCA, Article III. in Governance and its Limits

This discussion is made necessary by the ponencia’s patent misconceptions


555 regarding the role the President plays in governance as chief executive and
implementor of policies and the laws.
VOL. 779, JANUARY 12, 2016 555
Saguisag vs. Ochoa, Jr. IV.A. The Ponencia and My Objections

The U.S. is in Asia because of the geopolitical interests and the world In upholding the constitutionality of the EDCA, the ponencia holds that the
dominance that it seeks to maintain and preserve. 77 Asia is one region that has President’s power and duty to ensure the faithful execution of our laws include the
been in a flux because of the sense of nationalism that had lain dormant among its defense of our country as the commander-in-chief of the country’s armed forces.80 It
peoples, the economic progress that many of its countries are experiencing as the contends that these powers, combined with the President’s capacity as the
economic winds shift to the East, and the emergence of China that — at the very country’s sole organ in foreign affairs, empower the President to enter into
least — is now gradually being recognized as a regional power with the potential international agreements with other countries and give him the discretion to
for superpower status.78 The Philippines itself is encountering territorial problems determine whether an international agreement should be in the form of a treaty
with China because of the latter’s claims in the West Philippine Sea; the or executive agreement.
Philippines has chosen the path of peace in the dispute through international The patent misconception begins when the ponencia asserts that the President
arbitration.79 cannot function with crippled hands: “the manner of the President’s execution of the
EDCA and Article XVIII, Section 25 of the Constitution, in their larger law, even if not expressly granted by the law, is justified by necessity and limited
regional signification, mean that the Philippines would thereafter, not only by law since he must ‘take necessary and proper steps to carry into execution
only be bound as an American ally under the 1951 MDT, but as an active the law.’”81 It further adds that it is the President’s prerogative to do whatever is
participant as “pivot” and projection points in the grand American legal and necessary for the Philippines’ defense interests.82
strategy in Asia. While acknowledging the Constitution’s command that the entry of foreign
How the Philippines will react to all these developments is largely for the military bases, troops, and facilities must be
Executive and the people (through the Legislature) to determine. In making its _______________
decisions, they must — at the very least — show one and all that our country is
entitled to respect as an independent and sovereign nation. This respect must 80 Ponencia, pp. 337-341.
come primarily from within the Philippines and the Filipinos themselves, 81 Id., at p. 340.
from the nation’s own sense of self-respect: in negative terms, the Filipino 82 Id., at p. 341.
nation cannot attain self-respect unless it shows its respect for its own
_______________
557
77 Vine, David, Base Nation: How U.S. Military Bases Abroad Harm America VOL. 779, JANUARY 12, 2016 557
and the World, pp. 300-301 (2015).
78 Brzesinski, supra note 59 at pp. 151-193. Saguisag vs. Ochoa, Jr.
79 The arbitration case was filed before the Permanent Court of Arbitration
on January 22, 2013. See Republic of the Philippines v. The People’s Republic of in a treaty, the ponencia asserts that the EDCA should be examined in relation
China, Permanent Court of Arbitration, available at http://www.pca- with this requirement alone, as the President’s wide authority in external affairs
cpa.org/showpage65f2.html?pag_id= 1529. should be subject only to the limited amount of checks and restrictions under
the Constitution.83
It is within this framework that the ponencia concludes that the requirement Let it be noted that noble objectives do not authorize the President to bypass
under Article XVIII, Section 25 of the Constitution is limited to the initial entry of constitutional safeguards and limits to his powers. To emphasize this point, we
foreign military bases, troops, and facilities. Thus, once a treaty has allowed the only need to refer to Article VI, Section 23(2) of the Constitution:
entry of foreign military bases, troops, and facilities into the Philippines,
the ponencia posits that the President may enter into subsequent executive (2) In times of war or other national emergency, the Congress
agreements that involve “detail adjustments” of existing treaties. 84 may by law authorize the President, for a limited period and subject to
I cannot fully agree with the ponencia’s approach and with its such restrictions as it may prescribe, to exercise powers necessary and
conclusions. proper to carry out a declared national policy. Unless sooner
First and foremost, the ponencia overlooks that as Chief Executive, the withdrawn by resolution of the Congress, such
President’s role is not simply to execute the laws. This important function is _______________
preceded by the President’s foremost duty to preserve and defend the
Constitution, the highest law of the land. The President’s oath, quoted by 86 Although the ponencia recognized constitutional provisions that restrict or
the ponencia itself, in fact, states: limit the President’s prerogative in concluding international agreements
(see ponencia, pp. 357-371), it contradictorily asserts that “[n]o court can tell the
I do solemnly swear (or affirm) that I will faithfully and President to desist from choosing an executive agreement over a treaty to embody
conscientiously fulfill my duties as President (or Vice President or an international agreement, unless the case falls squarely within Article VIII, Sec.
Acting President) of the Philippines, preserve and defend its 25” and that “[t]he President had the choice to enter into the EDCA by way of an
Constitution, execute its laws, do justice to every man, and consecrate executive agreement or a treaty.” See ponencia, p. 371.
myself to the service of the Nation. So help me God.85 [Emphasis supplied]

The supremacy of the Constitution means that in the performance of his duties, 559
the President should always be guided and kept in check by the safeguards that VOL. 779, JANUARY 12, 2016 559
were crafted by the
_______________ Saguisag vs. Ochoa, Jr.

83 Id., at pp. 337-375. power shall cease upon the next adjournment thereof. [Emphasis supplied]
84 Id., at pp. 340-355, 375-446. Thus, the President cannot, by himself, usurp the prerogatives of a coequal
85 Constitution, Article VII, Section 5. branch to carry out what he believes is necessary for the country’s defense
interests. His position as the Commander-in-Chief of the Armed Forces of the
Philippines (AFP) does not give him the sole discretion to increase our military’s
558 defensive capabilities; his role as commander-in-chief only gives him control of the
military’s chain of command. It grants him the power to call out the armed forces
558 SUPREME COURT REPORTS ANNOTATED
to prevent/suppress lawless violence, invasion, insurrection, or rebellion. 87
Saguisag vs. Ochoa, Jr. The modernization of the military, in particular, is a joint responsibility of the
political branches of the State because the Congress is responsible for crafting
framers of the Constitution and ratified by the people. The Constitution relevant laws88 and for allocating funds for the AFP through the General
prescribes the limitations to the otherwise awesome powers of the Executive who Appropriations Act.89 The increase or decrease of funds and the extent of defense
wields the power of the sword and shares in the power of the purse. initiatives to be undertaken are national policy matters that the President cannot
I also do not agree that constitutional limitations, such as the need for Senate undertake alone.
concurrence in treaties, can be disregarded if they unduly “tie the hands” of the
President.86 These limitations are democratic safeguards that place the IV.B. The President’s Foreign Relations
responsibility over national policy beyond the hands of a single official. Their Power should be Interpreted in the Con-
existence is the hallmark of a strong and healthy democracy. In treaty-making, text of the Separation of Powers Doctrine
this is how the people participate — through their duly-elected Senate — or
directly when the Congress so requires. When the Constitution so dictates, the We cannot also interpret a provision in the Constitution in
President must act through the medium of a treaty and is left with no discretion on isolation and separately from the rest of the Constitution. Similarly, we cannot
the matter. This is the situation under Article XVIII, Section 25 of the Constitution, determine whether the Executive’s acts had been committed with grave abuse of
whose application is currently in dispute. discretion without considering his authority in the context of the powers of the
other branches of government.
_______________ Saguisag vs. Ochoa, Jr.

87 Constitution, Article VII, Section 18. Notably, this limitation is a not a new rule; the legislative branch of
88 The Constitution vests legislative power upon the Congress of the government has been participating in the treaty-making process by giving (or
Philippines. Thus, the Congress has the power to determine the subject matters it withholding) its consent to treaties since the 1935 Constitution. Section 10(7),
can legislate upon. See Constitution, Article VI, Section 1. Article VII of the 1935 Constitution provides:
89 Constitution, Article VI, Section 25.
Sec. 10. (7) The President shall have the power, with the concurrence
of two-thirds of all the Members of the Senate, to make treaties x x x.
560
560 SUPREME COURT REPORTS ANNOTATED This tradition of legislative participation continued despite our presidential-
parliamentary form of government under the 1973 Constitution, that is markedly
Saguisag vs. Ochoa, Jr.
different from the tripartite form of government that traditionally prevailed in the
country. Section 14(1) Article VIII of the 1973 Constitution stated:
While the President’s role as the country’s lead official in the conduct of foreign
affairs is beyond question, his authority is not without limit. When examined
Sec. 14. (1) Except as otherwise provided in this Constitution, no
within the larger context of how our tripartite system of government works (where
treaty shall be valid and effective unless concurred in by a majority of all
each branch of government is supreme within its sphere but coordinate with the
the Members of the Batasang Pambansa.
others), we can see that the conduct of foreign affairs, particularly when it comes
to international agreements, is a shared function among all three branches of
That we have consistently included the participation of the legislative branch
government.
in the treaty-making process is not without an important reason: it provides a
The President is undeniably the chief architect of foreign policy and is the
check on the Executive in the field of foreign relations. By requiring the
country’s representative in international affairs.90 He is vested with the authority
concurrence of the Legislature in the treaties entered into by the President, the
to preside over the nation’s foreign relations which involve, among others, dealing
Constitution ensures a healthy system of checks and balances necessary in the
with foreign states and governments, extending or withholding recognition,
nation’s pursuit of political maturity and growth.
maintaining diplomatic relations, and entering into treaties.91 In the realm of
Under this system, the functions of government are divided among three
treaty-making, the President has the sole authority to negotiate with other
branches of government, each one supreme within its own sphere: the executive
States.92
administers and enforces laws; the legislature formulates and enacts laws; and the
judiciary settles cases arising out of the enforcement of these
IV.B(1) Separation of Powers
and the Treaty-Making Process
562
This wide grant of authority, however, does not give him the license to conduct
foreign affairs to the point of disregarding or bypassing the separation of powers 562 SUPREME COURT REPORTS ANNOTATED
that underlies our established constitutional system. Saguisag vs. Ochoa, Jr.
Thus, while the President has the sole authority to negotiate and enter into
treaties, Article VII, Section 21 of the 1987 Constitution at the same time provides laws.93 The requirement of Senate concurrence to the executive’s treaty-
the limitation that two-thirds of the members of the Senate should give their making powers is a check on the prerogative of the Executive, in the same manner
concurrence for the treaty to be valid and effective. that the Executive’s veto on laws passed by Congress94 is a check on the latter’s
_______________ legislative powers.
Even the executive agreements that the President enters into without
90 Pimentel, Jr. v. Office of the Executive Secretary, 501 Phil. 303, 317-318; Senate concurrence has legislative participation — they are implementations of
462 SCRA 622, 632 (2005). existing laws Congress has passed or of treaties that the Senate had assented
91 Id. to.95 The President’s authority to negotiate and ratify these executive agreements
92 Id. springs from his power to ensure that these laws and treaties are executed.96
The judicial branch of government’s participation in international agreements
is largely passive, and is only triggered when cases reach the courts. The courts,
561 in the exercise of their judicial power, have the duty to ensure that the Executive
VOL. 779, JANUARY 12, 2016 561 and Legislature stay within their spheres of competence;97 they ensure as well that
constitutional standards and limitations set by the Constitution for the Executive authority can derive support from “congressional inertia, indifference or
and the Congress to follow are not violated. quiescence.”101
Article VIII, Section 5 of the Constitution is even more explicit, as it gives the _______________
Supreme Court the jurisdiction “to review by appeal or certiorari all cases in which
the constitutionality or validity of any treaty, international or executive 98 343 U.S. 579 (1952).
agreement, law x x x is in question.” 99 Id., at p. 635.
Thus, entry into international agreements is a shared function among the 100 Id., at p. 637.
three branches of government. In this light and in the context that the President’s 101 Id.
actions should be viewed under our tripartite system of government, I cannot
agree
564
_______________
564 SUPREME COURT REPORTS ANNOTATED

93 Angara v. Electoral Commission, 63 Phil. 139 (1936). Saguisag vs. Ochoa, Jr.
94 Constitution, Article VI, Section 27(2).
95 Commissioner of Customs v. Eastern Sea Trading, 113 Phil. 333, 338-340; Third, “when the President takes measures incompatible with the
3 SCRA 351, 357 (1961). expressed or implied will of Congress, his power is at its lowest ebb,”102 and
96 Constitution, Article VII, Sections 5 and 17. the Court can sustain his actions “only by disabling the Congress from acting upon
97 Angara v. Electoral Commission, supra at pp. 157-159. the subject.”103
This framework has been recently adopted by the U.S. Supreme Court
in Medellin v. Texas,104 a case involving the President’s foreign affairs powers and
563 one that can be directly instructive in deciding the present case.
In examining the validity of an executive act, the Court takes into
VOL. 779, JANUARY 12, 2016 563
consideration the varying degrees of authority that the President possesses. Acts
Saguisag vs. Ochoa, Jr. of the President with the authorization of Congress should have the “widest
latitude of judicial interpretation”105 and should be “supported by the strongest of
with the ponencia’s assertion that the case should be examined solely presumptions.”106 For the judiciary to overrule the executive action, it must decide
and strictly through the constitutional limitation found in Article XVIII, that the government itself lacks the power. In contrast, executive acts that are
Section 25 of the Constitution. without congressional imprimatur would have to be very carefully
examined.
IV.B(2) Standards in Examining the President’s Treaty-Making
Powers IV.B(3) The Senate Objection to EDCA
as an Executive Agreement
Because the Executive’s foreign relations power operates within the larger
constitutional framework of separation of powers, I find the examination of the In the present cases, the President’s act of treating the EDCA as an
President’s actions through this larger framework to be the better approach in the executive agreement has been disputed by the Senate, although the Senate is
present cases. This analytical framework, incidentally, is not the result of my not an active party in the present cases.
original and independent thought; it was devised by U.S. Supreme Court Associate _______________
Justice Robert Jackson in his Concurring Opinion in Youngstown Sheet & Tube
Co. v. Sawyer.98 102 Id.
Justice Jackson’s framework for evaluating executive action categorizes the 103 Id., at pp. 637-638.
President’s actions into three: first, when the President acts with authority from 104 552 U.S. 491 (2008).
the Congress, his authority is at its maximum, as it includes all the powers he 105 Id.; Youngstown Sheet & Tube Co. v. Sawyer, supra note 98 at p. 637.
possesses in his own right and everything that Congress can delegate.99 106 Id.
Second, “when the President acts in the absence of either a congressional
grant or denial of authority, he can only rely on his own independent powers,
but there is a [twilight zone where] he and Congress may have concurrent 565
authority, or where its distribution is uncertain.”100 In this situation, presidential
VOL. 779, JANUARY 12, 2016 565
Saguisag vs. Ochoa, Jr.
In general, the President’s foreign affairs power must be exercised in
On November 10, 2015, the Senate sent the Supreme Court a copy of Senate compliance with Article VII, Section 21 of the Constitution, which requires the
Resolution No. 1414107 expressing its sentiment that the EDCA should have been submission of treaties the President ratified, to the Senate for its concurrence. The
entered into in the form of a treaty. Furthermore, and as will be explained in the Senate may either concur in, or withhold consent to, the submitted treaties.
succeeding portions of this Dissenting Opinion, the EDCA’s provisions are not all Significantly, not all the international agreements that the President enters
within the terms of the two treaties properly ratified by the Senate — the 1951 into are required to be sent to the Senate for concurrence. Jurisprudence
MDT and 1998 VFA; hence, the President could not have drawn his authority from recognizes that the President may enter into executive agreements with other
these agreements. countries,109 and these agreements — under the proper conditions — do not
Thus, contrary to the ponencia’s assertion that the President’s act of treating require Senate concurrence to be valid and enforceable in the Philippines.110
the EDCA as an executive agreement should be subject to the “least amount of _______________
checks and restrictions under the Constitution,”108 this presidential action should
actually be very carefully examined, in light of the Senate’s own expressed 109 See Land Bank of the Philippines v. Atlanta Industries, Inc., G.R. No.
sentiments on the matter. 193796, July 2, 2014, 729 SCRA 12, 30-31, citing Bayan Muna v. Romulo, 656 Phil.
The mandatory character of the executive-legislative power sharing should be 246, 269-274; 641 SCRA 244, 258 (2011); Neri v. Senate Committee on
particularly true with respect to the EDCA, as its adoption signifies Philippine Accountability of Public Officers and Investigations, 586 Phil. 135, 168; 549 SCRA
participation in America’s pivot strategy by making our country one of the 77, 105 (2008), citing Usaffe Veterans Association, Inc. v. Treasurer of the
“pivot” or projection points that would enforce America’s military Philippines, 105 Phil. 1030, 1038 (1959); Commissioner of Customs v. Eastern Sea
strategy. In taking this kind of step, the Senate must simply be there to give its Trading, supra note 95.
consent, as the Constitution envisions in situations involving the entry of foreign 110 Id.
military bases, troops, and facilities into the country.
In these lights, I propose that we examine the President’s act of treating the
EDCA not simply by the standard of whether it complies with the limitation under 567
Article XVIII,
VOL. 779, JANUARY 12, 2016 567
_______________
Saguisag vs. Ochoa, Jr.
107 Senate Resolution No. 1414 was entitled as the “Resolution expressing the
strong sense of the Senate that any treaty ratified by the President of the IV.C(2) Treaties and Executive Agreements under Article VII, Section
Philippines should be concurred in by the Senate, otherwise the treaty becomes 21
invalid and ineffective.” It was signed by thirteen Senators: Senators Defensor- Where lies the difference, it may well be asked, since both a treaty and
Santiago, Angara, Cayetano, P., Ejercito, Estrada, Guingona III, Lapid, Marcos, an executive agreement fall under the general title of international
Jr., Osmeña III, Pimentel III, Recto, Revilla, Jr., and Villar. Available agreement?
at https://www.senate.gov.ph/listdata/2175018478!.pdf. An executive agreement emanates from the President’s duty to execute the
108 Ponencia, p. 375. laws faithfully.111 They trace their validity from existing laws or from treaties that
have been authorized by the legislative branch of government.112 In short, they
implement laws and treaties.
566 In contrast, treaties are international agreements that do not originate solely
from the President’s duty as the executor of the country’s laws, but from the shared
566 SUPREME COURT REPORTS ANNOTATED
function that the Constitution mandates between the President and the
Saguisag vs. Ochoa, Jr. Senate.113 They therefore need concurrence from the Senate after presidential
ratification, in order to fulfill the constitutional shared function requirement. 114
Section 25 of the Constitution, but in the context of how our government Jurisprudential definitions of treaties and executive agreements are
functions, and of other relevant provisions in the Constitution. conceptually drawn from these distinctions although in Bayan Muna v.
Romulo,115 we simply differentiated treaties from executive agreements in this
IV.C. Constitutional Standards in Allowing wise:
the Entry of Foreign Military Bases, Troops,
and Facilities in the Philippines Article 2 of the Vienna Convention on the Law of Treaties: An
international agreement concluded between states in written form and
IV.C(1) Article VII, Section 21 of the Con- governed by international law, whether embodied in a single instrument or
stitution and Treaty-Making
in two or more related instruments and whatever its particular 569
designation. International agreements may be in the VOL. 779, JANUARY 12, 2016 569
_______________
Saguisag vs. Ochoa, Jr.
111 Constitution, Article VII, Sections 5 and 17.
112 Commissioner of Customs v. Eastern Sea Trading, supra note 95. domestic sphere — carry no such effect.120 They cannot contravene statutory
113 Constitution, Article VII, Section 21. See also Bayan Muna v. enactments and treaties and would be invalid if they do so.121
Romulo, supranote 109 at pp. 269-270; p. 263. Again, this difference in impact is traceable to the source of their authority;
114 Bayan Muna v. Romulo, id. since a treaty has the approval of both the President and the Senate, it has the
115 Id. same impact as a statute. In contrast, since an executive agreement springs from
the President’s power to execute laws, it cannot amend or violate existing treaties,
and must be in accord with and be made pursuant to existing laws and treaties.122
568 Accordingly, the terms and objectives of the presidential entry into an
international agreement dictates the form the agreement must take. When an
568 SUPREME COURT REPORTS ANNOTATED international agreement is made merely to implement an existing law or
Saguisag vs. Ochoa, Jr. treaty, then it can properly take the form of an executive agreement.123
In contrast, when an international agreement involves the introduction of
form of (1) treaties that require legislative concurrence after a new subject matter or the amendment of existing agreements or laws and
executive ratification; or (2) executive agreements that are similar has not passed the required executive and legislative processes, then it should
to treaties, except that they do not require legislative properly be in the form of a treaty.124
concurrence and are usually less formal and deal with a narrower range To reiterate, the consequence of the violation of this norm impacts on the
of subject matters than treaties.116[Emphases supplied] enforceability of the international agreement in the domestic sphere; should an
executive agreement amend or contravene statutory enactments and treaties, then
Bayan Muna likewise did not distinguish between treaties and executive it is void and cannot be enforced in the Philippines for lack of the proper authority
agreements in terms of their binding effects on the contracting States on the part of the issuer.
concerned.117 But neither one can contravene the Constitution. _______________
This ambiguity perhaps might have been the root of the general statement that
the Executive generally has the discretion to determine whether an international 120 Gonzales v. Hechanova, 118 Phil. 1065, 1079; 9 SCRA 230, 242 (1963).
obligation should be in the form of a treaty or an executive agreement. This general 121 Adolfo v. CFI of Zambales, 145 Phil. 264, 266-268; 34 SCRA 166, 170
statement, however, is far from complete and should be qualified because (1970).
the Executive’s exercise of discretion is affected and should be dictated by the 122 Bayan Muna v. Romulo, supra note 109 at pp. 1079-1080; p. 279.
demands of the enforceability of the obligations the international agreement 123 Id.
creates in the domestic sphere. 124 Id.
Between a treaty and an executive agreement, a treaty exists on a higher plane
as it carries the authority of the President and the Senate.118 Treaties have the
status, effect, and impact of statutory law in the Philippines; they can amend or 570
prevail over prior statutory enactments.119 570 SUPREME COURT REPORTS ANNOTATED
Executive agreements — which exist at the level of implementing rules and
regulations or administrative orders in the Saguisag vs. Ochoa, Jr.
_______________
In judicial terms, the distinctions and their consequences mean that an
116 Id., at p. 269; pp. 258-259. executive agreement that creates new obligations or amends existing
117 Id. ones, has been issued with grave abuse of discretion amounting to a lack
118 Id., at p. 270; p. 259, citing Henkin, Foreign Affairs and the United States of or in excess of jurisdiction, and can he judicially nullified under the
Constitution, p. 224 (2nd ed., 1996), and Borchard, Edwin, Treaties and Executive courts’ power of judicial review.
Agreements – Reply, Yale Law Journal, June 1945.
119 Id. IV.C(3) Joint Reading of Article VII,
Section 21 and Article XVIII, Section 25
The dynamics that Article VII, Section 21 embody, should be read into Article the country on varying grounds, lengths or periods of time — all of which can
XVIII, Section 25 of the 1987 Constitution, which specifically covers and applies to change the nature of the obligations under existing treaties.
the entry of foreign military bases, troops, or facilities into the country. _______________
It is on the basis of this joint reading that the ponencia’sconclusion — that
Article XVIII, Section 25 applies only to the initial entry of foreign military bases, 125 BAYAN (Bagong Alyansang Makabayan) v. Zamora, supra note 69.
troops, and facilities in the country — is essentially incorrect. 126 Id., at p. 653; p. 484.
Article XVIII, Section 25 does not provide for any such limitation in its
applicability. Neither is there anything in the language of the provision that
remotely implies this consequence. What it simply states is that foreign military 572
bases, troops, and facilities may only be present in Philippine soil in accordance 572 SUPREME COURT REPORTS ANNOTATED
with a treaty concurred in by the Senate.
When the terms of Article XVIII, Section 25 treaty does not provide for Saguisag vs. Ochoa, Jr.
situations or arrangements subsequent to the initial entry of foreign military
bases, troops, or facilities in the country and the subsequent arrangements are still IV.C(4) The Dissent’s Analytical Approach
attributed to the same treaty made pursuant to Section 25, the combined reading
of Article VII, Section 21 and Article XVIII, Section 25 must now come into play. Given these parameters, I propose that we examine the constitutionality of the
This combined reading simply means that after the initial entry of foreign Executive’s act of entering into the obligations found in the EDCA in the form of
military bases, troops, or facilities in the Philippines under a duly ratified treaty, an executive agreement with these two questions:
subsequent arrangements relating to foreign military bases, troops or facilities (1) Does the EDCA involve the introduction into the Philippines of
that are claimed to be based on the same treaty, should be examined foreign military bases, troops, or facilities that call for its examination
under Article XVIII, Section 25?
(2) Does the EDCA impose new obligations, or amend or go beyond
571 existing ones, regarding the presence of foreign military bases, troops, or
VOL. 779, JANUARY 12, 2016 571 facilities in the Philippines?
If the EDCA introduces foreign military bases, troops, or facilities in the
Saguisag vs. Ochoa, Jr. Philippines within the contemplation of Article XVIII, Section 25 of the 1987
Constitution, and if these obligations are different from those found in our existing
based on the treaty-executive agreement distinctions recognized by treaty obligations with the U.S., then the EDCA cannot be enforced in the
jurisprudence under Article VII, Section 21 of the Constitution. Philippines without the Senate’s concurrence. The ponencia is then incorrect
In other words, any subsequent international agreement referring to military and the Dissent must prevail.
bases, troops or facilities should be examined based on whether it creates a Conversely, if the EDCA merely implements present treaty obligations —
new obligation or implements an existing one.The determination of this particularly those under the 1951 MDT and the 1998 VFA — then the President
question rests with the Executive but the treaty-executive agreement distinctions was well within his powers in the execution of our present treaty
should limit the Executive’s discretion when the new international agreement obligations. The ponencia is correct and the Dissent therefore fails.
relates to a new obligation (or a change in an existing obligation) as the presence
of foreign military bases, troops, or facilities in the Philippines should then be V. The Application of Article XVIII,
effected through another treaty. Section 25 to the EDCA
To put it more bluntly, Article XVIII, Section 25 effectively removes the
Executive’s discretion in deciding the form of an international agreement because V.A. The Article XVIII, Section 25 Dispute
of this provision’s explicit directive to use a treaty as the medium for new
obligations created.
In Bayan v. Zamora,125 our conclusion supported this position. We explained 573
that Article XVIII, Section 25 makes no distinction as to whether the presence of
VOL. 779, JANUARY 12, 2016 573
foreign military bases, troops, or facilities may be transient or permanent. 126 By
concluding that the permanence of foreign military bases, troops, or facilities is Saguisag vs. Ochoa, Jr.
immaterial to the application of Article XVIII, Section 25, we effectively
acknowledged that subsequent agreements that amend or introduce new When the subject of an international agreement falls under Article XVIII,
obligations to existing treaties that previously allowed the entry of foreign military Section 25 of the Constitution, the President — by constitutional command —
bases, troops or facilities, should be the subject of another treaty as they may enter must enter into a treaty subject to the concurrence of the Senate and, when
Congress so desires, of the people through a national referendum.
This rule opens the door for Court intervention pursuant to its duty to uphold contemporaneous developments and usages that give full and effective meaning to
the Constitution and its further duty (under its power of judicial review) to pass the provision.
upon any grave abuse of discretion committed by any official or agency of Separately from textual interpretation considerations and as part of the
government. It is under this constitutionally-mandated terms that this Court history of Article XVIII, Section 25, the basic concept of sovereignty that underlies
invokes its power to review the constitutionality of the President’s actions in it should not be forgot-
handling the EDCA. _______________
Within this framework, the issue these cases present is clear. The bottom line
question is whether the President gravely abused his discretion in 127 Ponencia, p. 341.
executing the EDCA as an executive agreement; the alleged existence of grave 128 Id., at p. 347.
abuse of discretion constitutes the actual case or controversy that allows the 129 Bolos v. Bolos, G.R. No. 186400, 20 October 2010, 634 SCRA 429, 437.
exercise of judicial power. Whether grave abuse exists, in turn, depends on the 130 Ponencia, pp. 356-357.
determination of whether the terms of the EDCA imposed new or amended 131 Id., at pp. 354-355.
existing obligations involving foreign military bases, troops, and facilities 132 Id.
in the Philippines.
If the EDCA does, then it should have been in the form of a treaty submitted
to the Senate for its concurrence. In resolving this question, I am guided first, by 575
the text of the, Constitution itself and the meaning of its operative words in both
VOL. 779, JANUARY 12, 2016 575
their original and contemporaneous senses; second, by the spirit that motivated
the framing of Article XVIII, Section 25; and third, by jurisprudence interpreting Saguisag vs. Ochoa, Jr.
this provision.
The ponencia lays the premise that the President may enter into an executive ten.133 Sovereignty means the full right and power of the nation to govern
agreement on foreign military bases, troops, or facilities if: itself, its people, and its territory without any interference from outside sources or
(a) it is not the instrument that allows the presence of foreign military bases, entities.134 Within its territory, a nation reigns supreme. If it will allow
troops, or facilities; or interference at all, such interference should be under the terms the nation allows
and has accepted;135beyond those terms, the primacy of sovereignty is the rule.136
_______________
574
574 SUPREME COURT REPORTS ANNOTATED 133 IV Record, Constitutional Commission, pp. 84, 659 and 661 (September
16, 1986), which reads:
Saguisag vs. Ochoa, Jr. MR. AZCUNA: After the agreement expires in 1991, the question, therefore,
is: Should we extend a new treaty for these bases to stay put in 1991 in our
(b) it merely aims to implement an existing law or treaty.127 territory? The position of the committee is that it should not, because the presence
of such bases is a derogation of Philippine sovereignty.
The ponencia follows this premise with the position that Article XVIII, Section It is said that we should leave these matters to be decided by the executive,
25 refers only to the initial entry of bases, troops, or facilities, and not to the since the President conducts foreign relations and this is a question of foreign
activities done after entry.128 policy. I disagree, Madam President. This is not simply a question of foreign policy;
In construing Article XVIII, Section 25, the ponencia invokes the rule of verba this is a question of national sovereignty. x x x
legis, a cardinal rule of construction stating that when the law is clear and free FR. BERNAS: My question is: Is it the position of the committee that the
from any doubt or ambiguity, then there is no room for construction or presence of foreign military bases in the country under any circumstances is a
interpretation, only application.129 The law must be given its literal meaning and derogation of national sovereignty?
applied without attempted interpretation.130 The ponencia asserts that the plain MR. AZCUNA: It is difficult to imagine a situation based on existing facts
meaning of “allowed in” refers solely to the initial entry. 131 Thus, after entry, any where it would not. x x x
subsequent acts involving foreign military troops, bases, or facilities no longer fall 134 Id.
under the coverage of Article XVIII, Section 25.132 135 See Tañada v. Angara, 338 Phil. 546, 593; 272 SCRA 18, 68 (1997),
I believe that the ponencia’s approach and interpretation are incorrect citing Reagan v. Commissioner of Internal Revenue, 141 Phil. 621, 625; 30 SCRA
because they are overly simplistic. The proper understanding of Article XVIII, 968, 973 (1969), where the Court discussed the concept of auto-limitation, viz.: “It
Section 25 must take into account the many considerations that bear upon its plain is to be admitted that any State may by its consent, express or implied, submit to
terms, among them, the treaty-executive agreement distinctions under Article VII, a restriction of its sovereignty rights. That is the concept of sovereignty as auto-
Section 21 that I discussed above; the history of Article XVIII, Section 25; the limitation which, in the succinct language of Jellinek, ‘is the property of a state-
motivations that drove the framers to adopt the provision; and the current and
force due to which it has the exclusive capacity of legal-self determination and self- 1991 of the Agreement between the Republic of the Philippines and the United
restriction.’ A State then, if it chooses to, may refrain from the exercise of what States of America concerning Military Bases x x x.”
otherwise is illimitable competence.” Purely and simply, the framers of the Constitution in 1986 then looked forward
136 Id. to the expiration of the U.S. bases coming in 1991 and wanted the terms of any
future foreign military presence governed by the Constitution itself. Behind this
intent is the deeper policy expressed under Article II, Section 7 of the Constitution
576 —
576 SUPREME COURT REPORTS ANNOTATED
The State shall pursue an independent foreign policy. In its
Saguisag vs. Ochoa, Jr. relations with other states the paramount consideration shall be national
sovereignty, territorial integrity, national interest, and the right to
Thus, if interference were to be allowed at all, or if exceptions to full self-determination.
sovereignty within a territory would be allowed, or if there would be any ambiguity
in the extent of an exception granted, the interference, exception or ambiguity During the constitutional deliberation on Article XVIII, Section 25, two views
must be resolved in favor of the fullest exercise of sovereignty under the obtaining were espoused on the presence of military bases in the Philippines. One view was
circumstances. Conversely, if any ambiguity exists at all in the terms of the that espoused by the anti-bases group; the other group supported the view that
exception or in the terms of the resulting treaty, then such terms should be this should be left to the policy makers.
interpreted restrictively in favor of the widest application of the restrictions Commissioner Adolfo Azcuna expressed the sentiment of the first group when
embodied in the Constitution and the laws. he stated in his privilege speech on 16 September 1986 that:
The ponencia cannot be incorrect in stating the rule that when terms are clear
and categorical, no need for any forced constitutional construction exists;137 we After the agreement expires in 1991, the question therefore, is: Should we
need not divine any further meaning but must only apply terms in the sense that extend a new treaty for these bases to stay put in 1991 in our territory? The
they are ordinarily understood. position of the committee is that it should not, because the presence of such
A flaw, however, exists in the ponencia’s application of verba legis as Article bases is a derogation of Philippine sovereignty.
XVIII, Section 25 is neither plain nor that simple.
As pointed out above, it must be read together with Article VII, Section 21 for It is said that we should leave these matters to be decided by the executive,
the general rules on the treaty-making process. It also expressly refers to a since the President conducts foreign relations and this is a question of
historical incident — the then coming expiration of the 1947 MBA. From these foreign policy. I disagree, Madam President. This is not simple a question
takeoff points, the Article XVIII, Section 25 proceeds to a list of the matters it of foreign policy; this is a question of national sovereignty.
specifically addresses — foreign military bases, troops, or facilities.
All these bring up the question that has so far been left undiscussed — what
are the circumstances that led to the expiration of the 1947 MBA and what 577
are the foreign military bases, troops, and facilities that Article XVIII,
VOL. 779, JANUARY 12, 2016 577
Section 25 refers to?
Saguisag vs. Ochoa, Jr.
V.B. The History and Intent of
Article XVIII, Section 25 And the Constitution is anything at all, it is a definition of the
_______________ parameters of the sovereignty of the people.138

137 Ponencia, pp. 356-357. On the other hand, the second group posited that the decision to allow foreign
bases into the country should be left to the policy makers. Commissioner Jose
Bengzon expressed the position of this group that:
577
x x x this is neither the time nor the forum to insist on our views for we
VOL. 779, JANUARY 12, 2016 577
know not what lies in the future. It would be foolhardy to second-guess the
Saguisag vs. Ochoa, Jr. events that will shape the world, our region and our country by 1991. It
would be sheer irresponsibility and a disservice to the highest calibre to our
The history of Article XVIII, Section 25 of the Constitution is practically own country if we were to tie down the hands of our future governments
summed up in the introductory phrase of the provision — “After the expiration in and future generations.139
Despite his view that the presence of foreign military bases in the Philippines MR. ROMULO: Madam President, may I propose my amendment to the
would lead to a derogation of national security, Commissioner Azcuna conceded Bernas amendment: “AFTER THE EXPIRATION OF THE RP-US AGREEMENT
that this would not be the case if the agreement would allow the foreign military IN 1991, FOREIGN MILITARY BASES, TROOPS OR FACILITIES SHALL NOT
bases, troops, and facilities to be embodied in a treaty.140 BE ALLOWED IN THE PHILIPPINE TERRITORY EXCEPT UNDER THE
_______________ TERMS OF A TREATY DULY CONCURRED IN BY THE SENATE, AND WHEN
CONGRESS SO REQUIRES RATIFIED BY A MAJORITY OF THE VOTES CAST
138 III Record, Constitutional Commission 86 (16 September 1986), p. 659. BY THE PEOPLE IN A REFERENDUM HELD FOR THAT PURPOSE AND
139 IV Record, Constitutional Commission 82 (13 September 1986), pp. 617- RECOGNIZED AS A TREATY BY THE OTHER CONTRACTING STATE.”
618. 142 IV Record, Constitutional Commission 86 (18 September 1986), p. 780,
140 IV Record, Constitutional Commission 84 (16 September 1986), pp. 661- which reads:
662, which reads: FR. BERNAS: On the other hand, Madam President, if we place it in the
FR. BERNAS. My question is: is it the position of the committee that the Transitory Provisions and mention only the American State, the conclusion might
presence of foreign military bases in the country under any circumstances is a be drawn that this applies only to foreign
derogation of national sovereignty?
MR. AZCUNA: It is difficult to imagine a situation based on existing facts
where it would not. However, in the abstract, it is possible that it would not be that 580
much of a derogation. I have in mind, Madam President, the argument that has 580 SUPREME COURT REPORTS ANNOTATED
been presented. Is that the reason why there are U.S. bases in England, in Spain
and in Turkey? And it is not being claimed that their sovereignty is being Saguisag vs. Ochoa, Jr.
derogated. Our situation is different from theirs because we did not lease or rent
sions of the Constitution is primarily a balance of the preservation of the
national sovereignty and openness to the establishment of foreign bases,
579 troops, or facilities in the country.
Article XVIII, Section 25 imposed three requirements that must be complied
VOL. 779, JANUARY 12, 2016 579 with for an agreement to be considered valid insofar as the Philippines is
Saguisag vs. Ochoa, Jr. concerned. These three requirements are: (1) the agreement must be embodied in
a treaty; (2) the treaty must be duly concurred in by 2/3 votes of all the members
After a series of debates, Commissioner Ricardo Romulo proposed an of the Senate;143 and (3) the agreement must be recognized as a treaty by the other
alternative formulation that is now the current Article XVIII, Section 25.141 He State.
explained that this is an explicit ban on all foreign military bases other than those On the second requirement, the two-thirds concurrence of all the members of
of the U.S.142 Based on the discussions, the spirit of the basing provi- the Senate, the people’s representative,144
_______________ _______________

these bases to the U.S. The US retained them from us as a colonial power. military bases of the United States. The conclusion might be drawn that the
xxxx principle does not apply to other states.
FR. BERNAS: Does the first sentence tolerate a situation radically different MR. ROMULO: That is certainly not our meaning. We do not wish any other
from what obtains now? In other words, if we understand sovereignty as auto foreign military base here and I think the phrase which says: “NO FOREIGN
limitation, as a people’s power to give up certain goods in order to obtain something MILITARY BASES, TROOPS OR FACILITIES...” makes that very clear even if it
which may be more valuable, would it be possible under this first sentence for the is in the Transitory Provisions.
nation to negotiate some kind of a treaty agreement that would not derogate 143 Bayan (Bagong Alyansang Makabayan) v. Zamora, supra note 69 at p.
against sovereignty? 652; pp. 483-487, stating that:
MR. AZCUNA: Yes. For example, Madam President, if it is negotiated on a Undoubtedly, Section 25, Article XVIII, which specifically deals with treaties
basis of true sovereign equality, such as a mutual ASEAN defense agreement involving foreign military bases, troops, or facilities, should apply in the instant
wherein an ASEAN force is created and this ASEAN force is a foreign military case. To a certain extent and in a limited sense, however, the provisions of Section
force and may have a basis in the member ASEAN countries, this kind of a 21, Article VII will find applicability with regard to the issue and for the sole
situation, I think would not derogate from sovereignty. purpose of determining the number of votes required to obtain the valid
141 IV Record, Constitutional Commission 86 (18 September 1986), p. 787, concurrence of the Senate, as will be further discussed hereunder.
which reads: xxxx
As noted, the “concurrence requirement” under Section 25, Article XVIII must
be construed in relation to the provisions of Section 21, Article VII. In a more
particular language, the concurrence of the Senate contemplated under Section 25, 581
Article XVIII means that at least two-thirds of all the members of the Senate VOL. 779, JANUARY 12, 2016 581
favorably vote to concur with the treaty, the VFA in the instant case.
144 Constitution, Article VII, Section 21. See also Bernas, Joaquin, The 1987 Saguisag vs. Ochoa, Jr.
Constitution of the Republic of the Philippines: A Commentary, pp. 487-488 (1995).
may be viewed as the people’s “voluntary submission” of their sovereignty so
they can reap the greater benefits of the agreement that the President, as
580 policymaker, entered into.
When the Congress so requires, the agreement should be ratified by a majority
580 SUPREME COURT REPORTS ANNOTATED
of the votes cast by the people in a national referendum held for that
Saguisag vs. Ochoa, Jr. purpose.145 This additional requirement evinces the framers’ intent to emphasize
the people’s direct participation in treaty-making.
sions of the Constitution is primarily a balance of the preservation of the In Bayan v. Zamora,146 the Court relaxed the third requirement when it ruled
national sovereignty and openness to the establishment of foreign bases, that it is sufficient that “the other contracting party accepts or acknowledges the
troops, or facilities in the country. agreement as a treaty.” In that case, since the U.S. had already declared its full
Article XVIII, Section 25 imposed three requirements that must be complied commitment to the 1998 VFA,147 we declared that it was unnecessary for the U.S.
with for an agreement to be considered valid insofar as the Philippines is to further submit the agreement to the U.S. Senate.148
concerned. These three requirements are: (1) the agreement must be embodied in This history highlights the importance of the issue now before us, and stresses
a treaty; (2) the treaty must be duly concurred in by 2/3 votes of all the members as well how seriously the Constitution regards the Senate concurrence
of the Senate;143 and (3) the agreement must be recognized as a treaty by the other requirement. Thus, the issue can neither be simply glossed over nor disregarded
State. on the basis of stretched legal technicalities. In case of doubt, as above discussed,
On the second requirement, the two-thirds concurrence of all the members of such doubt should be resolved strictly in favor of what the Constitution requires in
the Senate, the people’s representative,144 its widest sense.
_______________
V.C. Historical Roots of the U.S. Bases in the Philippines
military bases of the United States. The conclusion might be drawn that the
principle does not apply to other states. As a U.S. colony after the Treaty of Paris of 1898, the whole Philippines could
MR. ROMULO: That is certainly not our meaning. We do not wish any other be equated to one big American base: the U.S. had sovereignty and had a free hand
foreign military base here and I think the phrase which says: “NO FOREIGN on how to deal with defense matters and its military forces in the Philippines.
MILITARY BASES, TROOPS OR FACILITIES...” makes that very clear even if it _______________
is in the Transitory Provisions.
143 Bayan (Bagong Alyansang Makabayan) v. Zamora, supra note 69 at p. 145 Constitution, Article XVIII, Section 25.
652; pp. 483-487, stating that: 146 Bayan (Bagong Alyansang Makabayan) v. Zamora, supra note 69.
Undoubtedly, Section 25, Article XVIII, which specifically deals with treaties 147 Id., at p. 659; p. 488.
involving foreign military bases, troops, or facilities, should apply in the instant 148 Id., at pp. 656-659; p. 488.
case. To a certain extent and in a limited sense, however, the provisions of Section
21, Article VII will find applicability with regard to the issue and for the sole
purpose of determining the number of votes required to obtain the valid 582
concurrence of the Senate, as will be further discussed hereunder. 582 SUPREME COURT REPORTS ANNOTATED
xxxx
As noted, the “concurrence requirement” under Section 25, Article XVIII must Saguisag vs. Ochoa, Jr.
be construed in relation to the provisions of Section 21, Article VII. In a more
particular language, the concurrence of the Senate contemplated under Section 25, The Tydings-McDuffie Act of 1934 provided for the Philippines’ self-
Article XVIII means that at least two-thirds of all the members of the Senate government and specified a procedural framework for the drafting of a constitution
favorably vote to concur with the treaty, the VFA in the instant case. for the government of the Commonwealth of the Philippines149 within two years
144 Constitution, Article VII, Section 21. See also Bernas, Joaquin, The 1987 from the Act’s enactment.150The Act, more importantly, mandated the recognition
Constitution of the Republic of the Philippines: A Commentary, pp. 487-488 (1995). by the U.S. of the independence of the Philippine Islands as a separate and self-
governing nation after a ten-year transition period.151
Prior to independence, the Act allowed the U.S to maintain military forces in armed forces and personnel were granted rent-free access up to the expiration
the Philippines and to call all military forces of the Philippine government into of the Agreement.156
U.S. military service.152 The Act empowered the U.S. President, within two
years following independence, to negotiate for the establishment of U.S. _______________
naval reservations and fueling stations in the Philippine Islands.153
The negotiations for American bases that took place after independence 154 The 1947 MBA Whereas Clause, par. 7, states:
resulted in the 1947 MBA. THEREFORE, the Governments of the Republic of the Philippines and of the
United States of America agree upon the following terms for the delimitation,
V.C(1) The 1947 Military Bases Agreement establishment, maintenance, and operation of military bases in the Philippines.
155 1947 MBA, Article XXIX; see Annexes A and B of the 1947 MBA.
The 1947 MBA between the Philippines and the U.S. was signed on March 16, 156 The 1947 MBA Whereas clause states:
1947. The agreement officially allowed Whereas, the Governments of the Republic of the Philippines and of the United
_______________ States of America are desirous of cooperating in the common defense of their two
countries through arrangements consonant with the procedures and objectives of
149 The Tydings-McDuffie Act, also known as the Philippine Independence the United Nations, and particularly through a grant to the United States of
Act, was entitled “An Act to Provide for the Complete Independence of the America by the Republic of the Philippines in the exercise of its title and
Philippine Islands, to provide for the Adoption of a Constitution and a Form of sovereignty, of the use, free of rent, in furtherance of the mutual interest of both
Government for the Philippine Islands, and for other purposes.” It was signed into countries, of certain lands of the public domain. x x x (Emphases supplied)
law by President Franklin D. Roosevelt on March 24, 1934 and was approved by
the Philippine Senate on May 1, 1934. See Encyclopedia Britannica, Tydings-
McDuffie Act, available at http:/www.britanica.com/topic/Tydings-McDuffie- 584
Act and http://www.philippine-history.org/tydings-mcduffie-law.htm.
150 Tydings-McDuffie Act, Section 3. 584 SUPREME COURT REPORTS ANNOTATED
151 Id., Section 10. Saguisag vs. Ochoa, Jr.
152 Id., Section 2(12). See also Ordinance appended to 1935 Constitution,
Section 1(12).
153 Id., Section 10(b).
 The bases were for the mutual protection and cooperation of the two countries
and for this purpose were for their use as U.S. and Philippine military
installations.157
583
 The U.S. had the right, power and authority necessary for the establishment,
VOL. 779, JANUARY 12, 2016 583 operation, and defense of the bases and their control,158 specifically:
Saguisag vs. Ochoa, Jr. o To operate, maintain, utilize, occupy, garrison, and control the bases;
o To improve and deepen the harbors, channels and entrances and anchorage,
the U.S. to establish, maintain, and operate air and naval bases in the and to construct and maintain necessary roads and bridges accessing the
country.154 It provided for about 23 listed bases and facilities for use by Americans bases;
for a period of 99 years.155 The most important of these bases were the 180,000- o To control the operation and safety of the bases and all the structures and
acre Clark Air Base in Pampanga, then the biggest American airbase outside of facilities in them;
the continental U.S.A., and the Subic Naval Base in Zambales. o To acquire right-of-way by agreement and to construct telecommunication
The bases covered by the 1947 MBA were fixed bases where and other facilities;
American structures and facilities had been built and arms, weapons, and o To construct, install, maintain and employ on any base any type of facilities,
equipment were deployed and stored, and where troops and civilian weapons, substance, device, or vessel as may be necessary;
personnel were stationed, together with their families. o To bring into the Philippines members of the U.S. military forces and U.S.
Other provisions of the 29-article 1947 MBA were the following: nationals employed under contract by the U.S. with the families, as well as
technical personnel of other nationalities not otherwise excluded from the
Philippines.
 The bases were properties over which the U.S. originally exercised sovereignty
 The Philippine government was prohibited from granting any bases to other
but this was subsequently transferred to the Philippines pursuant to
nations without U.S. consent.159
the Romulo-Murphy Agreement of 1979. After the transfer, the U.S. and its
military arms, ammunitions, supplies, equipment, vessels, services, and training
for the latter’s defense forces.
 The U.S. was permitted to recruit Filipino citizens, on voluntary basis, for The 1953 Exchange of Notes Constituting an Agreement Extending the
service in the American military.160 Agreement Between the Government of the Republic of the Philippines and the
Government of the United States of America on Military Assistance to the
Philippines (1953 Agreement) clarified that the 1947 Agreement would remain in
force until terminated by any of the parties.
_______________

157 1947 MBA, Whereas Clause, Articles II and III. 586


158 Id., Articles II, III, IV, VI, and VII.
586 SUPREME COURT REPORTS ANNOTATED
159 Id., Article XXV(1).
160 Id., Article XXVII. Saguisag vs. Ochoa, Jr.

On September 16, 1966, the Ramos-Rusk Agreement reduced its term to 25


585 years starting from that year.
VOL. 779, JANUARY 12, 2016 585 A review of the 1947 MBA in 1979 led to the formal transfer of control of Clark
and Subic bases to the Philippines.163 Thus, these bases became Philippine
Saguisag vs. Ochoa, Jr. military installations containing U.S. military facilities. The review also
provided that each base would be under a Filipino base commander;
the Philippine flag was to fly singly in the bases; the Philippine
 The U.S. base commanders had the right to tax, distribute utilities, hand out government was to provide security along the bases’ perimeters; and the
licenses, search without warrants, and deport undesirables.161 review of the agreements would take place every five years starting in 1979. 164
On September 16, 1991, the Philippine Senate rejected the proposed RP-US
Treaty of Friendship, Cooperation and Security that would have extended the life
Complementing the signing of the 1947 MBA was the signing of the Military
of the bases for 10 more years.165The 1947 MBA was terminated on December 21,
Assistance Agreement of 1947 and the 1951 MDT.
1992 when the 25-year tenure lapsed. This prompted the U.S. to vacate its bases
Over the years, various provisions of the 1947 MBA were amended, gradually
effective at the end of December 1992.166
delimiting U.S. control over the bases.162
_______________
_______________
163 See Romulo-Murphy Exchange of Notes of 1979.
164 See Official Gazette, Report of President Marcos to the Batasang
161 Id., Articles XI, XII, XIII, XIV, and XV.
Pambansa, January 15, 1979.
162 The Ramos-Rusk Agreement of 1966 reduced the term of the 1947 Bases
165 Bayan (Bagong Alyansang Makabayan) v. Zamora, supra note 69 at p.
Treaty to a total of 44 years or until 1991.
632; p. 464, which states:
The Bohlen-Serrano Memorandum of Agreement provided for the return to the
In view of the impending expiration of the RP-U.S. Military Bases Agreement
Philippines of 17 U.S. military bases.
in 1991, the Philippines and the U.S. negotiated for a possible extension of the
The Romulo-Murphy exchange of Notes of 1979 recognized Philippine
military bases agreement. On September 16, 1991, the Philippine Senate rejected
sovereignty over the Clark and Subic Bases, reduced the area that could be used
the proposed RP-U.S. Treaty of Friendship, Cooperation and Security which, in
by the U.S. military, and provided for the mandatory review of the 1947 Bases
effect, would have extended the presence of U.S. military bases in the Philippines.
Treaty every five years.
166 Philippine Communications Satellite Corporation v. Globe Telecom,
The Romualdez-Armacost Agreement of 1983 revised the 1947 Bases Treaty,
Inc., 473 Phil. 116, 122; 429 SCRA 153, 156 (2004), which states:
particularly pertaining to the operational use of military/bases by the U.S.
On 31 December 1991, the Philippine Government sent a Note
government within the context of Philippine sovereignty, including the need for
Verbale to the U.S. Government through the U.S. Embassy, notifying it of
prior consultation with the Philippine government on the former’s use of the bases
the Philippines’ termination of the RP-US Military Bases Agreement.
for military combat operations or the establishment of long-range missiles.
The Note Verbale stated that since the RP-US Military Bases Agreement,
The 1947 Military Assistance Agreement (1947 MAA) entered into by the
as amended, shall ter-
President with the U.S. pursuant to the authority granted under Republic Act No.
9. The Agreement established the conditions under which the U.S. military
assistance would be granted to the Philippines, particularly the provision of
587 SOFA on June 19, 1951, to define the terms for the deployment and status of its
VOL. 779, JANUARY 12, 2016 587 military forces in these countries.171Most of the other NATO states, however,
_______________
Saguisag vs. Ochoa, Jr.
169 Mason, R. Chuck, “Status of Forces Agreement (SOFA): What is it, how is
The departure of the U.S. warship Bellau Wood marked the closure of it utilized?” U.S. Congressional Research Service Report for Congress (March 15,
American military bases in the country.167 2012). Available at https://wvvw.fas.org/sgp/crs/natsec/RL34531.pdf.
With the expiration of the 1947 MBA, the detailed arrangements for the 170 For an illustrated depiction of the increase of U.S. military bases around
presence of U.S. military forces and facilities in the Philippines, particularly those the world before (1939) and after (1945) World War III, see Vine, David, supra note
listed above, similarly ended, leaving only the general arrangements under the 77 at pp. 32-36.
1951 Mutual MDT. 171 See Mason, supra, stating that the U.S. and Germany entered into a
supplemental agreement to the NATO-SOFA (as provided in 14 U.S.T. 531;
V.C(2) The 1951 Mutual Defense Treaty T.I.A.S. 5351. Signed at Bonn, August 3, 1959. Entered into force July 1, 1963) and
additional exchange of notes related to specific issues (14 U.S.T. 689; T.I.A.S. 5352;
The 1951 MDT was signed on August 30, 1951, while the U.S. was establishing 490 U.N.T.S. 30. Signed at Bonn, August 3, 1959. Entered into force July 1, 1963).
a number of bilateral defense alliances with key Asian States as it positioned itself Also, the Manila Pact entered into on September 8, 1954 by the U.S., the
to contain communist expansion in Asia in the period following World War II and Philippines, Australia, France, New Zealand, Pakistan, and Thailand, whereby the
the Korean War. Despite periods of drift, its relationship with its Asian allies parties agreed, among others, to: settle any international disputes in which they
provided the U.S. support and assistance throughout the Cold War and during the may be involved by peaceful means in such a manner that international peace and
Vietnam War.168 security and justice are not endangered, and to refrain in their international
The 1951 MDT provided the general terms of the defense alliance between the relations from the threat or use of force in any manner inconsistent with the
U.S. and the Philippines; the more detailed terms were reflected in the earlier 1947 purposes of the United Nations; and separately and jointly, by means of continuous
MBA that expired and was not renewed in 1991. and effective self-help and mutual aid will maintain and develop their individual
_______________ and collective capacity to resist armed attack and to prevent and counter
subversive activities directed from without against their territorial integrity and
minate on 31 December 1992, the withdrawal of all U.S. military forces political stability. See Southeast Asia Collective Defense Treaty (September
from Subic Naval Base should be completed by said date.
167 Anderson, Gerald, Subic Bay From Magellan to Pinatubo: The History of
the US Naval Station, Subic Bay, p. 181 (2006), available 589
at https://books.google.com.ph/books?id=OfPs0NH5EuAC&printsec=frontcover&d
VOL. 779, JANUARY 12, 2016 589
q=subic+bay+from+magellan+to+pinatubo&hl=en&sa=X&ved=0ahUKEwjvitrLr
NjJAhUBJ5QKHcBICAUQ6AEIJDAA#v=onepage&q=subic%20bay%20from%20 Saguisag vs. Ochoa, Jr.
magellan%20to%20pinatubo&f=false.
168 Vaughn, Bruce, “U.S. Strategic and Defense Relationships in the Asia- required ratification and implementing legislation, with additional
Pacific Region” U.S. Congressional Research Service Report for Congress (January agreements to implement the NATO-SOFA.172
22, 2007). Available at https://www.fas.org/sgp/crs/row/RL33821.pdf. The 1951 MDT provides for an alliance — that both nations would support one
another if either the Philippines or the U.S. would be attacked by an external
party.173 It states that each party shall either, separately or jointly, through
588 mutual aid, acquire, develop and maintain their capacity to resist armed
588 SUPREME COURT REPORTS ANNOTATED attack.174 It provides for a mode of consultations to determine the 1951 MDT’s
appropriate implementation measures and when either of the parties determines
Saguisag vs. Ochoa, Jr. that their territorial integrity, political independence, or national security is
threatened by armed attack in the Pacific.175 An attack on either party will be
The 1947 MBA and the 1951 MDT were the counterparts of U.S. agreements acted upon in accordance with their constitutional processes and any armed attack
with the North Atlantic Treaty Organization (NATO)countries. One of those on either party will be brought to the attention of the United Nations for
agreements was the NATO Status of Forces Agreement (NATO-SOFA), a immediate action.176
multilateral agreement that applies to all the NATO member countries. 169 The accord defines the meaning of an armed attack as including armed attacks
After the World War II, the U.S. maintained various European by a hostile power on a metropolitan area of either party, on the island territories
bases.170 Despite the presence of these bases, the U.S. entered into the NATO-
under their jurisdiction in the Pacific, or on their armed forces, public vessels, or 178 Id., Article VIII.
aircrafts in the Pacific.177The U.S. government guar- 179 Lim v. Executive Secretary, 430 Phil. 555, 562; 380 SCRA 739, 742-743
_______________ (2002), which states: These so-called “Balikatan” exercises are the largest
combined training operations involving Filipino and American troops. In theory,
8, 1954), 209 U.N.T.S. 28-30, available they are a simulation of joint military maneuvers pursuant to the Mutual Defense
at https://treaties.un.org/doc/Publication/UNTS/Volume%20209/v209.pdf. Treaty, a bilateral defense agreement entered into by the Philippines and the
172 For example, the U.S. entered into supplementary agreement with the United States in 1951.
Federal Republic of Germany (which acceded to the NATO-SOFA in 1963) with 180 See Moderno, H. Marcos, “A Decade of US Troops in Mindanao: Revisiting
respect to allied forces stationed permanently in Germany, see Fleck, Dieter, The the Visiting Forces Agreement (2),” MindaNews,
Handbook of the Law on Visiting Forces, p. 353 (2001).
173 The 1951 MDT states the Parties’ objective “[d]esiring to declare publicly
and formally their sense of unity and their common determination to defend 591
themselves against external armed attack, so that no potential aggressor could be VOL. 779, JANUARY 12, 2016 591
under the illusion that either of them stands alone in the Pacific Area.”
174 1951 MDT, Article II. Saguisag vs. Ochoa, Jr.
175 Id., Article III.
176 Id., Article IV. In line with the American approach, the 1998 VFA allows the rotational
177 Id., Article V. presence of U.S. military forces and their operations anywhere in the Philippines
for a temporary but undefined length of time to train and inter-operate with the
Philippine armed forces and to use their facilities. The Philippines retains
590 jurisdiction over criminal cases, including capital offenses, involving U.S. troops.181
In BAYAN v. Zamora,182 the Court held that although the agreement did not
590 SUPREME COURT REPORTS ANNOTATED entail the permanent basing of a foreign military force, it required a treaty because
Saguisag vs. Ochoa, Jr. Article XVIII, Section 25 of the Constitution covers not only the presence of bases
but also the presence of “troops.”183 As a treaty, the 1998 VFA required the
anteed to defend the security of the Philippines against external aggression concurrence of the Senate pursuant to Article VII, Section 21 of the Constitution.
but not necessarily against internal subversion. The treaty expressly stipulates
that its terms are indefinite and would last until one or both parties terminate the _______________
agreement by a one-year advance notice.178 The treaty subsequently became the
basis for an annual joint exercise, known as Balikatan, between the Philippines April 24, 2012, available at http://www.mindanews.com/special-
and the U.S.179 reports/2012/04/24/a-decade-of-us-troops-in-mindanao-revisiting-the-visiting-
On the whole, the 1951 MDT embodied an alliance and defense agreement, forces-agreement-2/.
focused as it is on joint action and defenses against armed external attacks. It made 181 1998 VFA, Article V.
no provision for bases, troops, or facilities which the 1947 MBA contained and 182 Bayan (Bagong Alyansang Makabayan) v. Zamora, supra note 69.
which lapsed when the MBA’s term expired. 183 Id., at p. 652; p. 483, which states:
On the whole, the VFA is an agreement which defines the treatment of United
V.C(3) The 1998 Visiting Forces Agreement States troops and personnel visiting the Philippines. It provides for the guidelines
to govern such visits of military personnel, and further defines the rights of the
The 1998 VFA came after the expiration of the 1947 MBA in 1991 and opened United States and the Philippine government in the matter of criminal
a limited window for the presence of American troops in the Philippines. It was jurisdiction, movement of vessel and aircraft, importation and exportation of
entered into during the era when the U.S. was envisioning “access” as a new equipment, materials and supplies.
approach in maintaining its presence in Southeast Asia. Instead of permanent Undoubtedly, Section 25, Article XVIII, which specifically deals with treaties
bases, the approach sought bilateral arrangements — like those with Singapore — involving foreign military bases, troops, or facilities, should apply in the instant
for training, exercises, and interoperability to allow for uninterrupted case. To a certain extent and in a limited sense, however, the provisions of Section
forward deployment in the Asian region; their continued presence in the region 21, Article VII will find applicability with regard to the issue and for the sole
assures faster response to developments in flash points in the eastern purpose of determining the number of votes required to obtain the valid
hemisphere.180 concurrence of the Senate, as will be further discussed hereunder.
_______________
592 Goldberg, in time for the official state visit by U.S. President Barack Obama. The
592 SUPREME COURT REPORTS ANNOTATED 10-year accord is the second military agreement between the U.S. and the
Philippines (the first being the 1998 VFA) since American troops withdrew from
Saguisag vs. Ochoa, Jr. its Philippines naval base in 1992.
The agreement allows the U.S. to station troops and operations on Philippine
The Court also held that the Philippines is bound to accept an official territory without establishing a permanent base189 and with the stipulation that
declaration by the U.S. to satisfy the requirement that the other contracting party the U.S. is not allowed to store or position any nuclear weapons on Philippine
must recognize the agreement as a treaty.184It noted that the Vienna Convention territory.190
on the Law of Treaties leaves each state free to choose its form of giving consent to The EDCA was entered into for the following purposes:
a treaty.185
1. This Agreement deepens defense cooperation between the Parties and
V.D. The EDCA maintains and develops their individual and collective capacities, in
furtherance of Article II of the MDT, which states that “the Parties
As heretofore outlined, the U.S. adopted the “Pivot to Asia” strategy beginning separately and jointly by self-help and mutual aid will maintain and
2009 under the administration of President Barack Obama. In the develop their individual and collective capacity to resist armed attack,” and
article Explaining the U.S. Pivot to Asia, Kurt Campbell and Brian Andrews within the context of the VFA. This includes:
enumerated six key efforts under the U.S.’s “Pivot to Asia” policy, namely:
alliances; improving relationships with emerging powers; economic statecraft; (a) Supporting the Parties’ shared goal of improving interoperability of
engaging with multi-lateral institutions; support for universal values; and the Parties’ forces and for the Armed Forces of the Philippines (“AFP”),
increasing military presence.186 addressing short-term capabilities gaps; promoting long-term
On military presence, the operative word is “presence”: the forward modernization, and helping maintain and develop additional maritime
deployment of U.S. military forces in Asia.187 The EDCA perfectly fits the security, maritime domain awareness, and humanitarian assistance and
American strategy as it allows the prepositioning of equipment and disaster relief capabilities; and
supplies in agreed locations to enhance the U.S.’s “development of a geo- _______________
_______________
188 Id.
184 Id., at p. 657; p. 488, which states: 189 EDCA, Preamble, par. 5.
This Court is of the firm view that the phrase recognized as a treaty means 190 Id., Article IV, par. 6.
that the other contracting party accepts or acknowledges the agreement as a
treaty. To require the other contracting state, the United States of America in this
case, to submit the VFA to the United States Senate for concurrence pursuant to 594
its Constitution, is to accord strict meaning to the phrase.
594 SUPREME COURT REPORTS ANNOTATED
185 Bernas, Joaquin, supra note 144 at pp. 1400-1401.
186 See Campbell, Kurt & Andrews, Brian, Explaining the US ‘Pivot’ to Asia, Saguisag vs. Ochoa, Jr.
pp. 3-8, August 2013, Chatham House, available
at https://www.chathamhouse.org/sites/files/chathamhouse/public/Research/Amer (b) Authorizing access to Agreed Location in the territory of the Philippines
icas/0813pp_pivottoasia.pdf. by United States forces on a rotational basis as mutually determined by the
187 Id., at p. 8. Parties.
2. In furtherance of the MDT, the Parties mutually agree that this Agreement
provides the principal provisions and necessary authorizations with respect to
593 Agreed Locations.
VOL. 779, JANUARY 12, 2016 593 3. The Parties agree that the United States may undertake the following
types of activities in the territory of the Philippines in relation to its access to and
Saguisag vs. Ochoa, Jr. use of Agreed Locations: security cooperation exercises; joint and combined
training activities; humanitarian assistance and disaster relief activities; and such
other activities as may be agreed upon by the Parties.191
graphically dispersed, politically sustainable force posture in the
region.”188 To summarize, the EDCA has two main purposes:
The EDCA was signed on April 28, 2014, in Manila, by Philippine Defense
Secretary Voltaire Gazmin, and U.S. Ambassador to the Philippines Philip
First, it is intended as a framework for activities for defense cooperation in special meaning. In fact, this meaning was the compelling force that convinced the
accordance with the 1951 MDT and the 1998 VFA. framers to include Article XVIII, Section 25 in the 1987 Constitution.
Second, it grants to the U.S. military the right to use certain identified More specifically, when the framers of the 1987 Constitution referred to foreign
portions of the Philippine territory referred to in the EDCA as Agreed military bases, they had in mind the
Locations. This right is fleshed out in the EDCA when the agreement identifies the
privileges granted to the US in bringing in troops and facilities, in constructing
structures, and in conducting activities.192 596
The EDCA is effective for 10 years, unless both the U.S. and the Philippines 596 SUPREME COURT REPORTS ANNOTATED
formally agree to alter it.193 The U.S. is bound to hand over any and all facilities
in the “Agreed Locations” to the Philippine government upon the termination of Saguisag vs. Ochoa, Jr.
the Agreement.
In terms of contents, EDCA may be divided into two: then existing 1947 MBA.194 This is apparent from the text of the provision itself
_______________ which makes direct reference to the treaty, as well as from the exchanges of the
framers of the 1987 Constitution prior to their vote on the proposed provision.195
191 Id., Article I. In construing the meaning of statutes and of the Constitution, one aim is to
192 Id., Article III. discover the meaning that the framers attached to the particular word or phrase
193 Id., Article XII(4). employed.196 The pertinent statute or provision of the Constitution must then be
“construed as it was intended to be understood when it was passed.”197
Thus, a proper interpretation of the meaning of foreign military bases must
595 take into account how it was understood by the framers in accordance with how
the 1947 MBA established U.S. military bases in the Philippines. It is in this
VOL. 779, JANUARY 12, 2016 595 technical and precise meaning that the term military base was used. It is this kind
Saguisag vs. Ochoa, Jr. of military bases that Article XVIII, Section 25 intends to cover, subject to specific
qualifications.
First, it reiterates the purposes of the 1951 MDT and the 1998 VFA in that it Hence, the concept of military bases as illustrated in the 1947 MBA should be
affirms the continued conduct of joint activities between the U.S. and the taken into account in ascertaining whether the EDCA contemplates the
Philippines in pursuit of defense cooperation. establishment of foreign military bases. This reality renders a comparison of the
Second, it contains an entirely new agreement pertaining to Agreed 1947 MBA and the EDCA appropriate.
Locations, the right of the U.S. military to stay in these areas and conduct _______________
activities which may not be imbued with mutuality of interests since they do not
involve defense cooperation. 194 V Record, Constitutional Commission, p. 105. (October 11, 1986), which
The latter provides support for two interrelated arguments that I will forward reads:
in this Opinion. First, the EDCA refers to the presence of foreign military bases, Mr. Benntagen: Point of information. I have with me a book of Patricia M.
troops, and facilities in this jurisdiction. Second, the EDCA is not a mere Paez, The Bases Factor, the authority on US relations. And reference to the
implementation of, but goes beyond, the 1951 MDT and the 1998 VFA. It is an agreement reads this way: Agreement between the Republic of the Philippines and
agreement that introduces new terms and obligations not found in the 1951 MDT the United States of America concerning military bases.
and the 1998 VFA, and thus requires the concurrence of the Senate. Mr. Azcuna: That is the official title. Why do we not use that? After the
expiration of the agreement x x x.
V.D(1) Does the EDCA involve the entry 195 Id.
of military bases to the Philippines as en- 196 Alcantara, Samson, Statutes, p. 58 (1997 ed.); see also Agpalo,
visioned under Article XVIII, Section 25? Ruben, Statutory Construction, 6th ed., p. 282.
197 Ernesto v. Court of Appeals, 216 Phil. 319, 327-328; 131 SCRA 347, 356
V.D(1)(i) The Concept of a Foreign Military Base (1984).

A reading of the EDCA will reveal that it pertains to the presence in this
country of a foreign military base or the modern equivalent of one. While Article 597
XVIII, Section 25 mentions no definition of what a foreign military base, troops, or VOL. 779, JANUARY 12, 2016 597
facility is, these terms, at the time the 1987 Constitution was drafted, carried a
Saguisag vs. Ochoa, Jr.
To clarify this position, it is not that the framers of the 1987 Constitution had
in mind the specific existing foreign military bases under the 1947 MBA when they While the 1947 MBA grants broader powers to the U.S., due perhaps to the
drafted Article XVIII Section 25. Such a position unjustifiably assumes that the geopolitical context under which the agreement was forged (the 1947 MBA had an
framers lacked foresight and merely allowed themselves to be solely limited by the international, in contrast with EDCA’s Asian, focus), the EDCA and the 1947 MBA
existing facts. essentially pursue the same purpose — the identification of portions of
Rather, my position is that it is the concept of a foreign military Philippine territory over which the U.S. is granted certain rights for its
base under the 1947 MBA, and not the specific military bases listed in its military activities.
Annexes, that should be determinative of what the Constitution intends to These rights may be categorized into four:
cover. The foreign military base concept should necessarily be adjusted, (1) the right to construct structures and other facilities for the proper
too, to take into account the developments under the new U.S. “Pivot to functioning of the bases;
Asia” strategy. (2) the right to perform activities for the defense or security of the bases or
Agreed Locations;
V.D(1)(ii) EDCA and the 1947 MBA Compared (3) the right to preposition defense equipment, supplies and materiel; and
(4) other related rights such as the use of public utilities and public services.
A first material point to note is that the obligations under the EDCA are
similar to the obligations found in the 1947 MBA. To support this view, I Only those who refuse to see cannot discern these undeniable parallelisms.
present below a side by side comparison of the relevant provisions of the EDCA
and the 1947 MBA.
602
602 SUPREME COURT REPORTS ANNOTATED

598 Saguisag vs. Ochoa, Jr.


598 SUPREME COURT REPORTS ANNOTATED
Further, even independently of the concept of military bases under the 1947
Saguisag vs. Ochoa, Jr. MBA, the provisions of the EDCA itself provide a compelling argument that it
seeks to allow in this country what Article XVIII, Section 25 intends to regulate.
There exists no rigid definition of a military base. However, it is a term used
in the field of military operations and thus has a generally accepted
connotation. The U.S. Department of Defense (DoD) Dictionary of Military and
599 Associated Terms defines a base as “an area or locality containing installations
VOL. 779, JANUARY 12, 2016 599 which provide logistic or other support”; home carrier.198
Under our laws, we find the definition of a military base in Presidential Decree
Saguisag vs. Ochoa, Jr. No. 1227 which states that a military base is “any military, air, naval, coast guard
reservation, base, fort, camp, arsenal, yard, station, or installation in the
Philippines.”199 A military base connotes the presence, in a relatively permanent
degree, of troops and facilities in a particular area.200
In 2004, the U.S. DoD released Strengthening U.S. Global Defense Posture, a
600 report to U.S. Congress about the renewed U.S. global position.201 The U.S. DoD
600 SUPREME COURT REPORTS ANNOTATED redefined and reclassified their military bases in three categories:
_______________
Saguisag vs. Ochoa, Jr.
198 US Department of Defense, Joint Publication 1-02, Department of Defense
Dictionary of Military and Associated Terms, p. 21 (2015), available at
<http://www.dtic.mil/doctrine/new_pubs/jp1_02.pdf>.
199 Presidential Decree No. 1227, Section 2.
601
200 IV Record, Constitutional Commission, p. 86 (September 18, 1986):
VOL. 779, JANUARY 12, 2016 601 Fr. Bernas: By the term ‘bases,’ were we thinking of permanent bases?
Saguisag vs. Ochoa, Jr. Mr. Maambong: Yes.
201 US DoD, Strengthening U.S. Global Defense Posture: Report to Congress, Saguisag vs. Ochoa, Jr.
U.S. Department of Defense, pp. 10-11 (2004). Available
at http://www.dnizhawaii.org/wp-content/uploads/2008/12/global_posture.pdf. The third classification of military bases is a Cooperative Security Location,
described as follows:
603 Cooperative Security Location (CSL)
VOL. 779, JANUARY 12, 2016 603
Saguisag vs. Ochoa, Jr. Cooperative security locations will be facilities with little or no permanent
U.S. presence. Instead they will be maintained with periodic service,
contractor, or host-nation support. CSLs will provide contingency access
Main Operating Base (MOB) and be a focal point for security cooperation activities. A current example
of a CSL is in Dakar, Senegal, where the U.S. Air Force has negotiated
Main operating bases, with permanently stationed combat forces and contingency landing, logistics, and fuel contracting arrangements, and
robust infrastructure, will be characterized by command and control which served as a staging area for the 2003 peace support operation in
structures, family support facilities, and strengthened force protection Liberia.204
measures. Examples include Ramstein Air Base (Germany), Kadena Air
Base (Okinawa, Japan), and Camp Humphreys (Korea). The GDPR emphasized that the U.S.’s plan is to establish a network of
FOSs and CSLs in Asia-Pacific to support the global war on terrorism and
Forward Operating Site (FOS) to provide multiple avenues of access for contingency operations. These
facilities serve to expand training opportunities for the U.S. and the host-country.
Forward operating site will be an expandable “warm facilities” maintained FOSs and CSLs allow U.S. forces to use these areas in times of crisis
with a limited US military support presence and while avoiding the impression of establishing a permanent presence.205 Notably,
possibly prepositioned equipment. FOSs will support rotational rather these access agreements are less expensive to operate and maintain than
than permanently stationed forces and be a focus for bilateral and regional MOBs.206 Moreover, FOSs and CSLs allow overseas military presence with a
training. Examples include the Sembawang port facility in Singapore and lighter footprint.207
Soto Cano Air Base in Honduras. To go back to the EDCA, it notably allows the U.S. to use the Agreed Locations
for the following activities: “training,
The following are the key characteristics of an FOS: _______________
First, an FOS is an expandable/scalable facility. Andrew Krepinevich and
Robert Work noted that an FOS can support both small and large forces, and can 204 US DoD, supra note 201.
be readily expanded to serve as expeditionary or campaign bases should a crisis 205 Charbonneau, Bruno and Cox, Wayne, Locating Global Order: American
erupt nearby.202 Power and Canadian Security after 9/11, p. 65 (2010).
Second, the facility is maintained or “kept warm” by limited U.S. military 206 Pettyjohn, Stacie, “Minimalist International Interventions: For the Future
support personnel or U.S. military contractors. It hosts rotational rather than US Overseas Presence, Access Agreement Are Key” Summer 2013, RAND
permanently stationed forces. An FOS may also house prepositioned equipment. Corporation, available at http://www.rand.org/pubs/periodicals/rand-
Finally, an FOS facility does not need to be owned by the U.S. (i.e., the review/issues/2013/summer/for-the-future-us-overseaspresence.html.
Sembawang Port Facility and the Paya Lebar Airfield in Singapore). FOSs are 207 Id., at p. 2.
generally bases that support forward-deployed rather than forward-
based forces.203
_______________ 605
VOL. 779, JANUARY 12, 2016 605
202 Krepinevich, Andrew and Work, Robert, A New Global Defense Posture for Saguisag vs. Ochoa, Jr.
the Second Transoceanic Era, p. 19 (2007).
203 Id., at p. 18. transit, support and related activities, refueling of aircraft; bunkering of vessels;
temporary maintenance of vehicles, vessels, and aircraft; temporary
accommodation of personnel; communications; prepositioning of equipment,
604 supplies, and materiel; deploying forces and materiel and such other activities as
604 SUPREME COURT REPORTS ANNOTATED the Parties may agree.”208
In order to carry out these activities, the EDCA allows U.S. military Further, as we held in Bayan, Article XVIII, Section 25 refers to three different
personnel to enter and remain in Philippine territory. It grants the U.S. the right to situations: the presence of foreign military bases, troops, or facilities.212 Even
construct structures and assemblies.209 It also allows the U.S. to preposition defense assuming that the EDCA is not a basing agreement, it nevertheless involves
equipment, supplies and materiel.210 The U.S. personnel may also use the Agreed the deployment of troops and facilities in Philip-
Locations to refuel aircraft and bunker vessels.211 _______________
Stockpiling of military materiel in the Philippines is explicitly permitted under
the following EDCA provisions: 212 BAYAN (Bagong Alyansang Makabayan) v. Zamora, supra note 69 at p.
1. Article III, par. 1: The activities allowed on the agreed locations 653; p. 482.
include: (i) the prepositioning of equipment, supplies, and materiel; (ii)
deploying forces and materiel; and (iii) such other activities as the Parties
may agree. 607
2. Article IV, par. 1: U.S. forces are allowed to preposition and
VOL. 779, JANUARY 12, 2016 607
store defense equipment, supplies, material (“prepositioned
materiel”), including, but not limited to, humanitarian assistance and Saguisag vs. Ochoa, Jr.
disaster relief equipment, supplies, and materiel, at agreed locations.
3. Article IV, par. 3: The prepositioned materiel is for the pine soil. As I have already stated, the EDCA allows U.S. forces to enter and
exclusive use of U.S. forces and full title shall belong to the U.S. remain in the Philippines. It defines U.S. forces to include U.S. military and
_______________ civilian personnel and U.S. Armed Forces property, equipment, and
materiel.213 The EDCA itself provides that the U.S. can deploy forces and materiel
208 EDCA, Article III, Section 1. in the Agreed Locations.214
209 Id., Article V, Sec. 2. In like manner x x x such that, the provision contemplates three different
210 Id., Article IV, Sec. 1. situations — a military treaty the subject of which could be either (a) foreign bases,
211 Id. (b) foreign troops, or (c) foreign facilities — any of the three standing alone places
it under the coverage of Section 25, Article XVIII.
That the EDCA allows this arrangement for an initial period of 10 years, to
606 continue automatically unless terminated,215 is further proof that it pertains to the
presence in Philippine soil of foreign military bases, troops, and facilities on a more
606 SUPREME COURT REPORTS ANNOTATED
or less permanent basis.
Saguisag vs. Ochoa, Jr. Note, at this point, that the Senators, during the ratification of the 1998 VFA,
observed that it only covers temporary visits of U.S. troops and personnel in the
4. Article IV, par. 4: The U.S. forces and U.S. contractors shall country. These Senators gave their consent to the 1998 VFA on the
have unimpeded access to the agreed locations for all matters relating knowledge that the U.S. forces’ stay in the country may last only up to three
to the prepositioning and storage of defense equipment, supplies, weeks to six months per batch.216
and materiel, including delivery, management, inspection, use, This temporary stay of U.S. forces in the Philippines under the 1998 VFA
maintenance, and removal of such equipment, supplies and means that it does not cover, or approve of, a more permanent stay of US forces and
materiel. their equipment in the Philippines. Significantly, this is the key characteristic of
the Agreed Locations in the EDCA. For, if the EDCA had not envisioned the stay
Notably, neither the 1951 MDT nor the 1998 VFA authorized of U.S. forces and equipment in the Agreed Locations in the Philippines for a period
stockpiling. The 1951 MDT focused on developing the Philippines and the U.S.’s longer than envisioned in the 1998 VFA, it would not have added obligations
capacity to resist an armed attack while 1998 VFA focused on the entry and exit regarding the storage of their equipment and materiel. The more permanent
of US troops in the country. No provision in either treaty specifically allows nature of the EDCA, in contrast to the 1998 VFA,
stockpiling of military materiel. _______________
In sum, the Agreed Locations mentioned in the EDCA are areas where the U.S.
can perform military activities in structures built by its personnel. The extent of 213 EDCA, Article II, Section 2.
the U.S.’ right to use of the Agreed Locations is broad enough to include even 214 Id., Article III, Sec. 1.
the stockpiling of weapons and the shelter and repair of vessels over which the U.S. 215 Id., Article XII, Sec. 4.
personnel has exclusive control. Clearly, this is a military base as this term is 216 The senators argued the precise length of time but agreed that it would
ordinarily understood. not exceed six months. See Senate deliberations on P.S. Res. No. 443 – Visiting
Forces Agreement, May 17, 1999, Record and Archives Service, Vol. 133, pp. 23- rangements provided under the 1947 MBA for the presence of U.S. bases,
25. troops and facilities also ended, leaving only the 1951 MDT and its general
terms in place.
Under this situation, the detailed arrangements that expired with the 1947
608 MBA were not carried over to the 1951 MDT as this treaty only generally provided
608 SUPREME COURT REPORTS ANNOTATED for the defense and alliance relationship between the U.S. and the Philippines.
Thus, there were no specific policies on military bases, troops, and facilities that
Saguisag vs. Ochoa, Jr. could be implemented and operationalized by subsequent executive agreements on
the basis of the MDT.
indicates a change in the tenor of the agreement in the EDCA, one that does In particular, the terms of the 1947 MBA that had expired and had not
not merely implement the 1998 VFA. been renewed cannot be deemed carried over to the 1951 MDT. If any such
future agreements would be made after the effectivity of the 1987
V.D(2) Does the EDCA Merely Implement Constitution, then such agreements would be governed by Article XVIII,
the 1951 MDT? Section 25 of the new Constitution.
Significantly, when the 1987 Constitution and its Article XVIII, Section 25 took
This question responds to the ponencia’s argument that the EDCA can be effect, no absolute prohibition against the introduction of new U.S. bases, troops,
embodied in an executive agreement because it merely provides implementing and facilities was put in place. In fact the 1951 MDT then still existed as a general
details for the 1951 MDT.217 defense alliance of the Philippines and the U.S. against armed attack by third
parties. But the introduction of military bases or their equivalent, of troops, and of
V.D(2)(i) The Effects of the Expiration military facilities into the Philippines can now only take place by way of a treaty
of the 1947 MBA and of the Adoption of concurred in by the Senate.
the 1987 Constitution
V.D(2)(ii) The 1951 MDT examined in light
The sequence of events relating to American bases, troops, and facilities in the of the EDCA
Philippines that took place since Philippine independence, is critical in responding
to the question in caption. It should be remembered that as a condition under the That the EDCA is purely an implementation of the 1951 MDT and does not
Tydings-McDuffie Act for the grant of Philippine independence, the Philippines need to be in the form of a treaty, is not tenable for two reasons.
was bound to negotiate with the U.S. for bases in the Philippines, resulting in the First, the EDCA grants rights and privileges to the U.S. that go well beyond
1947 MBA. what is contemplated in the 1951 MDT and the 1998 VFA.
This agreement contained the detailed terms relating to the existence and
operation of American bases and the presence of American forces and facilities in
the Philippines. As its title denotes, the 1951 MDT is the treaty providing for 610
alliance and mutual defense against armed attack on either country; it only
610 SUPREME COURT REPORTS ANNOTATED
generally contained the defense and alliance relationship between the
Philippines and the U.S. Saguisag vs. Ochoa, Jr.
In 1987, the Philippines adopted a new Constitution. This Charter directly
looked forward to the expiration of the 1947 MBA and provided for the terms under Second, even the assumptions that the EDCA is indeed a mere
which foreign military bases, troops, and facilities would thereafter be allowed into implementation of both the earlier 1951 MDT and the 1998 VFA, this assumption
the Philippines. The 1947 MBA expired in 1991 and no replacement treaty took its by no means provides an argument in favor of treating the EDCA as an executive
place; thus, all the detailed ar- agreement. Notably, the 1998 VFA is also recognized as an implementation of
_______________ the 1951 MDT yet the Government deemed it necessary to have it embodied in a
separate treaty concurred in by the Senate.
217 Ponencia, pp. 375-402. On the first argument, an analysis of the 1951 MDT, the 1998 VFA, and the
EDCA reveals that the EDCA is a stand-alone agreement.
The 1951 MDT is a treaty intended for the collective defense of its signatory
609 countries (i.e., the U.S. and the Philippines) against external armed attack. This
is apparent from its declaration of policies which states, among others, that the
VOL. 779, JANUARY 12, 2016 609
U.S. and the Philippines have agreed to the MDT in pursuit of their desire to —
Saguisag vs. Ochoa, Jr.
x x x declare publicly and formally their sense of unity and their
common determination to defend themselves against external armed Article II. In order more effectively to achieve the objective of this
attack, so that no potential aggressor could be under the illusion that Treaty, the Parties separately and jointly by self-help and mutual aid
either of them stands alone in the Pacific area.218 will maintain and develop
_______________
The rest of the text of the 1951 MDT consistently highlights this goal.
Its Article II states that the parties shall “separately and jointly by self-help and 219 Supra note 179 at pp. 571-572; p. 752.
mutual aid maintain and develop their individual and collective capacity to resist
armed attack.” Article III provides that the parties shall “consult together”
regarding the implementation of the MDT whenever in their opinion the 612
“territorial integrity, political independence or security of either of the parties is
612 SUPREME COURT REPORTS ANNOTATED
threatened by external armed attack in the Pacific.” Article IV declares that an
armed attack in the Pacific area on either of the par- Saguisag vs. Ochoa, Jr.
_______________
their individual and collective capacity to resist armed attack.
218 1951 MDT, Preamble, par. 3. Article IV. Each Party recognizes that an armed attackin the
Pacific area on either of the Parties would be dangerous to its own peace
and safety and declares that it would act to meet the common dangers
611 in accordance with its constitutional processes.
Any such armed attack and all measures taken as a result thereof shall
VOL. 779, JANUARY 12, 2016 611
be immediately reported to the Security Council of the United Nations.
Saguisag vs. Ochoa, Jr. Such measures shall be terminated when the Security Council has taken
the measures necessary to restore and maintain international peace and
ties would be dangerous to each other’s peace and safety and thus they would security.
act to meet the common danger. Article V then proceeds to define an armed attack Article V. For purposes of ARTICLE IV, an armed attack on either
as to include an armed attack on “the metropolitan territory of either parties or on of the Parties is deemed to include an armed attack on the metropolitan
the island territories under its jurisdiction in the Pacific Ocean, its armed forces, territory of either of the Parties, or on the island territories under its
public vessels and aircrafts in the Pacific.” jurisdiction in the Pacific Ocean, its armed forces, public vessels or aircraft
This Court has had occasion to explain the nature of the 1951 MDT. In Lim v. in the Pacific.
Executive Secretary,219 we said —
(Fortunately, the limits of the 1951 MDT have not been tested in actual
x x x The MDT has been described as the core of the defense operation since neither the Philippines nor the U.S. has as yet been the subject of
relationship between the Philippines and its traditional ally, the United an armed attack in the Pacific region.)
States. Its aim is to enhance the strategic and technological capabilities of In relating the 1951 MDT to the EDCA, I glean from the ponencia the intent
our armed forces through joint training with its American counterparts to seize the term “mutual aid” in developing the contracting parties’ collective
x x x. [Emphasis supplied] capacity to resist an armed attack, as basis for the US to establish a military base
or a military facility or station military troops in the Philippines.220 This reading,
Thus, the essence of the 1951 MDT is the conduct of joint activities by the U.S. however, would be a novel one in the context of American agreements with other
and the Philippines in accordance with the dictates of collective defense against an Asian countries with their own alliance and MDTs with the U.S.
attack in the Pacific. This is a focus that the EDCA lacks. _______________

V.D(2)(iii) The 1951 MDT Compared with Other 220 Ponencia, pp. 378-383.
Defense Alliance Agreements

Our military obligations to the U.S. under the 1951 MDT are (1) to maintain 613
and develop our military capacity to resist armed attack, and (2) to recognize that VOL. 779, JANUARY 12, 2016 613
an armed attack against the U.S. in the Pacific is an attack on the Philippines and
to meet the common danger in accordance with our constitutional process. The Saguisag vs. Ochoa, Jr.
relevant provisions read:
Note that Article II of the RP-U.S. 1951 MDT is similar to the following U.S. the right to access and use of areas and facilities of the other contracting
provisions in other MDTs: party. Thus:
(1) The 1953 US-South Korean MDT
Article IV
Article II (US-Korea)
The Parties will consult together whenever, in the opinion of either of The Republic of Korea grants, and the United States of America
them, the political independence or security of either of the Parties is accepts, the right to dispose United States land, air and sea forces
threatened by external armed attack. Separately and jointly, by self-help in and about the territory of the Republic of Korea as determined by
and mutual aid, the Parties will maintain and develop appropriate mutual agreement.225
means to deter armed attack and will take suitable measures in _______________
consultation and agreement to implement this Treaty and to further its
purposes.221 223 Treaty of Mutual Cooperation and Security, U.S.-Japan, January 19,
1960, 373 U.N.T.S. 188, available
(2) The 1954 US-Taiwan (Republic of China) MDT at https://treaties.un.org/doc/Publication/UNTS/Volume%20373/v373.pdf.
224 The US-Taiwan MDT states that self-help and mutual aid will be utilized
Article II by the Parties to resist not only an armed attack but also “communist subversive
In order more effectively to achieve the objective of this Treaty, the activities directed against Taiwan’s territorial integrity and political stability.”
Parties separately and jointly by self-help and mutual aid will maintain Moreover, the US-Korean Treaty adds the phrase “whenever, in the opinion of
and develop their individual and collective capacity to resist armed either of them, the political independence or security of either of the Parties is
attack and communist subversive activities directed from without against threatened by external armed attack” and uses the phrase “means to deter [an]
their territorial integrity and political stability.222 armed attack”) instead of “maintain and develop x x x their capacities
to resist armed attack.”
(3) the 1960 US-Japan Treaty of Mutual Co-operation and Security 225 Mutual Defense Treaty, U.S.-South Korea, supra note 221.
_______________

221 Mutual Defense Treaty, U.S.-South Korea, October 1, 1953, 238 U.N.T.S. 615
202, 204, available VOL. 779, JANUARY 12, 2016 615
at https://treaties.un.org/doc/Publication/UNTS/Volume%20238/v238.pdf.
222 Mutual Defense Treaty, U.S.-Taiwan, December 10, 1954, 248 U.N.T.S. Saguisag vs. Ochoa, Jr.
214, available
at https://treaties.un.org/doc/Publication/UNTS/Volume%20248/v238.pdf. Article VII
(US-Taiwan)
The Government of the Republic of China (Taiwan) grants, and the
614 Government of the United States of America accepts, the right to dispose
such United States land, air and sea forces in and about Taiwan
614 SUPREME COURT REPORTS ANNOTATED
and the Pescadores as may be required for their defense, as determined
Saguisag vs. Ochoa, Jr. by mutual agreement.226
Article VI
Article III (US-Japan)
The Parties, individually and in cooperation with each other, by means For the purpose of contributing to the security of Japan and the
of continuous and effective self-help and mutual aid will maintain and maintenance of international peace and security in the Far East, the
develop, subject to their constitutional provisions, their capacities to United States of America is granted the use by its land, air and
resist armed attack.223 naval forces of facilities and areas in Japan.
The use of these facilities and areas as well as the status of United
With little variance,224 these articles are essentially identical to Article II of States armed forces in Japan shall be governed by a separate agreement,
the RP-U.S. 1951 MDT. replacing the Administrative Agreement under Article III of the Security
But notably, despite the existence of the above mentioned provisions, all three Treaty between Japan and the United States of America, signed at Tokyo
treaties also saw the need to include a separate provision explicitly granting the on February 28, 1952, as amended, and by such other arrangements as may
be agreed upon.227
The Philippines’ primary obligation under the 1998 VFA, is to facilitate the
These three articles do not have any counterpart in the RP-US 1951 MDT. entry and departure of U.S. personnel in rela-
Understandably perhaps, counterpart provisions are not in the 1951 MDT as our
commitment to grant the U.S. use and access to areas and facilities in the
Philippine territory was embodied in an earlier agreement, the 1947 MBA (which, 617
however, expired, thus ending the use and access grants to the U.S. and its armed VOL. 779, JANUARY 12, 2016 617
forces).
In my view, the implication of the above quoted provisions in the US-South Saguisag vs. Ochoa, Jr.
Korea, US-Taiwan, and US-Japan treaties
_______________ tion with “covered activities”;228 it merely defines the treatment of U.S.
personnel visiting the Philippines; hence, its name.229 It is in fact a counterpart of
226 Mutual Defense Treaty, U.S.-Taiwan, supra note 222. the NATO-SOFA that the U.S. forged in Europe.
227 Treaty of Mutual Cooperation and Security, U.S.-Japan, supra note 223. The Preamble of the VFA defines its objectives — to govern the terms of visits
of “elements of the United States Armed Forces” to the Philippines, while the body
of the agreement contains the agreed conditions. To quote from the relevant
616 provisions of the 1998 VFA:
616 SUPREME COURT REPORTS ANNOTATED VISITING FORCES AGREEMENT
Saguisag vs. Ochoa, Jr. Preamble

(on “mutual aid”) is clear: the obligation to provide mutual aid under The Government of the Republic of the Philippines and the Government
Article II of the RP-US 1951 MDT (and its counterpart of the United States of America,
provisions) does not include the obligation to allow the entry and the Reaffirming their faith in the purposes and principles of the Charter of
stationing of U.S. troops or the establishment of military bases or the United Nations and their desire to strengthen international and
facilities. regional security in the Pacific area;
In light particularly of the constitutional developments in 1987, the 1951 MDT Reaffirming their obligations under the Mutual Defense Treaty of
cannot be invoked as an umbrella agreement that would legally justify the grant August 30, 1951;
to the U.S. of entry, access, and use of Philippine-owned areas or facilities without Noting that from time to time elements of the United States
Senate concurrence. These activities, which the EDCA seeks to do allegedly armed forces may visit the Republic of the Philippines;
pursuant to the 1951 MDT, do not fall within the purview of our commitments _______________
under the earlier treaty.
228 1998 VFA, Article III(1).
V.D(3) Does the EDCA Merely Implement the 229 BAYAN (Bagong Alyansang Makabayan) v. Zamora, supra note 69. On
1998 VFA? the whole, the VFA is an agreement which defines the treatment of United States
troops and personnel visiting the Philippines. It provides for the guidelines to
Is the EDCA merely an agreement implementing the 1998 VFA which already govern such visits of military personnel, and further defines the rights of the
allows the limited entry of U.S. military troops and the construction of facilities? United States and the Philippine government in the matter of criminal
The quick and short answer to the above question is — No, the EDCA does not jurisdiction, movement of vessel and aircraft, importation and exportation of
implement the 1998 VFA as the EDCA in fact provides a wider arrangement than equipment, materials and supplies.
the 1998 VFA with respect to the entry of military bases, troops, and facilities
into the Philippines. A naughty view is that the 1998 VFA should form part of the
EDCA and not the other way around. Another reality, based on the treaty- 618
executive agreement distinctions discussed above, is that the EDCA introduces 618 SUPREME COURT REPORTS ANNOTATED
new arrangements and obligations to those existing under the 1998 VFA;
hence, the EDCA should be in the form of a treaty. Saguisag vs. Ochoa, Jr.

V.D(3)(i) The 1998 Visiting Forces Agreement Considering that cooperation between the Republic of the Philippines
and the United States promotes their common security interests;
Recognizing the desirability of defining the treatment of United States
personnel visiting the Republic of the Philippines;
Have agreed as follows: On the whole, the VFA is an agreement which defines the treatment of
United States troops and personnel visiting the Philippines. It provides for
Article I: Definitions the guidelines to govern such visits of military personnel, and further
As used in this Agreement, “United States personnel” means United defines the rights of the United States and the Philippine government in
States military and civilian personnel temporarily in the Philippines in the matter of criminal jurisdiction, movement of vessel and aircraft,
connection with activities approved by the Philippine importation and exportation of equipment, materials and supplies.
Government. x x x
In Lim v. Executive Secretary,231 this Court further explained:
xxxx
Article III: Entry and Departure The VFA provides the “regulatory mechanism” by which “United
1. The Government of the Philippines shall facilitate the States military and civilian personnel [may visit] temporarily in
admission of United States personnel and their departure from the the Philippines in connection with activities approved by the Phil-
Philippines in connection with activities covered by this Agreement. _______________
xxx
230 Id.
As the ponencia correctly observed, the 1998 VFA itself does not specify what 231 Supra note 179 at p. 572; p. 752.
“activities” would allow the entry of U.S. troops into the Philippines. The parties
left this open and recognized that the activities that shall require the entry of U.S.
troops are subject to future agreements and the approval by the Philippine 620
Government.
620 SUPREME COURT REPORTS ANNOTATED
How this approval, however, will be secured is far from certain. What is certain
is that beyond the restrictive “visits” that the 1998 VFA mentions, nothing else is Saguisag vs. Ochoa, Jr.
said under the express terms of the Agreement.
Harking back to the 1947 MBA and its clear and certain terms, what comes ippine Government.” It contains provisions relative to entry and
out boldly is that the 1998 VFA is not an agreement that covers “activities” departure of American personnel, driving and vehicle registration, criminal
in the way that the 1947 MBA did; it is simply an agreement regulating the jurisdiction, claims, importation and exportation, movement of vessels and
status of and the treatment to be accorded to U.S. aircraft, as well as the duration of the agreement and its termination.
[Emphasis supplied]

619 The 1998 VFA allows the entry of U.S. military personnel to Philippine
VOL. 779, JANUARY 12, 2016 619 territory and grants the U.S. specific rights; it is essentially an agreement
governing the rules for the visit of “US armed forces in the Philippines from time
Saguisag vs. Ochoa, Jr. to time”232 in pursuit of cooperation to promote “common security interests”; it is
essentially a treaty governing the sojourn of US forces in this country for joint
armed forces personnel and their aircraft and vehicles while visiting exercises.233
the Philippines. The agreement itself does not authorize U.S. troops to Significantly, the 1951 MDT and the 1998 VFA contain a similar feature —
permanently stay in the Philippines nor authorize any activity related to the joint activities in pursuit of common security interests. The EDCA, on the other
establishment and the operation of bases, as these activities had been defined hand, goes beyond the terms of the 1951 MDT and the 1998 VFA.
under the 1947 MBA. As explained above, the EDCA has two purposes. First, it is an agreement for
As discussed under the treaty-executive agreement distinctions above, if the conduct of joint activities in accordance with the 1951 MDT and the 1998 VFA.
indeed the activities would be in line with the original intent of the 1998 VFA, then This, however, is not the centerpiece of the EDCA. Its centerpiece is the
an executive agreement would suffice as an implementing agreement. On the other introduction of agreed Locations which are portions of the Philippine
hand, if the activity would be a modification of the 1998 VFA or would be beyond territory whose use is granted to the U.S.234 The EDCA then proceeds to list
its terms and would entail the establishment of a military base or facility or their the rights that the U.S. has over the Agreed Locations.235
equivalent, and the introduction of troops, then, a treaty duly concurred in by the _______________
Senate would be the appropriate medium of the U.S.-Philippines agreement.
This Court has had the opportunity to examine the 1998 VFA in BAYAN230 and 232 1998 VFA, Preamble, par. 4.
described the agreement in this wise — 233 Lim v. Executive Secretary, supra note 179 at p. 575; p. 755. In this
manner, visiting US forces may sojourn in Philippine territory for purposes other
than military. As conceived, the joint exercises may include training on new
techniques of patrol and surveillance to protect the nations marine resources, sea These rights, granted to the U.S. under the EDCA, do not contain an
search-and-rescue operations to assist vessels in distress, disaster relief element of mutuality in the sense that mutuality is reflected in the 1951 MDT
operations, civic action projects such as the building of school houses, medical and and the 1998 VFA. As these rights go beyond the earlier treaties and are, in fact,
humanitarian missions, and the like. independent sources of rights and obligations between the U.S. and the
234 EDCA, Article II(4). Philippines, they cannot be mere details of implementation of both the 1951 MDT
235 Id., Article III(1). and the 1998 VFA.
And, as pointed out earlier, the Agreed Locations under the EDCA are akin to
the military bases contemplated under the 1947 MBA. Thus, by its own terms, the
621 EDCA is not only a military base agreement outside the provisions of the 1951
VOL. 779, JANUARY 12, 2016 621 NlDT and the 1998 VFA, but a piecemeal introduction of military bases in the
Philippines.
Saguisag vs. Ochoa, Jr. Note that, at this point, there exists no agreement on the establishment of U.S.
military bases in the Philippines; the EDCA reintroduces a modernized
A reading of the EDCA’s provisions shows that the rights and privileges version of the fixed military base concept contemplated and
granted to the U.S. do not always carry a concomitant right on the part of the operationalized under the 1947 MBA.
Philippines nor do they involve joint exercises. While the EDCA mentions that the
Agreed Locations may be used for “security cooperation exercises”236 and “joint and V.D(4) The 1951 MDT and 1998 VFA in con-
combined training activities,”237 the provisions of the EDCA also provide for the junction with the EDCA
conduct of other activities beyond the 1951 MDT and the 1998 VFA.
Within the Agreed Locations, the U.S. may conduct trainings for its troops, An additional dimension that the EDCA introduces — the treatment of U.S.
transit, support and related activities.238 The EDCA also allows the U.S. to use the forces and U.S. contractors — reveals that it does not merely implement the
Agreed Locations to refuel aircraft, bunker vessels, temporarily maintain vehicles, 1951 MDT and the 1998 VFA, but adds to the obligations in these agreements.
vessels and aircraft.239Significantly, it does not provide for any qualification on the To support its conclusion that the EDCA implements the provisions in the 1951
purpose for the entry of these vessels, vehicles, and aircraft into Philippine MDT and the 1998 VFA, the ponencia points out that the EDCA references 1951
jurisdiction. MDT and the 1998 VFA in allowing the entry of U.S. personnel and U.S. forces in
The EDCA also permits the temporary accommodation of personnel,240 again the Philippines, and that the entry of U.S. contractors (who had not been
without any qualification as to the purpose of their visit. The U.S. forces may also mentioned in the 1998 VFA) do not contradict the obligations found in the 1998
engage in communications activities including the use of its own radio VFA.
spectrum,241 similarly without any limitation as to the purpose by which such The ponencia further notes that the U.S. contractors had been expressly
communications shall be carried out. excluded from the definition of U.S. personnel and U.S. forces, in line with their
Further, within the Agreed Locations, the U.S. can also preposition defense definitions in the 1998
equipment, supplies, and materiel over which the U.S. forces shall have exclusive
use and control.242 Clearly, the right to deploy weapons can be undertaken even if it
is not in the pursuit of joint activities for common security interests. 623
_______________
VOL. 779, JANUARY 12, 2016 623
236 Id., Article I(3). Saguisag vs. Ochoa, Jr.
237 Id.
238 Id., Article III(1). VFA.243 They are not entitled to the same privileges that U.S. Personnel and
239 Id. U.S. forces enjoy under the 1998 VFA, but would have to comply with Philippine
240 Id. law to enter the Philippines.
241 Id., Article VII(2) The ponencia proceeds to argue that the lack of dissimilarities between the
242 Id., Article IV(1), (3). 1998 VFA and the EDCA point to the conclusion that the EDCA implements the
1998 VFA. By limiting the entry of persons under the EDCA to the categories
under the 1998 VFA, the EDCA merely implements what had already been agreed
622 upon under the 1998 VFA. The U.S. forces’s authorization to perform activities
under the EDCA does not change the nature of the EDCA as the 1998 VFA’s
622 SUPREME COURT REPORTS ANNOTATED
implementing agreement, as the term “joint exercises” under the 1998 VFA
Saguisag vs. Ochoa, Jr.
denotes a wide range of activities that include the additional activities under the characterization, should be decisive in determining whether Section 25, Article
EDCA. XVIII applies.
That the 1998 VFA and the EDCA are not dissimilar in terms of their Lastly, even assuming that the EDCA is an implementation of the 1951 MDT
treatment of U.S. forces and U.S. personnel, does not automatically mean that the and the 1998 VFA, the practice of the Government reveals that even when an
EDCA simply implements the 1998 VFA, given the additional obligations that the agreement is considered as an implementation of a prior treaty, the concurrence
EDCA introduces for the Philippine government. of the Senate must still be sought.
As earlier discussed, the EDCA introduces military bases in the Philippines Early in the Senate deliberations on the 1998 VFA, the senator-sponsors
within the concept of the 1987 Constitution, and it is in light of these additional characterized it merely as a subsidiary or
obligations that the EDCA’s affirmation of the 1998 VFA should be
viewed: the EDCA adds new dimensions to the treatment of U.S. Personnel
and U.S. forces provided in the 1998 VFA, and these dimensions cannot be 625
ignored in determining whether the EDCA merely implements the 1998 VOL. 779, JANUARY 12, 2016 625
VFA.
Thus, while the EDCA affirms the treatment of U.S. personnel and U.S. forces Saguisag vs. Ochoa, Jr.
in the Philippines, it at the same time introduces the Philippines’ obligation
to recognize the authority of U.S. Forces in the “Agreed Locations.” Under implementing agreement to the 1951 MDT.244 Nevertheless, Senator Tatad,
the EDCA, U.S. forces can now preposition and store defense equipment, one of the 1998 VFA’s co-sponsors, recognized that Article XVIII, Section 25 of the
supplies, and materiel at Agreed Locations. They Constitution prohibits the 1998 VFA from being executed as a mere executive
_______________ agreement,245 for which reason it was sent to the Senate for concurrence.
The senators agreed during the deliberations that an agreement implementing
243 Ponencia, pp. 385-388. the 1951 MDT requires Senate concurrence.246 This is because the agreement,
despite implementing or affirming the 1951 MDT, allows the entry of U.S. troops
in the Philippines, a matter covered by Article XVIII, Section 25 of the
624 Constitution.
Indeed, the 1998 VFA has been consistently treated as an implementation of
624 SUPREME COURT REPORTS ANNOTATED the 1951 MDT. Nevertheless, the Government correctly chose to enter into the
Saguisag vs. Ochoa, Jr. international agreement in the form of a treaty duly concurred in by the Senate,
because it involves the entry of foreign military troops independent of, and in
shall have unimpeded access to Agreed Locations for all matters relating to addition to, the general agreements in the 1951 MDT.
the prepositioning and storage of defense equipment, supplies, and materiel. In the same manner, the EDCA, which purportedly implements and
Lastly, the EDCA authorizes the U.S. forces to exercise all rights and complements both the 1951 MDT and the 1998 VFA, should have likewise been
authorities within the Agreed Locationsthat are necessary for their submitted to the Senate for its concurrence because of the new obligations it
operational control or defense. In contrast, the 1998 VFA only refers to the tax and introduces.
duty-free entry of U.S. Government equipment in connection with the activities _______________
during their visit.
In the same manner, and despite being in a different class as U.S. personnel 244 Id.
and U.S. forces, U.S. contractors are also allowed “unimpeded access” to the 245 Senate deliberations, May 25, 1999, AM, p. 17, which reads:
Agreed Locations when it comes to all matters relating to the prepositioning Senator Tatad. x x x Mr. President, distinguished colleagues, the Visiting
and storage of defense equipment, supplies and materiel. Forces Agreement does not create a new policy or a new relationship. It simply
Thus, these groups of people (U.S. personnel, U.S. forces and U.S. contractors) seeks to implement and reinforce what already exists.
have been referred to in the EDCA not merely to implement the 1998 VFA, but to For that purpose, an executive agreement might have sufficed, were
further their roles in the Agreed Locations that the EDCA authorizes. there no constitutional constraints. But the Constitution requires the
From these perspectives, the EDCA cannot be considered to be a simple Senate to concur in all international agreements. So the Senate must concur
implementation of the 1998 VFA. Rather, it is a continuation of the 1998 VFA in the Visiting Forces Agreement, even if the U.S. Constitution does not require
under new dimensions. These dimensions should not and cannot be hidden behind the U.S. Senate to give its advice and consent.
reaffirmations of existing 1998 VFA obligations. These added dimensions reinforce 246 Senate Resolution No. 1414, supra note 107.
the idea of military bases, as it allows them access to the Agreed Locations that,
as I had earlier mentioned, is the cornerstone of the EDCA. From the legal end, the
obligations under the EDCA, not its policy declarations and 626
626 SUPREME COURT REPORTS ANNOTATED without the consent of the Filipino people and against the constitutional standards
they set, if EDCA would be enforced without the benefit of Senate concurrence.
Saguisag vs. Ochoa, Jr. 2. Under the “pivot to Asia strategy,” the operative word is “presence” which
means ready access to equipment, supplies, and materiel by troops who can be
To reiterate, the EDCA allows for a more permanent presence of U.S. troops ferried from safer locations and immediately be brought to the scene of action from
and military equipment in the Philippines (akin to establishing a base), which was the Agreed Locations. The EDCA provides such presence through the Agreed
not contemplated under the 1998 VFA. Thus, despite having been treated as an Locations; the access to these secured locations; the prepositioning and storage of
implementation of the 1951 MDT and the 1998 VFA, the new obligations under defense (read as “military”) equipment, supplies, and materiel; and the forward
the EDCA calls for the application of Article XVIII, Section 25 of the Constitution jump-off point for the deployment of troops to whatever scene of action there may
and its submission to the Senate for concurrence. be that Philippine locations may serve best.
3. From the point of view of “troops” that Article XVIII, Section 25 likewise
V.E. The EDCA: the Actual and Operational View regulates through Senate concurrence, note that in the EDCA, contractual
employees are mentioned together or side-by-side with the military. This is a
As my last point, let me just say that the ponencia can engage in a lot of relatively recent development where contractual em-
rationalizations and technical distinctions on why the EDCA provisions do _______________
not amount to or equate with the operation of military bases and the introduction
of troops and facilities into the Philippines. The ponencia cannot escape the 248 Strengthening U.S. Global Defense Posture: Report to
conclusion that translated to actual operational reality: Congress, supra note 201.
1. The activities described in the EDCA are no different from the operation of 249 Id.
a military base in the 1947 sense, except that under the current U.S. strategy, a
fixed base in the 1947 sense is hardly ever established because the expenses and
administrative problems accompanying a fixed base can now be avoided. A military 628
“facility” can very well serve the same purposes as a fixed military base under
current technological advances in weaponry, transportation, and 628 SUPREME COURT REPORTS ANNOTATED
communications.247 The U.S. can achieve the same results at less expense and with Saguisag vs. Ochoa, Jr.
lesser problems if it would have guaranteed access to and control of speci-
_______________ ployees are used to provide the same services and serve hand in hand or as
replacement or to augment regular military forces. The U.S. has put these
247 During the latter part of the Cold War, the term “facilities” was frequently contractual employees to good use in various local theaters of conflict, notably in
substituted for the word “bases” to soften the negative political overtones normally Iraq, Afghanistan and Syria.250The U.S. has reportedly resorted to the use, not
associated with the basing of foreign troops in a sovereign country. In line with only of regular military forces, but of contractual employees who may provide the
this thinking, the Stockholm International Peace Research Institute uses the term same services as military forces and who can increase their numbers without
foreign military presence (FMP) to describe bases/facilities that house foreign alerting the U.S. public to the actual number of troops maintained.
troops in a sovereign state. See Krepinevich and Work, supra note 202.
VI. Conclusion and the Question of Remedy

627 Based on all the above considerations, I conclude that the EDCA, instead of
VOL. 779, JANUARY 12, 2016 627 being in implementation of the 1951 MDT and the 1998 VFA, is significantly
broader in scope than these two treaties, and effectively added to what the 1951
Saguisag vs. Ochoa, Jr. MDT and the 1998 VFA provide.
The EDCA is thus a new agreement that touches on military bases, troops, and
fied areas such as the Agreed Locations that the EDCA conveniently provides. facilities beyond the scope of the 1951 MDT and the 1998 VFA, and should be
FOSs or CSLs, as defined above, are expandable “warm facilities” maintained covered by a treaty pursuant to Article XVIII, Section 25 and Article VII, Section
with limited U.S. military support presenceand 21, both of the 1987 Constitution. Without the referral and concurrence by the
possibly prepositioned equipment.248 FOSs will support rotational rather than Senate, the EDCA is constitutionally deficient and, hence, cannot be enforced in
permanently stationed forces, and will be a focus for bilateral and regional training our country.
and for the deployment of troops and stored and prepositioned equipment, supplies, To remedy the deficiency, the best recourse RECOMMENDED TO THE
and materiel.249 COURT under the circumstances is for the Court to suspend the operations of
As has already been mentioned, examples include the Sembawang port facility its rules on the finality of its rulings and for the Court to give the President
in Singapore and Soto Cano Air Base in Honduras. The Philippines will soon follow
ninety (90) days from the service of its Decision, whether or not a motion 1 Heneral Luna, Dir. Jerrold Tarog, Artikulo Uno Productions (2015). The
for reconsideration is filed, inclusion of this quote is to emphasize its metaphor and not meant in any way to
_______________ denigrate the human dignity of commercial sex workers.

250 See Gomez del Prado, Jose, Privatization of War: Mercenaries, Private
Military and Security Companies, Global Research, November 8, 2010, available 630
at http://www.globalresearch.ca/the-privitazation-of-war-mercenaries-private- 630 SUPREME COURT REPORTS ANNOTATED
military-and-security-companies-pmsc/21826.
Saguisag vs. Ochoa, Jr.

629 America concerning Military Bases, foreign military bases, troops, or


facilities shall not be allowed in the Philippines except under a treaty duly
VOL. 779, JANUARY 12, 2016 629 concurred in by the Senate and, when the Congress so requires, ratified by
Saguisag vs. Ochoa, Jr. a majority of the votes cast by the people in a national referendum held for
that purpose, and recognized as a treaty by the other contracting State.
the OPTION to refer the EDCA to the Senate for its consideration and
concurrence. In a disturbing turn of events, the majority of this court just succeeded in
The referral to the Senate shall serve as a main or supplemental motion for amending this constitutional provision. At the very least, it emasculated its text
reconsideration that addresses the deficiency, rendering the effects of the Court’s and weakened its spirit.
Decision moot and academic. Otherwise, the conclusion that the President An agreement signed by our Secretary of Defense and the Ambassador of the
committed grave abuse of discretion by entering into an executive agreement United States that grants United States military personnel and their contractors
instead of a treaty, and by certifying to the completeness of Philippine internal operational control over unspecified locations within Philippine territory in order
process, shall be fully effective. to pre-position military equipment as well as to use as launching pads for
As my last point, we must not forget that the disputed executive operations in various parts of the globe is not binding until it is concurred in by
agreement that the President entered into is with the Americans from the Senate. This is in accordance with Article XVIII, Section 25 and Article VII,
whom we trace the roots of our present Constitution. The Americans are a Section 21 of the Constitution.
people who place the highest value in their respect for their Constitution. Furthermore, the Enhanced Defense Cooperation Agreement (EDCA) does not
This should be no less than the spirit that should move us in adhering to simply implement the Agreement Between the Government of the United States
our own Constitution. To accord a lesser respect for our own Constitution of America and the Government of the Republic of the Philippines Regarding the
is to invite America’s disrespect for the Philippines as a coequal sovereign Treatment of United States Armed Forces Visiting the Philippines (Visiting Forces
and independent nation. Agreement or VFA). The EDCA substantially modifies or amends the VFA. An
executive agreement cannot amend a treaty. Nor can any executive agreement
DISSENTING OPINION amend any statute, most especially a constitutional provision.
The EDCA substantially modifies or amends the VFA in the following aspects:
“Para kayong mga birhen na naniniwala sa pag-ibig ng isang puta!”1 First, the EDCA does not only regulate the “visits” of foreign troops. It also
– Heneral Luna kina Pedro Paterno, Felix Buencamino, at allows the temporary stationing on a rotational basis of US military personnel and
Emilio Aguinaldo noong sinabi nila na nangako ang mga Amerikano their contractors in physical locations with permanent facilities and pre-positioned
na kikilalanin nila ang kasarinlan ng mga Pilipino military materiel.

LEONEN, J.:
631
1987 Constitution, Article XVIII, Section 25: VOL. 779, JANUARY 12, 2016 631

After the expiration in 1991 of the Agreement between the Republic of the Saguisag vs. Ochoa, Jr.
Philippines and the United States of
_______________ Second, unlike the VFA, the EDCA allows pre-positioning of military materiel,
which can include various types of warships, fighter planes, bombers, and vessels,
as well as land and amphibious vehicles and their corresponding ammunition.
Third, the VFA contemplates the entry of troops for various training exercises.
The EDCA allows our territory to be used by the United States to launch military
and paramilitary operations to be conducted within our territory or against targets On October 5, 1998, President Joseph E. Estrada, through respondent
in other states. Secretary of Foreign Affairs, ratified the VFA.
Fourth, the EDCA introduces the following concepts not contemplated in the On October 6, 1998, the President, acting through respondent
VFA or in the 1951 Mutual Defense Treaty, namely: (a) agreed locations; (b) Executive Secretary Ronalda Zamora, officially transmitted to the Senate
contractors; (c) pre-positioning of military materiel; and (d) operational control. of the Philippines, the Instrument of Ratification, the letter of the President
Lastly, the VFA does not have provisions that may be construed as a restriction and the VFA, for concurrence pursuant to Section 21, Article VII of the 1987
or modification of obligations found in existing statutes. The EDCA contains Constitution. The Senate, in turn, referred the VFA to its Committee on
provisions that may affect various statutes, including (a) the jurisdiction of courts, Foreign Relations, chaired by Senator Blas F. Ople, and its Committee on
(b) local autonomy, and (c) taxation. National Defense and Security, chaired by Senator Rodolfo G. Biazon, for
There is no showing that the new matters covered in the EDCA were their joint consideration and rec-
contemplated by the Senate when it approved the VFA. Senate Resolution No. 105, _______________
Series of 2015, which expresses the sentiment of that legislative chamber, is a
definite and unequivocal articulation of the Senate: the VFA was not intended to 2 Bayan (Bagong Alyansang Makabayan) v. Zamora, 396 Phil. 623; 342 SCRA
cover the matters now included in the EDCA. In the view of the Senate reading 449 (2000) [Per J. Buena, En Banc].
the same provisions of the Constitution as we do, the EDCA should be in treaty
form.
The EDCA, in its current form, is only an official and formal memorial of 633
agreed provisions resulting from the negotiations with the United States. The
VOL. 779, JANUARY 12, 2016 633
President has the discretion to submit the agreement to the Senate for
concurrence. The EDCA is a treaty and requires Senate concurrence. Saguisag vs. Ochoa, Jr.

I ommendation. Thereafter, joint public hearings were held by the two


Committees.
The EDCA should comply with Article XVIII, Section 25 of the Constitution. On May 3, 1999, the Committees submitted Proposed Senate
Resolution No. 443 recommending the concurrence of the Senate to the VFA
and the creation of a Legislative Oversight Committee to oversee its
632 implementation. Debates then ensued.
632 SUPREME COURT REPORTS ANNOTATED On May 27, 1999, Proposed Senate Resolution No. 443 was approved by
the Senate, by a two-thirds (2/3) vote of its members. Senate Resolution No.
Saguisag vs. Ochoa, Jr. 443 was then renumbered as Senate Resolution No. 18.
On June 1, 1999, the VFA officially entered into force after an Exchange
BAYAN v. Zamora2 interpreted the scope of this provision when it discussed of Notes between respondent Secretary Siazon and United States
the constitutionality of the VFA. Similar to the EDCA, the VFA was a product of Ambassador Hubbard.3 (Citations omitted)
negotiations between the two governments relating to mutual security interests.
Unlike the EDCA, however, the VFA was submitted to the Senate for concurrence, BAYAN held that Article XVIII, Section 25 of the Constitution applies to the
thus: VFA:

On July 18, 1997, the United States panel, headed by US Defense Section 25, Article XVIII disallows foreign military bases, troops, or
Deputy Assistant Secretary for Asia Pacific Kurt Campbell, met with the facilities in the country, unless the following conditions are sufficiently
Philippine panel, headed by Foreign Affairs Undersecretary Rodolfo met, viz.: (a) it must be under a treaty; (b) the treaty must be duly
Severino, Jr., to exchange notes on “the complementing strategic interests concurred in by the Senate and, when so required by Congress, ratified by
of the United States and the Philippines in the Asia-Pacific region.” Both a majority of the votes cast by the people in a national referendum; and (c)
sides discussed, among other things, the possible elements of the Visiting recognized as a treaty by the other contracting state.
Forces Agreement (VFA for brevity). Negotiations by both panels on the There is no dispute as to the presence of the first two requisites in the
VFA led to a consolidated draft text, which in turn resulted [in] a final case of the VFA. The concurrence handed by the Senate through Resolution
series of conferences and negotiations that culminated in Manila on No. 18 is in accordance with the provisions of the Constitution, whether
January 12 and 13, 1998. Thereafter, then President Fidel V. Ramos under the general requirement in Section 21, Article VII, or the specific
approved the VFA, which was respectively signed by public respondent mandate mentioned in Section 25, Article XVIII, the provision in the latter
Secretary Siazon and Unites States Ambassador Thomas Hubbard on
February 10, 1998.
article requiring ratification by a majority of the votes cast in a national
referendum being unnecessary since Congress has not required it. of America in this case, to submit the VFA to the United States Senate
_______________ for concurrence pursuant to its Constitution, is to accord strict meaning to
the phrase.4
3 Id., at pp. 632-637; pp. 464-469.
Lim v. Executive Secretary5 further explored the scope of the VFA as it dealt
with the constitutionality of the Terms of Reference of the “Balikatan 02-1” joint
634 military exercises between the Philippines and the United States:
634 SUPREME COURT REPORTS ANNOTATED
The Terms of Reference rightly fall within the context of the VFA.
Saguisag vs. Ochoa, Jr. After studied reflection, it appeared farfetched that the ambiguity
surrounding the meaning of the word “activities” arose from accident. In
As to the matter of voting, Section 21, Article VII particularly requires our view, it was deliberately made that way to give both parties a certain
that a treaty or international agreement, to be valid and effective, must be leeway in negotiation. In this manner, visiting US forces may sojourn in
concurred in by at least two-thirds of all the members of the Senate. On the Philippine territory for purposes other than military. As conceived, the joint
other hand, Section 25, Article XVIII simply provides that the treaty be exercises may include training on new techniques of patrol and surveillance
“duly concurred in by the Senate.” to protect the nation’s marine resources, sea search-and-rescue operations
Applying the foregoing constitutional provisions, a two-thirds vote of all to assist vessels in distress, disaster relief operations, civic action projects
the members of the Senate is clearly required so that the concurrence such as the building of school houses, medical and humanitarian missions,
contemplated by law may be validly obtained and deemed present. While it and the like.
is true that Section 25, Article XVIII requires, among other things, that the Under these auspices, the VFA gives legitimacy to the
treaty — the VFA, in the instant case — be “duly concurred in by the current Balikatan exercises. It is only logical to assume that “Balikatan 02-
Senate,” it is very true however that said provision must be related and 1,” a “mutual anti-terrorism advising, assisting and training exercise,” falls
viewed in light of the clear mandate embodied in Section 21, Article VII, under the umbrella of sanctioned or allowable activities in the context of
which in more specific terms, requires that the concurrence of a treaty, or the agreement. Both the history and intent of the Mutual Defense Treaty
international agreement, be made by a two-thirds vote of all the members and the VFA support the conclusion that combat-related activities — as
of the Senate. Indeed, Section 25, Article XVIII must not be treated in opposed to combat itself — such as the one subject of the instant petition,
isolation to Section 21, Article VII. are indeed authorized.
As noted, the “concurrence requirement” under Section 25, Article _______________
XVIII must be construed in relation to the provisions of Section 21, Article
VII. In a more particular language, the concurrence of the Senate 4 Id., at pp. 654-657; pp. 486-488.
contemplated under Section 25, Article XVIII means that at least two- 5 430 Phil. 555; 380 SCRA 739 (2002) [Per J. De Leon, Jr., En Banc].
thirds of all the members of the Senate favorably vote to concur with the
treaty — the VFA in the instant case.
.... 636
Having resolved that the first two requisites prescribed in Section 25,
636 SUPREME COURT REPORTS ANNOTATED
Article XVIII are present, we shall now pass upon and delve on the
requirement that the VFA should be recognized as a treaty by the United Saguisag vs. Ochoa, Jr.
States of America.
.... That is not the end of the matter, though. Granted that “Balikatan 02-
This Court is of the firm view that the phrase “recognized as a treaty” 1” is permitted under the terms of the VFA, what may US forces
means that the other contracting party accepts or acknowledges the legitimately do in furtherance of their aim to provide advice, assistance and
agreement as a treaty. To require the other contracting state, the United training in the global effort against terrorism? Differently phrased, may
States American troops actually engage in combat in Philippine territory? The
Terms of Reference are explicit enough. Paragraph 8 of Section I stipulates
that US exercise participants may not engage in combat “except in self-
635 defense.” We wryly note that this sentiment is admirable in the abstract
VOL. 779, JANUARY 12, 2016 635 but difficult in implementation. The target of “Balikatan 02-1,” the Abu
Sayyaf, cannot reasonably be expected to sit idly while the battle is brought
Saguisag vs. Ochoa, Jr.
to their very doorstep. They cannot be expected to pick and choose their law as part of the law of the land and adheres to the policy of peace,
targets for they will not have the luxury of doing so. We state this point if equality, justice, freedom, cooperation, and amity with all nations.
only to signify our awareness that the parties straddle a fine line, observing ....
the honored legal maxim “Nemo potest facere per alium quod non potest Section 7. The State shall pursue an independent foreign policy. In
facere per directum.” The indirect violation is actually petitioners’ worry, its relations with other states the paramount consideration shall be
that in reality, “Balikatan 02-1” is actually a war principally conducted by national sovereignty, territorial integrity, national interest, and the right
the United States government, and that the provision on self-defense serves to self-determination.
only as camouflage to conceal the true nature of the exercise. A clear
pronouncement on this matter thereby becomes crucial. Article 2(4) of the Charter of the United Nations similarly provides that “[a]ll
In our considered opinion, neither the MDT nor the VFA allow foreign Members shall refrain in their international relations from the threat or use of
troops to engage in an offensive war on Philippine territory.6 (Emphasis force against the territorial integrity or political independence of any state, or
supplied) _______________

Nicolas v. Romulo7 involved the grant of custody of Lance Corporal Daniel 8 Id., at pp. 284-285; p. 461.
Smith to the United States pursuant to the VFA and reiterated the ruling
in Bayan:
638
[A]s an implementing agreement of the RP-US Mutual Defense Treaty,
638 SUPREME COURT REPORTS ANNOTATED
it was not necessary to submit the VFA
_______________ Saguisag vs. Ochoa, Jr.

6 Id., at pp. 575-576; pp. 755-756. “Nemo palest facere per alium quod non in any other manner inconsistent with the Purposes of the United Nations.”9
palest facere per directum” translates to “No one is allowed to do indirectly what Our use of force is not completely proscribed as the Charter of the United
he is prohibited to do directly.” Nations provides for the inherent right of individual or collective self-defense:
7 598 Phil. 262; 578 SCRA 438 (2009) [Per J. Azcuna, En Banc].
CHAPTER VII: ACTION WITH RESPECT TO THREATS TO THE
PEACE, BREACHES OF THE PEACE, AND ACTS OF AGGRESSION
637
. . . .
VOL. 779, JANUARY 12, 2016 637
Saguisag vs. Ochoa, Jr. Article 51. Nothing in the present Charter shall impair the inherent
right of individual or collective self-defen[s]e if an armed attack occurs
to the US Senate for advice and consent, but merely to the US Congress under against a Member of the United Nations, until the Security Council has
the Case-Zablocki Act within 60 days of its ratification. It is for this reason that taken measures necessary to maintain international peace and security.
the US has certified that it recognizes the VFA as a binding international Measures taken by Members in the exercise of this right of self-defen[s]e
agreement, i.e., a treaty, and this substantially complies with the requirements of shall be immediately reported to the Security Council and shall not in any
Art. XVIII, Sec. 25 of our Constitution.8 way affect the authority and responsibility of the Security Council under
The controversy now before us involves more than the VFA. Reading the the present Charter to take at any time such action as it deems necessary
entirety of the Constitution is necessary to fully appreciate the context of the in order to maintain or restore international peace and security.10
interpretation of Article XVIII, Section 25.
Furthermore, falling within the penumbra on the use of force are preemptive
II self-defense,11 self-help, and humanitarian interventions.12
_______________
Foreign policy indeed includes security alliances and defense cooperation
among states. In the conduct of negotiations and in the implementation of any 9 Charter of United Nations, Chapter I, Art. 2(4)
valid and binding international agreement, Article II of the Constitution requires: <http://www.un.org/en/documents/charter/chapter1.shtml> (visited January 11,
2016).
Section 2. The Philippines renounces war as an instrument of 10 Charter of United Nations, Chapter VII, Art. 51
national policy, adopts the generally accepted principles of international <http://www.un.org/en/documents/charter/chapter7.shtml> (visited January 11,
2016). See Military and Paramilitary Activities in and Against Nicaragua 640 SUPREME COURT REPORTS ANNOTATED
(Nicaragua v. United States of America), I.C.J. 1984 I.C.J. 39
11 See Anthony Clark Arend, International Law and the Preemptive Use of Saguisag vs. Ochoa, Jr.
Military Force, THE WASHINGTON QUARTERLY 26:2, 89-103
(2003). See Higgins, Rosalyn, Problems and Process: International Law and How Generally, the President’s discretion is plenary in matters falling within
We Use It, pp. 242-243 (1994), citing US executive functions. He is the chief executive,15 having the power of control over
all executive departments, bureaus, and offices.16 Further, “by constitutional fiat
and by the intrinsic nature of his office, the President, as head of State, is the sole
639 organ and authority in the external affairs of the country [and] [i]n many ways,
the President is the chief architect of the nation’s foreign policy.”17
VOL. 779, JANUARY 12, 2016 639
The President is also the Commander-in-Chief of all armed forces of the
Saguisag vs. Ochoa, Jr. Philippines.18 He has the power to “call out such armed forces to prevent or
Another exception would be the collective security system set up under the suppress lawless violence, invasion or rebellion . . . suspend the privilege of the
Charter of the United Nations, with the Security Council acting in accordance with writ of habeas corpus or place the Philippines or any part thereof under martial
Chapter VII of the Charter. Under Article 42: law”19 subject to the conditions and requisites under the provision.
Should the Security Council consider that measures provided for in Article 41 However, the President’s discretion to allow our participation in the use of force
would be inadequate or have proved to be inadequate, it may take such action by — whether by committing our own military assets and personnel or by allowing
air, sea, or land forces as may be necessary to maintain or restore international our territory to be used as waypoints, refueling or staging areas — is also
peace and security. Such action may include demonstrations, blockade, and other constrained by the Constitution. In this sense, the power of the President as
operations by air, sea, or land forces of Members of the United Nations.13 Commander-in-Chief and head of state is limited by the sovereign through
_______________ judicially determinable constitutional parameters.
_______________
Secretary of State Webster in his diplomatic note in the 1842 Caroline
Case. According to Professor Higgins, under customary international law, 11, 2016). See also Enforcement action through regional arrangements under
preemptive self-defense may be resorted to when the necessity is “instant, Articles 52(1) and 53(1) of the United Nations Charter.
overwhelming, and leav[es] no choice of means, and no moment for deliberation.” <http://www.un.org/en/sections/un-charter/chapter-viii/index.html> (visited
12 See Higgins, id., at pp. 245-248 (1994). See Keynote address by Jacques January 11, 2016).
Forster, Vice President of the International Committee of the Red Cross, presented 15 Const., Art. VII, Sec. 1.
at the Ninth Annual Seminar on International Humanitarian Law for Diplomats 16 Const., Art. VII, Sec. 17.
accredited to the United Nations, Geneva, 8-9 March 2000 17 Supra note 2 at p. 663; p. 494.
<https://www.icrc.org/eng/resources/documents/misc/57jqjk.htm> (visited January 18 Const., Art. VII, Sec. 18.
11, 2016): “The use of force by the international community should come within 19 Id.
the scope of the United Nations Charter. International humanitarian law cannot
be invoked to justify armed intervention because it has nothing to do with the right
of States to use force. Its role is strictly limited to setting limits to armed force 641
irrespective of the legitimacy of its use.” See also United Nations Security Council VOL. 779, JANUARY 12, 2016 641
Resolution 1674 (2006) on the concept of Responsibility to Protect
<http://www.un.org/en/ga/search/view_doc.asp?symbol=S/RES/1674(2006)> Saguisag vs. Ochoa, Jr.
(visited January 11, 2016).
13 Charter of United Nations, Chapter VII, Art. 42 III
<http://www.un.org/en/sections/charter/chapter7.shtml> (visited January 11,
2016). With respect to the use of or threat to use force, we can discern a gradation of
We fall within this exception when we participate in the enforcement of the interrelations of the legislative and executive powers to ensure that we pursue “an
resolutions of the Security Council.14 independent foreign policy” in the context of our history.
14 See Charter of United Nations, Chapter VII, Art. 44 Article VI, Section 23 of the Constitution covers declarations of a state of war.
<http://www.un.org/en/documents/charter/chapter7.shtml> (visited January It is vested solely in Congress, thus:

Section 23. (1) The Congress, by a vote of two-thirds of both Houses


640 in joint session assembled, voting separately, shall have the sole power to
declare the existence of a state of war.
(2) In times of war or other national emergency, the Congress may, by national referendum; and (c) recognized as a treaty by the other
law, authorize the President, for a limited period and subject to such contracting state.20(Emphasis supplied)
restrictions as it may prescribe, to exercise powers necessary and proper to
carry out a declared national policy. Unless sooner withdrawn by resolution “Foreign military bases, troops, and facilities” should not be read together but
of the Congress, such powers shall cease upon the next adjournment separately. Again, in BAYAN:
thereof. _______________

Informed by our history and to ensure that the independence of our foreign 20 Supra note 2 at pp. 651-655; pp. 482-486.
policy is not compromised by the presence of foreign bases, troops, or facilities, the
Constitution now provides for treaty recognition, Senate concurrence, and public
ratification when required by Congress through Article XVIII, Section 25, thus: 643
VOL. 779, JANUARY 12, 2016 643
Section 25. After the expiration in 1991 of the Agreement between
the Republic of the Philippines and the United States of America Saguisag vs. Ochoa, Jr.
concerning Military Bases, foreign military bases, troops, or facilities shall
not be allowed in the Philippines except under a treaty duly concurred in Moreover, it is specious to argue that Section 25, Article XVIII is
by the Senate and, when the Congress so requires, ratified by a majority of inapplicable to mere transient agreements for the reason that there is no
the votes cast by the people in a national referendum held for that purpose, permanent placing of structure for the establishment of a military base. On
and recognized as a treaty by the other contracting State. this score, the Constitution makes no distinction between “transient” and
“permanent.” Certainly, we find nothing in Section 25, Article XVIII that
requires foreign troops or facilities to be stationed or placed permanently
642 in the Philippines.
642 SUPREME COURT REPORTS ANNOTATED It is a rudiment in legal hermeneutics that when no distinction is made
by law, the Court should not distinguish — Ubi lex non distinguit nec nos
Saguisag vs. Ochoa, Jr. distinguire debemos.
In like manner, we do not subscribe to the argument that Section 25,
The prohibition in Article XVIII, Section 25 relates only to international Article XVIII is not controlling since no foreign military bases, but merely
agreements involving foreign military bases, troops, or facilities. It does not foreign troops and facilities, are involved in the VFA. Notably, a perusal of
prohibit the President from entering into other types of agreements that relate to said constitutional provision reveals that the proscription covers “foreign
other aspects of his powers as Commander-in-Chief. military bases, troops, or facilities.” Stated differently, this prohibition is
In BAYAN: not limited to the entry of troops or facilities without any foreign bases
being established. The clause does not refer to “foreign military bases,
Section 25, Article XVIII is a special provision that applies to troops, or facilities” collectively but treats them as separate and
treaties which involve the presence of foreign military bases, troops independent subjects. The use of comma and the disjunctive word “or”
or facilities in the Philippines. Under this provision, the concurrence of clearly signifies disassociation and independence of one thing from the
the Senate is only one of the requisites to render compliance with the others included in the enumeration, such that, the provision contemplates
constitutional requirements and to consider the agreement binding on the three different situations — a military treaty the subject of which could be
Philippines. Section 25, Article XVIII further requires that “foreign either (a) foreign bases, (b) foreign troops, or (c) foreign facilities — any of
military bases, troops, or facilities” may be allowed in the the three standing alone places it under the coverage of Section 25, Article
Philippines only by virtue of a treaty duly concurred in by the XVIII.
Senate, ratified by a majority of the votes cast in a national To this end, the intention of the framers of the Charter, as manifested
referendum held for that purpose if so required by Congress, and during the deliberations of the 1986 Constitutional Commission, is
recognized as such by the other contracting state. consistent with this interpretation:
.... “MR. MAAMBONG. I just want to address a question or two
Section 25, Article XVIII disallows foreign military bases, troops, to Commissoner Bernas.
or facilities in the country, unless the following conditions are
sufficiently met, viz.: (a) it must be under a treaty; (b) the treaty must
be duly concurred in by the Senate and, when so required by 644
Congress, ratified by a majority of the votes cast by the people in a
644 SUPREME COURT REPORTS ANNOTATED
Saguisag vs. Ochoa, Jr. distinction between their initial entry and subsequent activities. Its very structure
shows that Article XVIII, Section 25 is not a mere gateway for the entry of foreign
This formulation speaks of three things: foreign military bases, troops or facilities into the Philippines for them to carry out any activity later on.
troops or facilities. My first question is: If the country does enter The provision contains measures designed to protect our country in the broader
into such kind of a treaty, must it cover the three — bases, troops or scheme of international relations. Military presence shapes both foreign policy and
facilities — or could the treaty entered into cover only one or two? political relations. War — or the threat thereof through the position of troops,
FR. BERNAS. Definitely, it can cover only one. Whether it basing, and provision of military facilities — is an extension of politic, thus:
covers only one or it covers three, the requirements will be the same. The use of military force is a means to a higher end — the political
MR. MAAMBONG. In other words, the Philippine government object. War is a tool that policy uses to achieve its objectives and, as such,
can enter into a treaty covering not bases but merely troops? has a measure of rational utility. So, the purpose for which the use of force
FR. BERNAS. Yes. is intended will be the major determinant of the course and character of a
MR. MAAMBONG. I cannot find any reason why the war. As Clausewitz explains, war “is controlled by its political object,”
government can enter into a treaty covering only troops. which will set its course,
FR. BERNAS. Why not? Probably if we stretch our _______________
imagination a little bit more, we will find some. We just want to
cover everything.” 21 Id., at pp. 653-655; pp. 484-486.
Moreover, military bases established within the territory of another 22 Ponencia, p. 353.
state is no longer viable because of the alternatives offered by new means 23 Id., at p. 354.
and weapons of warfare such as nuclear weapons, guided missiles as well
as huge sea vessels that can stay afloat in the sea even for months and
years without returning to their home country. These military warships are 646
actually used as substitutes for a land-home base not only of military 646 SUPREME COURT REPORTS ANNOTATED
aircraft but also of military personnel and facilities. Besides, vessels are
Saguisag vs. Ochoa, Jr.
mobile as compared to a land-based military headquarters.
At this juncture, we shall then resolve the issue of whether or not the
prescribe the scale of means and effort which is required, and makes its
requirements of Section 25 were complied with when the Senate gave its
influence felt throughout down to the smallest operational detail.24
concurrence to the VFA.
Section 25, Article XVIII disallows foreign military bases, troops, or
With respect to the entry and presence of foreign military bases, troops, and
facilities in the country, unless the following conditions are sufficiently
facilities, Article XVIII, Section 25 of the 1987 Constitution enables government to
met, viz.: (a) it must be
politically negotiate with other states from a position of equality. The authority is
not exclusively granted to the President. It is shared with the Congress. The
Senate participates because no foreign base, troop, or facility may enter unless it
645
is authorized by a treaty.
VOL. 779, JANUARY 12, 2016 645 There is more evidence in the text of the provision of a sovereign intent to
Saguisag vs. Ochoa, Jr. require conscious, deliberate, and public discussion regarding these issues.
The provision gives Congress, consisting of the Senate and the House of
under a treaty; (b) the treaty must be duly concurred in by the Senate Representatives, the option to require that the treaty become effective only when
and, when so required by congress, ratified by a majority of the votes cast approved by a majority of the people in a referendum. Furthermore, there is the
by the people in a national referendum; and (c) recognized as a treaty by additional requirement that the authority will be absent if the other state does not
the other contracting state.21 (Citations omitted) treat the same instrument that allows their bases, troops, and facilities to enter
our territory as a treaty.
The ponencia, among others, interprets “shall not be allowed” as being limited The provision ensures equality by requiring a higher level of public scrutiny.
to the “initial entry” of bases, troops, or facilities.22Subsequent acts are treated as Unlike in the past when we bargained with the United States from a position of
no longer being subject to Article XVIII, Section 25 and are, therefore, only limited weakness, the Constitution opens the legislative forum so that we use the freedoms
by other constitutional provisions and relevant laws.23 that we have won since 1946 to ensure a fair agreement. Legislative hearings make
This interpretation is specious and ahistorical. the agreements more publicly legible. They allow more criticism to be addressed.
There is nothing in Article XVIII, Section 25 that defines the extent and scope Public forums clarify to the United States and other foreign military powers
of the presence of foreign military bases, troops, or facilities, thereby justifying a interested in the Philippines the full extent of interest and
_______________ United States Senate. In the eyes of Philippine law, therefore, the
Military Bases Agreement was a treaty, but by the laws of the United
24 Thomas Waldman, Politics and War: Clausewitz’s Paradoxical Equation, States, it was a mere executive agreement. This asymmetry in the
AUTUMN 2 (2010) legal treatment of the Military Bases Agreement by the two countries
<http://strategicstudiesinstitute.army.mil/pubs/parameters/Articles/201Oautumn was believed to be a slur to our sovereignty. Thus, in the debate among
/Waldman.pdf> (visited January 11, 2016). the Constitutional Commissioners, the unmistakable intention of the
commission emerged that this anomalous asymmetry must never be
repeated. To correct this historical aberration, Sec. 25, Art. XVIII of
647 the Constitution requires that the treaty allowing the presence of
VOL. 779, JANUARY 12, 2016 647 foreign military bases, troops, and facilities should also be
“recognized as a treaty by the other contacting party.” In plain
Saguisag vs. Ochoa, Jr. language, recognition of the United States as the other contracting party of
the VFA should be by the US President with the advice and consent of
the various standpoints of our different constituents. As a mechanism of public the US Senate.
participation, it also assures our treaty partners of the durability of the various The following exchanges manifest this intention:
obligations in these types of security arrangements. “MR. OPLE. Will either of the two gentlemen yield to just one
The EDCA was negotiated in private between representatives of the President question for clarification? Is there anything in this formulation,
and the United States. The complete text of the negotiations was presented to the whether that of Commissioner Bernas or of Commissioner Romulo,
public in time for the visit of the President of the United States. During its that will prevent the Philippine government from abrogating the
presentation, the President’s representatives took the position that no further existing bases agreement?
public discussion would be held that might affect the terms of the EDCA. The FR. BERNAS. To my understanding, none.
President presented the EDCA as a final product withdrawn from Senate or MR. ROMULO. I concur with Commissioner Bernas.
Congressional input. The President curtailed even the possibility of full public MR. OPLE. I was very keen to put this question because I had taken
participation through a Congressional Resolution calling for a referendum on this the position from the beginning — and this is embodied in a resolution filed
matter. by Commissioners Natividad, Maambong and Regalado — that it is very
The Separate Opinion of former Chief Justice Puno in BAYAN provides a important that the government of the Republic of the Philippines be in a
picture of how the Constitutional Commission recognized the lopsided relationship position to terminate or abrogate the bases agreement as one of the options
of the United States and the Philippines despite the 1951 Mutual Defense Treaty . . . . we have acknowledged starting at the committee level that the bases
and the 1947 Agreement Between the United States of America and the Republic agreement was ratified by our Senate; it is a treaty under Philippine law.
of the Philippines Concerning Military Bases (1947 Military Bases Agreement): But as far as the Americans are con-
To determine compliance of the VFA with the requirements of Sec. 25,
Art. XVIII of the Constitution, it is necessary to ascertain the intent of
the framers of the Constitution as well as the will of the Filipino 649
people who ratified the fundamental law. This exercise would VOL. 779, JANUARY 12, 2016 649
inevitably take us back to the period in our history when U.S.
military presence was entrenched in Philippine territory with the Saguisag vs. Ochoa, Jr.
establishment and operation of U.S. Military Bases in several parts
of the archipelago under the 1947 R.P.-U.S. Military Bases cerned, the Senate never took cognizance of this and therefore, it is an
Agreement. As articulated by Constitutional Commissioner Blas F. Ople executive agreement. That creates a wholly unacceptable asymmetry
in the 1986 Constitutional Commission deliberations on this provision, the between the two countries. Therefore, in my opinion, the right step to take,
1947 RP-US Military Bases Agreement was ratified by the Philippine if the government of our country will deem it in the national interest to
Senate, but not by the terminate this agreement or even to renegotiate it, is that we must begin
with a clean slate; we should not be burdened by the flaws of the 1947
Military Bases Agreement. . .
648 MR. ROMULO. Madam President, I think the two phrases in the
648 SUPREME COURT REPORTS ANNOTATED Bernas formulation take care of Commissioner Ople’s concerns.
The first says ‘EXCEPT UNDER THE TERMS OF A TREATY.’
Saguisag vs. Ochoa, Jr. That means that if it is to be renegotiated, it must be under the
terms of a new treaty. The second is the concluding phrase which
says: ‘AND RECOGNIZED AS A TREATY BY THE OTHER
CONTRACTING STATE.’
.... 651
MR. SUAREZ. Is the proposal prospective and not retroactive in VOL. 779, JANUARY 12, 2016 651
character?
FR. BERNAS. Yes, it is prospective because it does not touch the Saguisag vs. Ochoa, Jr.
validity of the present agreement. However, if a decision should be
arrived at that the present agreement is invalid, then even prior to ens defense cooperation”26 between the Philippines and the United States.
1991, this becomes operative right away. However, like the 1947 Military Bases Agreement, it is the agreement more than
MR. SUAREZ. In other words, we do not impress the previous any other that will extensively shape our foreign policy.
agreements with a valid character, neither do we say that they are
null and void ab initio as claimed by many of us here. IV
FR. BERNAS. The position I hold is that it is not the function of
this Commission to pass judgment on the validity or invalidity of Article VII, Section 21 of the Constitution complements Article XVIII, Section
the subsisting agreement. 25 as it provides for the requisite Senate concurrence, thus:

Section 21. No treaty or international agreement shall be valid and


650 effective unless concurred in by at least two-thirds of all the Members of
the Senate.
650 SUPREME COURT REPORTS ANNOTATED
Saguisag vs. Ochoa, Jr. The provision covers both “treaty and international agreement.” Treaties are
traditionally understood as international agreements entered into between states
MR. SUAREZ . . . the proposal requires recognition of this treaty or by states with international organizations with international legal
by the other contracting nation. How would that recognition be personalities.27 The deliberate inclusion of the term “international agreement” is
expressed by that other contracting nation? That is in accordance the subject of a number of academic discussions pertaining to foreign relations and
with their constitutional or legislative process, I assume. international law. Its addition cannot be mere surplus. Certainly, Senate
FR. BERNAS. As Commissioner Romulo indicated, since this concurrence should cover more than treaties.
certainly would refer only to the United States, because it is only That the President may enter into international agreements as chief architect
the United States that would have the possibility of being allowed of the Philippines’ foreign policy has long been acknowledged.28 However, whether
to have treaties here, then we would have to require that the Senate an international
of the United States concur in the treaty because under American _______________
constitutional law, there must be concurrence on the part of the
Senate of the United States to conclude treaties. 26 Agreement between the Government of the Philippines and the
.... Government of the United States of America on Enhanced Defense Cooperation
FR. BERNAS. When I say that the other contracting state must (2014), Art. 1, Sec. 1.
recognize it as a treaty, by that I mean it must perform all the acts 27 See Vienna Convention on the Law of Treaties (1969), Art. 2(1)(a) and
required for the agreement to reach the status of a treaty under their Vienna Convention on the Law of Treaties between States and International
jurisdiction.”25(Emphasis supplied) Organizations or between International Organizations, Art. 2(1)(a) (1986).
28 Supra note 2; and Pimentel, Jr. v. Office of the Executive Secretary, 501
By allowing the entry of United States military personnel, their deployment Phil. 303; 462 SCRA 622 (2005) [Per J. Puno, En Banc].
into undefined missions here and abroad, and their use of military assets staged
from our territory against their present and future enemies based on a general
provision in the VFA, the majority now undermines the measures built into our 652
present Constitution to allow the Senate, Congress and our People to participate
652 SUPREME COURT REPORTS ANNOTATED
in the shaping of foreign policy. The EDCA may be an agreement that “deep-
_______________ Saguisag vs. Ochoa, Jr.

25 J. Puno, Dissenting Opinion in Bayan (Bagong Alyansang Makabayan) v.


Zamora, supra note 2 at pp. 672-675; pp. 503-505. agreement is to be regarded as a treaty or as an executive agreement depends
on the subject matter covered by and the temporal nature of the
agreement.29 Commissioner of Customs v. Eastern Sea Trading30 differentiated the agreement also carries no legal significance. Provided the instruments
international agreements that require Senate concurrence from those that do not: possess the elements of an agreement under international law, they are to
be taken equally as “treaty” without regard to the descriptive names by
International agreements involving political issues or changes of which they are designated, such as “protocol,” “charter,” “covenant,”
national policy and those involving international arrangements of a “exchange of notes,” “modus vivendi,” “convention,” or “executive
permanent character usually take the form of treaties. But international agreement.”32 (Emphasis supplied, citations omitted)
agreements embodying adjustments of detail carrying out well-established
national policies and traditions and those involving arrangements of a more Under Article 2(2)33 of the Vienna Convention on the Law of Treaties, in
or less temporary nature usually take the form of executive relation to Article 2(1)(a),34 the designation and
agreements.31 (Emphasis in the original) _______________

Indeed, the distinction made in Commissioner of Customs in terms of 32 Magallona, Merlin M., A Primer in International Law, pp. 62-64 (1997).
international agreements must be clarified depending on whether it is viewed from 33 Article 2. USE OF TERMS
an international law or domestic law perspective. Dean Merlin M. Magallona ....
summarizes the differences between the two perspectives: 2. The provisions of paragraph 1 regarding the use of terms in the present
Convention are without prejudice to the use of those terms or to the meanings
From the standpoint of Philippine constitutional law, a treaty is to be which may be given to them in the internal law of any State.
distinguished from an executive agreement, as the Supreme Court has done 34 1. For the purposes of the present Convention:
in Commissioner of Customs v. Eastern Sea Trading where it declares that (a) “Treaty” means an international agreement concluded between States in
“the concurrence of [the Senate] is required by our fundamental law in the written form and governed by international law, whether embodied in a single
making of ‘treaties’ . . . which are, however, distinct and different from instrument or in two or more related instruments and whatever its particular
‘executive agreements,’ which may be validly entered into without such designation.
concurrence.”
_______________
654
See also Exec. Order No. 292 (1987), Book IV, Title I, Sec. 3(1) and 20. 654 SUPREME COURT REPORTS ANNOTATED
29 Commissioner of Customs v. Eastern Sea Trading, 113 Phil. 333; 3 SCRA
351 (1961) [Per J. Concepcion, En Banc]. Saguisag vs. Ochoa, Jr.
30 Id.
31 Id., at p. 338; p. 356. treatment given to an international agreement is subject to the treatment
given by the internal law of the state party.35 Paragraph 2 of Article 2 specifically
safeguards the states’ usage of the terms “treaty” and “international agreement”
653 under their internal laws.36
Within the context of our Constitution, the requirement for Senate concurrence
VOL. 779, JANUARY 12, 2016 653
in Article VII, Section 21 of the Constitution connotes a special field of state
Saguisag vs. Ochoa, Jr. policies, interests, and issues relating to foreign relations that the Executive
cannot validly cover in an executive agreement:
Thus, the distinction rests on the application of Senate concurrence as a
constitutional requirement. As stated above, an executive agreement is outside the coverage of
However, from the standpoint of international law, no such distinction Article VII, Section 21 of the Constitution and hence not subject to Senate
is drawn. Note that for purposes of the Vienna Convention on the Law of concurrence. However, the demarcation line between a treaty and an
Treaties, in Article 2(1)(a) the term “treaty” is understood as “an executive agreement as to the subject matter or content of their coverage is
international agreement concluded between States in written form and ill-defined. The courts have not provided reliable guidelines as to the scope
governed by international law, whether embodied in a single instrument or of executive-agreement authority in relation to treaty-making power.
in two or more related instruments and whatever its particular If executive-agreement authority is un-contained, and if what may be the
designation.” . . . The Philippines is a party to the Convention which is proper subject matter of a treaty may also be included within the scope of
already in force. In the use of the term “treaty,” Article 2(1)(a) of the Vienna executive-agreement power, the constitutional requirement of Senate
Convention on the Law of Treaties between States and International concurrence could be rendered meaningless. The requirement could be
Organizations, which is not yet in force, the designation or appellation of circumvented by an expedient resort to executive agreement.
The definite provision for Senate concurrence in the Constitution
indomitably signifies that there must be a regime of national interests,
policies and problems which the Executive branch of the government cannot 656
deal with in terms of foreign relations except through treaties concurred in 656 SUPREME COURT REPORTS ANNOTATED
by the Senate under Article VII, Section 21 of
Saguisag vs. Ochoa, Jr.
_______________
V
35 See Magallona, Merlin M., The Supreme Court and International Law:
Problems and Approaches in Philippine Practice, in International Relations The Solicitor General, on behalf of government, proposes that we should view
Pamphlet Series No. 12, 16-17 (2010). the EDCA merely as an implementation of both the Mutual Defense Treaty and
36 See 1 Cortien, Olivier and Klein, Pierre, The Vienna Conventions on the the VFA. In his view, since both the Mutual Defense Treaty and the VFA have
Law of Treaties: A Commentary, pp. 34 and 55 (2011). been submitted to the Senate and concurred in validly under the governing
constitutional provisions at that time, there is no longer any need to have an
655 implementing agreement similarly submitted for Senate concurrence.
The Chief Justice, writing for the majority of this court, agrees with the
VOL. 779, JANUARY 12, 2016 655 position of the Solicitor General.
Saguisag vs. Ochoa, Jr. I disagree.
The proposal of the Solicitor General cannot be accepted for the following
the Constitution. The problem is how to define that regime, i.e., that reasons: (1) the Mutual Defense Treaty, entered into in 1951 and ratified in 1952,
which is outside the scope of executive-agreement power of the President and cannot trump the constitutional provision Article XVIII, Section 25; (2) even the
which exclusively belongs to treaty-making as subject to Senate VFA, which could have been also argued as implementing the Mutual Defense
concurrence.37 (Emphasis supplied) Treaty, was presented to the Senate for ratification; (3) the EDCA contains
significant and material obligations not contemplated by the VFA; and (4)
Thus, Article VII, Section 21 may cover some but not all types of executive assuming arguendo that the EDCA only provides the details for the full
agreements. Definitely, the determination of its coverage does not depend on the implementation of the VFA, Article XVIII, Section 25 still requires that it at least
nomenclature assigned by the President. be submitted to the Senate for concurrence, given the history and context of the
Executive agreements are international agreements that pertain to mere constitutional provision.
adjustments of detail that carry out well-entrenched national policies and
traditions in line with the functions of the Executive. It includes enforcement of VI
existing and valid treaties where the provisions are clear. It involves arrangements
that are of a temporary nature. More importantly, it does not amend existing The 1951 Mutual Defense Treaty cannot be the treaty contemplated in Article
treaties, statutes, or the Constitution. XVIII, Section 25. Its implementation through an executive agreement, which
In contrast, international agreements that are considered treaties under our allows foreign military bases, troops, and facilities, is not enough. If the Mutual
Constitution involve key political issues or changes of national policy. These Defense Treaty is the basis for the EDCA as a mere executive agreement, Article
agreements are of a permanent character. It requires concurrence by at least two- XVIII, Section 25 of the Constitution will make no sense. An absurd interpretation
thirds of all the members of the Senate. of the Constitution is no valid interpretation.
Even if we assume that the EDCA’s nomenclature as an “executive agreement”
is correct, it is still the type of international agreement that needs to be submitted
to the Senate for concurrence. It involves a key political issue that substantially 657
alters or reshapes our national and foreign policy. VOL. 779, JANUARY 12, 2016 657
Fundamentally however, the President’s classification of the EDCA as a mere
“executive agreement” is invalid. Article XVIII, Section 25 requires that the Saguisag vs. Ochoa, Jr.
presence of foreign troops, bases, and facilities must be covered by an
internationally binding agreement in the form of a treaty concurred in by the The Mutual Defense Treaty was entered into by representatives of the
Senate. Philippines and the United States on August 30, 1951 and concurred in by the
_______________ Philippine Senate on May 12, 1952. The treaty acknowledges that this is in the
context of our obligations under the Charter of the United Nations. Thus, Article I
37 Magallona, supra note 32 at pp. 66-67. of the Mutual Defense Treaty provides:
The Parties undertake, as set forth in the Charter of the United of the broad terms of the 1951 treaty yet did not expressly mention the 1951
Nations, to settle any international disputes in which they may be involved Mutual Defense Treaty in Article XVIII, Section 25. We can conclude, with sturdy
by peaceful means in such a manner that international peace and security and unassailable logic, that the 1951 treaty is not the treaty contemplated in
and justice are not endangered and to refrain in their international Article XVIII, Section 25.
relations from the threat or use of force in any manner inconsistent with Besides, the Executive also viewed the VFA as an implementation of the 1951
the purposes of the United Nations. Mutual Defense Treaty. Yet, it was still submitted to the Senate for concurrence.
Parenthetically, Article 62 of the Vienna Convention on the Law of
Further, the treaty expresses the desire of the parties to “maintain and develop Treaties38 provides for the principle of “rebus sic stan-
their individual and collective capacity to resist armed attack.” Thus, in Article III
of the Treaty: _______________

In order more effectively to achieve the objective of this Treaty, the 38 Article 62. Fundamental Change of Circumstances
Parties separately and jointly by self-help and mutual aid will maintain 1. A fundamental change of circumstances which has occurred with regard to
and develop their individual and collective capacity to resist armed attack. those existing at the time of the conclusion of a treaty, and which was not foreseen
by the par-
While these provisions in the 1951 Mutual Defense Treaty could reasonably be
interpreted to include activities done jointly by the Philippines and the United
States, nothing in International Law nor in the Constitution can be reasonably 659
read as referring to this treaty for the authorization for “foreign military bases,
VOL. 779, JANUARY 12, 2016 659
troops, or facilities” after the ratification of the 1987 Constitution.
Again, the constitutional provision reads: Saguisag vs. Ochoa, Jr.

Section 25. After the expiration in 1991 of the Agreement tibus,” in that a fundamental change of circumstances may be a ground to
between the Republic of the Philippines and the United States of terminate or withdraw from a treaty.39 Dean Merlin M. Magallona is of the view
America concerning Military Bases, foreign military bases, troops or that there has been a fundamental change in circumstances that allows the
facilities Philippines to terminate the 1951 Mutual Defense Treaty.40 Although we should
acknowledge this suggestion during the oral arguments by petitioners, we do not
need to go into such an issue and at this time to be able to resolve the controversies
658 in this case. We await a case that will provide a clearer factual backdrop properly
658 SUPREME COURT REPORTS ANNOTATED pleaded by the parties.
_______________
Saguisag vs. Ochoa, Jr.
ties, may not be invoked as a ground for terminating or withdrawing from the
shall not be allowed in the Philippines except under a treaty duly treaty unless:
concurred in by the Senate and, when the Congress so requires, ratified by a. The existence of those circumstances constituted an essential basis of the
a majority of the votes cast by the people in a national referendum held for consent of the parties to be bound by the treaty; and
that purpose, and recognized as a treaty by the other contracting State. b. The effect of the change is radically to transform the extent of obligations
(Emphasis supplied) still to be performed under the treaty.
2. A fundamental change of circumstances may not be invoked as a ground for
There is a time stamp to the obligation under this provision. The prohibition terminating or withdrawing from a treaty:
against “foreign military bases, troops, or facilities,” unless covered by treaty or a. If the treaty establishes a boundary; or
allowed through a referendum, becomes effective “after the expiration in 1991 of b. If the fundamental change is the result of a breach by the party invoking it
the Agreement . . . concerning Military Bases.” The treaty about to expire refers to either of an obligation under the treaty or of any other international obligation
the 1947 Military Bases Agreement as amended. This was still in effect at the time owed to any other party to the treaty.
of the drafting, submission, and ratification of the 1987 Constitution. 3. If, under the foregoing paragraphs, a party may invoke a fundamental
The constitutional timeline is unequivocal. change of circumstances as a ground for terminating or withdrawing from a treaty
The 1951 Mutual Defense Treaty was in effect at the time of the ratification of it may also invoke the change as a ground for suspending the operation of the
the Constitution in 1987. It was also in effect even after the expiration of the treaty.
Military Bases Agreement in 1991. We could reasonably assume that those who
drafted and ratified the 1987 Constitution were aware of this legal situation and
Vienna Convention of the Law of Treaties (1969)
<https://treaties.un.org/doc/Publication/UNTSNolume%201155/volume-1155-I- This asymmetry in the legal treatment of the Military Bases
18232-English.pdf> (visited January 11, 2016). Agreement by the two countries was believed to be a slur to our
39 Vienna Convention of the Law of Treaties, Art. 62 (1969) sovereignty.41 (Emphasis supplied)
<https://treaties.un.org/doc/Publication/UNTSN/Volume%201155/volume-1155-I-
18232-English.pdf> (visited January 11, 2016). In Nicolas, Associate Justice Antonio T. Carpio himself underscored the non-
40 Magallona, Merlin M., A Critical Review of the EDCA, p. 29 (2014) treaty status of the Visiting Forces Agreement in light of Medellin v. Texas42 in his
(Unpublished), annexed to petitioners’ Memorandum. Separate Opinion, thus:

Under Medellin, the VFA is indisputably not enforceable as domestic


660 federal law in the United States. On the other hand, since the Philippine
660 SUPREME COURT REPORTS ANNOTATED Senate ratified the VFA, the VFA constitutes domestic law in the
Philippines. This unequal legal status of the VFA violates Section 25,
Saguisag vs. Ochoa, Jr. Article XVIII of the Philippine Constitution, which specifically requires
that a treaty involving the presence of foreign troops in the Philippines
In addition, the Mutual Defense Treaty is not the treaty contemplated by must be equally binding on the Philippines and on the other contracting
Article XVIII, Section 25 on account of its subject matter. In paragraph 5 of its State.
Preamble, the Mutual Defense Treaty articulates the parties’ desire “to strengthen In short, the Philippine Constitution bars the efficacy of such a treaty
their present efforts to collective defense for the preservation of peace and security that is enforceable as domestic law only in the Philippines but
pending the development of a more comprehensive system of regional security in unenforceable as domestic law in the other contracting State. The
the Pacific Area.” Article II further clarifies the treaty’s purpose: Philippines is a sovereign and independent State. It is no longer a colony of
Article II the United States. This Court should not countenance an unequal treaty
In order more effectively to achieve the objective of this Treaty, the that is not only contrary to the express mandate of the Philippine
Parties separately and jointly by self-help and mutual aid will maintain Constitution, but also an affront to the sovereignty, dignity and
and develop their individual and collective capacity to resist armed independence of the Philippine State.
attack. (Emphasis supplied) There is no dispute that Section 25, Article XVIII of the Philippine
Constitution governs the constitutionality of the VFA. Section 25 states:
Clearly, none of its provisions provide specifically for the presence of a base, _______________
troops, or facilities that will put it within the ambit of Article XVIII, Section 25.
Its main aim is to provide support against state enemies effectively and efficiently. 41 J. Puno, Dissenting Opinion in Bayan (Bagong Alyansang Makabayan) v.
Thus, for instance, foreign military bases were covered in the 1947 Military Bases Zamora, supra note 2 at pp. 672-673; p. 503.
Agreement. 42 128 S.Ct. 1346; 170 L.Ed.2d 190.
The VFA cannot also be said to be the treaty required in Article XVIII, Section
25. This is because the United States, as the other contracting party, has never
treated it as such under its own domestic laws. The VFA has the same status as 662
that of the 1947 Military Bases Agreement in that it is merely an executive
agreement on the part of United States: 662 SUPREME COURT REPORTS ANNOTATED
Saguisag vs. Ochoa, Jr.
As articulated by Constitutional Commissioner Blas F. Ople in the 1986
Constitutional Commission deliberations on this provision, the 1947 RP- Section 25. After the expiration in 1991 of the Agreement
US Military Bases Agreement was ratified by the Philippine Senate, between the Republic of the Philippines and the United States of
but not by the United States Senate. In the eyes of Philippine law, America concerning Military Bases, foreign military bases, troops,
therefore, the Military Bases Agreement was a treaty, but by the laws or facilities shall not be allowed in the Philippines except under a
of the United States, it was a mere executive agreement. treaty duly concurred in by the Senate and, when the Congress so
requires, ratified by a majority of the votes cast by the people in a
national referendum held for that purpose, and recognized as a
661 treaty by the other contracting State.
VOL. 779, JANUARY 12, 2016 661 The clear intent of the phrase “recognized as a treaty by the other
contracting State” is to insure that the treaty has the same legal effect
Saguisag vs. Ochoa, Jr.
on the Philippines as on the other contracting State. This requirement is
unique to agreements involving the presence of foreign troops in the
Philippines, along with the requirement, if Congress is so minded, to hold 664
a national referendum for the ratification of such a treaty. 664 SUPREME COURT REPORTS ANNOTATED
The deliberations of the Constitutional Commission reveal the
sensitivity of the framers to the “unacceptable asymmetry” of the then Saguisag vs. Ochoa, Jr.
existing military bases agreement between the Philippines and the United
States. The Philippine Senate had ratified the military bases agreement So, for these reasons, I oppose the deletion of this section
but the United States Government refused to submit the same to the U.S. because, first of all, as I said, it does not prevent
Senate for ratification. Commissioner Blas Ople explained this renegotiation. Second, it respects the sovereignty of our people and
“unacceptable asymmetry” in this manner: the people will be in a better position to judge whether to accept the
. . . But I think we have acknowledged starting at the committee treaty or not, because then they will be voting not just on an
level that the bases agreement was ratified by our Senate; it is a abstraction but they will be voting after examination of the terms of
treaty under Philippine law. But as far as the Americans are the treaty negotiated by our government. And third, the
concerned, the Senate never took cognizance of this and, requirement that it be recognized as a treaty by the other
therefore, it is an executive agreement. That creates a contracting nation places us on the same level as any other
wholly unacceptable asymmetry between the two countries. contracting party.
Therefore, in my opinion, the right step The following exchanges in the Constitutional Commission explain
further the meaning of the phrase “recognized as a treaty by the other
contracting State”:
663 FR. BERNAS: Let me be concrete, Madam President, in our
circumstances. Suppose they were to have this situation where our
VOL. 779, JANUARY 12, 2016 663
government were to negotiate a treaty with the United States, and
Saguisag vs. Ochoa, Jr. then the two executive departments in the ordinary course of
negotiation come to an agreement. As our Constitution is taking
to take, if the government of our country will deem it in the shape now, if this is to be a treaty at all, it will have to be submitted
national interest to terminate this agreement or even to renegotiate to our Senate for its ratification. Suppose, therefore, that what was
it, is that we must begin with a clean slate; we should not be agreed upon between the United States and the executive
burdened by the flaws of the 1947 Military Bases Agreement. I think department of the Philippines is submitted and ratified by the
that is a very important point. I am glad to be reassured by the two Senate, then it is further submitted to the people for its ratification
Gentlemen that there is nothing in these proposals that will bar the and subsequently, we ask the United States: “Complete the
Philippine government at the proper time from exercising the option process by accepting it as a treaty through ratification by
of abrogation or termination. your Senate as the United States Constitution requires,”
Eventually, the Constitutional Commission required that any would such an arrangement be in derogation of sovereignty?
agreement involving the presence of foreign troops in the Philippines must
be “recognized as a treaty by the other contracting State.” This
means that the other contracting State must recognize the agreement as a 665
treaty, as distinguished from any other agreement, and if its constitutional VOL. 779, JANUARY 12, 2016 665
processes require, submit the agreement to its proper legislative body for
ratification as a treaty. As explained by Commissioner Father Joaquin Saguisag vs. Ochoa, Jr.
Bernas, S.J., during the deliberations of the Constitutional Commission:
Third, on the last phrase “AND RECOGNIZED AS A TREATY MR. NOLLEDO: Under the circumstances the Commissioner
BY THE OTHER CONTRACTING NATION,” we enter into a just mentioned, Madam President, on the basis of the provision of
treaty and we want the other contracting party to respect Section 1 that “sovereignty resides in the Filipino people,” then we
that document as a document possessing force in the same would not consider that a derogation of our sovereignty on the basis
way that we respect it. The present situation we have is that the and expectation that there was a plebiscite.
bases agreement is a treaty as far as we are concerned, but it is only xxx xxx xxx
an executive agreement as far as the United States is concerned, FR. BERNAS: As Commissioner Romulo indicated, since this
because the treaty process was never completed in the United States certainly would refer only to the United States, because it is only
because the agreement was not ratified by the Senate. the United States that would have the possibility of being allowed
to have treaties here, then we would have to require that the Senate _______________
of the United States concur in the treaty because under American
constitutional law, there must be concurrence on the part of the 43 J. Carpio, Dissenting Opinion in Nicolas v. Romulo, supra note 7 at pp.
Senate of the United States to conclude treaties. 308-312; pp. 488-491.
MR. SUAREZ: Thank you for the clarification.
Under the 1935 Constitution, if I recall it correctly, treaties and
agreements entered into require an exchange of ratification. I 667
remember that is how it was worded. We do not have in mind here
VOL. 779, JANUARY 12, 2016 667
an exchange of ratification by the Senate of the United States and
by the Senate of the Philippines, for instance, but only an approval Saguisag vs. Ochoa, Jr.
or a recognition by the Senate of the United States of that treaty.
FR. BERNAS: When I say that the other contracting state pines. A relationship where both parties are on equal footing must be
must recognize it as a treaty, by that I mean it must perform demanded, and from one state to another. The Philippine government must be firm
all the acts required for that agreement to reach the status in requiring that the United States establish stability in its international
of a treaty under their jurisdiction. commitment, both by legislation and jurisprudence.
The doctrine laid down in BAYAN, insofar as the VFA is concerned, should now
be revisited in light of new circumstances and challenges in foreign policy and
666 international relations.
666 SUPREME COURT REPORTS ANNOTATED
VII
Saguisag vs. Ochoa, Jr.
Even if we assume that the Mutual Defense Treaty and the VFA are the
Thus, Section 25, Article XVIII of the Philippine Constitution requires treaties contemplated by Article XVIII, Section 25 of the Constitution, this court
that any agreement involving the presence of foreign troops in the must determine whether the EDCA is a valid executive agreement as argued by
Philippines must be equally legally binding both on the Philippines respondents.
and on the other contracting State. This means the treaty must be It is not. The EDCA modifies these two agreements.
enforceable under Philippine domestic law as well as under the domestic Respondents claim that the EDCA is an executive agreement and merely
law of the other contracting State. Even Justice Adolfo S. Azcuna, implements the Mutual Defense Treaty and VFA.44 In arguing that the EDCA
the ponente of the majority opinion, and who was himself a member of the implements the Mutual Defense Treaty, respondents state that the latter has two
Constitutional Commission, expressly admits this when he states in operative principles: (1) the Principle of Defensive Reaction under Article IV;45 and
his ponencia: (2) the Principle of Defensive Preparation under Article II.46
The provision is thus designed to ensure that any agreement _______________
allowing the presence of foreign military bases, troops or facilities
in Philippine territory shall be equally binding on the 44 Respondents’ Memorandum, pp. 15-16.
Philippines and the foreign sovereign State involved. The 45 ARTICLE IV. Each Party recognizes that an armed attack in the Pacific
idea is to prevent a recurrence of the situation where the area on either of the Parties would be dangerous to its own peace and safety and
terms and conditions governing the presence of foreign declares that it would act to meet the common dangers in accordance with its
armed forces in our territory were binding on us but not constitutional processes. Any such armed attack and all measures taken as a result
upon the foreign State. thereof shall be immediately reported to the Security Council of the United
An “equally binding” treaty means exactly what it says — the treaty Nations, Such measures shall be terminated when the Security Council has taken
is enforceable as domestic law in the Philippines and likewise enforceable the measures necessary to restore and maintain international peace and security.
as domestic law in the other contracting State.43 (Emphasis in the original, 46 ARTICLE II. In order more effectively to achieve the objective of this
citations omitted) Treaty, the Parties separately and jointly by self-help and

Surprisingly, through his Concurring Opinion in this case, Associate Justice


Carpio has now abandoned his earlier views. 668
This court’s interpretation of a treaty under Article XVIII, Section 25
668 SUPREME COURT REPORTS ANNOTATED
in BAYAN, which did away with the requirement that the agreement be
recognized as a treaty by the other contracting party, has resulted in an absurd Saguisag vs. Ochoa, Jr.
situation of political asymmetry between the United States and the Philip-
According to respondents, “[t]he primary concern of the EDCA is the Principle capabilities” are activities that are authorized to be undertaken in the
of Defensive Preparation in order to enhance both parties’ abilities, if required, to Philippines under the VFA.
operationalize the Principle of Defensive Reaction.”47 The specific goals 63. Article II of the EDCA reiterates the definition of “United States
enumerated in the EDCA demonstrate this: personnel” in the VFA which means “United States military and civilian
personnel temporarily in the Philippines in connection with activities
56. The specific purposes of the EDCA — to “[s]upport the Parties’ approved by the Philippines.”
shared goal of improving interoperability of the Parties’ forces, and for the 64. Article III of the EDCA provides for the “Agreed Locations” where
Armed Forces of the Philippines (‘AFP’), [to address its] short-term the Philippines authorizes US to “conduct the following activities”:
capabilities gaps, promoting long-term modernization, and helping “training; transit; support and related activities; refueling of aircraft;
maintain and develop additional maritime security, maritime domain bunkering of vessels; temporary maintenance of vehicles, vessels and
awareness, and humanitarian assistance and disaster relief capabilities” aircraft; temporary accommodation of personnel; communications;
properly fall within the MDT’s objective of developing the defense prepositioning of equipment, supplies and materiel; deploying forces and
capabilities of the Philippines and the US. The EDCA implements the MDT materiel; and such other activities as the Parties may agree.”
by providing for a mechanism that promotes optimal cooperation between 65. Article IV of the EDCA authorizes the prepositioning and storing
the US and the Philippines.48 of defense equipment, supplies and materiel. Under Article IV in relation
to Article III of the EDCA, the “prepositioning of equipment, supplies and
Similarly, respondents allege that the EDCA implements the VFA in relation materiel” is an “activity” to be approved by the Philippine Government
to the entry of United States troops and personnel, importation and exportation of “through bilateral security mechanisms, such as the MDB and SEB.”
equipment, materials, supplies, and other property, and movement of vessels and 66. In sum, what the EDCA does is to enhance the existing contractual
aircraft in the Philippines.49 Respondents rely on this court’s pronouncement security apparatus between the Philippines and the US, set up through the
in Lim that combat-related activities are allowed under the VFA: MDT and the VFA. It is the duty of the Honorable Court to allow this
security apparatus enough breathing space to respond to perceived,
61. Article I of the EDCA provides that its purposes are to support anticipated, and actual exigencies.
“the Parties’ shared goal of improving interop-
_______________
670
mutual aid will maintain and develop their individual and collective capacity 670 SUPREME COURT REPORTS ANNOTATED
to resist armed attack.
47 Respondents’ Memorandum, p. 15. Saguisag vs. Ochoa, Jr.
48 Id., at p. 16.
49 Id., citing Agreement between the Government Republic of the Philippines As discussed earlier, an executive agreement merely provides for the detailed
and the Government of the United States of America Regarding the Treatment of adjustments of national policies or principles already existing in other treaties,
United States Armed Forces Visiting the Philippines (1998), Arts. I, VII, and VIII. statutes, or the Constitution. It involves only the enforcement of clear and specific
provisions of the Constitution, law, or treaty. It cannot amend nor invalidate an
existing statute, treaty, or provision in the Constitution. It includes agreements
669 that are of a temporary nature.
This is not the case with the EDCA.
VOL. 779, JANUARY 12, 2016 669 The EDCA contains significant and material obligations not contemplated by
Saguisag vs. Ochoa, Jr. the VFA. As an executive agreement, it cannot be given any legal effect. The EDCA
substantially modifies and amends the VFA in at least the following aspects:
erability of the Parties’ forces, and for the Armed Forces of the First, the EDCA does not only regulate the “visits” of foreign troops. It allows
Philippines (‘AFP’), [to address its] short-term capabilities gaps, promoting the temporary stationing on a rotational basis of United States military personnel
long-term modernization, and helping maintain and develop additional and their contractors on physical locations with permanent facilities and pre-
maritime security, maritime domain awareness, and humanitarian positioned military materiel.
assistance and disaster relief capabilities.” Second, unlike the VFA, the EDCA allows the pre-positioning of military
62. The Honorable Court in Lim ruled that these activities are materiel, which can include various types of warships, fighter planes, bombers,
already covered by the VFA. Under Lim,“maritime security, maritime land and amphibious vehicles, and their corresponding ammunition.
domain awareness, and humanitarian assistance and disaster relief Third, the VFA contemplates the entry of troops for various training exercises.
The EDCA allows our territory to be used by the United States to launch military
and paramilitary operations conducted in other states.
Fourth, the EDCA introduces new concepts not contemplated in the VFA, 672 SUPREME COURT REPORTS ANNOTATED
namely: (a) agreed locations; (b) contractors; (c) pre-positioning of military
materiel; and (d) operational control. Saguisag vs. Ochoa, Jr.
Lastly, the VFA did not have provisions that may have been construed as a
restriction or modification of obligations found in existing statutes. The EDCA Article III
contains provisions that Agreed Locations
1. With consideration of the views of the Parties, the Philippines hereby
authorizes and agrees that United States forces, United States contractors,
671 and vehicles, vessels, and aircraft operated by or for United States forces
may conduct the following activities with respect to Agreed Locations:
VOL. 779, JANUARY 12, 2016 671
training; transit; support and related activities; refueling of aircraft;
Saguisag vs. Ochoa, Jr. bunkering of vessels; temporary maintenance of vehicles, vessels, arid
aircraft; temporary accommodation of personnel; communications;
may affect various statutes including, among others, (a) the jurisdiction of prepositioning of equipment, supplies, and materiel; deploying forces and
courts, (b) local autonomy, and (c) taxation. materiel; and such other activities as the Parties may agree.
2. When requested, the Designated Authority of the Philippines shall
VIII assist in facilitating transit or temporary access by United States forces to
public land and facilities(including roads, ports, and airfields), including
Article I(1)(b) of the EDCA authorizes United States forces access to “Agreed those owned or controlled by local governments, and to other land and
Locations” in the Philippines on a rotational basis.50 Even while the concept of facilities (including roads, ports, and airfields).
“rotation” may refer to incidental and transient presence of foreign troops and 3. Given the mutuality of benefits, the Parties agree that
contractors, the nature of the “Agreed Locations” is eerily similar to and, therefore, the Philippines shall make Agreed Locations available to United States
amounts to basing agreements. forces without rental or similar costs. United States forces shall cover their
“Agreed Locations” has been defined by the EDCA in Article II(4) as: necessary operation expenses with respect to their activities at the Agreed
Locations.
Facilities and areas that are provided by the Government of the 4. The Philippines hereby grants to the United States, through bilateral
Philippines through the AFP and that United states forces, United security mechanisms, such as the MDB and SEB, operational control of
States contractors, and others as mutually agreed, shall have the Agreed Locations for construction activities and authority to undertake such
right to access and use pursuant to this Agreement. Such agreed activities on, and make alterations and improvements to, Agreed Locations.
Locations may be listed in an annex to be appended to this Agreement, and United States forces shall consult on issues regarding such construction;
may be further described in implementing agreements. (Emphasis alterations, and improvements on the Parties’ shared intent that the
supplied) technical requirements and construction standards of any such projects
undertaken by or on behalf of United States
As treaties, the 1947 Military Bases Agreement and its various amendments
specified the actual location of the physical locations of United States troops and
facilities. The EDCA, however, now delegates the identification of the location not 673
to a select Senate Committee or a public body but simply to our military VOL. 779, JANUARY 12, 2016 673
representatives in the Mutual Defense Board and the Security Enhancement
Board. Saguisag vs. Ochoa, Jr.
More importantly, the extent of access and use allowed to United States forces
and contractors under the EDCA is broad. It is set out in Article III: forces should be consistent with the requirements and standards of both
_______________ Parties.
....
50 (b) Authorizing access to Agreed Locations in the territory of the 6. United States forces shall be responsible on the basis of proportionate
Philippines by United States forces on a rotational basis, as mutually determined use for construction, development, operation, and maintenance costs at
by the Parties. Agreed Locations. Specific funding arrangements may be fined in
Implementing arrangements. (Emphasis supplied)

672 Parsing the provisions carefully, we find that the Agreed Locations may be
used for:
mercenary groups that have been used by the United States in other parts of the
(1) training; world. The EDCA covers these types of operations within and outside Philippine
(2) transit; territory. Again, the consequences to Philippine foreign policy in cases where
(3) support and related activities; targets are found in neighboring countries would be immeasurable.
(4) refueling of aircraft;
(5) bunkering of vessels;
(6) temporary maintenance of vehicles, vessels, and aircraft; 675
(7) temporary accommodation of personnel; VOL. 779, JANUARY 12, 2016 675
(8) communications;
(9) pre-positioning of equipment, supplies, and materiel; Saguisag vs. Ochoa, Jr.
(10) deploying forces and materiel; and
(11) other activities as the parties may agree. The Visiting Forces Agreement does not cover these sample activities. Nor does
it cover United States contractors.
There is no hierarchy among these activities. In other words, functions (2) to
(11) need not be supportive only of training or transit. Function (10), which IX
pertains to deployment of United States forces and materiel, can be done
independently of whether there are training exercises or whether the troops are Blanket authority over Agreed Locations is granted under Article VI, Section
only in transit. 3 of the EDCA. The United States forces are given a broad range of powers with
The permission to do all these activities is explicit in the EDCA. Government regard to the Agreed Locations that are “necessary for their operational control or
has already authorized and agreed that “United States forces, United States defense.”51 This authority extends to the protection of United States forces and
contractors, and vehicles, contractors. In addition, the United States is merely obligated to coordinate with
Philippine authorities the measures they will take in case they deem it necessary
to take action.
674
In contrast, the Mutual Defense Treaty is different. It is specific to the
674 SUPREME COURT REPORTS ANNOTATED
maintenance and development of the Philippines and the United States’ individual
Saguisag vs. Ochoa, Jr. and collective capacity to resist armed attack. The parties’ goal under the Mutual
Defense Treaty is to enhance collective defense mechanisms for the preservation
vessels, and aircraft operated by or for United States forces” may conduct all of peace and security in the Pacific area.52
these activities. Carefully breaking down this clause in Article III(1) of the EDCA, While certain activities such as “joint RP-US military exercises for the purpose
the authorization is already granted to: of developing the capability to resist an armed attack fall . . . under the provisions
(a) “United States forces”; of the RP-US Mutual Defense Treaty,”53 the alleged principles of Defensive
(b) “United States contractors”; and Reaction and Defensive Preparation do not license the ceding
(c) “vehicles, vessels, and aircraft operated by or for United States forces.” _______________
United States military forces will not only be allowed to “visit” Philippine
territory to do a transient military training exercise with their Philippine 51 Agreement between the Government of the Philippines and the
counterparts. They are also allowed to execute, among others, the following Government of the United States of America on Enhanced Defense Cooperation
scenarios: (2014), Art. VI(3). United States forces are authorized to exercise all rights and
One: Parts of Philippine territory may be used as staging areas for special or authorities within Agreed Locations that are necessary for their operational
regular United States military personnel for intervention in conflict areas in the control or defense, including taking appropriate measures to protect United States
Southeast Asian region. This can be in the form of landing rights given to their forces and United States contractors. The United States should coordinate such
fighter jets and stealth bombers or way stations for SEALS or other special units measures with appropriate authorities of the Philippines.
entering foreign territory in states not officially at war with the Philippines. 52 Mutual Defense Treaty between the Republic of the Philippines and the
Two: Parts of Philippine territory may be used to supplement overt United States of America (1951), Preamble, par. 4.
communication systems of the United States forces. For instance, cyberwarfare 53 Nicolas v. Romulo, supra note 7 at p. 284; p. 461.
targeting a state hostile to the United States can be launched from any of the
Agreed Locations to pursue their interests even if this will not augur well to
Philippine foreign policy. 676
Three: Parts of Philippine territory may be used to plan, deploy, and supply
676 SUPREME COURT REPORTS ANNOTATED
covert operations done by United States contractors such as Blackwater and other
Saguisag vs. Ochoa, Jr. 3. No permanent US basing and support facilities shall be established.
Temporary structures such as those for troop billeting, classroom instruction and
of authority and control over specific portions of the Philippines to foreign messing may be set up for use by RP and US Forces during the Exercise.
military forces without compliance with the Constitutional requirements. 54 Such 4. The Exercise shall be implemented jointly by RP and US Exercise Co-
grant of authority and control over Agreed Locations to foreign military forces Directors under the authority of the Chief of Staff, AFP. In no instance will US
involves a drastic change in national policy and cannot be done in a mere executive Forces operate independently during field training exercises (FTX). AFP and US
agreement. Unit Commanders will retain command over their respective forces under the
Moreover, nothing in the VFA provides for the use of Agreed Locations to overall authority of the Exercise Co-Directors. RP and US participants shall
United States forces or personnel, considering that the VFA focuses on the comply with operational instructions of the AFP during the FTX.
visitation of United States armed forces to the Philippines in relation to joint 5. The exercise shall be conducted and completed within a period of not more
military exercises: than six months, with the projected participation of 660 US personnel and 3,800
RP Forces. The Chief of Staff, AFP shall direct the Exercise Co-Directors to wind
Preamble up and terminate the Exercise and other activities within the six month Exercise
The Government of the United States of America and the Government of the period.
Republic of the Philippines, 6. The Exercise is a mutual counter-terrorism advising, assisting and
Reaffirming their faith in the purposes and principles of the Charter of the training Exercise relative to Philippine efforts against the ASG, and will be
United Nations and their desire to strengthen international and regional security conducted on the Island of Basilan. Further advising, assisting and training
in the Pacific area; exercises shall be conducted in Malagutay and the Zamboanga area. Related
Reaffirming their obligations under the Mutual Defense Treaty of August 30, activities in Cebu will be for support of the Exercise.
1951; 7. Only 160 US Forces organized in 12-man Special Forces Teams shall be
Noting that from time to time elements of the United States armed forces may deployed with AFP field commanders. The US teams shall remain at the Battalion
visit the Republic of the Philippines; Headquarters and, when approved, Company Tactical headquarters where they
Considering that cooperation between the United States and the Republic of the can observe and assess the performance of the AFP Forces.
Philippines promotes their common security interests; 8. US exercise participants shall not engage in combat, without prejudice to
Recognizing the desirability of defining the treatment of United States their right of self-defense.
personnel visiting the Republic of the Philippines[.] (Emphasis supplied)
_______________
678
54 See Const., Art. XVIII, Sec. 25. 678 SUPREME COURT REPORTS ANNOTATED
Saguisag vs. Ochoa, Jr.

677 joint military exercises is covered by the VFA. Hence, under


VOL. 779, JANUARY 12, 2016 677 _______________
Saguisag vs. Ochoa, Jr.
9. These terms of Reference are for purposes of this Exercise only and do not
In Lim, the Terms of Reference55 of the “Balikatan 02-1” create additional legal obligations between the US Government and the Republic
_______________ of the Philippines.
II. EXERCISE LEVEL
1. TRAINING
55 The Terms of Reference provides:
a. The Exercise shall involve the conduct of mutual military assisting, advising
I. POLICY LEVEL
and training of RP and US Forces with the primary objective of enhancing the
1. The Exercise shall be Consistent with the Philippine Constitution and all
operational capabilities of both forces to combat terrorism.
its activities shall be in consonance with the laws of the land and the provisions of
b. At no time shall US Forces operate independently within RP territory.
the RP-US Visiting Forces Agreement (VFA).
c. Flight plans of all aircraft involved in the exercise will comply with the local
2. The conduct of this training Exercise is in accordance with pertinent
air traffic regulations.
United Nations resolutions against global terrorism as understood by the
2. ADMINISTRATION & LOGISTICS
respective parties.
a. RP and US participants shall be given a country and area briefing at the
start of the Exercise. This briefing shall acquaint US Forces on the culture and
sensitivities of the Filipinos and the provisions of the VFA. The briefing shall also security, maritime domain awareness, and humanitarian assistance and
promote the full cooperation on the part of the RP and US participants for the disaster relief capabilities; and
successful conduct of the Exercise. (b) Authorizing access to Agreed Locations in the territory of the
b. RP and US participating forces may share, in accordance with their Philippines by United States forces on a rotational basis, as mutually
respective laws and regulations, in the use of their resources, equipment and other determined by the Parties.
assets. They will use their respective logistics channels. _______________
c. Medical evaluation shall be jointly planned and executed utilizing RP and
US assets and resources. 56 Lim v. Executive Secretary, supra note 5.
d. Legal liaison officers from each respective party shall be appointed by the 57 Id., at p. 564; p. 745.
Exercise Directors.
3. PUBLIC AFFAIRS
a. Combined RP-US Information Bureaus shall be established at the Exercise 680
Directorate m Zamboanga City and at GHQ, AFP in Camp Aguinaldo, Quezon
680 SUPREME COURT REPORTS ANNOTATED
City.
b. Local media relations will be the concern of the AFP and all public affairs Saguisag vs. Ochoa, Jr.
guidelines shall be jointly developed by RP and US Forces.
c. Socio-Economic Assistance Projects shall be planned and executed jointly by 2. In furtherance of the MDT, the Parties mutually agree that this
RP and US Forces in accordance with their respective laws and regulations, and Agreement provides the principal provisions and necessary authorizations
in consultation with community and local government officials. with respect to Agreed Locations.
3. The Parties agree that the United States may undertake the
following types of activities in the territory of the Philippines in relation to
679 its access to and use of Agreed Locations: security cooperation exercises; joint
VOL. 779, JANUARY 12, 2016 679 and combined training activities; humanitarian assistance and disaster
relief activities; and such other activities as may be agreed upon by the
Saguisag vs. Ochoa, Jr. Parties. (Emphasis supplied)

the VFA, activities such as joint exercises, which “include training on new The VFA was ratified in 1998. However, in 2011, the Obama Administration
techniques of patrol and surveillance to protect the nation’s marine resources, sea announced its plan of intensifying its presence in the Asia-Pacific region.58 The
search-and-rescue operations to assist vessels in distress, disaster relief United States hinges this pivot on maritime peace and security in the region in
operations, civic action projects such as the building of school houses, medical and relation to a stable international economic order.59 Hence, their Department of
humanitarian missions, and the like,”56 are authorized. However, Lim specifically Defense enumerates three maritime objectives: “to safeguard the freedom of the
provided for the context of the conduct of the combat-related activities under the seas; deter conflict and coercion; and promote adherence to international law and
VFA: President George W. Bush’s international antiterrorism campaign as a result standards.”60
of the events on September 11, 2001.57 To achieve these objectives, the United States conducts operations, exercises,
Meanwhile, the EDCA unduly expands the scope of authorized activities to and training with several countries it
Agreed Locations with only a vague reference to the VFA: _______________

Article I 58 Manyin, Mark E., Pivot to the Pacific? The Obama Administration’s
Purpose and Scope “Rebalancing” Toward Asia (2012)
<https://www.fas.org/sgp/crs/natsec/R42448.pdf> (visited January 11,
1. This Agreement deepens defense cooperation between the Parties 2016). See Odom, Jonathan G., What Does a “Pivot” or “Rebalance” Look Like?
and maintains and develops their individual and collective capacities, in Elements of the US Strategic Turn Towards Security in the Asia-Pacific Region and
furtherance of Article II of the MDT, which states that the Parties Its Waters, 14 APLPJ 2-8 (2013); O’Rourke, Ronald, Maritime Territorial and
separately and jointly by self-help and mutual aid will maintain and Exclusive Economic Zone (EEZ) Disputes Involving China: Issues for
develop their individual capacity to resist armed attack, and within the Congress (2015) <https://www.fas.org/sgp/crs/row/R42784.pdf> (visited January
context of VFA. This includes: 11, 2016).
(a) Supporting the Parties’ shared goal of improving interoperability 59 United States Department of Defense, The Asia-Pacific Maritime Security
of the Parties’ forces, and for the Armed Forces of the Philippines (“AFP”), Strategy: Achieving US National Security Objectives in a Changing Environment,
addressing short-term capabilities gaps, promoting long-term pp. 1-2 <http://www.defense.gov/Portals/1/Documents/pubs/NDAA%20A-
modernization, and helping maintain and develop additional maritime
P_Maritime_SecuritY_Strategy-08142015-1300-FINALFORMAT.PDF (visited
January 11, 2016). ....
60 Id., at p. 1. In Southeast Asia, the Department is honing an already robust bilateral
exercise program with our treaty ally, the Republic of the Philippines, to
assist it with establishing a minimum credible defense more effectively. We
681 are conducting more than 400 planned events with the Philippines in 2015,
VOL. 779, JANUARY 12, 2016 681 including our premier joint exercise, Balikatan, which this year was the
largest and most sophisticated ever. During this year’s Balikatan, more than
Saguisag vs. Ochoa, Jr. 15,000 US, Philippine, and Australian military personnel exercised
operations involving a territorial defense scenario in the Sulu Sea, with
considers allies in the region.61 Nevertheless, key to the United States’ military personnel from Japan observing.62 (Emphasis supplied)
strategy is the enhancement of its forward presence in the Asia-Pacific:
These changes in United States policy are reflected in the EDCA and not in the
Force Posture VFA. Thus, there is a substantial change of objectives.
If, indeed, the goal is only to enhance mutual defense capabilities under the
One of the most important efforts the Department of Defense has Mutual Defense Treaty through conduct of joint military exercises authorized by
underway is to enhance our forward presence by bringing our finest the VFA, then it behooves this court to ask the purpose of providing control and
capabilities, assets, and people to the Asia-Pacific region. The U.S. military authority over Agreed Locations here in the Philippines when it is outside the
presence has underwritten security and stability in the Asia-Pacific region coverage of both the Mutual Defense Treaty and the VFA. Through a vague
for more than 60 years. Our forward presence not only serves to deter reference to the VFA, respondents fail to establish how the EDCA merely
regional conflict and coercion, it also allows us to respond rapidly to implements the VFA. They cannot claim that the provisions of the EDCA merely
maritime crises. Working in concert with regional allies and partners make use of the authority previously granted under the VFA. What is clear is that
enables us to respond more effectively to these crises. the Agreed Locations become a platform for the United States to execute its new
The United States maintains 368,000 military personnel in the Asia- military strategy and strengthen its presence in the Asia-Pacific, which is clearly
Pacific region, of which approximately 97,000 are west of the International outside the coverage of the VFA.
Date Line. Over the next five years, the US. Navy will increase the number In addition, the EDCA does not merely implement the Mutual Defense Treaty
of ships assigned to Pacific Fleet outside of US territory by approximately and VFA when it provides for the entry of United States private contractors into
30 percent, greatly improving our ability to maintain a more regular and the Philippines.
persistent maritime presence in the Pacific. And by 2020, 60 percent of naval _______________
and overseas air assets will be homeported in the Pacific region. The
Department will also enhance Marine Corps presence by developing a more 62 Id., at pp. 22-23.
distributed and sustainable laydown model.
Enhancing our forward presence also involves using existing assets in
new ways, across the entire region, with an emphasis on operational 683
flexibility and maximizing the value of US assets despite the tyranny of
distance. This is why the Department is working to develop a more VOL. 779, JANUARY 12, 2016 683
distributed, resilient, and sustainable posture. As part of this effort, the Saguisag vs. Ochoa, Jr.
United States will maintain its presence in Northeast Asia, while
enhancing defense posture across the Western Pacific, Southeast Asia, and In the EDCA, United States contractors are defined as follows:
the Indian Ocean.
_______________ 3. “United States contractors” means companies and firms, and their
employees, under contract or subcontract to or on behalf of the United
61 Id., at pp. 23-24. States Department of Defense. United States contractors are not included
as part of the definition of United States personnel in this Agreement,
including within the context of the VFA.63 (Emphasis supplied)
682
682 SUPREME COURT REPORTS ANNOTATED This definition admits that the VFA does not provide for the entry of
contractors into Philippine territory. The activities that United States contractors
Saguisag vs. Ochoa, Jr. are allowed to undertake are specific to United States forces or personnel only as
can be gleaned from this court’s decisions in BAYAN, Lim, and Nicolas. Hence, the
extensive authority granted to United States contractors cannot be sourced from Respondents, through the Office of the Solicitor General, insist that the EDCA
the VFA: is an implementing agreement of the Mutual Defense Treaty and the VFA. They
do so based on the conclusion that all treaties or agreements entered into by the
Article II Philippines pursuant to certain principles contained in the Mutual Defense Treaty
DEFINITIONS may be considered subservient to these treaties. This will substantially weaken
.... the spirit of
4. “Agreed Locations” means facilities and areas that are provided by
the Government of the Philippines through the AFP and that United States
forces, United States contractors,and others as mutually agreed, shall have 685
the right to access and use pursuant to this Agreement. Such Agreed VOL. 779, JANUARY 12, 2016 685
Locations may be listed in an annex to be appended to this Agreement, and
may be further described in implementing arrangements. Saguisag vs. Ochoa, Jr.
....
_______________ Article XVIII, Section 25 and the sovereign desire to achieve an independent
foreign policy.
63 Agreement between the Government of the Philippines and the
Government of the United States of America on Enhanced Defense Cooperation X
(2014), Art. II(3).
The EDCA authorizes the use of Philippine territory as bases of operations.
Although not as permanent as those set up pursuant to the 1947 Military Bases
684 Agreement, they are still foreign military bases within the contemplation of Article
XVIII, Section 25 of the Constitution.
684 SUPREME COURT REPORTS ANNOTATED The development and use of these Agreed Locations are clearly within the
Saguisag vs. Ochoa, Jr. discretion of the United States. The retention of ownership by the Philippines
under Article V(1)64 of the EDCA does not temper the wide latitude accorded to the
Article III other contracting party. At best, the United States’ only obligation is to consult
AGREED LOCATIONS and coordinate with our government. Under the EDCA, the consent of the
1. With consideration of the views of the Parties, the Philippines Philippine government does not extend to the operations and activities to be
hereby authorizes and agrees that United States forces, United States conducted by the United States forces and contractors. Operational control
contractors, and vehicles, vessels, and aircraft operated by or for United remains solely with the United States government. The agreement did not create
States forces may conduct the following activities with respect to Agreed a distinction between domestic and international operations. Ownership of the
Locations: training; transit; support and related activities; refuel big of Agreed Locations under the EDCA is a diluted concept, with the Philippine
aircraft; bunkering of vessels; temporary maintenance of vehicles, vessels, government devoid of any authority to set the parameters for what may and may
and aircraft; temporary accommodation of personnel; communications; not be conducted within the confines of these areas.
prepositioning of equipment, supplies, and materiel; deploying forces and What constitutes a “base” in the context of United States-Philippine relations
materiel; and such other activities as the Parties may agree. may be explored by revisiting the 1947 Military Bases Agreement. 65 In one of the
.... agreement’s pream-
Article IV _______________
EQUIPMENT, SUPPLIES, AND MATERIEL
.... 64 “The Philippines shall retain ownership of and title to Agreed Locations.”
4. United States forces and United States contractors shall have 65 A copy is contained in Treaties and Other International Agreements of the
unimpeded access to Agreed Locations for all matters relating to the United States of America 1776-1949, as compiled under the direction of Charles I.
prepositioning and storage of defense equipment, supplies, and materiel, Bevans, LL.B., Assistant Legal Adviser, Department of
including delivery, management, inspection, use, maintenance, and removal State <http://kahimyang.info/kauswagan/Downloads.xhtml?sortorder=znoblair>
of such equipment, supplies and materiel. (visited November 5, 2015).
5. The Parties share an intent that United States contractors may
carry out such matters in accordance with, and to the extent permissible
under, United States laws, regulations, and policies. (Emphasis supplied) 686
686 SUPREME COURT REPORTS ANNOTATED d) the right to acquire, as may be agreed between the two
Governments, such rights of way, and to construct thereon, as may be
Saguisag vs. Ochoa, Jr. required for military purposes, wire and radio communications facilities,
including sub-marine and subterranean cables, pipe lines and spur tracks
bular clauses, the United States and Philippine governments agreed that in from railroads to bases, and the right, as may be agreed upon between the
line with cooperation and common defense, the United States shall be granted the two Governments to construct the necessary facilities;
use of certain lands of the public domain in the Philippines, free of rent.66 In line e) to construct, install, maintain, and employ on any base any type of
with the promotion of mutual security and territorial defense, the extent of rights facilities, weapons, substance, device, vessel or vehicle on or under the
of the contracting parties in the use of these lands was described in Article III of ground, in the air or on or under the water that may be requisite or
the agreement: appropriate, including meteorological systems, aerial and water navigation
lights, radio and radar apparatus and electronic devices, of any desired
Article III power, type of emission and frequency.
Description of Rights 3. In the exercise of the above mentioned rights, power and authority,
the United States agrees that the powers granted to it will not be used
1. It is mutually agreed that the United States shall have the rights, unreasonably or, unless required by military necessity determined by the
power and authority within the bases which are necessary for the two Governments, so as to interfere with the necessary rights of navigation,
establishment, use, operation and defense thereof or appropriate for the aviation, communication, or land travel within the territories of the
control thereof and all the rights, power and authority within the limits of Philippines. In the practical application outside the bases of the rights,
territorial waters and air space adjacent to, or in the vicinity of, the bases power and authority granted in this Article there shall be, as the occasion
which are necessary to provide access to them, or appropriate for their requires, consultation between the two Governments. (Emphasis supplied)
control.
2. Such rights, power and authority shall include, inter alia, the right, The bases contemplated by the 1947 Military Bases Agreement contain the
power and authority: elements of (a) absolute control of space; (b) the presence of a foreign command;
a) to construct (including dredging and filling), operate, maintain, and (c) having a purpose of a military nature. The agreement also relegates the
utilize, occupy, garrison and control the bases; role of the Philippine government to a mere “consultant” in cases of applications
b) to improve and deepen the harbors, channels, entrances and falling outside the terms provided in Article III.
anchorages, and to construct or maintain necessary roads and bridges
affording access to the bases;
c) to control (including the right to prohibit) insofar as may be 688
required for the efficient operation and safety of
688 SUPREME COURT REPORTS ANNOTATED
_______________
Saguisag vs. Ochoa, Jr.
66 WHEREAS, the Governments of the United States of America and of the
Republic of the Philippines are desirous of cooperating in the common defense of The EDCA contains similar elements.
their two countries through arrangements consonant with the procedures and However, the EDCA has an open-ended duration. Despite having an initial
objectives of the United Nations, and particularly through a grant to the United term of 10 years, Article XII(4) specifically provides for the automatic continuation
States of America by the Republic of the Philippines in the exercise of its title and of the agreement’s effectivity until a party communicates its intent to terminate. 67
sovereignty, of the use, free of rent, in furtherance of the mutual interest of both The purpose of the Agreed Locations is also open-ended. At best, its definition
countries, of certain lands of the public domain. and description of rights provide that the areas shall be for the use of United States
forces and contractors. However, short of referring to Agreed Locations as bases,
the EDCA enumerates activities that tend to be military in nature, such as
687 bunkering of vessels, pre-positioning of equipment, supplies, and materiel, and
VOL. 779, JANUARY 12, 2016 687 deploying forces and materiel.68 The United States is also allowed to undertake the
construction of permanent facilities,69 as well as to use utilities and its own
Saguisag vs. Ochoa, Jr. telecommunications systems.70
_______________
the bases, and within the limits of military necessity, anchorages,
moorings, landings, takeoffs, movements and operation of ships and 67 4. This Agreement shall have an initial term of ten years, and thereafter,
waterborne craft, aircraft and other vehicles on water, in the air or on land it shall continue in force automatically unless terminated by either Party by giving
comprising or in the vicinity of the bases;
one year’s written notice through diplomatic channels of its intention to terminate [“ITU”]). This shall include the right to utilize such means and services as required
this Agreement. to ensure the full ability to operate telecommunication systems, and the right to
68 Agreement between the Government of the Philippines and the use all necessary radio spectrum allocated for this purpose. Consistent with the
Government of the United States of America on Enhanced Defense Cooperation 1992 Constitution and Convention of the ITU, United States forces shall not
(2014), Art. III(1). interfere with frequencies in use by local operators. Use of the radio spectrum shall
69 Agreement between the Government of the Philippines and the be free of cost to the United States.
Government of the United States of America on Enhanced Defense Cooperation 71 Agreement between the Government of the Philippines and the
(2014), Art. V(4) provides: All buildings, non-relocatable structures, and Government of the United States of America on Enhanced Defense Cooperation
assemblies affixed to the land, in the Agreed Locations, including ones altered or (2014), Art. III(4).
improved by United States forces, remain the property of the Philippines.
Permanent buildings constructed by United States forces become the property of
the Philippines, once constructed, but shall be used by United States forces until 690
no longer required by United States forces. 690 SUPREME COURT REPORTS ANNOTATED
70 Agreement between the Government of the Philippines and the
Government of the United States of America on Enhanced Defense Cooperation Saguisag vs. Ochoa, Jr.
(2014), Art. VII provides for the use of utilities and communication systems:
4. The Parties shall take all reasonable measures to ensure the protection,
safety, and security of United States property from seizure by or conversion to the
689 use of any party other than the United States, without the prior written consent
of the United States. (Citation omitted)
VOL. 779, JANUARY 12, 2016 689
The United States Department of Defense Dictionary of Military and
Saguisag vs. Ochoa, Jr. Associated Terms72 defines “operational control” as:

Most significant is the Philippine government’s grant to the United States [O]perational control — The authority to perform those functions of
government of operational control over the Agreed Locations:71 command over subordinate forces involving organizing and employing
commands and forces, assigning tasks, designating objectives, and giving
Article VI authoritative direction necessary to accomplish the mission. Also called
Security OPCON.

.... Similar to the 1947 Military Bases Agreement, the role of the Philippine
3. United States forces are authorized to exercise all rights and government has been reduced to that of a consultant, except that the EDCA
authorities within Agreed Locations that are necessary for their avoided the use of this label.
operational control or defense, including taking appropriate measures to In some respects, too, the EDCA is similar to the Treaty of Friendship,
protect United States forces and United States contractors. The United Cooperation and Security between the Government of the Republic of the
States should coordinate such measures with appropriate authorities of the Philippines and the Government of the United States of America, which was
Philippines. rejected by the
_______________ _______________

1. The Philippines hereby grants to United States forces and United States 72 November 8, 2010, As Amended Through June 15, 2015
contractors the use of water, electricity, and other public utilities on terms and <http://fas.org/irp/doddir/dod/jpl_02.pdf> (visited November 5, 2015):
conditions, including rates or charges, no less favorable than those available to the 1. Scope
AFP or the Government of the Philippines in like circumstances, less charges for The Joint Publication 1-02, Department of Defense Dictionary of Military and
taxes and similar fees, which will be for the account of the Philippine Government. Associated Terms sets forth standard US military and associated terminology to
United States forces’ costs shall be equal to their pro rata share of the use of such encompass the joint activity of the Armed Forces of the United States. These
utilities. military and associated terms, together with their definitions, constitute approved
2. The Parties recognize that it may be necessary for United States forces to Department of Defense (DOD) terminology for general use by all DOD components.
use the radio spectrum. The Philippines authorizes the United States to operate 2. Purpose
its own telecommunication systems (as telecommunication is defined in the 1992 This publication supplements standard English-language dictionaries and
Constitution and Convention of the International Telecommunication Union standardizes military and associated terminology to improve communication and
mutual understanding within DOD, with other federal agencies, and among the 692 SUPREME COURT REPORTS ANNOTATED
United States and its allies.
Saguisag vs. Ochoa, Jr.

691 equipment, including essential utility systems such as energy and


water production and distribution systems and heating and air
VOL. 779, JANUARY 12, 2016 691
conditioning systems that are an integral part of such buildings and
Saguisag vs. Ochoa, Jr. structures, which are essential to the habitability and general use of such
improvements and are permanently attached to or integrated into the
Philippine Senate in 1991. This rejected treaty73 defines installations as: property.

“Installations” on the base authorized for use by the United States The treaty, which was not concurred in by the Senate, sets the parameters for
forces are buildings and structures to include non-removable buildings, defense cooperation and the use of installations in several provisions:
structures, and equipment therein owned by the Government of the Article IV
Philippines, grounds, land or sea areas specifically delineated for the Use of Installations by the US Forces
purpose. “Non-removable buildings and structures” refer to buildings,
structures, and other improvements permanently affixed to the ground, 1. Subject to the provisions of this Agreement, the Government of the
and such Philippines authorizes the Government of the United States to continue to
use for military purposes certain installations in Subic Naval Base.
_______________ 2. The installations shall be used solely for the purposes authorized
under this Agreement, and such other purposes as may be mutually agreed
73 This treaty contains a Supplementary Agreement on Installations and upon.
Military operating Procedures (Supplementary Agreement Number Two), which 3. Ownership of all existing non-removable buildings and structures
provides: in Subic Naval Base is with the Government of the Philippines which has
ARTICLE 1 title over them. The Government of the Philippines shall also become owner
PURPOSES OF THE UNITED STATES MILITARY PRESENCE IN THE of all non-removable buildings and structures that shall henceforth be
PHILIPPINES constructed in Subic Naval Base immediately after their completion, with
The Government of the Republic of the Philippines authorizes the Government title thereto being vested with the Government of the Philippines.
of the United States of America to station United States forces in the Philippines, 4. The Government of the United States shall not remove, relocate,
and in connection therewith to use certain installations in Subic Naval Base, which demolish, reconstruct or undertake major external alterations of non-
is a Philippine military base, designated training areas and air spaces, and such removable buildings and structures in Subic Naval Base without the
other areas as may be mutually agreed, for the following purposes and under the approval of the Philippine commander. The United States shall also not
terms and conditions stipulated in this Agreement: construct any removable or non-removable buildings or structures without
a. training of United States forces and joint training of United States forces the approval of the Philippine Commander. The Philippine Commander
with Philippine forces; will grant such
b. servicing, provisioning, maintenance, support and accommodation of
United States forces;
c. logistics supply and maintenance points for support of United States forces; 693
d. transit point for United States forces and United States military personnel; VOL. 779, JANUARY 12, 2016 693
e. projecting or operating United States forces from the installations under
conditions of peace or war, provided that military combat operations of United Saguisag vs. Ochoa, Jr.
States forces directly launched from installations on the base authorized for
United States use shall be subject to prior approval of the Government of the approval for reasons of safety as determined jointly by the Philippine
Philippines; and United States Commanders.
f. such other purposes, consistent with this Agreement, as may be mutually ....
agreed. 8. The Government of the United States shall bear costs of operations
and maintenance of the installations authorized for use in accordance with
Annex B to this Agreement.
692 9. The Government of the Philippines will, upon request, assist the
United States authorities in obtaining water, electricity, telephone and
other utilities. Such utilities shall be provided to the Government of the XI
United States, United States contractors and United States personnel for
activities under this Agreement at the rates, terms and conditions not less Thus, the EDCA amends the VFA. Since the VFA is a treaty, the EDCA cannot
favorable than those available to the military forces of the Philippine be implemented.
government, and free of duties, taxes, and other charges. Treaties, being of the same status as that of municipal law, may be modified
.... either by another statute or by the Constitu-
Article VII
Defense Cooperation and Use of Philippine Installations
1. Recognizing that cooperation in the areas of defense and security 695
serves their mutual interest and contributes to the maintenance of peace, VOL. 779, JANUARY 12, 2016 695
and reaffirming their existing defense relationship, the two Governments
shall pursue their common concerns in defense and security. Saguisag vs. Ochoa, Jr.
2. The two Governments recognize the need to readjust their defense
and security relationship to respond to existing realities in the national, tion itself.74 Treaties such as the VFA cannot be amended by an executive
regional, and global environment. To this end, the Government of the agreement.
Republic of the Philippines allows the Government of the United States to
use installations in Subic Naval Base for a specified period, under specific XII
conditions set forth in Supplementary Agreement Number Two: Agreement
on Installations and Military Operating Procedures and Supplementary Petitioners invoke this court’s power of judicial review to determine whether
Agreement Number Three: Agreement on the Status of Forces. respondents from the Executive Branch exceeded their powers and prerogatives in
entering into this agreement on behalf of the Philippines “in utter disregard of the
national sovereignty, territorial integrity and national interest provision of the
694 Constitution, Section 25 of the Transitory provisions of the Constitution, Section
21 and other provisions of the Philippine Constitution and various Philippine laws
694 SUPREME COURT REPORTS ANNOTATED
and principles of international law.”75
Saguisag vs. Ochoa, Jr. Petitioners submit that all requisites for this court to exercise its power of
judicial review are present.76 Petitioners in G.R. No. 212444 discussed that they
3. Both governments shall also cooperate in the maintenance, had legal standing and they raised justiciable issues. Petitioners in G.R. No.
upgrading and modernization of the defense and security capabilities of the 212426 similarly discussed their legal standing, the existence of an actual case or
armed forces of both countries, particularly of those of the Republic of the controversy involving a conflict of legal rights, and the ripeness of the case for
Philippines. In accordance with the common desire of the Parties to adjudication.77
improve their defense relationship through balanced, mutual contributions Respondents counter that only the Senate may sue on matters involving
to their common defense, the Government of the United States shall, constitutional prerogatives, and none of the petitioners are Senators. 78 They
subject to the constitutional procedures and to United States Congressional submit that “[t]he silence and active non-participation of the Senate in the current
action, provide security assistance to the Government of the Philippines to proceedings is an affirmation of the President’s characterization of the EDCA as
assist in the modernization and enhancement of the capabilities of the an executive agreement,”79 and “there is no such
Armed Forces of the Philippines and to support appropriate economic _______________
programs.
74 See Gonzales v. Hechanova, 118 Phil. 1065; 9 SCRA 230 (1963)
The 1987 Constitution does not proscribe the establishment of permanent or [Per J.Concepcion, En Banc] and Ichong v. Hernandez, 101 Phil. 1155 (1957)
temporary foreign military bases. However, the Constitution now requires that [Per J.Labrador, En Banc].
decisions on the presence of foreign military bases, troops, and facilities be not the 75 Memorandum for Petitioners Bayan, et al., pp. 3-4.
sole prerogative of the President and certainly not the prerogative at all of the 76 Id., at pp. 19-25; Memorandum for Petitioners Saguisag, pp. 11-17;
Secretary of Defense or Philippine Representatives to the Mutual Defense Board Memorandum for Petitioners-in-Intervention KMU, pp. 5-6.
and the Security Enhancement Board. 77 Memorandum for Petitioners Saguisag, pp. 11-17.
Absent any transmission by the President to the Senate, the EDCA remains a 78 Memorandum for Respondents, pp. 4-5.
formal official memorial of the results of intensive negotiations only. It has no legal 79 Id., at p. 6.
effect whatsoever, and any implementation at this stage will be grave abuse of
discretion.
696 legislature, but only asserts the solemn and sacred obligation assigned to
696 SUPREME COURT REPORTS ANNOTATED it by the Constitution to determine conflicting claims of authority under the
Constitution and to establish for the parties in an actual controversy the
Saguisag vs. Ochoa, Jr. rights which that instrument secures and guarantees to them. This is in
truth all that is involved in what is termed “judicial supremacy” which
actual conflict between the Executive and the Senate.”80 They add that the properly is the power of judicial review under the Constitution. Even then,
overuse of the transcendental importance exception “has cheapened the value of this power of judicial review is limited to actual cases and controversies to
the Constitution’s safeguards to adjudication.”81 be exercised after full opportunity of argument by the parties, and limited
Article VIII, Section 1 of the Constitution now clarifies the extent of this court’s further to the constitutional question raised or the very lis mota presented.
power of judicial review “to determine whether or not there has been a grave abuse Any attempt at abstraction could only lead to dialectics and barren legal
of discretion amounting to lack or excess of jurisdiction on the part of any branch questions and to sterile conclusions unrelated to actualities. Narrowed as
or instrumentality of the Government.”82 its function is in this manner, the judiciary does not pass upon questions of
The 1936 landmark case of Angara v. Electoral Commission83explained the wisdom, justice or expediency of legislation. More than that, courts accord
fundamental principle of separation of powers among government branches and the presumption of constitutionality to legislative enactments, not only
this court’s duty to mediate in the allocation of their constitutional boundaries: because the legislature is presumed to abide by the Constitution but also
because the judiciary in the determination of actual cases and controversies
In times of social disquietude or political excitement, the great must reflect the wisdom and justice of the people as expressed through their
landmarks of the Constitution are apt to be forgotten or marred, if not representatives in the executive and legislative departments of the
entirely obliterated. In cases of conflict, the judicial department is the only governments of the government.84
constitutional organ which can be called upon to determine the proper
allocation of powers between the several departments and among the Jurisprudence abounds on these four requisites for the exercise of judicial
integral or constituent units thereof. review. It must be shown that an actual case or controversy exists; that petitioners
. . . The Constitution sets forth in no uncertain language the restrictions have legal standing; that they raised the constitutionality question at the earliest
and limitations upon governmental powers and agencies. If these _______________
restrictions and limitations are transcended it would be inconceivable if the
Constitution had not provided for a mechanism by which to direct the 84 Id., at pp. 157-159.
course of government along constitutional channels, for then the
distribution of powers would be mere verbiage, the bill of rights mere
expressions of sentiment, and the principles of good government mere 698
political apothegms. Certainly, the limitation and restric-
_______________ 698 SUPREME COURT REPORTS ANNOTATED
Saguisag vs. Ochoa, Jr.
80 Id., at p. 7.
81 Id., at p. 8. possible opportunity; and that the constitutionality question is the very lis
82 Const., Art. VIII, Sec. 1. mota of the case.85
83 63 Phil. 139 (1936) [Per J. Laurel, En Banc]. This court can only exercise its power of judicial review after determining the
presence of all requisites, such as an actual case or controversy, in consideration
of the doctrine of separation of powers. It cannot issue advisory opinions nor
697 overstep into the review of the policy behind actions by the two other coequal
VOL. 779, JANUARY 12, 2016 697 branches of government. It cannot assume jurisdiction over political questions.

Saguisag vs. Ochoa, Jr. XIII

tions embodied in our Constitution are real as they should be in any The requirement for an actual case or controversy acknowledges that courts
living constitution. . . . should refrain from rendering advisory opinions concerning actions by the other
The Constitution is a definition of the powers of government. . . The branches of government.86
Constitution itself has provided for the instrumentality of the judiciary as Courts resolve issues resulting from adversarial positions based on existing
the rational way. And when the judiciary mediates to allocate facts established by the parties who seek the court’s application or interpretation
constitutional boundaries, it does not assert any superiority over the other of a legal provision that affects them.87It is not for this court to trigger or reenact
departments; it does not in reality nullify or invalidate an act of the
the political debates that resulted in the enactment of laws after considering Secretary of Justice, id., at pp. 534-535; and In the Matter of Save the Supreme
broadly construed factual circumstances to allow a general application by the Court Judicial Independence and Fiscal Autonomy Movement v. Abolition of
Executive.88 Judiciary Development Fund (JDF) and Reduction of Fiscal Autonomy, id.
The requisite actual case or controversy means the existence of “a conflict of 91 J. Leonen, Dissenting Opinion in Imbong v. Ochoa, Jr., G.R. Nos. 204819,
legal rights, an assertion of opposite April 8, 2014, 721 SCRA 146, 731 [Per J. Mendoza, En Banc], citing Angara v.
_______________ Electoral Commission, supra note 83 at p. 158; and Guingona, Jr. v. Court of
Appeals, 354 Phil. 415, 429; 292 SCRA 402, 413-414 (1998) [Per J. Panganiban,
85 See Francisco, Jr. v. Nagmamalasakit na mga Manananggol ng mga First Division].
Manggagawang Pilipino, Inc., 460 Phil. 830, 892; 415 SCRA 44, 133 (2003) 92 Id.
[Per J.Carpio-Morales, En Banc].
86 Lozano v. Nograles, 607 Phil. 334, 340; 589 SCRA 354, 358 (2009)
[Per CJ.Puno, En Banc]. See also J. Leonen, Dissenting and Concurring Opinion 700
in Disini, Jr. v. Secretary of Justice, G.R. Nos. 203335, February 18, 2014, 716 700 SUPREME COURT REPORTS ANNOTATED
SCRA 237, 535 [Per J. Abad, En Banc].
87 Diocese of Bacolod v. COMELEC, G.R. No. 205728, January 21, 2015, 747 Saguisag vs. Ochoa, Jr.
SCRA 1 [Per J. Leonen, En Banc].
88 Id. tion,93 as “[r]igor in determining whether controversies brought before us are
justiciable avoids the counter majoritarian difficulties attributed to the
judiciary.”94
699 Abstract cases include those where another political department has yet to act.
In other words, a case not ripe for adjudication is not yet a concrete case.
VOL. 779, JANUARY 12, 2016 699 Republic of the Philippines v. Roque95 clarified the concept of having an actual
Saguisag vs. Ochoa, Jr. case or controversy and the aspect of ripeness:

legal claims susceptible of judicial resolution; the case must not be moot or Pertinently, a justiciable controversy refers to an existing case or
academic or based on extralegal or other similar considerations not cognizable by controversy that is appropriate or ripe for judicial determination, not one
a court of justice.”89 It means the pleadings show “an active antagonistic assertion that is conjectural or merely anticipatory. Corollary thereto, by “ripening
of a legal right, on the one hand, and a denial thereof on the other; that is, it must seeds” it is meant, not that sufficient accrued facts may be dispensed with,
concern a real and not a merely theoretical question or issue.”90 but that a dispute may be tried at its inception before it has accumulated
Thus, it is not this court’s duty to “rule on abstract and speculative issues the asperity, distemper, animosity, passion, and violence of a full blown
barren of actual facts.”91 Ruling on abstract cases presents the danger of battle that looms ahead. The concept describes a state of facts indicating
foreclosing litigation between real parties, and rendering advisory opinions imminent and inevitable litigation provided that the issue is not settled and
presents the danger of a court that substitutes its own imagination and predicts stabilized by tranquilizing declaration.
facts, acts, or events that may or may not happen. 92 Facts based on judicial proof A perusal of private respondents’ petition for declaratory relief would
must frame the court’s discre- show that they have failed to demonstrate how they are left to sustain or
_______________ are in immediate danger to sustain some direct injury as a result of the
enforcement of the assailed provisions of RA 9372. Not far removed from
89 Information Technology Foundation of the Philippines v. COMELEC, 499 the factual milieu in the Southern Hemisphere cases, private respondents
Phil. 281, 304; 460 SCRA 291, 312 (2005) [Per J. Panganiban, En Banc], only assert general interests as citizens, and taxpayers and infractions
citing Republic v. Tan, G.R. No. 145255, March 30, 2004, 426 SCRA 485 which the government could prospectively commit if the enforcement of the
[Per J. Carpio-Morales, Third Division]. See also J. Leonen, Dissenting and said law would remain untrammelled. As their petition would disclose,
Concurring Opinion in Disini, Jr. v. Secretary of Justice, supra note 86 at p. 534; private respondents’ fear of prosecution was solely based on remarks of
and In the Matter of Save the Supreme Court Judicial Independence and Fiscal certain
Autonomy Movement v. Abolition of Judiciary Development Fund (JDF) and _______________
Reduction of Fiscal Autonomy, UDK-15143, January 21, 2015, 746 SCRA 352
[Per J. Leonen, En Banc]. 93 Id.
90 Information Technology Foundation of the Philippines v. COMELEC, id., 94 Id., at p. 721.
at p. 305; pp. 312-313, citing De Lumen v. Republic, 50 OG No. 2, February 14, 95 G.R. No. 204603, September 24, 2013, 706 SCRA 273 [Per J. Perlas-
1952, 578. See also J. Leonen, Dissenting and Concurring Opinion in Disini, Jr. v. Bernabe, En Banc].
There is still a political act that must happen before the agreement can become
valid and binding. The Senate can still address the constitutional challenges with
701 respect to the contents of the EDCA. Thus, the challenges to the substantive
VOL. 779, JANUARY 12, 2016 701 content of the EDCA are, at present, in the nature of political questions.
However, the nature of the EDCA, whether it is a treaty or merely an executive
Saguisag vs. Ochoa, Jr. agreement, is ripe for adjudication.
In 1957, Tañada v. Cuenca99 explained the concept of political questions as
government officials which were addressed to the general public. They, referring to issues that depend not on the legality of a measure but on the wisdom
however failed to show how these remarks tended towards any behind it:
prosecutorial or governmental action geared towards the implementation
of RA 9372 against them. In other words, there was no particular, real or As already adverted to, the objection to our jurisdiction hinges on the
imminent threat to any of them. As held in Southern Hemisphere: question whether the issue before us is political or not. In this connection,
Without any justiciable controversy, the petitions have become Willoughby lucidly states:
pleas for declaratory relief, over which the Court has no original “Elsewhere in this treatise the well-known and well-established
jurisdiction. Then again, declaratory actions characterized by principle is considered that it is not within the province of the courts
“double contingency” where both the activity the petitioners intend to pass judgment upon the policy of legislative or executive action.
to undertake and the anticipated reaction to it of a public official are Where, therefore, discretionary powers are granted by the
merely theorized, lie beyond judicial review for lack of ripeness. Constitution or by statute, the manner in which those powers are
The possibility of abuse in the implementation of RA 9372 does exercised is not subject to judicial review. The courts, therefore,
not avail to take the present petitions out of the realm of the surreal _______________
and merely imagined. Such possibility is not peculiar to RA 93 72
since the exercise of any power granted by law may be abused. 98 Id., at p. 369; p. 383, citing Lozano v. Nograles, supra note 86, in turn
Allegations of abuse must be anchored on real events before courts citing Guingona, Jr. v. Court of Appeals, supra note 91 at pp. 427-428; pp. 414-415.
may step in to settle actual controversies involving rights which are 99 103 Phil. 1051 (1957) [Per J. Concepcion, En Banc].
legally demandable and enforceable.96 (Emphasis supplied,
citations omitted)
703
Our courts generally treat the issue of ripeness for adjudication in terms of
actual injury to the plaintiff.97 The question is whether “the act being challenged VOL. 779, JANUARY 12, 2016 703
has had a direct adverse Saguisag vs. Ochoa, Jr.
_______________
concern themselves only with the question as to the existence
96 Id., at pp. 284-285. See also J. Leonen, Dissenting and Concurring Opinion and extent of these discretionary powers.
in Disini, Jr. v. Secretary of Justice, supra note 86 at pp. 536-537. As distinguished from the judicial, the legislative and executive
97 Lawyers Against Monopoly and Poverty (LAMP) v. Secretary of Budget and departments are spoken of as the political departments of
Management, 686 Phil. 357; 670 SCRA 373 (2012) [Per J. Mendoza, En Banc]. government because in very many cases their action is necessarily
dictated by considerations of public or political policy. These
considerations of public or political policy of course will not
702 permit the legislature to violate constitutional provisions, or
702 SUPREME COURT REPORTS ANNOTATED the executive to exercise authority not granted him by the
Constitution or by statute, but, within these limits, they do
Saguisag vs. Ochoa, Jr. permit the departments, separately or together, to recognize that a
certain set of facts exists or that a given status exists, and these
effect on the individual challenging it.”98 The Petitions are premature. Since determinations, together with the consequences that flow
the Senate has yet to act and the President has yet to transmit to the Senate, there therefrom, may not be traversed in the courts.”
is no right that has been violated as yet. To the same effect is the language used in Corpus Juris Secundum,
from which we quote:
XIV “It is well-settled doctrine that political questions are not within
the province of the judiciary, except to the extent that power to deal
with such questions has been conferred upon the courts by express
constitutional or statutory provisions. tionary acts cannot be controllable, not primarily because they
It is not easy, however, to define the phrase ‘political question,’ are of a political nature, but because the Constitution and laws have
nor to determine what matters fall within its scope. It is frequently placed the particular matter under his control. But every officer
used to designate all questions that the outside the scope of the under a constitutional government must act according to law and
judicial questions, which under the constitution, are to be decided subject him to the restraining and controlling power of the people,
by the people in their sovereign capacity, or in regard to which acting through the courts, as well as through the executive or the
full discretionary authority has been delegated to the legislative Legislature. One department is just as representative as the other,
or executive branch of the government.” and the judiciary is the department which is charged with the special
duty of determining the limitations which the law places upon all
official action. The recognition of this principle, unknown except in
704 Great Britain and America, is necessary, to ‘the end that the
704 SUPREME COURT REPORTS ANNOTATED government may be one of laws and not men’ — words which
Webster said were the greatest contained in any written
Saguisag vs. Ochoa, Jr. constitutional document.”
In short, the term “political question” connotes, in legal parlance, what
Thus, it has been repeatedly held that the question whether certain it means in ordinary parlance, namely, a question of policy. In other words,
amendments to the Constitution are invalid for noncompliance with in the language of Corpus Juris Secundum (supra), it refers to “those
the procedure therein prescribed, is not a political one and may be settled questions which, under the Constitution, are to be decided by the people in
by the Courts. their sovereign capacity, or in regard to which full discretionary
In the case of In re McConaughy, the nature of political question was authority has been delegated to the Legislature or executive branch of the
considered carefully. The Court said: Government.” It is concerned with issues dependent upon the wisdom, not
“At the threshold of the case we are met with the assertion that legality, of a particular measure.100 (Emphasis supplied, citations omitted)
the questions involved are political, and not judicial. If this is
correct, the court has no jurisdiction as the certificate of the state Francisco, Jr. v. Nagmamalasakit na mga Manananggol ng mga
canvassing board would then be final, regardless of the actual vote Manggagawang Pilipino, Inc.101 involved the second impeachment Complaint filed
upon the amendment. The question thus raised is a fundamental against former Chief Justice Hilario Davide before the House of Representatives
one; but it has been so often decided contrary to the view contended and raised
for by the Attorney General that it would seem, to be finally settled. _______________
....
. . . What is generally meant, when it is said that a question is 100 Id., at pp. 1065-1067.
political, and not judicial, is that it is a matter which is to be 101 Supra note 85.
exercised by the people in their primary political capacity, or
that it has been specifically delegated to some other
department or particular officer of the government, with 706
discretionary power to act. Thus the Legislature may in its
discretion determine whether it will pass a law or submit a proposed 706 SUPREME COURT REPORTS ANNOTATED
constitutional amendment to the people. The courts have no judicial Saguisag vs. Ochoa, Jr.
control over such matters, not merely because they involve political
question, but because they are matters which the people have by the the issue of whether this raised a political question. It traced the evolution of
Constitution delegated to the Legislature. The Governor may jurisprudence on the political question doctrine and the effect of this court’s
exercise the powers delegated-to him, free from judicial control, so expanded power of judicial review under the present Constitution on this doctrine:
long as he observes the laws and acts within the limits of the power
conferred. His discre- As pointed out by amicus curiae former dean Pacifico Agabin of the UP
College of Law, this Court has in fact in a number of cases taken jurisdiction
over questions which are not truly political following the effectivity of the
705 present Constitution.
VOL. 779, JANUARY 12, 2016 705 In Marcos v. Manglapus, this Court, speaking through Madame Justice
Irene Cortes, held:
Saguisag vs. Ochoa, Jr.
The present Constitution limits resort to the political question constitutional bodies, including this court. A policy of deference and respect for the
doctrine and broadens the scope of judicial inquiry into areas which allocation of such power by the sovereign to a legislative chamber requires that we
the Court, under previous constitutions, would have normally left to refrain from making clear and categorical rulings on the constitutional challenges
the political departments to decide. . . . to the content of the EDCA.
In Bengzon v. Senate Blue Ribbon Committee, through Justice Teodoro
Padilla, this Court declared: XV
The “allocation of constitutional boundaries” is a task that this
Court must perform under the Constitution. Moreover, as held in a It is true that we have, on certain occasions, substantially overridden the
recent case, (t)he political question doctrine neither interposes an requirements of justiciability when there is an imminent threat to the violation of
obstacle to judicial determination of the rival claims. The constitutional rights. In Garcia v. Drilon,105 I stated that:
jurisdiction to delimit constitutional boundaries has been given to
this Court. It cannot abdicate that obligation mandated by the 1987 I am aware of our precedents where this Court has waived questions
Constitution, although said provision by no means does away with relating to the justiciability of the constitutional issues raised when they
the applicability of the principle in appropriate cases. have “transcendental importance” to the public. In my view, this
And in Daza v. Singson, speaking through Justice Isagani Cruz, this accommodates our power to promulgate guidance “concerning the
Court ruled: protection and enforcement of constitutional rights.” We choose to rule
In the case now before us, the jurisdictional objection becomes squarely on the constitutional issues in a petition wanting all or some of
even less tenable and decisive. The reason is that, even if we the technical requisites to meet out general doctrines on justiciability but
raising clear conditions showing imminent threat to fundamental
rights. The imminence and clarity of the threat to fundamental
707 constitutional rights outweigh the necessity for prudence. In a sense,
VOL. 779, JANUARY 12, 2016 707 our exceptional doctrine relating to constitutional issues of “transcendental
importance” prevents courts from the paralysis of procedural niceties when
Saguisag vs. Ochoa, Jr. clearly faced with the need for substantial protection. 106 (Emphasis
supplied, citations omitted)
were to assume that the issue presented before us was political
in nature, we would still not be precluded from resolving it under There is, however, no need to invoke these exceptions. The imminence of the
the expanded jurisdiction conferred upon us that now covers, in implementation of the EDCA and, therefore,
proper cases, even the political question. . . . the clarity of the impending threat to constitutional rights do not appear cogent if
.... we declare that the EDCA, without Senate concurrence, is not yet valid and
In our jurisdiction, the determination of a truly political question from binding as a treaty or fully complying with the requirements of Article XVIII,
a nonjusticiable political question lies in the answer to the question Section 25.
of whether there are constitutionally imposed limits on powers or
functions conferred upon political bodies. If there are, then our XVI
courts are duty-bound to examine whether the branch or
instrumentality of the government properly acted within such The proposed disposition of this case does not in any way discount the
limits[.]102 (Emphasis supplied) deployment of the expertise of the Executive as it conducts foreign policy. Nor
should we arrogate executive discretion by compelling the President to transmit
In Diocese of Bacolod v. COMELEC,103 this court held that the political the agreement to the Senate for concurrence.107
question doctrine never precludes this court’s exercise of its power of judicial Nevertheless, the judiciary has the duty to ensure that the acts of all branches
review when the act of a constitutional body infringes upon a fundamental of government comply with the fundamental nature of the Constitution. 108 While
individual or collective right.104However, this will only be true if there is no other the EDCA is a formal and official memorial of the results of negotiations between
constitutional body to whom the discretion to make inquiry is preliminarily the Philippines and the United States, it is not yet effective until the Senate
granted by the sovereign. concurs or there is compliance with Congressional action to submit the agreement
Ruling on the challenge to the content of the EDCA will preclude and interfere to a national referendum in accordance with Article XVIII, Section 25 of the
with any future action on the part of the Senate as it inquires into and deliberates Constitution.
as to whether it should give its concurrence to the agreement or whether it should It is, thus, now up to the President. Should he desire to continue the policy
advise the President to reopen negotiations to amend some of its provisions. It is embedded in the EDCA, with deliberate dispatch he can certainly transmit the
the Senate, through Article VII, Section 21 in relation to Article XVIII, Section 25, agreement to the Senate for the latter to initiate the process to concur with the
that was given the discretion to make this initial inquiry exclusive of all other
agreement. After all, on these matters, the sovereign, speaking through the statement is that it will not interfere in those types of differences we have with
Constitution, has assumed that the exercise of wisdom is not within the sole China, among others.
domain of the President. Wisdom, in allowing foreign military bases, troops, or The inequality of the Mutual Defense Treaty is best presented by the image of
facilities, is likewise within the province of nationally elected Senators of the a commissioned but rusting and dilapidated warship beached in a shoal in the
Republic. West Philippine Sea. This ship is manned by a handful of gallant heroic marines,
and by the provisions of the Mutual Defense Treaty, an attack on this ship — as a
public vessel — is what we are relying upon to trigger mutual defense with the
On these matters, the Constitution rightly assumes that no one person — United States.
because of the exigencies and their consequences — has a monopoly of wisdom. We remain a permanent ally of the United States. For decades, we relied on
In my view, the same security concerns that moved the President with haste to them for the training of our troops and the provision of military materiel. For
ratify the EDCA signed by his Secretary of Defense will be the same security decades, we hosted their bases. Yet, our armed forces remain woefully equipped.
concerns — and more — that will move the Senate to consider the agreement with Unlike in many of their other allies, no modern US-made fighter jet exists in our
dispatch. There are matters of national consequence where the views of an elected Air Force. We have no credible missile defense. Our Navy’s most powerful assets
President can be enriched by the views of an elected Senate. Certainly, the now include a destroyer that was decommissioned by the United States Coast
participation of the public through these mechanisms is as critical as the foreign Guard.
policy directions that the EDCA frames. It is now suggested that these will change with the EDCA. It is now suggested
By abbreviating the constitutional process, this court makes itself vulnerable that this court should act to make that change possible. Impliedly, it is thus also
to a reasonable impression that we do not have the courage to enforce every word, suggested that the Senate, or Congress, or the People in a referendum as provided
phrase, and punctuation in the Constitution promulgated by our People. We will in our Constitution, will be less patriotic than this court or the President.
stand weak, as an institution and by implication as a state, in the community of There has never been a time in our history — and will never be a time in the
nations. In clear unequivocal words, the basic instrument through which we exist future — when the national interest of the United States was subservient to ours.
requires that we interpret its words to make real an independent foreign policy. It We cannot stake our future on how we imagine the United States will behave in
requires measures be fully publicly discussed before any foreign resource capable the future. We should learn from our history. If we wish the United States to
of making war with our neighbors and at the command of a foreign sovereign — behave in a way that we expect, then our government should demand clear
foreign military bases, troops and facilities — becomes effective. commitments for assistance to our primary interests. The likelihood that this will
Instead, the majority succumbed to a narrative of dependence to a superpower. happen increases when agreements with them run through the gauntlet of public
Our collective memories are perilously short. Our sense of history is wanting. opinion before they become effective.
The Americans did not recognize the Declaration of Independence of 1898, Certainly, this is what the Constitution provides. Certainly, this is the least
which was made possible by the blood of our ancestors. They ignored their that we should guarantee as a court of law.
agreements with the Filipino revolutionaries when they entered Intramuros and
staged the surrender of the Spanish colonizers to them. They ignored our Final Note
politicians when they negotiated the Treaty of Paris. Not a single Filipino was
there — not even as an observer. They triggered armed conflict with the Filipino In 1991, there was the “Senate that Said No” to the extension of the stay of
revolutionaries. The schools they put up attempted to block out the inhumanity military bases of the United States within Philippine territory. That historical
and barbarism in the conflict that followed. Only a few remember the massacres decision defined the patriotism implicit in our sovereignty. That single collective
of Samar, of Bud Dajo, and of other places in our country. In the memory of many act of courage was supposed to usher opportunities to achieve the vision of our
Filipinos today, these brutalities have been practically erased. Constitution for a more meaningful but equal relationship with the American
Filipino veterans of World War II who fought gallantly with the Americans, empire. That act was the pinnacle of decades of people’s struggles.
now gray and ailing, still await equal treatment with United States war veterans.
Filipina comfort women of that war still seek just treatment and receive no succor History will now record that in 2016, it is this Supreme Court that said yes to
from the ally with and for whom they bled and suffered. the EDCA. This decision now darkens the colors of what is left of our sovereignty
The 1951 Mutual Defense Treaty and the Visiting Forces Agreement was in as defined in our Constitution. The majority’s take is the aftermath of squandered
effect when the Chinese invaded certain features within our Exclusive Economic opportunity. We surrender to the dual narrative of expediency and a hegemonic
Zone in the West Philippine Sea. The Americans did not come to our aid. The view of the world from the eyes of a single superpower. The opinion of the majority
President of the United States visited and, on the occasion of that visit, our own of this Supreme Court affirms executive privileges and definitively precludes
President announced the completion of the EDCA. No clear, unequivocal, and Senate and/or Congressional oversight in the crafting of the most important
binding commitment was given with respect to the applicability of the Mutual policies in our relations with the United States and, implicitly, its enemies and its
Defense Treaty to the entirety of our valid legal claims in the West Philippine Sea. allies. In its hurry to abbreviate the constitutional process, the majority also
The commitment of the United States remains ambiguous. The United States’
excludes the possibility that our people directly participate in a referendum called
to affirm the EDCA.
Article XVIII, Section 25 does not sanction the surreptitious executive approval
of the entry of United States military bases or any of its euphemisms (i.e., “Agreed
Locations”) through strained and acrobatic implication from an ambiguous and
completely different treaty provision.
The majority succeeds in emasculating our Constitution. Effectively, this court
erases the blood, sweat, and tears shed by our martyrs.
I register more than my disagreement. I mourn that this court has allowed this
government to acquiesce into collective subservience to the Executive power
contrary to the spirit of our basic law.
I dissent.
ACCORDINGLY, I vote to PARTIALLY GRANT the Petitions and
to DECLARE the Enhanced Defense Cooperation Agreement (EDCA) between
the Republic of the Philippines and the United States of America as a formal and
official memorial of the results of the negotiations concerning the allowance
of United States military bases, troops, or facilities in the Philippines, which
is NOT EFFECTIVE until it complies with the requisites of Article XVIII, Section
25 of the 1987 Philippine Constitution, namely: (1) that the agreement must be
in the form of a treaty; (2) that the treaty must be duly concurred in by the
Philippine Senate and, when so required by Congress, ratified by a majority of
votes cast by the people in a national referendum; and (3) that the agreement is
either (a) recognized as a treaty or (b) accepted or acknowledged as a treaty by the
United States before it becomes valid, binding, and effective.
Petitions dismissed.
Notes.—Article XVIII, Sec. 25 of the Constitution is designed to ensure that
any agreement allowing the presence of foreign military bases, troops or facilities
in Philippine territory shall be equally binding on the Philippines and the foreign
sovereign State involved, the idea being to prevent a recurrence of what happened
in the past. (Nicolas vs. Romulo, 578 SCRA 438 [2009])
It is settled that “the conduct of the foreign relations of our government is
committed by the Constitution to the executive and legislative — ‘the political’ —
departments of the government, and the propriety of what may be done in the
exercise of this political power is not subject to judicial inquiry or decision.” (Arigo
vs. Swift,735 SCRA 102 [2014])

——o0o——

Vous aimerez peut-être aussi